41
ECN-0102-2013-001-000076193 먼저 답안지에 성명 학년 과목코드를 기입하십시오 , , . 문항을 읽고 맞는 답을 답란에 표시하십시오. 문항배점은 문항위에 표시된 배점표를 참고하십시오. 다음 중 사회방언의 사례로 볼 수 없는 것은? 무속인들은 머리 를 석거리 놀라다 를 노재다 라 한다 ' ' ' ', ' ' . 재래시장에서 를 먹주대삼패라 한다 '1,2,3' , , . 군대에서 병사들은 장군이 온다 를 별이 떴다 라 한다 " ." .” . ④ 대학생들은 단과대학 을 단대 사범대학 을 사대라 한다 ' ' ,‘ . ⑤ 심마니들은 산삼 을 심이나 심메 호랑이 를 산개라 한다 ,‘ . 사회 방언의 부정적인 요인으로 적절하지 않은 것은? 계층별 위화감 세대 간의 대화 단절 현상 구성원들 간의 불신 현상 ( ) 不信 ④ 직업별 언어 특성에 대한 몰이해 ⑤ 같은 집단 내에서의 심리적 불안감 차별적 표현에 대한 대안적 표현 의 예로 적절하지 않은 것은? 성별어에 대한 설명으로 옳지 않은 것은? ① 성별어는 남성어와 여성어로 사회 방언에 속한다. ② 여성의 대화에는 주로 동의를 구하는 부가 의문문이 사용 되고 있어. ③ 맞아 여성의 대화는 대부분 동조적 협조적이라 할 수 있 , , 지. ④ 반면에 남성의 대화는 단정적이라 할 수 있는 것 같아. ⑤ 그래 주로 비격식체를 사용하면서 말이야 , . 사회 방언 중 언어적 차별 표현 에 해당하는 것이 아닌 것은? 처녀작 집사람 혼혈 ④ 절름발이 ⑤ 흑인 언어적 차별 표현 부정적 영향 1) 스스로 열등감을 갖게 됨 왜곡된 이성관을 갖게 함 남자 미용사 여걸 여장부 ex) , ( ), ( ) 女傑 女丈夫 신체 인종 직업 등에 대한 편견을 양산함 , , , 모순적이고 양면적인 생활 태도를 지닐 수도 있음 여자는 조신해야 한다 남자가 한 입으로 두 말 하 ex) , 냐? 해소방법 2) 콘텐츠산업 진흥법 시행령 조에 의한 표시 33 ◇「 제작연월일 1) : 2013-08-22 제작자 2) : 교육지대㈜ 이 콘텐츠는 콘텐츠산업 진흥법 에 따라 최초 3) 제작일부터 년간 보호됩니다 5 . 콘텐츠산업 진흥법 외에도 저작권법 에 의하여 ◇「 보호되는 콘텐츠의 경우 그 콘텐츠의 전부 또는 일부 , 무단으로 복제하거나 전송하는 것은 콘텐츠산업 진 흥법 외에도 저작권법에 의한 법적 책임을 질 수 있습 니다. 학기 2013 2 중간고사 대비 현대고 대비 국어

현대고대비 국어 - chamsoriedu.com 「콘텐츠산업진흥 법」외 에도 저작권 의하여 ... 다른주체에게어떤동작을하도록만드는것을나타내는

  • Upload
    others

  • View
    0

  • Download
    0

Embed Size (px)

Citation preview

Page 1: 현대고대비 국어 - chamsoriedu.com 「콘텐츠산업진흥 법」외 에도 저작권 의하여 ... 다른주체에게어떤동작을하도록만드는것을나타내는

ECN-0102-2013-001-000076193

저 답안 에 성 학년 과 코드를 입하 bull

항을 읽고 맞는 답을 답란에 표 하 bull

항 점은 항위에 표 된 점표를 참고하 bull

zb1) 다음 사회 언의 사 볼 없는 것은

들 리 거리 다 재다 다 lsquo rsquo ①

재 시 에 주 삼 다123 lsquo rsquo ②

에 병사들 다 별 떴다 다 ldquo rdquo ③

④ 생들 단과 단 사 사 다 lsquo rsquo

⑤ 심마니들 산삼 심 심 산개 다lsquo rsquo lsquo rsquo

zb2) 사회 언의 정적인 인으 적절하 않은 것은

계 별 감①

간 단 상②

원들 간 신 상( )不信③

④ 직업별 언어 특 에 몰

⑤ 같 집단 내에 심리 감

zb3) 차 적 표현에 대한 대안적 표현의 적절하lsquo rsquo

않은 것은

zb4) 성 어에 대한 설 으 않은 것은

① 별어는 어 여 어 사 언에 다

② 여 에는 주 동 는 가 사

고 어

③ 맞 여 는 동

④ 에 는 단 는 것 같

⑤ 그 주 격식체 사 말

zb5) 사회 언 언어적 차 표현에 해당하는 것이lsquo rsquo

아닌 것은

① 집사② ③

④ ⑤

언어 별

1)

열등감 갖게

곡 갖게

미 사 여걸 여ex) ( ) ( )女傑 女丈夫

신체 직업 등에 편견 산

고 생 태도 지닐 도

여 는 신 다 가 말ex)

2)

콘텐츠산업 진흥법 시행령 제 조에 의한 표시33「 」

제작연월일1) 2013-08-22

제작자2) 교육지대

이 콘텐츠는 콘텐츠산업 진흥법 에 따라 최초3) 「 」

제작일부터 년간 보호됩니다5

콘텐츠산업 진흥법 외에도 저작권법 에 의하여 「 」 「 」

보호되는 콘텐츠의 경우 그 콘텐츠의 전부 또는 일부

를 무단으로 복제하거나 전송하는 것은 콘텐츠산업 진

흥법 외에도 저작권법에 의한 법적 책임을 질 수 있습

니다

고년 학기2013 2

중간고사 대비

현대고 대비국어

년 학 간고사 대비2013 2 현대고 대비

ECN-0102-2013-001-000076193

가 과 등 가 운 곳에 실

언어 별

만들어

울 가다 울 가다ex) rarr

살색 ( )rarr

( )rarr

직 태도3)

동( )和而不同 태도 지

zb6) 과 에 적절한 말을 쓰

zb7) 에서 장 리적 태도의 화 높임 표lt gt

현의 계가 적절한 것은

보lt gt

마루S

느리 신 식 들 시 지

시열 에 계신다

말 다 말 우리 지

첫 신 탄 게 어 냐

우리 꿔 우리

시 지 감사 말 우리 집 에

경사 어 님 지 에 가 는 시

느리 거 사 없 말 드 릴게

마 어 님도 지 뻐 실 거 늘 편

시고 내 가 감 사 드리러 갈게 마 편

시고 몸 심 느리 지가 편 (

에게 꾼다 어) (

는다 지 들 죠)

시열 그 들었다 게 경사냐 도 우리

집 징 가 보다 허허허

① 시열과 는 랫사 들에게 체 사 고

② 사에 는 청 는 다 4

③ 는 지 느리에게 심리 태도 드러

내 십시 체 사 고 다

④ 는 지 들 느리에게 격식체 상

사 여 심리 거리감 드러내고 다

⑤ 는 느리에게 신 것에 고마움 심리 태

도 드러내 사 고 다

zb8) 다음 의 에 대한 설 으 적절하lt gt - 않

은 것은

lt gt

아 채민이 너 공 열 히 - 하는 나

채민 에휴 넌 내 마음 라 hellip

채민 엄마가 나에게 큰 을 어 이 에 또 성적이 떨

어 휴대폰 빼앗아 리 대 러 나는 눈 이

맺힐 것 같아

아 전 코너에 네

채민 러게 말이야 전히 쫒 는 마음으 공 하

고 있어

① 주어가 동 는 주동 다

② 주어가 다 주체에게 어 동 도 시키고

③ 주어 극 태도가 드러 다

④ 진 주체 동 고 볼 없다

⑤ 동 미사 사 동 다 lsquo- -rsquo

가 어 상에 거 낮

는 주체1) 2) 상 객체 3)

등 다 주동사동 주어가 직 동 거

다 주체에게 어 동 도 만드는 것 타내는

다 능동 동 주어가 동

거 다 주체에 어 동 당

타내는 다

zb9) 다음 친 주체 높임 표현 성격이 가장 이 적

인 것은

① 그 살림 시

② 그 가 없 십니다

③ 그 님 직도 귀가 시다

④ 생님 에 가 편 시지

⑤ 지 주에 시골에 사시는 니께 다

zb10) 다음 높임 이 른 것은

년 학 간고사 대비2013 2 현대고 대비

ECN-0102-2013-001-000076193

① 지 지는 고 습니다

② 지 지께 는 고 계십니다

③ 지 지께 는 고 계시 니다

④ 사 님 에 에 갔습니다

⑤ 사 님 님께 는 에 에 가 습니다

가( ) S1

소장 자네에게 을 게 되어 정말 안하네 자네에게 뒷

일을 맡겨야겠네

나 자 네 여 는 당입니다 한때( ) 1 당에 쫓

도 했 니다 하 만 선거 후 전에 의 있는 정책

으 선거전을 서 당 위 를 끌어 린 것이

당을 고 리를 이끈 인으 고 있 니다

자 네 여 는 당입니다 선거 후 전 터 당2

위 가 한풀 이 서 세에 었는데 일 의

들의 눈에는 눈 이 맺혀 있 도 합니다

다 과량 사 액이 삼켜져서 위장 흡 에 의한 전( )

작 도 일어날 있 니다 따라서 액이 삼켜 않도

주의해야 합니다

라 통의 날을 맞아 늘 에서는 통사고 절 으( ) lsquo rsquo lsquo

이 캠페인을 내 곳곳에서 다채 게 펼쳤 니다 늘rsquo

행사는 많은 민들의 호응을 이끌어 냈 니다 계

자는 제 통사고 생률을 낮출 있는 체적인 정책도

마 하고 있다고 말했 니다

마 통의 날을 맞아 휴일인 늘 내 곳곳에서는 다양한( ) lsquo rsquo

통의 날 행사가lsquo rsquo 개최되었 니다 써 터 통의 날 lsquo rsquo

행사가 생색내 에 나 않는다는 비판이 제 되고 있 니

다 체적인 정책적 뒷 침이 마 되 않는다 이러한 비

판의 소리는 점점 더 커 것으 전망됩니다

zb11) 다음 조건 에 맞게 나 의 을 능동 으 을lt gt ( ) lsquo rsquo lsquo rsquo

피동 으 고쳐 각각 한 장으 서 하

조건lt gt

은 이가 을를 다 조의 능동 으1 lsquo- ( ) - ( ) - rsquo

은 이가 에 다 조의 피동 으2 lsquo- ( ) - - rsquo

에 사 할 단어는 나에서 찾아3 lsquo-rsquo ( )

서 어의 제 높임은 상 없이 장을 만들 것4

가 역도 앞 낮( ) S 38 ( )

냉장고를 열고 펄떡거리는 어들이 들어 있는 낚

망을 내는 고 를 매 탕과 함 맛

있게 을 고 있는 과 아이들 인다

현정 적 적 만날 맛난 거 어 좋 는 허 만 이제 나( )

도 값을 해야 헐튼디 맨날 이 만 하고 선생님

리 역도는 언제 가르쳐 대

당황해서 둘러대듯 초 체 이 한 ( ) helliphellip

거야 많이 어 근엄한 말투 역도는 일단 이 을 ( )

해야 할 있는 거다 략 lt gt

나 학 동장 치 낮( ) S 40 ( )

여 후 에게 똥침을 놓고 도망간다 장난 럽게 달

리 를 하고 있는 역도 들 치에 앉아서 아이들을 라

는 에게 다가 는 현정

현정 주장으 서 할 말 있 니다 훈 위 고 장난쳐도

혼내 도 않으 고 리고 역도 는 여 이처럼 꼬 꼬

한 아이들만 뽑 니

아이들을 딱하잖냐 래도 자들이 착하다 주장 ( )

이 이해 좀 해

답답한 표정 는 현정

zb12) 가 나 의 장 을 이 는 소에 대한 설 으 적( ) ( )

절하 않은 것은

① 가 청 는 동 다( ) ( )

② 가 시간 상 낮 다( ) ( )

③ 가 에 공간 상 달 다( ) ( )

④ 가 에 심리 태도는 다 게( ) ( )

타 다

⑤ 가 에 지 다 통( ) ( )

에 심리 거리감 타낸다

가( )

가 각시 본 듯도 고

상( )天上 경( )白玉京 엇 니별 고( ) 離別

년 학 간고사 대비2013 2 현대고 대비

ECN-0102-2013-001-000076193

다 믄 눌 가시 고

어 여 고 내 드러 보

내 얼 거동 님 가마

엇 보시고 다 실

도 님 미 업

어 러 돗

시 엇 다 신고

누어 각 고 니러 여 니

내 몸 지 죄 뫼 티 시니

원망 사 허믈 랴

워 니 믈 타시 다( ) 造物

각 마

략( )

님다 쇼식 므 쟈 니( )消息

도 거 다 사 가

내 업다 어드러 가쟛 말고

거니 거니 뫼 가니

니 개 므 고

산쳔 어 거니( ) (A)山川 월( )日月 엇 보

지쳑 거든 쳔리 보랴( ) ( ) 咫尺 千里

리 믈 가 보쟈 니

믈결 어 고

샤공 어 가고 만 걸 니

강 쟈 보니( )江天

님다 쇼식 득 고( )消息

략( )

근 역진 간 드니( )力盡

지극 님 보니( )精誠

얼 마 늘거( ) ( ) 玉 半

근 말 슬 쟈 니

눈믈 니 말 들 어

못다 여 니( )情

계 ( )鷄聲 엇 돗 고

어 허 다 님 어 간고( ) 虛事

결 니러 열고 보니( )窓

어엿 그림재 다

리 싀여 여 월( )落月

님 겨신 ( )窓 드시 리

각시님 니 쇼

미 곡- lt gt-

( )

내 님믈 그리 우니다니

(B)산 동새( )山 슷 다

니시 거 신

월 시리 다( ) 殘月曉星

시 도 님

벼 시니 뉘러시니 가

과 도 허믈도 만 업 다( ) ( ) 過 千萬

마리신

니미 마 니 시니 가

님 도 드 샤

과- lt gt-

zb13) 가 의 에 대한 설 으 적절하( ) ~ 않은 것은

① 과 달리 고 다

② 별 신 탓 여 고 다

③ 별 다 가 고 다

④ 산에 는 망 취 지 못 lsquo rsquo

⑤ 실 룰 없는 망 실 시키고 신

다 재 변 고 다

사( ) 5春詞

고 볃티 믉결 다

어 어

그믈 주어 랴 시 가

지 지 어( ) ( ) ( )至匊悤 至匊悤 於思臥

탁 가 니 고 도 니 다( ) ( )濯纓歌 興

사( ) 2夏詞

닙 싸 고 쟝만 마

닫 드러 닫 드러

청 립 사 가 냐( ) ( )靑蒻笠 綠蓑衣

년 학 간고사 대비2013 2 현대고 대비

ECN-0102-2013-001-000076193

지 지 어( ) ( ) ( )至匊悤 至匊悤 於思臥

심 내 가 가( ) ( )無心 白鷗

사( )3秋詞

운 니러 고 티 니 다( )白雲

돋 돋

믈 믈 동 가쟈( ) ( )西湖 東湖ㅣ

지 지 어( ) ( ) ( )至匊悤 至匊悤 於思臥

료 곳마다 경 다( ) ( )白蘋紅蓼 景

동사( )4冬詞

간 눈 갠 후 에 경믈 달 고( ) ( )後 景物

어 어

만경 리 듸 쳔 산( ) ( )萬頃琉璃 千疊玉山

지 지 어( ) ( ) ( )至匊悤 至匊悤 於思臥

계 가 계 가 간 니 다( ) ( ) ( )仙界 佛界 人間ㄴ ㄴ

zb14) 위 을 의 조 형태적 특 에서 어lt gt (1)

떤 차이가 있는 를 서 하고 인해 노래의 아 (2)

름다 에 어떤 향을 주었는 에 대해 서 하

보lt gt

십 경 여 가삼간 지어내니

달 청 맡겨 고

강산 들 곳 없 니 러보고 보리

가 가 각시 본 듯도 고( )

샹 경 엇 니 별 고天 上 白 玉 京 離 別

다 믄 눌 가시 고

어 여 고 내 사 드러 보

내 얼 거동 님 가마

엇 보시고 다 실

도 님 미 업

태 어 러 돗

시 낫 엇 다 신고

누어 생각 고 니러 여 니

내 몸 지 죄 뫼가티 싸 시니

원망 사 허믈 랴

워 니 믈 타시 다造 物

님다 쇼 식 므 쟈 니( ) 消 息

도 거 다 내 사 가

내 마암 둘 대 업다 어드러 가잔 말고

잡거니 거니 놉픈 뫼해 라가니

은 카니 안개난 사 일고

산 쳔이 어둡거니山 川 일日 月 을 엇디

쳑을 라거든 쳔 리랄 라 랴咫 尺 千 里

찰하리 가의 가 히나 자 하니

람이야 결이야 어둥졍 된뎌이고

샤공은 어대 가고 븬 만 걸 나니

강 텬의 혼쟈 셔셔 디난 해랄 니江 天

님다히 쇼 이 더 아득한뎌이고消 息

다 쳠 리 듕만 도 니( ) 茅

벽 쳥 등 눌 갓 고半 壁 靑 燈

라 나리 헤뜨 니니

져근덧 녁 하야力 盡 풋잠을 잠간 드니

졍 셩이 하야 꿈의 님을 니精 誠

가탄 얼 이 이나마 늘거셰라玉 半

마암의 근 말삼 카장 삷쟈 하니

눈 이 라 나니 말인들 어이하

졍을 다하야 이조차 여하니情

뎐된 계 셩의 잠은 엇디 돗던고鷄 聲

라( ) (A)찰하리 여디여 낙 이나 되야 이셔洛 月

님 겨 창 안해 드 비최리라窓

(B)각 님 달이야카니 잔 비나 되쇼셔

zb15) 위 가 라 에 대한 설 으 적절하( )-( ) 않은 것은

① 민들 과 삶 담고 다

② 여 체 식 어 다

③ 사미 곡과 께 가사 미 다

④ 우리말 사가 평가

년 학 간고사 대비2013 2 현대고 대비

ECN-0102-2013-001-000076193

고 다

⑤ 님 미 사 는 마lsquo rsquo

가사 다

가 춘사( ) [ 5]

고은 티 쬐얀 결이 름 다

이어라 이어라

을 주어 두랴 낙 노흘일가

총 총 어( ) ( ) ( )至匊葱 至匊葱 於思臥

탁 가 의 흥 이 나니 고 도 니 다( ) ( )濯纓歌 興

하사[ 2]

년닙희 두고 찬으란 쟝만 마라

닫 드러라 닫 드러라

청약립 은 써 잇노라 녹사의 가져 냐( ) ( )靑蒻笠 綠蓑衣

총 총 어( ) ( ) ( )至匊葱 至匊葱 於思臥

내 좃 가 제 좃 가( ) ( )無心 白鷗

추사[ 3]

이 니러나고 나 티 흐느 다( )白雲

돋 라라 돋 라라

의 셔호 혈 의 동호 가쟈( ) ( )西湖 東湖ㅣ

총 총 어사( ) ( ) ( )至菊悤 至菊悤 於思臥

빈홍 곳마다 경 이 다( ) ( )白蘋紅蓼 景

동사[ 4]

간 의 눈 갠 후 에 경 이 달랃고야( ) ( )後 景物

이어라 이어라

압희 만경류리 듸희 쳔텹 산( ) ( )萬頃琉璃 千疊玉山

총 총 어( ) ( ) ( )至菊悤 至菊悤 於思臥

선계 가 블계 가 인간 이 아니 다( ) ( ) ( )仙界 佛界 人間ㄴ ㄴ

윤선도 어 사 사- ( )漁父四時詞「 」

나 살어리 살어리랏다( ) 쳥산 애 살어리랏다( ) 靑山

위랑 래 랑 고 쳥산 애 살어리랏다 ( ) 靑山

얄리얄리 얄랑셩 얄라리 얄라

러라 러라 새여 자고 니러 러라 새여

널라 름 한 나도 자고 니러 니 라

얄리얄리 얄라셩 얄라리 얄라

청산별곡- ( )靑山別曲

다 청산은 엇뎨 야 만고애 프르르( ) ( ) ( ) 靑山 萬古

유 는 엇뎨하야 주야애 디 아니난고( ) ( )流水 晝夜

리도 치디 마라 만고상청 호리라( ) 萬古常靑

도산 이곡- ( )陶山十二曲

라 개를 여라 이나 르되 개 치 얄 랴( )

님 꼬리를 홰홰 치 치뛰락 나리뛰락 겨서 내

닷고 님 뒷 을 동 동 르락 나으락 캉캉

도리 암캐

이 릇 릇 날 들 너 이 이 랴

작자 상-

마 빈천 을 랴 고 에 드러가니( ) ( ) ( )貧賤 權門

침 업 흥졍을 뉘 져 쟈 리

강산과 풍 을 달나 니 는 리 리

조찬한-

정정 이랬거니 아람도리 큰 솔이 혀( ) ( )伐木丁丁

도 하이 골이 어 아리 소릴 쩌르 돌아 도 하이 다

람 도 좇 않고 새도 않어 은산 고 가 차라리

뼈를 저리 는데 눈과 이 종이 담 회 나 달도 름을

다 흰 뜻은 한 이골을 걸음이란다 절 이 여섯

판에 여섯 고 고 라간 뒤 조찰히 늙은 사나이의 남

내음새를 는가 름은 람도 일 않고 고 에 히

흔들리 노니 견디란다 차고 연 히 픔도 꿈도( )兀然

없이 장 산 속 겨 한 내-

정 장 산- 「 」

zb16) 다 에 나타난 상과 표현을 활 해 작 연 을 해( )

것이다 적절하 않은 것은

① 몰 는 도에 몸 맡 고 는 고통

욱 답게 가꾸 는 느님 산 겠지

② 돌 틈 뚫고 어 민들 보 리 진 고통

다가 도 민들 럼 강 게 살 지 겠는가

③ 벽에 런 움 없 어지고 는 폭포

여 시 는 죽 워 지 는 강

지 는가

④ 상 겨울 도 다리 꿋꿋 살 가고

겠지 망 지 는 삶 얼마 다운가

년 학 간고사 대비2013 2 현대고 대비

ECN-0102-2013-001-000076193

⑤ 막 늘 울리고 는 귀 미 리는

지새우는

가( ) 빈천( )貧賤을 랴 고 에 드러가니( )權門

침 업 흥졍을 뉘 져 쟈 리

강산과 풍 을 달나 니 는 리 리

나 청강 에( ) ( )淸江 비 듯는 소 어 읍

만산 홍 이 휘드르 는고야( )滿山紅綠

두어라 춘풍 이 날이리 을 어라( ) 春風

다 청산은 어찌 야 만고에 푸르르( ) ( ) ( )靑山 萬古

유 어찌 야 주야애 디 아니 고( ) ( )流水 晝夜

리도 치 말아 만고상청 하리라( ) 萬古常靑

라( ) 개를 여라 이나 르되 개 치 얄 랴

님 리를 홰홰 치 치 락 리 락

겨서 내닷고 고 님 뒷 을 동 동

르락 나으락 캉캉 도리 암

이 릇 릇 날 들 너 이 이 랴

zb17) 의 적 능을 비 한 내 으 가장 적절한

것은

① 는 가 지닌 실 계 다

② 는 내 갈등 심 시킨다

③ 는 가 는 상 각시킨( )愛着

④ 는 달리 내 갈등 시킨다

⑤ 는 달리 다 사 들 간 가

강 다

가 님다히 쇼 을 아 나 아쟈 니( ) ( )消息

도 거의 다 일이나 사 가

내 둘 업다 어드러 가쟛말고

잡거니 거니 놉픈 뫼 라가니

은 니 안개 일고

산쳔 이 어둡거니 일 을 엇디( ) ( )山川 日月

쳑 을 거든 쳔 리 라 랴( ) ( )咫尺 千里

하리 의 가 히나 쟈 니

람이야 결이야 어둥졍 된뎌이고

샤공은 어 가고 븬 만 걸 니

강텬 의 혼쟈 셔서 디 니( )江天

( )

쳠 자리의 듕만 도라 니( )茅簷

쳥등 은 눌 위 야 갓 고( )半壁靑燈

리 헤 니니

져근덧 녁 야 픗 을 잠간 드니( )力盡

졍셩 이 야 의 님을 니( )精誠

가 얼 이 이나마 늘거셰라( ) ( )玉 半

의 근 말 장 쟈 니

눈 이 라 나니 말인들 어이

졍 을 다 야 이조차 여 니( )情

뎐된 계셩 의 은 엇디 돗던고( )鷄聲

어 허 다 이 님이 어 간고( )虛事

결의 니러 안자 창 을 열고 라 니( )窓

어엿븐 림재 날 조 이 다

하리 여디여 낙 이나 되야이셔( )落月

님 겨 창 안 드 비최리라( )窓

나 내 님 리자 니다니( )

산 접동새 난 이 이다

아니 거츠르 아으

잔 효성이 아 리이다

넉 라도 님은 녀져라 아으

더 니 뉘러 니잇가

과도 허 도 천만 없소이다

힛마리 뎌

읏븐뎌 아으

니 나 마 니 니 잇가

아소 님하 도람 드르샤 쇼셔

zb18) 맥으 아 에 들어갈 적절한 것은

① 시 엇 다 신고

② 다 믄 눌 보 가시 고

③ 님다 쇼식 욱 득 고( )消息

④ 원망 사 허믈 랴

년 학 간고사 대비2013 2 현대고 대비

ECN-0102-2013-001-000076193

⑤ 죠 뫼 티 시 가( ) ( ) 粥早飯 朝夕

가( )

거리

공신 후 심 늦도( )劉尋

식 없어 과 께 산에 드리고 신

태몽 꾼 에 만고 웅 상 지닌 들

낳 키운다 그 후 신 들 에 역심 ( )逆心

담 귀 등 심 여 리 귀

보내고 지 죽 는 도망 가다

가 만 죽 고 에 경 가는 들 도움

살 다 그러 사 에 심 귀

보고 담 여 고 강 주가 승상

득 여 고 신 사 삼는다 그 후 강 승상

에게 심에 상 리지만 여움

사 귀 가게 다 강 승상 몸 는

연 과 헤어 리 다

경쇠 리 들리 에 들어가니 색

에 게 단청 누각과 큰 집들( )丹靑

다 주 보니 ( ) (一柱門 黃金

산 사 어 었다 산) lsquo rsquo 大字

들어가 고승 다 그( ) ( ) 山門 高僧

거동 보니 눈 눈 듯 고

변 같 귀는 어 에 늘어 니( ) 白邊

맑고 어 골격과 신 평 니었

팔염주를 에 걸고 육환장 을 고서 흑포( )六環杖

장삼에 떨어 송낙 쓰고 나 유생을 고 말[ ] 松蘿

하 를

소 이 연 하여 유상공 는 행차를 동 에 나ldquo

가 맞이하 하 으니 소 의 함을 서하

rdquo

유생이 크게 놀라 말하 다

천한 인생으 팔자가 하여 어 서 를 잃고ldquo

정처없이 다니다가 연히 이곳에 대사를 만난 것인데

토 대하 소생의 성은 어떻게 알고 있 니 rdquo

노 이 답하여 말하 를

어제 남악 형산 의 화선 이 소 의 절에ldquo ( ) ( )男樂 衡山

어서 소 에게 탁하 를 내일 낮 경에 남경 lsquo 12

동성 안에 사는 유 의 아들 충 이가 것이니 내쫓

말고 잘 대접하라 하셨 니다 마침 소 이 찾아 나rsquo

다가 상공의 차람새를 니 남경 사람이 에 알아

았 니다rdquo

유생이 말을 듣고 한편으 쁘고 한편으 퍼하

서 노 을 따라 들어가니 여러 들이 합장 하

가 했다 노 이 에 들어가 저녁 을 은 후에

을 편히 니 이곳은 선경 이었다 세상의 일을( ) 仙境

두 잊고 일 이 편안하 다 이후 는 노 과 함

서 도 이 탐 하고 경도 확하게 의 게 되었( )兵書

다 이 게 되니 대 천 에 가객 은 없 ( ) ( )大明天地 佳客

고 덕산 속에 리 른 만 있더라 래 ( ) 廣德山

이 천상 사람으 살아 있는 처를 만나 이한

을 니 재주 민함을 누가 당할 있겠는가

낮으 공 하더라

유충 전- -

웬늠 어가 사 싸다냐( ) ldquo rdquo

내가 가 막 런거 니

보통것 닐러 그 어낸ldquo ( )

틀어 주 그 가 루 러 허 에

싶어 키 틀어 주 그 가( )

루 허 우간 곡 틀어주는 루 못 는

웂는 고 닝께 고 지 들

어 사는 고 가 다는 건 에 그 집에

rdquo

그런 단 어들 어 새벽에 떼죽 거

다 고 어 보니 죄다 허 게 집어진

는 것 었다

총 가 내화를 꿴 뛰어나 만 아 소 없는

일이었다

어떻게 된 거야ldquo rdquo

한동안 넋나간 듯이 서 있던 총 가 하고많은 사람

에 하필이 유자를 겨냥하 은 말이었다

쎄유 아마 새에 고뿔이 들었던 개비네유ldquo rdquo

유자는 러 딴청을 하 다

야 고 가 에서 감 가 들어 죽는 고 두ldquo

어rdquo

총 는 가 혐의자 나 되는 것처럼 화풀이를( )嫌疑者

하 드는 것이었다

는 비위가 상해서

야 팔자가 사나서 이런 후 에 살라니|

여러 가 다 객고가 쌓여서 조 두 안 좋았을 테 helliphellip

런디다가 릇쓰 이 가락을 트는 대 디립

다 춰댔으니 과 해서 살끼두 다소 있었을 테 helliphellip

래 들어서 키 는 새끼덜일 이 다다 탈이 많은

이니 ldquohelliphellip

는 트의 독성을 충 히 내 않고 고 를 넣

은 것이 탈이었으 니 하 서도 러 참으 의 을 떨

었다 략 - -

마리가 마리 값 간다는 워ldquo

그냥 내뻔지 거시 허 싼 고 는 맛

겄다 싶 허 게 눌 강 어helliphellip

허 마늘 통 다

년 학 간고사 대비2013 2 현대고 대비

ECN-0102-2013-001-000076193

게 지 고뿌 지 rdquo

어 어째ldquo rdquo ldquo rdquo

런 도 것들 같 니ldquo ( ) rdquo殘忍無道 helliphellip

는 탱 여 지 못 다 보( ) 憤氣撐天

니 는 는 다 동원 여 통 쳤

생각 여 는 눈 다

달리 리헐 감ldquo rdquo

들 고 말 니었다 그가

는 것 그 말고는 없었 에 그 게 뒷동

달 거 다

는 우 럽고 식 짝 없는 랫것들 고

다 공연 신 가고 득 것

없다고 단 는지 결 웬만큼 고루 어

그 것들 쪽 에다 고 어주지ldquo

고 그 그걸 주 어 에 에 helliphellip

눈 없는 독 들 rdquohelliphellip helliphellip

고 말 럼 얼거리 들어가 리는 것 었

- ( ) -兪子小傳

zb19) 위 나 를 읽고 평가한 것으 적절하( ) 않은 것

① 사 리 통 감과 사실 고

격 과 달 고

② 는 가 재 컫는lsquo rsquo lsquo rsquo

미 가진 여 는 것 겠

③ 는 식 말 는 웃 상lsquo rsquo

여 는 미 지니고

④ 는 어가 죽 짐 지만 내색 지 고lsquo rsquo

말 고

⑤ 언어 통 가들 여 우리 통

것들 역 고

가 체 거리( ) [ ]

나라 종 연간에 정언주 의 을 하고( )正言注簿

있던 유 은 늦도 자 이 없어 한탄하다가 남악 형산lsquo rsquo

에 치성을 드리고 이한 태 을 꾼 뒤 아들을 낳아 이름

을 충 이라 고 키 다 이때 조정의 하들 에 역

을 품은 정한담 최일 등이 가달의 침입에 대한( ) 逆心

유 의 유화적 입장을 제 삼아 유 을 함하여 양

내고 유 의 에 을 러 충 자마저 살해하

한다 러나 충 은 천 조 정한담의 마 에서 어

나 많은 고난을 겪다가 은퇴한 재상 강희주를 만나 사위

가 된다 강희주는 유 을 하 고 상소를 으나 정

한담의 공격을 아 양을 가게 되고 강희주의 가족은

난을 피하여 두 흩어 다 충 은 강 소저 이 하고

사의 노 을 만나 를 때를 다린다 이

때 남적과 적이 를 들고 나라에 쳐들어 자 정한

담은 자 출전하여 남적에게 항 하고 남적의 선 장이

되어 천자를 공격한다 정한담에게 여러 패한 천자가

항 하 할 음 충 이 등장하여 남적의 선 정 걸

을 죽이고 천자를 출한다 충 은 단 으 란 을

쳐 고 정한담을 사 잡는다 리고 호 에게 ( )胡王

잡혀간 황후 태후 태자를 출하 유 에서 고생하

던 아 유 과 장인 강희주를 한다 또한 이 하

던 어 니 아내를 찾고 정한담 일파를 리친 뒤 높은

에 라서 화를 누린다

사 들 별 고 없 다니었다( )

마 마 돌 다니 걸 여 고

어 곤 다 에는 동쪽에 고

에는 쪽에 니 가 에 리는 엽

가는 없 니 늘 다니는 었

다 얼 말 죽 사 같고 림새가 말

니었다 가슴 에 고 등

삼태 헌 에 니 달 ( )奇男子

가 도리어 걸 었 담 만 열 도 ( )傅說

고 만났고 만 갈( ) ( ) 慇 武丁

도 탕 만났( ) ( ) (伊尹 成湯 渭

여상 도 주 만났는) ( ) ( ) ( ) 水 呂尙 周 文王

월 같 러가 도 어느 열 살

늘과 집 삼고 사 에 쳐 거리에

어 다가 곳에 니 다 ( ) 楚

지 다가 사 보고 가에 다다( )長沙

니 망 가에는 원 리가 슬 고 가

가 내리는 사 에는 갈매 가 갈 뿐 었다

쪽 돌 보니 가 우거 고

가 사 보 었다 그곳에

가니 는 사( ) 汨羅水

는 다 주 가 쓰고 죽고

곳 었다

마 감 여 에 가 사 살펴보니

에는 삼 고 그 에( ) 屈三閭

는 만고 월 과 지 가는 그 들( )風月

가 어 었다( ) 路程記

동쪽 벽 에 새 운 어 거늘 그

보니 월 에 경 주 는 간신에게ldquo ( )敗

보고 연경 귀 가다가 에 죽 rdquo

거늘 그 보고 에 거꾸러

통곡 말

[A]ldquo우리 연경 간 만 니 에

지 살 상에 엇 겠는가

에 고 에 었 니

상에 살 것 가 도 께 지리 rdquo

년 학 간고사 대비2013 2 현대고 대비

ECN-0102-2013-001-000076193

고 가에 내 가니 울 리가 에 지 사

쳤는지 심 심 것 가

신 심 것 가

다( ) 강 승상에게는 들 없고 다만 만

었다 가 낳 에 가 색

타고 내 에게 말 는 ldquo

니다 미원 과 연 맺고 ( ) ( )紫薇垣 緣分

었는 께 강 집 보내 에

니 게 여겨 주십시 거늘 rdquo

미 가운 낳 니 가 고 거동

단 다 시 짓 쓰 고 는 (音

없었 니 여 가운 지 는 짝) 律

룰 만 사 없었다 가 사 여 사 감

게 고 지 못 고 염 는 만다

다가 당에 거 고 식같 러 내니

고귀 상 루 말 다 어 울 도 다( ) 相

귀 사 없고 웅 걸( )富貴爵祿

만고 었다 승상 매우 뻐 내당 ( )內堂

들어가 에게 사 니 역시 매우 거

워 말 다

ldquo 도 마 사 는 승상께

그 게 말 시니 상 여러 말 지 말고

사 도 시다rdquo

상이 에 나 충 의 손을 잡고 결혼과 하여 ldquo

너에게 히 할 말이 있다 내가 늙은 말년에 딸

하나만을 두었는데 니 너 하늘이 정해 필

임이 하다 이제 년고락 을 너에게 탁 ( )百年苦樂

하겠다 하 대 충 이 릎을 꿇고 앉아 눈 을 흘리rdquo

여쭈었다

소자의 을 해주 고 또 하 에 두고자 하ldquo ( )膝下

니 감사하 이를 데가 없 니다 다만 가 속에 통탄

할 일이 사 쳐 있 니다 소자가 이 없어 양친 ( )兩親

의 생사를 른 채 결혼하여 아내를 얻는 것은 자 으

서 할 도리가 아닙니다 이것이 한 러 뿐입니다 rdquo

승상 그 말 듣고 슬 에 어 고

것 에 맞 어 변 게 리ldquo

는 다 집 시 공 도 여 ( )始祖公

고 가 에 가가 어진 만 개 공신

었 니 도 러워 말 시고 시 rdquo

택 여 니 다운 신 과 신

습 늘에 죄 짓고 간 상에 내 신

혼 를 다 끝내고 으 들어가 사 을 살펴 니 빛

나고 빛난 것이 한 입으 는 다 말하 어 고 하나

는 다 하 어 더라 에 켠 환한 촛 ( )新房

아래 은 에 랑과 가 평생의 연 을 맺었( )緣分

으니 서 사랑하 주고 은 말을 어떻게 다 헤아릴

있으 어떻게 다 하리 을 낸 후에 이튿날

상 를 니 상 거 마음을 이 하

더라

각 생 강 승상 집 쪽( )

늘 보고 없 가 신 신 생각 니

없고 어 없었다 는 어떻게 도리가 없다

여 산 에 들어가 리 고 어 도 닦

고 다 그 산 보고 가다

가 곳에 다다 니 에 큰 산 었다 많 우

리 골짜 가 늘 는 가운 색

에 고 갖가지 가 짝 어 었 ( )花草

다 신 산 생각 고 들어가니 경개 ( )景槪

가 매우 뛰어 고 경 산 다 산 리에 들

리는 것 리 보 는 것 울 청산뿐

었다 가 고 울 어 가

니 들 많 가지들 못 어 동

에 늘어 들거리 는( ) 洞口

우거진 가지에 갖 들 다 었다( ) 春情

계상 에는 공 는 늘( ) 花溪上

에 걸린 폭포가 벽 는 리는 산사( )層巖絶壁

쇠 리 객 에 는 듯 늘( ) ( ) 寒山寺 客船

에 싸여 는 습 산

그린 여 병 러 듯 다 경쇠 리가 들

리 에 들어가니 색 에

게 단청 누각과 큰 집들 다( ) 丹靑

주 보니( ) ( ) lsquo一柱門 黃金大字

산 사 어 었다 산 들rsquo ( )山門

어가 고승 다 그 거 보니( ) 高僧

눈 눈 듯 고 변 같 ( )白邊

귀는 어 에 늘어 니 맑고 어 골격

과 신 평 니었다 염주

에 걸고 짚고 포 삼에 어진( )六環仗

쓰고 생 보고 말

승 연 여 상공 시는 동 에ldquo

가 맞 지 못 니 승 십시 rdquo

생 크게 말 다

생 가 여 어 고ldquo

없 다니다가 우연 곳에 사 만 것

그 시 생 어떻게 고 습니 rdquo

승 답 여 말

어 산 승 에ldquo ( ) ( )南岳 衡山

시어 승에게 탁 내 낮 시경에 경 lsquo 12

동 에 사는 심 들 가 것 니 내쫓

지 말고 습니다 마 승rsquo

다가 상공 림새 보니 경 사 에 보

습니다rdquo

zb20) 위 의 친 에서 서 자의 개입이 드러나~

는 이 아닌 것은

① 달 가 도리어 걸 었( ) 奇男子

② 신 심 것 가

년 학 간고사 대비2013 2 현대고 대비

ECN-0102-2013-001-000076193

③ 다운 신 과 신 습 늘에 죄 짓고

간 상에 내 신 다

④ 사 주고 말 어떻게 다 헤 릴

어떻게 다 리

⑤ 신 산 생각 고 들어가니 경개 가 ( )景槪

매우 뛰어 고 경 산 다

거리 연[ ] ( )弘治

간 에 공신 후 에 언(1488~1505) ( )正言

주 는 벼슬 심 늦도 식( ) ( )主簿 劉尋

없어 과 께 산에 드리고 신 태

몽 꾼 에 만고 웅 상 지닌 들 낳

키운다 그 후 신 들 에 역심( )逆心

담 귀 등 심 여 리 귀 보내

고 지 죽 는 도망 가다가

만 다 에 에 어 니

헤어지게 다

에 에 어 니 헤어지게

다 그 후 사 들에게 우연 돌

생 다가 어느 열 살 었다 열 살

지 다가 우연 귀 견 는

그것 그 살 도 었고 그

귀 본 신도 지 죽고 마 고

크게 운다

( )

에는 강 주 는 재상 살고 었

니 시 에 과거에 격 여 승상 벼슬 다가 간

신 만 벼슬 그만 고 고 돌 었

다 그러 신 지 가 지 못 여 상

가 못 결 는 상 여 원 니

신 들 그 직간 꺼 다 그 에 도

담과 귀가 강 승상 가 미워 다

강 승상 마 본 에 갔다가 돌 는[A][ ( )本府

에 우편 주 에 다가 색( ) ( ) 右便 酒店

에 어리었는 청룡 에 지 늘

여 통곡 고 사 는 꿈 꾸었다] 마

상 게 생각 여 새 다리다가 새벽

닭 울고 가 달 갔다 가 보니

과연 어 동 가 가에 울고 는지 달

들어 그 고 사 에 어 말

는 어 어 에 어 가ldquo

닭 곳에 우느냐 니 울rdquo

그 고 답 여 말 다

는 경 동 에 사는 언 주 공 들ldquo

니다 께 간신 만 연경 귀 가

시다가 에 죽 사 에 는 닭에

도 에 죽고 니다rdquo

강 승상 말 듣고 크게 낯 변 말

것 웬 말 냐 근 동 ldquo (老

못 갔 니 그 사 변 여)患

런 변 었단 말 가 주 는 신 다

같 에 벼슬 다가 는 가 많 들어 고

돌 는 주 가 게 꿈 에 생

각 겠느냐 생각지 못 다 미 지 간

지지 말고 께 가 략rdquo ( ) hellip hellip

죽게 주 사당에 단 도 러운

겠느냐 말 말고 시는지 rdquo

어 없어 강 승상 가니 그곳

월계 었다

다( )

가가 고 지 사 들 가( )櫛比

통 는 리가 과

답게 꾸민 누각과 큰 집들 늘 고

게 식 가 어 들 태운 가고

었다 략 강 승상에게는 들 없고 ( ) hellip hellip

다만 만 었다 가 낳 에

가 색 타고 내 에게 말

는 니다 미원 과ldquo ( )紫薇垣

연 맺고 었는 께 강 집( )緣分

보내 에 니 게 여겨 주십시

rdquo

거늘 미 가운 낳 니 가

고 거동 단 다 시 짓 쓰 고

는 없었 니( )音律 여 가운

지 는 짝 룰 만 사 없었다 가 사

여 사 감 게 고 지 못 고 염 는 만다

다가 당에 거 고 식같

러 내니 고귀 상 루 말 다 ( )相

어 울 도 다 귀 사 없 ( )富貴爵祿

고 웅 걸 만고 었다 승상 매우 뻐

내당 들어가 에게 사 니( ) 內堂

역시 매우 거워 말 다 도 마 ldquo

사 는 승상께 그 게 말 시니

상 여러 말 지 말고 사 도 시다rdquo

( )

승상 에 고 결 과 ldquo

여 에게 말 다 내가 늙 말 에 지

만 었는 지 보니 늘

다 에게 탁 겠 ( )

다 신 꿇고 눈 리rdquo

여 었다 주시고 슬 에 ldquo ( )膝下

고 시니 감사 룰 가 없습니다 다만 가슴

에 통탄 사 쳐 습니다 복 없어

생사 결 여 내 얻는 것( )兩親

식 도리가 닙니다 것 러울 뿐 니

다rdquo

상 그 말 듣고 슬 에 어 고 말

것 에 맞 어 웅변 ldquo

년 학 간고사 대비2013 2 현대고 대비

ECN-0102-2013-001-000076193

게 리 는 다 집 시 공도 여

고 가 에 가가 어진 만 개 공신

었 니 도 러워 마 시고 rdquo 시

택 여 니 운 신 과 신

습 늘에 죄 짓고 간 상에 내 신

다 략 지낸 후에 튿 승상 ( ) hellip hellip

니 승상 거운 마 지 못

마( )

듯 월 러 생 열다 살 었

다 에 승상 어진 사 얻고 만 에 근심 없었

다만 주 가 간신 에

죽 것 생각 마 곧 어 곤

다 그 에 주 원통 어

없 고 여 시 가 거늘 생 만

여 다

말 감격 러우 간신 에 가득 여ldquo

고 니 께 상 듣지 니 것

니다rdquo

승상 듣지 고 가

퇴 재상 공달 집에 거 고 상 지어

승지 러 께 리

( )

뒷 거리 강 승상 에게 상 리지[ ]

만 여움 사 귀 가게 다 강 승상

몸 는 연 가 헤어

리 다 산 들어간 룡사 승 만

게 다 승 만 우 다릴

과 들고 략 다 담

원 여 에게 복 고 어

공격 다 담에게 여러 가( ) 天子

복 등 여 다 단

신 리쳐 담 사 고 에게

간 후 태후 태 여 지에 고생

지 심과 강 주 여 개 다 헤

어 어 니 내 고 담 리

벼슬에 귀 누리게 다

zb21) 위 의 인 간 계를 같이 나타냈을lt gt

때 에 대한 이해 가장 적절하 ~ 않은 것은

① 계에 주 는 계 심 열

상 에 다고 다( ) 水深火熱

② 계는 견원지간 고 다( ) 犬猿之間

③ 계는 달리 막역지 계 고( )莫逆之交

④ 연결 사 컬어 재 가 고( )才子佳人

⑤ 는 생 과 볼 ( )匹夫匹婦

가 재 는 는 심 고 매사에 생( )

각 고 능 도 어 가 에게 많lsquo rsquo

도움 사 다 그는 에게 거 에

꺼리 없 거 났다고 는

매우 싫어 고 신 들

는 사 다

내가 지 리에( ) 1970

사 실에 지 월간ldquo

편집 고 어 었다rdquo

어느 없 가 쑥 다 도 어 10

후 다 산 시 럼 어 엇 어 ( ) lsquo怡山

다시 만 랴 니 그는 재 그룹 승 운rsquo

사가 고 는 고 거 누

주는 가 없는 가가 어 다시 만 게 것

었다

다 보통 것 닐러 그( ) ldquo 어낸 ( )

틀어주 가 루 러 허 에

싶어 키 틀어주 그( )

가 루 허 우간 곡 틀어 주는 루 못

는 는 고 닝께 고 지

들어 사는 고 가 다는 건 에 그 집에

rdquo

그런 단 어들 어 새벽에 떼죽 거

다 고 어 보니 죄다 허 게 집어진

는 것 었다 가 실내 꿴 뛰어 지만

없는 었다

어떻게 된 거야 한동안 넋나간 듯이 서 있던 총ldquo rdquo

가 하고많은 사람 에 하필이 유자를 겨냥하 은

말이었다 쎄유 아마 새에 고뿔이 들었던 개비네 ldquo

유rdquo

유자는 러 딴청을 하 다 야 고 가 에서 ldquo

감 가 들어 죽는 고 두 어rdquo 총 는 가 혐의

자 나 되는 것처럼 화풀이를 하 드는 것이었다( )嫌疑者

라 이 어쩌 어 유( ) ldquo rdquo ldquo rdquo

애유 이런 잔인 도 한 것들 같으니ldquo ( ) rdquo殘忍無道 helliphellip

총 는 탱천 하여 쩌 를 하 다( ) 憤氣撐天

아하니 아는 자는 다 동 하여 호통을 쳤으 하나 혈

압을 생각하여 참는 눈치 다 달리 처리헐 두 ldquo

잖은감유rdquo

총 의 성 을 덧들이 고 한 말이 아니었다 가 할

년 학 간고사 대비2013 2 현대고 대비

ECN-0102-2013-001-000076193

있는 것이 말고는 없었 때 에 게 뒷동

산을 달은 거 다

이 유자소전- lsquo rsquo

zb22) 의 상황을 속담으 표현한 것으 적절한 것은

① 루 곳 게 마 다

② 에 맞고 강에 눈 다

③ 늘 도 다

④ 도 사 다

⑤ 에 가도 신만 리 다

거리 공신 후[ ]

에 주 는 벼슬 심 늦도( )主簿

식 없어 과 께 산에 드리고 신

태몽 꾼 에 만고 웅 상 지닌 들

낳 키운다 그 후 신 들 에 역심

담 귀 등 심 여 리 귀 보내고

지 죽 는 도망 간다 그

만 고 에 에 어 니

헤어지게 다 지 가 사 들에

사 들 별 고 없 다니었다

마 마 돌 다니 걸 여 고

어 곤 다 에는 동쪽에 고 에

는 쪽에 니 가 에 리는 엽 가는

없 니 늘 다니는 었다

얼 말 죽 사 같고 림새가 말 니었

다 가슴 에 고 등 삼태

헌 에 니 달 가 도리 ( )奇男子

어 걸 었 담 만 열 도 ( ) ( )傅說 慇

고 만났고 만 갈( ) ( )武丁 伊尹

도 탕 만났( ) ( )成湯 渭水

여상 도 주 만났는 월( ) ( ) ( ) 呂尙 周 文王

같 러가 도 어느 열 살

늘과 집 삼고 사 에 쳐 거리에

어 다가 곳에 니 다 ( ) 楚

지 다가 사 보고 가에 다다( )長沙

니 망 가에는 원 리가 슬 고 가

가 내리는 사 에는 갈매 가 갈 뿐 었다

쪽 돌 보니 가 우거 고

가 사 보 었다 그곳에

가니 는 사( ) 汨羅水

는 다 주 가 쓰고 죽고

곳 었다

마 감 여 에 가 사 살펴보니

에는 삼 고 그 에( ) 屈三閭

는 만고 월 과 지 가는 그 들( )風月

가 어 었다( ) 路程記

동쪽 벽 에 새 운 어 거늘 그

보니

월 에 경 주 는 간신에게ldquo ( )敗

보고 연경 귀 가다가 에 죽 rdquo

거늘 그 보고 에 거꾸러

통곡 말

우리 연경 간 만 니ldquo ( )燕京

에 지 살 상에 엇 겠는

가 에 고 에 었 니

상에 살 것 가 도 께 지리 rdquo

고 가에 내 가니 울 리가 에 지

사 쳤는지 심 심 것 가

에는 강 주 는 재상 살고 었

니 시 에 과거에 격 여 승상 벼슬 다가 간

신 만 벼슬 그만 고 고 돌 었

다 그러 신 지 가 지 못 여 상

가 못 결 는 상 여 원 니

신 들 그 직간 꺼 다 그 에 도

담과 귀가 강 승상 가 미워 다 강 승상 마

본 에 갔다가 돌 는 에 우편 주( ) ( )本府 右便

에 다가 색 에 어리었는 청룡( ) 酒店

에 지 늘 여 통곡 고

사 는 꿈 꾸었다 마 상 게 생

각 여 새 다리다가 새벽닭 울고

가 달 갔다 가 보니 과연 어 동 가

가에 울고 는지 달 들어 그

고 사 에 어 말

는 어 어 에 어 가ldquo

닭 곳에 우느냐rdquo

니 울 그 고 답 여 말 다

는 경 동 에 사는 언 주 공 들ldquo

니다 께 간신 만 연경 귀 가

시다가 에 죽 사 에 는 닭에

도 에 죽고 니다rdquo

강 승상 말 듣고 크게 낯 변 말

것 웬 말 냐 근 동ldquo ( )老患

못 갔 니 그 사 변 여 런 변

었단 말 가 주 는 신 다 같

에 벼슬 다가 는 가 많 들어 고 돌

는 주 가 게 꿈 에 생각

겠느냐 생각지 못 다 미 지 간 지지

말고 께 가 rdquo

뒷 거리 강 승상 도움 죽 고[ ]

년 학 간고사 대비2013 2 현대고 대비

ECN-0102-2013-001-000076193

고 그 과 결 여 사 가 다 그러 강

승상 에게 울린 상 강 승상 귀 가고

과 헤어 리 승 만 게 다

승 우 다릴 과

들고 략 다 담 원

여 에게 복 고 어 (天

공격 다 담에게 여러 가 복) 子

등 여 다 단신

리쳐 담 사 고 에게 간

후 태후 태 여 지에 고생 지

심과 강 주 여 개 다 헤어

어 니 내 고 담 리 벼

슬에 귀 누리게 다

미상- lsquo ( )-劉忠烈傳

zb23) 위 과 의 서사 조를 비 한 것으 적절하lt gt

않은 것은

보lt gt

믿지 고 결 여 곱

낳 다 곱째 공주 낳 가

리게 다 리 만 고 진 공주는 lsquo rsquo

리공 미 리공 에 키워진다 월

러 과 가 죽 병에 걸 는 승에 는

어 산다고 다 여 들에게 탁

지만 거 리 는다 리 는 과

승 다 승 지 가는 에 많

만 지만 보살 도움 사 도 다

그러 승 신과 결 여 시

들어 주겠다고 다 리 는 그 결

여 들 곱 낳 후에 신

얻게 다 돌 리 는

에 과 상여 만 지만 여 과

살 낸다 훗 리 그 공 우 죽 사

승 도 는 신 다

리-lsquo rsquo-

① 복 결말에 고 다

② 웅 에 탕 고 다

③ 시 겨 내고 귀 누리는lsquo rsquo

보 리 는 월 재 신 다lt gt lsquo rsquo

④ 과 보 리 는lsquo rsquo lt gt lsquo rsquo

도움과 어 신 능 극복 고

⑤ 등 여 시 겪는lsquo rsquo

보 리 는 닌 지lt gt lsquo rsquo

림 시 겪는다

가 각 고 에( ) ( )却說

살 없었다 략 사 들 슬 에 어 lt gt

가에 내 고 가고 싶 가 고 후

워 경

사 들 별 고 없 다니었다 lt

략 얼 말 죽 사 같고 림새가 말gt

니었다 가슴 에 고 등

삼태 헌 에 니 달 가 ( )奇男子

도리어 걸 었 담 만 열 도( )傅說

고 만났고 만 갈( ) ( ) 殷 武丁

도 탕 만났( ) ( ) (伊尹 成湯 渭

여상 도 주 만났는) ( ) ( ) ( )水 呂尙 周 文王

월 같 러가 도 어느 열 살

늘과 집 삼고 사 에 쳐 거리에

어 다가 곳에 니 다 ( ) 楚

지 다가 사 보고 가에 다다( )長沙

니 망 가에는 원 리가 슬 고 가

가 내리는 사 에는 갈매 가 갈 뿐 었다

쪽 돌 보니 가 우거 고

가 사 보 었다 그곳에

가니 는 사( ) 汨羅水

는 다 주 가 쓰고 죽고

곳 었다

에는 강 주 는 재상 살고( )

었 니 시 에 과거에 격 여 승상 벼슬 다

가 간신 만 벼슬 그만 고 고 돌

었다 략 강 승상 마 본 에 갔다가 돌 lt gt ( )本府

는 에 우편 주 에 다가 색( ) ( ) 右便 酒店

에 어리었는 청룡 에 지

늘 여 통곡 고 사 는 꿈 꾸

었다 마 상 게 생각 여 새 다리다

가 새벽닭 울고 달 갔다 가

보니 과연 어 동 가 가에 울고 는지

달 들어 그 고 사 에

어 말

는 어 어 에 어 가ldquo

닭 곳에 우느냐rdquo

니 울 그 고 답 여 말 다 lt

략gt

년 학 간고사 대비2013 2 현대고 대비

ECN-0102-2013-001-000076193

생각 여 가 고 시 는ldquo ( )大人

상에 다시없는 니다 살 엇 겠습니

에 돌 가시고

가에 돌 가 니 살 마 없습니

다 략 어 없어 강 승상 가니rdquo lt gt

그곳 월계 었다

다 강 승상에게는 들 없고 다만 만( )

었다 가 낳 에 가 색

타고 내 에게 말

는 니다 미원 과ldquo ( )紫微垣

연 맺고 었는 께 강 집( )緣分

보내 에 니 게 여겨 주십시

rdquo

거늘 미 가운 낳 니 가

고 거동 단 다 시 짓 쓰 고

는 없었 니 여 가운( ) 音律

지 는 짝 룰 만 사 없었다 가 사

여 사 감 게 고 지 못 고 염 는 만다

다가 당에 거 고 식같 러

내니 고귀 상 루 말 다 어 ( )相

울 도 다 귀 사 없고 ( )富貴爵祿

웅 걸 만고 었다 승상 매우 뻐 내

당 들어가 에게 사 니 역( ) 內堂

시 매우 거워 말 다

도 마 사 는 승상께ldquo

그 게 말 시니 상 여러 말 지 말고 사

도 시다 략 시 택 여rdquo lt gt

니 다운 신 과 신 습 늘에 죄

짓고 간 상에 내 신 다

다 내고 들어가 사 살펴보니

고 것 는 다 말 어 고

는 다 어 신 에 ( )新房

에 신 과 신 가 평생 연 맺었( )緣分

니 사 주고 말 어떻게 다 헤 릴

어떻게 다 리 지낸 후에 튿 승

상 니 승상 거운 마 지 못

( ) 듯 월 러 생 열다 살

었다 에 승상 어진 사 얻고 만 에 근심

없었 다만 주 가 간신

에 죽 것 생각 마 곧 어

곤 다 그 에 주 원통

어 없 고 여 시 가 거늘 략 lt gt

략 거리

강 승상 에게 상 리지만 여움

사 귀 가게 다 강 승상 몸 는

연 과 헤어 리 다

마 각 생 강 승상 집 쪽( )

늘 보고 없 가 신 신 생각 니

없고 어 없었다 는 어떻게 도리가 없다

여 산 에 들어가 리 고 어 도 닦

고 다 그 산 보고 가

다가 곳에 다다 니 에 큰 산 었다 많

우리 골짜 가 늘 는 가운 색

에 고 갖가지 가 짝 어 ( )花草

었다 략 주 보니 lt gt ( ) (一柱門 黃

산 룡사 어 었다) lsquo rsquo 金大字

산 들어가 고승 다 그( ) ( ) 山門 高僧

거동 보니 눈 눈 듯 고

변 같 귀는 어 에 늘어 니( ) 白邊

맑고 어 골격과 신 평 니었

다 염주 에 걸고 짚고 포 ( )六環杖

삼에 어진 쓰고 생 보고 말

승 연 여 상공 시는 동 에ldquo

가 맞 지 못 니 승 십시 rdquo

생 크게 말 다

생 가 여 어 고ldquo

없 다니다가 우연 곳에 사 만 것

그 시 생 어떻게 고 습니

rdquo

승 답 여 말

어 산 승 에ldquo ( ) ( )南岳 衡山

시어 승에게 탁 내 낮 시경에 경 lsquo 12

동 에 사는 심 들 가 것 니 내쫓

지 말고 습니다 마 승rsquo

다가 상공 림새 보니 경 사 에 보

습니다rdquo

생 그 말 듣고 편 고 편( )

슬 승 들어가니 여러 승 들

가워 다 승 에 들어가

후에 그 편 니 곳 경 었다 상( ) 仙境

고 신 편 다 후 는 승과

께 병 도 탐 고 경도 게( )兵書

게 었다 게 니 지 에 가객 ( ) ( )大明天地 佳客

년 학 간고사 대비2013 2 현대고 대비

ECN-0102-2013-001-000076193

없고 산 에 리 만 본 ( ) 廣德山

신 상 사 살 는 만

우고 늘 월 신 과 늘 ( )日月聖神

산 신 들 다 니 그 재( ) 名山神靈

주 민 누가 당 겠는가 낮 공

zb24) 다 에 해당하는 내 으 적절하( ) 않은 것은

① 강 티 통 당시 능 다

② 상계 지상계 경 는 원 계 드러

③ 실에 어 없는 실 가 타 는

④ 뛰어 재주 어 가진 고

등 다

⑤ 가 직 개 여 평가 내리는

편집 평 타 다lsquo rsquo

가 본격 가 동 것 지( )

다 단 상 에2003 lsquo rsquo

들어가 드럼 연주 다 취미 생 달리

들었다는 보 우 가 들ldquo

어 틱 린 도 다 고 말 다rdquo

경 는 가 망 없( ) lsquo

티 원 고 답 다 신과 같 시각rsquo

는 습 상상 만 도 감동

다 시각 연주 동시에

열 상 는

티 원 그런 열 경 럽다는 것 다

다 역시 엄청 다 본( )

에 복 들

고쳐 가고 다 신 에 얼

마 지는 고 리가 는 지도 생님

가 훈 고 많 고쳐 다

고 말 다

그러 직도 에 지 는 다 그는

체격 지 못 게 가 큰 만

체 운동 훈 과 께 체 늘 동 50

는 게 고 말 다

에게는 꿈 다 통 누 가( )

주겠다는 것 그 꿈 다 신 극복 는

과 에 큰 경험 들도 느 게

주고 싶다는 것 다

마 슬 마다( ) ldquo 통

낼 었 것 럼 고통 는 사 들

고 겠다 고rdquo

말 다 달 루 첫 낸 lsquo rsquo

첫 드 심 집에 는 리듬 드 2

루 에 도 보고 싶다 집 에는 직(RampB) 3 4

사 곡 도 보 고 싶다고 포 다middot

zb25) 에서 가장 유사한 의 를 닌 어를lt gt

찾아 쓰

lt gt

나는 이제 너에게도 픔을 주겠다

사랑 다 소 한 픔을 주겠다

겨 거리에서 개 놓고

살아 추위 떨고 있는 할 니에게

값을 으 서 뻐하던 너를 위하여

나는 픔의 평등한 얼 을 여 주겠다

내가 어둠 속에서 너를 를 때

단 한 도 평등하게 어주 않은

가마니에 덮인 동사자가

다 얼어 죽을 때

가마니 한 장조차 덮어주 않은

한 너의 사랑을 위해

흘릴 르는 너의 눈 을 위해

나는 너에게 이제 너에게도 다림을 주겠다

지 울 포동 여고 생들17

틈 없 가득 체 에 맑 울

다 죽 듣 생들 사 에

연 는 탄 다 객들 도 는lsquo rsquo

가 보 주 공 맹 가 운 는

단 그룹사운드 루 보컬 맡고 는lsquo rsquo

시각 지 었다17 1

근 다만과 가 거lsquo rsquo lsquo

꿈 고 퇴 내가 다rsquo

간 간에 지 지 연 생들 짧lsquo rsquo lsquo rsquo

가 운 듯 리에 어

연 다 내 사 고 퇴lsquo rsquo

과 루 들 결 다시 돌lsquo rsquo

들 고 사 들 에 당당

것 니다 내 태어

볼 없었 크고 열여

년 학 간고사 대비2013 2 현대고 대비

ECN-0102-2013-001-000076193

에도 고 시 얻지 못 다

감지 없는 시각 상태 다

신 지에 고 상 원망 도

단다 어느 가 에 시각 에 ldquo

어 그런 듣고 다 보니 내가 게 lsquo

살 는지 도 눈 고 싶rsquo lsquohelliphellip

보 는 생각만 들 고 그 가 들에게rsquo

도 내고 들도 고 많 었죠 들 rdquo

었 지 새 는 에 쑥 러운 색

어났다

생에 것 단연 었다lsquo rsquo

공연에 거 꿈lsquo rsquo

는 다 특 가사 갑게 는 운 lsquo

벽 에 당당 마주 어 언 가 그 벽

고 늘 어 거운 상도

없죠 내 삶 에 웃 그 께

는 다고 다rsquo

들었 그냥 런 도 고만 여ldquo lsquo rsquo

겼죠 그런 꾸 가사 미 새 다 보

니 통 는 가사 는 생각 들 고 (

가 게는 시각 는 생각 들고 들) ( )

마다 듣고 큰 얻었어 rdquo

에 진지 게 가에 미 가

zb26) 의 에 들어갈 말 적절한 것은lt gt ~

lt gt

난 난 꿈이 있었죠

고 찢겨 남 하여도

내 가 히 과 같이 간 했던 꿈

혹 때 누 가가 뜻 를 비 음

내 등 뒤에 흘릴 때도

난 참아야 했죠 참을 있었죠

날을 위해

늘 걱정하듯 말하죠

헛된 꿈은 독이라고

세상은 끝이 정해 책처럼

이 돌이킬 없는

현 이라고 helliphellip

래 난 난 꿈이 있어

꿈을 믿어

나를 켜

저 차갑게 서 있는 이란 앞에

당당히 마주칠 있어

출처 가 거위의 꿈 작사 이적 작곡 동률- lsquo rsquo ( )

① ② ③ ④ ⑤

가 떴다는 들 만 지만( ) lsquo rsquo

늘 겸 다 에 주 연 우승 지 간에도 3

단 생님께 만 지 고 고 만ldquo rdquo

큼 늘 겸 신 계 가

고 다

에게는 꿈 다 통 누 가

주겠다는 것 그 꿈 다 신 극복 는 과

에 큰 경험 들도 느 게 주

고 싶다는 것 다

슬 마다 통 낼ldquo

었 것 럼 고통 는 사 들

고 겠다 고rdquo

말 다 달 루 첫 낸lsquo rsquo

첫 드 심 집에 는 리듬 2

루 에 도 보고 싶다(RampB) 집 에는 직34

사 곡 도 보 고 싶다고 포 다

미 는( ) (26) 어 헤헤헤 웃다가 어ldquo rdquo

허허허 웃었다ldquo rdquo ldquo rdquo 같 도 고

상 다 는 같 도 다( ) 壯丁 킹 들lsquo

다 는 역도 보 그 다 지만 그는rsquo

뷰에 지 다 운동만 지 ldquo

것 지 간에 여러 사 도 역rdquo helliphellip

었다 그런 엇 그 마 움직 는지 보 쯤

지 담 사 다 훈 없어 그는 티

지 림 었다 태 다 갔다 는 습

마 집 럼 편 게 보 다

주말에는 주 엇 보내

주말에도 별 주 에 청ldquo

고 에 가고 도 쳐

에 듣고 보 에 갈 가 별 없

어 산 시 게 고 들어 2002

거 매 여 지냅니다 시 과 지훈 rdquo

다 근 간 과 진실 그리고 싶어( )

가 다 근에게 그것 진리 다 거 다 없

거 고 다 없 는 것 진리

다 근 진리는 후 쪽 었다 신산( )辛酸 삶

었 질곡( )桎梏 역사 에 지냈 가

눈에 든 것 료 단 료 게 보

것 었다 그것 그 에 겨우겨우

슬 슬 생 어가는 간들 었다

리 과 단 리 고리에 검 마

없 거리 돌

상 것 없는 등 근에게 상

과 진실 엄 ( )儼存 다는 사실 리는 가

실 고 가 과 역경 에 도 근 내 포

없었 후 보루( )堡壘 다 도 365

도 간 근 여

시 것 다

년 학 간고사 대비2013 2 현대고 대비

ECN-0102-2013-001-000076193

다 공주 그림 가 근 경- ( ) ldquo rdquo(

2009)

zb27) 작가의 주 적인 각이 드러난 것은~

① ② ③ ④ ⑤

가 신 지에 고 상 원망( )

도 단다 어느 가 에 시각 에 ldquo

어 그런 듣고 다 보니 내가 lsquo

게 살 는지 도 눈 고 싶rsquo lsquohelliphellip

보 는 생각만 들 고 그 가 들에게rsquo

도 내고 들도 고 많 었죠 들었rdquo

지 새 는 에 쑥쓰러운 색

어났다 략 [ ]

경 는 가 망 없 티lsquo

원 고 답 다 신과 같 시각rsquo

는 습 상상 만 도 감동

다 시각 연주 동시에

열 상 는 티

원 그런 열 경 럽다는 것 다 략 [ ]

슬 마다 통 낼ldquo

었 것 럼 고통 는 사 들

고 겠다 고rdquo

말 다 달 루 첫 낸 lsquo rsquo

첫 드 심 집에 는 리듬 2

루 에 도 보고 싶다 집 에는 직(RampB) 3 4

사 곡 도 보 고 싶다고 포 다

식 누 가-

고 싶어

다 역도 미 담 고 사( )

질 주말에는 주 엇 보내[ 1]

답 주말에도 별 주 에[ ] ldquo

청 고 에 가고 도 쳐

에 듣고 보 에 갈 가 별

없어 rdquo

질 계 고 슬슬 도 는 것 닙니[ 2]

답 다 들 눈 에 보 고 뿐 보[ ] ldquo

다 열심 고 어 상에 도 들지만 상

지키는 것 들다고 에 도달

그것 지키 훨 많 rdquo

질 들 살 고 리 는[ 3]

거운 들 체 리느 는다

답 가 고 게 체 어[ ] ldquo ( ) 級

느 도 계가 니 살 는 것도 고역 지만

살 우는 것 들어 는 살

체 리 고 어도 어도 실 갔다

쑥 어 rdquo

질 거리에 슷 연 여 들[ 4]

보는 간 상 지

답 상 다 체 게 리지 못[ ] ldquo

거 주변에 는 그 거 누 보지

못 고 뻐지고 싶 에 체 리는 에

타 워 지만 는 어울 는 것보다 는

시간 운동만 는 건 니에 사복 lsquo rsquo

고 사복 는 말에 들 웃지만 늘 운동복

고 지내니 사러 갈 도 어 rdquo

질 역도가 말 단 식 운동 니[ 5]

답 가 내는 만 클 업 보[ ] ldquo

그러니 만 쓰는 식 운동 니다

만 다고 거운 것 들 는 건 니거든 연

도 고 가지 동 에 도 여러 가지

복 들

보식 역도 여 미-

zb28) 가 에 대한 설 으( ) 않은 것은

① 시각 우 지 시 에 지

고 망 가는 태도 달 고 다

② 언어 과 언어 복 사 여

담 내 생각 게 는 가

③ 직 감 그 마 것

럼 생생 게 느껴지는 과 주고 간 내

없 리 어 억 게 다

④ 담 내 식 리 여 담 삶 습

과 가 시 여 독 에게 감동과 훈 다

⑤ 직 진 담 직 누

지 못 는 독 에게 생생 상 달 주고

담 욱 게 다

zb29) 나 의 각 의 의도를 설 한 것으 적절하( ) 않

년 학 간고사 대비2013 2 현대고 대비

ECN-0102-2013-001-000076193

은 것은

① 질 담 상 보여 주 것 다1

② 질 담 과 그에 삶 태도 보여2

주 것 다

③ 질 역도 겪는 어 움에 역도3

과 것 다

④ 질 같 연 여 갖는 고민 는지 말4

주 는 것 다

⑤ 질 역도가 과 고 운동 는 것5

담 가 말 주 는 것 다

가 만진 것 다( ) 3

감 달 다고 다 억 에( ) 音感

지워 지만 당시 청 탁 리도

다고 다 드럼 웠다 4

에 갈 마다 드럼 는 리가 신 게 들

다고 다 눈 볼 가 없 니 엔ldquo

는 는 님 틱 에 여 주

다 드럼과 연 맺 과 들 주었다rdquo

식 누 가-

고 싶어

역( ) 도가 말 단 식 운동 니

가 내는 만 클 업에 보ldquo

그러니 만 쓰는 식 운동 니다 만

다고 거운 것 들 는 건 니거든 연

도 고 가지 동 에 도 여러 가지 복

들 시 는 상 상

드는 상 에 맞춰 실 에 는 여러

펼쳐집니다rdquo

략( )

늘 에 는 어 만 것 같

가 에 사 고 사 사ldquo

겠어 든 에 가 경 만 고

울 는 사 겠어 rdquo

보식 역도 여 미-

다 가 운 는 어 어( ) ldquo rdquohelliphellip

월 새벽 시 태 없 거웠고1965 5 6 1

는 없 그 병원에 퇴원 집

가는 마지막 마 고 마 내 거 다

가 죽 간신 에 실 다 사는 어느5 lsquo

가 죽 는 말 가 식 다 신rsquo

상에 각 시키는( )刻印 에 실

어느 가는 후 민 가가 근 었다lsquo rsquo

는 간 과 진실 그 다는( ) ldquo

에 단 평 견 가지고 다 내

가 그리는 간상 단 고 다 지 다 는 그들

가 에 는 평 지 니 그리고 어린

들 미지 겨 그린다rdquo

마 근 간 과 진실 그리고 싶어( )

가 다 근에게 그것 진리 다 거 다 없

거 고 다 없 는 것 진리

다 근 진리는 후 쪽 었다 신산(辛酸 삶)

었 질곡(桎梏 역사 에 지냈)

가 눈에 든 것 료 단 료 게

보 것 었다 그것 그 에 겨우겨우

슬 슬 생 어가는 간들 었

다 리 과 단 리 고리에 검

마 없 거리 돌

상 것 없는 등 근에게 상에

과 진실 엄 다는 사실 리는 가( )儼存

실 고 가 과 역경 에 도 근 내

포 없었 후 보루(堡壘 다 도)

도 간 근365

여 시 것 다

월 강원도 림리에( ) 1914 2 21

삼 독 태어났다 어 근 복

그것 그리 가지 못 다 근 곱 살

지는 산 산업에 실 고 답마 에 내

갔다 근 그림 럼 쫓 다니 가 시 것

다 상 진 것도 가 었다

러 가 에도 고 근 가 꿈꾸었다 근

가 꿈꾸게 것 보통 업

원색도1926 만lsquo rsquo 었다

공주 그림 가 근 경-

zb30) 에 대한 설 가장 른 것은~

① 역도가 과 운동 도 질

② 리는 는 다 lsquo rsquo

③ 들었지만 그럭 럭 는 다 lsquo rsquo

④ 가 게 보 시 말 다

⑤ 보 병 는 지 상 lsquo rsquo

는 말 다

년 학 간고사 대비2013 2 현대고 대비

ECN-0102-2013-001-000076193

시간 많지 다 청량리 생 병원

마지막 상 경 릿 게 들어 다 그 는 십

만 큰 가 상 말 다

지 못 들 마 갈 고 돗

도시민들 싹 싹 탔다 가 시

월에 병원에 원 가 폐 진 몸도4 ( )疲弊

갈 미 지 못 고 었다 가는 얼마( ) 解渴

지 생 에 생각 가

마감 는 신 평생 십 만에

가 과 많 닮 다고 생각 지는

가 운 는 어 어ldquo rdquo 1965helliphellip

월 새벽 시 태 없 거웠고 는5 6 1

없 그 병원에 퇴원 집 가

는 마지막 마 고 마 내 거 다 가

죽 간신 에 실 다 사는 어느 가5 lsquo

죽 는 말 가 식 다 신rsquo

상에 각 시키는 에 실 어느( ) lsquo刻印

가는 후 민 가가 근 었다rsquo

ldquo 는 간 과 진실 그 다는 에

단 평 견 가지고 다 내가 그

리는 간상 단 고 다 지 다 는 그들 가

에 는 평 지 니 그리고 어린 들

미지 겨 그린다rdquo

근 간 과 진실 그리고 싶어 가

다 근에게 그것 진리 다 거 다 없 거

고 다 없 는 것 진리다

근 진리는 후 쪽 었다 신산 삶 ( )辛酸

었 질곡 역사 에 지냈 가 눈에( )桎梏

든 것 료 단 료 게 보 것

었다 그것 그 에 겨우겨우 슬

슬 생 어가는 간들 었다 리

과 단 리 고리에 검 마

없 거리 돌 상

것 없는 등 근에게 상에 과 진실

엄 다는 사실 리는 가 실( )儼存

고 가 과 역경 에 도 근 내 포 없었

후 보루 다 도 도( ) 365堡壘

간 근 여 시 것

간에 지닌 가 근 1914 2

월 강원도 림리에 삼 독21

태어났다 어 근 복 그것 그리

가지 못 다 근 곱 살 지는 산

사업에 실 고 답마 에 내 갔다 근

그림 럼 쫓 다니 가 시 것 다 상

진 것도 가 었다 러 가 에도

고 근 가 꿈꾸었다 근 가 꿈꾸게

것 보통 업 원색1926

도 만 었다lsquo rsquo

그림 가 근 경 공주- ldquo rdquo ( 2009)

zb31) 다음 이 같은 의 성 소에 해당하 않은

것은

사건 평① ② ③

④ 주 ⑤ 경

가 운 는 어 어ldquo rdquo 1965helliphellip

월 새벽 시 태 없 거웠고 는5 6 1

없 그 병원에 퇴원 집 가

는 마지막 마 고 마 내 거 다 가

죽 간신 에 실 다 사는 어느 가5 lsquo

죽 는 말 가 식 다 신rsquo

상에 각 시키는 에 실 어느( ) lsquo刻印

가는 후 민 가가 근 었다rsquo

는 간 과 진실 그 다는 에ldquo

단 평 견 가지고 다 내가 그

리는 간상 단 고 다 지 다 는 그들 가

에 는 평 지 니 그리고 어린 들

미지 겨 그린다rdquo

근 간 과 진실 그리고 싶어 가

다 근에게 그것 진리 다 거 다 없 거

고 다 없 는 것 진리다

근 진리는 후 쪽 었다 신산 삶 ( )辛酸

었 질곡 역사 에 지냈 가 눈에( )桎梏

든 것 료 단 료 게 보 것

었다 그것 그 에 겨우겨우 슬

슬 생 어가는 간들 었다 리

과 단 리 고리에 검 마

없 거리 돌 상

것 없는 등 근에게 상에 과 진실

엄 다는 사실 리는 가 실( )儼存

고 가 과 역경 에 도 근 내 포 없었

후 보루 다 도 도( ) 365堡壘

간 근 여 시 것

간에 지닌 가 근 1914 2

월 강원도 림리에 삼 독21

태어났다 어 근 복 그것 그리

가지 못 다 근 곱 살 지는 산

사업에 실 고 답마 에 내 갔다 근

그림 럼 쫓 다니 가 시 것 다 상

진 것도 가 었다 러 가 에도

고 근 가 꿈꾸었다 근 가 꿈꾸게

것 보통 업 원색1926

도 만 었다lsquo rsquo

공주 그림 가 근 경- ldquo rdquo ( 2009)

년 학 간고사 대비2013 2 현대고 대비

ECN-0102-2013-001-000076193

zb32) 위 을 작성하는 과정에서 되어 활 된 자

어 것은

신 사 료① 연보②

고③ ④ 들과 담

⑤ 에 평

는 간 과 진실 그 다는 에ldquo

단 평 견 가지고 다 내가 그

리는 간상 단 고 다 지 다 는 그들 가

에 는 평 지 니 그리고 어린 들

미지 겨 그린다rdquo

근 간 과 진실 그리고 싶어 가

다 근에게 그것 진리 다 거 다 없 거

고 다 없 는 것 진리다

근 진리는 후 쪽 었다 신산 삶 ( )辛酸

었 질곡 역사 에 지냈 가( )桎梏

눈에 든 것 료 단 료 게 보

것 었다 그것 그 에 겨우겨우

슬 슬 생 어가는 간들 었다

리 과 단 리 고리에 검 마

없 거리 돌 상

것 없는 등 근에게 상에 과

진실 엄 다는 사실 리는 가 실( )儼存

고 가 과 역경 에 도 근 내 포

없었 후 보루 다 도 도( ) 365堡壘

간 근 여 시

것 다

간에 지닌 가 근 1914 2

월 강원도 림리에 삼 독21

태어났다 어 근 복 그것 그리

가지 못 다 근 곱 살 지는 산

사업에 실 고 답마 에 내 갔다 근

그림 럼 쫓 다니 가 시 것 다 상

진 것도 가 었다 러 가 에도

고 근 가 꿈꾸었다 근 가 꿈꾸게

것 보통 업 원색1926

도 만 었다lsquo rsquo

질 루 마 가 도 린다 경건

움 느껴지는 경 다 훗 근 그림에

과 는 거 것( )裸木

만 간과 연 엮어 가는 경건 움lsquo rsquo

니었

같 가가 고 싶었 근에게 그 꿈에 다

가가는 지 다 다 가 지망생들 규 미

상 에 진 고

에 지만 근 다 다 근

미 에 운 것 보통 시 미 시간

다 그런 그에게 없는 연습 가가

통 다 가 귀 시 지 도

얻는 뛸 듯 뻤지만 마 도 가 에

듯 는 었 에 어린 근 주 에

에 그림 그리고 지우고 복( )粉板

시간 가는 게 루 보냈다

근 그 갈 가가 것 열여( )渴求

었 다가 미1932 lsquo rsquo ( lsquo

미 에 다 다는 고 마rsquo) lsquo rsquo

가 근 집 고도 지는 시골 경

그린 그림 다 후 근 에 1943 22

지 미 에 그림 고

에 걸쳐 다 미 근 가

동 는 었다

공주 그림 가 근 경- ldquo rdquo ( 2009)

zb33) 위 의 내 과 일치하는 것은

가 근 가 꿈 포 다①

근 당 가들과 께 에 다②

살 근 가 걷20③

게 었다

④ 만 통 근 역경 겨내는lsquo rsquo

느 다

⑤ 근 간 과 진실 그리 에 그 에

드러 는 간상 단 다

계 시 주 근 건강

걸었다 신 과 간에 상 다 건강

신 는 눈에도 다 근 쪽 눈 뿌 게

보 지 과에 다 다 시 지지 고 결

내 었다 시 지만 마 막막

다 늦어 결 근 쪽 눈 고 말 다

쪽 눈 근에게는 쪽 눈 었고

계 었다 그 근 는 여 그lsquo rsquo

다 근 에 같 그림 그 었다1950

시 그림 는 여 쪽lsquo rsquo

고 어 마주 고 는 그림1963

여 과 동 다 마 복

그린 듯 눈 내리 새 게 다 지

사 다 근 게 복 것

복 상과 타 는 근 상

가 떳떳 단 었고 근 그리고

간 과 진실 에 다가가 가 근다

운 었다 근 신에게 당당 지 그리고

그 다 근 그림에 단 복 보다

년 학 간고사 대비2013 2 현대고 대비

ECN-0102-2013-001-000076193

태 도 그리고 극 보다 과

얻 여 었다 과 통

근 그리고 는 재 고 에 질

만들고 특 것 다

공주 그림 가 근 경- ldquo rdquo( 2009)

zb34) 의 이유에 대해 추 한 것으 적절하 않은 것

상과 타 시도①

보다 과 얻②

근 신에게 당당 지③

④ 간 과 진실 에 다가

⑤ 태 도 얻

근 가가 었지만 그 다니 가

럼 어지지 다 복과 쟁 거쳐 시

는 가 근에게 생계 사 에

운 사 다 에 키에 건( ) 178cm死鬪

체 근 에 동 역 업( )荷役

가 생계 다 쟁

에는 동에 운 상우 주 미

죄 사 에 그림 그리는 시 다 그곳에

에 동 역 업 것에

결 것 럼 보 다 지만 그런 것만도

니었다 그림 그리는 고는 지만 매 근

는 극 간 과 별 없는 경 리 그림

벽에 그리는 것 었다 우도 리 없었다 근

트 는 우 그림 그 다 생

계 그림 단 것 다

후 근 지 신 계 리에 미

엑 리 겼다 근 곳에

건 사 크 에 미 들 ( )

상 상 그 다 근 갖 다 겪

냈다 그리고 결 그 돈

신동에 어 사리 집 마 다 마 ㄷ

루 심 쪽에는 과 엌 쪽에는 건

었다 건 주고 근 가 에

여 살 다 심 에는 지 집어

쓰고 지만 곳 근 가 에게 러웠

보 리 다 근 과 마루 업실 삼 그림

그 다 신동 마루는 근 그림에 등 는 lsquo rsquo

같 상들 지 다 시 고

에 들 폐허가

가 업실 었다

공주 그림 가 근 경- ldquo rdquo( 2009)

zb35) 위 에 대한 설 으 적절한 것은

업 시 여 훈과 감동 다①

에 주 평 드러 다②

사 사 등 식 과 ③

④ 다 근거 시 여 삶에

⑤ 살 시 사 경 께 여

습 시 다

가 시간 많지 다 청량리 생 병원( )

마지막 상 경 릿 게 들어 다 그 는

십 만 큰 가 상 말 다

지 못 들 마 갈 고 돗

도시민들 싹 싹 탔다 가 시

월에 병원에 원4 가 폐( )疲弊

진 몸도 갈 미 지 못 고 었다( )解渴 가는

얼마 지 생 에 생각

가 마감 는 신 평생 십 만에

가 과 많 닮 다고 생각 지는

가 운 는 어 어( ) ldquo rdquohelliphellip

월 새벽 시1965 5 6 1 태 없 거웠고

는 없 그 병원에 퇴원 집

가는 마지막 마 고 마 내 거 다

가 죽 간신 에 실 다 사는 어느5 lsquo

가 죽 는 말 가 식 다 신rsquo

상에 각 시키는 에 실( )刻印

어느 가는 후 민 가가 근 었다lsquo rsquo

다 는 간 과 진실 그 다는( ) ldquo

에 단 평 견 가지고 다 내

가 그리는 간상 단 고 다 지 다 는 가

에 는 평 지 니 그리고 어린 들

미지 겨 그린다rdquo

근 간 과 진실 그리고 싶어( )

가 다 근에게 그것 진리 다 거 다 없

년 학 간고사 대비2013 2 현대고 대비

ECN-0102-2013-001-000076193

거 고 다 없 는 것 진리

다 근 진리는 후 쪽 었다 신산( )辛酸 삶

었 질곡 역사 에 지냈( )桎梏

가 눈에 든 것 료 단 료 게 보

것 었다 그것 그 에 겨우겨우

슬 슬 생 어가는 간들 었다

리 과 단 리 고리에 검

마 없 거리 돌

상 것 없는 등 근에게 상에

과 진실 엄 다는 사실 리는 가 실( )儼存

고 가 과 역경 에 도 근 내 포

없었 후 보루 다( ) 堡壘 도 365

도 간 근 여

시 것 다

마 같 가가 고 싶었 근에게 그 꿈( )

에 다가가는 지 다 다 가 지망생들

규 미 상 에 진 고

에 지만 근 다 다 근

미 에 운 것 보통 시 미 시간

다 그런 그에게 없는 연습 가가

통 다 가 귀 시 지 도

얻는 뛸 듯 뻤지만 마 도 (

는 었 에 어린 근 주 에)

에 그림 그리고 지우고( )粉板

복 시간 가는 게 루 보냈다

zb36) 전 의 성 소가 아닌 것을 고르

① 평 ② 사건 ③ 경

④ ⑤ 훈

늘 지 상에 살고 는 사 들 억 도가10

고 그리 지 통 고 는 사 들( )知的

그보다 훨 많 억 도는 고 지 20

통 다 그런 지 고 2500

그리 간 보는 과 사 에

매우 달 뿐만 니 과 에 도 극

루고 었다 미 운 그런 들

살고 는 동 과 사 들 사고 식에

큰 가 다는 다

고 그리 들 우주 개별 고 독립

사 들 생각 지만 고 들 우

주 연 질 간주 다 같( ) 看做

각 도 들에게는 연 질

었지만 그리 들에게는 미 들 결 었

다 고 과 그리 들 사 같

는 동 과 사 에 도 견 다

지심리 미 마 드 겐트 는

살 들에 에 지 다

연 동 과 상 다 과 같 실험

다 크 만든 미드 도 보

여 주고 그 상 닥 고 주었다lsquo (Dax)rsquo

실 닥 는 재 지 는 것 실험 가lsquo rsquo

만들어 낸 다 그런 다 개 다 체 보

여 주었는 는 미드 지만 틱

만들었고 다 는 재료는 크 지만

달 다 그러고 어 것 닥 지 사 들에게 고 lsquo rsquo

게 니 들 주 같 고 는

체 택 고 동 들 같 재료 만들어진 체

택 다 러 는 심지어 살짜리

들에게 도 타났다 것 곧 과 동

다 상 보고 다는 것 미 다

개별 사 보고 고 동 연 질 보

고 는 것 다

동 들 주변 상 에 맞 어 동 고

에 다 사 들 태도 동에 보다 많

주 울 다 동 가 미시간 에

에 경험 다 그는 미식

경 보러 가게 었는 경 체는 매우 재미 었

주변 들 동에 질 다 그 는

들 계 어 상태 경 다

어 들 에 에 그 시 가 계 가

진 것 다 상 살펴 는 말 들 lsquo rsquo

에 그는 에 시 어 도 뒷사

생각 곧 다시 곤 것 다 그런 그에게 뒷

사 고 지 는 들 동 럼

어 웠다

생각 지도 리 드 니 벳-

zb37) 다음 위 의 내 전개 으 만 인lt gt

것은

lt gt

대조의 통해 대상이 닌 특성을 설 하고 있다

일화를 제 하여 자 의 주장을 뒷 침하고 있다

유추의 을 사 하여 독자의 의해를 돕고 있다

대상이 형성되는 과정을 간적 서에 따라 서 하고 있

① ②

③ ④

년 학 간고사 대비2013 2 현대고 대비

ECN-0102-2013-001-000076193

가 우리가 말 고 쓰는 든 단어가 사 에 는( )

것 니다 사 격에 가 는 지만

어 사 과 같 특별 는 사 니lsquo rsquo

단어 격 보 단어가 사 에

등재 어 다 리 리 사 는 단어 도 그

것 시 사 는 어 고 사 에

격 보 것 니다

러 얼 은 사전에 를 있는가 이에 대한 답lsquo rsquo

은 얼 이 유행어인가 아닌가에 따라 갈라 다 이 단어lsquo rsquo

는 년 어 자 에 랐고 쓰이고 있으2002 lsquo rsquo

유행어라고 하 에는 생 이 다 런데 계속

을 유 하 서 사전에 등재될 자격을 획득할 것인가 이

에 대한 답을 내리 는 히 어 다

여 서 가 를 고 해 볼 있다 첫 는 이 단어

를 써야 할 필 가 속적으 있는가 하는 점이다

상주의 열풍에 휩 인 사회 위 에 편 해서 퍼 말

이 얼 인데 과연 런 위 가 속될 것인가 이에lsquo rsquo

대해 필자의 생각은 정적이다 사회 위 가 뀌

런 말을 쓸 일이 없어 것이다

다음은 단어의 성이다 단어의 성이 사회적으 거

감이 없으 계속 사 될 가능성이 높다 런 에서

얼 은 좋은 조건이 아니다 익히 알 졌듯이 이lsquo rsquo

말은 얼 과 청소년층에서 속어 사 하는 이 결합lsquo rsquo lsquo rsquo

된 말이다 얼 에서 얼 을 리하는 조어 도 lsquo rsquo lsquo -rsquo

어에서는 매 낯선 이다 이것만으 도 거 감을 갖

는 사람들이 있다 더 나 속어 결합한 말이다 얼 lsquo rsquo

이 널리 퍼졌다 해도 은 여전히 청소년층의 속어lsquo rsquo

남아 있다 속어는 자연 럽게 아 자리에서나 쓰 에는

담 러 말이다 러한 담을 하고 사

역을 넓혀 가는 속어도 없 는 않다 특히 얼 은 lsquo rsquo

에도 종종 등장한다 만큼 거 감이 많이 희석되었다

고 할 있다 러나 일상의 자연 러 대화에서도 거

리낌 없이 등장하는가 게 는 되 않았다고 생

각한다

얼 이 유사어인 쌈 등을 만들어 내고lsquo rsquo lsquo rsquo

있으니 살아남을 있을 것이라고 는 견해도 있을 것

이다 러나 간이 나 서 유사어를 포함하여 든

말이 사라 사 는 많다 유사어가 많다는 것이 생 을

유 할 있는 절대적인 조건은 아니다

나 언젠가 터 사람들은 어느 단에서 얼 이 가장( )

쁜 사람을 가리켜 얼 이라고 르고 있다 이 얼lsquo rsquo lsquo rsquo

이라는 단어가 최근 어사전에 라 항간에 논란이 일고

있다 아닌 게 아니라 얼 은 유행어처럼 인다 생 lsquo rsquo

도 리 래되 않은 것 같고 언제 사라 도 알

없다 게다가 젊은이들 사이에서 주 쓰일 뿐이다 이런

단어를 사전에 는다는 게 하 이 없어 이 도

한다

러나 속단은 이다 차근차근 따져 볼 일이다

선 얼 이 일 적 유행어인 아닌 주의 게 들여다lsquo rsquo

볼 필 가 있다 유행어란 유행에 따라 빠르게 유포되었

다가 단 간 내에 소 되는 단어나 를 가리킨다

얼 은 인터넷을 통해 속히 퍼 말이다 하 만 일lsquo rsquo

적인 유행어처럼 단 간 내에 사라 않았을 뿐 아니라

현재 도 잦은 빈도 사 되고 있고 앞으 도 상당

간 사 될 것으 측된다 한 언 재단의 뉴 검 lsquo rsquo

색 사이트에 따르 얼 은 년 에 처음 나타난lsquo rsquo 2001

이후 꾸 히 사 되고 있다

이 같은 사 빈도는 얼 이 일 적 유행어 는 현lsquo rsquo

저히 다르다는 것을 여 다 장 간의 생존 만으 도

얼 은 이 한 어의 어휘 에 를 자격을 얻었다lsquo rsquo

고 할 있다 더 이 이라는 비 적 정제된 매체에

높은 빈도 쓰이고 있 않은가 사 빈도 측 에서

필통이나 연필과 같은 단어 대등하거나 더 많이 쓰lsquo rsquo lsquo rsquo

다는 것은 결코 가 게 볼 일이 아니다

이제는 사전이 언어 현 을 빠르게 하는 게 덕인

대가 되었다 세계적으 유 한 의 사전들도 경쟁

적으 어를 고 있다

하 만 얼 은 젊은이들이나 쓰는 속어라고 흠을 잡을lsquo rsquo

도 르겠다 얼 이 주 젊은 층에서 많이 쓰 lsquo rsquo

는 속어임에 틀림없다 러나 어사전에 표 적이고 품

위 있는 말만 어야 한다고 생각한다 것은 커다란

해다 당장 아 어사전이나 펼쳐 라 속어는

설과 같은 비어나 죄자들이 쓰는 은어 어

마니 같은 소 의 사람만이 쓰는 말 도 라 있

않은가 사전은 말 치에 일정 빈도 이상 나타나는 말이

라 말이든 다 할 있다

zb38) 가 나 에 대한 다음의 설( ) ( ) 않은 것은

① 가 는 얼짱 사 에 등재 것에( ) ( ) lsquo rsquo

보 고 다

② 사 등재 가는 단어 격에( )

고 고 는 언 들 언어 사 도에 고 다 ( )

③ 가 얼짱 어지만 신 과 같 매( ) ( ) lsquo rsquo

체에 도 사 는 말 는 고 다

④ 가는 얼짱 어 보고 크게 가지 근( ) lsquo rsquo 3

거 들어 뒷 고 다

⑤ 는 얼짱 어 는 다 특 다는( ) lsquo rsquo

근거 에도 크게 가지 근거 가 들어 주 2

뒷 고 다

가 늘 지 상에 살고 는 사 들 억( ) 10

도가 고 그리 지 통 고 는 사 들

그보다 훨 많 억 도는 고 지 20

통 다 그런 지 고 2500

년 학 간고사 대비2013 2 현대고 대비

ECN-0102-2013-001-000076193

그리 간 보는 과 사 에

매우 달 뿐만 니 과 에 도 극

루고 었다 미 운 그런 들

살고 는 동 과 사 들 사고 식에

큰 가 다는 다

고 그리 들 우주 개별 고 독립

사 들 생각 지만 고 들 우

주 연 질 간주 다 같 각

도 들에게는 연 질 었지

만 그리 들에게는 미 들 결 었다

고 과 그리 들 사 같 는

동 과 사 에 도 견 다

인 리학자인 츠 이마이 디드 겐트너는 두

살이 채 안 된 아이들에서 터 성인에 이르 다양한

연 대의 동양인과 서양인을 대상으 다음과 같은 험

을 했다 저 코르크 만든 피라 드 양의 도형을

여 주고 대상의 이름을 닥 라고 알 주었다lsquo (Dax)rsquo

제 닥 는 존재하 않는 것으 험자가 임의lsquo rsquo

만들어 낸 이름이다 런 다음 두 개의 다른 체를

여 주었는데 하나는 피라 드 양이 만 하얀 플라 틱

으 만들었고 다른 하나는 재 는 코르크 만 양이

달랐다 러고 나서 어떤 것이 닥 인 사람들에게 고 lsquo rsquo

르게 했더니 서양인들은 주 같은 양을 하고 있는

체를 선택했고 동양인들은 같은 재 만들어 체를

선택했다 이러한 차이는 성인은 어 두 살 리

아이들에게서도 나타났다 이것은 곧 서양인과 동양인은

서 다른 세상을 고 있다는 것을 의 한다 략 ( )

는 아주 단 하 서도 인상적인 험을 했다

험에는 동서양의 대학생들이 참여했다 는 험 참가자

들에게 컴퓨터 화 을 통해 속 장 을 담은 애니 이션

을 여 주었다 화 의 앙에는 초점의 역할을 하는 커

다란 고 한 마리가 있었고 주위에는 다른 생

들과 초 자갈 거품 등이 함 제 되었다 화 을

두 씩 후 참가자들은 자 이 것을 회상해 라는

를 았다

결과 서양인 대학생들과 동양인 대학생 두 앙

의 초점 역할을 했던 고 를 동일한 정도 언 했으

나 경 소 위 거품 초 다른 생 들 에 ( )

대해서는 동양인 대학생들이 서양인 대학생들 다 60

이상 더 많이 언 했다 뿐만 아니라 동양인 학생들은 서

양인 학생들에 비해 개 적인 고 다 전체적인 계

를 더 언 하는 경향을 다 략 또한 경의 일 ( )

를 화 킨 림을 제 하 을 때 동양인 대학생들은 대

경의 화를 알아챘 만 서양인 대학생들은 경

의 화를 거의 알아차리 했다 략 ( )

따라서 서양인들만을 대상으 연 한 화lsquo

편성 결 은 잘 된 것일 도 있다 각 과정과 인rsquo

과정의 어떤 이 화 편적이고 어떤 이

화에 따라 달라 는 는 앞으 많은 연 를 통하여 논의

되어야 한다

나 어떤 의 에서 리 두는 이 화적이다 리( )

안에는 다른 사람들과 더 친 한 계를 유 하 는 상호

의존성과 다른 사람들 터 독립적인 존재 살아가 는

독립성이 혼재한다 따라서 이 에서 어떤 특성이 더 강

하게 각되는 상황에 놓이느냐에 따라 서 다른 화적

특 을 일 있다 결 리 두는 어떤 경 에는

동양인처럼 행동하고 어떤 경 에는 서양인처럼 행동하는

것이다

zb39) 가 에 대한 다음의 설( ) 않은 것은

① 는 신 주 뒷 닥 실험과lsquo rsquo lsquo

니 실험 근거 시 다rsquo

② 동 들 상 간 공통 보다는 에 식

는 강 다

③ 들 주변 맥 에는 심 경 어 사건

과 사건 사 계에 상 민감 다

④ 는 동 과 틀린 지 고 는 것lsquo rsquo

니 다 고 다 lsquo rsquo

⑤ 가에 우리 사 들 개 시 가 원( )

집 경 말 고 는 것 개 보다는

에 고 는 것에 다

늘 지 상에 살고 는 사 들 억 도가10

고 그리 지 통 고 는 사 들( )知的

그보다 훨 많 억 도는 고 지 20

통 다 그런 지 고 2500

그리 간 보는 과 사 에

매우 달 뿐만 니 과 에 도 극

루고 었다 미 운 그런 들

살고 는 동 과 사 들 사고 식에

큰 가 다는 다

지심리 미 마 드 겐트 는 동

과 상 다 과 같 실험 다

크 만든 미드 도 보여 주고 그

상 닥 고 주었다 그런 다lsquo (Dax)rsquo

개 다 체 보여 주었는 는 미드

지만 틱 만들었고 다 는 재료는

크 지만 달 다 그러고 어 것 닥 lsquo

지 사 들에게 고 게 니 들 주 같rsquo

고 는 체 택 고 동 들 같

재료 만들어진 체 택 다 러 는

심지어 살짜리 들에게 도 타났다 것

곧 과 동 다 상 보고 다는

것 미 다 개별 사 보고 고 동

년 학 간고사 대비2013 2 현대고 대비

ECN-0102-2013-001-000076193

연 질 보고 는 것 다

동 들 주변 상 에 맞 어 동 고

에 다 사 들 태도 동에 보다

많 주 울 다 동 가 미시간

에 에 경험 다 그는 미

식 경 보러 가게 었는 경 체는 매우 재

미 었 주변 들 동에 질 다 그

는 들 계 어 상태 경

다 어 들 에 에 그 시 가 계

가 진 것 다 뒷사 고 지 는 들

동 럼 어 웠다

그는 경험에 어 얻어 동 들lsquo

각도 상 본다 는 가 우고rsquo

검 여 주 단 도 상 실험 실

시 다 그는 실험 가 들에게 컴퓨 통

담 니 보여 주었다

에는 역 는 커다 고 마리가 었

고 주 에는 다 생 들과 갈 거 등

께 시 었다 본 후 가 들

신 본 것 상 보 는 지시 다

그 결과 생들과 동 생

역 고 동 도 언

경 거 다 생 들에 ( )

는 동 생들 생들보다 60

상 많 언 다 뿐만 니 동 생들

생들에 개별 고 보다 체 계

언 는 경 보 다 경 변 시

킨 그림 시 동 생들 경

변 지만 생들 경 변

거 리지 못 다

지 지 들만 상 연 lsquo

보편 결 못 것 도 다 지각 과 과rsquo

지 과 어 보편 고 어

에 달 지는지는 많 연 통 여

어 다

리 드 니 벳 생각 지도 사- ldquo rdquo( 2004)

zb40) 위 에 대한 설 으 가장 적절한 것은

① 동 과 생 식 강 고 다

② 가지 실험 통 쓴 고 다

③ 닥 실험에 사 본질에 동 사

상에 주 다

④ 니 실험에 동 과 에 지

각 도에 가 다

⑤ 쓴 는 보편 연 에 드러 우월 에

에 근 고 다

가 동 들 주변 상 에 맞 어 동 고( )

에 다 사 들 태도 동에 보다 많

주 울 다 동 가 미시간 에

에 경험 다 그는 미식

경 보러 가게 었는 경 체는 매우 재미 었

주변 들 동에 질 다 그 는

들 계 어 상태 경 다

어 들 에 에 그 시 가 계 가

진 것 다 상 살펴lsquo 는 말 들rsquo

에 그는 에 시 어 도 뒷사

생각 곧 다시 곤 것 다 그런 그에게

뒷사 고 지 는 들 동 럼

어 웠다

그는 경험에 어 얻어( ) 동 들lsquo

각도 상 본다 는 가 우고rsquo

검 여 주 단 도 상 실험

실시 다 실험에는 동 생들 여 다

그는 실험 가 들에게 컴퓨 통

담 니 보여 주었다 에는

역 는 커다 고 마리가 었고 주 에는

다 생 들과 갈 거 등 께 시

었다 본 후 가 들 신 본 것

상 보 는 지시 다

다 그 결과 생들과 동 생( )

역 고 동 도 언

경 거 다 생 들 에 ( )

는 동 생들 생들보다 60

상 많 언 다 뿐만 니 동 생들

생들에 개별 고 보다 체 계

언 는 경 보 다 들어 동

생들 상 체 연못 럼 보 어ldquo 같rdquo

체 맥 언 시 었지만

생들 상 어 같 큰 고 가 쪽 움ldquo

직 어 같 역 고rdquo

언 시 다 경 변 시킨 그

림 시 동 생들 경 변

지만 생들 경 변 거

리지 못 다

년 학 간고사 대비2013 2 현대고 대비

ECN-0102-2013-001-000076193

게 볼 동 들 보다는 큰 그( )

림 보 에 사 과 체 맥 연결시 지각

는 경 고 체에 특 떼어 내

어 독립 보는 것 낯 어 다 에

들 사 에 고 주변 맥 에는 심 경

에 사건과 사건 사 계에 상

민감 편 다

마 지 지( ) 들만 상 연

보편 결 못 것 도 다lsquo rsquo 지각 과

과 지 과 어 보편 고 어

에 달 지는지는 많 연 통 여

어 다

리 드 니 벳 생각 지도 사- ldquo rdquo( 2004)

zb41) 의 하는 가~ 다른 것은

① ② ③

④ ⑤

얼마 그 에 동 사고 식과

사고 식 보여 주는 내 다

들 에 는 탕 고 같 게

어 겨 고 미 에 는 그 크 럼 큰 고

어리 주고 원 는 어 도 는

상 고 생각 다는 것 다 러

는 어떻게 생 것 고 과 그리 거슬

러 가 보 그 단 다

고 연 경 체 경 생 에

다 벼 사는 공동 업과 경험 많 연 역

에 고 들 연 웃과

게 지내 고 탁 연 들

들 지 연 럽게 들 다 민들

웃과 동 게 뿐만 니 는 집 과

게 다

동 시 는 생태 경 에 살 결과

들 다 사 들 사 상 에 주

울 게 었고 는 곧 체 상 과 간 사

계 시 는 낳게 었다 신 가

가 는 체에 는 원 는 동시

에 다 사 들 그 사 포 체 맥 에

다 들 간 사 연

계 체 계에 주 울 는 사고 체계

게 었다

그러 그리 연 경 그 었다 산

지 연결 는 지 건 그리고 역

에 다 런 들 업에 다 사 과

동 므 공동체에

다고 다 고 그리 들

들과는 달리 보 내 감 지 들과

지 크게 느 지 못 다 그

견 다 경우 주 쟁 통 결 는 갖

게 었다

신 사 간 계들 루어진 커다

트워크 에 게 당연 사 역시 연

계들 체 식 게 다 어 상

원 도 그 개체가 체 맥 과

계 에 고 다 게 체 맥 에 주

울 다 보 상 복 과 가변 식 게 고

상에 재 는 많 변 들 사 에 재 는 들도

게 다 들 주 태도 보

는 경우가 많다 쟁 결

통 결 보다는 통 결

는 보 다

그러 고 그리 들 개개 사 사 독

에 주 울 다 사 사 체에

어 그들 사 에 재 는 공통 규 주

고 다 상 원 에도 사

체 내 주 고 다 그들

체 여 탕 체

는 주 태도 시 고 특 사 어

주에 는지 여 그 주에 는 규

견 다 에 는 쟁 식 리

같 리 사고 체계가 달 게 었다

리 드 니 벳 생각 지도 사- ldquo rdquo( 2004)

zb42) 위 에서 사 된 설 과 가장 유사한 것은

① 크톱 컴퓨 는 본체 니 마우 루

어 다

② 곡과 시 리 는 지 과 사 루어 다는 공통

지니고 다

③ 경 고 것과는 달리

경 본 연 태 그 주변 경

④ 벽돌 능 에 사계 내내

습도가 지 다

⑤ 잰느 체 체 지닌 재 체가 없

는 재 눌 다

년 학 간고사 대비2013 2 현대고 대비

ECN-0102-2013-001-000076193

zb43) 는 립 앙 도서 이 정의 일 이다lt gt

도서 장과 이 자의 리 의 정의 연결이

적절하 않은 것은

lt gt

제 조 서 유8 ( )

도서 장은 다른 이 자의 안전을 위협하거나 도서 의①

서를 란하게 할 가 있는 자에 대하여는 도서 출입

을 제한할 있다

도서 장은 이 자가 제 조 각 호의 어느 하나의 행위를 하7②

을 때에는 이 을 하게 하거나 도서 출입을 제한할

있다

제 조자 의 대출9 ( )

도서 자 는 다음 각 호의 경 대출할 있다①

상호대차도서 간에 자 를 류하는 것을 말한다 등 다1 ( )

른 도서 과의 협 을 위하여 필 한 경

공 이 공 행 상 필 하는 경2

에 도서 장이 필 하다고 인정하는 경3

대출이 가능한 도서 자 의 위는 도서 장이 정하는②

에 따른다

제 조 상10 ( )

이 자가 도서 자 설을 더럽히거나 찢거나 뜨①

쓰게 하거나 잃어 린 경 에는 상하여야 한다

도서 장은 제 항에 따른 상 을 정하여 게 하여야1②

한다

제 조이 절차 등11 ( )

이 칙에서 정한 것 에 도서 자 설의 이 절차

이 제한 등에 필 한 사항은 도서 장이 정한다

출처 립 앙 도서- (httpwwwnlgokr)

① 는 도 리 다8

② 도 는 리 다9 1

③ 료 지 는 도 리 다9 2

④ 도 료 변상에 리10 1

⑤ 는 에 도 리 다11

3

도 다 각 같다①

공 공 다만 연1

연 간 다

매월 째 째 월2

도 도 리 그 사3

가 다고 는

도 에 미리 게1 3②

시 여 다

4

도 시간 도 여 게시 다

5

도 료 시 는 는 도①

지에 등 후

등 에 사 도②

7

는 다 각 여 는 니 다

도 료 시 상 리1 lsquo rsquo

도 료 시 훼 는2 middot

지 가 닌 곳에 식 거 담3

우는

도 보 등 보 검색열4 middot

그 에 도 질 지 여 도5

여 게시 사 는

8

도 다 거 도①

질 게 우 가 는 에 여는 도

도 가 각 어느7②

에는 지 게 거 도

9

도 료는 다 각 경우 다①

상 도 간에 료 는 것 말1 (

다 등 다 도 과 여 경우)

공 원 공 상 는 경우2

그 에 도 다고 는 경우3

가능 도 료 는 도②

는 에 다

10

년 학 간고사 대비2013 2 현대고 대비

ECN-0102-2013-001-000076193

가 도 료 시 럽 거 거①

못 쓰게 거 어 린 경우에는 변상 여

도 에 변상 여 게시1②

여 다

zb44) 위 에서 도서 장이 게 해야 할 사항에 해당하는

것을 두 쓰

년 학 간고사 대비2013 2 현대고 대비

ECN-0102-2013-001-000076193

립 도 규

1 ( )

규 립 도 립 어린 청 도(

포 다 료 시 열 시 말) (

다 에 사 규 립 도)

편 진 다

2 ( )

규 립 도 도 다 에( lsquo rsquo )

고 는 도 에 도lsquo rsquo 2 2

료 에 여 다 다만 특 료 귀

료 등 료 에 사 립 도

도 다 다( lsquo rsquo )

3 ( )

도 다 각 같다①

공 공 다만 연1

연 간 다

매월 째 째 월2

도 도 리 그 사3

가 다고 는

도 에 미리 게1 3②

시 여 다

시간4 ( )

도 시간 도 여 게시 다

등 등5 ( )

도 료 시 는 는 도①

지에 등 후

등 에 사 도②

사 료6 ( )

도 료 시 에 사 료는 도

7 ( )

는 다 각 여 는 니 다

도 료 시 상 리1 lsquo rsquo

도 료 시 훼 는2 middot

지 가 닌 곳에 식 거 담3

우는

도 보 등 보 검색열4 middot

그 에 도 질 지 여 도5

여 게시 사 는

질 지8 ( )

도 다 거 도①

질 게 우 가 는 에 여는 도

도 가 각 어느7②

에는 지 게 거 도

료9 ( )

도 료는 다 각 경우 다①

상 도 간에 료 는 것 말1 (

다 등 다 도 과 여 경우)

공 원 공 상 는 경우2

그 에 도 다고 는 경우3

가능 도 료 는 도②

는 에 다

변상10 ( )

가 도 료 시 럽 거 거①

못 쓰게 거 어 린 경우에는 변상 여

도 에 변상 여 게시1②

여 다

등 규 에 것 에 도11 ( )

료 시 등에 사

도 다

립 도- (httpwwwnlgokr)

zb45) 도서 장의 리 있는 조항으 적절하 않

은 것은

① ② ③ ④ ⑤

년 학 간고사 대비2013 2 현대고 대비

ECN-0102-2013-001-000076193

1 ( )

사가 공 는lsquo rsquo

과 여 사 원과 리

사 타 사 규

니다

개 보 보7 ( )

사는 보통신망 등 계 는 에lsquo rsquo lsquo rsquo

원 개 보 보 니다 개lsquo rsquo

보 보 사 에 는 사 개lsquo rsquo

보 취 니다 다만 사는 다 lsquo rsquo

사 계 통 공 는 경우 원 lsquo rsquo

등 개 보 당 사에 습니lsquo rsquo

원 리에8 (lsquo rsquo lsquo rsquo lsquo rsquo

)

원 에 리lsquo rsquo lsquo rsquo lsquo rsquo①

원에게 가 도 여 는lsquo rsquo 3

니다

사는 원 가 개 보 우 가lsquo rsquo lsquo rsquo lsquo rsquo②

거 사 경우 는 미 에 어 거 lsquo

사 사 운 우 가 는 경우 당rsquo lsquo rsquo

습니다lsquo rsquo

원 가 도 거lsquo rsquo lsquo rsquo lsquo rsquo 3③

가 사 고 지 경우에는 시 사에lsquo rsquo

통지 고 사 내에 니다lsquo rsquo

경우에 당 원 사에 그 사실3 lsquo rsquo lsquo rsquo④

통지 지 거 통지 도 사 내에 지 lsquo rsquo

생 경우 사는 지지 습니다lsquo rsquo

사10 (lsquo rsquo )

사는 과 지 미lsquo rsquo①

에 는 지 계 고

공 여 다 여 니다lsquo rsquo

사는 원 게lsquo rsquo lsquo rsquo lsquo rsquo②

도 개 보 신 보 포 보 보 시( )

갖 어 개 보 취 공시 고

니다

사는 과 여 원lsquo rsquo lsquo rsquo③

견 만 당 다고 경우에는

리 여 니다 원 견 만 사 lsquo rsquo

에 는 게시 거 우편 등 통 여

원에게 리 과 결과 달 니다lsquo rsquo

원11 (lsquo rsquo )

원 다 여 는 니다lsquo rsquo ①

신청 는 변경 시 허 내 등1

타 보 도2

사가 게시 보 변경3 lsquo rsquo

사가 보 보 컴퓨 그4 lsquo rsquo (

등 등 신 는 게시)

사 타 등 지 재산 에5 lsquo rsquo 3

사 타 상 거 업6 lsquo rsquo 3

는 폭 시지 상 타 공7 middot middot

에 는 보 에 공개 는 게시 는lsquo rsquo

사 동 없 리 사8 lsquo rsquo

타 거 당9

게시15 (lsquo rsquo )

원 내에 게시 는 게시 게재 는lsquo rsquo lsquo rsquo lsquo rsquo

경우 원 사가 게시 복 lsquo rsquo lsquo rsquo lsquo rsquo middot middot

등 태 언 등에 공 는

것 내에 다 원 본 게시 등 lsquo rsquo lsquo rsquo

크 능 등 여 복 는 등 태

는 것 동 것 니다

- (wwwnavercom)

zb46) 위 은 인터넷 포털사이트의 회 가입을 위한 이

약 의 일 이다 이 약 을 만드는 과정에서 생각한

내 으 적절하 않은 것은

개 보 보 가 지에 별 눠①

겠어

원 가 만들게 에②

시 주어 겠어

원들 게재 게시 다 원 크 다③

는 것 지

④ 원 지 는 뿐만 니 사가 지 는

도 께 달 지

리에 가 생 경우 사가⑤

에 다는 도 듯

1 ( )

사가 공 는lsquo rsquo

과 여 사 원과 리

사 타 사 규

년 학 간고사 대비2013 2 현대고 대비

ECN-0102-2013-001-000076193

니다

개 보 보7 ( )

사는 보통신망 등 계 는 에lsquo rsquo lsquo rsquo

원 개 보 보 니다 개lsquo rsquo

보 보 사 에 는 사 개lsquo rsquo

보 취 니다 다만 사는 다 lsquo rsquo

사 계 통 공 는 경우 원 lsquo rsquo

등 개 보 당 사에 습니lsquo rsquo

원 리에8 (lsquo rsquo lsquo rsquo lsquo rsquo

)

원 에 리lsquo rsquo lsquo rsquo lsquo rsquo①

원에게 가 도 여 는lsquo rsquo 3

니다

사는 원 가 개 보 우 가lsquo rsquo lsquo rsquo lsquo rsquo②

거 사 경우 는 미 에 어 거 lsquo

사 사 운 우 가 는 경우 당rsquo lsquo rsquo

습니다lsquo rsquo

원 가 도 거lsquo rsquo lsquo rsquo lsquo rsquo 3③

가 사 고 지 경우에는 시 사에lsquo rsquo

통지 고 사 내에 니다lsquo rsquo

경우에 당 원 사에 그 사실3 lsquo rsquo lsquo rsquo④

통지 지 거 통지 도 사 내에 지 lsquo rsquo

생 경우 사는 지지 습니다lsquo rsquo

원에 통지9 (lsquo rsquo )

사는 특 다 원에게 통지 경우lsquo rsquo lsquo rsquo

공지 게시 통 상 게시 개별 통지에7

갈 습니다

사10 (lsquo rsquo )

사는 과 지 미lsquo rsquo①

에 는 지 계 고

공 여 다 여 니다lsquo rsquo

사는 원 게lsquo rsquo lsquo rsquo lsquo rsquo②

도 개 보 신 보 포 보 보 시( )

갖 어 개 보 취 공시 고

니다

사는 과 여 원lsquo rsquo lsquo rsquo③

견 만 당 다고 경우에는

리 여 니다 원 견 만 사 lsquo rsquo

에 는 게시 거 우편 등 통 여

원에게 리 과 결과 달 니다lsquo rsquo

원11 (lsquo rsquo )

원 다 여 는 니다lsquo rsquo ①

신청 는 변경 시 허 내 등1

타 보 도2

사가 게시 보 변경3 lsquo rsquo

사가 보 보 컴퓨 그4 lsquo rsquo (

등 등 신 는 게시)

사 타 등 지 재산 에5 lsquo rsquo 3

사 타 상 거 업6 lsquo rsquo 3

는 폭 시지 상 타 공7 middot middot

에 는 보 에 공개 는 게시 는lsquo rsquo

사 동 없 리 사8 lsquo rsquo

타 거 당9

원 계 규 내lsquo rsquo lsquo②

여 공지 주 사 사가 통지 는rsquo lsquo rsquo

사 등 여 타 사 업 에 lsquo rsquo

는 여 는 니다

- (wwwnavercom)

zb47) 위 약 의 조항에서 같은 제점을 하lt gt

고 있는 조항은

lt gt

제휴 회사에 회 의 아이디 개인 정 를 전송할 있도

한 조항은 고객에게 당한 조항이다

1 7 8① ② ③

④ 9 ⑤ 10

립 도 규

1 ( )

규 립 도 립 어린 청 도(

포 다 료 시 열 시 말) (

다 에 사 규 립 도)

편 진 다

2 ( )

규 립 도 도 다 에( lsquo rsquo )

고 는 도 에 도lsquo rsquo 2 2

료 에 여 다 다만 특 료 귀

료 등 료 에 사 립 도

도 다 다( lsquo rsquo )

3 ( )

도 다 각 같다①

공 공 다만 연1

연 간 다

년 학 간고사 대비2013 2 현대고 대비

ECN-0102-2013-001-000076193

매월 째 째 월2

도 도 리 그 사3

가 다고 는

도 에 미리 게1 3②

시 여 다

시간4 ( )

도 시간 도 여 게시 다

등 등5 ( )

도 료 시 는 는 도①

지에 등 후

등 에 사 도②

사 료6 ( )

도 료 시 에 사 료는 도

7 ( )

는 다 각 여 는 니 다

도 료 시 상 리1 lsquo rsquo

도 료 시 훼 는2 middot

지 가 닌 곳에 식 거 담3

우는

도 보 등 보 검색열4 middot

그 에 도 질 지 여 도5

여 게시 사 는

질 지8 ( )

도 다 거 도①

질 게 우 가 는 에 여는 도

도 가 각 어느7②

에는 지 게 거 도

료9 ( )

도 료는 다 각 경우 다①

상 도 간에 료 는 것 말1 (

다 등 다 도 과 여 경우)

공 원 공 상 는 경우2

그 에 도 다고 는 경우3

가능 도 료 는 도②

는 에 다

변상10 ( )

가 도 료 시 럽 거 거①

못 쓰게 거 어 린 경우에는 변상 여

도 에 변상 여 게시1②

여 다

등 규 에 것 에 도11 ( )

료 시 등에 사

도 다

립 도- (httpwwwnlgokr)

zb48) 다음 정 리 의 의 으 볼 때 가장

이 적인 것은

도 시간 도 여 게시 다①

등 에 사 도②

가능 도 료 는 도 는③

에 다

④ 도 에 변상 여 게10 1

시 여 다

⑤ 도 가 각 어느7

에는 지 거 도

zb49) 를 참고하여 이 어의 성격을 설 한lt gt

것으 적절하 않은 것은

① 보 에 는 어 시 상 고 어 시lt gt lsquo rsquo

에 보여주고 다

② 진 어 어원에 견 고 다

에는 타 어 들어가는 것 다 lsquo rsquo

③ 에 들어갈 말 각각 고 어 어 신 어~

들 언어는 질 격 강 통 없었다

④ 시 우리 에 가 었지만 지 계

과 달리 들 통 사 달 어 웠

년 학 간고사 대비2013 2 현대고 대비

ECN-0102-2013-001-000076193

⑤ 크 몽골 만주 공통어가 우리 어 같

계열에 다는 에 사 특 짐

가( )

善化公主主隱 공주님

他密只嫁良置古 몰 결 고

薯童房乙 맛

夜矣卯乙抱遣去如 에 몰 고 가다

( )

始汝 會隱日恚見隱扐 만 에 본

恥隱汝衣淸隱笑 맑 웃

고 시 여 공 크다 만 다[ ] ( ) ( ) ( ) ( )始 汝 會扐

내다 에 보다 견( ) ( )恚 見 다( )隱

럽다 맑다 청 웃( ) ( ) ( ) ( )恥 衣 淸 笑

zb50) 위의 나 를 함 고 음에 답하( ) lt gt

보lt gt

( )素那或云金川 白城郡蛇山人也

운 사산

는 고 다 는( )[ ( ) ] (素那 金川 白城

사산 사 다) ( ) 郡 蛇山

삼 사- lsquo rsquo 47

에 제 된 단어 의 표 리를 조건(1) lt gt ( ) lt gt

에 맞게 서 하

건lt gt

lsquo 었고 었다 태rsquo

에 제 된 단어 동일한 표 리에(2) lt gt ( )

의해 적은 것을 나 에서 찾아 조건 에 맞게 서 하( ) lt gt

건lt gt

에 당 는 각각( ) 개 쓸 것2 단

당 는 가 여러 개 어도 개만 쓸 것 각2

개 과 도 쪽에 개만2 2

드시 지 것( )

과 동 원리 것lsquo 고

과 동 원리 것 다rsquo

태 것

가( )

素那(或云金川) 白城郡蛇山人也

소나 또는 천 이라 한다 는 성 사( ) ( ) ( )素那 金川 白城郡〔 〕

산 사람이다 현대어 풀이( ) ( )蛇山

나( )

紫布岩乎希 회

執音乎手母牛放敎遣 자 손 암쇼 노히 고

吾 不喩慙 伊賜等肹 肹 나 안디 리샤

花 折叱肹 可獻乎理音如 고 것거 도림다

다 향찰은 리말을 리 으 적은 표 이었 만 생( )

은 고 대를 넘 하고 끊어 고 말았다 랜 세

동안 갈고 닦아 체계적이었던 향찰 표 이 사라졌

을 인은 크게 두 가 나누어 생각해 볼 있다

하나는 족 사회의 한 선호도에서 찾을 있다 라 때

향찰은 주 족 계 에서 사 했을 것으 인다 한 을

알 하고서는 한자를 활 하여 리말을 리 으 표

하 란 가능하 때 이다 런데 족들은 간이 흐

를 향찰과 같은 리 표 을 익혀 사 하 다는

아 한 을 대 사 하는 쪽을 선호하게 되었다 더 이

고 초에 인재 등 을 위해 과거제도가 행되 서 한 선

호도가 더 높아졌고 결 향찰은 소 되고 말았다

또 다른 가능성은 한 어의 특성에서 찾을 있다

터 한 과 일 세 나라는 한자 화 에 속해 다

당연한 이야 겠 만 표의 자인 한자는 어를 표 하

에 매 적절하다 어의 음절은 성 ( ) ( )聲母 韻母

이 어 고 여 에 성조가 추가되어 최종 소리가 결정된

다 래서 어는 단음절을 하나의 한자 표 하 된

다 에 초성 성 종성의 세 가 소가 하나의 음절

년 학 간고사 대비2013 2 현대고 대비

ECN-0102-2013-001-000076193

을 이 는 한 어는 음절 조가 잡하고 음절의 가 많아

서 한자 차 만으 한 어의 소리를 만족 럽게 표 할

없었다 를 들어 한 어에서는 어 니 같이 음절 lsquo rsquo

이 어 단어가 얼마든 있으나 어는( ) 複數音節

자 하나 나타내 만이다lsquo [m ]rsquo 母 ǔ

한편 일 어의 표 은 핵 적 단어는 한자 적고 토는

가나라는 일 의 자 적는 이다 적인 의 를 나

타내는 은 표의 자인 한자 적고 적 계를 나

타내는 토는 표음 자 적는 셈이니 자세히 살펴

리의 향찰 표 을 쏙 빼닮았음을 알 있다 한 어 같

은 착어이 서도 일 어에만 향찰과 유사한 표 이 살아

남은 것은 일 어의 특 때 이다 일 어는 하나의 자음과

음의 결합으 음절을 이 고 침이 거의 없는 음절 언어

이다 이러한 음절의 특색에다가 토가 달한 착어라는 점

이 향찰과 유사한 표 이 살아남을 있는 비결이었다

하 만 같은 착어라도 다양한 음소 침이 달한 한

어는 향찰 표 하는 데 근 적으 한계가 있었다

zb51) 다 하여 의 행에 대한 탐 한 결과( ) lt gt 2

않은 것은

보lt gt

善花公主主隱 공주니믄 공주님( )

----------------------------------------

-

他密只嫁良置古 그 지 얼어 고 몰 결(

----------------------------------------

-

薯童房乙 맛 맛( )

夜矣卯乙抱遺去如 몰 고 가다 에 몰 고(

가다)

주동 역 동- (薯童謠『 』

에 2 ( )他密只嫁良置古

얼다 시집가다 결 다 말 lsquo rsquo

① 실질 미 지니고 므 타 타lsquo ( )rsquo lsquo [ ]

② 에 실질 미 타내고 지 는lsquo rsquo lsquo [ ]rsquo lsquo [ ]密只 密 只

계 타내는

③ 얼어는 실질 미 포 고 므 가lsquo rsquo lsquo [ ]rsquo嫁

것lsquo [ ]rsquo 良

④ 고 어간 는 실질 미 지니고 므lsquo rsquo lsquo -rsquo

것lsquo [ ]rsquo 置

⑤ 고 어미 고는 계 타내고 므lsquo rsquo lsquo- rsquo

고 것lsquo [ ]rsquo 古

가( )

엉 훈 민middot middot middot middot middot世 宗 御 製 訓 民 正 音

말 미 듕 귁에 달middot middot middot middot middot middot middot middot中 國 文 字

니 런middot middot middot middot middot middot 어린middot middot middot middot百 姓

니 고 도 내 들middot middot middot middot middot middot middot middot middot 시러middot

펴 몯middot 미middot middot 니 내middot middot middot middot middot middot middot middot 爲

어엿middot 겨 새middot middot middot 믈여듧middot middot middot middot字 니middot middot middot

사 마다 니겨 킈 middot middot middot middot middot middot middot middot middot便 安

고 미니middot middot middot middot

본 는 상( ) (象

원리에 만들어진 본) ( )形 ㄱ ㄴ ㅁ ㅅ ㅇ

에 는 가 원리에( )加劃

그리고( )ㅋ ㄷ ㅌ ㅂ ㅍ ㅈ ㅊ ㆆ ㅎ

쓰는 병 원리에 만들어진( )竝書

마지막 체( ) ( )異體ㄲ ㄸ ㅃ ㅆ ㅉ ㆅ

ᅀ 다 상 원리에 ㅇ ㄹ

지 는 삼재 상 본 본( ) ( ) ( 天地人 三才

탕 므림과 림에 ) (初ㅡ ㅣ

재)( ) ( )( )出字 再出字ㅗ ㅏ ㅜ ㅓ ㅛ ㅑ ㅜ ㅕ

병 그리고 들 에 다시( )ㅘ ㅝ ㅣ

( )ㅣ ㅢ ㅚ ㅐ ㅟ ㅔ ㆉ ㅒ ㆌ ㅖ ㅙ ㅞ

zb52) 가 에 대한 설 으 르 않은 것을( ) 두 고르

① 어쓰 규 지키고 다

② 리 고 다

③ 말 미 미 등 어 사 다lsquo rsquo

④ 개 지 다

년 학 간고사 대비2013 2 현대고 대비

ECN-0102-2013-001-000076193

⑤ 어 원 에 가 도 고 다

엉 훈 민世 宗 御 製 訓 民 正 音

말 미 듕귁에 달 니

런 어린 니 고 도middot

내 들 시러 펴 몯 미 니middot

내 어엿 겨 새 믈여듧

사 마다 니겨middot 킈 고

미니

훈민 언 본- lsquo rsquo 5 (1459 )

zb53) 위의 에 대한 현대어 풀이가 르~ 않은 것

① 우리 말 과 달

② 어리 말 고 는 것 어도

③ 신 생각 마 껏 펼 는 사 많다

④ 게 생각 여

⑤ 사 마다 게

zb54) 훈민정음 언해 에는 한 을 창제한 동 가 드러나

있다 훈민정음 창제의 정 과 내 이 잘 연결된 것

① 주 신 말 미 듕귁에 달

② 민 신 내 어 겨

③ 신 뻔 킈 고 미니

④ 실 신 사 마다 니겨

⑤ 귀 신 계 주 는 훈민 신과 거리가

가 엉 훈 민( ) middot middot middot middot middot世 宗 御 製 訓 民 正 音 

말 미 귁에 中 國 달 文 字

니 런 어린 니 百 姓

고 도 내 들 시러 펴 몯

미 니 내 어엿 爲 겨 새

믈여듧 니 사 마다 니 字

겨 킈 고 미니 便 安

훈민 언 본- lsquo ( )rsquo ( ) 5 (1459 )訓民正音 世祖

( )

[ 1 ]

동 룡 샤 마다 복( ) ( ) ( )海東 六龍 天福

시니 고 동( ) ( )古聖 同符 시니

[ 2 ]

매 니 곶 여

미 므 니 그 내 러

가 니

[ 125 ]

우 미리( )千世 샨( )定 에( )漢水北 累仁

누 개 샤 복 업 시니( ) ( ) 開國 卜年

신( )聖神 니 샤도 경 근민 샤 욱( )敬天勤民

드시리 다

님 쇼 산 가( ) ( )洛水 山行

미드니 가

어 가- lsquo ( )rsquo 27龍飛御天歌

다 우리신 니쓰고 다만 만 쓰( )

거 샹 귀쳔 다보게 러 귀

여 쓴 도 신 보 가 고 신 에

말 어 보게 각 에 사 들

고 본 몬 능통 후에

죠 죠 니

드 도 만 공 에 사

드 미 죠 고 고 여 보 죠

보다 얼마가 거시 어신고 니 첫

가 죠 니 죠

민 들 어 신 샹

귀쳔 도보고 어보 가 만 늘

고 폐 에 만쓴 죠 민

도 러보지못 고 보니 그게 엇지

심 니 리 보 가 어 운건 다

니 쳣 말마 지 니 고 그

쓰 에 가 우 지 지

몰 거 본후에 가 어 지

고 그니 쓴편지 쟝 보

년 학 간고사 대비2013 2 현대고 대비

ECN-0102-2013-001-000076193

쓴것보다 듸 보고 그 마 니 쓴 고

어 못

그런고 에 리 과 가

만 쓴 못 민 말만 듯고

고 편 그 못 보니 그사 단

병신 못 다고 그사 식 사

니 만 고 다 과 그사

만 고 다 과 업 사 보다 식 고

죠 도 고 각 과

견 고 실 직 귀쳔 간에 그

고도 다 것 몰 귀죡 보다

사 우리 신 귀쳔 다 업

시 신 보고 과 지 게 랴

시니 샹 귀쳔 간에 우리 신 걸

간 보 새지각과 새 걸 미리

독립신- lsquo (1896)rsquo

zb55) 친 어 나의 제 장( ) 2 매 함축적

의 가 가장 유사한 것은

① 지 눈 내리고 매 득 니 내 여 가

사- lsquo rsquo

② 도 어 리듯 그 게 어 다

주 사- lsquo rsquo

③ 눈 살 다 죽 어 린 과 체 여

눈 새벽 지 도 살 다

눈- lsquo rsquo

④ 삶 근심과 고단 에 돌 거니는 여 거 는

여 리 내린 살가지 에 눈 리 눈 리

택 그 생 에- lsquo rsquo

⑤ 늘 러 고 러

청룡 룡 어 개 루 우

신경림 계- lsquo rsquo

zb56) 친 를 위 가 나 에 나타난A B ( ) ( )

세 어의 특 에 의거하여 세 어 표 하

그 산 고 공 도 맑지만

A

주변에 쓰 리는 어리 사 많다

B

건lt gt

식 가 에 타 어 특징에( ) ( )

거 과 어쓰 는 고 지 말 것

A

B

zb57) 가 의( ) 달 아ㆍ 다 의 ( ) 나셔에서 알 있는

세 어 개화 어의 특 을 비 하여 조건 에lt gt

맞게 서 하

건lt gt

어에 는lsquo 개

어에 는 다 태rsquo

zb58) 은 가 는 다 에 나 는 절lt 1gt ( ) lt 2gt ( )

일 를 췌한 것이다 의 의 가 lt 1gt (1)~(2)

유사한 말을 에서 찾아 쓰lt 2gt

보lt 1gt

런 (1) 어린 니 고百 姓

도 내 들 시러 펴 몯 미

사 마다 (2) 니겨 便 安

킈 고 미니

보lt 2gt

죠 고 고 여 보 죠

보다 얼마가 거시 어신고 니 첫 가

죠 니 죠 민

들 어 신 샹 귀쳔

도보고 어보 가 만 늘 고

폐 에 만쓴 죠 민 도

러보지못 고 보니 그게 엇지 심

니 리

년 학 간고사 대비2013 2 현대고 대비

ECN-0102-2013-001-000076193

lt 1 gt

동 룡 샤 마다 복 시( ) ( ) ( )海東 六龍 天福

고 동 시니( ) ( )古聖 同符

lt 2 gt

(A) 매 니 곶

여 니

미 므 니 그 내

러 가 니

lt125 gt

우 미리 샨 에( ) ( ) ( ) 千世 定 漢水北 累

누 개 샤 복 업 시 니( ) ( ) 仁開國 卜年 聖

신( ) 神 니 샤도 경 근민 샤( ) 敬天勤民

욱 드 시 리 다

님 쇼 산 가 ( ) ( )洛水 山行

미드니 가

- lt gt龍飛御天歌

zb59) 장과 내 상 유사한 성격의 조는125

① 뫼 고 고 고 고

어 그린 많고 많고 고 고

어 러 는 울고 울고 가느니

도 견- lt gt

② 강 에 드니 몸 다

그믈 고 가니

뒷 뫼 엄 언 니( )藥

-

③ 말 없는 청산 태 없는 다

값 없는 청 없는 월

에 병 없는 몸 별 없 늙 리

-

④ 가마귀 골에 가지 마

낸 가마귀 새

청강에 것 시 몸 러 가( ) 淸江

-

⑤ 진 골에( ) 白雪

가 매 는 어느 곳에 었는고

에 갈 곳 몰( ) 夕陽

색-

zb60) 위 에 나타난 세 어의 특 으 적절하 않은

것은

① 룡 어 주격 사에 당 는 가 사( ) lsquo rsquo六龍

고 다

② 샤 어에도 어 주체 쓰 다

는 것 다

③ 매 어 달리 사 택에 어

가 지 지지 고 다

④ 므 원 상 직 어 지 다

⑤ 드시리 다 주체 과 상 께 사

고 다

수고 하셨습니다hearts hearts

년 학 간고사 대비2013 2 현대고 대비

ECN-0102-2013-001-000076193

보닷컴에 공 는 별 보는 고등

들 여 주 는

들 습니다 슷 동 지

가 복 는 것 도가

니 복 여 습 시고 거 시

니다

정답 해설

1) 정답[ ] ④

해설 다른 것은 두 특정 업이나 단 내에서 사[ ]

하는 일종의 은어 사회 언에 해당한다 러나

는 언이 아니라 단과대학을 여서 단대 사lsquo rsquo lsquo rsquo lsquo④

대학을 여서 사대라고 한 말에 해당하 일rsquo lsquo rsquo

사회에서도 널리 쓰이 사회 언이라 할

없다

2) 정답[ ] ⑤

해설 사회 언은 같은 단 내에서 쓰이는 언어이[ ] lsquo rsquo

동일 단끼리는 단결 과 친 감을 형성하는

능을 하 리적 안감이 일어나 않는다

3) 정답[ ] ③

해설 사람이라는 차 적 표현에 대한 대안적 표현이[ ]lsquo rsquo

인 아내 처 등으 볼 있다lsquo rsquo

4) 정답[ ]⑤

해설 남성은 주 격 체를 사 한다[ ]

5) 정답[ ] ⑤

해설 흑인은 검다라는 뜻을 가 고 있을 뿐 인[ ]lsquo rsquo lsquo rsquo lsquo rsquo

다 열등한 뜻을 내포하 않는다

6) 정답 살 색 첫 작품[ ] - -

해설 살색 혹은 킨색은 한 인의 피 색을 뜻[ ] lsquo rsquo lsquo rsquo

하는 것으 인종 차 을 추 고 출 이주민

의 평등 을 침해할 있어 년 표 이2005

살 색으 이름을 꾸었다 처녀작은 처녀라lsquo rsquo lsquo rsquo lsquo rsquo

는 단어가 가 고 있는 곡된 성 인 을 한 것

으 첫 작품정도 꾸어 사 하는 것이 좋다lsquo rsquo

7) 정답[ ] ⑤

해설 호는 아들에게 해체를 사 하고 있다[ ] ① ②

장 을 성하는 청자는 자 의 아 느리 아lsquo

들 세 이다 호는 아 느리에게 해rsquo ③

체를 사 하고 있다 호가 느리 아 에게 ④

사 한 해 체 아들에게 사 한 해체는 두 비lsquo rsquo lsquo rsquo

격 체에 해당한다 호는 자 의 아랫사람인 ⑤

느리에게 아들과 마찬가 해체를 사 하는 것이

상 이 만 임 을 한 느리에게 고마 과 쁨

존 의 표 를 하 위해 자 의 아 에게 말하듯

해 체를 사 하고 있다

8) 정답[ ] ③

9) 정답[ ] ⑤

10) 정답[ ] ①

해설 청자 할아 가 장의 주체 아 다 높을[ ] ( ) ( )

경 에는 압존 에 의해 장의 주체를 높이 않는lsquo rsquo

다 러 아 서가 아닌 아 는으 계 lsquo rsquo lsquo rsquo lsquo

니다 가 아닌 있 니다 표현하는 것이 르rsquo lsquo rsquo

11) 정답 당이 당을 쫒았다 당이[ ]

당에 다

해설[ ]

12) 정답[ ] ⑤

해설 서 다른 높임표현을 통해 청자에 대해 리[ ] ⑤

적 거리감을 나타내는 인 은 이 아니라 현정이

다 가 에서 현정은 에게 해 체를 사 함으 써 ( )

친근감을 드러낸다 나 에서 연 을 게을리하는 역 ( )

도 들 때 에 화가 난 현정이 선생님에게 항의하

는 장 에서는 하 체를 사 하여 리적 거리lsquo rsquo

가 어졌음을 나타내고 있다

13) 정답[ ] ①

해설 는 는 얼 빛이 날과 어찌 다르 고[ ] lsquo rsquo

라는 뜻으 전과 달리 임이 화자를 않고

있음을 알 있다

14) 정답 달리 후 가 있다 이를 통해 경[ ] lt gt

쾌한 음악성을 형성하고 노 젓는 상황을 체적으

형상화하는 역할을 한다

15) 정답[ ] ①

16) 정답[ ] ⑤

해설 다 의 자연은 를 성찰하게 하는 대상[ ] ( )⑤

이자 정의 대상이다 의 자연은 자 의 상황과 ⑤

처 를 드러내는 경으 서의 역할을 하 이

이 없다

17) 정답[ ] ③

해설 는 빈천 을 해결하고자 했으나 강산[ ] lsquo ( )rsquo 貧賤③

과 풍 을 달라는 에 거절하 다고 함으 써 자

연에 대한 애정을 드러내고 있으 는 않는

임에 대한 망을 개에게 전가 켜서 임에 대한 리

을 드러내고 있다

18) 정답[ ] ③

년 학 간고사 대비2013 2 현대고 대비

ECN-0102-2013-001-000076193

19) 정답[ ] ⑤

해설 고상한 음악가의 이름을 리말 꽝 럽[ ]

게 꿈으 써 언어유희를 통해 음을 유 하고 있

다 이는 고상한 척하는 총 를 비꼼으 써 비판적

태도를 드러내는 것이 대상을 꽝 럽게 표현

하여 총 의 허 과 사치를 풍자하고 있다

20) 정답[ ] ⑤

해설 는 작품 속 경에 대한 설 이 드러나는 것이[ ]

서 자의 주 적인 견해가 접적으 드러나는 것이

아니다

21) 정답[ ] ⑤

22) 정답[ ] ②

23) 정답[ ] ④

24) 정답[ ] ①

해설 적강 티프는 주인공의 비 한 출생이나 능[ ] ①

과 이 있는 것으 조정의 능함을 풍자하는lsquo rsquo

것과는 거리가 다

25) 정답 픔 나[ ] ( )

해설 의 음악은 고통 는 사람들을 위 하고 아픔[ ] lsquo rsquo

을 치유해 주는 능을 한다고 할 있다 의 lt gt

픔 도 소 된 이 과 더 어 살아가는 따뜻한 마음lsquo rsquo

을 상 한다

26) 정답[ ] ⑤

해설 에게 선천적으 주어 각 장애라는 역경[ ]

은 의 이라는 가사 연 을 있다lsquo rsquo

27) 정답[ ] ④

해설 는 장 란 선 에게 은 개인적인 인상을[ ]

소녀 장정 등으 표현한 것이다lsquo rsquo

28) 정답[ ] ②

해설 담자가 피 담자의 언어적 표현이나 비언어[ ]②

적 표현 하 독자는 담의 위 나 피

담자의 감정 상태를 알 있다 이를 통해 독자는

담 상황을 더 생생하게 느낄 있고 피 담자

를 더 잘 이해할 있게 된다

29) 정답[ ]③

해설 일상생활과 역도 선 서의 성과에 된 것에서[ ]

역도를 하 서 겪는 어 과 내적 고민으 화제를

전화하 위한 것이다

30) 정답[ ] ①

해설 릿속에 새겨 넣듯 이 억되도 함 세상[ ] ② ③

살이가 힘들고 고생 러 속 하여 자유를 ④

가 없는 고통의 상태를 비유적으 이르는 말

적의 침입을 막 위해 쌓은 축 켜야 할⑤

대상을 비유적으 이르는 말이다

31) 정답[ ] ④

해설 이 의 종류는 전 으 인 사건 경[ ] lsquo

비평을 성 소 삼는다rsquo

32) 정답[ ] ④

해설 근은 삼대독자 태어났음을 에서 확인할[ ]

있다 형제들과의 담은 이뤄 가 없다

33) 정답[ ] ⑤

해설 근은 가난에도 하고 화가를 꿈꾸었다[ ] (3

단 또한 다른 화가 망생들은 정 육을)

위해 상 학 학 해 유학 에 랐 만

근은 다른 을 찾아야 했다 단 세에(5 ) 18

근은 조선 전람회에 입선하 다 단 의(6 )

만종은 인간과 자연이 엮어 가는 경건한 조화 을lsquo rsquo

나타낸다

34) 정답[ ] ①

해설 근이 속에서도 창작활동을 추 않고[ ]

하는 닭은 은 세상과 타협할 르는

근이 세상의 이해를 하 위한 가장 떳떳한 단

이 때 이다

35) 정답[ ] ⑤

해설 전 은 서 자의 주 적인 평이 리는 것이[ ]

만 위 제 은 인 이 살았던 대 사회적 경

을 통해 객 적인 인 의 을 제 하고 있다

36) 정답[ ] ⑤

해설 전 은 인 사건 경 비평이라는[ ] lsquo rsquo⑤

성 이 어져 있다

37) 정답[ ] ①

해설 이 은 동양인과 서양인의 사고 에 차이가[ ]

있다는 것을 대조를 통해 설 하고 있다 또 쓴이

의 제자가 축 경 를 러 가서 경험한 일화를

통해 동양인이 서양인에 비해 주 상황에 더 많은

주의를 인다는 주장을 뒷 침하고 있다

38) 정답[ ] ④

39) 정답[ ] ②

40) 정답[ ] ②

41) 정답[ ] ④

42) 정답[ ] ③

43) 정답[ ] ④

44) 정답 도서 의 휴 일 도서 의 이 간 도서의[ ]

해설 도서 장은 임의 정한 휴 일과 도서 이[ ]

간 도서의 상 등을 게 할 의 가 있다

년 학 간고사 대비2013 2 현대고 대비

ECN-0102-2013-001-000076193

45) 정답[ ] ①

해설 제 조의 정 휴 일 의 휴 일의 사전 게[ ] 3

는 도서 장의 의 조항에 속한다

46) 정답[ ] ①

해설 개인 정 호 의 를 제 하 했 만 항[ ]

나눠서 제 하 않고 대 나열하고 있다

47) 정답[ ] ②

해설 제 조의 내 을 회사는 다른 회사 협[ ] 7 lsquo

계약을 통해 서비 를 제공하는 경 회 의 아이디

등 개인 정 를 해당 회사에 전송할 있다는 내rsquo

이 있으 의 제점을 제 할 있다②

48) 정답[ ] ④

해설 는 도서 장의 의 에 해당하고 나 는 도[ ] ④

서 장의 리에 해당한다

49) 정답[ ] ③

50) 정답 은 음독으 적었고 은 훈독으 적었[ ] (1)

다 과 동일한 표 리 적은 것은 이고 (2) ce

과 동일한 표 리 적은 것은 이다ab

51) 정답[ ] ③

52) 정답[ ] ①②

53) 정답[ ] ③

54) 정답[ ] ③

55) 정답[ ] ①

56) 정답 른 죠코 어린 노 하니라[ ] A B

57) 정답 세 어에서는 활 형이 칙적으[ ] lsquo rsquoㄹㅇ

나타났 만 개화 어에서는 활 형이 쓰 다 lsquo rsquo ㄹㄴ

58) 정답 호 가 흔[ ] (1) (2)

59) 정답[ ] ④

60) 정답[ ] ③

Page 2: 현대고대비 국어 - chamsoriedu.com 「콘텐츠산업진흥 법」외 에도 저작권 의하여 ... 다른주체에게어떤동작을하도록만드는것을나타내는

년 학 간고사 대비2013 2 현대고 대비

ECN-0102-2013-001-000076193

가 과 등 가 운 곳에 실

언어 별

만들어

울 가다 울 가다ex) rarr

살색 ( )rarr

( )rarr

직 태도3)

동( )和而不同 태도 지

zb6) 과 에 적절한 말을 쓰

zb7) 에서 장 리적 태도의 화 높임 표lt gt

현의 계가 적절한 것은

보lt gt

마루S

느리 신 식 들 시 지

시열 에 계신다

말 다 말 우리 지

첫 신 탄 게 어 냐

우리 꿔 우리

시 지 감사 말 우리 집 에

경사 어 님 지 에 가 는 시

느리 거 사 없 말 드 릴게

마 어 님도 지 뻐 실 거 늘 편

시고 내 가 감 사 드리러 갈게 마 편

시고 몸 심 느리 지가 편 (

에게 꾼다 어) (

는다 지 들 죠)

시열 그 들었다 게 경사냐 도 우리

집 징 가 보다 허허허

① 시열과 는 랫사 들에게 체 사 고

② 사에 는 청 는 다 4

③ 는 지 느리에게 심리 태도 드러

내 십시 체 사 고 다

④ 는 지 들 느리에게 격식체 상

사 여 심리 거리감 드러내고 다

⑤ 는 느리에게 신 것에 고마움 심리 태

도 드러내 사 고 다

zb8) 다음 의 에 대한 설 으 적절하lt gt - 않

은 것은

lt gt

아 채민이 너 공 열 히 - 하는 나

채민 에휴 넌 내 마음 라 hellip

채민 엄마가 나에게 큰 을 어 이 에 또 성적이 떨

어 휴대폰 빼앗아 리 대 러 나는 눈 이

맺힐 것 같아

아 전 코너에 네

채민 러게 말이야 전히 쫒 는 마음으 공 하

고 있어

① 주어가 동 는 주동 다

② 주어가 다 주체에게 어 동 도 시키고

③ 주어 극 태도가 드러 다

④ 진 주체 동 고 볼 없다

⑤ 동 미사 사 동 다 lsquo- -rsquo

가 어 상에 거 낮

는 주체1) 2) 상 객체 3)

등 다 주동사동 주어가 직 동 거

다 주체에게 어 동 도 만드는 것 타내는

다 능동 동 주어가 동

거 다 주체에 어 동 당

타내는 다

zb9) 다음 친 주체 높임 표현 성격이 가장 이 적

인 것은

① 그 살림 시

② 그 가 없 십니다

③ 그 님 직도 귀가 시다

④ 생님 에 가 편 시지

⑤ 지 주에 시골에 사시는 니께 다

zb10) 다음 높임 이 른 것은

년 학 간고사 대비2013 2 현대고 대비

ECN-0102-2013-001-000076193

① 지 지는 고 습니다

② 지 지께 는 고 계십니다

③ 지 지께 는 고 계시 니다

④ 사 님 에 에 갔습니다

⑤ 사 님 님께 는 에 에 가 습니다

가( ) S1

소장 자네에게 을 게 되어 정말 안하네 자네에게 뒷

일을 맡겨야겠네

나 자 네 여 는 당입니다 한때( ) 1 당에 쫓

도 했 니다 하 만 선거 후 전에 의 있는 정책

으 선거전을 서 당 위 를 끌어 린 것이

당을 고 리를 이끈 인으 고 있 니다

자 네 여 는 당입니다 선거 후 전 터 당2

위 가 한풀 이 서 세에 었는데 일 의

들의 눈에는 눈 이 맺혀 있 도 합니다

다 과량 사 액이 삼켜져서 위장 흡 에 의한 전( )

작 도 일어날 있 니다 따라서 액이 삼켜 않도

주의해야 합니다

라 통의 날을 맞아 늘 에서는 통사고 절 으( ) lsquo rsquo lsquo

이 캠페인을 내 곳곳에서 다채 게 펼쳤 니다 늘rsquo

행사는 많은 민들의 호응을 이끌어 냈 니다 계

자는 제 통사고 생률을 낮출 있는 체적인 정책도

마 하고 있다고 말했 니다

마 통의 날을 맞아 휴일인 늘 내 곳곳에서는 다양한( ) lsquo rsquo

통의 날 행사가lsquo rsquo 개최되었 니다 써 터 통의 날 lsquo rsquo

행사가 생색내 에 나 않는다는 비판이 제 되고 있 니

다 체적인 정책적 뒷 침이 마 되 않는다 이러한 비

판의 소리는 점점 더 커 것으 전망됩니다

zb11) 다음 조건 에 맞게 나 의 을 능동 으 을lt gt ( ) lsquo rsquo lsquo rsquo

피동 으 고쳐 각각 한 장으 서 하

조건lt gt

은 이가 을를 다 조의 능동 으1 lsquo- ( ) - ( ) - rsquo

은 이가 에 다 조의 피동 으2 lsquo- ( ) - - rsquo

에 사 할 단어는 나에서 찾아3 lsquo-rsquo ( )

서 어의 제 높임은 상 없이 장을 만들 것4

가 역도 앞 낮( ) S 38 ( )

냉장고를 열고 펄떡거리는 어들이 들어 있는 낚

망을 내는 고 를 매 탕과 함 맛

있게 을 고 있는 과 아이들 인다

현정 적 적 만날 맛난 거 어 좋 는 허 만 이제 나( )

도 값을 해야 헐튼디 맨날 이 만 하고 선생님

리 역도는 언제 가르쳐 대

당황해서 둘러대듯 초 체 이 한 ( ) helliphellip

거야 많이 어 근엄한 말투 역도는 일단 이 을 ( )

해야 할 있는 거다 략 lt gt

나 학 동장 치 낮( ) S 40 ( )

여 후 에게 똥침을 놓고 도망간다 장난 럽게 달

리 를 하고 있는 역도 들 치에 앉아서 아이들을 라

는 에게 다가 는 현정

현정 주장으 서 할 말 있 니다 훈 위 고 장난쳐도

혼내 도 않으 고 리고 역도 는 여 이처럼 꼬 꼬

한 아이들만 뽑 니

아이들을 딱하잖냐 래도 자들이 착하다 주장 ( )

이 이해 좀 해

답답한 표정 는 현정

zb12) 가 나 의 장 을 이 는 소에 대한 설 으 적( ) ( )

절하 않은 것은

① 가 청 는 동 다( ) ( )

② 가 시간 상 낮 다( ) ( )

③ 가 에 공간 상 달 다( ) ( )

④ 가 에 심리 태도는 다 게( ) ( )

타 다

⑤ 가 에 지 다 통( ) ( )

에 심리 거리감 타낸다

가( )

가 각시 본 듯도 고

상( )天上 경( )白玉京 엇 니별 고( ) 離別

년 학 간고사 대비2013 2 현대고 대비

ECN-0102-2013-001-000076193

다 믄 눌 가시 고

어 여 고 내 드러 보

내 얼 거동 님 가마

엇 보시고 다 실

도 님 미 업

어 러 돗

시 엇 다 신고

누어 각 고 니러 여 니

내 몸 지 죄 뫼 티 시니

원망 사 허믈 랴

워 니 믈 타시 다( ) 造物

각 마

략( )

님다 쇼식 므 쟈 니( )消息

도 거 다 사 가

내 업다 어드러 가쟛 말고

거니 거니 뫼 가니

니 개 므 고

산쳔 어 거니( ) (A)山川 월( )日月 엇 보

지쳑 거든 쳔리 보랴( ) ( ) 咫尺 千里

리 믈 가 보쟈 니

믈결 어 고

샤공 어 가고 만 걸 니

강 쟈 보니( )江天

님다 쇼식 득 고( )消息

략( )

근 역진 간 드니( )力盡

지극 님 보니( )精誠

얼 마 늘거( ) ( ) 玉 半

근 말 슬 쟈 니

눈믈 니 말 들 어

못다 여 니( )情

계 ( )鷄聲 엇 돗 고

어 허 다 님 어 간고( ) 虛事

결 니러 열고 보니( )窓

어엿 그림재 다

리 싀여 여 월( )落月

님 겨신 ( )窓 드시 리

각시님 니 쇼

미 곡- lt gt-

( )

내 님믈 그리 우니다니

(B)산 동새( )山 슷 다

니시 거 신

월 시리 다( ) 殘月曉星

시 도 님

벼 시니 뉘러시니 가

과 도 허믈도 만 업 다( ) ( ) 過 千萬

마리신

니미 마 니 시니 가

님 도 드 샤

과- lt gt-

zb13) 가 의 에 대한 설 으 적절하( ) ~ 않은 것은

① 과 달리 고 다

② 별 신 탓 여 고 다

③ 별 다 가 고 다

④ 산에 는 망 취 지 못 lsquo rsquo

⑤ 실 룰 없는 망 실 시키고 신

다 재 변 고 다

사( ) 5春詞

고 볃티 믉결 다

어 어

그믈 주어 랴 시 가

지 지 어( ) ( ) ( )至匊悤 至匊悤 於思臥

탁 가 니 고 도 니 다( ) ( )濯纓歌 興

사( ) 2夏詞

닙 싸 고 쟝만 마

닫 드러 닫 드러

청 립 사 가 냐( ) ( )靑蒻笠 綠蓑衣

년 학 간고사 대비2013 2 현대고 대비

ECN-0102-2013-001-000076193

지 지 어( ) ( ) ( )至匊悤 至匊悤 於思臥

심 내 가 가( ) ( )無心 白鷗

사( )3秋詞

운 니러 고 티 니 다( )白雲

돋 돋

믈 믈 동 가쟈( ) ( )西湖 東湖ㅣ

지 지 어( ) ( ) ( )至匊悤 至匊悤 於思臥

료 곳마다 경 다( ) ( )白蘋紅蓼 景

동사( )4冬詞

간 눈 갠 후 에 경믈 달 고( ) ( )後 景物

어 어

만경 리 듸 쳔 산( ) ( )萬頃琉璃 千疊玉山

지 지 어( ) ( ) ( )至匊悤 至匊悤 於思臥

계 가 계 가 간 니 다( ) ( ) ( )仙界 佛界 人間ㄴ ㄴ

zb14) 위 을 의 조 형태적 특 에서 어lt gt (1)

떤 차이가 있는 를 서 하고 인해 노래의 아 (2)

름다 에 어떤 향을 주었는 에 대해 서 하

보lt gt

십 경 여 가삼간 지어내니

달 청 맡겨 고

강산 들 곳 없 니 러보고 보리

가 가 각시 본 듯도 고( )

샹 경 엇 니 별 고天 上 白 玉 京 離 別

다 믄 눌 가시 고

어 여 고 내 사 드러 보

내 얼 거동 님 가마

엇 보시고 다 실

도 님 미 업

태 어 러 돗

시 낫 엇 다 신고

누어 생각 고 니러 여 니

내 몸 지 죄 뫼가티 싸 시니

원망 사 허믈 랴

워 니 믈 타시 다造 物

님다 쇼 식 므 쟈 니( ) 消 息

도 거 다 내 사 가

내 마암 둘 대 업다 어드러 가잔 말고

잡거니 거니 놉픈 뫼해 라가니

은 카니 안개난 사 일고

산 쳔이 어둡거니山 川 일日 月 을 엇디

쳑을 라거든 쳔 리랄 라 랴咫 尺 千 里

찰하리 가의 가 히나 자 하니

람이야 결이야 어둥졍 된뎌이고

샤공은 어대 가고 븬 만 걸 나니

강 텬의 혼쟈 셔셔 디난 해랄 니江 天

님다히 쇼 이 더 아득한뎌이고消 息

다 쳠 리 듕만 도 니( ) 茅

벽 쳥 등 눌 갓 고半 壁 靑 燈

라 나리 헤뜨 니니

져근덧 녁 하야力 盡 풋잠을 잠간 드니

졍 셩이 하야 꿈의 님을 니精 誠

가탄 얼 이 이나마 늘거셰라玉 半

마암의 근 말삼 카장 삷쟈 하니

눈 이 라 나니 말인들 어이하

졍을 다하야 이조차 여하니情

뎐된 계 셩의 잠은 엇디 돗던고鷄 聲

라( ) (A)찰하리 여디여 낙 이나 되야 이셔洛 月

님 겨 창 안해 드 비최리라窓

(B)각 님 달이야카니 잔 비나 되쇼셔

zb15) 위 가 라 에 대한 설 으 적절하( )-( ) 않은 것은

① 민들 과 삶 담고 다

② 여 체 식 어 다

③ 사미 곡과 께 가사 미 다

④ 우리말 사가 평가

년 학 간고사 대비2013 2 현대고 대비

ECN-0102-2013-001-000076193

고 다

⑤ 님 미 사 는 마lsquo rsquo

가사 다

가 춘사( ) [ 5]

고은 티 쬐얀 결이 름 다

이어라 이어라

을 주어 두랴 낙 노흘일가

총 총 어( ) ( ) ( )至匊葱 至匊葱 於思臥

탁 가 의 흥 이 나니 고 도 니 다( ) ( )濯纓歌 興

하사[ 2]

년닙희 두고 찬으란 쟝만 마라

닫 드러라 닫 드러라

청약립 은 써 잇노라 녹사의 가져 냐( ) ( )靑蒻笠 綠蓑衣

총 총 어( ) ( ) ( )至匊葱 至匊葱 於思臥

내 좃 가 제 좃 가( ) ( )無心 白鷗

추사[ 3]

이 니러나고 나 티 흐느 다( )白雲

돋 라라 돋 라라

의 셔호 혈 의 동호 가쟈( ) ( )西湖 東湖ㅣ

총 총 어사( ) ( ) ( )至菊悤 至菊悤 於思臥

빈홍 곳마다 경 이 다( ) ( )白蘋紅蓼 景

동사[ 4]

간 의 눈 갠 후 에 경 이 달랃고야( ) ( )後 景物

이어라 이어라

압희 만경류리 듸희 쳔텹 산( ) ( )萬頃琉璃 千疊玉山

총 총 어( ) ( ) ( )至菊悤 至菊悤 於思臥

선계 가 블계 가 인간 이 아니 다( ) ( ) ( )仙界 佛界 人間ㄴ ㄴ

윤선도 어 사 사- ( )漁父四時詞「 」

나 살어리 살어리랏다( ) 쳥산 애 살어리랏다( ) 靑山

위랑 래 랑 고 쳥산 애 살어리랏다 ( ) 靑山

얄리얄리 얄랑셩 얄라리 얄라

러라 러라 새여 자고 니러 러라 새여

널라 름 한 나도 자고 니러 니 라

얄리얄리 얄라셩 얄라리 얄라

청산별곡- ( )靑山別曲

다 청산은 엇뎨 야 만고애 프르르( ) ( ) ( ) 靑山 萬古

유 는 엇뎨하야 주야애 디 아니난고( ) ( )流水 晝夜

리도 치디 마라 만고상청 호리라( ) 萬古常靑

도산 이곡- ( )陶山十二曲

라 개를 여라 이나 르되 개 치 얄 랴( )

님 꼬리를 홰홰 치 치뛰락 나리뛰락 겨서 내

닷고 님 뒷 을 동 동 르락 나으락 캉캉

도리 암캐

이 릇 릇 날 들 너 이 이 랴

작자 상-

마 빈천 을 랴 고 에 드러가니( ) ( ) ( )貧賤 權門

침 업 흥졍을 뉘 져 쟈 리

강산과 풍 을 달나 니 는 리 리

조찬한-

정정 이랬거니 아람도리 큰 솔이 혀( ) ( )伐木丁丁

도 하이 골이 어 아리 소릴 쩌르 돌아 도 하이 다

람 도 좇 않고 새도 않어 은산 고 가 차라리

뼈를 저리 는데 눈과 이 종이 담 회 나 달도 름을

다 흰 뜻은 한 이골을 걸음이란다 절 이 여섯

판에 여섯 고 고 라간 뒤 조찰히 늙은 사나이의 남

내음새를 는가 름은 람도 일 않고 고 에 히

흔들리 노니 견디란다 차고 연 히 픔도 꿈도( )兀然

없이 장 산 속 겨 한 내-

정 장 산- 「 」

zb16) 다 에 나타난 상과 표현을 활 해 작 연 을 해( )

것이다 적절하 않은 것은

① 몰 는 도에 몸 맡 고 는 고통

욱 답게 가꾸 는 느님 산 겠지

② 돌 틈 뚫고 어 민들 보 리 진 고통

다가 도 민들 럼 강 게 살 지 겠는가

③ 벽에 런 움 없 어지고 는 폭포

여 시 는 죽 워 지 는 강

지 는가

④ 상 겨울 도 다리 꿋꿋 살 가고

겠지 망 지 는 삶 얼마 다운가

년 학 간고사 대비2013 2 현대고 대비

ECN-0102-2013-001-000076193

⑤ 막 늘 울리고 는 귀 미 리는

지새우는

가( ) 빈천( )貧賤을 랴 고 에 드러가니( )權門

침 업 흥졍을 뉘 져 쟈 리

강산과 풍 을 달나 니 는 리 리

나 청강 에( ) ( )淸江 비 듯는 소 어 읍

만산 홍 이 휘드르 는고야( )滿山紅綠

두어라 춘풍 이 날이리 을 어라( ) 春風

다 청산은 어찌 야 만고에 푸르르( ) ( ) ( )靑山 萬古

유 어찌 야 주야애 디 아니 고( ) ( )流水 晝夜

리도 치 말아 만고상청 하리라( ) 萬古常靑

라( ) 개를 여라 이나 르되 개 치 얄 랴

님 리를 홰홰 치 치 락 리 락

겨서 내닷고 고 님 뒷 을 동 동

르락 나으락 캉캉 도리 암

이 릇 릇 날 들 너 이 이 랴

zb17) 의 적 능을 비 한 내 으 가장 적절한

것은

① 는 가 지닌 실 계 다

② 는 내 갈등 심 시킨다

③ 는 가 는 상 각시킨( )愛着

④ 는 달리 내 갈등 시킨다

⑤ 는 달리 다 사 들 간 가

강 다

가 님다히 쇼 을 아 나 아쟈 니( ) ( )消息

도 거의 다 일이나 사 가

내 둘 업다 어드러 가쟛말고

잡거니 거니 놉픈 뫼 라가니

은 니 안개 일고

산쳔 이 어둡거니 일 을 엇디( ) ( )山川 日月

쳑 을 거든 쳔 리 라 랴( ) ( )咫尺 千里

하리 의 가 히나 쟈 니

람이야 결이야 어둥졍 된뎌이고

샤공은 어 가고 븬 만 걸 니

강텬 의 혼쟈 셔서 디 니( )江天

( )

쳠 자리의 듕만 도라 니( )茅簷

쳥등 은 눌 위 야 갓 고( )半壁靑燈

리 헤 니니

져근덧 녁 야 픗 을 잠간 드니( )力盡

졍셩 이 야 의 님을 니( )精誠

가 얼 이 이나마 늘거셰라( ) ( )玉 半

의 근 말 장 쟈 니

눈 이 라 나니 말인들 어이

졍 을 다 야 이조차 여 니( )情

뎐된 계셩 의 은 엇디 돗던고( )鷄聲

어 허 다 이 님이 어 간고( )虛事

결의 니러 안자 창 을 열고 라 니( )窓

어엿븐 림재 날 조 이 다

하리 여디여 낙 이나 되야이셔( )落月

님 겨 창 안 드 비최리라( )窓

나 내 님 리자 니다니( )

산 접동새 난 이 이다

아니 거츠르 아으

잔 효성이 아 리이다

넉 라도 님은 녀져라 아으

더 니 뉘러 니잇가

과도 허 도 천만 없소이다

힛마리 뎌

읏븐뎌 아으

니 나 마 니 니 잇가

아소 님하 도람 드르샤 쇼셔

zb18) 맥으 아 에 들어갈 적절한 것은

① 시 엇 다 신고

② 다 믄 눌 보 가시 고

③ 님다 쇼식 욱 득 고( )消息

④ 원망 사 허믈 랴

년 학 간고사 대비2013 2 현대고 대비

ECN-0102-2013-001-000076193

⑤ 죠 뫼 티 시 가( ) ( ) 粥早飯 朝夕

가( )

거리

공신 후 심 늦도( )劉尋

식 없어 과 께 산에 드리고 신

태몽 꾼 에 만고 웅 상 지닌 들

낳 키운다 그 후 신 들 에 역심 ( )逆心

담 귀 등 심 여 리 귀

보내고 지 죽 는 도망 가다

가 만 죽 고 에 경 가는 들 도움

살 다 그러 사 에 심 귀

보고 담 여 고 강 주가 승상

득 여 고 신 사 삼는다 그 후 강 승상

에게 심에 상 리지만 여움

사 귀 가게 다 강 승상 몸 는

연 과 헤어 리 다

경쇠 리 들리 에 들어가니 색

에 게 단청 누각과 큰 집들( )丹靑

다 주 보니 ( ) (一柱門 黃金

산 사 어 었다 산) lsquo rsquo 大字

들어가 고승 다 그( ) ( ) 山門 高僧

거동 보니 눈 눈 듯 고

변 같 귀는 어 에 늘어 니( ) 白邊

맑고 어 골격과 신 평 니었

팔염주를 에 걸고 육환장 을 고서 흑포( )六環杖

장삼에 떨어 송낙 쓰고 나 유생을 고 말[ ] 松蘿

하 를

소 이 연 하여 유상공 는 행차를 동 에 나ldquo

가 맞이하 하 으니 소 의 함을 서하

rdquo

유생이 크게 놀라 말하 다

천한 인생으 팔자가 하여 어 서 를 잃고ldquo

정처없이 다니다가 연히 이곳에 대사를 만난 것인데

토 대하 소생의 성은 어떻게 알고 있 니 rdquo

노 이 답하여 말하 를

어제 남악 형산 의 화선 이 소 의 절에ldquo ( ) ( )男樂 衡山

어서 소 에게 탁하 를 내일 낮 경에 남경 lsquo 12

동성 안에 사는 유 의 아들 충 이가 것이니 내쫓

말고 잘 대접하라 하셨 니다 마침 소 이 찾아 나rsquo

다가 상공의 차람새를 니 남경 사람이 에 알아

았 니다rdquo

유생이 말을 듣고 한편으 쁘고 한편으 퍼하

서 노 을 따라 들어가니 여러 들이 합장 하

가 했다 노 이 에 들어가 저녁 을 은 후에

을 편히 니 이곳은 선경 이었다 세상의 일을( ) 仙境

두 잊고 일 이 편안하 다 이후 는 노 과 함

서 도 이 탐 하고 경도 확하게 의 게 되었( )兵書

다 이 게 되니 대 천 에 가객 은 없 ( ) ( )大明天地 佳客

고 덕산 속에 리 른 만 있더라 래 ( ) 廣德山

이 천상 사람으 살아 있는 처를 만나 이한

을 니 재주 민함을 누가 당할 있겠는가

낮으 공 하더라

유충 전- -

웬늠 어가 사 싸다냐( ) ldquo rdquo

내가 가 막 런거 니

보통것 닐러 그 어낸ldquo ( )

틀어 주 그 가 루 러 허 에

싶어 키 틀어 주 그 가( )

루 허 우간 곡 틀어주는 루 못 는

웂는 고 닝께 고 지 들

어 사는 고 가 다는 건 에 그 집에

rdquo

그런 단 어들 어 새벽에 떼죽 거

다 고 어 보니 죄다 허 게 집어진

는 것 었다

총 가 내화를 꿴 뛰어나 만 아 소 없는

일이었다

어떻게 된 거야ldquo rdquo

한동안 넋나간 듯이 서 있던 총 가 하고많은 사람

에 하필이 유자를 겨냥하 은 말이었다

쎄유 아마 새에 고뿔이 들었던 개비네유ldquo rdquo

유자는 러 딴청을 하 다

야 고 가 에서 감 가 들어 죽는 고 두ldquo

어rdquo

총 는 가 혐의자 나 되는 것처럼 화풀이를( )嫌疑者

하 드는 것이었다

는 비위가 상해서

야 팔자가 사나서 이런 후 에 살라니|

여러 가 다 객고가 쌓여서 조 두 안 좋았을 테 helliphellip

런디다가 릇쓰 이 가락을 트는 대 디립

다 춰댔으니 과 해서 살끼두 다소 있었을 테 helliphellip

래 들어서 키 는 새끼덜일 이 다다 탈이 많은

이니 ldquohelliphellip

는 트의 독성을 충 히 내 않고 고 를 넣

은 것이 탈이었으 니 하 서도 러 참으 의 을 떨

었다 략 - -

마리가 마리 값 간다는 워ldquo

그냥 내뻔지 거시 허 싼 고 는 맛

겄다 싶 허 게 눌 강 어helliphellip

허 마늘 통 다

년 학 간고사 대비2013 2 현대고 대비

ECN-0102-2013-001-000076193

게 지 고뿌 지 rdquo

어 어째ldquo rdquo ldquo rdquo

런 도 것들 같 니ldquo ( ) rdquo殘忍無道 helliphellip

는 탱 여 지 못 다 보( ) 憤氣撐天

니 는 는 다 동원 여 통 쳤

생각 여 는 눈 다

달리 리헐 감ldquo rdquo

들 고 말 니었다 그가

는 것 그 말고는 없었 에 그 게 뒷동

달 거 다

는 우 럽고 식 짝 없는 랫것들 고

다 공연 신 가고 득 것

없다고 단 는지 결 웬만큼 고루 어

그 것들 쪽 에다 고 어주지ldquo

고 그 그걸 주 어 에 에 helliphellip

눈 없는 독 들 rdquohelliphellip helliphellip

고 말 럼 얼거리 들어가 리는 것 었

- ( ) -兪子小傳

zb19) 위 나 를 읽고 평가한 것으 적절하( ) 않은 것

① 사 리 통 감과 사실 고

격 과 달 고

② 는 가 재 컫는lsquo rsquo lsquo rsquo

미 가진 여 는 것 겠

③ 는 식 말 는 웃 상lsquo rsquo

여 는 미 지니고

④ 는 어가 죽 짐 지만 내색 지 고lsquo rsquo

말 고

⑤ 언어 통 가들 여 우리 통

것들 역 고

가 체 거리( ) [ ]

나라 종 연간에 정언주 의 을 하고( )正言注簿

있던 유 은 늦도 자 이 없어 한탄하다가 남악 형산lsquo rsquo

에 치성을 드리고 이한 태 을 꾼 뒤 아들을 낳아 이름

을 충 이라 고 키 다 이때 조정의 하들 에 역

을 품은 정한담 최일 등이 가달의 침입에 대한( ) 逆心

유 의 유화적 입장을 제 삼아 유 을 함하여 양

내고 유 의 에 을 러 충 자마저 살해하

한다 러나 충 은 천 조 정한담의 마 에서 어

나 많은 고난을 겪다가 은퇴한 재상 강희주를 만나 사위

가 된다 강희주는 유 을 하 고 상소를 으나 정

한담의 공격을 아 양을 가게 되고 강희주의 가족은

난을 피하여 두 흩어 다 충 은 강 소저 이 하고

사의 노 을 만나 를 때를 다린다 이

때 남적과 적이 를 들고 나라에 쳐들어 자 정한

담은 자 출전하여 남적에게 항 하고 남적의 선 장이

되어 천자를 공격한다 정한담에게 여러 패한 천자가

항 하 할 음 충 이 등장하여 남적의 선 정 걸

을 죽이고 천자를 출한다 충 은 단 으 란 을

쳐 고 정한담을 사 잡는다 리고 호 에게 ( )胡王

잡혀간 황후 태후 태자를 출하 유 에서 고생하

던 아 유 과 장인 강희주를 한다 또한 이 하

던 어 니 아내를 찾고 정한담 일파를 리친 뒤 높은

에 라서 화를 누린다

사 들 별 고 없 다니었다( )

마 마 돌 다니 걸 여 고

어 곤 다 에는 동쪽에 고

에는 쪽에 니 가 에 리는 엽

가는 없 니 늘 다니는 었

다 얼 말 죽 사 같고 림새가 말

니었다 가슴 에 고 등

삼태 헌 에 니 달 ( )奇男子

가 도리어 걸 었 담 만 열 도 ( )傅說

고 만났고 만 갈( ) ( ) 慇 武丁

도 탕 만났( ) ( ) (伊尹 成湯 渭

여상 도 주 만났는) ( ) ( ) ( ) 水 呂尙 周 文王

월 같 러가 도 어느 열 살

늘과 집 삼고 사 에 쳐 거리에

어 다가 곳에 니 다 ( ) 楚

지 다가 사 보고 가에 다다( )長沙

니 망 가에는 원 리가 슬 고 가

가 내리는 사 에는 갈매 가 갈 뿐 었다

쪽 돌 보니 가 우거 고

가 사 보 었다 그곳에

가니 는 사( ) 汨羅水

는 다 주 가 쓰고 죽고

곳 었다

마 감 여 에 가 사 살펴보니

에는 삼 고 그 에( ) 屈三閭

는 만고 월 과 지 가는 그 들( )風月

가 어 었다( ) 路程記

동쪽 벽 에 새 운 어 거늘 그

보니 월 에 경 주 는 간신에게ldquo ( )敗

보고 연경 귀 가다가 에 죽 rdquo

거늘 그 보고 에 거꾸러

통곡 말

[A]ldquo우리 연경 간 만 니 에

지 살 상에 엇 겠는가

에 고 에 었 니

상에 살 것 가 도 께 지리 rdquo

년 학 간고사 대비2013 2 현대고 대비

ECN-0102-2013-001-000076193

고 가에 내 가니 울 리가 에 지 사

쳤는지 심 심 것 가

신 심 것 가

다( ) 강 승상에게는 들 없고 다만 만

었다 가 낳 에 가 색

타고 내 에게 말 는 ldquo

니다 미원 과 연 맺고 ( ) ( )紫薇垣 緣分

었는 께 강 집 보내 에

니 게 여겨 주십시 거늘 rdquo

미 가운 낳 니 가 고 거동

단 다 시 짓 쓰 고 는 (音

없었 니 여 가운 지 는 짝) 律

룰 만 사 없었다 가 사 여 사 감

게 고 지 못 고 염 는 만다

다가 당에 거 고 식같 러 내니

고귀 상 루 말 다 어 울 도 다( ) 相

귀 사 없고 웅 걸( )富貴爵祿

만고 었다 승상 매우 뻐 내당 ( )內堂

들어가 에게 사 니 역시 매우 거

워 말 다

ldquo 도 마 사 는 승상께

그 게 말 시니 상 여러 말 지 말고

사 도 시다rdquo

상이 에 나 충 의 손을 잡고 결혼과 하여 ldquo

너에게 히 할 말이 있다 내가 늙은 말년에 딸

하나만을 두었는데 니 너 하늘이 정해 필

임이 하다 이제 년고락 을 너에게 탁 ( )百年苦樂

하겠다 하 대 충 이 릎을 꿇고 앉아 눈 을 흘리rdquo

여쭈었다

소자의 을 해주 고 또 하 에 두고자 하ldquo ( )膝下

니 감사하 이를 데가 없 니다 다만 가 속에 통탄

할 일이 사 쳐 있 니다 소자가 이 없어 양친 ( )兩親

의 생사를 른 채 결혼하여 아내를 얻는 것은 자 으

서 할 도리가 아닙니다 이것이 한 러 뿐입니다 rdquo

승상 그 말 듣고 슬 에 어 고

것 에 맞 어 변 게 리ldquo

는 다 집 시 공 도 여 ( )始祖公

고 가 에 가가 어진 만 개 공신

었 니 도 러워 말 시고 시 rdquo

택 여 니 다운 신 과 신

습 늘에 죄 짓고 간 상에 내 신

혼 를 다 끝내고 으 들어가 사 을 살펴 니 빛

나고 빛난 것이 한 입으 는 다 말하 어 고 하나

는 다 하 어 더라 에 켠 환한 촛 ( )新房

아래 은 에 랑과 가 평생의 연 을 맺었( )緣分

으니 서 사랑하 주고 은 말을 어떻게 다 헤아릴

있으 어떻게 다 하리 을 낸 후에 이튿날

상 를 니 상 거 마음을 이 하

더라

각 생 강 승상 집 쪽( )

늘 보고 없 가 신 신 생각 니

없고 어 없었다 는 어떻게 도리가 없다

여 산 에 들어가 리 고 어 도 닦

고 다 그 산 보고 가다

가 곳에 다다 니 에 큰 산 었다 많 우

리 골짜 가 늘 는 가운 색

에 고 갖가지 가 짝 어 었 ( )花草

다 신 산 생각 고 들어가니 경개 ( )景槪

가 매우 뛰어 고 경 산 다 산 리에 들

리는 것 리 보 는 것 울 청산뿐

었다 가 고 울 어 가

니 들 많 가지들 못 어 동

에 늘어 들거리 는( ) 洞口

우거진 가지에 갖 들 다 었다( ) 春情

계상 에는 공 는 늘( ) 花溪上

에 걸린 폭포가 벽 는 리는 산사( )層巖絶壁

쇠 리 객 에 는 듯 늘( ) ( ) 寒山寺 客船

에 싸여 는 습 산

그린 여 병 러 듯 다 경쇠 리가 들

리 에 들어가니 색 에

게 단청 누각과 큰 집들 다( ) 丹靑

주 보니( ) ( ) lsquo一柱門 黃金大字

산 사 어 었다 산 들rsquo ( )山門

어가 고승 다 그 거 보니( ) 高僧

눈 눈 듯 고 변 같 ( )白邊

귀는 어 에 늘어 니 맑고 어 골격

과 신 평 니었다 염주

에 걸고 짚고 포 삼에 어진( )六環仗

쓰고 생 보고 말

승 연 여 상공 시는 동 에ldquo

가 맞 지 못 니 승 십시 rdquo

생 크게 말 다

생 가 여 어 고ldquo

없 다니다가 우연 곳에 사 만 것

그 시 생 어떻게 고 습니 rdquo

승 답 여 말

어 산 승 에ldquo ( ) ( )南岳 衡山

시어 승에게 탁 내 낮 시경에 경 lsquo 12

동 에 사는 심 들 가 것 니 내쫓

지 말고 습니다 마 승rsquo

다가 상공 림새 보니 경 사 에 보

습니다rdquo

zb20) 위 의 친 에서 서 자의 개입이 드러나~

는 이 아닌 것은

① 달 가 도리어 걸 었( ) 奇男子

② 신 심 것 가

년 학 간고사 대비2013 2 현대고 대비

ECN-0102-2013-001-000076193

③ 다운 신 과 신 습 늘에 죄 짓고

간 상에 내 신 다

④ 사 주고 말 어떻게 다 헤 릴

어떻게 다 리

⑤ 신 산 생각 고 들어가니 경개 가 ( )景槪

매우 뛰어 고 경 산 다

거리 연[ ] ( )弘治

간 에 공신 후 에 언(1488~1505) ( )正言

주 는 벼슬 심 늦도 식( ) ( )主簿 劉尋

없어 과 께 산에 드리고 신 태

몽 꾼 에 만고 웅 상 지닌 들 낳

키운다 그 후 신 들 에 역심( )逆心

담 귀 등 심 여 리 귀 보내

고 지 죽 는 도망 가다가

만 다 에 에 어 니

헤어지게 다

에 에 어 니 헤어지게

다 그 후 사 들에게 우연 돌

생 다가 어느 열 살 었다 열 살

지 다가 우연 귀 견 는

그것 그 살 도 었고 그

귀 본 신도 지 죽고 마 고

크게 운다

( )

에는 강 주 는 재상 살고 었

니 시 에 과거에 격 여 승상 벼슬 다가 간

신 만 벼슬 그만 고 고 돌 었

다 그러 신 지 가 지 못 여 상

가 못 결 는 상 여 원 니

신 들 그 직간 꺼 다 그 에 도

담과 귀가 강 승상 가 미워 다

강 승상 마 본 에 갔다가 돌 는[A][ ( )本府

에 우편 주 에 다가 색( ) ( ) 右便 酒店

에 어리었는 청룡 에 지 늘

여 통곡 고 사 는 꿈 꾸었다] 마

상 게 생각 여 새 다리다가 새벽

닭 울고 가 달 갔다 가 보니

과연 어 동 가 가에 울고 는지 달

들어 그 고 사 에 어 말

는 어 어 에 어 가ldquo

닭 곳에 우느냐 니 울rdquo

그 고 답 여 말 다

는 경 동 에 사는 언 주 공 들ldquo

니다 께 간신 만 연경 귀 가

시다가 에 죽 사 에 는 닭에

도 에 죽고 니다rdquo

강 승상 말 듣고 크게 낯 변 말

것 웬 말 냐 근 동 ldquo (老

못 갔 니 그 사 변 여)患

런 변 었단 말 가 주 는 신 다

같 에 벼슬 다가 는 가 많 들어 고

돌 는 주 가 게 꿈 에 생

각 겠느냐 생각지 못 다 미 지 간

지지 말고 께 가 략rdquo ( ) hellip hellip

죽게 주 사당에 단 도 러운

겠느냐 말 말고 시는지 rdquo

어 없어 강 승상 가니 그곳

월계 었다

다( )

가가 고 지 사 들 가( )櫛比

통 는 리가 과

답게 꾸민 누각과 큰 집들 늘 고

게 식 가 어 들 태운 가고

었다 략 강 승상에게는 들 없고 ( ) hellip hellip

다만 만 었다 가 낳 에

가 색 타고 내 에게 말

는 니다 미원 과ldquo ( )紫薇垣

연 맺고 었는 께 강 집( )緣分

보내 에 니 게 여겨 주십시

rdquo

거늘 미 가운 낳 니 가

고 거동 단 다 시 짓 쓰 고

는 없었 니( )音律 여 가운

지 는 짝 룰 만 사 없었다 가 사

여 사 감 게 고 지 못 고 염 는 만다

다가 당에 거 고 식같

러 내니 고귀 상 루 말 다 ( )相

어 울 도 다 귀 사 없 ( )富貴爵祿

고 웅 걸 만고 었다 승상 매우 뻐

내당 들어가 에게 사 니( ) 內堂

역시 매우 거워 말 다 도 마 ldquo

사 는 승상께 그 게 말 시니

상 여러 말 지 말고 사 도 시다rdquo

( )

승상 에 고 결 과 ldquo

여 에게 말 다 내가 늙 말 에 지

만 었는 지 보니 늘

다 에게 탁 겠 ( )

다 신 꿇고 눈 리rdquo

여 었다 주시고 슬 에 ldquo ( )膝下

고 시니 감사 룰 가 없습니다 다만 가슴

에 통탄 사 쳐 습니다 복 없어

생사 결 여 내 얻는 것( )兩親

식 도리가 닙니다 것 러울 뿐 니

다rdquo

상 그 말 듣고 슬 에 어 고 말

것 에 맞 어 웅변 ldquo

년 학 간고사 대비2013 2 현대고 대비

ECN-0102-2013-001-000076193

게 리 는 다 집 시 공도 여

고 가 에 가가 어진 만 개 공신

었 니 도 러워 마 시고 rdquo 시

택 여 니 운 신 과 신

습 늘에 죄 짓고 간 상에 내 신

다 략 지낸 후에 튿 승상 ( ) hellip hellip

니 승상 거운 마 지 못

마( )

듯 월 러 생 열다 살 었

다 에 승상 어진 사 얻고 만 에 근심 없었

다만 주 가 간신 에

죽 것 생각 마 곧 어 곤

다 그 에 주 원통 어

없 고 여 시 가 거늘 생 만

여 다

말 감격 러우 간신 에 가득 여ldquo

고 니 께 상 듣지 니 것

니다rdquo

승상 듣지 고 가

퇴 재상 공달 집에 거 고 상 지어

승지 러 께 리

( )

뒷 거리 강 승상 에게 상 리지[ ]

만 여움 사 귀 가게 다 강 승상

몸 는 연 가 헤어

리 다 산 들어간 룡사 승 만

게 다 승 만 우 다릴

과 들고 략 다 담

원 여 에게 복 고 어

공격 다 담에게 여러 가( ) 天子

복 등 여 다 단

신 리쳐 담 사 고 에게

간 후 태후 태 여 지에 고생

지 심과 강 주 여 개 다 헤

어 어 니 내 고 담 리

벼슬에 귀 누리게 다

zb21) 위 의 인 간 계를 같이 나타냈을lt gt

때 에 대한 이해 가장 적절하 ~ 않은 것은

① 계에 주 는 계 심 열

상 에 다고 다( ) 水深火熱

② 계는 견원지간 고 다( ) 犬猿之間

③ 계는 달리 막역지 계 고( )莫逆之交

④ 연결 사 컬어 재 가 고( )才子佳人

⑤ 는 생 과 볼 ( )匹夫匹婦

가 재 는 는 심 고 매사에 생( )

각 고 능 도 어 가 에게 많lsquo rsquo

도움 사 다 그는 에게 거 에

꺼리 없 거 났다고 는

매우 싫어 고 신 들

는 사 다

내가 지 리에( ) 1970

사 실에 지 월간ldquo

편집 고 어 었다rdquo

어느 없 가 쑥 다 도 어 10

후 다 산 시 럼 어 엇 어 ( ) lsquo怡山

다시 만 랴 니 그는 재 그룹 승 운rsquo

사가 고 는 고 거 누

주는 가 없는 가가 어 다시 만 게 것

었다

다 보통 것 닐러 그( ) ldquo 어낸 ( )

틀어주 가 루 러 허 에

싶어 키 틀어주 그( )

가 루 허 우간 곡 틀어 주는 루 못

는 는 고 닝께 고 지

들어 사는 고 가 다는 건 에 그 집에

rdquo

그런 단 어들 어 새벽에 떼죽 거

다 고 어 보니 죄다 허 게 집어진

는 것 었다 가 실내 꿴 뛰어 지만

없는 었다

어떻게 된 거야 한동안 넋나간 듯이 서 있던 총ldquo rdquo

가 하고많은 사람 에 하필이 유자를 겨냥하 은

말이었다 쎄유 아마 새에 고뿔이 들었던 개비네 ldquo

유rdquo

유자는 러 딴청을 하 다 야 고 가 에서 ldquo

감 가 들어 죽는 고 두 어rdquo 총 는 가 혐의

자 나 되는 것처럼 화풀이를 하 드는 것이었다( )嫌疑者

라 이 어쩌 어 유( ) ldquo rdquo ldquo rdquo

애유 이런 잔인 도 한 것들 같으니ldquo ( ) rdquo殘忍無道 helliphellip

총 는 탱천 하여 쩌 를 하 다( ) 憤氣撐天

아하니 아는 자는 다 동 하여 호통을 쳤으 하나 혈

압을 생각하여 참는 눈치 다 달리 처리헐 두 ldquo

잖은감유rdquo

총 의 성 을 덧들이 고 한 말이 아니었다 가 할

년 학 간고사 대비2013 2 현대고 대비

ECN-0102-2013-001-000076193

있는 것이 말고는 없었 때 에 게 뒷동

산을 달은 거 다

이 유자소전- lsquo rsquo

zb22) 의 상황을 속담으 표현한 것으 적절한 것은

① 루 곳 게 마 다

② 에 맞고 강에 눈 다

③ 늘 도 다

④ 도 사 다

⑤ 에 가도 신만 리 다

거리 공신 후[ ]

에 주 는 벼슬 심 늦도( )主簿

식 없어 과 께 산에 드리고 신

태몽 꾼 에 만고 웅 상 지닌 들

낳 키운다 그 후 신 들 에 역심

담 귀 등 심 여 리 귀 보내고

지 죽 는 도망 간다 그

만 고 에 에 어 니

헤어지게 다 지 가 사 들에

사 들 별 고 없 다니었다

마 마 돌 다니 걸 여 고

어 곤 다 에는 동쪽에 고 에

는 쪽에 니 가 에 리는 엽 가는

없 니 늘 다니는 었다

얼 말 죽 사 같고 림새가 말 니었

다 가슴 에 고 등 삼태

헌 에 니 달 가 도리 ( )奇男子

어 걸 었 담 만 열 도 ( ) ( )傅說 慇

고 만났고 만 갈( ) ( )武丁 伊尹

도 탕 만났( ) ( )成湯 渭水

여상 도 주 만났는 월( ) ( ) ( ) 呂尙 周 文王

같 러가 도 어느 열 살

늘과 집 삼고 사 에 쳐 거리에

어 다가 곳에 니 다 ( ) 楚

지 다가 사 보고 가에 다다( )長沙

니 망 가에는 원 리가 슬 고 가

가 내리는 사 에는 갈매 가 갈 뿐 었다

쪽 돌 보니 가 우거 고

가 사 보 었다 그곳에

가니 는 사( ) 汨羅水

는 다 주 가 쓰고 죽고

곳 었다

마 감 여 에 가 사 살펴보니

에는 삼 고 그 에( ) 屈三閭

는 만고 월 과 지 가는 그 들( )風月

가 어 었다( ) 路程記

동쪽 벽 에 새 운 어 거늘 그

보니

월 에 경 주 는 간신에게ldquo ( )敗

보고 연경 귀 가다가 에 죽 rdquo

거늘 그 보고 에 거꾸러

통곡 말

우리 연경 간 만 니ldquo ( )燕京

에 지 살 상에 엇 겠는

가 에 고 에 었 니

상에 살 것 가 도 께 지리 rdquo

고 가에 내 가니 울 리가 에 지

사 쳤는지 심 심 것 가

에는 강 주 는 재상 살고 었

니 시 에 과거에 격 여 승상 벼슬 다가 간

신 만 벼슬 그만 고 고 돌 었

다 그러 신 지 가 지 못 여 상

가 못 결 는 상 여 원 니

신 들 그 직간 꺼 다 그 에 도

담과 귀가 강 승상 가 미워 다 강 승상 마

본 에 갔다가 돌 는 에 우편 주( ) ( )本府 右便

에 다가 색 에 어리었는 청룡( ) 酒店

에 지 늘 여 통곡 고

사 는 꿈 꾸었다 마 상 게 생

각 여 새 다리다가 새벽닭 울고

가 달 갔다 가 보니 과연 어 동 가

가에 울고 는지 달 들어 그

고 사 에 어 말

는 어 어 에 어 가ldquo

닭 곳에 우느냐rdquo

니 울 그 고 답 여 말 다

는 경 동 에 사는 언 주 공 들ldquo

니다 께 간신 만 연경 귀 가

시다가 에 죽 사 에 는 닭에

도 에 죽고 니다rdquo

강 승상 말 듣고 크게 낯 변 말

것 웬 말 냐 근 동ldquo ( )老患

못 갔 니 그 사 변 여 런 변

었단 말 가 주 는 신 다 같

에 벼슬 다가 는 가 많 들어 고 돌

는 주 가 게 꿈 에 생각

겠느냐 생각지 못 다 미 지 간 지지

말고 께 가 rdquo

뒷 거리 강 승상 도움 죽 고[ ]

년 학 간고사 대비2013 2 현대고 대비

ECN-0102-2013-001-000076193

고 그 과 결 여 사 가 다 그러 강

승상 에게 울린 상 강 승상 귀 가고

과 헤어 리 승 만 게 다

승 우 다릴 과

들고 략 다 담 원

여 에게 복 고 어 (天

공격 다 담에게 여러 가 복) 子

등 여 다 단신

리쳐 담 사 고 에게 간

후 태후 태 여 지에 고생 지

심과 강 주 여 개 다 헤어

어 니 내 고 담 리 벼

슬에 귀 누리게 다

미상- lsquo ( )-劉忠烈傳

zb23) 위 과 의 서사 조를 비 한 것으 적절하lt gt

않은 것은

보lt gt

믿지 고 결 여 곱

낳 다 곱째 공주 낳 가

리게 다 리 만 고 진 공주는 lsquo rsquo

리공 미 리공 에 키워진다 월

러 과 가 죽 병에 걸 는 승에 는

어 산다고 다 여 들에게 탁

지만 거 리 는다 리 는 과

승 다 승 지 가는 에 많

만 지만 보살 도움 사 도 다

그러 승 신과 결 여 시

들어 주겠다고 다 리 는 그 결

여 들 곱 낳 후에 신

얻게 다 돌 리 는

에 과 상여 만 지만 여 과

살 낸다 훗 리 그 공 우 죽 사

승 도 는 신 다

리-lsquo rsquo-

① 복 결말에 고 다

② 웅 에 탕 고 다

③ 시 겨 내고 귀 누리는lsquo rsquo

보 리 는 월 재 신 다lt gt lsquo rsquo

④ 과 보 리 는lsquo rsquo lt gt lsquo rsquo

도움과 어 신 능 극복 고

⑤ 등 여 시 겪는lsquo rsquo

보 리 는 닌 지lt gt lsquo rsquo

림 시 겪는다

가 각 고 에( ) ( )却說

살 없었다 략 사 들 슬 에 어 lt gt

가에 내 고 가고 싶 가 고 후

워 경

사 들 별 고 없 다니었다 lt

략 얼 말 죽 사 같고 림새가 말gt

니었다 가슴 에 고 등

삼태 헌 에 니 달 가 ( )奇男子

도리어 걸 었 담 만 열 도( )傅說

고 만났고 만 갈( ) ( ) 殷 武丁

도 탕 만났( ) ( ) (伊尹 成湯 渭

여상 도 주 만났는) ( ) ( ) ( )水 呂尙 周 文王

월 같 러가 도 어느 열 살

늘과 집 삼고 사 에 쳐 거리에

어 다가 곳에 니 다 ( ) 楚

지 다가 사 보고 가에 다다( )長沙

니 망 가에는 원 리가 슬 고 가

가 내리는 사 에는 갈매 가 갈 뿐 었다

쪽 돌 보니 가 우거 고

가 사 보 었다 그곳에

가니 는 사( ) 汨羅水

는 다 주 가 쓰고 죽고

곳 었다

에는 강 주 는 재상 살고( )

었 니 시 에 과거에 격 여 승상 벼슬 다

가 간신 만 벼슬 그만 고 고 돌

었다 략 강 승상 마 본 에 갔다가 돌 lt gt ( )本府

는 에 우편 주 에 다가 색( ) ( ) 右便 酒店

에 어리었는 청룡 에 지

늘 여 통곡 고 사 는 꿈 꾸

었다 마 상 게 생각 여 새 다리다

가 새벽닭 울고 달 갔다 가

보니 과연 어 동 가 가에 울고 는지

달 들어 그 고 사 에

어 말

는 어 어 에 어 가ldquo

닭 곳에 우느냐rdquo

니 울 그 고 답 여 말 다 lt

략gt

년 학 간고사 대비2013 2 현대고 대비

ECN-0102-2013-001-000076193

생각 여 가 고 시 는ldquo ( )大人

상에 다시없는 니다 살 엇 겠습니

에 돌 가시고

가에 돌 가 니 살 마 없습니

다 략 어 없어 강 승상 가니rdquo lt gt

그곳 월계 었다

다 강 승상에게는 들 없고 다만 만( )

었다 가 낳 에 가 색

타고 내 에게 말

는 니다 미원 과ldquo ( )紫微垣

연 맺고 었는 께 강 집( )緣分

보내 에 니 게 여겨 주십시

rdquo

거늘 미 가운 낳 니 가

고 거동 단 다 시 짓 쓰 고

는 없었 니 여 가운( ) 音律

지 는 짝 룰 만 사 없었다 가 사

여 사 감 게 고 지 못 고 염 는 만다

다가 당에 거 고 식같 러

내니 고귀 상 루 말 다 어 ( )相

울 도 다 귀 사 없고 ( )富貴爵祿

웅 걸 만고 었다 승상 매우 뻐 내

당 들어가 에게 사 니 역( ) 內堂

시 매우 거워 말 다

도 마 사 는 승상께ldquo

그 게 말 시니 상 여러 말 지 말고 사

도 시다 략 시 택 여rdquo lt gt

니 다운 신 과 신 습 늘에 죄

짓고 간 상에 내 신 다

다 내고 들어가 사 살펴보니

고 것 는 다 말 어 고

는 다 어 신 에 ( )新房

에 신 과 신 가 평생 연 맺었( )緣分

니 사 주고 말 어떻게 다 헤 릴

어떻게 다 리 지낸 후에 튿 승

상 니 승상 거운 마 지 못

( ) 듯 월 러 생 열다 살

었다 에 승상 어진 사 얻고 만 에 근심

없었 다만 주 가 간신

에 죽 것 생각 마 곧 어

곤 다 그 에 주 원통

어 없 고 여 시 가 거늘 략 lt gt

략 거리

강 승상 에게 상 리지만 여움

사 귀 가게 다 강 승상 몸 는

연 과 헤어 리 다

마 각 생 강 승상 집 쪽( )

늘 보고 없 가 신 신 생각 니

없고 어 없었다 는 어떻게 도리가 없다

여 산 에 들어가 리 고 어 도 닦

고 다 그 산 보고 가

다가 곳에 다다 니 에 큰 산 었다 많

우리 골짜 가 늘 는 가운 색

에 고 갖가지 가 짝 어 ( )花草

었다 략 주 보니 lt gt ( ) (一柱門 黃

산 룡사 어 었다) lsquo rsquo 金大字

산 들어가 고승 다 그( ) ( ) 山門 高僧

거동 보니 눈 눈 듯 고

변 같 귀는 어 에 늘어 니( ) 白邊

맑고 어 골격과 신 평 니었

다 염주 에 걸고 짚고 포 ( )六環杖

삼에 어진 쓰고 생 보고 말

승 연 여 상공 시는 동 에ldquo

가 맞 지 못 니 승 십시 rdquo

생 크게 말 다

생 가 여 어 고ldquo

없 다니다가 우연 곳에 사 만 것

그 시 생 어떻게 고 습니

rdquo

승 답 여 말

어 산 승 에ldquo ( ) ( )南岳 衡山

시어 승에게 탁 내 낮 시경에 경 lsquo 12

동 에 사는 심 들 가 것 니 내쫓

지 말고 습니다 마 승rsquo

다가 상공 림새 보니 경 사 에 보

습니다rdquo

생 그 말 듣고 편 고 편( )

슬 승 들어가니 여러 승 들

가워 다 승 에 들어가

후에 그 편 니 곳 경 었다 상( ) 仙境

고 신 편 다 후 는 승과

께 병 도 탐 고 경도 게( )兵書

게 었다 게 니 지 에 가객 ( ) ( )大明天地 佳客

년 학 간고사 대비2013 2 현대고 대비

ECN-0102-2013-001-000076193

없고 산 에 리 만 본 ( ) 廣德山

신 상 사 살 는 만

우고 늘 월 신 과 늘 ( )日月聖神

산 신 들 다 니 그 재( ) 名山神靈

주 민 누가 당 겠는가 낮 공

zb24) 다 에 해당하는 내 으 적절하( ) 않은 것은

① 강 티 통 당시 능 다

② 상계 지상계 경 는 원 계 드러

③ 실에 어 없는 실 가 타 는

④ 뛰어 재주 어 가진 고

등 다

⑤ 가 직 개 여 평가 내리는

편집 평 타 다lsquo rsquo

가 본격 가 동 것 지( )

다 단 상 에2003 lsquo rsquo

들어가 드럼 연주 다 취미 생 달리

들었다는 보 우 가 들ldquo

어 틱 린 도 다 고 말 다rdquo

경 는 가 망 없( ) lsquo

티 원 고 답 다 신과 같 시각rsquo

는 습 상상 만 도 감동

다 시각 연주 동시에

열 상 는

티 원 그런 열 경 럽다는 것 다

다 역시 엄청 다 본( )

에 복 들

고쳐 가고 다 신 에 얼

마 지는 고 리가 는 지도 생님

가 훈 고 많 고쳐 다

고 말 다

그러 직도 에 지 는 다 그는

체격 지 못 게 가 큰 만

체 운동 훈 과 께 체 늘 동 50

는 게 고 말 다

에게는 꿈 다 통 누 가( )

주겠다는 것 그 꿈 다 신 극복 는

과 에 큰 경험 들도 느 게

주고 싶다는 것 다

마 슬 마다( ) ldquo 통

낼 었 것 럼 고통 는 사 들

고 겠다 고rdquo

말 다 달 루 첫 낸 lsquo rsquo

첫 드 심 집에 는 리듬 드 2

루 에 도 보고 싶다 집 에는 직(RampB) 3 4

사 곡 도 보 고 싶다고 포 다middot

zb25) 에서 가장 유사한 의 를 닌 어를lt gt

찾아 쓰

lt gt

나는 이제 너에게도 픔을 주겠다

사랑 다 소 한 픔을 주겠다

겨 거리에서 개 놓고

살아 추위 떨고 있는 할 니에게

값을 으 서 뻐하던 너를 위하여

나는 픔의 평등한 얼 을 여 주겠다

내가 어둠 속에서 너를 를 때

단 한 도 평등하게 어주 않은

가마니에 덮인 동사자가

다 얼어 죽을 때

가마니 한 장조차 덮어주 않은

한 너의 사랑을 위해

흘릴 르는 너의 눈 을 위해

나는 너에게 이제 너에게도 다림을 주겠다

지 울 포동 여고 생들17

틈 없 가득 체 에 맑 울

다 죽 듣 생들 사 에

연 는 탄 다 객들 도 는lsquo rsquo

가 보 주 공 맹 가 운 는

단 그룹사운드 루 보컬 맡고 는lsquo rsquo

시각 지 었다17 1

근 다만과 가 거lsquo rsquo lsquo

꿈 고 퇴 내가 다rsquo

간 간에 지 지 연 생들 짧lsquo rsquo lsquo rsquo

가 운 듯 리에 어

연 다 내 사 고 퇴lsquo rsquo

과 루 들 결 다시 돌lsquo rsquo

들 고 사 들 에 당당

것 니다 내 태어

볼 없었 크고 열여

년 학 간고사 대비2013 2 현대고 대비

ECN-0102-2013-001-000076193

에도 고 시 얻지 못 다

감지 없는 시각 상태 다

신 지에 고 상 원망 도

단다 어느 가 에 시각 에 ldquo

어 그런 듣고 다 보니 내가 게 lsquo

살 는지 도 눈 고 싶rsquo lsquohelliphellip

보 는 생각만 들 고 그 가 들에게rsquo

도 내고 들도 고 많 었죠 들 rdquo

었 지 새 는 에 쑥 러운 색

어났다

생에 것 단연 었다lsquo rsquo

공연에 거 꿈lsquo rsquo

는 다 특 가사 갑게 는 운 lsquo

벽 에 당당 마주 어 언 가 그 벽

고 늘 어 거운 상도

없죠 내 삶 에 웃 그 께

는 다고 다rsquo

들었 그냥 런 도 고만 여ldquo lsquo rsquo

겼죠 그런 꾸 가사 미 새 다 보

니 통 는 가사 는 생각 들 고 (

가 게는 시각 는 생각 들고 들) ( )

마다 듣고 큰 얻었어 rdquo

에 진지 게 가에 미 가

zb26) 의 에 들어갈 말 적절한 것은lt gt ~

lt gt

난 난 꿈이 있었죠

고 찢겨 남 하여도

내 가 히 과 같이 간 했던 꿈

혹 때 누 가가 뜻 를 비 음

내 등 뒤에 흘릴 때도

난 참아야 했죠 참을 있었죠

날을 위해

늘 걱정하듯 말하죠

헛된 꿈은 독이라고

세상은 끝이 정해 책처럼

이 돌이킬 없는

현 이라고 helliphellip

래 난 난 꿈이 있어

꿈을 믿어

나를 켜

저 차갑게 서 있는 이란 앞에

당당히 마주칠 있어

출처 가 거위의 꿈 작사 이적 작곡 동률- lsquo rsquo ( )

① ② ③ ④ ⑤

가 떴다는 들 만 지만( ) lsquo rsquo

늘 겸 다 에 주 연 우승 지 간에도 3

단 생님께 만 지 고 고 만ldquo rdquo

큼 늘 겸 신 계 가

고 다

에게는 꿈 다 통 누 가

주겠다는 것 그 꿈 다 신 극복 는 과

에 큰 경험 들도 느 게 주

고 싶다는 것 다

슬 마다 통 낼ldquo

었 것 럼 고통 는 사 들

고 겠다 고rdquo

말 다 달 루 첫 낸lsquo rsquo

첫 드 심 집에 는 리듬 2

루 에 도 보고 싶다(RampB) 집 에는 직34

사 곡 도 보 고 싶다고 포 다

미 는( ) (26) 어 헤헤헤 웃다가 어ldquo rdquo

허허허 웃었다ldquo rdquo ldquo rdquo 같 도 고

상 다 는 같 도 다( ) 壯丁 킹 들lsquo

다 는 역도 보 그 다 지만 그는rsquo

뷰에 지 다 운동만 지 ldquo

것 지 간에 여러 사 도 역rdquo helliphellip

었다 그런 엇 그 마 움직 는지 보 쯤

지 담 사 다 훈 없어 그는 티

지 림 었다 태 다 갔다 는 습

마 집 럼 편 게 보 다

주말에는 주 엇 보내

주말에도 별 주 에 청ldquo

고 에 가고 도 쳐

에 듣고 보 에 갈 가 별 없

어 산 시 게 고 들어 2002

거 매 여 지냅니다 시 과 지훈 rdquo

다 근 간 과 진실 그리고 싶어( )

가 다 근에게 그것 진리 다 거 다 없

거 고 다 없 는 것 진리

다 근 진리는 후 쪽 었다 신산( )辛酸 삶

었 질곡( )桎梏 역사 에 지냈 가

눈에 든 것 료 단 료 게 보

것 었다 그것 그 에 겨우겨우

슬 슬 생 어가는 간들 었다

리 과 단 리 고리에 검 마

없 거리 돌

상 것 없는 등 근에게 상

과 진실 엄 ( )儼存 다는 사실 리는 가

실 고 가 과 역경 에 도 근 내 포

없었 후 보루( )堡壘 다 도 365

도 간 근 여

시 것 다

년 학 간고사 대비2013 2 현대고 대비

ECN-0102-2013-001-000076193

다 공주 그림 가 근 경- ( ) ldquo rdquo(

2009)

zb27) 작가의 주 적인 각이 드러난 것은~

① ② ③ ④ ⑤

가 신 지에 고 상 원망( )

도 단다 어느 가 에 시각 에 ldquo

어 그런 듣고 다 보니 내가 lsquo

게 살 는지 도 눈 고 싶rsquo lsquohelliphellip

보 는 생각만 들 고 그 가 들에게rsquo

도 내고 들도 고 많 었죠 들었rdquo

지 새 는 에 쑥쓰러운 색

어났다 략 [ ]

경 는 가 망 없 티lsquo

원 고 답 다 신과 같 시각rsquo

는 습 상상 만 도 감동

다 시각 연주 동시에

열 상 는 티

원 그런 열 경 럽다는 것 다 략 [ ]

슬 마다 통 낼ldquo

었 것 럼 고통 는 사 들

고 겠다 고rdquo

말 다 달 루 첫 낸 lsquo rsquo

첫 드 심 집에 는 리듬 2

루 에 도 보고 싶다 집 에는 직(RampB) 3 4

사 곡 도 보 고 싶다고 포 다

식 누 가-

고 싶어

다 역도 미 담 고 사( )

질 주말에는 주 엇 보내[ 1]

답 주말에도 별 주 에[ ] ldquo

청 고 에 가고 도 쳐

에 듣고 보 에 갈 가 별

없어 rdquo

질 계 고 슬슬 도 는 것 닙니[ 2]

답 다 들 눈 에 보 고 뿐 보[ ] ldquo

다 열심 고 어 상에 도 들지만 상

지키는 것 들다고 에 도달

그것 지키 훨 많 rdquo

질 들 살 고 리 는[ 3]

거운 들 체 리느 는다

답 가 고 게 체 어[ ] ldquo ( ) 級

느 도 계가 니 살 는 것도 고역 지만

살 우는 것 들어 는 살

체 리 고 어도 어도 실 갔다

쑥 어 rdquo

질 거리에 슷 연 여 들[ 4]

보는 간 상 지

답 상 다 체 게 리지 못[ ] ldquo

거 주변에 는 그 거 누 보지

못 고 뻐지고 싶 에 체 리는 에

타 워 지만 는 어울 는 것보다 는

시간 운동만 는 건 니에 사복 lsquo rsquo

고 사복 는 말에 들 웃지만 늘 운동복

고 지내니 사러 갈 도 어 rdquo

질 역도가 말 단 식 운동 니[ 5]

답 가 내는 만 클 업 보[ ] ldquo

그러니 만 쓰는 식 운동 니다

만 다고 거운 것 들 는 건 니거든 연

도 고 가지 동 에 도 여러 가지

복 들

보식 역도 여 미-

zb28) 가 에 대한 설 으( ) 않은 것은

① 시각 우 지 시 에 지

고 망 가는 태도 달 고 다

② 언어 과 언어 복 사 여

담 내 생각 게 는 가

③ 직 감 그 마 것

럼 생생 게 느껴지는 과 주고 간 내

없 리 어 억 게 다

④ 담 내 식 리 여 담 삶 습

과 가 시 여 독 에게 감동과 훈 다

⑤ 직 진 담 직 누

지 못 는 독 에게 생생 상 달 주고

담 욱 게 다

zb29) 나 의 각 의 의도를 설 한 것으 적절하( ) 않

년 학 간고사 대비2013 2 현대고 대비

ECN-0102-2013-001-000076193

은 것은

① 질 담 상 보여 주 것 다1

② 질 담 과 그에 삶 태도 보여2

주 것 다

③ 질 역도 겪는 어 움에 역도3

과 것 다

④ 질 같 연 여 갖는 고민 는지 말4

주 는 것 다

⑤ 질 역도가 과 고 운동 는 것5

담 가 말 주 는 것 다

가 만진 것 다( ) 3

감 달 다고 다 억 에( ) 音感

지워 지만 당시 청 탁 리도

다고 다 드럼 웠다 4

에 갈 마다 드럼 는 리가 신 게 들

다고 다 눈 볼 가 없 니 엔ldquo

는 는 님 틱 에 여 주

다 드럼과 연 맺 과 들 주었다rdquo

식 누 가-

고 싶어

역( ) 도가 말 단 식 운동 니

가 내는 만 클 업에 보ldquo

그러니 만 쓰는 식 운동 니다 만

다고 거운 것 들 는 건 니거든 연

도 고 가지 동 에 도 여러 가지 복

들 시 는 상 상

드는 상 에 맞춰 실 에 는 여러

펼쳐집니다rdquo

략( )

늘 에 는 어 만 것 같

가 에 사 고 사 사ldquo

겠어 든 에 가 경 만 고

울 는 사 겠어 rdquo

보식 역도 여 미-

다 가 운 는 어 어( ) ldquo rdquohelliphellip

월 새벽 시 태 없 거웠고1965 5 6 1

는 없 그 병원에 퇴원 집

가는 마지막 마 고 마 내 거 다

가 죽 간신 에 실 다 사는 어느5 lsquo

가 죽 는 말 가 식 다 신rsquo

상에 각 시키는( )刻印 에 실

어느 가는 후 민 가가 근 었다lsquo rsquo

는 간 과 진실 그 다는( ) ldquo

에 단 평 견 가지고 다 내

가 그리는 간상 단 고 다 지 다 는 그들

가 에 는 평 지 니 그리고 어린

들 미지 겨 그린다rdquo

마 근 간 과 진실 그리고 싶어( )

가 다 근에게 그것 진리 다 거 다 없

거 고 다 없 는 것 진리

다 근 진리는 후 쪽 었다 신산(辛酸 삶)

었 질곡(桎梏 역사 에 지냈)

가 눈에 든 것 료 단 료 게

보 것 었다 그것 그 에 겨우겨우

슬 슬 생 어가는 간들 었

다 리 과 단 리 고리에 검

마 없 거리 돌

상 것 없는 등 근에게 상에

과 진실 엄 다는 사실 리는 가( )儼存

실 고 가 과 역경 에 도 근 내

포 없었 후 보루(堡壘 다 도)

도 간 근365

여 시 것 다

월 강원도 림리에( ) 1914 2 21

삼 독 태어났다 어 근 복

그것 그리 가지 못 다 근 곱 살

지는 산 산업에 실 고 답마 에 내

갔다 근 그림 럼 쫓 다니 가 시 것

다 상 진 것도 가 었다

러 가 에도 고 근 가 꿈꾸었다 근

가 꿈꾸게 것 보통 업

원색도1926 만lsquo rsquo 었다

공주 그림 가 근 경-

zb30) 에 대한 설 가장 른 것은~

① 역도가 과 운동 도 질

② 리는 는 다 lsquo rsquo

③ 들었지만 그럭 럭 는 다 lsquo rsquo

④ 가 게 보 시 말 다

⑤ 보 병 는 지 상 lsquo rsquo

는 말 다

년 학 간고사 대비2013 2 현대고 대비

ECN-0102-2013-001-000076193

시간 많지 다 청량리 생 병원

마지막 상 경 릿 게 들어 다 그 는 십

만 큰 가 상 말 다

지 못 들 마 갈 고 돗

도시민들 싹 싹 탔다 가 시

월에 병원에 원 가 폐 진 몸도4 ( )疲弊

갈 미 지 못 고 었다 가는 얼마( ) 解渴

지 생 에 생각 가

마감 는 신 평생 십 만에

가 과 많 닮 다고 생각 지는

가 운 는 어 어ldquo rdquo 1965helliphellip

월 새벽 시 태 없 거웠고 는5 6 1

없 그 병원에 퇴원 집 가

는 마지막 마 고 마 내 거 다 가

죽 간신 에 실 다 사는 어느 가5 lsquo

죽 는 말 가 식 다 신rsquo

상에 각 시키는 에 실 어느( ) lsquo刻印

가는 후 민 가가 근 었다rsquo

ldquo 는 간 과 진실 그 다는 에

단 평 견 가지고 다 내가 그

리는 간상 단 고 다 지 다 는 그들 가

에 는 평 지 니 그리고 어린 들

미지 겨 그린다rdquo

근 간 과 진실 그리고 싶어 가

다 근에게 그것 진리 다 거 다 없 거

고 다 없 는 것 진리다

근 진리는 후 쪽 었다 신산 삶 ( )辛酸

었 질곡 역사 에 지냈 가 눈에( )桎梏

든 것 료 단 료 게 보 것

었다 그것 그 에 겨우겨우 슬

슬 생 어가는 간들 었다 리

과 단 리 고리에 검 마

없 거리 돌 상

것 없는 등 근에게 상에 과 진실

엄 다는 사실 리는 가 실( )儼存

고 가 과 역경 에 도 근 내 포 없었

후 보루 다 도 도( ) 365堡壘

간 근 여 시 것

간에 지닌 가 근 1914 2

월 강원도 림리에 삼 독21

태어났다 어 근 복 그것 그리

가지 못 다 근 곱 살 지는 산

사업에 실 고 답마 에 내 갔다 근

그림 럼 쫓 다니 가 시 것 다 상

진 것도 가 었다 러 가 에도

고 근 가 꿈꾸었다 근 가 꿈꾸게

것 보통 업 원색1926

도 만 었다lsquo rsquo

그림 가 근 경 공주- ldquo rdquo ( 2009)

zb31) 다음 이 같은 의 성 소에 해당하 않은

것은

사건 평① ② ③

④ 주 ⑤ 경

가 운 는 어 어ldquo rdquo 1965helliphellip

월 새벽 시 태 없 거웠고 는5 6 1

없 그 병원에 퇴원 집 가

는 마지막 마 고 마 내 거 다 가

죽 간신 에 실 다 사는 어느 가5 lsquo

죽 는 말 가 식 다 신rsquo

상에 각 시키는 에 실 어느( ) lsquo刻印

가는 후 민 가가 근 었다rsquo

는 간 과 진실 그 다는 에ldquo

단 평 견 가지고 다 내가 그

리는 간상 단 고 다 지 다 는 그들 가

에 는 평 지 니 그리고 어린 들

미지 겨 그린다rdquo

근 간 과 진실 그리고 싶어 가

다 근에게 그것 진리 다 거 다 없 거

고 다 없 는 것 진리다

근 진리는 후 쪽 었다 신산 삶 ( )辛酸

었 질곡 역사 에 지냈 가 눈에( )桎梏

든 것 료 단 료 게 보 것

었다 그것 그 에 겨우겨우 슬

슬 생 어가는 간들 었다 리

과 단 리 고리에 검 마

없 거리 돌 상

것 없는 등 근에게 상에 과 진실

엄 다는 사실 리는 가 실( )儼存

고 가 과 역경 에 도 근 내 포 없었

후 보루 다 도 도( ) 365堡壘

간 근 여 시 것

간에 지닌 가 근 1914 2

월 강원도 림리에 삼 독21

태어났다 어 근 복 그것 그리

가지 못 다 근 곱 살 지는 산

사업에 실 고 답마 에 내 갔다 근

그림 럼 쫓 다니 가 시 것 다 상

진 것도 가 었다 러 가 에도

고 근 가 꿈꾸었다 근 가 꿈꾸게

것 보통 업 원색1926

도 만 었다lsquo rsquo

공주 그림 가 근 경- ldquo rdquo ( 2009)

년 학 간고사 대비2013 2 현대고 대비

ECN-0102-2013-001-000076193

zb32) 위 을 작성하는 과정에서 되어 활 된 자

어 것은

신 사 료① 연보②

고③ ④ 들과 담

⑤ 에 평

는 간 과 진실 그 다는 에ldquo

단 평 견 가지고 다 내가 그

리는 간상 단 고 다 지 다 는 그들 가

에 는 평 지 니 그리고 어린 들

미지 겨 그린다rdquo

근 간 과 진실 그리고 싶어 가

다 근에게 그것 진리 다 거 다 없 거

고 다 없 는 것 진리다

근 진리는 후 쪽 었다 신산 삶 ( )辛酸

었 질곡 역사 에 지냈 가( )桎梏

눈에 든 것 료 단 료 게 보

것 었다 그것 그 에 겨우겨우

슬 슬 생 어가는 간들 었다

리 과 단 리 고리에 검 마

없 거리 돌 상

것 없는 등 근에게 상에 과

진실 엄 다는 사실 리는 가 실( )儼存

고 가 과 역경 에 도 근 내 포

없었 후 보루 다 도 도( ) 365堡壘

간 근 여 시

것 다

간에 지닌 가 근 1914 2

월 강원도 림리에 삼 독21

태어났다 어 근 복 그것 그리

가지 못 다 근 곱 살 지는 산

사업에 실 고 답마 에 내 갔다 근

그림 럼 쫓 다니 가 시 것 다 상

진 것도 가 었다 러 가 에도

고 근 가 꿈꾸었다 근 가 꿈꾸게

것 보통 업 원색1926

도 만 었다lsquo rsquo

질 루 마 가 도 린다 경건

움 느껴지는 경 다 훗 근 그림에

과 는 거 것( )裸木

만 간과 연 엮어 가는 경건 움lsquo rsquo

니었

같 가가 고 싶었 근에게 그 꿈에 다

가가는 지 다 다 가 지망생들 규 미

상 에 진 고

에 지만 근 다 다 근

미 에 운 것 보통 시 미 시간

다 그런 그에게 없는 연습 가가

통 다 가 귀 시 지 도

얻는 뛸 듯 뻤지만 마 도 가 에

듯 는 었 에 어린 근 주 에

에 그림 그리고 지우고 복( )粉板

시간 가는 게 루 보냈다

근 그 갈 가가 것 열여( )渴求

었 다가 미1932 lsquo rsquo ( lsquo

미 에 다 다는 고 마rsquo) lsquo rsquo

가 근 집 고도 지는 시골 경

그린 그림 다 후 근 에 1943 22

지 미 에 그림 고

에 걸쳐 다 미 근 가

동 는 었다

공주 그림 가 근 경- ldquo rdquo ( 2009)

zb33) 위 의 내 과 일치하는 것은

가 근 가 꿈 포 다①

근 당 가들과 께 에 다②

살 근 가 걷20③

게 었다

④ 만 통 근 역경 겨내는lsquo rsquo

느 다

⑤ 근 간 과 진실 그리 에 그 에

드러 는 간상 단 다

계 시 주 근 건강

걸었다 신 과 간에 상 다 건강

신 는 눈에도 다 근 쪽 눈 뿌 게

보 지 과에 다 다 시 지지 고 결

내 었다 시 지만 마 막막

다 늦어 결 근 쪽 눈 고 말 다

쪽 눈 근에게는 쪽 눈 었고

계 었다 그 근 는 여 그lsquo rsquo

다 근 에 같 그림 그 었다1950

시 그림 는 여 쪽lsquo rsquo

고 어 마주 고 는 그림1963

여 과 동 다 마 복

그린 듯 눈 내리 새 게 다 지

사 다 근 게 복 것

복 상과 타 는 근 상

가 떳떳 단 었고 근 그리고

간 과 진실 에 다가가 가 근다

운 었다 근 신에게 당당 지 그리고

그 다 근 그림에 단 복 보다

년 학 간고사 대비2013 2 현대고 대비

ECN-0102-2013-001-000076193

태 도 그리고 극 보다 과

얻 여 었다 과 통

근 그리고 는 재 고 에 질

만들고 특 것 다

공주 그림 가 근 경- ldquo rdquo( 2009)

zb34) 의 이유에 대해 추 한 것으 적절하 않은 것

상과 타 시도①

보다 과 얻②

근 신에게 당당 지③

④ 간 과 진실 에 다가

⑤ 태 도 얻

근 가가 었지만 그 다니 가

럼 어지지 다 복과 쟁 거쳐 시

는 가 근에게 생계 사 에

운 사 다 에 키에 건( ) 178cm死鬪

체 근 에 동 역 업( )荷役

가 생계 다 쟁

에는 동에 운 상우 주 미

죄 사 에 그림 그리는 시 다 그곳에

에 동 역 업 것에

결 것 럼 보 다 지만 그런 것만도

니었다 그림 그리는 고는 지만 매 근

는 극 간 과 별 없는 경 리 그림

벽에 그리는 것 었다 우도 리 없었다 근

트 는 우 그림 그 다 생

계 그림 단 것 다

후 근 지 신 계 리에 미

엑 리 겼다 근 곳에

건 사 크 에 미 들 ( )

상 상 그 다 근 갖 다 겪

냈다 그리고 결 그 돈

신동에 어 사리 집 마 다 마 ㄷ

루 심 쪽에는 과 엌 쪽에는 건

었다 건 주고 근 가 에

여 살 다 심 에는 지 집어

쓰고 지만 곳 근 가 에게 러웠

보 리 다 근 과 마루 업실 삼 그림

그 다 신동 마루는 근 그림에 등 는 lsquo rsquo

같 상들 지 다 시 고

에 들 폐허가

가 업실 었다

공주 그림 가 근 경- ldquo rdquo( 2009)

zb35) 위 에 대한 설 으 적절한 것은

업 시 여 훈과 감동 다①

에 주 평 드러 다②

사 사 등 식 과 ③

④ 다 근거 시 여 삶에

⑤ 살 시 사 경 께 여

습 시 다

가 시간 많지 다 청량리 생 병원( )

마지막 상 경 릿 게 들어 다 그 는

십 만 큰 가 상 말 다

지 못 들 마 갈 고 돗

도시민들 싹 싹 탔다 가 시

월에 병원에 원4 가 폐( )疲弊

진 몸도 갈 미 지 못 고 었다( )解渴 가는

얼마 지 생 에 생각

가 마감 는 신 평생 십 만에

가 과 많 닮 다고 생각 지는

가 운 는 어 어( ) ldquo rdquohelliphellip

월 새벽 시1965 5 6 1 태 없 거웠고

는 없 그 병원에 퇴원 집

가는 마지막 마 고 마 내 거 다

가 죽 간신 에 실 다 사는 어느5 lsquo

가 죽 는 말 가 식 다 신rsquo

상에 각 시키는 에 실( )刻印

어느 가는 후 민 가가 근 었다lsquo rsquo

다 는 간 과 진실 그 다는( ) ldquo

에 단 평 견 가지고 다 내

가 그리는 간상 단 고 다 지 다 는 가

에 는 평 지 니 그리고 어린 들

미지 겨 그린다rdquo

근 간 과 진실 그리고 싶어( )

가 다 근에게 그것 진리 다 거 다 없

년 학 간고사 대비2013 2 현대고 대비

ECN-0102-2013-001-000076193

거 고 다 없 는 것 진리

다 근 진리는 후 쪽 었다 신산( )辛酸 삶

었 질곡 역사 에 지냈( )桎梏

가 눈에 든 것 료 단 료 게 보

것 었다 그것 그 에 겨우겨우

슬 슬 생 어가는 간들 었다

리 과 단 리 고리에 검

마 없 거리 돌

상 것 없는 등 근에게 상에

과 진실 엄 다는 사실 리는 가 실( )儼存

고 가 과 역경 에 도 근 내 포

없었 후 보루 다( ) 堡壘 도 365

도 간 근 여

시 것 다

마 같 가가 고 싶었 근에게 그 꿈( )

에 다가가는 지 다 다 가 지망생들

규 미 상 에 진 고

에 지만 근 다 다 근

미 에 운 것 보통 시 미 시간

다 그런 그에게 없는 연습 가가

통 다 가 귀 시 지 도

얻는 뛸 듯 뻤지만 마 도 (

는 었 에 어린 근 주 에)

에 그림 그리고 지우고( )粉板

복 시간 가는 게 루 보냈다

zb36) 전 의 성 소가 아닌 것을 고르

① 평 ② 사건 ③ 경

④ ⑤ 훈

늘 지 상에 살고 는 사 들 억 도가10

고 그리 지 통 고 는 사 들( )知的

그보다 훨 많 억 도는 고 지 20

통 다 그런 지 고 2500

그리 간 보는 과 사 에

매우 달 뿐만 니 과 에 도 극

루고 었다 미 운 그런 들

살고 는 동 과 사 들 사고 식에

큰 가 다는 다

고 그리 들 우주 개별 고 독립

사 들 생각 지만 고 들 우

주 연 질 간주 다 같( ) 看做

각 도 들에게는 연 질

었지만 그리 들에게는 미 들 결 었

다 고 과 그리 들 사 같

는 동 과 사 에 도 견 다

지심리 미 마 드 겐트 는

살 들에 에 지 다

연 동 과 상 다 과 같 실험

다 크 만든 미드 도 보

여 주고 그 상 닥 고 주었다lsquo (Dax)rsquo

실 닥 는 재 지 는 것 실험 가lsquo rsquo

만들어 낸 다 그런 다 개 다 체 보

여 주었는 는 미드 지만 틱

만들었고 다 는 재료는 크 지만

달 다 그러고 어 것 닥 지 사 들에게 고 lsquo rsquo

게 니 들 주 같 고 는

체 택 고 동 들 같 재료 만들어진 체

택 다 러 는 심지어 살짜리

들에게 도 타났다 것 곧 과 동

다 상 보고 다는 것 미 다

개별 사 보고 고 동 연 질 보

고 는 것 다

동 들 주변 상 에 맞 어 동 고

에 다 사 들 태도 동에 보다 많

주 울 다 동 가 미시간 에

에 경험 다 그는 미식

경 보러 가게 었는 경 체는 매우 재미 었

주변 들 동에 질 다 그 는

들 계 어 상태 경 다

어 들 에 에 그 시 가 계 가

진 것 다 상 살펴 는 말 들 lsquo rsquo

에 그는 에 시 어 도 뒷사

생각 곧 다시 곤 것 다 그런 그에게 뒷

사 고 지 는 들 동 럼

어 웠다

생각 지도 리 드 니 벳-

zb37) 다음 위 의 내 전개 으 만 인lt gt

것은

lt gt

대조의 통해 대상이 닌 특성을 설 하고 있다

일화를 제 하여 자 의 주장을 뒷 침하고 있다

유추의 을 사 하여 독자의 의해를 돕고 있다

대상이 형성되는 과정을 간적 서에 따라 서 하고 있

① ②

③ ④

년 학 간고사 대비2013 2 현대고 대비

ECN-0102-2013-001-000076193

가 우리가 말 고 쓰는 든 단어가 사 에 는( )

것 니다 사 격에 가 는 지만

어 사 과 같 특별 는 사 니lsquo rsquo

단어 격 보 단어가 사 에

등재 어 다 리 리 사 는 단어 도 그

것 시 사 는 어 고 사 에

격 보 것 니다

러 얼 은 사전에 를 있는가 이에 대한 답lsquo rsquo

은 얼 이 유행어인가 아닌가에 따라 갈라 다 이 단어lsquo rsquo

는 년 어 자 에 랐고 쓰이고 있으2002 lsquo rsquo

유행어라고 하 에는 생 이 다 런데 계속

을 유 하 서 사전에 등재될 자격을 획득할 것인가 이

에 대한 답을 내리 는 히 어 다

여 서 가 를 고 해 볼 있다 첫 는 이 단어

를 써야 할 필 가 속적으 있는가 하는 점이다

상주의 열풍에 휩 인 사회 위 에 편 해서 퍼 말

이 얼 인데 과연 런 위 가 속될 것인가 이에lsquo rsquo

대해 필자의 생각은 정적이다 사회 위 가 뀌

런 말을 쓸 일이 없어 것이다

다음은 단어의 성이다 단어의 성이 사회적으 거

감이 없으 계속 사 될 가능성이 높다 런 에서

얼 은 좋은 조건이 아니다 익히 알 졌듯이 이lsquo rsquo

말은 얼 과 청소년층에서 속어 사 하는 이 결합lsquo rsquo lsquo rsquo

된 말이다 얼 에서 얼 을 리하는 조어 도 lsquo rsquo lsquo -rsquo

어에서는 매 낯선 이다 이것만으 도 거 감을 갖

는 사람들이 있다 더 나 속어 결합한 말이다 얼 lsquo rsquo

이 널리 퍼졌다 해도 은 여전히 청소년층의 속어lsquo rsquo

남아 있다 속어는 자연 럽게 아 자리에서나 쓰 에는

담 러 말이다 러한 담을 하고 사

역을 넓혀 가는 속어도 없 는 않다 특히 얼 은 lsquo rsquo

에도 종종 등장한다 만큼 거 감이 많이 희석되었다

고 할 있다 러나 일상의 자연 러 대화에서도 거

리낌 없이 등장하는가 게 는 되 않았다고 생

각한다

얼 이 유사어인 쌈 등을 만들어 내고lsquo rsquo lsquo rsquo

있으니 살아남을 있을 것이라고 는 견해도 있을 것

이다 러나 간이 나 서 유사어를 포함하여 든

말이 사라 사 는 많다 유사어가 많다는 것이 생 을

유 할 있는 절대적인 조건은 아니다

나 언젠가 터 사람들은 어느 단에서 얼 이 가장( )

쁜 사람을 가리켜 얼 이라고 르고 있다 이 얼lsquo rsquo lsquo rsquo

이라는 단어가 최근 어사전에 라 항간에 논란이 일고

있다 아닌 게 아니라 얼 은 유행어처럼 인다 생 lsquo rsquo

도 리 래되 않은 것 같고 언제 사라 도 알

없다 게다가 젊은이들 사이에서 주 쓰일 뿐이다 이런

단어를 사전에 는다는 게 하 이 없어 이 도

한다

러나 속단은 이다 차근차근 따져 볼 일이다

선 얼 이 일 적 유행어인 아닌 주의 게 들여다lsquo rsquo

볼 필 가 있다 유행어란 유행에 따라 빠르게 유포되었

다가 단 간 내에 소 되는 단어나 를 가리킨다

얼 은 인터넷을 통해 속히 퍼 말이다 하 만 일lsquo rsquo

적인 유행어처럼 단 간 내에 사라 않았을 뿐 아니라

현재 도 잦은 빈도 사 되고 있고 앞으 도 상당

간 사 될 것으 측된다 한 언 재단의 뉴 검 lsquo rsquo

색 사이트에 따르 얼 은 년 에 처음 나타난lsquo rsquo 2001

이후 꾸 히 사 되고 있다

이 같은 사 빈도는 얼 이 일 적 유행어 는 현lsquo rsquo

저히 다르다는 것을 여 다 장 간의 생존 만으 도

얼 은 이 한 어의 어휘 에 를 자격을 얻었다lsquo rsquo

고 할 있다 더 이 이라는 비 적 정제된 매체에

높은 빈도 쓰이고 있 않은가 사 빈도 측 에서

필통이나 연필과 같은 단어 대등하거나 더 많이 쓰lsquo rsquo lsquo rsquo

다는 것은 결코 가 게 볼 일이 아니다

이제는 사전이 언어 현 을 빠르게 하는 게 덕인

대가 되었다 세계적으 유 한 의 사전들도 경쟁

적으 어를 고 있다

하 만 얼 은 젊은이들이나 쓰는 속어라고 흠을 잡을lsquo rsquo

도 르겠다 얼 이 주 젊은 층에서 많이 쓰 lsquo rsquo

는 속어임에 틀림없다 러나 어사전에 표 적이고 품

위 있는 말만 어야 한다고 생각한다 것은 커다란

해다 당장 아 어사전이나 펼쳐 라 속어는

설과 같은 비어나 죄자들이 쓰는 은어 어

마니 같은 소 의 사람만이 쓰는 말 도 라 있

않은가 사전은 말 치에 일정 빈도 이상 나타나는 말이

라 말이든 다 할 있다

zb38) 가 나 에 대한 다음의 설( ) ( ) 않은 것은

① 가 는 얼짱 사 에 등재 것에( ) ( ) lsquo rsquo

보 고 다

② 사 등재 가는 단어 격에( )

고 고 는 언 들 언어 사 도에 고 다 ( )

③ 가 얼짱 어지만 신 과 같 매( ) ( ) lsquo rsquo

체에 도 사 는 말 는 고 다

④ 가는 얼짱 어 보고 크게 가지 근( ) lsquo rsquo 3

거 들어 뒷 고 다

⑤ 는 얼짱 어 는 다 특 다는( ) lsquo rsquo

근거 에도 크게 가지 근거 가 들어 주 2

뒷 고 다

가 늘 지 상에 살고 는 사 들 억( ) 10

도가 고 그리 지 통 고 는 사 들

그보다 훨 많 억 도는 고 지 20

통 다 그런 지 고 2500

년 학 간고사 대비2013 2 현대고 대비

ECN-0102-2013-001-000076193

그리 간 보는 과 사 에

매우 달 뿐만 니 과 에 도 극

루고 었다 미 운 그런 들

살고 는 동 과 사 들 사고 식에

큰 가 다는 다

고 그리 들 우주 개별 고 독립

사 들 생각 지만 고 들 우

주 연 질 간주 다 같 각

도 들에게는 연 질 었지

만 그리 들에게는 미 들 결 었다

고 과 그리 들 사 같 는

동 과 사 에 도 견 다

인 리학자인 츠 이마이 디드 겐트너는 두

살이 채 안 된 아이들에서 터 성인에 이르 다양한

연 대의 동양인과 서양인을 대상으 다음과 같은 험

을 했다 저 코르크 만든 피라 드 양의 도형을

여 주고 대상의 이름을 닥 라고 알 주었다lsquo (Dax)rsquo

제 닥 는 존재하 않는 것으 험자가 임의lsquo rsquo

만들어 낸 이름이다 런 다음 두 개의 다른 체를

여 주었는데 하나는 피라 드 양이 만 하얀 플라 틱

으 만들었고 다른 하나는 재 는 코르크 만 양이

달랐다 러고 나서 어떤 것이 닥 인 사람들에게 고 lsquo rsquo

르게 했더니 서양인들은 주 같은 양을 하고 있는

체를 선택했고 동양인들은 같은 재 만들어 체를

선택했다 이러한 차이는 성인은 어 두 살 리

아이들에게서도 나타났다 이것은 곧 서양인과 동양인은

서 다른 세상을 고 있다는 것을 의 한다 략 ( )

는 아주 단 하 서도 인상적인 험을 했다

험에는 동서양의 대학생들이 참여했다 는 험 참가자

들에게 컴퓨터 화 을 통해 속 장 을 담은 애니 이션

을 여 주었다 화 의 앙에는 초점의 역할을 하는 커

다란 고 한 마리가 있었고 주위에는 다른 생

들과 초 자갈 거품 등이 함 제 되었다 화 을

두 씩 후 참가자들은 자 이 것을 회상해 라는

를 았다

결과 서양인 대학생들과 동양인 대학생 두 앙

의 초점 역할을 했던 고 를 동일한 정도 언 했으

나 경 소 위 거품 초 다른 생 들 에 ( )

대해서는 동양인 대학생들이 서양인 대학생들 다 60

이상 더 많이 언 했다 뿐만 아니라 동양인 학생들은 서

양인 학생들에 비해 개 적인 고 다 전체적인 계

를 더 언 하는 경향을 다 략 또한 경의 일 ( )

를 화 킨 림을 제 하 을 때 동양인 대학생들은 대

경의 화를 알아챘 만 서양인 대학생들은 경

의 화를 거의 알아차리 했다 략 ( )

따라서 서양인들만을 대상으 연 한 화lsquo

편성 결 은 잘 된 것일 도 있다 각 과정과 인rsquo

과정의 어떤 이 화 편적이고 어떤 이

화에 따라 달라 는 는 앞으 많은 연 를 통하여 논의

되어야 한다

나 어떤 의 에서 리 두는 이 화적이다 리( )

안에는 다른 사람들과 더 친 한 계를 유 하 는 상호

의존성과 다른 사람들 터 독립적인 존재 살아가 는

독립성이 혼재한다 따라서 이 에서 어떤 특성이 더 강

하게 각되는 상황에 놓이느냐에 따라 서 다른 화적

특 을 일 있다 결 리 두는 어떤 경 에는

동양인처럼 행동하고 어떤 경 에는 서양인처럼 행동하는

것이다

zb39) 가 에 대한 다음의 설( ) 않은 것은

① 는 신 주 뒷 닥 실험과lsquo rsquo lsquo

니 실험 근거 시 다rsquo

② 동 들 상 간 공통 보다는 에 식

는 강 다

③ 들 주변 맥 에는 심 경 어 사건

과 사건 사 계에 상 민감 다

④ 는 동 과 틀린 지 고 는 것lsquo rsquo

니 다 고 다 lsquo rsquo

⑤ 가에 우리 사 들 개 시 가 원( )

집 경 말 고 는 것 개 보다는

에 고 는 것에 다

늘 지 상에 살고 는 사 들 억 도가10

고 그리 지 통 고 는 사 들( )知的

그보다 훨 많 억 도는 고 지 20

통 다 그런 지 고 2500

그리 간 보는 과 사 에

매우 달 뿐만 니 과 에 도 극

루고 었다 미 운 그런 들

살고 는 동 과 사 들 사고 식에

큰 가 다는 다

지심리 미 마 드 겐트 는 동

과 상 다 과 같 실험 다

크 만든 미드 도 보여 주고 그

상 닥 고 주었다 그런 다lsquo (Dax)rsquo

개 다 체 보여 주었는 는 미드

지만 틱 만들었고 다 는 재료는

크 지만 달 다 그러고 어 것 닥 lsquo

지 사 들에게 고 게 니 들 주 같rsquo

고 는 체 택 고 동 들 같

재료 만들어진 체 택 다 러 는

심지어 살짜리 들에게 도 타났다 것

곧 과 동 다 상 보고 다는

것 미 다 개별 사 보고 고 동

년 학 간고사 대비2013 2 현대고 대비

ECN-0102-2013-001-000076193

연 질 보고 는 것 다

동 들 주변 상 에 맞 어 동 고

에 다 사 들 태도 동에 보다

많 주 울 다 동 가 미시간

에 에 경험 다 그는 미

식 경 보러 가게 었는 경 체는 매우 재

미 었 주변 들 동에 질 다 그

는 들 계 어 상태 경

다 어 들 에 에 그 시 가 계

가 진 것 다 뒷사 고 지 는 들

동 럼 어 웠다

그는 경험에 어 얻어 동 들lsquo

각도 상 본다 는 가 우고rsquo

검 여 주 단 도 상 실험 실

시 다 그는 실험 가 들에게 컴퓨 통

담 니 보여 주었다

에는 역 는 커다 고 마리가 었

고 주 에는 다 생 들과 갈 거 등

께 시 었다 본 후 가 들

신 본 것 상 보 는 지시 다

그 결과 생들과 동 생

역 고 동 도 언

경 거 다 생 들에 ( )

는 동 생들 생들보다 60

상 많 언 다 뿐만 니 동 생들

생들에 개별 고 보다 체 계

언 는 경 보 다 경 변 시

킨 그림 시 동 생들 경

변 지만 생들 경 변

거 리지 못 다

지 지 들만 상 연 lsquo

보편 결 못 것 도 다 지각 과 과rsquo

지 과 어 보편 고 어

에 달 지는지는 많 연 통 여

어 다

리 드 니 벳 생각 지도 사- ldquo rdquo( 2004)

zb40) 위 에 대한 설 으 가장 적절한 것은

① 동 과 생 식 강 고 다

② 가지 실험 통 쓴 고 다

③ 닥 실험에 사 본질에 동 사

상에 주 다

④ 니 실험에 동 과 에 지

각 도에 가 다

⑤ 쓴 는 보편 연 에 드러 우월 에

에 근 고 다

가 동 들 주변 상 에 맞 어 동 고( )

에 다 사 들 태도 동에 보다 많

주 울 다 동 가 미시간 에

에 경험 다 그는 미식

경 보러 가게 었는 경 체는 매우 재미 었

주변 들 동에 질 다 그 는

들 계 어 상태 경 다

어 들 에 에 그 시 가 계 가

진 것 다 상 살펴lsquo 는 말 들rsquo

에 그는 에 시 어 도 뒷사

생각 곧 다시 곤 것 다 그런 그에게

뒷사 고 지 는 들 동 럼

어 웠다

그는 경험에 어 얻어( ) 동 들lsquo

각도 상 본다 는 가 우고rsquo

검 여 주 단 도 상 실험

실시 다 실험에는 동 생들 여 다

그는 실험 가 들에게 컴퓨 통

담 니 보여 주었다 에는

역 는 커다 고 마리가 었고 주 에는

다 생 들과 갈 거 등 께 시

었다 본 후 가 들 신 본 것

상 보 는 지시 다

다 그 결과 생들과 동 생( )

역 고 동 도 언

경 거 다 생 들 에 ( )

는 동 생들 생들보다 60

상 많 언 다 뿐만 니 동 생들

생들에 개별 고 보다 체 계

언 는 경 보 다 들어 동

생들 상 체 연못 럼 보 어ldquo 같rdquo

체 맥 언 시 었지만

생들 상 어 같 큰 고 가 쪽 움ldquo

직 어 같 역 고rdquo

언 시 다 경 변 시킨 그

림 시 동 생들 경 변

지만 생들 경 변 거

리지 못 다

년 학 간고사 대비2013 2 현대고 대비

ECN-0102-2013-001-000076193

게 볼 동 들 보다는 큰 그( )

림 보 에 사 과 체 맥 연결시 지각

는 경 고 체에 특 떼어 내

어 독립 보는 것 낯 어 다 에

들 사 에 고 주변 맥 에는 심 경

에 사건과 사건 사 계에 상

민감 편 다

마 지 지( ) 들만 상 연

보편 결 못 것 도 다lsquo rsquo 지각 과

과 지 과 어 보편 고 어

에 달 지는지는 많 연 통 여

어 다

리 드 니 벳 생각 지도 사- ldquo rdquo( 2004)

zb41) 의 하는 가~ 다른 것은

① ② ③

④ ⑤

얼마 그 에 동 사고 식과

사고 식 보여 주는 내 다

들 에 는 탕 고 같 게

어 겨 고 미 에 는 그 크 럼 큰 고

어리 주고 원 는 어 도 는

상 고 생각 다는 것 다 러

는 어떻게 생 것 고 과 그리 거슬

러 가 보 그 단 다

고 연 경 체 경 생 에

다 벼 사는 공동 업과 경험 많 연 역

에 고 들 연 웃과

게 지내 고 탁 연 들

들 지 연 럽게 들 다 민들

웃과 동 게 뿐만 니 는 집 과

게 다

동 시 는 생태 경 에 살 결과

들 다 사 들 사 상 에 주

울 게 었고 는 곧 체 상 과 간 사

계 시 는 낳게 었다 신 가

가 는 체에 는 원 는 동시

에 다 사 들 그 사 포 체 맥 에

다 들 간 사 연

계 체 계에 주 울 는 사고 체계

게 었다

그러 그리 연 경 그 었다 산

지 연결 는 지 건 그리고 역

에 다 런 들 업에 다 사 과

동 므 공동체에

다고 다 고 그리 들

들과는 달리 보 내 감 지 들과

지 크게 느 지 못 다 그

견 다 경우 주 쟁 통 결 는 갖

게 었다

신 사 간 계들 루어진 커다

트워크 에 게 당연 사 역시 연

계들 체 식 게 다 어 상

원 도 그 개체가 체 맥 과

계 에 고 다 게 체 맥 에 주

울 다 보 상 복 과 가변 식 게 고

상에 재 는 많 변 들 사 에 재 는 들도

게 다 들 주 태도 보

는 경우가 많다 쟁 결

통 결 보다는 통 결

는 보 다

그러 고 그리 들 개개 사 사 독

에 주 울 다 사 사 체에

어 그들 사 에 재 는 공통 규 주

고 다 상 원 에도 사

체 내 주 고 다 그들

체 여 탕 체

는 주 태도 시 고 특 사 어

주에 는지 여 그 주에 는 규

견 다 에 는 쟁 식 리

같 리 사고 체계가 달 게 었다

리 드 니 벳 생각 지도 사- ldquo rdquo( 2004)

zb42) 위 에서 사 된 설 과 가장 유사한 것은

① 크톱 컴퓨 는 본체 니 마우 루

어 다

② 곡과 시 리 는 지 과 사 루어 다는 공통

지니고 다

③ 경 고 것과는 달리

경 본 연 태 그 주변 경

④ 벽돌 능 에 사계 내내

습도가 지 다

⑤ 잰느 체 체 지닌 재 체가 없

는 재 눌 다

년 학 간고사 대비2013 2 현대고 대비

ECN-0102-2013-001-000076193

zb43) 는 립 앙 도서 이 정의 일 이다lt gt

도서 장과 이 자의 리 의 정의 연결이

적절하 않은 것은

lt gt

제 조 서 유8 ( )

도서 장은 다른 이 자의 안전을 위협하거나 도서 의①

서를 란하게 할 가 있는 자에 대하여는 도서 출입

을 제한할 있다

도서 장은 이 자가 제 조 각 호의 어느 하나의 행위를 하7②

을 때에는 이 을 하게 하거나 도서 출입을 제한할

있다

제 조자 의 대출9 ( )

도서 자 는 다음 각 호의 경 대출할 있다①

상호대차도서 간에 자 를 류하는 것을 말한다 등 다1 ( )

른 도서 과의 협 을 위하여 필 한 경

공 이 공 행 상 필 하는 경2

에 도서 장이 필 하다고 인정하는 경3

대출이 가능한 도서 자 의 위는 도서 장이 정하는②

에 따른다

제 조 상10 ( )

이 자가 도서 자 설을 더럽히거나 찢거나 뜨①

쓰게 하거나 잃어 린 경 에는 상하여야 한다

도서 장은 제 항에 따른 상 을 정하여 게 하여야1②

한다

제 조이 절차 등11 ( )

이 칙에서 정한 것 에 도서 자 설의 이 절차

이 제한 등에 필 한 사항은 도서 장이 정한다

출처 립 앙 도서- (httpwwwnlgokr)

① 는 도 리 다8

② 도 는 리 다9 1

③ 료 지 는 도 리 다9 2

④ 도 료 변상에 리10 1

⑤ 는 에 도 리 다11

3

도 다 각 같다①

공 공 다만 연1

연 간 다

매월 째 째 월2

도 도 리 그 사3

가 다고 는

도 에 미리 게1 3②

시 여 다

4

도 시간 도 여 게시 다

5

도 료 시 는 는 도①

지에 등 후

등 에 사 도②

7

는 다 각 여 는 니 다

도 료 시 상 리1 lsquo rsquo

도 료 시 훼 는2 middot

지 가 닌 곳에 식 거 담3

우는

도 보 등 보 검색열4 middot

그 에 도 질 지 여 도5

여 게시 사 는

8

도 다 거 도①

질 게 우 가 는 에 여는 도

도 가 각 어느7②

에는 지 게 거 도

9

도 료는 다 각 경우 다①

상 도 간에 료 는 것 말1 (

다 등 다 도 과 여 경우)

공 원 공 상 는 경우2

그 에 도 다고 는 경우3

가능 도 료 는 도②

는 에 다

10

년 학 간고사 대비2013 2 현대고 대비

ECN-0102-2013-001-000076193

가 도 료 시 럽 거 거①

못 쓰게 거 어 린 경우에는 변상 여

도 에 변상 여 게시1②

여 다

zb44) 위 에서 도서 장이 게 해야 할 사항에 해당하는

것을 두 쓰

년 학 간고사 대비2013 2 현대고 대비

ECN-0102-2013-001-000076193

립 도 규

1 ( )

규 립 도 립 어린 청 도(

포 다 료 시 열 시 말) (

다 에 사 규 립 도)

편 진 다

2 ( )

규 립 도 도 다 에( lsquo rsquo )

고 는 도 에 도lsquo rsquo 2 2

료 에 여 다 다만 특 료 귀

료 등 료 에 사 립 도

도 다 다( lsquo rsquo )

3 ( )

도 다 각 같다①

공 공 다만 연1

연 간 다

매월 째 째 월2

도 도 리 그 사3

가 다고 는

도 에 미리 게1 3②

시 여 다

시간4 ( )

도 시간 도 여 게시 다

등 등5 ( )

도 료 시 는 는 도①

지에 등 후

등 에 사 도②

사 료6 ( )

도 료 시 에 사 료는 도

7 ( )

는 다 각 여 는 니 다

도 료 시 상 리1 lsquo rsquo

도 료 시 훼 는2 middot

지 가 닌 곳에 식 거 담3

우는

도 보 등 보 검색열4 middot

그 에 도 질 지 여 도5

여 게시 사 는

질 지8 ( )

도 다 거 도①

질 게 우 가 는 에 여는 도

도 가 각 어느7②

에는 지 게 거 도

료9 ( )

도 료는 다 각 경우 다①

상 도 간에 료 는 것 말1 (

다 등 다 도 과 여 경우)

공 원 공 상 는 경우2

그 에 도 다고 는 경우3

가능 도 료 는 도②

는 에 다

변상10 ( )

가 도 료 시 럽 거 거①

못 쓰게 거 어 린 경우에는 변상 여

도 에 변상 여 게시1②

여 다

등 규 에 것 에 도11 ( )

료 시 등에 사

도 다

립 도- (httpwwwnlgokr)

zb45) 도서 장의 리 있는 조항으 적절하 않

은 것은

① ② ③ ④ ⑤

년 학 간고사 대비2013 2 현대고 대비

ECN-0102-2013-001-000076193

1 ( )

사가 공 는lsquo rsquo

과 여 사 원과 리

사 타 사 규

니다

개 보 보7 ( )

사는 보통신망 등 계 는 에lsquo rsquo lsquo rsquo

원 개 보 보 니다 개lsquo rsquo

보 보 사 에 는 사 개lsquo rsquo

보 취 니다 다만 사는 다 lsquo rsquo

사 계 통 공 는 경우 원 lsquo rsquo

등 개 보 당 사에 습니lsquo rsquo

원 리에8 (lsquo rsquo lsquo rsquo lsquo rsquo

)

원 에 리lsquo rsquo lsquo rsquo lsquo rsquo①

원에게 가 도 여 는lsquo rsquo 3

니다

사는 원 가 개 보 우 가lsquo rsquo lsquo rsquo lsquo rsquo②

거 사 경우 는 미 에 어 거 lsquo

사 사 운 우 가 는 경우 당rsquo lsquo rsquo

습니다lsquo rsquo

원 가 도 거lsquo rsquo lsquo rsquo lsquo rsquo 3③

가 사 고 지 경우에는 시 사에lsquo rsquo

통지 고 사 내에 니다lsquo rsquo

경우에 당 원 사에 그 사실3 lsquo rsquo lsquo rsquo④

통지 지 거 통지 도 사 내에 지 lsquo rsquo

생 경우 사는 지지 습니다lsquo rsquo

사10 (lsquo rsquo )

사는 과 지 미lsquo rsquo①

에 는 지 계 고

공 여 다 여 니다lsquo rsquo

사는 원 게lsquo rsquo lsquo rsquo lsquo rsquo②

도 개 보 신 보 포 보 보 시( )

갖 어 개 보 취 공시 고

니다

사는 과 여 원lsquo rsquo lsquo rsquo③

견 만 당 다고 경우에는

리 여 니다 원 견 만 사 lsquo rsquo

에 는 게시 거 우편 등 통 여

원에게 리 과 결과 달 니다lsquo rsquo

원11 (lsquo rsquo )

원 다 여 는 니다lsquo rsquo ①

신청 는 변경 시 허 내 등1

타 보 도2

사가 게시 보 변경3 lsquo rsquo

사가 보 보 컴퓨 그4 lsquo rsquo (

등 등 신 는 게시)

사 타 등 지 재산 에5 lsquo rsquo 3

사 타 상 거 업6 lsquo rsquo 3

는 폭 시지 상 타 공7 middot middot

에 는 보 에 공개 는 게시 는lsquo rsquo

사 동 없 리 사8 lsquo rsquo

타 거 당9

게시15 (lsquo rsquo )

원 내에 게시 는 게시 게재 는lsquo rsquo lsquo rsquo lsquo rsquo

경우 원 사가 게시 복 lsquo rsquo lsquo rsquo lsquo rsquo middot middot

등 태 언 등에 공 는

것 내에 다 원 본 게시 등 lsquo rsquo lsquo rsquo

크 능 등 여 복 는 등 태

는 것 동 것 니다

- (wwwnavercom)

zb46) 위 은 인터넷 포털사이트의 회 가입을 위한 이

약 의 일 이다 이 약 을 만드는 과정에서 생각한

내 으 적절하 않은 것은

개 보 보 가 지에 별 눠①

겠어

원 가 만들게 에②

시 주어 겠어

원들 게재 게시 다 원 크 다③

는 것 지

④ 원 지 는 뿐만 니 사가 지 는

도 께 달 지

리에 가 생 경우 사가⑤

에 다는 도 듯

1 ( )

사가 공 는lsquo rsquo

과 여 사 원과 리

사 타 사 규

년 학 간고사 대비2013 2 현대고 대비

ECN-0102-2013-001-000076193

니다

개 보 보7 ( )

사는 보통신망 등 계 는 에lsquo rsquo lsquo rsquo

원 개 보 보 니다 개lsquo rsquo

보 보 사 에 는 사 개lsquo rsquo

보 취 니다 다만 사는 다 lsquo rsquo

사 계 통 공 는 경우 원 lsquo rsquo

등 개 보 당 사에 습니lsquo rsquo

원 리에8 (lsquo rsquo lsquo rsquo lsquo rsquo

)

원 에 리lsquo rsquo lsquo rsquo lsquo rsquo①

원에게 가 도 여 는lsquo rsquo 3

니다

사는 원 가 개 보 우 가lsquo rsquo lsquo rsquo lsquo rsquo②

거 사 경우 는 미 에 어 거 lsquo

사 사 운 우 가 는 경우 당rsquo lsquo rsquo

습니다lsquo rsquo

원 가 도 거lsquo rsquo lsquo rsquo lsquo rsquo 3③

가 사 고 지 경우에는 시 사에lsquo rsquo

통지 고 사 내에 니다lsquo rsquo

경우에 당 원 사에 그 사실3 lsquo rsquo lsquo rsquo④

통지 지 거 통지 도 사 내에 지 lsquo rsquo

생 경우 사는 지지 습니다lsquo rsquo

원에 통지9 (lsquo rsquo )

사는 특 다 원에게 통지 경우lsquo rsquo lsquo rsquo

공지 게시 통 상 게시 개별 통지에7

갈 습니다

사10 (lsquo rsquo )

사는 과 지 미lsquo rsquo①

에 는 지 계 고

공 여 다 여 니다lsquo rsquo

사는 원 게lsquo rsquo lsquo rsquo lsquo rsquo②

도 개 보 신 보 포 보 보 시( )

갖 어 개 보 취 공시 고

니다

사는 과 여 원lsquo rsquo lsquo rsquo③

견 만 당 다고 경우에는

리 여 니다 원 견 만 사 lsquo rsquo

에 는 게시 거 우편 등 통 여

원에게 리 과 결과 달 니다lsquo rsquo

원11 (lsquo rsquo )

원 다 여 는 니다lsquo rsquo ①

신청 는 변경 시 허 내 등1

타 보 도2

사가 게시 보 변경3 lsquo rsquo

사가 보 보 컴퓨 그4 lsquo rsquo (

등 등 신 는 게시)

사 타 등 지 재산 에5 lsquo rsquo 3

사 타 상 거 업6 lsquo rsquo 3

는 폭 시지 상 타 공7 middot middot

에 는 보 에 공개 는 게시 는lsquo rsquo

사 동 없 리 사8 lsquo rsquo

타 거 당9

원 계 규 내lsquo rsquo lsquo②

여 공지 주 사 사가 통지 는rsquo lsquo rsquo

사 등 여 타 사 업 에 lsquo rsquo

는 여 는 니다

- (wwwnavercom)

zb47) 위 약 의 조항에서 같은 제점을 하lt gt

고 있는 조항은

lt gt

제휴 회사에 회 의 아이디 개인 정 를 전송할 있도

한 조항은 고객에게 당한 조항이다

1 7 8① ② ③

④ 9 ⑤ 10

립 도 규

1 ( )

규 립 도 립 어린 청 도(

포 다 료 시 열 시 말) (

다 에 사 규 립 도)

편 진 다

2 ( )

규 립 도 도 다 에( lsquo rsquo )

고 는 도 에 도lsquo rsquo 2 2

료 에 여 다 다만 특 료 귀

료 등 료 에 사 립 도

도 다 다( lsquo rsquo )

3 ( )

도 다 각 같다①

공 공 다만 연1

연 간 다

년 학 간고사 대비2013 2 현대고 대비

ECN-0102-2013-001-000076193

매월 째 째 월2

도 도 리 그 사3

가 다고 는

도 에 미리 게1 3②

시 여 다

시간4 ( )

도 시간 도 여 게시 다

등 등5 ( )

도 료 시 는 는 도①

지에 등 후

등 에 사 도②

사 료6 ( )

도 료 시 에 사 료는 도

7 ( )

는 다 각 여 는 니 다

도 료 시 상 리1 lsquo rsquo

도 료 시 훼 는2 middot

지 가 닌 곳에 식 거 담3

우는

도 보 등 보 검색열4 middot

그 에 도 질 지 여 도5

여 게시 사 는

질 지8 ( )

도 다 거 도①

질 게 우 가 는 에 여는 도

도 가 각 어느7②

에는 지 게 거 도

료9 ( )

도 료는 다 각 경우 다①

상 도 간에 료 는 것 말1 (

다 등 다 도 과 여 경우)

공 원 공 상 는 경우2

그 에 도 다고 는 경우3

가능 도 료 는 도②

는 에 다

변상10 ( )

가 도 료 시 럽 거 거①

못 쓰게 거 어 린 경우에는 변상 여

도 에 변상 여 게시1②

여 다

등 규 에 것 에 도11 ( )

료 시 등에 사

도 다

립 도- (httpwwwnlgokr)

zb48) 다음 정 리 의 의 으 볼 때 가장

이 적인 것은

도 시간 도 여 게시 다①

등 에 사 도②

가능 도 료 는 도 는③

에 다

④ 도 에 변상 여 게10 1

시 여 다

⑤ 도 가 각 어느7

에는 지 거 도

zb49) 를 참고하여 이 어의 성격을 설 한lt gt

것으 적절하 않은 것은

① 보 에 는 어 시 상 고 어 시lt gt lsquo rsquo

에 보여주고 다

② 진 어 어원에 견 고 다

에는 타 어 들어가는 것 다 lsquo rsquo

③ 에 들어갈 말 각각 고 어 어 신 어~

들 언어는 질 격 강 통 없었다

④ 시 우리 에 가 었지만 지 계

과 달리 들 통 사 달 어 웠

년 학 간고사 대비2013 2 현대고 대비

ECN-0102-2013-001-000076193

⑤ 크 몽골 만주 공통어가 우리 어 같

계열에 다는 에 사 특 짐

가( )

善化公主主隱 공주님

他密只嫁良置古 몰 결 고

薯童房乙 맛

夜矣卯乙抱遣去如 에 몰 고 가다

( )

始汝 會隱日恚見隱扐 만 에 본

恥隱汝衣淸隱笑 맑 웃

고 시 여 공 크다 만 다[ ] ( ) ( ) ( ) ( )始 汝 會扐

내다 에 보다 견( ) ( )恚 見 다( )隱

럽다 맑다 청 웃( ) ( ) ( ) ( )恥 衣 淸 笑

zb50) 위의 나 를 함 고 음에 답하( ) lt gt

보lt gt

( )素那或云金川 白城郡蛇山人也

운 사산

는 고 다 는( )[ ( ) ] (素那 金川 白城

사산 사 다) ( ) 郡 蛇山

삼 사- lsquo rsquo 47

에 제 된 단어 의 표 리를 조건(1) lt gt ( ) lt gt

에 맞게 서 하

건lt gt

lsquo 었고 었다 태rsquo

에 제 된 단어 동일한 표 리에(2) lt gt ( )

의해 적은 것을 나 에서 찾아 조건 에 맞게 서 하( ) lt gt

건lt gt

에 당 는 각각( ) 개 쓸 것2 단

당 는 가 여러 개 어도 개만 쓸 것 각2

개 과 도 쪽에 개만2 2

드시 지 것( )

과 동 원리 것lsquo 고

과 동 원리 것 다rsquo

태 것

가( )

素那(或云金川) 白城郡蛇山人也

소나 또는 천 이라 한다 는 성 사( ) ( ) ( )素那 金川 白城郡〔 〕

산 사람이다 현대어 풀이( ) ( )蛇山

나( )

紫布岩乎希 회

執音乎手母牛放敎遣 자 손 암쇼 노히 고

吾 不喩慙 伊賜等肹 肹 나 안디 리샤

花 折叱肹 可獻乎理音如 고 것거 도림다

다 향찰은 리말을 리 으 적은 표 이었 만 생( )

은 고 대를 넘 하고 끊어 고 말았다 랜 세

동안 갈고 닦아 체계적이었던 향찰 표 이 사라졌

을 인은 크게 두 가 나누어 생각해 볼 있다

하나는 족 사회의 한 선호도에서 찾을 있다 라 때

향찰은 주 족 계 에서 사 했을 것으 인다 한 을

알 하고서는 한자를 활 하여 리말을 리 으 표

하 란 가능하 때 이다 런데 족들은 간이 흐

를 향찰과 같은 리 표 을 익혀 사 하 다는

아 한 을 대 사 하는 쪽을 선호하게 되었다 더 이

고 초에 인재 등 을 위해 과거제도가 행되 서 한 선

호도가 더 높아졌고 결 향찰은 소 되고 말았다

또 다른 가능성은 한 어의 특성에서 찾을 있다

터 한 과 일 세 나라는 한자 화 에 속해 다

당연한 이야 겠 만 표의 자인 한자는 어를 표 하

에 매 적절하다 어의 음절은 성 ( ) ( )聲母 韻母

이 어 고 여 에 성조가 추가되어 최종 소리가 결정된

다 래서 어는 단음절을 하나의 한자 표 하 된

다 에 초성 성 종성의 세 가 소가 하나의 음절

년 학 간고사 대비2013 2 현대고 대비

ECN-0102-2013-001-000076193

을 이 는 한 어는 음절 조가 잡하고 음절의 가 많아

서 한자 차 만으 한 어의 소리를 만족 럽게 표 할

없었다 를 들어 한 어에서는 어 니 같이 음절 lsquo rsquo

이 어 단어가 얼마든 있으나 어는( ) 複數音節

자 하나 나타내 만이다lsquo [m ]rsquo 母 ǔ

한편 일 어의 표 은 핵 적 단어는 한자 적고 토는

가나라는 일 의 자 적는 이다 적인 의 를 나

타내는 은 표의 자인 한자 적고 적 계를 나

타내는 토는 표음 자 적는 셈이니 자세히 살펴

리의 향찰 표 을 쏙 빼닮았음을 알 있다 한 어 같

은 착어이 서도 일 어에만 향찰과 유사한 표 이 살아

남은 것은 일 어의 특 때 이다 일 어는 하나의 자음과

음의 결합으 음절을 이 고 침이 거의 없는 음절 언어

이다 이러한 음절의 특색에다가 토가 달한 착어라는 점

이 향찰과 유사한 표 이 살아남을 있는 비결이었다

하 만 같은 착어라도 다양한 음소 침이 달한 한

어는 향찰 표 하는 데 근 적으 한계가 있었다

zb51) 다 하여 의 행에 대한 탐 한 결과( ) lt gt 2

않은 것은

보lt gt

善花公主主隱 공주니믄 공주님( )

----------------------------------------

-

他密只嫁良置古 그 지 얼어 고 몰 결(

----------------------------------------

-

薯童房乙 맛 맛( )

夜矣卯乙抱遺去如 몰 고 가다 에 몰 고(

가다)

주동 역 동- (薯童謠『 』

에 2 ( )他密只嫁良置古

얼다 시집가다 결 다 말 lsquo rsquo

① 실질 미 지니고 므 타 타lsquo ( )rsquo lsquo [ ]

② 에 실질 미 타내고 지 는lsquo rsquo lsquo [ ]rsquo lsquo [ ]密只 密 只

계 타내는

③ 얼어는 실질 미 포 고 므 가lsquo rsquo lsquo [ ]rsquo嫁

것lsquo [ ]rsquo 良

④ 고 어간 는 실질 미 지니고 므lsquo rsquo lsquo -rsquo

것lsquo [ ]rsquo 置

⑤ 고 어미 고는 계 타내고 므lsquo rsquo lsquo- rsquo

고 것lsquo [ ]rsquo 古

가( )

엉 훈 민middot middot middot middot middot世 宗 御 製 訓 民 正 音

말 미 듕 귁에 달middot middot middot middot middot middot middot middot中 國 文 字

니 런middot middot middot middot middot middot 어린middot middot middot middot百 姓

니 고 도 내 들middot middot middot middot middot middot middot middot middot 시러middot

펴 몯middot 미middot middot 니 내middot middot middot middot middot middot middot middot 爲

어엿middot 겨 새middot middot middot 믈여듧middot middot middot middot字 니middot middot middot

사 마다 니겨 킈 middot middot middot middot middot middot middot middot middot便 安

고 미니middot middot middot middot

본 는 상( ) (象

원리에 만들어진 본) ( )形 ㄱ ㄴ ㅁ ㅅ ㅇ

에 는 가 원리에( )加劃

그리고( )ㅋ ㄷ ㅌ ㅂ ㅍ ㅈ ㅊ ㆆ ㅎ

쓰는 병 원리에 만들어진( )竝書

마지막 체( ) ( )異體ㄲ ㄸ ㅃ ㅆ ㅉ ㆅ

ᅀ 다 상 원리에 ㅇ ㄹ

지 는 삼재 상 본 본( ) ( ) ( 天地人 三才

탕 므림과 림에 ) (初ㅡ ㅣ

재)( ) ( )( )出字 再出字ㅗ ㅏ ㅜ ㅓ ㅛ ㅑ ㅜ ㅕ

병 그리고 들 에 다시( )ㅘ ㅝ ㅣ

( )ㅣ ㅢ ㅚ ㅐ ㅟ ㅔ ㆉ ㅒ ㆌ ㅖ ㅙ ㅞ

zb52) 가 에 대한 설 으 르 않은 것을( ) 두 고르

① 어쓰 규 지키고 다

② 리 고 다

③ 말 미 미 등 어 사 다lsquo rsquo

④ 개 지 다

년 학 간고사 대비2013 2 현대고 대비

ECN-0102-2013-001-000076193

⑤ 어 원 에 가 도 고 다

엉 훈 민世 宗 御 製 訓 民 正 音

말 미 듕귁에 달 니

런 어린 니 고 도middot

내 들 시러 펴 몯 미 니middot

내 어엿 겨 새 믈여듧

사 마다 니겨middot 킈 고

미니

훈민 언 본- lsquo rsquo 5 (1459 )

zb53) 위의 에 대한 현대어 풀이가 르~ 않은 것

① 우리 말 과 달

② 어리 말 고 는 것 어도

③ 신 생각 마 껏 펼 는 사 많다

④ 게 생각 여

⑤ 사 마다 게

zb54) 훈민정음 언해 에는 한 을 창제한 동 가 드러나

있다 훈민정음 창제의 정 과 내 이 잘 연결된 것

① 주 신 말 미 듕귁에 달

② 민 신 내 어 겨

③ 신 뻔 킈 고 미니

④ 실 신 사 마다 니겨

⑤ 귀 신 계 주 는 훈민 신과 거리가

가 엉 훈 민( ) middot middot middot middot middot世 宗 御 製 訓 民 正 音 

말 미 귁에 中 國 달 文 字

니 런 어린 니 百 姓

고 도 내 들 시러 펴 몯

미 니 내 어엿 爲 겨 새

믈여듧 니 사 마다 니 字

겨 킈 고 미니 便 安

훈민 언 본- lsquo ( )rsquo ( ) 5 (1459 )訓民正音 世祖

( )

[ 1 ]

동 룡 샤 마다 복( ) ( ) ( )海東 六龍 天福

시니 고 동( ) ( )古聖 同符 시니

[ 2 ]

매 니 곶 여

미 므 니 그 내 러

가 니

[ 125 ]

우 미리( )千世 샨( )定 에( )漢水北 累仁

누 개 샤 복 업 시니( ) ( ) 開國 卜年

신( )聖神 니 샤도 경 근민 샤 욱( )敬天勤民

드시리 다

님 쇼 산 가( ) ( )洛水 山行

미드니 가

어 가- lsquo ( )rsquo 27龍飛御天歌

다 우리신 니쓰고 다만 만 쓰( )

거 샹 귀쳔 다보게 러 귀

여 쓴 도 신 보 가 고 신 에

말 어 보게 각 에 사 들

고 본 몬 능통 후에

죠 죠 니

드 도 만 공 에 사

드 미 죠 고 고 여 보 죠

보다 얼마가 거시 어신고 니 첫

가 죠 니 죠

민 들 어 신 샹

귀쳔 도보고 어보 가 만 늘

고 폐 에 만쓴 죠 민

도 러보지못 고 보니 그게 엇지

심 니 리 보 가 어 운건 다

니 쳣 말마 지 니 고 그

쓰 에 가 우 지 지

몰 거 본후에 가 어 지

고 그니 쓴편지 쟝 보

년 학 간고사 대비2013 2 현대고 대비

ECN-0102-2013-001-000076193

쓴것보다 듸 보고 그 마 니 쓴 고

어 못

그런고 에 리 과 가

만 쓴 못 민 말만 듯고

고 편 그 못 보니 그사 단

병신 못 다고 그사 식 사

니 만 고 다 과 그사

만 고 다 과 업 사 보다 식 고

죠 도 고 각 과

견 고 실 직 귀쳔 간에 그

고도 다 것 몰 귀죡 보다

사 우리 신 귀쳔 다 업

시 신 보고 과 지 게 랴

시니 샹 귀쳔 간에 우리 신 걸

간 보 새지각과 새 걸 미리

독립신- lsquo (1896)rsquo

zb55) 친 어 나의 제 장( ) 2 매 함축적

의 가 가장 유사한 것은

① 지 눈 내리고 매 득 니 내 여 가

사- lsquo rsquo

② 도 어 리듯 그 게 어 다

주 사- lsquo rsquo

③ 눈 살 다 죽 어 린 과 체 여

눈 새벽 지 도 살 다

눈- lsquo rsquo

④ 삶 근심과 고단 에 돌 거니는 여 거 는

여 리 내린 살가지 에 눈 리 눈 리

택 그 생 에- lsquo rsquo

⑤ 늘 러 고 러

청룡 룡 어 개 루 우

신경림 계- lsquo rsquo

zb56) 친 를 위 가 나 에 나타난A B ( ) ( )

세 어의 특 에 의거하여 세 어 표 하

그 산 고 공 도 맑지만

A

주변에 쓰 리는 어리 사 많다

B

건lt gt

식 가 에 타 어 특징에( ) ( )

거 과 어쓰 는 고 지 말 것

A

B

zb57) 가 의( ) 달 아ㆍ 다 의 ( ) 나셔에서 알 있는

세 어 개화 어의 특 을 비 하여 조건 에lt gt

맞게 서 하

건lt gt

어에 는lsquo 개

어에 는 다 태rsquo

zb58) 은 가 는 다 에 나 는 절lt 1gt ( ) lt 2gt ( )

일 를 췌한 것이다 의 의 가 lt 1gt (1)~(2)

유사한 말을 에서 찾아 쓰lt 2gt

보lt 1gt

런 (1) 어린 니 고百 姓

도 내 들 시러 펴 몯 미

사 마다 (2) 니겨 便 安

킈 고 미니

보lt 2gt

죠 고 고 여 보 죠

보다 얼마가 거시 어신고 니 첫 가

죠 니 죠 민

들 어 신 샹 귀쳔

도보고 어보 가 만 늘 고

폐 에 만쓴 죠 민 도

러보지못 고 보니 그게 엇지 심

니 리

년 학 간고사 대비2013 2 현대고 대비

ECN-0102-2013-001-000076193

lt 1 gt

동 룡 샤 마다 복 시( ) ( ) ( )海東 六龍 天福

고 동 시니( ) ( )古聖 同符

lt 2 gt

(A) 매 니 곶

여 니

미 므 니 그 내

러 가 니

lt125 gt

우 미리 샨 에( ) ( ) ( ) 千世 定 漢水北 累

누 개 샤 복 업 시 니( ) ( ) 仁開國 卜年 聖

신( ) 神 니 샤도 경 근민 샤( ) 敬天勤民

욱 드 시 리 다

님 쇼 산 가 ( ) ( )洛水 山行

미드니 가

- lt gt龍飛御天歌

zb59) 장과 내 상 유사한 성격의 조는125

① 뫼 고 고 고 고

어 그린 많고 많고 고 고

어 러 는 울고 울고 가느니

도 견- lt gt

② 강 에 드니 몸 다

그믈 고 가니

뒷 뫼 엄 언 니( )藥

-

③ 말 없는 청산 태 없는 다

값 없는 청 없는 월

에 병 없는 몸 별 없 늙 리

-

④ 가마귀 골에 가지 마

낸 가마귀 새

청강에 것 시 몸 러 가( ) 淸江

-

⑤ 진 골에( ) 白雪

가 매 는 어느 곳에 었는고

에 갈 곳 몰( ) 夕陽

색-

zb60) 위 에 나타난 세 어의 특 으 적절하 않은

것은

① 룡 어 주격 사에 당 는 가 사( ) lsquo rsquo六龍

고 다

② 샤 어에도 어 주체 쓰 다

는 것 다

③ 매 어 달리 사 택에 어

가 지 지지 고 다

④ 므 원 상 직 어 지 다

⑤ 드시리 다 주체 과 상 께 사

고 다

수고 하셨습니다hearts hearts

년 학 간고사 대비2013 2 현대고 대비

ECN-0102-2013-001-000076193

보닷컴에 공 는 별 보는 고등

들 여 주 는

들 습니다 슷 동 지

가 복 는 것 도가

니 복 여 습 시고 거 시

니다

정답 해설

1) 정답[ ] ④

해설 다른 것은 두 특정 업이나 단 내에서 사[ ]

하는 일종의 은어 사회 언에 해당한다 러나

는 언이 아니라 단과대학을 여서 단대 사lsquo rsquo lsquo rsquo lsquo④

대학을 여서 사대라고 한 말에 해당하 일rsquo lsquo rsquo

사회에서도 널리 쓰이 사회 언이라 할

없다

2) 정답[ ] ⑤

해설 사회 언은 같은 단 내에서 쓰이는 언어이[ ] lsquo rsquo

동일 단끼리는 단결 과 친 감을 형성하는

능을 하 리적 안감이 일어나 않는다

3) 정답[ ] ③

해설 사람이라는 차 적 표현에 대한 대안적 표현이[ ]lsquo rsquo

인 아내 처 등으 볼 있다lsquo rsquo

4) 정답[ ]⑤

해설 남성은 주 격 체를 사 한다[ ]

5) 정답[ ] ⑤

해설 흑인은 검다라는 뜻을 가 고 있을 뿐 인[ ]lsquo rsquo lsquo rsquo lsquo rsquo

다 열등한 뜻을 내포하 않는다

6) 정답 살 색 첫 작품[ ] - -

해설 살색 혹은 킨색은 한 인의 피 색을 뜻[ ] lsquo rsquo lsquo rsquo

하는 것으 인종 차 을 추 고 출 이주민

의 평등 을 침해할 있어 년 표 이2005

살 색으 이름을 꾸었다 처녀작은 처녀라lsquo rsquo lsquo rsquo lsquo rsquo

는 단어가 가 고 있는 곡된 성 인 을 한 것

으 첫 작품정도 꾸어 사 하는 것이 좋다lsquo rsquo

7) 정답[ ] ⑤

해설 호는 아들에게 해체를 사 하고 있다[ ] ① ②

장 을 성하는 청자는 자 의 아 느리 아lsquo

들 세 이다 호는 아 느리에게 해rsquo ③

체를 사 하고 있다 호가 느리 아 에게 ④

사 한 해 체 아들에게 사 한 해체는 두 비lsquo rsquo lsquo rsquo

격 체에 해당한다 호는 자 의 아랫사람인 ⑤

느리에게 아들과 마찬가 해체를 사 하는 것이

상 이 만 임 을 한 느리에게 고마 과 쁨

존 의 표 를 하 위해 자 의 아 에게 말하듯

해 체를 사 하고 있다

8) 정답[ ] ③

9) 정답[ ] ⑤

10) 정답[ ] ①

해설 청자 할아 가 장의 주체 아 다 높을[ ] ( ) ( )

경 에는 압존 에 의해 장의 주체를 높이 않는lsquo rsquo

다 러 아 서가 아닌 아 는으 계 lsquo rsquo lsquo rsquo lsquo

니다 가 아닌 있 니다 표현하는 것이 르rsquo lsquo rsquo

11) 정답 당이 당을 쫒았다 당이[ ]

당에 다

해설[ ]

12) 정답[ ] ⑤

해설 서 다른 높임표현을 통해 청자에 대해 리[ ] ⑤

적 거리감을 나타내는 인 은 이 아니라 현정이

다 가 에서 현정은 에게 해 체를 사 함으 써 ( )

친근감을 드러낸다 나 에서 연 을 게을리하는 역 ( )

도 들 때 에 화가 난 현정이 선생님에게 항의하

는 장 에서는 하 체를 사 하여 리적 거리lsquo rsquo

가 어졌음을 나타내고 있다

13) 정답[ ] ①

해설 는 는 얼 빛이 날과 어찌 다르 고[ ] lsquo rsquo

라는 뜻으 전과 달리 임이 화자를 않고

있음을 알 있다

14) 정답 달리 후 가 있다 이를 통해 경[ ] lt gt

쾌한 음악성을 형성하고 노 젓는 상황을 체적으

형상화하는 역할을 한다

15) 정답[ ] ①

16) 정답[ ] ⑤

해설 다 의 자연은 를 성찰하게 하는 대상[ ] ( )⑤

이자 정의 대상이다 의 자연은 자 의 상황과 ⑤

처 를 드러내는 경으 서의 역할을 하 이

이 없다

17) 정답[ ] ③

해설 는 빈천 을 해결하고자 했으나 강산[ ] lsquo ( )rsquo 貧賤③

과 풍 을 달라는 에 거절하 다고 함으 써 자

연에 대한 애정을 드러내고 있으 는 않는

임에 대한 망을 개에게 전가 켜서 임에 대한 리

을 드러내고 있다

18) 정답[ ] ③

년 학 간고사 대비2013 2 현대고 대비

ECN-0102-2013-001-000076193

19) 정답[ ] ⑤

해설 고상한 음악가의 이름을 리말 꽝 럽[ ]

게 꿈으 써 언어유희를 통해 음을 유 하고 있

다 이는 고상한 척하는 총 를 비꼼으 써 비판적

태도를 드러내는 것이 대상을 꽝 럽게 표현

하여 총 의 허 과 사치를 풍자하고 있다

20) 정답[ ] ⑤

해설 는 작품 속 경에 대한 설 이 드러나는 것이[ ]

서 자의 주 적인 견해가 접적으 드러나는 것이

아니다

21) 정답[ ] ⑤

22) 정답[ ] ②

23) 정답[ ] ④

24) 정답[ ] ①

해설 적강 티프는 주인공의 비 한 출생이나 능[ ] ①

과 이 있는 것으 조정의 능함을 풍자하는lsquo rsquo

것과는 거리가 다

25) 정답 픔 나[ ] ( )

해설 의 음악은 고통 는 사람들을 위 하고 아픔[ ] lsquo rsquo

을 치유해 주는 능을 한다고 할 있다 의 lt gt

픔 도 소 된 이 과 더 어 살아가는 따뜻한 마음lsquo rsquo

을 상 한다

26) 정답[ ] ⑤

해설 에게 선천적으 주어 각 장애라는 역경[ ]

은 의 이라는 가사 연 을 있다lsquo rsquo

27) 정답[ ] ④

해설 는 장 란 선 에게 은 개인적인 인상을[ ]

소녀 장정 등으 표현한 것이다lsquo rsquo

28) 정답[ ] ②

해설 담자가 피 담자의 언어적 표현이나 비언어[ ]②

적 표현 하 독자는 담의 위 나 피

담자의 감정 상태를 알 있다 이를 통해 독자는

담 상황을 더 생생하게 느낄 있고 피 담자

를 더 잘 이해할 있게 된다

29) 정답[ ]③

해설 일상생활과 역도 선 서의 성과에 된 것에서[ ]

역도를 하 서 겪는 어 과 내적 고민으 화제를

전화하 위한 것이다

30) 정답[ ] ①

해설 릿속에 새겨 넣듯 이 억되도 함 세상[ ] ② ③

살이가 힘들고 고생 러 속 하여 자유를 ④

가 없는 고통의 상태를 비유적으 이르는 말

적의 침입을 막 위해 쌓은 축 켜야 할⑤

대상을 비유적으 이르는 말이다

31) 정답[ ] ④

해설 이 의 종류는 전 으 인 사건 경[ ] lsquo

비평을 성 소 삼는다rsquo

32) 정답[ ] ④

해설 근은 삼대독자 태어났음을 에서 확인할[ ]

있다 형제들과의 담은 이뤄 가 없다

33) 정답[ ] ⑤

해설 근은 가난에도 하고 화가를 꿈꾸었다[ ] (3

단 또한 다른 화가 망생들은 정 육을)

위해 상 학 학 해 유학 에 랐 만

근은 다른 을 찾아야 했다 단 세에(5 ) 18

근은 조선 전람회에 입선하 다 단 의(6 )

만종은 인간과 자연이 엮어 가는 경건한 조화 을lsquo rsquo

나타낸다

34) 정답[ ] ①

해설 근이 속에서도 창작활동을 추 않고[ ]

하는 닭은 은 세상과 타협할 르는

근이 세상의 이해를 하 위한 가장 떳떳한 단

이 때 이다

35) 정답[ ] ⑤

해설 전 은 서 자의 주 적인 평이 리는 것이[ ]

만 위 제 은 인 이 살았던 대 사회적 경

을 통해 객 적인 인 의 을 제 하고 있다

36) 정답[ ] ⑤

해설 전 은 인 사건 경 비평이라는[ ] lsquo rsquo⑤

성 이 어져 있다

37) 정답[ ] ①

해설 이 은 동양인과 서양인의 사고 에 차이가[ ]

있다는 것을 대조를 통해 설 하고 있다 또 쓴이

의 제자가 축 경 를 러 가서 경험한 일화를

통해 동양인이 서양인에 비해 주 상황에 더 많은

주의를 인다는 주장을 뒷 침하고 있다

38) 정답[ ] ④

39) 정답[ ] ②

40) 정답[ ] ②

41) 정답[ ] ④

42) 정답[ ] ③

43) 정답[ ] ④

44) 정답 도서 의 휴 일 도서 의 이 간 도서의[ ]

해설 도서 장은 임의 정한 휴 일과 도서 이[ ]

간 도서의 상 등을 게 할 의 가 있다

년 학 간고사 대비2013 2 현대고 대비

ECN-0102-2013-001-000076193

45) 정답[ ] ①

해설 제 조의 정 휴 일 의 휴 일의 사전 게[ ] 3

는 도서 장의 의 조항에 속한다

46) 정답[ ] ①

해설 개인 정 호 의 를 제 하 했 만 항[ ]

나눠서 제 하 않고 대 나열하고 있다

47) 정답[ ] ②

해설 제 조의 내 을 회사는 다른 회사 협[ ] 7 lsquo

계약을 통해 서비 를 제공하는 경 회 의 아이디

등 개인 정 를 해당 회사에 전송할 있다는 내rsquo

이 있으 의 제점을 제 할 있다②

48) 정답[ ] ④

해설 는 도서 장의 의 에 해당하고 나 는 도[ ] ④

서 장의 리에 해당한다

49) 정답[ ] ③

50) 정답 은 음독으 적었고 은 훈독으 적었[ ] (1)

다 과 동일한 표 리 적은 것은 이고 (2) ce

과 동일한 표 리 적은 것은 이다ab

51) 정답[ ] ③

52) 정답[ ] ①②

53) 정답[ ] ③

54) 정답[ ] ③

55) 정답[ ] ①

56) 정답 른 죠코 어린 노 하니라[ ] A B

57) 정답 세 어에서는 활 형이 칙적으[ ] lsquo rsquoㄹㅇ

나타났 만 개화 어에서는 활 형이 쓰 다 lsquo rsquo ㄹㄴ

58) 정답 호 가 흔[ ] (1) (2)

59) 정답[ ] ④

60) 정답[ ] ③

Page 3: 현대고대비 국어 - chamsoriedu.com 「콘텐츠산업진흥 법」외 에도 저작권 의하여 ... 다른주체에게어떤동작을하도록만드는것을나타내는

년 학 간고사 대비2013 2 현대고 대비

ECN-0102-2013-001-000076193

① 지 지는 고 습니다

② 지 지께 는 고 계십니다

③ 지 지께 는 고 계시 니다

④ 사 님 에 에 갔습니다

⑤ 사 님 님께 는 에 에 가 습니다

가( ) S1

소장 자네에게 을 게 되어 정말 안하네 자네에게 뒷

일을 맡겨야겠네

나 자 네 여 는 당입니다 한때( ) 1 당에 쫓

도 했 니다 하 만 선거 후 전에 의 있는 정책

으 선거전을 서 당 위 를 끌어 린 것이

당을 고 리를 이끈 인으 고 있 니다

자 네 여 는 당입니다 선거 후 전 터 당2

위 가 한풀 이 서 세에 었는데 일 의

들의 눈에는 눈 이 맺혀 있 도 합니다

다 과량 사 액이 삼켜져서 위장 흡 에 의한 전( )

작 도 일어날 있 니다 따라서 액이 삼켜 않도

주의해야 합니다

라 통의 날을 맞아 늘 에서는 통사고 절 으( ) lsquo rsquo lsquo

이 캠페인을 내 곳곳에서 다채 게 펼쳤 니다 늘rsquo

행사는 많은 민들의 호응을 이끌어 냈 니다 계

자는 제 통사고 생률을 낮출 있는 체적인 정책도

마 하고 있다고 말했 니다

마 통의 날을 맞아 휴일인 늘 내 곳곳에서는 다양한( ) lsquo rsquo

통의 날 행사가lsquo rsquo 개최되었 니다 써 터 통의 날 lsquo rsquo

행사가 생색내 에 나 않는다는 비판이 제 되고 있 니

다 체적인 정책적 뒷 침이 마 되 않는다 이러한 비

판의 소리는 점점 더 커 것으 전망됩니다

zb11) 다음 조건 에 맞게 나 의 을 능동 으 을lt gt ( ) lsquo rsquo lsquo rsquo

피동 으 고쳐 각각 한 장으 서 하

조건lt gt

은 이가 을를 다 조의 능동 으1 lsquo- ( ) - ( ) - rsquo

은 이가 에 다 조의 피동 으2 lsquo- ( ) - - rsquo

에 사 할 단어는 나에서 찾아3 lsquo-rsquo ( )

서 어의 제 높임은 상 없이 장을 만들 것4

가 역도 앞 낮( ) S 38 ( )

냉장고를 열고 펄떡거리는 어들이 들어 있는 낚

망을 내는 고 를 매 탕과 함 맛

있게 을 고 있는 과 아이들 인다

현정 적 적 만날 맛난 거 어 좋 는 허 만 이제 나( )

도 값을 해야 헐튼디 맨날 이 만 하고 선생님

리 역도는 언제 가르쳐 대

당황해서 둘러대듯 초 체 이 한 ( ) helliphellip

거야 많이 어 근엄한 말투 역도는 일단 이 을 ( )

해야 할 있는 거다 략 lt gt

나 학 동장 치 낮( ) S 40 ( )

여 후 에게 똥침을 놓고 도망간다 장난 럽게 달

리 를 하고 있는 역도 들 치에 앉아서 아이들을 라

는 에게 다가 는 현정

현정 주장으 서 할 말 있 니다 훈 위 고 장난쳐도

혼내 도 않으 고 리고 역도 는 여 이처럼 꼬 꼬

한 아이들만 뽑 니

아이들을 딱하잖냐 래도 자들이 착하다 주장 ( )

이 이해 좀 해

답답한 표정 는 현정

zb12) 가 나 의 장 을 이 는 소에 대한 설 으 적( ) ( )

절하 않은 것은

① 가 청 는 동 다( ) ( )

② 가 시간 상 낮 다( ) ( )

③ 가 에 공간 상 달 다( ) ( )

④ 가 에 심리 태도는 다 게( ) ( )

타 다

⑤ 가 에 지 다 통( ) ( )

에 심리 거리감 타낸다

가( )

가 각시 본 듯도 고

상( )天上 경( )白玉京 엇 니별 고( ) 離別

년 학 간고사 대비2013 2 현대고 대비

ECN-0102-2013-001-000076193

다 믄 눌 가시 고

어 여 고 내 드러 보

내 얼 거동 님 가마

엇 보시고 다 실

도 님 미 업

어 러 돗

시 엇 다 신고

누어 각 고 니러 여 니

내 몸 지 죄 뫼 티 시니

원망 사 허믈 랴

워 니 믈 타시 다( ) 造物

각 마

략( )

님다 쇼식 므 쟈 니( )消息

도 거 다 사 가

내 업다 어드러 가쟛 말고

거니 거니 뫼 가니

니 개 므 고

산쳔 어 거니( ) (A)山川 월( )日月 엇 보

지쳑 거든 쳔리 보랴( ) ( ) 咫尺 千里

리 믈 가 보쟈 니

믈결 어 고

샤공 어 가고 만 걸 니

강 쟈 보니( )江天

님다 쇼식 득 고( )消息

략( )

근 역진 간 드니( )力盡

지극 님 보니( )精誠

얼 마 늘거( ) ( ) 玉 半

근 말 슬 쟈 니

눈믈 니 말 들 어

못다 여 니( )情

계 ( )鷄聲 엇 돗 고

어 허 다 님 어 간고( ) 虛事

결 니러 열고 보니( )窓

어엿 그림재 다

리 싀여 여 월( )落月

님 겨신 ( )窓 드시 리

각시님 니 쇼

미 곡- lt gt-

( )

내 님믈 그리 우니다니

(B)산 동새( )山 슷 다

니시 거 신

월 시리 다( ) 殘月曉星

시 도 님

벼 시니 뉘러시니 가

과 도 허믈도 만 업 다( ) ( ) 過 千萬

마리신

니미 마 니 시니 가

님 도 드 샤

과- lt gt-

zb13) 가 의 에 대한 설 으 적절하( ) ~ 않은 것은

① 과 달리 고 다

② 별 신 탓 여 고 다

③ 별 다 가 고 다

④ 산에 는 망 취 지 못 lsquo rsquo

⑤ 실 룰 없는 망 실 시키고 신

다 재 변 고 다

사( ) 5春詞

고 볃티 믉결 다

어 어

그믈 주어 랴 시 가

지 지 어( ) ( ) ( )至匊悤 至匊悤 於思臥

탁 가 니 고 도 니 다( ) ( )濯纓歌 興

사( ) 2夏詞

닙 싸 고 쟝만 마

닫 드러 닫 드러

청 립 사 가 냐( ) ( )靑蒻笠 綠蓑衣

년 학 간고사 대비2013 2 현대고 대비

ECN-0102-2013-001-000076193

지 지 어( ) ( ) ( )至匊悤 至匊悤 於思臥

심 내 가 가( ) ( )無心 白鷗

사( )3秋詞

운 니러 고 티 니 다( )白雲

돋 돋

믈 믈 동 가쟈( ) ( )西湖 東湖ㅣ

지 지 어( ) ( ) ( )至匊悤 至匊悤 於思臥

료 곳마다 경 다( ) ( )白蘋紅蓼 景

동사( )4冬詞

간 눈 갠 후 에 경믈 달 고( ) ( )後 景物

어 어

만경 리 듸 쳔 산( ) ( )萬頃琉璃 千疊玉山

지 지 어( ) ( ) ( )至匊悤 至匊悤 於思臥

계 가 계 가 간 니 다( ) ( ) ( )仙界 佛界 人間ㄴ ㄴ

zb14) 위 을 의 조 형태적 특 에서 어lt gt (1)

떤 차이가 있는 를 서 하고 인해 노래의 아 (2)

름다 에 어떤 향을 주었는 에 대해 서 하

보lt gt

십 경 여 가삼간 지어내니

달 청 맡겨 고

강산 들 곳 없 니 러보고 보리

가 가 각시 본 듯도 고( )

샹 경 엇 니 별 고天 上 白 玉 京 離 別

다 믄 눌 가시 고

어 여 고 내 사 드러 보

내 얼 거동 님 가마

엇 보시고 다 실

도 님 미 업

태 어 러 돗

시 낫 엇 다 신고

누어 생각 고 니러 여 니

내 몸 지 죄 뫼가티 싸 시니

원망 사 허믈 랴

워 니 믈 타시 다造 物

님다 쇼 식 므 쟈 니( ) 消 息

도 거 다 내 사 가

내 마암 둘 대 업다 어드러 가잔 말고

잡거니 거니 놉픈 뫼해 라가니

은 카니 안개난 사 일고

산 쳔이 어둡거니山 川 일日 月 을 엇디

쳑을 라거든 쳔 리랄 라 랴咫 尺 千 里

찰하리 가의 가 히나 자 하니

람이야 결이야 어둥졍 된뎌이고

샤공은 어대 가고 븬 만 걸 나니

강 텬의 혼쟈 셔셔 디난 해랄 니江 天

님다히 쇼 이 더 아득한뎌이고消 息

다 쳠 리 듕만 도 니( ) 茅

벽 쳥 등 눌 갓 고半 壁 靑 燈

라 나리 헤뜨 니니

져근덧 녁 하야力 盡 풋잠을 잠간 드니

졍 셩이 하야 꿈의 님을 니精 誠

가탄 얼 이 이나마 늘거셰라玉 半

마암의 근 말삼 카장 삷쟈 하니

눈 이 라 나니 말인들 어이하

졍을 다하야 이조차 여하니情

뎐된 계 셩의 잠은 엇디 돗던고鷄 聲

라( ) (A)찰하리 여디여 낙 이나 되야 이셔洛 月

님 겨 창 안해 드 비최리라窓

(B)각 님 달이야카니 잔 비나 되쇼셔

zb15) 위 가 라 에 대한 설 으 적절하( )-( ) 않은 것은

① 민들 과 삶 담고 다

② 여 체 식 어 다

③ 사미 곡과 께 가사 미 다

④ 우리말 사가 평가

년 학 간고사 대비2013 2 현대고 대비

ECN-0102-2013-001-000076193

고 다

⑤ 님 미 사 는 마lsquo rsquo

가사 다

가 춘사( ) [ 5]

고은 티 쬐얀 결이 름 다

이어라 이어라

을 주어 두랴 낙 노흘일가

총 총 어( ) ( ) ( )至匊葱 至匊葱 於思臥

탁 가 의 흥 이 나니 고 도 니 다( ) ( )濯纓歌 興

하사[ 2]

년닙희 두고 찬으란 쟝만 마라

닫 드러라 닫 드러라

청약립 은 써 잇노라 녹사의 가져 냐( ) ( )靑蒻笠 綠蓑衣

총 총 어( ) ( ) ( )至匊葱 至匊葱 於思臥

내 좃 가 제 좃 가( ) ( )無心 白鷗

추사[ 3]

이 니러나고 나 티 흐느 다( )白雲

돋 라라 돋 라라

의 셔호 혈 의 동호 가쟈( ) ( )西湖 東湖ㅣ

총 총 어사( ) ( ) ( )至菊悤 至菊悤 於思臥

빈홍 곳마다 경 이 다( ) ( )白蘋紅蓼 景

동사[ 4]

간 의 눈 갠 후 에 경 이 달랃고야( ) ( )後 景物

이어라 이어라

압희 만경류리 듸희 쳔텹 산( ) ( )萬頃琉璃 千疊玉山

총 총 어( ) ( ) ( )至菊悤 至菊悤 於思臥

선계 가 블계 가 인간 이 아니 다( ) ( ) ( )仙界 佛界 人間ㄴ ㄴ

윤선도 어 사 사- ( )漁父四時詞「 」

나 살어리 살어리랏다( ) 쳥산 애 살어리랏다( ) 靑山

위랑 래 랑 고 쳥산 애 살어리랏다 ( ) 靑山

얄리얄리 얄랑셩 얄라리 얄라

러라 러라 새여 자고 니러 러라 새여

널라 름 한 나도 자고 니러 니 라

얄리얄리 얄라셩 얄라리 얄라

청산별곡- ( )靑山別曲

다 청산은 엇뎨 야 만고애 프르르( ) ( ) ( ) 靑山 萬古

유 는 엇뎨하야 주야애 디 아니난고( ) ( )流水 晝夜

리도 치디 마라 만고상청 호리라( ) 萬古常靑

도산 이곡- ( )陶山十二曲

라 개를 여라 이나 르되 개 치 얄 랴( )

님 꼬리를 홰홰 치 치뛰락 나리뛰락 겨서 내

닷고 님 뒷 을 동 동 르락 나으락 캉캉

도리 암캐

이 릇 릇 날 들 너 이 이 랴

작자 상-

마 빈천 을 랴 고 에 드러가니( ) ( ) ( )貧賤 權門

침 업 흥졍을 뉘 져 쟈 리

강산과 풍 을 달나 니 는 리 리

조찬한-

정정 이랬거니 아람도리 큰 솔이 혀( ) ( )伐木丁丁

도 하이 골이 어 아리 소릴 쩌르 돌아 도 하이 다

람 도 좇 않고 새도 않어 은산 고 가 차라리

뼈를 저리 는데 눈과 이 종이 담 회 나 달도 름을

다 흰 뜻은 한 이골을 걸음이란다 절 이 여섯

판에 여섯 고 고 라간 뒤 조찰히 늙은 사나이의 남

내음새를 는가 름은 람도 일 않고 고 에 히

흔들리 노니 견디란다 차고 연 히 픔도 꿈도( )兀然

없이 장 산 속 겨 한 내-

정 장 산- 「 」

zb16) 다 에 나타난 상과 표현을 활 해 작 연 을 해( )

것이다 적절하 않은 것은

① 몰 는 도에 몸 맡 고 는 고통

욱 답게 가꾸 는 느님 산 겠지

② 돌 틈 뚫고 어 민들 보 리 진 고통

다가 도 민들 럼 강 게 살 지 겠는가

③ 벽에 런 움 없 어지고 는 폭포

여 시 는 죽 워 지 는 강

지 는가

④ 상 겨울 도 다리 꿋꿋 살 가고

겠지 망 지 는 삶 얼마 다운가

년 학 간고사 대비2013 2 현대고 대비

ECN-0102-2013-001-000076193

⑤ 막 늘 울리고 는 귀 미 리는

지새우는

가( ) 빈천( )貧賤을 랴 고 에 드러가니( )權門

침 업 흥졍을 뉘 져 쟈 리

강산과 풍 을 달나 니 는 리 리

나 청강 에( ) ( )淸江 비 듯는 소 어 읍

만산 홍 이 휘드르 는고야( )滿山紅綠

두어라 춘풍 이 날이리 을 어라( ) 春風

다 청산은 어찌 야 만고에 푸르르( ) ( ) ( )靑山 萬古

유 어찌 야 주야애 디 아니 고( ) ( )流水 晝夜

리도 치 말아 만고상청 하리라( ) 萬古常靑

라( ) 개를 여라 이나 르되 개 치 얄 랴

님 리를 홰홰 치 치 락 리 락

겨서 내닷고 고 님 뒷 을 동 동

르락 나으락 캉캉 도리 암

이 릇 릇 날 들 너 이 이 랴

zb17) 의 적 능을 비 한 내 으 가장 적절한

것은

① 는 가 지닌 실 계 다

② 는 내 갈등 심 시킨다

③ 는 가 는 상 각시킨( )愛着

④ 는 달리 내 갈등 시킨다

⑤ 는 달리 다 사 들 간 가

강 다

가 님다히 쇼 을 아 나 아쟈 니( ) ( )消息

도 거의 다 일이나 사 가

내 둘 업다 어드러 가쟛말고

잡거니 거니 놉픈 뫼 라가니

은 니 안개 일고

산쳔 이 어둡거니 일 을 엇디( ) ( )山川 日月

쳑 을 거든 쳔 리 라 랴( ) ( )咫尺 千里

하리 의 가 히나 쟈 니

람이야 결이야 어둥졍 된뎌이고

샤공은 어 가고 븬 만 걸 니

강텬 의 혼쟈 셔서 디 니( )江天

( )

쳠 자리의 듕만 도라 니( )茅簷

쳥등 은 눌 위 야 갓 고( )半壁靑燈

리 헤 니니

져근덧 녁 야 픗 을 잠간 드니( )力盡

졍셩 이 야 의 님을 니( )精誠

가 얼 이 이나마 늘거셰라( ) ( )玉 半

의 근 말 장 쟈 니

눈 이 라 나니 말인들 어이

졍 을 다 야 이조차 여 니( )情

뎐된 계셩 의 은 엇디 돗던고( )鷄聲

어 허 다 이 님이 어 간고( )虛事

결의 니러 안자 창 을 열고 라 니( )窓

어엿븐 림재 날 조 이 다

하리 여디여 낙 이나 되야이셔( )落月

님 겨 창 안 드 비최리라( )窓

나 내 님 리자 니다니( )

산 접동새 난 이 이다

아니 거츠르 아으

잔 효성이 아 리이다

넉 라도 님은 녀져라 아으

더 니 뉘러 니잇가

과도 허 도 천만 없소이다

힛마리 뎌

읏븐뎌 아으

니 나 마 니 니 잇가

아소 님하 도람 드르샤 쇼셔

zb18) 맥으 아 에 들어갈 적절한 것은

① 시 엇 다 신고

② 다 믄 눌 보 가시 고

③ 님다 쇼식 욱 득 고( )消息

④ 원망 사 허믈 랴

년 학 간고사 대비2013 2 현대고 대비

ECN-0102-2013-001-000076193

⑤ 죠 뫼 티 시 가( ) ( ) 粥早飯 朝夕

가( )

거리

공신 후 심 늦도( )劉尋

식 없어 과 께 산에 드리고 신

태몽 꾼 에 만고 웅 상 지닌 들

낳 키운다 그 후 신 들 에 역심 ( )逆心

담 귀 등 심 여 리 귀

보내고 지 죽 는 도망 가다

가 만 죽 고 에 경 가는 들 도움

살 다 그러 사 에 심 귀

보고 담 여 고 강 주가 승상

득 여 고 신 사 삼는다 그 후 강 승상

에게 심에 상 리지만 여움

사 귀 가게 다 강 승상 몸 는

연 과 헤어 리 다

경쇠 리 들리 에 들어가니 색

에 게 단청 누각과 큰 집들( )丹靑

다 주 보니 ( ) (一柱門 黃金

산 사 어 었다 산) lsquo rsquo 大字

들어가 고승 다 그( ) ( ) 山門 高僧

거동 보니 눈 눈 듯 고

변 같 귀는 어 에 늘어 니( ) 白邊

맑고 어 골격과 신 평 니었

팔염주를 에 걸고 육환장 을 고서 흑포( )六環杖

장삼에 떨어 송낙 쓰고 나 유생을 고 말[ ] 松蘿

하 를

소 이 연 하여 유상공 는 행차를 동 에 나ldquo

가 맞이하 하 으니 소 의 함을 서하

rdquo

유생이 크게 놀라 말하 다

천한 인생으 팔자가 하여 어 서 를 잃고ldquo

정처없이 다니다가 연히 이곳에 대사를 만난 것인데

토 대하 소생의 성은 어떻게 알고 있 니 rdquo

노 이 답하여 말하 를

어제 남악 형산 의 화선 이 소 의 절에ldquo ( ) ( )男樂 衡山

어서 소 에게 탁하 를 내일 낮 경에 남경 lsquo 12

동성 안에 사는 유 의 아들 충 이가 것이니 내쫓

말고 잘 대접하라 하셨 니다 마침 소 이 찾아 나rsquo

다가 상공의 차람새를 니 남경 사람이 에 알아

았 니다rdquo

유생이 말을 듣고 한편으 쁘고 한편으 퍼하

서 노 을 따라 들어가니 여러 들이 합장 하

가 했다 노 이 에 들어가 저녁 을 은 후에

을 편히 니 이곳은 선경 이었다 세상의 일을( ) 仙境

두 잊고 일 이 편안하 다 이후 는 노 과 함

서 도 이 탐 하고 경도 확하게 의 게 되었( )兵書

다 이 게 되니 대 천 에 가객 은 없 ( ) ( )大明天地 佳客

고 덕산 속에 리 른 만 있더라 래 ( ) 廣德山

이 천상 사람으 살아 있는 처를 만나 이한

을 니 재주 민함을 누가 당할 있겠는가

낮으 공 하더라

유충 전- -

웬늠 어가 사 싸다냐( ) ldquo rdquo

내가 가 막 런거 니

보통것 닐러 그 어낸ldquo ( )

틀어 주 그 가 루 러 허 에

싶어 키 틀어 주 그 가( )

루 허 우간 곡 틀어주는 루 못 는

웂는 고 닝께 고 지 들

어 사는 고 가 다는 건 에 그 집에

rdquo

그런 단 어들 어 새벽에 떼죽 거

다 고 어 보니 죄다 허 게 집어진

는 것 었다

총 가 내화를 꿴 뛰어나 만 아 소 없는

일이었다

어떻게 된 거야ldquo rdquo

한동안 넋나간 듯이 서 있던 총 가 하고많은 사람

에 하필이 유자를 겨냥하 은 말이었다

쎄유 아마 새에 고뿔이 들었던 개비네유ldquo rdquo

유자는 러 딴청을 하 다

야 고 가 에서 감 가 들어 죽는 고 두ldquo

어rdquo

총 는 가 혐의자 나 되는 것처럼 화풀이를( )嫌疑者

하 드는 것이었다

는 비위가 상해서

야 팔자가 사나서 이런 후 에 살라니|

여러 가 다 객고가 쌓여서 조 두 안 좋았을 테 helliphellip

런디다가 릇쓰 이 가락을 트는 대 디립

다 춰댔으니 과 해서 살끼두 다소 있었을 테 helliphellip

래 들어서 키 는 새끼덜일 이 다다 탈이 많은

이니 ldquohelliphellip

는 트의 독성을 충 히 내 않고 고 를 넣

은 것이 탈이었으 니 하 서도 러 참으 의 을 떨

었다 략 - -

마리가 마리 값 간다는 워ldquo

그냥 내뻔지 거시 허 싼 고 는 맛

겄다 싶 허 게 눌 강 어helliphellip

허 마늘 통 다

년 학 간고사 대비2013 2 현대고 대비

ECN-0102-2013-001-000076193

게 지 고뿌 지 rdquo

어 어째ldquo rdquo ldquo rdquo

런 도 것들 같 니ldquo ( ) rdquo殘忍無道 helliphellip

는 탱 여 지 못 다 보( ) 憤氣撐天

니 는 는 다 동원 여 통 쳤

생각 여 는 눈 다

달리 리헐 감ldquo rdquo

들 고 말 니었다 그가

는 것 그 말고는 없었 에 그 게 뒷동

달 거 다

는 우 럽고 식 짝 없는 랫것들 고

다 공연 신 가고 득 것

없다고 단 는지 결 웬만큼 고루 어

그 것들 쪽 에다 고 어주지ldquo

고 그 그걸 주 어 에 에 helliphellip

눈 없는 독 들 rdquohelliphellip helliphellip

고 말 럼 얼거리 들어가 리는 것 었

- ( ) -兪子小傳

zb19) 위 나 를 읽고 평가한 것으 적절하( ) 않은 것

① 사 리 통 감과 사실 고

격 과 달 고

② 는 가 재 컫는lsquo rsquo lsquo rsquo

미 가진 여 는 것 겠

③ 는 식 말 는 웃 상lsquo rsquo

여 는 미 지니고

④ 는 어가 죽 짐 지만 내색 지 고lsquo rsquo

말 고

⑤ 언어 통 가들 여 우리 통

것들 역 고

가 체 거리( ) [ ]

나라 종 연간에 정언주 의 을 하고( )正言注簿

있던 유 은 늦도 자 이 없어 한탄하다가 남악 형산lsquo rsquo

에 치성을 드리고 이한 태 을 꾼 뒤 아들을 낳아 이름

을 충 이라 고 키 다 이때 조정의 하들 에 역

을 품은 정한담 최일 등이 가달의 침입에 대한( ) 逆心

유 의 유화적 입장을 제 삼아 유 을 함하여 양

내고 유 의 에 을 러 충 자마저 살해하

한다 러나 충 은 천 조 정한담의 마 에서 어

나 많은 고난을 겪다가 은퇴한 재상 강희주를 만나 사위

가 된다 강희주는 유 을 하 고 상소를 으나 정

한담의 공격을 아 양을 가게 되고 강희주의 가족은

난을 피하여 두 흩어 다 충 은 강 소저 이 하고

사의 노 을 만나 를 때를 다린다 이

때 남적과 적이 를 들고 나라에 쳐들어 자 정한

담은 자 출전하여 남적에게 항 하고 남적의 선 장이

되어 천자를 공격한다 정한담에게 여러 패한 천자가

항 하 할 음 충 이 등장하여 남적의 선 정 걸

을 죽이고 천자를 출한다 충 은 단 으 란 을

쳐 고 정한담을 사 잡는다 리고 호 에게 ( )胡王

잡혀간 황후 태후 태자를 출하 유 에서 고생하

던 아 유 과 장인 강희주를 한다 또한 이 하

던 어 니 아내를 찾고 정한담 일파를 리친 뒤 높은

에 라서 화를 누린다

사 들 별 고 없 다니었다( )

마 마 돌 다니 걸 여 고

어 곤 다 에는 동쪽에 고

에는 쪽에 니 가 에 리는 엽

가는 없 니 늘 다니는 었

다 얼 말 죽 사 같고 림새가 말

니었다 가슴 에 고 등

삼태 헌 에 니 달 ( )奇男子

가 도리어 걸 었 담 만 열 도 ( )傅說

고 만났고 만 갈( ) ( ) 慇 武丁

도 탕 만났( ) ( ) (伊尹 成湯 渭

여상 도 주 만났는) ( ) ( ) ( ) 水 呂尙 周 文王

월 같 러가 도 어느 열 살

늘과 집 삼고 사 에 쳐 거리에

어 다가 곳에 니 다 ( ) 楚

지 다가 사 보고 가에 다다( )長沙

니 망 가에는 원 리가 슬 고 가

가 내리는 사 에는 갈매 가 갈 뿐 었다

쪽 돌 보니 가 우거 고

가 사 보 었다 그곳에

가니 는 사( ) 汨羅水

는 다 주 가 쓰고 죽고

곳 었다

마 감 여 에 가 사 살펴보니

에는 삼 고 그 에( ) 屈三閭

는 만고 월 과 지 가는 그 들( )風月

가 어 었다( ) 路程記

동쪽 벽 에 새 운 어 거늘 그

보니 월 에 경 주 는 간신에게ldquo ( )敗

보고 연경 귀 가다가 에 죽 rdquo

거늘 그 보고 에 거꾸러

통곡 말

[A]ldquo우리 연경 간 만 니 에

지 살 상에 엇 겠는가

에 고 에 었 니

상에 살 것 가 도 께 지리 rdquo

년 학 간고사 대비2013 2 현대고 대비

ECN-0102-2013-001-000076193

고 가에 내 가니 울 리가 에 지 사

쳤는지 심 심 것 가

신 심 것 가

다( ) 강 승상에게는 들 없고 다만 만

었다 가 낳 에 가 색

타고 내 에게 말 는 ldquo

니다 미원 과 연 맺고 ( ) ( )紫薇垣 緣分

었는 께 강 집 보내 에

니 게 여겨 주십시 거늘 rdquo

미 가운 낳 니 가 고 거동

단 다 시 짓 쓰 고 는 (音

없었 니 여 가운 지 는 짝) 律

룰 만 사 없었다 가 사 여 사 감

게 고 지 못 고 염 는 만다

다가 당에 거 고 식같 러 내니

고귀 상 루 말 다 어 울 도 다( ) 相

귀 사 없고 웅 걸( )富貴爵祿

만고 었다 승상 매우 뻐 내당 ( )內堂

들어가 에게 사 니 역시 매우 거

워 말 다

ldquo 도 마 사 는 승상께

그 게 말 시니 상 여러 말 지 말고

사 도 시다rdquo

상이 에 나 충 의 손을 잡고 결혼과 하여 ldquo

너에게 히 할 말이 있다 내가 늙은 말년에 딸

하나만을 두었는데 니 너 하늘이 정해 필

임이 하다 이제 년고락 을 너에게 탁 ( )百年苦樂

하겠다 하 대 충 이 릎을 꿇고 앉아 눈 을 흘리rdquo

여쭈었다

소자의 을 해주 고 또 하 에 두고자 하ldquo ( )膝下

니 감사하 이를 데가 없 니다 다만 가 속에 통탄

할 일이 사 쳐 있 니다 소자가 이 없어 양친 ( )兩親

의 생사를 른 채 결혼하여 아내를 얻는 것은 자 으

서 할 도리가 아닙니다 이것이 한 러 뿐입니다 rdquo

승상 그 말 듣고 슬 에 어 고

것 에 맞 어 변 게 리ldquo

는 다 집 시 공 도 여 ( )始祖公

고 가 에 가가 어진 만 개 공신

었 니 도 러워 말 시고 시 rdquo

택 여 니 다운 신 과 신

습 늘에 죄 짓고 간 상에 내 신

혼 를 다 끝내고 으 들어가 사 을 살펴 니 빛

나고 빛난 것이 한 입으 는 다 말하 어 고 하나

는 다 하 어 더라 에 켠 환한 촛 ( )新房

아래 은 에 랑과 가 평생의 연 을 맺었( )緣分

으니 서 사랑하 주고 은 말을 어떻게 다 헤아릴

있으 어떻게 다 하리 을 낸 후에 이튿날

상 를 니 상 거 마음을 이 하

더라

각 생 강 승상 집 쪽( )

늘 보고 없 가 신 신 생각 니

없고 어 없었다 는 어떻게 도리가 없다

여 산 에 들어가 리 고 어 도 닦

고 다 그 산 보고 가다

가 곳에 다다 니 에 큰 산 었다 많 우

리 골짜 가 늘 는 가운 색

에 고 갖가지 가 짝 어 었 ( )花草

다 신 산 생각 고 들어가니 경개 ( )景槪

가 매우 뛰어 고 경 산 다 산 리에 들

리는 것 리 보 는 것 울 청산뿐

었다 가 고 울 어 가

니 들 많 가지들 못 어 동

에 늘어 들거리 는( ) 洞口

우거진 가지에 갖 들 다 었다( ) 春情

계상 에는 공 는 늘( ) 花溪上

에 걸린 폭포가 벽 는 리는 산사( )層巖絶壁

쇠 리 객 에 는 듯 늘( ) ( ) 寒山寺 客船

에 싸여 는 습 산

그린 여 병 러 듯 다 경쇠 리가 들

리 에 들어가니 색 에

게 단청 누각과 큰 집들 다( ) 丹靑

주 보니( ) ( ) lsquo一柱門 黃金大字

산 사 어 었다 산 들rsquo ( )山門

어가 고승 다 그 거 보니( ) 高僧

눈 눈 듯 고 변 같 ( )白邊

귀는 어 에 늘어 니 맑고 어 골격

과 신 평 니었다 염주

에 걸고 짚고 포 삼에 어진( )六環仗

쓰고 생 보고 말

승 연 여 상공 시는 동 에ldquo

가 맞 지 못 니 승 십시 rdquo

생 크게 말 다

생 가 여 어 고ldquo

없 다니다가 우연 곳에 사 만 것

그 시 생 어떻게 고 습니 rdquo

승 답 여 말

어 산 승 에ldquo ( ) ( )南岳 衡山

시어 승에게 탁 내 낮 시경에 경 lsquo 12

동 에 사는 심 들 가 것 니 내쫓

지 말고 습니다 마 승rsquo

다가 상공 림새 보니 경 사 에 보

습니다rdquo

zb20) 위 의 친 에서 서 자의 개입이 드러나~

는 이 아닌 것은

① 달 가 도리어 걸 었( ) 奇男子

② 신 심 것 가

년 학 간고사 대비2013 2 현대고 대비

ECN-0102-2013-001-000076193

③ 다운 신 과 신 습 늘에 죄 짓고

간 상에 내 신 다

④ 사 주고 말 어떻게 다 헤 릴

어떻게 다 리

⑤ 신 산 생각 고 들어가니 경개 가 ( )景槪

매우 뛰어 고 경 산 다

거리 연[ ] ( )弘治

간 에 공신 후 에 언(1488~1505) ( )正言

주 는 벼슬 심 늦도 식( ) ( )主簿 劉尋

없어 과 께 산에 드리고 신 태

몽 꾼 에 만고 웅 상 지닌 들 낳

키운다 그 후 신 들 에 역심( )逆心

담 귀 등 심 여 리 귀 보내

고 지 죽 는 도망 가다가

만 다 에 에 어 니

헤어지게 다

에 에 어 니 헤어지게

다 그 후 사 들에게 우연 돌

생 다가 어느 열 살 었다 열 살

지 다가 우연 귀 견 는

그것 그 살 도 었고 그

귀 본 신도 지 죽고 마 고

크게 운다

( )

에는 강 주 는 재상 살고 었

니 시 에 과거에 격 여 승상 벼슬 다가 간

신 만 벼슬 그만 고 고 돌 었

다 그러 신 지 가 지 못 여 상

가 못 결 는 상 여 원 니

신 들 그 직간 꺼 다 그 에 도

담과 귀가 강 승상 가 미워 다

강 승상 마 본 에 갔다가 돌 는[A][ ( )本府

에 우편 주 에 다가 색( ) ( ) 右便 酒店

에 어리었는 청룡 에 지 늘

여 통곡 고 사 는 꿈 꾸었다] 마

상 게 생각 여 새 다리다가 새벽

닭 울고 가 달 갔다 가 보니

과연 어 동 가 가에 울고 는지 달

들어 그 고 사 에 어 말

는 어 어 에 어 가ldquo

닭 곳에 우느냐 니 울rdquo

그 고 답 여 말 다

는 경 동 에 사는 언 주 공 들ldquo

니다 께 간신 만 연경 귀 가

시다가 에 죽 사 에 는 닭에

도 에 죽고 니다rdquo

강 승상 말 듣고 크게 낯 변 말

것 웬 말 냐 근 동 ldquo (老

못 갔 니 그 사 변 여)患

런 변 었단 말 가 주 는 신 다

같 에 벼슬 다가 는 가 많 들어 고

돌 는 주 가 게 꿈 에 생

각 겠느냐 생각지 못 다 미 지 간

지지 말고 께 가 략rdquo ( ) hellip hellip

죽게 주 사당에 단 도 러운

겠느냐 말 말고 시는지 rdquo

어 없어 강 승상 가니 그곳

월계 었다

다( )

가가 고 지 사 들 가( )櫛比

통 는 리가 과

답게 꾸민 누각과 큰 집들 늘 고

게 식 가 어 들 태운 가고

었다 략 강 승상에게는 들 없고 ( ) hellip hellip

다만 만 었다 가 낳 에

가 색 타고 내 에게 말

는 니다 미원 과ldquo ( )紫薇垣

연 맺고 었는 께 강 집( )緣分

보내 에 니 게 여겨 주십시

rdquo

거늘 미 가운 낳 니 가

고 거동 단 다 시 짓 쓰 고

는 없었 니( )音律 여 가운

지 는 짝 룰 만 사 없었다 가 사

여 사 감 게 고 지 못 고 염 는 만다

다가 당에 거 고 식같

러 내니 고귀 상 루 말 다 ( )相

어 울 도 다 귀 사 없 ( )富貴爵祿

고 웅 걸 만고 었다 승상 매우 뻐

내당 들어가 에게 사 니( ) 內堂

역시 매우 거워 말 다 도 마 ldquo

사 는 승상께 그 게 말 시니

상 여러 말 지 말고 사 도 시다rdquo

( )

승상 에 고 결 과 ldquo

여 에게 말 다 내가 늙 말 에 지

만 었는 지 보니 늘

다 에게 탁 겠 ( )

다 신 꿇고 눈 리rdquo

여 었다 주시고 슬 에 ldquo ( )膝下

고 시니 감사 룰 가 없습니다 다만 가슴

에 통탄 사 쳐 습니다 복 없어

생사 결 여 내 얻는 것( )兩親

식 도리가 닙니다 것 러울 뿐 니

다rdquo

상 그 말 듣고 슬 에 어 고 말

것 에 맞 어 웅변 ldquo

년 학 간고사 대비2013 2 현대고 대비

ECN-0102-2013-001-000076193

게 리 는 다 집 시 공도 여

고 가 에 가가 어진 만 개 공신

었 니 도 러워 마 시고 rdquo 시

택 여 니 운 신 과 신

습 늘에 죄 짓고 간 상에 내 신

다 략 지낸 후에 튿 승상 ( ) hellip hellip

니 승상 거운 마 지 못

마( )

듯 월 러 생 열다 살 었

다 에 승상 어진 사 얻고 만 에 근심 없었

다만 주 가 간신 에

죽 것 생각 마 곧 어 곤

다 그 에 주 원통 어

없 고 여 시 가 거늘 생 만

여 다

말 감격 러우 간신 에 가득 여ldquo

고 니 께 상 듣지 니 것

니다rdquo

승상 듣지 고 가

퇴 재상 공달 집에 거 고 상 지어

승지 러 께 리

( )

뒷 거리 강 승상 에게 상 리지[ ]

만 여움 사 귀 가게 다 강 승상

몸 는 연 가 헤어

리 다 산 들어간 룡사 승 만

게 다 승 만 우 다릴

과 들고 략 다 담

원 여 에게 복 고 어

공격 다 담에게 여러 가( ) 天子

복 등 여 다 단

신 리쳐 담 사 고 에게

간 후 태후 태 여 지에 고생

지 심과 강 주 여 개 다 헤

어 어 니 내 고 담 리

벼슬에 귀 누리게 다

zb21) 위 의 인 간 계를 같이 나타냈을lt gt

때 에 대한 이해 가장 적절하 ~ 않은 것은

① 계에 주 는 계 심 열

상 에 다고 다( ) 水深火熱

② 계는 견원지간 고 다( ) 犬猿之間

③ 계는 달리 막역지 계 고( )莫逆之交

④ 연결 사 컬어 재 가 고( )才子佳人

⑤ 는 생 과 볼 ( )匹夫匹婦

가 재 는 는 심 고 매사에 생( )

각 고 능 도 어 가 에게 많lsquo rsquo

도움 사 다 그는 에게 거 에

꺼리 없 거 났다고 는

매우 싫어 고 신 들

는 사 다

내가 지 리에( ) 1970

사 실에 지 월간ldquo

편집 고 어 었다rdquo

어느 없 가 쑥 다 도 어 10

후 다 산 시 럼 어 엇 어 ( ) lsquo怡山

다시 만 랴 니 그는 재 그룹 승 운rsquo

사가 고 는 고 거 누

주는 가 없는 가가 어 다시 만 게 것

었다

다 보통 것 닐러 그( ) ldquo 어낸 ( )

틀어주 가 루 러 허 에

싶어 키 틀어주 그( )

가 루 허 우간 곡 틀어 주는 루 못

는 는 고 닝께 고 지

들어 사는 고 가 다는 건 에 그 집에

rdquo

그런 단 어들 어 새벽에 떼죽 거

다 고 어 보니 죄다 허 게 집어진

는 것 었다 가 실내 꿴 뛰어 지만

없는 었다

어떻게 된 거야 한동안 넋나간 듯이 서 있던 총ldquo rdquo

가 하고많은 사람 에 하필이 유자를 겨냥하 은

말이었다 쎄유 아마 새에 고뿔이 들었던 개비네 ldquo

유rdquo

유자는 러 딴청을 하 다 야 고 가 에서 ldquo

감 가 들어 죽는 고 두 어rdquo 총 는 가 혐의

자 나 되는 것처럼 화풀이를 하 드는 것이었다( )嫌疑者

라 이 어쩌 어 유( ) ldquo rdquo ldquo rdquo

애유 이런 잔인 도 한 것들 같으니ldquo ( ) rdquo殘忍無道 helliphellip

총 는 탱천 하여 쩌 를 하 다( ) 憤氣撐天

아하니 아는 자는 다 동 하여 호통을 쳤으 하나 혈

압을 생각하여 참는 눈치 다 달리 처리헐 두 ldquo

잖은감유rdquo

총 의 성 을 덧들이 고 한 말이 아니었다 가 할

년 학 간고사 대비2013 2 현대고 대비

ECN-0102-2013-001-000076193

있는 것이 말고는 없었 때 에 게 뒷동

산을 달은 거 다

이 유자소전- lsquo rsquo

zb22) 의 상황을 속담으 표현한 것으 적절한 것은

① 루 곳 게 마 다

② 에 맞고 강에 눈 다

③ 늘 도 다

④ 도 사 다

⑤ 에 가도 신만 리 다

거리 공신 후[ ]

에 주 는 벼슬 심 늦도( )主簿

식 없어 과 께 산에 드리고 신

태몽 꾼 에 만고 웅 상 지닌 들

낳 키운다 그 후 신 들 에 역심

담 귀 등 심 여 리 귀 보내고

지 죽 는 도망 간다 그

만 고 에 에 어 니

헤어지게 다 지 가 사 들에

사 들 별 고 없 다니었다

마 마 돌 다니 걸 여 고

어 곤 다 에는 동쪽에 고 에

는 쪽에 니 가 에 리는 엽 가는

없 니 늘 다니는 었다

얼 말 죽 사 같고 림새가 말 니었

다 가슴 에 고 등 삼태

헌 에 니 달 가 도리 ( )奇男子

어 걸 었 담 만 열 도 ( ) ( )傅說 慇

고 만났고 만 갈( ) ( )武丁 伊尹

도 탕 만났( ) ( )成湯 渭水

여상 도 주 만났는 월( ) ( ) ( ) 呂尙 周 文王

같 러가 도 어느 열 살

늘과 집 삼고 사 에 쳐 거리에

어 다가 곳에 니 다 ( ) 楚

지 다가 사 보고 가에 다다( )長沙

니 망 가에는 원 리가 슬 고 가

가 내리는 사 에는 갈매 가 갈 뿐 었다

쪽 돌 보니 가 우거 고

가 사 보 었다 그곳에

가니 는 사( ) 汨羅水

는 다 주 가 쓰고 죽고

곳 었다

마 감 여 에 가 사 살펴보니

에는 삼 고 그 에( ) 屈三閭

는 만고 월 과 지 가는 그 들( )風月

가 어 었다( ) 路程記

동쪽 벽 에 새 운 어 거늘 그

보니

월 에 경 주 는 간신에게ldquo ( )敗

보고 연경 귀 가다가 에 죽 rdquo

거늘 그 보고 에 거꾸러

통곡 말

우리 연경 간 만 니ldquo ( )燕京

에 지 살 상에 엇 겠는

가 에 고 에 었 니

상에 살 것 가 도 께 지리 rdquo

고 가에 내 가니 울 리가 에 지

사 쳤는지 심 심 것 가

에는 강 주 는 재상 살고 었

니 시 에 과거에 격 여 승상 벼슬 다가 간

신 만 벼슬 그만 고 고 돌 었

다 그러 신 지 가 지 못 여 상

가 못 결 는 상 여 원 니

신 들 그 직간 꺼 다 그 에 도

담과 귀가 강 승상 가 미워 다 강 승상 마

본 에 갔다가 돌 는 에 우편 주( ) ( )本府 右便

에 다가 색 에 어리었는 청룡( ) 酒店

에 지 늘 여 통곡 고

사 는 꿈 꾸었다 마 상 게 생

각 여 새 다리다가 새벽닭 울고

가 달 갔다 가 보니 과연 어 동 가

가에 울고 는지 달 들어 그

고 사 에 어 말

는 어 어 에 어 가ldquo

닭 곳에 우느냐rdquo

니 울 그 고 답 여 말 다

는 경 동 에 사는 언 주 공 들ldquo

니다 께 간신 만 연경 귀 가

시다가 에 죽 사 에 는 닭에

도 에 죽고 니다rdquo

강 승상 말 듣고 크게 낯 변 말

것 웬 말 냐 근 동ldquo ( )老患

못 갔 니 그 사 변 여 런 변

었단 말 가 주 는 신 다 같

에 벼슬 다가 는 가 많 들어 고 돌

는 주 가 게 꿈 에 생각

겠느냐 생각지 못 다 미 지 간 지지

말고 께 가 rdquo

뒷 거리 강 승상 도움 죽 고[ ]

년 학 간고사 대비2013 2 현대고 대비

ECN-0102-2013-001-000076193

고 그 과 결 여 사 가 다 그러 강

승상 에게 울린 상 강 승상 귀 가고

과 헤어 리 승 만 게 다

승 우 다릴 과

들고 략 다 담 원

여 에게 복 고 어 (天

공격 다 담에게 여러 가 복) 子

등 여 다 단신

리쳐 담 사 고 에게 간

후 태후 태 여 지에 고생 지

심과 강 주 여 개 다 헤어

어 니 내 고 담 리 벼

슬에 귀 누리게 다

미상- lsquo ( )-劉忠烈傳

zb23) 위 과 의 서사 조를 비 한 것으 적절하lt gt

않은 것은

보lt gt

믿지 고 결 여 곱

낳 다 곱째 공주 낳 가

리게 다 리 만 고 진 공주는 lsquo rsquo

리공 미 리공 에 키워진다 월

러 과 가 죽 병에 걸 는 승에 는

어 산다고 다 여 들에게 탁

지만 거 리 는다 리 는 과

승 다 승 지 가는 에 많

만 지만 보살 도움 사 도 다

그러 승 신과 결 여 시

들어 주겠다고 다 리 는 그 결

여 들 곱 낳 후에 신

얻게 다 돌 리 는

에 과 상여 만 지만 여 과

살 낸다 훗 리 그 공 우 죽 사

승 도 는 신 다

리-lsquo rsquo-

① 복 결말에 고 다

② 웅 에 탕 고 다

③ 시 겨 내고 귀 누리는lsquo rsquo

보 리 는 월 재 신 다lt gt lsquo rsquo

④ 과 보 리 는lsquo rsquo lt gt lsquo rsquo

도움과 어 신 능 극복 고

⑤ 등 여 시 겪는lsquo rsquo

보 리 는 닌 지lt gt lsquo rsquo

림 시 겪는다

가 각 고 에( ) ( )却說

살 없었다 략 사 들 슬 에 어 lt gt

가에 내 고 가고 싶 가 고 후

워 경

사 들 별 고 없 다니었다 lt

략 얼 말 죽 사 같고 림새가 말gt

니었다 가슴 에 고 등

삼태 헌 에 니 달 가 ( )奇男子

도리어 걸 었 담 만 열 도( )傅說

고 만났고 만 갈( ) ( ) 殷 武丁

도 탕 만났( ) ( ) (伊尹 成湯 渭

여상 도 주 만났는) ( ) ( ) ( )水 呂尙 周 文王

월 같 러가 도 어느 열 살

늘과 집 삼고 사 에 쳐 거리에

어 다가 곳에 니 다 ( ) 楚

지 다가 사 보고 가에 다다( )長沙

니 망 가에는 원 리가 슬 고 가

가 내리는 사 에는 갈매 가 갈 뿐 었다

쪽 돌 보니 가 우거 고

가 사 보 었다 그곳에

가니 는 사( ) 汨羅水

는 다 주 가 쓰고 죽고

곳 었다

에는 강 주 는 재상 살고( )

었 니 시 에 과거에 격 여 승상 벼슬 다

가 간신 만 벼슬 그만 고 고 돌

었다 략 강 승상 마 본 에 갔다가 돌 lt gt ( )本府

는 에 우편 주 에 다가 색( ) ( ) 右便 酒店

에 어리었는 청룡 에 지

늘 여 통곡 고 사 는 꿈 꾸

었다 마 상 게 생각 여 새 다리다

가 새벽닭 울고 달 갔다 가

보니 과연 어 동 가 가에 울고 는지

달 들어 그 고 사 에

어 말

는 어 어 에 어 가ldquo

닭 곳에 우느냐rdquo

니 울 그 고 답 여 말 다 lt

략gt

년 학 간고사 대비2013 2 현대고 대비

ECN-0102-2013-001-000076193

생각 여 가 고 시 는ldquo ( )大人

상에 다시없는 니다 살 엇 겠습니

에 돌 가시고

가에 돌 가 니 살 마 없습니

다 략 어 없어 강 승상 가니rdquo lt gt

그곳 월계 었다

다 강 승상에게는 들 없고 다만 만( )

었다 가 낳 에 가 색

타고 내 에게 말

는 니다 미원 과ldquo ( )紫微垣

연 맺고 었는 께 강 집( )緣分

보내 에 니 게 여겨 주십시

rdquo

거늘 미 가운 낳 니 가

고 거동 단 다 시 짓 쓰 고

는 없었 니 여 가운( ) 音律

지 는 짝 룰 만 사 없었다 가 사

여 사 감 게 고 지 못 고 염 는 만다

다가 당에 거 고 식같 러

내니 고귀 상 루 말 다 어 ( )相

울 도 다 귀 사 없고 ( )富貴爵祿

웅 걸 만고 었다 승상 매우 뻐 내

당 들어가 에게 사 니 역( ) 內堂

시 매우 거워 말 다

도 마 사 는 승상께ldquo

그 게 말 시니 상 여러 말 지 말고 사

도 시다 략 시 택 여rdquo lt gt

니 다운 신 과 신 습 늘에 죄

짓고 간 상에 내 신 다

다 내고 들어가 사 살펴보니

고 것 는 다 말 어 고

는 다 어 신 에 ( )新房

에 신 과 신 가 평생 연 맺었( )緣分

니 사 주고 말 어떻게 다 헤 릴

어떻게 다 리 지낸 후에 튿 승

상 니 승상 거운 마 지 못

( ) 듯 월 러 생 열다 살

었다 에 승상 어진 사 얻고 만 에 근심

없었 다만 주 가 간신

에 죽 것 생각 마 곧 어

곤 다 그 에 주 원통

어 없 고 여 시 가 거늘 략 lt gt

략 거리

강 승상 에게 상 리지만 여움

사 귀 가게 다 강 승상 몸 는

연 과 헤어 리 다

마 각 생 강 승상 집 쪽( )

늘 보고 없 가 신 신 생각 니

없고 어 없었다 는 어떻게 도리가 없다

여 산 에 들어가 리 고 어 도 닦

고 다 그 산 보고 가

다가 곳에 다다 니 에 큰 산 었다 많

우리 골짜 가 늘 는 가운 색

에 고 갖가지 가 짝 어 ( )花草

었다 략 주 보니 lt gt ( ) (一柱門 黃

산 룡사 어 었다) lsquo rsquo 金大字

산 들어가 고승 다 그( ) ( ) 山門 高僧

거동 보니 눈 눈 듯 고

변 같 귀는 어 에 늘어 니( ) 白邊

맑고 어 골격과 신 평 니었

다 염주 에 걸고 짚고 포 ( )六環杖

삼에 어진 쓰고 생 보고 말

승 연 여 상공 시는 동 에ldquo

가 맞 지 못 니 승 십시 rdquo

생 크게 말 다

생 가 여 어 고ldquo

없 다니다가 우연 곳에 사 만 것

그 시 생 어떻게 고 습니

rdquo

승 답 여 말

어 산 승 에ldquo ( ) ( )南岳 衡山

시어 승에게 탁 내 낮 시경에 경 lsquo 12

동 에 사는 심 들 가 것 니 내쫓

지 말고 습니다 마 승rsquo

다가 상공 림새 보니 경 사 에 보

습니다rdquo

생 그 말 듣고 편 고 편( )

슬 승 들어가니 여러 승 들

가워 다 승 에 들어가

후에 그 편 니 곳 경 었다 상( ) 仙境

고 신 편 다 후 는 승과

께 병 도 탐 고 경도 게( )兵書

게 었다 게 니 지 에 가객 ( ) ( )大明天地 佳客

년 학 간고사 대비2013 2 현대고 대비

ECN-0102-2013-001-000076193

없고 산 에 리 만 본 ( ) 廣德山

신 상 사 살 는 만

우고 늘 월 신 과 늘 ( )日月聖神

산 신 들 다 니 그 재( ) 名山神靈

주 민 누가 당 겠는가 낮 공

zb24) 다 에 해당하는 내 으 적절하( ) 않은 것은

① 강 티 통 당시 능 다

② 상계 지상계 경 는 원 계 드러

③ 실에 어 없는 실 가 타 는

④ 뛰어 재주 어 가진 고

등 다

⑤ 가 직 개 여 평가 내리는

편집 평 타 다lsquo rsquo

가 본격 가 동 것 지( )

다 단 상 에2003 lsquo rsquo

들어가 드럼 연주 다 취미 생 달리

들었다는 보 우 가 들ldquo

어 틱 린 도 다 고 말 다rdquo

경 는 가 망 없( ) lsquo

티 원 고 답 다 신과 같 시각rsquo

는 습 상상 만 도 감동

다 시각 연주 동시에

열 상 는

티 원 그런 열 경 럽다는 것 다

다 역시 엄청 다 본( )

에 복 들

고쳐 가고 다 신 에 얼

마 지는 고 리가 는 지도 생님

가 훈 고 많 고쳐 다

고 말 다

그러 직도 에 지 는 다 그는

체격 지 못 게 가 큰 만

체 운동 훈 과 께 체 늘 동 50

는 게 고 말 다

에게는 꿈 다 통 누 가( )

주겠다는 것 그 꿈 다 신 극복 는

과 에 큰 경험 들도 느 게

주고 싶다는 것 다

마 슬 마다( ) ldquo 통

낼 었 것 럼 고통 는 사 들

고 겠다 고rdquo

말 다 달 루 첫 낸 lsquo rsquo

첫 드 심 집에 는 리듬 드 2

루 에 도 보고 싶다 집 에는 직(RampB) 3 4

사 곡 도 보 고 싶다고 포 다middot

zb25) 에서 가장 유사한 의 를 닌 어를lt gt

찾아 쓰

lt gt

나는 이제 너에게도 픔을 주겠다

사랑 다 소 한 픔을 주겠다

겨 거리에서 개 놓고

살아 추위 떨고 있는 할 니에게

값을 으 서 뻐하던 너를 위하여

나는 픔의 평등한 얼 을 여 주겠다

내가 어둠 속에서 너를 를 때

단 한 도 평등하게 어주 않은

가마니에 덮인 동사자가

다 얼어 죽을 때

가마니 한 장조차 덮어주 않은

한 너의 사랑을 위해

흘릴 르는 너의 눈 을 위해

나는 너에게 이제 너에게도 다림을 주겠다

지 울 포동 여고 생들17

틈 없 가득 체 에 맑 울

다 죽 듣 생들 사 에

연 는 탄 다 객들 도 는lsquo rsquo

가 보 주 공 맹 가 운 는

단 그룹사운드 루 보컬 맡고 는lsquo rsquo

시각 지 었다17 1

근 다만과 가 거lsquo rsquo lsquo

꿈 고 퇴 내가 다rsquo

간 간에 지 지 연 생들 짧lsquo rsquo lsquo rsquo

가 운 듯 리에 어

연 다 내 사 고 퇴lsquo rsquo

과 루 들 결 다시 돌lsquo rsquo

들 고 사 들 에 당당

것 니다 내 태어

볼 없었 크고 열여

년 학 간고사 대비2013 2 현대고 대비

ECN-0102-2013-001-000076193

에도 고 시 얻지 못 다

감지 없는 시각 상태 다

신 지에 고 상 원망 도

단다 어느 가 에 시각 에 ldquo

어 그런 듣고 다 보니 내가 게 lsquo

살 는지 도 눈 고 싶rsquo lsquohelliphellip

보 는 생각만 들 고 그 가 들에게rsquo

도 내고 들도 고 많 었죠 들 rdquo

었 지 새 는 에 쑥 러운 색

어났다

생에 것 단연 었다lsquo rsquo

공연에 거 꿈lsquo rsquo

는 다 특 가사 갑게 는 운 lsquo

벽 에 당당 마주 어 언 가 그 벽

고 늘 어 거운 상도

없죠 내 삶 에 웃 그 께

는 다고 다rsquo

들었 그냥 런 도 고만 여ldquo lsquo rsquo

겼죠 그런 꾸 가사 미 새 다 보

니 통 는 가사 는 생각 들 고 (

가 게는 시각 는 생각 들고 들) ( )

마다 듣고 큰 얻었어 rdquo

에 진지 게 가에 미 가

zb26) 의 에 들어갈 말 적절한 것은lt gt ~

lt gt

난 난 꿈이 있었죠

고 찢겨 남 하여도

내 가 히 과 같이 간 했던 꿈

혹 때 누 가가 뜻 를 비 음

내 등 뒤에 흘릴 때도

난 참아야 했죠 참을 있었죠

날을 위해

늘 걱정하듯 말하죠

헛된 꿈은 독이라고

세상은 끝이 정해 책처럼

이 돌이킬 없는

현 이라고 helliphellip

래 난 난 꿈이 있어

꿈을 믿어

나를 켜

저 차갑게 서 있는 이란 앞에

당당히 마주칠 있어

출처 가 거위의 꿈 작사 이적 작곡 동률- lsquo rsquo ( )

① ② ③ ④ ⑤

가 떴다는 들 만 지만( ) lsquo rsquo

늘 겸 다 에 주 연 우승 지 간에도 3

단 생님께 만 지 고 고 만ldquo rdquo

큼 늘 겸 신 계 가

고 다

에게는 꿈 다 통 누 가

주겠다는 것 그 꿈 다 신 극복 는 과

에 큰 경험 들도 느 게 주

고 싶다는 것 다

슬 마다 통 낼ldquo

었 것 럼 고통 는 사 들

고 겠다 고rdquo

말 다 달 루 첫 낸lsquo rsquo

첫 드 심 집에 는 리듬 2

루 에 도 보고 싶다(RampB) 집 에는 직34

사 곡 도 보 고 싶다고 포 다

미 는( ) (26) 어 헤헤헤 웃다가 어ldquo rdquo

허허허 웃었다ldquo rdquo ldquo rdquo 같 도 고

상 다 는 같 도 다( ) 壯丁 킹 들lsquo

다 는 역도 보 그 다 지만 그는rsquo

뷰에 지 다 운동만 지 ldquo

것 지 간에 여러 사 도 역rdquo helliphellip

었다 그런 엇 그 마 움직 는지 보 쯤

지 담 사 다 훈 없어 그는 티

지 림 었다 태 다 갔다 는 습

마 집 럼 편 게 보 다

주말에는 주 엇 보내

주말에도 별 주 에 청ldquo

고 에 가고 도 쳐

에 듣고 보 에 갈 가 별 없

어 산 시 게 고 들어 2002

거 매 여 지냅니다 시 과 지훈 rdquo

다 근 간 과 진실 그리고 싶어( )

가 다 근에게 그것 진리 다 거 다 없

거 고 다 없 는 것 진리

다 근 진리는 후 쪽 었다 신산( )辛酸 삶

었 질곡( )桎梏 역사 에 지냈 가

눈에 든 것 료 단 료 게 보

것 었다 그것 그 에 겨우겨우

슬 슬 생 어가는 간들 었다

리 과 단 리 고리에 검 마

없 거리 돌

상 것 없는 등 근에게 상

과 진실 엄 ( )儼存 다는 사실 리는 가

실 고 가 과 역경 에 도 근 내 포

없었 후 보루( )堡壘 다 도 365

도 간 근 여

시 것 다

년 학 간고사 대비2013 2 현대고 대비

ECN-0102-2013-001-000076193

다 공주 그림 가 근 경- ( ) ldquo rdquo(

2009)

zb27) 작가의 주 적인 각이 드러난 것은~

① ② ③ ④ ⑤

가 신 지에 고 상 원망( )

도 단다 어느 가 에 시각 에 ldquo

어 그런 듣고 다 보니 내가 lsquo

게 살 는지 도 눈 고 싶rsquo lsquohelliphellip

보 는 생각만 들 고 그 가 들에게rsquo

도 내고 들도 고 많 었죠 들었rdquo

지 새 는 에 쑥쓰러운 색

어났다 략 [ ]

경 는 가 망 없 티lsquo

원 고 답 다 신과 같 시각rsquo

는 습 상상 만 도 감동

다 시각 연주 동시에

열 상 는 티

원 그런 열 경 럽다는 것 다 략 [ ]

슬 마다 통 낼ldquo

었 것 럼 고통 는 사 들

고 겠다 고rdquo

말 다 달 루 첫 낸 lsquo rsquo

첫 드 심 집에 는 리듬 2

루 에 도 보고 싶다 집 에는 직(RampB) 3 4

사 곡 도 보 고 싶다고 포 다

식 누 가-

고 싶어

다 역도 미 담 고 사( )

질 주말에는 주 엇 보내[ 1]

답 주말에도 별 주 에[ ] ldquo

청 고 에 가고 도 쳐

에 듣고 보 에 갈 가 별

없어 rdquo

질 계 고 슬슬 도 는 것 닙니[ 2]

답 다 들 눈 에 보 고 뿐 보[ ] ldquo

다 열심 고 어 상에 도 들지만 상

지키는 것 들다고 에 도달

그것 지키 훨 많 rdquo

질 들 살 고 리 는[ 3]

거운 들 체 리느 는다

답 가 고 게 체 어[ ] ldquo ( ) 級

느 도 계가 니 살 는 것도 고역 지만

살 우는 것 들어 는 살

체 리 고 어도 어도 실 갔다

쑥 어 rdquo

질 거리에 슷 연 여 들[ 4]

보는 간 상 지

답 상 다 체 게 리지 못[ ] ldquo

거 주변에 는 그 거 누 보지

못 고 뻐지고 싶 에 체 리는 에

타 워 지만 는 어울 는 것보다 는

시간 운동만 는 건 니에 사복 lsquo rsquo

고 사복 는 말에 들 웃지만 늘 운동복

고 지내니 사러 갈 도 어 rdquo

질 역도가 말 단 식 운동 니[ 5]

답 가 내는 만 클 업 보[ ] ldquo

그러니 만 쓰는 식 운동 니다

만 다고 거운 것 들 는 건 니거든 연

도 고 가지 동 에 도 여러 가지

복 들

보식 역도 여 미-

zb28) 가 에 대한 설 으( ) 않은 것은

① 시각 우 지 시 에 지

고 망 가는 태도 달 고 다

② 언어 과 언어 복 사 여

담 내 생각 게 는 가

③ 직 감 그 마 것

럼 생생 게 느껴지는 과 주고 간 내

없 리 어 억 게 다

④ 담 내 식 리 여 담 삶 습

과 가 시 여 독 에게 감동과 훈 다

⑤ 직 진 담 직 누

지 못 는 독 에게 생생 상 달 주고

담 욱 게 다

zb29) 나 의 각 의 의도를 설 한 것으 적절하( ) 않

년 학 간고사 대비2013 2 현대고 대비

ECN-0102-2013-001-000076193

은 것은

① 질 담 상 보여 주 것 다1

② 질 담 과 그에 삶 태도 보여2

주 것 다

③ 질 역도 겪는 어 움에 역도3

과 것 다

④ 질 같 연 여 갖는 고민 는지 말4

주 는 것 다

⑤ 질 역도가 과 고 운동 는 것5

담 가 말 주 는 것 다

가 만진 것 다( ) 3

감 달 다고 다 억 에( ) 音感

지워 지만 당시 청 탁 리도

다고 다 드럼 웠다 4

에 갈 마다 드럼 는 리가 신 게 들

다고 다 눈 볼 가 없 니 엔ldquo

는 는 님 틱 에 여 주

다 드럼과 연 맺 과 들 주었다rdquo

식 누 가-

고 싶어

역( ) 도가 말 단 식 운동 니

가 내는 만 클 업에 보ldquo

그러니 만 쓰는 식 운동 니다 만

다고 거운 것 들 는 건 니거든 연

도 고 가지 동 에 도 여러 가지 복

들 시 는 상 상

드는 상 에 맞춰 실 에 는 여러

펼쳐집니다rdquo

략( )

늘 에 는 어 만 것 같

가 에 사 고 사 사ldquo

겠어 든 에 가 경 만 고

울 는 사 겠어 rdquo

보식 역도 여 미-

다 가 운 는 어 어( ) ldquo rdquohelliphellip

월 새벽 시 태 없 거웠고1965 5 6 1

는 없 그 병원에 퇴원 집

가는 마지막 마 고 마 내 거 다

가 죽 간신 에 실 다 사는 어느5 lsquo

가 죽 는 말 가 식 다 신rsquo

상에 각 시키는( )刻印 에 실

어느 가는 후 민 가가 근 었다lsquo rsquo

는 간 과 진실 그 다는( ) ldquo

에 단 평 견 가지고 다 내

가 그리는 간상 단 고 다 지 다 는 그들

가 에 는 평 지 니 그리고 어린

들 미지 겨 그린다rdquo

마 근 간 과 진실 그리고 싶어( )

가 다 근에게 그것 진리 다 거 다 없

거 고 다 없 는 것 진리

다 근 진리는 후 쪽 었다 신산(辛酸 삶)

었 질곡(桎梏 역사 에 지냈)

가 눈에 든 것 료 단 료 게

보 것 었다 그것 그 에 겨우겨우

슬 슬 생 어가는 간들 었

다 리 과 단 리 고리에 검

마 없 거리 돌

상 것 없는 등 근에게 상에

과 진실 엄 다는 사실 리는 가( )儼存

실 고 가 과 역경 에 도 근 내

포 없었 후 보루(堡壘 다 도)

도 간 근365

여 시 것 다

월 강원도 림리에( ) 1914 2 21

삼 독 태어났다 어 근 복

그것 그리 가지 못 다 근 곱 살

지는 산 산업에 실 고 답마 에 내

갔다 근 그림 럼 쫓 다니 가 시 것

다 상 진 것도 가 었다

러 가 에도 고 근 가 꿈꾸었다 근

가 꿈꾸게 것 보통 업

원색도1926 만lsquo rsquo 었다

공주 그림 가 근 경-

zb30) 에 대한 설 가장 른 것은~

① 역도가 과 운동 도 질

② 리는 는 다 lsquo rsquo

③ 들었지만 그럭 럭 는 다 lsquo rsquo

④ 가 게 보 시 말 다

⑤ 보 병 는 지 상 lsquo rsquo

는 말 다

년 학 간고사 대비2013 2 현대고 대비

ECN-0102-2013-001-000076193

시간 많지 다 청량리 생 병원

마지막 상 경 릿 게 들어 다 그 는 십

만 큰 가 상 말 다

지 못 들 마 갈 고 돗

도시민들 싹 싹 탔다 가 시

월에 병원에 원 가 폐 진 몸도4 ( )疲弊

갈 미 지 못 고 었다 가는 얼마( ) 解渴

지 생 에 생각 가

마감 는 신 평생 십 만에

가 과 많 닮 다고 생각 지는

가 운 는 어 어ldquo rdquo 1965helliphellip

월 새벽 시 태 없 거웠고 는5 6 1

없 그 병원에 퇴원 집 가

는 마지막 마 고 마 내 거 다 가

죽 간신 에 실 다 사는 어느 가5 lsquo

죽 는 말 가 식 다 신rsquo

상에 각 시키는 에 실 어느( ) lsquo刻印

가는 후 민 가가 근 었다rsquo

ldquo 는 간 과 진실 그 다는 에

단 평 견 가지고 다 내가 그

리는 간상 단 고 다 지 다 는 그들 가

에 는 평 지 니 그리고 어린 들

미지 겨 그린다rdquo

근 간 과 진실 그리고 싶어 가

다 근에게 그것 진리 다 거 다 없 거

고 다 없 는 것 진리다

근 진리는 후 쪽 었다 신산 삶 ( )辛酸

었 질곡 역사 에 지냈 가 눈에( )桎梏

든 것 료 단 료 게 보 것

었다 그것 그 에 겨우겨우 슬

슬 생 어가는 간들 었다 리

과 단 리 고리에 검 마

없 거리 돌 상

것 없는 등 근에게 상에 과 진실

엄 다는 사실 리는 가 실( )儼存

고 가 과 역경 에 도 근 내 포 없었

후 보루 다 도 도( ) 365堡壘

간 근 여 시 것

간에 지닌 가 근 1914 2

월 강원도 림리에 삼 독21

태어났다 어 근 복 그것 그리

가지 못 다 근 곱 살 지는 산

사업에 실 고 답마 에 내 갔다 근

그림 럼 쫓 다니 가 시 것 다 상

진 것도 가 었다 러 가 에도

고 근 가 꿈꾸었다 근 가 꿈꾸게

것 보통 업 원색1926

도 만 었다lsquo rsquo

그림 가 근 경 공주- ldquo rdquo ( 2009)

zb31) 다음 이 같은 의 성 소에 해당하 않은

것은

사건 평① ② ③

④ 주 ⑤ 경

가 운 는 어 어ldquo rdquo 1965helliphellip

월 새벽 시 태 없 거웠고 는5 6 1

없 그 병원에 퇴원 집 가

는 마지막 마 고 마 내 거 다 가

죽 간신 에 실 다 사는 어느 가5 lsquo

죽 는 말 가 식 다 신rsquo

상에 각 시키는 에 실 어느( ) lsquo刻印

가는 후 민 가가 근 었다rsquo

는 간 과 진실 그 다는 에ldquo

단 평 견 가지고 다 내가 그

리는 간상 단 고 다 지 다 는 그들 가

에 는 평 지 니 그리고 어린 들

미지 겨 그린다rdquo

근 간 과 진실 그리고 싶어 가

다 근에게 그것 진리 다 거 다 없 거

고 다 없 는 것 진리다

근 진리는 후 쪽 었다 신산 삶 ( )辛酸

었 질곡 역사 에 지냈 가 눈에( )桎梏

든 것 료 단 료 게 보 것

었다 그것 그 에 겨우겨우 슬

슬 생 어가는 간들 었다 리

과 단 리 고리에 검 마

없 거리 돌 상

것 없는 등 근에게 상에 과 진실

엄 다는 사실 리는 가 실( )儼存

고 가 과 역경 에 도 근 내 포 없었

후 보루 다 도 도( ) 365堡壘

간 근 여 시 것

간에 지닌 가 근 1914 2

월 강원도 림리에 삼 독21

태어났다 어 근 복 그것 그리

가지 못 다 근 곱 살 지는 산

사업에 실 고 답마 에 내 갔다 근

그림 럼 쫓 다니 가 시 것 다 상

진 것도 가 었다 러 가 에도

고 근 가 꿈꾸었다 근 가 꿈꾸게

것 보통 업 원색1926

도 만 었다lsquo rsquo

공주 그림 가 근 경- ldquo rdquo ( 2009)

년 학 간고사 대비2013 2 현대고 대비

ECN-0102-2013-001-000076193

zb32) 위 을 작성하는 과정에서 되어 활 된 자

어 것은

신 사 료① 연보②

고③ ④ 들과 담

⑤ 에 평

는 간 과 진실 그 다는 에ldquo

단 평 견 가지고 다 내가 그

리는 간상 단 고 다 지 다 는 그들 가

에 는 평 지 니 그리고 어린 들

미지 겨 그린다rdquo

근 간 과 진실 그리고 싶어 가

다 근에게 그것 진리 다 거 다 없 거

고 다 없 는 것 진리다

근 진리는 후 쪽 었다 신산 삶 ( )辛酸

었 질곡 역사 에 지냈 가( )桎梏

눈에 든 것 료 단 료 게 보

것 었다 그것 그 에 겨우겨우

슬 슬 생 어가는 간들 었다

리 과 단 리 고리에 검 마

없 거리 돌 상

것 없는 등 근에게 상에 과

진실 엄 다는 사실 리는 가 실( )儼存

고 가 과 역경 에 도 근 내 포

없었 후 보루 다 도 도( ) 365堡壘

간 근 여 시

것 다

간에 지닌 가 근 1914 2

월 강원도 림리에 삼 독21

태어났다 어 근 복 그것 그리

가지 못 다 근 곱 살 지는 산

사업에 실 고 답마 에 내 갔다 근

그림 럼 쫓 다니 가 시 것 다 상

진 것도 가 었다 러 가 에도

고 근 가 꿈꾸었다 근 가 꿈꾸게

것 보통 업 원색1926

도 만 었다lsquo rsquo

질 루 마 가 도 린다 경건

움 느껴지는 경 다 훗 근 그림에

과 는 거 것( )裸木

만 간과 연 엮어 가는 경건 움lsquo rsquo

니었

같 가가 고 싶었 근에게 그 꿈에 다

가가는 지 다 다 가 지망생들 규 미

상 에 진 고

에 지만 근 다 다 근

미 에 운 것 보통 시 미 시간

다 그런 그에게 없는 연습 가가

통 다 가 귀 시 지 도

얻는 뛸 듯 뻤지만 마 도 가 에

듯 는 었 에 어린 근 주 에

에 그림 그리고 지우고 복( )粉板

시간 가는 게 루 보냈다

근 그 갈 가가 것 열여( )渴求

었 다가 미1932 lsquo rsquo ( lsquo

미 에 다 다는 고 마rsquo) lsquo rsquo

가 근 집 고도 지는 시골 경

그린 그림 다 후 근 에 1943 22

지 미 에 그림 고

에 걸쳐 다 미 근 가

동 는 었다

공주 그림 가 근 경- ldquo rdquo ( 2009)

zb33) 위 의 내 과 일치하는 것은

가 근 가 꿈 포 다①

근 당 가들과 께 에 다②

살 근 가 걷20③

게 었다

④ 만 통 근 역경 겨내는lsquo rsquo

느 다

⑤ 근 간 과 진실 그리 에 그 에

드러 는 간상 단 다

계 시 주 근 건강

걸었다 신 과 간에 상 다 건강

신 는 눈에도 다 근 쪽 눈 뿌 게

보 지 과에 다 다 시 지지 고 결

내 었다 시 지만 마 막막

다 늦어 결 근 쪽 눈 고 말 다

쪽 눈 근에게는 쪽 눈 었고

계 었다 그 근 는 여 그lsquo rsquo

다 근 에 같 그림 그 었다1950

시 그림 는 여 쪽lsquo rsquo

고 어 마주 고 는 그림1963

여 과 동 다 마 복

그린 듯 눈 내리 새 게 다 지

사 다 근 게 복 것

복 상과 타 는 근 상

가 떳떳 단 었고 근 그리고

간 과 진실 에 다가가 가 근다

운 었다 근 신에게 당당 지 그리고

그 다 근 그림에 단 복 보다

년 학 간고사 대비2013 2 현대고 대비

ECN-0102-2013-001-000076193

태 도 그리고 극 보다 과

얻 여 었다 과 통

근 그리고 는 재 고 에 질

만들고 특 것 다

공주 그림 가 근 경- ldquo rdquo( 2009)

zb34) 의 이유에 대해 추 한 것으 적절하 않은 것

상과 타 시도①

보다 과 얻②

근 신에게 당당 지③

④ 간 과 진실 에 다가

⑤ 태 도 얻

근 가가 었지만 그 다니 가

럼 어지지 다 복과 쟁 거쳐 시

는 가 근에게 생계 사 에

운 사 다 에 키에 건( ) 178cm死鬪

체 근 에 동 역 업( )荷役

가 생계 다 쟁

에는 동에 운 상우 주 미

죄 사 에 그림 그리는 시 다 그곳에

에 동 역 업 것에

결 것 럼 보 다 지만 그런 것만도

니었다 그림 그리는 고는 지만 매 근

는 극 간 과 별 없는 경 리 그림

벽에 그리는 것 었다 우도 리 없었다 근

트 는 우 그림 그 다 생

계 그림 단 것 다

후 근 지 신 계 리에 미

엑 리 겼다 근 곳에

건 사 크 에 미 들 ( )

상 상 그 다 근 갖 다 겪

냈다 그리고 결 그 돈

신동에 어 사리 집 마 다 마 ㄷ

루 심 쪽에는 과 엌 쪽에는 건

었다 건 주고 근 가 에

여 살 다 심 에는 지 집어

쓰고 지만 곳 근 가 에게 러웠

보 리 다 근 과 마루 업실 삼 그림

그 다 신동 마루는 근 그림에 등 는 lsquo rsquo

같 상들 지 다 시 고

에 들 폐허가

가 업실 었다

공주 그림 가 근 경- ldquo rdquo( 2009)

zb35) 위 에 대한 설 으 적절한 것은

업 시 여 훈과 감동 다①

에 주 평 드러 다②

사 사 등 식 과 ③

④ 다 근거 시 여 삶에

⑤ 살 시 사 경 께 여

습 시 다

가 시간 많지 다 청량리 생 병원( )

마지막 상 경 릿 게 들어 다 그 는

십 만 큰 가 상 말 다

지 못 들 마 갈 고 돗

도시민들 싹 싹 탔다 가 시

월에 병원에 원4 가 폐( )疲弊

진 몸도 갈 미 지 못 고 었다( )解渴 가는

얼마 지 생 에 생각

가 마감 는 신 평생 십 만에

가 과 많 닮 다고 생각 지는

가 운 는 어 어( ) ldquo rdquohelliphellip

월 새벽 시1965 5 6 1 태 없 거웠고

는 없 그 병원에 퇴원 집

가는 마지막 마 고 마 내 거 다

가 죽 간신 에 실 다 사는 어느5 lsquo

가 죽 는 말 가 식 다 신rsquo

상에 각 시키는 에 실( )刻印

어느 가는 후 민 가가 근 었다lsquo rsquo

다 는 간 과 진실 그 다는( ) ldquo

에 단 평 견 가지고 다 내

가 그리는 간상 단 고 다 지 다 는 가

에 는 평 지 니 그리고 어린 들

미지 겨 그린다rdquo

근 간 과 진실 그리고 싶어( )

가 다 근에게 그것 진리 다 거 다 없

년 학 간고사 대비2013 2 현대고 대비

ECN-0102-2013-001-000076193

거 고 다 없 는 것 진리

다 근 진리는 후 쪽 었다 신산( )辛酸 삶

었 질곡 역사 에 지냈( )桎梏

가 눈에 든 것 료 단 료 게 보

것 었다 그것 그 에 겨우겨우

슬 슬 생 어가는 간들 었다

리 과 단 리 고리에 검

마 없 거리 돌

상 것 없는 등 근에게 상에

과 진실 엄 다는 사실 리는 가 실( )儼存

고 가 과 역경 에 도 근 내 포

없었 후 보루 다( ) 堡壘 도 365

도 간 근 여

시 것 다

마 같 가가 고 싶었 근에게 그 꿈( )

에 다가가는 지 다 다 가 지망생들

규 미 상 에 진 고

에 지만 근 다 다 근

미 에 운 것 보통 시 미 시간

다 그런 그에게 없는 연습 가가

통 다 가 귀 시 지 도

얻는 뛸 듯 뻤지만 마 도 (

는 었 에 어린 근 주 에)

에 그림 그리고 지우고( )粉板

복 시간 가는 게 루 보냈다

zb36) 전 의 성 소가 아닌 것을 고르

① 평 ② 사건 ③ 경

④ ⑤ 훈

늘 지 상에 살고 는 사 들 억 도가10

고 그리 지 통 고 는 사 들( )知的

그보다 훨 많 억 도는 고 지 20

통 다 그런 지 고 2500

그리 간 보는 과 사 에

매우 달 뿐만 니 과 에 도 극

루고 었다 미 운 그런 들

살고 는 동 과 사 들 사고 식에

큰 가 다는 다

고 그리 들 우주 개별 고 독립

사 들 생각 지만 고 들 우

주 연 질 간주 다 같( ) 看做

각 도 들에게는 연 질

었지만 그리 들에게는 미 들 결 었

다 고 과 그리 들 사 같

는 동 과 사 에 도 견 다

지심리 미 마 드 겐트 는

살 들에 에 지 다

연 동 과 상 다 과 같 실험

다 크 만든 미드 도 보

여 주고 그 상 닥 고 주었다lsquo (Dax)rsquo

실 닥 는 재 지 는 것 실험 가lsquo rsquo

만들어 낸 다 그런 다 개 다 체 보

여 주었는 는 미드 지만 틱

만들었고 다 는 재료는 크 지만

달 다 그러고 어 것 닥 지 사 들에게 고 lsquo rsquo

게 니 들 주 같 고 는

체 택 고 동 들 같 재료 만들어진 체

택 다 러 는 심지어 살짜리

들에게 도 타났다 것 곧 과 동

다 상 보고 다는 것 미 다

개별 사 보고 고 동 연 질 보

고 는 것 다

동 들 주변 상 에 맞 어 동 고

에 다 사 들 태도 동에 보다 많

주 울 다 동 가 미시간 에

에 경험 다 그는 미식

경 보러 가게 었는 경 체는 매우 재미 었

주변 들 동에 질 다 그 는

들 계 어 상태 경 다

어 들 에 에 그 시 가 계 가

진 것 다 상 살펴 는 말 들 lsquo rsquo

에 그는 에 시 어 도 뒷사

생각 곧 다시 곤 것 다 그런 그에게 뒷

사 고 지 는 들 동 럼

어 웠다

생각 지도 리 드 니 벳-

zb37) 다음 위 의 내 전개 으 만 인lt gt

것은

lt gt

대조의 통해 대상이 닌 특성을 설 하고 있다

일화를 제 하여 자 의 주장을 뒷 침하고 있다

유추의 을 사 하여 독자의 의해를 돕고 있다

대상이 형성되는 과정을 간적 서에 따라 서 하고 있

① ②

③ ④

년 학 간고사 대비2013 2 현대고 대비

ECN-0102-2013-001-000076193

가 우리가 말 고 쓰는 든 단어가 사 에 는( )

것 니다 사 격에 가 는 지만

어 사 과 같 특별 는 사 니lsquo rsquo

단어 격 보 단어가 사 에

등재 어 다 리 리 사 는 단어 도 그

것 시 사 는 어 고 사 에

격 보 것 니다

러 얼 은 사전에 를 있는가 이에 대한 답lsquo rsquo

은 얼 이 유행어인가 아닌가에 따라 갈라 다 이 단어lsquo rsquo

는 년 어 자 에 랐고 쓰이고 있으2002 lsquo rsquo

유행어라고 하 에는 생 이 다 런데 계속

을 유 하 서 사전에 등재될 자격을 획득할 것인가 이

에 대한 답을 내리 는 히 어 다

여 서 가 를 고 해 볼 있다 첫 는 이 단어

를 써야 할 필 가 속적으 있는가 하는 점이다

상주의 열풍에 휩 인 사회 위 에 편 해서 퍼 말

이 얼 인데 과연 런 위 가 속될 것인가 이에lsquo rsquo

대해 필자의 생각은 정적이다 사회 위 가 뀌

런 말을 쓸 일이 없어 것이다

다음은 단어의 성이다 단어의 성이 사회적으 거

감이 없으 계속 사 될 가능성이 높다 런 에서

얼 은 좋은 조건이 아니다 익히 알 졌듯이 이lsquo rsquo

말은 얼 과 청소년층에서 속어 사 하는 이 결합lsquo rsquo lsquo rsquo

된 말이다 얼 에서 얼 을 리하는 조어 도 lsquo rsquo lsquo -rsquo

어에서는 매 낯선 이다 이것만으 도 거 감을 갖

는 사람들이 있다 더 나 속어 결합한 말이다 얼 lsquo rsquo

이 널리 퍼졌다 해도 은 여전히 청소년층의 속어lsquo rsquo

남아 있다 속어는 자연 럽게 아 자리에서나 쓰 에는

담 러 말이다 러한 담을 하고 사

역을 넓혀 가는 속어도 없 는 않다 특히 얼 은 lsquo rsquo

에도 종종 등장한다 만큼 거 감이 많이 희석되었다

고 할 있다 러나 일상의 자연 러 대화에서도 거

리낌 없이 등장하는가 게 는 되 않았다고 생

각한다

얼 이 유사어인 쌈 등을 만들어 내고lsquo rsquo lsquo rsquo

있으니 살아남을 있을 것이라고 는 견해도 있을 것

이다 러나 간이 나 서 유사어를 포함하여 든

말이 사라 사 는 많다 유사어가 많다는 것이 생 을

유 할 있는 절대적인 조건은 아니다

나 언젠가 터 사람들은 어느 단에서 얼 이 가장( )

쁜 사람을 가리켜 얼 이라고 르고 있다 이 얼lsquo rsquo lsquo rsquo

이라는 단어가 최근 어사전에 라 항간에 논란이 일고

있다 아닌 게 아니라 얼 은 유행어처럼 인다 생 lsquo rsquo

도 리 래되 않은 것 같고 언제 사라 도 알

없다 게다가 젊은이들 사이에서 주 쓰일 뿐이다 이런

단어를 사전에 는다는 게 하 이 없어 이 도

한다

러나 속단은 이다 차근차근 따져 볼 일이다

선 얼 이 일 적 유행어인 아닌 주의 게 들여다lsquo rsquo

볼 필 가 있다 유행어란 유행에 따라 빠르게 유포되었

다가 단 간 내에 소 되는 단어나 를 가리킨다

얼 은 인터넷을 통해 속히 퍼 말이다 하 만 일lsquo rsquo

적인 유행어처럼 단 간 내에 사라 않았을 뿐 아니라

현재 도 잦은 빈도 사 되고 있고 앞으 도 상당

간 사 될 것으 측된다 한 언 재단의 뉴 검 lsquo rsquo

색 사이트에 따르 얼 은 년 에 처음 나타난lsquo rsquo 2001

이후 꾸 히 사 되고 있다

이 같은 사 빈도는 얼 이 일 적 유행어 는 현lsquo rsquo

저히 다르다는 것을 여 다 장 간의 생존 만으 도

얼 은 이 한 어의 어휘 에 를 자격을 얻었다lsquo rsquo

고 할 있다 더 이 이라는 비 적 정제된 매체에

높은 빈도 쓰이고 있 않은가 사 빈도 측 에서

필통이나 연필과 같은 단어 대등하거나 더 많이 쓰lsquo rsquo lsquo rsquo

다는 것은 결코 가 게 볼 일이 아니다

이제는 사전이 언어 현 을 빠르게 하는 게 덕인

대가 되었다 세계적으 유 한 의 사전들도 경쟁

적으 어를 고 있다

하 만 얼 은 젊은이들이나 쓰는 속어라고 흠을 잡을lsquo rsquo

도 르겠다 얼 이 주 젊은 층에서 많이 쓰 lsquo rsquo

는 속어임에 틀림없다 러나 어사전에 표 적이고 품

위 있는 말만 어야 한다고 생각한다 것은 커다란

해다 당장 아 어사전이나 펼쳐 라 속어는

설과 같은 비어나 죄자들이 쓰는 은어 어

마니 같은 소 의 사람만이 쓰는 말 도 라 있

않은가 사전은 말 치에 일정 빈도 이상 나타나는 말이

라 말이든 다 할 있다

zb38) 가 나 에 대한 다음의 설( ) ( ) 않은 것은

① 가 는 얼짱 사 에 등재 것에( ) ( ) lsquo rsquo

보 고 다

② 사 등재 가는 단어 격에( )

고 고 는 언 들 언어 사 도에 고 다 ( )

③ 가 얼짱 어지만 신 과 같 매( ) ( ) lsquo rsquo

체에 도 사 는 말 는 고 다

④ 가는 얼짱 어 보고 크게 가지 근( ) lsquo rsquo 3

거 들어 뒷 고 다

⑤ 는 얼짱 어 는 다 특 다는( ) lsquo rsquo

근거 에도 크게 가지 근거 가 들어 주 2

뒷 고 다

가 늘 지 상에 살고 는 사 들 억( ) 10

도가 고 그리 지 통 고 는 사 들

그보다 훨 많 억 도는 고 지 20

통 다 그런 지 고 2500

년 학 간고사 대비2013 2 현대고 대비

ECN-0102-2013-001-000076193

그리 간 보는 과 사 에

매우 달 뿐만 니 과 에 도 극

루고 었다 미 운 그런 들

살고 는 동 과 사 들 사고 식에

큰 가 다는 다

고 그리 들 우주 개별 고 독립

사 들 생각 지만 고 들 우

주 연 질 간주 다 같 각

도 들에게는 연 질 었지

만 그리 들에게는 미 들 결 었다

고 과 그리 들 사 같 는

동 과 사 에 도 견 다

인 리학자인 츠 이마이 디드 겐트너는 두

살이 채 안 된 아이들에서 터 성인에 이르 다양한

연 대의 동양인과 서양인을 대상으 다음과 같은 험

을 했다 저 코르크 만든 피라 드 양의 도형을

여 주고 대상의 이름을 닥 라고 알 주었다lsquo (Dax)rsquo

제 닥 는 존재하 않는 것으 험자가 임의lsquo rsquo

만들어 낸 이름이다 런 다음 두 개의 다른 체를

여 주었는데 하나는 피라 드 양이 만 하얀 플라 틱

으 만들었고 다른 하나는 재 는 코르크 만 양이

달랐다 러고 나서 어떤 것이 닥 인 사람들에게 고 lsquo rsquo

르게 했더니 서양인들은 주 같은 양을 하고 있는

체를 선택했고 동양인들은 같은 재 만들어 체를

선택했다 이러한 차이는 성인은 어 두 살 리

아이들에게서도 나타났다 이것은 곧 서양인과 동양인은

서 다른 세상을 고 있다는 것을 의 한다 략 ( )

는 아주 단 하 서도 인상적인 험을 했다

험에는 동서양의 대학생들이 참여했다 는 험 참가자

들에게 컴퓨터 화 을 통해 속 장 을 담은 애니 이션

을 여 주었다 화 의 앙에는 초점의 역할을 하는 커

다란 고 한 마리가 있었고 주위에는 다른 생

들과 초 자갈 거품 등이 함 제 되었다 화 을

두 씩 후 참가자들은 자 이 것을 회상해 라는

를 았다

결과 서양인 대학생들과 동양인 대학생 두 앙

의 초점 역할을 했던 고 를 동일한 정도 언 했으

나 경 소 위 거품 초 다른 생 들 에 ( )

대해서는 동양인 대학생들이 서양인 대학생들 다 60

이상 더 많이 언 했다 뿐만 아니라 동양인 학생들은 서

양인 학생들에 비해 개 적인 고 다 전체적인 계

를 더 언 하는 경향을 다 략 또한 경의 일 ( )

를 화 킨 림을 제 하 을 때 동양인 대학생들은 대

경의 화를 알아챘 만 서양인 대학생들은 경

의 화를 거의 알아차리 했다 략 ( )

따라서 서양인들만을 대상으 연 한 화lsquo

편성 결 은 잘 된 것일 도 있다 각 과정과 인rsquo

과정의 어떤 이 화 편적이고 어떤 이

화에 따라 달라 는 는 앞으 많은 연 를 통하여 논의

되어야 한다

나 어떤 의 에서 리 두는 이 화적이다 리( )

안에는 다른 사람들과 더 친 한 계를 유 하 는 상호

의존성과 다른 사람들 터 독립적인 존재 살아가 는

독립성이 혼재한다 따라서 이 에서 어떤 특성이 더 강

하게 각되는 상황에 놓이느냐에 따라 서 다른 화적

특 을 일 있다 결 리 두는 어떤 경 에는

동양인처럼 행동하고 어떤 경 에는 서양인처럼 행동하는

것이다

zb39) 가 에 대한 다음의 설( ) 않은 것은

① 는 신 주 뒷 닥 실험과lsquo rsquo lsquo

니 실험 근거 시 다rsquo

② 동 들 상 간 공통 보다는 에 식

는 강 다

③ 들 주변 맥 에는 심 경 어 사건

과 사건 사 계에 상 민감 다

④ 는 동 과 틀린 지 고 는 것lsquo rsquo

니 다 고 다 lsquo rsquo

⑤ 가에 우리 사 들 개 시 가 원( )

집 경 말 고 는 것 개 보다는

에 고 는 것에 다

늘 지 상에 살고 는 사 들 억 도가10

고 그리 지 통 고 는 사 들( )知的

그보다 훨 많 억 도는 고 지 20

통 다 그런 지 고 2500

그리 간 보는 과 사 에

매우 달 뿐만 니 과 에 도 극

루고 었다 미 운 그런 들

살고 는 동 과 사 들 사고 식에

큰 가 다는 다

지심리 미 마 드 겐트 는 동

과 상 다 과 같 실험 다

크 만든 미드 도 보여 주고 그

상 닥 고 주었다 그런 다lsquo (Dax)rsquo

개 다 체 보여 주었는 는 미드

지만 틱 만들었고 다 는 재료는

크 지만 달 다 그러고 어 것 닥 lsquo

지 사 들에게 고 게 니 들 주 같rsquo

고 는 체 택 고 동 들 같

재료 만들어진 체 택 다 러 는

심지어 살짜리 들에게 도 타났다 것

곧 과 동 다 상 보고 다는

것 미 다 개별 사 보고 고 동

년 학 간고사 대비2013 2 현대고 대비

ECN-0102-2013-001-000076193

연 질 보고 는 것 다

동 들 주변 상 에 맞 어 동 고

에 다 사 들 태도 동에 보다

많 주 울 다 동 가 미시간

에 에 경험 다 그는 미

식 경 보러 가게 었는 경 체는 매우 재

미 었 주변 들 동에 질 다 그

는 들 계 어 상태 경

다 어 들 에 에 그 시 가 계

가 진 것 다 뒷사 고 지 는 들

동 럼 어 웠다

그는 경험에 어 얻어 동 들lsquo

각도 상 본다 는 가 우고rsquo

검 여 주 단 도 상 실험 실

시 다 그는 실험 가 들에게 컴퓨 통

담 니 보여 주었다

에는 역 는 커다 고 마리가 었

고 주 에는 다 생 들과 갈 거 등

께 시 었다 본 후 가 들

신 본 것 상 보 는 지시 다

그 결과 생들과 동 생

역 고 동 도 언

경 거 다 생 들에 ( )

는 동 생들 생들보다 60

상 많 언 다 뿐만 니 동 생들

생들에 개별 고 보다 체 계

언 는 경 보 다 경 변 시

킨 그림 시 동 생들 경

변 지만 생들 경 변

거 리지 못 다

지 지 들만 상 연 lsquo

보편 결 못 것 도 다 지각 과 과rsquo

지 과 어 보편 고 어

에 달 지는지는 많 연 통 여

어 다

리 드 니 벳 생각 지도 사- ldquo rdquo( 2004)

zb40) 위 에 대한 설 으 가장 적절한 것은

① 동 과 생 식 강 고 다

② 가지 실험 통 쓴 고 다

③ 닥 실험에 사 본질에 동 사

상에 주 다

④ 니 실험에 동 과 에 지

각 도에 가 다

⑤ 쓴 는 보편 연 에 드러 우월 에

에 근 고 다

가 동 들 주변 상 에 맞 어 동 고( )

에 다 사 들 태도 동에 보다 많

주 울 다 동 가 미시간 에

에 경험 다 그는 미식

경 보러 가게 었는 경 체는 매우 재미 었

주변 들 동에 질 다 그 는

들 계 어 상태 경 다

어 들 에 에 그 시 가 계 가

진 것 다 상 살펴lsquo 는 말 들rsquo

에 그는 에 시 어 도 뒷사

생각 곧 다시 곤 것 다 그런 그에게

뒷사 고 지 는 들 동 럼

어 웠다

그는 경험에 어 얻어( ) 동 들lsquo

각도 상 본다 는 가 우고rsquo

검 여 주 단 도 상 실험

실시 다 실험에는 동 생들 여 다

그는 실험 가 들에게 컴퓨 통

담 니 보여 주었다 에는

역 는 커다 고 마리가 었고 주 에는

다 생 들과 갈 거 등 께 시

었다 본 후 가 들 신 본 것

상 보 는 지시 다

다 그 결과 생들과 동 생( )

역 고 동 도 언

경 거 다 생 들 에 ( )

는 동 생들 생들보다 60

상 많 언 다 뿐만 니 동 생들

생들에 개별 고 보다 체 계

언 는 경 보 다 들어 동

생들 상 체 연못 럼 보 어ldquo 같rdquo

체 맥 언 시 었지만

생들 상 어 같 큰 고 가 쪽 움ldquo

직 어 같 역 고rdquo

언 시 다 경 변 시킨 그

림 시 동 생들 경 변

지만 생들 경 변 거

리지 못 다

년 학 간고사 대비2013 2 현대고 대비

ECN-0102-2013-001-000076193

게 볼 동 들 보다는 큰 그( )

림 보 에 사 과 체 맥 연결시 지각

는 경 고 체에 특 떼어 내

어 독립 보는 것 낯 어 다 에

들 사 에 고 주변 맥 에는 심 경

에 사건과 사건 사 계에 상

민감 편 다

마 지 지( ) 들만 상 연

보편 결 못 것 도 다lsquo rsquo 지각 과

과 지 과 어 보편 고 어

에 달 지는지는 많 연 통 여

어 다

리 드 니 벳 생각 지도 사- ldquo rdquo( 2004)

zb41) 의 하는 가~ 다른 것은

① ② ③

④ ⑤

얼마 그 에 동 사고 식과

사고 식 보여 주는 내 다

들 에 는 탕 고 같 게

어 겨 고 미 에 는 그 크 럼 큰 고

어리 주고 원 는 어 도 는

상 고 생각 다는 것 다 러

는 어떻게 생 것 고 과 그리 거슬

러 가 보 그 단 다

고 연 경 체 경 생 에

다 벼 사는 공동 업과 경험 많 연 역

에 고 들 연 웃과

게 지내 고 탁 연 들

들 지 연 럽게 들 다 민들

웃과 동 게 뿐만 니 는 집 과

게 다

동 시 는 생태 경 에 살 결과

들 다 사 들 사 상 에 주

울 게 었고 는 곧 체 상 과 간 사

계 시 는 낳게 었다 신 가

가 는 체에 는 원 는 동시

에 다 사 들 그 사 포 체 맥 에

다 들 간 사 연

계 체 계에 주 울 는 사고 체계

게 었다

그러 그리 연 경 그 었다 산

지 연결 는 지 건 그리고 역

에 다 런 들 업에 다 사 과

동 므 공동체에

다고 다 고 그리 들

들과는 달리 보 내 감 지 들과

지 크게 느 지 못 다 그

견 다 경우 주 쟁 통 결 는 갖

게 었다

신 사 간 계들 루어진 커다

트워크 에 게 당연 사 역시 연

계들 체 식 게 다 어 상

원 도 그 개체가 체 맥 과

계 에 고 다 게 체 맥 에 주

울 다 보 상 복 과 가변 식 게 고

상에 재 는 많 변 들 사 에 재 는 들도

게 다 들 주 태도 보

는 경우가 많다 쟁 결

통 결 보다는 통 결

는 보 다

그러 고 그리 들 개개 사 사 독

에 주 울 다 사 사 체에

어 그들 사 에 재 는 공통 규 주

고 다 상 원 에도 사

체 내 주 고 다 그들

체 여 탕 체

는 주 태도 시 고 특 사 어

주에 는지 여 그 주에 는 규

견 다 에 는 쟁 식 리

같 리 사고 체계가 달 게 었다

리 드 니 벳 생각 지도 사- ldquo rdquo( 2004)

zb42) 위 에서 사 된 설 과 가장 유사한 것은

① 크톱 컴퓨 는 본체 니 마우 루

어 다

② 곡과 시 리 는 지 과 사 루어 다는 공통

지니고 다

③ 경 고 것과는 달리

경 본 연 태 그 주변 경

④ 벽돌 능 에 사계 내내

습도가 지 다

⑤ 잰느 체 체 지닌 재 체가 없

는 재 눌 다

년 학 간고사 대비2013 2 현대고 대비

ECN-0102-2013-001-000076193

zb43) 는 립 앙 도서 이 정의 일 이다lt gt

도서 장과 이 자의 리 의 정의 연결이

적절하 않은 것은

lt gt

제 조 서 유8 ( )

도서 장은 다른 이 자의 안전을 위협하거나 도서 의①

서를 란하게 할 가 있는 자에 대하여는 도서 출입

을 제한할 있다

도서 장은 이 자가 제 조 각 호의 어느 하나의 행위를 하7②

을 때에는 이 을 하게 하거나 도서 출입을 제한할

있다

제 조자 의 대출9 ( )

도서 자 는 다음 각 호의 경 대출할 있다①

상호대차도서 간에 자 를 류하는 것을 말한다 등 다1 ( )

른 도서 과의 협 을 위하여 필 한 경

공 이 공 행 상 필 하는 경2

에 도서 장이 필 하다고 인정하는 경3

대출이 가능한 도서 자 의 위는 도서 장이 정하는②

에 따른다

제 조 상10 ( )

이 자가 도서 자 설을 더럽히거나 찢거나 뜨①

쓰게 하거나 잃어 린 경 에는 상하여야 한다

도서 장은 제 항에 따른 상 을 정하여 게 하여야1②

한다

제 조이 절차 등11 ( )

이 칙에서 정한 것 에 도서 자 설의 이 절차

이 제한 등에 필 한 사항은 도서 장이 정한다

출처 립 앙 도서- (httpwwwnlgokr)

① 는 도 리 다8

② 도 는 리 다9 1

③ 료 지 는 도 리 다9 2

④ 도 료 변상에 리10 1

⑤ 는 에 도 리 다11

3

도 다 각 같다①

공 공 다만 연1

연 간 다

매월 째 째 월2

도 도 리 그 사3

가 다고 는

도 에 미리 게1 3②

시 여 다

4

도 시간 도 여 게시 다

5

도 료 시 는 는 도①

지에 등 후

등 에 사 도②

7

는 다 각 여 는 니 다

도 료 시 상 리1 lsquo rsquo

도 료 시 훼 는2 middot

지 가 닌 곳에 식 거 담3

우는

도 보 등 보 검색열4 middot

그 에 도 질 지 여 도5

여 게시 사 는

8

도 다 거 도①

질 게 우 가 는 에 여는 도

도 가 각 어느7②

에는 지 게 거 도

9

도 료는 다 각 경우 다①

상 도 간에 료 는 것 말1 (

다 등 다 도 과 여 경우)

공 원 공 상 는 경우2

그 에 도 다고 는 경우3

가능 도 료 는 도②

는 에 다

10

년 학 간고사 대비2013 2 현대고 대비

ECN-0102-2013-001-000076193

가 도 료 시 럽 거 거①

못 쓰게 거 어 린 경우에는 변상 여

도 에 변상 여 게시1②

여 다

zb44) 위 에서 도서 장이 게 해야 할 사항에 해당하는

것을 두 쓰

년 학 간고사 대비2013 2 현대고 대비

ECN-0102-2013-001-000076193

립 도 규

1 ( )

규 립 도 립 어린 청 도(

포 다 료 시 열 시 말) (

다 에 사 규 립 도)

편 진 다

2 ( )

규 립 도 도 다 에( lsquo rsquo )

고 는 도 에 도lsquo rsquo 2 2

료 에 여 다 다만 특 료 귀

료 등 료 에 사 립 도

도 다 다( lsquo rsquo )

3 ( )

도 다 각 같다①

공 공 다만 연1

연 간 다

매월 째 째 월2

도 도 리 그 사3

가 다고 는

도 에 미리 게1 3②

시 여 다

시간4 ( )

도 시간 도 여 게시 다

등 등5 ( )

도 료 시 는 는 도①

지에 등 후

등 에 사 도②

사 료6 ( )

도 료 시 에 사 료는 도

7 ( )

는 다 각 여 는 니 다

도 료 시 상 리1 lsquo rsquo

도 료 시 훼 는2 middot

지 가 닌 곳에 식 거 담3

우는

도 보 등 보 검색열4 middot

그 에 도 질 지 여 도5

여 게시 사 는

질 지8 ( )

도 다 거 도①

질 게 우 가 는 에 여는 도

도 가 각 어느7②

에는 지 게 거 도

료9 ( )

도 료는 다 각 경우 다①

상 도 간에 료 는 것 말1 (

다 등 다 도 과 여 경우)

공 원 공 상 는 경우2

그 에 도 다고 는 경우3

가능 도 료 는 도②

는 에 다

변상10 ( )

가 도 료 시 럽 거 거①

못 쓰게 거 어 린 경우에는 변상 여

도 에 변상 여 게시1②

여 다

등 규 에 것 에 도11 ( )

료 시 등에 사

도 다

립 도- (httpwwwnlgokr)

zb45) 도서 장의 리 있는 조항으 적절하 않

은 것은

① ② ③ ④ ⑤

년 학 간고사 대비2013 2 현대고 대비

ECN-0102-2013-001-000076193

1 ( )

사가 공 는lsquo rsquo

과 여 사 원과 리

사 타 사 규

니다

개 보 보7 ( )

사는 보통신망 등 계 는 에lsquo rsquo lsquo rsquo

원 개 보 보 니다 개lsquo rsquo

보 보 사 에 는 사 개lsquo rsquo

보 취 니다 다만 사는 다 lsquo rsquo

사 계 통 공 는 경우 원 lsquo rsquo

등 개 보 당 사에 습니lsquo rsquo

원 리에8 (lsquo rsquo lsquo rsquo lsquo rsquo

)

원 에 리lsquo rsquo lsquo rsquo lsquo rsquo①

원에게 가 도 여 는lsquo rsquo 3

니다

사는 원 가 개 보 우 가lsquo rsquo lsquo rsquo lsquo rsquo②

거 사 경우 는 미 에 어 거 lsquo

사 사 운 우 가 는 경우 당rsquo lsquo rsquo

습니다lsquo rsquo

원 가 도 거lsquo rsquo lsquo rsquo lsquo rsquo 3③

가 사 고 지 경우에는 시 사에lsquo rsquo

통지 고 사 내에 니다lsquo rsquo

경우에 당 원 사에 그 사실3 lsquo rsquo lsquo rsquo④

통지 지 거 통지 도 사 내에 지 lsquo rsquo

생 경우 사는 지지 습니다lsquo rsquo

사10 (lsquo rsquo )

사는 과 지 미lsquo rsquo①

에 는 지 계 고

공 여 다 여 니다lsquo rsquo

사는 원 게lsquo rsquo lsquo rsquo lsquo rsquo②

도 개 보 신 보 포 보 보 시( )

갖 어 개 보 취 공시 고

니다

사는 과 여 원lsquo rsquo lsquo rsquo③

견 만 당 다고 경우에는

리 여 니다 원 견 만 사 lsquo rsquo

에 는 게시 거 우편 등 통 여

원에게 리 과 결과 달 니다lsquo rsquo

원11 (lsquo rsquo )

원 다 여 는 니다lsquo rsquo ①

신청 는 변경 시 허 내 등1

타 보 도2

사가 게시 보 변경3 lsquo rsquo

사가 보 보 컴퓨 그4 lsquo rsquo (

등 등 신 는 게시)

사 타 등 지 재산 에5 lsquo rsquo 3

사 타 상 거 업6 lsquo rsquo 3

는 폭 시지 상 타 공7 middot middot

에 는 보 에 공개 는 게시 는lsquo rsquo

사 동 없 리 사8 lsquo rsquo

타 거 당9

게시15 (lsquo rsquo )

원 내에 게시 는 게시 게재 는lsquo rsquo lsquo rsquo lsquo rsquo

경우 원 사가 게시 복 lsquo rsquo lsquo rsquo lsquo rsquo middot middot

등 태 언 등에 공 는

것 내에 다 원 본 게시 등 lsquo rsquo lsquo rsquo

크 능 등 여 복 는 등 태

는 것 동 것 니다

- (wwwnavercom)

zb46) 위 은 인터넷 포털사이트의 회 가입을 위한 이

약 의 일 이다 이 약 을 만드는 과정에서 생각한

내 으 적절하 않은 것은

개 보 보 가 지에 별 눠①

겠어

원 가 만들게 에②

시 주어 겠어

원들 게재 게시 다 원 크 다③

는 것 지

④ 원 지 는 뿐만 니 사가 지 는

도 께 달 지

리에 가 생 경우 사가⑤

에 다는 도 듯

1 ( )

사가 공 는lsquo rsquo

과 여 사 원과 리

사 타 사 규

년 학 간고사 대비2013 2 현대고 대비

ECN-0102-2013-001-000076193

니다

개 보 보7 ( )

사는 보통신망 등 계 는 에lsquo rsquo lsquo rsquo

원 개 보 보 니다 개lsquo rsquo

보 보 사 에 는 사 개lsquo rsquo

보 취 니다 다만 사는 다 lsquo rsquo

사 계 통 공 는 경우 원 lsquo rsquo

등 개 보 당 사에 습니lsquo rsquo

원 리에8 (lsquo rsquo lsquo rsquo lsquo rsquo

)

원 에 리lsquo rsquo lsquo rsquo lsquo rsquo①

원에게 가 도 여 는lsquo rsquo 3

니다

사는 원 가 개 보 우 가lsquo rsquo lsquo rsquo lsquo rsquo②

거 사 경우 는 미 에 어 거 lsquo

사 사 운 우 가 는 경우 당rsquo lsquo rsquo

습니다lsquo rsquo

원 가 도 거lsquo rsquo lsquo rsquo lsquo rsquo 3③

가 사 고 지 경우에는 시 사에lsquo rsquo

통지 고 사 내에 니다lsquo rsquo

경우에 당 원 사에 그 사실3 lsquo rsquo lsquo rsquo④

통지 지 거 통지 도 사 내에 지 lsquo rsquo

생 경우 사는 지지 습니다lsquo rsquo

원에 통지9 (lsquo rsquo )

사는 특 다 원에게 통지 경우lsquo rsquo lsquo rsquo

공지 게시 통 상 게시 개별 통지에7

갈 습니다

사10 (lsquo rsquo )

사는 과 지 미lsquo rsquo①

에 는 지 계 고

공 여 다 여 니다lsquo rsquo

사는 원 게lsquo rsquo lsquo rsquo lsquo rsquo②

도 개 보 신 보 포 보 보 시( )

갖 어 개 보 취 공시 고

니다

사는 과 여 원lsquo rsquo lsquo rsquo③

견 만 당 다고 경우에는

리 여 니다 원 견 만 사 lsquo rsquo

에 는 게시 거 우편 등 통 여

원에게 리 과 결과 달 니다lsquo rsquo

원11 (lsquo rsquo )

원 다 여 는 니다lsquo rsquo ①

신청 는 변경 시 허 내 등1

타 보 도2

사가 게시 보 변경3 lsquo rsquo

사가 보 보 컴퓨 그4 lsquo rsquo (

등 등 신 는 게시)

사 타 등 지 재산 에5 lsquo rsquo 3

사 타 상 거 업6 lsquo rsquo 3

는 폭 시지 상 타 공7 middot middot

에 는 보 에 공개 는 게시 는lsquo rsquo

사 동 없 리 사8 lsquo rsquo

타 거 당9

원 계 규 내lsquo rsquo lsquo②

여 공지 주 사 사가 통지 는rsquo lsquo rsquo

사 등 여 타 사 업 에 lsquo rsquo

는 여 는 니다

- (wwwnavercom)

zb47) 위 약 의 조항에서 같은 제점을 하lt gt

고 있는 조항은

lt gt

제휴 회사에 회 의 아이디 개인 정 를 전송할 있도

한 조항은 고객에게 당한 조항이다

1 7 8① ② ③

④ 9 ⑤ 10

립 도 규

1 ( )

규 립 도 립 어린 청 도(

포 다 료 시 열 시 말) (

다 에 사 규 립 도)

편 진 다

2 ( )

규 립 도 도 다 에( lsquo rsquo )

고 는 도 에 도lsquo rsquo 2 2

료 에 여 다 다만 특 료 귀

료 등 료 에 사 립 도

도 다 다( lsquo rsquo )

3 ( )

도 다 각 같다①

공 공 다만 연1

연 간 다

년 학 간고사 대비2013 2 현대고 대비

ECN-0102-2013-001-000076193

매월 째 째 월2

도 도 리 그 사3

가 다고 는

도 에 미리 게1 3②

시 여 다

시간4 ( )

도 시간 도 여 게시 다

등 등5 ( )

도 료 시 는 는 도①

지에 등 후

등 에 사 도②

사 료6 ( )

도 료 시 에 사 료는 도

7 ( )

는 다 각 여 는 니 다

도 료 시 상 리1 lsquo rsquo

도 료 시 훼 는2 middot

지 가 닌 곳에 식 거 담3

우는

도 보 등 보 검색열4 middot

그 에 도 질 지 여 도5

여 게시 사 는

질 지8 ( )

도 다 거 도①

질 게 우 가 는 에 여는 도

도 가 각 어느7②

에는 지 게 거 도

료9 ( )

도 료는 다 각 경우 다①

상 도 간에 료 는 것 말1 (

다 등 다 도 과 여 경우)

공 원 공 상 는 경우2

그 에 도 다고 는 경우3

가능 도 료 는 도②

는 에 다

변상10 ( )

가 도 료 시 럽 거 거①

못 쓰게 거 어 린 경우에는 변상 여

도 에 변상 여 게시1②

여 다

등 규 에 것 에 도11 ( )

료 시 등에 사

도 다

립 도- (httpwwwnlgokr)

zb48) 다음 정 리 의 의 으 볼 때 가장

이 적인 것은

도 시간 도 여 게시 다①

등 에 사 도②

가능 도 료 는 도 는③

에 다

④ 도 에 변상 여 게10 1

시 여 다

⑤ 도 가 각 어느7

에는 지 거 도

zb49) 를 참고하여 이 어의 성격을 설 한lt gt

것으 적절하 않은 것은

① 보 에 는 어 시 상 고 어 시lt gt lsquo rsquo

에 보여주고 다

② 진 어 어원에 견 고 다

에는 타 어 들어가는 것 다 lsquo rsquo

③ 에 들어갈 말 각각 고 어 어 신 어~

들 언어는 질 격 강 통 없었다

④ 시 우리 에 가 었지만 지 계

과 달리 들 통 사 달 어 웠

년 학 간고사 대비2013 2 현대고 대비

ECN-0102-2013-001-000076193

⑤ 크 몽골 만주 공통어가 우리 어 같

계열에 다는 에 사 특 짐

가( )

善化公主主隱 공주님

他密只嫁良置古 몰 결 고

薯童房乙 맛

夜矣卯乙抱遣去如 에 몰 고 가다

( )

始汝 會隱日恚見隱扐 만 에 본

恥隱汝衣淸隱笑 맑 웃

고 시 여 공 크다 만 다[ ] ( ) ( ) ( ) ( )始 汝 會扐

내다 에 보다 견( ) ( )恚 見 다( )隱

럽다 맑다 청 웃( ) ( ) ( ) ( )恥 衣 淸 笑

zb50) 위의 나 를 함 고 음에 답하( ) lt gt

보lt gt

( )素那或云金川 白城郡蛇山人也

운 사산

는 고 다 는( )[ ( ) ] (素那 金川 白城

사산 사 다) ( ) 郡 蛇山

삼 사- lsquo rsquo 47

에 제 된 단어 의 표 리를 조건(1) lt gt ( ) lt gt

에 맞게 서 하

건lt gt

lsquo 었고 었다 태rsquo

에 제 된 단어 동일한 표 리에(2) lt gt ( )

의해 적은 것을 나 에서 찾아 조건 에 맞게 서 하( ) lt gt

건lt gt

에 당 는 각각( ) 개 쓸 것2 단

당 는 가 여러 개 어도 개만 쓸 것 각2

개 과 도 쪽에 개만2 2

드시 지 것( )

과 동 원리 것lsquo 고

과 동 원리 것 다rsquo

태 것

가( )

素那(或云金川) 白城郡蛇山人也

소나 또는 천 이라 한다 는 성 사( ) ( ) ( )素那 金川 白城郡〔 〕

산 사람이다 현대어 풀이( ) ( )蛇山

나( )

紫布岩乎希 회

執音乎手母牛放敎遣 자 손 암쇼 노히 고

吾 不喩慙 伊賜等肹 肹 나 안디 리샤

花 折叱肹 可獻乎理音如 고 것거 도림다

다 향찰은 리말을 리 으 적은 표 이었 만 생( )

은 고 대를 넘 하고 끊어 고 말았다 랜 세

동안 갈고 닦아 체계적이었던 향찰 표 이 사라졌

을 인은 크게 두 가 나누어 생각해 볼 있다

하나는 족 사회의 한 선호도에서 찾을 있다 라 때

향찰은 주 족 계 에서 사 했을 것으 인다 한 을

알 하고서는 한자를 활 하여 리말을 리 으 표

하 란 가능하 때 이다 런데 족들은 간이 흐

를 향찰과 같은 리 표 을 익혀 사 하 다는

아 한 을 대 사 하는 쪽을 선호하게 되었다 더 이

고 초에 인재 등 을 위해 과거제도가 행되 서 한 선

호도가 더 높아졌고 결 향찰은 소 되고 말았다

또 다른 가능성은 한 어의 특성에서 찾을 있다

터 한 과 일 세 나라는 한자 화 에 속해 다

당연한 이야 겠 만 표의 자인 한자는 어를 표 하

에 매 적절하다 어의 음절은 성 ( ) ( )聲母 韻母

이 어 고 여 에 성조가 추가되어 최종 소리가 결정된

다 래서 어는 단음절을 하나의 한자 표 하 된

다 에 초성 성 종성의 세 가 소가 하나의 음절

년 학 간고사 대비2013 2 현대고 대비

ECN-0102-2013-001-000076193

을 이 는 한 어는 음절 조가 잡하고 음절의 가 많아

서 한자 차 만으 한 어의 소리를 만족 럽게 표 할

없었다 를 들어 한 어에서는 어 니 같이 음절 lsquo rsquo

이 어 단어가 얼마든 있으나 어는( ) 複數音節

자 하나 나타내 만이다lsquo [m ]rsquo 母 ǔ

한편 일 어의 표 은 핵 적 단어는 한자 적고 토는

가나라는 일 의 자 적는 이다 적인 의 를 나

타내는 은 표의 자인 한자 적고 적 계를 나

타내는 토는 표음 자 적는 셈이니 자세히 살펴

리의 향찰 표 을 쏙 빼닮았음을 알 있다 한 어 같

은 착어이 서도 일 어에만 향찰과 유사한 표 이 살아

남은 것은 일 어의 특 때 이다 일 어는 하나의 자음과

음의 결합으 음절을 이 고 침이 거의 없는 음절 언어

이다 이러한 음절의 특색에다가 토가 달한 착어라는 점

이 향찰과 유사한 표 이 살아남을 있는 비결이었다

하 만 같은 착어라도 다양한 음소 침이 달한 한

어는 향찰 표 하는 데 근 적으 한계가 있었다

zb51) 다 하여 의 행에 대한 탐 한 결과( ) lt gt 2

않은 것은

보lt gt

善花公主主隱 공주니믄 공주님( )

----------------------------------------

-

他密只嫁良置古 그 지 얼어 고 몰 결(

----------------------------------------

-

薯童房乙 맛 맛( )

夜矣卯乙抱遺去如 몰 고 가다 에 몰 고(

가다)

주동 역 동- (薯童謠『 』

에 2 ( )他密只嫁良置古

얼다 시집가다 결 다 말 lsquo rsquo

① 실질 미 지니고 므 타 타lsquo ( )rsquo lsquo [ ]

② 에 실질 미 타내고 지 는lsquo rsquo lsquo [ ]rsquo lsquo [ ]密只 密 只

계 타내는

③ 얼어는 실질 미 포 고 므 가lsquo rsquo lsquo [ ]rsquo嫁

것lsquo [ ]rsquo 良

④ 고 어간 는 실질 미 지니고 므lsquo rsquo lsquo -rsquo

것lsquo [ ]rsquo 置

⑤ 고 어미 고는 계 타내고 므lsquo rsquo lsquo- rsquo

고 것lsquo [ ]rsquo 古

가( )

엉 훈 민middot middot middot middot middot世 宗 御 製 訓 民 正 音

말 미 듕 귁에 달middot middot middot middot middot middot middot middot中 國 文 字

니 런middot middot middot middot middot middot 어린middot middot middot middot百 姓

니 고 도 내 들middot middot middot middot middot middot middot middot middot 시러middot

펴 몯middot 미middot middot 니 내middot middot middot middot middot middot middot middot 爲

어엿middot 겨 새middot middot middot 믈여듧middot middot middot middot字 니middot middot middot

사 마다 니겨 킈 middot middot middot middot middot middot middot middot middot便 安

고 미니middot middot middot middot

본 는 상( ) (象

원리에 만들어진 본) ( )形 ㄱ ㄴ ㅁ ㅅ ㅇ

에 는 가 원리에( )加劃

그리고( )ㅋ ㄷ ㅌ ㅂ ㅍ ㅈ ㅊ ㆆ ㅎ

쓰는 병 원리에 만들어진( )竝書

마지막 체( ) ( )異體ㄲ ㄸ ㅃ ㅆ ㅉ ㆅ

ᅀ 다 상 원리에 ㅇ ㄹ

지 는 삼재 상 본 본( ) ( ) ( 天地人 三才

탕 므림과 림에 ) (初ㅡ ㅣ

재)( ) ( )( )出字 再出字ㅗ ㅏ ㅜ ㅓ ㅛ ㅑ ㅜ ㅕ

병 그리고 들 에 다시( )ㅘ ㅝ ㅣ

( )ㅣ ㅢ ㅚ ㅐ ㅟ ㅔ ㆉ ㅒ ㆌ ㅖ ㅙ ㅞ

zb52) 가 에 대한 설 으 르 않은 것을( ) 두 고르

① 어쓰 규 지키고 다

② 리 고 다

③ 말 미 미 등 어 사 다lsquo rsquo

④ 개 지 다

년 학 간고사 대비2013 2 현대고 대비

ECN-0102-2013-001-000076193

⑤ 어 원 에 가 도 고 다

엉 훈 민世 宗 御 製 訓 民 正 音

말 미 듕귁에 달 니

런 어린 니 고 도middot

내 들 시러 펴 몯 미 니middot

내 어엿 겨 새 믈여듧

사 마다 니겨middot 킈 고

미니

훈민 언 본- lsquo rsquo 5 (1459 )

zb53) 위의 에 대한 현대어 풀이가 르~ 않은 것

① 우리 말 과 달

② 어리 말 고 는 것 어도

③ 신 생각 마 껏 펼 는 사 많다

④ 게 생각 여

⑤ 사 마다 게

zb54) 훈민정음 언해 에는 한 을 창제한 동 가 드러나

있다 훈민정음 창제의 정 과 내 이 잘 연결된 것

① 주 신 말 미 듕귁에 달

② 민 신 내 어 겨

③ 신 뻔 킈 고 미니

④ 실 신 사 마다 니겨

⑤ 귀 신 계 주 는 훈민 신과 거리가

가 엉 훈 민( ) middot middot middot middot middot世 宗 御 製 訓 民 正 音 

말 미 귁에 中 國 달 文 字

니 런 어린 니 百 姓

고 도 내 들 시러 펴 몯

미 니 내 어엿 爲 겨 새

믈여듧 니 사 마다 니 字

겨 킈 고 미니 便 安

훈민 언 본- lsquo ( )rsquo ( ) 5 (1459 )訓民正音 世祖

( )

[ 1 ]

동 룡 샤 마다 복( ) ( ) ( )海東 六龍 天福

시니 고 동( ) ( )古聖 同符 시니

[ 2 ]

매 니 곶 여

미 므 니 그 내 러

가 니

[ 125 ]

우 미리( )千世 샨( )定 에( )漢水北 累仁

누 개 샤 복 업 시니( ) ( ) 開國 卜年

신( )聖神 니 샤도 경 근민 샤 욱( )敬天勤民

드시리 다

님 쇼 산 가( ) ( )洛水 山行

미드니 가

어 가- lsquo ( )rsquo 27龍飛御天歌

다 우리신 니쓰고 다만 만 쓰( )

거 샹 귀쳔 다보게 러 귀

여 쓴 도 신 보 가 고 신 에

말 어 보게 각 에 사 들

고 본 몬 능통 후에

죠 죠 니

드 도 만 공 에 사

드 미 죠 고 고 여 보 죠

보다 얼마가 거시 어신고 니 첫

가 죠 니 죠

민 들 어 신 샹

귀쳔 도보고 어보 가 만 늘

고 폐 에 만쓴 죠 민

도 러보지못 고 보니 그게 엇지

심 니 리 보 가 어 운건 다

니 쳣 말마 지 니 고 그

쓰 에 가 우 지 지

몰 거 본후에 가 어 지

고 그니 쓴편지 쟝 보

년 학 간고사 대비2013 2 현대고 대비

ECN-0102-2013-001-000076193

쓴것보다 듸 보고 그 마 니 쓴 고

어 못

그런고 에 리 과 가

만 쓴 못 민 말만 듯고

고 편 그 못 보니 그사 단

병신 못 다고 그사 식 사

니 만 고 다 과 그사

만 고 다 과 업 사 보다 식 고

죠 도 고 각 과

견 고 실 직 귀쳔 간에 그

고도 다 것 몰 귀죡 보다

사 우리 신 귀쳔 다 업

시 신 보고 과 지 게 랴

시니 샹 귀쳔 간에 우리 신 걸

간 보 새지각과 새 걸 미리

독립신- lsquo (1896)rsquo

zb55) 친 어 나의 제 장( ) 2 매 함축적

의 가 가장 유사한 것은

① 지 눈 내리고 매 득 니 내 여 가

사- lsquo rsquo

② 도 어 리듯 그 게 어 다

주 사- lsquo rsquo

③ 눈 살 다 죽 어 린 과 체 여

눈 새벽 지 도 살 다

눈- lsquo rsquo

④ 삶 근심과 고단 에 돌 거니는 여 거 는

여 리 내린 살가지 에 눈 리 눈 리

택 그 생 에- lsquo rsquo

⑤ 늘 러 고 러

청룡 룡 어 개 루 우

신경림 계- lsquo rsquo

zb56) 친 를 위 가 나 에 나타난A B ( ) ( )

세 어의 특 에 의거하여 세 어 표 하

그 산 고 공 도 맑지만

A

주변에 쓰 리는 어리 사 많다

B

건lt gt

식 가 에 타 어 특징에( ) ( )

거 과 어쓰 는 고 지 말 것

A

B

zb57) 가 의( ) 달 아ㆍ 다 의 ( ) 나셔에서 알 있는

세 어 개화 어의 특 을 비 하여 조건 에lt gt

맞게 서 하

건lt gt

어에 는lsquo 개

어에 는 다 태rsquo

zb58) 은 가 는 다 에 나 는 절lt 1gt ( ) lt 2gt ( )

일 를 췌한 것이다 의 의 가 lt 1gt (1)~(2)

유사한 말을 에서 찾아 쓰lt 2gt

보lt 1gt

런 (1) 어린 니 고百 姓

도 내 들 시러 펴 몯 미

사 마다 (2) 니겨 便 安

킈 고 미니

보lt 2gt

죠 고 고 여 보 죠

보다 얼마가 거시 어신고 니 첫 가

죠 니 죠 민

들 어 신 샹 귀쳔

도보고 어보 가 만 늘 고

폐 에 만쓴 죠 민 도

러보지못 고 보니 그게 엇지 심

니 리

년 학 간고사 대비2013 2 현대고 대비

ECN-0102-2013-001-000076193

lt 1 gt

동 룡 샤 마다 복 시( ) ( ) ( )海東 六龍 天福

고 동 시니( ) ( )古聖 同符

lt 2 gt

(A) 매 니 곶

여 니

미 므 니 그 내

러 가 니

lt125 gt

우 미리 샨 에( ) ( ) ( ) 千世 定 漢水北 累

누 개 샤 복 업 시 니( ) ( ) 仁開國 卜年 聖

신( ) 神 니 샤도 경 근민 샤( ) 敬天勤民

욱 드 시 리 다

님 쇼 산 가 ( ) ( )洛水 山行

미드니 가

- lt gt龍飛御天歌

zb59) 장과 내 상 유사한 성격의 조는125

① 뫼 고 고 고 고

어 그린 많고 많고 고 고

어 러 는 울고 울고 가느니

도 견- lt gt

② 강 에 드니 몸 다

그믈 고 가니

뒷 뫼 엄 언 니( )藥

-

③ 말 없는 청산 태 없는 다

값 없는 청 없는 월

에 병 없는 몸 별 없 늙 리

-

④ 가마귀 골에 가지 마

낸 가마귀 새

청강에 것 시 몸 러 가( ) 淸江

-

⑤ 진 골에( ) 白雪

가 매 는 어느 곳에 었는고

에 갈 곳 몰( ) 夕陽

색-

zb60) 위 에 나타난 세 어의 특 으 적절하 않은

것은

① 룡 어 주격 사에 당 는 가 사( ) lsquo rsquo六龍

고 다

② 샤 어에도 어 주체 쓰 다

는 것 다

③ 매 어 달리 사 택에 어

가 지 지지 고 다

④ 므 원 상 직 어 지 다

⑤ 드시리 다 주체 과 상 께 사

고 다

수고 하셨습니다hearts hearts

년 학 간고사 대비2013 2 현대고 대비

ECN-0102-2013-001-000076193

보닷컴에 공 는 별 보는 고등

들 여 주 는

들 습니다 슷 동 지

가 복 는 것 도가

니 복 여 습 시고 거 시

니다

정답 해설

1) 정답[ ] ④

해설 다른 것은 두 특정 업이나 단 내에서 사[ ]

하는 일종의 은어 사회 언에 해당한다 러나

는 언이 아니라 단과대학을 여서 단대 사lsquo rsquo lsquo rsquo lsquo④

대학을 여서 사대라고 한 말에 해당하 일rsquo lsquo rsquo

사회에서도 널리 쓰이 사회 언이라 할

없다

2) 정답[ ] ⑤

해설 사회 언은 같은 단 내에서 쓰이는 언어이[ ] lsquo rsquo

동일 단끼리는 단결 과 친 감을 형성하는

능을 하 리적 안감이 일어나 않는다

3) 정답[ ] ③

해설 사람이라는 차 적 표현에 대한 대안적 표현이[ ]lsquo rsquo

인 아내 처 등으 볼 있다lsquo rsquo

4) 정답[ ]⑤

해설 남성은 주 격 체를 사 한다[ ]

5) 정답[ ] ⑤

해설 흑인은 검다라는 뜻을 가 고 있을 뿐 인[ ]lsquo rsquo lsquo rsquo lsquo rsquo

다 열등한 뜻을 내포하 않는다

6) 정답 살 색 첫 작품[ ] - -

해설 살색 혹은 킨색은 한 인의 피 색을 뜻[ ] lsquo rsquo lsquo rsquo

하는 것으 인종 차 을 추 고 출 이주민

의 평등 을 침해할 있어 년 표 이2005

살 색으 이름을 꾸었다 처녀작은 처녀라lsquo rsquo lsquo rsquo lsquo rsquo

는 단어가 가 고 있는 곡된 성 인 을 한 것

으 첫 작품정도 꾸어 사 하는 것이 좋다lsquo rsquo

7) 정답[ ] ⑤

해설 호는 아들에게 해체를 사 하고 있다[ ] ① ②

장 을 성하는 청자는 자 의 아 느리 아lsquo

들 세 이다 호는 아 느리에게 해rsquo ③

체를 사 하고 있다 호가 느리 아 에게 ④

사 한 해 체 아들에게 사 한 해체는 두 비lsquo rsquo lsquo rsquo

격 체에 해당한다 호는 자 의 아랫사람인 ⑤

느리에게 아들과 마찬가 해체를 사 하는 것이

상 이 만 임 을 한 느리에게 고마 과 쁨

존 의 표 를 하 위해 자 의 아 에게 말하듯

해 체를 사 하고 있다

8) 정답[ ] ③

9) 정답[ ] ⑤

10) 정답[ ] ①

해설 청자 할아 가 장의 주체 아 다 높을[ ] ( ) ( )

경 에는 압존 에 의해 장의 주체를 높이 않는lsquo rsquo

다 러 아 서가 아닌 아 는으 계 lsquo rsquo lsquo rsquo lsquo

니다 가 아닌 있 니다 표현하는 것이 르rsquo lsquo rsquo

11) 정답 당이 당을 쫒았다 당이[ ]

당에 다

해설[ ]

12) 정답[ ] ⑤

해설 서 다른 높임표현을 통해 청자에 대해 리[ ] ⑤

적 거리감을 나타내는 인 은 이 아니라 현정이

다 가 에서 현정은 에게 해 체를 사 함으 써 ( )

친근감을 드러낸다 나 에서 연 을 게을리하는 역 ( )

도 들 때 에 화가 난 현정이 선생님에게 항의하

는 장 에서는 하 체를 사 하여 리적 거리lsquo rsquo

가 어졌음을 나타내고 있다

13) 정답[ ] ①

해설 는 는 얼 빛이 날과 어찌 다르 고[ ] lsquo rsquo

라는 뜻으 전과 달리 임이 화자를 않고

있음을 알 있다

14) 정답 달리 후 가 있다 이를 통해 경[ ] lt gt

쾌한 음악성을 형성하고 노 젓는 상황을 체적으

형상화하는 역할을 한다

15) 정답[ ] ①

16) 정답[ ] ⑤

해설 다 의 자연은 를 성찰하게 하는 대상[ ] ( )⑤

이자 정의 대상이다 의 자연은 자 의 상황과 ⑤

처 를 드러내는 경으 서의 역할을 하 이

이 없다

17) 정답[ ] ③

해설 는 빈천 을 해결하고자 했으나 강산[ ] lsquo ( )rsquo 貧賤③

과 풍 을 달라는 에 거절하 다고 함으 써 자

연에 대한 애정을 드러내고 있으 는 않는

임에 대한 망을 개에게 전가 켜서 임에 대한 리

을 드러내고 있다

18) 정답[ ] ③

년 학 간고사 대비2013 2 현대고 대비

ECN-0102-2013-001-000076193

19) 정답[ ] ⑤

해설 고상한 음악가의 이름을 리말 꽝 럽[ ]

게 꿈으 써 언어유희를 통해 음을 유 하고 있

다 이는 고상한 척하는 총 를 비꼼으 써 비판적

태도를 드러내는 것이 대상을 꽝 럽게 표현

하여 총 의 허 과 사치를 풍자하고 있다

20) 정답[ ] ⑤

해설 는 작품 속 경에 대한 설 이 드러나는 것이[ ]

서 자의 주 적인 견해가 접적으 드러나는 것이

아니다

21) 정답[ ] ⑤

22) 정답[ ] ②

23) 정답[ ] ④

24) 정답[ ] ①

해설 적강 티프는 주인공의 비 한 출생이나 능[ ] ①

과 이 있는 것으 조정의 능함을 풍자하는lsquo rsquo

것과는 거리가 다

25) 정답 픔 나[ ] ( )

해설 의 음악은 고통 는 사람들을 위 하고 아픔[ ] lsquo rsquo

을 치유해 주는 능을 한다고 할 있다 의 lt gt

픔 도 소 된 이 과 더 어 살아가는 따뜻한 마음lsquo rsquo

을 상 한다

26) 정답[ ] ⑤

해설 에게 선천적으 주어 각 장애라는 역경[ ]

은 의 이라는 가사 연 을 있다lsquo rsquo

27) 정답[ ] ④

해설 는 장 란 선 에게 은 개인적인 인상을[ ]

소녀 장정 등으 표현한 것이다lsquo rsquo

28) 정답[ ] ②

해설 담자가 피 담자의 언어적 표현이나 비언어[ ]②

적 표현 하 독자는 담의 위 나 피

담자의 감정 상태를 알 있다 이를 통해 독자는

담 상황을 더 생생하게 느낄 있고 피 담자

를 더 잘 이해할 있게 된다

29) 정답[ ]③

해설 일상생활과 역도 선 서의 성과에 된 것에서[ ]

역도를 하 서 겪는 어 과 내적 고민으 화제를

전화하 위한 것이다

30) 정답[ ] ①

해설 릿속에 새겨 넣듯 이 억되도 함 세상[ ] ② ③

살이가 힘들고 고생 러 속 하여 자유를 ④

가 없는 고통의 상태를 비유적으 이르는 말

적의 침입을 막 위해 쌓은 축 켜야 할⑤

대상을 비유적으 이르는 말이다

31) 정답[ ] ④

해설 이 의 종류는 전 으 인 사건 경[ ] lsquo

비평을 성 소 삼는다rsquo

32) 정답[ ] ④

해설 근은 삼대독자 태어났음을 에서 확인할[ ]

있다 형제들과의 담은 이뤄 가 없다

33) 정답[ ] ⑤

해설 근은 가난에도 하고 화가를 꿈꾸었다[ ] (3

단 또한 다른 화가 망생들은 정 육을)

위해 상 학 학 해 유학 에 랐 만

근은 다른 을 찾아야 했다 단 세에(5 ) 18

근은 조선 전람회에 입선하 다 단 의(6 )

만종은 인간과 자연이 엮어 가는 경건한 조화 을lsquo rsquo

나타낸다

34) 정답[ ] ①

해설 근이 속에서도 창작활동을 추 않고[ ]

하는 닭은 은 세상과 타협할 르는

근이 세상의 이해를 하 위한 가장 떳떳한 단

이 때 이다

35) 정답[ ] ⑤

해설 전 은 서 자의 주 적인 평이 리는 것이[ ]

만 위 제 은 인 이 살았던 대 사회적 경

을 통해 객 적인 인 의 을 제 하고 있다

36) 정답[ ] ⑤

해설 전 은 인 사건 경 비평이라는[ ] lsquo rsquo⑤

성 이 어져 있다

37) 정답[ ] ①

해설 이 은 동양인과 서양인의 사고 에 차이가[ ]

있다는 것을 대조를 통해 설 하고 있다 또 쓴이

의 제자가 축 경 를 러 가서 경험한 일화를

통해 동양인이 서양인에 비해 주 상황에 더 많은

주의를 인다는 주장을 뒷 침하고 있다

38) 정답[ ] ④

39) 정답[ ] ②

40) 정답[ ] ②

41) 정답[ ] ④

42) 정답[ ] ③

43) 정답[ ] ④

44) 정답 도서 의 휴 일 도서 의 이 간 도서의[ ]

해설 도서 장은 임의 정한 휴 일과 도서 이[ ]

간 도서의 상 등을 게 할 의 가 있다

년 학 간고사 대비2013 2 현대고 대비

ECN-0102-2013-001-000076193

45) 정답[ ] ①

해설 제 조의 정 휴 일 의 휴 일의 사전 게[ ] 3

는 도서 장의 의 조항에 속한다

46) 정답[ ] ①

해설 개인 정 호 의 를 제 하 했 만 항[ ]

나눠서 제 하 않고 대 나열하고 있다

47) 정답[ ] ②

해설 제 조의 내 을 회사는 다른 회사 협[ ] 7 lsquo

계약을 통해 서비 를 제공하는 경 회 의 아이디

등 개인 정 를 해당 회사에 전송할 있다는 내rsquo

이 있으 의 제점을 제 할 있다②

48) 정답[ ] ④

해설 는 도서 장의 의 에 해당하고 나 는 도[ ] ④

서 장의 리에 해당한다

49) 정답[ ] ③

50) 정답 은 음독으 적었고 은 훈독으 적었[ ] (1)

다 과 동일한 표 리 적은 것은 이고 (2) ce

과 동일한 표 리 적은 것은 이다ab

51) 정답[ ] ③

52) 정답[ ] ①②

53) 정답[ ] ③

54) 정답[ ] ③

55) 정답[ ] ①

56) 정답 른 죠코 어린 노 하니라[ ] A B

57) 정답 세 어에서는 활 형이 칙적으[ ] lsquo rsquoㄹㅇ

나타났 만 개화 어에서는 활 형이 쓰 다 lsquo rsquo ㄹㄴ

58) 정답 호 가 흔[ ] (1) (2)

59) 정답[ ] ④

60) 정답[ ] ③

Page 4: 현대고대비 국어 - chamsoriedu.com 「콘텐츠산업진흥 법」외 에도 저작권 의하여 ... 다른주체에게어떤동작을하도록만드는것을나타내는

년 학 간고사 대비2013 2 현대고 대비

ECN-0102-2013-001-000076193

다 믄 눌 가시 고

어 여 고 내 드러 보

내 얼 거동 님 가마

엇 보시고 다 실

도 님 미 업

어 러 돗

시 엇 다 신고

누어 각 고 니러 여 니

내 몸 지 죄 뫼 티 시니

원망 사 허믈 랴

워 니 믈 타시 다( ) 造物

각 마

략( )

님다 쇼식 므 쟈 니( )消息

도 거 다 사 가

내 업다 어드러 가쟛 말고

거니 거니 뫼 가니

니 개 므 고

산쳔 어 거니( ) (A)山川 월( )日月 엇 보

지쳑 거든 쳔리 보랴( ) ( ) 咫尺 千里

리 믈 가 보쟈 니

믈결 어 고

샤공 어 가고 만 걸 니

강 쟈 보니( )江天

님다 쇼식 득 고( )消息

략( )

근 역진 간 드니( )力盡

지극 님 보니( )精誠

얼 마 늘거( ) ( ) 玉 半

근 말 슬 쟈 니

눈믈 니 말 들 어

못다 여 니( )情

계 ( )鷄聲 엇 돗 고

어 허 다 님 어 간고( ) 虛事

결 니러 열고 보니( )窓

어엿 그림재 다

리 싀여 여 월( )落月

님 겨신 ( )窓 드시 리

각시님 니 쇼

미 곡- lt gt-

( )

내 님믈 그리 우니다니

(B)산 동새( )山 슷 다

니시 거 신

월 시리 다( ) 殘月曉星

시 도 님

벼 시니 뉘러시니 가

과 도 허믈도 만 업 다( ) ( ) 過 千萬

마리신

니미 마 니 시니 가

님 도 드 샤

과- lt gt-

zb13) 가 의 에 대한 설 으 적절하( ) ~ 않은 것은

① 과 달리 고 다

② 별 신 탓 여 고 다

③ 별 다 가 고 다

④ 산에 는 망 취 지 못 lsquo rsquo

⑤ 실 룰 없는 망 실 시키고 신

다 재 변 고 다

사( ) 5春詞

고 볃티 믉결 다

어 어

그믈 주어 랴 시 가

지 지 어( ) ( ) ( )至匊悤 至匊悤 於思臥

탁 가 니 고 도 니 다( ) ( )濯纓歌 興

사( ) 2夏詞

닙 싸 고 쟝만 마

닫 드러 닫 드러

청 립 사 가 냐( ) ( )靑蒻笠 綠蓑衣

년 학 간고사 대비2013 2 현대고 대비

ECN-0102-2013-001-000076193

지 지 어( ) ( ) ( )至匊悤 至匊悤 於思臥

심 내 가 가( ) ( )無心 白鷗

사( )3秋詞

운 니러 고 티 니 다( )白雲

돋 돋

믈 믈 동 가쟈( ) ( )西湖 東湖ㅣ

지 지 어( ) ( ) ( )至匊悤 至匊悤 於思臥

료 곳마다 경 다( ) ( )白蘋紅蓼 景

동사( )4冬詞

간 눈 갠 후 에 경믈 달 고( ) ( )後 景物

어 어

만경 리 듸 쳔 산( ) ( )萬頃琉璃 千疊玉山

지 지 어( ) ( ) ( )至匊悤 至匊悤 於思臥

계 가 계 가 간 니 다( ) ( ) ( )仙界 佛界 人間ㄴ ㄴ

zb14) 위 을 의 조 형태적 특 에서 어lt gt (1)

떤 차이가 있는 를 서 하고 인해 노래의 아 (2)

름다 에 어떤 향을 주었는 에 대해 서 하

보lt gt

십 경 여 가삼간 지어내니

달 청 맡겨 고

강산 들 곳 없 니 러보고 보리

가 가 각시 본 듯도 고( )

샹 경 엇 니 별 고天 上 白 玉 京 離 別

다 믄 눌 가시 고

어 여 고 내 사 드러 보

내 얼 거동 님 가마

엇 보시고 다 실

도 님 미 업

태 어 러 돗

시 낫 엇 다 신고

누어 생각 고 니러 여 니

내 몸 지 죄 뫼가티 싸 시니

원망 사 허믈 랴

워 니 믈 타시 다造 物

님다 쇼 식 므 쟈 니( ) 消 息

도 거 다 내 사 가

내 마암 둘 대 업다 어드러 가잔 말고

잡거니 거니 놉픈 뫼해 라가니

은 카니 안개난 사 일고

산 쳔이 어둡거니山 川 일日 月 을 엇디

쳑을 라거든 쳔 리랄 라 랴咫 尺 千 里

찰하리 가의 가 히나 자 하니

람이야 결이야 어둥졍 된뎌이고

샤공은 어대 가고 븬 만 걸 나니

강 텬의 혼쟈 셔셔 디난 해랄 니江 天

님다히 쇼 이 더 아득한뎌이고消 息

다 쳠 리 듕만 도 니( ) 茅

벽 쳥 등 눌 갓 고半 壁 靑 燈

라 나리 헤뜨 니니

져근덧 녁 하야力 盡 풋잠을 잠간 드니

졍 셩이 하야 꿈의 님을 니精 誠

가탄 얼 이 이나마 늘거셰라玉 半

마암의 근 말삼 카장 삷쟈 하니

눈 이 라 나니 말인들 어이하

졍을 다하야 이조차 여하니情

뎐된 계 셩의 잠은 엇디 돗던고鷄 聲

라( ) (A)찰하리 여디여 낙 이나 되야 이셔洛 月

님 겨 창 안해 드 비최리라窓

(B)각 님 달이야카니 잔 비나 되쇼셔

zb15) 위 가 라 에 대한 설 으 적절하( )-( ) 않은 것은

① 민들 과 삶 담고 다

② 여 체 식 어 다

③ 사미 곡과 께 가사 미 다

④ 우리말 사가 평가

년 학 간고사 대비2013 2 현대고 대비

ECN-0102-2013-001-000076193

고 다

⑤ 님 미 사 는 마lsquo rsquo

가사 다

가 춘사( ) [ 5]

고은 티 쬐얀 결이 름 다

이어라 이어라

을 주어 두랴 낙 노흘일가

총 총 어( ) ( ) ( )至匊葱 至匊葱 於思臥

탁 가 의 흥 이 나니 고 도 니 다( ) ( )濯纓歌 興

하사[ 2]

년닙희 두고 찬으란 쟝만 마라

닫 드러라 닫 드러라

청약립 은 써 잇노라 녹사의 가져 냐( ) ( )靑蒻笠 綠蓑衣

총 총 어( ) ( ) ( )至匊葱 至匊葱 於思臥

내 좃 가 제 좃 가( ) ( )無心 白鷗

추사[ 3]

이 니러나고 나 티 흐느 다( )白雲

돋 라라 돋 라라

의 셔호 혈 의 동호 가쟈( ) ( )西湖 東湖ㅣ

총 총 어사( ) ( ) ( )至菊悤 至菊悤 於思臥

빈홍 곳마다 경 이 다( ) ( )白蘋紅蓼 景

동사[ 4]

간 의 눈 갠 후 에 경 이 달랃고야( ) ( )後 景物

이어라 이어라

압희 만경류리 듸희 쳔텹 산( ) ( )萬頃琉璃 千疊玉山

총 총 어( ) ( ) ( )至菊悤 至菊悤 於思臥

선계 가 블계 가 인간 이 아니 다( ) ( ) ( )仙界 佛界 人間ㄴ ㄴ

윤선도 어 사 사- ( )漁父四時詞「 」

나 살어리 살어리랏다( ) 쳥산 애 살어리랏다( ) 靑山

위랑 래 랑 고 쳥산 애 살어리랏다 ( ) 靑山

얄리얄리 얄랑셩 얄라리 얄라

러라 러라 새여 자고 니러 러라 새여

널라 름 한 나도 자고 니러 니 라

얄리얄리 얄라셩 얄라리 얄라

청산별곡- ( )靑山別曲

다 청산은 엇뎨 야 만고애 프르르( ) ( ) ( ) 靑山 萬古

유 는 엇뎨하야 주야애 디 아니난고( ) ( )流水 晝夜

리도 치디 마라 만고상청 호리라( ) 萬古常靑

도산 이곡- ( )陶山十二曲

라 개를 여라 이나 르되 개 치 얄 랴( )

님 꼬리를 홰홰 치 치뛰락 나리뛰락 겨서 내

닷고 님 뒷 을 동 동 르락 나으락 캉캉

도리 암캐

이 릇 릇 날 들 너 이 이 랴

작자 상-

마 빈천 을 랴 고 에 드러가니( ) ( ) ( )貧賤 權門

침 업 흥졍을 뉘 져 쟈 리

강산과 풍 을 달나 니 는 리 리

조찬한-

정정 이랬거니 아람도리 큰 솔이 혀( ) ( )伐木丁丁

도 하이 골이 어 아리 소릴 쩌르 돌아 도 하이 다

람 도 좇 않고 새도 않어 은산 고 가 차라리

뼈를 저리 는데 눈과 이 종이 담 회 나 달도 름을

다 흰 뜻은 한 이골을 걸음이란다 절 이 여섯

판에 여섯 고 고 라간 뒤 조찰히 늙은 사나이의 남

내음새를 는가 름은 람도 일 않고 고 에 히

흔들리 노니 견디란다 차고 연 히 픔도 꿈도( )兀然

없이 장 산 속 겨 한 내-

정 장 산- 「 」

zb16) 다 에 나타난 상과 표현을 활 해 작 연 을 해( )

것이다 적절하 않은 것은

① 몰 는 도에 몸 맡 고 는 고통

욱 답게 가꾸 는 느님 산 겠지

② 돌 틈 뚫고 어 민들 보 리 진 고통

다가 도 민들 럼 강 게 살 지 겠는가

③ 벽에 런 움 없 어지고 는 폭포

여 시 는 죽 워 지 는 강

지 는가

④ 상 겨울 도 다리 꿋꿋 살 가고

겠지 망 지 는 삶 얼마 다운가

년 학 간고사 대비2013 2 현대고 대비

ECN-0102-2013-001-000076193

⑤ 막 늘 울리고 는 귀 미 리는

지새우는

가( ) 빈천( )貧賤을 랴 고 에 드러가니( )權門

침 업 흥졍을 뉘 져 쟈 리

강산과 풍 을 달나 니 는 리 리

나 청강 에( ) ( )淸江 비 듯는 소 어 읍

만산 홍 이 휘드르 는고야( )滿山紅綠

두어라 춘풍 이 날이리 을 어라( ) 春風

다 청산은 어찌 야 만고에 푸르르( ) ( ) ( )靑山 萬古

유 어찌 야 주야애 디 아니 고( ) ( )流水 晝夜

리도 치 말아 만고상청 하리라( ) 萬古常靑

라( ) 개를 여라 이나 르되 개 치 얄 랴

님 리를 홰홰 치 치 락 리 락

겨서 내닷고 고 님 뒷 을 동 동

르락 나으락 캉캉 도리 암

이 릇 릇 날 들 너 이 이 랴

zb17) 의 적 능을 비 한 내 으 가장 적절한

것은

① 는 가 지닌 실 계 다

② 는 내 갈등 심 시킨다

③ 는 가 는 상 각시킨( )愛着

④ 는 달리 내 갈등 시킨다

⑤ 는 달리 다 사 들 간 가

강 다

가 님다히 쇼 을 아 나 아쟈 니( ) ( )消息

도 거의 다 일이나 사 가

내 둘 업다 어드러 가쟛말고

잡거니 거니 놉픈 뫼 라가니

은 니 안개 일고

산쳔 이 어둡거니 일 을 엇디( ) ( )山川 日月

쳑 을 거든 쳔 리 라 랴( ) ( )咫尺 千里

하리 의 가 히나 쟈 니

람이야 결이야 어둥졍 된뎌이고

샤공은 어 가고 븬 만 걸 니

강텬 의 혼쟈 셔서 디 니( )江天

( )

쳠 자리의 듕만 도라 니( )茅簷

쳥등 은 눌 위 야 갓 고( )半壁靑燈

리 헤 니니

져근덧 녁 야 픗 을 잠간 드니( )力盡

졍셩 이 야 의 님을 니( )精誠

가 얼 이 이나마 늘거셰라( ) ( )玉 半

의 근 말 장 쟈 니

눈 이 라 나니 말인들 어이

졍 을 다 야 이조차 여 니( )情

뎐된 계셩 의 은 엇디 돗던고( )鷄聲

어 허 다 이 님이 어 간고( )虛事

결의 니러 안자 창 을 열고 라 니( )窓

어엿븐 림재 날 조 이 다

하리 여디여 낙 이나 되야이셔( )落月

님 겨 창 안 드 비최리라( )窓

나 내 님 리자 니다니( )

산 접동새 난 이 이다

아니 거츠르 아으

잔 효성이 아 리이다

넉 라도 님은 녀져라 아으

더 니 뉘러 니잇가

과도 허 도 천만 없소이다

힛마리 뎌

읏븐뎌 아으

니 나 마 니 니 잇가

아소 님하 도람 드르샤 쇼셔

zb18) 맥으 아 에 들어갈 적절한 것은

① 시 엇 다 신고

② 다 믄 눌 보 가시 고

③ 님다 쇼식 욱 득 고( )消息

④ 원망 사 허믈 랴

년 학 간고사 대비2013 2 현대고 대비

ECN-0102-2013-001-000076193

⑤ 죠 뫼 티 시 가( ) ( ) 粥早飯 朝夕

가( )

거리

공신 후 심 늦도( )劉尋

식 없어 과 께 산에 드리고 신

태몽 꾼 에 만고 웅 상 지닌 들

낳 키운다 그 후 신 들 에 역심 ( )逆心

담 귀 등 심 여 리 귀

보내고 지 죽 는 도망 가다

가 만 죽 고 에 경 가는 들 도움

살 다 그러 사 에 심 귀

보고 담 여 고 강 주가 승상

득 여 고 신 사 삼는다 그 후 강 승상

에게 심에 상 리지만 여움

사 귀 가게 다 강 승상 몸 는

연 과 헤어 리 다

경쇠 리 들리 에 들어가니 색

에 게 단청 누각과 큰 집들( )丹靑

다 주 보니 ( ) (一柱門 黃金

산 사 어 었다 산) lsquo rsquo 大字

들어가 고승 다 그( ) ( ) 山門 高僧

거동 보니 눈 눈 듯 고

변 같 귀는 어 에 늘어 니( ) 白邊

맑고 어 골격과 신 평 니었

팔염주를 에 걸고 육환장 을 고서 흑포( )六環杖

장삼에 떨어 송낙 쓰고 나 유생을 고 말[ ] 松蘿

하 를

소 이 연 하여 유상공 는 행차를 동 에 나ldquo

가 맞이하 하 으니 소 의 함을 서하

rdquo

유생이 크게 놀라 말하 다

천한 인생으 팔자가 하여 어 서 를 잃고ldquo

정처없이 다니다가 연히 이곳에 대사를 만난 것인데

토 대하 소생의 성은 어떻게 알고 있 니 rdquo

노 이 답하여 말하 를

어제 남악 형산 의 화선 이 소 의 절에ldquo ( ) ( )男樂 衡山

어서 소 에게 탁하 를 내일 낮 경에 남경 lsquo 12

동성 안에 사는 유 의 아들 충 이가 것이니 내쫓

말고 잘 대접하라 하셨 니다 마침 소 이 찾아 나rsquo

다가 상공의 차람새를 니 남경 사람이 에 알아

았 니다rdquo

유생이 말을 듣고 한편으 쁘고 한편으 퍼하

서 노 을 따라 들어가니 여러 들이 합장 하

가 했다 노 이 에 들어가 저녁 을 은 후에

을 편히 니 이곳은 선경 이었다 세상의 일을( ) 仙境

두 잊고 일 이 편안하 다 이후 는 노 과 함

서 도 이 탐 하고 경도 확하게 의 게 되었( )兵書

다 이 게 되니 대 천 에 가객 은 없 ( ) ( )大明天地 佳客

고 덕산 속에 리 른 만 있더라 래 ( ) 廣德山

이 천상 사람으 살아 있는 처를 만나 이한

을 니 재주 민함을 누가 당할 있겠는가

낮으 공 하더라

유충 전- -

웬늠 어가 사 싸다냐( ) ldquo rdquo

내가 가 막 런거 니

보통것 닐러 그 어낸ldquo ( )

틀어 주 그 가 루 러 허 에

싶어 키 틀어 주 그 가( )

루 허 우간 곡 틀어주는 루 못 는

웂는 고 닝께 고 지 들

어 사는 고 가 다는 건 에 그 집에

rdquo

그런 단 어들 어 새벽에 떼죽 거

다 고 어 보니 죄다 허 게 집어진

는 것 었다

총 가 내화를 꿴 뛰어나 만 아 소 없는

일이었다

어떻게 된 거야ldquo rdquo

한동안 넋나간 듯이 서 있던 총 가 하고많은 사람

에 하필이 유자를 겨냥하 은 말이었다

쎄유 아마 새에 고뿔이 들었던 개비네유ldquo rdquo

유자는 러 딴청을 하 다

야 고 가 에서 감 가 들어 죽는 고 두ldquo

어rdquo

총 는 가 혐의자 나 되는 것처럼 화풀이를( )嫌疑者

하 드는 것이었다

는 비위가 상해서

야 팔자가 사나서 이런 후 에 살라니|

여러 가 다 객고가 쌓여서 조 두 안 좋았을 테 helliphellip

런디다가 릇쓰 이 가락을 트는 대 디립

다 춰댔으니 과 해서 살끼두 다소 있었을 테 helliphellip

래 들어서 키 는 새끼덜일 이 다다 탈이 많은

이니 ldquohelliphellip

는 트의 독성을 충 히 내 않고 고 를 넣

은 것이 탈이었으 니 하 서도 러 참으 의 을 떨

었다 략 - -

마리가 마리 값 간다는 워ldquo

그냥 내뻔지 거시 허 싼 고 는 맛

겄다 싶 허 게 눌 강 어helliphellip

허 마늘 통 다

년 학 간고사 대비2013 2 현대고 대비

ECN-0102-2013-001-000076193

게 지 고뿌 지 rdquo

어 어째ldquo rdquo ldquo rdquo

런 도 것들 같 니ldquo ( ) rdquo殘忍無道 helliphellip

는 탱 여 지 못 다 보( ) 憤氣撐天

니 는 는 다 동원 여 통 쳤

생각 여 는 눈 다

달리 리헐 감ldquo rdquo

들 고 말 니었다 그가

는 것 그 말고는 없었 에 그 게 뒷동

달 거 다

는 우 럽고 식 짝 없는 랫것들 고

다 공연 신 가고 득 것

없다고 단 는지 결 웬만큼 고루 어

그 것들 쪽 에다 고 어주지ldquo

고 그 그걸 주 어 에 에 helliphellip

눈 없는 독 들 rdquohelliphellip helliphellip

고 말 럼 얼거리 들어가 리는 것 었

- ( ) -兪子小傳

zb19) 위 나 를 읽고 평가한 것으 적절하( ) 않은 것

① 사 리 통 감과 사실 고

격 과 달 고

② 는 가 재 컫는lsquo rsquo lsquo rsquo

미 가진 여 는 것 겠

③ 는 식 말 는 웃 상lsquo rsquo

여 는 미 지니고

④ 는 어가 죽 짐 지만 내색 지 고lsquo rsquo

말 고

⑤ 언어 통 가들 여 우리 통

것들 역 고

가 체 거리( ) [ ]

나라 종 연간에 정언주 의 을 하고( )正言注簿

있던 유 은 늦도 자 이 없어 한탄하다가 남악 형산lsquo rsquo

에 치성을 드리고 이한 태 을 꾼 뒤 아들을 낳아 이름

을 충 이라 고 키 다 이때 조정의 하들 에 역

을 품은 정한담 최일 등이 가달의 침입에 대한( ) 逆心

유 의 유화적 입장을 제 삼아 유 을 함하여 양

내고 유 의 에 을 러 충 자마저 살해하

한다 러나 충 은 천 조 정한담의 마 에서 어

나 많은 고난을 겪다가 은퇴한 재상 강희주를 만나 사위

가 된다 강희주는 유 을 하 고 상소를 으나 정

한담의 공격을 아 양을 가게 되고 강희주의 가족은

난을 피하여 두 흩어 다 충 은 강 소저 이 하고

사의 노 을 만나 를 때를 다린다 이

때 남적과 적이 를 들고 나라에 쳐들어 자 정한

담은 자 출전하여 남적에게 항 하고 남적의 선 장이

되어 천자를 공격한다 정한담에게 여러 패한 천자가

항 하 할 음 충 이 등장하여 남적의 선 정 걸

을 죽이고 천자를 출한다 충 은 단 으 란 을

쳐 고 정한담을 사 잡는다 리고 호 에게 ( )胡王

잡혀간 황후 태후 태자를 출하 유 에서 고생하

던 아 유 과 장인 강희주를 한다 또한 이 하

던 어 니 아내를 찾고 정한담 일파를 리친 뒤 높은

에 라서 화를 누린다

사 들 별 고 없 다니었다( )

마 마 돌 다니 걸 여 고

어 곤 다 에는 동쪽에 고

에는 쪽에 니 가 에 리는 엽

가는 없 니 늘 다니는 었

다 얼 말 죽 사 같고 림새가 말

니었다 가슴 에 고 등

삼태 헌 에 니 달 ( )奇男子

가 도리어 걸 었 담 만 열 도 ( )傅說

고 만났고 만 갈( ) ( ) 慇 武丁

도 탕 만났( ) ( ) (伊尹 成湯 渭

여상 도 주 만났는) ( ) ( ) ( ) 水 呂尙 周 文王

월 같 러가 도 어느 열 살

늘과 집 삼고 사 에 쳐 거리에

어 다가 곳에 니 다 ( ) 楚

지 다가 사 보고 가에 다다( )長沙

니 망 가에는 원 리가 슬 고 가

가 내리는 사 에는 갈매 가 갈 뿐 었다

쪽 돌 보니 가 우거 고

가 사 보 었다 그곳에

가니 는 사( ) 汨羅水

는 다 주 가 쓰고 죽고

곳 었다

마 감 여 에 가 사 살펴보니

에는 삼 고 그 에( ) 屈三閭

는 만고 월 과 지 가는 그 들( )風月

가 어 었다( ) 路程記

동쪽 벽 에 새 운 어 거늘 그

보니 월 에 경 주 는 간신에게ldquo ( )敗

보고 연경 귀 가다가 에 죽 rdquo

거늘 그 보고 에 거꾸러

통곡 말

[A]ldquo우리 연경 간 만 니 에

지 살 상에 엇 겠는가

에 고 에 었 니

상에 살 것 가 도 께 지리 rdquo

년 학 간고사 대비2013 2 현대고 대비

ECN-0102-2013-001-000076193

고 가에 내 가니 울 리가 에 지 사

쳤는지 심 심 것 가

신 심 것 가

다( ) 강 승상에게는 들 없고 다만 만

었다 가 낳 에 가 색

타고 내 에게 말 는 ldquo

니다 미원 과 연 맺고 ( ) ( )紫薇垣 緣分

었는 께 강 집 보내 에

니 게 여겨 주십시 거늘 rdquo

미 가운 낳 니 가 고 거동

단 다 시 짓 쓰 고 는 (音

없었 니 여 가운 지 는 짝) 律

룰 만 사 없었다 가 사 여 사 감

게 고 지 못 고 염 는 만다

다가 당에 거 고 식같 러 내니

고귀 상 루 말 다 어 울 도 다( ) 相

귀 사 없고 웅 걸( )富貴爵祿

만고 었다 승상 매우 뻐 내당 ( )內堂

들어가 에게 사 니 역시 매우 거

워 말 다

ldquo 도 마 사 는 승상께

그 게 말 시니 상 여러 말 지 말고

사 도 시다rdquo

상이 에 나 충 의 손을 잡고 결혼과 하여 ldquo

너에게 히 할 말이 있다 내가 늙은 말년에 딸

하나만을 두었는데 니 너 하늘이 정해 필

임이 하다 이제 년고락 을 너에게 탁 ( )百年苦樂

하겠다 하 대 충 이 릎을 꿇고 앉아 눈 을 흘리rdquo

여쭈었다

소자의 을 해주 고 또 하 에 두고자 하ldquo ( )膝下

니 감사하 이를 데가 없 니다 다만 가 속에 통탄

할 일이 사 쳐 있 니다 소자가 이 없어 양친 ( )兩親

의 생사를 른 채 결혼하여 아내를 얻는 것은 자 으

서 할 도리가 아닙니다 이것이 한 러 뿐입니다 rdquo

승상 그 말 듣고 슬 에 어 고

것 에 맞 어 변 게 리ldquo

는 다 집 시 공 도 여 ( )始祖公

고 가 에 가가 어진 만 개 공신

었 니 도 러워 말 시고 시 rdquo

택 여 니 다운 신 과 신

습 늘에 죄 짓고 간 상에 내 신

혼 를 다 끝내고 으 들어가 사 을 살펴 니 빛

나고 빛난 것이 한 입으 는 다 말하 어 고 하나

는 다 하 어 더라 에 켠 환한 촛 ( )新房

아래 은 에 랑과 가 평생의 연 을 맺었( )緣分

으니 서 사랑하 주고 은 말을 어떻게 다 헤아릴

있으 어떻게 다 하리 을 낸 후에 이튿날

상 를 니 상 거 마음을 이 하

더라

각 생 강 승상 집 쪽( )

늘 보고 없 가 신 신 생각 니

없고 어 없었다 는 어떻게 도리가 없다

여 산 에 들어가 리 고 어 도 닦

고 다 그 산 보고 가다

가 곳에 다다 니 에 큰 산 었다 많 우

리 골짜 가 늘 는 가운 색

에 고 갖가지 가 짝 어 었 ( )花草

다 신 산 생각 고 들어가니 경개 ( )景槪

가 매우 뛰어 고 경 산 다 산 리에 들

리는 것 리 보 는 것 울 청산뿐

었다 가 고 울 어 가

니 들 많 가지들 못 어 동

에 늘어 들거리 는( ) 洞口

우거진 가지에 갖 들 다 었다( ) 春情

계상 에는 공 는 늘( ) 花溪上

에 걸린 폭포가 벽 는 리는 산사( )層巖絶壁

쇠 리 객 에 는 듯 늘( ) ( ) 寒山寺 客船

에 싸여 는 습 산

그린 여 병 러 듯 다 경쇠 리가 들

리 에 들어가니 색 에

게 단청 누각과 큰 집들 다( ) 丹靑

주 보니( ) ( ) lsquo一柱門 黃金大字

산 사 어 었다 산 들rsquo ( )山門

어가 고승 다 그 거 보니( ) 高僧

눈 눈 듯 고 변 같 ( )白邊

귀는 어 에 늘어 니 맑고 어 골격

과 신 평 니었다 염주

에 걸고 짚고 포 삼에 어진( )六環仗

쓰고 생 보고 말

승 연 여 상공 시는 동 에ldquo

가 맞 지 못 니 승 십시 rdquo

생 크게 말 다

생 가 여 어 고ldquo

없 다니다가 우연 곳에 사 만 것

그 시 생 어떻게 고 습니 rdquo

승 답 여 말

어 산 승 에ldquo ( ) ( )南岳 衡山

시어 승에게 탁 내 낮 시경에 경 lsquo 12

동 에 사는 심 들 가 것 니 내쫓

지 말고 습니다 마 승rsquo

다가 상공 림새 보니 경 사 에 보

습니다rdquo

zb20) 위 의 친 에서 서 자의 개입이 드러나~

는 이 아닌 것은

① 달 가 도리어 걸 었( ) 奇男子

② 신 심 것 가

년 학 간고사 대비2013 2 현대고 대비

ECN-0102-2013-001-000076193

③ 다운 신 과 신 습 늘에 죄 짓고

간 상에 내 신 다

④ 사 주고 말 어떻게 다 헤 릴

어떻게 다 리

⑤ 신 산 생각 고 들어가니 경개 가 ( )景槪

매우 뛰어 고 경 산 다

거리 연[ ] ( )弘治

간 에 공신 후 에 언(1488~1505) ( )正言

주 는 벼슬 심 늦도 식( ) ( )主簿 劉尋

없어 과 께 산에 드리고 신 태

몽 꾼 에 만고 웅 상 지닌 들 낳

키운다 그 후 신 들 에 역심( )逆心

담 귀 등 심 여 리 귀 보내

고 지 죽 는 도망 가다가

만 다 에 에 어 니

헤어지게 다

에 에 어 니 헤어지게

다 그 후 사 들에게 우연 돌

생 다가 어느 열 살 었다 열 살

지 다가 우연 귀 견 는

그것 그 살 도 었고 그

귀 본 신도 지 죽고 마 고

크게 운다

( )

에는 강 주 는 재상 살고 었

니 시 에 과거에 격 여 승상 벼슬 다가 간

신 만 벼슬 그만 고 고 돌 었

다 그러 신 지 가 지 못 여 상

가 못 결 는 상 여 원 니

신 들 그 직간 꺼 다 그 에 도

담과 귀가 강 승상 가 미워 다

강 승상 마 본 에 갔다가 돌 는[A][ ( )本府

에 우편 주 에 다가 색( ) ( ) 右便 酒店

에 어리었는 청룡 에 지 늘

여 통곡 고 사 는 꿈 꾸었다] 마

상 게 생각 여 새 다리다가 새벽

닭 울고 가 달 갔다 가 보니

과연 어 동 가 가에 울고 는지 달

들어 그 고 사 에 어 말

는 어 어 에 어 가ldquo

닭 곳에 우느냐 니 울rdquo

그 고 답 여 말 다

는 경 동 에 사는 언 주 공 들ldquo

니다 께 간신 만 연경 귀 가

시다가 에 죽 사 에 는 닭에

도 에 죽고 니다rdquo

강 승상 말 듣고 크게 낯 변 말

것 웬 말 냐 근 동 ldquo (老

못 갔 니 그 사 변 여)患

런 변 었단 말 가 주 는 신 다

같 에 벼슬 다가 는 가 많 들어 고

돌 는 주 가 게 꿈 에 생

각 겠느냐 생각지 못 다 미 지 간

지지 말고 께 가 략rdquo ( ) hellip hellip

죽게 주 사당에 단 도 러운

겠느냐 말 말고 시는지 rdquo

어 없어 강 승상 가니 그곳

월계 었다

다( )

가가 고 지 사 들 가( )櫛比

통 는 리가 과

답게 꾸민 누각과 큰 집들 늘 고

게 식 가 어 들 태운 가고

었다 략 강 승상에게는 들 없고 ( ) hellip hellip

다만 만 었다 가 낳 에

가 색 타고 내 에게 말

는 니다 미원 과ldquo ( )紫薇垣

연 맺고 었는 께 강 집( )緣分

보내 에 니 게 여겨 주십시

rdquo

거늘 미 가운 낳 니 가

고 거동 단 다 시 짓 쓰 고

는 없었 니( )音律 여 가운

지 는 짝 룰 만 사 없었다 가 사

여 사 감 게 고 지 못 고 염 는 만다

다가 당에 거 고 식같

러 내니 고귀 상 루 말 다 ( )相

어 울 도 다 귀 사 없 ( )富貴爵祿

고 웅 걸 만고 었다 승상 매우 뻐

내당 들어가 에게 사 니( ) 內堂

역시 매우 거워 말 다 도 마 ldquo

사 는 승상께 그 게 말 시니

상 여러 말 지 말고 사 도 시다rdquo

( )

승상 에 고 결 과 ldquo

여 에게 말 다 내가 늙 말 에 지

만 었는 지 보니 늘

다 에게 탁 겠 ( )

다 신 꿇고 눈 리rdquo

여 었다 주시고 슬 에 ldquo ( )膝下

고 시니 감사 룰 가 없습니다 다만 가슴

에 통탄 사 쳐 습니다 복 없어

생사 결 여 내 얻는 것( )兩親

식 도리가 닙니다 것 러울 뿐 니

다rdquo

상 그 말 듣고 슬 에 어 고 말

것 에 맞 어 웅변 ldquo

년 학 간고사 대비2013 2 현대고 대비

ECN-0102-2013-001-000076193

게 리 는 다 집 시 공도 여

고 가 에 가가 어진 만 개 공신

었 니 도 러워 마 시고 rdquo 시

택 여 니 운 신 과 신

습 늘에 죄 짓고 간 상에 내 신

다 략 지낸 후에 튿 승상 ( ) hellip hellip

니 승상 거운 마 지 못

마( )

듯 월 러 생 열다 살 었

다 에 승상 어진 사 얻고 만 에 근심 없었

다만 주 가 간신 에

죽 것 생각 마 곧 어 곤

다 그 에 주 원통 어

없 고 여 시 가 거늘 생 만

여 다

말 감격 러우 간신 에 가득 여ldquo

고 니 께 상 듣지 니 것

니다rdquo

승상 듣지 고 가

퇴 재상 공달 집에 거 고 상 지어

승지 러 께 리

( )

뒷 거리 강 승상 에게 상 리지[ ]

만 여움 사 귀 가게 다 강 승상

몸 는 연 가 헤어

리 다 산 들어간 룡사 승 만

게 다 승 만 우 다릴

과 들고 략 다 담

원 여 에게 복 고 어

공격 다 담에게 여러 가( ) 天子

복 등 여 다 단

신 리쳐 담 사 고 에게

간 후 태후 태 여 지에 고생

지 심과 강 주 여 개 다 헤

어 어 니 내 고 담 리

벼슬에 귀 누리게 다

zb21) 위 의 인 간 계를 같이 나타냈을lt gt

때 에 대한 이해 가장 적절하 ~ 않은 것은

① 계에 주 는 계 심 열

상 에 다고 다( ) 水深火熱

② 계는 견원지간 고 다( ) 犬猿之間

③ 계는 달리 막역지 계 고( )莫逆之交

④ 연결 사 컬어 재 가 고( )才子佳人

⑤ 는 생 과 볼 ( )匹夫匹婦

가 재 는 는 심 고 매사에 생( )

각 고 능 도 어 가 에게 많lsquo rsquo

도움 사 다 그는 에게 거 에

꺼리 없 거 났다고 는

매우 싫어 고 신 들

는 사 다

내가 지 리에( ) 1970

사 실에 지 월간ldquo

편집 고 어 었다rdquo

어느 없 가 쑥 다 도 어 10

후 다 산 시 럼 어 엇 어 ( ) lsquo怡山

다시 만 랴 니 그는 재 그룹 승 운rsquo

사가 고 는 고 거 누

주는 가 없는 가가 어 다시 만 게 것

었다

다 보통 것 닐러 그( ) ldquo 어낸 ( )

틀어주 가 루 러 허 에

싶어 키 틀어주 그( )

가 루 허 우간 곡 틀어 주는 루 못

는 는 고 닝께 고 지

들어 사는 고 가 다는 건 에 그 집에

rdquo

그런 단 어들 어 새벽에 떼죽 거

다 고 어 보니 죄다 허 게 집어진

는 것 었다 가 실내 꿴 뛰어 지만

없는 었다

어떻게 된 거야 한동안 넋나간 듯이 서 있던 총ldquo rdquo

가 하고많은 사람 에 하필이 유자를 겨냥하 은

말이었다 쎄유 아마 새에 고뿔이 들었던 개비네 ldquo

유rdquo

유자는 러 딴청을 하 다 야 고 가 에서 ldquo

감 가 들어 죽는 고 두 어rdquo 총 는 가 혐의

자 나 되는 것처럼 화풀이를 하 드는 것이었다( )嫌疑者

라 이 어쩌 어 유( ) ldquo rdquo ldquo rdquo

애유 이런 잔인 도 한 것들 같으니ldquo ( ) rdquo殘忍無道 helliphellip

총 는 탱천 하여 쩌 를 하 다( ) 憤氣撐天

아하니 아는 자는 다 동 하여 호통을 쳤으 하나 혈

압을 생각하여 참는 눈치 다 달리 처리헐 두 ldquo

잖은감유rdquo

총 의 성 을 덧들이 고 한 말이 아니었다 가 할

년 학 간고사 대비2013 2 현대고 대비

ECN-0102-2013-001-000076193

있는 것이 말고는 없었 때 에 게 뒷동

산을 달은 거 다

이 유자소전- lsquo rsquo

zb22) 의 상황을 속담으 표현한 것으 적절한 것은

① 루 곳 게 마 다

② 에 맞고 강에 눈 다

③ 늘 도 다

④ 도 사 다

⑤ 에 가도 신만 리 다

거리 공신 후[ ]

에 주 는 벼슬 심 늦도( )主簿

식 없어 과 께 산에 드리고 신

태몽 꾼 에 만고 웅 상 지닌 들

낳 키운다 그 후 신 들 에 역심

담 귀 등 심 여 리 귀 보내고

지 죽 는 도망 간다 그

만 고 에 에 어 니

헤어지게 다 지 가 사 들에

사 들 별 고 없 다니었다

마 마 돌 다니 걸 여 고

어 곤 다 에는 동쪽에 고 에

는 쪽에 니 가 에 리는 엽 가는

없 니 늘 다니는 었다

얼 말 죽 사 같고 림새가 말 니었

다 가슴 에 고 등 삼태

헌 에 니 달 가 도리 ( )奇男子

어 걸 었 담 만 열 도 ( ) ( )傅說 慇

고 만났고 만 갈( ) ( )武丁 伊尹

도 탕 만났( ) ( )成湯 渭水

여상 도 주 만났는 월( ) ( ) ( ) 呂尙 周 文王

같 러가 도 어느 열 살

늘과 집 삼고 사 에 쳐 거리에

어 다가 곳에 니 다 ( ) 楚

지 다가 사 보고 가에 다다( )長沙

니 망 가에는 원 리가 슬 고 가

가 내리는 사 에는 갈매 가 갈 뿐 었다

쪽 돌 보니 가 우거 고

가 사 보 었다 그곳에

가니 는 사( ) 汨羅水

는 다 주 가 쓰고 죽고

곳 었다

마 감 여 에 가 사 살펴보니

에는 삼 고 그 에( ) 屈三閭

는 만고 월 과 지 가는 그 들( )風月

가 어 었다( ) 路程記

동쪽 벽 에 새 운 어 거늘 그

보니

월 에 경 주 는 간신에게ldquo ( )敗

보고 연경 귀 가다가 에 죽 rdquo

거늘 그 보고 에 거꾸러

통곡 말

우리 연경 간 만 니ldquo ( )燕京

에 지 살 상에 엇 겠는

가 에 고 에 었 니

상에 살 것 가 도 께 지리 rdquo

고 가에 내 가니 울 리가 에 지

사 쳤는지 심 심 것 가

에는 강 주 는 재상 살고 었

니 시 에 과거에 격 여 승상 벼슬 다가 간

신 만 벼슬 그만 고 고 돌 었

다 그러 신 지 가 지 못 여 상

가 못 결 는 상 여 원 니

신 들 그 직간 꺼 다 그 에 도

담과 귀가 강 승상 가 미워 다 강 승상 마

본 에 갔다가 돌 는 에 우편 주( ) ( )本府 右便

에 다가 색 에 어리었는 청룡( ) 酒店

에 지 늘 여 통곡 고

사 는 꿈 꾸었다 마 상 게 생

각 여 새 다리다가 새벽닭 울고

가 달 갔다 가 보니 과연 어 동 가

가에 울고 는지 달 들어 그

고 사 에 어 말

는 어 어 에 어 가ldquo

닭 곳에 우느냐rdquo

니 울 그 고 답 여 말 다

는 경 동 에 사는 언 주 공 들ldquo

니다 께 간신 만 연경 귀 가

시다가 에 죽 사 에 는 닭에

도 에 죽고 니다rdquo

강 승상 말 듣고 크게 낯 변 말

것 웬 말 냐 근 동ldquo ( )老患

못 갔 니 그 사 변 여 런 변

었단 말 가 주 는 신 다 같

에 벼슬 다가 는 가 많 들어 고 돌

는 주 가 게 꿈 에 생각

겠느냐 생각지 못 다 미 지 간 지지

말고 께 가 rdquo

뒷 거리 강 승상 도움 죽 고[ ]

년 학 간고사 대비2013 2 현대고 대비

ECN-0102-2013-001-000076193

고 그 과 결 여 사 가 다 그러 강

승상 에게 울린 상 강 승상 귀 가고

과 헤어 리 승 만 게 다

승 우 다릴 과

들고 략 다 담 원

여 에게 복 고 어 (天

공격 다 담에게 여러 가 복) 子

등 여 다 단신

리쳐 담 사 고 에게 간

후 태후 태 여 지에 고생 지

심과 강 주 여 개 다 헤어

어 니 내 고 담 리 벼

슬에 귀 누리게 다

미상- lsquo ( )-劉忠烈傳

zb23) 위 과 의 서사 조를 비 한 것으 적절하lt gt

않은 것은

보lt gt

믿지 고 결 여 곱

낳 다 곱째 공주 낳 가

리게 다 리 만 고 진 공주는 lsquo rsquo

리공 미 리공 에 키워진다 월

러 과 가 죽 병에 걸 는 승에 는

어 산다고 다 여 들에게 탁

지만 거 리 는다 리 는 과

승 다 승 지 가는 에 많

만 지만 보살 도움 사 도 다

그러 승 신과 결 여 시

들어 주겠다고 다 리 는 그 결

여 들 곱 낳 후에 신

얻게 다 돌 리 는

에 과 상여 만 지만 여 과

살 낸다 훗 리 그 공 우 죽 사

승 도 는 신 다

리-lsquo rsquo-

① 복 결말에 고 다

② 웅 에 탕 고 다

③ 시 겨 내고 귀 누리는lsquo rsquo

보 리 는 월 재 신 다lt gt lsquo rsquo

④ 과 보 리 는lsquo rsquo lt gt lsquo rsquo

도움과 어 신 능 극복 고

⑤ 등 여 시 겪는lsquo rsquo

보 리 는 닌 지lt gt lsquo rsquo

림 시 겪는다

가 각 고 에( ) ( )却說

살 없었다 략 사 들 슬 에 어 lt gt

가에 내 고 가고 싶 가 고 후

워 경

사 들 별 고 없 다니었다 lt

략 얼 말 죽 사 같고 림새가 말gt

니었다 가슴 에 고 등

삼태 헌 에 니 달 가 ( )奇男子

도리어 걸 었 담 만 열 도( )傅說

고 만났고 만 갈( ) ( ) 殷 武丁

도 탕 만났( ) ( ) (伊尹 成湯 渭

여상 도 주 만났는) ( ) ( ) ( )水 呂尙 周 文王

월 같 러가 도 어느 열 살

늘과 집 삼고 사 에 쳐 거리에

어 다가 곳에 니 다 ( ) 楚

지 다가 사 보고 가에 다다( )長沙

니 망 가에는 원 리가 슬 고 가

가 내리는 사 에는 갈매 가 갈 뿐 었다

쪽 돌 보니 가 우거 고

가 사 보 었다 그곳에

가니 는 사( ) 汨羅水

는 다 주 가 쓰고 죽고

곳 었다

에는 강 주 는 재상 살고( )

었 니 시 에 과거에 격 여 승상 벼슬 다

가 간신 만 벼슬 그만 고 고 돌

었다 략 강 승상 마 본 에 갔다가 돌 lt gt ( )本府

는 에 우편 주 에 다가 색( ) ( ) 右便 酒店

에 어리었는 청룡 에 지

늘 여 통곡 고 사 는 꿈 꾸

었다 마 상 게 생각 여 새 다리다

가 새벽닭 울고 달 갔다 가

보니 과연 어 동 가 가에 울고 는지

달 들어 그 고 사 에

어 말

는 어 어 에 어 가ldquo

닭 곳에 우느냐rdquo

니 울 그 고 답 여 말 다 lt

략gt

년 학 간고사 대비2013 2 현대고 대비

ECN-0102-2013-001-000076193

생각 여 가 고 시 는ldquo ( )大人

상에 다시없는 니다 살 엇 겠습니

에 돌 가시고

가에 돌 가 니 살 마 없습니

다 략 어 없어 강 승상 가니rdquo lt gt

그곳 월계 었다

다 강 승상에게는 들 없고 다만 만( )

었다 가 낳 에 가 색

타고 내 에게 말

는 니다 미원 과ldquo ( )紫微垣

연 맺고 었는 께 강 집( )緣分

보내 에 니 게 여겨 주십시

rdquo

거늘 미 가운 낳 니 가

고 거동 단 다 시 짓 쓰 고

는 없었 니 여 가운( ) 音律

지 는 짝 룰 만 사 없었다 가 사

여 사 감 게 고 지 못 고 염 는 만다

다가 당에 거 고 식같 러

내니 고귀 상 루 말 다 어 ( )相

울 도 다 귀 사 없고 ( )富貴爵祿

웅 걸 만고 었다 승상 매우 뻐 내

당 들어가 에게 사 니 역( ) 內堂

시 매우 거워 말 다

도 마 사 는 승상께ldquo

그 게 말 시니 상 여러 말 지 말고 사

도 시다 략 시 택 여rdquo lt gt

니 다운 신 과 신 습 늘에 죄

짓고 간 상에 내 신 다

다 내고 들어가 사 살펴보니

고 것 는 다 말 어 고

는 다 어 신 에 ( )新房

에 신 과 신 가 평생 연 맺었( )緣分

니 사 주고 말 어떻게 다 헤 릴

어떻게 다 리 지낸 후에 튿 승

상 니 승상 거운 마 지 못

( ) 듯 월 러 생 열다 살

었다 에 승상 어진 사 얻고 만 에 근심

없었 다만 주 가 간신

에 죽 것 생각 마 곧 어

곤 다 그 에 주 원통

어 없 고 여 시 가 거늘 략 lt gt

략 거리

강 승상 에게 상 리지만 여움

사 귀 가게 다 강 승상 몸 는

연 과 헤어 리 다

마 각 생 강 승상 집 쪽( )

늘 보고 없 가 신 신 생각 니

없고 어 없었다 는 어떻게 도리가 없다

여 산 에 들어가 리 고 어 도 닦

고 다 그 산 보고 가

다가 곳에 다다 니 에 큰 산 었다 많

우리 골짜 가 늘 는 가운 색

에 고 갖가지 가 짝 어 ( )花草

었다 략 주 보니 lt gt ( ) (一柱門 黃

산 룡사 어 었다) lsquo rsquo 金大字

산 들어가 고승 다 그( ) ( ) 山門 高僧

거동 보니 눈 눈 듯 고

변 같 귀는 어 에 늘어 니( ) 白邊

맑고 어 골격과 신 평 니었

다 염주 에 걸고 짚고 포 ( )六環杖

삼에 어진 쓰고 생 보고 말

승 연 여 상공 시는 동 에ldquo

가 맞 지 못 니 승 십시 rdquo

생 크게 말 다

생 가 여 어 고ldquo

없 다니다가 우연 곳에 사 만 것

그 시 생 어떻게 고 습니

rdquo

승 답 여 말

어 산 승 에ldquo ( ) ( )南岳 衡山

시어 승에게 탁 내 낮 시경에 경 lsquo 12

동 에 사는 심 들 가 것 니 내쫓

지 말고 습니다 마 승rsquo

다가 상공 림새 보니 경 사 에 보

습니다rdquo

생 그 말 듣고 편 고 편( )

슬 승 들어가니 여러 승 들

가워 다 승 에 들어가

후에 그 편 니 곳 경 었다 상( ) 仙境

고 신 편 다 후 는 승과

께 병 도 탐 고 경도 게( )兵書

게 었다 게 니 지 에 가객 ( ) ( )大明天地 佳客

년 학 간고사 대비2013 2 현대고 대비

ECN-0102-2013-001-000076193

없고 산 에 리 만 본 ( ) 廣德山

신 상 사 살 는 만

우고 늘 월 신 과 늘 ( )日月聖神

산 신 들 다 니 그 재( ) 名山神靈

주 민 누가 당 겠는가 낮 공

zb24) 다 에 해당하는 내 으 적절하( ) 않은 것은

① 강 티 통 당시 능 다

② 상계 지상계 경 는 원 계 드러

③ 실에 어 없는 실 가 타 는

④ 뛰어 재주 어 가진 고

등 다

⑤ 가 직 개 여 평가 내리는

편집 평 타 다lsquo rsquo

가 본격 가 동 것 지( )

다 단 상 에2003 lsquo rsquo

들어가 드럼 연주 다 취미 생 달리

들었다는 보 우 가 들ldquo

어 틱 린 도 다 고 말 다rdquo

경 는 가 망 없( ) lsquo

티 원 고 답 다 신과 같 시각rsquo

는 습 상상 만 도 감동

다 시각 연주 동시에

열 상 는

티 원 그런 열 경 럽다는 것 다

다 역시 엄청 다 본( )

에 복 들

고쳐 가고 다 신 에 얼

마 지는 고 리가 는 지도 생님

가 훈 고 많 고쳐 다

고 말 다

그러 직도 에 지 는 다 그는

체격 지 못 게 가 큰 만

체 운동 훈 과 께 체 늘 동 50

는 게 고 말 다

에게는 꿈 다 통 누 가( )

주겠다는 것 그 꿈 다 신 극복 는

과 에 큰 경험 들도 느 게

주고 싶다는 것 다

마 슬 마다( ) ldquo 통

낼 었 것 럼 고통 는 사 들

고 겠다 고rdquo

말 다 달 루 첫 낸 lsquo rsquo

첫 드 심 집에 는 리듬 드 2

루 에 도 보고 싶다 집 에는 직(RampB) 3 4

사 곡 도 보 고 싶다고 포 다middot

zb25) 에서 가장 유사한 의 를 닌 어를lt gt

찾아 쓰

lt gt

나는 이제 너에게도 픔을 주겠다

사랑 다 소 한 픔을 주겠다

겨 거리에서 개 놓고

살아 추위 떨고 있는 할 니에게

값을 으 서 뻐하던 너를 위하여

나는 픔의 평등한 얼 을 여 주겠다

내가 어둠 속에서 너를 를 때

단 한 도 평등하게 어주 않은

가마니에 덮인 동사자가

다 얼어 죽을 때

가마니 한 장조차 덮어주 않은

한 너의 사랑을 위해

흘릴 르는 너의 눈 을 위해

나는 너에게 이제 너에게도 다림을 주겠다

지 울 포동 여고 생들17

틈 없 가득 체 에 맑 울

다 죽 듣 생들 사 에

연 는 탄 다 객들 도 는lsquo rsquo

가 보 주 공 맹 가 운 는

단 그룹사운드 루 보컬 맡고 는lsquo rsquo

시각 지 었다17 1

근 다만과 가 거lsquo rsquo lsquo

꿈 고 퇴 내가 다rsquo

간 간에 지 지 연 생들 짧lsquo rsquo lsquo rsquo

가 운 듯 리에 어

연 다 내 사 고 퇴lsquo rsquo

과 루 들 결 다시 돌lsquo rsquo

들 고 사 들 에 당당

것 니다 내 태어

볼 없었 크고 열여

년 학 간고사 대비2013 2 현대고 대비

ECN-0102-2013-001-000076193

에도 고 시 얻지 못 다

감지 없는 시각 상태 다

신 지에 고 상 원망 도

단다 어느 가 에 시각 에 ldquo

어 그런 듣고 다 보니 내가 게 lsquo

살 는지 도 눈 고 싶rsquo lsquohelliphellip

보 는 생각만 들 고 그 가 들에게rsquo

도 내고 들도 고 많 었죠 들 rdquo

었 지 새 는 에 쑥 러운 색

어났다

생에 것 단연 었다lsquo rsquo

공연에 거 꿈lsquo rsquo

는 다 특 가사 갑게 는 운 lsquo

벽 에 당당 마주 어 언 가 그 벽

고 늘 어 거운 상도

없죠 내 삶 에 웃 그 께

는 다고 다rsquo

들었 그냥 런 도 고만 여ldquo lsquo rsquo

겼죠 그런 꾸 가사 미 새 다 보

니 통 는 가사 는 생각 들 고 (

가 게는 시각 는 생각 들고 들) ( )

마다 듣고 큰 얻었어 rdquo

에 진지 게 가에 미 가

zb26) 의 에 들어갈 말 적절한 것은lt gt ~

lt gt

난 난 꿈이 있었죠

고 찢겨 남 하여도

내 가 히 과 같이 간 했던 꿈

혹 때 누 가가 뜻 를 비 음

내 등 뒤에 흘릴 때도

난 참아야 했죠 참을 있었죠

날을 위해

늘 걱정하듯 말하죠

헛된 꿈은 독이라고

세상은 끝이 정해 책처럼

이 돌이킬 없는

현 이라고 helliphellip

래 난 난 꿈이 있어

꿈을 믿어

나를 켜

저 차갑게 서 있는 이란 앞에

당당히 마주칠 있어

출처 가 거위의 꿈 작사 이적 작곡 동률- lsquo rsquo ( )

① ② ③ ④ ⑤

가 떴다는 들 만 지만( ) lsquo rsquo

늘 겸 다 에 주 연 우승 지 간에도 3

단 생님께 만 지 고 고 만ldquo rdquo

큼 늘 겸 신 계 가

고 다

에게는 꿈 다 통 누 가

주겠다는 것 그 꿈 다 신 극복 는 과

에 큰 경험 들도 느 게 주

고 싶다는 것 다

슬 마다 통 낼ldquo

었 것 럼 고통 는 사 들

고 겠다 고rdquo

말 다 달 루 첫 낸lsquo rsquo

첫 드 심 집에 는 리듬 2

루 에 도 보고 싶다(RampB) 집 에는 직34

사 곡 도 보 고 싶다고 포 다

미 는( ) (26) 어 헤헤헤 웃다가 어ldquo rdquo

허허허 웃었다ldquo rdquo ldquo rdquo 같 도 고

상 다 는 같 도 다( ) 壯丁 킹 들lsquo

다 는 역도 보 그 다 지만 그는rsquo

뷰에 지 다 운동만 지 ldquo

것 지 간에 여러 사 도 역rdquo helliphellip

었다 그런 엇 그 마 움직 는지 보 쯤

지 담 사 다 훈 없어 그는 티

지 림 었다 태 다 갔다 는 습

마 집 럼 편 게 보 다

주말에는 주 엇 보내

주말에도 별 주 에 청ldquo

고 에 가고 도 쳐

에 듣고 보 에 갈 가 별 없

어 산 시 게 고 들어 2002

거 매 여 지냅니다 시 과 지훈 rdquo

다 근 간 과 진실 그리고 싶어( )

가 다 근에게 그것 진리 다 거 다 없

거 고 다 없 는 것 진리

다 근 진리는 후 쪽 었다 신산( )辛酸 삶

었 질곡( )桎梏 역사 에 지냈 가

눈에 든 것 료 단 료 게 보

것 었다 그것 그 에 겨우겨우

슬 슬 생 어가는 간들 었다

리 과 단 리 고리에 검 마

없 거리 돌

상 것 없는 등 근에게 상

과 진실 엄 ( )儼存 다는 사실 리는 가

실 고 가 과 역경 에 도 근 내 포

없었 후 보루( )堡壘 다 도 365

도 간 근 여

시 것 다

년 학 간고사 대비2013 2 현대고 대비

ECN-0102-2013-001-000076193

다 공주 그림 가 근 경- ( ) ldquo rdquo(

2009)

zb27) 작가의 주 적인 각이 드러난 것은~

① ② ③ ④ ⑤

가 신 지에 고 상 원망( )

도 단다 어느 가 에 시각 에 ldquo

어 그런 듣고 다 보니 내가 lsquo

게 살 는지 도 눈 고 싶rsquo lsquohelliphellip

보 는 생각만 들 고 그 가 들에게rsquo

도 내고 들도 고 많 었죠 들었rdquo

지 새 는 에 쑥쓰러운 색

어났다 략 [ ]

경 는 가 망 없 티lsquo

원 고 답 다 신과 같 시각rsquo

는 습 상상 만 도 감동

다 시각 연주 동시에

열 상 는 티

원 그런 열 경 럽다는 것 다 략 [ ]

슬 마다 통 낼ldquo

었 것 럼 고통 는 사 들

고 겠다 고rdquo

말 다 달 루 첫 낸 lsquo rsquo

첫 드 심 집에 는 리듬 2

루 에 도 보고 싶다 집 에는 직(RampB) 3 4

사 곡 도 보 고 싶다고 포 다

식 누 가-

고 싶어

다 역도 미 담 고 사( )

질 주말에는 주 엇 보내[ 1]

답 주말에도 별 주 에[ ] ldquo

청 고 에 가고 도 쳐

에 듣고 보 에 갈 가 별

없어 rdquo

질 계 고 슬슬 도 는 것 닙니[ 2]

답 다 들 눈 에 보 고 뿐 보[ ] ldquo

다 열심 고 어 상에 도 들지만 상

지키는 것 들다고 에 도달

그것 지키 훨 많 rdquo

질 들 살 고 리 는[ 3]

거운 들 체 리느 는다

답 가 고 게 체 어[ ] ldquo ( ) 級

느 도 계가 니 살 는 것도 고역 지만

살 우는 것 들어 는 살

체 리 고 어도 어도 실 갔다

쑥 어 rdquo

질 거리에 슷 연 여 들[ 4]

보는 간 상 지

답 상 다 체 게 리지 못[ ] ldquo

거 주변에 는 그 거 누 보지

못 고 뻐지고 싶 에 체 리는 에

타 워 지만 는 어울 는 것보다 는

시간 운동만 는 건 니에 사복 lsquo rsquo

고 사복 는 말에 들 웃지만 늘 운동복

고 지내니 사러 갈 도 어 rdquo

질 역도가 말 단 식 운동 니[ 5]

답 가 내는 만 클 업 보[ ] ldquo

그러니 만 쓰는 식 운동 니다

만 다고 거운 것 들 는 건 니거든 연

도 고 가지 동 에 도 여러 가지

복 들

보식 역도 여 미-

zb28) 가 에 대한 설 으( ) 않은 것은

① 시각 우 지 시 에 지

고 망 가는 태도 달 고 다

② 언어 과 언어 복 사 여

담 내 생각 게 는 가

③ 직 감 그 마 것

럼 생생 게 느껴지는 과 주고 간 내

없 리 어 억 게 다

④ 담 내 식 리 여 담 삶 습

과 가 시 여 독 에게 감동과 훈 다

⑤ 직 진 담 직 누

지 못 는 독 에게 생생 상 달 주고

담 욱 게 다

zb29) 나 의 각 의 의도를 설 한 것으 적절하( ) 않

년 학 간고사 대비2013 2 현대고 대비

ECN-0102-2013-001-000076193

은 것은

① 질 담 상 보여 주 것 다1

② 질 담 과 그에 삶 태도 보여2

주 것 다

③ 질 역도 겪는 어 움에 역도3

과 것 다

④ 질 같 연 여 갖는 고민 는지 말4

주 는 것 다

⑤ 질 역도가 과 고 운동 는 것5

담 가 말 주 는 것 다

가 만진 것 다( ) 3

감 달 다고 다 억 에( ) 音感

지워 지만 당시 청 탁 리도

다고 다 드럼 웠다 4

에 갈 마다 드럼 는 리가 신 게 들

다고 다 눈 볼 가 없 니 엔ldquo

는 는 님 틱 에 여 주

다 드럼과 연 맺 과 들 주었다rdquo

식 누 가-

고 싶어

역( ) 도가 말 단 식 운동 니

가 내는 만 클 업에 보ldquo

그러니 만 쓰는 식 운동 니다 만

다고 거운 것 들 는 건 니거든 연

도 고 가지 동 에 도 여러 가지 복

들 시 는 상 상

드는 상 에 맞춰 실 에 는 여러

펼쳐집니다rdquo

략( )

늘 에 는 어 만 것 같

가 에 사 고 사 사ldquo

겠어 든 에 가 경 만 고

울 는 사 겠어 rdquo

보식 역도 여 미-

다 가 운 는 어 어( ) ldquo rdquohelliphellip

월 새벽 시 태 없 거웠고1965 5 6 1

는 없 그 병원에 퇴원 집

가는 마지막 마 고 마 내 거 다

가 죽 간신 에 실 다 사는 어느5 lsquo

가 죽 는 말 가 식 다 신rsquo

상에 각 시키는( )刻印 에 실

어느 가는 후 민 가가 근 었다lsquo rsquo

는 간 과 진실 그 다는( ) ldquo

에 단 평 견 가지고 다 내

가 그리는 간상 단 고 다 지 다 는 그들

가 에 는 평 지 니 그리고 어린

들 미지 겨 그린다rdquo

마 근 간 과 진실 그리고 싶어( )

가 다 근에게 그것 진리 다 거 다 없

거 고 다 없 는 것 진리

다 근 진리는 후 쪽 었다 신산(辛酸 삶)

었 질곡(桎梏 역사 에 지냈)

가 눈에 든 것 료 단 료 게

보 것 었다 그것 그 에 겨우겨우

슬 슬 생 어가는 간들 었

다 리 과 단 리 고리에 검

마 없 거리 돌

상 것 없는 등 근에게 상에

과 진실 엄 다는 사실 리는 가( )儼存

실 고 가 과 역경 에 도 근 내

포 없었 후 보루(堡壘 다 도)

도 간 근365

여 시 것 다

월 강원도 림리에( ) 1914 2 21

삼 독 태어났다 어 근 복

그것 그리 가지 못 다 근 곱 살

지는 산 산업에 실 고 답마 에 내

갔다 근 그림 럼 쫓 다니 가 시 것

다 상 진 것도 가 었다

러 가 에도 고 근 가 꿈꾸었다 근

가 꿈꾸게 것 보통 업

원색도1926 만lsquo rsquo 었다

공주 그림 가 근 경-

zb30) 에 대한 설 가장 른 것은~

① 역도가 과 운동 도 질

② 리는 는 다 lsquo rsquo

③ 들었지만 그럭 럭 는 다 lsquo rsquo

④ 가 게 보 시 말 다

⑤ 보 병 는 지 상 lsquo rsquo

는 말 다

년 학 간고사 대비2013 2 현대고 대비

ECN-0102-2013-001-000076193

시간 많지 다 청량리 생 병원

마지막 상 경 릿 게 들어 다 그 는 십

만 큰 가 상 말 다

지 못 들 마 갈 고 돗

도시민들 싹 싹 탔다 가 시

월에 병원에 원 가 폐 진 몸도4 ( )疲弊

갈 미 지 못 고 었다 가는 얼마( ) 解渴

지 생 에 생각 가

마감 는 신 평생 십 만에

가 과 많 닮 다고 생각 지는

가 운 는 어 어ldquo rdquo 1965helliphellip

월 새벽 시 태 없 거웠고 는5 6 1

없 그 병원에 퇴원 집 가

는 마지막 마 고 마 내 거 다 가

죽 간신 에 실 다 사는 어느 가5 lsquo

죽 는 말 가 식 다 신rsquo

상에 각 시키는 에 실 어느( ) lsquo刻印

가는 후 민 가가 근 었다rsquo

ldquo 는 간 과 진실 그 다는 에

단 평 견 가지고 다 내가 그

리는 간상 단 고 다 지 다 는 그들 가

에 는 평 지 니 그리고 어린 들

미지 겨 그린다rdquo

근 간 과 진실 그리고 싶어 가

다 근에게 그것 진리 다 거 다 없 거

고 다 없 는 것 진리다

근 진리는 후 쪽 었다 신산 삶 ( )辛酸

었 질곡 역사 에 지냈 가 눈에( )桎梏

든 것 료 단 료 게 보 것

었다 그것 그 에 겨우겨우 슬

슬 생 어가는 간들 었다 리

과 단 리 고리에 검 마

없 거리 돌 상

것 없는 등 근에게 상에 과 진실

엄 다는 사실 리는 가 실( )儼存

고 가 과 역경 에 도 근 내 포 없었

후 보루 다 도 도( ) 365堡壘

간 근 여 시 것

간에 지닌 가 근 1914 2

월 강원도 림리에 삼 독21

태어났다 어 근 복 그것 그리

가지 못 다 근 곱 살 지는 산

사업에 실 고 답마 에 내 갔다 근

그림 럼 쫓 다니 가 시 것 다 상

진 것도 가 었다 러 가 에도

고 근 가 꿈꾸었다 근 가 꿈꾸게

것 보통 업 원색1926

도 만 었다lsquo rsquo

그림 가 근 경 공주- ldquo rdquo ( 2009)

zb31) 다음 이 같은 의 성 소에 해당하 않은

것은

사건 평① ② ③

④ 주 ⑤ 경

가 운 는 어 어ldquo rdquo 1965helliphellip

월 새벽 시 태 없 거웠고 는5 6 1

없 그 병원에 퇴원 집 가

는 마지막 마 고 마 내 거 다 가

죽 간신 에 실 다 사는 어느 가5 lsquo

죽 는 말 가 식 다 신rsquo

상에 각 시키는 에 실 어느( ) lsquo刻印

가는 후 민 가가 근 었다rsquo

는 간 과 진실 그 다는 에ldquo

단 평 견 가지고 다 내가 그

리는 간상 단 고 다 지 다 는 그들 가

에 는 평 지 니 그리고 어린 들

미지 겨 그린다rdquo

근 간 과 진실 그리고 싶어 가

다 근에게 그것 진리 다 거 다 없 거

고 다 없 는 것 진리다

근 진리는 후 쪽 었다 신산 삶 ( )辛酸

었 질곡 역사 에 지냈 가 눈에( )桎梏

든 것 료 단 료 게 보 것

었다 그것 그 에 겨우겨우 슬

슬 생 어가는 간들 었다 리

과 단 리 고리에 검 마

없 거리 돌 상

것 없는 등 근에게 상에 과 진실

엄 다는 사실 리는 가 실( )儼存

고 가 과 역경 에 도 근 내 포 없었

후 보루 다 도 도( ) 365堡壘

간 근 여 시 것

간에 지닌 가 근 1914 2

월 강원도 림리에 삼 독21

태어났다 어 근 복 그것 그리

가지 못 다 근 곱 살 지는 산

사업에 실 고 답마 에 내 갔다 근

그림 럼 쫓 다니 가 시 것 다 상

진 것도 가 었다 러 가 에도

고 근 가 꿈꾸었다 근 가 꿈꾸게

것 보통 업 원색1926

도 만 었다lsquo rsquo

공주 그림 가 근 경- ldquo rdquo ( 2009)

년 학 간고사 대비2013 2 현대고 대비

ECN-0102-2013-001-000076193

zb32) 위 을 작성하는 과정에서 되어 활 된 자

어 것은

신 사 료① 연보②

고③ ④ 들과 담

⑤ 에 평

는 간 과 진실 그 다는 에ldquo

단 평 견 가지고 다 내가 그

리는 간상 단 고 다 지 다 는 그들 가

에 는 평 지 니 그리고 어린 들

미지 겨 그린다rdquo

근 간 과 진실 그리고 싶어 가

다 근에게 그것 진리 다 거 다 없 거

고 다 없 는 것 진리다

근 진리는 후 쪽 었다 신산 삶 ( )辛酸

었 질곡 역사 에 지냈 가( )桎梏

눈에 든 것 료 단 료 게 보

것 었다 그것 그 에 겨우겨우

슬 슬 생 어가는 간들 었다

리 과 단 리 고리에 검 마

없 거리 돌 상

것 없는 등 근에게 상에 과

진실 엄 다는 사실 리는 가 실( )儼存

고 가 과 역경 에 도 근 내 포

없었 후 보루 다 도 도( ) 365堡壘

간 근 여 시

것 다

간에 지닌 가 근 1914 2

월 강원도 림리에 삼 독21

태어났다 어 근 복 그것 그리

가지 못 다 근 곱 살 지는 산

사업에 실 고 답마 에 내 갔다 근

그림 럼 쫓 다니 가 시 것 다 상

진 것도 가 었다 러 가 에도

고 근 가 꿈꾸었다 근 가 꿈꾸게

것 보통 업 원색1926

도 만 었다lsquo rsquo

질 루 마 가 도 린다 경건

움 느껴지는 경 다 훗 근 그림에

과 는 거 것( )裸木

만 간과 연 엮어 가는 경건 움lsquo rsquo

니었

같 가가 고 싶었 근에게 그 꿈에 다

가가는 지 다 다 가 지망생들 규 미

상 에 진 고

에 지만 근 다 다 근

미 에 운 것 보통 시 미 시간

다 그런 그에게 없는 연습 가가

통 다 가 귀 시 지 도

얻는 뛸 듯 뻤지만 마 도 가 에

듯 는 었 에 어린 근 주 에

에 그림 그리고 지우고 복( )粉板

시간 가는 게 루 보냈다

근 그 갈 가가 것 열여( )渴求

었 다가 미1932 lsquo rsquo ( lsquo

미 에 다 다는 고 마rsquo) lsquo rsquo

가 근 집 고도 지는 시골 경

그린 그림 다 후 근 에 1943 22

지 미 에 그림 고

에 걸쳐 다 미 근 가

동 는 었다

공주 그림 가 근 경- ldquo rdquo ( 2009)

zb33) 위 의 내 과 일치하는 것은

가 근 가 꿈 포 다①

근 당 가들과 께 에 다②

살 근 가 걷20③

게 었다

④ 만 통 근 역경 겨내는lsquo rsquo

느 다

⑤ 근 간 과 진실 그리 에 그 에

드러 는 간상 단 다

계 시 주 근 건강

걸었다 신 과 간에 상 다 건강

신 는 눈에도 다 근 쪽 눈 뿌 게

보 지 과에 다 다 시 지지 고 결

내 었다 시 지만 마 막막

다 늦어 결 근 쪽 눈 고 말 다

쪽 눈 근에게는 쪽 눈 었고

계 었다 그 근 는 여 그lsquo rsquo

다 근 에 같 그림 그 었다1950

시 그림 는 여 쪽lsquo rsquo

고 어 마주 고 는 그림1963

여 과 동 다 마 복

그린 듯 눈 내리 새 게 다 지

사 다 근 게 복 것

복 상과 타 는 근 상

가 떳떳 단 었고 근 그리고

간 과 진실 에 다가가 가 근다

운 었다 근 신에게 당당 지 그리고

그 다 근 그림에 단 복 보다

년 학 간고사 대비2013 2 현대고 대비

ECN-0102-2013-001-000076193

태 도 그리고 극 보다 과

얻 여 었다 과 통

근 그리고 는 재 고 에 질

만들고 특 것 다

공주 그림 가 근 경- ldquo rdquo( 2009)

zb34) 의 이유에 대해 추 한 것으 적절하 않은 것

상과 타 시도①

보다 과 얻②

근 신에게 당당 지③

④ 간 과 진실 에 다가

⑤ 태 도 얻

근 가가 었지만 그 다니 가

럼 어지지 다 복과 쟁 거쳐 시

는 가 근에게 생계 사 에

운 사 다 에 키에 건( ) 178cm死鬪

체 근 에 동 역 업( )荷役

가 생계 다 쟁

에는 동에 운 상우 주 미

죄 사 에 그림 그리는 시 다 그곳에

에 동 역 업 것에

결 것 럼 보 다 지만 그런 것만도

니었다 그림 그리는 고는 지만 매 근

는 극 간 과 별 없는 경 리 그림

벽에 그리는 것 었다 우도 리 없었다 근

트 는 우 그림 그 다 생

계 그림 단 것 다

후 근 지 신 계 리에 미

엑 리 겼다 근 곳에

건 사 크 에 미 들 ( )

상 상 그 다 근 갖 다 겪

냈다 그리고 결 그 돈

신동에 어 사리 집 마 다 마 ㄷ

루 심 쪽에는 과 엌 쪽에는 건

었다 건 주고 근 가 에

여 살 다 심 에는 지 집어

쓰고 지만 곳 근 가 에게 러웠

보 리 다 근 과 마루 업실 삼 그림

그 다 신동 마루는 근 그림에 등 는 lsquo rsquo

같 상들 지 다 시 고

에 들 폐허가

가 업실 었다

공주 그림 가 근 경- ldquo rdquo( 2009)

zb35) 위 에 대한 설 으 적절한 것은

업 시 여 훈과 감동 다①

에 주 평 드러 다②

사 사 등 식 과 ③

④ 다 근거 시 여 삶에

⑤ 살 시 사 경 께 여

습 시 다

가 시간 많지 다 청량리 생 병원( )

마지막 상 경 릿 게 들어 다 그 는

십 만 큰 가 상 말 다

지 못 들 마 갈 고 돗

도시민들 싹 싹 탔다 가 시

월에 병원에 원4 가 폐( )疲弊

진 몸도 갈 미 지 못 고 었다( )解渴 가는

얼마 지 생 에 생각

가 마감 는 신 평생 십 만에

가 과 많 닮 다고 생각 지는

가 운 는 어 어( ) ldquo rdquohelliphellip

월 새벽 시1965 5 6 1 태 없 거웠고

는 없 그 병원에 퇴원 집

가는 마지막 마 고 마 내 거 다

가 죽 간신 에 실 다 사는 어느5 lsquo

가 죽 는 말 가 식 다 신rsquo

상에 각 시키는 에 실( )刻印

어느 가는 후 민 가가 근 었다lsquo rsquo

다 는 간 과 진실 그 다는( ) ldquo

에 단 평 견 가지고 다 내

가 그리는 간상 단 고 다 지 다 는 가

에 는 평 지 니 그리고 어린 들

미지 겨 그린다rdquo

근 간 과 진실 그리고 싶어( )

가 다 근에게 그것 진리 다 거 다 없

년 학 간고사 대비2013 2 현대고 대비

ECN-0102-2013-001-000076193

거 고 다 없 는 것 진리

다 근 진리는 후 쪽 었다 신산( )辛酸 삶

었 질곡 역사 에 지냈( )桎梏

가 눈에 든 것 료 단 료 게 보

것 었다 그것 그 에 겨우겨우

슬 슬 생 어가는 간들 었다

리 과 단 리 고리에 검

마 없 거리 돌

상 것 없는 등 근에게 상에

과 진실 엄 다는 사실 리는 가 실( )儼存

고 가 과 역경 에 도 근 내 포

없었 후 보루 다( ) 堡壘 도 365

도 간 근 여

시 것 다

마 같 가가 고 싶었 근에게 그 꿈( )

에 다가가는 지 다 다 가 지망생들

규 미 상 에 진 고

에 지만 근 다 다 근

미 에 운 것 보통 시 미 시간

다 그런 그에게 없는 연습 가가

통 다 가 귀 시 지 도

얻는 뛸 듯 뻤지만 마 도 (

는 었 에 어린 근 주 에)

에 그림 그리고 지우고( )粉板

복 시간 가는 게 루 보냈다

zb36) 전 의 성 소가 아닌 것을 고르

① 평 ② 사건 ③ 경

④ ⑤ 훈

늘 지 상에 살고 는 사 들 억 도가10

고 그리 지 통 고 는 사 들( )知的

그보다 훨 많 억 도는 고 지 20

통 다 그런 지 고 2500

그리 간 보는 과 사 에

매우 달 뿐만 니 과 에 도 극

루고 었다 미 운 그런 들

살고 는 동 과 사 들 사고 식에

큰 가 다는 다

고 그리 들 우주 개별 고 독립

사 들 생각 지만 고 들 우

주 연 질 간주 다 같( ) 看做

각 도 들에게는 연 질

었지만 그리 들에게는 미 들 결 었

다 고 과 그리 들 사 같

는 동 과 사 에 도 견 다

지심리 미 마 드 겐트 는

살 들에 에 지 다

연 동 과 상 다 과 같 실험

다 크 만든 미드 도 보

여 주고 그 상 닥 고 주었다lsquo (Dax)rsquo

실 닥 는 재 지 는 것 실험 가lsquo rsquo

만들어 낸 다 그런 다 개 다 체 보

여 주었는 는 미드 지만 틱

만들었고 다 는 재료는 크 지만

달 다 그러고 어 것 닥 지 사 들에게 고 lsquo rsquo

게 니 들 주 같 고 는

체 택 고 동 들 같 재료 만들어진 체

택 다 러 는 심지어 살짜리

들에게 도 타났다 것 곧 과 동

다 상 보고 다는 것 미 다

개별 사 보고 고 동 연 질 보

고 는 것 다

동 들 주변 상 에 맞 어 동 고

에 다 사 들 태도 동에 보다 많

주 울 다 동 가 미시간 에

에 경험 다 그는 미식

경 보러 가게 었는 경 체는 매우 재미 었

주변 들 동에 질 다 그 는

들 계 어 상태 경 다

어 들 에 에 그 시 가 계 가

진 것 다 상 살펴 는 말 들 lsquo rsquo

에 그는 에 시 어 도 뒷사

생각 곧 다시 곤 것 다 그런 그에게 뒷

사 고 지 는 들 동 럼

어 웠다

생각 지도 리 드 니 벳-

zb37) 다음 위 의 내 전개 으 만 인lt gt

것은

lt gt

대조의 통해 대상이 닌 특성을 설 하고 있다

일화를 제 하여 자 의 주장을 뒷 침하고 있다

유추의 을 사 하여 독자의 의해를 돕고 있다

대상이 형성되는 과정을 간적 서에 따라 서 하고 있

① ②

③ ④

년 학 간고사 대비2013 2 현대고 대비

ECN-0102-2013-001-000076193

가 우리가 말 고 쓰는 든 단어가 사 에 는( )

것 니다 사 격에 가 는 지만

어 사 과 같 특별 는 사 니lsquo rsquo

단어 격 보 단어가 사 에

등재 어 다 리 리 사 는 단어 도 그

것 시 사 는 어 고 사 에

격 보 것 니다

러 얼 은 사전에 를 있는가 이에 대한 답lsquo rsquo

은 얼 이 유행어인가 아닌가에 따라 갈라 다 이 단어lsquo rsquo

는 년 어 자 에 랐고 쓰이고 있으2002 lsquo rsquo

유행어라고 하 에는 생 이 다 런데 계속

을 유 하 서 사전에 등재될 자격을 획득할 것인가 이

에 대한 답을 내리 는 히 어 다

여 서 가 를 고 해 볼 있다 첫 는 이 단어

를 써야 할 필 가 속적으 있는가 하는 점이다

상주의 열풍에 휩 인 사회 위 에 편 해서 퍼 말

이 얼 인데 과연 런 위 가 속될 것인가 이에lsquo rsquo

대해 필자의 생각은 정적이다 사회 위 가 뀌

런 말을 쓸 일이 없어 것이다

다음은 단어의 성이다 단어의 성이 사회적으 거

감이 없으 계속 사 될 가능성이 높다 런 에서

얼 은 좋은 조건이 아니다 익히 알 졌듯이 이lsquo rsquo

말은 얼 과 청소년층에서 속어 사 하는 이 결합lsquo rsquo lsquo rsquo

된 말이다 얼 에서 얼 을 리하는 조어 도 lsquo rsquo lsquo -rsquo

어에서는 매 낯선 이다 이것만으 도 거 감을 갖

는 사람들이 있다 더 나 속어 결합한 말이다 얼 lsquo rsquo

이 널리 퍼졌다 해도 은 여전히 청소년층의 속어lsquo rsquo

남아 있다 속어는 자연 럽게 아 자리에서나 쓰 에는

담 러 말이다 러한 담을 하고 사

역을 넓혀 가는 속어도 없 는 않다 특히 얼 은 lsquo rsquo

에도 종종 등장한다 만큼 거 감이 많이 희석되었다

고 할 있다 러나 일상의 자연 러 대화에서도 거

리낌 없이 등장하는가 게 는 되 않았다고 생

각한다

얼 이 유사어인 쌈 등을 만들어 내고lsquo rsquo lsquo rsquo

있으니 살아남을 있을 것이라고 는 견해도 있을 것

이다 러나 간이 나 서 유사어를 포함하여 든

말이 사라 사 는 많다 유사어가 많다는 것이 생 을

유 할 있는 절대적인 조건은 아니다

나 언젠가 터 사람들은 어느 단에서 얼 이 가장( )

쁜 사람을 가리켜 얼 이라고 르고 있다 이 얼lsquo rsquo lsquo rsquo

이라는 단어가 최근 어사전에 라 항간에 논란이 일고

있다 아닌 게 아니라 얼 은 유행어처럼 인다 생 lsquo rsquo

도 리 래되 않은 것 같고 언제 사라 도 알

없다 게다가 젊은이들 사이에서 주 쓰일 뿐이다 이런

단어를 사전에 는다는 게 하 이 없어 이 도

한다

러나 속단은 이다 차근차근 따져 볼 일이다

선 얼 이 일 적 유행어인 아닌 주의 게 들여다lsquo rsquo

볼 필 가 있다 유행어란 유행에 따라 빠르게 유포되었

다가 단 간 내에 소 되는 단어나 를 가리킨다

얼 은 인터넷을 통해 속히 퍼 말이다 하 만 일lsquo rsquo

적인 유행어처럼 단 간 내에 사라 않았을 뿐 아니라

현재 도 잦은 빈도 사 되고 있고 앞으 도 상당

간 사 될 것으 측된다 한 언 재단의 뉴 검 lsquo rsquo

색 사이트에 따르 얼 은 년 에 처음 나타난lsquo rsquo 2001

이후 꾸 히 사 되고 있다

이 같은 사 빈도는 얼 이 일 적 유행어 는 현lsquo rsquo

저히 다르다는 것을 여 다 장 간의 생존 만으 도

얼 은 이 한 어의 어휘 에 를 자격을 얻었다lsquo rsquo

고 할 있다 더 이 이라는 비 적 정제된 매체에

높은 빈도 쓰이고 있 않은가 사 빈도 측 에서

필통이나 연필과 같은 단어 대등하거나 더 많이 쓰lsquo rsquo lsquo rsquo

다는 것은 결코 가 게 볼 일이 아니다

이제는 사전이 언어 현 을 빠르게 하는 게 덕인

대가 되었다 세계적으 유 한 의 사전들도 경쟁

적으 어를 고 있다

하 만 얼 은 젊은이들이나 쓰는 속어라고 흠을 잡을lsquo rsquo

도 르겠다 얼 이 주 젊은 층에서 많이 쓰 lsquo rsquo

는 속어임에 틀림없다 러나 어사전에 표 적이고 품

위 있는 말만 어야 한다고 생각한다 것은 커다란

해다 당장 아 어사전이나 펼쳐 라 속어는

설과 같은 비어나 죄자들이 쓰는 은어 어

마니 같은 소 의 사람만이 쓰는 말 도 라 있

않은가 사전은 말 치에 일정 빈도 이상 나타나는 말이

라 말이든 다 할 있다

zb38) 가 나 에 대한 다음의 설( ) ( ) 않은 것은

① 가 는 얼짱 사 에 등재 것에( ) ( ) lsquo rsquo

보 고 다

② 사 등재 가는 단어 격에( )

고 고 는 언 들 언어 사 도에 고 다 ( )

③ 가 얼짱 어지만 신 과 같 매( ) ( ) lsquo rsquo

체에 도 사 는 말 는 고 다

④ 가는 얼짱 어 보고 크게 가지 근( ) lsquo rsquo 3

거 들어 뒷 고 다

⑤ 는 얼짱 어 는 다 특 다는( ) lsquo rsquo

근거 에도 크게 가지 근거 가 들어 주 2

뒷 고 다

가 늘 지 상에 살고 는 사 들 억( ) 10

도가 고 그리 지 통 고 는 사 들

그보다 훨 많 억 도는 고 지 20

통 다 그런 지 고 2500

년 학 간고사 대비2013 2 현대고 대비

ECN-0102-2013-001-000076193

그리 간 보는 과 사 에

매우 달 뿐만 니 과 에 도 극

루고 었다 미 운 그런 들

살고 는 동 과 사 들 사고 식에

큰 가 다는 다

고 그리 들 우주 개별 고 독립

사 들 생각 지만 고 들 우

주 연 질 간주 다 같 각

도 들에게는 연 질 었지

만 그리 들에게는 미 들 결 었다

고 과 그리 들 사 같 는

동 과 사 에 도 견 다

인 리학자인 츠 이마이 디드 겐트너는 두

살이 채 안 된 아이들에서 터 성인에 이르 다양한

연 대의 동양인과 서양인을 대상으 다음과 같은 험

을 했다 저 코르크 만든 피라 드 양의 도형을

여 주고 대상의 이름을 닥 라고 알 주었다lsquo (Dax)rsquo

제 닥 는 존재하 않는 것으 험자가 임의lsquo rsquo

만들어 낸 이름이다 런 다음 두 개의 다른 체를

여 주었는데 하나는 피라 드 양이 만 하얀 플라 틱

으 만들었고 다른 하나는 재 는 코르크 만 양이

달랐다 러고 나서 어떤 것이 닥 인 사람들에게 고 lsquo rsquo

르게 했더니 서양인들은 주 같은 양을 하고 있는

체를 선택했고 동양인들은 같은 재 만들어 체를

선택했다 이러한 차이는 성인은 어 두 살 리

아이들에게서도 나타났다 이것은 곧 서양인과 동양인은

서 다른 세상을 고 있다는 것을 의 한다 략 ( )

는 아주 단 하 서도 인상적인 험을 했다

험에는 동서양의 대학생들이 참여했다 는 험 참가자

들에게 컴퓨터 화 을 통해 속 장 을 담은 애니 이션

을 여 주었다 화 의 앙에는 초점의 역할을 하는 커

다란 고 한 마리가 있었고 주위에는 다른 생

들과 초 자갈 거품 등이 함 제 되었다 화 을

두 씩 후 참가자들은 자 이 것을 회상해 라는

를 았다

결과 서양인 대학생들과 동양인 대학생 두 앙

의 초점 역할을 했던 고 를 동일한 정도 언 했으

나 경 소 위 거품 초 다른 생 들 에 ( )

대해서는 동양인 대학생들이 서양인 대학생들 다 60

이상 더 많이 언 했다 뿐만 아니라 동양인 학생들은 서

양인 학생들에 비해 개 적인 고 다 전체적인 계

를 더 언 하는 경향을 다 략 또한 경의 일 ( )

를 화 킨 림을 제 하 을 때 동양인 대학생들은 대

경의 화를 알아챘 만 서양인 대학생들은 경

의 화를 거의 알아차리 했다 략 ( )

따라서 서양인들만을 대상으 연 한 화lsquo

편성 결 은 잘 된 것일 도 있다 각 과정과 인rsquo

과정의 어떤 이 화 편적이고 어떤 이

화에 따라 달라 는 는 앞으 많은 연 를 통하여 논의

되어야 한다

나 어떤 의 에서 리 두는 이 화적이다 리( )

안에는 다른 사람들과 더 친 한 계를 유 하 는 상호

의존성과 다른 사람들 터 독립적인 존재 살아가 는

독립성이 혼재한다 따라서 이 에서 어떤 특성이 더 강

하게 각되는 상황에 놓이느냐에 따라 서 다른 화적

특 을 일 있다 결 리 두는 어떤 경 에는

동양인처럼 행동하고 어떤 경 에는 서양인처럼 행동하는

것이다

zb39) 가 에 대한 다음의 설( ) 않은 것은

① 는 신 주 뒷 닥 실험과lsquo rsquo lsquo

니 실험 근거 시 다rsquo

② 동 들 상 간 공통 보다는 에 식

는 강 다

③ 들 주변 맥 에는 심 경 어 사건

과 사건 사 계에 상 민감 다

④ 는 동 과 틀린 지 고 는 것lsquo rsquo

니 다 고 다 lsquo rsquo

⑤ 가에 우리 사 들 개 시 가 원( )

집 경 말 고 는 것 개 보다는

에 고 는 것에 다

늘 지 상에 살고 는 사 들 억 도가10

고 그리 지 통 고 는 사 들( )知的

그보다 훨 많 억 도는 고 지 20

통 다 그런 지 고 2500

그리 간 보는 과 사 에

매우 달 뿐만 니 과 에 도 극

루고 었다 미 운 그런 들

살고 는 동 과 사 들 사고 식에

큰 가 다는 다

지심리 미 마 드 겐트 는 동

과 상 다 과 같 실험 다

크 만든 미드 도 보여 주고 그

상 닥 고 주었다 그런 다lsquo (Dax)rsquo

개 다 체 보여 주었는 는 미드

지만 틱 만들었고 다 는 재료는

크 지만 달 다 그러고 어 것 닥 lsquo

지 사 들에게 고 게 니 들 주 같rsquo

고 는 체 택 고 동 들 같

재료 만들어진 체 택 다 러 는

심지어 살짜리 들에게 도 타났다 것

곧 과 동 다 상 보고 다는

것 미 다 개별 사 보고 고 동

년 학 간고사 대비2013 2 현대고 대비

ECN-0102-2013-001-000076193

연 질 보고 는 것 다

동 들 주변 상 에 맞 어 동 고

에 다 사 들 태도 동에 보다

많 주 울 다 동 가 미시간

에 에 경험 다 그는 미

식 경 보러 가게 었는 경 체는 매우 재

미 었 주변 들 동에 질 다 그

는 들 계 어 상태 경

다 어 들 에 에 그 시 가 계

가 진 것 다 뒷사 고 지 는 들

동 럼 어 웠다

그는 경험에 어 얻어 동 들lsquo

각도 상 본다 는 가 우고rsquo

검 여 주 단 도 상 실험 실

시 다 그는 실험 가 들에게 컴퓨 통

담 니 보여 주었다

에는 역 는 커다 고 마리가 었

고 주 에는 다 생 들과 갈 거 등

께 시 었다 본 후 가 들

신 본 것 상 보 는 지시 다

그 결과 생들과 동 생

역 고 동 도 언

경 거 다 생 들에 ( )

는 동 생들 생들보다 60

상 많 언 다 뿐만 니 동 생들

생들에 개별 고 보다 체 계

언 는 경 보 다 경 변 시

킨 그림 시 동 생들 경

변 지만 생들 경 변

거 리지 못 다

지 지 들만 상 연 lsquo

보편 결 못 것 도 다 지각 과 과rsquo

지 과 어 보편 고 어

에 달 지는지는 많 연 통 여

어 다

리 드 니 벳 생각 지도 사- ldquo rdquo( 2004)

zb40) 위 에 대한 설 으 가장 적절한 것은

① 동 과 생 식 강 고 다

② 가지 실험 통 쓴 고 다

③ 닥 실험에 사 본질에 동 사

상에 주 다

④ 니 실험에 동 과 에 지

각 도에 가 다

⑤ 쓴 는 보편 연 에 드러 우월 에

에 근 고 다

가 동 들 주변 상 에 맞 어 동 고( )

에 다 사 들 태도 동에 보다 많

주 울 다 동 가 미시간 에

에 경험 다 그는 미식

경 보러 가게 었는 경 체는 매우 재미 었

주변 들 동에 질 다 그 는

들 계 어 상태 경 다

어 들 에 에 그 시 가 계 가

진 것 다 상 살펴lsquo 는 말 들rsquo

에 그는 에 시 어 도 뒷사

생각 곧 다시 곤 것 다 그런 그에게

뒷사 고 지 는 들 동 럼

어 웠다

그는 경험에 어 얻어( ) 동 들lsquo

각도 상 본다 는 가 우고rsquo

검 여 주 단 도 상 실험

실시 다 실험에는 동 생들 여 다

그는 실험 가 들에게 컴퓨 통

담 니 보여 주었다 에는

역 는 커다 고 마리가 었고 주 에는

다 생 들과 갈 거 등 께 시

었다 본 후 가 들 신 본 것

상 보 는 지시 다

다 그 결과 생들과 동 생( )

역 고 동 도 언

경 거 다 생 들 에 ( )

는 동 생들 생들보다 60

상 많 언 다 뿐만 니 동 생들

생들에 개별 고 보다 체 계

언 는 경 보 다 들어 동

생들 상 체 연못 럼 보 어ldquo 같rdquo

체 맥 언 시 었지만

생들 상 어 같 큰 고 가 쪽 움ldquo

직 어 같 역 고rdquo

언 시 다 경 변 시킨 그

림 시 동 생들 경 변

지만 생들 경 변 거

리지 못 다

년 학 간고사 대비2013 2 현대고 대비

ECN-0102-2013-001-000076193

게 볼 동 들 보다는 큰 그( )

림 보 에 사 과 체 맥 연결시 지각

는 경 고 체에 특 떼어 내

어 독립 보는 것 낯 어 다 에

들 사 에 고 주변 맥 에는 심 경

에 사건과 사건 사 계에 상

민감 편 다

마 지 지( ) 들만 상 연

보편 결 못 것 도 다lsquo rsquo 지각 과

과 지 과 어 보편 고 어

에 달 지는지는 많 연 통 여

어 다

리 드 니 벳 생각 지도 사- ldquo rdquo( 2004)

zb41) 의 하는 가~ 다른 것은

① ② ③

④ ⑤

얼마 그 에 동 사고 식과

사고 식 보여 주는 내 다

들 에 는 탕 고 같 게

어 겨 고 미 에 는 그 크 럼 큰 고

어리 주고 원 는 어 도 는

상 고 생각 다는 것 다 러

는 어떻게 생 것 고 과 그리 거슬

러 가 보 그 단 다

고 연 경 체 경 생 에

다 벼 사는 공동 업과 경험 많 연 역

에 고 들 연 웃과

게 지내 고 탁 연 들

들 지 연 럽게 들 다 민들

웃과 동 게 뿐만 니 는 집 과

게 다

동 시 는 생태 경 에 살 결과

들 다 사 들 사 상 에 주

울 게 었고 는 곧 체 상 과 간 사

계 시 는 낳게 었다 신 가

가 는 체에 는 원 는 동시

에 다 사 들 그 사 포 체 맥 에

다 들 간 사 연

계 체 계에 주 울 는 사고 체계

게 었다

그러 그리 연 경 그 었다 산

지 연결 는 지 건 그리고 역

에 다 런 들 업에 다 사 과

동 므 공동체에

다고 다 고 그리 들

들과는 달리 보 내 감 지 들과

지 크게 느 지 못 다 그

견 다 경우 주 쟁 통 결 는 갖

게 었다

신 사 간 계들 루어진 커다

트워크 에 게 당연 사 역시 연

계들 체 식 게 다 어 상

원 도 그 개체가 체 맥 과

계 에 고 다 게 체 맥 에 주

울 다 보 상 복 과 가변 식 게 고

상에 재 는 많 변 들 사 에 재 는 들도

게 다 들 주 태도 보

는 경우가 많다 쟁 결

통 결 보다는 통 결

는 보 다

그러 고 그리 들 개개 사 사 독

에 주 울 다 사 사 체에

어 그들 사 에 재 는 공통 규 주

고 다 상 원 에도 사

체 내 주 고 다 그들

체 여 탕 체

는 주 태도 시 고 특 사 어

주에 는지 여 그 주에 는 규

견 다 에 는 쟁 식 리

같 리 사고 체계가 달 게 었다

리 드 니 벳 생각 지도 사- ldquo rdquo( 2004)

zb42) 위 에서 사 된 설 과 가장 유사한 것은

① 크톱 컴퓨 는 본체 니 마우 루

어 다

② 곡과 시 리 는 지 과 사 루어 다는 공통

지니고 다

③ 경 고 것과는 달리

경 본 연 태 그 주변 경

④ 벽돌 능 에 사계 내내

습도가 지 다

⑤ 잰느 체 체 지닌 재 체가 없

는 재 눌 다

년 학 간고사 대비2013 2 현대고 대비

ECN-0102-2013-001-000076193

zb43) 는 립 앙 도서 이 정의 일 이다lt gt

도서 장과 이 자의 리 의 정의 연결이

적절하 않은 것은

lt gt

제 조 서 유8 ( )

도서 장은 다른 이 자의 안전을 위협하거나 도서 의①

서를 란하게 할 가 있는 자에 대하여는 도서 출입

을 제한할 있다

도서 장은 이 자가 제 조 각 호의 어느 하나의 행위를 하7②

을 때에는 이 을 하게 하거나 도서 출입을 제한할

있다

제 조자 의 대출9 ( )

도서 자 는 다음 각 호의 경 대출할 있다①

상호대차도서 간에 자 를 류하는 것을 말한다 등 다1 ( )

른 도서 과의 협 을 위하여 필 한 경

공 이 공 행 상 필 하는 경2

에 도서 장이 필 하다고 인정하는 경3

대출이 가능한 도서 자 의 위는 도서 장이 정하는②

에 따른다

제 조 상10 ( )

이 자가 도서 자 설을 더럽히거나 찢거나 뜨①

쓰게 하거나 잃어 린 경 에는 상하여야 한다

도서 장은 제 항에 따른 상 을 정하여 게 하여야1②

한다

제 조이 절차 등11 ( )

이 칙에서 정한 것 에 도서 자 설의 이 절차

이 제한 등에 필 한 사항은 도서 장이 정한다

출처 립 앙 도서- (httpwwwnlgokr)

① 는 도 리 다8

② 도 는 리 다9 1

③ 료 지 는 도 리 다9 2

④ 도 료 변상에 리10 1

⑤ 는 에 도 리 다11

3

도 다 각 같다①

공 공 다만 연1

연 간 다

매월 째 째 월2

도 도 리 그 사3

가 다고 는

도 에 미리 게1 3②

시 여 다

4

도 시간 도 여 게시 다

5

도 료 시 는 는 도①

지에 등 후

등 에 사 도②

7

는 다 각 여 는 니 다

도 료 시 상 리1 lsquo rsquo

도 료 시 훼 는2 middot

지 가 닌 곳에 식 거 담3

우는

도 보 등 보 검색열4 middot

그 에 도 질 지 여 도5

여 게시 사 는

8

도 다 거 도①

질 게 우 가 는 에 여는 도

도 가 각 어느7②

에는 지 게 거 도

9

도 료는 다 각 경우 다①

상 도 간에 료 는 것 말1 (

다 등 다 도 과 여 경우)

공 원 공 상 는 경우2

그 에 도 다고 는 경우3

가능 도 료 는 도②

는 에 다

10

년 학 간고사 대비2013 2 현대고 대비

ECN-0102-2013-001-000076193

가 도 료 시 럽 거 거①

못 쓰게 거 어 린 경우에는 변상 여

도 에 변상 여 게시1②

여 다

zb44) 위 에서 도서 장이 게 해야 할 사항에 해당하는

것을 두 쓰

년 학 간고사 대비2013 2 현대고 대비

ECN-0102-2013-001-000076193

립 도 규

1 ( )

규 립 도 립 어린 청 도(

포 다 료 시 열 시 말) (

다 에 사 규 립 도)

편 진 다

2 ( )

규 립 도 도 다 에( lsquo rsquo )

고 는 도 에 도lsquo rsquo 2 2

료 에 여 다 다만 특 료 귀

료 등 료 에 사 립 도

도 다 다( lsquo rsquo )

3 ( )

도 다 각 같다①

공 공 다만 연1

연 간 다

매월 째 째 월2

도 도 리 그 사3

가 다고 는

도 에 미리 게1 3②

시 여 다

시간4 ( )

도 시간 도 여 게시 다

등 등5 ( )

도 료 시 는 는 도①

지에 등 후

등 에 사 도②

사 료6 ( )

도 료 시 에 사 료는 도

7 ( )

는 다 각 여 는 니 다

도 료 시 상 리1 lsquo rsquo

도 료 시 훼 는2 middot

지 가 닌 곳에 식 거 담3

우는

도 보 등 보 검색열4 middot

그 에 도 질 지 여 도5

여 게시 사 는

질 지8 ( )

도 다 거 도①

질 게 우 가 는 에 여는 도

도 가 각 어느7②

에는 지 게 거 도

료9 ( )

도 료는 다 각 경우 다①

상 도 간에 료 는 것 말1 (

다 등 다 도 과 여 경우)

공 원 공 상 는 경우2

그 에 도 다고 는 경우3

가능 도 료 는 도②

는 에 다

변상10 ( )

가 도 료 시 럽 거 거①

못 쓰게 거 어 린 경우에는 변상 여

도 에 변상 여 게시1②

여 다

등 규 에 것 에 도11 ( )

료 시 등에 사

도 다

립 도- (httpwwwnlgokr)

zb45) 도서 장의 리 있는 조항으 적절하 않

은 것은

① ② ③ ④ ⑤

년 학 간고사 대비2013 2 현대고 대비

ECN-0102-2013-001-000076193

1 ( )

사가 공 는lsquo rsquo

과 여 사 원과 리

사 타 사 규

니다

개 보 보7 ( )

사는 보통신망 등 계 는 에lsquo rsquo lsquo rsquo

원 개 보 보 니다 개lsquo rsquo

보 보 사 에 는 사 개lsquo rsquo

보 취 니다 다만 사는 다 lsquo rsquo

사 계 통 공 는 경우 원 lsquo rsquo

등 개 보 당 사에 습니lsquo rsquo

원 리에8 (lsquo rsquo lsquo rsquo lsquo rsquo

)

원 에 리lsquo rsquo lsquo rsquo lsquo rsquo①

원에게 가 도 여 는lsquo rsquo 3

니다

사는 원 가 개 보 우 가lsquo rsquo lsquo rsquo lsquo rsquo②

거 사 경우 는 미 에 어 거 lsquo

사 사 운 우 가 는 경우 당rsquo lsquo rsquo

습니다lsquo rsquo

원 가 도 거lsquo rsquo lsquo rsquo lsquo rsquo 3③

가 사 고 지 경우에는 시 사에lsquo rsquo

통지 고 사 내에 니다lsquo rsquo

경우에 당 원 사에 그 사실3 lsquo rsquo lsquo rsquo④

통지 지 거 통지 도 사 내에 지 lsquo rsquo

생 경우 사는 지지 습니다lsquo rsquo

사10 (lsquo rsquo )

사는 과 지 미lsquo rsquo①

에 는 지 계 고

공 여 다 여 니다lsquo rsquo

사는 원 게lsquo rsquo lsquo rsquo lsquo rsquo②

도 개 보 신 보 포 보 보 시( )

갖 어 개 보 취 공시 고

니다

사는 과 여 원lsquo rsquo lsquo rsquo③

견 만 당 다고 경우에는

리 여 니다 원 견 만 사 lsquo rsquo

에 는 게시 거 우편 등 통 여

원에게 리 과 결과 달 니다lsquo rsquo

원11 (lsquo rsquo )

원 다 여 는 니다lsquo rsquo ①

신청 는 변경 시 허 내 등1

타 보 도2

사가 게시 보 변경3 lsquo rsquo

사가 보 보 컴퓨 그4 lsquo rsquo (

등 등 신 는 게시)

사 타 등 지 재산 에5 lsquo rsquo 3

사 타 상 거 업6 lsquo rsquo 3

는 폭 시지 상 타 공7 middot middot

에 는 보 에 공개 는 게시 는lsquo rsquo

사 동 없 리 사8 lsquo rsquo

타 거 당9

게시15 (lsquo rsquo )

원 내에 게시 는 게시 게재 는lsquo rsquo lsquo rsquo lsquo rsquo

경우 원 사가 게시 복 lsquo rsquo lsquo rsquo lsquo rsquo middot middot

등 태 언 등에 공 는

것 내에 다 원 본 게시 등 lsquo rsquo lsquo rsquo

크 능 등 여 복 는 등 태

는 것 동 것 니다

- (wwwnavercom)

zb46) 위 은 인터넷 포털사이트의 회 가입을 위한 이

약 의 일 이다 이 약 을 만드는 과정에서 생각한

내 으 적절하 않은 것은

개 보 보 가 지에 별 눠①

겠어

원 가 만들게 에②

시 주어 겠어

원들 게재 게시 다 원 크 다③

는 것 지

④ 원 지 는 뿐만 니 사가 지 는

도 께 달 지

리에 가 생 경우 사가⑤

에 다는 도 듯

1 ( )

사가 공 는lsquo rsquo

과 여 사 원과 리

사 타 사 규

년 학 간고사 대비2013 2 현대고 대비

ECN-0102-2013-001-000076193

니다

개 보 보7 ( )

사는 보통신망 등 계 는 에lsquo rsquo lsquo rsquo

원 개 보 보 니다 개lsquo rsquo

보 보 사 에 는 사 개lsquo rsquo

보 취 니다 다만 사는 다 lsquo rsquo

사 계 통 공 는 경우 원 lsquo rsquo

등 개 보 당 사에 습니lsquo rsquo

원 리에8 (lsquo rsquo lsquo rsquo lsquo rsquo

)

원 에 리lsquo rsquo lsquo rsquo lsquo rsquo①

원에게 가 도 여 는lsquo rsquo 3

니다

사는 원 가 개 보 우 가lsquo rsquo lsquo rsquo lsquo rsquo②

거 사 경우 는 미 에 어 거 lsquo

사 사 운 우 가 는 경우 당rsquo lsquo rsquo

습니다lsquo rsquo

원 가 도 거lsquo rsquo lsquo rsquo lsquo rsquo 3③

가 사 고 지 경우에는 시 사에lsquo rsquo

통지 고 사 내에 니다lsquo rsquo

경우에 당 원 사에 그 사실3 lsquo rsquo lsquo rsquo④

통지 지 거 통지 도 사 내에 지 lsquo rsquo

생 경우 사는 지지 습니다lsquo rsquo

원에 통지9 (lsquo rsquo )

사는 특 다 원에게 통지 경우lsquo rsquo lsquo rsquo

공지 게시 통 상 게시 개별 통지에7

갈 습니다

사10 (lsquo rsquo )

사는 과 지 미lsquo rsquo①

에 는 지 계 고

공 여 다 여 니다lsquo rsquo

사는 원 게lsquo rsquo lsquo rsquo lsquo rsquo②

도 개 보 신 보 포 보 보 시( )

갖 어 개 보 취 공시 고

니다

사는 과 여 원lsquo rsquo lsquo rsquo③

견 만 당 다고 경우에는

리 여 니다 원 견 만 사 lsquo rsquo

에 는 게시 거 우편 등 통 여

원에게 리 과 결과 달 니다lsquo rsquo

원11 (lsquo rsquo )

원 다 여 는 니다lsquo rsquo ①

신청 는 변경 시 허 내 등1

타 보 도2

사가 게시 보 변경3 lsquo rsquo

사가 보 보 컴퓨 그4 lsquo rsquo (

등 등 신 는 게시)

사 타 등 지 재산 에5 lsquo rsquo 3

사 타 상 거 업6 lsquo rsquo 3

는 폭 시지 상 타 공7 middot middot

에 는 보 에 공개 는 게시 는lsquo rsquo

사 동 없 리 사8 lsquo rsquo

타 거 당9

원 계 규 내lsquo rsquo lsquo②

여 공지 주 사 사가 통지 는rsquo lsquo rsquo

사 등 여 타 사 업 에 lsquo rsquo

는 여 는 니다

- (wwwnavercom)

zb47) 위 약 의 조항에서 같은 제점을 하lt gt

고 있는 조항은

lt gt

제휴 회사에 회 의 아이디 개인 정 를 전송할 있도

한 조항은 고객에게 당한 조항이다

1 7 8① ② ③

④ 9 ⑤ 10

립 도 규

1 ( )

규 립 도 립 어린 청 도(

포 다 료 시 열 시 말) (

다 에 사 규 립 도)

편 진 다

2 ( )

규 립 도 도 다 에( lsquo rsquo )

고 는 도 에 도lsquo rsquo 2 2

료 에 여 다 다만 특 료 귀

료 등 료 에 사 립 도

도 다 다( lsquo rsquo )

3 ( )

도 다 각 같다①

공 공 다만 연1

연 간 다

년 학 간고사 대비2013 2 현대고 대비

ECN-0102-2013-001-000076193

매월 째 째 월2

도 도 리 그 사3

가 다고 는

도 에 미리 게1 3②

시 여 다

시간4 ( )

도 시간 도 여 게시 다

등 등5 ( )

도 료 시 는 는 도①

지에 등 후

등 에 사 도②

사 료6 ( )

도 료 시 에 사 료는 도

7 ( )

는 다 각 여 는 니 다

도 료 시 상 리1 lsquo rsquo

도 료 시 훼 는2 middot

지 가 닌 곳에 식 거 담3

우는

도 보 등 보 검색열4 middot

그 에 도 질 지 여 도5

여 게시 사 는

질 지8 ( )

도 다 거 도①

질 게 우 가 는 에 여는 도

도 가 각 어느7②

에는 지 게 거 도

료9 ( )

도 료는 다 각 경우 다①

상 도 간에 료 는 것 말1 (

다 등 다 도 과 여 경우)

공 원 공 상 는 경우2

그 에 도 다고 는 경우3

가능 도 료 는 도②

는 에 다

변상10 ( )

가 도 료 시 럽 거 거①

못 쓰게 거 어 린 경우에는 변상 여

도 에 변상 여 게시1②

여 다

등 규 에 것 에 도11 ( )

료 시 등에 사

도 다

립 도- (httpwwwnlgokr)

zb48) 다음 정 리 의 의 으 볼 때 가장

이 적인 것은

도 시간 도 여 게시 다①

등 에 사 도②

가능 도 료 는 도 는③

에 다

④ 도 에 변상 여 게10 1

시 여 다

⑤ 도 가 각 어느7

에는 지 거 도

zb49) 를 참고하여 이 어의 성격을 설 한lt gt

것으 적절하 않은 것은

① 보 에 는 어 시 상 고 어 시lt gt lsquo rsquo

에 보여주고 다

② 진 어 어원에 견 고 다

에는 타 어 들어가는 것 다 lsquo rsquo

③ 에 들어갈 말 각각 고 어 어 신 어~

들 언어는 질 격 강 통 없었다

④ 시 우리 에 가 었지만 지 계

과 달리 들 통 사 달 어 웠

년 학 간고사 대비2013 2 현대고 대비

ECN-0102-2013-001-000076193

⑤ 크 몽골 만주 공통어가 우리 어 같

계열에 다는 에 사 특 짐

가( )

善化公主主隱 공주님

他密只嫁良置古 몰 결 고

薯童房乙 맛

夜矣卯乙抱遣去如 에 몰 고 가다

( )

始汝 會隱日恚見隱扐 만 에 본

恥隱汝衣淸隱笑 맑 웃

고 시 여 공 크다 만 다[ ] ( ) ( ) ( ) ( )始 汝 會扐

내다 에 보다 견( ) ( )恚 見 다( )隱

럽다 맑다 청 웃( ) ( ) ( ) ( )恥 衣 淸 笑

zb50) 위의 나 를 함 고 음에 답하( ) lt gt

보lt gt

( )素那或云金川 白城郡蛇山人也

운 사산

는 고 다 는( )[ ( ) ] (素那 金川 白城

사산 사 다) ( ) 郡 蛇山

삼 사- lsquo rsquo 47

에 제 된 단어 의 표 리를 조건(1) lt gt ( ) lt gt

에 맞게 서 하

건lt gt

lsquo 었고 었다 태rsquo

에 제 된 단어 동일한 표 리에(2) lt gt ( )

의해 적은 것을 나 에서 찾아 조건 에 맞게 서 하( ) lt gt

건lt gt

에 당 는 각각( ) 개 쓸 것2 단

당 는 가 여러 개 어도 개만 쓸 것 각2

개 과 도 쪽에 개만2 2

드시 지 것( )

과 동 원리 것lsquo 고

과 동 원리 것 다rsquo

태 것

가( )

素那(或云金川) 白城郡蛇山人也

소나 또는 천 이라 한다 는 성 사( ) ( ) ( )素那 金川 白城郡〔 〕

산 사람이다 현대어 풀이( ) ( )蛇山

나( )

紫布岩乎希 회

執音乎手母牛放敎遣 자 손 암쇼 노히 고

吾 不喩慙 伊賜等肹 肹 나 안디 리샤

花 折叱肹 可獻乎理音如 고 것거 도림다

다 향찰은 리말을 리 으 적은 표 이었 만 생( )

은 고 대를 넘 하고 끊어 고 말았다 랜 세

동안 갈고 닦아 체계적이었던 향찰 표 이 사라졌

을 인은 크게 두 가 나누어 생각해 볼 있다

하나는 족 사회의 한 선호도에서 찾을 있다 라 때

향찰은 주 족 계 에서 사 했을 것으 인다 한 을

알 하고서는 한자를 활 하여 리말을 리 으 표

하 란 가능하 때 이다 런데 족들은 간이 흐

를 향찰과 같은 리 표 을 익혀 사 하 다는

아 한 을 대 사 하는 쪽을 선호하게 되었다 더 이

고 초에 인재 등 을 위해 과거제도가 행되 서 한 선

호도가 더 높아졌고 결 향찰은 소 되고 말았다

또 다른 가능성은 한 어의 특성에서 찾을 있다

터 한 과 일 세 나라는 한자 화 에 속해 다

당연한 이야 겠 만 표의 자인 한자는 어를 표 하

에 매 적절하다 어의 음절은 성 ( ) ( )聲母 韻母

이 어 고 여 에 성조가 추가되어 최종 소리가 결정된

다 래서 어는 단음절을 하나의 한자 표 하 된

다 에 초성 성 종성의 세 가 소가 하나의 음절

년 학 간고사 대비2013 2 현대고 대비

ECN-0102-2013-001-000076193

을 이 는 한 어는 음절 조가 잡하고 음절의 가 많아

서 한자 차 만으 한 어의 소리를 만족 럽게 표 할

없었다 를 들어 한 어에서는 어 니 같이 음절 lsquo rsquo

이 어 단어가 얼마든 있으나 어는( ) 複數音節

자 하나 나타내 만이다lsquo [m ]rsquo 母 ǔ

한편 일 어의 표 은 핵 적 단어는 한자 적고 토는

가나라는 일 의 자 적는 이다 적인 의 를 나

타내는 은 표의 자인 한자 적고 적 계를 나

타내는 토는 표음 자 적는 셈이니 자세히 살펴

리의 향찰 표 을 쏙 빼닮았음을 알 있다 한 어 같

은 착어이 서도 일 어에만 향찰과 유사한 표 이 살아

남은 것은 일 어의 특 때 이다 일 어는 하나의 자음과

음의 결합으 음절을 이 고 침이 거의 없는 음절 언어

이다 이러한 음절의 특색에다가 토가 달한 착어라는 점

이 향찰과 유사한 표 이 살아남을 있는 비결이었다

하 만 같은 착어라도 다양한 음소 침이 달한 한

어는 향찰 표 하는 데 근 적으 한계가 있었다

zb51) 다 하여 의 행에 대한 탐 한 결과( ) lt gt 2

않은 것은

보lt gt

善花公主主隱 공주니믄 공주님( )

----------------------------------------

-

他密只嫁良置古 그 지 얼어 고 몰 결(

----------------------------------------

-

薯童房乙 맛 맛( )

夜矣卯乙抱遺去如 몰 고 가다 에 몰 고(

가다)

주동 역 동- (薯童謠『 』

에 2 ( )他密只嫁良置古

얼다 시집가다 결 다 말 lsquo rsquo

① 실질 미 지니고 므 타 타lsquo ( )rsquo lsquo [ ]

② 에 실질 미 타내고 지 는lsquo rsquo lsquo [ ]rsquo lsquo [ ]密只 密 只

계 타내는

③ 얼어는 실질 미 포 고 므 가lsquo rsquo lsquo [ ]rsquo嫁

것lsquo [ ]rsquo 良

④ 고 어간 는 실질 미 지니고 므lsquo rsquo lsquo -rsquo

것lsquo [ ]rsquo 置

⑤ 고 어미 고는 계 타내고 므lsquo rsquo lsquo- rsquo

고 것lsquo [ ]rsquo 古

가( )

엉 훈 민middot middot middot middot middot世 宗 御 製 訓 民 正 音

말 미 듕 귁에 달middot middot middot middot middot middot middot middot中 國 文 字

니 런middot middot middot middot middot middot 어린middot middot middot middot百 姓

니 고 도 내 들middot middot middot middot middot middot middot middot middot 시러middot

펴 몯middot 미middot middot 니 내middot middot middot middot middot middot middot middot 爲

어엿middot 겨 새middot middot middot 믈여듧middot middot middot middot字 니middot middot middot

사 마다 니겨 킈 middot middot middot middot middot middot middot middot middot便 安

고 미니middot middot middot middot

본 는 상( ) (象

원리에 만들어진 본) ( )形 ㄱ ㄴ ㅁ ㅅ ㅇ

에 는 가 원리에( )加劃

그리고( )ㅋ ㄷ ㅌ ㅂ ㅍ ㅈ ㅊ ㆆ ㅎ

쓰는 병 원리에 만들어진( )竝書

마지막 체( ) ( )異體ㄲ ㄸ ㅃ ㅆ ㅉ ㆅ

ᅀ 다 상 원리에 ㅇ ㄹ

지 는 삼재 상 본 본( ) ( ) ( 天地人 三才

탕 므림과 림에 ) (初ㅡ ㅣ

재)( ) ( )( )出字 再出字ㅗ ㅏ ㅜ ㅓ ㅛ ㅑ ㅜ ㅕ

병 그리고 들 에 다시( )ㅘ ㅝ ㅣ

( )ㅣ ㅢ ㅚ ㅐ ㅟ ㅔ ㆉ ㅒ ㆌ ㅖ ㅙ ㅞ

zb52) 가 에 대한 설 으 르 않은 것을( ) 두 고르

① 어쓰 규 지키고 다

② 리 고 다

③ 말 미 미 등 어 사 다lsquo rsquo

④ 개 지 다

년 학 간고사 대비2013 2 현대고 대비

ECN-0102-2013-001-000076193

⑤ 어 원 에 가 도 고 다

엉 훈 민世 宗 御 製 訓 民 正 音

말 미 듕귁에 달 니

런 어린 니 고 도middot

내 들 시러 펴 몯 미 니middot

내 어엿 겨 새 믈여듧

사 마다 니겨middot 킈 고

미니

훈민 언 본- lsquo rsquo 5 (1459 )

zb53) 위의 에 대한 현대어 풀이가 르~ 않은 것

① 우리 말 과 달

② 어리 말 고 는 것 어도

③ 신 생각 마 껏 펼 는 사 많다

④ 게 생각 여

⑤ 사 마다 게

zb54) 훈민정음 언해 에는 한 을 창제한 동 가 드러나

있다 훈민정음 창제의 정 과 내 이 잘 연결된 것

① 주 신 말 미 듕귁에 달

② 민 신 내 어 겨

③ 신 뻔 킈 고 미니

④ 실 신 사 마다 니겨

⑤ 귀 신 계 주 는 훈민 신과 거리가

가 엉 훈 민( ) middot middot middot middot middot世 宗 御 製 訓 民 正 音 

말 미 귁에 中 國 달 文 字

니 런 어린 니 百 姓

고 도 내 들 시러 펴 몯

미 니 내 어엿 爲 겨 새

믈여듧 니 사 마다 니 字

겨 킈 고 미니 便 安

훈민 언 본- lsquo ( )rsquo ( ) 5 (1459 )訓民正音 世祖

( )

[ 1 ]

동 룡 샤 마다 복( ) ( ) ( )海東 六龍 天福

시니 고 동( ) ( )古聖 同符 시니

[ 2 ]

매 니 곶 여

미 므 니 그 내 러

가 니

[ 125 ]

우 미리( )千世 샨( )定 에( )漢水北 累仁

누 개 샤 복 업 시니( ) ( ) 開國 卜年

신( )聖神 니 샤도 경 근민 샤 욱( )敬天勤民

드시리 다

님 쇼 산 가( ) ( )洛水 山行

미드니 가

어 가- lsquo ( )rsquo 27龍飛御天歌

다 우리신 니쓰고 다만 만 쓰( )

거 샹 귀쳔 다보게 러 귀

여 쓴 도 신 보 가 고 신 에

말 어 보게 각 에 사 들

고 본 몬 능통 후에

죠 죠 니

드 도 만 공 에 사

드 미 죠 고 고 여 보 죠

보다 얼마가 거시 어신고 니 첫

가 죠 니 죠

민 들 어 신 샹

귀쳔 도보고 어보 가 만 늘

고 폐 에 만쓴 죠 민

도 러보지못 고 보니 그게 엇지

심 니 리 보 가 어 운건 다

니 쳣 말마 지 니 고 그

쓰 에 가 우 지 지

몰 거 본후에 가 어 지

고 그니 쓴편지 쟝 보

년 학 간고사 대비2013 2 현대고 대비

ECN-0102-2013-001-000076193

쓴것보다 듸 보고 그 마 니 쓴 고

어 못

그런고 에 리 과 가

만 쓴 못 민 말만 듯고

고 편 그 못 보니 그사 단

병신 못 다고 그사 식 사

니 만 고 다 과 그사

만 고 다 과 업 사 보다 식 고

죠 도 고 각 과

견 고 실 직 귀쳔 간에 그

고도 다 것 몰 귀죡 보다

사 우리 신 귀쳔 다 업

시 신 보고 과 지 게 랴

시니 샹 귀쳔 간에 우리 신 걸

간 보 새지각과 새 걸 미리

독립신- lsquo (1896)rsquo

zb55) 친 어 나의 제 장( ) 2 매 함축적

의 가 가장 유사한 것은

① 지 눈 내리고 매 득 니 내 여 가

사- lsquo rsquo

② 도 어 리듯 그 게 어 다

주 사- lsquo rsquo

③ 눈 살 다 죽 어 린 과 체 여

눈 새벽 지 도 살 다

눈- lsquo rsquo

④ 삶 근심과 고단 에 돌 거니는 여 거 는

여 리 내린 살가지 에 눈 리 눈 리

택 그 생 에- lsquo rsquo

⑤ 늘 러 고 러

청룡 룡 어 개 루 우

신경림 계- lsquo rsquo

zb56) 친 를 위 가 나 에 나타난A B ( ) ( )

세 어의 특 에 의거하여 세 어 표 하

그 산 고 공 도 맑지만

A

주변에 쓰 리는 어리 사 많다

B

건lt gt

식 가 에 타 어 특징에( ) ( )

거 과 어쓰 는 고 지 말 것

A

B

zb57) 가 의( ) 달 아ㆍ 다 의 ( ) 나셔에서 알 있는

세 어 개화 어의 특 을 비 하여 조건 에lt gt

맞게 서 하

건lt gt

어에 는lsquo 개

어에 는 다 태rsquo

zb58) 은 가 는 다 에 나 는 절lt 1gt ( ) lt 2gt ( )

일 를 췌한 것이다 의 의 가 lt 1gt (1)~(2)

유사한 말을 에서 찾아 쓰lt 2gt

보lt 1gt

런 (1) 어린 니 고百 姓

도 내 들 시러 펴 몯 미

사 마다 (2) 니겨 便 安

킈 고 미니

보lt 2gt

죠 고 고 여 보 죠

보다 얼마가 거시 어신고 니 첫 가

죠 니 죠 민

들 어 신 샹 귀쳔

도보고 어보 가 만 늘 고

폐 에 만쓴 죠 민 도

러보지못 고 보니 그게 엇지 심

니 리

년 학 간고사 대비2013 2 현대고 대비

ECN-0102-2013-001-000076193

lt 1 gt

동 룡 샤 마다 복 시( ) ( ) ( )海東 六龍 天福

고 동 시니( ) ( )古聖 同符

lt 2 gt

(A) 매 니 곶

여 니

미 므 니 그 내

러 가 니

lt125 gt

우 미리 샨 에( ) ( ) ( ) 千世 定 漢水北 累

누 개 샤 복 업 시 니( ) ( ) 仁開國 卜年 聖

신( ) 神 니 샤도 경 근민 샤( ) 敬天勤民

욱 드 시 리 다

님 쇼 산 가 ( ) ( )洛水 山行

미드니 가

- lt gt龍飛御天歌

zb59) 장과 내 상 유사한 성격의 조는125

① 뫼 고 고 고 고

어 그린 많고 많고 고 고

어 러 는 울고 울고 가느니

도 견- lt gt

② 강 에 드니 몸 다

그믈 고 가니

뒷 뫼 엄 언 니( )藥

-

③ 말 없는 청산 태 없는 다

값 없는 청 없는 월

에 병 없는 몸 별 없 늙 리

-

④ 가마귀 골에 가지 마

낸 가마귀 새

청강에 것 시 몸 러 가( ) 淸江

-

⑤ 진 골에( ) 白雪

가 매 는 어느 곳에 었는고

에 갈 곳 몰( ) 夕陽

색-

zb60) 위 에 나타난 세 어의 특 으 적절하 않은

것은

① 룡 어 주격 사에 당 는 가 사( ) lsquo rsquo六龍

고 다

② 샤 어에도 어 주체 쓰 다

는 것 다

③ 매 어 달리 사 택에 어

가 지 지지 고 다

④ 므 원 상 직 어 지 다

⑤ 드시리 다 주체 과 상 께 사

고 다

수고 하셨습니다hearts hearts

년 학 간고사 대비2013 2 현대고 대비

ECN-0102-2013-001-000076193

보닷컴에 공 는 별 보는 고등

들 여 주 는

들 습니다 슷 동 지

가 복 는 것 도가

니 복 여 습 시고 거 시

니다

정답 해설

1) 정답[ ] ④

해설 다른 것은 두 특정 업이나 단 내에서 사[ ]

하는 일종의 은어 사회 언에 해당한다 러나

는 언이 아니라 단과대학을 여서 단대 사lsquo rsquo lsquo rsquo lsquo④

대학을 여서 사대라고 한 말에 해당하 일rsquo lsquo rsquo

사회에서도 널리 쓰이 사회 언이라 할

없다

2) 정답[ ] ⑤

해설 사회 언은 같은 단 내에서 쓰이는 언어이[ ] lsquo rsquo

동일 단끼리는 단결 과 친 감을 형성하는

능을 하 리적 안감이 일어나 않는다

3) 정답[ ] ③

해설 사람이라는 차 적 표현에 대한 대안적 표현이[ ]lsquo rsquo

인 아내 처 등으 볼 있다lsquo rsquo

4) 정답[ ]⑤

해설 남성은 주 격 체를 사 한다[ ]

5) 정답[ ] ⑤

해설 흑인은 검다라는 뜻을 가 고 있을 뿐 인[ ]lsquo rsquo lsquo rsquo lsquo rsquo

다 열등한 뜻을 내포하 않는다

6) 정답 살 색 첫 작품[ ] - -

해설 살색 혹은 킨색은 한 인의 피 색을 뜻[ ] lsquo rsquo lsquo rsquo

하는 것으 인종 차 을 추 고 출 이주민

의 평등 을 침해할 있어 년 표 이2005

살 색으 이름을 꾸었다 처녀작은 처녀라lsquo rsquo lsquo rsquo lsquo rsquo

는 단어가 가 고 있는 곡된 성 인 을 한 것

으 첫 작품정도 꾸어 사 하는 것이 좋다lsquo rsquo

7) 정답[ ] ⑤

해설 호는 아들에게 해체를 사 하고 있다[ ] ① ②

장 을 성하는 청자는 자 의 아 느리 아lsquo

들 세 이다 호는 아 느리에게 해rsquo ③

체를 사 하고 있다 호가 느리 아 에게 ④

사 한 해 체 아들에게 사 한 해체는 두 비lsquo rsquo lsquo rsquo

격 체에 해당한다 호는 자 의 아랫사람인 ⑤

느리에게 아들과 마찬가 해체를 사 하는 것이

상 이 만 임 을 한 느리에게 고마 과 쁨

존 의 표 를 하 위해 자 의 아 에게 말하듯

해 체를 사 하고 있다

8) 정답[ ] ③

9) 정답[ ] ⑤

10) 정답[ ] ①

해설 청자 할아 가 장의 주체 아 다 높을[ ] ( ) ( )

경 에는 압존 에 의해 장의 주체를 높이 않는lsquo rsquo

다 러 아 서가 아닌 아 는으 계 lsquo rsquo lsquo rsquo lsquo

니다 가 아닌 있 니다 표현하는 것이 르rsquo lsquo rsquo

11) 정답 당이 당을 쫒았다 당이[ ]

당에 다

해설[ ]

12) 정답[ ] ⑤

해설 서 다른 높임표현을 통해 청자에 대해 리[ ] ⑤

적 거리감을 나타내는 인 은 이 아니라 현정이

다 가 에서 현정은 에게 해 체를 사 함으 써 ( )

친근감을 드러낸다 나 에서 연 을 게을리하는 역 ( )

도 들 때 에 화가 난 현정이 선생님에게 항의하

는 장 에서는 하 체를 사 하여 리적 거리lsquo rsquo

가 어졌음을 나타내고 있다

13) 정답[ ] ①

해설 는 는 얼 빛이 날과 어찌 다르 고[ ] lsquo rsquo

라는 뜻으 전과 달리 임이 화자를 않고

있음을 알 있다

14) 정답 달리 후 가 있다 이를 통해 경[ ] lt gt

쾌한 음악성을 형성하고 노 젓는 상황을 체적으

형상화하는 역할을 한다

15) 정답[ ] ①

16) 정답[ ] ⑤

해설 다 의 자연은 를 성찰하게 하는 대상[ ] ( )⑤

이자 정의 대상이다 의 자연은 자 의 상황과 ⑤

처 를 드러내는 경으 서의 역할을 하 이

이 없다

17) 정답[ ] ③

해설 는 빈천 을 해결하고자 했으나 강산[ ] lsquo ( )rsquo 貧賤③

과 풍 을 달라는 에 거절하 다고 함으 써 자

연에 대한 애정을 드러내고 있으 는 않는

임에 대한 망을 개에게 전가 켜서 임에 대한 리

을 드러내고 있다

18) 정답[ ] ③

년 학 간고사 대비2013 2 현대고 대비

ECN-0102-2013-001-000076193

19) 정답[ ] ⑤

해설 고상한 음악가의 이름을 리말 꽝 럽[ ]

게 꿈으 써 언어유희를 통해 음을 유 하고 있

다 이는 고상한 척하는 총 를 비꼼으 써 비판적

태도를 드러내는 것이 대상을 꽝 럽게 표현

하여 총 의 허 과 사치를 풍자하고 있다

20) 정답[ ] ⑤

해설 는 작품 속 경에 대한 설 이 드러나는 것이[ ]

서 자의 주 적인 견해가 접적으 드러나는 것이

아니다

21) 정답[ ] ⑤

22) 정답[ ] ②

23) 정답[ ] ④

24) 정답[ ] ①

해설 적강 티프는 주인공의 비 한 출생이나 능[ ] ①

과 이 있는 것으 조정의 능함을 풍자하는lsquo rsquo

것과는 거리가 다

25) 정답 픔 나[ ] ( )

해설 의 음악은 고통 는 사람들을 위 하고 아픔[ ] lsquo rsquo

을 치유해 주는 능을 한다고 할 있다 의 lt gt

픔 도 소 된 이 과 더 어 살아가는 따뜻한 마음lsquo rsquo

을 상 한다

26) 정답[ ] ⑤

해설 에게 선천적으 주어 각 장애라는 역경[ ]

은 의 이라는 가사 연 을 있다lsquo rsquo

27) 정답[ ] ④

해설 는 장 란 선 에게 은 개인적인 인상을[ ]

소녀 장정 등으 표현한 것이다lsquo rsquo

28) 정답[ ] ②

해설 담자가 피 담자의 언어적 표현이나 비언어[ ]②

적 표현 하 독자는 담의 위 나 피

담자의 감정 상태를 알 있다 이를 통해 독자는

담 상황을 더 생생하게 느낄 있고 피 담자

를 더 잘 이해할 있게 된다

29) 정답[ ]③

해설 일상생활과 역도 선 서의 성과에 된 것에서[ ]

역도를 하 서 겪는 어 과 내적 고민으 화제를

전화하 위한 것이다

30) 정답[ ] ①

해설 릿속에 새겨 넣듯 이 억되도 함 세상[ ] ② ③

살이가 힘들고 고생 러 속 하여 자유를 ④

가 없는 고통의 상태를 비유적으 이르는 말

적의 침입을 막 위해 쌓은 축 켜야 할⑤

대상을 비유적으 이르는 말이다

31) 정답[ ] ④

해설 이 의 종류는 전 으 인 사건 경[ ] lsquo

비평을 성 소 삼는다rsquo

32) 정답[ ] ④

해설 근은 삼대독자 태어났음을 에서 확인할[ ]

있다 형제들과의 담은 이뤄 가 없다

33) 정답[ ] ⑤

해설 근은 가난에도 하고 화가를 꿈꾸었다[ ] (3

단 또한 다른 화가 망생들은 정 육을)

위해 상 학 학 해 유학 에 랐 만

근은 다른 을 찾아야 했다 단 세에(5 ) 18

근은 조선 전람회에 입선하 다 단 의(6 )

만종은 인간과 자연이 엮어 가는 경건한 조화 을lsquo rsquo

나타낸다

34) 정답[ ] ①

해설 근이 속에서도 창작활동을 추 않고[ ]

하는 닭은 은 세상과 타협할 르는

근이 세상의 이해를 하 위한 가장 떳떳한 단

이 때 이다

35) 정답[ ] ⑤

해설 전 은 서 자의 주 적인 평이 리는 것이[ ]

만 위 제 은 인 이 살았던 대 사회적 경

을 통해 객 적인 인 의 을 제 하고 있다

36) 정답[ ] ⑤

해설 전 은 인 사건 경 비평이라는[ ] lsquo rsquo⑤

성 이 어져 있다

37) 정답[ ] ①

해설 이 은 동양인과 서양인의 사고 에 차이가[ ]

있다는 것을 대조를 통해 설 하고 있다 또 쓴이

의 제자가 축 경 를 러 가서 경험한 일화를

통해 동양인이 서양인에 비해 주 상황에 더 많은

주의를 인다는 주장을 뒷 침하고 있다

38) 정답[ ] ④

39) 정답[ ] ②

40) 정답[ ] ②

41) 정답[ ] ④

42) 정답[ ] ③

43) 정답[ ] ④

44) 정답 도서 의 휴 일 도서 의 이 간 도서의[ ]

해설 도서 장은 임의 정한 휴 일과 도서 이[ ]

간 도서의 상 등을 게 할 의 가 있다

년 학 간고사 대비2013 2 현대고 대비

ECN-0102-2013-001-000076193

45) 정답[ ] ①

해설 제 조의 정 휴 일 의 휴 일의 사전 게[ ] 3

는 도서 장의 의 조항에 속한다

46) 정답[ ] ①

해설 개인 정 호 의 를 제 하 했 만 항[ ]

나눠서 제 하 않고 대 나열하고 있다

47) 정답[ ] ②

해설 제 조의 내 을 회사는 다른 회사 협[ ] 7 lsquo

계약을 통해 서비 를 제공하는 경 회 의 아이디

등 개인 정 를 해당 회사에 전송할 있다는 내rsquo

이 있으 의 제점을 제 할 있다②

48) 정답[ ] ④

해설 는 도서 장의 의 에 해당하고 나 는 도[ ] ④

서 장의 리에 해당한다

49) 정답[ ] ③

50) 정답 은 음독으 적었고 은 훈독으 적었[ ] (1)

다 과 동일한 표 리 적은 것은 이고 (2) ce

과 동일한 표 리 적은 것은 이다ab

51) 정답[ ] ③

52) 정답[ ] ①②

53) 정답[ ] ③

54) 정답[ ] ③

55) 정답[ ] ①

56) 정답 른 죠코 어린 노 하니라[ ] A B

57) 정답 세 어에서는 활 형이 칙적으[ ] lsquo rsquoㄹㅇ

나타났 만 개화 어에서는 활 형이 쓰 다 lsquo rsquo ㄹㄴ

58) 정답 호 가 흔[ ] (1) (2)

59) 정답[ ] ④

60) 정답[ ] ③

Page 5: 현대고대비 국어 - chamsoriedu.com 「콘텐츠산업진흥 법」외 에도 저작권 의하여 ... 다른주체에게어떤동작을하도록만드는것을나타내는

년 학 간고사 대비2013 2 현대고 대비

ECN-0102-2013-001-000076193

지 지 어( ) ( ) ( )至匊悤 至匊悤 於思臥

심 내 가 가( ) ( )無心 白鷗

사( )3秋詞

운 니러 고 티 니 다( )白雲

돋 돋

믈 믈 동 가쟈( ) ( )西湖 東湖ㅣ

지 지 어( ) ( ) ( )至匊悤 至匊悤 於思臥

료 곳마다 경 다( ) ( )白蘋紅蓼 景

동사( )4冬詞

간 눈 갠 후 에 경믈 달 고( ) ( )後 景物

어 어

만경 리 듸 쳔 산( ) ( )萬頃琉璃 千疊玉山

지 지 어( ) ( ) ( )至匊悤 至匊悤 於思臥

계 가 계 가 간 니 다( ) ( ) ( )仙界 佛界 人間ㄴ ㄴ

zb14) 위 을 의 조 형태적 특 에서 어lt gt (1)

떤 차이가 있는 를 서 하고 인해 노래의 아 (2)

름다 에 어떤 향을 주었는 에 대해 서 하

보lt gt

십 경 여 가삼간 지어내니

달 청 맡겨 고

강산 들 곳 없 니 러보고 보리

가 가 각시 본 듯도 고( )

샹 경 엇 니 별 고天 上 白 玉 京 離 別

다 믄 눌 가시 고

어 여 고 내 사 드러 보

내 얼 거동 님 가마

엇 보시고 다 실

도 님 미 업

태 어 러 돗

시 낫 엇 다 신고

누어 생각 고 니러 여 니

내 몸 지 죄 뫼가티 싸 시니

원망 사 허믈 랴

워 니 믈 타시 다造 物

님다 쇼 식 므 쟈 니( ) 消 息

도 거 다 내 사 가

내 마암 둘 대 업다 어드러 가잔 말고

잡거니 거니 놉픈 뫼해 라가니

은 카니 안개난 사 일고

산 쳔이 어둡거니山 川 일日 月 을 엇디

쳑을 라거든 쳔 리랄 라 랴咫 尺 千 里

찰하리 가의 가 히나 자 하니

람이야 결이야 어둥졍 된뎌이고

샤공은 어대 가고 븬 만 걸 나니

강 텬의 혼쟈 셔셔 디난 해랄 니江 天

님다히 쇼 이 더 아득한뎌이고消 息

다 쳠 리 듕만 도 니( ) 茅

벽 쳥 등 눌 갓 고半 壁 靑 燈

라 나리 헤뜨 니니

져근덧 녁 하야力 盡 풋잠을 잠간 드니

졍 셩이 하야 꿈의 님을 니精 誠

가탄 얼 이 이나마 늘거셰라玉 半

마암의 근 말삼 카장 삷쟈 하니

눈 이 라 나니 말인들 어이하

졍을 다하야 이조차 여하니情

뎐된 계 셩의 잠은 엇디 돗던고鷄 聲

라( ) (A)찰하리 여디여 낙 이나 되야 이셔洛 月

님 겨 창 안해 드 비최리라窓

(B)각 님 달이야카니 잔 비나 되쇼셔

zb15) 위 가 라 에 대한 설 으 적절하( )-( ) 않은 것은

① 민들 과 삶 담고 다

② 여 체 식 어 다

③ 사미 곡과 께 가사 미 다

④ 우리말 사가 평가

년 학 간고사 대비2013 2 현대고 대비

ECN-0102-2013-001-000076193

고 다

⑤ 님 미 사 는 마lsquo rsquo

가사 다

가 춘사( ) [ 5]

고은 티 쬐얀 결이 름 다

이어라 이어라

을 주어 두랴 낙 노흘일가

총 총 어( ) ( ) ( )至匊葱 至匊葱 於思臥

탁 가 의 흥 이 나니 고 도 니 다( ) ( )濯纓歌 興

하사[ 2]

년닙희 두고 찬으란 쟝만 마라

닫 드러라 닫 드러라

청약립 은 써 잇노라 녹사의 가져 냐( ) ( )靑蒻笠 綠蓑衣

총 총 어( ) ( ) ( )至匊葱 至匊葱 於思臥

내 좃 가 제 좃 가( ) ( )無心 白鷗

추사[ 3]

이 니러나고 나 티 흐느 다( )白雲

돋 라라 돋 라라

의 셔호 혈 의 동호 가쟈( ) ( )西湖 東湖ㅣ

총 총 어사( ) ( ) ( )至菊悤 至菊悤 於思臥

빈홍 곳마다 경 이 다( ) ( )白蘋紅蓼 景

동사[ 4]

간 의 눈 갠 후 에 경 이 달랃고야( ) ( )後 景物

이어라 이어라

압희 만경류리 듸희 쳔텹 산( ) ( )萬頃琉璃 千疊玉山

총 총 어( ) ( ) ( )至菊悤 至菊悤 於思臥

선계 가 블계 가 인간 이 아니 다( ) ( ) ( )仙界 佛界 人間ㄴ ㄴ

윤선도 어 사 사- ( )漁父四時詞「 」

나 살어리 살어리랏다( ) 쳥산 애 살어리랏다( ) 靑山

위랑 래 랑 고 쳥산 애 살어리랏다 ( ) 靑山

얄리얄리 얄랑셩 얄라리 얄라

러라 러라 새여 자고 니러 러라 새여

널라 름 한 나도 자고 니러 니 라

얄리얄리 얄라셩 얄라리 얄라

청산별곡- ( )靑山別曲

다 청산은 엇뎨 야 만고애 프르르( ) ( ) ( ) 靑山 萬古

유 는 엇뎨하야 주야애 디 아니난고( ) ( )流水 晝夜

리도 치디 마라 만고상청 호리라( ) 萬古常靑

도산 이곡- ( )陶山十二曲

라 개를 여라 이나 르되 개 치 얄 랴( )

님 꼬리를 홰홰 치 치뛰락 나리뛰락 겨서 내

닷고 님 뒷 을 동 동 르락 나으락 캉캉

도리 암캐

이 릇 릇 날 들 너 이 이 랴

작자 상-

마 빈천 을 랴 고 에 드러가니( ) ( ) ( )貧賤 權門

침 업 흥졍을 뉘 져 쟈 리

강산과 풍 을 달나 니 는 리 리

조찬한-

정정 이랬거니 아람도리 큰 솔이 혀( ) ( )伐木丁丁

도 하이 골이 어 아리 소릴 쩌르 돌아 도 하이 다

람 도 좇 않고 새도 않어 은산 고 가 차라리

뼈를 저리 는데 눈과 이 종이 담 회 나 달도 름을

다 흰 뜻은 한 이골을 걸음이란다 절 이 여섯

판에 여섯 고 고 라간 뒤 조찰히 늙은 사나이의 남

내음새를 는가 름은 람도 일 않고 고 에 히

흔들리 노니 견디란다 차고 연 히 픔도 꿈도( )兀然

없이 장 산 속 겨 한 내-

정 장 산- 「 」

zb16) 다 에 나타난 상과 표현을 활 해 작 연 을 해( )

것이다 적절하 않은 것은

① 몰 는 도에 몸 맡 고 는 고통

욱 답게 가꾸 는 느님 산 겠지

② 돌 틈 뚫고 어 민들 보 리 진 고통

다가 도 민들 럼 강 게 살 지 겠는가

③ 벽에 런 움 없 어지고 는 폭포

여 시 는 죽 워 지 는 강

지 는가

④ 상 겨울 도 다리 꿋꿋 살 가고

겠지 망 지 는 삶 얼마 다운가

년 학 간고사 대비2013 2 현대고 대비

ECN-0102-2013-001-000076193

⑤ 막 늘 울리고 는 귀 미 리는

지새우는

가( ) 빈천( )貧賤을 랴 고 에 드러가니( )權門

침 업 흥졍을 뉘 져 쟈 리

강산과 풍 을 달나 니 는 리 리

나 청강 에( ) ( )淸江 비 듯는 소 어 읍

만산 홍 이 휘드르 는고야( )滿山紅綠

두어라 춘풍 이 날이리 을 어라( ) 春風

다 청산은 어찌 야 만고에 푸르르( ) ( ) ( )靑山 萬古

유 어찌 야 주야애 디 아니 고( ) ( )流水 晝夜

리도 치 말아 만고상청 하리라( ) 萬古常靑

라( ) 개를 여라 이나 르되 개 치 얄 랴

님 리를 홰홰 치 치 락 리 락

겨서 내닷고 고 님 뒷 을 동 동

르락 나으락 캉캉 도리 암

이 릇 릇 날 들 너 이 이 랴

zb17) 의 적 능을 비 한 내 으 가장 적절한

것은

① 는 가 지닌 실 계 다

② 는 내 갈등 심 시킨다

③ 는 가 는 상 각시킨( )愛着

④ 는 달리 내 갈등 시킨다

⑤ 는 달리 다 사 들 간 가

강 다

가 님다히 쇼 을 아 나 아쟈 니( ) ( )消息

도 거의 다 일이나 사 가

내 둘 업다 어드러 가쟛말고

잡거니 거니 놉픈 뫼 라가니

은 니 안개 일고

산쳔 이 어둡거니 일 을 엇디( ) ( )山川 日月

쳑 을 거든 쳔 리 라 랴( ) ( )咫尺 千里

하리 의 가 히나 쟈 니

람이야 결이야 어둥졍 된뎌이고

샤공은 어 가고 븬 만 걸 니

강텬 의 혼쟈 셔서 디 니( )江天

( )

쳠 자리의 듕만 도라 니( )茅簷

쳥등 은 눌 위 야 갓 고( )半壁靑燈

리 헤 니니

져근덧 녁 야 픗 을 잠간 드니( )力盡

졍셩 이 야 의 님을 니( )精誠

가 얼 이 이나마 늘거셰라( ) ( )玉 半

의 근 말 장 쟈 니

눈 이 라 나니 말인들 어이

졍 을 다 야 이조차 여 니( )情

뎐된 계셩 의 은 엇디 돗던고( )鷄聲

어 허 다 이 님이 어 간고( )虛事

결의 니러 안자 창 을 열고 라 니( )窓

어엿븐 림재 날 조 이 다

하리 여디여 낙 이나 되야이셔( )落月

님 겨 창 안 드 비최리라( )窓

나 내 님 리자 니다니( )

산 접동새 난 이 이다

아니 거츠르 아으

잔 효성이 아 리이다

넉 라도 님은 녀져라 아으

더 니 뉘러 니잇가

과도 허 도 천만 없소이다

힛마리 뎌

읏븐뎌 아으

니 나 마 니 니 잇가

아소 님하 도람 드르샤 쇼셔

zb18) 맥으 아 에 들어갈 적절한 것은

① 시 엇 다 신고

② 다 믄 눌 보 가시 고

③ 님다 쇼식 욱 득 고( )消息

④ 원망 사 허믈 랴

년 학 간고사 대비2013 2 현대고 대비

ECN-0102-2013-001-000076193

⑤ 죠 뫼 티 시 가( ) ( ) 粥早飯 朝夕

가( )

거리

공신 후 심 늦도( )劉尋

식 없어 과 께 산에 드리고 신

태몽 꾼 에 만고 웅 상 지닌 들

낳 키운다 그 후 신 들 에 역심 ( )逆心

담 귀 등 심 여 리 귀

보내고 지 죽 는 도망 가다

가 만 죽 고 에 경 가는 들 도움

살 다 그러 사 에 심 귀

보고 담 여 고 강 주가 승상

득 여 고 신 사 삼는다 그 후 강 승상

에게 심에 상 리지만 여움

사 귀 가게 다 강 승상 몸 는

연 과 헤어 리 다

경쇠 리 들리 에 들어가니 색

에 게 단청 누각과 큰 집들( )丹靑

다 주 보니 ( ) (一柱門 黃金

산 사 어 었다 산) lsquo rsquo 大字

들어가 고승 다 그( ) ( ) 山門 高僧

거동 보니 눈 눈 듯 고

변 같 귀는 어 에 늘어 니( ) 白邊

맑고 어 골격과 신 평 니었

팔염주를 에 걸고 육환장 을 고서 흑포( )六環杖

장삼에 떨어 송낙 쓰고 나 유생을 고 말[ ] 松蘿

하 를

소 이 연 하여 유상공 는 행차를 동 에 나ldquo

가 맞이하 하 으니 소 의 함을 서하

rdquo

유생이 크게 놀라 말하 다

천한 인생으 팔자가 하여 어 서 를 잃고ldquo

정처없이 다니다가 연히 이곳에 대사를 만난 것인데

토 대하 소생의 성은 어떻게 알고 있 니 rdquo

노 이 답하여 말하 를

어제 남악 형산 의 화선 이 소 의 절에ldquo ( ) ( )男樂 衡山

어서 소 에게 탁하 를 내일 낮 경에 남경 lsquo 12

동성 안에 사는 유 의 아들 충 이가 것이니 내쫓

말고 잘 대접하라 하셨 니다 마침 소 이 찾아 나rsquo

다가 상공의 차람새를 니 남경 사람이 에 알아

았 니다rdquo

유생이 말을 듣고 한편으 쁘고 한편으 퍼하

서 노 을 따라 들어가니 여러 들이 합장 하

가 했다 노 이 에 들어가 저녁 을 은 후에

을 편히 니 이곳은 선경 이었다 세상의 일을( ) 仙境

두 잊고 일 이 편안하 다 이후 는 노 과 함

서 도 이 탐 하고 경도 확하게 의 게 되었( )兵書

다 이 게 되니 대 천 에 가객 은 없 ( ) ( )大明天地 佳客

고 덕산 속에 리 른 만 있더라 래 ( ) 廣德山

이 천상 사람으 살아 있는 처를 만나 이한

을 니 재주 민함을 누가 당할 있겠는가

낮으 공 하더라

유충 전- -

웬늠 어가 사 싸다냐( ) ldquo rdquo

내가 가 막 런거 니

보통것 닐러 그 어낸ldquo ( )

틀어 주 그 가 루 러 허 에

싶어 키 틀어 주 그 가( )

루 허 우간 곡 틀어주는 루 못 는

웂는 고 닝께 고 지 들

어 사는 고 가 다는 건 에 그 집에

rdquo

그런 단 어들 어 새벽에 떼죽 거

다 고 어 보니 죄다 허 게 집어진

는 것 었다

총 가 내화를 꿴 뛰어나 만 아 소 없는

일이었다

어떻게 된 거야ldquo rdquo

한동안 넋나간 듯이 서 있던 총 가 하고많은 사람

에 하필이 유자를 겨냥하 은 말이었다

쎄유 아마 새에 고뿔이 들었던 개비네유ldquo rdquo

유자는 러 딴청을 하 다

야 고 가 에서 감 가 들어 죽는 고 두ldquo

어rdquo

총 는 가 혐의자 나 되는 것처럼 화풀이를( )嫌疑者

하 드는 것이었다

는 비위가 상해서

야 팔자가 사나서 이런 후 에 살라니|

여러 가 다 객고가 쌓여서 조 두 안 좋았을 테 helliphellip

런디다가 릇쓰 이 가락을 트는 대 디립

다 춰댔으니 과 해서 살끼두 다소 있었을 테 helliphellip

래 들어서 키 는 새끼덜일 이 다다 탈이 많은

이니 ldquohelliphellip

는 트의 독성을 충 히 내 않고 고 를 넣

은 것이 탈이었으 니 하 서도 러 참으 의 을 떨

었다 략 - -

마리가 마리 값 간다는 워ldquo

그냥 내뻔지 거시 허 싼 고 는 맛

겄다 싶 허 게 눌 강 어helliphellip

허 마늘 통 다

년 학 간고사 대비2013 2 현대고 대비

ECN-0102-2013-001-000076193

게 지 고뿌 지 rdquo

어 어째ldquo rdquo ldquo rdquo

런 도 것들 같 니ldquo ( ) rdquo殘忍無道 helliphellip

는 탱 여 지 못 다 보( ) 憤氣撐天

니 는 는 다 동원 여 통 쳤

생각 여 는 눈 다

달리 리헐 감ldquo rdquo

들 고 말 니었다 그가

는 것 그 말고는 없었 에 그 게 뒷동

달 거 다

는 우 럽고 식 짝 없는 랫것들 고

다 공연 신 가고 득 것

없다고 단 는지 결 웬만큼 고루 어

그 것들 쪽 에다 고 어주지ldquo

고 그 그걸 주 어 에 에 helliphellip

눈 없는 독 들 rdquohelliphellip helliphellip

고 말 럼 얼거리 들어가 리는 것 었

- ( ) -兪子小傳

zb19) 위 나 를 읽고 평가한 것으 적절하( ) 않은 것

① 사 리 통 감과 사실 고

격 과 달 고

② 는 가 재 컫는lsquo rsquo lsquo rsquo

미 가진 여 는 것 겠

③ 는 식 말 는 웃 상lsquo rsquo

여 는 미 지니고

④ 는 어가 죽 짐 지만 내색 지 고lsquo rsquo

말 고

⑤ 언어 통 가들 여 우리 통

것들 역 고

가 체 거리( ) [ ]

나라 종 연간에 정언주 의 을 하고( )正言注簿

있던 유 은 늦도 자 이 없어 한탄하다가 남악 형산lsquo rsquo

에 치성을 드리고 이한 태 을 꾼 뒤 아들을 낳아 이름

을 충 이라 고 키 다 이때 조정의 하들 에 역

을 품은 정한담 최일 등이 가달의 침입에 대한( ) 逆心

유 의 유화적 입장을 제 삼아 유 을 함하여 양

내고 유 의 에 을 러 충 자마저 살해하

한다 러나 충 은 천 조 정한담의 마 에서 어

나 많은 고난을 겪다가 은퇴한 재상 강희주를 만나 사위

가 된다 강희주는 유 을 하 고 상소를 으나 정

한담의 공격을 아 양을 가게 되고 강희주의 가족은

난을 피하여 두 흩어 다 충 은 강 소저 이 하고

사의 노 을 만나 를 때를 다린다 이

때 남적과 적이 를 들고 나라에 쳐들어 자 정한

담은 자 출전하여 남적에게 항 하고 남적의 선 장이

되어 천자를 공격한다 정한담에게 여러 패한 천자가

항 하 할 음 충 이 등장하여 남적의 선 정 걸

을 죽이고 천자를 출한다 충 은 단 으 란 을

쳐 고 정한담을 사 잡는다 리고 호 에게 ( )胡王

잡혀간 황후 태후 태자를 출하 유 에서 고생하

던 아 유 과 장인 강희주를 한다 또한 이 하

던 어 니 아내를 찾고 정한담 일파를 리친 뒤 높은

에 라서 화를 누린다

사 들 별 고 없 다니었다( )

마 마 돌 다니 걸 여 고

어 곤 다 에는 동쪽에 고

에는 쪽에 니 가 에 리는 엽

가는 없 니 늘 다니는 었

다 얼 말 죽 사 같고 림새가 말

니었다 가슴 에 고 등

삼태 헌 에 니 달 ( )奇男子

가 도리어 걸 었 담 만 열 도 ( )傅說

고 만났고 만 갈( ) ( ) 慇 武丁

도 탕 만났( ) ( ) (伊尹 成湯 渭

여상 도 주 만났는) ( ) ( ) ( ) 水 呂尙 周 文王

월 같 러가 도 어느 열 살

늘과 집 삼고 사 에 쳐 거리에

어 다가 곳에 니 다 ( ) 楚

지 다가 사 보고 가에 다다( )長沙

니 망 가에는 원 리가 슬 고 가

가 내리는 사 에는 갈매 가 갈 뿐 었다

쪽 돌 보니 가 우거 고

가 사 보 었다 그곳에

가니 는 사( ) 汨羅水

는 다 주 가 쓰고 죽고

곳 었다

마 감 여 에 가 사 살펴보니

에는 삼 고 그 에( ) 屈三閭

는 만고 월 과 지 가는 그 들( )風月

가 어 었다( ) 路程記

동쪽 벽 에 새 운 어 거늘 그

보니 월 에 경 주 는 간신에게ldquo ( )敗

보고 연경 귀 가다가 에 죽 rdquo

거늘 그 보고 에 거꾸러

통곡 말

[A]ldquo우리 연경 간 만 니 에

지 살 상에 엇 겠는가

에 고 에 었 니

상에 살 것 가 도 께 지리 rdquo

년 학 간고사 대비2013 2 현대고 대비

ECN-0102-2013-001-000076193

고 가에 내 가니 울 리가 에 지 사

쳤는지 심 심 것 가

신 심 것 가

다( ) 강 승상에게는 들 없고 다만 만

었다 가 낳 에 가 색

타고 내 에게 말 는 ldquo

니다 미원 과 연 맺고 ( ) ( )紫薇垣 緣分

었는 께 강 집 보내 에

니 게 여겨 주십시 거늘 rdquo

미 가운 낳 니 가 고 거동

단 다 시 짓 쓰 고 는 (音

없었 니 여 가운 지 는 짝) 律

룰 만 사 없었다 가 사 여 사 감

게 고 지 못 고 염 는 만다

다가 당에 거 고 식같 러 내니

고귀 상 루 말 다 어 울 도 다( ) 相

귀 사 없고 웅 걸( )富貴爵祿

만고 었다 승상 매우 뻐 내당 ( )內堂

들어가 에게 사 니 역시 매우 거

워 말 다

ldquo 도 마 사 는 승상께

그 게 말 시니 상 여러 말 지 말고

사 도 시다rdquo

상이 에 나 충 의 손을 잡고 결혼과 하여 ldquo

너에게 히 할 말이 있다 내가 늙은 말년에 딸

하나만을 두었는데 니 너 하늘이 정해 필

임이 하다 이제 년고락 을 너에게 탁 ( )百年苦樂

하겠다 하 대 충 이 릎을 꿇고 앉아 눈 을 흘리rdquo

여쭈었다

소자의 을 해주 고 또 하 에 두고자 하ldquo ( )膝下

니 감사하 이를 데가 없 니다 다만 가 속에 통탄

할 일이 사 쳐 있 니다 소자가 이 없어 양친 ( )兩親

의 생사를 른 채 결혼하여 아내를 얻는 것은 자 으

서 할 도리가 아닙니다 이것이 한 러 뿐입니다 rdquo

승상 그 말 듣고 슬 에 어 고

것 에 맞 어 변 게 리ldquo

는 다 집 시 공 도 여 ( )始祖公

고 가 에 가가 어진 만 개 공신

었 니 도 러워 말 시고 시 rdquo

택 여 니 다운 신 과 신

습 늘에 죄 짓고 간 상에 내 신

혼 를 다 끝내고 으 들어가 사 을 살펴 니 빛

나고 빛난 것이 한 입으 는 다 말하 어 고 하나

는 다 하 어 더라 에 켠 환한 촛 ( )新房

아래 은 에 랑과 가 평생의 연 을 맺었( )緣分

으니 서 사랑하 주고 은 말을 어떻게 다 헤아릴

있으 어떻게 다 하리 을 낸 후에 이튿날

상 를 니 상 거 마음을 이 하

더라

각 생 강 승상 집 쪽( )

늘 보고 없 가 신 신 생각 니

없고 어 없었다 는 어떻게 도리가 없다

여 산 에 들어가 리 고 어 도 닦

고 다 그 산 보고 가다

가 곳에 다다 니 에 큰 산 었다 많 우

리 골짜 가 늘 는 가운 색

에 고 갖가지 가 짝 어 었 ( )花草

다 신 산 생각 고 들어가니 경개 ( )景槪

가 매우 뛰어 고 경 산 다 산 리에 들

리는 것 리 보 는 것 울 청산뿐

었다 가 고 울 어 가

니 들 많 가지들 못 어 동

에 늘어 들거리 는( ) 洞口

우거진 가지에 갖 들 다 었다( ) 春情

계상 에는 공 는 늘( ) 花溪上

에 걸린 폭포가 벽 는 리는 산사( )層巖絶壁

쇠 리 객 에 는 듯 늘( ) ( ) 寒山寺 客船

에 싸여 는 습 산

그린 여 병 러 듯 다 경쇠 리가 들

리 에 들어가니 색 에

게 단청 누각과 큰 집들 다( ) 丹靑

주 보니( ) ( ) lsquo一柱門 黃金大字

산 사 어 었다 산 들rsquo ( )山門

어가 고승 다 그 거 보니( ) 高僧

눈 눈 듯 고 변 같 ( )白邊

귀는 어 에 늘어 니 맑고 어 골격

과 신 평 니었다 염주

에 걸고 짚고 포 삼에 어진( )六環仗

쓰고 생 보고 말

승 연 여 상공 시는 동 에ldquo

가 맞 지 못 니 승 십시 rdquo

생 크게 말 다

생 가 여 어 고ldquo

없 다니다가 우연 곳에 사 만 것

그 시 생 어떻게 고 습니 rdquo

승 답 여 말

어 산 승 에ldquo ( ) ( )南岳 衡山

시어 승에게 탁 내 낮 시경에 경 lsquo 12

동 에 사는 심 들 가 것 니 내쫓

지 말고 습니다 마 승rsquo

다가 상공 림새 보니 경 사 에 보

습니다rdquo

zb20) 위 의 친 에서 서 자의 개입이 드러나~

는 이 아닌 것은

① 달 가 도리어 걸 었( ) 奇男子

② 신 심 것 가

년 학 간고사 대비2013 2 현대고 대비

ECN-0102-2013-001-000076193

③ 다운 신 과 신 습 늘에 죄 짓고

간 상에 내 신 다

④ 사 주고 말 어떻게 다 헤 릴

어떻게 다 리

⑤ 신 산 생각 고 들어가니 경개 가 ( )景槪

매우 뛰어 고 경 산 다

거리 연[ ] ( )弘治

간 에 공신 후 에 언(1488~1505) ( )正言

주 는 벼슬 심 늦도 식( ) ( )主簿 劉尋

없어 과 께 산에 드리고 신 태

몽 꾼 에 만고 웅 상 지닌 들 낳

키운다 그 후 신 들 에 역심( )逆心

담 귀 등 심 여 리 귀 보내

고 지 죽 는 도망 가다가

만 다 에 에 어 니

헤어지게 다

에 에 어 니 헤어지게

다 그 후 사 들에게 우연 돌

생 다가 어느 열 살 었다 열 살

지 다가 우연 귀 견 는

그것 그 살 도 었고 그

귀 본 신도 지 죽고 마 고

크게 운다

( )

에는 강 주 는 재상 살고 었

니 시 에 과거에 격 여 승상 벼슬 다가 간

신 만 벼슬 그만 고 고 돌 었

다 그러 신 지 가 지 못 여 상

가 못 결 는 상 여 원 니

신 들 그 직간 꺼 다 그 에 도

담과 귀가 강 승상 가 미워 다

강 승상 마 본 에 갔다가 돌 는[A][ ( )本府

에 우편 주 에 다가 색( ) ( ) 右便 酒店

에 어리었는 청룡 에 지 늘

여 통곡 고 사 는 꿈 꾸었다] 마

상 게 생각 여 새 다리다가 새벽

닭 울고 가 달 갔다 가 보니

과연 어 동 가 가에 울고 는지 달

들어 그 고 사 에 어 말

는 어 어 에 어 가ldquo

닭 곳에 우느냐 니 울rdquo

그 고 답 여 말 다

는 경 동 에 사는 언 주 공 들ldquo

니다 께 간신 만 연경 귀 가

시다가 에 죽 사 에 는 닭에

도 에 죽고 니다rdquo

강 승상 말 듣고 크게 낯 변 말

것 웬 말 냐 근 동 ldquo (老

못 갔 니 그 사 변 여)患

런 변 었단 말 가 주 는 신 다

같 에 벼슬 다가 는 가 많 들어 고

돌 는 주 가 게 꿈 에 생

각 겠느냐 생각지 못 다 미 지 간

지지 말고 께 가 략rdquo ( ) hellip hellip

죽게 주 사당에 단 도 러운

겠느냐 말 말고 시는지 rdquo

어 없어 강 승상 가니 그곳

월계 었다

다( )

가가 고 지 사 들 가( )櫛比

통 는 리가 과

답게 꾸민 누각과 큰 집들 늘 고

게 식 가 어 들 태운 가고

었다 략 강 승상에게는 들 없고 ( ) hellip hellip

다만 만 었다 가 낳 에

가 색 타고 내 에게 말

는 니다 미원 과ldquo ( )紫薇垣

연 맺고 었는 께 강 집( )緣分

보내 에 니 게 여겨 주십시

rdquo

거늘 미 가운 낳 니 가

고 거동 단 다 시 짓 쓰 고

는 없었 니( )音律 여 가운

지 는 짝 룰 만 사 없었다 가 사

여 사 감 게 고 지 못 고 염 는 만다

다가 당에 거 고 식같

러 내니 고귀 상 루 말 다 ( )相

어 울 도 다 귀 사 없 ( )富貴爵祿

고 웅 걸 만고 었다 승상 매우 뻐

내당 들어가 에게 사 니( ) 內堂

역시 매우 거워 말 다 도 마 ldquo

사 는 승상께 그 게 말 시니

상 여러 말 지 말고 사 도 시다rdquo

( )

승상 에 고 결 과 ldquo

여 에게 말 다 내가 늙 말 에 지

만 었는 지 보니 늘

다 에게 탁 겠 ( )

다 신 꿇고 눈 리rdquo

여 었다 주시고 슬 에 ldquo ( )膝下

고 시니 감사 룰 가 없습니다 다만 가슴

에 통탄 사 쳐 습니다 복 없어

생사 결 여 내 얻는 것( )兩親

식 도리가 닙니다 것 러울 뿐 니

다rdquo

상 그 말 듣고 슬 에 어 고 말

것 에 맞 어 웅변 ldquo

년 학 간고사 대비2013 2 현대고 대비

ECN-0102-2013-001-000076193

게 리 는 다 집 시 공도 여

고 가 에 가가 어진 만 개 공신

었 니 도 러워 마 시고 rdquo 시

택 여 니 운 신 과 신

습 늘에 죄 짓고 간 상에 내 신

다 략 지낸 후에 튿 승상 ( ) hellip hellip

니 승상 거운 마 지 못

마( )

듯 월 러 생 열다 살 었

다 에 승상 어진 사 얻고 만 에 근심 없었

다만 주 가 간신 에

죽 것 생각 마 곧 어 곤

다 그 에 주 원통 어

없 고 여 시 가 거늘 생 만

여 다

말 감격 러우 간신 에 가득 여ldquo

고 니 께 상 듣지 니 것

니다rdquo

승상 듣지 고 가

퇴 재상 공달 집에 거 고 상 지어

승지 러 께 리

( )

뒷 거리 강 승상 에게 상 리지[ ]

만 여움 사 귀 가게 다 강 승상

몸 는 연 가 헤어

리 다 산 들어간 룡사 승 만

게 다 승 만 우 다릴

과 들고 략 다 담

원 여 에게 복 고 어

공격 다 담에게 여러 가( ) 天子

복 등 여 다 단

신 리쳐 담 사 고 에게

간 후 태후 태 여 지에 고생

지 심과 강 주 여 개 다 헤

어 어 니 내 고 담 리

벼슬에 귀 누리게 다

zb21) 위 의 인 간 계를 같이 나타냈을lt gt

때 에 대한 이해 가장 적절하 ~ 않은 것은

① 계에 주 는 계 심 열

상 에 다고 다( ) 水深火熱

② 계는 견원지간 고 다( ) 犬猿之間

③ 계는 달리 막역지 계 고( )莫逆之交

④ 연결 사 컬어 재 가 고( )才子佳人

⑤ 는 생 과 볼 ( )匹夫匹婦

가 재 는 는 심 고 매사에 생( )

각 고 능 도 어 가 에게 많lsquo rsquo

도움 사 다 그는 에게 거 에

꺼리 없 거 났다고 는

매우 싫어 고 신 들

는 사 다

내가 지 리에( ) 1970

사 실에 지 월간ldquo

편집 고 어 었다rdquo

어느 없 가 쑥 다 도 어 10

후 다 산 시 럼 어 엇 어 ( ) lsquo怡山

다시 만 랴 니 그는 재 그룹 승 운rsquo

사가 고 는 고 거 누

주는 가 없는 가가 어 다시 만 게 것

었다

다 보통 것 닐러 그( ) ldquo 어낸 ( )

틀어주 가 루 러 허 에

싶어 키 틀어주 그( )

가 루 허 우간 곡 틀어 주는 루 못

는 는 고 닝께 고 지

들어 사는 고 가 다는 건 에 그 집에

rdquo

그런 단 어들 어 새벽에 떼죽 거

다 고 어 보니 죄다 허 게 집어진

는 것 었다 가 실내 꿴 뛰어 지만

없는 었다

어떻게 된 거야 한동안 넋나간 듯이 서 있던 총ldquo rdquo

가 하고많은 사람 에 하필이 유자를 겨냥하 은

말이었다 쎄유 아마 새에 고뿔이 들었던 개비네 ldquo

유rdquo

유자는 러 딴청을 하 다 야 고 가 에서 ldquo

감 가 들어 죽는 고 두 어rdquo 총 는 가 혐의

자 나 되는 것처럼 화풀이를 하 드는 것이었다( )嫌疑者

라 이 어쩌 어 유( ) ldquo rdquo ldquo rdquo

애유 이런 잔인 도 한 것들 같으니ldquo ( ) rdquo殘忍無道 helliphellip

총 는 탱천 하여 쩌 를 하 다( ) 憤氣撐天

아하니 아는 자는 다 동 하여 호통을 쳤으 하나 혈

압을 생각하여 참는 눈치 다 달리 처리헐 두 ldquo

잖은감유rdquo

총 의 성 을 덧들이 고 한 말이 아니었다 가 할

년 학 간고사 대비2013 2 현대고 대비

ECN-0102-2013-001-000076193

있는 것이 말고는 없었 때 에 게 뒷동

산을 달은 거 다

이 유자소전- lsquo rsquo

zb22) 의 상황을 속담으 표현한 것으 적절한 것은

① 루 곳 게 마 다

② 에 맞고 강에 눈 다

③ 늘 도 다

④ 도 사 다

⑤ 에 가도 신만 리 다

거리 공신 후[ ]

에 주 는 벼슬 심 늦도( )主簿

식 없어 과 께 산에 드리고 신

태몽 꾼 에 만고 웅 상 지닌 들

낳 키운다 그 후 신 들 에 역심

담 귀 등 심 여 리 귀 보내고

지 죽 는 도망 간다 그

만 고 에 에 어 니

헤어지게 다 지 가 사 들에

사 들 별 고 없 다니었다

마 마 돌 다니 걸 여 고

어 곤 다 에는 동쪽에 고 에

는 쪽에 니 가 에 리는 엽 가는

없 니 늘 다니는 었다

얼 말 죽 사 같고 림새가 말 니었

다 가슴 에 고 등 삼태

헌 에 니 달 가 도리 ( )奇男子

어 걸 었 담 만 열 도 ( ) ( )傅說 慇

고 만났고 만 갈( ) ( )武丁 伊尹

도 탕 만났( ) ( )成湯 渭水

여상 도 주 만났는 월( ) ( ) ( ) 呂尙 周 文王

같 러가 도 어느 열 살

늘과 집 삼고 사 에 쳐 거리에

어 다가 곳에 니 다 ( ) 楚

지 다가 사 보고 가에 다다( )長沙

니 망 가에는 원 리가 슬 고 가

가 내리는 사 에는 갈매 가 갈 뿐 었다

쪽 돌 보니 가 우거 고

가 사 보 었다 그곳에

가니 는 사( ) 汨羅水

는 다 주 가 쓰고 죽고

곳 었다

마 감 여 에 가 사 살펴보니

에는 삼 고 그 에( ) 屈三閭

는 만고 월 과 지 가는 그 들( )風月

가 어 었다( ) 路程記

동쪽 벽 에 새 운 어 거늘 그

보니

월 에 경 주 는 간신에게ldquo ( )敗

보고 연경 귀 가다가 에 죽 rdquo

거늘 그 보고 에 거꾸러

통곡 말

우리 연경 간 만 니ldquo ( )燕京

에 지 살 상에 엇 겠는

가 에 고 에 었 니

상에 살 것 가 도 께 지리 rdquo

고 가에 내 가니 울 리가 에 지

사 쳤는지 심 심 것 가

에는 강 주 는 재상 살고 었

니 시 에 과거에 격 여 승상 벼슬 다가 간

신 만 벼슬 그만 고 고 돌 었

다 그러 신 지 가 지 못 여 상

가 못 결 는 상 여 원 니

신 들 그 직간 꺼 다 그 에 도

담과 귀가 강 승상 가 미워 다 강 승상 마

본 에 갔다가 돌 는 에 우편 주( ) ( )本府 右便

에 다가 색 에 어리었는 청룡( ) 酒店

에 지 늘 여 통곡 고

사 는 꿈 꾸었다 마 상 게 생

각 여 새 다리다가 새벽닭 울고

가 달 갔다 가 보니 과연 어 동 가

가에 울고 는지 달 들어 그

고 사 에 어 말

는 어 어 에 어 가ldquo

닭 곳에 우느냐rdquo

니 울 그 고 답 여 말 다

는 경 동 에 사는 언 주 공 들ldquo

니다 께 간신 만 연경 귀 가

시다가 에 죽 사 에 는 닭에

도 에 죽고 니다rdquo

강 승상 말 듣고 크게 낯 변 말

것 웬 말 냐 근 동ldquo ( )老患

못 갔 니 그 사 변 여 런 변

었단 말 가 주 는 신 다 같

에 벼슬 다가 는 가 많 들어 고 돌

는 주 가 게 꿈 에 생각

겠느냐 생각지 못 다 미 지 간 지지

말고 께 가 rdquo

뒷 거리 강 승상 도움 죽 고[ ]

년 학 간고사 대비2013 2 현대고 대비

ECN-0102-2013-001-000076193

고 그 과 결 여 사 가 다 그러 강

승상 에게 울린 상 강 승상 귀 가고

과 헤어 리 승 만 게 다

승 우 다릴 과

들고 략 다 담 원

여 에게 복 고 어 (天

공격 다 담에게 여러 가 복) 子

등 여 다 단신

리쳐 담 사 고 에게 간

후 태후 태 여 지에 고생 지

심과 강 주 여 개 다 헤어

어 니 내 고 담 리 벼

슬에 귀 누리게 다

미상- lsquo ( )-劉忠烈傳

zb23) 위 과 의 서사 조를 비 한 것으 적절하lt gt

않은 것은

보lt gt

믿지 고 결 여 곱

낳 다 곱째 공주 낳 가

리게 다 리 만 고 진 공주는 lsquo rsquo

리공 미 리공 에 키워진다 월

러 과 가 죽 병에 걸 는 승에 는

어 산다고 다 여 들에게 탁

지만 거 리 는다 리 는 과

승 다 승 지 가는 에 많

만 지만 보살 도움 사 도 다

그러 승 신과 결 여 시

들어 주겠다고 다 리 는 그 결

여 들 곱 낳 후에 신

얻게 다 돌 리 는

에 과 상여 만 지만 여 과

살 낸다 훗 리 그 공 우 죽 사

승 도 는 신 다

리-lsquo rsquo-

① 복 결말에 고 다

② 웅 에 탕 고 다

③ 시 겨 내고 귀 누리는lsquo rsquo

보 리 는 월 재 신 다lt gt lsquo rsquo

④ 과 보 리 는lsquo rsquo lt gt lsquo rsquo

도움과 어 신 능 극복 고

⑤ 등 여 시 겪는lsquo rsquo

보 리 는 닌 지lt gt lsquo rsquo

림 시 겪는다

가 각 고 에( ) ( )却說

살 없었다 략 사 들 슬 에 어 lt gt

가에 내 고 가고 싶 가 고 후

워 경

사 들 별 고 없 다니었다 lt

략 얼 말 죽 사 같고 림새가 말gt

니었다 가슴 에 고 등

삼태 헌 에 니 달 가 ( )奇男子

도리어 걸 었 담 만 열 도( )傅說

고 만났고 만 갈( ) ( ) 殷 武丁

도 탕 만났( ) ( ) (伊尹 成湯 渭

여상 도 주 만났는) ( ) ( ) ( )水 呂尙 周 文王

월 같 러가 도 어느 열 살

늘과 집 삼고 사 에 쳐 거리에

어 다가 곳에 니 다 ( ) 楚

지 다가 사 보고 가에 다다( )長沙

니 망 가에는 원 리가 슬 고 가

가 내리는 사 에는 갈매 가 갈 뿐 었다

쪽 돌 보니 가 우거 고

가 사 보 었다 그곳에

가니 는 사( ) 汨羅水

는 다 주 가 쓰고 죽고

곳 었다

에는 강 주 는 재상 살고( )

었 니 시 에 과거에 격 여 승상 벼슬 다

가 간신 만 벼슬 그만 고 고 돌

었다 략 강 승상 마 본 에 갔다가 돌 lt gt ( )本府

는 에 우편 주 에 다가 색( ) ( ) 右便 酒店

에 어리었는 청룡 에 지

늘 여 통곡 고 사 는 꿈 꾸

었다 마 상 게 생각 여 새 다리다

가 새벽닭 울고 달 갔다 가

보니 과연 어 동 가 가에 울고 는지

달 들어 그 고 사 에

어 말

는 어 어 에 어 가ldquo

닭 곳에 우느냐rdquo

니 울 그 고 답 여 말 다 lt

략gt

년 학 간고사 대비2013 2 현대고 대비

ECN-0102-2013-001-000076193

생각 여 가 고 시 는ldquo ( )大人

상에 다시없는 니다 살 엇 겠습니

에 돌 가시고

가에 돌 가 니 살 마 없습니

다 략 어 없어 강 승상 가니rdquo lt gt

그곳 월계 었다

다 강 승상에게는 들 없고 다만 만( )

었다 가 낳 에 가 색

타고 내 에게 말

는 니다 미원 과ldquo ( )紫微垣

연 맺고 었는 께 강 집( )緣分

보내 에 니 게 여겨 주십시

rdquo

거늘 미 가운 낳 니 가

고 거동 단 다 시 짓 쓰 고

는 없었 니 여 가운( ) 音律

지 는 짝 룰 만 사 없었다 가 사

여 사 감 게 고 지 못 고 염 는 만다

다가 당에 거 고 식같 러

내니 고귀 상 루 말 다 어 ( )相

울 도 다 귀 사 없고 ( )富貴爵祿

웅 걸 만고 었다 승상 매우 뻐 내

당 들어가 에게 사 니 역( ) 內堂

시 매우 거워 말 다

도 마 사 는 승상께ldquo

그 게 말 시니 상 여러 말 지 말고 사

도 시다 략 시 택 여rdquo lt gt

니 다운 신 과 신 습 늘에 죄

짓고 간 상에 내 신 다

다 내고 들어가 사 살펴보니

고 것 는 다 말 어 고

는 다 어 신 에 ( )新房

에 신 과 신 가 평생 연 맺었( )緣分

니 사 주고 말 어떻게 다 헤 릴

어떻게 다 리 지낸 후에 튿 승

상 니 승상 거운 마 지 못

( ) 듯 월 러 생 열다 살

었다 에 승상 어진 사 얻고 만 에 근심

없었 다만 주 가 간신

에 죽 것 생각 마 곧 어

곤 다 그 에 주 원통

어 없 고 여 시 가 거늘 략 lt gt

략 거리

강 승상 에게 상 리지만 여움

사 귀 가게 다 강 승상 몸 는

연 과 헤어 리 다

마 각 생 강 승상 집 쪽( )

늘 보고 없 가 신 신 생각 니

없고 어 없었다 는 어떻게 도리가 없다

여 산 에 들어가 리 고 어 도 닦

고 다 그 산 보고 가

다가 곳에 다다 니 에 큰 산 었다 많

우리 골짜 가 늘 는 가운 색

에 고 갖가지 가 짝 어 ( )花草

었다 략 주 보니 lt gt ( ) (一柱門 黃

산 룡사 어 었다) lsquo rsquo 金大字

산 들어가 고승 다 그( ) ( ) 山門 高僧

거동 보니 눈 눈 듯 고

변 같 귀는 어 에 늘어 니( ) 白邊

맑고 어 골격과 신 평 니었

다 염주 에 걸고 짚고 포 ( )六環杖

삼에 어진 쓰고 생 보고 말

승 연 여 상공 시는 동 에ldquo

가 맞 지 못 니 승 십시 rdquo

생 크게 말 다

생 가 여 어 고ldquo

없 다니다가 우연 곳에 사 만 것

그 시 생 어떻게 고 습니

rdquo

승 답 여 말

어 산 승 에ldquo ( ) ( )南岳 衡山

시어 승에게 탁 내 낮 시경에 경 lsquo 12

동 에 사는 심 들 가 것 니 내쫓

지 말고 습니다 마 승rsquo

다가 상공 림새 보니 경 사 에 보

습니다rdquo

생 그 말 듣고 편 고 편( )

슬 승 들어가니 여러 승 들

가워 다 승 에 들어가

후에 그 편 니 곳 경 었다 상( ) 仙境

고 신 편 다 후 는 승과

께 병 도 탐 고 경도 게( )兵書

게 었다 게 니 지 에 가객 ( ) ( )大明天地 佳客

년 학 간고사 대비2013 2 현대고 대비

ECN-0102-2013-001-000076193

없고 산 에 리 만 본 ( ) 廣德山

신 상 사 살 는 만

우고 늘 월 신 과 늘 ( )日月聖神

산 신 들 다 니 그 재( ) 名山神靈

주 민 누가 당 겠는가 낮 공

zb24) 다 에 해당하는 내 으 적절하( ) 않은 것은

① 강 티 통 당시 능 다

② 상계 지상계 경 는 원 계 드러

③ 실에 어 없는 실 가 타 는

④ 뛰어 재주 어 가진 고

등 다

⑤ 가 직 개 여 평가 내리는

편집 평 타 다lsquo rsquo

가 본격 가 동 것 지( )

다 단 상 에2003 lsquo rsquo

들어가 드럼 연주 다 취미 생 달리

들었다는 보 우 가 들ldquo

어 틱 린 도 다 고 말 다rdquo

경 는 가 망 없( ) lsquo

티 원 고 답 다 신과 같 시각rsquo

는 습 상상 만 도 감동

다 시각 연주 동시에

열 상 는

티 원 그런 열 경 럽다는 것 다

다 역시 엄청 다 본( )

에 복 들

고쳐 가고 다 신 에 얼

마 지는 고 리가 는 지도 생님

가 훈 고 많 고쳐 다

고 말 다

그러 직도 에 지 는 다 그는

체격 지 못 게 가 큰 만

체 운동 훈 과 께 체 늘 동 50

는 게 고 말 다

에게는 꿈 다 통 누 가( )

주겠다는 것 그 꿈 다 신 극복 는

과 에 큰 경험 들도 느 게

주고 싶다는 것 다

마 슬 마다( ) ldquo 통

낼 었 것 럼 고통 는 사 들

고 겠다 고rdquo

말 다 달 루 첫 낸 lsquo rsquo

첫 드 심 집에 는 리듬 드 2

루 에 도 보고 싶다 집 에는 직(RampB) 3 4

사 곡 도 보 고 싶다고 포 다middot

zb25) 에서 가장 유사한 의 를 닌 어를lt gt

찾아 쓰

lt gt

나는 이제 너에게도 픔을 주겠다

사랑 다 소 한 픔을 주겠다

겨 거리에서 개 놓고

살아 추위 떨고 있는 할 니에게

값을 으 서 뻐하던 너를 위하여

나는 픔의 평등한 얼 을 여 주겠다

내가 어둠 속에서 너를 를 때

단 한 도 평등하게 어주 않은

가마니에 덮인 동사자가

다 얼어 죽을 때

가마니 한 장조차 덮어주 않은

한 너의 사랑을 위해

흘릴 르는 너의 눈 을 위해

나는 너에게 이제 너에게도 다림을 주겠다

지 울 포동 여고 생들17

틈 없 가득 체 에 맑 울

다 죽 듣 생들 사 에

연 는 탄 다 객들 도 는lsquo rsquo

가 보 주 공 맹 가 운 는

단 그룹사운드 루 보컬 맡고 는lsquo rsquo

시각 지 었다17 1

근 다만과 가 거lsquo rsquo lsquo

꿈 고 퇴 내가 다rsquo

간 간에 지 지 연 생들 짧lsquo rsquo lsquo rsquo

가 운 듯 리에 어

연 다 내 사 고 퇴lsquo rsquo

과 루 들 결 다시 돌lsquo rsquo

들 고 사 들 에 당당

것 니다 내 태어

볼 없었 크고 열여

년 학 간고사 대비2013 2 현대고 대비

ECN-0102-2013-001-000076193

에도 고 시 얻지 못 다

감지 없는 시각 상태 다

신 지에 고 상 원망 도

단다 어느 가 에 시각 에 ldquo

어 그런 듣고 다 보니 내가 게 lsquo

살 는지 도 눈 고 싶rsquo lsquohelliphellip

보 는 생각만 들 고 그 가 들에게rsquo

도 내고 들도 고 많 었죠 들 rdquo

었 지 새 는 에 쑥 러운 색

어났다

생에 것 단연 었다lsquo rsquo

공연에 거 꿈lsquo rsquo

는 다 특 가사 갑게 는 운 lsquo

벽 에 당당 마주 어 언 가 그 벽

고 늘 어 거운 상도

없죠 내 삶 에 웃 그 께

는 다고 다rsquo

들었 그냥 런 도 고만 여ldquo lsquo rsquo

겼죠 그런 꾸 가사 미 새 다 보

니 통 는 가사 는 생각 들 고 (

가 게는 시각 는 생각 들고 들) ( )

마다 듣고 큰 얻었어 rdquo

에 진지 게 가에 미 가

zb26) 의 에 들어갈 말 적절한 것은lt gt ~

lt gt

난 난 꿈이 있었죠

고 찢겨 남 하여도

내 가 히 과 같이 간 했던 꿈

혹 때 누 가가 뜻 를 비 음

내 등 뒤에 흘릴 때도

난 참아야 했죠 참을 있었죠

날을 위해

늘 걱정하듯 말하죠

헛된 꿈은 독이라고

세상은 끝이 정해 책처럼

이 돌이킬 없는

현 이라고 helliphellip

래 난 난 꿈이 있어

꿈을 믿어

나를 켜

저 차갑게 서 있는 이란 앞에

당당히 마주칠 있어

출처 가 거위의 꿈 작사 이적 작곡 동률- lsquo rsquo ( )

① ② ③ ④ ⑤

가 떴다는 들 만 지만( ) lsquo rsquo

늘 겸 다 에 주 연 우승 지 간에도 3

단 생님께 만 지 고 고 만ldquo rdquo

큼 늘 겸 신 계 가

고 다

에게는 꿈 다 통 누 가

주겠다는 것 그 꿈 다 신 극복 는 과

에 큰 경험 들도 느 게 주

고 싶다는 것 다

슬 마다 통 낼ldquo

었 것 럼 고통 는 사 들

고 겠다 고rdquo

말 다 달 루 첫 낸lsquo rsquo

첫 드 심 집에 는 리듬 2

루 에 도 보고 싶다(RampB) 집 에는 직34

사 곡 도 보 고 싶다고 포 다

미 는( ) (26) 어 헤헤헤 웃다가 어ldquo rdquo

허허허 웃었다ldquo rdquo ldquo rdquo 같 도 고

상 다 는 같 도 다( ) 壯丁 킹 들lsquo

다 는 역도 보 그 다 지만 그는rsquo

뷰에 지 다 운동만 지 ldquo

것 지 간에 여러 사 도 역rdquo helliphellip

었다 그런 엇 그 마 움직 는지 보 쯤

지 담 사 다 훈 없어 그는 티

지 림 었다 태 다 갔다 는 습

마 집 럼 편 게 보 다

주말에는 주 엇 보내

주말에도 별 주 에 청ldquo

고 에 가고 도 쳐

에 듣고 보 에 갈 가 별 없

어 산 시 게 고 들어 2002

거 매 여 지냅니다 시 과 지훈 rdquo

다 근 간 과 진실 그리고 싶어( )

가 다 근에게 그것 진리 다 거 다 없

거 고 다 없 는 것 진리

다 근 진리는 후 쪽 었다 신산( )辛酸 삶

었 질곡( )桎梏 역사 에 지냈 가

눈에 든 것 료 단 료 게 보

것 었다 그것 그 에 겨우겨우

슬 슬 생 어가는 간들 었다

리 과 단 리 고리에 검 마

없 거리 돌

상 것 없는 등 근에게 상

과 진실 엄 ( )儼存 다는 사실 리는 가

실 고 가 과 역경 에 도 근 내 포

없었 후 보루( )堡壘 다 도 365

도 간 근 여

시 것 다

년 학 간고사 대비2013 2 현대고 대비

ECN-0102-2013-001-000076193

다 공주 그림 가 근 경- ( ) ldquo rdquo(

2009)

zb27) 작가의 주 적인 각이 드러난 것은~

① ② ③ ④ ⑤

가 신 지에 고 상 원망( )

도 단다 어느 가 에 시각 에 ldquo

어 그런 듣고 다 보니 내가 lsquo

게 살 는지 도 눈 고 싶rsquo lsquohelliphellip

보 는 생각만 들 고 그 가 들에게rsquo

도 내고 들도 고 많 었죠 들었rdquo

지 새 는 에 쑥쓰러운 색

어났다 략 [ ]

경 는 가 망 없 티lsquo

원 고 답 다 신과 같 시각rsquo

는 습 상상 만 도 감동

다 시각 연주 동시에

열 상 는 티

원 그런 열 경 럽다는 것 다 략 [ ]

슬 마다 통 낼ldquo

었 것 럼 고통 는 사 들

고 겠다 고rdquo

말 다 달 루 첫 낸 lsquo rsquo

첫 드 심 집에 는 리듬 2

루 에 도 보고 싶다 집 에는 직(RampB) 3 4

사 곡 도 보 고 싶다고 포 다

식 누 가-

고 싶어

다 역도 미 담 고 사( )

질 주말에는 주 엇 보내[ 1]

답 주말에도 별 주 에[ ] ldquo

청 고 에 가고 도 쳐

에 듣고 보 에 갈 가 별

없어 rdquo

질 계 고 슬슬 도 는 것 닙니[ 2]

답 다 들 눈 에 보 고 뿐 보[ ] ldquo

다 열심 고 어 상에 도 들지만 상

지키는 것 들다고 에 도달

그것 지키 훨 많 rdquo

질 들 살 고 리 는[ 3]

거운 들 체 리느 는다

답 가 고 게 체 어[ ] ldquo ( ) 級

느 도 계가 니 살 는 것도 고역 지만

살 우는 것 들어 는 살

체 리 고 어도 어도 실 갔다

쑥 어 rdquo

질 거리에 슷 연 여 들[ 4]

보는 간 상 지

답 상 다 체 게 리지 못[ ] ldquo

거 주변에 는 그 거 누 보지

못 고 뻐지고 싶 에 체 리는 에

타 워 지만 는 어울 는 것보다 는

시간 운동만 는 건 니에 사복 lsquo rsquo

고 사복 는 말에 들 웃지만 늘 운동복

고 지내니 사러 갈 도 어 rdquo

질 역도가 말 단 식 운동 니[ 5]

답 가 내는 만 클 업 보[ ] ldquo

그러니 만 쓰는 식 운동 니다

만 다고 거운 것 들 는 건 니거든 연

도 고 가지 동 에 도 여러 가지

복 들

보식 역도 여 미-

zb28) 가 에 대한 설 으( ) 않은 것은

① 시각 우 지 시 에 지

고 망 가는 태도 달 고 다

② 언어 과 언어 복 사 여

담 내 생각 게 는 가

③ 직 감 그 마 것

럼 생생 게 느껴지는 과 주고 간 내

없 리 어 억 게 다

④ 담 내 식 리 여 담 삶 습

과 가 시 여 독 에게 감동과 훈 다

⑤ 직 진 담 직 누

지 못 는 독 에게 생생 상 달 주고

담 욱 게 다

zb29) 나 의 각 의 의도를 설 한 것으 적절하( ) 않

년 학 간고사 대비2013 2 현대고 대비

ECN-0102-2013-001-000076193

은 것은

① 질 담 상 보여 주 것 다1

② 질 담 과 그에 삶 태도 보여2

주 것 다

③ 질 역도 겪는 어 움에 역도3

과 것 다

④ 질 같 연 여 갖는 고민 는지 말4

주 는 것 다

⑤ 질 역도가 과 고 운동 는 것5

담 가 말 주 는 것 다

가 만진 것 다( ) 3

감 달 다고 다 억 에( ) 音感

지워 지만 당시 청 탁 리도

다고 다 드럼 웠다 4

에 갈 마다 드럼 는 리가 신 게 들

다고 다 눈 볼 가 없 니 엔ldquo

는 는 님 틱 에 여 주

다 드럼과 연 맺 과 들 주었다rdquo

식 누 가-

고 싶어

역( ) 도가 말 단 식 운동 니

가 내는 만 클 업에 보ldquo

그러니 만 쓰는 식 운동 니다 만

다고 거운 것 들 는 건 니거든 연

도 고 가지 동 에 도 여러 가지 복

들 시 는 상 상

드는 상 에 맞춰 실 에 는 여러

펼쳐집니다rdquo

략( )

늘 에 는 어 만 것 같

가 에 사 고 사 사ldquo

겠어 든 에 가 경 만 고

울 는 사 겠어 rdquo

보식 역도 여 미-

다 가 운 는 어 어( ) ldquo rdquohelliphellip

월 새벽 시 태 없 거웠고1965 5 6 1

는 없 그 병원에 퇴원 집

가는 마지막 마 고 마 내 거 다

가 죽 간신 에 실 다 사는 어느5 lsquo

가 죽 는 말 가 식 다 신rsquo

상에 각 시키는( )刻印 에 실

어느 가는 후 민 가가 근 었다lsquo rsquo

는 간 과 진실 그 다는( ) ldquo

에 단 평 견 가지고 다 내

가 그리는 간상 단 고 다 지 다 는 그들

가 에 는 평 지 니 그리고 어린

들 미지 겨 그린다rdquo

마 근 간 과 진실 그리고 싶어( )

가 다 근에게 그것 진리 다 거 다 없

거 고 다 없 는 것 진리

다 근 진리는 후 쪽 었다 신산(辛酸 삶)

었 질곡(桎梏 역사 에 지냈)

가 눈에 든 것 료 단 료 게

보 것 었다 그것 그 에 겨우겨우

슬 슬 생 어가는 간들 었

다 리 과 단 리 고리에 검

마 없 거리 돌

상 것 없는 등 근에게 상에

과 진실 엄 다는 사실 리는 가( )儼存

실 고 가 과 역경 에 도 근 내

포 없었 후 보루(堡壘 다 도)

도 간 근365

여 시 것 다

월 강원도 림리에( ) 1914 2 21

삼 독 태어났다 어 근 복

그것 그리 가지 못 다 근 곱 살

지는 산 산업에 실 고 답마 에 내

갔다 근 그림 럼 쫓 다니 가 시 것

다 상 진 것도 가 었다

러 가 에도 고 근 가 꿈꾸었다 근

가 꿈꾸게 것 보통 업

원색도1926 만lsquo rsquo 었다

공주 그림 가 근 경-

zb30) 에 대한 설 가장 른 것은~

① 역도가 과 운동 도 질

② 리는 는 다 lsquo rsquo

③ 들었지만 그럭 럭 는 다 lsquo rsquo

④ 가 게 보 시 말 다

⑤ 보 병 는 지 상 lsquo rsquo

는 말 다

년 학 간고사 대비2013 2 현대고 대비

ECN-0102-2013-001-000076193

시간 많지 다 청량리 생 병원

마지막 상 경 릿 게 들어 다 그 는 십

만 큰 가 상 말 다

지 못 들 마 갈 고 돗

도시민들 싹 싹 탔다 가 시

월에 병원에 원 가 폐 진 몸도4 ( )疲弊

갈 미 지 못 고 었다 가는 얼마( ) 解渴

지 생 에 생각 가

마감 는 신 평생 십 만에

가 과 많 닮 다고 생각 지는

가 운 는 어 어ldquo rdquo 1965helliphellip

월 새벽 시 태 없 거웠고 는5 6 1

없 그 병원에 퇴원 집 가

는 마지막 마 고 마 내 거 다 가

죽 간신 에 실 다 사는 어느 가5 lsquo

죽 는 말 가 식 다 신rsquo

상에 각 시키는 에 실 어느( ) lsquo刻印

가는 후 민 가가 근 었다rsquo

ldquo 는 간 과 진실 그 다는 에

단 평 견 가지고 다 내가 그

리는 간상 단 고 다 지 다 는 그들 가

에 는 평 지 니 그리고 어린 들

미지 겨 그린다rdquo

근 간 과 진실 그리고 싶어 가

다 근에게 그것 진리 다 거 다 없 거

고 다 없 는 것 진리다

근 진리는 후 쪽 었다 신산 삶 ( )辛酸

었 질곡 역사 에 지냈 가 눈에( )桎梏

든 것 료 단 료 게 보 것

었다 그것 그 에 겨우겨우 슬

슬 생 어가는 간들 었다 리

과 단 리 고리에 검 마

없 거리 돌 상

것 없는 등 근에게 상에 과 진실

엄 다는 사실 리는 가 실( )儼存

고 가 과 역경 에 도 근 내 포 없었

후 보루 다 도 도( ) 365堡壘

간 근 여 시 것

간에 지닌 가 근 1914 2

월 강원도 림리에 삼 독21

태어났다 어 근 복 그것 그리

가지 못 다 근 곱 살 지는 산

사업에 실 고 답마 에 내 갔다 근

그림 럼 쫓 다니 가 시 것 다 상

진 것도 가 었다 러 가 에도

고 근 가 꿈꾸었다 근 가 꿈꾸게

것 보통 업 원색1926

도 만 었다lsquo rsquo

그림 가 근 경 공주- ldquo rdquo ( 2009)

zb31) 다음 이 같은 의 성 소에 해당하 않은

것은

사건 평① ② ③

④ 주 ⑤ 경

가 운 는 어 어ldquo rdquo 1965helliphellip

월 새벽 시 태 없 거웠고 는5 6 1

없 그 병원에 퇴원 집 가

는 마지막 마 고 마 내 거 다 가

죽 간신 에 실 다 사는 어느 가5 lsquo

죽 는 말 가 식 다 신rsquo

상에 각 시키는 에 실 어느( ) lsquo刻印

가는 후 민 가가 근 었다rsquo

는 간 과 진실 그 다는 에ldquo

단 평 견 가지고 다 내가 그

리는 간상 단 고 다 지 다 는 그들 가

에 는 평 지 니 그리고 어린 들

미지 겨 그린다rdquo

근 간 과 진실 그리고 싶어 가

다 근에게 그것 진리 다 거 다 없 거

고 다 없 는 것 진리다

근 진리는 후 쪽 었다 신산 삶 ( )辛酸

었 질곡 역사 에 지냈 가 눈에( )桎梏

든 것 료 단 료 게 보 것

었다 그것 그 에 겨우겨우 슬

슬 생 어가는 간들 었다 리

과 단 리 고리에 검 마

없 거리 돌 상

것 없는 등 근에게 상에 과 진실

엄 다는 사실 리는 가 실( )儼存

고 가 과 역경 에 도 근 내 포 없었

후 보루 다 도 도( ) 365堡壘

간 근 여 시 것

간에 지닌 가 근 1914 2

월 강원도 림리에 삼 독21

태어났다 어 근 복 그것 그리

가지 못 다 근 곱 살 지는 산

사업에 실 고 답마 에 내 갔다 근

그림 럼 쫓 다니 가 시 것 다 상

진 것도 가 었다 러 가 에도

고 근 가 꿈꾸었다 근 가 꿈꾸게

것 보통 업 원색1926

도 만 었다lsquo rsquo

공주 그림 가 근 경- ldquo rdquo ( 2009)

년 학 간고사 대비2013 2 현대고 대비

ECN-0102-2013-001-000076193

zb32) 위 을 작성하는 과정에서 되어 활 된 자

어 것은

신 사 료① 연보②

고③ ④ 들과 담

⑤ 에 평

는 간 과 진실 그 다는 에ldquo

단 평 견 가지고 다 내가 그

리는 간상 단 고 다 지 다 는 그들 가

에 는 평 지 니 그리고 어린 들

미지 겨 그린다rdquo

근 간 과 진실 그리고 싶어 가

다 근에게 그것 진리 다 거 다 없 거

고 다 없 는 것 진리다

근 진리는 후 쪽 었다 신산 삶 ( )辛酸

었 질곡 역사 에 지냈 가( )桎梏

눈에 든 것 료 단 료 게 보

것 었다 그것 그 에 겨우겨우

슬 슬 생 어가는 간들 었다

리 과 단 리 고리에 검 마

없 거리 돌 상

것 없는 등 근에게 상에 과

진실 엄 다는 사실 리는 가 실( )儼存

고 가 과 역경 에 도 근 내 포

없었 후 보루 다 도 도( ) 365堡壘

간 근 여 시

것 다

간에 지닌 가 근 1914 2

월 강원도 림리에 삼 독21

태어났다 어 근 복 그것 그리

가지 못 다 근 곱 살 지는 산

사업에 실 고 답마 에 내 갔다 근

그림 럼 쫓 다니 가 시 것 다 상

진 것도 가 었다 러 가 에도

고 근 가 꿈꾸었다 근 가 꿈꾸게

것 보통 업 원색1926

도 만 었다lsquo rsquo

질 루 마 가 도 린다 경건

움 느껴지는 경 다 훗 근 그림에

과 는 거 것( )裸木

만 간과 연 엮어 가는 경건 움lsquo rsquo

니었

같 가가 고 싶었 근에게 그 꿈에 다

가가는 지 다 다 가 지망생들 규 미

상 에 진 고

에 지만 근 다 다 근

미 에 운 것 보통 시 미 시간

다 그런 그에게 없는 연습 가가

통 다 가 귀 시 지 도

얻는 뛸 듯 뻤지만 마 도 가 에

듯 는 었 에 어린 근 주 에

에 그림 그리고 지우고 복( )粉板

시간 가는 게 루 보냈다

근 그 갈 가가 것 열여( )渴求

었 다가 미1932 lsquo rsquo ( lsquo

미 에 다 다는 고 마rsquo) lsquo rsquo

가 근 집 고도 지는 시골 경

그린 그림 다 후 근 에 1943 22

지 미 에 그림 고

에 걸쳐 다 미 근 가

동 는 었다

공주 그림 가 근 경- ldquo rdquo ( 2009)

zb33) 위 의 내 과 일치하는 것은

가 근 가 꿈 포 다①

근 당 가들과 께 에 다②

살 근 가 걷20③

게 었다

④ 만 통 근 역경 겨내는lsquo rsquo

느 다

⑤ 근 간 과 진실 그리 에 그 에

드러 는 간상 단 다

계 시 주 근 건강

걸었다 신 과 간에 상 다 건강

신 는 눈에도 다 근 쪽 눈 뿌 게

보 지 과에 다 다 시 지지 고 결

내 었다 시 지만 마 막막

다 늦어 결 근 쪽 눈 고 말 다

쪽 눈 근에게는 쪽 눈 었고

계 었다 그 근 는 여 그lsquo rsquo

다 근 에 같 그림 그 었다1950

시 그림 는 여 쪽lsquo rsquo

고 어 마주 고 는 그림1963

여 과 동 다 마 복

그린 듯 눈 내리 새 게 다 지

사 다 근 게 복 것

복 상과 타 는 근 상

가 떳떳 단 었고 근 그리고

간 과 진실 에 다가가 가 근다

운 었다 근 신에게 당당 지 그리고

그 다 근 그림에 단 복 보다

년 학 간고사 대비2013 2 현대고 대비

ECN-0102-2013-001-000076193

태 도 그리고 극 보다 과

얻 여 었다 과 통

근 그리고 는 재 고 에 질

만들고 특 것 다

공주 그림 가 근 경- ldquo rdquo( 2009)

zb34) 의 이유에 대해 추 한 것으 적절하 않은 것

상과 타 시도①

보다 과 얻②

근 신에게 당당 지③

④ 간 과 진실 에 다가

⑤ 태 도 얻

근 가가 었지만 그 다니 가

럼 어지지 다 복과 쟁 거쳐 시

는 가 근에게 생계 사 에

운 사 다 에 키에 건( ) 178cm死鬪

체 근 에 동 역 업( )荷役

가 생계 다 쟁

에는 동에 운 상우 주 미

죄 사 에 그림 그리는 시 다 그곳에

에 동 역 업 것에

결 것 럼 보 다 지만 그런 것만도

니었다 그림 그리는 고는 지만 매 근

는 극 간 과 별 없는 경 리 그림

벽에 그리는 것 었다 우도 리 없었다 근

트 는 우 그림 그 다 생

계 그림 단 것 다

후 근 지 신 계 리에 미

엑 리 겼다 근 곳에

건 사 크 에 미 들 ( )

상 상 그 다 근 갖 다 겪

냈다 그리고 결 그 돈

신동에 어 사리 집 마 다 마 ㄷ

루 심 쪽에는 과 엌 쪽에는 건

었다 건 주고 근 가 에

여 살 다 심 에는 지 집어

쓰고 지만 곳 근 가 에게 러웠

보 리 다 근 과 마루 업실 삼 그림

그 다 신동 마루는 근 그림에 등 는 lsquo rsquo

같 상들 지 다 시 고

에 들 폐허가

가 업실 었다

공주 그림 가 근 경- ldquo rdquo( 2009)

zb35) 위 에 대한 설 으 적절한 것은

업 시 여 훈과 감동 다①

에 주 평 드러 다②

사 사 등 식 과 ③

④ 다 근거 시 여 삶에

⑤ 살 시 사 경 께 여

습 시 다

가 시간 많지 다 청량리 생 병원( )

마지막 상 경 릿 게 들어 다 그 는

십 만 큰 가 상 말 다

지 못 들 마 갈 고 돗

도시민들 싹 싹 탔다 가 시

월에 병원에 원4 가 폐( )疲弊

진 몸도 갈 미 지 못 고 었다( )解渴 가는

얼마 지 생 에 생각

가 마감 는 신 평생 십 만에

가 과 많 닮 다고 생각 지는

가 운 는 어 어( ) ldquo rdquohelliphellip

월 새벽 시1965 5 6 1 태 없 거웠고

는 없 그 병원에 퇴원 집

가는 마지막 마 고 마 내 거 다

가 죽 간신 에 실 다 사는 어느5 lsquo

가 죽 는 말 가 식 다 신rsquo

상에 각 시키는 에 실( )刻印

어느 가는 후 민 가가 근 었다lsquo rsquo

다 는 간 과 진실 그 다는( ) ldquo

에 단 평 견 가지고 다 내

가 그리는 간상 단 고 다 지 다 는 가

에 는 평 지 니 그리고 어린 들

미지 겨 그린다rdquo

근 간 과 진실 그리고 싶어( )

가 다 근에게 그것 진리 다 거 다 없

년 학 간고사 대비2013 2 현대고 대비

ECN-0102-2013-001-000076193

거 고 다 없 는 것 진리

다 근 진리는 후 쪽 었다 신산( )辛酸 삶

었 질곡 역사 에 지냈( )桎梏

가 눈에 든 것 료 단 료 게 보

것 었다 그것 그 에 겨우겨우

슬 슬 생 어가는 간들 었다

리 과 단 리 고리에 검

마 없 거리 돌

상 것 없는 등 근에게 상에

과 진실 엄 다는 사실 리는 가 실( )儼存

고 가 과 역경 에 도 근 내 포

없었 후 보루 다( ) 堡壘 도 365

도 간 근 여

시 것 다

마 같 가가 고 싶었 근에게 그 꿈( )

에 다가가는 지 다 다 가 지망생들

규 미 상 에 진 고

에 지만 근 다 다 근

미 에 운 것 보통 시 미 시간

다 그런 그에게 없는 연습 가가

통 다 가 귀 시 지 도

얻는 뛸 듯 뻤지만 마 도 (

는 었 에 어린 근 주 에)

에 그림 그리고 지우고( )粉板

복 시간 가는 게 루 보냈다

zb36) 전 의 성 소가 아닌 것을 고르

① 평 ② 사건 ③ 경

④ ⑤ 훈

늘 지 상에 살고 는 사 들 억 도가10

고 그리 지 통 고 는 사 들( )知的

그보다 훨 많 억 도는 고 지 20

통 다 그런 지 고 2500

그리 간 보는 과 사 에

매우 달 뿐만 니 과 에 도 극

루고 었다 미 운 그런 들

살고 는 동 과 사 들 사고 식에

큰 가 다는 다

고 그리 들 우주 개별 고 독립

사 들 생각 지만 고 들 우

주 연 질 간주 다 같( ) 看做

각 도 들에게는 연 질

었지만 그리 들에게는 미 들 결 었

다 고 과 그리 들 사 같

는 동 과 사 에 도 견 다

지심리 미 마 드 겐트 는

살 들에 에 지 다

연 동 과 상 다 과 같 실험

다 크 만든 미드 도 보

여 주고 그 상 닥 고 주었다lsquo (Dax)rsquo

실 닥 는 재 지 는 것 실험 가lsquo rsquo

만들어 낸 다 그런 다 개 다 체 보

여 주었는 는 미드 지만 틱

만들었고 다 는 재료는 크 지만

달 다 그러고 어 것 닥 지 사 들에게 고 lsquo rsquo

게 니 들 주 같 고 는

체 택 고 동 들 같 재료 만들어진 체

택 다 러 는 심지어 살짜리

들에게 도 타났다 것 곧 과 동

다 상 보고 다는 것 미 다

개별 사 보고 고 동 연 질 보

고 는 것 다

동 들 주변 상 에 맞 어 동 고

에 다 사 들 태도 동에 보다 많

주 울 다 동 가 미시간 에

에 경험 다 그는 미식

경 보러 가게 었는 경 체는 매우 재미 었

주변 들 동에 질 다 그 는

들 계 어 상태 경 다

어 들 에 에 그 시 가 계 가

진 것 다 상 살펴 는 말 들 lsquo rsquo

에 그는 에 시 어 도 뒷사

생각 곧 다시 곤 것 다 그런 그에게 뒷

사 고 지 는 들 동 럼

어 웠다

생각 지도 리 드 니 벳-

zb37) 다음 위 의 내 전개 으 만 인lt gt

것은

lt gt

대조의 통해 대상이 닌 특성을 설 하고 있다

일화를 제 하여 자 의 주장을 뒷 침하고 있다

유추의 을 사 하여 독자의 의해를 돕고 있다

대상이 형성되는 과정을 간적 서에 따라 서 하고 있

① ②

③ ④

년 학 간고사 대비2013 2 현대고 대비

ECN-0102-2013-001-000076193

가 우리가 말 고 쓰는 든 단어가 사 에 는( )

것 니다 사 격에 가 는 지만

어 사 과 같 특별 는 사 니lsquo rsquo

단어 격 보 단어가 사 에

등재 어 다 리 리 사 는 단어 도 그

것 시 사 는 어 고 사 에

격 보 것 니다

러 얼 은 사전에 를 있는가 이에 대한 답lsquo rsquo

은 얼 이 유행어인가 아닌가에 따라 갈라 다 이 단어lsquo rsquo

는 년 어 자 에 랐고 쓰이고 있으2002 lsquo rsquo

유행어라고 하 에는 생 이 다 런데 계속

을 유 하 서 사전에 등재될 자격을 획득할 것인가 이

에 대한 답을 내리 는 히 어 다

여 서 가 를 고 해 볼 있다 첫 는 이 단어

를 써야 할 필 가 속적으 있는가 하는 점이다

상주의 열풍에 휩 인 사회 위 에 편 해서 퍼 말

이 얼 인데 과연 런 위 가 속될 것인가 이에lsquo rsquo

대해 필자의 생각은 정적이다 사회 위 가 뀌

런 말을 쓸 일이 없어 것이다

다음은 단어의 성이다 단어의 성이 사회적으 거

감이 없으 계속 사 될 가능성이 높다 런 에서

얼 은 좋은 조건이 아니다 익히 알 졌듯이 이lsquo rsquo

말은 얼 과 청소년층에서 속어 사 하는 이 결합lsquo rsquo lsquo rsquo

된 말이다 얼 에서 얼 을 리하는 조어 도 lsquo rsquo lsquo -rsquo

어에서는 매 낯선 이다 이것만으 도 거 감을 갖

는 사람들이 있다 더 나 속어 결합한 말이다 얼 lsquo rsquo

이 널리 퍼졌다 해도 은 여전히 청소년층의 속어lsquo rsquo

남아 있다 속어는 자연 럽게 아 자리에서나 쓰 에는

담 러 말이다 러한 담을 하고 사

역을 넓혀 가는 속어도 없 는 않다 특히 얼 은 lsquo rsquo

에도 종종 등장한다 만큼 거 감이 많이 희석되었다

고 할 있다 러나 일상의 자연 러 대화에서도 거

리낌 없이 등장하는가 게 는 되 않았다고 생

각한다

얼 이 유사어인 쌈 등을 만들어 내고lsquo rsquo lsquo rsquo

있으니 살아남을 있을 것이라고 는 견해도 있을 것

이다 러나 간이 나 서 유사어를 포함하여 든

말이 사라 사 는 많다 유사어가 많다는 것이 생 을

유 할 있는 절대적인 조건은 아니다

나 언젠가 터 사람들은 어느 단에서 얼 이 가장( )

쁜 사람을 가리켜 얼 이라고 르고 있다 이 얼lsquo rsquo lsquo rsquo

이라는 단어가 최근 어사전에 라 항간에 논란이 일고

있다 아닌 게 아니라 얼 은 유행어처럼 인다 생 lsquo rsquo

도 리 래되 않은 것 같고 언제 사라 도 알

없다 게다가 젊은이들 사이에서 주 쓰일 뿐이다 이런

단어를 사전에 는다는 게 하 이 없어 이 도

한다

러나 속단은 이다 차근차근 따져 볼 일이다

선 얼 이 일 적 유행어인 아닌 주의 게 들여다lsquo rsquo

볼 필 가 있다 유행어란 유행에 따라 빠르게 유포되었

다가 단 간 내에 소 되는 단어나 를 가리킨다

얼 은 인터넷을 통해 속히 퍼 말이다 하 만 일lsquo rsquo

적인 유행어처럼 단 간 내에 사라 않았을 뿐 아니라

현재 도 잦은 빈도 사 되고 있고 앞으 도 상당

간 사 될 것으 측된다 한 언 재단의 뉴 검 lsquo rsquo

색 사이트에 따르 얼 은 년 에 처음 나타난lsquo rsquo 2001

이후 꾸 히 사 되고 있다

이 같은 사 빈도는 얼 이 일 적 유행어 는 현lsquo rsquo

저히 다르다는 것을 여 다 장 간의 생존 만으 도

얼 은 이 한 어의 어휘 에 를 자격을 얻었다lsquo rsquo

고 할 있다 더 이 이라는 비 적 정제된 매체에

높은 빈도 쓰이고 있 않은가 사 빈도 측 에서

필통이나 연필과 같은 단어 대등하거나 더 많이 쓰lsquo rsquo lsquo rsquo

다는 것은 결코 가 게 볼 일이 아니다

이제는 사전이 언어 현 을 빠르게 하는 게 덕인

대가 되었다 세계적으 유 한 의 사전들도 경쟁

적으 어를 고 있다

하 만 얼 은 젊은이들이나 쓰는 속어라고 흠을 잡을lsquo rsquo

도 르겠다 얼 이 주 젊은 층에서 많이 쓰 lsquo rsquo

는 속어임에 틀림없다 러나 어사전에 표 적이고 품

위 있는 말만 어야 한다고 생각한다 것은 커다란

해다 당장 아 어사전이나 펼쳐 라 속어는

설과 같은 비어나 죄자들이 쓰는 은어 어

마니 같은 소 의 사람만이 쓰는 말 도 라 있

않은가 사전은 말 치에 일정 빈도 이상 나타나는 말이

라 말이든 다 할 있다

zb38) 가 나 에 대한 다음의 설( ) ( ) 않은 것은

① 가 는 얼짱 사 에 등재 것에( ) ( ) lsquo rsquo

보 고 다

② 사 등재 가는 단어 격에( )

고 고 는 언 들 언어 사 도에 고 다 ( )

③ 가 얼짱 어지만 신 과 같 매( ) ( ) lsquo rsquo

체에 도 사 는 말 는 고 다

④ 가는 얼짱 어 보고 크게 가지 근( ) lsquo rsquo 3

거 들어 뒷 고 다

⑤ 는 얼짱 어 는 다 특 다는( ) lsquo rsquo

근거 에도 크게 가지 근거 가 들어 주 2

뒷 고 다

가 늘 지 상에 살고 는 사 들 억( ) 10

도가 고 그리 지 통 고 는 사 들

그보다 훨 많 억 도는 고 지 20

통 다 그런 지 고 2500

년 학 간고사 대비2013 2 현대고 대비

ECN-0102-2013-001-000076193

그리 간 보는 과 사 에

매우 달 뿐만 니 과 에 도 극

루고 었다 미 운 그런 들

살고 는 동 과 사 들 사고 식에

큰 가 다는 다

고 그리 들 우주 개별 고 독립

사 들 생각 지만 고 들 우

주 연 질 간주 다 같 각

도 들에게는 연 질 었지

만 그리 들에게는 미 들 결 었다

고 과 그리 들 사 같 는

동 과 사 에 도 견 다

인 리학자인 츠 이마이 디드 겐트너는 두

살이 채 안 된 아이들에서 터 성인에 이르 다양한

연 대의 동양인과 서양인을 대상으 다음과 같은 험

을 했다 저 코르크 만든 피라 드 양의 도형을

여 주고 대상의 이름을 닥 라고 알 주었다lsquo (Dax)rsquo

제 닥 는 존재하 않는 것으 험자가 임의lsquo rsquo

만들어 낸 이름이다 런 다음 두 개의 다른 체를

여 주었는데 하나는 피라 드 양이 만 하얀 플라 틱

으 만들었고 다른 하나는 재 는 코르크 만 양이

달랐다 러고 나서 어떤 것이 닥 인 사람들에게 고 lsquo rsquo

르게 했더니 서양인들은 주 같은 양을 하고 있는

체를 선택했고 동양인들은 같은 재 만들어 체를

선택했다 이러한 차이는 성인은 어 두 살 리

아이들에게서도 나타났다 이것은 곧 서양인과 동양인은

서 다른 세상을 고 있다는 것을 의 한다 략 ( )

는 아주 단 하 서도 인상적인 험을 했다

험에는 동서양의 대학생들이 참여했다 는 험 참가자

들에게 컴퓨터 화 을 통해 속 장 을 담은 애니 이션

을 여 주었다 화 의 앙에는 초점의 역할을 하는 커

다란 고 한 마리가 있었고 주위에는 다른 생

들과 초 자갈 거품 등이 함 제 되었다 화 을

두 씩 후 참가자들은 자 이 것을 회상해 라는

를 았다

결과 서양인 대학생들과 동양인 대학생 두 앙

의 초점 역할을 했던 고 를 동일한 정도 언 했으

나 경 소 위 거품 초 다른 생 들 에 ( )

대해서는 동양인 대학생들이 서양인 대학생들 다 60

이상 더 많이 언 했다 뿐만 아니라 동양인 학생들은 서

양인 학생들에 비해 개 적인 고 다 전체적인 계

를 더 언 하는 경향을 다 략 또한 경의 일 ( )

를 화 킨 림을 제 하 을 때 동양인 대학생들은 대

경의 화를 알아챘 만 서양인 대학생들은 경

의 화를 거의 알아차리 했다 략 ( )

따라서 서양인들만을 대상으 연 한 화lsquo

편성 결 은 잘 된 것일 도 있다 각 과정과 인rsquo

과정의 어떤 이 화 편적이고 어떤 이

화에 따라 달라 는 는 앞으 많은 연 를 통하여 논의

되어야 한다

나 어떤 의 에서 리 두는 이 화적이다 리( )

안에는 다른 사람들과 더 친 한 계를 유 하 는 상호

의존성과 다른 사람들 터 독립적인 존재 살아가 는

독립성이 혼재한다 따라서 이 에서 어떤 특성이 더 강

하게 각되는 상황에 놓이느냐에 따라 서 다른 화적

특 을 일 있다 결 리 두는 어떤 경 에는

동양인처럼 행동하고 어떤 경 에는 서양인처럼 행동하는

것이다

zb39) 가 에 대한 다음의 설( ) 않은 것은

① 는 신 주 뒷 닥 실험과lsquo rsquo lsquo

니 실험 근거 시 다rsquo

② 동 들 상 간 공통 보다는 에 식

는 강 다

③ 들 주변 맥 에는 심 경 어 사건

과 사건 사 계에 상 민감 다

④ 는 동 과 틀린 지 고 는 것lsquo rsquo

니 다 고 다 lsquo rsquo

⑤ 가에 우리 사 들 개 시 가 원( )

집 경 말 고 는 것 개 보다는

에 고 는 것에 다

늘 지 상에 살고 는 사 들 억 도가10

고 그리 지 통 고 는 사 들( )知的

그보다 훨 많 억 도는 고 지 20

통 다 그런 지 고 2500

그리 간 보는 과 사 에

매우 달 뿐만 니 과 에 도 극

루고 었다 미 운 그런 들

살고 는 동 과 사 들 사고 식에

큰 가 다는 다

지심리 미 마 드 겐트 는 동

과 상 다 과 같 실험 다

크 만든 미드 도 보여 주고 그

상 닥 고 주었다 그런 다lsquo (Dax)rsquo

개 다 체 보여 주었는 는 미드

지만 틱 만들었고 다 는 재료는

크 지만 달 다 그러고 어 것 닥 lsquo

지 사 들에게 고 게 니 들 주 같rsquo

고 는 체 택 고 동 들 같

재료 만들어진 체 택 다 러 는

심지어 살짜리 들에게 도 타났다 것

곧 과 동 다 상 보고 다는

것 미 다 개별 사 보고 고 동

년 학 간고사 대비2013 2 현대고 대비

ECN-0102-2013-001-000076193

연 질 보고 는 것 다

동 들 주변 상 에 맞 어 동 고

에 다 사 들 태도 동에 보다

많 주 울 다 동 가 미시간

에 에 경험 다 그는 미

식 경 보러 가게 었는 경 체는 매우 재

미 었 주변 들 동에 질 다 그

는 들 계 어 상태 경

다 어 들 에 에 그 시 가 계

가 진 것 다 뒷사 고 지 는 들

동 럼 어 웠다

그는 경험에 어 얻어 동 들lsquo

각도 상 본다 는 가 우고rsquo

검 여 주 단 도 상 실험 실

시 다 그는 실험 가 들에게 컴퓨 통

담 니 보여 주었다

에는 역 는 커다 고 마리가 었

고 주 에는 다 생 들과 갈 거 등

께 시 었다 본 후 가 들

신 본 것 상 보 는 지시 다

그 결과 생들과 동 생

역 고 동 도 언

경 거 다 생 들에 ( )

는 동 생들 생들보다 60

상 많 언 다 뿐만 니 동 생들

생들에 개별 고 보다 체 계

언 는 경 보 다 경 변 시

킨 그림 시 동 생들 경

변 지만 생들 경 변

거 리지 못 다

지 지 들만 상 연 lsquo

보편 결 못 것 도 다 지각 과 과rsquo

지 과 어 보편 고 어

에 달 지는지는 많 연 통 여

어 다

리 드 니 벳 생각 지도 사- ldquo rdquo( 2004)

zb40) 위 에 대한 설 으 가장 적절한 것은

① 동 과 생 식 강 고 다

② 가지 실험 통 쓴 고 다

③ 닥 실험에 사 본질에 동 사

상에 주 다

④ 니 실험에 동 과 에 지

각 도에 가 다

⑤ 쓴 는 보편 연 에 드러 우월 에

에 근 고 다

가 동 들 주변 상 에 맞 어 동 고( )

에 다 사 들 태도 동에 보다 많

주 울 다 동 가 미시간 에

에 경험 다 그는 미식

경 보러 가게 었는 경 체는 매우 재미 었

주변 들 동에 질 다 그 는

들 계 어 상태 경 다

어 들 에 에 그 시 가 계 가

진 것 다 상 살펴lsquo 는 말 들rsquo

에 그는 에 시 어 도 뒷사

생각 곧 다시 곤 것 다 그런 그에게

뒷사 고 지 는 들 동 럼

어 웠다

그는 경험에 어 얻어( ) 동 들lsquo

각도 상 본다 는 가 우고rsquo

검 여 주 단 도 상 실험

실시 다 실험에는 동 생들 여 다

그는 실험 가 들에게 컴퓨 통

담 니 보여 주었다 에는

역 는 커다 고 마리가 었고 주 에는

다 생 들과 갈 거 등 께 시

었다 본 후 가 들 신 본 것

상 보 는 지시 다

다 그 결과 생들과 동 생( )

역 고 동 도 언

경 거 다 생 들 에 ( )

는 동 생들 생들보다 60

상 많 언 다 뿐만 니 동 생들

생들에 개별 고 보다 체 계

언 는 경 보 다 들어 동

생들 상 체 연못 럼 보 어ldquo 같rdquo

체 맥 언 시 었지만

생들 상 어 같 큰 고 가 쪽 움ldquo

직 어 같 역 고rdquo

언 시 다 경 변 시킨 그

림 시 동 생들 경 변

지만 생들 경 변 거

리지 못 다

년 학 간고사 대비2013 2 현대고 대비

ECN-0102-2013-001-000076193

게 볼 동 들 보다는 큰 그( )

림 보 에 사 과 체 맥 연결시 지각

는 경 고 체에 특 떼어 내

어 독립 보는 것 낯 어 다 에

들 사 에 고 주변 맥 에는 심 경

에 사건과 사건 사 계에 상

민감 편 다

마 지 지( ) 들만 상 연

보편 결 못 것 도 다lsquo rsquo 지각 과

과 지 과 어 보편 고 어

에 달 지는지는 많 연 통 여

어 다

리 드 니 벳 생각 지도 사- ldquo rdquo( 2004)

zb41) 의 하는 가~ 다른 것은

① ② ③

④ ⑤

얼마 그 에 동 사고 식과

사고 식 보여 주는 내 다

들 에 는 탕 고 같 게

어 겨 고 미 에 는 그 크 럼 큰 고

어리 주고 원 는 어 도 는

상 고 생각 다는 것 다 러

는 어떻게 생 것 고 과 그리 거슬

러 가 보 그 단 다

고 연 경 체 경 생 에

다 벼 사는 공동 업과 경험 많 연 역

에 고 들 연 웃과

게 지내 고 탁 연 들

들 지 연 럽게 들 다 민들

웃과 동 게 뿐만 니 는 집 과

게 다

동 시 는 생태 경 에 살 결과

들 다 사 들 사 상 에 주

울 게 었고 는 곧 체 상 과 간 사

계 시 는 낳게 었다 신 가

가 는 체에 는 원 는 동시

에 다 사 들 그 사 포 체 맥 에

다 들 간 사 연

계 체 계에 주 울 는 사고 체계

게 었다

그러 그리 연 경 그 었다 산

지 연결 는 지 건 그리고 역

에 다 런 들 업에 다 사 과

동 므 공동체에

다고 다 고 그리 들

들과는 달리 보 내 감 지 들과

지 크게 느 지 못 다 그

견 다 경우 주 쟁 통 결 는 갖

게 었다

신 사 간 계들 루어진 커다

트워크 에 게 당연 사 역시 연

계들 체 식 게 다 어 상

원 도 그 개체가 체 맥 과

계 에 고 다 게 체 맥 에 주

울 다 보 상 복 과 가변 식 게 고

상에 재 는 많 변 들 사 에 재 는 들도

게 다 들 주 태도 보

는 경우가 많다 쟁 결

통 결 보다는 통 결

는 보 다

그러 고 그리 들 개개 사 사 독

에 주 울 다 사 사 체에

어 그들 사 에 재 는 공통 규 주

고 다 상 원 에도 사

체 내 주 고 다 그들

체 여 탕 체

는 주 태도 시 고 특 사 어

주에 는지 여 그 주에 는 규

견 다 에 는 쟁 식 리

같 리 사고 체계가 달 게 었다

리 드 니 벳 생각 지도 사- ldquo rdquo( 2004)

zb42) 위 에서 사 된 설 과 가장 유사한 것은

① 크톱 컴퓨 는 본체 니 마우 루

어 다

② 곡과 시 리 는 지 과 사 루어 다는 공통

지니고 다

③ 경 고 것과는 달리

경 본 연 태 그 주변 경

④ 벽돌 능 에 사계 내내

습도가 지 다

⑤ 잰느 체 체 지닌 재 체가 없

는 재 눌 다

년 학 간고사 대비2013 2 현대고 대비

ECN-0102-2013-001-000076193

zb43) 는 립 앙 도서 이 정의 일 이다lt gt

도서 장과 이 자의 리 의 정의 연결이

적절하 않은 것은

lt gt

제 조 서 유8 ( )

도서 장은 다른 이 자의 안전을 위협하거나 도서 의①

서를 란하게 할 가 있는 자에 대하여는 도서 출입

을 제한할 있다

도서 장은 이 자가 제 조 각 호의 어느 하나의 행위를 하7②

을 때에는 이 을 하게 하거나 도서 출입을 제한할

있다

제 조자 의 대출9 ( )

도서 자 는 다음 각 호의 경 대출할 있다①

상호대차도서 간에 자 를 류하는 것을 말한다 등 다1 ( )

른 도서 과의 협 을 위하여 필 한 경

공 이 공 행 상 필 하는 경2

에 도서 장이 필 하다고 인정하는 경3

대출이 가능한 도서 자 의 위는 도서 장이 정하는②

에 따른다

제 조 상10 ( )

이 자가 도서 자 설을 더럽히거나 찢거나 뜨①

쓰게 하거나 잃어 린 경 에는 상하여야 한다

도서 장은 제 항에 따른 상 을 정하여 게 하여야1②

한다

제 조이 절차 등11 ( )

이 칙에서 정한 것 에 도서 자 설의 이 절차

이 제한 등에 필 한 사항은 도서 장이 정한다

출처 립 앙 도서- (httpwwwnlgokr)

① 는 도 리 다8

② 도 는 리 다9 1

③ 료 지 는 도 리 다9 2

④ 도 료 변상에 리10 1

⑤ 는 에 도 리 다11

3

도 다 각 같다①

공 공 다만 연1

연 간 다

매월 째 째 월2

도 도 리 그 사3

가 다고 는

도 에 미리 게1 3②

시 여 다

4

도 시간 도 여 게시 다

5

도 료 시 는 는 도①

지에 등 후

등 에 사 도②

7

는 다 각 여 는 니 다

도 료 시 상 리1 lsquo rsquo

도 료 시 훼 는2 middot

지 가 닌 곳에 식 거 담3

우는

도 보 등 보 검색열4 middot

그 에 도 질 지 여 도5

여 게시 사 는

8

도 다 거 도①

질 게 우 가 는 에 여는 도

도 가 각 어느7②

에는 지 게 거 도

9

도 료는 다 각 경우 다①

상 도 간에 료 는 것 말1 (

다 등 다 도 과 여 경우)

공 원 공 상 는 경우2

그 에 도 다고 는 경우3

가능 도 료 는 도②

는 에 다

10

년 학 간고사 대비2013 2 현대고 대비

ECN-0102-2013-001-000076193

가 도 료 시 럽 거 거①

못 쓰게 거 어 린 경우에는 변상 여

도 에 변상 여 게시1②

여 다

zb44) 위 에서 도서 장이 게 해야 할 사항에 해당하는

것을 두 쓰

년 학 간고사 대비2013 2 현대고 대비

ECN-0102-2013-001-000076193

립 도 규

1 ( )

규 립 도 립 어린 청 도(

포 다 료 시 열 시 말) (

다 에 사 규 립 도)

편 진 다

2 ( )

규 립 도 도 다 에( lsquo rsquo )

고 는 도 에 도lsquo rsquo 2 2

료 에 여 다 다만 특 료 귀

료 등 료 에 사 립 도

도 다 다( lsquo rsquo )

3 ( )

도 다 각 같다①

공 공 다만 연1

연 간 다

매월 째 째 월2

도 도 리 그 사3

가 다고 는

도 에 미리 게1 3②

시 여 다

시간4 ( )

도 시간 도 여 게시 다

등 등5 ( )

도 료 시 는 는 도①

지에 등 후

등 에 사 도②

사 료6 ( )

도 료 시 에 사 료는 도

7 ( )

는 다 각 여 는 니 다

도 료 시 상 리1 lsquo rsquo

도 료 시 훼 는2 middot

지 가 닌 곳에 식 거 담3

우는

도 보 등 보 검색열4 middot

그 에 도 질 지 여 도5

여 게시 사 는

질 지8 ( )

도 다 거 도①

질 게 우 가 는 에 여는 도

도 가 각 어느7②

에는 지 게 거 도

료9 ( )

도 료는 다 각 경우 다①

상 도 간에 료 는 것 말1 (

다 등 다 도 과 여 경우)

공 원 공 상 는 경우2

그 에 도 다고 는 경우3

가능 도 료 는 도②

는 에 다

변상10 ( )

가 도 료 시 럽 거 거①

못 쓰게 거 어 린 경우에는 변상 여

도 에 변상 여 게시1②

여 다

등 규 에 것 에 도11 ( )

료 시 등에 사

도 다

립 도- (httpwwwnlgokr)

zb45) 도서 장의 리 있는 조항으 적절하 않

은 것은

① ② ③ ④ ⑤

년 학 간고사 대비2013 2 현대고 대비

ECN-0102-2013-001-000076193

1 ( )

사가 공 는lsquo rsquo

과 여 사 원과 리

사 타 사 규

니다

개 보 보7 ( )

사는 보통신망 등 계 는 에lsquo rsquo lsquo rsquo

원 개 보 보 니다 개lsquo rsquo

보 보 사 에 는 사 개lsquo rsquo

보 취 니다 다만 사는 다 lsquo rsquo

사 계 통 공 는 경우 원 lsquo rsquo

등 개 보 당 사에 습니lsquo rsquo

원 리에8 (lsquo rsquo lsquo rsquo lsquo rsquo

)

원 에 리lsquo rsquo lsquo rsquo lsquo rsquo①

원에게 가 도 여 는lsquo rsquo 3

니다

사는 원 가 개 보 우 가lsquo rsquo lsquo rsquo lsquo rsquo②

거 사 경우 는 미 에 어 거 lsquo

사 사 운 우 가 는 경우 당rsquo lsquo rsquo

습니다lsquo rsquo

원 가 도 거lsquo rsquo lsquo rsquo lsquo rsquo 3③

가 사 고 지 경우에는 시 사에lsquo rsquo

통지 고 사 내에 니다lsquo rsquo

경우에 당 원 사에 그 사실3 lsquo rsquo lsquo rsquo④

통지 지 거 통지 도 사 내에 지 lsquo rsquo

생 경우 사는 지지 습니다lsquo rsquo

사10 (lsquo rsquo )

사는 과 지 미lsquo rsquo①

에 는 지 계 고

공 여 다 여 니다lsquo rsquo

사는 원 게lsquo rsquo lsquo rsquo lsquo rsquo②

도 개 보 신 보 포 보 보 시( )

갖 어 개 보 취 공시 고

니다

사는 과 여 원lsquo rsquo lsquo rsquo③

견 만 당 다고 경우에는

리 여 니다 원 견 만 사 lsquo rsquo

에 는 게시 거 우편 등 통 여

원에게 리 과 결과 달 니다lsquo rsquo

원11 (lsquo rsquo )

원 다 여 는 니다lsquo rsquo ①

신청 는 변경 시 허 내 등1

타 보 도2

사가 게시 보 변경3 lsquo rsquo

사가 보 보 컴퓨 그4 lsquo rsquo (

등 등 신 는 게시)

사 타 등 지 재산 에5 lsquo rsquo 3

사 타 상 거 업6 lsquo rsquo 3

는 폭 시지 상 타 공7 middot middot

에 는 보 에 공개 는 게시 는lsquo rsquo

사 동 없 리 사8 lsquo rsquo

타 거 당9

게시15 (lsquo rsquo )

원 내에 게시 는 게시 게재 는lsquo rsquo lsquo rsquo lsquo rsquo

경우 원 사가 게시 복 lsquo rsquo lsquo rsquo lsquo rsquo middot middot

등 태 언 등에 공 는

것 내에 다 원 본 게시 등 lsquo rsquo lsquo rsquo

크 능 등 여 복 는 등 태

는 것 동 것 니다

- (wwwnavercom)

zb46) 위 은 인터넷 포털사이트의 회 가입을 위한 이

약 의 일 이다 이 약 을 만드는 과정에서 생각한

내 으 적절하 않은 것은

개 보 보 가 지에 별 눠①

겠어

원 가 만들게 에②

시 주어 겠어

원들 게재 게시 다 원 크 다③

는 것 지

④ 원 지 는 뿐만 니 사가 지 는

도 께 달 지

리에 가 생 경우 사가⑤

에 다는 도 듯

1 ( )

사가 공 는lsquo rsquo

과 여 사 원과 리

사 타 사 규

년 학 간고사 대비2013 2 현대고 대비

ECN-0102-2013-001-000076193

니다

개 보 보7 ( )

사는 보통신망 등 계 는 에lsquo rsquo lsquo rsquo

원 개 보 보 니다 개lsquo rsquo

보 보 사 에 는 사 개lsquo rsquo

보 취 니다 다만 사는 다 lsquo rsquo

사 계 통 공 는 경우 원 lsquo rsquo

등 개 보 당 사에 습니lsquo rsquo

원 리에8 (lsquo rsquo lsquo rsquo lsquo rsquo

)

원 에 리lsquo rsquo lsquo rsquo lsquo rsquo①

원에게 가 도 여 는lsquo rsquo 3

니다

사는 원 가 개 보 우 가lsquo rsquo lsquo rsquo lsquo rsquo②

거 사 경우 는 미 에 어 거 lsquo

사 사 운 우 가 는 경우 당rsquo lsquo rsquo

습니다lsquo rsquo

원 가 도 거lsquo rsquo lsquo rsquo lsquo rsquo 3③

가 사 고 지 경우에는 시 사에lsquo rsquo

통지 고 사 내에 니다lsquo rsquo

경우에 당 원 사에 그 사실3 lsquo rsquo lsquo rsquo④

통지 지 거 통지 도 사 내에 지 lsquo rsquo

생 경우 사는 지지 습니다lsquo rsquo

원에 통지9 (lsquo rsquo )

사는 특 다 원에게 통지 경우lsquo rsquo lsquo rsquo

공지 게시 통 상 게시 개별 통지에7

갈 습니다

사10 (lsquo rsquo )

사는 과 지 미lsquo rsquo①

에 는 지 계 고

공 여 다 여 니다lsquo rsquo

사는 원 게lsquo rsquo lsquo rsquo lsquo rsquo②

도 개 보 신 보 포 보 보 시( )

갖 어 개 보 취 공시 고

니다

사는 과 여 원lsquo rsquo lsquo rsquo③

견 만 당 다고 경우에는

리 여 니다 원 견 만 사 lsquo rsquo

에 는 게시 거 우편 등 통 여

원에게 리 과 결과 달 니다lsquo rsquo

원11 (lsquo rsquo )

원 다 여 는 니다lsquo rsquo ①

신청 는 변경 시 허 내 등1

타 보 도2

사가 게시 보 변경3 lsquo rsquo

사가 보 보 컴퓨 그4 lsquo rsquo (

등 등 신 는 게시)

사 타 등 지 재산 에5 lsquo rsquo 3

사 타 상 거 업6 lsquo rsquo 3

는 폭 시지 상 타 공7 middot middot

에 는 보 에 공개 는 게시 는lsquo rsquo

사 동 없 리 사8 lsquo rsquo

타 거 당9

원 계 규 내lsquo rsquo lsquo②

여 공지 주 사 사가 통지 는rsquo lsquo rsquo

사 등 여 타 사 업 에 lsquo rsquo

는 여 는 니다

- (wwwnavercom)

zb47) 위 약 의 조항에서 같은 제점을 하lt gt

고 있는 조항은

lt gt

제휴 회사에 회 의 아이디 개인 정 를 전송할 있도

한 조항은 고객에게 당한 조항이다

1 7 8① ② ③

④ 9 ⑤ 10

립 도 규

1 ( )

규 립 도 립 어린 청 도(

포 다 료 시 열 시 말) (

다 에 사 규 립 도)

편 진 다

2 ( )

규 립 도 도 다 에( lsquo rsquo )

고 는 도 에 도lsquo rsquo 2 2

료 에 여 다 다만 특 료 귀

료 등 료 에 사 립 도

도 다 다( lsquo rsquo )

3 ( )

도 다 각 같다①

공 공 다만 연1

연 간 다

년 학 간고사 대비2013 2 현대고 대비

ECN-0102-2013-001-000076193

매월 째 째 월2

도 도 리 그 사3

가 다고 는

도 에 미리 게1 3②

시 여 다

시간4 ( )

도 시간 도 여 게시 다

등 등5 ( )

도 료 시 는 는 도①

지에 등 후

등 에 사 도②

사 료6 ( )

도 료 시 에 사 료는 도

7 ( )

는 다 각 여 는 니 다

도 료 시 상 리1 lsquo rsquo

도 료 시 훼 는2 middot

지 가 닌 곳에 식 거 담3

우는

도 보 등 보 검색열4 middot

그 에 도 질 지 여 도5

여 게시 사 는

질 지8 ( )

도 다 거 도①

질 게 우 가 는 에 여는 도

도 가 각 어느7②

에는 지 게 거 도

료9 ( )

도 료는 다 각 경우 다①

상 도 간에 료 는 것 말1 (

다 등 다 도 과 여 경우)

공 원 공 상 는 경우2

그 에 도 다고 는 경우3

가능 도 료 는 도②

는 에 다

변상10 ( )

가 도 료 시 럽 거 거①

못 쓰게 거 어 린 경우에는 변상 여

도 에 변상 여 게시1②

여 다

등 규 에 것 에 도11 ( )

료 시 등에 사

도 다

립 도- (httpwwwnlgokr)

zb48) 다음 정 리 의 의 으 볼 때 가장

이 적인 것은

도 시간 도 여 게시 다①

등 에 사 도②

가능 도 료 는 도 는③

에 다

④ 도 에 변상 여 게10 1

시 여 다

⑤ 도 가 각 어느7

에는 지 거 도

zb49) 를 참고하여 이 어의 성격을 설 한lt gt

것으 적절하 않은 것은

① 보 에 는 어 시 상 고 어 시lt gt lsquo rsquo

에 보여주고 다

② 진 어 어원에 견 고 다

에는 타 어 들어가는 것 다 lsquo rsquo

③ 에 들어갈 말 각각 고 어 어 신 어~

들 언어는 질 격 강 통 없었다

④ 시 우리 에 가 었지만 지 계

과 달리 들 통 사 달 어 웠

년 학 간고사 대비2013 2 현대고 대비

ECN-0102-2013-001-000076193

⑤ 크 몽골 만주 공통어가 우리 어 같

계열에 다는 에 사 특 짐

가( )

善化公主主隱 공주님

他密只嫁良置古 몰 결 고

薯童房乙 맛

夜矣卯乙抱遣去如 에 몰 고 가다

( )

始汝 會隱日恚見隱扐 만 에 본

恥隱汝衣淸隱笑 맑 웃

고 시 여 공 크다 만 다[ ] ( ) ( ) ( ) ( )始 汝 會扐

내다 에 보다 견( ) ( )恚 見 다( )隱

럽다 맑다 청 웃( ) ( ) ( ) ( )恥 衣 淸 笑

zb50) 위의 나 를 함 고 음에 답하( ) lt gt

보lt gt

( )素那或云金川 白城郡蛇山人也

운 사산

는 고 다 는( )[ ( ) ] (素那 金川 白城

사산 사 다) ( ) 郡 蛇山

삼 사- lsquo rsquo 47

에 제 된 단어 의 표 리를 조건(1) lt gt ( ) lt gt

에 맞게 서 하

건lt gt

lsquo 었고 었다 태rsquo

에 제 된 단어 동일한 표 리에(2) lt gt ( )

의해 적은 것을 나 에서 찾아 조건 에 맞게 서 하( ) lt gt

건lt gt

에 당 는 각각( ) 개 쓸 것2 단

당 는 가 여러 개 어도 개만 쓸 것 각2

개 과 도 쪽에 개만2 2

드시 지 것( )

과 동 원리 것lsquo 고

과 동 원리 것 다rsquo

태 것

가( )

素那(或云金川) 白城郡蛇山人也

소나 또는 천 이라 한다 는 성 사( ) ( ) ( )素那 金川 白城郡〔 〕

산 사람이다 현대어 풀이( ) ( )蛇山

나( )

紫布岩乎希 회

執音乎手母牛放敎遣 자 손 암쇼 노히 고

吾 不喩慙 伊賜等肹 肹 나 안디 리샤

花 折叱肹 可獻乎理音如 고 것거 도림다

다 향찰은 리말을 리 으 적은 표 이었 만 생( )

은 고 대를 넘 하고 끊어 고 말았다 랜 세

동안 갈고 닦아 체계적이었던 향찰 표 이 사라졌

을 인은 크게 두 가 나누어 생각해 볼 있다

하나는 족 사회의 한 선호도에서 찾을 있다 라 때

향찰은 주 족 계 에서 사 했을 것으 인다 한 을

알 하고서는 한자를 활 하여 리말을 리 으 표

하 란 가능하 때 이다 런데 족들은 간이 흐

를 향찰과 같은 리 표 을 익혀 사 하 다는

아 한 을 대 사 하는 쪽을 선호하게 되었다 더 이

고 초에 인재 등 을 위해 과거제도가 행되 서 한 선

호도가 더 높아졌고 결 향찰은 소 되고 말았다

또 다른 가능성은 한 어의 특성에서 찾을 있다

터 한 과 일 세 나라는 한자 화 에 속해 다

당연한 이야 겠 만 표의 자인 한자는 어를 표 하

에 매 적절하다 어의 음절은 성 ( ) ( )聲母 韻母

이 어 고 여 에 성조가 추가되어 최종 소리가 결정된

다 래서 어는 단음절을 하나의 한자 표 하 된

다 에 초성 성 종성의 세 가 소가 하나의 음절

년 학 간고사 대비2013 2 현대고 대비

ECN-0102-2013-001-000076193

을 이 는 한 어는 음절 조가 잡하고 음절의 가 많아

서 한자 차 만으 한 어의 소리를 만족 럽게 표 할

없었다 를 들어 한 어에서는 어 니 같이 음절 lsquo rsquo

이 어 단어가 얼마든 있으나 어는( ) 複數音節

자 하나 나타내 만이다lsquo [m ]rsquo 母 ǔ

한편 일 어의 표 은 핵 적 단어는 한자 적고 토는

가나라는 일 의 자 적는 이다 적인 의 를 나

타내는 은 표의 자인 한자 적고 적 계를 나

타내는 토는 표음 자 적는 셈이니 자세히 살펴

리의 향찰 표 을 쏙 빼닮았음을 알 있다 한 어 같

은 착어이 서도 일 어에만 향찰과 유사한 표 이 살아

남은 것은 일 어의 특 때 이다 일 어는 하나의 자음과

음의 결합으 음절을 이 고 침이 거의 없는 음절 언어

이다 이러한 음절의 특색에다가 토가 달한 착어라는 점

이 향찰과 유사한 표 이 살아남을 있는 비결이었다

하 만 같은 착어라도 다양한 음소 침이 달한 한

어는 향찰 표 하는 데 근 적으 한계가 있었다

zb51) 다 하여 의 행에 대한 탐 한 결과( ) lt gt 2

않은 것은

보lt gt

善花公主主隱 공주니믄 공주님( )

----------------------------------------

-

他密只嫁良置古 그 지 얼어 고 몰 결(

----------------------------------------

-

薯童房乙 맛 맛( )

夜矣卯乙抱遺去如 몰 고 가다 에 몰 고(

가다)

주동 역 동- (薯童謠『 』

에 2 ( )他密只嫁良置古

얼다 시집가다 결 다 말 lsquo rsquo

① 실질 미 지니고 므 타 타lsquo ( )rsquo lsquo [ ]

② 에 실질 미 타내고 지 는lsquo rsquo lsquo [ ]rsquo lsquo [ ]密只 密 只

계 타내는

③ 얼어는 실질 미 포 고 므 가lsquo rsquo lsquo [ ]rsquo嫁

것lsquo [ ]rsquo 良

④ 고 어간 는 실질 미 지니고 므lsquo rsquo lsquo -rsquo

것lsquo [ ]rsquo 置

⑤ 고 어미 고는 계 타내고 므lsquo rsquo lsquo- rsquo

고 것lsquo [ ]rsquo 古

가( )

엉 훈 민middot middot middot middot middot世 宗 御 製 訓 民 正 音

말 미 듕 귁에 달middot middot middot middot middot middot middot middot中 國 文 字

니 런middot middot middot middot middot middot 어린middot middot middot middot百 姓

니 고 도 내 들middot middot middot middot middot middot middot middot middot 시러middot

펴 몯middot 미middot middot 니 내middot middot middot middot middot middot middot middot 爲

어엿middot 겨 새middot middot middot 믈여듧middot middot middot middot字 니middot middot middot

사 마다 니겨 킈 middot middot middot middot middot middot middot middot middot便 安

고 미니middot middot middot middot

본 는 상( ) (象

원리에 만들어진 본) ( )形 ㄱ ㄴ ㅁ ㅅ ㅇ

에 는 가 원리에( )加劃

그리고( )ㅋ ㄷ ㅌ ㅂ ㅍ ㅈ ㅊ ㆆ ㅎ

쓰는 병 원리에 만들어진( )竝書

마지막 체( ) ( )異體ㄲ ㄸ ㅃ ㅆ ㅉ ㆅ

ᅀ 다 상 원리에 ㅇ ㄹ

지 는 삼재 상 본 본( ) ( ) ( 天地人 三才

탕 므림과 림에 ) (初ㅡ ㅣ

재)( ) ( )( )出字 再出字ㅗ ㅏ ㅜ ㅓ ㅛ ㅑ ㅜ ㅕ

병 그리고 들 에 다시( )ㅘ ㅝ ㅣ

( )ㅣ ㅢ ㅚ ㅐ ㅟ ㅔ ㆉ ㅒ ㆌ ㅖ ㅙ ㅞ

zb52) 가 에 대한 설 으 르 않은 것을( ) 두 고르

① 어쓰 규 지키고 다

② 리 고 다

③ 말 미 미 등 어 사 다lsquo rsquo

④ 개 지 다

년 학 간고사 대비2013 2 현대고 대비

ECN-0102-2013-001-000076193

⑤ 어 원 에 가 도 고 다

엉 훈 민世 宗 御 製 訓 民 正 音

말 미 듕귁에 달 니

런 어린 니 고 도middot

내 들 시러 펴 몯 미 니middot

내 어엿 겨 새 믈여듧

사 마다 니겨middot 킈 고

미니

훈민 언 본- lsquo rsquo 5 (1459 )

zb53) 위의 에 대한 현대어 풀이가 르~ 않은 것

① 우리 말 과 달

② 어리 말 고 는 것 어도

③ 신 생각 마 껏 펼 는 사 많다

④ 게 생각 여

⑤ 사 마다 게

zb54) 훈민정음 언해 에는 한 을 창제한 동 가 드러나

있다 훈민정음 창제의 정 과 내 이 잘 연결된 것

① 주 신 말 미 듕귁에 달

② 민 신 내 어 겨

③ 신 뻔 킈 고 미니

④ 실 신 사 마다 니겨

⑤ 귀 신 계 주 는 훈민 신과 거리가

가 엉 훈 민( ) middot middot middot middot middot世 宗 御 製 訓 民 正 音 

말 미 귁에 中 國 달 文 字

니 런 어린 니 百 姓

고 도 내 들 시러 펴 몯

미 니 내 어엿 爲 겨 새

믈여듧 니 사 마다 니 字

겨 킈 고 미니 便 安

훈민 언 본- lsquo ( )rsquo ( ) 5 (1459 )訓民正音 世祖

( )

[ 1 ]

동 룡 샤 마다 복( ) ( ) ( )海東 六龍 天福

시니 고 동( ) ( )古聖 同符 시니

[ 2 ]

매 니 곶 여

미 므 니 그 내 러

가 니

[ 125 ]

우 미리( )千世 샨( )定 에( )漢水北 累仁

누 개 샤 복 업 시니( ) ( ) 開國 卜年

신( )聖神 니 샤도 경 근민 샤 욱( )敬天勤民

드시리 다

님 쇼 산 가( ) ( )洛水 山行

미드니 가

어 가- lsquo ( )rsquo 27龍飛御天歌

다 우리신 니쓰고 다만 만 쓰( )

거 샹 귀쳔 다보게 러 귀

여 쓴 도 신 보 가 고 신 에

말 어 보게 각 에 사 들

고 본 몬 능통 후에

죠 죠 니

드 도 만 공 에 사

드 미 죠 고 고 여 보 죠

보다 얼마가 거시 어신고 니 첫

가 죠 니 죠

민 들 어 신 샹

귀쳔 도보고 어보 가 만 늘

고 폐 에 만쓴 죠 민

도 러보지못 고 보니 그게 엇지

심 니 리 보 가 어 운건 다

니 쳣 말마 지 니 고 그

쓰 에 가 우 지 지

몰 거 본후에 가 어 지

고 그니 쓴편지 쟝 보

년 학 간고사 대비2013 2 현대고 대비

ECN-0102-2013-001-000076193

쓴것보다 듸 보고 그 마 니 쓴 고

어 못

그런고 에 리 과 가

만 쓴 못 민 말만 듯고

고 편 그 못 보니 그사 단

병신 못 다고 그사 식 사

니 만 고 다 과 그사

만 고 다 과 업 사 보다 식 고

죠 도 고 각 과

견 고 실 직 귀쳔 간에 그

고도 다 것 몰 귀죡 보다

사 우리 신 귀쳔 다 업

시 신 보고 과 지 게 랴

시니 샹 귀쳔 간에 우리 신 걸

간 보 새지각과 새 걸 미리

독립신- lsquo (1896)rsquo

zb55) 친 어 나의 제 장( ) 2 매 함축적

의 가 가장 유사한 것은

① 지 눈 내리고 매 득 니 내 여 가

사- lsquo rsquo

② 도 어 리듯 그 게 어 다

주 사- lsquo rsquo

③ 눈 살 다 죽 어 린 과 체 여

눈 새벽 지 도 살 다

눈- lsquo rsquo

④ 삶 근심과 고단 에 돌 거니는 여 거 는

여 리 내린 살가지 에 눈 리 눈 리

택 그 생 에- lsquo rsquo

⑤ 늘 러 고 러

청룡 룡 어 개 루 우

신경림 계- lsquo rsquo

zb56) 친 를 위 가 나 에 나타난A B ( ) ( )

세 어의 특 에 의거하여 세 어 표 하

그 산 고 공 도 맑지만

A

주변에 쓰 리는 어리 사 많다

B

건lt gt

식 가 에 타 어 특징에( ) ( )

거 과 어쓰 는 고 지 말 것

A

B

zb57) 가 의( ) 달 아ㆍ 다 의 ( ) 나셔에서 알 있는

세 어 개화 어의 특 을 비 하여 조건 에lt gt

맞게 서 하

건lt gt

어에 는lsquo 개

어에 는 다 태rsquo

zb58) 은 가 는 다 에 나 는 절lt 1gt ( ) lt 2gt ( )

일 를 췌한 것이다 의 의 가 lt 1gt (1)~(2)

유사한 말을 에서 찾아 쓰lt 2gt

보lt 1gt

런 (1) 어린 니 고百 姓

도 내 들 시러 펴 몯 미

사 마다 (2) 니겨 便 安

킈 고 미니

보lt 2gt

죠 고 고 여 보 죠

보다 얼마가 거시 어신고 니 첫 가

죠 니 죠 민

들 어 신 샹 귀쳔

도보고 어보 가 만 늘 고

폐 에 만쓴 죠 민 도

러보지못 고 보니 그게 엇지 심

니 리

년 학 간고사 대비2013 2 현대고 대비

ECN-0102-2013-001-000076193

lt 1 gt

동 룡 샤 마다 복 시( ) ( ) ( )海東 六龍 天福

고 동 시니( ) ( )古聖 同符

lt 2 gt

(A) 매 니 곶

여 니

미 므 니 그 내

러 가 니

lt125 gt

우 미리 샨 에( ) ( ) ( ) 千世 定 漢水北 累

누 개 샤 복 업 시 니( ) ( ) 仁開國 卜年 聖

신( ) 神 니 샤도 경 근민 샤( ) 敬天勤民

욱 드 시 리 다

님 쇼 산 가 ( ) ( )洛水 山行

미드니 가

- lt gt龍飛御天歌

zb59) 장과 내 상 유사한 성격의 조는125

① 뫼 고 고 고 고

어 그린 많고 많고 고 고

어 러 는 울고 울고 가느니

도 견- lt gt

② 강 에 드니 몸 다

그믈 고 가니

뒷 뫼 엄 언 니( )藥

-

③ 말 없는 청산 태 없는 다

값 없는 청 없는 월

에 병 없는 몸 별 없 늙 리

-

④ 가마귀 골에 가지 마

낸 가마귀 새

청강에 것 시 몸 러 가( ) 淸江

-

⑤ 진 골에( ) 白雪

가 매 는 어느 곳에 었는고

에 갈 곳 몰( ) 夕陽

색-

zb60) 위 에 나타난 세 어의 특 으 적절하 않은

것은

① 룡 어 주격 사에 당 는 가 사( ) lsquo rsquo六龍

고 다

② 샤 어에도 어 주체 쓰 다

는 것 다

③ 매 어 달리 사 택에 어

가 지 지지 고 다

④ 므 원 상 직 어 지 다

⑤ 드시리 다 주체 과 상 께 사

고 다

수고 하셨습니다hearts hearts

년 학 간고사 대비2013 2 현대고 대비

ECN-0102-2013-001-000076193

보닷컴에 공 는 별 보는 고등

들 여 주 는

들 습니다 슷 동 지

가 복 는 것 도가

니 복 여 습 시고 거 시

니다

정답 해설

1) 정답[ ] ④

해설 다른 것은 두 특정 업이나 단 내에서 사[ ]

하는 일종의 은어 사회 언에 해당한다 러나

는 언이 아니라 단과대학을 여서 단대 사lsquo rsquo lsquo rsquo lsquo④

대학을 여서 사대라고 한 말에 해당하 일rsquo lsquo rsquo

사회에서도 널리 쓰이 사회 언이라 할

없다

2) 정답[ ] ⑤

해설 사회 언은 같은 단 내에서 쓰이는 언어이[ ] lsquo rsquo

동일 단끼리는 단결 과 친 감을 형성하는

능을 하 리적 안감이 일어나 않는다

3) 정답[ ] ③

해설 사람이라는 차 적 표현에 대한 대안적 표현이[ ]lsquo rsquo

인 아내 처 등으 볼 있다lsquo rsquo

4) 정답[ ]⑤

해설 남성은 주 격 체를 사 한다[ ]

5) 정답[ ] ⑤

해설 흑인은 검다라는 뜻을 가 고 있을 뿐 인[ ]lsquo rsquo lsquo rsquo lsquo rsquo

다 열등한 뜻을 내포하 않는다

6) 정답 살 색 첫 작품[ ] - -

해설 살색 혹은 킨색은 한 인의 피 색을 뜻[ ] lsquo rsquo lsquo rsquo

하는 것으 인종 차 을 추 고 출 이주민

의 평등 을 침해할 있어 년 표 이2005

살 색으 이름을 꾸었다 처녀작은 처녀라lsquo rsquo lsquo rsquo lsquo rsquo

는 단어가 가 고 있는 곡된 성 인 을 한 것

으 첫 작품정도 꾸어 사 하는 것이 좋다lsquo rsquo

7) 정답[ ] ⑤

해설 호는 아들에게 해체를 사 하고 있다[ ] ① ②

장 을 성하는 청자는 자 의 아 느리 아lsquo

들 세 이다 호는 아 느리에게 해rsquo ③

체를 사 하고 있다 호가 느리 아 에게 ④

사 한 해 체 아들에게 사 한 해체는 두 비lsquo rsquo lsquo rsquo

격 체에 해당한다 호는 자 의 아랫사람인 ⑤

느리에게 아들과 마찬가 해체를 사 하는 것이

상 이 만 임 을 한 느리에게 고마 과 쁨

존 의 표 를 하 위해 자 의 아 에게 말하듯

해 체를 사 하고 있다

8) 정답[ ] ③

9) 정답[ ] ⑤

10) 정답[ ] ①

해설 청자 할아 가 장의 주체 아 다 높을[ ] ( ) ( )

경 에는 압존 에 의해 장의 주체를 높이 않는lsquo rsquo

다 러 아 서가 아닌 아 는으 계 lsquo rsquo lsquo rsquo lsquo

니다 가 아닌 있 니다 표현하는 것이 르rsquo lsquo rsquo

11) 정답 당이 당을 쫒았다 당이[ ]

당에 다

해설[ ]

12) 정답[ ] ⑤

해설 서 다른 높임표현을 통해 청자에 대해 리[ ] ⑤

적 거리감을 나타내는 인 은 이 아니라 현정이

다 가 에서 현정은 에게 해 체를 사 함으 써 ( )

친근감을 드러낸다 나 에서 연 을 게을리하는 역 ( )

도 들 때 에 화가 난 현정이 선생님에게 항의하

는 장 에서는 하 체를 사 하여 리적 거리lsquo rsquo

가 어졌음을 나타내고 있다

13) 정답[ ] ①

해설 는 는 얼 빛이 날과 어찌 다르 고[ ] lsquo rsquo

라는 뜻으 전과 달리 임이 화자를 않고

있음을 알 있다

14) 정답 달리 후 가 있다 이를 통해 경[ ] lt gt

쾌한 음악성을 형성하고 노 젓는 상황을 체적으

형상화하는 역할을 한다

15) 정답[ ] ①

16) 정답[ ] ⑤

해설 다 의 자연은 를 성찰하게 하는 대상[ ] ( )⑤

이자 정의 대상이다 의 자연은 자 의 상황과 ⑤

처 를 드러내는 경으 서의 역할을 하 이

이 없다

17) 정답[ ] ③

해설 는 빈천 을 해결하고자 했으나 강산[ ] lsquo ( )rsquo 貧賤③

과 풍 을 달라는 에 거절하 다고 함으 써 자

연에 대한 애정을 드러내고 있으 는 않는

임에 대한 망을 개에게 전가 켜서 임에 대한 리

을 드러내고 있다

18) 정답[ ] ③

년 학 간고사 대비2013 2 현대고 대비

ECN-0102-2013-001-000076193

19) 정답[ ] ⑤

해설 고상한 음악가의 이름을 리말 꽝 럽[ ]

게 꿈으 써 언어유희를 통해 음을 유 하고 있

다 이는 고상한 척하는 총 를 비꼼으 써 비판적

태도를 드러내는 것이 대상을 꽝 럽게 표현

하여 총 의 허 과 사치를 풍자하고 있다

20) 정답[ ] ⑤

해설 는 작품 속 경에 대한 설 이 드러나는 것이[ ]

서 자의 주 적인 견해가 접적으 드러나는 것이

아니다

21) 정답[ ] ⑤

22) 정답[ ] ②

23) 정답[ ] ④

24) 정답[ ] ①

해설 적강 티프는 주인공의 비 한 출생이나 능[ ] ①

과 이 있는 것으 조정의 능함을 풍자하는lsquo rsquo

것과는 거리가 다

25) 정답 픔 나[ ] ( )

해설 의 음악은 고통 는 사람들을 위 하고 아픔[ ] lsquo rsquo

을 치유해 주는 능을 한다고 할 있다 의 lt gt

픔 도 소 된 이 과 더 어 살아가는 따뜻한 마음lsquo rsquo

을 상 한다

26) 정답[ ] ⑤

해설 에게 선천적으 주어 각 장애라는 역경[ ]

은 의 이라는 가사 연 을 있다lsquo rsquo

27) 정답[ ] ④

해설 는 장 란 선 에게 은 개인적인 인상을[ ]

소녀 장정 등으 표현한 것이다lsquo rsquo

28) 정답[ ] ②

해설 담자가 피 담자의 언어적 표현이나 비언어[ ]②

적 표현 하 독자는 담의 위 나 피

담자의 감정 상태를 알 있다 이를 통해 독자는

담 상황을 더 생생하게 느낄 있고 피 담자

를 더 잘 이해할 있게 된다

29) 정답[ ]③

해설 일상생활과 역도 선 서의 성과에 된 것에서[ ]

역도를 하 서 겪는 어 과 내적 고민으 화제를

전화하 위한 것이다

30) 정답[ ] ①

해설 릿속에 새겨 넣듯 이 억되도 함 세상[ ] ② ③

살이가 힘들고 고생 러 속 하여 자유를 ④

가 없는 고통의 상태를 비유적으 이르는 말

적의 침입을 막 위해 쌓은 축 켜야 할⑤

대상을 비유적으 이르는 말이다

31) 정답[ ] ④

해설 이 의 종류는 전 으 인 사건 경[ ] lsquo

비평을 성 소 삼는다rsquo

32) 정답[ ] ④

해설 근은 삼대독자 태어났음을 에서 확인할[ ]

있다 형제들과의 담은 이뤄 가 없다

33) 정답[ ] ⑤

해설 근은 가난에도 하고 화가를 꿈꾸었다[ ] (3

단 또한 다른 화가 망생들은 정 육을)

위해 상 학 학 해 유학 에 랐 만

근은 다른 을 찾아야 했다 단 세에(5 ) 18

근은 조선 전람회에 입선하 다 단 의(6 )

만종은 인간과 자연이 엮어 가는 경건한 조화 을lsquo rsquo

나타낸다

34) 정답[ ] ①

해설 근이 속에서도 창작활동을 추 않고[ ]

하는 닭은 은 세상과 타협할 르는

근이 세상의 이해를 하 위한 가장 떳떳한 단

이 때 이다

35) 정답[ ] ⑤

해설 전 은 서 자의 주 적인 평이 리는 것이[ ]

만 위 제 은 인 이 살았던 대 사회적 경

을 통해 객 적인 인 의 을 제 하고 있다

36) 정답[ ] ⑤

해설 전 은 인 사건 경 비평이라는[ ] lsquo rsquo⑤

성 이 어져 있다

37) 정답[ ] ①

해설 이 은 동양인과 서양인의 사고 에 차이가[ ]

있다는 것을 대조를 통해 설 하고 있다 또 쓴이

의 제자가 축 경 를 러 가서 경험한 일화를

통해 동양인이 서양인에 비해 주 상황에 더 많은

주의를 인다는 주장을 뒷 침하고 있다

38) 정답[ ] ④

39) 정답[ ] ②

40) 정답[ ] ②

41) 정답[ ] ④

42) 정답[ ] ③

43) 정답[ ] ④

44) 정답 도서 의 휴 일 도서 의 이 간 도서의[ ]

해설 도서 장은 임의 정한 휴 일과 도서 이[ ]

간 도서의 상 등을 게 할 의 가 있다

년 학 간고사 대비2013 2 현대고 대비

ECN-0102-2013-001-000076193

45) 정답[ ] ①

해설 제 조의 정 휴 일 의 휴 일의 사전 게[ ] 3

는 도서 장의 의 조항에 속한다

46) 정답[ ] ①

해설 개인 정 호 의 를 제 하 했 만 항[ ]

나눠서 제 하 않고 대 나열하고 있다

47) 정답[ ] ②

해설 제 조의 내 을 회사는 다른 회사 협[ ] 7 lsquo

계약을 통해 서비 를 제공하는 경 회 의 아이디

등 개인 정 를 해당 회사에 전송할 있다는 내rsquo

이 있으 의 제점을 제 할 있다②

48) 정답[ ] ④

해설 는 도서 장의 의 에 해당하고 나 는 도[ ] ④

서 장의 리에 해당한다

49) 정답[ ] ③

50) 정답 은 음독으 적었고 은 훈독으 적었[ ] (1)

다 과 동일한 표 리 적은 것은 이고 (2) ce

과 동일한 표 리 적은 것은 이다ab

51) 정답[ ] ③

52) 정답[ ] ①②

53) 정답[ ] ③

54) 정답[ ] ③

55) 정답[ ] ①

56) 정답 른 죠코 어린 노 하니라[ ] A B

57) 정답 세 어에서는 활 형이 칙적으[ ] lsquo rsquoㄹㅇ

나타났 만 개화 어에서는 활 형이 쓰 다 lsquo rsquo ㄹㄴ

58) 정답 호 가 흔[ ] (1) (2)

59) 정답[ ] ④

60) 정답[ ] ③

Page 6: 현대고대비 국어 - chamsoriedu.com 「콘텐츠산업진흥 법」외 에도 저작권 의하여 ... 다른주체에게어떤동작을하도록만드는것을나타내는

년 학 간고사 대비2013 2 현대고 대비

ECN-0102-2013-001-000076193

고 다

⑤ 님 미 사 는 마lsquo rsquo

가사 다

가 춘사( ) [ 5]

고은 티 쬐얀 결이 름 다

이어라 이어라

을 주어 두랴 낙 노흘일가

총 총 어( ) ( ) ( )至匊葱 至匊葱 於思臥

탁 가 의 흥 이 나니 고 도 니 다( ) ( )濯纓歌 興

하사[ 2]

년닙희 두고 찬으란 쟝만 마라

닫 드러라 닫 드러라

청약립 은 써 잇노라 녹사의 가져 냐( ) ( )靑蒻笠 綠蓑衣

총 총 어( ) ( ) ( )至匊葱 至匊葱 於思臥

내 좃 가 제 좃 가( ) ( )無心 白鷗

추사[ 3]

이 니러나고 나 티 흐느 다( )白雲

돋 라라 돋 라라

의 셔호 혈 의 동호 가쟈( ) ( )西湖 東湖ㅣ

총 총 어사( ) ( ) ( )至菊悤 至菊悤 於思臥

빈홍 곳마다 경 이 다( ) ( )白蘋紅蓼 景

동사[ 4]

간 의 눈 갠 후 에 경 이 달랃고야( ) ( )後 景物

이어라 이어라

압희 만경류리 듸희 쳔텹 산( ) ( )萬頃琉璃 千疊玉山

총 총 어( ) ( ) ( )至菊悤 至菊悤 於思臥

선계 가 블계 가 인간 이 아니 다( ) ( ) ( )仙界 佛界 人間ㄴ ㄴ

윤선도 어 사 사- ( )漁父四時詞「 」

나 살어리 살어리랏다( ) 쳥산 애 살어리랏다( ) 靑山

위랑 래 랑 고 쳥산 애 살어리랏다 ( ) 靑山

얄리얄리 얄랑셩 얄라리 얄라

러라 러라 새여 자고 니러 러라 새여

널라 름 한 나도 자고 니러 니 라

얄리얄리 얄라셩 얄라리 얄라

청산별곡- ( )靑山別曲

다 청산은 엇뎨 야 만고애 프르르( ) ( ) ( ) 靑山 萬古

유 는 엇뎨하야 주야애 디 아니난고( ) ( )流水 晝夜

리도 치디 마라 만고상청 호리라( ) 萬古常靑

도산 이곡- ( )陶山十二曲

라 개를 여라 이나 르되 개 치 얄 랴( )

님 꼬리를 홰홰 치 치뛰락 나리뛰락 겨서 내

닷고 님 뒷 을 동 동 르락 나으락 캉캉

도리 암캐

이 릇 릇 날 들 너 이 이 랴

작자 상-

마 빈천 을 랴 고 에 드러가니( ) ( ) ( )貧賤 權門

침 업 흥졍을 뉘 져 쟈 리

강산과 풍 을 달나 니 는 리 리

조찬한-

정정 이랬거니 아람도리 큰 솔이 혀( ) ( )伐木丁丁

도 하이 골이 어 아리 소릴 쩌르 돌아 도 하이 다

람 도 좇 않고 새도 않어 은산 고 가 차라리

뼈를 저리 는데 눈과 이 종이 담 회 나 달도 름을

다 흰 뜻은 한 이골을 걸음이란다 절 이 여섯

판에 여섯 고 고 라간 뒤 조찰히 늙은 사나이의 남

내음새를 는가 름은 람도 일 않고 고 에 히

흔들리 노니 견디란다 차고 연 히 픔도 꿈도( )兀然

없이 장 산 속 겨 한 내-

정 장 산- 「 」

zb16) 다 에 나타난 상과 표현을 활 해 작 연 을 해( )

것이다 적절하 않은 것은

① 몰 는 도에 몸 맡 고 는 고통

욱 답게 가꾸 는 느님 산 겠지

② 돌 틈 뚫고 어 민들 보 리 진 고통

다가 도 민들 럼 강 게 살 지 겠는가

③ 벽에 런 움 없 어지고 는 폭포

여 시 는 죽 워 지 는 강

지 는가

④ 상 겨울 도 다리 꿋꿋 살 가고

겠지 망 지 는 삶 얼마 다운가

년 학 간고사 대비2013 2 현대고 대비

ECN-0102-2013-001-000076193

⑤ 막 늘 울리고 는 귀 미 리는

지새우는

가( ) 빈천( )貧賤을 랴 고 에 드러가니( )權門

침 업 흥졍을 뉘 져 쟈 리

강산과 풍 을 달나 니 는 리 리

나 청강 에( ) ( )淸江 비 듯는 소 어 읍

만산 홍 이 휘드르 는고야( )滿山紅綠

두어라 춘풍 이 날이리 을 어라( ) 春風

다 청산은 어찌 야 만고에 푸르르( ) ( ) ( )靑山 萬古

유 어찌 야 주야애 디 아니 고( ) ( )流水 晝夜

리도 치 말아 만고상청 하리라( ) 萬古常靑

라( ) 개를 여라 이나 르되 개 치 얄 랴

님 리를 홰홰 치 치 락 리 락

겨서 내닷고 고 님 뒷 을 동 동

르락 나으락 캉캉 도리 암

이 릇 릇 날 들 너 이 이 랴

zb17) 의 적 능을 비 한 내 으 가장 적절한

것은

① 는 가 지닌 실 계 다

② 는 내 갈등 심 시킨다

③ 는 가 는 상 각시킨( )愛着

④ 는 달리 내 갈등 시킨다

⑤ 는 달리 다 사 들 간 가

강 다

가 님다히 쇼 을 아 나 아쟈 니( ) ( )消息

도 거의 다 일이나 사 가

내 둘 업다 어드러 가쟛말고

잡거니 거니 놉픈 뫼 라가니

은 니 안개 일고

산쳔 이 어둡거니 일 을 엇디( ) ( )山川 日月

쳑 을 거든 쳔 리 라 랴( ) ( )咫尺 千里

하리 의 가 히나 쟈 니

람이야 결이야 어둥졍 된뎌이고

샤공은 어 가고 븬 만 걸 니

강텬 의 혼쟈 셔서 디 니( )江天

( )

쳠 자리의 듕만 도라 니( )茅簷

쳥등 은 눌 위 야 갓 고( )半壁靑燈

리 헤 니니

져근덧 녁 야 픗 을 잠간 드니( )力盡

졍셩 이 야 의 님을 니( )精誠

가 얼 이 이나마 늘거셰라( ) ( )玉 半

의 근 말 장 쟈 니

눈 이 라 나니 말인들 어이

졍 을 다 야 이조차 여 니( )情

뎐된 계셩 의 은 엇디 돗던고( )鷄聲

어 허 다 이 님이 어 간고( )虛事

결의 니러 안자 창 을 열고 라 니( )窓

어엿븐 림재 날 조 이 다

하리 여디여 낙 이나 되야이셔( )落月

님 겨 창 안 드 비최리라( )窓

나 내 님 리자 니다니( )

산 접동새 난 이 이다

아니 거츠르 아으

잔 효성이 아 리이다

넉 라도 님은 녀져라 아으

더 니 뉘러 니잇가

과도 허 도 천만 없소이다

힛마리 뎌

읏븐뎌 아으

니 나 마 니 니 잇가

아소 님하 도람 드르샤 쇼셔

zb18) 맥으 아 에 들어갈 적절한 것은

① 시 엇 다 신고

② 다 믄 눌 보 가시 고

③ 님다 쇼식 욱 득 고( )消息

④ 원망 사 허믈 랴

년 학 간고사 대비2013 2 현대고 대비

ECN-0102-2013-001-000076193

⑤ 죠 뫼 티 시 가( ) ( ) 粥早飯 朝夕

가( )

거리

공신 후 심 늦도( )劉尋

식 없어 과 께 산에 드리고 신

태몽 꾼 에 만고 웅 상 지닌 들

낳 키운다 그 후 신 들 에 역심 ( )逆心

담 귀 등 심 여 리 귀

보내고 지 죽 는 도망 가다

가 만 죽 고 에 경 가는 들 도움

살 다 그러 사 에 심 귀

보고 담 여 고 강 주가 승상

득 여 고 신 사 삼는다 그 후 강 승상

에게 심에 상 리지만 여움

사 귀 가게 다 강 승상 몸 는

연 과 헤어 리 다

경쇠 리 들리 에 들어가니 색

에 게 단청 누각과 큰 집들( )丹靑

다 주 보니 ( ) (一柱門 黃金

산 사 어 었다 산) lsquo rsquo 大字

들어가 고승 다 그( ) ( ) 山門 高僧

거동 보니 눈 눈 듯 고

변 같 귀는 어 에 늘어 니( ) 白邊

맑고 어 골격과 신 평 니었

팔염주를 에 걸고 육환장 을 고서 흑포( )六環杖

장삼에 떨어 송낙 쓰고 나 유생을 고 말[ ] 松蘿

하 를

소 이 연 하여 유상공 는 행차를 동 에 나ldquo

가 맞이하 하 으니 소 의 함을 서하

rdquo

유생이 크게 놀라 말하 다

천한 인생으 팔자가 하여 어 서 를 잃고ldquo

정처없이 다니다가 연히 이곳에 대사를 만난 것인데

토 대하 소생의 성은 어떻게 알고 있 니 rdquo

노 이 답하여 말하 를

어제 남악 형산 의 화선 이 소 의 절에ldquo ( ) ( )男樂 衡山

어서 소 에게 탁하 를 내일 낮 경에 남경 lsquo 12

동성 안에 사는 유 의 아들 충 이가 것이니 내쫓

말고 잘 대접하라 하셨 니다 마침 소 이 찾아 나rsquo

다가 상공의 차람새를 니 남경 사람이 에 알아

았 니다rdquo

유생이 말을 듣고 한편으 쁘고 한편으 퍼하

서 노 을 따라 들어가니 여러 들이 합장 하

가 했다 노 이 에 들어가 저녁 을 은 후에

을 편히 니 이곳은 선경 이었다 세상의 일을( ) 仙境

두 잊고 일 이 편안하 다 이후 는 노 과 함

서 도 이 탐 하고 경도 확하게 의 게 되었( )兵書

다 이 게 되니 대 천 에 가객 은 없 ( ) ( )大明天地 佳客

고 덕산 속에 리 른 만 있더라 래 ( ) 廣德山

이 천상 사람으 살아 있는 처를 만나 이한

을 니 재주 민함을 누가 당할 있겠는가

낮으 공 하더라

유충 전- -

웬늠 어가 사 싸다냐( ) ldquo rdquo

내가 가 막 런거 니

보통것 닐러 그 어낸ldquo ( )

틀어 주 그 가 루 러 허 에

싶어 키 틀어 주 그 가( )

루 허 우간 곡 틀어주는 루 못 는

웂는 고 닝께 고 지 들

어 사는 고 가 다는 건 에 그 집에

rdquo

그런 단 어들 어 새벽에 떼죽 거

다 고 어 보니 죄다 허 게 집어진

는 것 었다

총 가 내화를 꿴 뛰어나 만 아 소 없는

일이었다

어떻게 된 거야ldquo rdquo

한동안 넋나간 듯이 서 있던 총 가 하고많은 사람

에 하필이 유자를 겨냥하 은 말이었다

쎄유 아마 새에 고뿔이 들었던 개비네유ldquo rdquo

유자는 러 딴청을 하 다

야 고 가 에서 감 가 들어 죽는 고 두ldquo

어rdquo

총 는 가 혐의자 나 되는 것처럼 화풀이를( )嫌疑者

하 드는 것이었다

는 비위가 상해서

야 팔자가 사나서 이런 후 에 살라니|

여러 가 다 객고가 쌓여서 조 두 안 좋았을 테 helliphellip

런디다가 릇쓰 이 가락을 트는 대 디립

다 춰댔으니 과 해서 살끼두 다소 있었을 테 helliphellip

래 들어서 키 는 새끼덜일 이 다다 탈이 많은

이니 ldquohelliphellip

는 트의 독성을 충 히 내 않고 고 를 넣

은 것이 탈이었으 니 하 서도 러 참으 의 을 떨

었다 략 - -

마리가 마리 값 간다는 워ldquo

그냥 내뻔지 거시 허 싼 고 는 맛

겄다 싶 허 게 눌 강 어helliphellip

허 마늘 통 다

년 학 간고사 대비2013 2 현대고 대비

ECN-0102-2013-001-000076193

게 지 고뿌 지 rdquo

어 어째ldquo rdquo ldquo rdquo

런 도 것들 같 니ldquo ( ) rdquo殘忍無道 helliphellip

는 탱 여 지 못 다 보( ) 憤氣撐天

니 는 는 다 동원 여 통 쳤

생각 여 는 눈 다

달리 리헐 감ldquo rdquo

들 고 말 니었다 그가

는 것 그 말고는 없었 에 그 게 뒷동

달 거 다

는 우 럽고 식 짝 없는 랫것들 고

다 공연 신 가고 득 것

없다고 단 는지 결 웬만큼 고루 어

그 것들 쪽 에다 고 어주지ldquo

고 그 그걸 주 어 에 에 helliphellip

눈 없는 독 들 rdquohelliphellip helliphellip

고 말 럼 얼거리 들어가 리는 것 었

- ( ) -兪子小傳

zb19) 위 나 를 읽고 평가한 것으 적절하( ) 않은 것

① 사 리 통 감과 사실 고

격 과 달 고

② 는 가 재 컫는lsquo rsquo lsquo rsquo

미 가진 여 는 것 겠

③ 는 식 말 는 웃 상lsquo rsquo

여 는 미 지니고

④ 는 어가 죽 짐 지만 내색 지 고lsquo rsquo

말 고

⑤ 언어 통 가들 여 우리 통

것들 역 고

가 체 거리( ) [ ]

나라 종 연간에 정언주 의 을 하고( )正言注簿

있던 유 은 늦도 자 이 없어 한탄하다가 남악 형산lsquo rsquo

에 치성을 드리고 이한 태 을 꾼 뒤 아들을 낳아 이름

을 충 이라 고 키 다 이때 조정의 하들 에 역

을 품은 정한담 최일 등이 가달의 침입에 대한( ) 逆心

유 의 유화적 입장을 제 삼아 유 을 함하여 양

내고 유 의 에 을 러 충 자마저 살해하

한다 러나 충 은 천 조 정한담의 마 에서 어

나 많은 고난을 겪다가 은퇴한 재상 강희주를 만나 사위

가 된다 강희주는 유 을 하 고 상소를 으나 정

한담의 공격을 아 양을 가게 되고 강희주의 가족은

난을 피하여 두 흩어 다 충 은 강 소저 이 하고

사의 노 을 만나 를 때를 다린다 이

때 남적과 적이 를 들고 나라에 쳐들어 자 정한

담은 자 출전하여 남적에게 항 하고 남적의 선 장이

되어 천자를 공격한다 정한담에게 여러 패한 천자가

항 하 할 음 충 이 등장하여 남적의 선 정 걸

을 죽이고 천자를 출한다 충 은 단 으 란 을

쳐 고 정한담을 사 잡는다 리고 호 에게 ( )胡王

잡혀간 황후 태후 태자를 출하 유 에서 고생하

던 아 유 과 장인 강희주를 한다 또한 이 하

던 어 니 아내를 찾고 정한담 일파를 리친 뒤 높은

에 라서 화를 누린다

사 들 별 고 없 다니었다( )

마 마 돌 다니 걸 여 고

어 곤 다 에는 동쪽에 고

에는 쪽에 니 가 에 리는 엽

가는 없 니 늘 다니는 었

다 얼 말 죽 사 같고 림새가 말

니었다 가슴 에 고 등

삼태 헌 에 니 달 ( )奇男子

가 도리어 걸 었 담 만 열 도 ( )傅說

고 만났고 만 갈( ) ( ) 慇 武丁

도 탕 만났( ) ( ) (伊尹 成湯 渭

여상 도 주 만났는) ( ) ( ) ( ) 水 呂尙 周 文王

월 같 러가 도 어느 열 살

늘과 집 삼고 사 에 쳐 거리에

어 다가 곳에 니 다 ( ) 楚

지 다가 사 보고 가에 다다( )長沙

니 망 가에는 원 리가 슬 고 가

가 내리는 사 에는 갈매 가 갈 뿐 었다

쪽 돌 보니 가 우거 고

가 사 보 었다 그곳에

가니 는 사( ) 汨羅水

는 다 주 가 쓰고 죽고

곳 었다

마 감 여 에 가 사 살펴보니

에는 삼 고 그 에( ) 屈三閭

는 만고 월 과 지 가는 그 들( )風月

가 어 었다( ) 路程記

동쪽 벽 에 새 운 어 거늘 그

보니 월 에 경 주 는 간신에게ldquo ( )敗

보고 연경 귀 가다가 에 죽 rdquo

거늘 그 보고 에 거꾸러

통곡 말

[A]ldquo우리 연경 간 만 니 에

지 살 상에 엇 겠는가

에 고 에 었 니

상에 살 것 가 도 께 지리 rdquo

년 학 간고사 대비2013 2 현대고 대비

ECN-0102-2013-001-000076193

고 가에 내 가니 울 리가 에 지 사

쳤는지 심 심 것 가

신 심 것 가

다( ) 강 승상에게는 들 없고 다만 만

었다 가 낳 에 가 색

타고 내 에게 말 는 ldquo

니다 미원 과 연 맺고 ( ) ( )紫薇垣 緣分

었는 께 강 집 보내 에

니 게 여겨 주십시 거늘 rdquo

미 가운 낳 니 가 고 거동

단 다 시 짓 쓰 고 는 (音

없었 니 여 가운 지 는 짝) 律

룰 만 사 없었다 가 사 여 사 감

게 고 지 못 고 염 는 만다

다가 당에 거 고 식같 러 내니

고귀 상 루 말 다 어 울 도 다( ) 相

귀 사 없고 웅 걸( )富貴爵祿

만고 었다 승상 매우 뻐 내당 ( )內堂

들어가 에게 사 니 역시 매우 거

워 말 다

ldquo 도 마 사 는 승상께

그 게 말 시니 상 여러 말 지 말고

사 도 시다rdquo

상이 에 나 충 의 손을 잡고 결혼과 하여 ldquo

너에게 히 할 말이 있다 내가 늙은 말년에 딸

하나만을 두었는데 니 너 하늘이 정해 필

임이 하다 이제 년고락 을 너에게 탁 ( )百年苦樂

하겠다 하 대 충 이 릎을 꿇고 앉아 눈 을 흘리rdquo

여쭈었다

소자의 을 해주 고 또 하 에 두고자 하ldquo ( )膝下

니 감사하 이를 데가 없 니다 다만 가 속에 통탄

할 일이 사 쳐 있 니다 소자가 이 없어 양친 ( )兩親

의 생사를 른 채 결혼하여 아내를 얻는 것은 자 으

서 할 도리가 아닙니다 이것이 한 러 뿐입니다 rdquo

승상 그 말 듣고 슬 에 어 고

것 에 맞 어 변 게 리ldquo

는 다 집 시 공 도 여 ( )始祖公

고 가 에 가가 어진 만 개 공신

었 니 도 러워 말 시고 시 rdquo

택 여 니 다운 신 과 신

습 늘에 죄 짓고 간 상에 내 신

혼 를 다 끝내고 으 들어가 사 을 살펴 니 빛

나고 빛난 것이 한 입으 는 다 말하 어 고 하나

는 다 하 어 더라 에 켠 환한 촛 ( )新房

아래 은 에 랑과 가 평생의 연 을 맺었( )緣分

으니 서 사랑하 주고 은 말을 어떻게 다 헤아릴

있으 어떻게 다 하리 을 낸 후에 이튿날

상 를 니 상 거 마음을 이 하

더라

각 생 강 승상 집 쪽( )

늘 보고 없 가 신 신 생각 니

없고 어 없었다 는 어떻게 도리가 없다

여 산 에 들어가 리 고 어 도 닦

고 다 그 산 보고 가다

가 곳에 다다 니 에 큰 산 었다 많 우

리 골짜 가 늘 는 가운 색

에 고 갖가지 가 짝 어 었 ( )花草

다 신 산 생각 고 들어가니 경개 ( )景槪

가 매우 뛰어 고 경 산 다 산 리에 들

리는 것 리 보 는 것 울 청산뿐

었다 가 고 울 어 가

니 들 많 가지들 못 어 동

에 늘어 들거리 는( ) 洞口

우거진 가지에 갖 들 다 었다( ) 春情

계상 에는 공 는 늘( ) 花溪上

에 걸린 폭포가 벽 는 리는 산사( )層巖絶壁

쇠 리 객 에 는 듯 늘( ) ( ) 寒山寺 客船

에 싸여 는 습 산

그린 여 병 러 듯 다 경쇠 리가 들

리 에 들어가니 색 에

게 단청 누각과 큰 집들 다( ) 丹靑

주 보니( ) ( ) lsquo一柱門 黃金大字

산 사 어 었다 산 들rsquo ( )山門

어가 고승 다 그 거 보니( ) 高僧

눈 눈 듯 고 변 같 ( )白邊

귀는 어 에 늘어 니 맑고 어 골격

과 신 평 니었다 염주

에 걸고 짚고 포 삼에 어진( )六環仗

쓰고 생 보고 말

승 연 여 상공 시는 동 에ldquo

가 맞 지 못 니 승 십시 rdquo

생 크게 말 다

생 가 여 어 고ldquo

없 다니다가 우연 곳에 사 만 것

그 시 생 어떻게 고 습니 rdquo

승 답 여 말

어 산 승 에ldquo ( ) ( )南岳 衡山

시어 승에게 탁 내 낮 시경에 경 lsquo 12

동 에 사는 심 들 가 것 니 내쫓

지 말고 습니다 마 승rsquo

다가 상공 림새 보니 경 사 에 보

습니다rdquo

zb20) 위 의 친 에서 서 자의 개입이 드러나~

는 이 아닌 것은

① 달 가 도리어 걸 었( ) 奇男子

② 신 심 것 가

년 학 간고사 대비2013 2 현대고 대비

ECN-0102-2013-001-000076193

③ 다운 신 과 신 습 늘에 죄 짓고

간 상에 내 신 다

④ 사 주고 말 어떻게 다 헤 릴

어떻게 다 리

⑤ 신 산 생각 고 들어가니 경개 가 ( )景槪

매우 뛰어 고 경 산 다

거리 연[ ] ( )弘治

간 에 공신 후 에 언(1488~1505) ( )正言

주 는 벼슬 심 늦도 식( ) ( )主簿 劉尋

없어 과 께 산에 드리고 신 태

몽 꾼 에 만고 웅 상 지닌 들 낳

키운다 그 후 신 들 에 역심( )逆心

담 귀 등 심 여 리 귀 보내

고 지 죽 는 도망 가다가

만 다 에 에 어 니

헤어지게 다

에 에 어 니 헤어지게

다 그 후 사 들에게 우연 돌

생 다가 어느 열 살 었다 열 살

지 다가 우연 귀 견 는

그것 그 살 도 었고 그

귀 본 신도 지 죽고 마 고

크게 운다

( )

에는 강 주 는 재상 살고 었

니 시 에 과거에 격 여 승상 벼슬 다가 간

신 만 벼슬 그만 고 고 돌 었

다 그러 신 지 가 지 못 여 상

가 못 결 는 상 여 원 니

신 들 그 직간 꺼 다 그 에 도

담과 귀가 강 승상 가 미워 다

강 승상 마 본 에 갔다가 돌 는[A][ ( )本府

에 우편 주 에 다가 색( ) ( ) 右便 酒店

에 어리었는 청룡 에 지 늘

여 통곡 고 사 는 꿈 꾸었다] 마

상 게 생각 여 새 다리다가 새벽

닭 울고 가 달 갔다 가 보니

과연 어 동 가 가에 울고 는지 달

들어 그 고 사 에 어 말

는 어 어 에 어 가ldquo

닭 곳에 우느냐 니 울rdquo

그 고 답 여 말 다

는 경 동 에 사는 언 주 공 들ldquo

니다 께 간신 만 연경 귀 가

시다가 에 죽 사 에 는 닭에

도 에 죽고 니다rdquo

강 승상 말 듣고 크게 낯 변 말

것 웬 말 냐 근 동 ldquo (老

못 갔 니 그 사 변 여)患

런 변 었단 말 가 주 는 신 다

같 에 벼슬 다가 는 가 많 들어 고

돌 는 주 가 게 꿈 에 생

각 겠느냐 생각지 못 다 미 지 간

지지 말고 께 가 략rdquo ( ) hellip hellip

죽게 주 사당에 단 도 러운

겠느냐 말 말고 시는지 rdquo

어 없어 강 승상 가니 그곳

월계 었다

다( )

가가 고 지 사 들 가( )櫛比

통 는 리가 과

답게 꾸민 누각과 큰 집들 늘 고

게 식 가 어 들 태운 가고

었다 략 강 승상에게는 들 없고 ( ) hellip hellip

다만 만 었다 가 낳 에

가 색 타고 내 에게 말

는 니다 미원 과ldquo ( )紫薇垣

연 맺고 었는 께 강 집( )緣分

보내 에 니 게 여겨 주십시

rdquo

거늘 미 가운 낳 니 가

고 거동 단 다 시 짓 쓰 고

는 없었 니( )音律 여 가운

지 는 짝 룰 만 사 없었다 가 사

여 사 감 게 고 지 못 고 염 는 만다

다가 당에 거 고 식같

러 내니 고귀 상 루 말 다 ( )相

어 울 도 다 귀 사 없 ( )富貴爵祿

고 웅 걸 만고 었다 승상 매우 뻐

내당 들어가 에게 사 니( ) 內堂

역시 매우 거워 말 다 도 마 ldquo

사 는 승상께 그 게 말 시니

상 여러 말 지 말고 사 도 시다rdquo

( )

승상 에 고 결 과 ldquo

여 에게 말 다 내가 늙 말 에 지

만 었는 지 보니 늘

다 에게 탁 겠 ( )

다 신 꿇고 눈 리rdquo

여 었다 주시고 슬 에 ldquo ( )膝下

고 시니 감사 룰 가 없습니다 다만 가슴

에 통탄 사 쳐 습니다 복 없어

생사 결 여 내 얻는 것( )兩親

식 도리가 닙니다 것 러울 뿐 니

다rdquo

상 그 말 듣고 슬 에 어 고 말

것 에 맞 어 웅변 ldquo

년 학 간고사 대비2013 2 현대고 대비

ECN-0102-2013-001-000076193

게 리 는 다 집 시 공도 여

고 가 에 가가 어진 만 개 공신

었 니 도 러워 마 시고 rdquo 시

택 여 니 운 신 과 신

습 늘에 죄 짓고 간 상에 내 신

다 략 지낸 후에 튿 승상 ( ) hellip hellip

니 승상 거운 마 지 못

마( )

듯 월 러 생 열다 살 었

다 에 승상 어진 사 얻고 만 에 근심 없었

다만 주 가 간신 에

죽 것 생각 마 곧 어 곤

다 그 에 주 원통 어

없 고 여 시 가 거늘 생 만

여 다

말 감격 러우 간신 에 가득 여ldquo

고 니 께 상 듣지 니 것

니다rdquo

승상 듣지 고 가

퇴 재상 공달 집에 거 고 상 지어

승지 러 께 리

( )

뒷 거리 강 승상 에게 상 리지[ ]

만 여움 사 귀 가게 다 강 승상

몸 는 연 가 헤어

리 다 산 들어간 룡사 승 만

게 다 승 만 우 다릴

과 들고 략 다 담

원 여 에게 복 고 어

공격 다 담에게 여러 가( ) 天子

복 등 여 다 단

신 리쳐 담 사 고 에게

간 후 태후 태 여 지에 고생

지 심과 강 주 여 개 다 헤

어 어 니 내 고 담 리

벼슬에 귀 누리게 다

zb21) 위 의 인 간 계를 같이 나타냈을lt gt

때 에 대한 이해 가장 적절하 ~ 않은 것은

① 계에 주 는 계 심 열

상 에 다고 다( ) 水深火熱

② 계는 견원지간 고 다( ) 犬猿之間

③ 계는 달리 막역지 계 고( )莫逆之交

④ 연결 사 컬어 재 가 고( )才子佳人

⑤ 는 생 과 볼 ( )匹夫匹婦

가 재 는 는 심 고 매사에 생( )

각 고 능 도 어 가 에게 많lsquo rsquo

도움 사 다 그는 에게 거 에

꺼리 없 거 났다고 는

매우 싫어 고 신 들

는 사 다

내가 지 리에( ) 1970

사 실에 지 월간ldquo

편집 고 어 었다rdquo

어느 없 가 쑥 다 도 어 10

후 다 산 시 럼 어 엇 어 ( ) lsquo怡山

다시 만 랴 니 그는 재 그룹 승 운rsquo

사가 고 는 고 거 누

주는 가 없는 가가 어 다시 만 게 것

었다

다 보통 것 닐러 그( ) ldquo 어낸 ( )

틀어주 가 루 러 허 에

싶어 키 틀어주 그( )

가 루 허 우간 곡 틀어 주는 루 못

는 는 고 닝께 고 지

들어 사는 고 가 다는 건 에 그 집에

rdquo

그런 단 어들 어 새벽에 떼죽 거

다 고 어 보니 죄다 허 게 집어진

는 것 었다 가 실내 꿴 뛰어 지만

없는 었다

어떻게 된 거야 한동안 넋나간 듯이 서 있던 총ldquo rdquo

가 하고많은 사람 에 하필이 유자를 겨냥하 은

말이었다 쎄유 아마 새에 고뿔이 들었던 개비네 ldquo

유rdquo

유자는 러 딴청을 하 다 야 고 가 에서 ldquo

감 가 들어 죽는 고 두 어rdquo 총 는 가 혐의

자 나 되는 것처럼 화풀이를 하 드는 것이었다( )嫌疑者

라 이 어쩌 어 유( ) ldquo rdquo ldquo rdquo

애유 이런 잔인 도 한 것들 같으니ldquo ( ) rdquo殘忍無道 helliphellip

총 는 탱천 하여 쩌 를 하 다( ) 憤氣撐天

아하니 아는 자는 다 동 하여 호통을 쳤으 하나 혈

압을 생각하여 참는 눈치 다 달리 처리헐 두 ldquo

잖은감유rdquo

총 의 성 을 덧들이 고 한 말이 아니었다 가 할

년 학 간고사 대비2013 2 현대고 대비

ECN-0102-2013-001-000076193

있는 것이 말고는 없었 때 에 게 뒷동

산을 달은 거 다

이 유자소전- lsquo rsquo

zb22) 의 상황을 속담으 표현한 것으 적절한 것은

① 루 곳 게 마 다

② 에 맞고 강에 눈 다

③ 늘 도 다

④ 도 사 다

⑤ 에 가도 신만 리 다

거리 공신 후[ ]

에 주 는 벼슬 심 늦도( )主簿

식 없어 과 께 산에 드리고 신

태몽 꾼 에 만고 웅 상 지닌 들

낳 키운다 그 후 신 들 에 역심

담 귀 등 심 여 리 귀 보내고

지 죽 는 도망 간다 그

만 고 에 에 어 니

헤어지게 다 지 가 사 들에

사 들 별 고 없 다니었다

마 마 돌 다니 걸 여 고

어 곤 다 에는 동쪽에 고 에

는 쪽에 니 가 에 리는 엽 가는

없 니 늘 다니는 었다

얼 말 죽 사 같고 림새가 말 니었

다 가슴 에 고 등 삼태

헌 에 니 달 가 도리 ( )奇男子

어 걸 었 담 만 열 도 ( ) ( )傅說 慇

고 만났고 만 갈( ) ( )武丁 伊尹

도 탕 만났( ) ( )成湯 渭水

여상 도 주 만났는 월( ) ( ) ( ) 呂尙 周 文王

같 러가 도 어느 열 살

늘과 집 삼고 사 에 쳐 거리에

어 다가 곳에 니 다 ( ) 楚

지 다가 사 보고 가에 다다( )長沙

니 망 가에는 원 리가 슬 고 가

가 내리는 사 에는 갈매 가 갈 뿐 었다

쪽 돌 보니 가 우거 고

가 사 보 었다 그곳에

가니 는 사( ) 汨羅水

는 다 주 가 쓰고 죽고

곳 었다

마 감 여 에 가 사 살펴보니

에는 삼 고 그 에( ) 屈三閭

는 만고 월 과 지 가는 그 들( )風月

가 어 었다( ) 路程記

동쪽 벽 에 새 운 어 거늘 그

보니

월 에 경 주 는 간신에게ldquo ( )敗

보고 연경 귀 가다가 에 죽 rdquo

거늘 그 보고 에 거꾸러

통곡 말

우리 연경 간 만 니ldquo ( )燕京

에 지 살 상에 엇 겠는

가 에 고 에 었 니

상에 살 것 가 도 께 지리 rdquo

고 가에 내 가니 울 리가 에 지

사 쳤는지 심 심 것 가

에는 강 주 는 재상 살고 었

니 시 에 과거에 격 여 승상 벼슬 다가 간

신 만 벼슬 그만 고 고 돌 었

다 그러 신 지 가 지 못 여 상

가 못 결 는 상 여 원 니

신 들 그 직간 꺼 다 그 에 도

담과 귀가 강 승상 가 미워 다 강 승상 마

본 에 갔다가 돌 는 에 우편 주( ) ( )本府 右便

에 다가 색 에 어리었는 청룡( ) 酒店

에 지 늘 여 통곡 고

사 는 꿈 꾸었다 마 상 게 생

각 여 새 다리다가 새벽닭 울고

가 달 갔다 가 보니 과연 어 동 가

가에 울고 는지 달 들어 그

고 사 에 어 말

는 어 어 에 어 가ldquo

닭 곳에 우느냐rdquo

니 울 그 고 답 여 말 다

는 경 동 에 사는 언 주 공 들ldquo

니다 께 간신 만 연경 귀 가

시다가 에 죽 사 에 는 닭에

도 에 죽고 니다rdquo

강 승상 말 듣고 크게 낯 변 말

것 웬 말 냐 근 동ldquo ( )老患

못 갔 니 그 사 변 여 런 변

었단 말 가 주 는 신 다 같

에 벼슬 다가 는 가 많 들어 고 돌

는 주 가 게 꿈 에 생각

겠느냐 생각지 못 다 미 지 간 지지

말고 께 가 rdquo

뒷 거리 강 승상 도움 죽 고[ ]

년 학 간고사 대비2013 2 현대고 대비

ECN-0102-2013-001-000076193

고 그 과 결 여 사 가 다 그러 강

승상 에게 울린 상 강 승상 귀 가고

과 헤어 리 승 만 게 다

승 우 다릴 과

들고 략 다 담 원

여 에게 복 고 어 (天

공격 다 담에게 여러 가 복) 子

등 여 다 단신

리쳐 담 사 고 에게 간

후 태후 태 여 지에 고생 지

심과 강 주 여 개 다 헤어

어 니 내 고 담 리 벼

슬에 귀 누리게 다

미상- lsquo ( )-劉忠烈傳

zb23) 위 과 의 서사 조를 비 한 것으 적절하lt gt

않은 것은

보lt gt

믿지 고 결 여 곱

낳 다 곱째 공주 낳 가

리게 다 리 만 고 진 공주는 lsquo rsquo

리공 미 리공 에 키워진다 월

러 과 가 죽 병에 걸 는 승에 는

어 산다고 다 여 들에게 탁

지만 거 리 는다 리 는 과

승 다 승 지 가는 에 많

만 지만 보살 도움 사 도 다

그러 승 신과 결 여 시

들어 주겠다고 다 리 는 그 결

여 들 곱 낳 후에 신

얻게 다 돌 리 는

에 과 상여 만 지만 여 과

살 낸다 훗 리 그 공 우 죽 사

승 도 는 신 다

리-lsquo rsquo-

① 복 결말에 고 다

② 웅 에 탕 고 다

③ 시 겨 내고 귀 누리는lsquo rsquo

보 리 는 월 재 신 다lt gt lsquo rsquo

④ 과 보 리 는lsquo rsquo lt gt lsquo rsquo

도움과 어 신 능 극복 고

⑤ 등 여 시 겪는lsquo rsquo

보 리 는 닌 지lt gt lsquo rsquo

림 시 겪는다

가 각 고 에( ) ( )却說

살 없었다 략 사 들 슬 에 어 lt gt

가에 내 고 가고 싶 가 고 후

워 경

사 들 별 고 없 다니었다 lt

략 얼 말 죽 사 같고 림새가 말gt

니었다 가슴 에 고 등

삼태 헌 에 니 달 가 ( )奇男子

도리어 걸 었 담 만 열 도( )傅說

고 만났고 만 갈( ) ( ) 殷 武丁

도 탕 만났( ) ( ) (伊尹 成湯 渭

여상 도 주 만났는) ( ) ( ) ( )水 呂尙 周 文王

월 같 러가 도 어느 열 살

늘과 집 삼고 사 에 쳐 거리에

어 다가 곳에 니 다 ( ) 楚

지 다가 사 보고 가에 다다( )長沙

니 망 가에는 원 리가 슬 고 가

가 내리는 사 에는 갈매 가 갈 뿐 었다

쪽 돌 보니 가 우거 고

가 사 보 었다 그곳에

가니 는 사( ) 汨羅水

는 다 주 가 쓰고 죽고

곳 었다

에는 강 주 는 재상 살고( )

었 니 시 에 과거에 격 여 승상 벼슬 다

가 간신 만 벼슬 그만 고 고 돌

었다 략 강 승상 마 본 에 갔다가 돌 lt gt ( )本府

는 에 우편 주 에 다가 색( ) ( ) 右便 酒店

에 어리었는 청룡 에 지

늘 여 통곡 고 사 는 꿈 꾸

었다 마 상 게 생각 여 새 다리다

가 새벽닭 울고 달 갔다 가

보니 과연 어 동 가 가에 울고 는지

달 들어 그 고 사 에

어 말

는 어 어 에 어 가ldquo

닭 곳에 우느냐rdquo

니 울 그 고 답 여 말 다 lt

략gt

년 학 간고사 대비2013 2 현대고 대비

ECN-0102-2013-001-000076193

생각 여 가 고 시 는ldquo ( )大人

상에 다시없는 니다 살 엇 겠습니

에 돌 가시고

가에 돌 가 니 살 마 없습니

다 략 어 없어 강 승상 가니rdquo lt gt

그곳 월계 었다

다 강 승상에게는 들 없고 다만 만( )

었다 가 낳 에 가 색

타고 내 에게 말

는 니다 미원 과ldquo ( )紫微垣

연 맺고 었는 께 강 집( )緣分

보내 에 니 게 여겨 주십시

rdquo

거늘 미 가운 낳 니 가

고 거동 단 다 시 짓 쓰 고

는 없었 니 여 가운( ) 音律

지 는 짝 룰 만 사 없었다 가 사

여 사 감 게 고 지 못 고 염 는 만다

다가 당에 거 고 식같 러

내니 고귀 상 루 말 다 어 ( )相

울 도 다 귀 사 없고 ( )富貴爵祿

웅 걸 만고 었다 승상 매우 뻐 내

당 들어가 에게 사 니 역( ) 內堂

시 매우 거워 말 다

도 마 사 는 승상께ldquo

그 게 말 시니 상 여러 말 지 말고 사

도 시다 략 시 택 여rdquo lt gt

니 다운 신 과 신 습 늘에 죄

짓고 간 상에 내 신 다

다 내고 들어가 사 살펴보니

고 것 는 다 말 어 고

는 다 어 신 에 ( )新房

에 신 과 신 가 평생 연 맺었( )緣分

니 사 주고 말 어떻게 다 헤 릴

어떻게 다 리 지낸 후에 튿 승

상 니 승상 거운 마 지 못

( ) 듯 월 러 생 열다 살

었다 에 승상 어진 사 얻고 만 에 근심

없었 다만 주 가 간신

에 죽 것 생각 마 곧 어

곤 다 그 에 주 원통

어 없 고 여 시 가 거늘 략 lt gt

략 거리

강 승상 에게 상 리지만 여움

사 귀 가게 다 강 승상 몸 는

연 과 헤어 리 다

마 각 생 강 승상 집 쪽( )

늘 보고 없 가 신 신 생각 니

없고 어 없었다 는 어떻게 도리가 없다

여 산 에 들어가 리 고 어 도 닦

고 다 그 산 보고 가

다가 곳에 다다 니 에 큰 산 었다 많

우리 골짜 가 늘 는 가운 색

에 고 갖가지 가 짝 어 ( )花草

었다 략 주 보니 lt gt ( ) (一柱門 黃

산 룡사 어 었다) lsquo rsquo 金大字

산 들어가 고승 다 그( ) ( ) 山門 高僧

거동 보니 눈 눈 듯 고

변 같 귀는 어 에 늘어 니( ) 白邊

맑고 어 골격과 신 평 니었

다 염주 에 걸고 짚고 포 ( )六環杖

삼에 어진 쓰고 생 보고 말

승 연 여 상공 시는 동 에ldquo

가 맞 지 못 니 승 십시 rdquo

생 크게 말 다

생 가 여 어 고ldquo

없 다니다가 우연 곳에 사 만 것

그 시 생 어떻게 고 습니

rdquo

승 답 여 말

어 산 승 에ldquo ( ) ( )南岳 衡山

시어 승에게 탁 내 낮 시경에 경 lsquo 12

동 에 사는 심 들 가 것 니 내쫓

지 말고 습니다 마 승rsquo

다가 상공 림새 보니 경 사 에 보

습니다rdquo

생 그 말 듣고 편 고 편( )

슬 승 들어가니 여러 승 들

가워 다 승 에 들어가

후에 그 편 니 곳 경 었다 상( ) 仙境

고 신 편 다 후 는 승과

께 병 도 탐 고 경도 게( )兵書

게 었다 게 니 지 에 가객 ( ) ( )大明天地 佳客

년 학 간고사 대비2013 2 현대고 대비

ECN-0102-2013-001-000076193

없고 산 에 리 만 본 ( ) 廣德山

신 상 사 살 는 만

우고 늘 월 신 과 늘 ( )日月聖神

산 신 들 다 니 그 재( ) 名山神靈

주 민 누가 당 겠는가 낮 공

zb24) 다 에 해당하는 내 으 적절하( ) 않은 것은

① 강 티 통 당시 능 다

② 상계 지상계 경 는 원 계 드러

③ 실에 어 없는 실 가 타 는

④ 뛰어 재주 어 가진 고

등 다

⑤ 가 직 개 여 평가 내리는

편집 평 타 다lsquo rsquo

가 본격 가 동 것 지( )

다 단 상 에2003 lsquo rsquo

들어가 드럼 연주 다 취미 생 달리

들었다는 보 우 가 들ldquo

어 틱 린 도 다 고 말 다rdquo

경 는 가 망 없( ) lsquo

티 원 고 답 다 신과 같 시각rsquo

는 습 상상 만 도 감동

다 시각 연주 동시에

열 상 는

티 원 그런 열 경 럽다는 것 다

다 역시 엄청 다 본( )

에 복 들

고쳐 가고 다 신 에 얼

마 지는 고 리가 는 지도 생님

가 훈 고 많 고쳐 다

고 말 다

그러 직도 에 지 는 다 그는

체격 지 못 게 가 큰 만

체 운동 훈 과 께 체 늘 동 50

는 게 고 말 다

에게는 꿈 다 통 누 가( )

주겠다는 것 그 꿈 다 신 극복 는

과 에 큰 경험 들도 느 게

주고 싶다는 것 다

마 슬 마다( ) ldquo 통

낼 었 것 럼 고통 는 사 들

고 겠다 고rdquo

말 다 달 루 첫 낸 lsquo rsquo

첫 드 심 집에 는 리듬 드 2

루 에 도 보고 싶다 집 에는 직(RampB) 3 4

사 곡 도 보 고 싶다고 포 다middot

zb25) 에서 가장 유사한 의 를 닌 어를lt gt

찾아 쓰

lt gt

나는 이제 너에게도 픔을 주겠다

사랑 다 소 한 픔을 주겠다

겨 거리에서 개 놓고

살아 추위 떨고 있는 할 니에게

값을 으 서 뻐하던 너를 위하여

나는 픔의 평등한 얼 을 여 주겠다

내가 어둠 속에서 너를 를 때

단 한 도 평등하게 어주 않은

가마니에 덮인 동사자가

다 얼어 죽을 때

가마니 한 장조차 덮어주 않은

한 너의 사랑을 위해

흘릴 르는 너의 눈 을 위해

나는 너에게 이제 너에게도 다림을 주겠다

지 울 포동 여고 생들17

틈 없 가득 체 에 맑 울

다 죽 듣 생들 사 에

연 는 탄 다 객들 도 는lsquo rsquo

가 보 주 공 맹 가 운 는

단 그룹사운드 루 보컬 맡고 는lsquo rsquo

시각 지 었다17 1

근 다만과 가 거lsquo rsquo lsquo

꿈 고 퇴 내가 다rsquo

간 간에 지 지 연 생들 짧lsquo rsquo lsquo rsquo

가 운 듯 리에 어

연 다 내 사 고 퇴lsquo rsquo

과 루 들 결 다시 돌lsquo rsquo

들 고 사 들 에 당당

것 니다 내 태어

볼 없었 크고 열여

년 학 간고사 대비2013 2 현대고 대비

ECN-0102-2013-001-000076193

에도 고 시 얻지 못 다

감지 없는 시각 상태 다

신 지에 고 상 원망 도

단다 어느 가 에 시각 에 ldquo

어 그런 듣고 다 보니 내가 게 lsquo

살 는지 도 눈 고 싶rsquo lsquohelliphellip

보 는 생각만 들 고 그 가 들에게rsquo

도 내고 들도 고 많 었죠 들 rdquo

었 지 새 는 에 쑥 러운 색

어났다

생에 것 단연 었다lsquo rsquo

공연에 거 꿈lsquo rsquo

는 다 특 가사 갑게 는 운 lsquo

벽 에 당당 마주 어 언 가 그 벽

고 늘 어 거운 상도

없죠 내 삶 에 웃 그 께

는 다고 다rsquo

들었 그냥 런 도 고만 여ldquo lsquo rsquo

겼죠 그런 꾸 가사 미 새 다 보

니 통 는 가사 는 생각 들 고 (

가 게는 시각 는 생각 들고 들) ( )

마다 듣고 큰 얻었어 rdquo

에 진지 게 가에 미 가

zb26) 의 에 들어갈 말 적절한 것은lt gt ~

lt gt

난 난 꿈이 있었죠

고 찢겨 남 하여도

내 가 히 과 같이 간 했던 꿈

혹 때 누 가가 뜻 를 비 음

내 등 뒤에 흘릴 때도

난 참아야 했죠 참을 있었죠

날을 위해

늘 걱정하듯 말하죠

헛된 꿈은 독이라고

세상은 끝이 정해 책처럼

이 돌이킬 없는

현 이라고 helliphellip

래 난 난 꿈이 있어

꿈을 믿어

나를 켜

저 차갑게 서 있는 이란 앞에

당당히 마주칠 있어

출처 가 거위의 꿈 작사 이적 작곡 동률- lsquo rsquo ( )

① ② ③ ④ ⑤

가 떴다는 들 만 지만( ) lsquo rsquo

늘 겸 다 에 주 연 우승 지 간에도 3

단 생님께 만 지 고 고 만ldquo rdquo

큼 늘 겸 신 계 가

고 다

에게는 꿈 다 통 누 가

주겠다는 것 그 꿈 다 신 극복 는 과

에 큰 경험 들도 느 게 주

고 싶다는 것 다

슬 마다 통 낼ldquo

었 것 럼 고통 는 사 들

고 겠다 고rdquo

말 다 달 루 첫 낸lsquo rsquo

첫 드 심 집에 는 리듬 2

루 에 도 보고 싶다(RampB) 집 에는 직34

사 곡 도 보 고 싶다고 포 다

미 는( ) (26) 어 헤헤헤 웃다가 어ldquo rdquo

허허허 웃었다ldquo rdquo ldquo rdquo 같 도 고

상 다 는 같 도 다( ) 壯丁 킹 들lsquo

다 는 역도 보 그 다 지만 그는rsquo

뷰에 지 다 운동만 지 ldquo

것 지 간에 여러 사 도 역rdquo helliphellip

었다 그런 엇 그 마 움직 는지 보 쯤

지 담 사 다 훈 없어 그는 티

지 림 었다 태 다 갔다 는 습

마 집 럼 편 게 보 다

주말에는 주 엇 보내

주말에도 별 주 에 청ldquo

고 에 가고 도 쳐

에 듣고 보 에 갈 가 별 없

어 산 시 게 고 들어 2002

거 매 여 지냅니다 시 과 지훈 rdquo

다 근 간 과 진실 그리고 싶어( )

가 다 근에게 그것 진리 다 거 다 없

거 고 다 없 는 것 진리

다 근 진리는 후 쪽 었다 신산( )辛酸 삶

었 질곡( )桎梏 역사 에 지냈 가

눈에 든 것 료 단 료 게 보

것 었다 그것 그 에 겨우겨우

슬 슬 생 어가는 간들 었다

리 과 단 리 고리에 검 마

없 거리 돌

상 것 없는 등 근에게 상

과 진실 엄 ( )儼存 다는 사실 리는 가

실 고 가 과 역경 에 도 근 내 포

없었 후 보루( )堡壘 다 도 365

도 간 근 여

시 것 다

년 학 간고사 대비2013 2 현대고 대비

ECN-0102-2013-001-000076193

다 공주 그림 가 근 경- ( ) ldquo rdquo(

2009)

zb27) 작가의 주 적인 각이 드러난 것은~

① ② ③ ④ ⑤

가 신 지에 고 상 원망( )

도 단다 어느 가 에 시각 에 ldquo

어 그런 듣고 다 보니 내가 lsquo

게 살 는지 도 눈 고 싶rsquo lsquohelliphellip

보 는 생각만 들 고 그 가 들에게rsquo

도 내고 들도 고 많 었죠 들었rdquo

지 새 는 에 쑥쓰러운 색

어났다 략 [ ]

경 는 가 망 없 티lsquo

원 고 답 다 신과 같 시각rsquo

는 습 상상 만 도 감동

다 시각 연주 동시에

열 상 는 티

원 그런 열 경 럽다는 것 다 략 [ ]

슬 마다 통 낼ldquo

었 것 럼 고통 는 사 들

고 겠다 고rdquo

말 다 달 루 첫 낸 lsquo rsquo

첫 드 심 집에 는 리듬 2

루 에 도 보고 싶다 집 에는 직(RampB) 3 4

사 곡 도 보 고 싶다고 포 다

식 누 가-

고 싶어

다 역도 미 담 고 사( )

질 주말에는 주 엇 보내[ 1]

답 주말에도 별 주 에[ ] ldquo

청 고 에 가고 도 쳐

에 듣고 보 에 갈 가 별

없어 rdquo

질 계 고 슬슬 도 는 것 닙니[ 2]

답 다 들 눈 에 보 고 뿐 보[ ] ldquo

다 열심 고 어 상에 도 들지만 상

지키는 것 들다고 에 도달

그것 지키 훨 많 rdquo

질 들 살 고 리 는[ 3]

거운 들 체 리느 는다

답 가 고 게 체 어[ ] ldquo ( ) 級

느 도 계가 니 살 는 것도 고역 지만

살 우는 것 들어 는 살

체 리 고 어도 어도 실 갔다

쑥 어 rdquo

질 거리에 슷 연 여 들[ 4]

보는 간 상 지

답 상 다 체 게 리지 못[ ] ldquo

거 주변에 는 그 거 누 보지

못 고 뻐지고 싶 에 체 리는 에

타 워 지만 는 어울 는 것보다 는

시간 운동만 는 건 니에 사복 lsquo rsquo

고 사복 는 말에 들 웃지만 늘 운동복

고 지내니 사러 갈 도 어 rdquo

질 역도가 말 단 식 운동 니[ 5]

답 가 내는 만 클 업 보[ ] ldquo

그러니 만 쓰는 식 운동 니다

만 다고 거운 것 들 는 건 니거든 연

도 고 가지 동 에 도 여러 가지

복 들

보식 역도 여 미-

zb28) 가 에 대한 설 으( ) 않은 것은

① 시각 우 지 시 에 지

고 망 가는 태도 달 고 다

② 언어 과 언어 복 사 여

담 내 생각 게 는 가

③ 직 감 그 마 것

럼 생생 게 느껴지는 과 주고 간 내

없 리 어 억 게 다

④ 담 내 식 리 여 담 삶 습

과 가 시 여 독 에게 감동과 훈 다

⑤ 직 진 담 직 누

지 못 는 독 에게 생생 상 달 주고

담 욱 게 다

zb29) 나 의 각 의 의도를 설 한 것으 적절하( ) 않

년 학 간고사 대비2013 2 현대고 대비

ECN-0102-2013-001-000076193

은 것은

① 질 담 상 보여 주 것 다1

② 질 담 과 그에 삶 태도 보여2

주 것 다

③ 질 역도 겪는 어 움에 역도3

과 것 다

④ 질 같 연 여 갖는 고민 는지 말4

주 는 것 다

⑤ 질 역도가 과 고 운동 는 것5

담 가 말 주 는 것 다

가 만진 것 다( ) 3

감 달 다고 다 억 에( ) 音感

지워 지만 당시 청 탁 리도

다고 다 드럼 웠다 4

에 갈 마다 드럼 는 리가 신 게 들

다고 다 눈 볼 가 없 니 엔ldquo

는 는 님 틱 에 여 주

다 드럼과 연 맺 과 들 주었다rdquo

식 누 가-

고 싶어

역( ) 도가 말 단 식 운동 니

가 내는 만 클 업에 보ldquo

그러니 만 쓰는 식 운동 니다 만

다고 거운 것 들 는 건 니거든 연

도 고 가지 동 에 도 여러 가지 복

들 시 는 상 상

드는 상 에 맞춰 실 에 는 여러

펼쳐집니다rdquo

략( )

늘 에 는 어 만 것 같

가 에 사 고 사 사ldquo

겠어 든 에 가 경 만 고

울 는 사 겠어 rdquo

보식 역도 여 미-

다 가 운 는 어 어( ) ldquo rdquohelliphellip

월 새벽 시 태 없 거웠고1965 5 6 1

는 없 그 병원에 퇴원 집

가는 마지막 마 고 마 내 거 다

가 죽 간신 에 실 다 사는 어느5 lsquo

가 죽 는 말 가 식 다 신rsquo

상에 각 시키는( )刻印 에 실

어느 가는 후 민 가가 근 었다lsquo rsquo

는 간 과 진실 그 다는( ) ldquo

에 단 평 견 가지고 다 내

가 그리는 간상 단 고 다 지 다 는 그들

가 에 는 평 지 니 그리고 어린

들 미지 겨 그린다rdquo

마 근 간 과 진실 그리고 싶어( )

가 다 근에게 그것 진리 다 거 다 없

거 고 다 없 는 것 진리

다 근 진리는 후 쪽 었다 신산(辛酸 삶)

었 질곡(桎梏 역사 에 지냈)

가 눈에 든 것 료 단 료 게

보 것 었다 그것 그 에 겨우겨우

슬 슬 생 어가는 간들 었

다 리 과 단 리 고리에 검

마 없 거리 돌

상 것 없는 등 근에게 상에

과 진실 엄 다는 사실 리는 가( )儼存

실 고 가 과 역경 에 도 근 내

포 없었 후 보루(堡壘 다 도)

도 간 근365

여 시 것 다

월 강원도 림리에( ) 1914 2 21

삼 독 태어났다 어 근 복

그것 그리 가지 못 다 근 곱 살

지는 산 산업에 실 고 답마 에 내

갔다 근 그림 럼 쫓 다니 가 시 것

다 상 진 것도 가 었다

러 가 에도 고 근 가 꿈꾸었다 근

가 꿈꾸게 것 보통 업

원색도1926 만lsquo rsquo 었다

공주 그림 가 근 경-

zb30) 에 대한 설 가장 른 것은~

① 역도가 과 운동 도 질

② 리는 는 다 lsquo rsquo

③ 들었지만 그럭 럭 는 다 lsquo rsquo

④ 가 게 보 시 말 다

⑤ 보 병 는 지 상 lsquo rsquo

는 말 다

년 학 간고사 대비2013 2 현대고 대비

ECN-0102-2013-001-000076193

시간 많지 다 청량리 생 병원

마지막 상 경 릿 게 들어 다 그 는 십

만 큰 가 상 말 다

지 못 들 마 갈 고 돗

도시민들 싹 싹 탔다 가 시

월에 병원에 원 가 폐 진 몸도4 ( )疲弊

갈 미 지 못 고 었다 가는 얼마( ) 解渴

지 생 에 생각 가

마감 는 신 평생 십 만에

가 과 많 닮 다고 생각 지는

가 운 는 어 어ldquo rdquo 1965helliphellip

월 새벽 시 태 없 거웠고 는5 6 1

없 그 병원에 퇴원 집 가

는 마지막 마 고 마 내 거 다 가

죽 간신 에 실 다 사는 어느 가5 lsquo

죽 는 말 가 식 다 신rsquo

상에 각 시키는 에 실 어느( ) lsquo刻印

가는 후 민 가가 근 었다rsquo

ldquo 는 간 과 진실 그 다는 에

단 평 견 가지고 다 내가 그

리는 간상 단 고 다 지 다 는 그들 가

에 는 평 지 니 그리고 어린 들

미지 겨 그린다rdquo

근 간 과 진실 그리고 싶어 가

다 근에게 그것 진리 다 거 다 없 거

고 다 없 는 것 진리다

근 진리는 후 쪽 었다 신산 삶 ( )辛酸

었 질곡 역사 에 지냈 가 눈에( )桎梏

든 것 료 단 료 게 보 것

었다 그것 그 에 겨우겨우 슬

슬 생 어가는 간들 었다 리

과 단 리 고리에 검 마

없 거리 돌 상

것 없는 등 근에게 상에 과 진실

엄 다는 사실 리는 가 실( )儼存

고 가 과 역경 에 도 근 내 포 없었

후 보루 다 도 도( ) 365堡壘

간 근 여 시 것

간에 지닌 가 근 1914 2

월 강원도 림리에 삼 독21

태어났다 어 근 복 그것 그리

가지 못 다 근 곱 살 지는 산

사업에 실 고 답마 에 내 갔다 근

그림 럼 쫓 다니 가 시 것 다 상

진 것도 가 었다 러 가 에도

고 근 가 꿈꾸었다 근 가 꿈꾸게

것 보통 업 원색1926

도 만 었다lsquo rsquo

그림 가 근 경 공주- ldquo rdquo ( 2009)

zb31) 다음 이 같은 의 성 소에 해당하 않은

것은

사건 평① ② ③

④ 주 ⑤ 경

가 운 는 어 어ldquo rdquo 1965helliphellip

월 새벽 시 태 없 거웠고 는5 6 1

없 그 병원에 퇴원 집 가

는 마지막 마 고 마 내 거 다 가

죽 간신 에 실 다 사는 어느 가5 lsquo

죽 는 말 가 식 다 신rsquo

상에 각 시키는 에 실 어느( ) lsquo刻印

가는 후 민 가가 근 었다rsquo

는 간 과 진실 그 다는 에ldquo

단 평 견 가지고 다 내가 그

리는 간상 단 고 다 지 다 는 그들 가

에 는 평 지 니 그리고 어린 들

미지 겨 그린다rdquo

근 간 과 진실 그리고 싶어 가

다 근에게 그것 진리 다 거 다 없 거

고 다 없 는 것 진리다

근 진리는 후 쪽 었다 신산 삶 ( )辛酸

었 질곡 역사 에 지냈 가 눈에( )桎梏

든 것 료 단 료 게 보 것

었다 그것 그 에 겨우겨우 슬

슬 생 어가는 간들 었다 리

과 단 리 고리에 검 마

없 거리 돌 상

것 없는 등 근에게 상에 과 진실

엄 다는 사실 리는 가 실( )儼存

고 가 과 역경 에 도 근 내 포 없었

후 보루 다 도 도( ) 365堡壘

간 근 여 시 것

간에 지닌 가 근 1914 2

월 강원도 림리에 삼 독21

태어났다 어 근 복 그것 그리

가지 못 다 근 곱 살 지는 산

사업에 실 고 답마 에 내 갔다 근

그림 럼 쫓 다니 가 시 것 다 상

진 것도 가 었다 러 가 에도

고 근 가 꿈꾸었다 근 가 꿈꾸게

것 보통 업 원색1926

도 만 었다lsquo rsquo

공주 그림 가 근 경- ldquo rdquo ( 2009)

년 학 간고사 대비2013 2 현대고 대비

ECN-0102-2013-001-000076193

zb32) 위 을 작성하는 과정에서 되어 활 된 자

어 것은

신 사 료① 연보②

고③ ④ 들과 담

⑤ 에 평

는 간 과 진실 그 다는 에ldquo

단 평 견 가지고 다 내가 그

리는 간상 단 고 다 지 다 는 그들 가

에 는 평 지 니 그리고 어린 들

미지 겨 그린다rdquo

근 간 과 진실 그리고 싶어 가

다 근에게 그것 진리 다 거 다 없 거

고 다 없 는 것 진리다

근 진리는 후 쪽 었다 신산 삶 ( )辛酸

었 질곡 역사 에 지냈 가( )桎梏

눈에 든 것 료 단 료 게 보

것 었다 그것 그 에 겨우겨우

슬 슬 생 어가는 간들 었다

리 과 단 리 고리에 검 마

없 거리 돌 상

것 없는 등 근에게 상에 과

진실 엄 다는 사실 리는 가 실( )儼存

고 가 과 역경 에 도 근 내 포

없었 후 보루 다 도 도( ) 365堡壘

간 근 여 시

것 다

간에 지닌 가 근 1914 2

월 강원도 림리에 삼 독21

태어났다 어 근 복 그것 그리

가지 못 다 근 곱 살 지는 산

사업에 실 고 답마 에 내 갔다 근

그림 럼 쫓 다니 가 시 것 다 상

진 것도 가 었다 러 가 에도

고 근 가 꿈꾸었다 근 가 꿈꾸게

것 보통 업 원색1926

도 만 었다lsquo rsquo

질 루 마 가 도 린다 경건

움 느껴지는 경 다 훗 근 그림에

과 는 거 것( )裸木

만 간과 연 엮어 가는 경건 움lsquo rsquo

니었

같 가가 고 싶었 근에게 그 꿈에 다

가가는 지 다 다 가 지망생들 규 미

상 에 진 고

에 지만 근 다 다 근

미 에 운 것 보통 시 미 시간

다 그런 그에게 없는 연습 가가

통 다 가 귀 시 지 도

얻는 뛸 듯 뻤지만 마 도 가 에

듯 는 었 에 어린 근 주 에

에 그림 그리고 지우고 복( )粉板

시간 가는 게 루 보냈다

근 그 갈 가가 것 열여( )渴求

었 다가 미1932 lsquo rsquo ( lsquo

미 에 다 다는 고 마rsquo) lsquo rsquo

가 근 집 고도 지는 시골 경

그린 그림 다 후 근 에 1943 22

지 미 에 그림 고

에 걸쳐 다 미 근 가

동 는 었다

공주 그림 가 근 경- ldquo rdquo ( 2009)

zb33) 위 의 내 과 일치하는 것은

가 근 가 꿈 포 다①

근 당 가들과 께 에 다②

살 근 가 걷20③

게 었다

④ 만 통 근 역경 겨내는lsquo rsquo

느 다

⑤ 근 간 과 진실 그리 에 그 에

드러 는 간상 단 다

계 시 주 근 건강

걸었다 신 과 간에 상 다 건강

신 는 눈에도 다 근 쪽 눈 뿌 게

보 지 과에 다 다 시 지지 고 결

내 었다 시 지만 마 막막

다 늦어 결 근 쪽 눈 고 말 다

쪽 눈 근에게는 쪽 눈 었고

계 었다 그 근 는 여 그lsquo rsquo

다 근 에 같 그림 그 었다1950

시 그림 는 여 쪽lsquo rsquo

고 어 마주 고 는 그림1963

여 과 동 다 마 복

그린 듯 눈 내리 새 게 다 지

사 다 근 게 복 것

복 상과 타 는 근 상

가 떳떳 단 었고 근 그리고

간 과 진실 에 다가가 가 근다

운 었다 근 신에게 당당 지 그리고

그 다 근 그림에 단 복 보다

년 학 간고사 대비2013 2 현대고 대비

ECN-0102-2013-001-000076193

태 도 그리고 극 보다 과

얻 여 었다 과 통

근 그리고 는 재 고 에 질

만들고 특 것 다

공주 그림 가 근 경- ldquo rdquo( 2009)

zb34) 의 이유에 대해 추 한 것으 적절하 않은 것

상과 타 시도①

보다 과 얻②

근 신에게 당당 지③

④ 간 과 진실 에 다가

⑤ 태 도 얻

근 가가 었지만 그 다니 가

럼 어지지 다 복과 쟁 거쳐 시

는 가 근에게 생계 사 에

운 사 다 에 키에 건( ) 178cm死鬪

체 근 에 동 역 업( )荷役

가 생계 다 쟁

에는 동에 운 상우 주 미

죄 사 에 그림 그리는 시 다 그곳에

에 동 역 업 것에

결 것 럼 보 다 지만 그런 것만도

니었다 그림 그리는 고는 지만 매 근

는 극 간 과 별 없는 경 리 그림

벽에 그리는 것 었다 우도 리 없었다 근

트 는 우 그림 그 다 생

계 그림 단 것 다

후 근 지 신 계 리에 미

엑 리 겼다 근 곳에

건 사 크 에 미 들 ( )

상 상 그 다 근 갖 다 겪

냈다 그리고 결 그 돈

신동에 어 사리 집 마 다 마 ㄷ

루 심 쪽에는 과 엌 쪽에는 건

었다 건 주고 근 가 에

여 살 다 심 에는 지 집어

쓰고 지만 곳 근 가 에게 러웠

보 리 다 근 과 마루 업실 삼 그림

그 다 신동 마루는 근 그림에 등 는 lsquo rsquo

같 상들 지 다 시 고

에 들 폐허가

가 업실 었다

공주 그림 가 근 경- ldquo rdquo( 2009)

zb35) 위 에 대한 설 으 적절한 것은

업 시 여 훈과 감동 다①

에 주 평 드러 다②

사 사 등 식 과 ③

④ 다 근거 시 여 삶에

⑤ 살 시 사 경 께 여

습 시 다

가 시간 많지 다 청량리 생 병원( )

마지막 상 경 릿 게 들어 다 그 는

십 만 큰 가 상 말 다

지 못 들 마 갈 고 돗

도시민들 싹 싹 탔다 가 시

월에 병원에 원4 가 폐( )疲弊

진 몸도 갈 미 지 못 고 었다( )解渴 가는

얼마 지 생 에 생각

가 마감 는 신 평생 십 만에

가 과 많 닮 다고 생각 지는

가 운 는 어 어( ) ldquo rdquohelliphellip

월 새벽 시1965 5 6 1 태 없 거웠고

는 없 그 병원에 퇴원 집

가는 마지막 마 고 마 내 거 다

가 죽 간신 에 실 다 사는 어느5 lsquo

가 죽 는 말 가 식 다 신rsquo

상에 각 시키는 에 실( )刻印

어느 가는 후 민 가가 근 었다lsquo rsquo

다 는 간 과 진실 그 다는( ) ldquo

에 단 평 견 가지고 다 내

가 그리는 간상 단 고 다 지 다 는 가

에 는 평 지 니 그리고 어린 들

미지 겨 그린다rdquo

근 간 과 진실 그리고 싶어( )

가 다 근에게 그것 진리 다 거 다 없

년 학 간고사 대비2013 2 현대고 대비

ECN-0102-2013-001-000076193

거 고 다 없 는 것 진리

다 근 진리는 후 쪽 었다 신산( )辛酸 삶

었 질곡 역사 에 지냈( )桎梏

가 눈에 든 것 료 단 료 게 보

것 었다 그것 그 에 겨우겨우

슬 슬 생 어가는 간들 었다

리 과 단 리 고리에 검

마 없 거리 돌

상 것 없는 등 근에게 상에

과 진실 엄 다는 사실 리는 가 실( )儼存

고 가 과 역경 에 도 근 내 포

없었 후 보루 다( ) 堡壘 도 365

도 간 근 여

시 것 다

마 같 가가 고 싶었 근에게 그 꿈( )

에 다가가는 지 다 다 가 지망생들

규 미 상 에 진 고

에 지만 근 다 다 근

미 에 운 것 보통 시 미 시간

다 그런 그에게 없는 연습 가가

통 다 가 귀 시 지 도

얻는 뛸 듯 뻤지만 마 도 (

는 었 에 어린 근 주 에)

에 그림 그리고 지우고( )粉板

복 시간 가는 게 루 보냈다

zb36) 전 의 성 소가 아닌 것을 고르

① 평 ② 사건 ③ 경

④ ⑤ 훈

늘 지 상에 살고 는 사 들 억 도가10

고 그리 지 통 고 는 사 들( )知的

그보다 훨 많 억 도는 고 지 20

통 다 그런 지 고 2500

그리 간 보는 과 사 에

매우 달 뿐만 니 과 에 도 극

루고 었다 미 운 그런 들

살고 는 동 과 사 들 사고 식에

큰 가 다는 다

고 그리 들 우주 개별 고 독립

사 들 생각 지만 고 들 우

주 연 질 간주 다 같( ) 看做

각 도 들에게는 연 질

었지만 그리 들에게는 미 들 결 었

다 고 과 그리 들 사 같

는 동 과 사 에 도 견 다

지심리 미 마 드 겐트 는

살 들에 에 지 다

연 동 과 상 다 과 같 실험

다 크 만든 미드 도 보

여 주고 그 상 닥 고 주었다lsquo (Dax)rsquo

실 닥 는 재 지 는 것 실험 가lsquo rsquo

만들어 낸 다 그런 다 개 다 체 보

여 주었는 는 미드 지만 틱

만들었고 다 는 재료는 크 지만

달 다 그러고 어 것 닥 지 사 들에게 고 lsquo rsquo

게 니 들 주 같 고 는

체 택 고 동 들 같 재료 만들어진 체

택 다 러 는 심지어 살짜리

들에게 도 타났다 것 곧 과 동

다 상 보고 다는 것 미 다

개별 사 보고 고 동 연 질 보

고 는 것 다

동 들 주변 상 에 맞 어 동 고

에 다 사 들 태도 동에 보다 많

주 울 다 동 가 미시간 에

에 경험 다 그는 미식

경 보러 가게 었는 경 체는 매우 재미 었

주변 들 동에 질 다 그 는

들 계 어 상태 경 다

어 들 에 에 그 시 가 계 가

진 것 다 상 살펴 는 말 들 lsquo rsquo

에 그는 에 시 어 도 뒷사

생각 곧 다시 곤 것 다 그런 그에게 뒷

사 고 지 는 들 동 럼

어 웠다

생각 지도 리 드 니 벳-

zb37) 다음 위 의 내 전개 으 만 인lt gt

것은

lt gt

대조의 통해 대상이 닌 특성을 설 하고 있다

일화를 제 하여 자 의 주장을 뒷 침하고 있다

유추의 을 사 하여 독자의 의해를 돕고 있다

대상이 형성되는 과정을 간적 서에 따라 서 하고 있

① ②

③ ④

년 학 간고사 대비2013 2 현대고 대비

ECN-0102-2013-001-000076193

가 우리가 말 고 쓰는 든 단어가 사 에 는( )

것 니다 사 격에 가 는 지만

어 사 과 같 특별 는 사 니lsquo rsquo

단어 격 보 단어가 사 에

등재 어 다 리 리 사 는 단어 도 그

것 시 사 는 어 고 사 에

격 보 것 니다

러 얼 은 사전에 를 있는가 이에 대한 답lsquo rsquo

은 얼 이 유행어인가 아닌가에 따라 갈라 다 이 단어lsquo rsquo

는 년 어 자 에 랐고 쓰이고 있으2002 lsquo rsquo

유행어라고 하 에는 생 이 다 런데 계속

을 유 하 서 사전에 등재될 자격을 획득할 것인가 이

에 대한 답을 내리 는 히 어 다

여 서 가 를 고 해 볼 있다 첫 는 이 단어

를 써야 할 필 가 속적으 있는가 하는 점이다

상주의 열풍에 휩 인 사회 위 에 편 해서 퍼 말

이 얼 인데 과연 런 위 가 속될 것인가 이에lsquo rsquo

대해 필자의 생각은 정적이다 사회 위 가 뀌

런 말을 쓸 일이 없어 것이다

다음은 단어의 성이다 단어의 성이 사회적으 거

감이 없으 계속 사 될 가능성이 높다 런 에서

얼 은 좋은 조건이 아니다 익히 알 졌듯이 이lsquo rsquo

말은 얼 과 청소년층에서 속어 사 하는 이 결합lsquo rsquo lsquo rsquo

된 말이다 얼 에서 얼 을 리하는 조어 도 lsquo rsquo lsquo -rsquo

어에서는 매 낯선 이다 이것만으 도 거 감을 갖

는 사람들이 있다 더 나 속어 결합한 말이다 얼 lsquo rsquo

이 널리 퍼졌다 해도 은 여전히 청소년층의 속어lsquo rsquo

남아 있다 속어는 자연 럽게 아 자리에서나 쓰 에는

담 러 말이다 러한 담을 하고 사

역을 넓혀 가는 속어도 없 는 않다 특히 얼 은 lsquo rsquo

에도 종종 등장한다 만큼 거 감이 많이 희석되었다

고 할 있다 러나 일상의 자연 러 대화에서도 거

리낌 없이 등장하는가 게 는 되 않았다고 생

각한다

얼 이 유사어인 쌈 등을 만들어 내고lsquo rsquo lsquo rsquo

있으니 살아남을 있을 것이라고 는 견해도 있을 것

이다 러나 간이 나 서 유사어를 포함하여 든

말이 사라 사 는 많다 유사어가 많다는 것이 생 을

유 할 있는 절대적인 조건은 아니다

나 언젠가 터 사람들은 어느 단에서 얼 이 가장( )

쁜 사람을 가리켜 얼 이라고 르고 있다 이 얼lsquo rsquo lsquo rsquo

이라는 단어가 최근 어사전에 라 항간에 논란이 일고

있다 아닌 게 아니라 얼 은 유행어처럼 인다 생 lsquo rsquo

도 리 래되 않은 것 같고 언제 사라 도 알

없다 게다가 젊은이들 사이에서 주 쓰일 뿐이다 이런

단어를 사전에 는다는 게 하 이 없어 이 도

한다

러나 속단은 이다 차근차근 따져 볼 일이다

선 얼 이 일 적 유행어인 아닌 주의 게 들여다lsquo rsquo

볼 필 가 있다 유행어란 유행에 따라 빠르게 유포되었

다가 단 간 내에 소 되는 단어나 를 가리킨다

얼 은 인터넷을 통해 속히 퍼 말이다 하 만 일lsquo rsquo

적인 유행어처럼 단 간 내에 사라 않았을 뿐 아니라

현재 도 잦은 빈도 사 되고 있고 앞으 도 상당

간 사 될 것으 측된다 한 언 재단의 뉴 검 lsquo rsquo

색 사이트에 따르 얼 은 년 에 처음 나타난lsquo rsquo 2001

이후 꾸 히 사 되고 있다

이 같은 사 빈도는 얼 이 일 적 유행어 는 현lsquo rsquo

저히 다르다는 것을 여 다 장 간의 생존 만으 도

얼 은 이 한 어의 어휘 에 를 자격을 얻었다lsquo rsquo

고 할 있다 더 이 이라는 비 적 정제된 매체에

높은 빈도 쓰이고 있 않은가 사 빈도 측 에서

필통이나 연필과 같은 단어 대등하거나 더 많이 쓰lsquo rsquo lsquo rsquo

다는 것은 결코 가 게 볼 일이 아니다

이제는 사전이 언어 현 을 빠르게 하는 게 덕인

대가 되었다 세계적으 유 한 의 사전들도 경쟁

적으 어를 고 있다

하 만 얼 은 젊은이들이나 쓰는 속어라고 흠을 잡을lsquo rsquo

도 르겠다 얼 이 주 젊은 층에서 많이 쓰 lsquo rsquo

는 속어임에 틀림없다 러나 어사전에 표 적이고 품

위 있는 말만 어야 한다고 생각한다 것은 커다란

해다 당장 아 어사전이나 펼쳐 라 속어는

설과 같은 비어나 죄자들이 쓰는 은어 어

마니 같은 소 의 사람만이 쓰는 말 도 라 있

않은가 사전은 말 치에 일정 빈도 이상 나타나는 말이

라 말이든 다 할 있다

zb38) 가 나 에 대한 다음의 설( ) ( ) 않은 것은

① 가 는 얼짱 사 에 등재 것에( ) ( ) lsquo rsquo

보 고 다

② 사 등재 가는 단어 격에( )

고 고 는 언 들 언어 사 도에 고 다 ( )

③ 가 얼짱 어지만 신 과 같 매( ) ( ) lsquo rsquo

체에 도 사 는 말 는 고 다

④ 가는 얼짱 어 보고 크게 가지 근( ) lsquo rsquo 3

거 들어 뒷 고 다

⑤ 는 얼짱 어 는 다 특 다는( ) lsquo rsquo

근거 에도 크게 가지 근거 가 들어 주 2

뒷 고 다

가 늘 지 상에 살고 는 사 들 억( ) 10

도가 고 그리 지 통 고 는 사 들

그보다 훨 많 억 도는 고 지 20

통 다 그런 지 고 2500

년 학 간고사 대비2013 2 현대고 대비

ECN-0102-2013-001-000076193

그리 간 보는 과 사 에

매우 달 뿐만 니 과 에 도 극

루고 었다 미 운 그런 들

살고 는 동 과 사 들 사고 식에

큰 가 다는 다

고 그리 들 우주 개별 고 독립

사 들 생각 지만 고 들 우

주 연 질 간주 다 같 각

도 들에게는 연 질 었지

만 그리 들에게는 미 들 결 었다

고 과 그리 들 사 같 는

동 과 사 에 도 견 다

인 리학자인 츠 이마이 디드 겐트너는 두

살이 채 안 된 아이들에서 터 성인에 이르 다양한

연 대의 동양인과 서양인을 대상으 다음과 같은 험

을 했다 저 코르크 만든 피라 드 양의 도형을

여 주고 대상의 이름을 닥 라고 알 주었다lsquo (Dax)rsquo

제 닥 는 존재하 않는 것으 험자가 임의lsquo rsquo

만들어 낸 이름이다 런 다음 두 개의 다른 체를

여 주었는데 하나는 피라 드 양이 만 하얀 플라 틱

으 만들었고 다른 하나는 재 는 코르크 만 양이

달랐다 러고 나서 어떤 것이 닥 인 사람들에게 고 lsquo rsquo

르게 했더니 서양인들은 주 같은 양을 하고 있는

체를 선택했고 동양인들은 같은 재 만들어 체를

선택했다 이러한 차이는 성인은 어 두 살 리

아이들에게서도 나타났다 이것은 곧 서양인과 동양인은

서 다른 세상을 고 있다는 것을 의 한다 략 ( )

는 아주 단 하 서도 인상적인 험을 했다

험에는 동서양의 대학생들이 참여했다 는 험 참가자

들에게 컴퓨터 화 을 통해 속 장 을 담은 애니 이션

을 여 주었다 화 의 앙에는 초점의 역할을 하는 커

다란 고 한 마리가 있었고 주위에는 다른 생

들과 초 자갈 거품 등이 함 제 되었다 화 을

두 씩 후 참가자들은 자 이 것을 회상해 라는

를 았다

결과 서양인 대학생들과 동양인 대학생 두 앙

의 초점 역할을 했던 고 를 동일한 정도 언 했으

나 경 소 위 거품 초 다른 생 들 에 ( )

대해서는 동양인 대학생들이 서양인 대학생들 다 60

이상 더 많이 언 했다 뿐만 아니라 동양인 학생들은 서

양인 학생들에 비해 개 적인 고 다 전체적인 계

를 더 언 하는 경향을 다 략 또한 경의 일 ( )

를 화 킨 림을 제 하 을 때 동양인 대학생들은 대

경의 화를 알아챘 만 서양인 대학생들은 경

의 화를 거의 알아차리 했다 략 ( )

따라서 서양인들만을 대상으 연 한 화lsquo

편성 결 은 잘 된 것일 도 있다 각 과정과 인rsquo

과정의 어떤 이 화 편적이고 어떤 이

화에 따라 달라 는 는 앞으 많은 연 를 통하여 논의

되어야 한다

나 어떤 의 에서 리 두는 이 화적이다 리( )

안에는 다른 사람들과 더 친 한 계를 유 하 는 상호

의존성과 다른 사람들 터 독립적인 존재 살아가 는

독립성이 혼재한다 따라서 이 에서 어떤 특성이 더 강

하게 각되는 상황에 놓이느냐에 따라 서 다른 화적

특 을 일 있다 결 리 두는 어떤 경 에는

동양인처럼 행동하고 어떤 경 에는 서양인처럼 행동하는

것이다

zb39) 가 에 대한 다음의 설( ) 않은 것은

① 는 신 주 뒷 닥 실험과lsquo rsquo lsquo

니 실험 근거 시 다rsquo

② 동 들 상 간 공통 보다는 에 식

는 강 다

③ 들 주변 맥 에는 심 경 어 사건

과 사건 사 계에 상 민감 다

④ 는 동 과 틀린 지 고 는 것lsquo rsquo

니 다 고 다 lsquo rsquo

⑤ 가에 우리 사 들 개 시 가 원( )

집 경 말 고 는 것 개 보다는

에 고 는 것에 다

늘 지 상에 살고 는 사 들 억 도가10

고 그리 지 통 고 는 사 들( )知的

그보다 훨 많 억 도는 고 지 20

통 다 그런 지 고 2500

그리 간 보는 과 사 에

매우 달 뿐만 니 과 에 도 극

루고 었다 미 운 그런 들

살고 는 동 과 사 들 사고 식에

큰 가 다는 다

지심리 미 마 드 겐트 는 동

과 상 다 과 같 실험 다

크 만든 미드 도 보여 주고 그

상 닥 고 주었다 그런 다lsquo (Dax)rsquo

개 다 체 보여 주었는 는 미드

지만 틱 만들었고 다 는 재료는

크 지만 달 다 그러고 어 것 닥 lsquo

지 사 들에게 고 게 니 들 주 같rsquo

고 는 체 택 고 동 들 같

재료 만들어진 체 택 다 러 는

심지어 살짜리 들에게 도 타났다 것

곧 과 동 다 상 보고 다는

것 미 다 개별 사 보고 고 동

년 학 간고사 대비2013 2 현대고 대비

ECN-0102-2013-001-000076193

연 질 보고 는 것 다

동 들 주변 상 에 맞 어 동 고

에 다 사 들 태도 동에 보다

많 주 울 다 동 가 미시간

에 에 경험 다 그는 미

식 경 보러 가게 었는 경 체는 매우 재

미 었 주변 들 동에 질 다 그

는 들 계 어 상태 경

다 어 들 에 에 그 시 가 계

가 진 것 다 뒷사 고 지 는 들

동 럼 어 웠다

그는 경험에 어 얻어 동 들lsquo

각도 상 본다 는 가 우고rsquo

검 여 주 단 도 상 실험 실

시 다 그는 실험 가 들에게 컴퓨 통

담 니 보여 주었다

에는 역 는 커다 고 마리가 었

고 주 에는 다 생 들과 갈 거 등

께 시 었다 본 후 가 들

신 본 것 상 보 는 지시 다

그 결과 생들과 동 생

역 고 동 도 언

경 거 다 생 들에 ( )

는 동 생들 생들보다 60

상 많 언 다 뿐만 니 동 생들

생들에 개별 고 보다 체 계

언 는 경 보 다 경 변 시

킨 그림 시 동 생들 경

변 지만 생들 경 변

거 리지 못 다

지 지 들만 상 연 lsquo

보편 결 못 것 도 다 지각 과 과rsquo

지 과 어 보편 고 어

에 달 지는지는 많 연 통 여

어 다

리 드 니 벳 생각 지도 사- ldquo rdquo( 2004)

zb40) 위 에 대한 설 으 가장 적절한 것은

① 동 과 생 식 강 고 다

② 가지 실험 통 쓴 고 다

③ 닥 실험에 사 본질에 동 사

상에 주 다

④ 니 실험에 동 과 에 지

각 도에 가 다

⑤ 쓴 는 보편 연 에 드러 우월 에

에 근 고 다

가 동 들 주변 상 에 맞 어 동 고( )

에 다 사 들 태도 동에 보다 많

주 울 다 동 가 미시간 에

에 경험 다 그는 미식

경 보러 가게 었는 경 체는 매우 재미 었

주변 들 동에 질 다 그 는

들 계 어 상태 경 다

어 들 에 에 그 시 가 계 가

진 것 다 상 살펴lsquo 는 말 들rsquo

에 그는 에 시 어 도 뒷사

생각 곧 다시 곤 것 다 그런 그에게

뒷사 고 지 는 들 동 럼

어 웠다

그는 경험에 어 얻어( ) 동 들lsquo

각도 상 본다 는 가 우고rsquo

검 여 주 단 도 상 실험

실시 다 실험에는 동 생들 여 다

그는 실험 가 들에게 컴퓨 통

담 니 보여 주었다 에는

역 는 커다 고 마리가 었고 주 에는

다 생 들과 갈 거 등 께 시

었다 본 후 가 들 신 본 것

상 보 는 지시 다

다 그 결과 생들과 동 생( )

역 고 동 도 언

경 거 다 생 들 에 ( )

는 동 생들 생들보다 60

상 많 언 다 뿐만 니 동 생들

생들에 개별 고 보다 체 계

언 는 경 보 다 들어 동

생들 상 체 연못 럼 보 어ldquo 같rdquo

체 맥 언 시 었지만

생들 상 어 같 큰 고 가 쪽 움ldquo

직 어 같 역 고rdquo

언 시 다 경 변 시킨 그

림 시 동 생들 경 변

지만 생들 경 변 거

리지 못 다

년 학 간고사 대비2013 2 현대고 대비

ECN-0102-2013-001-000076193

게 볼 동 들 보다는 큰 그( )

림 보 에 사 과 체 맥 연결시 지각

는 경 고 체에 특 떼어 내

어 독립 보는 것 낯 어 다 에

들 사 에 고 주변 맥 에는 심 경

에 사건과 사건 사 계에 상

민감 편 다

마 지 지( ) 들만 상 연

보편 결 못 것 도 다lsquo rsquo 지각 과

과 지 과 어 보편 고 어

에 달 지는지는 많 연 통 여

어 다

리 드 니 벳 생각 지도 사- ldquo rdquo( 2004)

zb41) 의 하는 가~ 다른 것은

① ② ③

④ ⑤

얼마 그 에 동 사고 식과

사고 식 보여 주는 내 다

들 에 는 탕 고 같 게

어 겨 고 미 에 는 그 크 럼 큰 고

어리 주고 원 는 어 도 는

상 고 생각 다는 것 다 러

는 어떻게 생 것 고 과 그리 거슬

러 가 보 그 단 다

고 연 경 체 경 생 에

다 벼 사는 공동 업과 경험 많 연 역

에 고 들 연 웃과

게 지내 고 탁 연 들

들 지 연 럽게 들 다 민들

웃과 동 게 뿐만 니 는 집 과

게 다

동 시 는 생태 경 에 살 결과

들 다 사 들 사 상 에 주

울 게 었고 는 곧 체 상 과 간 사

계 시 는 낳게 었다 신 가

가 는 체에 는 원 는 동시

에 다 사 들 그 사 포 체 맥 에

다 들 간 사 연

계 체 계에 주 울 는 사고 체계

게 었다

그러 그리 연 경 그 었다 산

지 연결 는 지 건 그리고 역

에 다 런 들 업에 다 사 과

동 므 공동체에

다고 다 고 그리 들

들과는 달리 보 내 감 지 들과

지 크게 느 지 못 다 그

견 다 경우 주 쟁 통 결 는 갖

게 었다

신 사 간 계들 루어진 커다

트워크 에 게 당연 사 역시 연

계들 체 식 게 다 어 상

원 도 그 개체가 체 맥 과

계 에 고 다 게 체 맥 에 주

울 다 보 상 복 과 가변 식 게 고

상에 재 는 많 변 들 사 에 재 는 들도

게 다 들 주 태도 보

는 경우가 많다 쟁 결

통 결 보다는 통 결

는 보 다

그러 고 그리 들 개개 사 사 독

에 주 울 다 사 사 체에

어 그들 사 에 재 는 공통 규 주

고 다 상 원 에도 사

체 내 주 고 다 그들

체 여 탕 체

는 주 태도 시 고 특 사 어

주에 는지 여 그 주에 는 규

견 다 에 는 쟁 식 리

같 리 사고 체계가 달 게 었다

리 드 니 벳 생각 지도 사- ldquo rdquo( 2004)

zb42) 위 에서 사 된 설 과 가장 유사한 것은

① 크톱 컴퓨 는 본체 니 마우 루

어 다

② 곡과 시 리 는 지 과 사 루어 다는 공통

지니고 다

③ 경 고 것과는 달리

경 본 연 태 그 주변 경

④ 벽돌 능 에 사계 내내

습도가 지 다

⑤ 잰느 체 체 지닌 재 체가 없

는 재 눌 다

년 학 간고사 대비2013 2 현대고 대비

ECN-0102-2013-001-000076193

zb43) 는 립 앙 도서 이 정의 일 이다lt gt

도서 장과 이 자의 리 의 정의 연결이

적절하 않은 것은

lt gt

제 조 서 유8 ( )

도서 장은 다른 이 자의 안전을 위협하거나 도서 의①

서를 란하게 할 가 있는 자에 대하여는 도서 출입

을 제한할 있다

도서 장은 이 자가 제 조 각 호의 어느 하나의 행위를 하7②

을 때에는 이 을 하게 하거나 도서 출입을 제한할

있다

제 조자 의 대출9 ( )

도서 자 는 다음 각 호의 경 대출할 있다①

상호대차도서 간에 자 를 류하는 것을 말한다 등 다1 ( )

른 도서 과의 협 을 위하여 필 한 경

공 이 공 행 상 필 하는 경2

에 도서 장이 필 하다고 인정하는 경3

대출이 가능한 도서 자 의 위는 도서 장이 정하는②

에 따른다

제 조 상10 ( )

이 자가 도서 자 설을 더럽히거나 찢거나 뜨①

쓰게 하거나 잃어 린 경 에는 상하여야 한다

도서 장은 제 항에 따른 상 을 정하여 게 하여야1②

한다

제 조이 절차 등11 ( )

이 칙에서 정한 것 에 도서 자 설의 이 절차

이 제한 등에 필 한 사항은 도서 장이 정한다

출처 립 앙 도서- (httpwwwnlgokr)

① 는 도 리 다8

② 도 는 리 다9 1

③ 료 지 는 도 리 다9 2

④ 도 료 변상에 리10 1

⑤ 는 에 도 리 다11

3

도 다 각 같다①

공 공 다만 연1

연 간 다

매월 째 째 월2

도 도 리 그 사3

가 다고 는

도 에 미리 게1 3②

시 여 다

4

도 시간 도 여 게시 다

5

도 료 시 는 는 도①

지에 등 후

등 에 사 도②

7

는 다 각 여 는 니 다

도 료 시 상 리1 lsquo rsquo

도 료 시 훼 는2 middot

지 가 닌 곳에 식 거 담3

우는

도 보 등 보 검색열4 middot

그 에 도 질 지 여 도5

여 게시 사 는

8

도 다 거 도①

질 게 우 가 는 에 여는 도

도 가 각 어느7②

에는 지 게 거 도

9

도 료는 다 각 경우 다①

상 도 간에 료 는 것 말1 (

다 등 다 도 과 여 경우)

공 원 공 상 는 경우2

그 에 도 다고 는 경우3

가능 도 료 는 도②

는 에 다

10

년 학 간고사 대비2013 2 현대고 대비

ECN-0102-2013-001-000076193

가 도 료 시 럽 거 거①

못 쓰게 거 어 린 경우에는 변상 여

도 에 변상 여 게시1②

여 다

zb44) 위 에서 도서 장이 게 해야 할 사항에 해당하는

것을 두 쓰

년 학 간고사 대비2013 2 현대고 대비

ECN-0102-2013-001-000076193

립 도 규

1 ( )

규 립 도 립 어린 청 도(

포 다 료 시 열 시 말) (

다 에 사 규 립 도)

편 진 다

2 ( )

규 립 도 도 다 에( lsquo rsquo )

고 는 도 에 도lsquo rsquo 2 2

료 에 여 다 다만 특 료 귀

료 등 료 에 사 립 도

도 다 다( lsquo rsquo )

3 ( )

도 다 각 같다①

공 공 다만 연1

연 간 다

매월 째 째 월2

도 도 리 그 사3

가 다고 는

도 에 미리 게1 3②

시 여 다

시간4 ( )

도 시간 도 여 게시 다

등 등5 ( )

도 료 시 는 는 도①

지에 등 후

등 에 사 도②

사 료6 ( )

도 료 시 에 사 료는 도

7 ( )

는 다 각 여 는 니 다

도 료 시 상 리1 lsquo rsquo

도 료 시 훼 는2 middot

지 가 닌 곳에 식 거 담3

우는

도 보 등 보 검색열4 middot

그 에 도 질 지 여 도5

여 게시 사 는

질 지8 ( )

도 다 거 도①

질 게 우 가 는 에 여는 도

도 가 각 어느7②

에는 지 게 거 도

료9 ( )

도 료는 다 각 경우 다①

상 도 간에 료 는 것 말1 (

다 등 다 도 과 여 경우)

공 원 공 상 는 경우2

그 에 도 다고 는 경우3

가능 도 료 는 도②

는 에 다

변상10 ( )

가 도 료 시 럽 거 거①

못 쓰게 거 어 린 경우에는 변상 여

도 에 변상 여 게시1②

여 다

등 규 에 것 에 도11 ( )

료 시 등에 사

도 다

립 도- (httpwwwnlgokr)

zb45) 도서 장의 리 있는 조항으 적절하 않

은 것은

① ② ③ ④ ⑤

년 학 간고사 대비2013 2 현대고 대비

ECN-0102-2013-001-000076193

1 ( )

사가 공 는lsquo rsquo

과 여 사 원과 리

사 타 사 규

니다

개 보 보7 ( )

사는 보통신망 등 계 는 에lsquo rsquo lsquo rsquo

원 개 보 보 니다 개lsquo rsquo

보 보 사 에 는 사 개lsquo rsquo

보 취 니다 다만 사는 다 lsquo rsquo

사 계 통 공 는 경우 원 lsquo rsquo

등 개 보 당 사에 습니lsquo rsquo

원 리에8 (lsquo rsquo lsquo rsquo lsquo rsquo

)

원 에 리lsquo rsquo lsquo rsquo lsquo rsquo①

원에게 가 도 여 는lsquo rsquo 3

니다

사는 원 가 개 보 우 가lsquo rsquo lsquo rsquo lsquo rsquo②

거 사 경우 는 미 에 어 거 lsquo

사 사 운 우 가 는 경우 당rsquo lsquo rsquo

습니다lsquo rsquo

원 가 도 거lsquo rsquo lsquo rsquo lsquo rsquo 3③

가 사 고 지 경우에는 시 사에lsquo rsquo

통지 고 사 내에 니다lsquo rsquo

경우에 당 원 사에 그 사실3 lsquo rsquo lsquo rsquo④

통지 지 거 통지 도 사 내에 지 lsquo rsquo

생 경우 사는 지지 습니다lsquo rsquo

사10 (lsquo rsquo )

사는 과 지 미lsquo rsquo①

에 는 지 계 고

공 여 다 여 니다lsquo rsquo

사는 원 게lsquo rsquo lsquo rsquo lsquo rsquo②

도 개 보 신 보 포 보 보 시( )

갖 어 개 보 취 공시 고

니다

사는 과 여 원lsquo rsquo lsquo rsquo③

견 만 당 다고 경우에는

리 여 니다 원 견 만 사 lsquo rsquo

에 는 게시 거 우편 등 통 여

원에게 리 과 결과 달 니다lsquo rsquo

원11 (lsquo rsquo )

원 다 여 는 니다lsquo rsquo ①

신청 는 변경 시 허 내 등1

타 보 도2

사가 게시 보 변경3 lsquo rsquo

사가 보 보 컴퓨 그4 lsquo rsquo (

등 등 신 는 게시)

사 타 등 지 재산 에5 lsquo rsquo 3

사 타 상 거 업6 lsquo rsquo 3

는 폭 시지 상 타 공7 middot middot

에 는 보 에 공개 는 게시 는lsquo rsquo

사 동 없 리 사8 lsquo rsquo

타 거 당9

게시15 (lsquo rsquo )

원 내에 게시 는 게시 게재 는lsquo rsquo lsquo rsquo lsquo rsquo

경우 원 사가 게시 복 lsquo rsquo lsquo rsquo lsquo rsquo middot middot

등 태 언 등에 공 는

것 내에 다 원 본 게시 등 lsquo rsquo lsquo rsquo

크 능 등 여 복 는 등 태

는 것 동 것 니다

- (wwwnavercom)

zb46) 위 은 인터넷 포털사이트의 회 가입을 위한 이

약 의 일 이다 이 약 을 만드는 과정에서 생각한

내 으 적절하 않은 것은

개 보 보 가 지에 별 눠①

겠어

원 가 만들게 에②

시 주어 겠어

원들 게재 게시 다 원 크 다③

는 것 지

④ 원 지 는 뿐만 니 사가 지 는

도 께 달 지

리에 가 생 경우 사가⑤

에 다는 도 듯

1 ( )

사가 공 는lsquo rsquo

과 여 사 원과 리

사 타 사 규

년 학 간고사 대비2013 2 현대고 대비

ECN-0102-2013-001-000076193

니다

개 보 보7 ( )

사는 보통신망 등 계 는 에lsquo rsquo lsquo rsquo

원 개 보 보 니다 개lsquo rsquo

보 보 사 에 는 사 개lsquo rsquo

보 취 니다 다만 사는 다 lsquo rsquo

사 계 통 공 는 경우 원 lsquo rsquo

등 개 보 당 사에 습니lsquo rsquo

원 리에8 (lsquo rsquo lsquo rsquo lsquo rsquo

)

원 에 리lsquo rsquo lsquo rsquo lsquo rsquo①

원에게 가 도 여 는lsquo rsquo 3

니다

사는 원 가 개 보 우 가lsquo rsquo lsquo rsquo lsquo rsquo②

거 사 경우 는 미 에 어 거 lsquo

사 사 운 우 가 는 경우 당rsquo lsquo rsquo

습니다lsquo rsquo

원 가 도 거lsquo rsquo lsquo rsquo lsquo rsquo 3③

가 사 고 지 경우에는 시 사에lsquo rsquo

통지 고 사 내에 니다lsquo rsquo

경우에 당 원 사에 그 사실3 lsquo rsquo lsquo rsquo④

통지 지 거 통지 도 사 내에 지 lsquo rsquo

생 경우 사는 지지 습니다lsquo rsquo

원에 통지9 (lsquo rsquo )

사는 특 다 원에게 통지 경우lsquo rsquo lsquo rsquo

공지 게시 통 상 게시 개별 통지에7

갈 습니다

사10 (lsquo rsquo )

사는 과 지 미lsquo rsquo①

에 는 지 계 고

공 여 다 여 니다lsquo rsquo

사는 원 게lsquo rsquo lsquo rsquo lsquo rsquo②

도 개 보 신 보 포 보 보 시( )

갖 어 개 보 취 공시 고

니다

사는 과 여 원lsquo rsquo lsquo rsquo③

견 만 당 다고 경우에는

리 여 니다 원 견 만 사 lsquo rsquo

에 는 게시 거 우편 등 통 여

원에게 리 과 결과 달 니다lsquo rsquo

원11 (lsquo rsquo )

원 다 여 는 니다lsquo rsquo ①

신청 는 변경 시 허 내 등1

타 보 도2

사가 게시 보 변경3 lsquo rsquo

사가 보 보 컴퓨 그4 lsquo rsquo (

등 등 신 는 게시)

사 타 등 지 재산 에5 lsquo rsquo 3

사 타 상 거 업6 lsquo rsquo 3

는 폭 시지 상 타 공7 middot middot

에 는 보 에 공개 는 게시 는lsquo rsquo

사 동 없 리 사8 lsquo rsquo

타 거 당9

원 계 규 내lsquo rsquo lsquo②

여 공지 주 사 사가 통지 는rsquo lsquo rsquo

사 등 여 타 사 업 에 lsquo rsquo

는 여 는 니다

- (wwwnavercom)

zb47) 위 약 의 조항에서 같은 제점을 하lt gt

고 있는 조항은

lt gt

제휴 회사에 회 의 아이디 개인 정 를 전송할 있도

한 조항은 고객에게 당한 조항이다

1 7 8① ② ③

④ 9 ⑤ 10

립 도 규

1 ( )

규 립 도 립 어린 청 도(

포 다 료 시 열 시 말) (

다 에 사 규 립 도)

편 진 다

2 ( )

규 립 도 도 다 에( lsquo rsquo )

고 는 도 에 도lsquo rsquo 2 2

료 에 여 다 다만 특 료 귀

료 등 료 에 사 립 도

도 다 다( lsquo rsquo )

3 ( )

도 다 각 같다①

공 공 다만 연1

연 간 다

년 학 간고사 대비2013 2 현대고 대비

ECN-0102-2013-001-000076193

매월 째 째 월2

도 도 리 그 사3

가 다고 는

도 에 미리 게1 3②

시 여 다

시간4 ( )

도 시간 도 여 게시 다

등 등5 ( )

도 료 시 는 는 도①

지에 등 후

등 에 사 도②

사 료6 ( )

도 료 시 에 사 료는 도

7 ( )

는 다 각 여 는 니 다

도 료 시 상 리1 lsquo rsquo

도 료 시 훼 는2 middot

지 가 닌 곳에 식 거 담3

우는

도 보 등 보 검색열4 middot

그 에 도 질 지 여 도5

여 게시 사 는

질 지8 ( )

도 다 거 도①

질 게 우 가 는 에 여는 도

도 가 각 어느7②

에는 지 게 거 도

료9 ( )

도 료는 다 각 경우 다①

상 도 간에 료 는 것 말1 (

다 등 다 도 과 여 경우)

공 원 공 상 는 경우2

그 에 도 다고 는 경우3

가능 도 료 는 도②

는 에 다

변상10 ( )

가 도 료 시 럽 거 거①

못 쓰게 거 어 린 경우에는 변상 여

도 에 변상 여 게시1②

여 다

등 규 에 것 에 도11 ( )

료 시 등에 사

도 다

립 도- (httpwwwnlgokr)

zb48) 다음 정 리 의 의 으 볼 때 가장

이 적인 것은

도 시간 도 여 게시 다①

등 에 사 도②

가능 도 료 는 도 는③

에 다

④ 도 에 변상 여 게10 1

시 여 다

⑤ 도 가 각 어느7

에는 지 거 도

zb49) 를 참고하여 이 어의 성격을 설 한lt gt

것으 적절하 않은 것은

① 보 에 는 어 시 상 고 어 시lt gt lsquo rsquo

에 보여주고 다

② 진 어 어원에 견 고 다

에는 타 어 들어가는 것 다 lsquo rsquo

③ 에 들어갈 말 각각 고 어 어 신 어~

들 언어는 질 격 강 통 없었다

④ 시 우리 에 가 었지만 지 계

과 달리 들 통 사 달 어 웠

년 학 간고사 대비2013 2 현대고 대비

ECN-0102-2013-001-000076193

⑤ 크 몽골 만주 공통어가 우리 어 같

계열에 다는 에 사 특 짐

가( )

善化公主主隱 공주님

他密只嫁良置古 몰 결 고

薯童房乙 맛

夜矣卯乙抱遣去如 에 몰 고 가다

( )

始汝 會隱日恚見隱扐 만 에 본

恥隱汝衣淸隱笑 맑 웃

고 시 여 공 크다 만 다[ ] ( ) ( ) ( ) ( )始 汝 會扐

내다 에 보다 견( ) ( )恚 見 다( )隱

럽다 맑다 청 웃( ) ( ) ( ) ( )恥 衣 淸 笑

zb50) 위의 나 를 함 고 음에 답하( ) lt gt

보lt gt

( )素那或云金川 白城郡蛇山人也

운 사산

는 고 다 는( )[ ( ) ] (素那 金川 白城

사산 사 다) ( ) 郡 蛇山

삼 사- lsquo rsquo 47

에 제 된 단어 의 표 리를 조건(1) lt gt ( ) lt gt

에 맞게 서 하

건lt gt

lsquo 었고 었다 태rsquo

에 제 된 단어 동일한 표 리에(2) lt gt ( )

의해 적은 것을 나 에서 찾아 조건 에 맞게 서 하( ) lt gt

건lt gt

에 당 는 각각( ) 개 쓸 것2 단

당 는 가 여러 개 어도 개만 쓸 것 각2

개 과 도 쪽에 개만2 2

드시 지 것( )

과 동 원리 것lsquo 고

과 동 원리 것 다rsquo

태 것

가( )

素那(或云金川) 白城郡蛇山人也

소나 또는 천 이라 한다 는 성 사( ) ( ) ( )素那 金川 白城郡〔 〕

산 사람이다 현대어 풀이( ) ( )蛇山

나( )

紫布岩乎希 회

執音乎手母牛放敎遣 자 손 암쇼 노히 고

吾 不喩慙 伊賜等肹 肹 나 안디 리샤

花 折叱肹 可獻乎理音如 고 것거 도림다

다 향찰은 리말을 리 으 적은 표 이었 만 생( )

은 고 대를 넘 하고 끊어 고 말았다 랜 세

동안 갈고 닦아 체계적이었던 향찰 표 이 사라졌

을 인은 크게 두 가 나누어 생각해 볼 있다

하나는 족 사회의 한 선호도에서 찾을 있다 라 때

향찰은 주 족 계 에서 사 했을 것으 인다 한 을

알 하고서는 한자를 활 하여 리말을 리 으 표

하 란 가능하 때 이다 런데 족들은 간이 흐

를 향찰과 같은 리 표 을 익혀 사 하 다는

아 한 을 대 사 하는 쪽을 선호하게 되었다 더 이

고 초에 인재 등 을 위해 과거제도가 행되 서 한 선

호도가 더 높아졌고 결 향찰은 소 되고 말았다

또 다른 가능성은 한 어의 특성에서 찾을 있다

터 한 과 일 세 나라는 한자 화 에 속해 다

당연한 이야 겠 만 표의 자인 한자는 어를 표 하

에 매 적절하다 어의 음절은 성 ( ) ( )聲母 韻母

이 어 고 여 에 성조가 추가되어 최종 소리가 결정된

다 래서 어는 단음절을 하나의 한자 표 하 된

다 에 초성 성 종성의 세 가 소가 하나의 음절

년 학 간고사 대비2013 2 현대고 대비

ECN-0102-2013-001-000076193

을 이 는 한 어는 음절 조가 잡하고 음절의 가 많아

서 한자 차 만으 한 어의 소리를 만족 럽게 표 할

없었다 를 들어 한 어에서는 어 니 같이 음절 lsquo rsquo

이 어 단어가 얼마든 있으나 어는( ) 複數音節

자 하나 나타내 만이다lsquo [m ]rsquo 母 ǔ

한편 일 어의 표 은 핵 적 단어는 한자 적고 토는

가나라는 일 의 자 적는 이다 적인 의 를 나

타내는 은 표의 자인 한자 적고 적 계를 나

타내는 토는 표음 자 적는 셈이니 자세히 살펴

리의 향찰 표 을 쏙 빼닮았음을 알 있다 한 어 같

은 착어이 서도 일 어에만 향찰과 유사한 표 이 살아

남은 것은 일 어의 특 때 이다 일 어는 하나의 자음과

음의 결합으 음절을 이 고 침이 거의 없는 음절 언어

이다 이러한 음절의 특색에다가 토가 달한 착어라는 점

이 향찰과 유사한 표 이 살아남을 있는 비결이었다

하 만 같은 착어라도 다양한 음소 침이 달한 한

어는 향찰 표 하는 데 근 적으 한계가 있었다

zb51) 다 하여 의 행에 대한 탐 한 결과( ) lt gt 2

않은 것은

보lt gt

善花公主主隱 공주니믄 공주님( )

----------------------------------------

-

他密只嫁良置古 그 지 얼어 고 몰 결(

----------------------------------------

-

薯童房乙 맛 맛( )

夜矣卯乙抱遺去如 몰 고 가다 에 몰 고(

가다)

주동 역 동- (薯童謠『 』

에 2 ( )他密只嫁良置古

얼다 시집가다 결 다 말 lsquo rsquo

① 실질 미 지니고 므 타 타lsquo ( )rsquo lsquo [ ]

② 에 실질 미 타내고 지 는lsquo rsquo lsquo [ ]rsquo lsquo [ ]密只 密 只

계 타내는

③ 얼어는 실질 미 포 고 므 가lsquo rsquo lsquo [ ]rsquo嫁

것lsquo [ ]rsquo 良

④ 고 어간 는 실질 미 지니고 므lsquo rsquo lsquo -rsquo

것lsquo [ ]rsquo 置

⑤ 고 어미 고는 계 타내고 므lsquo rsquo lsquo- rsquo

고 것lsquo [ ]rsquo 古

가( )

엉 훈 민middot middot middot middot middot世 宗 御 製 訓 民 正 音

말 미 듕 귁에 달middot middot middot middot middot middot middot middot中 國 文 字

니 런middot middot middot middot middot middot 어린middot middot middot middot百 姓

니 고 도 내 들middot middot middot middot middot middot middot middot middot 시러middot

펴 몯middot 미middot middot 니 내middot middot middot middot middot middot middot middot 爲

어엿middot 겨 새middot middot middot 믈여듧middot middot middot middot字 니middot middot middot

사 마다 니겨 킈 middot middot middot middot middot middot middot middot middot便 安

고 미니middot middot middot middot

본 는 상( ) (象

원리에 만들어진 본) ( )形 ㄱ ㄴ ㅁ ㅅ ㅇ

에 는 가 원리에( )加劃

그리고( )ㅋ ㄷ ㅌ ㅂ ㅍ ㅈ ㅊ ㆆ ㅎ

쓰는 병 원리에 만들어진( )竝書

마지막 체( ) ( )異體ㄲ ㄸ ㅃ ㅆ ㅉ ㆅ

ᅀ 다 상 원리에 ㅇ ㄹ

지 는 삼재 상 본 본( ) ( ) ( 天地人 三才

탕 므림과 림에 ) (初ㅡ ㅣ

재)( ) ( )( )出字 再出字ㅗ ㅏ ㅜ ㅓ ㅛ ㅑ ㅜ ㅕ

병 그리고 들 에 다시( )ㅘ ㅝ ㅣ

( )ㅣ ㅢ ㅚ ㅐ ㅟ ㅔ ㆉ ㅒ ㆌ ㅖ ㅙ ㅞ

zb52) 가 에 대한 설 으 르 않은 것을( ) 두 고르

① 어쓰 규 지키고 다

② 리 고 다

③ 말 미 미 등 어 사 다lsquo rsquo

④ 개 지 다

년 학 간고사 대비2013 2 현대고 대비

ECN-0102-2013-001-000076193

⑤ 어 원 에 가 도 고 다

엉 훈 민世 宗 御 製 訓 民 正 音

말 미 듕귁에 달 니

런 어린 니 고 도middot

내 들 시러 펴 몯 미 니middot

내 어엿 겨 새 믈여듧

사 마다 니겨middot 킈 고

미니

훈민 언 본- lsquo rsquo 5 (1459 )

zb53) 위의 에 대한 현대어 풀이가 르~ 않은 것

① 우리 말 과 달

② 어리 말 고 는 것 어도

③ 신 생각 마 껏 펼 는 사 많다

④ 게 생각 여

⑤ 사 마다 게

zb54) 훈민정음 언해 에는 한 을 창제한 동 가 드러나

있다 훈민정음 창제의 정 과 내 이 잘 연결된 것

① 주 신 말 미 듕귁에 달

② 민 신 내 어 겨

③ 신 뻔 킈 고 미니

④ 실 신 사 마다 니겨

⑤ 귀 신 계 주 는 훈민 신과 거리가

가 엉 훈 민( ) middot middot middot middot middot世 宗 御 製 訓 民 正 音 

말 미 귁에 中 國 달 文 字

니 런 어린 니 百 姓

고 도 내 들 시러 펴 몯

미 니 내 어엿 爲 겨 새

믈여듧 니 사 마다 니 字

겨 킈 고 미니 便 安

훈민 언 본- lsquo ( )rsquo ( ) 5 (1459 )訓民正音 世祖

( )

[ 1 ]

동 룡 샤 마다 복( ) ( ) ( )海東 六龍 天福

시니 고 동( ) ( )古聖 同符 시니

[ 2 ]

매 니 곶 여

미 므 니 그 내 러

가 니

[ 125 ]

우 미리( )千世 샨( )定 에( )漢水北 累仁

누 개 샤 복 업 시니( ) ( ) 開國 卜年

신( )聖神 니 샤도 경 근민 샤 욱( )敬天勤民

드시리 다

님 쇼 산 가( ) ( )洛水 山行

미드니 가

어 가- lsquo ( )rsquo 27龍飛御天歌

다 우리신 니쓰고 다만 만 쓰( )

거 샹 귀쳔 다보게 러 귀

여 쓴 도 신 보 가 고 신 에

말 어 보게 각 에 사 들

고 본 몬 능통 후에

죠 죠 니

드 도 만 공 에 사

드 미 죠 고 고 여 보 죠

보다 얼마가 거시 어신고 니 첫

가 죠 니 죠

민 들 어 신 샹

귀쳔 도보고 어보 가 만 늘

고 폐 에 만쓴 죠 민

도 러보지못 고 보니 그게 엇지

심 니 리 보 가 어 운건 다

니 쳣 말마 지 니 고 그

쓰 에 가 우 지 지

몰 거 본후에 가 어 지

고 그니 쓴편지 쟝 보

년 학 간고사 대비2013 2 현대고 대비

ECN-0102-2013-001-000076193

쓴것보다 듸 보고 그 마 니 쓴 고

어 못

그런고 에 리 과 가

만 쓴 못 민 말만 듯고

고 편 그 못 보니 그사 단

병신 못 다고 그사 식 사

니 만 고 다 과 그사

만 고 다 과 업 사 보다 식 고

죠 도 고 각 과

견 고 실 직 귀쳔 간에 그

고도 다 것 몰 귀죡 보다

사 우리 신 귀쳔 다 업

시 신 보고 과 지 게 랴

시니 샹 귀쳔 간에 우리 신 걸

간 보 새지각과 새 걸 미리

독립신- lsquo (1896)rsquo

zb55) 친 어 나의 제 장( ) 2 매 함축적

의 가 가장 유사한 것은

① 지 눈 내리고 매 득 니 내 여 가

사- lsquo rsquo

② 도 어 리듯 그 게 어 다

주 사- lsquo rsquo

③ 눈 살 다 죽 어 린 과 체 여

눈 새벽 지 도 살 다

눈- lsquo rsquo

④ 삶 근심과 고단 에 돌 거니는 여 거 는

여 리 내린 살가지 에 눈 리 눈 리

택 그 생 에- lsquo rsquo

⑤ 늘 러 고 러

청룡 룡 어 개 루 우

신경림 계- lsquo rsquo

zb56) 친 를 위 가 나 에 나타난A B ( ) ( )

세 어의 특 에 의거하여 세 어 표 하

그 산 고 공 도 맑지만

A

주변에 쓰 리는 어리 사 많다

B

건lt gt

식 가 에 타 어 특징에( ) ( )

거 과 어쓰 는 고 지 말 것

A

B

zb57) 가 의( ) 달 아ㆍ 다 의 ( ) 나셔에서 알 있는

세 어 개화 어의 특 을 비 하여 조건 에lt gt

맞게 서 하

건lt gt

어에 는lsquo 개

어에 는 다 태rsquo

zb58) 은 가 는 다 에 나 는 절lt 1gt ( ) lt 2gt ( )

일 를 췌한 것이다 의 의 가 lt 1gt (1)~(2)

유사한 말을 에서 찾아 쓰lt 2gt

보lt 1gt

런 (1) 어린 니 고百 姓

도 내 들 시러 펴 몯 미

사 마다 (2) 니겨 便 安

킈 고 미니

보lt 2gt

죠 고 고 여 보 죠

보다 얼마가 거시 어신고 니 첫 가

죠 니 죠 민

들 어 신 샹 귀쳔

도보고 어보 가 만 늘 고

폐 에 만쓴 죠 민 도

러보지못 고 보니 그게 엇지 심

니 리

년 학 간고사 대비2013 2 현대고 대비

ECN-0102-2013-001-000076193

lt 1 gt

동 룡 샤 마다 복 시( ) ( ) ( )海東 六龍 天福

고 동 시니( ) ( )古聖 同符

lt 2 gt

(A) 매 니 곶

여 니

미 므 니 그 내

러 가 니

lt125 gt

우 미리 샨 에( ) ( ) ( ) 千世 定 漢水北 累

누 개 샤 복 업 시 니( ) ( ) 仁開國 卜年 聖

신( ) 神 니 샤도 경 근민 샤( ) 敬天勤民

욱 드 시 리 다

님 쇼 산 가 ( ) ( )洛水 山行

미드니 가

- lt gt龍飛御天歌

zb59) 장과 내 상 유사한 성격의 조는125

① 뫼 고 고 고 고

어 그린 많고 많고 고 고

어 러 는 울고 울고 가느니

도 견- lt gt

② 강 에 드니 몸 다

그믈 고 가니

뒷 뫼 엄 언 니( )藥

-

③ 말 없는 청산 태 없는 다

값 없는 청 없는 월

에 병 없는 몸 별 없 늙 리

-

④ 가마귀 골에 가지 마

낸 가마귀 새

청강에 것 시 몸 러 가( ) 淸江

-

⑤ 진 골에( ) 白雪

가 매 는 어느 곳에 었는고

에 갈 곳 몰( ) 夕陽

색-

zb60) 위 에 나타난 세 어의 특 으 적절하 않은

것은

① 룡 어 주격 사에 당 는 가 사( ) lsquo rsquo六龍

고 다

② 샤 어에도 어 주체 쓰 다

는 것 다

③ 매 어 달리 사 택에 어

가 지 지지 고 다

④ 므 원 상 직 어 지 다

⑤ 드시리 다 주체 과 상 께 사

고 다

수고 하셨습니다hearts hearts

년 학 간고사 대비2013 2 현대고 대비

ECN-0102-2013-001-000076193

보닷컴에 공 는 별 보는 고등

들 여 주 는

들 습니다 슷 동 지

가 복 는 것 도가

니 복 여 습 시고 거 시

니다

정답 해설

1) 정답[ ] ④

해설 다른 것은 두 특정 업이나 단 내에서 사[ ]

하는 일종의 은어 사회 언에 해당한다 러나

는 언이 아니라 단과대학을 여서 단대 사lsquo rsquo lsquo rsquo lsquo④

대학을 여서 사대라고 한 말에 해당하 일rsquo lsquo rsquo

사회에서도 널리 쓰이 사회 언이라 할

없다

2) 정답[ ] ⑤

해설 사회 언은 같은 단 내에서 쓰이는 언어이[ ] lsquo rsquo

동일 단끼리는 단결 과 친 감을 형성하는

능을 하 리적 안감이 일어나 않는다

3) 정답[ ] ③

해설 사람이라는 차 적 표현에 대한 대안적 표현이[ ]lsquo rsquo

인 아내 처 등으 볼 있다lsquo rsquo

4) 정답[ ]⑤

해설 남성은 주 격 체를 사 한다[ ]

5) 정답[ ] ⑤

해설 흑인은 검다라는 뜻을 가 고 있을 뿐 인[ ]lsquo rsquo lsquo rsquo lsquo rsquo

다 열등한 뜻을 내포하 않는다

6) 정답 살 색 첫 작품[ ] - -

해설 살색 혹은 킨색은 한 인의 피 색을 뜻[ ] lsquo rsquo lsquo rsquo

하는 것으 인종 차 을 추 고 출 이주민

의 평등 을 침해할 있어 년 표 이2005

살 색으 이름을 꾸었다 처녀작은 처녀라lsquo rsquo lsquo rsquo lsquo rsquo

는 단어가 가 고 있는 곡된 성 인 을 한 것

으 첫 작품정도 꾸어 사 하는 것이 좋다lsquo rsquo

7) 정답[ ] ⑤

해설 호는 아들에게 해체를 사 하고 있다[ ] ① ②

장 을 성하는 청자는 자 의 아 느리 아lsquo

들 세 이다 호는 아 느리에게 해rsquo ③

체를 사 하고 있다 호가 느리 아 에게 ④

사 한 해 체 아들에게 사 한 해체는 두 비lsquo rsquo lsquo rsquo

격 체에 해당한다 호는 자 의 아랫사람인 ⑤

느리에게 아들과 마찬가 해체를 사 하는 것이

상 이 만 임 을 한 느리에게 고마 과 쁨

존 의 표 를 하 위해 자 의 아 에게 말하듯

해 체를 사 하고 있다

8) 정답[ ] ③

9) 정답[ ] ⑤

10) 정답[ ] ①

해설 청자 할아 가 장의 주체 아 다 높을[ ] ( ) ( )

경 에는 압존 에 의해 장의 주체를 높이 않는lsquo rsquo

다 러 아 서가 아닌 아 는으 계 lsquo rsquo lsquo rsquo lsquo

니다 가 아닌 있 니다 표현하는 것이 르rsquo lsquo rsquo

11) 정답 당이 당을 쫒았다 당이[ ]

당에 다

해설[ ]

12) 정답[ ] ⑤

해설 서 다른 높임표현을 통해 청자에 대해 리[ ] ⑤

적 거리감을 나타내는 인 은 이 아니라 현정이

다 가 에서 현정은 에게 해 체를 사 함으 써 ( )

친근감을 드러낸다 나 에서 연 을 게을리하는 역 ( )

도 들 때 에 화가 난 현정이 선생님에게 항의하

는 장 에서는 하 체를 사 하여 리적 거리lsquo rsquo

가 어졌음을 나타내고 있다

13) 정답[ ] ①

해설 는 는 얼 빛이 날과 어찌 다르 고[ ] lsquo rsquo

라는 뜻으 전과 달리 임이 화자를 않고

있음을 알 있다

14) 정답 달리 후 가 있다 이를 통해 경[ ] lt gt

쾌한 음악성을 형성하고 노 젓는 상황을 체적으

형상화하는 역할을 한다

15) 정답[ ] ①

16) 정답[ ] ⑤

해설 다 의 자연은 를 성찰하게 하는 대상[ ] ( )⑤

이자 정의 대상이다 의 자연은 자 의 상황과 ⑤

처 를 드러내는 경으 서의 역할을 하 이

이 없다

17) 정답[ ] ③

해설 는 빈천 을 해결하고자 했으나 강산[ ] lsquo ( )rsquo 貧賤③

과 풍 을 달라는 에 거절하 다고 함으 써 자

연에 대한 애정을 드러내고 있으 는 않는

임에 대한 망을 개에게 전가 켜서 임에 대한 리

을 드러내고 있다

18) 정답[ ] ③

년 학 간고사 대비2013 2 현대고 대비

ECN-0102-2013-001-000076193

19) 정답[ ] ⑤

해설 고상한 음악가의 이름을 리말 꽝 럽[ ]

게 꿈으 써 언어유희를 통해 음을 유 하고 있

다 이는 고상한 척하는 총 를 비꼼으 써 비판적

태도를 드러내는 것이 대상을 꽝 럽게 표현

하여 총 의 허 과 사치를 풍자하고 있다

20) 정답[ ] ⑤

해설 는 작품 속 경에 대한 설 이 드러나는 것이[ ]

서 자의 주 적인 견해가 접적으 드러나는 것이

아니다

21) 정답[ ] ⑤

22) 정답[ ] ②

23) 정답[ ] ④

24) 정답[ ] ①

해설 적강 티프는 주인공의 비 한 출생이나 능[ ] ①

과 이 있는 것으 조정의 능함을 풍자하는lsquo rsquo

것과는 거리가 다

25) 정답 픔 나[ ] ( )

해설 의 음악은 고통 는 사람들을 위 하고 아픔[ ] lsquo rsquo

을 치유해 주는 능을 한다고 할 있다 의 lt gt

픔 도 소 된 이 과 더 어 살아가는 따뜻한 마음lsquo rsquo

을 상 한다

26) 정답[ ] ⑤

해설 에게 선천적으 주어 각 장애라는 역경[ ]

은 의 이라는 가사 연 을 있다lsquo rsquo

27) 정답[ ] ④

해설 는 장 란 선 에게 은 개인적인 인상을[ ]

소녀 장정 등으 표현한 것이다lsquo rsquo

28) 정답[ ] ②

해설 담자가 피 담자의 언어적 표현이나 비언어[ ]②

적 표현 하 독자는 담의 위 나 피

담자의 감정 상태를 알 있다 이를 통해 독자는

담 상황을 더 생생하게 느낄 있고 피 담자

를 더 잘 이해할 있게 된다

29) 정답[ ]③

해설 일상생활과 역도 선 서의 성과에 된 것에서[ ]

역도를 하 서 겪는 어 과 내적 고민으 화제를

전화하 위한 것이다

30) 정답[ ] ①

해설 릿속에 새겨 넣듯 이 억되도 함 세상[ ] ② ③

살이가 힘들고 고생 러 속 하여 자유를 ④

가 없는 고통의 상태를 비유적으 이르는 말

적의 침입을 막 위해 쌓은 축 켜야 할⑤

대상을 비유적으 이르는 말이다

31) 정답[ ] ④

해설 이 의 종류는 전 으 인 사건 경[ ] lsquo

비평을 성 소 삼는다rsquo

32) 정답[ ] ④

해설 근은 삼대독자 태어났음을 에서 확인할[ ]

있다 형제들과의 담은 이뤄 가 없다

33) 정답[ ] ⑤

해설 근은 가난에도 하고 화가를 꿈꾸었다[ ] (3

단 또한 다른 화가 망생들은 정 육을)

위해 상 학 학 해 유학 에 랐 만

근은 다른 을 찾아야 했다 단 세에(5 ) 18

근은 조선 전람회에 입선하 다 단 의(6 )

만종은 인간과 자연이 엮어 가는 경건한 조화 을lsquo rsquo

나타낸다

34) 정답[ ] ①

해설 근이 속에서도 창작활동을 추 않고[ ]

하는 닭은 은 세상과 타협할 르는

근이 세상의 이해를 하 위한 가장 떳떳한 단

이 때 이다

35) 정답[ ] ⑤

해설 전 은 서 자의 주 적인 평이 리는 것이[ ]

만 위 제 은 인 이 살았던 대 사회적 경

을 통해 객 적인 인 의 을 제 하고 있다

36) 정답[ ] ⑤

해설 전 은 인 사건 경 비평이라는[ ] lsquo rsquo⑤

성 이 어져 있다

37) 정답[ ] ①

해설 이 은 동양인과 서양인의 사고 에 차이가[ ]

있다는 것을 대조를 통해 설 하고 있다 또 쓴이

의 제자가 축 경 를 러 가서 경험한 일화를

통해 동양인이 서양인에 비해 주 상황에 더 많은

주의를 인다는 주장을 뒷 침하고 있다

38) 정답[ ] ④

39) 정답[ ] ②

40) 정답[ ] ②

41) 정답[ ] ④

42) 정답[ ] ③

43) 정답[ ] ④

44) 정답 도서 의 휴 일 도서 의 이 간 도서의[ ]

해설 도서 장은 임의 정한 휴 일과 도서 이[ ]

간 도서의 상 등을 게 할 의 가 있다

년 학 간고사 대비2013 2 현대고 대비

ECN-0102-2013-001-000076193

45) 정답[ ] ①

해설 제 조의 정 휴 일 의 휴 일의 사전 게[ ] 3

는 도서 장의 의 조항에 속한다

46) 정답[ ] ①

해설 개인 정 호 의 를 제 하 했 만 항[ ]

나눠서 제 하 않고 대 나열하고 있다

47) 정답[ ] ②

해설 제 조의 내 을 회사는 다른 회사 협[ ] 7 lsquo

계약을 통해 서비 를 제공하는 경 회 의 아이디

등 개인 정 를 해당 회사에 전송할 있다는 내rsquo

이 있으 의 제점을 제 할 있다②

48) 정답[ ] ④

해설 는 도서 장의 의 에 해당하고 나 는 도[ ] ④

서 장의 리에 해당한다

49) 정답[ ] ③

50) 정답 은 음독으 적었고 은 훈독으 적었[ ] (1)

다 과 동일한 표 리 적은 것은 이고 (2) ce

과 동일한 표 리 적은 것은 이다ab

51) 정답[ ] ③

52) 정답[ ] ①②

53) 정답[ ] ③

54) 정답[ ] ③

55) 정답[ ] ①

56) 정답 른 죠코 어린 노 하니라[ ] A B

57) 정답 세 어에서는 활 형이 칙적으[ ] lsquo rsquoㄹㅇ

나타났 만 개화 어에서는 활 형이 쓰 다 lsquo rsquo ㄹㄴ

58) 정답 호 가 흔[ ] (1) (2)

59) 정답[ ] ④

60) 정답[ ] ③

Page 7: 현대고대비 국어 - chamsoriedu.com 「콘텐츠산업진흥 법」외 에도 저작권 의하여 ... 다른주체에게어떤동작을하도록만드는것을나타내는

년 학 간고사 대비2013 2 현대고 대비

ECN-0102-2013-001-000076193

⑤ 막 늘 울리고 는 귀 미 리는

지새우는

가( ) 빈천( )貧賤을 랴 고 에 드러가니( )權門

침 업 흥졍을 뉘 져 쟈 리

강산과 풍 을 달나 니 는 리 리

나 청강 에( ) ( )淸江 비 듯는 소 어 읍

만산 홍 이 휘드르 는고야( )滿山紅綠

두어라 춘풍 이 날이리 을 어라( ) 春風

다 청산은 어찌 야 만고에 푸르르( ) ( ) ( )靑山 萬古

유 어찌 야 주야애 디 아니 고( ) ( )流水 晝夜

리도 치 말아 만고상청 하리라( ) 萬古常靑

라( ) 개를 여라 이나 르되 개 치 얄 랴

님 리를 홰홰 치 치 락 리 락

겨서 내닷고 고 님 뒷 을 동 동

르락 나으락 캉캉 도리 암

이 릇 릇 날 들 너 이 이 랴

zb17) 의 적 능을 비 한 내 으 가장 적절한

것은

① 는 가 지닌 실 계 다

② 는 내 갈등 심 시킨다

③ 는 가 는 상 각시킨( )愛着

④ 는 달리 내 갈등 시킨다

⑤ 는 달리 다 사 들 간 가

강 다

가 님다히 쇼 을 아 나 아쟈 니( ) ( )消息

도 거의 다 일이나 사 가

내 둘 업다 어드러 가쟛말고

잡거니 거니 놉픈 뫼 라가니

은 니 안개 일고

산쳔 이 어둡거니 일 을 엇디( ) ( )山川 日月

쳑 을 거든 쳔 리 라 랴( ) ( )咫尺 千里

하리 의 가 히나 쟈 니

람이야 결이야 어둥졍 된뎌이고

샤공은 어 가고 븬 만 걸 니

강텬 의 혼쟈 셔서 디 니( )江天

( )

쳠 자리의 듕만 도라 니( )茅簷

쳥등 은 눌 위 야 갓 고( )半壁靑燈

리 헤 니니

져근덧 녁 야 픗 을 잠간 드니( )力盡

졍셩 이 야 의 님을 니( )精誠

가 얼 이 이나마 늘거셰라( ) ( )玉 半

의 근 말 장 쟈 니

눈 이 라 나니 말인들 어이

졍 을 다 야 이조차 여 니( )情

뎐된 계셩 의 은 엇디 돗던고( )鷄聲

어 허 다 이 님이 어 간고( )虛事

결의 니러 안자 창 을 열고 라 니( )窓

어엿븐 림재 날 조 이 다

하리 여디여 낙 이나 되야이셔( )落月

님 겨 창 안 드 비최리라( )窓

나 내 님 리자 니다니( )

산 접동새 난 이 이다

아니 거츠르 아으

잔 효성이 아 리이다

넉 라도 님은 녀져라 아으

더 니 뉘러 니잇가

과도 허 도 천만 없소이다

힛마리 뎌

읏븐뎌 아으

니 나 마 니 니 잇가

아소 님하 도람 드르샤 쇼셔

zb18) 맥으 아 에 들어갈 적절한 것은

① 시 엇 다 신고

② 다 믄 눌 보 가시 고

③ 님다 쇼식 욱 득 고( )消息

④ 원망 사 허믈 랴

년 학 간고사 대비2013 2 현대고 대비

ECN-0102-2013-001-000076193

⑤ 죠 뫼 티 시 가( ) ( ) 粥早飯 朝夕

가( )

거리

공신 후 심 늦도( )劉尋

식 없어 과 께 산에 드리고 신

태몽 꾼 에 만고 웅 상 지닌 들

낳 키운다 그 후 신 들 에 역심 ( )逆心

담 귀 등 심 여 리 귀

보내고 지 죽 는 도망 가다

가 만 죽 고 에 경 가는 들 도움

살 다 그러 사 에 심 귀

보고 담 여 고 강 주가 승상

득 여 고 신 사 삼는다 그 후 강 승상

에게 심에 상 리지만 여움

사 귀 가게 다 강 승상 몸 는

연 과 헤어 리 다

경쇠 리 들리 에 들어가니 색

에 게 단청 누각과 큰 집들( )丹靑

다 주 보니 ( ) (一柱門 黃金

산 사 어 었다 산) lsquo rsquo 大字

들어가 고승 다 그( ) ( ) 山門 高僧

거동 보니 눈 눈 듯 고

변 같 귀는 어 에 늘어 니( ) 白邊

맑고 어 골격과 신 평 니었

팔염주를 에 걸고 육환장 을 고서 흑포( )六環杖

장삼에 떨어 송낙 쓰고 나 유생을 고 말[ ] 松蘿

하 를

소 이 연 하여 유상공 는 행차를 동 에 나ldquo

가 맞이하 하 으니 소 의 함을 서하

rdquo

유생이 크게 놀라 말하 다

천한 인생으 팔자가 하여 어 서 를 잃고ldquo

정처없이 다니다가 연히 이곳에 대사를 만난 것인데

토 대하 소생의 성은 어떻게 알고 있 니 rdquo

노 이 답하여 말하 를

어제 남악 형산 의 화선 이 소 의 절에ldquo ( ) ( )男樂 衡山

어서 소 에게 탁하 를 내일 낮 경에 남경 lsquo 12

동성 안에 사는 유 의 아들 충 이가 것이니 내쫓

말고 잘 대접하라 하셨 니다 마침 소 이 찾아 나rsquo

다가 상공의 차람새를 니 남경 사람이 에 알아

았 니다rdquo

유생이 말을 듣고 한편으 쁘고 한편으 퍼하

서 노 을 따라 들어가니 여러 들이 합장 하

가 했다 노 이 에 들어가 저녁 을 은 후에

을 편히 니 이곳은 선경 이었다 세상의 일을( ) 仙境

두 잊고 일 이 편안하 다 이후 는 노 과 함

서 도 이 탐 하고 경도 확하게 의 게 되었( )兵書

다 이 게 되니 대 천 에 가객 은 없 ( ) ( )大明天地 佳客

고 덕산 속에 리 른 만 있더라 래 ( ) 廣德山

이 천상 사람으 살아 있는 처를 만나 이한

을 니 재주 민함을 누가 당할 있겠는가

낮으 공 하더라

유충 전- -

웬늠 어가 사 싸다냐( ) ldquo rdquo

내가 가 막 런거 니

보통것 닐러 그 어낸ldquo ( )

틀어 주 그 가 루 러 허 에

싶어 키 틀어 주 그 가( )

루 허 우간 곡 틀어주는 루 못 는

웂는 고 닝께 고 지 들

어 사는 고 가 다는 건 에 그 집에

rdquo

그런 단 어들 어 새벽에 떼죽 거

다 고 어 보니 죄다 허 게 집어진

는 것 었다

총 가 내화를 꿴 뛰어나 만 아 소 없는

일이었다

어떻게 된 거야ldquo rdquo

한동안 넋나간 듯이 서 있던 총 가 하고많은 사람

에 하필이 유자를 겨냥하 은 말이었다

쎄유 아마 새에 고뿔이 들었던 개비네유ldquo rdquo

유자는 러 딴청을 하 다

야 고 가 에서 감 가 들어 죽는 고 두ldquo

어rdquo

총 는 가 혐의자 나 되는 것처럼 화풀이를( )嫌疑者

하 드는 것이었다

는 비위가 상해서

야 팔자가 사나서 이런 후 에 살라니|

여러 가 다 객고가 쌓여서 조 두 안 좋았을 테 helliphellip

런디다가 릇쓰 이 가락을 트는 대 디립

다 춰댔으니 과 해서 살끼두 다소 있었을 테 helliphellip

래 들어서 키 는 새끼덜일 이 다다 탈이 많은

이니 ldquohelliphellip

는 트의 독성을 충 히 내 않고 고 를 넣

은 것이 탈이었으 니 하 서도 러 참으 의 을 떨

었다 략 - -

마리가 마리 값 간다는 워ldquo

그냥 내뻔지 거시 허 싼 고 는 맛

겄다 싶 허 게 눌 강 어helliphellip

허 마늘 통 다

년 학 간고사 대비2013 2 현대고 대비

ECN-0102-2013-001-000076193

게 지 고뿌 지 rdquo

어 어째ldquo rdquo ldquo rdquo

런 도 것들 같 니ldquo ( ) rdquo殘忍無道 helliphellip

는 탱 여 지 못 다 보( ) 憤氣撐天

니 는 는 다 동원 여 통 쳤

생각 여 는 눈 다

달리 리헐 감ldquo rdquo

들 고 말 니었다 그가

는 것 그 말고는 없었 에 그 게 뒷동

달 거 다

는 우 럽고 식 짝 없는 랫것들 고

다 공연 신 가고 득 것

없다고 단 는지 결 웬만큼 고루 어

그 것들 쪽 에다 고 어주지ldquo

고 그 그걸 주 어 에 에 helliphellip

눈 없는 독 들 rdquohelliphellip helliphellip

고 말 럼 얼거리 들어가 리는 것 었

- ( ) -兪子小傳

zb19) 위 나 를 읽고 평가한 것으 적절하( ) 않은 것

① 사 리 통 감과 사실 고

격 과 달 고

② 는 가 재 컫는lsquo rsquo lsquo rsquo

미 가진 여 는 것 겠

③ 는 식 말 는 웃 상lsquo rsquo

여 는 미 지니고

④ 는 어가 죽 짐 지만 내색 지 고lsquo rsquo

말 고

⑤ 언어 통 가들 여 우리 통

것들 역 고

가 체 거리( ) [ ]

나라 종 연간에 정언주 의 을 하고( )正言注簿

있던 유 은 늦도 자 이 없어 한탄하다가 남악 형산lsquo rsquo

에 치성을 드리고 이한 태 을 꾼 뒤 아들을 낳아 이름

을 충 이라 고 키 다 이때 조정의 하들 에 역

을 품은 정한담 최일 등이 가달의 침입에 대한( ) 逆心

유 의 유화적 입장을 제 삼아 유 을 함하여 양

내고 유 의 에 을 러 충 자마저 살해하

한다 러나 충 은 천 조 정한담의 마 에서 어

나 많은 고난을 겪다가 은퇴한 재상 강희주를 만나 사위

가 된다 강희주는 유 을 하 고 상소를 으나 정

한담의 공격을 아 양을 가게 되고 강희주의 가족은

난을 피하여 두 흩어 다 충 은 강 소저 이 하고

사의 노 을 만나 를 때를 다린다 이

때 남적과 적이 를 들고 나라에 쳐들어 자 정한

담은 자 출전하여 남적에게 항 하고 남적의 선 장이

되어 천자를 공격한다 정한담에게 여러 패한 천자가

항 하 할 음 충 이 등장하여 남적의 선 정 걸

을 죽이고 천자를 출한다 충 은 단 으 란 을

쳐 고 정한담을 사 잡는다 리고 호 에게 ( )胡王

잡혀간 황후 태후 태자를 출하 유 에서 고생하

던 아 유 과 장인 강희주를 한다 또한 이 하

던 어 니 아내를 찾고 정한담 일파를 리친 뒤 높은

에 라서 화를 누린다

사 들 별 고 없 다니었다( )

마 마 돌 다니 걸 여 고

어 곤 다 에는 동쪽에 고

에는 쪽에 니 가 에 리는 엽

가는 없 니 늘 다니는 었

다 얼 말 죽 사 같고 림새가 말

니었다 가슴 에 고 등

삼태 헌 에 니 달 ( )奇男子

가 도리어 걸 었 담 만 열 도 ( )傅說

고 만났고 만 갈( ) ( ) 慇 武丁

도 탕 만났( ) ( ) (伊尹 成湯 渭

여상 도 주 만났는) ( ) ( ) ( ) 水 呂尙 周 文王

월 같 러가 도 어느 열 살

늘과 집 삼고 사 에 쳐 거리에

어 다가 곳에 니 다 ( ) 楚

지 다가 사 보고 가에 다다( )長沙

니 망 가에는 원 리가 슬 고 가

가 내리는 사 에는 갈매 가 갈 뿐 었다

쪽 돌 보니 가 우거 고

가 사 보 었다 그곳에

가니 는 사( ) 汨羅水

는 다 주 가 쓰고 죽고

곳 었다

마 감 여 에 가 사 살펴보니

에는 삼 고 그 에( ) 屈三閭

는 만고 월 과 지 가는 그 들( )風月

가 어 었다( ) 路程記

동쪽 벽 에 새 운 어 거늘 그

보니 월 에 경 주 는 간신에게ldquo ( )敗

보고 연경 귀 가다가 에 죽 rdquo

거늘 그 보고 에 거꾸러

통곡 말

[A]ldquo우리 연경 간 만 니 에

지 살 상에 엇 겠는가

에 고 에 었 니

상에 살 것 가 도 께 지리 rdquo

년 학 간고사 대비2013 2 현대고 대비

ECN-0102-2013-001-000076193

고 가에 내 가니 울 리가 에 지 사

쳤는지 심 심 것 가

신 심 것 가

다( ) 강 승상에게는 들 없고 다만 만

었다 가 낳 에 가 색

타고 내 에게 말 는 ldquo

니다 미원 과 연 맺고 ( ) ( )紫薇垣 緣分

었는 께 강 집 보내 에

니 게 여겨 주십시 거늘 rdquo

미 가운 낳 니 가 고 거동

단 다 시 짓 쓰 고 는 (音

없었 니 여 가운 지 는 짝) 律

룰 만 사 없었다 가 사 여 사 감

게 고 지 못 고 염 는 만다

다가 당에 거 고 식같 러 내니

고귀 상 루 말 다 어 울 도 다( ) 相

귀 사 없고 웅 걸( )富貴爵祿

만고 었다 승상 매우 뻐 내당 ( )內堂

들어가 에게 사 니 역시 매우 거

워 말 다

ldquo 도 마 사 는 승상께

그 게 말 시니 상 여러 말 지 말고

사 도 시다rdquo

상이 에 나 충 의 손을 잡고 결혼과 하여 ldquo

너에게 히 할 말이 있다 내가 늙은 말년에 딸

하나만을 두었는데 니 너 하늘이 정해 필

임이 하다 이제 년고락 을 너에게 탁 ( )百年苦樂

하겠다 하 대 충 이 릎을 꿇고 앉아 눈 을 흘리rdquo

여쭈었다

소자의 을 해주 고 또 하 에 두고자 하ldquo ( )膝下

니 감사하 이를 데가 없 니다 다만 가 속에 통탄

할 일이 사 쳐 있 니다 소자가 이 없어 양친 ( )兩親

의 생사를 른 채 결혼하여 아내를 얻는 것은 자 으

서 할 도리가 아닙니다 이것이 한 러 뿐입니다 rdquo

승상 그 말 듣고 슬 에 어 고

것 에 맞 어 변 게 리ldquo

는 다 집 시 공 도 여 ( )始祖公

고 가 에 가가 어진 만 개 공신

었 니 도 러워 말 시고 시 rdquo

택 여 니 다운 신 과 신

습 늘에 죄 짓고 간 상에 내 신

혼 를 다 끝내고 으 들어가 사 을 살펴 니 빛

나고 빛난 것이 한 입으 는 다 말하 어 고 하나

는 다 하 어 더라 에 켠 환한 촛 ( )新房

아래 은 에 랑과 가 평생의 연 을 맺었( )緣分

으니 서 사랑하 주고 은 말을 어떻게 다 헤아릴

있으 어떻게 다 하리 을 낸 후에 이튿날

상 를 니 상 거 마음을 이 하

더라

각 생 강 승상 집 쪽( )

늘 보고 없 가 신 신 생각 니

없고 어 없었다 는 어떻게 도리가 없다

여 산 에 들어가 리 고 어 도 닦

고 다 그 산 보고 가다

가 곳에 다다 니 에 큰 산 었다 많 우

리 골짜 가 늘 는 가운 색

에 고 갖가지 가 짝 어 었 ( )花草

다 신 산 생각 고 들어가니 경개 ( )景槪

가 매우 뛰어 고 경 산 다 산 리에 들

리는 것 리 보 는 것 울 청산뿐

었다 가 고 울 어 가

니 들 많 가지들 못 어 동

에 늘어 들거리 는( ) 洞口

우거진 가지에 갖 들 다 었다( ) 春情

계상 에는 공 는 늘( ) 花溪上

에 걸린 폭포가 벽 는 리는 산사( )層巖絶壁

쇠 리 객 에 는 듯 늘( ) ( ) 寒山寺 客船

에 싸여 는 습 산

그린 여 병 러 듯 다 경쇠 리가 들

리 에 들어가니 색 에

게 단청 누각과 큰 집들 다( ) 丹靑

주 보니( ) ( ) lsquo一柱門 黃金大字

산 사 어 었다 산 들rsquo ( )山門

어가 고승 다 그 거 보니( ) 高僧

눈 눈 듯 고 변 같 ( )白邊

귀는 어 에 늘어 니 맑고 어 골격

과 신 평 니었다 염주

에 걸고 짚고 포 삼에 어진( )六環仗

쓰고 생 보고 말

승 연 여 상공 시는 동 에ldquo

가 맞 지 못 니 승 십시 rdquo

생 크게 말 다

생 가 여 어 고ldquo

없 다니다가 우연 곳에 사 만 것

그 시 생 어떻게 고 습니 rdquo

승 답 여 말

어 산 승 에ldquo ( ) ( )南岳 衡山

시어 승에게 탁 내 낮 시경에 경 lsquo 12

동 에 사는 심 들 가 것 니 내쫓

지 말고 습니다 마 승rsquo

다가 상공 림새 보니 경 사 에 보

습니다rdquo

zb20) 위 의 친 에서 서 자의 개입이 드러나~

는 이 아닌 것은

① 달 가 도리어 걸 었( ) 奇男子

② 신 심 것 가

년 학 간고사 대비2013 2 현대고 대비

ECN-0102-2013-001-000076193

③ 다운 신 과 신 습 늘에 죄 짓고

간 상에 내 신 다

④ 사 주고 말 어떻게 다 헤 릴

어떻게 다 리

⑤ 신 산 생각 고 들어가니 경개 가 ( )景槪

매우 뛰어 고 경 산 다

거리 연[ ] ( )弘治

간 에 공신 후 에 언(1488~1505) ( )正言

주 는 벼슬 심 늦도 식( ) ( )主簿 劉尋

없어 과 께 산에 드리고 신 태

몽 꾼 에 만고 웅 상 지닌 들 낳

키운다 그 후 신 들 에 역심( )逆心

담 귀 등 심 여 리 귀 보내

고 지 죽 는 도망 가다가

만 다 에 에 어 니

헤어지게 다

에 에 어 니 헤어지게

다 그 후 사 들에게 우연 돌

생 다가 어느 열 살 었다 열 살

지 다가 우연 귀 견 는

그것 그 살 도 었고 그

귀 본 신도 지 죽고 마 고

크게 운다

( )

에는 강 주 는 재상 살고 었

니 시 에 과거에 격 여 승상 벼슬 다가 간

신 만 벼슬 그만 고 고 돌 었

다 그러 신 지 가 지 못 여 상

가 못 결 는 상 여 원 니

신 들 그 직간 꺼 다 그 에 도

담과 귀가 강 승상 가 미워 다

강 승상 마 본 에 갔다가 돌 는[A][ ( )本府

에 우편 주 에 다가 색( ) ( ) 右便 酒店

에 어리었는 청룡 에 지 늘

여 통곡 고 사 는 꿈 꾸었다] 마

상 게 생각 여 새 다리다가 새벽

닭 울고 가 달 갔다 가 보니

과연 어 동 가 가에 울고 는지 달

들어 그 고 사 에 어 말

는 어 어 에 어 가ldquo

닭 곳에 우느냐 니 울rdquo

그 고 답 여 말 다

는 경 동 에 사는 언 주 공 들ldquo

니다 께 간신 만 연경 귀 가

시다가 에 죽 사 에 는 닭에

도 에 죽고 니다rdquo

강 승상 말 듣고 크게 낯 변 말

것 웬 말 냐 근 동 ldquo (老

못 갔 니 그 사 변 여)患

런 변 었단 말 가 주 는 신 다

같 에 벼슬 다가 는 가 많 들어 고

돌 는 주 가 게 꿈 에 생

각 겠느냐 생각지 못 다 미 지 간

지지 말고 께 가 략rdquo ( ) hellip hellip

죽게 주 사당에 단 도 러운

겠느냐 말 말고 시는지 rdquo

어 없어 강 승상 가니 그곳

월계 었다

다( )

가가 고 지 사 들 가( )櫛比

통 는 리가 과

답게 꾸민 누각과 큰 집들 늘 고

게 식 가 어 들 태운 가고

었다 략 강 승상에게는 들 없고 ( ) hellip hellip

다만 만 었다 가 낳 에

가 색 타고 내 에게 말

는 니다 미원 과ldquo ( )紫薇垣

연 맺고 었는 께 강 집( )緣分

보내 에 니 게 여겨 주십시

rdquo

거늘 미 가운 낳 니 가

고 거동 단 다 시 짓 쓰 고

는 없었 니( )音律 여 가운

지 는 짝 룰 만 사 없었다 가 사

여 사 감 게 고 지 못 고 염 는 만다

다가 당에 거 고 식같

러 내니 고귀 상 루 말 다 ( )相

어 울 도 다 귀 사 없 ( )富貴爵祿

고 웅 걸 만고 었다 승상 매우 뻐

내당 들어가 에게 사 니( ) 內堂

역시 매우 거워 말 다 도 마 ldquo

사 는 승상께 그 게 말 시니

상 여러 말 지 말고 사 도 시다rdquo

( )

승상 에 고 결 과 ldquo

여 에게 말 다 내가 늙 말 에 지

만 었는 지 보니 늘

다 에게 탁 겠 ( )

다 신 꿇고 눈 리rdquo

여 었다 주시고 슬 에 ldquo ( )膝下

고 시니 감사 룰 가 없습니다 다만 가슴

에 통탄 사 쳐 습니다 복 없어

생사 결 여 내 얻는 것( )兩親

식 도리가 닙니다 것 러울 뿐 니

다rdquo

상 그 말 듣고 슬 에 어 고 말

것 에 맞 어 웅변 ldquo

년 학 간고사 대비2013 2 현대고 대비

ECN-0102-2013-001-000076193

게 리 는 다 집 시 공도 여

고 가 에 가가 어진 만 개 공신

었 니 도 러워 마 시고 rdquo 시

택 여 니 운 신 과 신

습 늘에 죄 짓고 간 상에 내 신

다 략 지낸 후에 튿 승상 ( ) hellip hellip

니 승상 거운 마 지 못

마( )

듯 월 러 생 열다 살 었

다 에 승상 어진 사 얻고 만 에 근심 없었

다만 주 가 간신 에

죽 것 생각 마 곧 어 곤

다 그 에 주 원통 어

없 고 여 시 가 거늘 생 만

여 다

말 감격 러우 간신 에 가득 여ldquo

고 니 께 상 듣지 니 것

니다rdquo

승상 듣지 고 가

퇴 재상 공달 집에 거 고 상 지어

승지 러 께 리

( )

뒷 거리 강 승상 에게 상 리지[ ]

만 여움 사 귀 가게 다 강 승상

몸 는 연 가 헤어

리 다 산 들어간 룡사 승 만

게 다 승 만 우 다릴

과 들고 략 다 담

원 여 에게 복 고 어

공격 다 담에게 여러 가( ) 天子

복 등 여 다 단

신 리쳐 담 사 고 에게

간 후 태후 태 여 지에 고생

지 심과 강 주 여 개 다 헤

어 어 니 내 고 담 리

벼슬에 귀 누리게 다

zb21) 위 의 인 간 계를 같이 나타냈을lt gt

때 에 대한 이해 가장 적절하 ~ 않은 것은

① 계에 주 는 계 심 열

상 에 다고 다( ) 水深火熱

② 계는 견원지간 고 다( ) 犬猿之間

③ 계는 달리 막역지 계 고( )莫逆之交

④ 연결 사 컬어 재 가 고( )才子佳人

⑤ 는 생 과 볼 ( )匹夫匹婦

가 재 는 는 심 고 매사에 생( )

각 고 능 도 어 가 에게 많lsquo rsquo

도움 사 다 그는 에게 거 에

꺼리 없 거 났다고 는

매우 싫어 고 신 들

는 사 다

내가 지 리에( ) 1970

사 실에 지 월간ldquo

편집 고 어 었다rdquo

어느 없 가 쑥 다 도 어 10

후 다 산 시 럼 어 엇 어 ( ) lsquo怡山

다시 만 랴 니 그는 재 그룹 승 운rsquo

사가 고 는 고 거 누

주는 가 없는 가가 어 다시 만 게 것

었다

다 보통 것 닐러 그( ) ldquo 어낸 ( )

틀어주 가 루 러 허 에

싶어 키 틀어주 그( )

가 루 허 우간 곡 틀어 주는 루 못

는 는 고 닝께 고 지

들어 사는 고 가 다는 건 에 그 집에

rdquo

그런 단 어들 어 새벽에 떼죽 거

다 고 어 보니 죄다 허 게 집어진

는 것 었다 가 실내 꿴 뛰어 지만

없는 었다

어떻게 된 거야 한동안 넋나간 듯이 서 있던 총ldquo rdquo

가 하고많은 사람 에 하필이 유자를 겨냥하 은

말이었다 쎄유 아마 새에 고뿔이 들었던 개비네 ldquo

유rdquo

유자는 러 딴청을 하 다 야 고 가 에서 ldquo

감 가 들어 죽는 고 두 어rdquo 총 는 가 혐의

자 나 되는 것처럼 화풀이를 하 드는 것이었다( )嫌疑者

라 이 어쩌 어 유( ) ldquo rdquo ldquo rdquo

애유 이런 잔인 도 한 것들 같으니ldquo ( ) rdquo殘忍無道 helliphellip

총 는 탱천 하여 쩌 를 하 다( ) 憤氣撐天

아하니 아는 자는 다 동 하여 호통을 쳤으 하나 혈

압을 생각하여 참는 눈치 다 달리 처리헐 두 ldquo

잖은감유rdquo

총 의 성 을 덧들이 고 한 말이 아니었다 가 할

년 학 간고사 대비2013 2 현대고 대비

ECN-0102-2013-001-000076193

있는 것이 말고는 없었 때 에 게 뒷동

산을 달은 거 다

이 유자소전- lsquo rsquo

zb22) 의 상황을 속담으 표현한 것으 적절한 것은

① 루 곳 게 마 다

② 에 맞고 강에 눈 다

③ 늘 도 다

④ 도 사 다

⑤ 에 가도 신만 리 다

거리 공신 후[ ]

에 주 는 벼슬 심 늦도( )主簿

식 없어 과 께 산에 드리고 신

태몽 꾼 에 만고 웅 상 지닌 들

낳 키운다 그 후 신 들 에 역심

담 귀 등 심 여 리 귀 보내고

지 죽 는 도망 간다 그

만 고 에 에 어 니

헤어지게 다 지 가 사 들에

사 들 별 고 없 다니었다

마 마 돌 다니 걸 여 고

어 곤 다 에는 동쪽에 고 에

는 쪽에 니 가 에 리는 엽 가는

없 니 늘 다니는 었다

얼 말 죽 사 같고 림새가 말 니었

다 가슴 에 고 등 삼태

헌 에 니 달 가 도리 ( )奇男子

어 걸 었 담 만 열 도 ( ) ( )傅說 慇

고 만났고 만 갈( ) ( )武丁 伊尹

도 탕 만났( ) ( )成湯 渭水

여상 도 주 만났는 월( ) ( ) ( ) 呂尙 周 文王

같 러가 도 어느 열 살

늘과 집 삼고 사 에 쳐 거리에

어 다가 곳에 니 다 ( ) 楚

지 다가 사 보고 가에 다다( )長沙

니 망 가에는 원 리가 슬 고 가

가 내리는 사 에는 갈매 가 갈 뿐 었다

쪽 돌 보니 가 우거 고

가 사 보 었다 그곳에

가니 는 사( ) 汨羅水

는 다 주 가 쓰고 죽고

곳 었다

마 감 여 에 가 사 살펴보니

에는 삼 고 그 에( ) 屈三閭

는 만고 월 과 지 가는 그 들( )風月

가 어 었다( ) 路程記

동쪽 벽 에 새 운 어 거늘 그

보니

월 에 경 주 는 간신에게ldquo ( )敗

보고 연경 귀 가다가 에 죽 rdquo

거늘 그 보고 에 거꾸러

통곡 말

우리 연경 간 만 니ldquo ( )燕京

에 지 살 상에 엇 겠는

가 에 고 에 었 니

상에 살 것 가 도 께 지리 rdquo

고 가에 내 가니 울 리가 에 지

사 쳤는지 심 심 것 가

에는 강 주 는 재상 살고 었

니 시 에 과거에 격 여 승상 벼슬 다가 간

신 만 벼슬 그만 고 고 돌 었

다 그러 신 지 가 지 못 여 상

가 못 결 는 상 여 원 니

신 들 그 직간 꺼 다 그 에 도

담과 귀가 강 승상 가 미워 다 강 승상 마

본 에 갔다가 돌 는 에 우편 주( ) ( )本府 右便

에 다가 색 에 어리었는 청룡( ) 酒店

에 지 늘 여 통곡 고

사 는 꿈 꾸었다 마 상 게 생

각 여 새 다리다가 새벽닭 울고

가 달 갔다 가 보니 과연 어 동 가

가에 울고 는지 달 들어 그

고 사 에 어 말

는 어 어 에 어 가ldquo

닭 곳에 우느냐rdquo

니 울 그 고 답 여 말 다

는 경 동 에 사는 언 주 공 들ldquo

니다 께 간신 만 연경 귀 가

시다가 에 죽 사 에 는 닭에

도 에 죽고 니다rdquo

강 승상 말 듣고 크게 낯 변 말

것 웬 말 냐 근 동ldquo ( )老患

못 갔 니 그 사 변 여 런 변

었단 말 가 주 는 신 다 같

에 벼슬 다가 는 가 많 들어 고 돌

는 주 가 게 꿈 에 생각

겠느냐 생각지 못 다 미 지 간 지지

말고 께 가 rdquo

뒷 거리 강 승상 도움 죽 고[ ]

년 학 간고사 대비2013 2 현대고 대비

ECN-0102-2013-001-000076193

고 그 과 결 여 사 가 다 그러 강

승상 에게 울린 상 강 승상 귀 가고

과 헤어 리 승 만 게 다

승 우 다릴 과

들고 략 다 담 원

여 에게 복 고 어 (天

공격 다 담에게 여러 가 복) 子

등 여 다 단신

리쳐 담 사 고 에게 간

후 태후 태 여 지에 고생 지

심과 강 주 여 개 다 헤어

어 니 내 고 담 리 벼

슬에 귀 누리게 다

미상- lsquo ( )-劉忠烈傳

zb23) 위 과 의 서사 조를 비 한 것으 적절하lt gt

않은 것은

보lt gt

믿지 고 결 여 곱

낳 다 곱째 공주 낳 가

리게 다 리 만 고 진 공주는 lsquo rsquo

리공 미 리공 에 키워진다 월

러 과 가 죽 병에 걸 는 승에 는

어 산다고 다 여 들에게 탁

지만 거 리 는다 리 는 과

승 다 승 지 가는 에 많

만 지만 보살 도움 사 도 다

그러 승 신과 결 여 시

들어 주겠다고 다 리 는 그 결

여 들 곱 낳 후에 신

얻게 다 돌 리 는

에 과 상여 만 지만 여 과

살 낸다 훗 리 그 공 우 죽 사

승 도 는 신 다

리-lsquo rsquo-

① 복 결말에 고 다

② 웅 에 탕 고 다

③ 시 겨 내고 귀 누리는lsquo rsquo

보 리 는 월 재 신 다lt gt lsquo rsquo

④ 과 보 리 는lsquo rsquo lt gt lsquo rsquo

도움과 어 신 능 극복 고

⑤ 등 여 시 겪는lsquo rsquo

보 리 는 닌 지lt gt lsquo rsquo

림 시 겪는다

가 각 고 에( ) ( )却說

살 없었다 략 사 들 슬 에 어 lt gt

가에 내 고 가고 싶 가 고 후

워 경

사 들 별 고 없 다니었다 lt

략 얼 말 죽 사 같고 림새가 말gt

니었다 가슴 에 고 등

삼태 헌 에 니 달 가 ( )奇男子

도리어 걸 었 담 만 열 도( )傅說

고 만났고 만 갈( ) ( ) 殷 武丁

도 탕 만났( ) ( ) (伊尹 成湯 渭

여상 도 주 만났는) ( ) ( ) ( )水 呂尙 周 文王

월 같 러가 도 어느 열 살

늘과 집 삼고 사 에 쳐 거리에

어 다가 곳에 니 다 ( ) 楚

지 다가 사 보고 가에 다다( )長沙

니 망 가에는 원 리가 슬 고 가

가 내리는 사 에는 갈매 가 갈 뿐 었다

쪽 돌 보니 가 우거 고

가 사 보 었다 그곳에

가니 는 사( ) 汨羅水

는 다 주 가 쓰고 죽고

곳 었다

에는 강 주 는 재상 살고( )

었 니 시 에 과거에 격 여 승상 벼슬 다

가 간신 만 벼슬 그만 고 고 돌

었다 략 강 승상 마 본 에 갔다가 돌 lt gt ( )本府

는 에 우편 주 에 다가 색( ) ( ) 右便 酒店

에 어리었는 청룡 에 지

늘 여 통곡 고 사 는 꿈 꾸

었다 마 상 게 생각 여 새 다리다

가 새벽닭 울고 달 갔다 가

보니 과연 어 동 가 가에 울고 는지

달 들어 그 고 사 에

어 말

는 어 어 에 어 가ldquo

닭 곳에 우느냐rdquo

니 울 그 고 답 여 말 다 lt

략gt

년 학 간고사 대비2013 2 현대고 대비

ECN-0102-2013-001-000076193

생각 여 가 고 시 는ldquo ( )大人

상에 다시없는 니다 살 엇 겠습니

에 돌 가시고

가에 돌 가 니 살 마 없습니

다 략 어 없어 강 승상 가니rdquo lt gt

그곳 월계 었다

다 강 승상에게는 들 없고 다만 만( )

었다 가 낳 에 가 색

타고 내 에게 말

는 니다 미원 과ldquo ( )紫微垣

연 맺고 었는 께 강 집( )緣分

보내 에 니 게 여겨 주십시

rdquo

거늘 미 가운 낳 니 가

고 거동 단 다 시 짓 쓰 고

는 없었 니 여 가운( ) 音律

지 는 짝 룰 만 사 없었다 가 사

여 사 감 게 고 지 못 고 염 는 만다

다가 당에 거 고 식같 러

내니 고귀 상 루 말 다 어 ( )相

울 도 다 귀 사 없고 ( )富貴爵祿

웅 걸 만고 었다 승상 매우 뻐 내

당 들어가 에게 사 니 역( ) 內堂

시 매우 거워 말 다

도 마 사 는 승상께ldquo

그 게 말 시니 상 여러 말 지 말고 사

도 시다 략 시 택 여rdquo lt gt

니 다운 신 과 신 습 늘에 죄

짓고 간 상에 내 신 다

다 내고 들어가 사 살펴보니

고 것 는 다 말 어 고

는 다 어 신 에 ( )新房

에 신 과 신 가 평생 연 맺었( )緣分

니 사 주고 말 어떻게 다 헤 릴

어떻게 다 리 지낸 후에 튿 승

상 니 승상 거운 마 지 못

( ) 듯 월 러 생 열다 살

었다 에 승상 어진 사 얻고 만 에 근심

없었 다만 주 가 간신

에 죽 것 생각 마 곧 어

곤 다 그 에 주 원통

어 없 고 여 시 가 거늘 략 lt gt

략 거리

강 승상 에게 상 리지만 여움

사 귀 가게 다 강 승상 몸 는

연 과 헤어 리 다

마 각 생 강 승상 집 쪽( )

늘 보고 없 가 신 신 생각 니

없고 어 없었다 는 어떻게 도리가 없다

여 산 에 들어가 리 고 어 도 닦

고 다 그 산 보고 가

다가 곳에 다다 니 에 큰 산 었다 많

우리 골짜 가 늘 는 가운 색

에 고 갖가지 가 짝 어 ( )花草

었다 략 주 보니 lt gt ( ) (一柱門 黃

산 룡사 어 었다) lsquo rsquo 金大字

산 들어가 고승 다 그( ) ( ) 山門 高僧

거동 보니 눈 눈 듯 고

변 같 귀는 어 에 늘어 니( ) 白邊

맑고 어 골격과 신 평 니었

다 염주 에 걸고 짚고 포 ( )六環杖

삼에 어진 쓰고 생 보고 말

승 연 여 상공 시는 동 에ldquo

가 맞 지 못 니 승 십시 rdquo

생 크게 말 다

생 가 여 어 고ldquo

없 다니다가 우연 곳에 사 만 것

그 시 생 어떻게 고 습니

rdquo

승 답 여 말

어 산 승 에ldquo ( ) ( )南岳 衡山

시어 승에게 탁 내 낮 시경에 경 lsquo 12

동 에 사는 심 들 가 것 니 내쫓

지 말고 습니다 마 승rsquo

다가 상공 림새 보니 경 사 에 보

습니다rdquo

생 그 말 듣고 편 고 편( )

슬 승 들어가니 여러 승 들

가워 다 승 에 들어가

후에 그 편 니 곳 경 었다 상( ) 仙境

고 신 편 다 후 는 승과

께 병 도 탐 고 경도 게( )兵書

게 었다 게 니 지 에 가객 ( ) ( )大明天地 佳客

년 학 간고사 대비2013 2 현대고 대비

ECN-0102-2013-001-000076193

없고 산 에 리 만 본 ( ) 廣德山

신 상 사 살 는 만

우고 늘 월 신 과 늘 ( )日月聖神

산 신 들 다 니 그 재( ) 名山神靈

주 민 누가 당 겠는가 낮 공

zb24) 다 에 해당하는 내 으 적절하( ) 않은 것은

① 강 티 통 당시 능 다

② 상계 지상계 경 는 원 계 드러

③ 실에 어 없는 실 가 타 는

④ 뛰어 재주 어 가진 고

등 다

⑤ 가 직 개 여 평가 내리는

편집 평 타 다lsquo rsquo

가 본격 가 동 것 지( )

다 단 상 에2003 lsquo rsquo

들어가 드럼 연주 다 취미 생 달리

들었다는 보 우 가 들ldquo

어 틱 린 도 다 고 말 다rdquo

경 는 가 망 없( ) lsquo

티 원 고 답 다 신과 같 시각rsquo

는 습 상상 만 도 감동

다 시각 연주 동시에

열 상 는

티 원 그런 열 경 럽다는 것 다

다 역시 엄청 다 본( )

에 복 들

고쳐 가고 다 신 에 얼

마 지는 고 리가 는 지도 생님

가 훈 고 많 고쳐 다

고 말 다

그러 직도 에 지 는 다 그는

체격 지 못 게 가 큰 만

체 운동 훈 과 께 체 늘 동 50

는 게 고 말 다

에게는 꿈 다 통 누 가( )

주겠다는 것 그 꿈 다 신 극복 는

과 에 큰 경험 들도 느 게

주고 싶다는 것 다

마 슬 마다( ) ldquo 통

낼 었 것 럼 고통 는 사 들

고 겠다 고rdquo

말 다 달 루 첫 낸 lsquo rsquo

첫 드 심 집에 는 리듬 드 2

루 에 도 보고 싶다 집 에는 직(RampB) 3 4

사 곡 도 보 고 싶다고 포 다middot

zb25) 에서 가장 유사한 의 를 닌 어를lt gt

찾아 쓰

lt gt

나는 이제 너에게도 픔을 주겠다

사랑 다 소 한 픔을 주겠다

겨 거리에서 개 놓고

살아 추위 떨고 있는 할 니에게

값을 으 서 뻐하던 너를 위하여

나는 픔의 평등한 얼 을 여 주겠다

내가 어둠 속에서 너를 를 때

단 한 도 평등하게 어주 않은

가마니에 덮인 동사자가

다 얼어 죽을 때

가마니 한 장조차 덮어주 않은

한 너의 사랑을 위해

흘릴 르는 너의 눈 을 위해

나는 너에게 이제 너에게도 다림을 주겠다

지 울 포동 여고 생들17

틈 없 가득 체 에 맑 울

다 죽 듣 생들 사 에

연 는 탄 다 객들 도 는lsquo rsquo

가 보 주 공 맹 가 운 는

단 그룹사운드 루 보컬 맡고 는lsquo rsquo

시각 지 었다17 1

근 다만과 가 거lsquo rsquo lsquo

꿈 고 퇴 내가 다rsquo

간 간에 지 지 연 생들 짧lsquo rsquo lsquo rsquo

가 운 듯 리에 어

연 다 내 사 고 퇴lsquo rsquo

과 루 들 결 다시 돌lsquo rsquo

들 고 사 들 에 당당

것 니다 내 태어

볼 없었 크고 열여

년 학 간고사 대비2013 2 현대고 대비

ECN-0102-2013-001-000076193

에도 고 시 얻지 못 다

감지 없는 시각 상태 다

신 지에 고 상 원망 도

단다 어느 가 에 시각 에 ldquo

어 그런 듣고 다 보니 내가 게 lsquo

살 는지 도 눈 고 싶rsquo lsquohelliphellip

보 는 생각만 들 고 그 가 들에게rsquo

도 내고 들도 고 많 었죠 들 rdquo

었 지 새 는 에 쑥 러운 색

어났다

생에 것 단연 었다lsquo rsquo

공연에 거 꿈lsquo rsquo

는 다 특 가사 갑게 는 운 lsquo

벽 에 당당 마주 어 언 가 그 벽

고 늘 어 거운 상도

없죠 내 삶 에 웃 그 께

는 다고 다rsquo

들었 그냥 런 도 고만 여ldquo lsquo rsquo

겼죠 그런 꾸 가사 미 새 다 보

니 통 는 가사 는 생각 들 고 (

가 게는 시각 는 생각 들고 들) ( )

마다 듣고 큰 얻었어 rdquo

에 진지 게 가에 미 가

zb26) 의 에 들어갈 말 적절한 것은lt gt ~

lt gt

난 난 꿈이 있었죠

고 찢겨 남 하여도

내 가 히 과 같이 간 했던 꿈

혹 때 누 가가 뜻 를 비 음

내 등 뒤에 흘릴 때도

난 참아야 했죠 참을 있었죠

날을 위해

늘 걱정하듯 말하죠

헛된 꿈은 독이라고

세상은 끝이 정해 책처럼

이 돌이킬 없는

현 이라고 helliphellip

래 난 난 꿈이 있어

꿈을 믿어

나를 켜

저 차갑게 서 있는 이란 앞에

당당히 마주칠 있어

출처 가 거위의 꿈 작사 이적 작곡 동률- lsquo rsquo ( )

① ② ③ ④ ⑤

가 떴다는 들 만 지만( ) lsquo rsquo

늘 겸 다 에 주 연 우승 지 간에도 3

단 생님께 만 지 고 고 만ldquo rdquo

큼 늘 겸 신 계 가

고 다

에게는 꿈 다 통 누 가

주겠다는 것 그 꿈 다 신 극복 는 과

에 큰 경험 들도 느 게 주

고 싶다는 것 다

슬 마다 통 낼ldquo

었 것 럼 고통 는 사 들

고 겠다 고rdquo

말 다 달 루 첫 낸lsquo rsquo

첫 드 심 집에 는 리듬 2

루 에 도 보고 싶다(RampB) 집 에는 직34

사 곡 도 보 고 싶다고 포 다

미 는( ) (26) 어 헤헤헤 웃다가 어ldquo rdquo

허허허 웃었다ldquo rdquo ldquo rdquo 같 도 고

상 다 는 같 도 다( ) 壯丁 킹 들lsquo

다 는 역도 보 그 다 지만 그는rsquo

뷰에 지 다 운동만 지 ldquo

것 지 간에 여러 사 도 역rdquo helliphellip

었다 그런 엇 그 마 움직 는지 보 쯤

지 담 사 다 훈 없어 그는 티

지 림 었다 태 다 갔다 는 습

마 집 럼 편 게 보 다

주말에는 주 엇 보내

주말에도 별 주 에 청ldquo

고 에 가고 도 쳐

에 듣고 보 에 갈 가 별 없

어 산 시 게 고 들어 2002

거 매 여 지냅니다 시 과 지훈 rdquo

다 근 간 과 진실 그리고 싶어( )

가 다 근에게 그것 진리 다 거 다 없

거 고 다 없 는 것 진리

다 근 진리는 후 쪽 었다 신산( )辛酸 삶

었 질곡( )桎梏 역사 에 지냈 가

눈에 든 것 료 단 료 게 보

것 었다 그것 그 에 겨우겨우

슬 슬 생 어가는 간들 었다

리 과 단 리 고리에 검 마

없 거리 돌

상 것 없는 등 근에게 상

과 진실 엄 ( )儼存 다는 사실 리는 가

실 고 가 과 역경 에 도 근 내 포

없었 후 보루( )堡壘 다 도 365

도 간 근 여

시 것 다

년 학 간고사 대비2013 2 현대고 대비

ECN-0102-2013-001-000076193

다 공주 그림 가 근 경- ( ) ldquo rdquo(

2009)

zb27) 작가의 주 적인 각이 드러난 것은~

① ② ③ ④ ⑤

가 신 지에 고 상 원망( )

도 단다 어느 가 에 시각 에 ldquo

어 그런 듣고 다 보니 내가 lsquo

게 살 는지 도 눈 고 싶rsquo lsquohelliphellip

보 는 생각만 들 고 그 가 들에게rsquo

도 내고 들도 고 많 었죠 들었rdquo

지 새 는 에 쑥쓰러운 색

어났다 략 [ ]

경 는 가 망 없 티lsquo

원 고 답 다 신과 같 시각rsquo

는 습 상상 만 도 감동

다 시각 연주 동시에

열 상 는 티

원 그런 열 경 럽다는 것 다 략 [ ]

슬 마다 통 낼ldquo

었 것 럼 고통 는 사 들

고 겠다 고rdquo

말 다 달 루 첫 낸 lsquo rsquo

첫 드 심 집에 는 리듬 2

루 에 도 보고 싶다 집 에는 직(RampB) 3 4

사 곡 도 보 고 싶다고 포 다

식 누 가-

고 싶어

다 역도 미 담 고 사( )

질 주말에는 주 엇 보내[ 1]

답 주말에도 별 주 에[ ] ldquo

청 고 에 가고 도 쳐

에 듣고 보 에 갈 가 별

없어 rdquo

질 계 고 슬슬 도 는 것 닙니[ 2]

답 다 들 눈 에 보 고 뿐 보[ ] ldquo

다 열심 고 어 상에 도 들지만 상

지키는 것 들다고 에 도달

그것 지키 훨 많 rdquo

질 들 살 고 리 는[ 3]

거운 들 체 리느 는다

답 가 고 게 체 어[ ] ldquo ( ) 級

느 도 계가 니 살 는 것도 고역 지만

살 우는 것 들어 는 살

체 리 고 어도 어도 실 갔다

쑥 어 rdquo

질 거리에 슷 연 여 들[ 4]

보는 간 상 지

답 상 다 체 게 리지 못[ ] ldquo

거 주변에 는 그 거 누 보지

못 고 뻐지고 싶 에 체 리는 에

타 워 지만 는 어울 는 것보다 는

시간 운동만 는 건 니에 사복 lsquo rsquo

고 사복 는 말에 들 웃지만 늘 운동복

고 지내니 사러 갈 도 어 rdquo

질 역도가 말 단 식 운동 니[ 5]

답 가 내는 만 클 업 보[ ] ldquo

그러니 만 쓰는 식 운동 니다

만 다고 거운 것 들 는 건 니거든 연

도 고 가지 동 에 도 여러 가지

복 들

보식 역도 여 미-

zb28) 가 에 대한 설 으( ) 않은 것은

① 시각 우 지 시 에 지

고 망 가는 태도 달 고 다

② 언어 과 언어 복 사 여

담 내 생각 게 는 가

③ 직 감 그 마 것

럼 생생 게 느껴지는 과 주고 간 내

없 리 어 억 게 다

④ 담 내 식 리 여 담 삶 습

과 가 시 여 독 에게 감동과 훈 다

⑤ 직 진 담 직 누

지 못 는 독 에게 생생 상 달 주고

담 욱 게 다

zb29) 나 의 각 의 의도를 설 한 것으 적절하( ) 않

년 학 간고사 대비2013 2 현대고 대비

ECN-0102-2013-001-000076193

은 것은

① 질 담 상 보여 주 것 다1

② 질 담 과 그에 삶 태도 보여2

주 것 다

③ 질 역도 겪는 어 움에 역도3

과 것 다

④ 질 같 연 여 갖는 고민 는지 말4

주 는 것 다

⑤ 질 역도가 과 고 운동 는 것5

담 가 말 주 는 것 다

가 만진 것 다( ) 3

감 달 다고 다 억 에( ) 音感

지워 지만 당시 청 탁 리도

다고 다 드럼 웠다 4

에 갈 마다 드럼 는 리가 신 게 들

다고 다 눈 볼 가 없 니 엔ldquo

는 는 님 틱 에 여 주

다 드럼과 연 맺 과 들 주었다rdquo

식 누 가-

고 싶어

역( ) 도가 말 단 식 운동 니

가 내는 만 클 업에 보ldquo

그러니 만 쓰는 식 운동 니다 만

다고 거운 것 들 는 건 니거든 연

도 고 가지 동 에 도 여러 가지 복

들 시 는 상 상

드는 상 에 맞춰 실 에 는 여러

펼쳐집니다rdquo

략( )

늘 에 는 어 만 것 같

가 에 사 고 사 사ldquo

겠어 든 에 가 경 만 고

울 는 사 겠어 rdquo

보식 역도 여 미-

다 가 운 는 어 어( ) ldquo rdquohelliphellip

월 새벽 시 태 없 거웠고1965 5 6 1

는 없 그 병원에 퇴원 집

가는 마지막 마 고 마 내 거 다

가 죽 간신 에 실 다 사는 어느5 lsquo

가 죽 는 말 가 식 다 신rsquo

상에 각 시키는( )刻印 에 실

어느 가는 후 민 가가 근 었다lsquo rsquo

는 간 과 진실 그 다는( ) ldquo

에 단 평 견 가지고 다 내

가 그리는 간상 단 고 다 지 다 는 그들

가 에 는 평 지 니 그리고 어린

들 미지 겨 그린다rdquo

마 근 간 과 진실 그리고 싶어( )

가 다 근에게 그것 진리 다 거 다 없

거 고 다 없 는 것 진리

다 근 진리는 후 쪽 었다 신산(辛酸 삶)

었 질곡(桎梏 역사 에 지냈)

가 눈에 든 것 료 단 료 게

보 것 었다 그것 그 에 겨우겨우

슬 슬 생 어가는 간들 었

다 리 과 단 리 고리에 검

마 없 거리 돌

상 것 없는 등 근에게 상에

과 진실 엄 다는 사실 리는 가( )儼存

실 고 가 과 역경 에 도 근 내

포 없었 후 보루(堡壘 다 도)

도 간 근365

여 시 것 다

월 강원도 림리에( ) 1914 2 21

삼 독 태어났다 어 근 복

그것 그리 가지 못 다 근 곱 살

지는 산 산업에 실 고 답마 에 내

갔다 근 그림 럼 쫓 다니 가 시 것

다 상 진 것도 가 었다

러 가 에도 고 근 가 꿈꾸었다 근

가 꿈꾸게 것 보통 업

원색도1926 만lsquo rsquo 었다

공주 그림 가 근 경-

zb30) 에 대한 설 가장 른 것은~

① 역도가 과 운동 도 질

② 리는 는 다 lsquo rsquo

③ 들었지만 그럭 럭 는 다 lsquo rsquo

④ 가 게 보 시 말 다

⑤ 보 병 는 지 상 lsquo rsquo

는 말 다

년 학 간고사 대비2013 2 현대고 대비

ECN-0102-2013-001-000076193

시간 많지 다 청량리 생 병원

마지막 상 경 릿 게 들어 다 그 는 십

만 큰 가 상 말 다

지 못 들 마 갈 고 돗

도시민들 싹 싹 탔다 가 시

월에 병원에 원 가 폐 진 몸도4 ( )疲弊

갈 미 지 못 고 었다 가는 얼마( ) 解渴

지 생 에 생각 가

마감 는 신 평생 십 만에

가 과 많 닮 다고 생각 지는

가 운 는 어 어ldquo rdquo 1965helliphellip

월 새벽 시 태 없 거웠고 는5 6 1

없 그 병원에 퇴원 집 가

는 마지막 마 고 마 내 거 다 가

죽 간신 에 실 다 사는 어느 가5 lsquo

죽 는 말 가 식 다 신rsquo

상에 각 시키는 에 실 어느( ) lsquo刻印

가는 후 민 가가 근 었다rsquo

ldquo 는 간 과 진실 그 다는 에

단 평 견 가지고 다 내가 그

리는 간상 단 고 다 지 다 는 그들 가

에 는 평 지 니 그리고 어린 들

미지 겨 그린다rdquo

근 간 과 진실 그리고 싶어 가

다 근에게 그것 진리 다 거 다 없 거

고 다 없 는 것 진리다

근 진리는 후 쪽 었다 신산 삶 ( )辛酸

었 질곡 역사 에 지냈 가 눈에( )桎梏

든 것 료 단 료 게 보 것

었다 그것 그 에 겨우겨우 슬

슬 생 어가는 간들 었다 리

과 단 리 고리에 검 마

없 거리 돌 상

것 없는 등 근에게 상에 과 진실

엄 다는 사실 리는 가 실( )儼存

고 가 과 역경 에 도 근 내 포 없었

후 보루 다 도 도( ) 365堡壘

간 근 여 시 것

간에 지닌 가 근 1914 2

월 강원도 림리에 삼 독21

태어났다 어 근 복 그것 그리

가지 못 다 근 곱 살 지는 산

사업에 실 고 답마 에 내 갔다 근

그림 럼 쫓 다니 가 시 것 다 상

진 것도 가 었다 러 가 에도

고 근 가 꿈꾸었다 근 가 꿈꾸게

것 보통 업 원색1926

도 만 었다lsquo rsquo

그림 가 근 경 공주- ldquo rdquo ( 2009)

zb31) 다음 이 같은 의 성 소에 해당하 않은

것은

사건 평① ② ③

④ 주 ⑤ 경

가 운 는 어 어ldquo rdquo 1965helliphellip

월 새벽 시 태 없 거웠고 는5 6 1

없 그 병원에 퇴원 집 가

는 마지막 마 고 마 내 거 다 가

죽 간신 에 실 다 사는 어느 가5 lsquo

죽 는 말 가 식 다 신rsquo

상에 각 시키는 에 실 어느( ) lsquo刻印

가는 후 민 가가 근 었다rsquo

는 간 과 진실 그 다는 에ldquo

단 평 견 가지고 다 내가 그

리는 간상 단 고 다 지 다 는 그들 가

에 는 평 지 니 그리고 어린 들

미지 겨 그린다rdquo

근 간 과 진실 그리고 싶어 가

다 근에게 그것 진리 다 거 다 없 거

고 다 없 는 것 진리다

근 진리는 후 쪽 었다 신산 삶 ( )辛酸

었 질곡 역사 에 지냈 가 눈에( )桎梏

든 것 료 단 료 게 보 것

었다 그것 그 에 겨우겨우 슬

슬 생 어가는 간들 었다 리

과 단 리 고리에 검 마

없 거리 돌 상

것 없는 등 근에게 상에 과 진실

엄 다는 사실 리는 가 실( )儼存

고 가 과 역경 에 도 근 내 포 없었

후 보루 다 도 도( ) 365堡壘

간 근 여 시 것

간에 지닌 가 근 1914 2

월 강원도 림리에 삼 독21

태어났다 어 근 복 그것 그리

가지 못 다 근 곱 살 지는 산

사업에 실 고 답마 에 내 갔다 근

그림 럼 쫓 다니 가 시 것 다 상

진 것도 가 었다 러 가 에도

고 근 가 꿈꾸었다 근 가 꿈꾸게

것 보통 업 원색1926

도 만 었다lsquo rsquo

공주 그림 가 근 경- ldquo rdquo ( 2009)

년 학 간고사 대비2013 2 현대고 대비

ECN-0102-2013-001-000076193

zb32) 위 을 작성하는 과정에서 되어 활 된 자

어 것은

신 사 료① 연보②

고③ ④ 들과 담

⑤ 에 평

는 간 과 진실 그 다는 에ldquo

단 평 견 가지고 다 내가 그

리는 간상 단 고 다 지 다 는 그들 가

에 는 평 지 니 그리고 어린 들

미지 겨 그린다rdquo

근 간 과 진실 그리고 싶어 가

다 근에게 그것 진리 다 거 다 없 거

고 다 없 는 것 진리다

근 진리는 후 쪽 었다 신산 삶 ( )辛酸

었 질곡 역사 에 지냈 가( )桎梏

눈에 든 것 료 단 료 게 보

것 었다 그것 그 에 겨우겨우

슬 슬 생 어가는 간들 었다

리 과 단 리 고리에 검 마

없 거리 돌 상

것 없는 등 근에게 상에 과

진실 엄 다는 사실 리는 가 실( )儼存

고 가 과 역경 에 도 근 내 포

없었 후 보루 다 도 도( ) 365堡壘

간 근 여 시

것 다

간에 지닌 가 근 1914 2

월 강원도 림리에 삼 독21

태어났다 어 근 복 그것 그리

가지 못 다 근 곱 살 지는 산

사업에 실 고 답마 에 내 갔다 근

그림 럼 쫓 다니 가 시 것 다 상

진 것도 가 었다 러 가 에도

고 근 가 꿈꾸었다 근 가 꿈꾸게

것 보통 업 원색1926

도 만 었다lsquo rsquo

질 루 마 가 도 린다 경건

움 느껴지는 경 다 훗 근 그림에

과 는 거 것( )裸木

만 간과 연 엮어 가는 경건 움lsquo rsquo

니었

같 가가 고 싶었 근에게 그 꿈에 다

가가는 지 다 다 가 지망생들 규 미

상 에 진 고

에 지만 근 다 다 근

미 에 운 것 보통 시 미 시간

다 그런 그에게 없는 연습 가가

통 다 가 귀 시 지 도

얻는 뛸 듯 뻤지만 마 도 가 에

듯 는 었 에 어린 근 주 에

에 그림 그리고 지우고 복( )粉板

시간 가는 게 루 보냈다

근 그 갈 가가 것 열여( )渴求

었 다가 미1932 lsquo rsquo ( lsquo

미 에 다 다는 고 마rsquo) lsquo rsquo

가 근 집 고도 지는 시골 경

그린 그림 다 후 근 에 1943 22

지 미 에 그림 고

에 걸쳐 다 미 근 가

동 는 었다

공주 그림 가 근 경- ldquo rdquo ( 2009)

zb33) 위 의 내 과 일치하는 것은

가 근 가 꿈 포 다①

근 당 가들과 께 에 다②

살 근 가 걷20③

게 었다

④ 만 통 근 역경 겨내는lsquo rsquo

느 다

⑤ 근 간 과 진실 그리 에 그 에

드러 는 간상 단 다

계 시 주 근 건강

걸었다 신 과 간에 상 다 건강

신 는 눈에도 다 근 쪽 눈 뿌 게

보 지 과에 다 다 시 지지 고 결

내 었다 시 지만 마 막막

다 늦어 결 근 쪽 눈 고 말 다

쪽 눈 근에게는 쪽 눈 었고

계 었다 그 근 는 여 그lsquo rsquo

다 근 에 같 그림 그 었다1950

시 그림 는 여 쪽lsquo rsquo

고 어 마주 고 는 그림1963

여 과 동 다 마 복

그린 듯 눈 내리 새 게 다 지

사 다 근 게 복 것

복 상과 타 는 근 상

가 떳떳 단 었고 근 그리고

간 과 진실 에 다가가 가 근다

운 었다 근 신에게 당당 지 그리고

그 다 근 그림에 단 복 보다

년 학 간고사 대비2013 2 현대고 대비

ECN-0102-2013-001-000076193

태 도 그리고 극 보다 과

얻 여 었다 과 통

근 그리고 는 재 고 에 질

만들고 특 것 다

공주 그림 가 근 경- ldquo rdquo( 2009)

zb34) 의 이유에 대해 추 한 것으 적절하 않은 것

상과 타 시도①

보다 과 얻②

근 신에게 당당 지③

④ 간 과 진실 에 다가

⑤ 태 도 얻

근 가가 었지만 그 다니 가

럼 어지지 다 복과 쟁 거쳐 시

는 가 근에게 생계 사 에

운 사 다 에 키에 건( ) 178cm死鬪

체 근 에 동 역 업( )荷役

가 생계 다 쟁

에는 동에 운 상우 주 미

죄 사 에 그림 그리는 시 다 그곳에

에 동 역 업 것에

결 것 럼 보 다 지만 그런 것만도

니었다 그림 그리는 고는 지만 매 근

는 극 간 과 별 없는 경 리 그림

벽에 그리는 것 었다 우도 리 없었다 근

트 는 우 그림 그 다 생

계 그림 단 것 다

후 근 지 신 계 리에 미

엑 리 겼다 근 곳에

건 사 크 에 미 들 ( )

상 상 그 다 근 갖 다 겪

냈다 그리고 결 그 돈

신동에 어 사리 집 마 다 마 ㄷ

루 심 쪽에는 과 엌 쪽에는 건

었다 건 주고 근 가 에

여 살 다 심 에는 지 집어

쓰고 지만 곳 근 가 에게 러웠

보 리 다 근 과 마루 업실 삼 그림

그 다 신동 마루는 근 그림에 등 는 lsquo rsquo

같 상들 지 다 시 고

에 들 폐허가

가 업실 었다

공주 그림 가 근 경- ldquo rdquo( 2009)

zb35) 위 에 대한 설 으 적절한 것은

업 시 여 훈과 감동 다①

에 주 평 드러 다②

사 사 등 식 과 ③

④ 다 근거 시 여 삶에

⑤ 살 시 사 경 께 여

습 시 다

가 시간 많지 다 청량리 생 병원( )

마지막 상 경 릿 게 들어 다 그 는

십 만 큰 가 상 말 다

지 못 들 마 갈 고 돗

도시민들 싹 싹 탔다 가 시

월에 병원에 원4 가 폐( )疲弊

진 몸도 갈 미 지 못 고 었다( )解渴 가는

얼마 지 생 에 생각

가 마감 는 신 평생 십 만에

가 과 많 닮 다고 생각 지는

가 운 는 어 어( ) ldquo rdquohelliphellip

월 새벽 시1965 5 6 1 태 없 거웠고

는 없 그 병원에 퇴원 집

가는 마지막 마 고 마 내 거 다

가 죽 간신 에 실 다 사는 어느5 lsquo

가 죽 는 말 가 식 다 신rsquo

상에 각 시키는 에 실( )刻印

어느 가는 후 민 가가 근 었다lsquo rsquo

다 는 간 과 진실 그 다는( ) ldquo

에 단 평 견 가지고 다 내

가 그리는 간상 단 고 다 지 다 는 가

에 는 평 지 니 그리고 어린 들

미지 겨 그린다rdquo

근 간 과 진실 그리고 싶어( )

가 다 근에게 그것 진리 다 거 다 없

년 학 간고사 대비2013 2 현대고 대비

ECN-0102-2013-001-000076193

거 고 다 없 는 것 진리

다 근 진리는 후 쪽 었다 신산( )辛酸 삶

었 질곡 역사 에 지냈( )桎梏

가 눈에 든 것 료 단 료 게 보

것 었다 그것 그 에 겨우겨우

슬 슬 생 어가는 간들 었다

리 과 단 리 고리에 검

마 없 거리 돌

상 것 없는 등 근에게 상에

과 진실 엄 다는 사실 리는 가 실( )儼存

고 가 과 역경 에 도 근 내 포

없었 후 보루 다( ) 堡壘 도 365

도 간 근 여

시 것 다

마 같 가가 고 싶었 근에게 그 꿈( )

에 다가가는 지 다 다 가 지망생들

규 미 상 에 진 고

에 지만 근 다 다 근

미 에 운 것 보통 시 미 시간

다 그런 그에게 없는 연습 가가

통 다 가 귀 시 지 도

얻는 뛸 듯 뻤지만 마 도 (

는 었 에 어린 근 주 에)

에 그림 그리고 지우고( )粉板

복 시간 가는 게 루 보냈다

zb36) 전 의 성 소가 아닌 것을 고르

① 평 ② 사건 ③ 경

④ ⑤ 훈

늘 지 상에 살고 는 사 들 억 도가10

고 그리 지 통 고 는 사 들( )知的

그보다 훨 많 억 도는 고 지 20

통 다 그런 지 고 2500

그리 간 보는 과 사 에

매우 달 뿐만 니 과 에 도 극

루고 었다 미 운 그런 들

살고 는 동 과 사 들 사고 식에

큰 가 다는 다

고 그리 들 우주 개별 고 독립

사 들 생각 지만 고 들 우

주 연 질 간주 다 같( ) 看做

각 도 들에게는 연 질

었지만 그리 들에게는 미 들 결 었

다 고 과 그리 들 사 같

는 동 과 사 에 도 견 다

지심리 미 마 드 겐트 는

살 들에 에 지 다

연 동 과 상 다 과 같 실험

다 크 만든 미드 도 보

여 주고 그 상 닥 고 주었다lsquo (Dax)rsquo

실 닥 는 재 지 는 것 실험 가lsquo rsquo

만들어 낸 다 그런 다 개 다 체 보

여 주었는 는 미드 지만 틱

만들었고 다 는 재료는 크 지만

달 다 그러고 어 것 닥 지 사 들에게 고 lsquo rsquo

게 니 들 주 같 고 는

체 택 고 동 들 같 재료 만들어진 체

택 다 러 는 심지어 살짜리

들에게 도 타났다 것 곧 과 동

다 상 보고 다는 것 미 다

개별 사 보고 고 동 연 질 보

고 는 것 다

동 들 주변 상 에 맞 어 동 고

에 다 사 들 태도 동에 보다 많

주 울 다 동 가 미시간 에

에 경험 다 그는 미식

경 보러 가게 었는 경 체는 매우 재미 었

주변 들 동에 질 다 그 는

들 계 어 상태 경 다

어 들 에 에 그 시 가 계 가

진 것 다 상 살펴 는 말 들 lsquo rsquo

에 그는 에 시 어 도 뒷사

생각 곧 다시 곤 것 다 그런 그에게 뒷

사 고 지 는 들 동 럼

어 웠다

생각 지도 리 드 니 벳-

zb37) 다음 위 의 내 전개 으 만 인lt gt

것은

lt gt

대조의 통해 대상이 닌 특성을 설 하고 있다

일화를 제 하여 자 의 주장을 뒷 침하고 있다

유추의 을 사 하여 독자의 의해를 돕고 있다

대상이 형성되는 과정을 간적 서에 따라 서 하고 있

① ②

③ ④

년 학 간고사 대비2013 2 현대고 대비

ECN-0102-2013-001-000076193

가 우리가 말 고 쓰는 든 단어가 사 에 는( )

것 니다 사 격에 가 는 지만

어 사 과 같 특별 는 사 니lsquo rsquo

단어 격 보 단어가 사 에

등재 어 다 리 리 사 는 단어 도 그

것 시 사 는 어 고 사 에

격 보 것 니다

러 얼 은 사전에 를 있는가 이에 대한 답lsquo rsquo

은 얼 이 유행어인가 아닌가에 따라 갈라 다 이 단어lsquo rsquo

는 년 어 자 에 랐고 쓰이고 있으2002 lsquo rsquo

유행어라고 하 에는 생 이 다 런데 계속

을 유 하 서 사전에 등재될 자격을 획득할 것인가 이

에 대한 답을 내리 는 히 어 다

여 서 가 를 고 해 볼 있다 첫 는 이 단어

를 써야 할 필 가 속적으 있는가 하는 점이다

상주의 열풍에 휩 인 사회 위 에 편 해서 퍼 말

이 얼 인데 과연 런 위 가 속될 것인가 이에lsquo rsquo

대해 필자의 생각은 정적이다 사회 위 가 뀌

런 말을 쓸 일이 없어 것이다

다음은 단어의 성이다 단어의 성이 사회적으 거

감이 없으 계속 사 될 가능성이 높다 런 에서

얼 은 좋은 조건이 아니다 익히 알 졌듯이 이lsquo rsquo

말은 얼 과 청소년층에서 속어 사 하는 이 결합lsquo rsquo lsquo rsquo

된 말이다 얼 에서 얼 을 리하는 조어 도 lsquo rsquo lsquo -rsquo

어에서는 매 낯선 이다 이것만으 도 거 감을 갖

는 사람들이 있다 더 나 속어 결합한 말이다 얼 lsquo rsquo

이 널리 퍼졌다 해도 은 여전히 청소년층의 속어lsquo rsquo

남아 있다 속어는 자연 럽게 아 자리에서나 쓰 에는

담 러 말이다 러한 담을 하고 사

역을 넓혀 가는 속어도 없 는 않다 특히 얼 은 lsquo rsquo

에도 종종 등장한다 만큼 거 감이 많이 희석되었다

고 할 있다 러나 일상의 자연 러 대화에서도 거

리낌 없이 등장하는가 게 는 되 않았다고 생

각한다

얼 이 유사어인 쌈 등을 만들어 내고lsquo rsquo lsquo rsquo

있으니 살아남을 있을 것이라고 는 견해도 있을 것

이다 러나 간이 나 서 유사어를 포함하여 든

말이 사라 사 는 많다 유사어가 많다는 것이 생 을

유 할 있는 절대적인 조건은 아니다

나 언젠가 터 사람들은 어느 단에서 얼 이 가장( )

쁜 사람을 가리켜 얼 이라고 르고 있다 이 얼lsquo rsquo lsquo rsquo

이라는 단어가 최근 어사전에 라 항간에 논란이 일고

있다 아닌 게 아니라 얼 은 유행어처럼 인다 생 lsquo rsquo

도 리 래되 않은 것 같고 언제 사라 도 알

없다 게다가 젊은이들 사이에서 주 쓰일 뿐이다 이런

단어를 사전에 는다는 게 하 이 없어 이 도

한다

러나 속단은 이다 차근차근 따져 볼 일이다

선 얼 이 일 적 유행어인 아닌 주의 게 들여다lsquo rsquo

볼 필 가 있다 유행어란 유행에 따라 빠르게 유포되었

다가 단 간 내에 소 되는 단어나 를 가리킨다

얼 은 인터넷을 통해 속히 퍼 말이다 하 만 일lsquo rsquo

적인 유행어처럼 단 간 내에 사라 않았을 뿐 아니라

현재 도 잦은 빈도 사 되고 있고 앞으 도 상당

간 사 될 것으 측된다 한 언 재단의 뉴 검 lsquo rsquo

색 사이트에 따르 얼 은 년 에 처음 나타난lsquo rsquo 2001

이후 꾸 히 사 되고 있다

이 같은 사 빈도는 얼 이 일 적 유행어 는 현lsquo rsquo

저히 다르다는 것을 여 다 장 간의 생존 만으 도

얼 은 이 한 어의 어휘 에 를 자격을 얻었다lsquo rsquo

고 할 있다 더 이 이라는 비 적 정제된 매체에

높은 빈도 쓰이고 있 않은가 사 빈도 측 에서

필통이나 연필과 같은 단어 대등하거나 더 많이 쓰lsquo rsquo lsquo rsquo

다는 것은 결코 가 게 볼 일이 아니다

이제는 사전이 언어 현 을 빠르게 하는 게 덕인

대가 되었다 세계적으 유 한 의 사전들도 경쟁

적으 어를 고 있다

하 만 얼 은 젊은이들이나 쓰는 속어라고 흠을 잡을lsquo rsquo

도 르겠다 얼 이 주 젊은 층에서 많이 쓰 lsquo rsquo

는 속어임에 틀림없다 러나 어사전에 표 적이고 품

위 있는 말만 어야 한다고 생각한다 것은 커다란

해다 당장 아 어사전이나 펼쳐 라 속어는

설과 같은 비어나 죄자들이 쓰는 은어 어

마니 같은 소 의 사람만이 쓰는 말 도 라 있

않은가 사전은 말 치에 일정 빈도 이상 나타나는 말이

라 말이든 다 할 있다

zb38) 가 나 에 대한 다음의 설( ) ( ) 않은 것은

① 가 는 얼짱 사 에 등재 것에( ) ( ) lsquo rsquo

보 고 다

② 사 등재 가는 단어 격에( )

고 고 는 언 들 언어 사 도에 고 다 ( )

③ 가 얼짱 어지만 신 과 같 매( ) ( ) lsquo rsquo

체에 도 사 는 말 는 고 다

④ 가는 얼짱 어 보고 크게 가지 근( ) lsquo rsquo 3

거 들어 뒷 고 다

⑤ 는 얼짱 어 는 다 특 다는( ) lsquo rsquo

근거 에도 크게 가지 근거 가 들어 주 2

뒷 고 다

가 늘 지 상에 살고 는 사 들 억( ) 10

도가 고 그리 지 통 고 는 사 들

그보다 훨 많 억 도는 고 지 20

통 다 그런 지 고 2500

년 학 간고사 대비2013 2 현대고 대비

ECN-0102-2013-001-000076193

그리 간 보는 과 사 에

매우 달 뿐만 니 과 에 도 극

루고 었다 미 운 그런 들

살고 는 동 과 사 들 사고 식에

큰 가 다는 다

고 그리 들 우주 개별 고 독립

사 들 생각 지만 고 들 우

주 연 질 간주 다 같 각

도 들에게는 연 질 었지

만 그리 들에게는 미 들 결 었다

고 과 그리 들 사 같 는

동 과 사 에 도 견 다

인 리학자인 츠 이마이 디드 겐트너는 두

살이 채 안 된 아이들에서 터 성인에 이르 다양한

연 대의 동양인과 서양인을 대상으 다음과 같은 험

을 했다 저 코르크 만든 피라 드 양의 도형을

여 주고 대상의 이름을 닥 라고 알 주었다lsquo (Dax)rsquo

제 닥 는 존재하 않는 것으 험자가 임의lsquo rsquo

만들어 낸 이름이다 런 다음 두 개의 다른 체를

여 주었는데 하나는 피라 드 양이 만 하얀 플라 틱

으 만들었고 다른 하나는 재 는 코르크 만 양이

달랐다 러고 나서 어떤 것이 닥 인 사람들에게 고 lsquo rsquo

르게 했더니 서양인들은 주 같은 양을 하고 있는

체를 선택했고 동양인들은 같은 재 만들어 체를

선택했다 이러한 차이는 성인은 어 두 살 리

아이들에게서도 나타났다 이것은 곧 서양인과 동양인은

서 다른 세상을 고 있다는 것을 의 한다 략 ( )

는 아주 단 하 서도 인상적인 험을 했다

험에는 동서양의 대학생들이 참여했다 는 험 참가자

들에게 컴퓨터 화 을 통해 속 장 을 담은 애니 이션

을 여 주었다 화 의 앙에는 초점의 역할을 하는 커

다란 고 한 마리가 있었고 주위에는 다른 생

들과 초 자갈 거품 등이 함 제 되었다 화 을

두 씩 후 참가자들은 자 이 것을 회상해 라는

를 았다

결과 서양인 대학생들과 동양인 대학생 두 앙

의 초점 역할을 했던 고 를 동일한 정도 언 했으

나 경 소 위 거품 초 다른 생 들 에 ( )

대해서는 동양인 대학생들이 서양인 대학생들 다 60

이상 더 많이 언 했다 뿐만 아니라 동양인 학생들은 서

양인 학생들에 비해 개 적인 고 다 전체적인 계

를 더 언 하는 경향을 다 략 또한 경의 일 ( )

를 화 킨 림을 제 하 을 때 동양인 대학생들은 대

경의 화를 알아챘 만 서양인 대학생들은 경

의 화를 거의 알아차리 했다 략 ( )

따라서 서양인들만을 대상으 연 한 화lsquo

편성 결 은 잘 된 것일 도 있다 각 과정과 인rsquo

과정의 어떤 이 화 편적이고 어떤 이

화에 따라 달라 는 는 앞으 많은 연 를 통하여 논의

되어야 한다

나 어떤 의 에서 리 두는 이 화적이다 리( )

안에는 다른 사람들과 더 친 한 계를 유 하 는 상호

의존성과 다른 사람들 터 독립적인 존재 살아가 는

독립성이 혼재한다 따라서 이 에서 어떤 특성이 더 강

하게 각되는 상황에 놓이느냐에 따라 서 다른 화적

특 을 일 있다 결 리 두는 어떤 경 에는

동양인처럼 행동하고 어떤 경 에는 서양인처럼 행동하는

것이다

zb39) 가 에 대한 다음의 설( ) 않은 것은

① 는 신 주 뒷 닥 실험과lsquo rsquo lsquo

니 실험 근거 시 다rsquo

② 동 들 상 간 공통 보다는 에 식

는 강 다

③ 들 주변 맥 에는 심 경 어 사건

과 사건 사 계에 상 민감 다

④ 는 동 과 틀린 지 고 는 것lsquo rsquo

니 다 고 다 lsquo rsquo

⑤ 가에 우리 사 들 개 시 가 원( )

집 경 말 고 는 것 개 보다는

에 고 는 것에 다

늘 지 상에 살고 는 사 들 억 도가10

고 그리 지 통 고 는 사 들( )知的

그보다 훨 많 억 도는 고 지 20

통 다 그런 지 고 2500

그리 간 보는 과 사 에

매우 달 뿐만 니 과 에 도 극

루고 었다 미 운 그런 들

살고 는 동 과 사 들 사고 식에

큰 가 다는 다

지심리 미 마 드 겐트 는 동

과 상 다 과 같 실험 다

크 만든 미드 도 보여 주고 그

상 닥 고 주었다 그런 다lsquo (Dax)rsquo

개 다 체 보여 주었는 는 미드

지만 틱 만들었고 다 는 재료는

크 지만 달 다 그러고 어 것 닥 lsquo

지 사 들에게 고 게 니 들 주 같rsquo

고 는 체 택 고 동 들 같

재료 만들어진 체 택 다 러 는

심지어 살짜리 들에게 도 타났다 것

곧 과 동 다 상 보고 다는

것 미 다 개별 사 보고 고 동

년 학 간고사 대비2013 2 현대고 대비

ECN-0102-2013-001-000076193

연 질 보고 는 것 다

동 들 주변 상 에 맞 어 동 고

에 다 사 들 태도 동에 보다

많 주 울 다 동 가 미시간

에 에 경험 다 그는 미

식 경 보러 가게 었는 경 체는 매우 재

미 었 주변 들 동에 질 다 그

는 들 계 어 상태 경

다 어 들 에 에 그 시 가 계

가 진 것 다 뒷사 고 지 는 들

동 럼 어 웠다

그는 경험에 어 얻어 동 들lsquo

각도 상 본다 는 가 우고rsquo

검 여 주 단 도 상 실험 실

시 다 그는 실험 가 들에게 컴퓨 통

담 니 보여 주었다

에는 역 는 커다 고 마리가 었

고 주 에는 다 생 들과 갈 거 등

께 시 었다 본 후 가 들

신 본 것 상 보 는 지시 다

그 결과 생들과 동 생

역 고 동 도 언

경 거 다 생 들에 ( )

는 동 생들 생들보다 60

상 많 언 다 뿐만 니 동 생들

생들에 개별 고 보다 체 계

언 는 경 보 다 경 변 시

킨 그림 시 동 생들 경

변 지만 생들 경 변

거 리지 못 다

지 지 들만 상 연 lsquo

보편 결 못 것 도 다 지각 과 과rsquo

지 과 어 보편 고 어

에 달 지는지는 많 연 통 여

어 다

리 드 니 벳 생각 지도 사- ldquo rdquo( 2004)

zb40) 위 에 대한 설 으 가장 적절한 것은

① 동 과 생 식 강 고 다

② 가지 실험 통 쓴 고 다

③ 닥 실험에 사 본질에 동 사

상에 주 다

④ 니 실험에 동 과 에 지

각 도에 가 다

⑤ 쓴 는 보편 연 에 드러 우월 에

에 근 고 다

가 동 들 주변 상 에 맞 어 동 고( )

에 다 사 들 태도 동에 보다 많

주 울 다 동 가 미시간 에

에 경험 다 그는 미식

경 보러 가게 었는 경 체는 매우 재미 었

주변 들 동에 질 다 그 는

들 계 어 상태 경 다

어 들 에 에 그 시 가 계 가

진 것 다 상 살펴lsquo 는 말 들rsquo

에 그는 에 시 어 도 뒷사

생각 곧 다시 곤 것 다 그런 그에게

뒷사 고 지 는 들 동 럼

어 웠다

그는 경험에 어 얻어( ) 동 들lsquo

각도 상 본다 는 가 우고rsquo

검 여 주 단 도 상 실험

실시 다 실험에는 동 생들 여 다

그는 실험 가 들에게 컴퓨 통

담 니 보여 주었다 에는

역 는 커다 고 마리가 었고 주 에는

다 생 들과 갈 거 등 께 시

었다 본 후 가 들 신 본 것

상 보 는 지시 다

다 그 결과 생들과 동 생( )

역 고 동 도 언

경 거 다 생 들 에 ( )

는 동 생들 생들보다 60

상 많 언 다 뿐만 니 동 생들

생들에 개별 고 보다 체 계

언 는 경 보 다 들어 동

생들 상 체 연못 럼 보 어ldquo 같rdquo

체 맥 언 시 었지만

생들 상 어 같 큰 고 가 쪽 움ldquo

직 어 같 역 고rdquo

언 시 다 경 변 시킨 그

림 시 동 생들 경 변

지만 생들 경 변 거

리지 못 다

년 학 간고사 대비2013 2 현대고 대비

ECN-0102-2013-001-000076193

게 볼 동 들 보다는 큰 그( )

림 보 에 사 과 체 맥 연결시 지각

는 경 고 체에 특 떼어 내

어 독립 보는 것 낯 어 다 에

들 사 에 고 주변 맥 에는 심 경

에 사건과 사건 사 계에 상

민감 편 다

마 지 지( ) 들만 상 연

보편 결 못 것 도 다lsquo rsquo 지각 과

과 지 과 어 보편 고 어

에 달 지는지는 많 연 통 여

어 다

리 드 니 벳 생각 지도 사- ldquo rdquo( 2004)

zb41) 의 하는 가~ 다른 것은

① ② ③

④ ⑤

얼마 그 에 동 사고 식과

사고 식 보여 주는 내 다

들 에 는 탕 고 같 게

어 겨 고 미 에 는 그 크 럼 큰 고

어리 주고 원 는 어 도 는

상 고 생각 다는 것 다 러

는 어떻게 생 것 고 과 그리 거슬

러 가 보 그 단 다

고 연 경 체 경 생 에

다 벼 사는 공동 업과 경험 많 연 역

에 고 들 연 웃과

게 지내 고 탁 연 들

들 지 연 럽게 들 다 민들

웃과 동 게 뿐만 니 는 집 과

게 다

동 시 는 생태 경 에 살 결과

들 다 사 들 사 상 에 주

울 게 었고 는 곧 체 상 과 간 사

계 시 는 낳게 었다 신 가

가 는 체에 는 원 는 동시

에 다 사 들 그 사 포 체 맥 에

다 들 간 사 연

계 체 계에 주 울 는 사고 체계

게 었다

그러 그리 연 경 그 었다 산

지 연결 는 지 건 그리고 역

에 다 런 들 업에 다 사 과

동 므 공동체에

다고 다 고 그리 들

들과는 달리 보 내 감 지 들과

지 크게 느 지 못 다 그

견 다 경우 주 쟁 통 결 는 갖

게 었다

신 사 간 계들 루어진 커다

트워크 에 게 당연 사 역시 연

계들 체 식 게 다 어 상

원 도 그 개체가 체 맥 과

계 에 고 다 게 체 맥 에 주

울 다 보 상 복 과 가변 식 게 고

상에 재 는 많 변 들 사 에 재 는 들도

게 다 들 주 태도 보

는 경우가 많다 쟁 결

통 결 보다는 통 결

는 보 다

그러 고 그리 들 개개 사 사 독

에 주 울 다 사 사 체에

어 그들 사 에 재 는 공통 규 주

고 다 상 원 에도 사

체 내 주 고 다 그들

체 여 탕 체

는 주 태도 시 고 특 사 어

주에 는지 여 그 주에 는 규

견 다 에 는 쟁 식 리

같 리 사고 체계가 달 게 었다

리 드 니 벳 생각 지도 사- ldquo rdquo( 2004)

zb42) 위 에서 사 된 설 과 가장 유사한 것은

① 크톱 컴퓨 는 본체 니 마우 루

어 다

② 곡과 시 리 는 지 과 사 루어 다는 공통

지니고 다

③ 경 고 것과는 달리

경 본 연 태 그 주변 경

④ 벽돌 능 에 사계 내내

습도가 지 다

⑤ 잰느 체 체 지닌 재 체가 없

는 재 눌 다

년 학 간고사 대비2013 2 현대고 대비

ECN-0102-2013-001-000076193

zb43) 는 립 앙 도서 이 정의 일 이다lt gt

도서 장과 이 자의 리 의 정의 연결이

적절하 않은 것은

lt gt

제 조 서 유8 ( )

도서 장은 다른 이 자의 안전을 위협하거나 도서 의①

서를 란하게 할 가 있는 자에 대하여는 도서 출입

을 제한할 있다

도서 장은 이 자가 제 조 각 호의 어느 하나의 행위를 하7②

을 때에는 이 을 하게 하거나 도서 출입을 제한할

있다

제 조자 의 대출9 ( )

도서 자 는 다음 각 호의 경 대출할 있다①

상호대차도서 간에 자 를 류하는 것을 말한다 등 다1 ( )

른 도서 과의 협 을 위하여 필 한 경

공 이 공 행 상 필 하는 경2

에 도서 장이 필 하다고 인정하는 경3

대출이 가능한 도서 자 의 위는 도서 장이 정하는②

에 따른다

제 조 상10 ( )

이 자가 도서 자 설을 더럽히거나 찢거나 뜨①

쓰게 하거나 잃어 린 경 에는 상하여야 한다

도서 장은 제 항에 따른 상 을 정하여 게 하여야1②

한다

제 조이 절차 등11 ( )

이 칙에서 정한 것 에 도서 자 설의 이 절차

이 제한 등에 필 한 사항은 도서 장이 정한다

출처 립 앙 도서- (httpwwwnlgokr)

① 는 도 리 다8

② 도 는 리 다9 1

③ 료 지 는 도 리 다9 2

④ 도 료 변상에 리10 1

⑤ 는 에 도 리 다11

3

도 다 각 같다①

공 공 다만 연1

연 간 다

매월 째 째 월2

도 도 리 그 사3

가 다고 는

도 에 미리 게1 3②

시 여 다

4

도 시간 도 여 게시 다

5

도 료 시 는 는 도①

지에 등 후

등 에 사 도②

7

는 다 각 여 는 니 다

도 료 시 상 리1 lsquo rsquo

도 료 시 훼 는2 middot

지 가 닌 곳에 식 거 담3

우는

도 보 등 보 검색열4 middot

그 에 도 질 지 여 도5

여 게시 사 는

8

도 다 거 도①

질 게 우 가 는 에 여는 도

도 가 각 어느7②

에는 지 게 거 도

9

도 료는 다 각 경우 다①

상 도 간에 료 는 것 말1 (

다 등 다 도 과 여 경우)

공 원 공 상 는 경우2

그 에 도 다고 는 경우3

가능 도 료 는 도②

는 에 다

10

년 학 간고사 대비2013 2 현대고 대비

ECN-0102-2013-001-000076193

가 도 료 시 럽 거 거①

못 쓰게 거 어 린 경우에는 변상 여

도 에 변상 여 게시1②

여 다

zb44) 위 에서 도서 장이 게 해야 할 사항에 해당하는

것을 두 쓰

년 학 간고사 대비2013 2 현대고 대비

ECN-0102-2013-001-000076193

립 도 규

1 ( )

규 립 도 립 어린 청 도(

포 다 료 시 열 시 말) (

다 에 사 규 립 도)

편 진 다

2 ( )

규 립 도 도 다 에( lsquo rsquo )

고 는 도 에 도lsquo rsquo 2 2

료 에 여 다 다만 특 료 귀

료 등 료 에 사 립 도

도 다 다( lsquo rsquo )

3 ( )

도 다 각 같다①

공 공 다만 연1

연 간 다

매월 째 째 월2

도 도 리 그 사3

가 다고 는

도 에 미리 게1 3②

시 여 다

시간4 ( )

도 시간 도 여 게시 다

등 등5 ( )

도 료 시 는 는 도①

지에 등 후

등 에 사 도②

사 료6 ( )

도 료 시 에 사 료는 도

7 ( )

는 다 각 여 는 니 다

도 료 시 상 리1 lsquo rsquo

도 료 시 훼 는2 middot

지 가 닌 곳에 식 거 담3

우는

도 보 등 보 검색열4 middot

그 에 도 질 지 여 도5

여 게시 사 는

질 지8 ( )

도 다 거 도①

질 게 우 가 는 에 여는 도

도 가 각 어느7②

에는 지 게 거 도

료9 ( )

도 료는 다 각 경우 다①

상 도 간에 료 는 것 말1 (

다 등 다 도 과 여 경우)

공 원 공 상 는 경우2

그 에 도 다고 는 경우3

가능 도 료 는 도②

는 에 다

변상10 ( )

가 도 료 시 럽 거 거①

못 쓰게 거 어 린 경우에는 변상 여

도 에 변상 여 게시1②

여 다

등 규 에 것 에 도11 ( )

료 시 등에 사

도 다

립 도- (httpwwwnlgokr)

zb45) 도서 장의 리 있는 조항으 적절하 않

은 것은

① ② ③ ④ ⑤

년 학 간고사 대비2013 2 현대고 대비

ECN-0102-2013-001-000076193

1 ( )

사가 공 는lsquo rsquo

과 여 사 원과 리

사 타 사 규

니다

개 보 보7 ( )

사는 보통신망 등 계 는 에lsquo rsquo lsquo rsquo

원 개 보 보 니다 개lsquo rsquo

보 보 사 에 는 사 개lsquo rsquo

보 취 니다 다만 사는 다 lsquo rsquo

사 계 통 공 는 경우 원 lsquo rsquo

등 개 보 당 사에 습니lsquo rsquo

원 리에8 (lsquo rsquo lsquo rsquo lsquo rsquo

)

원 에 리lsquo rsquo lsquo rsquo lsquo rsquo①

원에게 가 도 여 는lsquo rsquo 3

니다

사는 원 가 개 보 우 가lsquo rsquo lsquo rsquo lsquo rsquo②

거 사 경우 는 미 에 어 거 lsquo

사 사 운 우 가 는 경우 당rsquo lsquo rsquo

습니다lsquo rsquo

원 가 도 거lsquo rsquo lsquo rsquo lsquo rsquo 3③

가 사 고 지 경우에는 시 사에lsquo rsquo

통지 고 사 내에 니다lsquo rsquo

경우에 당 원 사에 그 사실3 lsquo rsquo lsquo rsquo④

통지 지 거 통지 도 사 내에 지 lsquo rsquo

생 경우 사는 지지 습니다lsquo rsquo

사10 (lsquo rsquo )

사는 과 지 미lsquo rsquo①

에 는 지 계 고

공 여 다 여 니다lsquo rsquo

사는 원 게lsquo rsquo lsquo rsquo lsquo rsquo②

도 개 보 신 보 포 보 보 시( )

갖 어 개 보 취 공시 고

니다

사는 과 여 원lsquo rsquo lsquo rsquo③

견 만 당 다고 경우에는

리 여 니다 원 견 만 사 lsquo rsquo

에 는 게시 거 우편 등 통 여

원에게 리 과 결과 달 니다lsquo rsquo

원11 (lsquo rsquo )

원 다 여 는 니다lsquo rsquo ①

신청 는 변경 시 허 내 등1

타 보 도2

사가 게시 보 변경3 lsquo rsquo

사가 보 보 컴퓨 그4 lsquo rsquo (

등 등 신 는 게시)

사 타 등 지 재산 에5 lsquo rsquo 3

사 타 상 거 업6 lsquo rsquo 3

는 폭 시지 상 타 공7 middot middot

에 는 보 에 공개 는 게시 는lsquo rsquo

사 동 없 리 사8 lsquo rsquo

타 거 당9

게시15 (lsquo rsquo )

원 내에 게시 는 게시 게재 는lsquo rsquo lsquo rsquo lsquo rsquo

경우 원 사가 게시 복 lsquo rsquo lsquo rsquo lsquo rsquo middot middot

등 태 언 등에 공 는

것 내에 다 원 본 게시 등 lsquo rsquo lsquo rsquo

크 능 등 여 복 는 등 태

는 것 동 것 니다

- (wwwnavercom)

zb46) 위 은 인터넷 포털사이트의 회 가입을 위한 이

약 의 일 이다 이 약 을 만드는 과정에서 생각한

내 으 적절하 않은 것은

개 보 보 가 지에 별 눠①

겠어

원 가 만들게 에②

시 주어 겠어

원들 게재 게시 다 원 크 다③

는 것 지

④ 원 지 는 뿐만 니 사가 지 는

도 께 달 지

리에 가 생 경우 사가⑤

에 다는 도 듯

1 ( )

사가 공 는lsquo rsquo

과 여 사 원과 리

사 타 사 규

년 학 간고사 대비2013 2 현대고 대비

ECN-0102-2013-001-000076193

니다

개 보 보7 ( )

사는 보통신망 등 계 는 에lsquo rsquo lsquo rsquo

원 개 보 보 니다 개lsquo rsquo

보 보 사 에 는 사 개lsquo rsquo

보 취 니다 다만 사는 다 lsquo rsquo

사 계 통 공 는 경우 원 lsquo rsquo

등 개 보 당 사에 습니lsquo rsquo

원 리에8 (lsquo rsquo lsquo rsquo lsquo rsquo

)

원 에 리lsquo rsquo lsquo rsquo lsquo rsquo①

원에게 가 도 여 는lsquo rsquo 3

니다

사는 원 가 개 보 우 가lsquo rsquo lsquo rsquo lsquo rsquo②

거 사 경우 는 미 에 어 거 lsquo

사 사 운 우 가 는 경우 당rsquo lsquo rsquo

습니다lsquo rsquo

원 가 도 거lsquo rsquo lsquo rsquo lsquo rsquo 3③

가 사 고 지 경우에는 시 사에lsquo rsquo

통지 고 사 내에 니다lsquo rsquo

경우에 당 원 사에 그 사실3 lsquo rsquo lsquo rsquo④

통지 지 거 통지 도 사 내에 지 lsquo rsquo

생 경우 사는 지지 습니다lsquo rsquo

원에 통지9 (lsquo rsquo )

사는 특 다 원에게 통지 경우lsquo rsquo lsquo rsquo

공지 게시 통 상 게시 개별 통지에7

갈 습니다

사10 (lsquo rsquo )

사는 과 지 미lsquo rsquo①

에 는 지 계 고

공 여 다 여 니다lsquo rsquo

사는 원 게lsquo rsquo lsquo rsquo lsquo rsquo②

도 개 보 신 보 포 보 보 시( )

갖 어 개 보 취 공시 고

니다

사는 과 여 원lsquo rsquo lsquo rsquo③

견 만 당 다고 경우에는

리 여 니다 원 견 만 사 lsquo rsquo

에 는 게시 거 우편 등 통 여

원에게 리 과 결과 달 니다lsquo rsquo

원11 (lsquo rsquo )

원 다 여 는 니다lsquo rsquo ①

신청 는 변경 시 허 내 등1

타 보 도2

사가 게시 보 변경3 lsquo rsquo

사가 보 보 컴퓨 그4 lsquo rsquo (

등 등 신 는 게시)

사 타 등 지 재산 에5 lsquo rsquo 3

사 타 상 거 업6 lsquo rsquo 3

는 폭 시지 상 타 공7 middot middot

에 는 보 에 공개 는 게시 는lsquo rsquo

사 동 없 리 사8 lsquo rsquo

타 거 당9

원 계 규 내lsquo rsquo lsquo②

여 공지 주 사 사가 통지 는rsquo lsquo rsquo

사 등 여 타 사 업 에 lsquo rsquo

는 여 는 니다

- (wwwnavercom)

zb47) 위 약 의 조항에서 같은 제점을 하lt gt

고 있는 조항은

lt gt

제휴 회사에 회 의 아이디 개인 정 를 전송할 있도

한 조항은 고객에게 당한 조항이다

1 7 8① ② ③

④ 9 ⑤ 10

립 도 규

1 ( )

규 립 도 립 어린 청 도(

포 다 료 시 열 시 말) (

다 에 사 규 립 도)

편 진 다

2 ( )

규 립 도 도 다 에( lsquo rsquo )

고 는 도 에 도lsquo rsquo 2 2

료 에 여 다 다만 특 료 귀

료 등 료 에 사 립 도

도 다 다( lsquo rsquo )

3 ( )

도 다 각 같다①

공 공 다만 연1

연 간 다

년 학 간고사 대비2013 2 현대고 대비

ECN-0102-2013-001-000076193

매월 째 째 월2

도 도 리 그 사3

가 다고 는

도 에 미리 게1 3②

시 여 다

시간4 ( )

도 시간 도 여 게시 다

등 등5 ( )

도 료 시 는 는 도①

지에 등 후

등 에 사 도②

사 료6 ( )

도 료 시 에 사 료는 도

7 ( )

는 다 각 여 는 니 다

도 료 시 상 리1 lsquo rsquo

도 료 시 훼 는2 middot

지 가 닌 곳에 식 거 담3

우는

도 보 등 보 검색열4 middot

그 에 도 질 지 여 도5

여 게시 사 는

질 지8 ( )

도 다 거 도①

질 게 우 가 는 에 여는 도

도 가 각 어느7②

에는 지 게 거 도

료9 ( )

도 료는 다 각 경우 다①

상 도 간에 료 는 것 말1 (

다 등 다 도 과 여 경우)

공 원 공 상 는 경우2

그 에 도 다고 는 경우3

가능 도 료 는 도②

는 에 다

변상10 ( )

가 도 료 시 럽 거 거①

못 쓰게 거 어 린 경우에는 변상 여

도 에 변상 여 게시1②

여 다

등 규 에 것 에 도11 ( )

료 시 등에 사

도 다

립 도- (httpwwwnlgokr)

zb48) 다음 정 리 의 의 으 볼 때 가장

이 적인 것은

도 시간 도 여 게시 다①

등 에 사 도②

가능 도 료 는 도 는③

에 다

④ 도 에 변상 여 게10 1

시 여 다

⑤ 도 가 각 어느7

에는 지 거 도

zb49) 를 참고하여 이 어의 성격을 설 한lt gt

것으 적절하 않은 것은

① 보 에 는 어 시 상 고 어 시lt gt lsquo rsquo

에 보여주고 다

② 진 어 어원에 견 고 다

에는 타 어 들어가는 것 다 lsquo rsquo

③ 에 들어갈 말 각각 고 어 어 신 어~

들 언어는 질 격 강 통 없었다

④ 시 우리 에 가 었지만 지 계

과 달리 들 통 사 달 어 웠

년 학 간고사 대비2013 2 현대고 대비

ECN-0102-2013-001-000076193

⑤ 크 몽골 만주 공통어가 우리 어 같

계열에 다는 에 사 특 짐

가( )

善化公主主隱 공주님

他密只嫁良置古 몰 결 고

薯童房乙 맛

夜矣卯乙抱遣去如 에 몰 고 가다

( )

始汝 會隱日恚見隱扐 만 에 본

恥隱汝衣淸隱笑 맑 웃

고 시 여 공 크다 만 다[ ] ( ) ( ) ( ) ( )始 汝 會扐

내다 에 보다 견( ) ( )恚 見 다( )隱

럽다 맑다 청 웃( ) ( ) ( ) ( )恥 衣 淸 笑

zb50) 위의 나 를 함 고 음에 답하( ) lt gt

보lt gt

( )素那或云金川 白城郡蛇山人也

운 사산

는 고 다 는( )[ ( ) ] (素那 金川 白城

사산 사 다) ( ) 郡 蛇山

삼 사- lsquo rsquo 47

에 제 된 단어 의 표 리를 조건(1) lt gt ( ) lt gt

에 맞게 서 하

건lt gt

lsquo 었고 었다 태rsquo

에 제 된 단어 동일한 표 리에(2) lt gt ( )

의해 적은 것을 나 에서 찾아 조건 에 맞게 서 하( ) lt gt

건lt gt

에 당 는 각각( ) 개 쓸 것2 단

당 는 가 여러 개 어도 개만 쓸 것 각2

개 과 도 쪽에 개만2 2

드시 지 것( )

과 동 원리 것lsquo 고

과 동 원리 것 다rsquo

태 것

가( )

素那(或云金川) 白城郡蛇山人也

소나 또는 천 이라 한다 는 성 사( ) ( ) ( )素那 金川 白城郡〔 〕

산 사람이다 현대어 풀이( ) ( )蛇山

나( )

紫布岩乎希 회

執音乎手母牛放敎遣 자 손 암쇼 노히 고

吾 不喩慙 伊賜等肹 肹 나 안디 리샤

花 折叱肹 可獻乎理音如 고 것거 도림다

다 향찰은 리말을 리 으 적은 표 이었 만 생( )

은 고 대를 넘 하고 끊어 고 말았다 랜 세

동안 갈고 닦아 체계적이었던 향찰 표 이 사라졌

을 인은 크게 두 가 나누어 생각해 볼 있다

하나는 족 사회의 한 선호도에서 찾을 있다 라 때

향찰은 주 족 계 에서 사 했을 것으 인다 한 을

알 하고서는 한자를 활 하여 리말을 리 으 표

하 란 가능하 때 이다 런데 족들은 간이 흐

를 향찰과 같은 리 표 을 익혀 사 하 다는

아 한 을 대 사 하는 쪽을 선호하게 되었다 더 이

고 초에 인재 등 을 위해 과거제도가 행되 서 한 선

호도가 더 높아졌고 결 향찰은 소 되고 말았다

또 다른 가능성은 한 어의 특성에서 찾을 있다

터 한 과 일 세 나라는 한자 화 에 속해 다

당연한 이야 겠 만 표의 자인 한자는 어를 표 하

에 매 적절하다 어의 음절은 성 ( ) ( )聲母 韻母

이 어 고 여 에 성조가 추가되어 최종 소리가 결정된

다 래서 어는 단음절을 하나의 한자 표 하 된

다 에 초성 성 종성의 세 가 소가 하나의 음절

년 학 간고사 대비2013 2 현대고 대비

ECN-0102-2013-001-000076193

을 이 는 한 어는 음절 조가 잡하고 음절의 가 많아

서 한자 차 만으 한 어의 소리를 만족 럽게 표 할

없었다 를 들어 한 어에서는 어 니 같이 음절 lsquo rsquo

이 어 단어가 얼마든 있으나 어는( ) 複數音節

자 하나 나타내 만이다lsquo [m ]rsquo 母 ǔ

한편 일 어의 표 은 핵 적 단어는 한자 적고 토는

가나라는 일 의 자 적는 이다 적인 의 를 나

타내는 은 표의 자인 한자 적고 적 계를 나

타내는 토는 표음 자 적는 셈이니 자세히 살펴

리의 향찰 표 을 쏙 빼닮았음을 알 있다 한 어 같

은 착어이 서도 일 어에만 향찰과 유사한 표 이 살아

남은 것은 일 어의 특 때 이다 일 어는 하나의 자음과

음의 결합으 음절을 이 고 침이 거의 없는 음절 언어

이다 이러한 음절의 특색에다가 토가 달한 착어라는 점

이 향찰과 유사한 표 이 살아남을 있는 비결이었다

하 만 같은 착어라도 다양한 음소 침이 달한 한

어는 향찰 표 하는 데 근 적으 한계가 있었다

zb51) 다 하여 의 행에 대한 탐 한 결과( ) lt gt 2

않은 것은

보lt gt

善花公主主隱 공주니믄 공주님( )

----------------------------------------

-

他密只嫁良置古 그 지 얼어 고 몰 결(

----------------------------------------

-

薯童房乙 맛 맛( )

夜矣卯乙抱遺去如 몰 고 가다 에 몰 고(

가다)

주동 역 동- (薯童謠『 』

에 2 ( )他密只嫁良置古

얼다 시집가다 결 다 말 lsquo rsquo

① 실질 미 지니고 므 타 타lsquo ( )rsquo lsquo [ ]

② 에 실질 미 타내고 지 는lsquo rsquo lsquo [ ]rsquo lsquo [ ]密只 密 只

계 타내는

③ 얼어는 실질 미 포 고 므 가lsquo rsquo lsquo [ ]rsquo嫁

것lsquo [ ]rsquo 良

④ 고 어간 는 실질 미 지니고 므lsquo rsquo lsquo -rsquo

것lsquo [ ]rsquo 置

⑤ 고 어미 고는 계 타내고 므lsquo rsquo lsquo- rsquo

고 것lsquo [ ]rsquo 古

가( )

엉 훈 민middot middot middot middot middot世 宗 御 製 訓 民 正 音

말 미 듕 귁에 달middot middot middot middot middot middot middot middot中 國 文 字

니 런middot middot middot middot middot middot 어린middot middot middot middot百 姓

니 고 도 내 들middot middot middot middot middot middot middot middot middot 시러middot

펴 몯middot 미middot middot 니 내middot middot middot middot middot middot middot middot 爲

어엿middot 겨 새middot middot middot 믈여듧middot middot middot middot字 니middot middot middot

사 마다 니겨 킈 middot middot middot middot middot middot middot middot middot便 安

고 미니middot middot middot middot

본 는 상( ) (象

원리에 만들어진 본) ( )形 ㄱ ㄴ ㅁ ㅅ ㅇ

에 는 가 원리에( )加劃

그리고( )ㅋ ㄷ ㅌ ㅂ ㅍ ㅈ ㅊ ㆆ ㅎ

쓰는 병 원리에 만들어진( )竝書

마지막 체( ) ( )異體ㄲ ㄸ ㅃ ㅆ ㅉ ㆅ

ᅀ 다 상 원리에 ㅇ ㄹ

지 는 삼재 상 본 본( ) ( ) ( 天地人 三才

탕 므림과 림에 ) (初ㅡ ㅣ

재)( ) ( )( )出字 再出字ㅗ ㅏ ㅜ ㅓ ㅛ ㅑ ㅜ ㅕ

병 그리고 들 에 다시( )ㅘ ㅝ ㅣ

( )ㅣ ㅢ ㅚ ㅐ ㅟ ㅔ ㆉ ㅒ ㆌ ㅖ ㅙ ㅞ

zb52) 가 에 대한 설 으 르 않은 것을( ) 두 고르

① 어쓰 규 지키고 다

② 리 고 다

③ 말 미 미 등 어 사 다lsquo rsquo

④ 개 지 다

년 학 간고사 대비2013 2 현대고 대비

ECN-0102-2013-001-000076193

⑤ 어 원 에 가 도 고 다

엉 훈 민世 宗 御 製 訓 民 正 音

말 미 듕귁에 달 니

런 어린 니 고 도middot

내 들 시러 펴 몯 미 니middot

내 어엿 겨 새 믈여듧

사 마다 니겨middot 킈 고

미니

훈민 언 본- lsquo rsquo 5 (1459 )

zb53) 위의 에 대한 현대어 풀이가 르~ 않은 것

① 우리 말 과 달

② 어리 말 고 는 것 어도

③ 신 생각 마 껏 펼 는 사 많다

④ 게 생각 여

⑤ 사 마다 게

zb54) 훈민정음 언해 에는 한 을 창제한 동 가 드러나

있다 훈민정음 창제의 정 과 내 이 잘 연결된 것

① 주 신 말 미 듕귁에 달

② 민 신 내 어 겨

③ 신 뻔 킈 고 미니

④ 실 신 사 마다 니겨

⑤ 귀 신 계 주 는 훈민 신과 거리가

가 엉 훈 민( ) middot middot middot middot middot世 宗 御 製 訓 民 正 音 

말 미 귁에 中 國 달 文 字

니 런 어린 니 百 姓

고 도 내 들 시러 펴 몯

미 니 내 어엿 爲 겨 새

믈여듧 니 사 마다 니 字

겨 킈 고 미니 便 安

훈민 언 본- lsquo ( )rsquo ( ) 5 (1459 )訓民正音 世祖

( )

[ 1 ]

동 룡 샤 마다 복( ) ( ) ( )海東 六龍 天福

시니 고 동( ) ( )古聖 同符 시니

[ 2 ]

매 니 곶 여

미 므 니 그 내 러

가 니

[ 125 ]

우 미리( )千世 샨( )定 에( )漢水北 累仁

누 개 샤 복 업 시니( ) ( ) 開國 卜年

신( )聖神 니 샤도 경 근민 샤 욱( )敬天勤民

드시리 다

님 쇼 산 가( ) ( )洛水 山行

미드니 가

어 가- lsquo ( )rsquo 27龍飛御天歌

다 우리신 니쓰고 다만 만 쓰( )

거 샹 귀쳔 다보게 러 귀

여 쓴 도 신 보 가 고 신 에

말 어 보게 각 에 사 들

고 본 몬 능통 후에

죠 죠 니

드 도 만 공 에 사

드 미 죠 고 고 여 보 죠

보다 얼마가 거시 어신고 니 첫

가 죠 니 죠

민 들 어 신 샹

귀쳔 도보고 어보 가 만 늘

고 폐 에 만쓴 죠 민

도 러보지못 고 보니 그게 엇지

심 니 리 보 가 어 운건 다

니 쳣 말마 지 니 고 그

쓰 에 가 우 지 지

몰 거 본후에 가 어 지

고 그니 쓴편지 쟝 보

년 학 간고사 대비2013 2 현대고 대비

ECN-0102-2013-001-000076193

쓴것보다 듸 보고 그 마 니 쓴 고

어 못

그런고 에 리 과 가

만 쓴 못 민 말만 듯고

고 편 그 못 보니 그사 단

병신 못 다고 그사 식 사

니 만 고 다 과 그사

만 고 다 과 업 사 보다 식 고

죠 도 고 각 과

견 고 실 직 귀쳔 간에 그

고도 다 것 몰 귀죡 보다

사 우리 신 귀쳔 다 업

시 신 보고 과 지 게 랴

시니 샹 귀쳔 간에 우리 신 걸

간 보 새지각과 새 걸 미리

독립신- lsquo (1896)rsquo

zb55) 친 어 나의 제 장( ) 2 매 함축적

의 가 가장 유사한 것은

① 지 눈 내리고 매 득 니 내 여 가

사- lsquo rsquo

② 도 어 리듯 그 게 어 다

주 사- lsquo rsquo

③ 눈 살 다 죽 어 린 과 체 여

눈 새벽 지 도 살 다

눈- lsquo rsquo

④ 삶 근심과 고단 에 돌 거니는 여 거 는

여 리 내린 살가지 에 눈 리 눈 리

택 그 생 에- lsquo rsquo

⑤ 늘 러 고 러

청룡 룡 어 개 루 우

신경림 계- lsquo rsquo

zb56) 친 를 위 가 나 에 나타난A B ( ) ( )

세 어의 특 에 의거하여 세 어 표 하

그 산 고 공 도 맑지만

A

주변에 쓰 리는 어리 사 많다

B

건lt gt

식 가 에 타 어 특징에( ) ( )

거 과 어쓰 는 고 지 말 것

A

B

zb57) 가 의( ) 달 아ㆍ 다 의 ( ) 나셔에서 알 있는

세 어 개화 어의 특 을 비 하여 조건 에lt gt

맞게 서 하

건lt gt

어에 는lsquo 개

어에 는 다 태rsquo

zb58) 은 가 는 다 에 나 는 절lt 1gt ( ) lt 2gt ( )

일 를 췌한 것이다 의 의 가 lt 1gt (1)~(2)

유사한 말을 에서 찾아 쓰lt 2gt

보lt 1gt

런 (1) 어린 니 고百 姓

도 내 들 시러 펴 몯 미

사 마다 (2) 니겨 便 安

킈 고 미니

보lt 2gt

죠 고 고 여 보 죠

보다 얼마가 거시 어신고 니 첫 가

죠 니 죠 민

들 어 신 샹 귀쳔

도보고 어보 가 만 늘 고

폐 에 만쓴 죠 민 도

러보지못 고 보니 그게 엇지 심

니 리

년 학 간고사 대비2013 2 현대고 대비

ECN-0102-2013-001-000076193

lt 1 gt

동 룡 샤 마다 복 시( ) ( ) ( )海東 六龍 天福

고 동 시니( ) ( )古聖 同符

lt 2 gt

(A) 매 니 곶

여 니

미 므 니 그 내

러 가 니

lt125 gt

우 미리 샨 에( ) ( ) ( ) 千世 定 漢水北 累

누 개 샤 복 업 시 니( ) ( ) 仁開國 卜年 聖

신( ) 神 니 샤도 경 근민 샤( ) 敬天勤民

욱 드 시 리 다

님 쇼 산 가 ( ) ( )洛水 山行

미드니 가

- lt gt龍飛御天歌

zb59) 장과 내 상 유사한 성격의 조는125

① 뫼 고 고 고 고

어 그린 많고 많고 고 고

어 러 는 울고 울고 가느니

도 견- lt gt

② 강 에 드니 몸 다

그믈 고 가니

뒷 뫼 엄 언 니( )藥

-

③ 말 없는 청산 태 없는 다

값 없는 청 없는 월

에 병 없는 몸 별 없 늙 리

-

④ 가마귀 골에 가지 마

낸 가마귀 새

청강에 것 시 몸 러 가( ) 淸江

-

⑤ 진 골에( ) 白雪

가 매 는 어느 곳에 었는고

에 갈 곳 몰( ) 夕陽

색-

zb60) 위 에 나타난 세 어의 특 으 적절하 않은

것은

① 룡 어 주격 사에 당 는 가 사( ) lsquo rsquo六龍

고 다

② 샤 어에도 어 주체 쓰 다

는 것 다

③ 매 어 달리 사 택에 어

가 지 지지 고 다

④ 므 원 상 직 어 지 다

⑤ 드시리 다 주체 과 상 께 사

고 다

수고 하셨습니다hearts hearts

년 학 간고사 대비2013 2 현대고 대비

ECN-0102-2013-001-000076193

보닷컴에 공 는 별 보는 고등

들 여 주 는

들 습니다 슷 동 지

가 복 는 것 도가

니 복 여 습 시고 거 시

니다

정답 해설

1) 정답[ ] ④

해설 다른 것은 두 특정 업이나 단 내에서 사[ ]

하는 일종의 은어 사회 언에 해당한다 러나

는 언이 아니라 단과대학을 여서 단대 사lsquo rsquo lsquo rsquo lsquo④

대학을 여서 사대라고 한 말에 해당하 일rsquo lsquo rsquo

사회에서도 널리 쓰이 사회 언이라 할

없다

2) 정답[ ] ⑤

해설 사회 언은 같은 단 내에서 쓰이는 언어이[ ] lsquo rsquo

동일 단끼리는 단결 과 친 감을 형성하는

능을 하 리적 안감이 일어나 않는다

3) 정답[ ] ③

해설 사람이라는 차 적 표현에 대한 대안적 표현이[ ]lsquo rsquo

인 아내 처 등으 볼 있다lsquo rsquo

4) 정답[ ]⑤

해설 남성은 주 격 체를 사 한다[ ]

5) 정답[ ] ⑤

해설 흑인은 검다라는 뜻을 가 고 있을 뿐 인[ ]lsquo rsquo lsquo rsquo lsquo rsquo

다 열등한 뜻을 내포하 않는다

6) 정답 살 색 첫 작품[ ] - -

해설 살색 혹은 킨색은 한 인의 피 색을 뜻[ ] lsquo rsquo lsquo rsquo

하는 것으 인종 차 을 추 고 출 이주민

의 평등 을 침해할 있어 년 표 이2005

살 색으 이름을 꾸었다 처녀작은 처녀라lsquo rsquo lsquo rsquo lsquo rsquo

는 단어가 가 고 있는 곡된 성 인 을 한 것

으 첫 작품정도 꾸어 사 하는 것이 좋다lsquo rsquo

7) 정답[ ] ⑤

해설 호는 아들에게 해체를 사 하고 있다[ ] ① ②

장 을 성하는 청자는 자 의 아 느리 아lsquo

들 세 이다 호는 아 느리에게 해rsquo ③

체를 사 하고 있다 호가 느리 아 에게 ④

사 한 해 체 아들에게 사 한 해체는 두 비lsquo rsquo lsquo rsquo

격 체에 해당한다 호는 자 의 아랫사람인 ⑤

느리에게 아들과 마찬가 해체를 사 하는 것이

상 이 만 임 을 한 느리에게 고마 과 쁨

존 의 표 를 하 위해 자 의 아 에게 말하듯

해 체를 사 하고 있다

8) 정답[ ] ③

9) 정답[ ] ⑤

10) 정답[ ] ①

해설 청자 할아 가 장의 주체 아 다 높을[ ] ( ) ( )

경 에는 압존 에 의해 장의 주체를 높이 않는lsquo rsquo

다 러 아 서가 아닌 아 는으 계 lsquo rsquo lsquo rsquo lsquo

니다 가 아닌 있 니다 표현하는 것이 르rsquo lsquo rsquo

11) 정답 당이 당을 쫒았다 당이[ ]

당에 다

해설[ ]

12) 정답[ ] ⑤

해설 서 다른 높임표현을 통해 청자에 대해 리[ ] ⑤

적 거리감을 나타내는 인 은 이 아니라 현정이

다 가 에서 현정은 에게 해 체를 사 함으 써 ( )

친근감을 드러낸다 나 에서 연 을 게을리하는 역 ( )

도 들 때 에 화가 난 현정이 선생님에게 항의하

는 장 에서는 하 체를 사 하여 리적 거리lsquo rsquo

가 어졌음을 나타내고 있다

13) 정답[ ] ①

해설 는 는 얼 빛이 날과 어찌 다르 고[ ] lsquo rsquo

라는 뜻으 전과 달리 임이 화자를 않고

있음을 알 있다

14) 정답 달리 후 가 있다 이를 통해 경[ ] lt gt

쾌한 음악성을 형성하고 노 젓는 상황을 체적으

형상화하는 역할을 한다

15) 정답[ ] ①

16) 정답[ ] ⑤

해설 다 의 자연은 를 성찰하게 하는 대상[ ] ( )⑤

이자 정의 대상이다 의 자연은 자 의 상황과 ⑤

처 를 드러내는 경으 서의 역할을 하 이

이 없다

17) 정답[ ] ③

해설 는 빈천 을 해결하고자 했으나 강산[ ] lsquo ( )rsquo 貧賤③

과 풍 을 달라는 에 거절하 다고 함으 써 자

연에 대한 애정을 드러내고 있으 는 않는

임에 대한 망을 개에게 전가 켜서 임에 대한 리

을 드러내고 있다

18) 정답[ ] ③

년 학 간고사 대비2013 2 현대고 대비

ECN-0102-2013-001-000076193

19) 정답[ ] ⑤

해설 고상한 음악가의 이름을 리말 꽝 럽[ ]

게 꿈으 써 언어유희를 통해 음을 유 하고 있

다 이는 고상한 척하는 총 를 비꼼으 써 비판적

태도를 드러내는 것이 대상을 꽝 럽게 표현

하여 총 의 허 과 사치를 풍자하고 있다

20) 정답[ ] ⑤

해설 는 작품 속 경에 대한 설 이 드러나는 것이[ ]

서 자의 주 적인 견해가 접적으 드러나는 것이

아니다

21) 정답[ ] ⑤

22) 정답[ ] ②

23) 정답[ ] ④

24) 정답[ ] ①

해설 적강 티프는 주인공의 비 한 출생이나 능[ ] ①

과 이 있는 것으 조정의 능함을 풍자하는lsquo rsquo

것과는 거리가 다

25) 정답 픔 나[ ] ( )

해설 의 음악은 고통 는 사람들을 위 하고 아픔[ ] lsquo rsquo

을 치유해 주는 능을 한다고 할 있다 의 lt gt

픔 도 소 된 이 과 더 어 살아가는 따뜻한 마음lsquo rsquo

을 상 한다

26) 정답[ ] ⑤

해설 에게 선천적으 주어 각 장애라는 역경[ ]

은 의 이라는 가사 연 을 있다lsquo rsquo

27) 정답[ ] ④

해설 는 장 란 선 에게 은 개인적인 인상을[ ]

소녀 장정 등으 표현한 것이다lsquo rsquo

28) 정답[ ] ②

해설 담자가 피 담자의 언어적 표현이나 비언어[ ]②

적 표현 하 독자는 담의 위 나 피

담자의 감정 상태를 알 있다 이를 통해 독자는

담 상황을 더 생생하게 느낄 있고 피 담자

를 더 잘 이해할 있게 된다

29) 정답[ ]③

해설 일상생활과 역도 선 서의 성과에 된 것에서[ ]

역도를 하 서 겪는 어 과 내적 고민으 화제를

전화하 위한 것이다

30) 정답[ ] ①

해설 릿속에 새겨 넣듯 이 억되도 함 세상[ ] ② ③

살이가 힘들고 고생 러 속 하여 자유를 ④

가 없는 고통의 상태를 비유적으 이르는 말

적의 침입을 막 위해 쌓은 축 켜야 할⑤

대상을 비유적으 이르는 말이다

31) 정답[ ] ④

해설 이 의 종류는 전 으 인 사건 경[ ] lsquo

비평을 성 소 삼는다rsquo

32) 정답[ ] ④

해설 근은 삼대독자 태어났음을 에서 확인할[ ]

있다 형제들과의 담은 이뤄 가 없다

33) 정답[ ] ⑤

해설 근은 가난에도 하고 화가를 꿈꾸었다[ ] (3

단 또한 다른 화가 망생들은 정 육을)

위해 상 학 학 해 유학 에 랐 만

근은 다른 을 찾아야 했다 단 세에(5 ) 18

근은 조선 전람회에 입선하 다 단 의(6 )

만종은 인간과 자연이 엮어 가는 경건한 조화 을lsquo rsquo

나타낸다

34) 정답[ ] ①

해설 근이 속에서도 창작활동을 추 않고[ ]

하는 닭은 은 세상과 타협할 르는

근이 세상의 이해를 하 위한 가장 떳떳한 단

이 때 이다

35) 정답[ ] ⑤

해설 전 은 서 자의 주 적인 평이 리는 것이[ ]

만 위 제 은 인 이 살았던 대 사회적 경

을 통해 객 적인 인 의 을 제 하고 있다

36) 정답[ ] ⑤

해설 전 은 인 사건 경 비평이라는[ ] lsquo rsquo⑤

성 이 어져 있다

37) 정답[ ] ①

해설 이 은 동양인과 서양인의 사고 에 차이가[ ]

있다는 것을 대조를 통해 설 하고 있다 또 쓴이

의 제자가 축 경 를 러 가서 경험한 일화를

통해 동양인이 서양인에 비해 주 상황에 더 많은

주의를 인다는 주장을 뒷 침하고 있다

38) 정답[ ] ④

39) 정답[ ] ②

40) 정답[ ] ②

41) 정답[ ] ④

42) 정답[ ] ③

43) 정답[ ] ④

44) 정답 도서 의 휴 일 도서 의 이 간 도서의[ ]

해설 도서 장은 임의 정한 휴 일과 도서 이[ ]

간 도서의 상 등을 게 할 의 가 있다

년 학 간고사 대비2013 2 현대고 대비

ECN-0102-2013-001-000076193

45) 정답[ ] ①

해설 제 조의 정 휴 일 의 휴 일의 사전 게[ ] 3

는 도서 장의 의 조항에 속한다

46) 정답[ ] ①

해설 개인 정 호 의 를 제 하 했 만 항[ ]

나눠서 제 하 않고 대 나열하고 있다

47) 정답[ ] ②

해설 제 조의 내 을 회사는 다른 회사 협[ ] 7 lsquo

계약을 통해 서비 를 제공하는 경 회 의 아이디

등 개인 정 를 해당 회사에 전송할 있다는 내rsquo

이 있으 의 제점을 제 할 있다②

48) 정답[ ] ④

해설 는 도서 장의 의 에 해당하고 나 는 도[ ] ④

서 장의 리에 해당한다

49) 정답[ ] ③

50) 정답 은 음독으 적었고 은 훈독으 적었[ ] (1)

다 과 동일한 표 리 적은 것은 이고 (2) ce

과 동일한 표 리 적은 것은 이다ab

51) 정답[ ] ③

52) 정답[ ] ①②

53) 정답[ ] ③

54) 정답[ ] ③

55) 정답[ ] ①

56) 정답 른 죠코 어린 노 하니라[ ] A B

57) 정답 세 어에서는 활 형이 칙적으[ ] lsquo rsquoㄹㅇ

나타났 만 개화 어에서는 활 형이 쓰 다 lsquo rsquo ㄹㄴ

58) 정답 호 가 흔[ ] (1) (2)

59) 정답[ ] ④

60) 정답[ ] ③

Page 8: 현대고대비 국어 - chamsoriedu.com 「콘텐츠산업진흥 법」외 에도 저작권 의하여 ... 다른주체에게어떤동작을하도록만드는것을나타내는

년 학 간고사 대비2013 2 현대고 대비

ECN-0102-2013-001-000076193

⑤ 죠 뫼 티 시 가( ) ( ) 粥早飯 朝夕

가( )

거리

공신 후 심 늦도( )劉尋

식 없어 과 께 산에 드리고 신

태몽 꾼 에 만고 웅 상 지닌 들

낳 키운다 그 후 신 들 에 역심 ( )逆心

담 귀 등 심 여 리 귀

보내고 지 죽 는 도망 가다

가 만 죽 고 에 경 가는 들 도움

살 다 그러 사 에 심 귀

보고 담 여 고 강 주가 승상

득 여 고 신 사 삼는다 그 후 강 승상

에게 심에 상 리지만 여움

사 귀 가게 다 강 승상 몸 는

연 과 헤어 리 다

경쇠 리 들리 에 들어가니 색

에 게 단청 누각과 큰 집들( )丹靑

다 주 보니 ( ) (一柱門 黃金

산 사 어 었다 산) lsquo rsquo 大字

들어가 고승 다 그( ) ( ) 山門 高僧

거동 보니 눈 눈 듯 고

변 같 귀는 어 에 늘어 니( ) 白邊

맑고 어 골격과 신 평 니었

팔염주를 에 걸고 육환장 을 고서 흑포( )六環杖

장삼에 떨어 송낙 쓰고 나 유생을 고 말[ ] 松蘿

하 를

소 이 연 하여 유상공 는 행차를 동 에 나ldquo

가 맞이하 하 으니 소 의 함을 서하

rdquo

유생이 크게 놀라 말하 다

천한 인생으 팔자가 하여 어 서 를 잃고ldquo

정처없이 다니다가 연히 이곳에 대사를 만난 것인데

토 대하 소생의 성은 어떻게 알고 있 니 rdquo

노 이 답하여 말하 를

어제 남악 형산 의 화선 이 소 의 절에ldquo ( ) ( )男樂 衡山

어서 소 에게 탁하 를 내일 낮 경에 남경 lsquo 12

동성 안에 사는 유 의 아들 충 이가 것이니 내쫓

말고 잘 대접하라 하셨 니다 마침 소 이 찾아 나rsquo

다가 상공의 차람새를 니 남경 사람이 에 알아

았 니다rdquo

유생이 말을 듣고 한편으 쁘고 한편으 퍼하

서 노 을 따라 들어가니 여러 들이 합장 하

가 했다 노 이 에 들어가 저녁 을 은 후에

을 편히 니 이곳은 선경 이었다 세상의 일을( ) 仙境

두 잊고 일 이 편안하 다 이후 는 노 과 함

서 도 이 탐 하고 경도 확하게 의 게 되었( )兵書

다 이 게 되니 대 천 에 가객 은 없 ( ) ( )大明天地 佳客

고 덕산 속에 리 른 만 있더라 래 ( ) 廣德山

이 천상 사람으 살아 있는 처를 만나 이한

을 니 재주 민함을 누가 당할 있겠는가

낮으 공 하더라

유충 전- -

웬늠 어가 사 싸다냐( ) ldquo rdquo

내가 가 막 런거 니

보통것 닐러 그 어낸ldquo ( )

틀어 주 그 가 루 러 허 에

싶어 키 틀어 주 그 가( )

루 허 우간 곡 틀어주는 루 못 는

웂는 고 닝께 고 지 들

어 사는 고 가 다는 건 에 그 집에

rdquo

그런 단 어들 어 새벽에 떼죽 거

다 고 어 보니 죄다 허 게 집어진

는 것 었다

총 가 내화를 꿴 뛰어나 만 아 소 없는

일이었다

어떻게 된 거야ldquo rdquo

한동안 넋나간 듯이 서 있던 총 가 하고많은 사람

에 하필이 유자를 겨냥하 은 말이었다

쎄유 아마 새에 고뿔이 들었던 개비네유ldquo rdquo

유자는 러 딴청을 하 다

야 고 가 에서 감 가 들어 죽는 고 두ldquo

어rdquo

총 는 가 혐의자 나 되는 것처럼 화풀이를( )嫌疑者

하 드는 것이었다

는 비위가 상해서

야 팔자가 사나서 이런 후 에 살라니|

여러 가 다 객고가 쌓여서 조 두 안 좋았을 테 helliphellip

런디다가 릇쓰 이 가락을 트는 대 디립

다 춰댔으니 과 해서 살끼두 다소 있었을 테 helliphellip

래 들어서 키 는 새끼덜일 이 다다 탈이 많은

이니 ldquohelliphellip

는 트의 독성을 충 히 내 않고 고 를 넣

은 것이 탈이었으 니 하 서도 러 참으 의 을 떨

었다 략 - -

마리가 마리 값 간다는 워ldquo

그냥 내뻔지 거시 허 싼 고 는 맛

겄다 싶 허 게 눌 강 어helliphellip

허 마늘 통 다

년 학 간고사 대비2013 2 현대고 대비

ECN-0102-2013-001-000076193

게 지 고뿌 지 rdquo

어 어째ldquo rdquo ldquo rdquo

런 도 것들 같 니ldquo ( ) rdquo殘忍無道 helliphellip

는 탱 여 지 못 다 보( ) 憤氣撐天

니 는 는 다 동원 여 통 쳤

생각 여 는 눈 다

달리 리헐 감ldquo rdquo

들 고 말 니었다 그가

는 것 그 말고는 없었 에 그 게 뒷동

달 거 다

는 우 럽고 식 짝 없는 랫것들 고

다 공연 신 가고 득 것

없다고 단 는지 결 웬만큼 고루 어

그 것들 쪽 에다 고 어주지ldquo

고 그 그걸 주 어 에 에 helliphellip

눈 없는 독 들 rdquohelliphellip helliphellip

고 말 럼 얼거리 들어가 리는 것 었

- ( ) -兪子小傳

zb19) 위 나 를 읽고 평가한 것으 적절하( ) 않은 것

① 사 리 통 감과 사실 고

격 과 달 고

② 는 가 재 컫는lsquo rsquo lsquo rsquo

미 가진 여 는 것 겠

③ 는 식 말 는 웃 상lsquo rsquo

여 는 미 지니고

④ 는 어가 죽 짐 지만 내색 지 고lsquo rsquo

말 고

⑤ 언어 통 가들 여 우리 통

것들 역 고

가 체 거리( ) [ ]

나라 종 연간에 정언주 의 을 하고( )正言注簿

있던 유 은 늦도 자 이 없어 한탄하다가 남악 형산lsquo rsquo

에 치성을 드리고 이한 태 을 꾼 뒤 아들을 낳아 이름

을 충 이라 고 키 다 이때 조정의 하들 에 역

을 품은 정한담 최일 등이 가달의 침입에 대한( ) 逆心

유 의 유화적 입장을 제 삼아 유 을 함하여 양

내고 유 의 에 을 러 충 자마저 살해하

한다 러나 충 은 천 조 정한담의 마 에서 어

나 많은 고난을 겪다가 은퇴한 재상 강희주를 만나 사위

가 된다 강희주는 유 을 하 고 상소를 으나 정

한담의 공격을 아 양을 가게 되고 강희주의 가족은

난을 피하여 두 흩어 다 충 은 강 소저 이 하고

사의 노 을 만나 를 때를 다린다 이

때 남적과 적이 를 들고 나라에 쳐들어 자 정한

담은 자 출전하여 남적에게 항 하고 남적의 선 장이

되어 천자를 공격한다 정한담에게 여러 패한 천자가

항 하 할 음 충 이 등장하여 남적의 선 정 걸

을 죽이고 천자를 출한다 충 은 단 으 란 을

쳐 고 정한담을 사 잡는다 리고 호 에게 ( )胡王

잡혀간 황후 태후 태자를 출하 유 에서 고생하

던 아 유 과 장인 강희주를 한다 또한 이 하

던 어 니 아내를 찾고 정한담 일파를 리친 뒤 높은

에 라서 화를 누린다

사 들 별 고 없 다니었다( )

마 마 돌 다니 걸 여 고

어 곤 다 에는 동쪽에 고

에는 쪽에 니 가 에 리는 엽

가는 없 니 늘 다니는 었

다 얼 말 죽 사 같고 림새가 말

니었다 가슴 에 고 등

삼태 헌 에 니 달 ( )奇男子

가 도리어 걸 었 담 만 열 도 ( )傅說

고 만났고 만 갈( ) ( ) 慇 武丁

도 탕 만났( ) ( ) (伊尹 成湯 渭

여상 도 주 만났는) ( ) ( ) ( ) 水 呂尙 周 文王

월 같 러가 도 어느 열 살

늘과 집 삼고 사 에 쳐 거리에

어 다가 곳에 니 다 ( ) 楚

지 다가 사 보고 가에 다다( )長沙

니 망 가에는 원 리가 슬 고 가

가 내리는 사 에는 갈매 가 갈 뿐 었다

쪽 돌 보니 가 우거 고

가 사 보 었다 그곳에

가니 는 사( ) 汨羅水

는 다 주 가 쓰고 죽고

곳 었다

마 감 여 에 가 사 살펴보니

에는 삼 고 그 에( ) 屈三閭

는 만고 월 과 지 가는 그 들( )風月

가 어 었다( ) 路程記

동쪽 벽 에 새 운 어 거늘 그

보니 월 에 경 주 는 간신에게ldquo ( )敗

보고 연경 귀 가다가 에 죽 rdquo

거늘 그 보고 에 거꾸러

통곡 말

[A]ldquo우리 연경 간 만 니 에

지 살 상에 엇 겠는가

에 고 에 었 니

상에 살 것 가 도 께 지리 rdquo

년 학 간고사 대비2013 2 현대고 대비

ECN-0102-2013-001-000076193

고 가에 내 가니 울 리가 에 지 사

쳤는지 심 심 것 가

신 심 것 가

다( ) 강 승상에게는 들 없고 다만 만

었다 가 낳 에 가 색

타고 내 에게 말 는 ldquo

니다 미원 과 연 맺고 ( ) ( )紫薇垣 緣分

었는 께 강 집 보내 에

니 게 여겨 주십시 거늘 rdquo

미 가운 낳 니 가 고 거동

단 다 시 짓 쓰 고 는 (音

없었 니 여 가운 지 는 짝) 律

룰 만 사 없었다 가 사 여 사 감

게 고 지 못 고 염 는 만다

다가 당에 거 고 식같 러 내니

고귀 상 루 말 다 어 울 도 다( ) 相

귀 사 없고 웅 걸( )富貴爵祿

만고 었다 승상 매우 뻐 내당 ( )內堂

들어가 에게 사 니 역시 매우 거

워 말 다

ldquo 도 마 사 는 승상께

그 게 말 시니 상 여러 말 지 말고

사 도 시다rdquo

상이 에 나 충 의 손을 잡고 결혼과 하여 ldquo

너에게 히 할 말이 있다 내가 늙은 말년에 딸

하나만을 두었는데 니 너 하늘이 정해 필

임이 하다 이제 년고락 을 너에게 탁 ( )百年苦樂

하겠다 하 대 충 이 릎을 꿇고 앉아 눈 을 흘리rdquo

여쭈었다

소자의 을 해주 고 또 하 에 두고자 하ldquo ( )膝下

니 감사하 이를 데가 없 니다 다만 가 속에 통탄

할 일이 사 쳐 있 니다 소자가 이 없어 양친 ( )兩親

의 생사를 른 채 결혼하여 아내를 얻는 것은 자 으

서 할 도리가 아닙니다 이것이 한 러 뿐입니다 rdquo

승상 그 말 듣고 슬 에 어 고

것 에 맞 어 변 게 리ldquo

는 다 집 시 공 도 여 ( )始祖公

고 가 에 가가 어진 만 개 공신

었 니 도 러워 말 시고 시 rdquo

택 여 니 다운 신 과 신

습 늘에 죄 짓고 간 상에 내 신

혼 를 다 끝내고 으 들어가 사 을 살펴 니 빛

나고 빛난 것이 한 입으 는 다 말하 어 고 하나

는 다 하 어 더라 에 켠 환한 촛 ( )新房

아래 은 에 랑과 가 평생의 연 을 맺었( )緣分

으니 서 사랑하 주고 은 말을 어떻게 다 헤아릴

있으 어떻게 다 하리 을 낸 후에 이튿날

상 를 니 상 거 마음을 이 하

더라

각 생 강 승상 집 쪽( )

늘 보고 없 가 신 신 생각 니

없고 어 없었다 는 어떻게 도리가 없다

여 산 에 들어가 리 고 어 도 닦

고 다 그 산 보고 가다

가 곳에 다다 니 에 큰 산 었다 많 우

리 골짜 가 늘 는 가운 색

에 고 갖가지 가 짝 어 었 ( )花草

다 신 산 생각 고 들어가니 경개 ( )景槪

가 매우 뛰어 고 경 산 다 산 리에 들

리는 것 리 보 는 것 울 청산뿐

었다 가 고 울 어 가

니 들 많 가지들 못 어 동

에 늘어 들거리 는( ) 洞口

우거진 가지에 갖 들 다 었다( ) 春情

계상 에는 공 는 늘( ) 花溪上

에 걸린 폭포가 벽 는 리는 산사( )層巖絶壁

쇠 리 객 에 는 듯 늘( ) ( ) 寒山寺 客船

에 싸여 는 습 산

그린 여 병 러 듯 다 경쇠 리가 들

리 에 들어가니 색 에

게 단청 누각과 큰 집들 다( ) 丹靑

주 보니( ) ( ) lsquo一柱門 黃金大字

산 사 어 었다 산 들rsquo ( )山門

어가 고승 다 그 거 보니( ) 高僧

눈 눈 듯 고 변 같 ( )白邊

귀는 어 에 늘어 니 맑고 어 골격

과 신 평 니었다 염주

에 걸고 짚고 포 삼에 어진( )六環仗

쓰고 생 보고 말

승 연 여 상공 시는 동 에ldquo

가 맞 지 못 니 승 십시 rdquo

생 크게 말 다

생 가 여 어 고ldquo

없 다니다가 우연 곳에 사 만 것

그 시 생 어떻게 고 습니 rdquo

승 답 여 말

어 산 승 에ldquo ( ) ( )南岳 衡山

시어 승에게 탁 내 낮 시경에 경 lsquo 12

동 에 사는 심 들 가 것 니 내쫓

지 말고 습니다 마 승rsquo

다가 상공 림새 보니 경 사 에 보

습니다rdquo

zb20) 위 의 친 에서 서 자의 개입이 드러나~

는 이 아닌 것은

① 달 가 도리어 걸 었( ) 奇男子

② 신 심 것 가

년 학 간고사 대비2013 2 현대고 대비

ECN-0102-2013-001-000076193

③ 다운 신 과 신 습 늘에 죄 짓고

간 상에 내 신 다

④ 사 주고 말 어떻게 다 헤 릴

어떻게 다 리

⑤ 신 산 생각 고 들어가니 경개 가 ( )景槪

매우 뛰어 고 경 산 다

거리 연[ ] ( )弘治

간 에 공신 후 에 언(1488~1505) ( )正言

주 는 벼슬 심 늦도 식( ) ( )主簿 劉尋

없어 과 께 산에 드리고 신 태

몽 꾼 에 만고 웅 상 지닌 들 낳

키운다 그 후 신 들 에 역심( )逆心

담 귀 등 심 여 리 귀 보내

고 지 죽 는 도망 가다가

만 다 에 에 어 니

헤어지게 다

에 에 어 니 헤어지게

다 그 후 사 들에게 우연 돌

생 다가 어느 열 살 었다 열 살

지 다가 우연 귀 견 는

그것 그 살 도 었고 그

귀 본 신도 지 죽고 마 고

크게 운다

( )

에는 강 주 는 재상 살고 었

니 시 에 과거에 격 여 승상 벼슬 다가 간

신 만 벼슬 그만 고 고 돌 었

다 그러 신 지 가 지 못 여 상

가 못 결 는 상 여 원 니

신 들 그 직간 꺼 다 그 에 도

담과 귀가 강 승상 가 미워 다

강 승상 마 본 에 갔다가 돌 는[A][ ( )本府

에 우편 주 에 다가 색( ) ( ) 右便 酒店

에 어리었는 청룡 에 지 늘

여 통곡 고 사 는 꿈 꾸었다] 마

상 게 생각 여 새 다리다가 새벽

닭 울고 가 달 갔다 가 보니

과연 어 동 가 가에 울고 는지 달

들어 그 고 사 에 어 말

는 어 어 에 어 가ldquo

닭 곳에 우느냐 니 울rdquo

그 고 답 여 말 다

는 경 동 에 사는 언 주 공 들ldquo

니다 께 간신 만 연경 귀 가

시다가 에 죽 사 에 는 닭에

도 에 죽고 니다rdquo

강 승상 말 듣고 크게 낯 변 말

것 웬 말 냐 근 동 ldquo (老

못 갔 니 그 사 변 여)患

런 변 었단 말 가 주 는 신 다

같 에 벼슬 다가 는 가 많 들어 고

돌 는 주 가 게 꿈 에 생

각 겠느냐 생각지 못 다 미 지 간

지지 말고 께 가 략rdquo ( ) hellip hellip

죽게 주 사당에 단 도 러운

겠느냐 말 말고 시는지 rdquo

어 없어 강 승상 가니 그곳

월계 었다

다( )

가가 고 지 사 들 가( )櫛比

통 는 리가 과

답게 꾸민 누각과 큰 집들 늘 고

게 식 가 어 들 태운 가고

었다 략 강 승상에게는 들 없고 ( ) hellip hellip

다만 만 었다 가 낳 에

가 색 타고 내 에게 말

는 니다 미원 과ldquo ( )紫薇垣

연 맺고 었는 께 강 집( )緣分

보내 에 니 게 여겨 주십시

rdquo

거늘 미 가운 낳 니 가

고 거동 단 다 시 짓 쓰 고

는 없었 니( )音律 여 가운

지 는 짝 룰 만 사 없었다 가 사

여 사 감 게 고 지 못 고 염 는 만다

다가 당에 거 고 식같

러 내니 고귀 상 루 말 다 ( )相

어 울 도 다 귀 사 없 ( )富貴爵祿

고 웅 걸 만고 었다 승상 매우 뻐

내당 들어가 에게 사 니( ) 內堂

역시 매우 거워 말 다 도 마 ldquo

사 는 승상께 그 게 말 시니

상 여러 말 지 말고 사 도 시다rdquo

( )

승상 에 고 결 과 ldquo

여 에게 말 다 내가 늙 말 에 지

만 었는 지 보니 늘

다 에게 탁 겠 ( )

다 신 꿇고 눈 리rdquo

여 었다 주시고 슬 에 ldquo ( )膝下

고 시니 감사 룰 가 없습니다 다만 가슴

에 통탄 사 쳐 습니다 복 없어

생사 결 여 내 얻는 것( )兩親

식 도리가 닙니다 것 러울 뿐 니

다rdquo

상 그 말 듣고 슬 에 어 고 말

것 에 맞 어 웅변 ldquo

년 학 간고사 대비2013 2 현대고 대비

ECN-0102-2013-001-000076193

게 리 는 다 집 시 공도 여

고 가 에 가가 어진 만 개 공신

었 니 도 러워 마 시고 rdquo 시

택 여 니 운 신 과 신

습 늘에 죄 짓고 간 상에 내 신

다 략 지낸 후에 튿 승상 ( ) hellip hellip

니 승상 거운 마 지 못

마( )

듯 월 러 생 열다 살 었

다 에 승상 어진 사 얻고 만 에 근심 없었

다만 주 가 간신 에

죽 것 생각 마 곧 어 곤

다 그 에 주 원통 어

없 고 여 시 가 거늘 생 만

여 다

말 감격 러우 간신 에 가득 여ldquo

고 니 께 상 듣지 니 것

니다rdquo

승상 듣지 고 가

퇴 재상 공달 집에 거 고 상 지어

승지 러 께 리

( )

뒷 거리 강 승상 에게 상 리지[ ]

만 여움 사 귀 가게 다 강 승상

몸 는 연 가 헤어

리 다 산 들어간 룡사 승 만

게 다 승 만 우 다릴

과 들고 략 다 담

원 여 에게 복 고 어

공격 다 담에게 여러 가( ) 天子

복 등 여 다 단

신 리쳐 담 사 고 에게

간 후 태후 태 여 지에 고생

지 심과 강 주 여 개 다 헤

어 어 니 내 고 담 리

벼슬에 귀 누리게 다

zb21) 위 의 인 간 계를 같이 나타냈을lt gt

때 에 대한 이해 가장 적절하 ~ 않은 것은

① 계에 주 는 계 심 열

상 에 다고 다( ) 水深火熱

② 계는 견원지간 고 다( ) 犬猿之間

③ 계는 달리 막역지 계 고( )莫逆之交

④ 연결 사 컬어 재 가 고( )才子佳人

⑤ 는 생 과 볼 ( )匹夫匹婦

가 재 는 는 심 고 매사에 생( )

각 고 능 도 어 가 에게 많lsquo rsquo

도움 사 다 그는 에게 거 에

꺼리 없 거 났다고 는

매우 싫어 고 신 들

는 사 다

내가 지 리에( ) 1970

사 실에 지 월간ldquo

편집 고 어 었다rdquo

어느 없 가 쑥 다 도 어 10

후 다 산 시 럼 어 엇 어 ( ) lsquo怡山

다시 만 랴 니 그는 재 그룹 승 운rsquo

사가 고 는 고 거 누

주는 가 없는 가가 어 다시 만 게 것

었다

다 보통 것 닐러 그( ) ldquo 어낸 ( )

틀어주 가 루 러 허 에

싶어 키 틀어주 그( )

가 루 허 우간 곡 틀어 주는 루 못

는 는 고 닝께 고 지

들어 사는 고 가 다는 건 에 그 집에

rdquo

그런 단 어들 어 새벽에 떼죽 거

다 고 어 보니 죄다 허 게 집어진

는 것 었다 가 실내 꿴 뛰어 지만

없는 었다

어떻게 된 거야 한동안 넋나간 듯이 서 있던 총ldquo rdquo

가 하고많은 사람 에 하필이 유자를 겨냥하 은

말이었다 쎄유 아마 새에 고뿔이 들었던 개비네 ldquo

유rdquo

유자는 러 딴청을 하 다 야 고 가 에서 ldquo

감 가 들어 죽는 고 두 어rdquo 총 는 가 혐의

자 나 되는 것처럼 화풀이를 하 드는 것이었다( )嫌疑者

라 이 어쩌 어 유( ) ldquo rdquo ldquo rdquo

애유 이런 잔인 도 한 것들 같으니ldquo ( ) rdquo殘忍無道 helliphellip

총 는 탱천 하여 쩌 를 하 다( ) 憤氣撐天

아하니 아는 자는 다 동 하여 호통을 쳤으 하나 혈

압을 생각하여 참는 눈치 다 달리 처리헐 두 ldquo

잖은감유rdquo

총 의 성 을 덧들이 고 한 말이 아니었다 가 할

년 학 간고사 대비2013 2 현대고 대비

ECN-0102-2013-001-000076193

있는 것이 말고는 없었 때 에 게 뒷동

산을 달은 거 다

이 유자소전- lsquo rsquo

zb22) 의 상황을 속담으 표현한 것으 적절한 것은

① 루 곳 게 마 다

② 에 맞고 강에 눈 다

③ 늘 도 다

④ 도 사 다

⑤ 에 가도 신만 리 다

거리 공신 후[ ]

에 주 는 벼슬 심 늦도( )主簿

식 없어 과 께 산에 드리고 신

태몽 꾼 에 만고 웅 상 지닌 들

낳 키운다 그 후 신 들 에 역심

담 귀 등 심 여 리 귀 보내고

지 죽 는 도망 간다 그

만 고 에 에 어 니

헤어지게 다 지 가 사 들에

사 들 별 고 없 다니었다

마 마 돌 다니 걸 여 고

어 곤 다 에는 동쪽에 고 에

는 쪽에 니 가 에 리는 엽 가는

없 니 늘 다니는 었다

얼 말 죽 사 같고 림새가 말 니었

다 가슴 에 고 등 삼태

헌 에 니 달 가 도리 ( )奇男子

어 걸 었 담 만 열 도 ( ) ( )傅說 慇

고 만났고 만 갈( ) ( )武丁 伊尹

도 탕 만났( ) ( )成湯 渭水

여상 도 주 만났는 월( ) ( ) ( ) 呂尙 周 文王

같 러가 도 어느 열 살

늘과 집 삼고 사 에 쳐 거리에

어 다가 곳에 니 다 ( ) 楚

지 다가 사 보고 가에 다다( )長沙

니 망 가에는 원 리가 슬 고 가

가 내리는 사 에는 갈매 가 갈 뿐 었다

쪽 돌 보니 가 우거 고

가 사 보 었다 그곳에

가니 는 사( ) 汨羅水

는 다 주 가 쓰고 죽고

곳 었다

마 감 여 에 가 사 살펴보니

에는 삼 고 그 에( ) 屈三閭

는 만고 월 과 지 가는 그 들( )風月

가 어 었다( ) 路程記

동쪽 벽 에 새 운 어 거늘 그

보니

월 에 경 주 는 간신에게ldquo ( )敗

보고 연경 귀 가다가 에 죽 rdquo

거늘 그 보고 에 거꾸러

통곡 말

우리 연경 간 만 니ldquo ( )燕京

에 지 살 상에 엇 겠는

가 에 고 에 었 니

상에 살 것 가 도 께 지리 rdquo

고 가에 내 가니 울 리가 에 지

사 쳤는지 심 심 것 가

에는 강 주 는 재상 살고 었

니 시 에 과거에 격 여 승상 벼슬 다가 간

신 만 벼슬 그만 고 고 돌 었

다 그러 신 지 가 지 못 여 상

가 못 결 는 상 여 원 니

신 들 그 직간 꺼 다 그 에 도

담과 귀가 강 승상 가 미워 다 강 승상 마

본 에 갔다가 돌 는 에 우편 주( ) ( )本府 右便

에 다가 색 에 어리었는 청룡( ) 酒店

에 지 늘 여 통곡 고

사 는 꿈 꾸었다 마 상 게 생

각 여 새 다리다가 새벽닭 울고

가 달 갔다 가 보니 과연 어 동 가

가에 울고 는지 달 들어 그

고 사 에 어 말

는 어 어 에 어 가ldquo

닭 곳에 우느냐rdquo

니 울 그 고 답 여 말 다

는 경 동 에 사는 언 주 공 들ldquo

니다 께 간신 만 연경 귀 가

시다가 에 죽 사 에 는 닭에

도 에 죽고 니다rdquo

강 승상 말 듣고 크게 낯 변 말

것 웬 말 냐 근 동ldquo ( )老患

못 갔 니 그 사 변 여 런 변

었단 말 가 주 는 신 다 같

에 벼슬 다가 는 가 많 들어 고 돌

는 주 가 게 꿈 에 생각

겠느냐 생각지 못 다 미 지 간 지지

말고 께 가 rdquo

뒷 거리 강 승상 도움 죽 고[ ]

년 학 간고사 대비2013 2 현대고 대비

ECN-0102-2013-001-000076193

고 그 과 결 여 사 가 다 그러 강

승상 에게 울린 상 강 승상 귀 가고

과 헤어 리 승 만 게 다

승 우 다릴 과

들고 략 다 담 원

여 에게 복 고 어 (天

공격 다 담에게 여러 가 복) 子

등 여 다 단신

리쳐 담 사 고 에게 간

후 태후 태 여 지에 고생 지

심과 강 주 여 개 다 헤어

어 니 내 고 담 리 벼

슬에 귀 누리게 다

미상- lsquo ( )-劉忠烈傳

zb23) 위 과 의 서사 조를 비 한 것으 적절하lt gt

않은 것은

보lt gt

믿지 고 결 여 곱

낳 다 곱째 공주 낳 가

리게 다 리 만 고 진 공주는 lsquo rsquo

리공 미 리공 에 키워진다 월

러 과 가 죽 병에 걸 는 승에 는

어 산다고 다 여 들에게 탁

지만 거 리 는다 리 는 과

승 다 승 지 가는 에 많

만 지만 보살 도움 사 도 다

그러 승 신과 결 여 시

들어 주겠다고 다 리 는 그 결

여 들 곱 낳 후에 신

얻게 다 돌 리 는

에 과 상여 만 지만 여 과

살 낸다 훗 리 그 공 우 죽 사

승 도 는 신 다

리-lsquo rsquo-

① 복 결말에 고 다

② 웅 에 탕 고 다

③ 시 겨 내고 귀 누리는lsquo rsquo

보 리 는 월 재 신 다lt gt lsquo rsquo

④ 과 보 리 는lsquo rsquo lt gt lsquo rsquo

도움과 어 신 능 극복 고

⑤ 등 여 시 겪는lsquo rsquo

보 리 는 닌 지lt gt lsquo rsquo

림 시 겪는다

가 각 고 에( ) ( )却說

살 없었다 략 사 들 슬 에 어 lt gt

가에 내 고 가고 싶 가 고 후

워 경

사 들 별 고 없 다니었다 lt

략 얼 말 죽 사 같고 림새가 말gt

니었다 가슴 에 고 등

삼태 헌 에 니 달 가 ( )奇男子

도리어 걸 었 담 만 열 도( )傅說

고 만났고 만 갈( ) ( ) 殷 武丁

도 탕 만났( ) ( ) (伊尹 成湯 渭

여상 도 주 만났는) ( ) ( ) ( )水 呂尙 周 文王

월 같 러가 도 어느 열 살

늘과 집 삼고 사 에 쳐 거리에

어 다가 곳에 니 다 ( ) 楚

지 다가 사 보고 가에 다다( )長沙

니 망 가에는 원 리가 슬 고 가

가 내리는 사 에는 갈매 가 갈 뿐 었다

쪽 돌 보니 가 우거 고

가 사 보 었다 그곳에

가니 는 사( ) 汨羅水

는 다 주 가 쓰고 죽고

곳 었다

에는 강 주 는 재상 살고( )

었 니 시 에 과거에 격 여 승상 벼슬 다

가 간신 만 벼슬 그만 고 고 돌

었다 략 강 승상 마 본 에 갔다가 돌 lt gt ( )本府

는 에 우편 주 에 다가 색( ) ( ) 右便 酒店

에 어리었는 청룡 에 지

늘 여 통곡 고 사 는 꿈 꾸

었다 마 상 게 생각 여 새 다리다

가 새벽닭 울고 달 갔다 가

보니 과연 어 동 가 가에 울고 는지

달 들어 그 고 사 에

어 말

는 어 어 에 어 가ldquo

닭 곳에 우느냐rdquo

니 울 그 고 답 여 말 다 lt

략gt

년 학 간고사 대비2013 2 현대고 대비

ECN-0102-2013-001-000076193

생각 여 가 고 시 는ldquo ( )大人

상에 다시없는 니다 살 엇 겠습니

에 돌 가시고

가에 돌 가 니 살 마 없습니

다 략 어 없어 강 승상 가니rdquo lt gt

그곳 월계 었다

다 강 승상에게는 들 없고 다만 만( )

었다 가 낳 에 가 색

타고 내 에게 말

는 니다 미원 과ldquo ( )紫微垣

연 맺고 었는 께 강 집( )緣分

보내 에 니 게 여겨 주십시

rdquo

거늘 미 가운 낳 니 가

고 거동 단 다 시 짓 쓰 고

는 없었 니 여 가운( ) 音律

지 는 짝 룰 만 사 없었다 가 사

여 사 감 게 고 지 못 고 염 는 만다

다가 당에 거 고 식같 러

내니 고귀 상 루 말 다 어 ( )相

울 도 다 귀 사 없고 ( )富貴爵祿

웅 걸 만고 었다 승상 매우 뻐 내

당 들어가 에게 사 니 역( ) 內堂

시 매우 거워 말 다

도 마 사 는 승상께ldquo

그 게 말 시니 상 여러 말 지 말고 사

도 시다 략 시 택 여rdquo lt gt

니 다운 신 과 신 습 늘에 죄

짓고 간 상에 내 신 다

다 내고 들어가 사 살펴보니

고 것 는 다 말 어 고

는 다 어 신 에 ( )新房

에 신 과 신 가 평생 연 맺었( )緣分

니 사 주고 말 어떻게 다 헤 릴

어떻게 다 리 지낸 후에 튿 승

상 니 승상 거운 마 지 못

( ) 듯 월 러 생 열다 살

었다 에 승상 어진 사 얻고 만 에 근심

없었 다만 주 가 간신

에 죽 것 생각 마 곧 어

곤 다 그 에 주 원통

어 없 고 여 시 가 거늘 략 lt gt

략 거리

강 승상 에게 상 리지만 여움

사 귀 가게 다 강 승상 몸 는

연 과 헤어 리 다

마 각 생 강 승상 집 쪽( )

늘 보고 없 가 신 신 생각 니

없고 어 없었다 는 어떻게 도리가 없다

여 산 에 들어가 리 고 어 도 닦

고 다 그 산 보고 가

다가 곳에 다다 니 에 큰 산 었다 많

우리 골짜 가 늘 는 가운 색

에 고 갖가지 가 짝 어 ( )花草

었다 략 주 보니 lt gt ( ) (一柱門 黃

산 룡사 어 었다) lsquo rsquo 金大字

산 들어가 고승 다 그( ) ( ) 山門 高僧

거동 보니 눈 눈 듯 고

변 같 귀는 어 에 늘어 니( ) 白邊

맑고 어 골격과 신 평 니었

다 염주 에 걸고 짚고 포 ( )六環杖

삼에 어진 쓰고 생 보고 말

승 연 여 상공 시는 동 에ldquo

가 맞 지 못 니 승 십시 rdquo

생 크게 말 다

생 가 여 어 고ldquo

없 다니다가 우연 곳에 사 만 것

그 시 생 어떻게 고 습니

rdquo

승 답 여 말

어 산 승 에ldquo ( ) ( )南岳 衡山

시어 승에게 탁 내 낮 시경에 경 lsquo 12

동 에 사는 심 들 가 것 니 내쫓

지 말고 습니다 마 승rsquo

다가 상공 림새 보니 경 사 에 보

습니다rdquo

생 그 말 듣고 편 고 편( )

슬 승 들어가니 여러 승 들

가워 다 승 에 들어가

후에 그 편 니 곳 경 었다 상( ) 仙境

고 신 편 다 후 는 승과

께 병 도 탐 고 경도 게( )兵書

게 었다 게 니 지 에 가객 ( ) ( )大明天地 佳客

년 학 간고사 대비2013 2 현대고 대비

ECN-0102-2013-001-000076193

없고 산 에 리 만 본 ( ) 廣德山

신 상 사 살 는 만

우고 늘 월 신 과 늘 ( )日月聖神

산 신 들 다 니 그 재( ) 名山神靈

주 민 누가 당 겠는가 낮 공

zb24) 다 에 해당하는 내 으 적절하( ) 않은 것은

① 강 티 통 당시 능 다

② 상계 지상계 경 는 원 계 드러

③ 실에 어 없는 실 가 타 는

④ 뛰어 재주 어 가진 고

등 다

⑤ 가 직 개 여 평가 내리는

편집 평 타 다lsquo rsquo

가 본격 가 동 것 지( )

다 단 상 에2003 lsquo rsquo

들어가 드럼 연주 다 취미 생 달리

들었다는 보 우 가 들ldquo

어 틱 린 도 다 고 말 다rdquo

경 는 가 망 없( ) lsquo

티 원 고 답 다 신과 같 시각rsquo

는 습 상상 만 도 감동

다 시각 연주 동시에

열 상 는

티 원 그런 열 경 럽다는 것 다

다 역시 엄청 다 본( )

에 복 들

고쳐 가고 다 신 에 얼

마 지는 고 리가 는 지도 생님

가 훈 고 많 고쳐 다

고 말 다

그러 직도 에 지 는 다 그는

체격 지 못 게 가 큰 만

체 운동 훈 과 께 체 늘 동 50

는 게 고 말 다

에게는 꿈 다 통 누 가( )

주겠다는 것 그 꿈 다 신 극복 는

과 에 큰 경험 들도 느 게

주고 싶다는 것 다

마 슬 마다( ) ldquo 통

낼 었 것 럼 고통 는 사 들

고 겠다 고rdquo

말 다 달 루 첫 낸 lsquo rsquo

첫 드 심 집에 는 리듬 드 2

루 에 도 보고 싶다 집 에는 직(RampB) 3 4

사 곡 도 보 고 싶다고 포 다middot

zb25) 에서 가장 유사한 의 를 닌 어를lt gt

찾아 쓰

lt gt

나는 이제 너에게도 픔을 주겠다

사랑 다 소 한 픔을 주겠다

겨 거리에서 개 놓고

살아 추위 떨고 있는 할 니에게

값을 으 서 뻐하던 너를 위하여

나는 픔의 평등한 얼 을 여 주겠다

내가 어둠 속에서 너를 를 때

단 한 도 평등하게 어주 않은

가마니에 덮인 동사자가

다 얼어 죽을 때

가마니 한 장조차 덮어주 않은

한 너의 사랑을 위해

흘릴 르는 너의 눈 을 위해

나는 너에게 이제 너에게도 다림을 주겠다

지 울 포동 여고 생들17

틈 없 가득 체 에 맑 울

다 죽 듣 생들 사 에

연 는 탄 다 객들 도 는lsquo rsquo

가 보 주 공 맹 가 운 는

단 그룹사운드 루 보컬 맡고 는lsquo rsquo

시각 지 었다17 1

근 다만과 가 거lsquo rsquo lsquo

꿈 고 퇴 내가 다rsquo

간 간에 지 지 연 생들 짧lsquo rsquo lsquo rsquo

가 운 듯 리에 어

연 다 내 사 고 퇴lsquo rsquo

과 루 들 결 다시 돌lsquo rsquo

들 고 사 들 에 당당

것 니다 내 태어

볼 없었 크고 열여

년 학 간고사 대비2013 2 현대고 대비

ECN-0102-2013-001-000076193

에도 고 시 얻지 못 다

감지 없는 시각 상태 다

신 지에 고 상 원망 도

단다 어느 가 에 시각 에 ldquo

어 그런 듣고 다 보니 내가 게 lsquo

살 는지 도 눈 고 싶rsquo lsquohelliphellip

보 는 생각만 들 고 그 가 들에게rsquo

도 내고 들도 고 많 었죠 들 rdquo

었 지 새 는 에 쑥 러운 색

어났다

생에 것 단연 었다lsquo rsquo

공연에 거 꿈lsquo rsquo

는 다 특 가사 갑게 는 운 lsquo

벽 에 당당 마주 어 언 가 그 벽

고 늘 어 거운 상도

없죠 내 삶 에 웃 그 께

는 다고 다rsquo

들었 그냥 런 도 고만 여ldquo lsquo rsquo

겼죠 그런 꾸 가사 미 새 다 보

니 통 는 가사 는 생각 들 고 (

가 게는 시각 는 생각 들고 들) ( )

마다 듣고 큰 얻었어 rdquo

에 진지 게 가에 미 가

zb26) 의 에 들어갈 말 적절한 것은lt gt ~

lt gt

난 난 꿈이 있었죠

고 찢겨 남 하여도

내 가 히 과 같이 간 했던 꿈

혹 때 누 가가 뜻 를 비 음

내 등 뒤에 흘릴 때도

난 참아야 했죠 참을 있었죠

날을 위해

늘 걱정하듯 말하죠

헛된 꿈은 독이라고

세상은 끝이 정해 책처럼

이 돌이킬 없는

현 이라고 helliphellip

래 난 난 꿈이 있어

꿈을 믿어

나를 켜

저 차갑게 서 있는 이란 앞에

당당히 마주칠 있어

출처 가 거위의 꿈 작사 이적 작곡 동률- lsquo rsquo ( )

① ② ③ ④ ⑤

가 떴다는 들 만 지만( ) lsquo rsquo

늘 겸 다 에 주 연 우승 지 간에도 3

단 생님께 만 지 고 고 만ldquo rdquo

큼 늘 겸 신 계 가

고 다

에게는 꿈 다 통 누 가

주겠다는 것 그 꿈 다 신 극복 는 과

에 큰 경험 들도 느 게 주

고 싶다는 것 다

슬 마다 통 낼ldquo

었 것 럼 고통 는 사 들

고 겠다 고rdquo

말 다 달 루 첫 낸lsquo rsquo

첫 드 심 집에 는 리듬 2

루 에 도 보고 싶다(RampB) 집 에는 직34

사 곡 도 보 고 싶다고 포 다

미 는( ) (26) 어 헤헤헤 웃다가 어ldquo rdquo

허허허 웃었다ldquo rdquo ldquo rdquo 같 도 고

상 다 는 같 도 다( ) 壯丁 킹 들lsquo

다 는 역도 보 그 다 지만 그는rsquo

뷰에 지 다 운동만 지 ldquo

것 지 간에 여러 사 도 역rdquo helliphellip

었다 그런 엇 그 마 움직 는지 보 쯤

지 담 사 다 훈 없어 그는 티

지 림 었다 태 다 갔다 는 습

마 집 럼 편 게 보 다

주말에는 주 엇 보내

주말에도 별 주 에 청ldquo

고 에 가고 도 쳐

에 듣고 보 에 갈 가 별 없

어 산 시 게 고 들어 2002

거 매 여 지냅니다 시 과 지훈 rdquo

다 근 간 과 진실 그리고 싶어( )

가 다 근에게 그것 진리 다 거 다 없

거 고 다 없 는 것 진리

다 근 진리는 후 쪽 었다 신산( )辛酸 삶

었 질곡( )桎梏 역사 에 지냈 가

눈에 든 것 료 단 료 게 보

것 었다 그것 그 에 겨우겨우

슬 슬 생 어가는 간들 었다

리 과 단 리 고리에 검 마

없 거리 돌

상 것 없는 등 근에게 상

과 진실 엄 ( )儼存 다는 사실 리는 가

실 고 가 과 역경 에 도 근 내 포

없었 후 보루( )堡壘 다 도 365

도 간 근 여

시 것 다

년 학 간고사 대비2013 2 현대고 대비

ECN-0102-2013-001-000076193

다 공주 그림 가 근 경- ( ) ldquo rdquo(

2009)

zb27) 작가의 주 적인 각이 드러난 것은~

① ② ③ ④ ⑤

가 신 지에 고 상 원망( )

도 단다 어느 가 에 시각 에 ldquo

어 그런 듣고 다 보니 내가 lsquo

게 살 는지 도 눈 고 싶rsquo lsquohelliphellip

보 는 생각만 들 고 그 가 들에게rsquo

도 내고 들도 고 많 었죠 들었rdquo

지 새 는 에 쑥쓰러운 색

어났다 략 [ ]

경 는 가 망 없 티lsquo

원 고 답 다 신과 같 시각rsquo

는 습 상상 만 도 감동

다 시각 연주 동시에

열 상 는 티

원 그런 열 경 럽다는 것 다 략 [ ]

슬 마다 통 낼ldquo

었 것 럼 고통 는 사 들

고 겠다 고rdquo

말 다 달 루 첫 낸 lsquo rsquo

첫 드 심 집에 는 리듬 2

루 에 도 보고 싶다 집 에는 직(RampB) 3 4

사 곡 도 보 고 싶다고 포 다

식 누 가-

고 싶어

다 역도 미 담 고 사( )

질 주말에는 주 엇 보내[ 1]

답 주말에도 별 주 에[ ] ldquo

청 고 에 가고 도 쳐

에 듣고 보 에 갈 가 별

없어 rdquo

질 계 고 슬슬 도 는 것 닙니[ 2]

답 다 들 눈 에 보 고 뿐 보[ ] ldquo

다 열심 고 어 상에 도 들지만 상

지키는 것 들다고 에 도달

그것 지키 훨 많 rdquo

질 들 살 고 리 는[ 3]

거운 들 체 리느 는다

답 가 고 게 체 어[ ] ldquo ( ) 級

느 도 계가 니 살 는 것도 고역 지만

살 우는 것 들어 는 살

체 리 고 어도 어도 실 갔다

쑥 어 rdquo

질 거리에 슷 연 여 들[ 4]

보는 간 상 지

답 상 다 체 게 리지 못[ ] ldquo

거 주변에 는 그 거 누 보지

못 고 뻐지고 싶 에 체 리는 에

타 워 지만 는 어울 는 것보다 는

시간 운동만 는 건 니에 사복 lsquo rsquo

고 사복 는 말에 들 웃지만 늘 운동복

고 지내니 사러 갈 도 어 rdquo

질 역도가 말 단 식 운동 니[ 5]

답 가 내는 만 클 업 보[ ] ldquo

그러니 만 쓰는 식 운동 니다

만 다고 거운 것 들 는 건 니거든 연

도 고 가지 동 에 도 여러 가지

복 들

보식 역도 여 미-

zb28) 가 에 대한 설 으( ) 않은 것은

① 시각 우 지 시 에 지

고 망 가는 태도 달 고 다

② 언어 과 언어 복 사 여

담 내 생각 게 는 가

③ 직 감 그 마 것

럼 생생 게 느껴지는 과 주고 간 내

없 리 어 억 게 다

④ 담 내 식 리 여 담 삶 습

과 가 시 여 독 에게 감동과 훈 다

⑤ 직 진 담 직 누

지 못 는 독 에게 생생 상 달 주고

담 욱 게 다

zb29) 나 의 각 의 의도를 설 한 것으 적절하( ) 않

년 학 간고사 대비2013 2 현대고 대비

ECN-0102-2013-001-000076193

은 것은

① 질 담 상 보여 주 것 다1

② 질 담 과 그에 삶 태도 보여2

주 것 다

③ 질 역도 겪는 어 움에 역도3

과 것 다

④ 질 같 연 여 갖는 고민 는지 말4

주 는 것 다

⑤ 질 역도가 과 고 운동 는 것5

담 가 말 주 는 것 다

가 만진 것 다( ) 3

감 달 다고 다 억 에( ) 音感

지워 지만 당시 청 탁 리도

다고 다 드럼 웠다 4

에 갈 마다 드럼 는 리가 신 게 들

다고 다 눈 볼 가 없 니 엔ldquo

는 는 님 틱 에 여 주

다 드럼과 연 맺 과 들 주었다rdquo

식 누 가-

고 싶어

역( ) 도가 말 단 식 운동 니

가 내는 만 클 업에 보ldquo

그러니 만 쓰는 식 운동 니다 만

다고 거운 것 들 는 건 니거든 연

도 고 가지 동 에 도 여러 가지 복

들 시 는 상 상

드는 상 에 맞춰 실 에 는 여러

펼쳐집니다rdquo

략( )

늘 에 는 어 만 것 같

가 에 사 고 사 사ldquo

겠어 든 에 가 경 만 고

울 는 사 겠어 rdquo

보식 역도 여 미-

다 가 운 는 어 어( ) ldquo rdquohelliphellip

월 새벽 시 태 없 거웠고1965 5 6 1

는 없 그 병원에 퇴원 집

가는 마지막 마 고 마 내 거 다

가 죽 간신 에 실 다 사는 어느5 lsquo

가 죽 는 말 가 식 다 신rsquo

상에 각 시키는( )刻印 에 실

어느 가는 후 민 가가 근 었다lsquo rsquo

는 간 과 진실 그 다는( ) ldquo

에 단 평 견 가지고 다 내

가 그리는 간상 단 고 다 지 다 는 그들

가 에 는 평 지 니 그리고 어린

들 미지 겨 그린다rdquo

마 근 간 과 진실 그리고 싶어( )

가 다 근에게 그것 진리 다 거 다 없

거 고 다 없 는 것 진리

다 근 진리는 후 쪽 었다 신산(辛酸 삶)

었 질곡(桎梏 역사 에 지냈)

가 눈에 든 것 료 단 료 게

보 것 었다 그것 그 에 겨우겨우

슬 슬 생 어가는 간들 었

다 리 과 단 리 고리에 검

마 없 거리 돌

상 것 없는 등 근에게 상에

과 진실 엄 다는 사실 리는 가( )儼存

실 고 가 과 역경 에 도 근 내

포 없었 후 보루(堡壘 다 도)

도 간 근365

여 시 것 다

월 강원도 림리에( ) 1914 2 21

삼 독 태어났다 어 근 복

그것 그리 가지 못 다 근 곱 살

지는 산 산업에 실 고 답마 에 내

갔다 근 그림 럼 쫓 다니 가 시 것

다 상 진 것도 가 었다

러 가 에도 고 근 가 꿈꾸었다 근

가 꿈꾸게 것 보통 업

원색도1926 만lsquo rsquo 었다

공주 그림 가 근 경-

zb30) 에 대한 설 가장 른 것은~

① 역도가 과 운동 도 질

② 리는 는 다 lsquo rsquo

③ 들었지만 그럭 럭 는 다 lsquo rsquo

④ 가 게 보 시 말 다

⑤ 보 병 는 지 상 lsquo rsquo

는 말 다

년 학 간고사 대비2013 2 현대고 대비

ECN-0102-2013-001-000076193

시간 많지 다 청량리 생 병원

마지막 상 경 릿 게 들어 다 그 는 십

만 큰 가 상 말 다

지 못 들 마 갈 고 돗

도시민들 싹 싹 탔다 가 시

월에 병원에 원 가 폐 진 몸도4 ( )疲弊

갈 미 지 못 고 었다 가는 얼마( ) 解渴

지 생 에 생각 가

마감 는 신 평생 십 만에

가 과 많 닮 다고 생각 지는

가 운 는 어 어ldquo rdquo 1965helliphellip

월 새벽 시 태 없 거웠고 는5 6 1

없 그 병원에 퇴원 집 가

는 마지막 마 고 마 내 거 다 가

죽 간신 에 실 다 사는 어느 가5 lsquo

죽 는 말 가 식 다 신rsquo

상에 각 시키는 에 실 어느( ) lsquo刻印

가는 후 민 가가 근 었다rsquo

ldquo 는 간 과 진실 그 다는 에

단 평 견 가지고 다 내가 그

리는 간상 단 고 다 지 다 는 그들 가

에 는 평 지 니 그리고 어린 들

미지 겨 그린다rdquo

근 간 과 진실 그리고 싶어 가

다 근에게 그것 진리 다 거 다 없 거

고 다 없 는 것 진리다

근 진리는 후 쪽 었다 신산 삶 ( )辛酸

었 질곡 역사 에 지냈 가 눈에( )桎梏

든 것 료 단 료 게 보 것

었다 그것 그 에 겨우겨우 슬

슬 생 어가는 간들 었다 리

과 단 리 고리에 검 마

없 거리 돌 상

것 없는 등 근에게 상에 과 진실

엄 다는 사실 리는 가 실( )儼存

고 가 과 역경 에 도 근 내 포 없었

후 보루 다 도 도( ) 365堡壘

간 근 여 시 것

간에 지닌 가 근 1914 2

월 강원도 림리에 삼 독21

태어났다 어 근 복 그것 그리

가지 못 다 근 곱 살 지는 산

사업에 실 고 답마 에 내 갔다 근

그림 럼 쫓 다니 가 시 것 다 상

진 것도 가 었다 러 가 에도

고 근 가 꿈꾸었다 근 가 꿈꾸게

것 보통 업 원색1926

도 만 었다lsquo rsquo

그림 가 근 경 공주- ldquo rdquo ( 2009)

zb31) 다음 이 같은 의 성 소에 해당하 않은

것은

사건 평① ② ③

④ 주 ⑤ 경

가 운 는 어 어ldquo rdquo 1965helliphellip

월 새벽 시 태 없 거웠고 는5 6 1

없 그 병원에 퇴원 집 가

는 마지막 마 고 마 내 거 다 가

죽 간신 에 실 다 사는 어느 가5 lsquo

죽 는 말 가 식 다 신rsquo

상에 각 시키는 에 실 어느( ) lsquo刻印

가는 후 민 가가 근 었다rsquo

는 간 과 진실 그 다는 에ldquo

단 평 견 가지고 다 내가 그

리는 간상 단 고 다 지 다 는 그들 가

에 는 평 지 니 그리고 어린 들

미지 겨 그린다rdquo

근 간 과 진실 그리고 싶어 가

다 근에게 그것 진리 다 거 다 없 거

고 다 없 는 것 진리다

근 진리는 후 쪽 었다 신산 삶 ( )辛酸

었 질곡 역사 에 지냈 가 눈에( )桎梏

든 것 료 단 료 게 보 것

었다 그것 그 에 겨우겨우 슬

슬 생 어가는 간들 었다 리

과 단 리 고리에 검 마

없 거리 돌 상

것 없는 등 근에게 상에 과 진실

엄 다는 사실 리는 가 실( )儼存

고 가 과 역경 에 도 근 내 포 없었

후 보루 다 도 도( ) 365堡壘

간 근 여 시 것

간에 지닌 가 근 1914 2

월 강원도 림리에 삼 독21

태어났다 어 근 복 그것 그리

가지 못 다 근 곱 살 지는 산

사업에 실 고 답마 에 내 갔다 근

그림 럼 쫓 다니 가 시 것 다 상

진 것도 가 었다 러 가 에도

고 근 가 꿈꾸었다 근 가 꿈꾸게

것 보통 업 원색1926

도 만 었다lsquo rsquo

공주 그림 가 근 경- ldquo rdquo ( 2009)

년 학 간고사 대비2013 2 현대고 대비

ECN-0102-2013-001-000076193

zb32) 위 을 작성하는 과정에서 되어 활 된 자

어 것은

신 사 료① 연보②

고③ ④ 들과 담

⑤ 에 평

는 간 과 진실 그 다는 에ldquo

단 평 견 가지고 다 내가 그

리는 간상 단 고 다 지 다 는 그들 가

에 는 평 지 니 그리고 어린 들

미지 겨 그린다rdquo

근 간 과 진실 그리고 싶어 가

다 근에게 그것 진리 다 거 다 없 거

고 다 없 는 것 진리다

근 진리는 후 쪽 었다 신산 삶 ( )辛酸

었 질곡 역사 에 지냈 가( )桎梏

눈에 든 것 료 단 료 게 보

것 었다 그것 그 에 겨우겨우

슬 슬 생 어가는 간들 었다

리 과 단 리 고리에 검 마

없 거리 돌 상

것 없는 등 근에게 상에 과

진실 엄 다는 사실 리는 가 실( )儼存

고 가 과 역경 에 도 근 내 포

없었 후 보루 다 도 도( ) 365堡壘

간 근 여 시

것 다

간에 지닌 가 근 1914 2

월 강원도 림리에 삼 독21

태어났다 어 근 복 그것 그리

가지 못 다 근 곱 살 지는 산

사업에 실 고 답마 에 내 갔다 근

그림 럼 쫓 다니 가 시 것 다 상

진 것도 가 었다 러 가 에도

고 근 가 꿈꾸었다 근 가 꿈꾸게

것 보통 업 원색1926

도 만 었다lsquo rsquo

질 루 마 가 도 린다 경건

움 느껴지는 경 다 훗 근 그림에

과 는 거 것( )裸木

만 간과 연 엮어 가는 경건 움lsquo rsquo

니었

같 가가 고 싶었 근에게 그 꿈에 다

가가는 지 다 다 가 지망생들 규 미

상 에 진 고

에 지만 근 다 다 근

미 에 운 것 보통 시 미 시간

다 그런 그에게 없는 연습 가가

통 다 가 귀 시 지 도

얻는 뛸 듯 뻤지만 마 도 가 에

듯 는 었 에 어린 근 주 에

에 그림 그리고 지우고 복( )粉板

시간 가는 게 루 보냈다

근 그 갈 가가 것 열여( )渴求

었 다가 미1932 lsquo rsquo ( lsquo

미 에 다 다는 고 마rsquo) lsquo rsquo

가 근 집 고도 지는 시골 경

그린 그림 다 후 근 에 1943 22

지 미 에 그림 고

에 걸쳐 다 미 근 가

동 는 었다

공주 그림 가 근 경- ldquo rdquo ( 2009)

zb33) 위 의 내 과 일치하는 것은

가 근 가 꿈 포 다①

근 당 가들과 께 에 다②

살 근 가 걷20③

게 었다

④ 만 통 근 역경 겨내는lsquo rsquo

느 다

⑤ 근 간 과 진실 그리 에 그 에

드러 는 간상 단 다

계 시 주 근 건강

걸었다 신 과 간에 상 다 건강

신 는 눈에도 다 근 쪽 눈 뿌 게

보 지 과에 다 다 시 지지 고 결

내 었다 시 지만 마 막막

다 늦어 결 근 쪽 눈 고 말 다

쪽 눈 근에게는 쪽 눈 었고

계 었다 그 근 는 여 그lsquo rsquo

다 근 에 같 그림 그 었다1950

시 그림 는 여 쪽lsquo rsquo

고 어 마주 고 는 그림1963

여 과 동 다 마 복

그린 듯 눈 내리 새 게 다 지

사 다 근 게 복 것

복 상과 타 는 근 상

가 떳떳 단 었고 근 그리고

간 과 진실 에 다가가 가 근다

운 었다 근 신에게 당당 지 그리고

그 다 근 그림에 단 복 보다

년 학 간고사 대비2013 2 현대고 대비

ECN-0102-2013-001-000076193

태 도 그리고 극 보다 과

얻 여 었다 과 통

근 그리고 는 재 고 에 질

만들고 특 것 다

공주 그림 가 근 경- ldquo rdquo( 2009)

zb34) 의 이유에 대해 추 한 것으 적절하 않은 것

상과 타 시도①

보다 과 얻②

근 신에게 당당 지③

④ 간 과 진실 에 다가

⑤ 태 도 얻

근 가가 었지만 그 다니 가

럼 어지지 다 복과 쟁 거쳐 시

는 가 근에게 생계 사 에

운 사 다 에 키에 건( ) 178cm死鬪

체 근 에 동 역 업( )荷役

가 생계 다 쟁

에는 동에 운 상우 주 미

죄 사 에 그림 그리는 시 다 그곳에

에 동 역 업 것에

결 것 럼 보 다 지만 그런 것만도

니었다 그림 그리는 고는 지만 매 근

는 극 간 과 별 없는 경 리 그림

벽에 그리는 것 었다 우도 리 없었다 근

트 는 우 그림 그 다 생

계 그림 단 것 다

후 근 지 신 계 리에 미

엑 리 겼다 근 곳에

건 사 크 에 미 들 ( )

상 상 그 다 근 갖 다 겪

냈다 그리고 결 그 돈

신동에 어 사리 집 마 다 마 ㄷ

루 심 쪽에는 과 엌 쪽에는 건

었다 건 주고 근 가 에

여 살 다 심 에는 지 집어

쓰고 지만 곳 근 가 에게 러웠

보 리 다 근 과 마루 업실 삼 그림

그 다 신동 마루는 근 그림에 등 는 lsquo rsquo

같 상들 지 다 시 고

에 들 폐허가

가 업실 었다

공주 그림 가 근 경- ldquo rdquo( 2009)

zb35) 위 에 대한 설 으 적절한 것은

업 시 여 훈과 감동 다①

에 주 평 드러 다②

사 사 등 식 과 ③

④ 다 근거 시 여 삶에

⑤ 살 시 사 경 께 여

습 시 다

가 시간 많지 다 청량리 생 병원( )

마지막 상 경 릿 게 들어 다 그 는

십 만 큰 가 상 말 다

지 못 들 마 갈 고 돗

도시민들 싹 싹 탔다 가 시

월에 병원에 원4 가 폐( )疲弊

진 몸도 갈 미 지 못 고 었다( )解渴 가는

얼마 지 생 에 생각

가 마감 는 신 평생 십 만에

가 과 많 닮 다고 생각 지는

가 운 는 어 어( ) ldquo rdquohelliphellip

월 새벽 시1965 5 6 1 태 없 거웠고

는 없 그 병원에 퇴원 집

가는 마지막 마 고 마 내 거 다

가 죽 간신 에 실 다 사는 어느5 lsquo

가 죽 는 말 가 식 다 신rsquo

상에 각 시키는 에 실( )刻印

어느 가는 후 민 가가 근 었다lsquo rsquo

다 는 간 과 진실 그 다는( ) ldquo

에 단 평 견 가지고 다 내

가 그리는 간상 단 고 다 지 다 는 가

에 는 평 지 니 그리고 어린 들

미지 겨 그린다rdquo

근 간 과 진실 그리고 싶어( )

가 다 근에게 그것 진리 다 거 다 없

년 학 간고사 대비2013 2 현대고 대비

ECN-0102-2013-001-000076193

거 고 다 없 는 것 진리

다 근 진리는 후 쪽 었다 신산( )辛酸 삶

었 질곡 역사 에 지냈( )桎梏

가 눈에 든 것 료 단 료 게 보

것 었다 그것 그 에 겨우겨우

슬 슬 생 어가는 간들 었다

리 과 단 리 고리에 검

마 없 거리 돌

상 것 없는 등 근에게 상에

과 진실 엄 다는 사실 리는 가 실( )儼存

고 가 과 역경 에 도 근 내 포

없었 후 보루 다( ) 堡壘 도 365

도 간 근 여

시 것 다

마 같 가가 고 싶었 근에게 그 꿈( )

에 다가가는 지 다 다 가 지망생들

규 미 상 에 진 고

에 지만 근 다 다 근

미 에 운 것 보통 시 미 시간

다 그런 그에게 없는 연습 가가

통 다 가 귀 시 지 도

얻는 뛸 듯 뻤지만 마 도 (

는 었 에 어린 근 주 에)

에 그림 그리고 지우고( )粉板

복 시간 가는 게 루 보냈다

zb36) 전 의 성 소가 아닌 것을 고르

① 평 ② 사건 ③ 경

④ ⑤ 훈

늘 지 상에 살고 는 사 들 억 도가10

고 그리 지 통 고 는 사 들( )知的

그보다 훨 많 억 도는 고 지 20

통 다 그런 지 고 2500

그리 간 보는 과 사 에

매우 달 뿐만 니 과 에 도 극

루고 었다 미 운 그런 들

살고 는 동 과 사 들 사고 식에

큰 가 다는 다

고 그리 들 우주 개별 고 독립

사 들 생각 지만 고 들 우

주 연 질 간주 다 같( ) 看做

각 도 들에게는 연 질

었지만 그리 들에게는 미 들 결 었

다 고 과 그리 들 사 같

는 동 과 사 에 도 견 다

지심리 미 마 드 겐트 는

살 들에 에 지 다

연 동 과 상 다 과 같 실험

다 크 만든 미드 도 보

여 주고 그 상 닥 고 주었다lsquo (Dax)rsquo

실 닥 는 재 지 는 것 실험 가lsquo rsquo

만들어 낸 다 그런 다 개 다 체 보

여 주었는 는 미드 지만 틱

만들었고 다 는 재료는 크 지만

달 다 그러고 어 것 닥 지 사 들에게 고 lsquo rsquo

게 니 들 주 같 고 는

체 택 고 동 들 같 재료 만들어진 체

택 다 러 는 심지어 살짜리

들에게 도 타났다 것 곧 과 동

다 상 보고 다는 것 미 다

개별 사 보고 고 동 연 질 보

고 는 것 다

동 들 주변 상 에 맞 어 동 고

에 다 사 들 태도 동에 보다 많

주 울 다 동 가 미시간 에

에 경험 다 그는 미식

경 보러 가게 었는 경 체는 매우 재미 었

주변 들 동에 질 다 그 는

들 계 어 상태 경 다

어 들 에 에 그 시 가 계 가

진 것 다 상 살펴 는 말 들 lsquo rsquo

에 그는 에 시 어 도 뒷사

생각 곧 다시 곤 것 다 그런 그에게 뒷

사 고 지 는 들 동 럼

어 웠다

생각 지도 리 드 니 벳-

zb37) 다음 위 의 내 전개 으 만 인lt gt

것은

lt gt

대조의 통해 대상이 닌 특성을 설 하고 있다

일화를 제 하여 자 의 주장을 뒷 침하고 있다

유추의 을 사 하여 독자의 의해를 돕고 있다

대상이 형성되는 과정을 간적 서에 따라 서 하고 있

① ②

③ ④

년 학 간고사 대비2013 2 현대고 대비

ECN-0102-2013-001-000076193

가 우리가 말 고 쓰는 든 단어가 사 에 는( )

것 니다 사 격에 가 는 지만

어 사 과 같 특별 는 사 니lsquo rsquo

단어 격 보 단어가 사 에

등재 어 다 리 리 사 는 단어 도 그

것 시 사 는 어 고 사 에

격 보 것 니다

러 얼 은 사전에 를 있는가 이에 대한 답lsquo rsquo

은 얼 이 유행어인가 아닌가에 따라 갈라 다 이 단어lsquo rsquo

는 년 어 자 에 랐고 쓰이고 있으2002 lsquo rsquo

유행어라고 하 에는 생 이 다 런데 계속

을 유 하 서 사전에 등재될 자격을 획득할 것인가 이

에 대한 답을 내리 는 히 어 다

여 서 가 를 고 해 볼 있다 첫 는 이 단어

를 써야 할 필 가 속적으 있는가 하는 점이다

상주의 열풍에 휩 인 사회 위 에 편 해서 퍼 말

이 얼 인데 과연 런 위 가 속될 것인가 이에lsquo rsquo

대해 필자의 생각은 정적이다 사회 위 가 뀌

런 말을 쓸 일이 없어 것이다

다음은 단어의 성이다 단어의 성이 사회적으 거

감이 없으 계속 사 될 가능성이 높다 런 에서

얼 은 좋은 조건이 아니다 익히 알 졌듯이 이lsquo rsquo

말은 얼 과 청소년층에서 속어 사 하는 이 결합lsquo rsquo lsquo rsquo

된 말이다 얼 에서 얼 을 리하는 조어 도 lsquo rsquo lsquo -rsquo

어에서는 매 낯선 이다 이것만으 도 거 감을 갖

는 사람들이 있다 더 나 속어 결합한 말이다 얼 lsquo rsquo

이 널리 퍼졌다 해도 은 여전히 청소년층의 속어lsquo rsquo

남아 있다 속어는 자연 럽게 아 자리에서나 쓰 에는

담 러 말이다 러한 담을 하고 사

역을 넓혀 가는 속어도 없 는 않다 특히 얼 은 lsquo rsquo

에도 종종 등장한다 만큼 거 감이 많이 희석되었다

고 할 있다 러나 일상의 자연 러 대화에서도 거

리낌 없이 등장하는가 게 는 되 않았다고 생

각한다

얼 이 유사어인 쌈 등을 만들어 내고lsquo rsquo lsquo rsquo

있으니 살아남을 있을 것이라고 는 견해도 있을 것

이다 러나 간이 나 서 유사어를 포함하여 든

말이 사라 사 는 많다 유사어가 많다는 것이 생 을

유 할 있는 절대적인 조건은 아니다

나 언젠가 터 사람들은 어느 단에서 얼 이 가장( )

쁜 사람을 가리켜 얼 이라고 르고 있다 이 얼lsquo rsquo lsquo rsquo

이라는 단어가 최근 어사전에 라 항간에 논란이 일고

있다 아닌 게 아니라 얼 은 유행어처럼 인다 생 lsquo rsquo

도 리 래되 않은 것 같고 언제 사라 도 알

없다 게다가 젊은이들 사이에서 주 쓰일 뿐이다 이런

단어를 사전에 는다는 게 하 이 없어 이 도

한다

러나 속단은 이다 차근차근 따져 볼 일이다

선 얼 이 일 적 유행어인 아닌 주의 게 들여다lsquo rsquo

볼 필 가 있다 유행어란 유행에 따라 빠르게 유포되었

다가 단 간 내에 소 되는 단어나 를 가리킨다

얼 은 인터넷을 통해 속히 퍼 말이다 하 만 일lsquo rsquo

적인 유행어처럼 단 간 내에 사라 않았을 뿐 아니라

현재 도 잦은 빈도 사 되고 있고 앞으 도 상당

간 사 될 것으 측된다 한 언 재단의 뉴 검 lsquo rsquo

색 사이트에 따르 얼 은 년 에 처음 나타난lsquo rsquo 2001

이후 꾸 히 사 되고 있다

이 같은 사 빈도는 얼 이 일 적 유행어 는 현lsquo rsquo

저히 다르다는 것을 여 다 장 간의 생존 만으 도

얼 은 이 한 어의 어휘 에 를 자격을 얻었다lsquo rsquo

고 할 있다 더 이 이라는 비 적 정제된 매체에

높은 빈도 쓰이고 있 않은가 사 빈도 측 에서

필통이나 연필과 같은 단어 대등하거나 더 많이 쓰lsquo rsquo lsquo rsquo

다는 것은 결코 가 게 볼 일이 아니다

이제는 사전이 언어 현 을 빠르게 하는 게 덕인

대가 되었다 세계적으 유 한 의 사전들도 경쟁

적으 어를 고 있다

하 만 얼 은 젊은이들이나 쓰는 속어라고 흠을 잡을lsquo rsquo

도 르겠다 얼 이 주 젊은 층에서 많이 쓰 lsquo rsquo

는 속어임에 틀림없다 러나 어사전에 표 적이고 품

위 있는 말만 어야 한다고 생각한다 것은 커다란

해다 당장 아 어사전이나 펼쳐 라 속어는

설과 같은 비어나 죄자들이 쓰는 은어 어

마니 같은 소 의 사람만이 쓰는 말 도 라 있

않은가 사전은 말 치에 일정 빈도 이상 나타나는 말이

라 말이든 다 할 있다

zb38) 가 나 에 대한 다음의 설( ) ( ) 않은 것은

① 가 는 얼짱 사 에 등재 것에( ) ( ) lsquo rsquo

보 고 다

② 사 등재 가는 단어 격에( )

고 고 는 언 들 언어 사 도에 고 다 ( )

③ 가 얼짱 어지만 신 과 같 매( ) ( ) lsquo rsquo

체에 도 사 는 말 는 고 다

④ 가는 얼짱 어 보고 크게 가지 근( ) lsquo rsquo 3

거 들어 뒷 고 다

⑤ 는 얼짱 어 는 다 특 다는( ) lsquo rsquo

근거 에도 크게 가지 근거 가 들어 주 2

뒷 고 다

가 늘 지 상에 살고 는 사 들 억( ) 10

도가 고 그리 지 통 고 는 사 들

그보다 훨 많 억 도는 고 지 20

통 다 그런 지 고 2500

년 학 간고사 대비2013 2 현대고 대비

ECN-0102-2013-001-000076193

그리 간 보는 과 사 에

매우 달 뿐만 니 과 에 도 극

루고 었다 미 운 그런 들

살고 는 동 과 사 들 사고 식에

큰 가 다는 다

고 그리 들 우주 개별 고 독립

사 들 생각 지만 고 들 우

주 연 질 간주 다 같 각

도 들에게는 연 질 었지

만 그리 들에게는 미 들 결 었다

고 과 그리 들 사 같 는

동 과 사 에 도 견 다

인 리학자인 츠 이마이 디드 겐트너는 두

살이 채 안 된 아이들에서 터 성인에 이르 다양한

연 대의 동양인과 서양인을 대상으 다음과 같은 험

을 했다 저 코르크 만든 피라 드 양의 도형을

여 주고 대상의 이름을 닥 라고 알 주었다lsquo (Dax)rsquo

제 닥 는 존재하 않는 것으 험자가 임의lsquo rsquo

만들어 낸 이름이다 런 다음 두 개의 다른 체를

여 주었는데 하나는 피라 드 양이 만 하얀 플라 틱

으 만들었고 다른 하나는 재 는 코르크 만 양이

달랐다 러고 나서 어떤 것이 닥 인 사람들에게 고 lsquo rsquo

르게 했더니 서양인들은 주 같은 양을 하고 있는

체를 선택했고 동양인들은 같은 재 만들어 체를

선택했다 이러한 차이는 성인은 어 두 살 리

아이들에게서도 나타났다 이것은 곧 서양인과 동양인은

서 다른 세상을 고 있다는 것을 의 한다 략 ( )

는 아주 단 하 서도 인상적인 험을 했다

험에는 동서양의 대학생들이 참여했다 는 험 참가자

들에게 컴퓨터 화 을 통해 속 장 을 담은 애니 이션

을 여 주었다 화 의 앙에는 초점의 역할을 하는 커

다란 고 한 마리가 있었고 주위에는 다른 생

들과 초 자갈 거품 등이 함 제 되었다 화 을

두 씩 후 참가자들은 자 이 것을 회상해 라는

를 았다

결과 서양인 대학생들과 동양인 대학생 두 앙

의 초점 역할을 했던 고 를 동일한 정도 언 했으

나 경 소 위 거품 초 다른 생 들 에 ( )

대해서는 동양인 대학생들이 서양인 대학생들 다 60

이상 더 많이 언 했다 뿐만 아니라 동양인 학생들은 서

양인 학생들에 비해 개 적인 고 다 전체적인 계

를 더 언 하는 경향을 다 략 또한 경의 일 ( )

를 화 킨 림을 제 하 을 때 동양인 대학생들은 대

경의 화를 알아챘 만 서양인 대학생들은 경

의 화를 거의 알아차리 했다 략 ( )

따라서 서양인들만을 대상으 연 한 화lsquo

편성 결 은 잘 된 것일 도 있다 각 과정과 인rsquo

과정의 어떤 이 화 편적이고 어떤 이

화에 따라 달라 는 는 앞으 많은 연 를 통하여 논의

되어야 한다

나 어떤 의 에서 리 두는 이 화적이다 리( )

안에는 다른 사람들과 더 친 한 계를 유 하 는 상호

의존성과 다른 사람들 터 독립적인 존재 살아가 는

독립성이 혼재한다 따라서 이 에서 어떤 특성이 더 강

하게 각되는 상황에 놓이느냐에 따라 서 다른 화적

특 을 일 있다 결 리 두는 어떤 경 에는

동양인처럼 행동하고 어떤 경 에는 서양인처럼 행동하는

것이다

zb39) 가 에 대한 다음의 설( ) 않은 것은

① 는 신 주 뒷 닥 실험과lsquo rsquo lsquo

니 실험 근거 시 다rsquo

② 동 들 상 간 공통 보다는 에 식

는 강 다

③ 들 주변 맥 에는 심 경 어 사건

과 사건 사 계에 상 민감 다

④ 는 동 과 틀린 지 고 는 것lsquo rsquo

니 다 고 다 lsquo rsquo

⑤ 가에 우리 사 들 개 시 가 원( )

집 경 말 고 는 것 개 보다는

에 고 는 것에 다

늘 지 상에 살고 는 사 들 억 도가10

고 그리 지 통 고 는 사 들( )知的

그보다 훨 많 억 도는 고 지 20

통 다 그런 지 고 2500

그리 간 보는 과 사 에

매우 달 뿐만 니 과 에 도 극

루고 었다 미 운 그런 들

살고 는 동 과 사 들 사고 식에

큰 가 다는 다

지심리 미 마 드 겐트 는 동

과 상 다 과 같 실험 다

크 만든 미드 도 보여 주고 그

상 닥 고 주었다 그런 다lsquo (Dax)rsquo

개 다 체 보여 주었는 는 미드

지만 틱 만들었고 다 는 재료는

크 지만 달 다 그러고 어 것 닥 lsquo

지 사 들에게 고 게 니 들 주 같rsquo

고 는 체 택 고 동 들 같

재료 만들어진 체 택 다 러 는

심지어 살짜리 들에게 도 타났다 것

곧 과 동 다 상 보고 다는

것 미 다 개별 사 보고 고 동

년 학 간고사 대비2013 2 현대고 대비

ECN-0102-2013-001-000076193

연 질 보고 는 것 다

동 들 주변 상 에 맞 어 동 고

에 다 사 들 태도 동에 보다

많 주 울 다 동 가 미시간

에 에 경험 다 그는 미

식 경 보러 가게 었는 경 체는 매우 재

미 었 주변 들 동에 질 다 그

는 들 계 어 상태 경

다 어 들 에 에 그 시 가 계

가 진 것 다 뒷사 고 지 는 들

동 럼 어 웠다

그는 경험에 어 얻어 동 들lsquo

각도 상 본다 는 가 우고rsquo

검 여 주 단 도 상 실험 실

시 다 그는 실험 가 들에게 컴퓨 통

담 니 보여 주었다

에는 역 는 커다 고 마리가 었

고 주 에는 다 생 들과 갈 거 등

께 시 었다 본 후 가 들

신 본 것 상 보 는 지시 다

그 결과 생들과 동 생

역 고 동 도 언

경 거 다 생 들에 ( )

는 동 생들 생들보다 60

상 많 언 다 뿐만 니 동 생들

생들에 개별 고 보다 체 계

언 는 경 보 다 경 변 시

킨 그림 시 동 생들 경

변 지만 생들 경 변

거 리지 못 다

지 지 들만 상 연 lsquo

보편 결 못 것 도 다 지각 과 과rsquo

지 과 어 보편 고 어

에 달 지는지는 많 연 통 여

어 다

리 드 니 벳 생각 지도 사- ldquo rdquo( 2004)

zb40) 위 에 대한 설 으 가장 적절한 것은

① 동 과 생 식 강 고 다

② 가지 실험 통 쓴 고 다

③ 닥 실험에 사 본질에 동 사

상에 주 다

④ 니 실험에 동 과 에 지

각 도에 가 다

⑤ 쓴 는 보편 연 에 드러 우월 에

에 근 고 다

가 동 들 주변 상 에 맞 어 동 고( )

에 다 사 들 태도 동에 보다 많

주 울 다 동 가 미시간 에

에 경험 다 그는 미식

경 보러 가게 었는 경 체는 매우 재미 었

주변 들 동에 질 다 그 는

들 계 어 상태 경 다

어 들 에 에 그 시 가 계 가

진 것 다 상 살펴lsquo 는 말 들rsquo

에 그는 에 시 어 도 뒷사

생각 곧 다시 곤 것 다 그런 그에게

뒷사 고 지 는 들 동 럼

어 웠다

그는 경험에 어 얻어( ) 동 들lsquo

각도 상 본다 는 가 우고rsquo

검 여 주 단 도 상 실험

실시 다 실험에는 동 생들 여 다

그는 실험 가 들에게 컴퓨 통

담 니 보여 주었다 에는

역 는 커다 고 마리가 었고 주 에는

다 생 들과 갈 거 등 께 시

었다 본 후 가 들 신 본 것

상 보 는 지시 다

다 그 결과 생들과 동 생( )

역 고 동 도 언

경 거 다 생 들 에 ( )

는 동 생들 생들보다 60

상 많 언 다 뿐만 니 동 생들

생들에 개별 고 보다 체 계

언 는 경 보 다 들어 동

생들 상 체 연못 럼 보 어ldquo 같rdquo

체 맥 언 시 었지만

생들 상 어 같 큰 고 가 쪽 움ldquo

직 어 같 역 고rdquo

언 시 다 경 변 시킨 그

림 시 동 생들 경 변

지만 생들 경 변 거

리지 못 다

년 학 간고사 대비2013 2 현대고 대비

ECN-0102-2013-001-000076193

게 볼 동 들 보다는 큰 그( )

림 보 에 사 과 체 맥 연결시 지각

는 경 고 체에 특 떼어 내

어 독립 보는 것 낯 어 다 에

들 사 에 고 주변 맥 에는 심 경

에 사건과 사건 사 계에 상

민감 편 다

마 지 지( ) 들만 상 연

보편 결 못 것 도 다lsquo rsquo 지각 과

과 지 과 어 보편 고 어

에 달 지는지는 많 연 통 여

어 다

리 드 니 벳 생각 지도 사- ldquo rdquo( 2004)

zb41) 의 하는 가~ 다른 것은

① ② ③

④ ⑤

얼마 그 에 동 사고 식과

사고 식 보여 주는 내 다

들 에 는 탕 고 같 게

어 겨 고 미 에 는 그 크 럼 큰 고

어리 주고 원 는 어 도 는

상 고 생각 다는 것 다 러

는 어떻게 생 것 고 과 그리 거슬

러 가 보 그 단 다

고 연 경 체 경 생 에

다 벼 사는 공동 업과 경험 많 연 역

에 고 들 연 웃과

게 지내 고 탁 연 들

들 지 연 럽게 들 다 민들

웃과 동 게 뿐만 니 는 집 과

게 다

동 시 는 생태 경 에 살 결과

들 다 사 들 사 상 에 주

울 게 었고 는 곧 체 상 과 간 사

계 시 는 낳게 었다 신 가

가 는 체에 는 원 는 동시

에 다 사 들 그 사 포 체 맥 에

다 들 간 사 연

계 체 계에 주 울 는 사고 체계

게 었다

그러 그리 연 경 그 었다 산

지 연결 는 지 건 그리고 역

에 다 런 들 업에 다 사 과

동 므 공동체에

다고 다 고 그리 들

들과는 달리 보 내 감 지 들과

지 크게 느 지 못 다 그

견 다 경우 주 쟁 통 결 는 갖

게 었다

신 사 간 계들 루어진 커다

트워크 에 게 당연 사 역시 연

계들 체 식 게 다 어 상

원 도 그 개체가 체 맥 과

계 에 고 다 게 체 맥 에 주

울 다 보 상 복 과 가변 식 게 고

상에 재 는 많 변 들 사 에 재 는 들도

게 다 들 주 태도 보

는 경우가 많다 쟁 결

통 결 보다는 통 결

는 보 다

그러 고 그리 들 개개 사 사 독

에 주 울 다 사 사 체에

어 그들 사 에 재 는 공통 규 주

고 다 상 원 에도 사

체 내 주 고 다 그들

체 여 탕 체

는 주 태도 시 고 특 사 어

주에 는지 여 그 주에 는 규

견 다 에 는 쟁 식 리

같 리 사고 체계가 달 게 었다

리 드 니 벳 생각 지도 사- ldquo rdquo( 2004)

zb42) 위 에서 사 된 설 과 가장 유사한 것은

① 크톱 컴퓨 는 본체 니 마우 루

어 다

② 곡과 시 리 는 지 과 사 루어 다는 공통

지니고 다

③ 경 고 것과는 달리

경 본 연 태 그 주변 경

④ 벽돌 능 에 사계 내내

습도가 지 다

⑤ 잰느 체 체 지닌 재 체가 없

는 재 눌 다

년 학 간고사 대비2013 2 현대고 대비

ECN-0102-2013-001-000076193

zb43) 는 립 앙 도서 이 정의 일 이다lt gt

도서 장과 이 자의 리 의 정의 연결이

적절하 않은 것은

lt gt

제 조 서 유8 ( )

도서 장은 다른 이 자의 안전을 위협하거나 도서 의①

서를 란하게 할 가 있는 자에 대하여는 도서 출입

을 제한할 있다

도서 장은 이 자가 제 조 각 호의 어느 하나의 행위를 하7②

을 때에는 이 을 하게 하거나 도서 출입을 제한할

있다

제 조자 의 대출9 ( )

도서 자 는 다음 각 호의 경 대출할 있다①

상호대차도서 간에 자 를 류하는 것을 말한다 등 다1 ( )

른 도서 과의 협 을 위하여 필 한 경

공 이 공 행 상 필 하는 경2

에 도서 장이 필 하다고 인정하는 경3

대출이 가능한 도서 자 의 위는 도서 장이 정하는②

에 따른다

제 조 상10 ( )

이 자가 도서 자 설을 더럽히거나 찢거나 뜨①

쓰게 하거나 잃어 린 경 에는 상하여야 한다

도서 장은 제 항에 따른 상 을 정하여 게 하여야1②

한다

제 조이 절차 등11 ( )

이 칙에서 정한 것 에 도서 자 설의 이 절차

이 제한 등에 필 한 사항은 도서 장이 정한다

출처 립 앙 도서- (httpwwwnlgokr)

① 는 도 리 다8

② 도 는 리 다9 1

③ 료 지 는 도 리 다9 2

④ 도 료 변상에 리10 1

⑤ 는 에 도 리 다11

3

도 다 각 같다①

공 공 다만 연1

연 간 다

매월 째 째 월2

도 도 리 그 사3

가 다고 는

도 에 미리 게1 3②

시 여 다

4

도 시간 도 여 게시 다

5

도 료 시 는 는 도①

지에 등 후

등 에 사 도②

7

는 다 각 여 는 니 다

도 료 시 상 리1 lsquo rsquo

도 료 시 훼 는2 middot

지 가 닌 곳에 식 거 담3

우는

도 보 등 보 검색열4 middot

그 에 도 질 지 여 도5

여 게시 사 는

8

도 다 거 도①

질 게 우 가 는 에 여는 도

도 가 각 어느7②

에는 지 게 거 도

9

도 료는 다 각 경우 다①

상 도 간에 료 는 것 말1 (

다 등 다 도 과 여 경우)

공 원 공 상 는 경우2

그 에 도 다고 는 경우3

가능 도 료 는 도②

는 에 다

10

년 학 간고사 대비2013 2 현대고 대비

ECN-0102-2013-001-000076193

가 도 료 시 럽 거 거①

못 쓰게 거 어 린 경우에는 변상 여

도 에 변상 여 게시1②

여 다

zb44) 위 에서 도서 장이 게 해야 할 사항에 해당하는

것을 두 쓰

년 학 간고사 대비2013 2 현대고 대비

ECN-0102-2013-001-000076193

립 도 규

1 ( )

규 립 도 립 어린 청 도(

포 다 료 시 열 시 말) (

다 에 사 규 립 도)

편 진 다

2 ( )

규 립 도 도 다 에( lsquo rsquo )

고 는 도 에 도lsquo rsquo 2 2

료 에 여 다 다만 특 료 귀

료 등 료 에 사 립 도

도 다 다( lsquo rsquo )

3 ( )

도 다 각 같다①

공 공 다만 연1

연 간 다

매월 째 째 월2

도 도 리 그 사3

가 다고 는

도 에 미리 게1 3②

시 여 다

시간4 ( )

도 시간 도 여 게시 다

등 등5 ( )

도 료 시 는 는 도①

지에 등 후

등 에 사 도②

사 료6 ( )

도 료 시 에 사 료는 도

7 ( )

는 다 각 여 는 니 다

도 료 시 상 리1 lsquo rsquo

도 료 시 훼 는2 middot

지 가 닌 곳에 식 거 담3

우는

도 보 등 보 검색열4 middot

그 에 도 질 지 여 도5

여 게시 사 는

질 지8 ( )

도 다 거 도①

질 게 우 가 는 에 여는 도

도 가 각 어느7②

에는 지 게 거 도

료9 ( )

도 료는 다 각 경우 다①

상 도 간에 료 는 것 말1 (

다 등 다 도 과 여 경우)

공 원 공 상 는 경우2

그 에 도 다고 는 경우3

가능 도 료 는 도②

는 에 다

변상10 ( )

가 도 료 시 럽 거 거①

못 쓰게 거 어 린 경우에는 변상 여

도 에 변상 여 게시1②

여 다

등 규 에 것 에 도11 ( )

료 시 등에 사

도 다

립 도- (httpwwwnlgokr)

zb45) 도서 장의 리 있는 조항으 적절하 않

은 것은

① ② ③ ④ ⑤

년 학 간고사 대비2013 2 현대고 대비

ECN-0102-2013-001-000076193

1 ( )

사가 공 는lsquo rsquo

과 여 사 원과 리

사 타 사 규

니다

개 보 보7 ( )

사는 보통신망 등 계 는 에lsquo rsquo lsquo rsquo

원 개 보 보 니다 개lsquo rsquo

보 보 사 에 는 사 개lsquo rsquo

보 취 니다 다만 사는 다 lsquo rsquo

사 계 통 공 는 경우 원 lsquo rsquo

등 개 보 당 사에 습니lsquo rsquo

원 리에8 (lsquo rsquo lsquo rsquo lsquo rsquo

)

원 에 리lsquo rsquo lsquo rsquo lsquo rsquo①

원에게 가 도 여 는lsquo rsquo 3

니다

사는 원 가 개 보 우 가lsquo rsquo lsquo rsquo lsquo rsquo②

거 사 경우 는 미 에 어 거 lsquo

사 사 운 우 가 는 경우 당rsquo lsquo rsquo

습니다lsquo rsquo

원 가 도 거lsquo rsquo lsquo rsquo lsquo rsquo 3③

가 사 고 지 경우에는 시 사에lsquo rsquo

통지 고 사 내에 니다lsquo rsquo

경우에 당 원 사에 그 사실3 lsquo rsquo lsquo rsquo④

통지 지 거 통지 도 사 내에 지 lsquo rsquo

생 경우 사는 지지 습니다lsquo rsquo

사10 (lsquo rsquo )

사는 과 지 미lsquo rsquo①

에 는 지 계 고

공 여 다 여 니다lsquo rsquo

사는 원 게lsquo rsquo lsquo rsquo lsquo rsquo②

도 개 보 신 보 포 보 보 시( )

갖 어 개 보 취 공시 고

니다

사는 과 여 원lsquo rsquo lsquo rsquo③

견 만 당 다고 경우에는

리 여 니다 원 견 만 사 lsquo rsquo

에 는 게시 거 우편 등 통 여

원에게 리 과 결과 달 니다lsquo rsquo

원11 (lsquo rsquo )

원 다 여 는 니다lsquo rsquo ①

신청 는 변경 시 허 내 등1

타 보 도2

사가 게시 보 변경3 lsquo rsquo

사가 보 보 컴퓨 그4 lsquo rsquo (

등 등 신 는 게시)

사 타 등 지 재산 에5 lsquo rsquo 3

사 타 상 거 업6 lsquo rsquo 3

는 폭 시지 상 타 공7 middot middot

에 는 보 에 공개 는 게시 는lsquo rsquo

사 동 없 리 사8 lsquo rsquo

타 거 당9

게시15 (lsquo rsquo )

원 내에 게시 는 게시 게재 는lsquo rsquo lsquo rsquo lsquo rsquo

경우 원 사가 게시 복 lsquo rsquo lsquo rsquo lsquo rsquo middot middot

등 태 언 등에 공 는

것 내에 다 원 본 게시 등 lsquo rsquo lsquo rsquo

크 능 등 여 복 는 등 태

는 것 동 것 니다

- (wwwnavercom)

zb46) 위 은 인터넷 포털사이트의 회 가입을 위한 이

약 의 일 이다 이 약 을 만드는 과정에서 생각한

내 으 적절하 않은 것은

개 보 보 가 지에 별 눠①

겠어

원 가 만들게 에②

시 주어 겠어

원들 게재 게시 다 원 크 다③

는 것 지

④ 원 지 는 뿐만 니 사가 지 는

도 께 달 지

리에 가 생 경우 사가⑤

에 다는 도 듯

1 ( )

사가 공 는lsquo rsquo

과 여 사 원과 리

사 타 사 규

년 학 간고사 대비2013 2 현대고 대비

ECN-0102-2013-001-000076193

니다

개 보 보7 ( )

사는 보통신망 등 계 는 에lsquo rsquo lsquo rsquo

원 개 보 보 니다 개lsquo rsquo

보 보 사 에 는 사 개lsquo rsquo

보 취 니다 다만 사는 다 lsquo rsquo

사 계 통 공 는 경우 원 lsquo rsquo

등 개 보 당 사에 습니lsquo rsquo

원 리에8 (lsquo rsquo lsquo rsquo lsquo rsquo

)

원 에 리lsquo rsquo lsquo rsquo lsquo rsquo①

원에게 가 도 여 는lsquo rsquo 3

니다

사는 원 가 개 보 우 가lsquo rsquo lsquo rsquo lsquo rsquo②

거 사 경우 는 미 에 어 거 lsquo

사 사 운 우 가 는 경우 당rsquo lsquo rsquo

습니다lsquo rsquo

원 가 도 거lsquo rsquo lsquo rsquo lsquo rsquo 3③

가 사 고 지 경우에는 시 사에lsquo rsquo

통지 고 사 내에 니다lsquo rsquo

경우에 당 원 사에 그 사실3 lsquo rsquo lsquo rsquo④

통지 지 거 통지 도 사 내에 지 lsquo rsquo

생 경우 사는 지지 습니다lsquo rsquo

원에 통지9 (lsquo rsquo )

사는 특 다 원에게 통지 경우lsquo rsquo lsquo rsquo

공지 게시 통 상 게시 개별 통지에7

갈 습니다

사10 (lsquo rsquo )

사는 과 지 미lsquo rsquo①

에 는 지 계 고

공 여 다 여 니다lsquo rsquo

사는 원 게lsquo rsquo lsquo rsquo lsquo rsquo②

도 개 보 신 보 포 보 보 시( )

갖 어 개 보 취 공시 고

니다

사는 과 여 원lsquo rsquo lsquo rsquo③

견 만 당 다고 경우에는

리 여 니다 원 견 만 사 lsquo rsquo

에 는 게시 거 우편 등 통 여

원에게 리 과 결과 달 니다lsquo rsquo

원11 (lsquo rsquo )

원 다 여 는 니다lsquo rsquo ①

신청 는 변경 시 허 내 등1

타 보 도2

사가 게시 보 변경3 lsquo rsquo

사가 보 보 컴퓨 그4 lsquo rsquo (

등 등 신 는 게시)

사 타 등 지 재산 에5 lsquo rsquo 3

사 타 상 거 업6 lsquo rsquo 3

는 폭 시지 상 타 공7 middot middot

에 는 보 에 공개 는 게시 는lsquo rsquo

사 동 없 리 사8 lsquo rsquo

타 거 당9

원 계 규 내lsquo rsquo lsquo②

여 공지 주 사 사가 통지 는rsquo lsquo rsquo

사 등 여 타 사 업 에 lsquo rsquo

는 여 는 니다

- (wwwnavercom)

zb47) 위 약 의 조항에서 같은 제점을 하lt gt

고 있는 조항은

lt gt

제휴 회사에 회 의 아이디 개인 정 를 전송할 있도

한 조항은 고객에게 당한 조항이다

1 7 8① ② ③

④ 9 ⑤ 10

립 도 규

1 ( )

규 립 도 립 어린 청 도(

포 다 료 시 열 시 말) (

다 에 사 규 립 도)

편 진 다

2 ( )

규 립 도 도 다 에( lsquo rsquo )

고 는 도 에 도lsquo rsquo 2 2

료 에 여 다 다만 특 료 귀

료 등 료 에 사 립 도

도 다 다( lsquo rsquo )

3 ( )

도 다 각 같다①

공 공 다만 연1

연 간 다

년 학 간고사 대비2013 2 현대고 대비

ECN-0102-2013-001-000076193

매월 째 째 월2

도 도 리 그 사3

가 다고 는

도 에 미리 게1 3②

시 여 다

시간4 ( )

도 시간 도 여 게시 다

등 등5 ( )

도 료 시 는 는 도①

지에 등 후

등 에 사 도②

사 료6 ( )

도 료 시 에 사 료는 도

7 ( )

는 다 각 여 는 니 다

도 료 시 상 리1 lsquo rsquo

도 료 시 훼 는2 middot

지 가 닌 곳에 식 거 담3

우는

도 보 등 보 검색열4 middot

그 에 도 질 지 여 도5

여 게시 사 는

질 지8 ( )

도 다 거 도①

질 게 우 가 는 에 여는 도

도 가 각 어느7②

에는 지 게 거 도

료9 ( )

도 료는 다 각 경우 다①

상 도 간에 료 는 것 말1 (

다 등 다 도 과 여 경우)

공 원 공 상 는 경우2

그 에 도 다고 는 경우3

가능 도 료 는 도②

는 에 다

변상10 ( )

가 도 료 시 럽 거 거①

못 쓰게 거 어 린 경우에는 변상 여

도 에 변상 여 게시1②

여 다

등 규 에 것 에 도11 ( )

료 시 등에 사

도 다

립 도- (httpwwwnlgokr)

zb48) 다음 정 리 의 의 으 볼 때 가장

이 적인 것은

도 시간 도 여 게시 다①

등 에 사 도②

가능 도 료 는 도 는③

에 다

④ 도 에 변상 여 게10 1

시 여 다

⑤ 도 가 각 어느7

에는 지 거 도

zb49) 를 참고하여 이 어의 성격을 설 한lt gt

것으 적절하 않은 것은

① 보 에 는 어 시 상 고 어 시lt gt lsquo rsquo

에 보여주고 다

② 진 어 어원에 견 고 다

에는 타 어 들어가는 것 다 lsquo rsquo

③ 에 들어갈 말 각각 고 어 어 신 어~

들 언어는 질 격 강 통 없었다

④ 시 우리 에 가 었지만 지 계

과 달리 들 통 사 달 어 웠

년 학 간고사 대비2013 2 현대고 대비

ECN-0102-2013-001-000076193

⑤ 크 몽골 만주 공통어가 우리 어 같

계열에 다는 에 사 특 짐

가( )

善化公主主隱 공주님

他密只嫁良置古 몰 결 고

薯童房乙 맛

夜矣卯乙抱遣去如 에 몰 고 가다

( )

始汝 會隱日恚見隱扐 만 에 본

恥隱汝衣淸隱笑 맑 웃

고 시 여 공 크다 만 다[ ] ( ) ( ) ( ) ( )始 汝 會扐

내다 에 보다 견( ) ( )恚 見 다( )隱

럽다 맑다 청 웃( ) ( ) ( ) ( )恥 衣 淸 笑

zb50) 위의 나 를 함 고 음에 답하( ) lt gt

보lt gt

( )素那或云金川 白城郡蛇山人也

운 사산

는 고 다 는( )[ ( ) ] (素那 金川 白城

사산 사 다) ( ) 郡 蛇山

삼 사- lsquo rsquo 47

에 제 된 단어 의 표 리를 조건(1) lt gt ( ) lt gt

에 맞게 서 하

건lt gt

lsquo 었고 었다 태rsquo

에 제 된 단어 동일한 표 리에(2) lt gt ( )

의해 적은 것을 나 에서 찾아 조건 에 맞게 서 하( ) lt gt

건lt gt

에 당 는 각각( ) 개 쓸 것2 단

당 는 가 여러 개 어도 개만 쓸 것 각2

개 과 도 쪽에 개만2 2

드시 지 것( )

과 동 원리 것lsquo 고

과 동 원리 것 다rsquo

태 것

가( )

素那(或云金川) 白城郡蛇山人也

소나 또는 천 이라 한다 는 성 사( ) ( ) ( )素那 金川 白城郡〔 〕

산 사람이다 현대어 풀이( ) ( )蛇山

나( )

紫布岩乎希 회

執音乎手母牛放敎遣 자 손 암쇼 노히 고

吾 不喩慙 伊賜等肹 肹 나 안디 리샤

花 折叱肹 可獻乎理音如 고 것거 도림다

다 향찰은 리말을 리 으 적은 표 이었 만 생( )

은 고 대를 넘 하고 끊어 고 말았다 랜 세

동안 갈고 닦아 체계적이었던 향찰 표 이 사라졌

을 인은 크게 두 가 나누어 생각해 볼 있다

하나는 족 사회의 한 선호도에서 찾을 있다 라 때

향찰은 주 족 계 에서 사 했을 것으 인다 한 을

알 하고서는 한자를 활 하여 리말을 리 으 표

하 란 가능하 때 이다 런데 족들은 간이 흐

를 향찰과 같은 리 표 을 익혀 사 하 다는

아 한 을 대 사 하는 쪽을 선호하게 되었다 더 이

고 초에 인재 등 을 위해 과거제도가 행되 서 한 선

호도가 더 높아졌고 결 향찰은 소 되고 말았다

또 다른 가능성은 한 어의 특성에서 찾을 있다

터 한 과 일 세 나라는 한자 화 에 속해 다

당연한 이야 겠 만 표의 자인 한자는 어를 표 하

에 매 적절하다 어의 음절은 성 ( ) ( )聲母 韻母

이 어 고 여 에 성조가 추가되어 최종 소리가 결정된

다 래서 어는 단음절을 하나의 한자 표 하 된

다 에 초성 성 종성의 세 가 소가 하나의 음절

년 학 간고사 대비2013 2 현대고 대비

ECN-0102-2013-001-000076193

을 이 는 한 어는 음절 조가 잡하고 음절의 가 많아

서 한자 차 만으 한 어의 소리를 만족 럽게 표 할

없었다 를 들어 한 어에서는 어 니 같이 음절 lsquo rsquo

이 어 단어가 얼마든 있으나 어는( ) 複數音節

자 하나 나타내 만이다lsquo [m ]rsquo 母 ǔ

한편 일 어의 표 은 핵 적 단어는 한자 적고 토는

가나라는 일 의 자 적는 이다 적인 의 를 나

타내는 은 표의 자인 한자 적고 적 계를 나

타내는 토는 표음 자 적는 셈이니 자세히 살펴

리의 향찰 표 을 쏙 빼닮았음을 알 있다 한 어 같

은 착어이 서도 일 어에만 향찰과 유사한 표 이 살아

남은 것은 일 어의 특 때 이다 일 어는 하나의 자음과

음의 결합으 음절을 이 고 침이 거의 없는 음절 언어

이다 이러한 음절의 특색에다가 토가 달한 착어라는 점

이 향찰과 유사한 표 이 살아남을 있는 비결이었다

하 만 같은 착어라도 다양한 음소 침이 달한 한

어는 향찰 표 하는 데 근 적으 한계가 있었다

zb51) 다 하여 의 행에 대한 탐 한 결과( ) lt gt 2

않은 것은

보lt gt

善花公主主隱 공주니믄 공주님( )

----------------------------------------

-

他密只嫁良置古 그 지 얼어 고 몰 결(

----------------------------------------

-

薯童房乙 맛 맛( )

夜矣卯乙抱遺去如 몰 고 가다 에 몰 고(

가다)

주동 역 동- (薯童謠『 』

에 2 ( )他密只嫁良置古

얼다 시집가다 결 다 말 lsquo rsquo

① 실질 미 지니고 므 타 타lsquo ( )rsquo lsquo [ ]

② 에 실질 미 타내고 지 는lsquo rsquo lsquo [ ]rsquo lsquo [ ]密只 密 只

계 타내는

③ 얼어는 실질 미 포 고 므 가lsquo rsquo lsquo [ ]rsquo嫁

것lsquo [ ]rsquo 良

④ 고 어간 는 실질 미 지니고 므lsquo rsquo lsquo -rsquo

것lsquo [ ]rsquo 置

⑤ 고 어미 고는 계 타내고 므lsquo rsquo lsquo- rsquo

고 것lsquo [ ]rsquo 古

가( )

엉 훈 민middot middot middot middot middot世 宗 御 製 訓 民 正 音

말 미 듕 귁에 달middot middot middot middot middot middot middot middot中 國 文 字

니 런middot middot middot middot middot middot 어린middot middot middot middot百 姓

니 고 도 내 들middot middot middot middot middot middot middot middot middot 시러middot

펴 몯middot 미middot middot 니 내middot middot middot middot middot middot middot middot 爲

어엿middot 겨 새middot middot middot 믈여듧middot middot middot middot字 니middot middot middot

사 마다 니겨 킈 middot middot middot middot middot middot middot middot middot便 安

고 미니middot middot middot middot

본 는 상( ) (象

원리에 만들어진 본) ( )形 ㄱ ㄴ ㅁ ㅅ ㅇ

에 는 가 원리에( )加劃

그리고( )ㅋ ㄷ ㅌ ㅂ ㅍ ㅈ ㅊ ㆆ ㅎ

쓰는 병 원리에 만들어진( )竝書

마지막 체( ) ( )異體ㄲ ㄸ ㅃ ㅆ ㅉ ㆅ

ᅀ 다 상 원리에 ㅇ ㄹ

지 는 삼재 상 본 본( ) ( ) ( 天地人 三才

탕 므림과 림에 ) (初ㅡ ㅣ

재)( ) ( )( )出字 再出字ㅗ ㅏ ㅜ ㅓ ㅛ ㅑ ㅜ ㅕ

병 그리고 들 에 다시( )ㅘ ㅝ ㅣ

( )ㅣ ㅢ ㅚ ㅐ ㅟ ㅔ ㆉ ㅒ ㆌ ㅖ ㅙ ㅞ

zb52) 가 에 대한 설 으 르 않은 것을( ) 두 고르

① 어쓰 규 지키고 다

② 리 고 다

③ 말 미 미 등 어 사 다lsquo rsquo

④ 개 지 다

년 학 간고사 대비2013 2 현대고 대비

ECN-0102-2013-001-000076193

⑤ 어 원 에 가 도 고 다

엉 훈 민世 宗 御 製 訓 民 正 音

말 미 듕귁에 달 니

런 어린 니 고 도middot

내 들 시러 펴 몯 미 니middot

내 어엿 겨 새 믈여듧

사 마다 니겨middot 킈 고

미니

훈민 언 본- lsquo rsquo 5 (1459 )

zb53) 위의 에 대한 현대어 풀이가 르~ 않은 것

① 우리 말 과 달

② 어리 말 고 는 것 어도

③ 신 생각 마 껏 펼 는 사 많다

④ 게 생각 여

⑤ 사 마다 게

zb54) 훈민정음 언해 에는 한 을 창제한 동 가 드러나

있다 훈민정음 창제의 정 과 내 이 잘 연결된 것

① 주 신 말 미 듕귁에 달

② 민 신 내 어 겨

③ 신 뻔 킈 고 미니

④ 실 신 사 마다 니겨

⑤ 귀 신 계 주 는 훈민 신과 거리가

가 엉 훈 민( ) middot middot middot middot middot世 宗 御 製 訓 民 正 音 

말 미 귁에 中 國 달 文 字

니 런 어린 니 百 姓

고 도 내 들 시러 펴 몯

미 니 내 어엿 爲 겨 새

믈여듧 니 사 마다 니 字

겨 킈 고 미니 便 安

훈민 언 본- lsquo ( )rsquo ( ) 5 (1459 )訓民正音 世祖

( )

[ 1 ]

동 룡 샤 마다 복( ) ( ) ( )海東 六龍 天福

시니 고 동( ) ( )古聖 同符 시니

[ 2 ]

매 니 곶 여

미 므 니 그 내 러

가 니

[ 125 ]

우 미리( )千世 샨( )定 에( )漢水北 累仁

누 개 샤 복 업 시니( ) ( ) 開國 卜年

신( )聖神 니 샤도 경 근민 샤 욱( )敬天勤民

드시리 다

님 쇼 산 가( ) ( )洛水 山行

미드니 가

어 가- lsquo ( )rsquo 27龍飛御天歌

다 우리신 니쓰고 다만 만 쓰( )

거 샹 귀쳔 다보게 러 귀

여 쓴 도 신 보 가 고 신 에

말 어 보게 각 에 사 들

고 본 몬 능통 후에

죠 죠 니

드 도 만 공 에 사

드 미 죠 고 고 여 보 죠

보다 얼마가 거시 어신고 니 첫

가 죠 니 죠

민 들 어 신 샹

귀쳔 도보고 어보 가 만 늘

고 폐 에 만쓴 죠 민

도 러보지못 고 보니 그게 엇지

심 니 리 보 가 어 운건 다

니 쳣 말마 지 니 고 그

쓰 에 가 우 지 지

몰 거 본후에 가 어 지

고 그니 쓴편지 쟝 보

년 학 간고사 대비2013 2 현대고 대비

ECN-0102-2013-001-000076193

쓴것보다 듸 보고 그 마 니 쓴 고

어 못

그런고 에 리 과 가

만 쓴 못 민 말만 듯고

고 편 그 못 보니 그사 단

병신 못 다고 그사 식 사

니 만 고 다 과 그사

만 고 다 과 업 사 보다 식 고

죠 도 고 각 과

견 고 실 직 귀쳔 간에 그

고도 다 것 몰 귀죡 보다

사 우리 신 귀쳔 다 업

시 신 보고 과 지 게 랴

시니 샹 귀쳔 간에 우리 신 걸

간 보 새지각과 새 걸 미리

독립신- lsquo (1896)rsquo

zb55) 친 어 나의 제 장( ) 2 매 함축적

의 가 가장 유사한 것은

① 지 눈 내리고 매 득 니 내 여 가

사- lsquo rsquo

② 도 어 리듯 그 게 어 다

주 사- lsquo rsquo

③ 눈 살 다 죽 어 린 과 체 여

눈 새벽 지 도 살 다

눈- lsquo rsquo

④ 삶 근심과 고단 에 돌 거니는 여 거 는

여 리 내린 살가지 에 눈 리 눈 리

택 그 생 에- lsquo rsquo

⑤ 늘 러 고 러

청룡 룡 어 개 루 우

신경림 계- lsquo rsquo

zb56) 친 를 위 가 나 에 나타난A B ( ) ( )

세 어의 특 에 의거하여 세 어 표 하

그 산 고 공 도 맑지만

A

주변에 쓰 리는 어리 사 많다

B

건lt gt

식 가 에 타 어 특징에( ) ( )

거 과 어쓰 는 고 지 말 것

A

B

zb57) 가 의( ) 달 아ㆍ 다 의 ( ) 나셔에서 알 있는

세 어 개화 어의 특 을 비 하여 조건 에lt gt

맞게 서 하

건lt gt

어에 는lsquo 개

어에 는 다 태rsquo

zb58) 은 가 는 다 에 나 는 절lt 1gt ( ) lt 2gt ( )

일 를 췌한 것이다 의 의 가 lt 1gt (1)~(2)

유사한 말을 에서 찾아 쓰lt 2gt

보lt 1gt

런 (1) 어린 니 고百 姓

도 내 들 시러 펴 몯 미

사 마다 (2) 니겨 便 安

킈 고 미니

보lt 2gt

죠 고 고 여 보 죠

보다 얼마가 거시 어신고 니 첫 가

죠 니 죠 민

들 어 신 샹 귀쳔

도보고 어보 가 만 늘 고

폐 에 만쓴 죠 민 도

러보지못 고 보니 그게 엇지 심

니 리

년 학 간고사 대비2013 2 현대고 대비

ECN-0102-2013-001-000076193

lt 1 gt

동 룡 샤 마다 복 시( ) ( ) ( )海東 六龍 天福

고 동 시니( ) ( )古聖 同符

lt 2 gt

(A) 매 니 곶

여 니

미 므 니 그 내

러 가 니

lt125 gt

우 미리 샨 에( ) ( ) ( ) 千世 定 漢水北 累

누 개 샤 복 업 시 니( ) ( ) 仁開國 卜年 聖

신( ) 神 니 샤도 경 근민 샤( ) 敬天勤民

욱 드 시 리 다

님 쇼 산 가 ( ) ( )洛水 山行

미드니 가

- lt gt龍飛御天歌

zb59) 장과 내 상 유사한 성격의 조는125

① 뫼 고 고 고 고

어 그린 많고 많고 고 고

어 러 는 울고 울고 가느니

도 견- lt gt

② 강 에 드니 몸 다

그믈 고 가니

뒷 뫼 엄 언 니( )藥

-

③ 말 없는 청산 태 없는 다

값 없는 청 없는 월

에 병 없는 몸 별 없 늙 리

-

④ 가마귀 골에 가지 마

낸 가마귀 새

청강에 것 시 몸 러 가( ) 淸江

-

⑤ 진 골에( ) 白雪

가 매 는 어느 곳에 었는고

에 갈 곳 몰( ) 夕陽

색-

zb60) 위 에 나타난 세 어의 특 으 적절하 않은

것은

① 룡 어 주격 사에 당 는 가 사( ) lsquo rsquo六龍

고 다

② 샤 어에도 어 주체 쓰 다

는 것 다

③ 매 어 달리 사 택에 어

가 지 지지 고 다

④ 므 원 상 직 어 지 다

⑤ 드시리 다 주체 과 상 께 사

고 다

수고 하셨습니다hearts hearts

년 학 간고사 대비2013 2 현대고 대비

ECN-0102-2013-001-000076193

보닷컴에 공 는 별 보는 고등

들 여 주 는

들 습니다 슷 동 지

가 복 는 것 도가

니 복 여 습 시고 거 시

니다

정답 해설

1) 정답[ ] ④

해설 다른 것은 두 특정 업이나 단 내에서 사[ ]

하는 일종의 은어 사회 언에 해당한다 러나

는 언이 아니라 단과대학을 여서 단대 사lsquo rsquo lsquo rsquo lsquo④

대학을 여서 사대라고 한 말에 해당하 일rsquo lsquo rsquo

사회에서도 널리 쓰이 사회 언이라 할

없다

2) 정답[ ] ⑤

해설 사회 언은 같은 단 내에서 쓰이는 언어이[ ] lsquo rsquo

동일 단끼리는 단결 과 친 감을 형성하는

능을 하 리적 안감이 일어나 않는다

3) 정답[ ] ③

해설 사람이라는 차 적 표현에 대한 대안적 표현이[ ]lsquo rsquo

인 아내 처 등으 볼 있다lsquo rsquo

4) 정답[ ]⑤

해설 남성은 주 격 체를 사 한다[ ]

5) 정답[ ] ⑤

해설 흑인은 검다라는 뜻을 가 고 있을 뿐 인[ ]lsquo rsquo lsquo rsquo lsquo rsquo

다 열등한 뜻을 내포하 않는다

6) 정답 살 색 첫 작품[ ] - -

해설 살색 혹은 킨색은 한 인의 피 색을 뜻[ ] lsquo rsquo lsquo rsquo

하는 것으 인종 차 을 추 고 출 이주민

의 평등 을 침해할 있어 년 표 이2005

살 색으 이름을 꾸었다 처녀작은 처녀라lsquo rsquo lsquo rsquo lsquo rsquo

는 단어가 가 고 있는 곡된 성 인 을 한 것

으 첫 작품정도 꾸어 사 하는 것이 좋다lsquo rsquo

7) 정답[ ] ⑤

해설 호는 아들에게 해체를 사 하고 있다[ ] ① ②

장 을 성하는 청자는 자 의 아 느리 아lsquo

들 세 이다 호는 아 느리에게 해rsquo ③

체를 사 하고 있다 호가 느리 아 에게 ④

사 한 해 체 아들에게 사 한 해체는 두 비lsquo rsquo lsquo rsquo

격 체에 해당한다 호는 자 의 아랫사람인 ⑤

느리에게 아들과 마찬가 해체를 사 하는 것이

상 이 만 임 을 한 느리에게 고마 과 쁨

존 의 표 를 하 위해 자 의 아 에게 말하듯

해 체를 사 하고 있다

8) 정답[ ] ③

9) 정답[ ] ⑤

10) 정답[ ] ①

해설 청자 할아 가 장의 주체 아 다 높을[ ] ( ) ( )

경 에는 압존 에 의해 장의 주체를 높이 않는lsquo rsquo

다 러 아 서가 아닌 아 는으 계 lsquo rsquo lsquo rsquo lsquo

니다 가 아닌 있 니다 표현하는 것이 르rsquo lsquo rsquo

11) 정답 당이 당을 쫒았다 당이[ ]

당에 다

해설[ ]

12) 정답[ ] ⑤

해설 서 다른 높임표현을 통해 청자에 대해 리[ ] ⑤

적 거리감을 나타내는 인 은 이 아니라 현정이

다 가 에서 현정은 에게 해 체를 사 함으 써 ( )

친근감을 드러낸다 나 에서 연 을 게을리하는 역 ( )

도 들 때 에 화가 난 현정이 선생님에게 항의하

는 장 에서는 하 체를 사 하여 리적 거리lsquo rsquo

가 어졌음을 나타내고 있다

13) 정답[ ] ①

해설 는 는 얼 빛이 날과 어찌 다르 고[ ] lsquo rsquo

라는 뜻으 전과 달리 임이 화자를 않고

있음을 알 있다

14) 정답 달리 후 가 있다 이를 통해 경[ ] lt gt

쾌한 음악성을 형성하고 노 젓는 상황을 체적으

형상화하는 역할을 한다

15) 정답[ ] ①

16) 정답[ ] ⑤

해설 다 의 자연은 를 성찰하게 하는 대상[ ] ( )⑤

이자 정의 대상이다 의 자연은 자 의 상황과 ⑤

처 를 드러내는 경으 서의 역할을 하 이

이 없다

17) 정답[ ] ③

해설 는 빈천 을 해결하고자 했으나 강산[ ] lsquo ( )rsquo 貧賤③

과 풍 을 달라는 에 거절하 다고 함으 써 자

연에 대한 애정을 드러내고 있으 는 않는

임에 대한 망을 개에게 전가 켜서 임에 대한 리

을 드러내고 있다

18) 정답[ ] ③

년 학 간고사 대비2013 2 현대고 대비

ECN-0102-2013-001-000076193

19) 정답[ ] ⑤

해설 고상한 음악가의 이름을 리말 꽝 럽[ ]

게 꿈으 써 언어유희를 통해 음을 유 하고 있

다 이는 고상한 척하는 총 를 비꼼으 써 비판적

태도를 드러내는 것이 대상을 꽝 럽게 표현

하여 총 의 허 과 사치를 풍자하고 있다

20) 정답[ ] ⑤

해설 는 작품 속 경에 대한 설 이 드러나는 것이[ ]

서 자의 주 적인 견해가 접적으 드러나는 것이

아니다

21) 정답[ ] ⑤

22) 정답[ ] ②

23) 정답[ ] ④

24) 정답[ ] ①

해설 적강 티프는 주인공의 비 한 출생이나 능[ ] ①

과 이 있는 것으 조정의 능함을 풍자하는lsquo rsquo

것과는 거리가 다

25) 정답 픔 나[ ] ( )

해설 의 음악은 고통 는 사람들을 위 하고 아픔[ ] lsquo rsquo

을 치유해 주는 능을 한다고 할 있다 의 lt gt

픔 도 소 된 이 과 더 어 살아가는 따뜻한 마음lsquo rsquo

을 상 한다

26) 정답[ ] ⑤

해설 에게 선천적으 주어 각 장애라는 역경[ ]

은 의 이라는 가사 연 을 있다lsquo rsquo

27) 정답[ ] ④

해설 는 장 란 선 에게 은 개인적인 인상을[ ]

소녀 장정 등으 표현한 것이다lsquo rsquo

28) 정답[ ] ②

해설 담자가 피 담자의 언어적 표현이나 비언어[ ]②

적 표현 하 독자는 담의 위 나 피

담자의 감정 상태를 알 있다 이를 통해 독자는

담 상황을 더 생생하게 느낄 있고 피 담자

를 더 잘 이해할 있게 된다

29) 정답[ ]③

해설 일상생활과 역도 선 서의 성과에 된 것에서[ ]

역도를 하 서 겪는 어 과 내적 고민으 화제를

전화하 위한 것이다

30) 정답[ ] ①

해설 릿속에 새겨 넣듯 이 억되도 함 세상[ ] ② ③

살이가 힘들고 고생 러 속 하여 자유를 ④

가 없는 고통의 상태를 비유적으 이르는 말

적의 침입을 막 위해 쌓은 축 켜야 할⑤

대상을 비유적으 이르는 말이다

31) 정답[ ] ④

해설 이 의 종류는 전 으 인 사건 경[ ] lsquo

비평을 성 소 삼는다rsquo

32) 정답[ ] ④

해설 근은 삼대독자 태어났음을 에서 확인할[ ]

있다 형제들과의 담은 이뤄 가 없다

33) 정답[ ] ⑤

해설 근은 가난에도 하고 화가를 꿈꾸었다[ ] (3

단 또한 다른 화가 망생들은 정 육을)

위해 상 학 학 해 유학 에 랐 만

근은 다른 을 찾아야 했다 단 세에(5 ) 18

근은 조선 전람회에 입선하 다 단 의(6 )

만종은 인간과 자연이 엮어 가는 경건한 조화 을lsquo rsquo

나타낸다

34) 정답[ ] ①

해설 근이 속에서도 창작활동을 추 않고[ ]

하는 닭은 은 세상과 타협할 르는

근이 세상의 이해를 하 위한 가장 떳떳한 단

이 때 이다

35) 정답[ ] ⑤

해설 전 은 서 자의 주 적인 평이 리는 것이[ ]

만 위 제 은 인 이 살았던 대 사회적 경

을 통해 객 적인 인 의 을 제 하고 있다

36) 정답[ ] ⑤

해설 전 은 인 사건 경 비평이라는[ ] lsquo rsquo⑤

성 이 어져 있다

37) 정답[ ] ①

해설 이 은 동양인과 서양인의 사고 에 차이가[ ]

있다는 것을 대조를 통해 설 하고 있다 또 쓴이

의 제자가 축 경 를 러 가서 경험한 일화를

통해 동양인이 서양인에 비해 주 상황에 더 많은

주의를 인다는 주장을 뒷 침하고 있다

38) 정답[ ] ④

39) 정답[ ] ②

40) 정답[ ] ②

41) 정답[ ] ④

42) 정답[ ] ③

43) 정답[ ] ④

44) 정답 도서 의 휴 일 도서 의 이 간 도서의[ ]

해설 도서 장은 임의 정한 휴 일과 도서 이[ ]

간 도서의 상 등을 게 할 의 가 있다

년 학 간고사 대비2013 2 현대고 대비

ECN-0102-2013-001-000076193

45) 정답[ ] ①

해설 제 조의 정 휴 일 의 휴 일의 사전 게[ ] 3

는 도서 장의 의 조항에 속한다

46) 정답[ ] ①

해설 개인 정 호 의 를 제 하 했 만 항[ ]

나눠서 제 하 않고 대 나열하고 있다

47) 정답[ ] ②

해설 제 조의 내 을 회사는 다른 회사 협[ ] 7 lsquo

계약을 통해 서비 를 제공하는 경 회 의 아이디

등 개인 정 를 해당 회사에 전송할 있다는 내rsquo

이 있으 의 제점을 제 할 있다②

48) 정답[ ] ④

해설 는 도서 장의 의 에 해당하고 나 는 도[ ] ④

서 장의 리에 해당한다

49) 정답[ ] ③

50) 정답 은 음독으 적었고 은 훈독으 적었[ ] (1)

다 과 동일한 표 리 적은 것은 이고 (2) ce

과 동일한 표 리 적은 것은 이다ab

51) 정답[ ] ③

52) 정답[ ] ①②

53) 정답[ ] ③

54) 정답[ ] ③

55) 정답[ ] ①

56) 정답 른 죠코 어린 노 하니라[ ] A B

57) 정답 세 어에서는 활 형이 칙적으[ ] lsquo rsquoㄹㅇ

나타났 만 개화 어에서는 활 형이 쓰 다 lsquo rsquo ㄹㄴ

58) 정답 호 가 흔[ ] (1) (2)

59) 정답[ ] ④

60) 정답[ ] ③

Page 9: 현대고대비 국어 - chamsoriedu.com 「콘텐츠산업진흥 법」외 에도 저작권 의하여 ... 다른주체에게어떤동작을하도록만드는것을나타내는

년 학 간고사 대비2013 2 현대고 대비

ECN-0102-2013-001-000076193

게 지 고뿌 지 rdquo

어 어째ldquo rdquo ldquo rdquo

런 도 것들 같 니ldquo ( ) rdquo殘忍無道 helliphellip

는 탱 여 지 못 다 보( ) 憤氣撐天

니 는 는 다 동원 여 통 쳤

생각 여 는 눈 다

달리 리헐 감ldquo rdquo

들 고 말 니었다 그가

는 것 그 말고는 없었 에 그 게 뒷동

달 거 다

는 우 럽고 식 짝 없는 랫것들 고

다 공연 신 가고 득 것

없다고 단 는지 결 웬만큼 고루 어

그 것들 쪽 에다 고 어주지ldquo

고 그 그걸 주 어 에 에 helliphellip

눈 없는 독 들 rdquohelliphellip helliphellip

고 말 럼 얼거리 들어가 리는 것 었

- ( ) -兪子小傳

zb19) 위 나 를 읽고 평가한 것으 적절하( ) 않은 것

① 사 리 통 감과 사실 고

격 과 달 고

② 는 가 재 컫는lsquo rsquo lsquo rsquo

미 가진 여 는 것 겠

③ 는 식 말 는 웃 상lsquo rsquo

여 는 미 지니고

④ 는 어가 죽 짐 지만 내색 지 고lsquo rsquo

말 고

⑤ 언어 통 가들 여 우리 통

것들 역 고

가 체 거리( ) [ ]

나라 종 연간에 정언주 의 을 하고( )正言注簿

있던 유 은 늦도 자 이 없어 한탄하다가 남악 형산lsquo rsquo

에 치성을 드리고 이한 태 을 꾼 뒤 아들을 낳아 이름

을 충 이라 고 키 다 이때 조정의 하들 에 역

을 품은 정한담 최일 등이 가달의 침입에 대한( ) 逆心

유 의 유화적 입장을 제 삼아 유 을 함하여 양

내고 유 의 에 을 러 충 자마저 살해하

한다 러나 충 은 천 조 정한담의 마 에서 어

나 많은 고난을 겪다가 은퇴한 재상 강희주를 만나 사위

가 된다 강희주는 유 을 하 고 상소를 으나 정

한담의 공격을 아 양을 가게 되고 강희주의 가족은

난을 피하여 두 흩어 다 충 은 강 소저 이 하고

사의 노 을 만나 를 때를 다린다 이

때 남적과 적이 를 들고 나라에 쳐들어 자 정한

담은 자 출전하여 남적에게 항 하고 남적의 선 장이

되어 천자를 공격한다 정한담에게 여러 패한 천자가

항 하 할 음 충 이 등장하여 남적의 선 정 걸

을 죽이고 천자를 출한다 충 은 단 으 란 을

쳐 고 정한담을 사 잡는다 리고 호 에게 ( )胡王

잡혀간 황후 태후 태자를 출하 유 에서 고생하

던 아 유 과 장인 강희주를 한다 또한 이 하

던 어 니 아내를 찾고 정한담 일파를 리친 뒤 높은

에 라서 화를 누린다

사 들 별 고 없 다니었다( )

마 마 돌 다니 걸 여 고

어 곤 다 에는 동쪽에 고

에는 쪽에 니 가 에 리는 엽

가는 없 니 늘 다니는 었

다 얼 말 죽 사 같고 림새가 말

니었다 가슴 에 고 등

삼태 헌 에 니 달 ( )奇男子

가 도리어 걸 었 담 만 열 도 ( )傅說

고 만났고 만 갈( ) ( ) 慇 武丁

도 탕 만났( ) ( ) (伊尹 成湯 渭

여상 도 주 만났는) ( ) ( ) ( ) 水 呂尙 周 文王

월 같 러가 도 어느 열 살

늘과 집 삼고 사 에 쳐 거리에

어 다가 곳에 니 다 ( ) 楚

지 다가 사 보고 가에 다다( )長沙

니 망 가에는 원 리가 슬 고 가

가 내리는 사 에는 갈매 가 갈 뿐 었다

쪽 돌 보니 가 우거 고

가 사 보 었다 그곳에

가니 는 사( ) 汨羅水

는 다 주 가 쓰고 죽고

곳 었다

마 감 여 에 가 사 살펴보니

에는 삼 고 그 에( ) 屈三閭

는 만고 월 과 지 가는 그 들( )風月

가 어 었다( ) 路程記

동쪽 벽 에 새 운 어 거늘 그

보니 월 에 경 주 는 간신에게ldquo ( )敗

보고 연경 귀 가다가 에 죽 rdquo

거늘 그 보고 에 거꾸러

통곡 말

[A]ldquo우리 연경 간 만 니 에

지 살 상에 엇 겠는가

에 고 에 었 니

상에 살 것 가 도 께 지리 rdquo

년 학 간고사 대비2013 2 현대고 대비

ECN-0102-2013-001-000076193

고 가에 내 가니 울 리가 에 지 사

쳤는지 심 심 것 가

신 심 것 가

다( ) 강 승상에게는 들 없고 다만 만

었다 가 낳 에 가 색

타고 내 에게 말 는 ldquo

니다 미원 과 연 맺고 ( ) ( )紫薇垣 緣分

었는 께 강 집 보내 에

니 게 여겨 주십시 거늘 rdquo

미 가운 낳 니 가 고 거동

단 다 시 짓 쓰 고 는 (音

없었 니 여 가운 지 는 짝) 律

룰 만 사 없었다 가 사 여 사 감

게 고 지 못 고 염 는 만다

다가 당에 거 고 식같 러 내니

고귀 상 루 말 다 어 울 도 다( ) 相

귀 사 없고 웅 걸( )富貴爵祿

만고 었다 승상 매우 뻐 내당 ( )內堂

들어가 에게 사 니 역시 매우 거

워 말 다

ldquo 도 마 사 는 승상께

그 게 말 시니 상 여러 말 지 말고

사 도 시다rdquo

상이 에 나 충 의 손을 잡고 결혼과 하여 ldquo

너에게 히 할 말이 있다 내가 늙은 말년에 딸

하나만을 두었는데 니 너 하늘이 정해 필

임이 하다 이제 년고락 을 너에게 탁 ( )百年苦樂

하겠다 하 대 충 이 릎을 꿇고 앉아 눈 을 흘리rdquo

여쭈었다

소자의 을 해주 고 또 하 에 두고자 하ldquo ( )膝下

니 감사하 이를 데가 없 니다 다만 가 속에 통탄

할 일이 사 쳐 있 니다 소자가 이 없어 양친 ( )兩親

의 생사를 른 채 결혼하여 아내를 얻는 것은 자 으

서 할 도리가 아닙니다 이것이 한 러 뿐입니다 rdquo

승상 그 말 듣고 슬 에 어 고

것 에 맞 어 변 게 리ldquo

는 다 집 시 공 도 여 ( )始祖公

고 가 에 가가 어진 만 개 공신

었 니 도 러워 말 시고 시 rdquo

택 여 니 다운 신 과 신

습 늘에 죄 짓고 간 상에 내 신

혼 를 다 끝내고 으 들어가 사 을 살펴 니 빛

나고 빛난 것이 한 입으 는 다 말하 어 고 하나

는 다 하 어 더라 에 켠 환한 촛 ( )新房

아래 은 에 랑과 가 평생의 연 을 맺었( )緣分

으니 서 사랑하 주고 은 말을 어떻게 다 헤아릴

있으 어떻게 다 하리 을 낸 후에 이튿날

상 를 니 상 거 마음을 이 하

더라

각 생 강 승상 집 쪽( )

늘 보고 없 가 신 신 생각 니

없고 어 없었다 는 어떻게 도리가 없다

여 산 에 들어가 리 고 어 도 닦

고 다 그 산 보고 가다

가 곳에 다다 니 에 큰 산 었다 많 우

리 골짜 가 늘 는 가운 색

에 고 갖가지 가 짝 어 었 ( )花草

다 신 산 생각 고 들어가니 경개 ( )景槪

가 매우 뛰어 고 경 산 다 산 리에 들

리는 것 리 보 는 것 울 청산뿐

었다 가 고 울 어 가

니 들 많 가지들 못 어 동

에 늘어 들거리 는( ) 洞口

우거진 가지에 갖 들 다 었다( ) 春情

계상 에는 공 는 늘( ) 花溪上

에 걸린 폭포가 벽 는 리는 산사( )層巖絶壁

쇠 리 객 에 는 듯 늘( ) ( ) 寒山寺 客船

에 싸여 는 습 산

그린 여 병 러 듯 다 경쇠 리가 들

리 에 들어가니 색 에

게 단청 누각과 큰 집들 다( ) 丹靑

주 보니( ) ( ) lsquo一柱門 黃金大字

산 사 어 었다 산 들rsquo ( )山門

어가 고승 다 그 거 보니( ) 高僧

눈 눈 듯 고 변 같 ( )白邊

귀는 어 에 늘어 니 맑고 어 골격

과 신 평 니었다 염주

에 걸고 짚고 포 삼에 어진( )六環仗

쓰고 생 보고 말

승 연 여 상공 시는 동 에ldquo

가 맞 지 못 니 승 십시 rdquo

생 크게 말 다

생 가 여 어 고ldquo

없 다니다가 우연 곳에 사 만 것

그 시 생 어떻게 고 습니 rdquo

승 답 여 말

어 산 승 에ldquo ( ) ( )南岳 衡山

시어 승에게 탁 내 낮 시경에 경 lsquo 12

동 에 사는 심 들 가 것 니 내쫓

지 말고 습니다 마 승rsquo

다가 상공 림새 보니 경 사 에 보

습니다rdquo

zb20) 위 의 친 에서 서 자의 개입이 드러나~

는 이 아닌 것은

① 달 가 도리어 걸 었( ) 奇男子

② 신 심 것 가

년 학 간고사 대비2013 2 현대고 대비

ECN-0102-2013-001-000076193

③ 다운 신 과 신 습 늘에 죄 짓고

간 상에 내 신 다

④ 사 주고 말 어떻게 다 헤 릴

어떻게 다 리

⑤ 신 산 생각 고 들어가니 경개 가 ( )景槪

매우 뛰어 고 경 산 다

거리 연[ ] ( )弘治

간 에 공신 후 에 언(1488~1505) ( )正言

주 는 벼슬 심 늦도 식( ) ( )主簿 劉尋

없어 과 께 산에 드리고 신 태

몽 꾼 에 만고 웅 상 지닌 들 낳

키운다 그 후 신 들 에 역심( )逆心

담 귀 등 심 여 리 귀 보내

고 지 죽 는 도망 가다가

만 다 에 에 어 니

헤어지게 다

에 에 어 니 헤어지게

다 그 후 사 들에게 우연 돌

생 다가 어느 열 살 었다 열 살

지 다가 우연 귀 견 는

그것 그 살 도 었고 그

귀 본 신도 지 죽고 마 고

크게 운다

( )

에는 강 주 는 재상 살고 었

니 시 에 과거에 격 여 승상 벼슬 다가 간

신 만 벼슬 그만 고 고 돌 었

다 그러 신 지 가 지 못 여 상

가 못 결 는 상 여 원 니

신 들 그 직간 꺼 다 그 에 도

담과 귀가 강 승상 가 미워 다

강 승상 마 본 에 갔다가 돌 는[A][ ( )本府

에 우편 주 에 다가 색( ) ( ) 右便 酒店

에 어리었는 청룡 에 지 늘

여 통곡 고 사 는 꿈 꾸었다] 마

상 게 생각 여 새 다리다가 새벽

닭 울고 가 달 갔다 가 보니

과연 어 동 가 가에 울고 는지 달

들어 그 고 사 에 어 말

는 어 어 에 어 가ldquo

닭 곳에 우느냐 니 울rdquo

그 고 답 여 말 다

는 경 동 에 사는 언 주 공 들ldquo

니다 께 간신 만 연경 귀 가

시다가 에 죽 사 에 는 닭에

도 에 죽고 니다rdquo

강 승상 말 듣고 크게 낯 변 말

것 웬 말 냐 근 동 ldquo (老

못 갔 니 그 사 변 여)患

런 변 었단 말 가 주 는 신 다

같 에 벼슬 다가 는 가 많 들어 고

돌 는 주 가 게 꿈 에 생

각 겠느냐 생각지 못 다 미 지 간

지지 말고 께 가 략rdquo ( ) hellip hellip

죽게 주 사당에 단 도 러운

겠느냐 말 말고 시는지 rdquo

어 없어 강 승상 가니 그곳

월계 었다

다( )

가가 고 지 사 들 가( )櫛比

통 는 리가 과

답게 꾸민 누각과 큰 집들 늘 고

게 식 가 어 들 태운 가고

었다 략 강 승상에게는 들 없고 ( ) hellip hellip

다만 만 었다 가 낳 에

가 색 타고 내 에게 말

는 니다 미원 과ldquo ( )紫薇垣

연 맺고 었는 께 강 집( )緣分

보내 에 니 게 여겨 주십시

rdquo

거늘 미 가운 낳 니 가

고 거동 단 다 시 짓 쓰 고

는 없었 니( )音律 여 가운

지 는 짝 룰 만 사 없었다 가 사

여 사 감 게 고 지 못 고 염 는 만다

다가 당에 거 고 식같

러 내니 고귀 상 루 말 다 ( )相

어 울 도 다 귀 사 없 ( )富貴爵祿

고 웅 걸 만고 었다 승상 매우 뻐

내당 들어가 에게 사 니( ) 內堂

역시 매우 거워 말 다 도 마 ldquo

사 는 승상께 그 게 말 시니

상 여러 말 지 말고 사 도 시다rdquo

( )

승상 에 고 결 과 ldquo

여 에게 말 다 내가 늙 말 에 지

만 었는 지 보니 늘

다 에게 탁 겠 ( )

다 신 꿇고 눈 리rdquo

여 었다 주시고 슬 에 ldquo ( )膝下

고 시니 감사 룰 가 없습니다 다만 가슴

에 통탄 사 쳐 습니다 복 없어

생사 결 여 내 얻는 것( )兩親

식 도리가 닙니다 것 러울 뿐 니

다rdquo

상 그 말 듣고 슬 에 어 고 말

것 에 맞 어 웅변 ldquo

년 학 간고사 대비2013 2 현대고 대비

ECN-0102-2013-001-000076193

게 리 는 다 집 시 공도 여

고 가 에 가가 어진 만 개 공신

었 니 도 러워 마 시고 rdquo 시

택 여 니 운 신 과 신

습 늘에 죄 짓고 간 상에 내 신

다 략 지낸 후에 튿 승상 ( ) hellip hellip

니 승상 거운 마 지 못

마( )

듯 월 러 생 열다 살 었

다 에 승상 어진 사 얻고 만 에 근심 없었

다만 주 가 간신 에

죽 것 생각 마 곧 어 곤

다 그 에 주 원통 어

없 고 여 시 가 거늘 생 만

여 다

말 감격 러우 간신 에 가득 여ldquo

고 니 께 상 듣지 니 것

니다rdquo

승상 듣지 고 가

퇴 재상 공달 집에 거 고 상 지어

승지 러 께 리

( )

뒷 거리 강 승상 에게 상 리지[ ]

만 여움 사 귀 가게 다 강 승상

몸 는 연 가 헤어

리 다 산 들어간 룡사 승 만

게 다 승 만 우 다릴

과 들고 략 다 담

원 여 에게 복 고 어

공격 다 담에게 여러 가( ) 天子

복 등 여 다 단

신 리쳐 담 사 고 에게

간 후 태후 태 여 지에 고생

지 심과 강 주 여 개 다 헤

어 어 니 내 고 담 리

벼슬에 귀 누리게 다

zb21) 위 의 인 간 계를 같이 나타냈을lt gt

때 에 대한 이해 가장 적절하 ~ 않은 것은

① 계에 주 는 계 심 열

상 에 다고 다( ) 水深火熱

② 계는 견원지간 고 다( ) 犬猿之間

③ 계는 달리 막역지 계 고( )莫逆之交

④ 연결 사 컬어 재 가 고( )才子佳人

⑤ 는 생 과 볼 ( )匹夫匹婦

가 재 는 는 심 고 매사에 생( )

각 고 능 도 어 가 에게 많lsquo rsquo

도움 사 다 그는 에게 거 에

꺼리 없 거 났다고 는

매우 싫어 고 신 들

는 사 다

내가 지 리에( ) 1970

사 실에 지 월간ldquo

편집 고 어 었다rdquo

어느 없 가 쑥 다 도 어 10

후 다 산 시 럼 어 엇 어 ( ) lsquo怡山

다시 만 랴 니 그는 재 그룹 승 운rsquo

사가 고 는 고 거 누

주는 가 없는 가가 어 다시 만 게 것

었다

다 보통 것 닐러 그( ) ldquo 어낸 ( )

틀어주 가 루 러 허 에

싶어 키 틀어주 그( )

가 루 허 우간 곡 틀어 주는 루 못

는 는 고 닝께 고 지

들어 사는 고 가 다는 건 에 그 집에

rdquo

그런 단 어들 어 새벽에 떼죽 거

다 고 어 보니 죄다 허 게 집어진

는 것 었다 가 실내 꿴 뛰어 지만

없는 었다

어떻게 된 거야 한동안 넋나간 듯이 서 있던 총ldquo rdquo

가 하고많은 사람 에 하필이 유자를 겨냥하 은

말이었다 쎄유 아마 새에 고뿔이 들었던 개비네 ldquo

유rdquo

유자는 러 딴청을 하 다 야 고 가 에서 ldquo

감 가 들어 죽는 고 두 어rdquo 총 는 가 혐의

자 나 되는 것처럼 화풀이를 하 드는 것이었다( )嫌疑者

라 이 어쩌 어 유( ) ldquo rdquo ldquo rdquo

애유 이런 잔인 도 한 것들 같으니ldquo ( ) rdquo殘忍無道 helliphellip

총 는 탱천 하여 쩌 를 하 다( ) 憤氣撐天

아하니 아는 자는 다 동 하여 호통을 쳤으 하나 혈

압을 생각하여 참는 눈치 다 달리 처리헐 두 ldquo

잖은감유rdquo

총 의 성 을 덧들이 고 한 말이 아니었다 가 할

년 학 간고사 대비2013 2 현대고 대비

ECN-0102-2013-001-000076193

있는 것이 말고는 없었 때 에 게 뒷동

산을 달은 거 다

이 유자소전- lsquo rsquo

zb22) 의 상황을 속담으 표현한 것으 적절한 것은

① 루 곳 게 마 다

② 에 맞고 강에 눈 다

③ 늘 도 다

④ 도 사 다

⑤ 에 가도 신만 리 다

거리 공신 후[ ]

에 주 는 벼슬 심 늦도( )主簿

식 없어 과 께 산에 드리고 신

태몽 꾼 에 만고 웅 상 지닌 들

낳 키운다 그 후 신 들 에 역심

담 귀 등 심 여 리 귀 보내고

지 죽 는 도망 간다 그

만 고 에 에 어 니

헤어지게 다 지 가 사 들에

사 들 별 고 없 다니었다

마 마 돌 다니 걸 여 고

어 곤 다 에는 동쪽에 고 에

는 쪽에 니 가 에 리는 엽 가는

없 니 늘 다니는 었다

얼 말 죽 사 같고 림새가 말 니었

다 가슴 에 고 등 삼태

헌 에 니 달 가 도리 ( )奇男子

어 걸 었 담 만 열 도 ( ) ( )傅說 慇

고 만났고 만 갈( ) ( )武丁 伊尹

도 탕 만났( ) ( )成湯 渭水

여상 도 주 만났는 월( ) ( ) ( ) 呂尙 周 文王

같 러가 도 어느 열 살

늘과 집 삼고 사 에 쳐 거리에

어 다가 곳에 니 다 ( ) 楚

지 다가 사 보고 가에 다다( )長沙

니 망 가에는 원 리가 슬 고 가

가 내리는 사 에는 갈매 가 갈 뿐 었다

쪽 돌 보니 가 우거 고

가 사 보 었다 그곳에

가니 는 사( ) 汨羅水

는 다 주 가 쓰고 죽고

곳 었다

마 감 여 에 가 사 살펴보니

에는 삼 고 그 에( ) 屈三閭

는 만고 월 과 지 가는 그 들( )風月

가 어 었다( ) 路程記

동쪽 벽 에 새 운 어 거늘 그

보니

월 에 경 주 는 간신에게ldquo ( )敗

보고 연경 귀 가다가 에 죽 rdquo

거늘 그 보고 에 거꾸러

통곡 말

우리 연경 간 만 니ldquo ( )燕京

에 지 살 상에 엇 겠는

가 에 고 에 었 니

상에 살 것 가 도 께 지리 rdquo

고 가에 내 가니 울 리가 에 지

사 쳤는지 심 심 것 가

에는 강 주 는 재상 살고 었

니 시 에 과거에 격 여 승상 벼슬 다가 간

신 만 벼슬 그만 고 고 돌 었

다 그러 신 지 가 지 못 여 상

가 못 결 는 상 여 원 니

신 들 그 직간 꺼 다 그 에 도

담과 귀가 강 승상 가 미워 다 강 승상 마

본 에 갔다가 돌 는 에 우편 주( ) ( )本府 右便

에 다가 색 에 어리었는 청룡( ) 酒店

에 지 늘 여 통곡 고

사 는 꿈 꾸었다 마 상 게 생

각 여 새 다리다가 새벽닭 울고

가 달 갔다 가 보니 과연 어 동 가

가에 울고 는지 달 들어 그

고 사 에 어 말

는 어 어 에 어 가ldquo

닭 곳에 우느냐rdquo

니 울 그 고 답 여 말 다

는 경 동 에 사는 언 주 공 들ldquo

니다 께 간신 만 연경 귀 가

시다가 에 죽 사 에 는 닭에

도 에 죽고 니다rdquo

강 승상 말 듣고 크게 낯 변 말

것 웬 말 냐 근 동ldquo ( )老患

못 갔 니 그 사 변 여 런 변

었단 말 가 주 는 신 다 같

에 벼슬 다가 는 가 많 들어 고 돌

는 주 가 게 꿈 에 생각

겠느냐 생각지 못 다 미 지 간 지지

말고 께 가 rdquo

뒷 거리 강 승상 도움 죽 고[ ]

년 학 간고사 대비2013 2 현대고 대비

ECN-0102-2013-001-000076193

고 그 과 결 여 사 가 다 그러 강

승상 에게 울린 상 강 승상 귀 가고

과 헤어 리 승 만 게 다

승 우 다릴 과

들고 략 다 담 원

여 에게 복 고 어 (天

공격 다 담에게 여러 가 복) 子

등 여 다 단신

리쳐 담 사 고 에게 간

후 태후 태 여 지에 고생 지

심과 강 주 여 개 다 헤어

어 니 내 고 담 리 벼

슬에 귀 누리게 다

미상- lsquo ( )-劉忠烈傳

zb23) 위 과 의 서사 조를 비 한 것으 적절하lt gt

않은 것은

보lt gt

믿지 고 결 여 곱

낳 다 곱째 공주 낳 가

리게 다 리 만 고 진 공주는 lsquo rsquo

리공 미 리공 에 키워진다 월

러 과 가 죽 병에 걸 는 승에 는

어 산다고 다 여 들에게 탁

지만 거 리 는다 리 는 과

승 다 승 지 가는 에 많

만 지만 보살 도움 사 도 다

그러 승 신과 결 여 시

들어 주겠다고 다 리 는 그 결

여 들 곱 낳 후에 신

얻게 다 돌 리 는

에 과 상여 만 지만 여 과

살 낸다 훗 리 그 공 우 죽 사

승 도 는 신 다

리-lsquo rsquo-

① 복 결말에 고 다

② 웅 에 탕 고 다

③ 시 겨 내고 귀 누리는lsquo rsquo

보 리 는 월 재 신 다lt gt lsquo rsquo

④ 과 보 리 는lsquo rsquo lt gt lsquo rsquo

도움과 어 신 능 극복 고

⑤ 등 여 시 겪는lsquo rsquo

보 리 는 닌 지lt gt lsquo rsquo

림 시 겪는다

가 각 고 에( ) ( )却說

살 없었다 략 사 들 슬 에 어 lt gt

가에 내 고 가고 싶 가 고 후

워 경

사 들 별 고 없 다니었다 lt

략 얼 말 죽 사 같고 림새가 말gt

니었다 가슴 에 고 등

삼태 헌 에 니 달 가 ( )奇男子

도리어 걸 었 담 만 열 도( )傅說

고 만났고 만 갈( ) ( ) 殷 武丁

도 탕 만났( ) ( ) (伊尹 成湯 渭

여상 도 주 만났는) ( ) ( ) ( )水 呂尙 周 文王

월 같 러가 도 어느 열 살

늘과 집 삼고 사 에 쳐 거리에

어 다가 곳에 니 다 ( ) 楚

지 다가 사 보고 가에 다다( )長沙

니 망 가에는 원 리가 슬 고 가

가 내리는 사 에는 갈매 가 갈 뿐 었다

쪽 돌 보니 가 우거 고

가 사 보 었다 그곳에

가니 는 사( ) 汨羅水

는 다 주 가 쓰고 죽고

곳 었다

에는 강 주 는 재상 살고( )

었 니 시 에 과거에 격 여 승상 벼슬 다

가 간신 만 벼슬 그만 고 고 돌

었다 략 강 승상 마 본 에 갔다가 돌 lt gt ( )本府

는 에 우편 주 에 다가 색( ) ( ) 右便 酒店

에 어리었는 청룡 에 지

늘 여 통곡 고 사 는 꿈 꾸

었다 마 상 게 생각 여 새 다리다

가 새벽닭 울고 달 갔다 가

보니 과연 어 동 가 가에 울고 는지

달 들어 그 고 사 에

어 말

는 어 어 에 어 가ldquo

닭 곳에 우느냐rdquo

니 울 그 고 답 여 말 다 lt

략gt

년 학 간고사 대비2013 2 현대고 대비

ECN-0102-2013-001-000076193

생각 여 가 고 시 는ldquo ( )大人

상에 다시없는 니다 살 엇 겠습니

에 돌 가시고

가에 돌 가 니 살 마 없습니

다 략 어 없어 강 승상 가니rdquo lt gt

그곳 월계 었다

다 강 승상에게는 들 없고 다만 만( )

었다 가 낳 에 가 색

타고 내 에게 말

는 니다 미원 과ldquo ( )紫微垣

연 맺고 었는 께 강 집( )緣分

보내 에 니 게 여겨 주십시

rdquo

거늘 미 가운 낳 니 가

고 거동 단 다 시 짓 쓰 고

는 없었 니 여 가운( ) 音律

지 는 짝 룰 만 사 없었다 가 사

여 사 감 게 고 지 못 고 염 는 만다

다가 당에 거 고 식같 러

내니 고귀 상 루 말 다 어 ( )相

울 도 다 귀 사 없고 ( )富貴爵祿

웅 걸 만고 었다 승상 매우 뻐 내

당 들어가 에게 사 니 역( ) 內堂

시 매우 거워 말 다

도 마 사 는 승상께ldquo

그 게 말 시니 상 여러 말 지 말고 사

도 시다 략 시 택 여rdquo lt gt

니 다운 신 과 신 습 늘에 죄

짓고 간 상에 내 신 다

다 내고 들어가 사 살펴보니

고 것 는 다 말 어 고

는 다 어 신 에 ( )新房

에 신 과 신 가 평생 연 맺었( )緣分

니 사 주고 말 어떻게 다 헤 릴

어떻게 다 리 지낸 후에 튿 승

상 니 승상 거운 마 지 못

( ) 듯 월 러 생 열다 살

었다 에 승상 어진 사 얻고 만 에 근심

없었 다만 주 가 간신

에 죽 것 생각 마 곧 어

곤 다 그 에 주 원통

어 없 고 여 시 가 거늘 략 lt gt

략 거리

강 승상 에게 상 리지만 여움

사 귀 가게 다 강 승상 몸 는

연 과 헤어 리 다

마 각 생 강 승상 집 쪽( )

늘 보고 없 가 신 신 생각 니

없고 어 없었다 는 어떻게 도리가 없다

여 산 에 들어가 리 고 어 도 닦

고 다 그 산 보고 가

다가 곳에 다다 니 에 큰 산 었다 많

우리 골짜 가 늘 는 가운 색

에 고 갖가지 가 짝 어 ( )花草

었다 략 주 보니 lt gt ( ) (一柱門 黃

산 룡사 어 었다) lsquo rsquo 金大字

산 들어가 고승 다 그( ) ( ) 山門 高僧

거동 보니 눈 눈 듯 고

변 같 귀는 어 에 늘어 니( ) 白邊

맑고 어 골격과 신 평 니었

다 염주 에 걸고 짚고 포 ( )六環杖

삼에 어진 쓰고 생 보고 말

승 연 여 상공 시는 동 에ldquo

가 맞 지 못 니 승 십시 rdquo

생 크게 말 다

생 가 여 어 고ldquo

없 다니다가 우연 곳에 사 만 것

그 시 생 어떻게 고 습니

rdquo

승 답 여 말

어 산 승 에ldquo ( ) ( )南岳 衡山

시어 승에게 탁 내 낮 시경에 경 lsquo 12

동 에 사는 심 들 가 것 니 내쫓

지 말고 습니다 마 승rsquo

다가 상공 림새 보니 경 사 에 보

습니다rdquo

생 그 말 듣고 편 고 편( )

슬 승 들어가니 여러 승 들

가워 다 승 에 들어가

후에 그 편 니 곳 경 었다 상( ) 仙境

고 신 편 다 후 는 승과

께 병 도 탐 고 경도 게( )兵書

게 었다 게 니 지 에 가객 ( ) ( )大明天地 佳客

년 학 간고사 대비2013 2 현대고 대비

ECN-0102-2013-001-000076193

없고 산 에 리 만 본 ( ) 廣德山

신 상 사 살 는 만

우고 늘 월 신 과 늘 ( )日月聖神

산 신 들 다 니 그 재( ) 名山神靈

주 민 누가 당 겠는가 낮 공

zb24) 다 에 해당하는 내 으 적절하( ) 않은 것은

① 강 티 통 당시 능 다

② 상계 지상계 경 는 원 계 드러

③ 실에 어 없는 실 가 타 는

④ 뛰어 재주 어 가진 고

등 다

⑤ 가 직 개 여 평가 내리는

편집 평 타 다lsquo rsquo

가 본격 가 동 것 지( )

다 단 상 에2003 lsquo rsquo

들어가 드럼 연주 다 취미 생 달리

들었다는 보 우 가 들ldquo

어 틱 린 도 다 고 말 다rdquo

경 는 가 망 없( ) lsquo

티 원 고 답 다 신과 같 시각rsquo

는 습 상상 만 도 감동

다 시각 연주 동시에

열 상 는

티 원 그런 열 경 럽다는 것 다

다 역시 엄청 다 본( )

에 복 들

고쳐 가고 다 신 에 얼

마 지는 고 리가 는 지도 생님

가 훈 고 많 고쳐 다

고 말 다

그러 직도 에 지 는 다 그는

체격 지 못 게 가 큰 만

체 운동 훈 과 께 체 늘 동 50

는 게 고 말 다

에게는 꿈 다 통 누 가( )

주겠다는 것 그 꿈 다 신 극복 는

과 에 큰 경험 들도 느 게

주고 싶다는 것 다

마 슬 마다( ) ldquo 통

낼 었 것 럼 고통 는 사 들

고 겠다 고rdquo

말 다 달 루 첫 낸 lsquo rsquo

첫 드 심 집에 는 리듬 드 2

루 에 도 보고 싶다 집 에는 직(RampB) 3 4

사 곡 도 보 고 싶다고 포 다middot

zb25) 에서 가장 유사한 의 를 닌 어를lt gt

찾아 쓰

lt gt

나는 이제 너에게도 픔을 주겠다

사랑 다 소 한 픔을 주겠다

겨 거리에서 개 놓고

살아 추위 떨고 있는 할 니에게

값을 으 서 뻐하던 너를 위하여

나는 픔의 평등한 얼 을 여 주겠다

내가 어둠 속에서 너를 를 때

단 한 도 평등하게 어주 않은

가마니에 덮인 동사자가

다 얼어 죽을 때

가마니 한 장조차 덮어주 않은

한 너의 사랑을 위해

흘릴 르는 너의 눈 을 위해

나는 너에게 이제 너에게도 다림을 주겠다

지 울 포동 여고 생들17

틈 없 가득 체 에 맑 울

다 죽 듣 생들 사 에

연 는 탄 다 객들 도 는lsquo rsquo

가 보 주 공 맹 가 운 는

단 그룹사운드 루 보컬 맡고 는lsquo rsquo

시각 지 었다17 1

근 다만과 가 거lsquo rsquo lsquo

꿈 고 퇴 내가 다rsquo

간 간에 지 지 연 생들 짧lsquo rsquo lsquo rsquo

가 운 듯 리에 어

연 다 내 사 고 퇴lsquo rsquo

과 루 들 결 다시 돌lsquo rsquo

들 고 사 들 에 당당

것 니다 내 태어

볼 없었 크고 열여

년 학 간고사 대비2013 2 현대고 대비

ECN-0102-2013-001-000076193

에도 고 시 얻지 못 다

감지 없는 시각 상태 다

신 지에 고 상 원망 도

단다 어느 가 에 시각 에 ldquo

어 그런 듣고 다 보니 내가 게 lsquo

살 는지 도 눈 고 싶rsquo lsquohelliphellip

보 는 생각만 들 고 그 가 들에게rsquo

도 내고 들도 고 많 었죠 들 rdquo

었 지 새 는 에 쑥 러운 색

어났다

생에 것 단연 었다lsquo rsquo

공연에 거 꿈lsquo rsquo

는 다 특 가사 갑게 는 운 lsquo

벽 에 당당 마주 어 언 가 그 벽

고 늘 어 거운 상도

없죠 내 삶 에 웃 그 께

는 다고 다rsquo

들었 그냥 런 도 고만 여ldquo lsquo rsquo

겼죠 그런 꾸 가사 미 새 다 보

니 통 는 가사 는 생각 들 고 (

가 게는 시각 는 생각 들고 들) ( )

마다 듣고 큰 얻었어 rdquo

에 진지 게 가에 미 가

zb26) 의 에 들어갈 말 적절한 것은lt gt ~

lt gt

난 난 꿈이 있었죠

고 찢겨 남 하여도

내 가 히 과 같이 간 했던 꿈

혹 때 누 가가 뜻 를 비 음

내 등 뒤에 흘릴 때도

난 참아야 했죠 참을 있었죠

날을 위해

늘 걱정하듯 말하죠

헛된 꿈은 독이라고

세상은 끝이 정해 책처럼

이 돌이킬 없는

현 이라고 helliphellip

래 난 난 꿈이 있어

꿈을 믿어

나를 켜

저 차갑게 서 있는 이란 앞에

당당히 마주칠 있어

출처 가 거위의 꿈 작사 이적 작곡 동률- lsquo rsquo ( )

① ② ③ ④ ⑤

가 떴다는 들 만 지만( ) lsquo rsquo

늘 겸 다 에 주 연 우승 지 간에도 3

단 생님께 만 지 고 고 만ldquo rdquo

큼 늘 겸 신 계 가

고 다

에게는 꿈 다 통 누 가

주겠다는 것 그 꿈 다 신 극복 는 과

에 큰 경험 들도 느 게 주

고 싶다는 것 다

슬 마다 통 낼ldquo

었 것 럼 고통 는 사 들

고 겠다 고rdquo

말 다 달 루 첫 낸lsquo rsquo

첫 드 심 집에 는 리듬 2

루 에 도 보고 싶다(RampB) 집 에는 직34

사 곡 도 보 고 싶다고 포 다

미 는( ) (26) 어 헤헤헤 웃다가 어ldquo rdquo

허허허 웃었다ldquo rdquo ldquo rdquo 같 도 고

상 다 는 같 도 다( ) 壯丁 킹 들lsquo

다 는 역도 보 그 다 지만 그는rsquo

뷰에 지 다 운동만 지 ldquo

것 지 간에 여러 사 도 역rdquo helliphellip

었다 그런 엇 그 마 움직 는지 보 쯤

지 담 사 다 훈 없어 그는 티

지 림 었다 태 다 갔다 는 습

마 집 럼 편 게 보 다

주말에는 주 엇 보내

주말에도 별 주 에 청ldquo

고 에 가고 도 쳐

에 듣고 보 에 갈 가 별 없

어 산 시 게 고 들어 2002

거 매 여 지냅니다 시 과 지훈 rdquo

다 근 간 과 진실 그리고 싶어( )

가 다 근에게 그것 진리 다 거 다 없

거 고 다 없 는 것 진리

다 근 진리는 후 쪽 었다 신산( )辛酸 삶

었 질곡( )桎梏 역사 에 지냈 가

눈에 든 것 료 단 료 게 보

것 었다 그것 그 에 겨우겨우

슬 슬 생 어가는 간들 었다

리 과 단 리 고리에 검 마

없 거리 돌

상 것 없는 등 근에게 상

과 진실 엄 ( )儼存 다는 사실 리는 가

실 고 가 과 역경 에 도 근 내 포

없었 후 보루( )堡壘 다 도 365

도 간 근 여

시 것 다

년 학 간고사 대비2013 2 현대고 대비

ECN-0102-2013-001-000076193

다 공주 그림 가 근 경- ( ) ldquo rdquo(

2009)

zb27) 작가의 주 적인 각이 드러난 것은~

① ② ③ ④ ⑤

가 신 지에 고 상 원망( )

도 단다 어느 가 에 시각 에 ldquo

어 그런 듣고 다 보니 내가 lsquo

게 살 는지 도 눈 고 싶rsquo lsquohelliphellip

보 는 생각만 들 고 그 가 들에게rsquo

도 내고 들도 고 많 었죠 들었rdquo

지 새 는 에 쑥쓰러운 색

어났다 략 [ ]

경 는 가 망 없 티lsquo

원 고 답 다 신과 같 시각rsquo

는 습 상상 만 도 감동

다 시각 연주 동시에

열 상 는 티

원 그런 열 경 럽다는 것 다 략 [ ]

슬 마다 통 낼ldquo

었 것 럼 고통 는 사 들

고 겠다 고rdquo

말 다 달 루 첫 낸 lsquo rsquo

첫 드 심 집에 는 리듬 2

루 에 도 보고 싶다 집 에는 직(RampB) 3 4

사 곡 도 보 고 싶다고 포 다

식 누 가-

고 싶어

다 역도 미 담 고 사( )

질 주말에는 주 엇 보내[ 1]

답 주말에도 별 주 에[ ] ldquo

청 고 에 가고 도 쳐

에 듣고 보 에 갈 가 별

없어 rdquo

질 계 고 슬슬 도 는 것 닙니[ 2]

답 다 들 눈 에 보 고 뿐 보[ ] ldquo

다 열심 고 어 상에 도 들지만 상

지키는 것 들다고 에 도달

그것 지키 훨 많 rdquo

질 들 살 고 리 는[ 3]

거운 들 체 리느 는다

답 가 고 게 체 어[ ] ldquo ( ) 級

느 도 계가 니 살 는 것도 고역 지만

살 우는 것 들어 는 살

체 리 고 어도 어도 실 갔다

쑥 어 rdquo

질 거리에 슷 연 여 들[ 4]

보는 간 상 지

답 상 다 체 게 리지 못[ ] ldquo

거 주변에 는 그 거 누 보지

못 고 뻐지고 싶 에 체 리는 에

타 워 지만 는 어울 는 것보다 는

시간 운동만 는 건 니에 사복 lsquo rsquo

고 사복 는 말에 들 웃지만 늘 운동복

고 지내니 사러 갈 도 어 rdquo

질 역도가 말 단 식 운동 니[ 5]

답 가 내는 만 클 업 보[ ] ldquo

그러니 만 쓰는 식 운동 니다

만 다고 거운 것 들 는 건 니거든 연

도 고 가지 동 에 도 여러 가지

복 들

보식 역도 여 미-

zb28) 가 에 대한 설 으( ) 않은 것은

① 시각 우 지 시 에 지

고 망 가는 태도 달 고 다

② 언어 과 언어 복 사 여

담 내 생각 게 는 가

③ 직 감 그 마 것

럼 생생 게 느껴지는 과 주고 간 내

없 리 어 억 게 다

④ 담 내 식 리 여 담 삶 습

과 가 시 여 독 에게 감동과 훈 다

⑤ 직 진 담 직 누

지 못 는 독 에게 생생 상 달 주고

담 욱 게 다

zb29) 나 의 각 의 의도를 설 한 것으 적절하( ) 않

년 학 간고사 대비2013 2 현대고 대비

ECN-0102-2013-001-000076193

은 것은

① 질 담 상 보여 주 것 다1

② 질 담 과 그에 삶 태도 보여2

주 것 다

③ 질 역도 겪는 어 움에 역도3

과 것 다

④ 질 같 연 여 갖는 고민 는지 말4

주 는 것 다

⑤ 질 역도가 과 고 운동 는 것5

담 가 말 주 는 것 다

가 만진 것 다( ) 3

감 달 다고 다 억 에( ) 音感

지워 지만 당시 청 탁 리도

다고 다 드럼 웠다 4

에 갈 마다 드럼 는 리가 신 게 들

다고 다 눈 볼 가 없 니 엔ldquo

는 는 님 틱 에 여 주

다 드럼과 연 맺 과 들 주었다rdquo

식 누 가-

고 싶어

역( ) 도가 말 단 식 운동 니

가 내는 만 클 업에 보ldquo

그러니 만 쓰는 식 운동 니다 만

다고 거운 것 들 는 건 니거든 연

도 고 가지 동 에 도 여러 가지 복

들 시 는 상 상

드는 상 에 맞춰 실 에 는 여러

펼쳐집니다rdquo

략( )

늘 에 는 어 만 것 같

가 에 사 고 사 사ldquo

겠어 든 에 가 경 만 고

울 는 사 겠어 rdquo

보식 역도 여 미-

다 가 운 는 어 어( ) ldquo rdquohelliphellip

월 새벽 시 태 없 거웠고1965 5 6 1

는 없 그 병원에 퇴원 집

가는 마지막 마 고 마 내 거 다

가 죽 간신 에 실 다 사는 어느5 lsquo

가 죽 는 말 가 식 다 신rsquo

상에 각 시키는( )刻印 에 실

어느 가는 후 민 가가 근 었다lsquo rsquo

는 간 과 진실 그 다는( ) ldquo

에 단 평 견 가지고 다 내

가 그리는 간상 단 고 다 지 다 는 그들

가 에 는 평 지 니 그리고 어린

들 미지 겨 그린다rdquo

마 근 간 과 진실 그리고 싶어( )

가 다 근에게 그것 진리 다 거 다 없

거 고 다 없 는 것 진리

다 근 진리는 후 쪽 었다 신산(辛酸 삶)

었 질곡(桎梏 역사 에 지냈)

가 눈에 든 것 료 단 료 게

보 것 었다 그것 그 에 겨우겨우

슬 슬 생 어가는 간들 었

다 리 과 단 리 고리에 검

마 없 거리 돌

상 것 없는 등 근에게 상에

과 진실 엄 다는 사실 리는 가( )儼存

실 고 가 과 역경 에 도 근 내

포 없었 후 보루(堡壘 다 도)

도 간 근365

여 시 것 다

월 강원도 림리에( ) 1914 2 21

삼 독 태어났다 어 근 복

그것 그리 가지 못 다 근 곱 살

지는 산 산업에 실 고 답마 에 내

갔다 근 그림 럼 쫓 다니 가 시 것

다 상 진 것도 가 었다

러 가 에도 고 근 가 꿈꾸었다 근

가 꿈꾸게 것 보통 업

원색도1926 만lsquo rsquo 었다

공주 그림 가 근 경-

zb30) 에 대한 설 가장 른 것은~

① 역도가 과 운동 도 질

② 리는 는 다 lsquo rsquo

③ 들었지만 그럭 럭 는 다 lsquo rsquo

④ 가 게 보 시 말 다

⑤ 보 병 는 지 상 lsquo rsquo

는 말 다

년 학 간고사 대비2013 2 현대고 대비

ECN-0102-2013-001-000076193

시간 많지 다 청량리 생 병원

마지막 상 경 릿 게 들어 다 그 는 십

만 큰 가 상 말 다

지 못 들 마 갈 고 돗

도시민들 싹 싹 탔다 가 시

월에 병원에 원 가 폐 진 몸도4 ( )疲弊

갈 미 지 못 고 었다 가는 얼마( ) 解渴

지 생 에 생각 가

마감 는 신 평생 십 만에

가 과 많 닮 다고 생각 지는

가 운 는 어 어ldquo rdquo 1965helliphellip

월 새벽 시 태 없 거웠고 는5 6 1

없 그 병원에 퇴원 집 가

는 마지막 마 고 마 내 거 다 가

죽 간신 에 실 다 사는 어느 가5 lsquo

죽 는 말 가 식 다 신rsquo

상에 각 시키는 에 실 어느( ) lsquo刻印

가는 후 민 가가 근 었다rsquo

ldquo 는 간 과 진실 그 다는 에

단 평 견 가지고 다 내가 그

리는 간상 단 고 다 지 다 는 그들 가

에 는 평 지 니 그리고 어린 들

미지 겨 그린다rdquo

근 간 과 진실 그리고 싶어 가

다 근에게 그것 진리 다 거 다 없 거

고 다 없 는 것 진리다

근 진리는 후 쪽 었다 신산 삶 ( )辛酸

었 질곡 역사 에 지냈 가 눈에( )桎梏

든 것 료 단 료 게 보 것

었다 그것 그 에 겨우겨우 슬

슬 생 어가는 간들 었다 리

과 단 리 고리에 검 마

없 거리 돌 상

것 없는 등 근에게 상에 과 진실

엄 다는 사실 리는 가 실( )儼存

고 가 과 역경 에 도 근 내 포 없었

후 보루 다 도 도( ) 365堡壘

간 근 여 시 것

간에 지닌 가 근 1914 2

월 강원도 림리에 삼 독21

태어났다 어 근 복 그것 그리

가지 못 다 근 곱 살 지는 산

사업에 실 고 답마 에 내 갔다 근

그림 럼 쫓 다니 가 시 것 다 상

진 것도 가 었다 러 가 에도

고 근 가 꿈꾸었다 근 가 꿈꾸게

것 보통 업 원색1926

도 만 었다lsquo rsquo

그림 가 근 경 공주- ldquo rdquo ( 2009)

zb31) 다음 이 같은 의 성 소에 해당하 않은

것은

사건 평① ② ③

④ 주 ⑤ 경

가 운 는 어 어ldquo rdquo 1965helliphellip

월 새벽 시 태 없 거웠고 는5 6 1

없 그 병원에 퇴원 집 가

는 마지막 마 고 마 내 거 다 가

죽 간신 에 실 다 사는 어느 가5 lsquo

죽 는 말 가 식 다 신rsquo

상에 각 시키는 에 실 어느( ) lsquo刻印

가는 후 민 가가 근 었다rsquo

는 간 과 진실 그 다는 에ldquo

단 평 견 가지고 다 내가 그

리는 간상 단 고 다 지 다 는 그들 가

에 는 평 지 니 그리고 어린 들

미지 겨 그린다rdquo

근 간 과 진실 그리고 싶어 가

다 근에게 그것 진리 다 거 다 없 거

고 다 없 는 것 진리다

근 진리는 후 쪽 었다 신산 삶 ( )辛酸

었 질곡 역사 에 지냈 가 눈에( )桎梏

든 것 료 단 료 게 보 것

었다 그것 그 에 겨우겨우 슬

슬 생 어가는 간들 었다 리

과 단 리 고리에 검 마

없 거리 돌 상

것 없는 등 근에게 상에 과 진실

엄 다는 사실 리는 가 실( )儼存

고 가 과 역경 에 도 근 내 포 없었

후 보루 다 도 도( ) 365堡壘

간 근 여 시 것

간에 지닌 가 근 1914 2

월 강원도 림리에 삼 독21

태어났다 어 근 복 그것 그리

가지 못 다 근 곱 살 지는 산

사업에 실 고 답마 에 내 갔다 근

그림 럼 쫓 다니 가 시 것 다 상

진 것도 가 었다 러 가 에도

고 근 가 꿈꾸었다 근 가 꿈꾸게

것 보통 업 원색1926

도 만 었다lsquo rsquo

공주 그림 가 근 경- ldquo rdquo ( 2009)

년 학 간고사 대비2013 2 현대고 대비

ECN-0102-2013-001-000076193

zb32) 위 을 작성하는 과정에서 되어 활 된 자

어 것은

신 사 료① 연보②

고③ ④ 들과 담

⑤ 에 평

는 간 과 진실 그 다는 에ldquo

단 평 견 가지고 다 내가 그

리는 간상 단 고 다 지 다 는 그들 가

에 는 평 지 니 그리고 어린 들

미지 겨 그린다rdquo

근 간 과 진실 그리고 싶어 가

다 근에게 그것 진리 다 거 다 없 거

고 다 없 는 것 진리다

근 진리는 후 쪽 었다 신산 삶 ( )辛酸

었 질곡 역사 에 지냈 가( )桎梏

눈에 든 것 료 단 료 게 보

것 었다 그것 그 에 겨우겨우

슬 슬 생 어가는 간들 었다

리 과 단 리 고리에 검 마

없 거리 돌 상

것 없는 등 근에게 상에 과

진실 엄 다는 사실 리는 가 실( )儼存

고 가 과 역경 에 도 근 내 포

없었 후 보루 다 도 도( ) 365堡壘

간 근 여 시

것 다

간에 지닌 가 근 1914 2

월 강원도 림리에 삼 독21

태어났다 어 근 복 그것 그리

가지 못 다 근 곱 살 지는 산

사업에 실 고 답마 에 내 갔다 근

그림 럼 쫓 다니 가 시 것 다 상

진 것도 가 었다 러 가 에도

고 근 가 꿈꾸었다 근 가 꿈꾸게

것 보통 업 원색1926

도 만 었다lsquo rsquo

질 루 마 가 도 린다 경건

움 느껴지는 경 다 훗 근 그림에

과 는 거 것( )裸木

만 간과 연 엮어 가는 경건 움lsquo rsquo

니었

같 가가 고 싶었 근에게 그 꿈에 다

가가는 지 다 다 가 지망생들 규 미

상 에 진 고

에 지만 근 다 다 근

미 에 운 것 보통 시 미 시간

다 그런 그에게 없는 연습 가가

통 다 가 귀 시 지 도

얻는 뛸 듯 뻤지만 마 도 가 에

듯 는 었 에 어린 근 주 에

에 그림 그리고 지우고 복( )粉板

시간 가는 게 루 보냈다

근 그 갈 가가 것 열여( )渴求

었 다가 미1932 lsquo rsquo ( lsquo

미 에 다 다는 고 마rsquo) lsquo rsquo

가 근 집 고도 지는 시골 경

그린 그림 다 후 근 에 1943 22

지 미 에 그림 고

에 걸쳐 다 미 근 가

동 는 었다

공주 그림 가 근 경- ldquo rdquo ( 2009)

zb33) 위 의 내 과 일치하는 것은

가 근 가 꿈 포 다①

근 당 가들과 께 에 다②

살 근 가 걷20③

게 었다

④ 만 통 근 역경 겨내는lsquo rsquo

느 다

⑤ 근 간 과 진실 그리 에 그 에

드러 는 간상 단 다

계 시 주 근 건강

걸었다 신 과 간에 상 다 건강

신 는 눈에도 다 근 쪽 눈 뿌 게

보 지 과에 다 다 시 지지 고 결

내 었다 시 지만 마 막막

다 늦어 결 근 쪽 눈 고 말 다

쪽 눈 근에게는 쪽 눈 었고

계 었다 그 근 는 여 그lsquo rsquo

다 근 에 같 그림 그 었다1950

시 그림 는 여 쪽lsquo rsquo

고 어 마주 고 는 그림1963

여 과 동 다 마 복

그린 듯 눈 내리 새 게 다 지

사 다 근 게 복 것

복 상과 타 는 근 상

가 떳떳 단 었고 근 그리고

간 과 진실 에 다가가 가 근다

운 었다 근 신에게 당당 지 그리고

그 다 근 그림에 단 복 보다

년 학 간고사 대비2013 2 현대고 대비

ECN-0102-2013-001-000076193

태 도 그리고 극 보다 과

얻 여 었다 과 통

근 그리고 는 재 고 에 질

만들고 특 것 다

공주 그림 가 근 경- ldquo rdquo( 2009)

zb34) 의 이유에 대해 추 한 것으 적절하 않은 것

상과 타 시도①

보다 과 얻②

근 신에게 당당 지③

④ 간 과 진실 에 다가

⑤ 태 도 얻

근 가가 었지만 그 다니 가

럼 어지지 다 복과 쟁 거쳐 시

는 가 근에게 생계 사 에

운 사 다 에 키에 건( ) 178cm死鬪

체 근 에 동 역 업( )荷役

가 생계 다 쟁

에는 동에 운 상우 주 미

죄 사 에 그림 그리는 시 다 그곳에

에 동 역 업 것에

결 것 럼 보 다 지만 그런 것만도

니었다 그림 그리는 고는 지만 매 근

는 극 간 과 별 없는 경 리 그림

벽에 그리는 것 었다 우도 리 없었다 근

트 는 우 그림 그 다 생

계 그림 단 것 다

후 근 지 신 계 리에 미

엑 리 겼다 근 곳에

건 사 크 에 미 들 ( )

상 상 그 다 근 갖 다 겪

냈다 그리고 결 그 돈

신동에 어 사리 집 마 다 마 ㄷ

루 심 쪽에는 과 엌 쪽에는 건

었다 건 주고 근 가 에

여 살 다 심 에는 지 집어

쓰고 지만 곳 근 가 에게 러웠

보 리 다 근 과 마루 업실 삼 그림

그 다 신동 마루는 근 그림에 등 는 lsquo rsquo

같 상들 지 다 시 고

에 들 폐허가

가 업실 었다

공주 그림 가 근 경- ldquo rdquo( 2009)

zb35) 위 에 대한 설 으 적절한 것은

업 시 여 훈과 감동 다①

에 주 평 드러 다②

사 사 등 식 과 ③

④ 다 근거 시 여 삶에

⑤ 살 시 사 경 께 여

습 시 다

가 시간 많지 다 청량리 생 병원( )

마지막 상 경 릿 게 들어 다 그 는

십 만 큰 가 상 말 다

지 못 들 마 갈 고 돗

도시민들 싹 싹 탔다 가 시

월에 병원에 원4 가 폐( )疲弊

진 몸도 갈 미 지 못 고 었다( )解渴 가는

얼마 지 생 에 생각

가 마감 는 신 평생 십 만에

가 과 많 닮 다고 생각 지는

가 운 는 어 어( ) ldquo rdquohelliphellip

월 새벽 시1965 5 6 1 태 없 거웠고

는 없 그 병원에 퇴원 집

가는 마지막 마 고 마 내 거 다

가 죽 간신 에 실 다 사는 어느5 lsquo

가 죽 는 말 가 식 다 신rsquo

상에 각 시키는 에 실( )刻印

어느 가는 후 민 가가 근 었다lsquo rsquo

다 는 간 과 진실 그 다는( ) ldquo

에 단 평 견 가지고 다 내

가 그리는 간상 단 고 다 지 다 는 가

에 는 평 지 니 그리고 어린 들

미지 겨 그린다rdquo

근 간 과 진실 그리고 싶어( )

가 다 근에게 그것 진리 다 거 다 없

년 학 간고사 대비2013 2 현대고 대비

ECN-0102-2013-001-000076193

거 고 다 없 는 것 진리

다 근 진리는 후 쪽 었다 신산( )辛酸 삶

었 질곡 역사 에 지냈( )桎梏

가 눈에 든 것 료 단 료 게 보

것 었다 그것 그 에 겨우겨우

슬 슬 생 어가는 간들 었다

리 과 단 리 고리에 검

마 없 거리 돌

상 것 없는 등 근에게 상에

과 진실 엄 다는 사실 리는 가 실( )儼存

고 가 과 역경 에 도 근 내 포

없었 후 보루 다( ) 堡壘 도 365

도 간 근 여

시 것 다

마 같 가가 고 싶었 근에게 그 꿈( )

에 다가가는 지 다 다 가 지망생들

규 미 상 에 진 고

에 지만 근 다 다 근

미 에 운 것 보통 시 미 시간

다 그런 그에게 없는 연습 가가

통 다 가 귀 시 지 도

얻는 뛸 듯 뻤지만 마 도 (

는 었 에 어린 근 주 에)

에 그림 그리고 지우고( )粉板

복 시간 가는 게 루 보냈다

zb36) 전 의 성 소가 아닌 것을 고르

① 평 ② 사건 ③ 경

④ ⑤ 훈

늘 지 상에 살고 는 사 들 억 도가10

고 그리 지 통 고 는 사 들( )知的

그보다 훨 많 억 도는 고 지 20

통 다 그런 지 고 2500

그리 간 보는 과 사 에

매우 달 뿐만 니 과 에 도 극

루고 었다 미 운 그런 들

살고 는 동 과 사 들 사고 식에

큰 가 다는 다

고 그리 들 우주 개별 고 독립

사 들 생각 지만 고 들 우

주 연 질 간주 다 같( ) 看做

각 도 들에게는 연 질

었지만 그리 들에게는 미 들 결 었

다 고 과 그리 들 사 같

는 동 과 사 에 도 견 다

지심리 미 마 드 겐트 는

살 들에 에 지 다

연 동 과 상 다 과 같 실험

다 크 만든 미드 도 보

여 주고 그 상 닥 고 주었다lsquo (Dax)rsquo

실 닥 는 재 지 는 것 실험 가lsquo rsquo

만들어 낸 다 그런 다 개 다 체 보

여 주었는 는 미드 지만 틱

만들었고 다 는 재료는 크 지만

달 다 그러고 어 것 닥 지 사 들에게 고 lsquo rsquo

게 니 들 주 같 고 는

체 택 고 동 들 같 재료 만들어진 체

택 다 러 는 심지어 살짜리

들에게 도 타났다 것 곧 과 동

다 상 보고 다는 것 미 다

개별 사 보고 고 동 연 질 보

고 는 것 다

동 들 주변 상 에 맞 어 동 고

에 다 사 들 태도 동에 보다 많

주 울 다 동 가 미시간 에

에 경험 다 그는 미식

경 보러 가게 었는 경 체는 매우 재미 었

주변 들 동에 질 다 그 는

들 계 어 상태 경 다

어 들 에 에 그 시 가 계 가

진 것 다 상 살펴 는 말 들 lsquo rsquo

에 그는 에 시 어 도 뒷사

생각 곧 다시 곤 것 다 그런 그에게 뒷

사 고 지 는 들 동 럼

어 웠다

생각 지도 리 드 니 벳-

zb37) 다음 위 의 내 전개 으 만 인lt gt

것은

lt gt

대조의 통해 대상이 닌 특성을 설 하고 있다

일화를 제 하여 자 의 주장을 뒷 침하고 있다

유추의 을 사 하여 독자의 의해를 돕고 있다

대상이 형성되는 과정을 간적 서에 따라 서 하고 있

① ②

③ ④

년 학 간고사 대비2013 2 현대고 대비

ECN-0102-2013-001-000076193

가 우리가 말 고 쓰는 든 단어가 사 에 는( )

것 니다 사 격에 가 는 지만

어 사 과 같 특별 는 사 니lsquo rsquo

단어 격 보 단어가 사 에

등재 어 다 리 리 사 는 단어 도 그

것 시 사 는 어 고 사 에

격 보 것 니다

러 얼 은 사전에 를 있는가 이에 대한 답lsquo rsquo

은 얼 이 유행어인가 아닌가에 따라 갈라 다 이 단어lsquo rsquo

는 년 어 자 에 랐고 쓰이고 있으2002 lsquo rsquo

유행어라고 하 에는 생 이 다 런데 계속

을 유 하 서 사전에 등재될 자격을 획득할 것인가 이

에 대한 답을 내리 는 히 어 다

여 서 가 를 고 해 볼 있다 첫 는 이 단어

를 써야 할 필 가 속적으 있는가 하는 점이다

상주의 열풍에 휩 인 사회 위 에 편 해서 퍼 말

이 얼 인데 과연 런 위 가 속될 것인가 이에lsquo rsquo

대해 필자의 생각은 정적이다 사회 위 가 뀌

런 말을 쓸 일이 없어 것이다

다음은 단어의 성이다 단어의 성이 사회적으 거

감이 없으 계속 사 될 가능성이 높다 런 에서

얼 은 좋은 조건이 아니다 익히 알 졌듯이 이lsquo rsquo

말은 얼 과 청소년층에서 속어 사 하는 이 결합lsquo rsquo lsquo rsquo

된 말이다 얼 에서 얼 을 리하는 조어 도 lsquo rsquo lsquo -rsquo

어에서는 매 낯선 이다 이것만으 도 거 감을 갖

는 사람들이 있다 더 나 속어 결합한 말이다 얼 lsquo rsquo

이 널리 퍼졌다 해도 은 여전히 청소년층의 속어lsquo rsquo

남아 있다 속어는 자연 럽게 아 자리에서나 쓰 에는

담 러 말이다 러한 담을 하고 사

역을 넓혀 가는 속어도 없 는 않다 특히 얼 은 lsquo rsquo

에도 종종 등장한다 만큼 거 감이 많이 희석되었다

고 할 있다 러나 일상의 자연 러 대화에서도 거

리낌 없이 등장하는가 게 는 되 않았다고 생

각한다

얼 이 유사어인 쌈 등을 만들어 내고lsquo rsquo lsquo rsquo

있으니 살아남을 있을 것이라고 는 견해도 있을 것

이다 러나 간이 나 서 유사어를 포함하여 든

말이 사라 사 는 많다 유사어가 많다는 것이 생 을

유 할 있는 절대적인 조건은 아니다

나 언젠가 터 사람들은 어느 단에서 얼 이 가장( )

쁜 사람을 가리켜 얼 이라고 르고 있다 이 얼lsquo rsquo lsquo rsquo

이라는 단어가 최근 어사전에 라 항간에 논란이 일고

있다 아닌 게 아니라 얼 은 유행어처럼 인다 생 lsquo rsquo

도 리 래되 않은 것 같고 언제 사라 도 알

없다 게다가 젊은이들 사이에서 주 쓰일 뿐이다 이런

단어를 사전에 는다는 게 하 이 없어 이 도

한다

러나 속단은 이다 차근차근 따져 볼 일이다

선 얼 이 일 적 유행어인 아닌 주의 게 들여다lsquo rsquo

볼 필 가 있다 유행어란 유행에 따라 빠르게 유포되었

다가 단 간 내에 소 되는 단어나 를 가리킨다

얼 은 인터넷을 통해 속히 퍼 말이다 하 만 일lsquo rsquo

적인 유행어처럼 단 간 내에 사라 않았을 뿐 아니라

현재 도 잦은 빈도 사 되고 있고 앞으 도 상당

간 사 될 것으 측된다 한 언 재단의 뉴 검 lsquo rsquo

색 사이트에 따르 얼 은 년 에 처음 나타난lsquo rsquo 2001

이후 꾸 히 사 되고 있다

이 같은 사 빈도는 얼 이 일 적 유행어 는 현lsquo rsquo

저히 다르다는 것을 여 다 장 간의 생존 만으 도

얼 은 이 한 어의 어휘 에 를 자격을 얻었다lsquo rsquo

고 할 있다 더 이 이라는 비 적 정제된 매체에

높은 빈도 쓰이고 있 않은가 사 빈도 측 에서

필통이나 연필과 같은 단어 대등하거나 더 많이 쓰lsquo rsquo lsquo rsquo

다는 것은 결코 가 게 볼 일이 아니다

이제는 사전이 언어 현 을 빠르게 하는 게 덕인

대가 되었다 세계적으 유 한 의 사전들도 경쟁

적으 어를 고 있다

하 만 얼 은 젊은이들이나 쓰는 속어라고 흠을 잡을lsquo rsquo

도 르겠다 얼 이 주 젊은 층에서 많이 쓰 lsquo rsquo

는 속어임에 틀림없다 러나 어사전에 표 적이고 품

위 있는 말만 어야 한다고 생각한다 것은 커다란

해다 당장 아 어사전이나 펼쳐 라 속어는

설과 같은 비어나 죄자들이 쓰는 은어 어

마니 같은 소 의 사람만이 쓰는 말 도 라 있

않은가 사전은 말 치에 일정 빈도 이상 나타나는 말이

라 말이든 다 할 있다

zb38) 가 나 에 대한 다음의 설( ) ( ) 않은 것은

① 가 는 얼짱 사 에 등재 것에( ) ( ) lsquo rsquo

보 고 다

② 사 등재 가는 단어 격에( )

고 고 는 언 들 언어 사 도에 고 다 ( )

③ 가 얼짱 어지만 신 과 같 매( ) ( ) lsquo rsquo

체에 도 사 는 말 는 고 다

④ 가는 얼짱 어 보고 크게 가지 근( ) lsquo rsquo 3

거 들어 뒷 고 다

⑤ 는 얼짱 어 는 다 특 다는( ) lsquo rsquo

근거 에도 크게 가지 근거 가 들어 주 2

뒷 고 다

가 늘 지 상에 살고 는 사 들 억( ) 10

도가 고 그리 지 통 고 는 사 들

그보다 훨 많 억 도는 고 지 20

통 다 그런 지 고 2500

년 학 간고사 대비2013 2 현대고 대비

ECN-0102-2013-001-000076193

그리 간 보는 과 사 에

매우 달 뿐만 니 과 에 도 극

루고 었다 미 운 그런 들

살고 는 동 과 사 들 사고 식에

큰 가 다는 다

고 그리 들 우주 개별 고 독립

사 들 생각 지만 고 들 우

주 연 질 간주 다 같 각

도 들에게는 연 질 었지

만 그리 들에게는 미 들 결 었다

고 과 그리 들 사 같 는

동 과 사 에 도 견 다

인 리학자인 츠 이마이 디드 겐트너는 두

살이 채 안 된 아이들에서 터 성인에 이르 다양한

연 대의 동양인과 서양인을 대상으 다음과 같은 험

을 했다 저 코르크 만든 피라 드 양의 도형을

여 주고 대상의 이름을 닥 라고 알 주었다lsquo (Dax)rsquo

제 닥 는 존재하 않는 것으 험자가 임의lsquo rsquo

만들어 낸 이름이다 런 다음 두 개의 다른 체를

여 주었는데 하나는 피라 드 양이 만 하얀 플라 틱

으 만들었고 다른 하나는 재 는 코르크 만 양이

달랐다 러고 나서 어떤 것이 닥 인 사람들에게 고 lsquo rsquo

르게 했더니 서양인들은 주 같은 양을 하고 있는

체를 선택했고 동양인들은 같은 재 만들어 체를

선택했다 이러한 차이는 성인은 어 두 살 리

아이들에게서도 나타났다 이것은 곧 서양인과 동양인은

서 다른 세상을 고 있다는 것을 의 한다 략 ( )

는 아주 단 하 서도 인상적인 험을 했다

험에는 동서양의 대학생들이 참여했다 는 험 참가자

들에게 컴퓨터 화 을 통해 속 장 을 담은 애니 이션

을 여 주었다 화 의 앙에는 초점의 역할을 하는 커

다란 고 한 마리가 있었고 주위에는 다른 생

들과 초 자갈 거품 등이 함 제 되었다 화 을

두 씩 후 참가자들은 자 이 것을 회상해 라는

를 았다

결과 서양인 대학생들과 동양인 대학생 두 앙

의 초점 역할을 했던 고 를 동일한 정도 언 했으

나 경 소 위 거품 초 다른 생 들 에 ( )

대해서는 동양인 대학생들이 서양인 대학생들 다 60

이상 더 많이 언 했다 뿐만 아니라 동양인 학생들은 서

양인 학생들에 비해 개 적인 고 다 전체적인 계

를 더 언 하는 경향을 다 략 또한 경의 일 ( )

를 화 킨 림을 제 하 을 때 동양인 대학생들은 대

경의 화를 알아챘 만 서양인 대학생들은 경

의 화를 거의 알아차리 했다 략 ( )

따라서 서양인들만을 대상으 연 한 화lsquo

편성 결 은 잘 된 것일 도 있다 각 과정과 인rsquo

과정의 어떤 이 화 편적이고 어떤 이

화에 따라 달라 는 는 앞으 많은 연 를 통하여 논의

되어야 한다

나 어떤 의 에서 리 두는 이 화적이다 리( )

안에는 다른 사람들과 더 친 한 계를 유 하 는 상호

의존성과 다른 사람들 터 독립적인 존재 살아가 는

독립성이 혼재한다 따라서 이 에서 어떤 특성이 더 강

하게 각되는 상황에 놓이느냐에 따라 서 다른 화적

특 을 일 있다 결 리 두는 어떤 경 에는

동양인처럼 행동하고 어떤 경 에는 서양인처럼 행동하는

것이다

zb39) 가 에 대한 다음의 설( ) 않은 것은

① 는 신 주 뒷 닥 실험과lsquo rsquo lsquo

니 실험 근거 시 다rsquo

② 동 들 상 간 공통 보다는 에 식

는 강 다

③ 들 주변 맥 에는 심 경 어 사건

과 사건 사 계에 상 민감 다

④ 는 동 과 틀린 지 고 는 것lsquo rsquo

니 다 고 다 lsquo rsquo

⑤ 가에 우리 사 들 개 시 가 원( )

집 경 말 고 는 것 개 보다는

에 고 는 것에 다

늘 지 상에 살고 는 사 들 억 도가10

고 그리 지 통 고 는 사 들( )知的

그보다 훨 많 억 도는 고 지 20

통 다 그런 지 고 2500

그리 간 보는 과 사 에

매우 달 뿐만 니 과 에 도 극

루고 었다 미 운 그런 들

살고 는 동 과 사 들 사고 식에

큰 가 다는 다

지심리 미 마 드 겐트 는 동

과 상 다 과 같 실험 다

크 만든 미드 도 보여 주고 그

상 닥 고 주었다 그런 다lsquo (Dax)rsquo

개 다 체 보여 주었는 는 미드

지만 틱 만들었고 다 는 재료는

크 지만 달 다 그러고 어 것 닥 lsquo

지 사 들에게 고 게 니 들 주 같rsquo

고 는 체 택 고 동 들 같

재료 만들어진 체 택 다 러 는

심지어 살짜리 들에게 도 타났다 것

곧 과 동 다 상 보고 다는

것 미 다 개별 사 보고 고 동

년 학 간고사 대비2013 2 현대고 대비

ECN-0102-2013-001-000076193

연 질 보고 는 것 다

동 들 주변 상 에 맞 어 동 고

에 다 사 들 태도 동에 보다

많 주 울 다 동 가 미시간

에 에 경험 다 그는 미

식 경 보러 가게 었는 경 체는 매우 재

미 었 주변 들 동에 질 다 그

는 들 계 어 상태 경

다 어 들 에 에 그 시 가 계

가 진 것 다 뒷사 고 지 는 들

동 럼 어 웠다

그는 경험에 어 얻어 동 들lsquo

각도 상 본다 는 가 우고rsquo

검 여 주 단 도 상 실험 실

시 다 그는 실험 가 들에게 컴퓨 통

담 니 보여 주었다

에는 역 는 커다 고 마리가 었

고 주 에는 다 생 들과 갈 거 등

께 시 었다 본 후 가 들

신 본 것 상 보 는 지시 다

그 결과 생들과 동 생

역 고 동 도 언

경 거 다 생 들에 ( )

는 동 생들 생들보다 60

상 많 언 다 뿐만 니 동 생들

생들에 개별 고 보다 체 계

언 는 경 보 다 경 변 시

킨 그림 시 동 생들 경

변 지만 생들 경 변

거 리지 못 다

지 지 들만 상 연 lsquo

보편 결 못 것 도 다 지각 과 과rsquo

지 과 어 보편 고 어

에 달 지는지는 많 연 통 여

어 다

리 드 니 벳 생각 지도 사- ldquo rdquo( 2004)

zb40) 위 에 대한 설 으 가장 적절한 것은

① 동 과 생 식 강 고 다

② 가지 실험 통 쓴 고 다

③ 닥 실험에 사 본질에 동 사

상에 주 다

④ 니 실험에 동 과 에 지

각 도에 가 다

⑤ 쓴 는 보편 연 에 드러 우월 에

에 근 고 다

가 동 들 주변 상 에 맞 어 동 고( )

에 다 사 들 태도 동에 보다 많

주 울 다 동 가 미시간 에

에 경험 다 그는 미식

경 보러 가게 었는 경 체는 매우 재미 었

주변 들 동에 질 다 그 는

들 계 어 상태 경 다

어 들 에 에 그 시 가 계 가

진 것 다 상 살펴lsquo 는 말 들rsquo

에 그는 에 시 어 도 뒷사

생각 곧 다시 곤 것 다 그런 그에게

뒷사 고 지 는 들 동 럼

어 웠다

그는 경험에 어 얻어( ) 동 들lsquo

각도 상 본다 는 가 우고rsquo

검 여 주 단 도 상 실험

실시 다 실험에는 동 생들 여 다

그는 실험 가 들에게 컴퓨 통

담 니 보여 주었다 에는

역 는 커다 고 마리가 었고 주 에는

다 생 들과 갈 거 등 께 시

었다 본 후 가 들 신 본 것

상 보 는 지시 다

다 그 결과 생들과 동 생( )

역 고 동 도 언

경 거 다 생 들 에 ( )

는 동 생들 생들보다 60

상 많 언 다 뿐만 니 동 생들

생들에 개별 고 보다 체 계

언 는 경 보 다 들어 동

생들 상 체 연못 럼 보 어ldquo 같rdquo

체 맥 언 시 었지만

생들 상 어 같 큰 고 가 쪽 움ldquo

직 어 같 역 고rdquo

언 시 다 경 변 시킨 그

림 시 동 생들 경 변

지만 생들 경 변 거

리지 못 다

년 학 간고사 대비2013 2 현대고 대비

ECN-0102-2013-001-000076193

게 볼 동 들 보다는 큰 그( )

림 보 에 사 과 체 맥 연결시 지각

는 경 고 체에 특 떼어 내

어 독립 보는 것 낯 어 다 에

들 사 에 고 주변 맥 에는 심 경

에 사건과 사건 사 계에 상

민감 편 다

마 지 지( ) 들만 상 연

보편 결 못 것 도 다lsquo rsquo 지각 과

과 지 과 어 보편 고 어

에 달 지는지는 많 연 통 여

어 다

리 드 니 벳 생각 지도 사- ldquo rdquo( 2004)

zb41) 의 하는 가~ 다른 것은

① ② ③

④ ⑤

얼마 그 에 동 사고 식과

사고 식 보여 주는 내 다

들 에 는 탕 고 같 게

어 겨 고 미 에 는 그 크 럼 큰 고

어리 주고 원 는 어 도 는

상 고 생각 다는 것 다 러

는 어떻게 생 것 고 과 그리 거슬

러 가 보 그 단 다

고 연 경 체 경 생 에

다 벼 사는 공동 업과 경험 많 연 역

에 고 들 연 웃과

게 지내 고 탁 연 들

들 지 연 럽게 들 다 민들

웃과 동 게 뿐만 니 는 집 과

게 다

동 시 는 생태 경 에 살 결과

들 다 사 들 사 상 에 주

울 게 었고 는 곧 체 상 과 간 사

계 시 는 낳게 었다 신 가

가 는 체에 는 원 는 동시

에 다 사 들 그 사 포 체 맥 에

다 들 간 사 연

계 체 계에 주 울 는 사고 체계

게 었다

그러 그리 연 경 그 었다 산

지 연결 는 지 건 그리고 역

에 다 런 들 업에 다 사 과

동 므 공동체에

다고 다 고 그리 들

들과는 달리 보 내 감 지 들과

지 크게 느 지 못 다 그

견 다 경우 주 쟁 통 결 는 갖

게 었다

신 사 간 계들 루어진 커다

트워크 에 게 당연 사 역시 연

계들 체 식 게 다 어 상

원 도 그 개체가 체 맥 과

계 에 고 다 게 체 맥 에 주

울 다 보 상 복 과 가변 식 게 고

상에 재 는 많 변 들 사 에 재 는 들도

게 다 들 주 태도 보

는 경우가 많다 쟁 결

통 결 보다는 통 결

는 보 다

그러 고 그리 들 개개 사 사 독

에 주 울 다 사 사 체에

어 그들 사 에 재 는 공통 규 주

고 다 상 원 에도 사

체 내 주 고 다 그들

체 여 탕 체

는 주 태도 시 고 특 사 어

주에 는지 여 그 주에 는 규

견 다 에 는 쟁 식 리

같 리 사고 체계가 달 게 었다

리 드 니 벳 생각 지도 사- ldquo rdquo( 2004)

zb42) 위 에서 사 된 설 과 가장 유사한 것은

① 크톱 컴퓨 는 본체 니 마우 루

어 다

② 곡과 시 리 는 지 과 사 루어 다는 공통

지니고 다

③ 경 고 것과는 달리

경 본 연 태 그 주변 경

④ 벽돌 능 에 사계 내내

습도가 지 다

⑤ 잰느 체 체 지닌 재 체가 없

는 재 눌 다

년 학 간고사 대비2013 2 현대고 대비

ECN-0102-2013-001-000076193

zb43) 는 립 앙 도서 이 정의 일 이다lt gt

도서 장과 이 자의 리 의 정의 연결이

적절하 않은 것은

lt gt

제 조 서 유8 ( )

도서 장은 다른 이 자의 안전을 위협하거나 도서 의①

서를 란하게 할 가 있는 자에 대하여는 도서 출입

을 제한할 있다

도서 장은 이 자가 제 조 각 호의 어느 하나의 행위를 하7②

을 때에는 이 을 하게 하거나 도서 출입을 제한할

있다

제 조자 의 대출9 ( )

도서 자 는 다음 각 호의 경 대출할 있다①

상호대차도서 간에 자 를 류하는 것을 말한다 등 다1 ( )

른 도서 과의 협 을 위하여 필 한 경

공 이 공 행 상 필 하는 경2

에 도서 장이 필 하다고 인정하는 경3

대출이 가능한 도서 자 의 위는 도서 장이 정하는②

에 따른다

제 조 상10 ( )

이 자가 도서 자 설을 더럽히거나 찢거나 뜨①

쓰게 하거나 잃어 린 경 에는 상하여야 한다

도서 장은 제 항에 따른 상 을 정하여 게 하여야1②

한다

제 조이 절차 등11 ( )

이 칙에서 정한 것 에 도서 자 설의 이 절차

이 제한 등에 필 한 사항은 도서 장이 정한다

출처 립 앙 도서- (httpwwwnlgokr)

① 는 도 리 다8

② 도 는 리 다9 1

③ 료 지 는 도 리 다9 2

④ 도 료 변상에 리10 1

⑤ 는 에 도 리 다11

3

도 다 각 같다①

공 공 다만 연1

연 간 다

매월 째 째 월2

도 도 리 그 사3

가 다고 는

도 에 미리 게1 3②

시 여 다

4

도 시간 도 여 게시 다

5

도 료 시 는 는 도①

지에 등 후

등 에 사 도②

7

는 다 각 여 는 니 다

도 료 시 상 리1 lsquo rsquo

도 료 시 훼 는2 middot

지 가 닌 곳에 식 거 담3

우는

도 보 등 보 검색열4 middot

그 에 도 질 지 여 도5

여 게시 사 는

8

도 다 거 도①

질 게 우 가 는 에 여는 도

도 가 각 어느7②

에는 지 게 거 도

9

도 료는 다 각 경우 다①

상 도 간에 료 는 것 말1 (

다 등 다 도 과 여 경우)

공 원 공 상 는 경우2

그 에 도 다고 는 경우3

가능 도 료 는 도②

는 에 다

10

년 학 간고사 대비2013 2 현대고 대비

ECN-0102-2013-001-000076193

가 도 료 시 럽 거 거①

못 쓰게 거 어 린 경우에는 변상 여

도 에 변상 여 게시1②

여 다

zb44) 위 에서 도서 장이 게 해야 할 사항에 해당하는

것을 두 쓰

년 학 간고사 대비2013 2 현대고 대비

ECN-0102-2013-001-000076193

립 도 규

1 ( )

규 립 도 립 어린 청 도(

포 다 료 시 열 시 말) (

다 에 사 규 립 도)

편 진 다

2 ( )

규 립 도 도 다 에( lsquo rsquo )

고 는 도 에 도lsquo rsquo 2 2

료 에 여 다 다만 특 료 귀

료 등 료 에 사 립 도

도 다 다( lsquo rsquo )

3 ( )

도 다 각 같다①

공 공 다만 연1

연 간 다

매월 째 째 월2

도 도 리 그 사3

가 다고 는

도 에 미리 게1 3②

시 여 다

시간4 ( )

도 시간 도 여 게시 다

등 등5 ( )

도 료 시 는 는 도①

지에 등 후

등 에 사 도②

사 료6 ( )

도 료 시 에 사 료는 도

7 ( )

는 다 각 여 는 니 다

도 료 시 상 리1 lsquo rsquo

도 료 시 훼 는2 middot

지 가 닌 곳에 식 거 담3

우는

도 보 등 보 검색열4 middot

그 에 도 질 지 여 도5

여 게시 사 는

질 지8 ( )

도 다 거 도①

질 게 우 가 는 에 여는 도

도 가 각 어느7②

에는 지 게 거 도

료9 ( )

도 료는 다 각 경우 다①

상 도 간에 료 는 것 말1 (

다 등 다 도 과 여 경우)

공 원 공 상 는 경우2

그 에 도 다고 는 경우3

가능 도 료 는 도②

는 에 다

변상10 ( )

가 도 료 시 럽 거 거①

못 쓰게 거 어 린 경우에는 변상 여

도 에 변상 여 게시1②

여 다

등 규 에 것 에 도11 ( )

료 시 등에 사

도 다

립 도- (httpwwwnlgokr)

zb45) 도서 장의 리 있는 조항으 적절하 않

은 것은

① ② ③ ④ ⑤

년 학 간고사 대비2013 2 현대고 대비

ECN-0102-2013-001-000076193

1 ( )

사가 공 는lsquo rsquo

과 여 사 원과 리

사 타 사 규

니다

개 보 보7 ( )

사는 보통신망 등 계 는 에lsquo rsquo lsquo rsquo

원 개 보 보 니다 개lsquo rsquo

보 보 사 에 는 사 개lsquo rsquo

보 취 니다 다만 사는 다 lsquo rsquo

사 계 통 공 는 경우 원 lsquo rsquo

등 개 보 당 사에 습니lsquo rsquo

원 리에8 (lsquo rsquo lsquo rsquo lsquo rsquo

)

원 에 리lsquo rsquo lsquo rsquo lsquo rsquo①

원에게 가 도 여 는lsquo rsquo 3

니다

사는 원 가 개 보 우 가lsquo rsquo lsquo rsquo lsquo rsquo②

거 사 경우 는 미 에 어 거 lsquo

사 사 운 우 가 는 경우 당rsquo lsquo rsquo

습니다lsquo rsquo

원 가 도 거lsquo rsquo lsquo rsquo lsquo rsquo 3③

가 사 고 지 경우에는 시 사에lsquo rsquo

통지 고 사 내에 니다lsquo rsquo

경우에 당 원 사에 그 사실3 lsquo rsquo lsquo rsquo④

통지 지 거 통지 도 사 내에 지 lsquo rsquo

생 경우 사는 지지 습니다lsquo rsquo

사10 (lsquo rsquo )

사는 과 지 미lsquo rsquo①

에 는 지 계 고

공 여 다 여 니다lsquo rsquo

사는 원 게lsquo rsquo lsquo rsquo lsquo rsquo②

도 개 보 신 보 포 보 보 시( )

갖 어 개 보 취 공시 고

니다

사는 과 여 원lsquo rsquo lsquo rsquo③

견 만 당 다고 경우에는

리 여 니다 원 견 만 사 lsquo rsquo

에 는 게시 거 우편 등 통 여

원에게 리 과 결과 달 니다lsquo rsquo

원11 (lsquo rsquo )

원 다 여 는 니다lsquo rsquo ①

신청 는 변경 시 허 내 등1

타 보 도2

사가 게시 보 변경3 lsquo rsquo

사가 보 보 컴퓨 그4 lsquo rsquo (

등 등 신 는 게시)

사 타 등 지 재산 에5 lsquo rsquo 3

사 타 상 거 업6 lsquo rsquo 3

는 폭 시지 상 타 공7 middot middot

에 는 보 에 공개 는 게시 는lsquo rsquo

사 동 없 리 사8 lsquo rsquo

타 거 당9

게시15 (lsquo rsquo )

원 내에 게시 는 게시 게재 는lsquo rsquo lsquo rsquo lsquo rsquo

경우 원 사가 게시 복 lsquo rsquo lsquo rsquo lsquo rsquo middot middot

등 태 언 등에 공 는

것 내에 다 원 본 게시 등 lsquo rsquo lsquo rsquo

크 능 등 여 복 는 등 태

는 것 동 것 니다

- (wwwnavercom)

zb46) 위 은 인터넷 포털사이트의 회 가입을 위한 이

약 의 일 이다 이 약 을 만드는 과정에서 생각한

내 으 적절하 않은 것은

개 보 보 가 지에 별 눠①

겠어

원 가 만들게 에②

시 주어 겠어

원들 게재 게시 다 원 크 다③

는 것 지

④ 원 지 는 뿐만 니 사가 지 는

도 께 달 지

리에 가 생 경우 사가⑤

에 다는 도 듯

1 ( )

사가 공 는lsquo rsquo

과 여 사 원과 리

사 타 사 규

년 학 간고사 대비2013 2 현대고 대비

ECN-0102-2013-001-000076193

니다

개 보 보7 ( )

사는 보통신망 등 계 는 에lsquo rsquo lsquo rsquo

원 개 보 보 니다 개lsquo rsquo

보 보 사 에 는 사 개lsquo rsquo

보 취 니다 다만 사는 다 lsquo rsquo

사 계 통 공 는 경우 원 lsquo rsquo

등 개 보 당 사에 습니lsquo rsquo

원 리에8 (lsquo rsquo lsquo rsquo lsquo rsquo

)

원 에 리lsquo rsquo lsquo rsquo lsquo rsquo①

원에게 가 도 여 는lsquo rsquo 3

니다

사는 원 가 개 보 우 가lsquo rsquo lsquo rsquo lsquo rsquo②

거 사 경우 는 미 에 어 거 lsquo

사 사 운 우 가 는 경우 당rsquo lsquo rsquo

습니다lsquo rsquo

원 가 도 거lsquo rsquo lsquo rsquo lsquo rsquo 3③

가 사 고 지 경우에는 시 사에lsquo rsquo

통지 고 사 내에 니다lsquo rsquo

경우에 당 원 사에 그 사실3 lsquo rsquo lsquo rsquo④

통지 지 거 통지 도 사 내에 지 lsquo rsquo

생 경우 사는 지지 습니다lsquo rsquo

원에 통지9 (lsquo rsquo )

사는 특 다 원에게 통지 경우lsquo rsquo lsquo rsquo

공지 게시 통 상 게시 개별 통지에7

갈 습니다

사10 (lsquo rsquo )

사는 과 지 미lsquo rsquo①

에 는 지 계 고

공 여 다 여 니다lsquo rsquo

사는 원 게lsquo rsquo lsquo rsquo lsquo rsquo②

도 개 보 신 보 포 보 보 시( )

갖 어 개 보 취 공시 고

니다

사는 과 여 원lsquo rsquo lsquo rsquo③

견 만 당 다고 경우에는

리 여 니다 원 견 만 사 lsquo rsquo

에 는 게시 거 우편 등 통 여

원에게 리 과 결과 달 니다lsquo rsquo

원11 (lsquo rsquo )

원 다 여 는 니다lsquo rsquo ①

신청 는 변경 시 허 내 등1

타 보 도2

사가 게시 보 변경3 lsquo rsquo

사가 보 보 컴퓨 그4 lsquo rsquo (

등 등 신 는 게시)

사 타 등 지 재산 에5 lsquo rsquo 3

사 타 상 거 업6 lsquo rsquo 3

는 폭 시지 상 타 공7 middot middot

에 는 보 에 공개 는 게시 는lsquo rsquo

사 동 없 리 사8 lsquo rsquo

타 거 당9

원 계 규 내lsquo rsquo lsquo②

여 공지 주 사 사가 통지 는rsquo lsquo rsquo

사 등 여 타 사 업 에 lsquo rsquo

는 여 는 니다

- (wwwnavercom)

zb47) 위 약 의 조항에서 같은 제점을 하lt gt

고 있는 조항은

lt gt

제휴 회사에 회 의 아이디 개인 정 를 전송할 있도

한 조항은 고객에게 당한 조항이다

1 7 8① ② ③

④ 9 ⑤ 10

립 도 규

1 ( )

규 립 도 립 어린 청 도(

포 다 료 시 열 시 말) (

다 에 사 규 립 도)

편 진 다

2 ( )

규 립 도 도 다 에( lsquo rsquo )

고 는 도 에 도lsquo rsquo 2 2

료 에 여 다 다만 특 료 귀

료 등 료 에 사 립 도

도 다 다( lsquo rsquo )

3 ( )

도 다 각 같다①

공 공 다만 연1

연 간 다

년 학 간고사 대비2013 2 현대고 대비

ECN-0102-2013-001-000076193

매월 째 째 월2

도 도 리 그 사3

가 다고 는

도 에 미리 게1 3②

시 여 다

시간4 ( )

도 시간 도 여 게시 다

등 등5 ( )

도 료 시 는 는 도①

지에 등 후

등 에 사 도②

사 료6 ( )

도 료 시 에 사 료는 도

7 ( )

는 다 각 여 는 니 다

도 료 시 상 리1 lsquo rsquo

도 료 시 훼 는2 middot

지 가 닌 곳에 식 거 담3

우는

도 보 등 보 검색열4 middot

그 에 도 질 지 여 도5

여 게시 사 는

질 지8 ( )

도 다 거 도①

질 게 우 가 는 에 여는 도

도 가 각 어느7②

에는 지 게 거 도

료9 ( )

도 료는 다 각 경우 다①

상 도 간에 료 는 것 말1 (

다 등 다 도 과 여 경우)

공 원 공 상 는 경우2

그 에 도 다고 는 경우3

가능 도 료 는 도②

는 에 다

변상10 ( )

가 도 료 시 럽 거 거①

못 쓰게 거 어 린 경우에는 변상 여

도 에 변상 여 게시1②

여 다

등 규 에 것 에 도11 ( )

료 시 등에 사

도 다

립 도- (httpwwwnlgokr)

zb48) 다음 정 리 의 의 으 볼 때 가장

이 적인 것은

도 시간 도 여 게시 다①

등 에 사 도②

가능 도 료 는 도 는③

에 다

④ 도 에 변상 여 게10 1

시 여 다

⑤ 도 가 각 어느7

에는 지 거 도

zb49) 를 참고하여 이 어의 성격을 설 한lt gt

것으 적절하 않은 것은

① 보 에 는 어 시 상 고 어 시lt gt lsquo rsquo

에 보여주고 다

② 진 어 어원에 견 고 다

에는 타 어 들어가는 것 다 lsquo rsquo

③ 에 들어갈 말 각각 고 어 어 신 어~

들 언어는 질 격 강 통 없었다

④ 시 우리 에 가 었지만 지 계

과 달리 들 통 사 달 어 웠

년 학 간고사 대비2013 2 현대고 대비

ECN-0102-2013-001-000076193

⑤ 크 몽골 만주 공통어가 우리 어 같

계열에 다는 에 사 특 짐

가( )

善化公主主隱 공주님

他密只嫁良置古 몰 결 고

薯童房乙 맛

夜矣卯乙抱遣去如 에 몰 고 가다

( )

始汝 會隱日恚見隱扐 만 에 본

恥隱汝衣淸隱笑 맑 웃

고 시 여 공 크다 만 다[ ] ( ) ( ) ( ) ( )始 汝 會扐

내다 에 보다 견( ) ( )恚 見 다( )隱

럽다 맑다 청 웃( ) ( ) ( ) ( )恥 衣 淸 笑

zb50) 위의 나 를 함 고 음에 답하( ) lt gt

보lt gt

( )素那或云金川 白城郡蛇山人也

운 사산

는 고 다 는( )[ ( ) ] (素那 金川 白城

사산 사 다) ( ) 郡 蛇山

삼 사- lsquo rsquo 47

에 제 된 단어 의 표 리를 조건(1) lt gt ( ) lt gt

에 맞게 서 하

건lt gt

lsquo 었고 었다 태rsquo

에 제 된 단어 동일한 표 리에(2) lt gt ( )

의해 적은 것을 나 에서 찾아 조건 에 맞게 서 하( ) lt gt

건lt gt

에 당 는 각각( ) 개 쓸 것2 단

당 는 가 여러 개 어도 개만 쓸 것 각2

개 과 도 쪽에 개만2 2

드시 지 것( )

과 동 원리 것lsquo 고

과 동 원리 것 다rsquo

태 것

가( )

素那(或云金川) 白城郡蛇山人也

소나 또는 천 이라 한다 는 성 사( ) ( ) ( )素那 金川 白城郡〔 〕

산 사람이다 현대어 풀이( ) ( )蛇山

나( )

紫布岩乎希 회

執音乎手母牛放敎遣 자 손 암쇼 노히 고

吾 不喩慙 伊賜等肹 肹 나 안디 리샤

花 折叱肹 可獻乎理音如 고 것거 도림다

다 향찰은 리말을 리 으 적은 표 이었 만 생( )

은 고 대를 넘 하고 끊어 고 말았다 랜 세

동안 갈고 닦아 체계적이었던 향찰 표 이 사라졌

을 인은 크게 두 가 나누어 생각해 볼 있다

하나는 족 사회의 한 선호도에서 찾을 있다 라 때

향찰은 주 족 계 에서 사 했을 것으 인다 한 을

알 하고서는 한자를 활 하여 리말을 리 으 표

하 란 가능하 때 이다 런데 족들은 간이 흐

를 향찰과 같은 리 표 을 익혀 사 하 다는

아 한 을 대 사 하는 쪽을 선호하게 되었다 더 이

고 초에 인재 등 을 위해 과거제도가 행되 서 한 선

호도가 더 높아졌고 결 향찰은 소 되고 말았다

또 다른 가능성은 한 어의 특성에서 찾을 있다

터 한 과 일 세 나라는 한자 화 에 속해 다

당연한 이야 겠 만 표의 자인 한자는 어를 표 하

에 매 적절하다 어의 음절은 성 ( ) ( )聲母 韻母

이 어 고 여 에 성조가 추가되어 최종 소리가 결정된

다 래서 어는 단음절을 하나의 한자 표 하 된

다 에 초성 성 종성의 세 가 소가 하나의 음절

년 학 간고사 대비2013 2 현대고 대비

ECN-0102-2013-001-000076193

을 이 는 한 어는 음절 조가 잡하고 음절의 가 많아

서 한자 차 만으 한 어의 소리를 만족 럽게 표 할

없었다 를 들어 한 어에서는 어 니 같이 음절 lsquo rsquo

이 어 단어가 얼마든 있으나 어는( ) 複數音節

자 하나 나타내 만이다lsquo [m ]rsquo 母 ǔ

한편 일 어의 표 은 핵 적 단어는 한자 적고 토는

가나라는 일 의 자 적는 이다 적인 의 를 나

타내는 은 표의 자인 한자 적고 적 계를 나

타내는 토는 표음 자 적는 셈이니 자세히 살펴

리의 향찰 표 을 쏙 빼닮았음을 알 있다 한 어 같

은 착어이 서도 일 어에만 향찰과 유사한 표 이 살아

남은 것은 일 어의 특 때 이다 일 어는 하나의 자음과

음의 결합으 음절을 이 고 침이 거의 없는 음절 언어

이다 이러한 음절의 특색에다가 토가 달한 착어라는 점

이 향찰과 유사한 표 이 살아남을 있는 비결이었다

하 만 같은 착어라도 다양한 음소 침이 달한 한

어는 향찰 표 하는 데 근 적으 한계가 있었다

zb51) 다 하여 의 행에 대한 탐 한 결과( ) lt gt 2

않은 것은

보lt gt

善花公主主隱 공주니믄 공주님( )

----------------------------------------

-

他密只嫁良置古 그 지 얼어 고 몰 결(

----------------------------------------

-

薯童房乙 맛 맛( )

夜矣卯乙抱遺去如 몰 고 가다 에 몰 고(

가다)

주동 역 동- (薯童謠『 』

에 2 ( )他密只嫁良置古

얼다 시집가다 결 다 말 lsquo rsquo

① 실질 미 지니고 므 타 타lsquo ( )rsquo lsquo [ ]

② 에 실질 미 타내고 지 는lsquo rsquo lsquo [ ]rsquo lsquo [ ]密只 密 只

계 타내는

③ 얼어는 실질 미 포 고 므 가lsquo rsquo lsquo [ ]rsquo嫁

것lsquo [ ]rsquo 良

④ 고 어간 는 실질 미 지니고 므lsquo rsquo lsquo -rsquo

것lsquo [ ]rsquo 置

⑤ 고 어미 고는 계 타내고 므lsquo rsquo lsquo- rsquo

고 것lsquo [ ]rsquo 古

가( )

엉 훈 민middot middot middot middot middot世 宗 御 製 訓 民 正 音

말 미 듕 귁에 달middot middot middot middot middot middot middot middot中 國 文 字

니 런middot middot middot middot middot middot 어린middot middot middot middot百 姓

니 고 도 내 들middot middot middot middot middot middot middot middot middot 시러middot

펴 몯middot 미middot middot 니 내middot middot middot middot middot middot middot middot 爲

어엿middot 겨 새middot middot middot 믈여듧middot middot middot middot字 니middot middot middot

사 마다 니겨 킈 middot middot middot middot middot middot middot middot middot便 安

고 미니middot middot middot middot

본 는 상( ) (象

원리에 만들어진 본) ( )形 ㄱ ㄴ ㅁ ㅅ ㅇ

에 는 가 원리에( )加劃

그리고( )ㅋ ㄷ ㅌ ㅂ ㅍ ㅈ ㅊ ㆆ ㅎ

쓰는 병 원리에 만들어진( )竝書

마지막 체( ) ( )異體ㄲ ㄸ ㅃ ㅆ ㅉ ㆅ

ᅀ 다 상 원리에 ㅇ ㄹ

지 는 삼재 상 본 본( ) ( ) ( 天地人 三才

탕 므림과 림에 ) (初ㅡ ㅣ

재)( ) ( )( )出字 再出字ㅗ ㅏ ㅜ ㅓ ㅛ ㅑ ㅜ ㅕ

병 그리고 들 에 다시( )ㅘ ㅝ ㅣ

( )ㅣ ㅢ ㅚ ㅐ ㅟ ㅔ ㆉ ㅒ ㆌ ㅖ ㅙ ㅞ

zb52) 가 에 대한 설 으 르 않은 것을( ) 두 고르

① 어쓰 규 지키고 다

② 리 고 다

③ 말 미 미 등 어 사 다lsquo rsquo

④ 개 지 다

년 학 간고사 대비2013 2 현대고 대비

ECN-0102-2013-001-000076193

⑤ 어 원 에 가 도 고 다

엉 훈 민世 宗 御 製 訓 民 正 音

말 미 듕귁에 달 니

런 어린 니 고 도middot

내 들 시러 펴 몯 미 니middot

내 어엿 겨 새 믈여듧

사 마다 니겨middot 킈 고

미니

훈민 언 본- lsquo rsquo 5 (1459 )

zb53) 위의 에 대한 현대어 풀이가 르~ 않은 것

① 우리 말 과 달

② 어리 말 고 는 것 어도

③ 신 생각 마 껏 펼 는 사 많다

④ 게 생각 여

⑤ 사 마다 게

zb54) 훈민정음 언해 에는 한 을 창제한 동 가 드러나

있다 훈민정음 창제의 정 과 내 이 잘 연결된 것

① 주 신 말 미 듕귁에 달

② 민 신 내 어 겨

③ 신 뻔 킈 고 미니

④ 실 신 사 마다 니겨

⑤ 귀 신 계 주 는 훈민 신과 거리가

가 엉 훈 민( ) middot middot middot middot middot世 宗 御 製 訓 民 正 音 

말 미 귁에 中 國 달 文 字

니 런 어린 니 百 姓

고 도 내 들 시러 펴 몯

미 니 내 어엿 爲 겨 새

믈여듧 니 사 마다 니 字

겨 킈 고 미니 便 安

훈민 언 본- lsquo ( )rsquo ( ) 5 (1459 )訓民正音 世祖

( )

[ 1 ]

동 룡 샤 마다 복( ) ( ) ( )海東 六龍 天福

시니 고 동( ) ( )古聖 同符 시니

[ 2 ]

매 니 곶 여

미 므 니 그 내 러

가 니

[ 125 ]

우 미리( )千世 샨( )定 에( )漢水北 累仁

누 개 샤 복 업 시니( ) ( ) 開國 卜年

신( )聖神 니 샤도 경 근민 샤 욱( )敬天勤民

드시리 다

님 쇼 산 가( ) ( )洛水 山行

미드니 가

어 가- lsquo ( )rsquo 27龍飛御天歌

다 우리신 니쓰고 다만 만 쓰( )

거 샹 귀쳔 다보게 러 귀

여 쓴 도 신 보 가 고 신 에

말 어 보게 각 에 사 들

고 본 몬 능통 후에

죠 죠 니

드 도 만 공 에 사

드 미 죠 고 고 여 보 죠

보다 얼마가 거시 어신고 니 첫

가 죠 니 죠

민 들 어 신 샹

귀쳔 도보고 어보 가 만 늘

고 폐 에 만쓴 죠 민

도 러보지못 고 보니 그게 엇지

심 니 리 보 가 어 운건 다

니 쳣 말마 지 니 고 그

쓰 에 가 우 지 지

몰 거 본후에 가 어 지

고 그니 쓴편지 쟝 보

년 학 간고사 대비2013 2 현대고 대비

ECN-0102-2013-001-000076193

쓴것보다 듸 보고 그 마 니 쓴 고

어 못

그런고 에 리 과 가

만 쓴 못 민 말만 듯고

고 편 그 못 보니 그사 단

병신 못 다고 그사 식 사

니 만 고 다 과 그사

만 고 다 과 업 사 보다 식 고

죠 도 고 각 과

견 고 실 직 귀쳔 간에 그

고도 다 것 몰 귀죡 보다

사 우리 신 귀쳔 다 업

시 신 보고 과 지 게 랴

시니 샹 귀쳔 간에 우리 신 걸

간 보 새지각과 새 걸 미리

독립신- lsquo (1896)rsquo

zb55) 친 어 나의 제 장( ) 2 매 함축적

의 가 가장 유사한 것은

① 지 눈 내리고 매 득 니 내 여 가

사- lsquo rsquo

② 도 어 리듯 그 게 어 다

주 사- lsquo rsquo

③ 눈 살 다 죽 어 린 과 체 여

눈 새벽 지 도 살 다

눈- lsquo rsquo

④ 삶 근심과 고단 에 돌 거니는 여 거 는

여 리 내린 살가지 에 눈 리 눈 리

택 그 생 에- lsquo rsquo

⑤ 늘 러 고 러

청룡 룡 어 개 루 우

신경림 계- lsquo rsquo

zb56) 친 를 위 가 나 에 나타난A B ( ) ( )

세 어의 특 에 의거하여 세 어 표 하

그 산 고 공 도 맑지만

A

주변에 쓰 리는 어리 사 많다

B

건lt gt

식 가 에 타 어 특징에( ) ( )

거 과 어쓰 는 고 지 말 것

A

B

zb57) 가 의( ) 달 아ㆍ 다 의 ( ) 나셔에서 알 있는

세 어 개화 어의 특 을 비 하여 조건 에lt gt

맞게 서 하

건lt gt

어에 는lsquo 개

어에 는 다 태rsquo

zb58) 은 가 는 다 에 나 는 절lt 1gt ( ) lt 2gt ( )

일 를 췌한 것이다 의 의 가 lt 1gt (1)~(2)

유사한 말을 에서 찾아 쓰lt 2gt

보lt 1gt

런 (1) 어린 니 고百 姓

도 내 들 시러 펴 몯 미

사 마다 (2) 니겨 便 安

킈 고 미니

보lt 2gt

죠 고 고 여 보 죠

보다 얼마가 거시 어신고 니 첫 가

죠 니 죠 민

들 어 신 샹 귀쳔

도보고 어보 가 만 늘 고

폐 에 만쓴 죠 민 도

러보지못 고 보니 그게 엇지 심

니 리

년 학 간고사 대비2013 2 현대고 대비

ECN-0102-2013-001-000076193

lt 1 gt

동 룡 샤 마다 복 시( ) ( ) ( )海東 六龍 天福

고 동 시니( ) ( )古聖 同符

lt 2 gt

(A) 매 니 곶

여 니

미 므 니 그 내

러 가 니

lt125 gt

우 미리 샨 에( ) ( ) ( ) 千世 定 漢水北 累

누 개 샤 복 업 시 니( ) ( ) 仁開國 卜年 聖

신( ) 神 니 샤도 경 근민 샤( ) 敬天勤民

욱 드 시 리 다

님 쇼 산 가 ( ) ( )洛水 山行

미드니 가

- lt gt龍飛御天歌

zb59) 장과 내 상 유사한 성격의 조는125

① 뫼 고 고 고 고

어 그린 많고 많고 고 고

어 러 는 울고 울고 가느니

도 견- lt gt

② 강 에 드니 몸 다

그믈 고 가니

뒷 뫼 엄 언 니( )藥

-

③ 말 없는 청산 태 없는 다

값 없는 청 없는 월

에 병 없는 몸 별 없 늙 리

-

④ 가마귀 골에 가지 마

낸 가마귀 새

청강에 것 시 몸 러 가( ) 淸江

-

⑤ 진 골에( ) 白雪

가 매 는 어느 곳에 었는고

에 갈 곳 몰( ) 夕陽

색-

zb60) 위 에 나타난 세 어의 특 으 적절하 않은

것은

① 룡 어 주격 사에 당 는 가 사( ) lsquo rsquo六龍

고 다

② 샤 어에도 어 주체 쓰 다

는 것 다

③ 매 어 달리 사 택에 어

가 지 지지 고 다

④ 므 원 상 직 어 지 다

⑤ 드시리 다 주체 과 상 께 사

고 다

수고 하셨습니다hearts hearts

년 학 간고사 대비2013 2 현대고 대비

ECN-0102-2013-001-000076193

보닷컴에 공 는 별 보는 고등

들 여 주 는

들 습니다 슷 동 지

가 복 는 것 도가

니 복 여 습 시고 거 시

니다

정답 해설

1) 정답[ ] ④

해설 다른 것은 두 특정 업이나 단 내에서 사[ ]

하는 일종의 은어 사회 언에 해당한다 러나

는 언이 아니라 단과대학을 여서 단대 사lsquo rsquo lsquo rsquo lsquo④

대학을 여서 사대라고 한 말에 해당하 일rsquo lsquo rsquo

사회에서도 널리 쓰이 사회 언이라 할

없다

2) 정답[ ] ⑤

해설 사회 언은 같은 단 내에서 쓰이는 언어이[ ] lsquo rsquo

동일 단끼리는 단결 과 친 감을 형성하는

능을 하 리적 안감이 일어나 않는다

3) 정답[ ] ③

해설 사람이라는 차 적 표현에 대한 대안적 표현이[ ]lsquo rsquo

인 아내 처 등으 볼 있다lsquo rsquo

4) 정답[ ]⑤

해설 남성은 주 격 체를 사 한다[ ]

5) 정답[ ] ⑤

해설 흑인은 검다라는 뜻을 가 고 있을 뿐 인[ ]lsquo rsquo lsquo rsquo lsquo rsquo

다 열등한 뜻을 내포하 않는다

6) 정답 살 색 첫 작품[ ] - -

해설 살색 혹은 킨색은 한 인의 피 색을 뜻[ ] lsquo rsquo lsquo rsquo

하는 것으 인종 차 을 추 고 출 이주민

의 평등 을 침해할 있어 년 표 이2005

살 색으 이름을 꾸었다 처녀작은 처녀라lsquo rsquo lsquo rsquo lsquo rsquo

는 단어가 가 고 있는 곡된 성 인 을 한 것

으 첫 작품정도 꾸어 사 하는 것이 좋다lsquo rsquo

7) 정답[ ] ⑤

해설 호는 아들에게 해체를 사 하고 있다[ ] ① ②

장 을 성하는 청자는 자 의 아 느리 아lsquo

들 세 이다 호는 아 느리에게 해rsquo ③

체를 사 하고 있다 호가 느리 아 에게 ④

사 한 해 체 아들에게 사 한 해체는 두 비lsquo rsquo lsquo rsquo

격 체에 해당한다 호는 자 의 아랫사람인 ⑤

느리에게 아들과 마찬가 해체를 사 하는 것이

상 이 만 임 을 한 느리에게 고마 과 쁨

존 의 표 를 하 위해 자 의 아 에게 말하듯

해 체를 사 하고 있다

8) 정답[ ] ③

9) 정답[ ] ⑤

10) 정답[ ] ①

해설 청자 할아 가 장의 주체 아 다 높을[ ] ( ) ( )

경 에는 압존 에 의해 장의 주체를 높이 않는lsquo rsquo

다 러 아 서가 아닌 아 는으 계 lsquo rsquo lsquo rsquo lsquo

니다 가 아닌 있 니다 표현하는 것이 르rsquo lsquo rsquo

11) 정답 당이 당을 쫒았다 당이[ ]

당에 다

해설[ ]

12) 정답[ ] ⑤

해설 서 다른 높임표현을 통해 청자에 대해 리[ ] ⑤

적 거리감을 나타내는 인 은 이 아니라 현정이

다 가 에서 현정은 에게 해 체를 사 함으 써 ( )

친근감을 드러낸다 나 에서 연 을 게을리하는 역 ( )

도 들 때 에 화가 난 현정이 선생님에게 항의하

는 장 에서는 하 체를 사 하여 리적 거리lsquo rsquo

가 어졌음을 나타내고 있다

13) 정답[ ] ①

해설 는 는 얼 빛이 날과 어찌 다르 고[ ] lsquo rsquo

라는 뜻으 전과 달리 임이 화자를 않고

있음을 알 있다

14) 정답 달리 후 가 있다 이를 통해 경[ ] lt gt

쾌한 음악성을 형성하고 노 젓는 상황을 체적으

형상화하는 역할을 한다

15) 정답[ ] ①

16) 정답[ ] ⑤

해설 다 의 자연은 를 성찰하게 하는 대상[ ] ( )⑤

이자 정의 대상이다 의 자연은 자 의 상황과 ⑤

처 를 드러내는 경으 서의 역할을 하 이

이 없다

17) 정답[ ] ③

해설 는 빈천 을 해결하고자 했으나 강산[ ] lsquo ( )rsquo 貧賤③

과 풍 을 달라는 에 거절하 다고 함으 써 자

연에 대한 애정을 드러내고 있으 는 않는

임에 대한 망을 개에게 전가 켜서 임에 대한 리

을 드러내고 있다

18) 정답[ ] ③

년 학 간고사 대비2013 2 현대고 대비

ECN-0102-2013-001-000076193

19) 정답[ ] ⑤

해설 고상한 음악가의 이름을 리말 꽝 럽[ ]

게 꿈으 써 언어유희를 통해 음을 유 하고 있

다 이는 고상한 척하는 총 를 비꼼으 써 비판적

태도를 드러내는 것이 대상을 꽝 럽게 표현

하여 총 의 허 과 사치를 풍자하고 있다

20) 정답[ ] ⑤

해설 는 작품 속 경에 대한 설 이 드러나는 것이[ ]

서 자의 주 적인 견해가 접적으 드러나는 것이

아니다

21) 정답[ ] ⑤

22) 정답[ ] ②

23) 정답[ ] ④

24) 정답[ ] ①

해설 적강 티프는 주인공의 비 한 출생이나 능[ ] ①

과 이 있는 것으 조정의 능함을 풍자하는lsquo rsquo

것과는 거리가 다

25) 정답 픔 나[ ] ( )

해설 의 음악은 고통 는 사람들을 위 하고 아픔[ ] lsquo rsquo

을 치유해 주는 능을 한다고 할 있다 의 lt gt

픔 도 소 된 이 과 더 어 살아가는 따뜻한 마음lsquo rsquo

을 상 한다

26) 정답[ ] ⑤

해설 에게 선천적으 주어 각 장애라는 역경[ ]

은 의 이라는 가사 연 을 있다lsquo rsquo

27) 정답[ ] ④

해설 는 장 란 선 에게 은 개인적인 인상을[ ]

소녀 장정 등으 표현한 것이다lsquo rsquo

28) 정답[ ] ②

해설 담자가 피 담자의 언어적 표현이나 비언어[ ]②

적 표현 하 독자는 담의 위 나 피

담자의 감정 상태를 알 있다 이를 통해 독자는

담 상황을 더 생생하게 느낄 있고 피 담자

를 더 잘 이해할 있게 된다

29) 정답[ ]③

해설 일상생활과 역도 선 서의 성과에 된 것에서[ ]

역도를 하 서 겪는 어 과 내적 고민으 화제를

전화하 위한 것이다

30) 정답[ ] ①

해설 릿속에 새겨 넣듯 이 억되도 함 세상[ ] ② ③

살이가 힘들고 고생 러 속 하여 자유를 ④

가 없는 고통의 상태를 비유적으 이르는 말

적의 침입을 막 위해 쌓은 축 켜야 할⑤

대상을 비유적으 이르는 말이다

31) 정답[ ] ④

해설 이 의 종류는 전 으 인 사건 경[ ] lsquo

비평을 성 소 삼는다rsquo

32) 정답[ ] ④

해설 근은 삼대독자 태어났음을 에서 확인할[ ]

있다 형제들과의 담은 이뤄 가 없다

33) 정답[ ] ⑤

해설 근은 가난에도 하고 화가를 꿈꾸었다[ ] (3

단 또한 다른 화가 망생들은 정 육을)

위해 상 학 학 해 유학 에 랐 만

근은 다른 을 찾아야 했다 단 세에(5 ) 18

근은 조선 전람회에 입선하 다 단 의(6 )

만종은 인간과 자연이 엮어 가는 경건한 조화 을lsquo rsquo

나타낸다

34) 정답[ ] ①

해설 근이 속에서도 창작활동을 추 않고[ ]

하는 닭은 은 세상과 타협할 르는

근이 세상의 이해를 하 위한 가장 떳떳한 단

이 때 이다

35) 정답[ ] ⑤

해설 전 은 서 자의 주 적인 평이 리는 것이[ ]

만 위 제 은 인 이 살았던 대 사회적 경

을 통해 객 적인 인 의 을 제 하고 있다

36) 정답[ ] ⑤

해설 전 은 인 사건 경 비평이라는[ ] lsquo rsquo⑤

성 이 어져 있다

37) 정답[ ] ①

해설 이 은 동양인과 서양인의 사고 에 차이가[ ]

있다는 것을 대조를 통해 설 하고 있다 또 쓴이

의 제자가 축 경 를 러 가서 경험한 일화를

통해 동양인이 서양인에 비해 주 상황에 더 많은

주의를 인다는 주장을 뒷 침하고 있다

38) 정답[ ] ④

39) 정답[ ] ②

40) 정답[ ] ②

41) 정답[ ] ④

42) 정답[ ] ③

43) 정답[ ] ④

44) 정답 도서 의 휴 일 도서 의 이 간 도서의[ ]

해설 도서 장은 임의 정한 휴 일과 도서 이[ ]

간 도서의 상 등을 게 할 의 가 있다

년 학 간고사 대비2013 2 현대고 대비

ECN-0102-2013-001-000076193

45) 정답[ ] ①

해설 제 조의 정 휴 일 의 휴 일의 사전 게[ ] 3

는 도서 장의 의 조항에 속한다

46) 정답[ ] ①

해설 개인 정 호 의 를 제 하 했 만 항[ ]

나눠서 제 하 않고 대 나열하고 있다

47) 정답[ ] ②

해설 제 조의 내 을 회사는 다른 회사 협[ ] 7 lsquo

계약을 통해 서비 를 제공하는 경 회 의 아이디

등 개인 정 를 해당 회사에 전송할 있다는 내rsquo

이 있으 의 제점을 제 할 있다②

48) 정답[ ] ④

해설 는 도서 장의 의 에 해당하고 나 는 도[ ] ④

서 장의 리에 해당한다

49) 정답[ ] ③

50) 정답 은 음독으 적었고 은 훈독으 적었[ ] (1)

다 과 동일한 표 리 적은 것은 이고 (2) ce

과 동일한 표 리 적은 것은 이다ab

51) 정답[ ] ③

52) 정답[ ] ①②

53) 정답[ ] ③

54) 정답[ ] ③

55) 정답[ ] ①

56) 정답 른 죠코 어린 노 하니라[ ] A B

57) 정답 세 어에서는 활 형이 칙적으[ ] lsquo rsquoㄹㅇ

나타났 만 개화 어에서는 활 형이 쓰 다 lsquo rsquo ㄹㄴ

58) 정답 호 가 흔[ ] (1) (2)

59) 정답[ ] ④

60) 정답[ ] ③

Page 10: 현대고대비 국어 - chamsoriedu.com 「콘텐츠산업진흥 법」외 에도 저작권 의하여 ... 다른주체에게어떤동작을하도록만드는것을나타내는

년 학 간고사 대비2013 2 현대고 대비

ECN-0102-2013-001-000076193

고 가에 내 가니 울 리가 에 지 사

쳤는지 심 심 것 가

신 심 것 가

다( ) 강 승상에게는 들 없고 다만 만

었다 가 낳 에 가 색

타고 내 에게 말 는 ldquo

니다 미원 과 연 맺고 ( ) ( )紫薇垣 緣分

었는 께 강 집 보내 에

니 게 여겨 주십시 거늘 rdquo

미 가운 낳 니 가 고 거동

단 다 시 짓 쓰 고 는 (音

없었 니 여 가운 지 는 짝) 律

룰 만 사 없었다 가 사 여 사 감

게 고 지 못 고 염 는 만다

다가 당에 거 고 식같 러 내니

고귀 상 루 말 다 어 울 도 다( ) 相

귀 사 없고 웅 걸( )富貴爵祿

만고 었다 승상 매우 뻐 내당 ( )內堂

들어가 에게 사 니 역시 매우 거

워 말 다

ldquo 도 마 사 는 승상께

그 게 말 시니 상 여러 말 지 말고

사 도 시다rdquo

상이 에 나 충 의 손을 잡고 결혼과 하여 ldquo

너에게 히 할 말이 있다 내가 늙은 말년에 딸

하나만을 두었는데 니 너 하늘이 정해 필

임이 하다 이제 년고락 을 너에게 탁 ( )百年苦樂

하겠다 하 대 충 이 릎을 꿇고 앉아 눈 을 흘리rdquo

여쭈었다

소자의 을 해주 고 또 하 에 두고자 하ldquo ( )膝下

니 감사하 이를 데가 없 니다 다만 가 속에 통탄

할 일이 사 쳐 있 니다 소자가 이 없어 양친 ( )兩親

의 생사를 른 채 결혼하여 아내를 얻는 것은 자 으

서 할 도리가 아닙니다 이것이 한 러 뿐입니다 rdquo

승상 그 말 듣고 슬 에 어 고

것 에 맞 어 변 게 리ldquo

는 다 집 시 공 도 여 ( )始祖公

고 가 에 가가 어진 만 개 공신

었 니 도 러워 말 시고 시 rdquo

택 여 니 다운 신 과 신

습 늘에 죄 짓고 간 상에 내 신

혼 를 다 끝내고 으 들어가 사 을 살펴 니 빛

나고 빛난 것이 한 입으 는 다 말하 어 고 하나

는 다 하 어 더라 에 켠 환한 촛 ( )新房

아래 은 에 랑과 가 평생의 연 을 맺었( )緣分

으니 서 사랑하 주고 은 말을 어떻게 다 헤아릴

있으 어떻게 다 하리 을 낸 후에 이튿날

상 를 니 상 거 마음을 이 하

더라

각 생 강 승상 집 쪽( )

늘 보고 없 가 신 신 생각 니

없고 어 없었다 는 어떻게 도리가 없다

여 산 에 들어가 리 고 어 도 닦

고 다 그 산 보고 가다

가 곳에 다다 니 에 큰 산 었다 많 우

리 골짜 가 늘 는 가운 색

에 고 갖가지 가 짝 어 었 ( )花草

다 신 산 생각 고 들어가니 경개 ( )景槪

가 매우 뛰어 고 경 산 다 산 리에 들

리는 것 리 보 는 것 울 청산뿐

었다 가 고 울 어 가

니 들 많 가지들 못 어 동

에 늘어 들거리 는( ) 洞口

우거진 가지에 갖 들 다 었다( ) 春情

계상 에는 공 는 늘( ) 花溪上

에 걸린 폭포가 벽 는 리는 산사( )層巖絶壁

쇠 리 객 에 는 듯 늘( ) ( ) 寒山寺 客船

에 싸여 는 습 산

그린 여 병 러 듯 다 경쇠 리가 들

리 에 들어가니 색 에

게 단청 누각과 큰 집들 다( ) 丹靑

주 보니( ) ( ) lsquo一柱門 黃金大字

산 사 어 었다 산 들rsquo ( )山門

어가 고승 다 그 거 보니( ) 高僧

눈 눈 듯 고 변 같 ( )白邊

귀는 어 에 늘어 니 맑고 어 골격

과 신 평 니었다 염주

에 걸고 짚고 포 삼에 어진( )六環仗

쓰고 생 보고 말

승 연 여 상공 시는 동 에ldquo

가 맞 지 못 니 승 십시 rdquo

생 크게 말 다

생 가 여 어 고ldquo

없 다니다가 우연 곳에 사 만 것

그 시 생 어떻게 고 습니 rdquo

승 답 여 말

어 산 승 에ldquo ( ) ( )南岳 衡山

시어 승에게 탁 내 낮 시경에 경 lsquo 12

동 에 사는 심 들 가 것 니 내쫓

지 말고 습니다 마 승rsquo

다가 상공 림새 보니 경 사 에 보

습니다rdquo

zb20) 위 의 친 에서 서 자의 개입이 드러나~

는 이 아닌 것은

① 달 가 도리어 걸 었( ) 奇男子

② 신 심 것 가

년 학 간고사 대비2013 2 현대고 대비

ECN-0102-2013-001-000076193

③ 다운 신 과 신 습 늘에 죄 짓고

간 상에 내 신 다

④ 사 주고 말 어떻게 다 헤 릴

어떻게 다 리

⑤ 신 산 생각 고 들어가니 경개 가 ( )景槪

매우 뛰어 고 경 산 다

거리 연[ ] ( )弘治

간 에 공신 후 에 언(1488~1505) ( )正言

주 는 벼슬 심 늦도 식( ) ( )主簿 劉尋

없어 과 께 산에 드리고 신 태

몽 꾼 에 만고 웅 상 지닌 들 낳

키운다 그 후 신 들 에 역심( )逆心

담 귀 등 심 여 리 귀 보내

고 지 죽 는 도망 가다가

만 다 에 에 어 니

헤어지게 다

에 에 어 니 헤어지게

다 그 후 사 들에게 우연 돌

생 다가 어느 열 살 었다 열 살

지 다가 우연 귀 견 는

그것 그 살 도 었고 그

귀 본 신도 지 죽고 마 고

크게 운다

( )

에는 강 주 는 재상 살고 었

니 시 에 과거에 격 여 승상 벼슬 다가 간

신 만 벼슬 그만 고 고 돌 었

다 그러 신 지 가 지 못 여 상

가 못 결 는 상 여 원 니

신 들 그 직간 꺼 다 그 에 도

담과 귀가 강 승상 가 미워 다

강 승상 마 본 에 갔다가 돌 는[A][ ( )本府

에 우편 주 에 다가 색( ) ( ) 右便 酒店

에 어리었는 청룡 에 지 늘

여 통곡 고 사 는 꿈 꾸었다] 마

상 게 생각 여 새 다리다가 새벽

닭 울고 가 달 갔다 가 보니

과연 어 동 가 가에 울고 는지 달

들어 그 고 사 에 어 말

는 어 어 에 어 가ldquo

닭 곳에 우느냐 니 울rdquo

그 고 답 여 말 다

는 경 동 에 사는 언 주 공 들ldquo

니다 께 간신 만 연경 귀 가

시다가 에 죽 사 에 는 닭에

도 에 죽고 니다rdquo

강 승상 말 듣고 크게 낯 변 말

것 웬 말 냐 근 동 ldquo (老

못 갔 니 그 사 변 여)患

런 변 었단 말 가 주 는 신 다

같 에 벼슬 다가 는 가 많 들어 고

돌 는 주 가 게 꿈 에 생

각 겠느냐 생각지 못 다 미 지 간

지지 말고 께 가 략rdquo ( ) hellip hellip

죽게 주 사당에 단 도 러운

겠느냐 말 말고 시는지 rdquo

어 없어 강 승상 가니 그곳

월계 었다

다( )

가가 고 지 사 들 가( )櫛比

통 는 리가 과

답게 꾸민 누각과 큰 집들 늘 고

게 식 가 어 들 태운 가고

었다 략 강 승상에게는 들 없고 ( ) hellip hellip

다만 만 었다 가 낳 에

가 색 타고 내 에게 말

는 니다 미원 과ldquo ( )紫薇垣

연 맺고 었는 께 강 집( )緣分

보내 에 니 게 여겨 주십시

rdquo

거늘 미 가운 낳 니 가

고 거동 단 다 시 짓 쓰 고

는 없었 니( )音律 여 가운

지 는 짝 룰 만 사 없었다 가 사

여 사 감 게 고 지 못 고 염 는 만다

다가 당에 거 고 식같

러 내니 고귀 상 루 말 다 ( )相

어 울 도 다 귀 사 없 ( )富貴爵祿

고 웅 걸 만고 었다 승상 매우 뻐

내당 들어가 에게 사 니( ) 內堂

역시 매우 거워 말 다 도 마 ldquo

사 는 승상께 그 게 말 시니

상 여러 말 지 말고 사 도 시다rdquo

( )

승상 에 고 결 과 ldquo

여 에게 말 다 내가 늙 말 에 지

만 었는 지 보니 늘

다 에게 탁 겠 ( )

다 신 꿇고 눈 리rdquo

여 었다 주시고 슬 에 ldquo ( )膝下

고 시니 감사 룰 가 없습니다 다만 가슴

에 통탄 사 쳐 습니다 복 없어

생사 결 여 내 얻는 것( )兩親

식 도리가 닙니다 것 러울 뿐 니

다rdquo

상 그 말 듣고 슬 에 어 고 말

것 에 맞 어 웅변 ldquo

년 학 간고사 대비2013 2 현대고 대비

ECN-0102-2013-001-000076193

게 리 는 다 집 시 공도 여

고 가 에 가가 어진 만 개 공신

었 니 도 러워 마 시고 rdquo 시

택 여 니 운 신 과 신

습 늘에 죄 짓고 간 상에 내 신

다 략 지낸 후에 튿 승상 ( ) hellip hellip

니 승상 거운 마 지 못

마( )

듯 월 러 생 열다 살 었

다 에 승상 어진 사 얻고 만 에 근심 없었

다만 주 가 간신 에

죽 것 생각 마 곧 어 곤

다 그 에 주 원통 어

없 고 여 시 가 거늘 생 만

여 다

말 감격 러우 간신 에 가득 여ldquo

고 니 께 상 듣지 니 것

니다rdquo

승상 듣지 고 가

퇴 재상 공달 집에 거 고 상 지어

승지 러 께 리

( )

뒷 거리 강 승상 에게 상 리지[ ]

만 여움 사 귀 가게 다 강 승상

몸 는 연 가 헤어

리 다 산 들어간 룡사 승 만

게 다 승 만 우 다릴

과 들고 략 다 담

원 여 에게 복 고 어

공격 다 담에게 여러 가( ) 天子

복 등 여 다 단

신 리쳐 담 사 고 에게

간 후 태후 태 여 지에 고생

지 심과 강 주 여 개 다 헤

어 어 니 내 고 담 리

벼슬에 귀 누리게 다

zb21) 위 의 인 간 계를 같이 나타냈을lt gt

때 에 대한 이해 가장 적절하 ~ 않은 것은

① 계에 주 는 계 심 열

상 에 다고 다( ) 水深火熱

② 계는 견원지간 고 다( ) 犬猿之間

③ 계는 달리 막역지 계 고( )莫逆之交

④ 연결 사 컬어 재 가 고( )才子佳人

⑤ 는 생 과 볼 ( )匹夫匹婦

가 재 는 는 심 고 매사에 생( )

각 고 능 도 어 가 에게 많lsquo rsquo

도움 사 다 그는 에게 거 에

꺼리 없 거 났다고 는

매우 싫어 고 신 들

는 사 다

내가 지 리에( ) 1970

사 실에 지 월간ldquo

편집 고 어 었다rdquo

어느 없 가 쑥 다 도 어 10

후 다 산 시 럼 어 엇 어 ( ) lsquo怡山

다시 만 랴 니 그는 재 그룹 승 운rsquo

사가 고 는 고 거 누

주는 가 없는 가가 어 다시 만 게 것

었다

다 보통 것 닐러 그( ) ldquo 어낸 ( )

틀어주 가 루 러 허 에

싶어 키 틀어주 그( )

가 루 허 우간 곡 틀어 주는 루 못

는 는 고 닝께 고 지

들어 사는 고 가 다는 건 에 그 집에

rdquo

그런 단 어들 어 새벽에 떼죽 거

다 고 어 보니 죄다 허 게 집어진

는 것 었다 가 실내 꿴 뛰어 지만

없는 었다

어떻게 된 거야 한동안 넋나간 듯이 서 있던 총ldquo rdquo

가 하고많은 사람 에 하필이 유자를 겨냥하 은

말이었다 쎄유 아마 새에 고뿔이 들었던 개비네 ldquo

유rdquo

유자는 러 딴청을 하 다 야 고 가 에서 ldquo

감 가 들어 죽는 고 두 어rdquo 총 는 가 혐의

자 나 되는 것처럼 화풀이를 하 드는 것이었다( )嫌疑者

라 이 어쩌 어 유( ) ldquo rdquo ldquo rdquo

애유 이런 잔인 도 한 것들 같으니ldquo ( ) rdquo殘忍無道 helliphellip

총 는 탱천 하여 쩌 를 하 다( ) 憤氣撐天

아하니 아는 자는 다 동 하여 호통을 쳤으 하나 혈

압을 생각하여 참는 눈치 다 달리 처리헐 두 ldquo

잖은감유rdquo

총 의 성 을 덧들이 고 한 말이 아니었다 가 할

년 학 간고사 대비2013 2 현대고 대비

ECN-0102-2013-001-000076193

있는 것이 말고는 없었 때 에 게 뒷동

산을 달은 거 다

이 유자소전- lsquo rsquo

zb22) 의 상황을 속담으 표현한 것으 적절한 것은

① 루 곳 게 마 다

② 에 맞고 강에 눈 다

③ 늘 도 다

④ 도 사 다

⑤ 에 가도 신만 리 다

거리 공신 후[ ]

에 주 는 벼슬 심 늦도( )主簿

식 없어 과 께 산에 드리고 신

태몽 꾼 에 만고 웅 상 지닌 들

낳 키운다 그 후 신 들 에 역심

담 귀 등 심 여 리 귀 보내고

지 죽 는 도망 간다 그

만 고 에 에 어 니

헤어지게 다 지 가 사 들에

사 들 별 고 없 다니었다

마 마 돌 다니 걸 여 고

어 곤 다 에는 동쪽에 고 에

는 쪽에 니 가 에 리는 엽 가는

없 니 늘 다니는 었다

얼 말 죽 사 같고 림새가 말 니었

다 가슴 에 고 등 삼태

헌 에 니 달 가 도리 ( )奇男子

어 걸 었 담 만 열 도 ( ) ( )傅說 慇

고 만났고 만 갈( ) ( )武丁 伊尹

도 탕 만났( ) ( )成湯 渭水

여상 도 주 만났는 월( ) ( ) ( ) 呂尙 周 文王

같 러가 도 어느 열 살

늘과 집 삼고 사 에 쳐 거리에

어 다가 곳에 니 다 ( ) 楚

지 다가 사 보고 가에 다다( )長沙

니 망 가에는 원 리가 슬 고 가

가 내리는 사 에는 갈매 가 갈 뿐 었다

쪽 돌 보니 가 우거 고

가 사 보 었다 그곳에

가니 는 사( ) 汨羅水

는 다 주 가 쓰고 죽고

곳 었다

마 감 여 에 가 사 살펴보니

에는 삼 고 그 에( ) 屈三閭

는 만고 월 과 지 가는 그 들( )風月

가 어 었다( ) 路程記

동쪽 벽 에 새 운 어 거늘 그

보니

월 에 경 주 는 간신에게ldquo ( )敗

보고 연경 귀 가다가 에 죽 rdquo

거늘 그 보고 에 거꾸러

통곡 말

우리 연경 간 만 니ldquo ( )燕京

에 지 살 상에 엇 겠는

가 에 고 에 었 니

상에 살 것 가 도 께 지리 rdquo

고 가에 내 가니 울 리가 에 지

사 쳤는지 심 심 것 가

에는 강 주 는 재상 살고 었

니 시 에 과거에 격 여 승상 벼슬 다가 간

신 만 벼슬 그만 고 고 돌 었

다 그러 신 지 가 지 못 여 상

가 못 결 는 상 여 원 니

신 들 그 직간 꺼 다 그 에 도

담과 귀가 강 승상 가 미워 다 강 승상 마

본 에 갔다가 돌 는 에 우편 주( ) ( )本府 右便

에 다가 색 에 어리었는 청룡( ) 酒店

에 지 늘 여 통곡 고

사 는 꿈 꾸었다 마 상 게 생

각 여 새 다리다가 새벽닭 울고

가 달 갔다 가 보니 과연 어 동 가

가에 울고 는지 달 들어 그

고 사 에 어 말

는 어 어 에 어 가ldquo

닭 곳에 우느냐rdquo

니 울 그 고 답 여 말 다

는 경 동 에 사는 언 주 공 들ldquo

니다 께 간신 만 연경 귀 가

시다가 에 죽 사 에 는 닭에

도 에 죽고 니다rdquo

강 승상 말 듣고 크게 낯 변 말

것 웬 말 냐 근 동ldquo ( )老患

못 갔 니 그 사 변 여 런 변

었단 말 가 주 는 신 다 같

에 벼슬 다가 는 가 많 들어 고 돌

는 주 가 게 꿈 에 생각

겠느냐 생각지 못 다 미 지 간 지지

말고 께 가 rdquo

뒷 거리 강 승상 도움 죽 고[ ]

년 학 간고사 대비2013 2 현대고 대비

ECN-0102-2013-001-000076193

고 그 과 결 여 사 가 다 그러 강

승상 에게 울린 상 강 승상 귀 가고

과 헤어 리 승 만 게 다

승 우 다릴 과

들고 략 다 담 원

여 에게 복 고 어 (天

공격 다 담에게 여러 가 복) 子

등 여 다 단신

리쳐 담 사 고 에게 간

후 태후 태 여 지에 고생 지

심과 강 주 여 개 다 헤어

어 니 내 고 담 리 벼

슬에 귀 누리게 다

미상- lsquo ( )-劉忠烈傳

zb23) 위 과 의 서사 조를 비 한 것으 적절하lt gt

않은 것은

보lt gt

믿지 고 결 여 곱

낳 다 곱째 공주 낳 가

리게 다 리 만 고 진 공주는 lsquo rsquo

리공 미 리공 에 키워진다 월

러 과 가 죽 병에 걸 는 승에 는

어 산다고 다 여 들에게 탁

지만 거 리 는다 리 는 과

승 다 승 지 가는 에 많

만 지만 보살 도움 사 도 다

그러 승 신과 결 여 시

들어 주겠다고 다 리 는 그 결

여 들 곱 낳 후에 신

얻게 다 돌 리 는

에 과 상여 만 지만 여 과

살 낸다 훗 리 그 공 우 죽 사

승 도 는 신 다

리-lsquo rsquo-

① 복 결말에 고 다

② 웅 에 탕 고 다

③ 시 겨 내고 귀 누리는lsquo rsquo

보 리 는 월 재 신 다lt gt lsquo rsquo

④ 과 보 리 는lsquo rsquo lt gt lsquo rsquo

도움과 어 신 능 극복 고

⑤ 등 여 시 겪는lsquo rsquo

보 리 는 닌 지lt gt lsquo rsquo

림 시 겪는다

가 각 고 에( ) ( )却說

살 없었다 략 사 들 슬 에 어 lt gt

가에 내 고 가고 싶 가 고 후

워 경

사 들 별 고 없 다니었다 lt

략 얼 말 죽 사 같고 림새가 말gt

니었다 가슴 에 고 등

삼태 헌 에 니 달 가 ( )奇男子

도리어 걸 었 담 만 열 도( )傅說

고 만났고 만 갈( ) ( ) 殷 武丁

도 탕 만났( ) ( ) (伊尹 成湯 渭

여상 도 주 만났는) ( ) ( ) ( )水 呂尙 周 文王

월 같 러가 도 어느 열 살

늘과 집 삼고 사 에 쳐 거리에

어 다가 곳에 니 다 ( ) 楚

지 다가 사 보고 가에 다다( )長沙

니 망 가에는 원 리가 슬 고 가

가 내리는 사 에는 갈매 가 갈 뿐 었다

쪽 돌 보니 가 우거 고

가 사 보 었다 그곳에

가니 는 사( ) 汨羅水

는 다 주 가 쓰고 죽고

곳 었다

에는 강 주 는 재상 살고( )

었 니 시 에 과거에 격 여 승상 벼슬 다

가 간신 만 벼슬 그만 고 고 돌

었다 략 강 승상 마 본 에 갔다가 돌 lt gt ( )本府

는 에 우편 주 에 다가 색( ) ( ) 右便 酒店

에 어리었는 청룡 에 지

늘 여 통곡 고 사 는 꿈 꾸

었다 마 상 게 생각 여 새 다리다

가 새벽닭 울고 달 갔다 가

보니 과연 어 동 가 가에 울고 는지

달 들어 그 고 사 에

어 말

는 어 어 에 어 가ldquo

닭 곳에 우느냐rdquo

니 울 그 고 답 여 말 다 lt

략gt

년 학 간고사 대비2013 2 현대고 대비

ECN-0102-2013-001-000076193

생각 여 가 고 시 는ldquo ( )大人

상에 다시없는 니다 살 엇 겠습니

에 돌 가시고

가에 돌 가 니 살 마 없습니

다 략 어 없어 강 승상 가니rdquo lt gt

그곳 월계 었다

다 강 승상에게는 들 없고 다만 만( )

었다 가 낳 에 가 색

타고 내 에게 말

는 니다 미원 과ldquo ( )紫微垣

연 맺고 었는 께 강 집( )緣分

보내 에 니 게 여겨 주십시

rdquo

거늘 미 가운 낳 니 가

고 거동 단 다 시 짓 쓰 고

는 없었 니 여 가운( ) 音律

지 는 짝 룰 만 사 없었다 가 사

여 사 감 게 고 지 못 고 염 는 만다

다가 당에 거 고 식같 러

내니 고귀 상 루 말 다 어 ( )相

울 도 다 귀 사 없고 ( )富貴爵祿

웅 걸 만고 었다 승상 매우 뻐 내

당 들어가 에게 사 니 역( ) 內堂

시 매우 거워 말 다

도 마 사 는 승상께ldquo

그 게 말 시니 상 여러 말 지 말고 사

도 시다 략 시 택 여rdquo lt gt

니 다운 신 과 신 습 늘에 죄

짓고 간 상에 내 신 다

다 내고 들어가 사 살펴보니

고 것 는 다 말 어 고

는 다 어 신 에 ( )新房

에 신 과 신 가 평생 연 맺었( )緣分

니 사 주고 말 어떻게 다 헤 릴

어떻게 다 리 지낸 후에 튿 승

상 니 승상 거운 마 지 못

( ) 듯 월 러 생 열다 살

었다 에 승상 어진 사 얻고 만 에 근심

없었 다만 주 가 간신

에 죽 것 생각 마 곧 어

곤 다 그 에 주 원통

어 없 고 여 시 가 거늘 략 lt gt

략 거리

강 승상 에게 상 리지만 여움

사 귀 가게 다 강 승상 몸 는

연 과 헤어 리 다

마 각 생 강 승상 집 쪽( )

늘 보고 없 가 신 신 생각 니

없고 어 없었다 는 어떻게 도리가 없다

여 산 에 들어가 리 고 어 도 닦

고 다 그 산 보고 가

다가 곳에 다다 니 에 큰 산 었다 많

우리 골짜 가 늘 는 가운 색

에 고 갖가지 가 짝 어 ( )花草

었다 략 주 보니 lt gt ( ) (一柱門 黃

산 룡사 어 었다) lsquo rsquo 金大字

산 들어가 고승 다 그( ) ( ) 山門 高僧

거동 보니 눈 눈 듯 고

변 같 귀는 어 에 늘어 니( ) 白邊

맑고 어 골격과 신 평 니었

다 염주 에 걸고 짚고 포 ( )六環杖

삼에 어진 쓰고 생 보고 말

승 연 여 상공 시는 동 에ldquo

가 맞 지 못 니 승 십시 rdquo

생 크게 말 다

생 가 여 어 고ldquo

없 다니다가 우연 곳에 사 만 것

그 시 생 어떻게 고 습니

rdquo

승 답 여 말

어 산 승 에ldquo ( ) ( )南岳 衡山

시어 승에게 탁 내 낮 시경에 경 lsquo 12

동 에 사는 심 들 가 것 니 내쫓

지 말고 습니다 마 승rsquo

다가 상공 림새 보니 경 사 에 보

습니다rdquo

생 그 말 듣고 편 고 편( )

슬 승 들어가니 여러 승 들

가워 다 승 에 들어가

후에 그 편 니 곳 경 었다 상( ) 仙境

고 신 편 다 후 는 승과

께 병 도 탐 고 경도 게( )兵書

게 었다 게 니 지 에 가객 ( ) ( )大明天地 佳客

년 학 간고사 대비2013 2 현대고 대비

ECN-0102-2013-001-000076193

없고 산 에 리 만 본 ( ) 廣德山

신 상 사 살 는 만

우고 늘 월 신 과 늘 ( )日月聖神

산 신 들 다 니 그 재( ) 名山神靈

주 민 누가 당 겠는가 낮 공

zb24) 다 에 해당하는 내 으 적절하( ) 않은 것은

① 강 티 통 당시 능 다

② 상계 지상계 경 는 원 계 드러

③ 실에 어 없는 실 가 타 는

④ 뛰어 재주 어 가진 고

등 다

⑤ 가 직 개 여 평가 내리는

편집 평 타 다lsquo rsquo

가 본격 가 동 것 지( )

다 단 상 에2003 lsquo rsquo

들어가 드럼 연주 다 취미 생 달리

들었다는 보 우 가 들ldquo

어 틱 린 도 다 고 말 다rdquo

경 는 가 망 없( ) lsquo

티 원 고 답 다 신과 같 시각rsquo

는 습 상상 만 도 감동

다 시각 연주 동시에

열 상 는

티 원 그런 열 경 럽다는 것 다

다 역시 엄청 다 본( )

에 복 들

고쳐 가고 다 신 에 얼

마 지는 고 리가 는 지도 생님

가 훈 고 많 고쳐 다

고 말 다

그러 직도 에 지 는 다 그는

체격 지 못 게 가 큰 만

체 운동 훈 과 께 체 늘 동 50

는 게 고 말 다

에게는 꿈 다 통 누 가( )

주겠다는 것 그 꿈 다 신 극복 는

과 에 큰 경험 들도 느 게

주고 싶다는 것 다

마 슬 마다( ) ldquo 통

낼 었 것 럼 고통 는 사 들

고 겠다 고rdquo

말 다 달 루 첫 낸 lsquo rsquo

첫 드 심 집에 는 리듬 드 2

루 에 도 보고 싶다 집 에는 직(RampB) 3 4

사 곡 도 보 고 싶다고 포 다middot

zb25) 에서 가장 유사한 의 를 닌 어를lt gt

찾아 쓰

lt gt

나는 이제 너에게도 픔을 주겠다

사랑 다 소 한 픔을 주겠다

겨 거리에서 개 놓고

살아 추위 떨고 있는 할 니에게

값을 으 서 뻐하던 너를 위하여

나는 픔의 평등한 얼 을 여 주겠다

내가 어둠 속에서 너를 를 때

단 한 도 평등하게 어주 않은

가마니에 덮인 동사자가

다 얼어 죽을 때

가마니 한 장조차 덮어주 않은

한 너의 사랑을 위해

흘릴 르는 너의 눈 을 위해

나는 너에게 이제 너에게도 다림을 주겠다

지 울 포동 여고 생들17

틈 없 가득 체 에 맑 울

다 죽 듣 생들 사 에

연 는 탄 다 객들 도 는lsquo rsquo

가 보 주 공 맹 가 운 는

단 그룹사운드 루 보컬 맡고 는lsquo rsquo

시각 지 었다17 1

근 다만과 가 거lsquo rsquo lsquo

꿈 고 퇴 내가 다rsquo

간 간에 지 지 연 생들 짧lsquo rsquo lsquo rsquo

가 운 듯 리에 어

연 다 내 사 고 퇴lsquo rsquo

과 루 들 결 다시 돌lsquo rsquo

들 고 사 들 에 당당

것 니다 내 태어

볼 없었 크고 열여

년 학 간고사 대비2013 2 현대고 대비

ECN-0102-2013-001-000076193

에도 고 시 얻지 못 다

감지 없는 시각 상태 다

신 지에 고 상 원망 도

단다 어느 가 에 시각 에 ldquo

어 그런 듣고 다 보니 내가 게 lsquo

살 는지 도 눈 고 싶rsquo lsquohelliphellip

보 는 생각만 들 고 그 가 들에게rsquo

도 내고 들도 고 많 었죠 들 rdquo

었 지 새 는 에 쑥 러운 색

어났다

생에 것 단연 었다lsquo rsquo

공연에 거 꿈lsquo rsquo

는 다 특 가사 갑게 는 운 lsquo

벽 에 당당 마주 어 언 가 그 벽

고 늘 어 거운 상도

없죠 내 삶 에 웃 그 께

는 다고 다rsquo

들었 그냥 런 도 고만 여ldquo lsquo rsquo

겼죠 그런 꾸 가사 미 새 다 보

니 통 는 가사 는 생각 들 고 (

가 게는 시각 는 생각 들고 들) ( )

마다 듣고 큰 얻었어 rdquo

에 진지 게 가에 미 가

zb26) 의 에 들어갈 말 적절한 것은lt gt ~

lt gt

난 난 꿈이 있었죠

고 찢겨 남 하여도

내 가 히 과 같이 간 했던 꿈

혹 때 누 가가 뜻 를 비 음

내 등 뒤에 흘릴 때도

난 참아야 했죠 참을 있었죠

날을 위해

늘 걱정하듯 말하죠

헛된 꿈은 독이라고

세상은 끝이 정해 책처럼

이 돌이킬 없는

현 이라고 helliphellip

래 난 난 꿈이 있어

꿈을 믿어

나를 켜

저 차갑게 서 있는 이란 앞에

당당히 마주칠 있어

출처 가 거위의 꿈 작사 이적 작곡 동률- lsquo rsquo ( )

① ② ③ ④ ⑤

가 떴다는 들 만 지만( ) lsquo rsquo

늘 겸 다 에 주 연 우승 지 간에도 3

단 생님께 만 지 고 고 만ldquo rdquo

큼 늘 겸 신 계 가

고 다

에게는 꿈 다 통 누 가

주겠다는 것 그 꿈 다 신 극복 는 과

에 큰 경험 들도 느 게 주

고 싶다는 것 다

슬 마다 통 낼ldquo

었 것 럼 고통 는 사 들

고 겠다 고rdquo

말 다 달 루 첫 낸lsquo rsquo

첫 드 심 집에 는 리듬 2

루 에 도 보고 싶다(RampB) 집 에는 직34

사 곡 도 보 고 싶다고 포 다

미 는( ) (26) 어 헤헤헤 웃다가 어ldquo rdquo

허허허 웃었다ldquo rdquo ldquo rdquo 같 도 고

상 다 는 같 도 다( ) 壯丁 킹 들lsquo

다 는 역도 보 그 다 지만 그는rsquo

뷰에 지 다 운동만 지 ldquo

것 지 간에 여러 사 도 역rdquo helliphellip

었다 그런 엇 그 마 움직 는지 보 쯤

지 담 사 다 훈 없어 그는 티

지 림 었다 태 다 갔다 는 습

마 집 럼 편 게 보 다

주말에는 주 엇 보내

주말에도 별 주 에 청ldquo

고 에 가고 도 쳐

에 듣고 보 에 갈 가 별 없

어 산 시 게 고 들어 2002

거 매 여 지냅니다 시 과 지훈 rdquo

다 근 간 과 진실 그리고 싶어( )

가 다 근에게 그것 진리 다 거 다 없

거 고 다 없 는 것 진리

다 근 진리는 후 쪽 었다 신산( )辛酸 삶

었 질곡( )桎梏 역사 에 지냈 가

눈에 든 것 료 단 료 게 보

것 었다 그것 그 에 겨우겨우

슬 슬 생 어가는 간들 었다

리 과 단 리 고리에 검 마

없 거리 돌

상 것 없는 등 근에게 상

과 진실 엄 ( )儼存 다는 사실 리는 가

실 고 가 과 역경 에 도 근 내 포

없었 후 보루( )堡壘 다 도 365

도 간 근 여

시 것 다

년 학 간고사 대비2013 2 현대고 대비

ECN-0102-2013-001-000076193

다 공주 그림 가 근 경- ( ) ldquo rdquo(

2009)

zb27) 작가의 주 적인 각이 드러난 것은~

① ② ③ ④ ⑤

가 신 지에 고 상 원망( )

도 단다 어느 가 에 시각 에 ldquo

어 그런 듣고 다 보니 내가 lsquo

게 살 는지 도 눈 고 싶rsquo lsquohelliphellip

보 는 생각만 들 고 그 가 들에게rsquo

도 내고 들도 고 많 었죠 들었rdquo

지 새 는 에 쑥쓰러운 색

어났다 략 [ ]

경 는 가 망 없 티lsquo

원 고 답 다 신과 같 시각rsquo

는 습 상상 만 도 감동

다 시각 연주 동시에

열 상 는 티

원 그런 열 경 럽다는 것 다 략 [ ]

슬 마다 통 낼ldquo

었 것 럼 고통 는 사 들

고 겠다 고rdquo

말 다 달 루 첫 낸 lsquo rsquo

첫 드 심 집에 는 리듬 2

루 에 도 보고 싶다 집 에는 직(RampB) 3 4

사 곡 도 보 고 싶다고 포 다

식 누 가-

고 싶어

다 역도 미 담 고 사( )

질 주말에는 주 엇 보내[ 1]

답 주말에도 별 주 에[ ] ldquo

청 고 에 가고 도 쳐

에 듣고 보 에 갈 가 별

없어 rdquo

질 계 고 슬슬 도 는 것 닙니[ 2]

답 다 들 눈 에 보 고 뿐 보[ ] ldquo

다 열심 고 어 상에 도 들지만 상

지키는 것 들다고 에 도달

그것 지키 훨 많 rdquo

질 들 살 고 리 는[ 3]

거운 들 체 리느 는다

답 가 고 게 체 어[ ] ldquo ( ) 級

느 도 계가 니 살 는 것도 고역 지만

살 우는 것 들어 는 살

체 리 고 어도 어도 실 갔다

쑥 어 rdquo

질 거리에 슷 연 여 들[ 4]

보는 간 상 지

답 상 다 체 게 리지 못[ ] ldquo

거 주변에 는 그 거 누 보지

못 고 뻐지고 싶 에 체 리는 에

타 워 지만 는 어울 는 것보다 는

시간 운동만 는 건 니에 사복 lsquo rsquo

고 사복 는 말에 들 웃지만 늘 운동복

고 지내니 사러 갈 도 어 rdquo

질 역도가 말 단 식 운동 니[ 5]

답 가 내는 만 클 업 보[ ] ldquo

그러니 만 쓰는 식 운동 니다

만 다고 거운 것 들 는 건 니거든 연

도 고 가지 동 에 도 여러 가지

복 들

보식 역도 여 미-

zb28) 가 에 대한 설 으( ) 않은 것은

① 시각 우 지 시 에 지

고 망 가는 태도 달 고 다

② 언어 과 언어 복 사 여

담 내 생각 게 는 가

③ 직 감 그 마 것

럼 생생 게 느껴지는 과 주고 간 내

없 리 어 억 게 다

④ 담 내 식 리 여 담 삶 습

과 가 시 여 독 에게 감동과 훈 다

⑤ 직 진 담 직 누

지 못 는 독 에게 생생 상 달 주고

담 욱 게 다

zb29) 나 의 각 의 의도를 설 한 것으 적절하( ) 않

년 학 간고사 대비2013 2 현대고 대비

ECN-0102-2013-001-000076193

은 것은

① 질 담 상 보여 주 것 다1

② 질 담 과 그에 삶 태도 보여2

주 것 다

③ 질 역도 겪는 어 움에 역도3

과 것 다

④ 질 같 연 여 갖는 고민 는지 말4

주 는 것 다

⑤ 질 역도가 과 고 운동 는 것5

담 가 말 주 는 것 다

가 만진 것 다( ) 3

감 달 다고 다 억 에( ) 音感

지워 지만 당시 청 탁 리도

다고 다 드럼 웠다 4

에 갈 마다 드럼 는 리가 신 게 들

다고 다 눈 볼 가 없 니 엔ldquo

는 는 님 틱 에 여 주

다 드럼과 연 맺 과 들 주었다rdquo

식 누 가-

고 싶어

역( ) 도가 말 단 식 운동 니

가 내는 만 클 업에 보ldquo

그러니 만 쓰는 식 운동 니다 만

다고 거운 것 들 는 건 니거든 연

도 고 가지 동 에 도 여러 가지 복

들 시 는 상 상

드는 상 에 맞춰 실 에 는 여러

펼쳐집니다rdquo

략( )

늘 에 는 어 만 것 같

가 에 사 고 사 사ldquo

겠어 든 에 가 경 만 고

울 는 사 겠어 rdquo

보식 역도 여 미-

다 가 운 는 어 어( ) ldquo rdquohelliphellip

월 새벽 시 태 없 거웠고1965 5 6 1

는 없 그 병원에 퇴원 집

가는 마지막 마 고 마 내 거 다

가 죽 간신 에 실 다 사는 어느5 lsquo

가 죽 는 말 가 식 다 신rsquo

상에 각 시키는( )刻印 에 실

어느 가는 후 민 가가 근 었다lsquo rsquo

는 간 과 진실 그 다는( ) ldquo

에 단 평 견 가지고 다 내

가 그리는 간상 단 고 다 지 다 는 그들

가 에 는 평 지 니 그리고 어린

들 미지 겨 그린다rdquo

마 근 간 과 진실 그리고 싶어( )

가 다 근에게 그것 진리 다 거 다 없

거 고 다 없 는 것 진리

다 근 진리는 후 쪽 었다 신산(辛酸 삶)

었 질곡(桎梏 역사 에 지냈)

가 눈에 든 것 료 단 료 게

보 것 었다 그것 그 에 겨우겨우

슬 슬 생 어가는 간들 었

다 리 과 단 리 고리에 검

마 없 거리 돌

상 것 없는 등 근에게 상에

과 진실 엄 다는 사실 리는 가( )儼存

실 고 가 과 역경 에 도 근 내

포 없었 후 보루(堡壘 다 도)

도 간 근365

여 시 것 다

월 강원도 림리에( ) 1914 2 21

삼 독 태어났다 어 근 복

그것 그리 가지 못 다 근 곱 살

지는 산 산업에 실 고 답마 에 내

갔다 근 그림 럼 쫓 다니 가 시 것

다 상 진 것도 가 었다

러 가 에도 고 근 가 꿈꾸었다 근

가 꿈꾸게 것 보통 업

원색도1926 만lsquo rsquo 었다

공주 그림 가 근 경-

zb30) 에 대한 설 가장 른 것은~

① 역도가 과 운동 도 질

② 리는 는 다 lsquo rsquo

③ 들었지만 그럭 럭 는 다 lsquo rsquo

④ 가 게 보 시 말 다

⑤ 보 병 는 지 상 lsquo rsquo

는 말 다

년 학 간고사 대비2013 2 현대고 대비

ECN-0102-2013-001-000076193

시간 많지 다 청량리 생 병원

마지막 상 경 릿 게 들어 다 그 는 십

만 큰 가 상 말 다

지 못 들 마 갈 고 돗

도시민들 싹 싹 탔다 가 시

월에 병원에 원 가 폐 진 몸도4 ( )疲弊

갈 미 지 못 고 었다 가는 얼마( ) 解渴

지 생 에 생각 가

마감 는 신 평생 십 만에

가 과 많 닮 다고 생각 지는

가 운 는 어 어ldquo rdquo 1965helliphellip

월 새벽 시 태 없 거웠고 는5 6 1

없 그 병원에 퇴원 집 가

는 마지막 마 고 마 내 거 다 가

죽 간신 에 실 다 사는 어느 가5 lsquo

죽 는 말 가 식 다 신rsquo

상에 각 시키는 에 실 어느( ) lsquo刻印

가는 후 민 가가 근 었다rsquo

ldquo 는 간 과 진실 그 다는 에

단 평 견 가지고 다 내가 그

리는 간상 단 고 다 지 다 는 그들 가

에 는 평 지 니 그리고 어린 들

미지 겨 그린다rdquo

근 간 과 진실 그리고 싶어 가

다 근에게 그것 진리 다 거 다 없 거

고 다 없 는 것 진리다

근 진리는 후 쪽 었다 신산 삶 ( )辛酸

었 질곡 역사 에 지냈 가 눈에( )桎梏

든 것 료 단 료 게 보 것

었다 그것 그 에 겨우겨우 슬

슬 생 어가는 간들 었다 리

과 단 리 고리에 검 마

없 거리 돌 상

것 없는 등 근에게 상에 과 진실

엄 다는 사실 리는 가 실( )儼存

고 가 과 역경 에 도 근 내 포 없었

후 보루 다 도 도( ) 365堡壘

간 근 여 시 것

간에 지닌 가 근 1914 2

월 강원도 림리에 삼 독21

태어났다 어 근 복 그것 그리

가지 못 다 근 곱 살 지는 산

사업에 실 고 답마 에 내 갔다 근

그림 럼 쫓 다니 가 시 것 다 상

진 것도 가 었다 러 가 에도

고 근 가 꿈꾸었다 근 가 꿈꾸게

것 보통 업 원색1926

도 만 었다lsquo rsquo

그림 가 근 경 공주- ldquo rdquo ( 2009)

zb31) 다음 이 같은 의 성 소에 해당하 않은

것은

사건 평① ② ③

④ 주 ⑤ 경

가 운 는 어 어ldquo rdquo 1965helliphellip

월 새벽 시 태 없 거웠고 는5 6 1

없 그 병원에 퇴원 집 가

는 마지막 마 고 마 내 거 다 가

죽 간신 에 실 다 사는 어느 가5 lsquo

죽 는 말 가 식 다 신rsquo

상에 각 시키는 에 실 어느( ) lsquo刻印

가는 후 민 가가 근 었다rsquo

는 간 과 진실 그 다는 에ldquo

단 평 견 가지고 다 내가 그

리는 간상 단 고 다 지 다 는 그들 가

에 는 평 지 니 그리고 어린 들

미지 겨 그린다rdquo

근 간 과 진실 그리고 싶어 가

다 근에게 그것 진리 다 거 다 없 거

고 다 없 는 것 진리다

근 진리는 후 쪽 었다 신산 삶 ( )辛酸

었 질곡 역사 에 지냈 가 눈에( )桎梏

든 것 료 단 료 게 보 것

었다 그것 그 에 겨우겨우 슬

슬 생 어가는 간들 었다 리

과 단 리 고리에 검 마

없 거리 돌 상

것 없는 등 근에게 상에 과 진실

엄 다는 사실 리는 가 실( )儼存

고 가 과 역경 에 도 근 내 포 없었

후 보루 다 도 도( ) 365堡壘

간 근 여 시 것

간에 지닌 가 근 1914 2

월 강원도 림리에 삼 독21

태어났다 어 근 복 그것 그리

가지 못 다 근 곱 살 지는 산

사업에 실 고 답마 에 내 갔다 근

그림 럼 쫓 다니 가 시 것 다 상

진 것도 가 었다 러 가 에도

고 근 가 꿈꾸었다 근 가 꿈꾸게

것 보통 업 원색1926

도 만 었다lsquo rsquo

공주 그림 가 근 경- ldquo rdquo ( 2009)

년 학 간고사 대비2013 2 현대고 대비

ECN-0102-2013-001-000076193

zb32) 위 을 작성하는 과정에서 되어 활 된 자

어 것은

신 사 료① 연보②

고③ ④ 들과 담

⑤ 에 평

는 간 과 진실 그 다는 에ldquo

단 평 견 가지고 다 내가 그

리는 간상 단 고 다 지 다 는 그들 가

에 는 평 지 니 그리고 어린 들

미지 겨 그린다rdquo

근 간 과 진실 그리고 싶어 가

다 근에게 그것 진리 다 거 다 없 거

고 다 없 는 것 진리다

근 진리는 후 쪽 었다 신산 삶 ( )辛酸

었 질곡 역사 에 지냈 가( )桎梏

눈에 든 것 료 단 료 게 보

것 었다 그것 그 에 겨우겨우

슬 슬 생 어가는 간들 었다

리 과 단 리 고리에 검 마

없 거리 돌 상

것 없는 등 근에게 상에 과

진실 엄 다는 사실 리는 가 실( )儼存

고 가 과 역경 에 도 근 내 포

없었 후 보루 다 도 도( ) 365堡壘

간 근 여 시

것 다

간에 지닌 가 근 1914 2

월 강원도 림리에 삼 독21

태어났다 어 근 복 그것 그리

가지 못 다 근 곱 살 지는 산

사업에 실 고 답마 에 내 갔다 근

그림 럼 쫓 다니 가 시 것 다 상

진 것도 가 었다 러 가 에도

고 근 가 꿈꾸었다 근 가 꿈꾸게

것 보통 업 원색1926

도 만 었다lsquo rsquo

질 루 마 가 도 린다 경건

움 느껴지는 경 다 훗 근 그림에

과 는 거 것( )裸木

만 간과 연 엮어 가는 경건 움lsquo rsquo

니었

같 가가 고 싶었 근에게 그 꿈에 다

가가는 지 다 다 가 지망생들 규 미

상 에 진 고

에 지만 근 다 다 근

미 에 운 것 보통 시 미 시간

다 그런 그에게 없는 연습 가가

통 다 가 귀 시 지 도

얻는 뛸 듯 뻤지만 마 도 가 에

듯 는 었 에 어린 근 주 에

에 그림 그리고 지우고 복( )粉板

시간 가는 게 루 보냈다

근 그 갈 가가 것 열여( )渴求

었 다가 미1932 lsquo rsquo ( lsquo

미 에 다 다는 고 마rsquo) lsquo rsquo

가 근 집 고도 지는 시골 경

그린 그림 다 후 근 에 1943 22

지 미 에 그림 고

에 걸쳐 다 미 근 가

동 는 었다

공주 그림 가 근 경- ldquo rdquo ( 2009)

zb33) 위 의 내 과 일치하는 것은

가 근 가 꿈 포 다①

근 당 가들과 께 에 다②

살 근 가 걷20③

게 었다

④ 만 통 근 역경 겨내는lsquo rsquo

느 다

⑤ 근 간 과 진실 그리 에 그 에

드러 는 간상 단 다

계 시 주 근 건강

걸었다 신 과 간에 상 다 건강

신 는 눈에도 다 근 쪽 눈 뿌 게

보 지 과에 다 다 시 지지 고 결

내 었다 시 지만 마 막막

다 늦어 결 근 쪽 눈 고 말 다

쪽 눈 근에게는 쪽 눈 었고

계 었다 그 근 는 여 그lsquo rsquo

다 근 에 같 그림 그 었다1950

시 그림 는 여 쪽lsquo rsquo

고 어 마주 고 는 그림1963

여 과 동 다 마 복

그린 듯 눈 내리 새 게 다 지

사 다 근 게 복 것

복 상과 타 는 근 상

가 떳떳 단 었고 근 그리고

간 과 진실 에 다가가 가 근다

운 었다 근 신에게 당당 지 그리고

그 다 근 그림에 단 복 보다

년 학 간고사 대비2013 2 현대고 대비

ECN-0102-2013-001-000076193

태 도 그리고 극 보다 과

얻 여 었다 과 통

근 그리고 는 재 고 에 질

만들고 특 것 다

공주 그림 가 근 경- ldquo rdquo( 2009)

zb34) 의 이유에 대해 추 한 것으 적절하 않은 것

상과 타 시도①

보다 과 얻②

근 신에게 당당 지③

④ 간 과 진실 에 다가

⑤ 태 도 얻

근 가가 었지만 그 다니 가

럼 어지지 다 복과 쟁 거쳐 시

는 가 근에게 생계 사 에

운 사 다 에 키에 건( ) 178cm死鬪

체 근 에 동 역 업( )荷役

가 생계 다 쟁

에는 동에 운 상우 주 미

죄 사 에 그림 그리는 시 다 그곳에

에 동 역 업 것에

결 것 럼 보 다 지만 그런 것만도

니었다 그림 그리는 고는 지만 매 근

는 극 간 과 별 없는 경 리 그림

벽에 그리는 것 었다 우도 리 없었다 근

트 는 우 그림 그 다 생

계 그림 단 것 다

후 근 지 신 계 리에 미

엑 리 겼다 근 곳에

건 사 크 에 미 들 ( )

상 상 그 다 근 갖 다 겪

냈다 그리고 결 그 돈

신동에 어 사리 집 마 다 마 ㄷ

루 심 쪽에는 과 엌 쪽에는 건

었다 건 주고 근 가 에

여 살 다 심 에는 지 집어

쓰고 지만 곳 근 가 에게 러웠

보 리 다 근 과 마루 업실 삼 그림

그 다 신동 마루는 근 그림에 등 는 lsquo rsquo

같 상들 지 다 시 고

에 들 폐허가

가 업실 었다

공주 그림 가 근 경- ldquo rdquo( 2009)

zb35) 위 에 대한 설 으 적절한 것은

업 시 여 훈과 감동 다①

에 주 평 드러 다②

사 사 등 식 과 ③

④ 다 근거 시 여 삶에

⑤ 살 시 사 경 께 여

습 시 다

가 시간 많지 다 청량리 생 병원( )

마지막 상 경 릿 게 들어 다 그 는

십 만 큰 가 상 말 다

지 못 들 마 갈 고 돗

도시민들 싹 싹 탔다 가 시

월에 병원에 원4 가 폐( )疲弊

진 몸도 갈 미 지 못 고 었다( )解渴 가는

얼마 지 생 에 생각

가 마감 는 신 평생 십 만에

가 과 많 닮 다고 생각 지는

가 운 는 어 어( ) ldquo rdquohelliphellip

월 새벽 시1965 5 6 1 태 없 거웠고

는 없 그 병원에 퇴원 집

가는 마지막 마 고 마 내 거 다

가 죽 간신 에 실 다 사는 어느5 lsquo

가 죽 는 말 가 식 다 신rsquo

상에 각 시키는 에 실( )刻印

어느 가는 후 민 가가 근 었다lsquo rsquo

다 는 간 과 진실 그 다는( ) ldquo

에 단 평 견 가지고 다 내

가 그리는 간상 단 고 다 지 다 는 가

에 는 평 지 니 그리고 어린 들

미지 겨 그린다rdquo

근 간 과 진실 그리고 싶어( )

가 다 근에게 그것 진리 다 거 다 없

년 학 간고사 대비2013 2 현대고 대비

ECN-0102-2013-001-000076193

거 고 다 없 는 것 진리

다 근 진리는 후 쪽 었다 신산( )辛酸 삶

었 질곡 역사 에 지냈( )桎梏

가 눈에 든 것 료 단 료 게 보

것 었다 그것 그 에 겨우겨우

슬 슬 생 어가는 간들 었다

리 과 단 리 고리에 검

마 없 거리 돌

상 것 없는 등 근에게 상에

과 진실 엄 다는 사실 리는 가 실( )儼存

고 가 과 역경 에 도 근 내 포

없었 후 보루 다( ) 堡壘 도 365

도 간 근 여

시 것 다

마 같 가가 고 싶었 근에게 그 꿈( )

에 다가가는 지 다 다 가 지망생들

규 미 상 에 진 고

에 지만 근 다 다 근

미 에 운 것 보통 시 미 시간

다 그런 그에게 없는 연습 가가

통 다 가 귀 시 지 도

얻는 뛸 듯 뻤지만 마 도 (

는 었 에 어린 근 주 에)

에 그림 그리고 지우고( )粉板

복 시간 가는 게 루 보냈다

zb36) 전 의 성 소가 아닌 것을 고르

① 평 ② 사건 ③ 경

④ ⑤ 훈

늘 지 상에 살고 는 사 들 억 도가10

고 그리 지 통 고 는 사 들( )知的

그보다 훨 많 억 도는 고 지 20

통 다 그런 지 고 2500

그리 간 보는 과 사 에

매우 달 뿐만 니 과 에 도 극

루고 었다 미 운 그런 들

살고 는 동 과 사 들 사고 식에

큰 가 다는 다

고 그리 들 우주 개별 고 독립

사 들 생각 지만 고 들 우

주 연 질 간주 다 같( ) 看做

각 도 들에게는 연 질

었지만 그리 들에게는 미 들 결 었

다 고 과 그리 들 사 같

는 동 과 사 에 도 견 다

지심리 미 마 드 겐트 는

살 들에 에 지 다

연 동 과 상 다 과 같 실험

다 크 만든 미드 도 보

여 주고 그 상 닥 고 주었다lsquo (Dax)rsquo

실 닥 는 재 지 는 것 실험 가lsquo rsquo

만들어 낸 다 그런 다 개 다 체 보

여 주었는 는 미드 지만 틱

만들었고 다 는 재료는 크 지만

달 다 그러고 어 것 닥 지 사 들에게 고 lsquo rsquo

게 니 들 주 같 고 는

체 택 고 동 들 같 재료 만들어진 체

택 다 러 는 심지어 살짜리

들에게 도 타났다 것 곧 과 동

다 상 보고 다는 것 미 다

개별 사 보고 고 동 연 질 보

고 는 것 다

동 들 주변 상 에 맞 어 동 고

에 다 사 들 태도 동에 보다 많

주 울 다 동 가 미시간 에

에 경험 다 그는 미식

경 보러 가게 었는 경 체는 매우 재미 었

주변 들 동에 질 다 그 는

들 계 어 상태 경 다

어 들 에 에 그 시 가 계 가

진 것 다 상 살펴 는 말 들 lsquo rsquo

에 그는 에 시 어 도 뒷사

생각 곧 다시 곤 것 다 그런 그에게 뒷

사 고 지 는 들 동 럼

어 웠다

생각 지도 리 드 니 벳-

zb37) 다음 위 의 내 전개 으 만 인lt gt

것은

lt gt

대조의 통해 대상이 닌 특성을 설 하고 있다

일화를 제 하여 자 의 주장을 뒷 침하고 있다

유추의 을 사 하여 독자의 의해를 돕고 있다

대상이 형성되는 과정을 간적 서에 따라 서 하고 있

① ②

③ ④

년 학 간고사 대비2013 2 현대고 대비

ECN-0102-2013-001-000076193

가 우리가 말 고 쓰는 든 단어가 사 에 는( )

것 니다 사 격에 가 는 지만

어 사 과 같 특별 는 사 니lsquo rsquo

단어 격 보 단어가 사 에

등재 어 다 리 리 사 는 단어 도 그

것 시 사 는 어 고 사 에

격 보 것 니다

러 얼 은 사전에 를 있는가 이에 대한 답lsquo rsquo

은 얼 이 유행어인가 아닌가에 따라 갈라 다 이 단어lsquo rsquo

는 년 어 자 에 랐고 쓰이고 있으2002 lsquo rsquo

유행어라고 하 에는 생 이 다 런데 계속

을 유 하 서 사전에 등재될 자격을 획득할 것인가 이

에 대한 답을 내리 는 히 어 다

여 서 가 를 고 해 볼 있다 첫 는 이 단어

를 써야 할 필 가 속적으 있는가 하는 점이다

상주의 열풍에 휩 인 사회 위 에 편 해서 퍼 말

이 얼 인데 과연 런 위 가 속될 것인가 이에lsquo rsquo

대해 필자의 생각은 정적이다 사회 위 가 뀌

런 말을 쓸 일이 없어 것이다

다음은 단어의 성이다 단어의 성이 사회적으 거

감이 없으 계속 사 될 가능성이 높다 런 에서

얼 은 좋은 조건이 아니다 익히 알 졌듯이 이lsquo rsquo

말은 얼 과 청소년층에서 속어 사 하는 이 결합lsquo rsquo lsquo rsquo

된 말이다 얼 에서 얼 을 리하는 조어 도 lsquo rsquo lsquo -rsquo

어에서는 매 낯선 이다 이것만으 도 거 감을 갖

는 사람들이 있다 더 나 속어 결합한 말이다 얼 lsquo rsquo

이 널리 퍼졌다 해도 은 여전히 청소년층의 속어lsquo rsquo

남아 있다 속어는 자연 럽게 아 자리에서나 쓰 에는

담 러 말이다 러한 담을 하고 사

역을 넓혀 가는 속어도 없 는 않다 특히 얼 은 lsquo rsquo

에도 종종 등장한다 만큼 거 감이 많이 희석되었다

고 할 있다 러나 일상의 자연 러 대화에서도 거

리낌 없이 등장하는가 게 는 되 않았다고 생

각한다

얼 이 유사어인 쌈 등을 만들어 내고lsquo rsquo lsquo rsquo

있으니 살아남을 있을 것이라고 는 견해도 있을 것

이다 러나 간이 나 서 유사어를 포함하여 든

말이 사라 사 는 많다 유사어가 많다는 것이 생 을

유 할 있는 절대적인 조건은 아니다

나 언젠가 터 사람들은 어느 단에서 얼 이 가장( )

쁜 사람을 가리켜 얼 이라고 르고 있다 이 얼lsquo rsquo lsquo rsquo

이라는 단어가 최근 어사전에 라 항간에 논란이 일고

있다 아닌 게 아니라 얼 은 유행어처럼 인다 생 lsquo rsquo

도 리 래되 않은 것 같고 언제 사라 도 알

없다 게다가 젊은이들 사이에서 주 쓰일 뿐이다 이런

단어를 사전에 는다는 게 하 이 없어 이 도

한다

러나 속단은 이다 차근차근 따져 볼 일이다

선 얼 이 일 적 유행어인 아닌 주의 게 들여다lsquo rsquo

볼 필 가 있다 유행어란 유행에 따라 빠르게 유포되었

다가 단 간 내에 소 되는 단어나 를 가리킨다

얼 은 인터넷을 통해 속히 퍼 말이다 하 만 일lsquo rsquo

적인 유행어처럼 단 간 내에 사라 않았을 뿐 아니라

현재 도 잦은 빈도 사 되고 있고 앞으 도 상당

간 사 될 것으 측된다 한 언 재단의 뉴 검 lsquo rsquo

색 사이트에 따르 얼 은 년 에 처음 나타난lsquo rsquo 2001

이후 꾸 히 사 되고 있다

이 같은 사 빈도는 얼 이 일 적 유행어 는 현lsquo rsquo

저히 다르다는 것을 여 다 장 간의 생존 만으 도

얼 은 이 한 어의 어휘 에 를 자격을 얻었다lsquo rsquo

고 할 있다 더 이 이라는 비 적 정제된 매체에

높은 빈도 쓰이고 있 않은가 사 빈도 측 에서

필통이나 연필과 같은 단어 대등하거나 더 많이 쓰lsquo rsquo lsquo rsquo

다는 것은 결코 가 게 볼 일이 아니다

이제는 사전이 언어 현 을 빠르게 하는 게 덕인

대가 되었다 세계적으 유 한 의 사전들도 경쟁

적으 어를 고 있다

하 만 얼 은 젊은이들이나 쓰는 속어라고 흠을 잡을lsquo rsquo

도 르겠다 얼 이 주 젊은 층에서 많이 쓰 lsquo rsquo

는 속어임에 틀림없다 러나 어사전에 표 적이고 품

위 있는 말만 어야 한다고 생각한다 것은 커다란

해다 당장 아 어사전이나 펼쳐 라 속어는

설과 같은 비어나 죄자들이 쓰는 은어 어

마니 같은 소 의 사람만이 쓰는 말 도 라 있

않은가 사전은 말 치에 일정 빈도 이상 나타나는 말이

라 말이든 다 할 있다

zb38) 가 나 에 대한 다음의 설( ) ( ) 않은 것은

① 가 는 얼짱 사 에 등재 것에( ) ( ) lsquo rsquo

보 고 다

② 사 등재 가는 단어 격에( )

고 고 는 언 들 언어 사 도에 고 다 ( )

③ 가 얼짱 어지만 신 과 같 매( ) ( ) lsquo rsquo

체에 도 사 는 말 는 고 다

④ 가는 얼짱 어 보고 크게 가지 근( ) lsquo rsquo 3

거 들어 뒷 고 다

⑤ 는 얼짱 어 는 다 특 다는( ) lsquo rsquo

근거 에도 크게 가지 근거 가 들어 주 2

뒷 고 다

가 늘 지 상에 살고 는 사 들 억( ) 10

도가 고 그리 지 통 고 는 사 들

그보다 훨 많 억 도는 고 지 20

통 다 그런 지 고 2500

년 학 간고사 대비2013 2 현대고 대비

ECN-0102-2013-001-000076193

그리 간 보는 과 사 에

매우 달 뿐만 니 과 에 도 극

루고 었다 미 운 그런 들

살고 는 동 과 사 들 사고 식에

큰 가 다는 다

고 그리 들 우주 개별 고 독립

사 들 생각 지만 고 들 우

주 연 질 간주 다 같 각

도 들에게는 연 질 었지

만 그리 들에게는 미 들 결 었다

고 과 그리 들 사 같 는

동 과 사 에 도 견 다

인 리학자인 츠 이마이 디드 겐트너는 두

살이 채 안 된 아이들에서 터 성인에 이르 다양한

연 대의 동양인과 서양인을 대상으 다음과 같은 험

을 했다 저 코르크 만든 피라 드 양의 도형을

여 주고 대상의 이름을 닥 라고 알 주었다lsquo (Dax)rsquo

제 닥 는 존재하 않는 것으 험자가 임의lsquo rsquo

만들어 낸 이름이다 런 다음 두 개의 다른 체를

여 주었는데 하나는 피라 드 양이 만 하얀 플라 틱

으 만들었고 다른 하나는 재 는 코르크 만 양이

달랐다 러고 나서 어떤 것이 닥 인 사람들에게 고 lsquo rsquo

르게 했더니 서양인들은 주 같은 양을 하고 있는

체를 선택했고 동양인들은 같은 재 만들어 체를

선택했다 이러한 차이는 성인은 어 두 살 리

아이들에게서도 나타났다 이것은 곧 서양인과 동양인은

서 다른 세상을 고 있다는 것을 의 한다 략 ( )

는 아주 단 하 서도 인상적인 험을 했다

험에는 동서양의 대학생들이 참여했다 는 험 참가자

들에게 컴퓨터 화 을 통해 속 장 을 담은 애니 이션

을 여 주었다 화 의 앙에는 초점의 역할을 하는 커

다란 고 한 마리가 있었고 주위에는 다른 생

들과 초 자갈 거품 등이 함 제 되었다 화 을

두 씩 후 참가자들은 자 이 것을 회상해 라는

를 았다

결과 서양인 대학생들과 동양인 대학생 두 앙

의 초점 역할을 했던 고 를 동일한 정도 언 했으

나 경 소 위 거품 초 다른 생 들 에 ( )

대해서는 동양인 대학생들이 서양인 대학생들 다 60

이상 더 많이 언 했다 뿐만 아니라 동양인 학생들은 서

양인 학생들에 비해 개 적인 고 다 전체적인 계

를 더 언 하는 경향을 다 략 또한 경의 일 ( )

를 화 킨 림을 제 하 을 때 동양인 대학생들은 대

경의 화를 알아챘 만 서양인 대학생들은 경

의 화를 거의 알아차리 했다 략 ( )

따라서 서양인들만을 대상으 연 한 화lsquo

편성 결 은 잘 된 것일 도 있다 각 과정과 인rsquo

과정의 어떤 이 화 편적이고 어떤 이

화에 따라 달라 는 는 앞으 많은 연 를 통하여 논의

되어야 한다

나 어떤 의 에서 리 두는 이 화적이다 리( )

안에는 다른 사람들과 더 친 한 계를 유 하 는 상호

의존성과 다른 사람들 터 독립적인 존재 살아가 는

독립성이 혼재한다 따라서 이 에서 어떤 특성이 더 강

하게 각되는 상황에 놓이느냐에 따라 서 다른 화적

특 을 일 있다 결 리 두는 어떤 경 에는

동양인처럼 행동하고 어떤 경 에는 서양인처럼 행동하는

것이다

zb39) 가 에 대한 다음의 설( ) 않은 것은

① 는 신 주 뒷 닥 실험과lsquo rsquo lsquo

니 실험 근거 시 다rsquo

② 동 들 상 간 공통 보다는 에 식

는 강 다

③ 들 주변 맥 에는 심 경 어 사건

과 사건 사 계에 상 민감 다

④ 는 동 과 틀린 지 고 는 것lsquo rsquo

니 다 고 다 lsquo rsquo

⑤ 가에 우리 사 들 개 시 가 원( )

집 경 말 고 는 것 개 보다는

에 고 는 것에 다

늘 지 상에 살고 는 사 들 억 도가10

고 그리 지 통 고 는 사 들( )知的

그보다 훨 많 억 도는 고 지 20

통 다 그런 지 고 2500

그리 간 보는 과 사 에

매우 달 뿐만 니 과 에 도 극

루고 었다 미 운 그런 들

살고 는 동 과 사 들 사고 식에

큰 가 다는 다

지심리 미 마 드 겐트 는 동

과 상 다 과 같 실험 다

크 만든 미드 도 보여 주고 그

상 닥 고 주었다 그런 다lsquo (Dax)rsquo

개 다 체 보여 주었는 는 미드

지만 틱 만들었고 다 는 재료는

크 지만 달 다 그러고 어 것 닥 lsquo

지 사 들에게 고 게 니 들 주 같rsquo

고 는 체 택 고 동 들 같

재료 만들어진 체 택 다 러 는

심지어 살짜리 들에게 도 타났다 것

곧 과 동 다 상 보고 다는

것 미 다 개별 사 보고 고 동

년 학 간고사 대비2013 2 현대고 대비

ECN-0102-2013-001-000076193

연 질 보고 는 것 다

동 들 주변 상 에 맞 어 동 고

에 다 사 들 태도 동에 보다

많 주 울 다 동 가 미시간

에 에 경험 다 그는 미

식 경 보러 가게 었는 경 체는 매우 재

미 었 주변 들 동에 질 다 그

는 들 계 어 상태 경

다 어 들 에 에 그 시 가 계

가 진 것 다 뒷사 고 지 는 들

동 럼 어 웠다

그는 경험에 어 얻어 동 들lsquo

각도 상 본다 는 가 우고rsquo

검 여 주 단 도 상 실험 실

시 다 그는 실험 가 들에게 컴퓨 통

담 니 보여 주었다

에는 역 는 커다 고 마리가 었

고 주 에는 다 생 들과 갈 거 등

께 시 었다 본 후 가 들

신 본 것 상 보 는 지시 다

그 결과 생들과 동 생

역 고 동 도 언

경 거 다 생 들에 ( )

는 동 생들 생들보다 60

상 많 언 다 뿐만 니 동 생들

생들에 개별 고 보다 체 계

언 는 경 보 다 경 변 시

킨 그림 시 동 생들 경

변 지만 생들 경 변

거 리지 못 다

지 지 들만 상 연 lsquo

보편 결 못 것 도 다 지각 과 과rsquo

지 과 어 보편 고 어

에 달 지는지는 많 연 통 여

어 다

리 드 니 벳 생각 지도 사- ldquo rdquo( 2004)

zb40) 위 에 대한 설 으 가장 적절한 것은

① 동 과 생 식 강 고 다

② 가지 실험 통 쓴 고 다

③ 닥 실험에 사 본질에 동 사

상에 주 다

④ 니 실험에 동 과 에 지

각 도에 가 다

⑤ 쓴 는 보편 연 에 드러 우월 에

에 근 고 다

가 동 들 주변 상 에 맞 어 동 고( )

에 다 사 들 태도 동에 보다 많

주 울 다 동 가 미시간 에

에 경험 다 그는 미식

경 보러 가게 었는 경 체는 매우 재미 었

주변 들 동에 질 다 그 는

들 계 어 상태 경 다

어 들 에 에 그 시 가 계 가

진 것 다 상 살펴lsquo 는 말 들rsquo

에 그는 에 시 어 도 뒷사

생각 곧 다시 곤 것 다 그런 그에게

뒷사 고 지 는 들 동 럼

어 웠다

그는 경험에 어 얻어( ) 동 들lsquo

각도 상 본다 는 가 우고rsquo

검 여 주 단 도 상 실험

실시 다 실험에는 동 생들 여 다

그는 실험 가 들에게 컴퓨 통

담 니 보여 주었다 에는

역 는 커다 고 마리가 었고 주 에는

다 생 들과 갈 거 등 께 시

었다 본 후 가 들 신 본 것

상 보 는 지시 다

다 그 결과 생들과 동 생( )

역 고 동 도 언

경 거 다 생 들 에 ( )

는 동 생들 생들보다 60

상 많 언 다 뿐만 니 동 생들

생들에 개별 고 보다 체 계

언 는 경 보 다 들어 동

생들 상 체 연못 럼 보 어ldquo 같rdquo

체 맥 언 시 었지만

생들 상 어 같 큰 고 가 쪽 움ldquo

직 어 같 역 고rdquo

언 시 다 경 변 시킨 그

림 시 동 생들 경 변

지만 생들 경 변 거

리지 못 다

년 학 간고사 대비2013 2 현대고 대비

ECN-0102-2013-001-000076193

게 볼 동 들 보다는 큰 그( )

림 보 에 사 과 체 맥 연결시 지각

는 경 고 체에 특 떼어 내

어 독립 보는 것 낯 어 다 에

들 사 에 고 주변 맥 에는 심 경

에 사건과 사건 사 계에 상

민감 편 다

마 지 지( ) 들만 상 연

보편 결 못 것 도 다lsquo rsquo 지각 과

과 지 과 어 보편 고 어

에 달 지는지는 많 연 통 여

어 다

리 드 니 벳 생각 지도 사- ldquo rdquo( 2004)

zb41) 의 하는 가~ 다른 것은

① ② ③

④ ⑤

얼마 그 에 동 사고 식과

사고 식 보여 주는 내 다

들 에 는 탕 고 같 게

어 겨 고 미 에 는 그 크 럼 큰 고

어리 주고 원 는 어 도 는

상 고 생각 다는 것 다 러

는 어떻게 생 것 고 과 그리 거슬

러 가 보 그 단 다

고 연 경 체 경 생 에

다 벼 사는 공동 업과 경험 많 연 역

에 고 들 연 웃과

게 지내 고 탁 연 들

들 지 연 럽게 들 다 민들

웃과 동 게 뿐만 니 는 집 과

게 다

동 시 는 생태 경 에 살 결과

들 다 사 들 사 상 에 주

울 게 었고 는 곧 체 상 과 간 사

계 시 는 낳게 었다 신 가

가 는 체에 는 원 는 동시

에 다 사 들 그 사 포 체 맥 에

다 들 간 사 연

계 체 계에 주 울 는 사고 체계

게 었다

그러 그리 연 경 그 었다 산

지 연결 는 지 건 그리고 역

에 다 런 들 업에 다 사 과

동 므 공동체에

다고 다 고 그리 들

들과는 달리 보 내 감 지 들과

지 크게 느 지 못 다 그

견 다 경우 주 쟁 통 결 는 갖

게 었다

신 사 간 계들 루어진 커다

트워크 에 게 당연 사 역시 연

계들 체 식 게 다 어 상

원 도 그 개체가 체 맥 과

계 에 고 다 게 체 맥 에 주

울 다 보 상 복 과 가변 식 게 고

상에 재 는 많 변 들 사 에 재 는 들도

게 다 들 주 태도 보

는 경우가 많다 쟁 결

통 결 보다는 통 결

는 보 다

그러 고 그리 들 개개 사 사 독

에 주 울 다 사 사 체에

어 그들 사 에 재 는 공통 규 주

고 다 상 원 에도 사

체 내 주 고 다 그들

체 여 탕 체

는 주 태도 시 고 특 사 어

주에 는지 여 그 주에 는 규

견 다 에 는 쟁 식 리

같 리 사고 체계가 달 게 었다

리 드 니 벳 생각 지도 사- ldquo rdquo( 2004)

zb42) 위 에서 사 된 설 과 가장 유사한 것은

① 크톱 컴퓨 는 본체 니 마우 루

어 다

② 곡과 시 리 는 지 과 사 루어 다는 공통

지니고 다

③ 경 고 것과는 달리

경 본 연 태 그 주변 경

④ 벽돌 능 에 사계 내내

습도가 지 다

⑤ 잰느 체 체 지닌 재 체가 없

는 재 눌 다

년 학 간고사 대비2013 2 현대고 대비

ECN-0102-2013-001-000076193

zb43) 는 립 앙 도서 이 정의 일 이다lt gt

도서 장과 이 자의 리 의 정의 연결이

적절하 않은 것은

lt gt

제 조 서 유8 ( )

도서 장은 다른 이 자의 안전을 위협하거나 도서 의①

서를 란하게 할 가 있는 자에 대하여는 도서 출입

을 제한할 있다

도서 장은 이 자가 제 조 각 호의 어느 하나의 행위를 하7②

을 때에는 이 을 하게 하거나 도서 출입을 제한할

있다

제 조자 의 대출9 ( )

도서 자 는 다음 각 호의 경 대출할 있다①

상호대차도서 간에 자 를 류하는 것을 말한다 등 다1 ( )

른 도서 과의 협 을 위하여 필 한 경

공 이 공 행 상 필 하는 경2

에 도서 장이 필 하다고 인정하는 경3

대출이 가능한 도서 자 의 위는 도서 장이 정하는②

에 따른다

제 조 상10 ( )

이 자가 도서 자 설을 더럽히거나 찢거나 뜨①

쓰게 하거나 잃어 린 경 에는 상하여야 한다

도서 장은 제 항에 따른 상 을 정하여 게 하여야1②

한다

제 조이 절차 등11 ( )

이 칙에서 정한 것 에 도서 자 설의 이 절차

이 제한 등에 필 한 사항은 도서 장이 정한다

출처 립 앙 도서- (httpwwwnlgokr)

① 는 도 리 다8

② 도 는 리 다9 1

③ 료 지 는 도 리 다9 2

④ 도 료 변상에 리10 1

⑤ 는 에 도 리 다11

3

도 다 각 같다①

공 공 다만 연1

연 간 다

매월 째 째 월2

도 도 리 그 사3

가 다고 는

도 에 미리 게1 3②

시 여 다

4

도 시간 도 여 게시 다

5

도 료 시 는 는 도①

지에 등 후

등 에 사 도②

7

는 다 각 여 는 니 다

도 료 시 상 리1 lsquo rsquo

도 료 시 훼 는2 middot

지 가 닌 곳에 식 거 담3

우는

도 보 등 보 검색열4 middot

그 에 도 질 지 여 도5

여 게시 사 는

8

도 다 거 도①

질 게 우 가 는 에 여는 도

도 가 각 어느7②

에는 지 게 거 도

9

도 료는 다 각 경우 다①

상 도 간에 료 는 것 말1 (

다 등 다 도 과 여 경우)

공 원 공 상 는 경우2

그 에 도 다고 는 경우3

가능 도 료 는 도②

는 에 다

10

년 학 간고사 대비2013 2 현대고 대비

ECN-0102-2013-001-000076193

가 도 료 시 럽 거 거①

못 쓰게 거 어 린 경우에는 변상 여

도 에 변상 여 게시1②

여 다

zb44) 위 에서 도서 장이 게 해야 할 사항에 해당하는

것을 두 쓰

년 학 간고사 대비2013 2 현대고 대비

ECN-0102-2013-001-000076193

립 도 규

1 ( )

규 립 도 립 어린 청 도(

포 다 료 시 열 시 말) (

다 에 사 규 립 도)

편 진 다

2 ( )

규 립 도 도 다 에( lsquo rsquo )

고 는 도 에 도lsquo rsquo 2 2

료 에 여 다 다만 특 료 귀

료 등 료 에 사 립 도

도 다 다( lsquo rsquo )

3 ( )

도 다 각 같다①

공 공 다만 연1

연 간 다

매월 째 째 월2

도 도 리 그 사3

가 다고 는

도 에 미리 게1 3②

시 여 다

시간4 ( )

도 시간 도 여 게시 다

등 등5 ( )

도 료 시 는 는 도①

지에 등 후

등 에 사 도②

사 료6 ( )

도 료 시 에 사 료는 도

7 ( )

는 다 각 여 는 니 다

도 료 시 상 리1 lsquo rsquo

도 료 시 훼 는2 middot

지 가 닌 곳에 식 거 담3

우는

도 보 등 보 검색열4 middot

그 에 도 질 지 여 도5

여 게시 사 는

질 지8 ( )

도 다 거 도①

질 게 우 가 는 에 여는 도

도 가 각 어느7②

에는 지 게 거 도

료9 ( )

도 료는 다 각 경우 다①

상 도 간에 료 는 것 말1 (

다 등 다 도 과 여 경우)

공 원 공 상 는 경우2

그 에 도 다고 는 경우3

가능 도 료 는 도②

는 에 다

변상10 ( )

가 도 료 시 럽 거 거①

못 쓰게 거 어 린 경우에는 변상 여

도 에 변상 여 게시1②

여 다

등 규 에 것 에 도11 ( )

료 시 등에 사

도 다

립 도- (httpwwwnlgokr)

zb45) 도서 장의 리 있는 조항으 적절하 않

은 것은

① ② ③ ④ ⑤

년 학 간고사 대비2013 2 현대고 대비

ECN-0102-2013-001-000076193

1 ( )

사가 공 는lsquo rsquo

과 여 사 원과 리

사 타 사 규

니다

개 보 보7 ( )

사는 보통신망 등 계 는 에lsquo rsquo lsquo rsquo

원 개 보 보 니다 개lsquo rsquo

보 보 사 에 는 사 개lsquo rsquo

보 취 니다 다만 사는 다 lsquo rsquo

사 계 통 공 는 경우 원 lsquo rsquo

등 개 보 당 사에 습니lsquo rsquo

원 리에8 (lsquo rsquo lsquo rsquo lsquo rsquo

)

원 에 리lsquo rsquo lsquo rsquo lsquo rsquo①

원에게 가 도 여 는lsquo rsquo 3

니다

사는 원 가 개 보 우 가lsquo rsquo lsquo rsquo lsquo rsquo②

거 사 경우 는 미 에 어 거 lsquo

사 사 운 우 가 는 경우 당rsquo lsquo rsquo

습니다lsquo rsquo

원 가 도 거lsquo rsquo lsquo rsquo lsquo rsquo 3③

가 사 고 지 경우에는 시 사에lsquo rsquo

통지 고 사 내에 니다lsquo rsquo

경우에 당 원 사에 그 사실3 lsquo rsquo lsquo rsquo④

통지 지 거 통지 도 사 내에 지 lsquo rsquo

생 경우 사는 지지 습니다lsquo rsquo

사10 (lsquo rsquo )

사는 과 지 미lsquo rsquo①

에 는 지 계 고

공 여 다 여 니다lsquo rsquo

사는 원 게lsquo rsquo lsquo rsquo lsquo rsquo②

도 개 보 신 보 포 보 보 시( )

갖 어 개 보 취 공시 고

니다

사는 과 여 원lsquo rsquo lsquo rsquo③

견 만 당 다고 경우에는

리 여 니다 원 견 만 사 lsquo rsquo

에 는 게시 거 우편 등 통 여

원에게 리 과 결과 달 니다lsquo rsquo

원11 (lsquo rsquo )

원 다 여 는 니다lsquo rsquo ①

신청 는 변경 시 허 내 등1

타 보 도2

사가 게시 보 변경3 lsquo rsquo

사가 보 보 컴퓨 그4 lsquo rsquo (

등 등 신 는 게시)

사 타 등 지 재산 에5 lsquo rsquo 3

사 타 상 거 업6 lsquo rsquo 3

는 폭 시지 상 타 공7 middot middot

에 는 보 에 공개 는 게시 는lsquo rsquo

사 동 없 리 사8 lsquo rsquo

타 거 당9

게시15 (lsquo rsquo )

원 내에 게시 는 게시 게재 는lsquo rsquo lsquo rsquo lsquo rsquo

경우 원 사가 게시 복 lsquo rsquo lsquo rsquo lsquo rsquo middot middot

등 태 언 등에 공 는

것 내에 다 원 본 게시 등 lsquo rsquo lsquo rsquo

크 능 등 여 복 는 등 태

는 것 동 것 니다

- (wwwnavercom)

zb46) 위 은 인터넷 포털사이트의 회 가입을 위한 이

약 의 일 이다 이 약 을 만드는 과정에서 생각한

내 으 적절하 않은 것은

개 보 보 가 지에 별 눠①

겠어

원 가 만들게 에②

시 주어 겠어

원들 게재 게시 다 원 크 다③

는 것 지

④ 원 지 는 뿐만 니 사가 지 는

도 께 달 지

리에 가 생 경우 사가⑤

에 다는 도 듯

1 ( )

사가 공 는lsquo rsquo

과 여 사 원과 리

사 타 사 규

년 학 간고사 대비2013 2 현대고 대비

ECN-0102-2013-001-000076193

니다

개 보 보7 ( )

사는 보통신망 등 계 는 에lsquo rsquo lsquo rsquo

원 개 보 보 니다 개lsquo rsquo

보 보 사 에 는 사 개lsquo rsquo

보 취 니다 다만 사는 다 lsquo rsquo

사 계 통 공 는 경우 원 lsquo rsquo

등 개 보 당 사에 습니lsquo rsquo

원 리에8 (lsquo rsquo lsquo rsquo lsquo rsquo

)

원 에 리lsquo rsquo lsquo rsquo lsquo rsquo①

원에게 가 도 여 는lsquo rsquo 3

니다

사는 원 가 개 보 우 가lsquo rsquo lsquo rsquo lsquo rsquo②

거 사 경우 는 미 에 어 거 lsquo

사 사 운 우 가 는 경우 당rsquo lsquo rsquo

습니다lsquo rsquo

원 가 도 거lsquo rsquo lsquo rsquo lsquo rsquo 3③

가 사 고 지 경우에는 시 사에lsquo rsquo

통지 고 사 내에 니다lsquo rsquo

경우에 당 원 사에 그 사실3 lsquo rsquo lsquo rsquo④

통지 지 거 통지 도 사 내에 지 lsquo rsquo

생 경우 사는 지지 습니다lsquo rsquo

원에 통지9 (lsquo rsquo )

사는 특 다 원에게 통지 경우lsquo rsquo lsquo rsquo

공지 게시 통 상 게시 개별 통지에7

갈 습니다

사10 (lsquo rsquo )

사는 과 지 미lsquo rsquo①

에 는 지 계 고

공 여 다 여 니다lsquo rsquo

사는 원 게lsquo rsquo lsquo rsquo lsquo rsquo②

도 개 보 신 보 포 보 보 시( )

갖 어 개 보 취 공시 고

니다

사는 과 여 원lsquo rsquo lsquo rsquo③

견 만 당 다고 경우에는

리 여 니다 원 견 만 사 lsquo rsquo

에 는 게시 거 우편 등 통 여

원에게 리 과 결과 달 니다lsquo rsquo

원11 (lsquo rsquo )

원 다 여 는 니다lsquo rsquo ①

신청 는 변경 시 허 내 등1

타 보 도2

사가 게시 보 변경3 lsquo rsquo

사가 보 보 컴퓨 그4 lsquo rsquo (

등 등 신 는 게시)

사 타 등 지 재산 에5 lsquo rsquo 3

사 타 상 거 업6 lsquo rsquo 3

는 폭 시지 상 타 공7 middot middot

에 는 보 에 공개 는 게시 는lsquo rsquo

사 동 없 리 사8 lsquo rsquo

타 거 당9

원 계 규 내lsquo rsquo lsquo②

여 공지 주 사 사가 통지 는rsquo lsquo rsquo

사 등 여 타 사 업 에 lsquo rsquo

는 여 는 니다

- (wwwnavercom)

zb47) 위 약 의 조항에서 같은 제점을 하lt gt

고 있는 조항은

lt gt

제휴 회사에 회 의 아이디 개인 정 를 전송할 있도

한 조항은 고객에게 당한 조항이다

1 7 8① ② ③

④ 9 ⑤ 10

립 도 규

1 ( )

규 립 도 립 어린 청 도(

포 다 료 시 열 시 말) (

다 에 사 규 립 도)

편 진 다

2 ( )

규 립 도 도 다 에( lsquo rsquo )

고 는 도 에 도lsquo rsquo 2 2

료 에 여 다 다만 특 료 귀

료 등 료 에 사 립 도

도 다 다( lsquo rsquo )

3 ( )

도 다 각 같다①

공 공 다만 연1

연 간 다

년 학 간고사 대비2013 2 현대고 대비

ECN-0102-2013-001-000076193

매월 째 째 월2

도 도 리 그 사3

가 다고 는

도 에 미리 게1 3②

시 여 다

시간4 ( )

도 시간 도 여 게시 다

등 등5 ( )

도 료 시 는 는 도①

지에 등 후

등 에 사 도②

사 료6 ( )

도 료 시 에 사 료는 도

7 ( )

는 다 각 여 는 니 다

도 료 시 상 리1 lsquo rsquo

도 료 시 훼 는2 middot

지 가 닌 곳에 식 거 담3

우는

도 보 등 보 검색열4 middot

그 에 도 질 지 여 도5

여 게시 사 는

질 지8 ( )

도 다 거 도①

질 게 우 가 는 에 여는 도

도 가 각 어느7②

에는 지 게 거 도

료9 ( )

도 료는 다 각 경우 다①

상 도 간에 료 는 것 말1 (

다 등 다 도 과 여 경우)

공 원 공 상 는 경우2

그 에 도 다고 는 경우3

가능 도 료 는 도②

는 에 다

변상10 ( )

가 도 료 시 럽 거 거①

못 쓰게 거 어 린 경우에는 변상 여

도 에 변상 여 게시1②

여 다

등 규 에 것 에 도11 ( )

료 시 등에 사

도 다

립 도- (httpwwwnlgokr)

zb48) 다음 정 리 의 의 으 볼 때 가장

이 적인 것은

도 시간 도 여 게시 다①

등 에 사 도②

가능 도 료 는 도 는③

에 다

④ 도 에 변상 여 게10 1

시 여 다

⑤ 도 가 각 어느7

에는 지 거 도

zb49) 를 참고하여 이 어의 성격을 설 한lt gt

것으 적절하 않은 것은

① 보 에 는 어 시 상 고 어 시lt gt lsquo rsquo

에 보여주고 다

② 진 어 어원에 견 고 다

에는 타 어 들어가는 것 다 lsquo rsquo

③ 에 들어갈 말 각각 고 어 어 신 어~

들 언어는 질 격 강 통 없었다

④ 시 우리 에 가 었지만 지 계

과 달리 들 통 사 달 어 웠

년 학 간고사 대비2013 2 현대고 대비

ECN-0102-2013-001-000076193

⑤ 크 몽골 만주 공통어가 우리 어 같

계열에 다는 에 사 특 짐

가( )

善化公主主隱 공주님

他密只嫁良置古 몰 결 고

薯童房乙 맛

夜矣卯乙抱遣去如 에 몰 고 가다

( )

始汝 會隱日恚見隱扐 만 에 본

恥隱汝衣淸隱笑 맑 웃

고 시 여 공 크다 만 다[ ] ( ) ( ) ( ) ( )始 汝 會扐

내다 에 보다 견( ) ( )恚 見 다( )隱

럽다 맑다 청 웃( ) ( ) ( ) ( )恥 衣 淸 笑

zb50) 위의 나 를 함 고 음에 답하( ) lt gt

보lt gt

( )素那或云金川 白城郡蛇山人也

운 사산

는 고 다 는( )[ ( ) ] (素那 金川 白城

사산 사 다) ( ) 郡 蛇山

삼 사- lsquo rsquo 47

에 제 된 단어 의 표 리를 조건(1) lt gt ( ) lt gt

에 맞게 서 하

건lt gt

lsquo 었고 었다 태rsquo

에 제 된 단어 동일한 표 리에(2) lt gt ( )

의해 적은 것을 나 에서 찾아 조건 에 맞게 서 하( ) lt gt

건lt gt

에 당 는 각각( ) 개 쓸 것2 단

당 는 가 여러 개 어도 개만 쓸 것 각2

개 과 도 쪽에 개만2 2

드시 지 것( )

과 동 원리 것lsquo 고

과 동 원리 것 다rsquo

태 것

가( )

素那(或云金川) 白城郡蛇山人也

소나 또는 천 이라 한다 는 성 사( ) ( ) ( )素那 金川 白城郡〔 〕

산 사람이다 현대어 풀이( ) ( )蛇山

나( )

紫布岩乎希 회

執音乎手母牛放敎遣 자 손 암쇼 노히 고

吾 不喩慙 伊賜等肹 肹 나 안디 리샤

花 折叱肹 可獻乎理音如 고 것거 도림다

다 향찰은 리말을 리 으 적은 표 이었 만 생( )

은 고 대를 넘 하고 끊어 고 말았다 랜 세

동안 갈고 닦아 체계적이었던 향찰 표 이 사라졌

을 인은 크게 두 가 나누어 생각해 볼 있다

하나는 족 사회의 한 선호도에서 찾을 있다 라 때

향찰은 주 족 계 에서 사 했을 것으 인다 한 을

알 하고서는 한자를 활 하여 리말을 리 으 표

하 란 가능하 때 이다 런데 족들은 간이 흐

를 향찰과 같은 리 표 을 익혀 사 하 다는

아 한 을 대 사 하는 쪽을 선호하게 되었다 더 이

고 초에 인재 등 을 위해 과거제도가 행되 서 한 선

호도가 더 높아졌고 결 향찰은 소 되고 말았다

또 다른 가능성은 한 어의 특성에서 찾을 있다

터 한 과 일 세 나라는 한자 화 에 속해 다

당연한 이야 겠 만 표의 자인 한자는 어를 표 하

에 매 적절하다 어의 음절은 성 ( ) ( )聲母 韻母

이 어 고 여 에 성조가 추가되어 최종 소리가 결정된

다 래서 어는 단음절을 하나의 한자 표 하 된

다 에 초성 성 종성의 세 가 소가 하나의 음절

년 학 간고사 대비2013 2 현대고 대비

ECN-0102-2013-001-000076193

을 이 는 한 어는 음절 조가 잡하고 음절의 가 많아

서 한자 차 만으 한 어의 소리를 만족 럽게 표 할

없었다 를 들어 한 어에서는 어 니 같이 음절 lsquo rsquo

이 어 단어가 얼마든 있으나 어는( ) 複數音節

자 하나 나타내 만이다lsquo [m ]rsquo 母 ǔ

한편 일 어의 표 은 핵 적 단어는 한자 적고 토는

가나라는 일 의 자 적는 이다 적인 의 를 나

타내는 은 표의 자인 한자 적고 적 계를 나

타내는 토는 표음 자 적는 셈이니 자세히 살펴

리의 향찰 표 을 쏙 빼닮았음을 알 있다 한 어 같

은 착어이 서도 일 어에만 향찰과 유사한 표 이 살아

남은 것은 일 어의 특 때 이다 일 어는 하나의 자음과

음의 결합으 음절을 이 고 침이 거의 없는 음절 언어

이다 이러한 음절의 특색에다가 토가 달한 착어라는 점

이 향찰과 유사한 표 이 살아남을 있는 비결이었다

하 만 같은 착어라도 다양한 음소 침이 달한 한

어는 향찰 표 하는 데 근 적으 한계가 있었다

zb51) 다 하여 의 행에 대한 탐 한 결과( ) lt gt 2

않은 것은

보lt gt

善花公主主隱 공주니믄 공주님( )

----------------------------------------

-

他密只嫁良置古 그 지 얼어 고 몰 결(

----------------------------------------

-

薯童房乙 맛 맛( )

夜矣卯乙抱遺去如 몰 고 가다 에 몰 고(

가다)

주동 역 동- (薯童謠『 』

에 2 ( )他密只嫁良置古

얼다 시집가다 결 다 말 lsquo rsquo

① 실질 미 지니고 므 타 타lsquo ( )rsquo lsquo [ ]

② 에 실질 미 타내고 지 는lsquo rsquo lsquo [ ]rsquo lsquo [ ]密只 密 只

계 타내는

③ 얼어는 실질 미 포 고 므 가lsquo rsquo lsquo [ ]rsquo嫁

것lsquo [ ]rsquo 良

④ 고 어간 는 실질 미 지니고 므lsquo rsquo lsquo -rsquo

것lsquo [ ]rsquo 置

⑤ 고 어미 고는 계 타내고 므lsquo rsquo lsquo- rsquo

고 것lsquo [ ]rsquo 古

가( )

엉 훈 민middot middot middot middot middot世 宗 御 製 訓 民 正 音

말 미 듕 귁에 달middot middot middot middot middot middot middot middot中 國 文 字

니 런middot middot middot middot middot middot 어린middot middot middot middot百 姓

니 고 도 내 들middot middot middot middot middot middot middot middot middot 시러middot

펴 몯middot 미middot middot 니 내middot middot middot middot middot middot middot middot 爲

어엿middot 겨 새middot middot middot 믈여듧middot middot middot middot字 니middot middot middot

사 마다 니겨 킈 middot middot middot middot middot middot middot middot middot便 安

고 미니middot middot middot middot

본 는 상( ) (象

원리에 만들어진 본) ( )形 ㄱ ㄴ ㅁ ㅅ ㅇ

에 는 가 원리에( )加劃

그리고( )ㅋ ㄷ ㅌ ㅂ ㅍ ㅈ ㅊ ㆆ ㅎ

쓰는 병 원리에 만들어진( )竝書

마지막 체( ) ( )異體ㄲ ㄸ ㅃ ㅆ ㅉ ㆅ

ᅀ 다 상 원리에 ㅇ ㄹ

지 는 삼재 상 본 본( ) ( ) ( 天地人 三才

탕 므림과 림에 ) (初ㅡ ㅣ

재)( ) ( )( )出字 再出字ㅗ ㅏ ㅜ ㅓ ㅛ ㅑ ㅜ ㅕ

병 그리고 들 에 다시( )ㅘ ㅝ ㅣ

( )ㅣ ㅢ ㅚ ㅐ ㅟ ㅔ ㆉ ㅒ ㆌ ㅖ ㅙ ㅞ

zb52) 가 에 대한 설 으 르 않은 것을( ) 두 고르

① 어쓰 규 지키고 다

② 리 고 다

③ 말 미 미 등 어 사 다lsquo rsquo

④ 개 지 다

년 학 간고사 대비2013 2 현대고 대비

ECN-0102-2013-001-000076193

⑤ 어 원 에 가 도 고 다

엉 훈 민世 宗 御 製 訓 民 正 音

말 미 듕귁에 달 니

런 어린 니 고 도middot

내 들 시러 펴 몯 미 니middot

내 어엿 겨 새 믈여듧

사 마다 니겨middot 킈 고

미니

훈민 언 본- lsquo rsquo 5 (1459 )

zb53) 위의 에 대한 현대어 풀이가 르~ 않은 것

① 우리 말 과 달

② 어리 말 고 는 것 어도

③ 신 생각 마 껏 펼 는 사 많다

④ 게 생각 여

⑤ 사 마다 게

zb54) 훈민정음 언해 에는 한 을 창제한 동 가 드러나

있다 훈민정음 창제의 정 과 내 이 잘 연결된 것

① 주 신 말 미 듕귁에 달

② 민 신 내 어 겨

③ 신 뻔 킈 고 미니

④ 실 신 사 마다 니겨

⑤ 귀 신 계 주 는 훈민 신과 거리가

가 엉 훈 민( ) middot middot middot middot middot世 宗 御 製 訓 民 正 音 

말 미 귁에 中 國 달 文 字

니 런 어린 니 百 姓

고 도 내 들 시러 펴 몯

미 니 내 어엿 爲 겨 새

믈여듧 니 사 마다 니 字

겨 킈 고 미니 便 安

훈민 언 본- lsquo ( )rsquo ( ) 5 (1459 )訓民正音 世祖

( )

[ 1 ]

동 룡 샤 마다 복( ) ( ) ( )海東 六龍 天福

시니 고 동( ) ( )古聖 同符 시니

[ 2 ]

매 니 곶 여

미 므 니 그 내 러

가 니

[ 125 ]

우 미리( )千世 샨( )定 에( )漢水北 累仁

누 개 샤 복 업 시니( ) ( ) 開國 卜年

신( )聖神 니 샤도 경 근민 샤 욱( )敬天勤民

드시리 다

님 쇼 산 가( ) ( )洛水 山行

미드니 가

어 가- lsquo ( )rsquo 27龍飛御天歌

다 우리신 니쓰고 다만 만 쓰( )

거 샹 귀쳔 다보게 러 귀

여 쓴 도 신 보 가 고 신 에

말 어 보게 각 에 사 들

고 본 몬 능통 후에

죠 죠 니

드 도 만 공 에 사

드 미 죠 고 고 여 보 죠

보다 얼마가 거시 어신고 니 첫

가 죠 니 죠

민 들 어 신 샹

귀쳔 도보고 어보 가 만 늘

고 폐 에 만쓴 죠 민

도 러보지못 고 보니 그게 엇지

심 니 리 보 가 어 운건 다

니 쳣 말마 지 니 고 그

쓰 에 가 우 지 지

몰 거 본후에 가 어 지

고 그니 쓴편지 쟝 보

년 학 간고사 대비2013 2 현대고 대비

ECN-0102-2013-001-000076193

쓴것보다 듸 보고 그 마 니 쓴 고

어 못

그런고 에 리 과 가

만 쓴 못 민 말만 듯고

고 편 그 못 보니 그사 단

병신 못 다고 그사 식 사

니 만 고 다 과 그사

만 고 다 과 업 사 보다 식 고

죠 도 고 각 과

견 고 실 직 귀쳔 간에 그

고도 다 것 몰 귀죡 보다

사 우리 신 귀쳔 다 업

시 신 보고 과 지 게 랴

시니 샹 귀쳔 간에 우리 신 걸

간 보 새지각과 새 걸 미리

독립신- lsquo (1896)rsquo

zb55) 친 어 나의 제 장( ) 2 매 함축적

의 가 가장 유사한 것은

① 지 눈 내리고 매 득 니 내 여 가

사- lsquo rsquo

② 도 어 리듯 그 게 어 다

주 사- lsquo rsquo

③ 눈 살 다 죽 어 린 과 체 여

눈 새벽 지 도 살 다

눈- lsquo rsquo

④ 삶 근심과 고단 에 돌 거니는 여 거 는

여 리 내린 살가지 에 눈 리 눈 리

택 그 생 에- lsquo rsquo

⑤ 늘 러 고 러

청룡 룡 어 개 루 우

신경림 계- lsquo rsquo

zb56) 친 를 위 가 나 에 나타난A B ( ) ( )

세 어의 특 에 의거하여 세 어 표 하

그 산 고 공 도 맑지만

A

주변에 쓰 리는 어리 사 많다

B

건lt gt

식 가 에 타 어 특징에( ) ( )

거 과 어쓰 는 고 지 말 것

A

B

zb57) 가 의( ) 달 아ㆍ 다 의 ( ) 나셔에서 알 있는

세 어 개화 어의 특 을 비 하여 조건 에lt gt

맞게 서 하

건lt gt

어에 는lsquo 개

어에 는 다 태rsquo

zb58) 은 가 는 다 에 나 는 절lt 1gt ( ) lt 2gt ( )

일 를 췌한 것이다 의 의 가 lt 1gt (1)~(2)

유사한 말을 에서 찾아 쓰lt 2gt

보lt 1gt

런 (1) 어린 니 고百 姓

도 내 들 시러 펴 몯 미

사 마다 (2) 니겨 便 安

킈 고 미니

보lt 2gt

죠 고 고 여 보 죠

보다 얼마가 거시 어신고 니 첫 가

죠 니 죠 민

들 어 신 샹 귀쳔

도보고 어보 가 만 늘 고

폐 에 만쓴 죠 민 도

러보지못 고 보니 그게 엇지 심

니 리

년 학 간고사 대비2013 2 현대고 대비

ECN-0102-2013-001-000076193

lt 1 gt

동 룡 샤 마다 복 시( ) ( ) ( )海東 六龍 天福

고 동 시니( ) ( )古聖 同符

lt 2 gt

(A) 매 니 곶

여 니

미 므 니 그 내

러 가 니

lt125 gt

우 미리 샨 에( ) ( ) ( ) 千世 定 漢水北 累

누 개 샤 복 업 시 니( ) ( ) 仁開國 卜年 聖

신( ) 神 니 샤도 경 근민 샤( ) 敬天勤民

욱 드 시 리 다

님 쇼 산 가 ( ) ( )洛水 山行

미드니 가

- lt gt龍飛御天歌

zb59) 장과 내 상 유사한 성격의 조는125

① 뫼 고 고 고 고

어 그린 많고 많고 고 고

어 러 는 울고 울고 가느니

도 견- lt gt

② 강 에 드니 몸 다

그믈 고 가니

뒷 뫼 엄 언 니( )藥

-

③ 말 없는 청산 태 없는 다

값 없는 청 없는 월

에 병 없는 몸 별 없 늙 리

-

④ 가마귀 골에 가지 마

낸 가마귀 새

청강에 것 시 몸 러 가( ) 淸江

-

⑤ 진 골에( ) 白雪

가 매 는 어느 곳에 었는고

에 갈 곳 몰( ) 夕陽

색-

zb60) 위 에 나타난 세 어의 특 으 적절하 않은

것은

① 룡 어 주격 사에 당 는 가 사( ) lsquo rsquo六龍

고 다

② 샤 어에도 어 주체 쓰 다

는 것 다

③ 매 어 달리 사 택에 어

가 지 지지 고 다

④ 므 원 상 직 어 지 다

⑤ 드시리 다 주체 과 상 께 사

고 다

수고 하셨습니다hearts hearts

년 학 간고사 대비2013 2 현대고 대비

ECN-0102-2013-001-000076193

보닷컴에 공 는 별 보는 고등

들 여 주 는

들 습니다 슷 동 지

가 복 는 것 도가

니 복 여 습 시고 거 시

니다

정답 해설

1) 정답[ ] ④

해설 다른 것은 두 특정 업이나 단 내에서 사[ ]

하는 일종의 은어 사회 언에 해당한다 러나

는 언이 아니라 단과대학을 여서 단대 사lsquo rsquo lsquo rsquo lsquo④

대학을 여서 사대라고 한 말에 해당하 일rsquo lsquo rsquo

사회에서도 널리 쓰이 사회 언이라 할

없다

2) 정답[ ] ⑤

해설 사회 언은 같은 단 내에서 쓰이는 언어이[ ] lsquo rsquo

동일 단끼리는 단결 과 친 감을 형성하는

능을 하 리적 안감이 일어나 않는다

3) 정답[ ] ③

해설 사람이라는 차 적 표현에 대한 대안적 표현이[ ]lsquo rsquo

인 아내 처 등으 볼 있다lsquo rsquo

4) 정답[ ]⑤

해설 남성은 주 격 체를 사 한다[ ]

5) 정답[ ] ⑤

해설 흑인은 검다라는 뜻을 가 고 있을 뿐 인[ ]lsquo rsquo lsquo rsquo lsquo rsquo

다 열등한 뜻을 내포하 않는다

6) 정답 살 색 첫 작품[ ] - -

해설 살색 혹은 킨색은 한 인의 피 색을 뜻[ ] lsquo rsquo lsquo rsquo

하는 것으 인종 차 을 추 고 출 이주민

의 평등 을 침해할 있어 년 표 이2005

살 색으 이름을 꾸었다 처녀작은 처녀라lsquo rsquo lsquo rsquo lsquo rsquo

는 단어가 가 고 있는 곡된 성 인 을 한 것

으 첫 작품정도 꾸어 사 하는 것이 좋다lsquo rsquo

7) 정답[ ] ⑤

해설 호는 아들에게 해체를 사 하고 있다[ ] ① ②

장 을 성하는 청자는 자 의 아 느리 아lsquo

들 세 이다 호는 아 느리에게 해rsquo ③

체를 사 하고 있다 호가 느리 아 에게 ④

사 한 해 체 아들에게 사 한 해체는 두 비lsquo rsquo lsquo rsquo

격 체에 해당한다 호는 자 의 아랫사람인 ⑤

느리에게 아들과 마찬가 해체를 사 하는 것이

상 이 만 임 을 한 느리에게 고마 과 쁨

존 의 표 를 하 위해 자 의 아 에게 말하듯

해 체를 사 하고 있다

8) 정답[ ] ③

9) 정답[ ] ⑤

10) 정답[ ] ①

해설 청자 할아 가 장의 주체 아 다 높을[ ] ( ) ( )

경 에는 압존 에 의해 장의 주체를 높이 않는lsquo rsquo

다 러 아 서가 아닌 아 는으 계 lsquo rsquo lsquo rsquo lsquo

니다 가 아닌 있 니다 표현하는 것이 르rsquo lsquo rsquo

11) 정답 당이 당을 쫒았다 당이[ ]

당에 다

해설[ ]

12) 정답[ ] ⑤

해설 서 다른 높임표현을 통해 청자에 대해 리[ ] ⑤

적 거리감을 나타내는 인 은 이 아니라 현정이

다 가 에서 현정은 에게 해 체를 사 함으 써 ( )

친근감을 드러낸다 나 에서 연 을 게을리하는 역 ( )

도 들 때 에 화가 난 현정이 선생님에게 항의하

는 장 에서는 하 체를 사 하여 리적 거리lsquo rsquo

가 어졌음을 나타내고 있다

13) 정답[ ] ①

해설 는 는 얼 빛이 날과 어찌 다르 고[ ] lsquo rsquo

라는 뜻으 전과 달리 임이 화자를 않고

있음을 알 있다

14) 정답 달리 후 가 있다 이를 통해 경[ ] lt gt

쾌한 음악성을 형성하고 노 젓는 상황을 체적으

형상화하는 역할을 한다

15) 정답[ ] ①

16) 정답[ ] ⑤

해설 다 의 자연은 를 성찰하게 하는 대상[ ] ( )⑤

이자 정의 대상이다 의 자연은 자 의 상황과 ⑤

처 를 드러내는 경으 서의 역할을 하 이

이 없다

17) 정답[ ] ③

해설 는 빈천 을 해결하고자 했으나 강산[ ] lsquo ( )rsquo 貧賤③

과 풍 을 달라는 에 거절하 다고 함으 써 자

연에 대한 애정을 드러내고 있으 는 않는

임에 대한 망을 개에게 전가 켜서 임에 대한 리

을 드러내고 있다

18) 정답[ ] ③

년 학 간고사 대비2013 2 현대고 대비

ECN-0102-2013-001-000076193

19) 정답[ ] ⑤

해설 고상한 음악가의 이름을 리말 꽝 럽[ ]

게 꿈으 써 언어유희를 통해 음을 유 하고 있

다 이는 고상한 척하는 총 를 비꼼으 써 비판적

태도를 드러내는 것이 대상을 꽝 럽게 표현

하여 총 의 허 과 사치를 풍자하고 있다

20) 정답[ ] ⑤

해설 는 작품 속 경에 대한 설 이 드러나는 것이[ ]

서 자의 주 적인 견해가 접적으 드러나는 것이

아니다

21) 정답[ ] ⑤

22) 정답[ ] ②

23) 정답[ ] ④

24) 정답[ ] ①

해설 적강 티프는 주인공의 비 한 출생이나 능[ ] ①

과 이 있는 것으 조정의 능함을 풍자하는lsquo rsquo

것과는 거리가 다

25) 정답 픔 나[ ] ( )

해설 의 음악은 고통 는 사람들을 위 하고 아픔[ ] lsquo rsquo

을 치유해 주는 능을 한다고 할 있다 의 lt gt

픔 도 소 된 이 과 더 어 살아가는 따뜻한 마음lsquo rsquo

을 상 한다

26) 정답[ ] ⑤

해설 에게 선천적으 주어 각 장애라는 역경[ ]

은 의 이라는 가사 연 을 있다lsquo rsquo

27) 정답[ ] ④

해설 는 장 란 선 에게 은 개인적인 인상을[ ]

소녀 장정 등으 표현한 것이다lsquo rsquo

28) 정답[ ] ②

해설 담자가 피 담자의 언어적 표현이나 비언어[ ]②

적 표현 하 독자는 담의 위 나 피

담자의 감정 상태를 알 있다 이를 통해 독자는

담 상황을 더 생생하게 느낄 있고 피 담자

를 더 잘 이해할 있게 된다

29) 정답[ ]③

해설 일상생활과 역도 선 서의 성과에 된 것에서[ ]

역도를 하 서 겪는 어 과 내적 고민으 화제를

전화하 위한 것이다

30) 정답[ ] ①

해설 릿속에 새겨 넣듯 이 억되도 함 세상[ ] ② ③

살이가 힘들고 고생 러 속 하여 자유를 ④

가 없는 고통의 상태를 비유적으 이르는 말

적의 침입을 막 위해 쌓은 축 켜야 할⑤

대상을 비유적으 이르는 말이다

31) 정답[ ] ④

해설 이 의 종류는 전 으 인 사건 경[ ] lsquo

비평을 성 소 삼는다rsquo

32) 정답[ ] ④

해설 근은 삼대독자 태어났음을 에서 확인할[ ]

있다 형제들과의 담은 이뤄 가 없다

33) 정답[ ] ⑤

해설 근은 가난에도 하고 화가를 꿈꾸었다[ ] (3

단 또한 다른 화가 망생들은 정 육을)

위해 상 학 학 해 유학 에 랐 만

근은 다른 을 찾아야 했다 단 세에(5 ) 18

근은 조선 전람회에 입선하 다 단 의(6 )

만종은 인간과 자연이 엮어 가는 경건한 조화 을lsquo rsquo

나타낸다

34) 정답[ ] ①

해설 근이 속에서도 창작활동을 추 않고[ ]

하는 닭은 은 세상과 타협할 르는

근이 세상의 이해를 하 위한 가장 떳떳한 단

이 때 이다

35) 정답[ ] ⑤

해설 전 은 서 자의 주 적인 평이 리는 것이[ ]

만 위 제 은 인 이 살았던 대 사회적 경

을 통해 객 적인 인 의 을 제 하고 있다

36) 정답[ ] ⑤

해설 전 은 인 사건 경 비평이라는[ ] lsquo rsquo⑤

성 이 어져 있다

37) 정답[ ] ①

해설 이 은 동양인과 서양인의 사고 에 차이가[ ]

있다는 것을 대조를 통해 설 하고 있다 또 쓴이

의 제자가 축 경 를 러 가서 경험한 일화를

통해 동양인이 서양인에 비해 주 상황에 더 많은

주의를 인다는 주장을 뒷 침하고 있다

38) 정답[ ] ④

39) 정답[ ] ②

40) 정답[ ] ②

41) 정답[ ] ④

42) 정답[ ] ③

43) 정답[ ] ④

44) 정답 도서 의 휴 일 도서 의 이 간 도서의[ ]

해설 도서 장은 임의 정한 휴 일과 도서 이[ ]

간 도서의 상 등을 게 할 의 가 있다

년 학 간고사 대비2013 2 현대고 대비

ECN-0102-2013-001-000076193

45) 정답[ ] ①

해설 제 조의 정 휴 일 의 휴 일의 사전 게[ ] 3

는 도서 장의 의 조항에 속한다

46) 정답[ ] ①

해설 개인 정 호 의 를 제 하 했 만 항[ ]

나눠서 제 하 않고 대 나열하고 있다

47) 정답[ ] ②

해설 제 조의 내 을 회사는 다른 회사 협[ ] 7 lsquo

계약을 통해 서비 를 제공하는 경 회 의 아이디

등 개인 정 를 해당 회사에 전송할 있다는 내rsquo

이 있으 의 제점을 제 할 있다②

48) 정답[ ] ④

해설 는 도서 장의 의 에 해당하고 나 는 도[ ] ④

서 장의 리에 해당한다

49) 정답[ ] ③

50) 정답 은 음독으 적었고 은 훈독으 적었[ ] (1)

다 과 동일한 표 리 적은 것은 이고 (2) ce

과 동일한 표 리 적은 것은 이다ab

51) 정답[ ] ③

52) 정답[ ] ①②

53) 정답[ ] ③

54) 정답[ ] ③

55) 정답[ ] ①

56) 정답 른 죠코 어린 노 하니라[ ] A B

57) 정답 세 어에서는 활 형이 칙적으[ ] lsquo rsquoㄹㅇ

나타났 만 개화 어에서는 활 형이 쓰 다 lsquo rsquo ㄹㄴ

58) 정답 호 가 흔[ ] (1) (2)

59) 정답[ ] ④

60) 정답[ ] ③

Page 11: 현대고대비 국어 - chamsoriedu.com 「콘텐츠산업진흥 법」외 에도 저작권 의하여 ... 다른주체에게어떤동작을하도록만드는것을나타내는

년 학 간고사 대비2013 2 현대고 대비

ECN-0102-2013-001-000076193

③ 다운 신 과 신 습 늘에 죄 짓고

간 상에 내 신 다

④ 사 주고 말 어떻게 다 헤 릴

어떻게 다 리

⑤ 신 산 생각 고 들어가니 경개 가 ( )景槪

매우 뛰어 고 경 산 다

거리 연[ ] ( )弘治

간 에 공신 후 에 언(1488~1505) ( )正言

주 는 벼슬 심 늦도 식( ) ( )主簿 劉尋

없어 과 께 산에 드리고 신 태

몽 꾼 에 만고 웅 상 지닌 들 낳

키운다 그 후 신 들 에 역심( )逆心

담 귀 등 심 여 리 귀 보내

고 지 죽 는 도망 가다가

만 다 에 에 어 니

헤어지게 다

에 에 어 니 헤어지게

다 그 후 사 들에게 우연 돌

생 다가 어느 열 살 었다 열 살

지 다가 우연 귀 견 는

그것 그 살 도 었고 그

귀 본 신도 지 죽고 마 고

크게 운다

( )

에는 강 주 는 재상 살고 었

니 시 에 과거에 격 여 승상 벼슬 다가 간

신 만 벼슬 그만 고 고 돌 었

다 그러 신 지 가 지 못 여 상

가 못 결 는 상 여 원 니

신 들 그 직간 꺼 다 그 에 도

담과 귀가 강 승상 가 미워 다

강 승상 마 본 에 갔다가 돌 는[A][ ( )本府

에 우편 주 에 다가 색( ) ( ) 右便 酒店

에 어리었는 청룡 에 지 늘

여 통곡 고 사 는 꿈 꾸었다] 마

상 게 생각 여 새 다리다가 새벽

닭 울고 가 달 갔다 가 보니

과연 어 동 가 가에 울고 는지 달

들어 그 고 사 에 어 말

는 어 어 에 어 가ldquo

닭 곳에 우느냐 니 울rdquo

그 고 답 여 말 다

는 경 동 에 사는 언 주 공 들ldquo

니다 께 간신 만 연경 귀 가

시다가 에 죽 사 에 는 닭에

도 에 죽고 니다rdquo

강 승상 말 듣고 크게 낯 변 말

것 웬 말 냐 근 동 ldquo (老

못 갔 니 그 사 변 여)患

런 변 었단 말 가 주 는 신 다

같 에 벼슬 다가 는 가 많 들어 고

돌 는 주 가 게 꿈 에 생

각 겠느냐 생각지 못 다 미 지 간

지지 말고 께 가 략rdquo ( ) hellip hellip

죽게 주 사당에 단 도 러운

겠느냐 말 말고 시는지 rdquo

어 없어 강 승상 가니 그곳

월계 었다

다( )

가가 고 지 사 들 가( )櫛比

통 는 리가 과

답게 꾸민 누각과 큰 집들 늘 고

게 식 가 어 들 태운 가고

었다 략 강 승상에게는 들 없고 ( ) hellip hellip

다만 만 었다 가 낳 에

가 색 타고 내 에게 말

는 니다 미원 과ldquo ( )紫薇垣

연 맺고 었는 께 강 집( )緣分

보내 에 니 게 여겨 주십시

rdquo

거늘 미 가운 낳 니 가

고 거동 단 다 시 짓 쓰 고

는 없었 니( )音律 여 가운

지 는 짝 룰 만 사 없었다 가 사

여 사 감 게 고 지 못 고 염 는 만다

다가 당에 거 고 식같

러 내니 고귀 상 루 말 다 ( )相

어 울 도 다 귀 사 없 ( )富貴爵祿

고 웅 걸 만고 었다 승상 매우 뻐

내당 들어가 에게 사 니( ) 內堂

역시 매우 거워 말 다 도 마 ldquo

사 는 승상께 그 게 말 시니

상 여러 말 지 말고 사 도 시다rdquo

( )

승상 에 고 결 과 ldquo

여 에게 말 다 내가 늙 말 에 지

만 었는 지 보니 늘

다 에게 탁 겠 ( )

다 신 꿇고 눈 리rdquo

여 었다 주시고 슬 에 ldquo ( )膝下

고 시니 감사 룰 가 없습니다 다만 가슴

에 통탄 사 쳐 습니다 복 없어

생사 결 여 내 얻는 것( )兩親

식 도리가 닙니다 것 러울 뿐 니

다rdquo

상 그 말 듣고 슬 에 어 고 말

것 에 맞 어 웅변 ldquo

년 학 간고사 대비2013 2 현대고 대비

ECN-0102-2013-001-000076193

게 리 는 다 집 시 공도 여

고 가 에 가가 어진 만 개 공신

었 니 도 러워 마 시고 rdquo 시

택 여 니 운 신 과 신

습 늘에 죄 짓고 간 상에 내 신

다 략 지낸 후에 튿 승상 ( ) hellip hellip

니 승상 거운 마 지 못

마( )

듯 월 러 생 열다 살 었

다 에 승상 어진 사 얻고 만 에 근심 없었

다만 주 가 간신 에

죽 것 생각 마 곧 어 곤

다 그 에 주 원통 어

없 고 여 시 가 거늘 생 만

여 다

말 감격 러우 간신 에 가득 여ldquo

고 니 께 상 듣지 니 것

니다rdquo

승상 듣지 고 가

퇴 재상 공달 집에 거 고 상 지어

승지 러 께 리

( )

뒷 거리 강 승상 에게 상 리지[ ]

만 여움 사 귀 가게 다 강 승상

몸 는 연 가 헤어

리 다 산 들어간 룡사 승 만

게 다 승 만 우 다릴

과 들고 략 다 담

원 여 에게 복 고 어

공격 다 담에게 여러 가( ) 天子

복 등 여 다 단

신 리쳐 담 사 고 에게

간 후 태후 태 여 지에 고생

지 심과 강 주 여 개 다 헤

어 어 니 내 고 담 리

벼슬에 귀 누리게 다

zb21) 위 의 인 간 계를 같이 나타냈을lt gt

때 에 대한 이해 가장 적절하 ~ 않은 것은

① 계에 주 는 계 심 열

상 에 다고 다( ) 水深火熱

② 계는 견원지간 고 다( ) 犬猿之間

③ 계는 달리 막역지 계 고( )莫逆之交

④ 연결 사 컬어 재 가 고( )才子佳人

⑤ 는 생 과 볼 ( )匹夫匹婦

가 재 는 는 심 고 매사에 생( )

각 고 능 도 어 가 에게 많lsquo rsquo

도움 사 다 그는 에게 거 에

꺼리 없 거 났다고 는

매우 싫어 고 신 들

는 사 다

내가 지 리에( ) 1970

사 실에 지 월간ldquo

편집 고 어 었다rdquo

어느 없 가 쑥 다 도 어 10

후 다 산 시 럼 어 엇 어 ( ) lsquo怡山

다시 만 랴 니 그는 재 그룹 승 운rsquo

사가 고 는 고 거 누

주는 가 없는 가가 어 다시 만 게 것

었다

다 보통 것 닐러 그( ) ldquo 어낸 ( )

틀어주 가 루 러 허 에

싶어 키 틀어주 그( )

가 루 허 우간 곡 틀어 주는 루 못

는 는 고 닝께 고 지

들어 사는 고 가 다는 건 에 그 집에

rdquo

그런 단 어들 어 새벽에 떼죽 거

다 고 어 보니 죄다 허 게 집어진

는 것 었다 가 실내 꿴 뛰어 지만

없는 었다

어떻게 된 거야 한동안 넋나간 듯이 서 있던 총ldquo rdquo

가 하고많은 사람 에 하필이 유자를 겨냥하 은

말이었다 쎄유 아마 새에 고뿔이 들었던 개비네 ldquo

유rdquo

유자는 러 딴청을 하 다 야 고 가 에서 ldquo

감 가 들어 죽는 고 두 어rdquo 총 는 가 혐의

자 나 되는 것처럼 화풀이를 하 드는 것이었다( )嫌疑者

라 이 어쩌 어 유( ) ldquo rdquo ldquo rdquo

애유 이런 잔인 도 한 것들 같으니ldquo ( ) rdquo殘忍無道 helliphellip

총 는 탱천 하여 쩌 를 하 다( ) 憤氣撐天

아하니 아는 자는 다 동 하여 호통을 쳤으 하나 혈

압을 생각하여 참는 눈치 다 달리 처리헐 두 ldquo

잖은감유rdquo

총 의 성 을 덧들이 고 한 말이 아니었다 가 할

년 학 간고사 대비2013 2 현대고 대비

ECN-0102-2013-001-000076193

있는 것이 말고는 없었 때 에 게 뒷동

산을 달은 거 다

이 유자소전- lsquo rsquo

zb22) 의 상황을 속담으 표현한 것으 적절한 것은

① 루 곳 게 마 다

② 에 맞고 강에 눈 다

③ 늘 도 다

④ 도 사 다

⑤ 에 가도 신만 리 다

거리 공신 후[ ]

에 주 는 벼슬 심 늦도( )主簿

식 없어 과 께 산에 드리고 신

태몽 꾼 에 만고 웅 상 지닌 들

낳 키운다 그 후 신 들 에 역심

담 귀 등 심 여 리 귀 보내고

지 죽 는 도망 간다 그

만 고 에 에 어 니

헤어지게 다 지 가 사 들에

사 들 별 고 없 다니었다

마 마 돌 다니 걸 여 고

어 곤 다 에는 동쪽에 고 에

는 쪽에 니 가 에 리는 엽 가는

없 니 늘 다니는 었다

얼 말 죽 사 같고 림새가 말 니었

다 가슴 에 고 등 삼태

헌 에 니 달 가 도리 ( )奇男子

어 걸 었 담 만 열 도 ( ) ( )傅說 慇

고 만났고 만 갈( ) ( )武丁 伊尹

도 탕 만났( ) ( )成湯 渭水

여상 도 주 만났는 월( ) ( ) ( ) 呂尙 周 文王

같 러가 도 어느 열 살

늘과 집 삼고 사 에 쳐 거리에

어 다가 곳에 니 다 ( ) 楚

지 다가 사 보고 가에 다다( )長沙

니 망 가에는 원 리가 슬 고 가

가 내리는 사 에는 갈매 가 갈 뿐 었다

쪽 돌 보니 가 우거 고

가 사 보 었다 그곳에

가니 는 사( ) 汨羅水

는 다 주 가 쓰고 죽고

곳 었다

마 감 여 에 가 사 살펴보니

에는 삼 고 그 에( ) 屈三閭

는 만고 월 과 지 가는 그 들( )風月

가 어 었다( ) 路程記

동쪽 벽 에 새 운 어 거늘 그

보니

월 에 경 주 는 간신에게ldquo ( )敗

보고 연경 귀 가다가 에 죽 rdquo

거늘 그 보고 에 거꾸러

통곡 말

우리 연경 간 만 니ldquo ( )燕京

에 지 살 상에 엇 겠는

가 에 고 에 었 니

상에 살 것 가 도 께 지리 rdquo

고 가에 내 가니 울 리가 에 지

사 쳤는지 심 심 것 가

에는 강 주 는 재상 살고 었

니 시 에 과거에 격 여 승상 벼슬 다가 간

신 만 벼슬 그만 고 고 돌 었

다 그러 신 지 가 지 못 여 상

가 못 결 는 상 여 원 니

신 들 그 직간 꺼 다 그 에 도

담과 귀가 강 승상 가 미워 다 강 승상 마

본 에 갔다가 돌 는 에 우편 주( ) ( )本府 右便

에 다가 색 에 어리었는 청룡( ) 酒店

에 지 늘 여 통곡 고

사 는 꿈 꾸었다 마 상 게 생

각 여 새 다리다가 새벽닭 울고

가 달 갔다 가 보니 과연 어 동 가

가에 울고 는지 달 들어 그

고 사 에 어 말

는 어 어 에 어 가ldquo

닭 곳에 우느냐rdquo

니 울 그 고 답 여 말 다

는 경 동 에 사는 언 주 공 들ldquo

니다 께 간신 만 연경 귀 가

시다가 에 죽 사 에 는 닭에

도 에 죽고 니다rdquo

강 승상 말 듣고 크게 낯 변 말

것 웬 말 냐 근 동ldquo ( )老患

못 갔 니 그 사 변 여 런 변

었단 말 가 주 는 신 다 같

에 벼슬 다가 는 가 많 들어 고 돌

는 주 가 게 꿈 에 생각

겠느냐 생각지 못 다 미 지 간 지지

말고 께 가 rdquo

뒷 거리 강 승상 도움 죽 고[ ]

년 학 간고사 대비2013 2 현대고 대비

ECN-0102-2013-001-000076193

고 그 과 결 여 사 가 다 그러 강

승상 에게 울린 상 강 승상 귀 가고

과 헤어 리 승 만 게 다

승 우 다릴 과

들고 략 다 담 원

여 에게 복 고 어 (天

공격 다 담에게 여러 가 복) 子

등 여 다 단신

리쳐 담 사 고 에게 간

후 태후 태 여 지에 고생 지

심과 강 주 여 개 다 헤어

어 니 내 고 담 리 벼

슬에 귀 누리게 다

미상- lsquo ( )-劉忠烈傳

zb23) 위 과 의 서사 조를 비 한 것으 적절하lt gt

않은 것은

보lt gt

믿지 고 결 여 곱

낳 다 곱째 공주 낳 가

리게 다 리 만 고 진 공주는 lsquo rsquo

리공 미 리공 에 키워진다 월

러 과 가 죽 병에 걸 는 승에 는

어 산다고 다 여 들에게 탁

지만 거 리 는다 리 는 과

승 다 승 지 가는 에 많

만 지만 보살 도움 사 도 다

그러 승 신과 결 여 시

들어 주겠다고 다 리 는 그 결

여 들 곱 낳 후에 신

얻게 다 돌 리 는

에 과 상여 만 지만 여 과

살 낸다 훗 리 그 공 우 죽 사

승 도 는 신 다

리-lsquo rsquo-

① 복 결말에 고 다

② 웅 에 탕 고 다

③ 시 겨 내고 귀 누리는lsquo rsquo

보 리 는 월 재 신 다lt gt lsquo rsquo

④ 과 보 리 는lsquo rsquo lt gt lsquo rsquo

도움과 어 신 능 극복 고

⑤ 등 여 시 겪는lsquo rsquo

보 리 는 닌 지lt gt lsquo rsquo

림 시 겪는다

가 각 고 에( ) ( )却說

살 없었다 략 사 들 슬 에 어 lt gt

가에 내 고 가고 싶 가 고 후

워 경

사 들 별 고 없 다니었다 lt

략 얼 말 죽 사 같고 림새가 말gt

니었다 가슴 에 고 등

삼태 헌 에 니 달 가 ( )奇男子

도리어 걸 었 담 만 열 도( )傅說

고 만났고 만 갈( ) ( ) 殷 武丁

도 탕 만났( ) ( ) (伊尹 成湯 渭

여상 도 주 만났는) ( ) ( ) ( )水 呂尙 周 文王

월 같 러가 도 어느 열 살

늘과 집 삼고 사 에 쳐 거리에

어 다가 곳에 니 다 ( ) 楚

지 다가 사 보고 가에 다다( )長沙

니 망 가에는 원 리가 슬 고 가

가 내리는 사 에는 갈매 가 갈 뿐 었다

쪽 돌 보니 가 우거 고

가 사 보 었다 그곳에

가니 는 사( ) 汨羅水

는 다 주 가 쓰고 죽고

곳 었다

에는 강 주 는 재상 살고( )

었 니 시 에 과거에 격 여 승상 벼슬 다

가 간신 만 벼슬 그만 고 고 돌

었다 략 강 승상 마 본 에 갔다가 돌 lt gt ( )本府

는 에 우편 주 에 다가 색( ) ( ) 右便 酒店

에 어리었는 청룡 에 지

늘 여 통곡 고 사 는 꿈 꾸

었다 마 상 게 생각 여 새 다리다

가 새벽닭 울고 달 갔다 가

보니 과연 어 동 가 가에 울고 는지

달 들어 그 고 사 에

어 말

는 어 어 에 어 가ldquo

닭 곳에 우느냐rdquo

니 울 그 고 답 여 말 다 lt

략gt

년 학 간고사 대비2013 2 현대고 대비

ECN-0102-2013-001-000076193

생각 여 가 고 시 는ldquo ( )大人

상에 다시없는 니다 살 엇 겠습니

에 돌 가시고

가에 돌 가 니 살 마 없습니

다 략 어 없어 강 승상 가니rdquo lt gt

그곳 월계 었다

다 강 승상에게는 들 없고 다만 만( )

었다 가 낳 에 가 색

타고 내 에게 말

는 니다 미원 과ldquo ( )紫微垣

연 맺고 었는 께 강 집( )緣分

보내 에 니 게 여겨 주십시

rdquo

거늘 미 가운 낳 니 가

고 거동 단 다 시 짓 쓰 고

는 없었 니 여 가운( ) 音律

지 는 짝 룰 만 사 없었다 가 사

여 사 감 게 고 지 못 고 염 는 만다

다가 당에 거 고 식같 러

내니 고귀 상 루 말 다 어 ( )相

울 도 다 귀 사 없고 ( )富貴爵祿

웅 걸 만고 었다 승상 매우 뻐 내

당 들어가 에게 사 니 역( ) 內堂

시 매우 거워 말 다

도 마 사 는 승상께ldquo

그 게 말 시니 상 여러 말 지 말고 사

도 시다 략 시 택 여rdquo lt gt

니 다운 신 과 신 습 늘에 죄

짓고 간 상에 내 신 다

다 내고 들어가 사 살펴보니

고 것 는 다 말 어 고

는 다 어 신 에 ( )新房

에 신 과 신 가 평생 연 맺었( )緣分

니 사 주고 말 어떻게 다 헤 릴

어떻게 다 리 지낸 후에 튿 승

상 니 승상 거운 마 지 못

( ) 듯 월 러 생 열다 살

었다 에 승상 어진 사 얻고 만 에 근심

없었 다만 주 가 간신

에 죽 것 생각 마 곧 어

곤 다 그 에 주 원통

어 없 고 여 시 가 거늘 략 lt gt

략 거리

강 승상 에게 상 리지만 여움

사 귀 가게 다 강 승상 몸 는

연 과 헤어 리 다

마 각 생 강 승상 집 쪽( )

늘 보고 없 가 신 신 생각 니

없고 어 없었다 는 어떻게 도리가 없다

여 산 에 들어가 리 고 어 도 닦

고 다 그 산 보고 가

다가 곳에 다다 니 에 큰 산 었다 많

우리 골짜 가 늘 는 가운 색

에 고 갖가지 가 짝 어 ( )花草

었다 략 주 보니 lt gt ( ) (一柱門 黃

산 룡사 어 었다) lsquo rsquo 金大字

산 들어가 고승 다 그( ) ( ) 山門 高僧

거동 보니 눈 눈 듯 고

변 같 귀는 어 에 늘어 니( ) 白邊

맑고 어 골격과 신 평 니었

다 염주 에 걸고 짚고 포 ( )六環杖

삼에 어진 쓰고 생 보고 말

승 연 여 상공 시는 동 에ldquo

가 맞 지 못 니 승 십시 rdquo

생 크게 말 다

생 가 여 어 고ldquo

없 다니다가 우연 곳에 사 만 것

그 시 생 어떻게 고 습니

rdquo

승 답 여 말

어 산 승 에ldquo ( ) ( )南岳 衡山

시어 승에게 탁 내 낮 시경에 경 lsquo 12

동 에 사는 심 들 가 것 니 내쫓

지 말고 습니다 마 승rsquo

다가 상공 림새 보니 경 사 에 보

습니다rdquo

생 그 말 듣고 편 고 편( )

슬 승 들어가니 여러 승 들

가워 다 승 에 들어가

후에 그 편 니 곳 경 었다 상( ) 仙境

고 신 편 다 후 는 승과

께 병 도 탐 고 경도 게( )兵書

게 었다 게 니 지 에 가객 ( ) ( )大明天地 佳客

년 학 간고사 대비2013 2 현대고 대비

ECN-0102-2013-001-000076193

없고 산 에 리 만 본 ( ) 廣德山

신 상 사 살 는 만

우고 늘 월 신 과 늘 ( )日月聖神

산 신 들 다 니 그 재( ) 名山神靈

주 민 누가 당 겠는가 낮 공

zb24) 다 에 해당하는 내 으 적절하( ) 않은 것은

① 강 티 통 당시 능 다

② 상계 지상계 경 는 원 계 드러

③ 실에 어 없는 실 가 타 는

④ 뛰어 재주 어 가진 고

등 다

⑤ 가 직 개 여 평가 내리는

편집 평 타 다lsquo rsquo

가 본격 가 동 것 지( )

다 단 상 에2003 lsquo rsquo

들어가 드럼 연주 다 취미 생 달리

들었다는 보 우 가 들ldquo

어 틱 린 도 다 고 말 다rdquo

경 는 가 망 없( ) lsquo

티 원 고 답 다 신과 같 시각rsquo

는 습 상상 만 도 감동

다 시각 연주 동시에

열 상 는

티 원 그런 열 경 럽다는 것 다

다 역시 엄청 다 본( )

에 복 들

고쳐 가고 다 신 에 얼

마 지는 고 리가 는 지도 생님

가 훈 고 많 고쳐 다

고 말 다

그러 직도 에 지 는 다 그는

체격 지 못 게 가 큰 만

체 운동 훈 과 께 체 늘 동 50

는 게 고 말 다

에게는 꿈 다 통 누 가( )

주겠다는 것 그 꿈 다 신 극복 는

과 에 큰 경험 들도 느 게

주고 싶다는 것 다

마 슬 마다( ) ldquo 통

낼 었 것 럼 고통 는 사 들

고 겠다 고rdquo

말 다 달 루 첫 낸 lsquo rsquo

첫 드 심 집에 는 리듬 드 2

루 에 도 보고 싶다 집 에는 직(RampB) 3 4

사 곡 도 보 고 싶다고 포 다middot

zb25) 에서 가장 유사한 의 를 닌 어를lt gt

찾아 쓰

lt gt

나는 이제 너에게도 픔을 주겠다

사랑 다 소 한 픔을 주겠다

겨 거리에서 개 놓고

살아 추위 떨고 있는 할 니에게

값을 으 서 뻐하던 너를 위하여

나는 픔의 평등한 얼 을 여 주겠다

내가 어둠 속에서 너를 를 때

단 한 도 평등하게 어주 않은

가마니에 덮인 동사자가

다 얼어 죽을 때

가마니 한 장조차 덮어주 않은

한 너의 사랑을 위해

흘릴 르는 너의 눈 을 위해

나는 너에게 이제 너에게도 다림을 주겠다

지 울 포동 여고 생들17

틈 없 가득 체 에 맑 울

다 죽 듣 생들 사 에

연 는 탄 다 객들 도 는lsquo rsquo

가 보 주 공 맹 가 운 는

단 그룹사운드 루 보컬 맡고 는lsquo rsquo

시각 지 었다17 1

근 다만과 가 거lsquo rsquo lsquo

꿈 고 퇴 내가 다rsquo

간 간에 지 지 연 생들 짧lsquo rsquo lsquo rsquo

가 운 듯 리에 어

연 다 내 사 고 퇴lsquo rsquo

과 루 들 결 다시 돌lsquo rsquo

들 고 사 들 에 당당

것 니다 내 태어

볼 없었 크고 열여

년 학 간고사 대비2013 2 현대고 대비

ECN-0102-2013-001-000076193

에도 고 시 얻지 못 다

감지 없는 시각 상태 다

신 지에 고 상 원망 도

단다 어느 가 에 시각 에 ldquo

어 그런 듣고 다 보니 내가 게 lsquo

살 는지 도 눈 고 싶rsquo lsquohelliphellip

보 는 생각만 들 고 그 가 들에게rsquo

도 내고 들도 고 많 었죠 들 rdquo

었 지 새 는 에 쑥 러운 색

어났다

생에 것 단연 었다lsquo rsquo

공연에 거 꿈lsquo rsquo

는 다 특 가사 갑게 는 운 lsquo

벽 에 당당 마주 어 언 가 그 벽

고 늘 어 거운 상도

없죠 내 삶 에 웃 그 께

는 다고 다rsquo

들었 그냥 런 도 고만 여ldquo lsquo rsquo

겼죠 그런 꾸 가사 미 새 다 보

니 통 는 가사 는 생각 들 고 (

가 게는 시각 는 생각 들고 들) ( )

마다 듣고 큰 얻었어 rdquo

에 진지 게 가에 미 가

zb26) 의 에 들어갈 말 적절한 것은lt gt ~

lt gt

난 난 꿈이 있었죠

고 찢겨 남 하여도

내 가 히 과 같이 간 했던 꿈

혹 때 누 가가 뜻 를 비 음

내 등 뒤에 흘릴 때도

난 참아야 했죠 참을 있었죠

날을 위해

늘 걱정하듯 말하죠

헛된 꿈은 독이라고

세상은 끝이 정해 책처럼

이 돌이킬 없는

현 이라고 helliphellip

래 난 난 꿈이 있어

꿈을 믿어

나를 켜

저 차갑게 서 있는 이란 앞에

당당히 마주칠 있어

출처 가 거위의 꿈 작사 이적 작곡 동률- lsquo rsquo ( )

① ② ③ ④ ⑤

가 떴다는 들 만 지만( ) lsquo rsquo

늘 겸 다 에 주 연 우승 지 간에도 3

단 생님께 만 지 고 고 만ldquo rdquo

큼 늘 겸 신 계 가

고 다

에게는 꿈 다 통 누 가

주겠다는 것 그 꿈 다 신 극복 는 과

에 큰 경험 들도 느 게 주

고 싶다는 것 다

슬 마다 통 낼ldquo

었 것 럼 고통 는 사 들

고 겠다 고rdquo

말 다 달 루 첫 낸lsquo rsquo

첫 드 심 집에 는 리듬 2

루 에 도 보고 싶다(RampB) 집 에는 직34

사 곡 도 보 고 싶다고 포 다

미 는( ) (26) 어 헤헤헤 웃다가 어ldquo rdquo

허허허 웃었다ldquo rdquo ldquo rdquo 같 도 고

상 다 는 같 도 다( ) 壯丁 킹 들lsquo

다 는 역도 보 그 다 지만 그는rsquo

뷰에 지 다 운동만 지 ldquo

것 지 간에 여러 사 도 역rdquo helliphellip

었다 그런 엇 그 마 움직 는지 보 쯤

지 담 사 다 훈 없어 그는 티

지 림 었다 태 다 갔다 는 습

마 집 럼 편 게 보 다

주말에는 주 엇 보내

주말에도 별 주 에 청ldquo

고 에 가고 도 쳐

에 듣고 보 에 갈 가 별 없

어 산 시 게 고 들어 2002

거 매 여 지냅니다 시 과 지훈 rdquo

다 근 간 과 진실 그리고 싶어( )

가 다 근에게 그것 진리 다 거 다 없

거 고 다 없 는 것 진리

다 근 진리는 후 쪽 었다 신산( )辛酸 삶

었 질곡( )桎梏 역사 에 지냈 가

눈에 든 것 료 단 료 게 보

것 었다 그것 그 에 겨우겨우

슬 슬 생 어가는 간들 었다

리 과 단 리 고리에 검 마

없 거리 돌

상 것 없는 등 근에게 상

과 진실 엄 ( )儼存 다는 사실 리는 가

실 고 가 과 역경 에 도 근 내 포

없었 후 보루( )堡壘 다 도 365

도 간 근 여

시 것 다

년 학 간고사 대비2013 2 현대고 대비

ECN-0102-2013-001-000076193

다 공주 그림 가 근 경- ( ) ldquo rdquo(

2009)

zb27) 작가의 주 적인 각이 드러난 것은~

① ② ③ ④ ⑤

가 신 지에 고 상 원망( )

도 단다 어느 가 에 시각 에 ldquo

어 그런 듣고 다 보니 내가 lsquo

게 살 는지 도 눈 고 싶rsquo lsquohelliphellip

보 는 생각만 들 고 그 가 들에게rsquo

도 내고 들도 고 많 었죠 들었rdquo

지 새 는 에 쑥쓰러운 색

어났다 략 [ ]

경 는 가 망 없 티lsquo

원 고 답 다 신과 같 시각rsquo

는 습 상상 만 도 감동

다 시각 연주 동시에

열 상 는 티

원 그런 열 경 럽다는 것 다 략 [ ]

슬 마다 통 낼ldquo

었 것 럼 고통 는 사 들

고 겠다 고rdquo

말 다 달 루 첫 낸 lsquo rsquo

첫 드 심 집에 는 리듬 2

루 에 도 보고 싶다 집 에는 직(RampB) 3 4

사 곡 도 보 고 싶다고 포 다

식 누 가-

고 싶어

다 역도 미 담 고 사( )

질 주말에는 주 엇 보내[ 1]

답 주말에도 별 주 에[ ] ldquo

청 고 에 가고 도 쳐

에 듣고 보 에 갈 가 별

없어 rdquo

질 계 고 슬슬 도 는 것 닙니[ 2]

답 다 들 눈 에 보 고 뿐 보[ ] ldquo

다 열심 고 어 상에 도 들지만 상

지키는 것 들다고 에 도달

그것 지키 훨 많 rdquo

질 들 살 고 리 는[ 3]

거운 들 체 리느 는다

답 가 고 게 체 어[ ] ldquo ( ) 級

느 도 계가 니 살 는 것도 고역 지만

살 우는 것 들어 는 살

체 리 고 어도 어도 실 갔다

쑥 어 rdquo

질 거리에 슷 연 여 들[ 4]

보는 간 상 지

답 상 다 체 게 리지 못[ ] ldquo

거 주변에 는 그 거 누 보지

못 고 뻐지고 싶 에 체 리는 에

타 워 지만 는 어울 는 것보다 는

시간 운동만 는 건 니에 사복 lsquo rsquo

고 사복 는 말에 들 웃지만 늘 운동복

고 지내니 사러 갈 도 어 rdquo

질 역도가 말 단 식 운동 니[ 5]

답 가 내는 만 클 업 보[ ] ldquo

그러니 만 쓰는 식 운동 니다

만 다고 거운 것 들 는 건 니거든 연

도 고 가지 동 에 도 여러 가지

복 들

보식 역도 여 미-

zb28) 가 에 대한 설 으( ) 않은 것은

① 시각 우 지 시 에 지

고 망 가는 태도 달 고 다

② 언어 과 언어 복 사 여

담 내 생각 게 는 가

③ 직 감 그 마 것

럼 생생 게 느껴지는 과 주고 간 내

없 리 어 억 게 다

④ 담 내 식 리 여 담 삶 습

과 가 시 여 독 에게 감동과 훈 다

⑤ 직 진 담 직 누

지 못 는 독 에게 생생 상 달 주고

담 욱 게 다

zb29) 나 의 각 의 의도를 설 한 것으 적절하( ) 않

년 학 간고사 대비2013 2 현대고 대비

ECN-0102-2013-001-000076193

은 것은

① 질 담 상 보여 주 것 다1

② 질 담 과 그에 삶 태도 보여2

주 것 다

③ 질 역도 겪는 어 움에 역도3

과 것 다

④ 질 같 연 여 갖는 고민 는지 말4

주 는 것 다

⑤ 질 역도가 과 고 운동 는 것5

담 가 말 주 는 것 다

가 만진 것 다( ) 3

감 달 다고 다 억 에( ) 音感

지워 지만 당시 청 탁 리도

다고 다 드럼 웠다 4

에 갈 마다 드럼 는 리가 신 게 들

다고 다 눈 볼 가 없 니 엔ldquo

는 는 님 틱 에 여 주

다 드럼과 연 맺 과 들 주었다rdquo

식 누 가-

고 싶어

역( ) 도가 말 단 식 운동 니

가 내는 만 클 업에 보ldquo

그러니 만 쓰는 식 운동 니다 만

다고 거운 것 들 는 건 니거든 연

도 고 가지 동 에 도 여러 가지 복

들 시 는 상 상

드는 상 에 맞춰 실 에 는 여러

펼쳐집니다rdquo

략( )

늘 에 는 어 만 것 같

가 에 사 고 사 사ldquo

겠어 든 에 가 경 만 고

울 는 사 겠어 rdquo

보식 역도 여 미-

다 가 운 는 어 어( ) ldquo rdquohelliphellip

월 새벽 시 태 없 거웠고1965 5 6 1

는 없 그 병원에 퇴원 집

가는 마지막 마 고 마 내 거 다

가 죽 간신 에 실 다 사는 어느5 lsquo

가 죽 는 말 가 식 다 신rsquo

상에 각 시키는( )刻印 에 실

어느 가는 후 민 가가 근 었다lsquo rsquo

는 간 과 진실 그 다는( ) ldquo

에 단 평 견 가지고 다 내

가 그리는 간상 단 고 다 지 다 는 그들

가 에 는 평 지 니 그리고 어린

들 미지 겨 그린다rdquo

마 근 간 과 진실 그리고 싶어( )

가 다 근에게 그것 진리 다 거 다 없

거 고 다 없 는 것 진리

다 근 진리는 후 쪽 었다 신산(辛酸 삶)

었 질곡(桎梏 역사 에 지냈)

가 눈에 든 것 료 단 료 게

보 것 었다 그것 그 에 겨우겨우

슬 슬 생 어가는 간들 었

다 리 과 단 리 고리에 검

마 없 거리 돌

상 것 없는 등 근에게 상에

과 진실 엄 다는 사실 리는 가( )儼存

실 고 가 과 역경 에 도 근 내

포 없었 후 보루(堡壘 다 도)

도 간 근365

여 시 것 다

월 강원도 림리에( ) 1914 2 21

삼 독 태어났다 어 근 복

그것 그리 가지 못 다 근 곱 살

지는 산 산업에 실 고 답마 에 내

갔다 근 그림 럼 쫓 다니 가 시 것

다 상 진 것도 가 었다

러 가 에도 고 근 가 꿈꾸었다 근

가 꿈꾸게 것 보통 업

원색도1926 만lsquo rsquo 었다

공주 그림 가 근 경-

zb30) 에 대한 설 가장 른 것은~

① 역도가 과 운동 도 질

② 리는 는 다 lsquo rsquo

③ 들었지만 그럭 럭 는 다 lsquo rsquo

④ 가 게 보 시 말 다

⑤ 보 병 는 지 상 lsquo rsquo

는 말 다

년 학 간고사 대비2013 2 현대고 대비

ECN-0102-2013-001-000076193

시간 많지 다 청량리 생 병원

마지막 상 경 릿 게 들어 다 그 는 십

만 큰 가 상 말 다

지 못 들 마 갈 고 돗

도시민들 싹 싹 탔다 가 시

월에 병원에 원 가 폐 진 몸도4 ( )疲弊

갈 미 지 못 고 었다 가는 얼마( ) 解渴

지 생 에 생각 가

마감 는 신 평생 십 만에

가 과 많 닮 다고 생각 지는

가 운 는 어 어ldquo rdquo 1965helliphellip

월 새벽 시 태 없 거웠고 는5 6 1

없 그 병원에 퇴원 집 가

는 마지막 마 고 마 내 거 다 가

죽 간신 에 실 다 사는 어느 가5 lsquo

죽 는 말 가 식 다 신rsquo

상에 각 시키는 에 실 어느( ) lsquo刻印

가는 후 민 가가 근 었다rsquo

ldquo 는 간 과 진실 그 다는 에

단 평 견 가지고 다 내가 그

리는 간상 단 고 다 지 다 는 그들 가

에 는 평 지 니 그리고 어린 들

미지 겨 그린다rdquo

근 간 과 진실 그리고 싶어 가

다 근에게 그것 진리 다 거 다 없 거

고 다 없 는 것 진리다

근 진리는 후 쪽 었다 신산 삶 ( )辛酸

었 질곡 역사 에 지냈 가 눈에( )桎梏

든 것 료 단 료 게 보 것

었다 그것 그 에 겨우겨우 슬

슬 생 어가는 간들 었다 리

과 단 리 고리에 검 마

없 거리 돌 상

것 없는 등 근에게 상에 과 진실

엄 다는 사실 리는 가 실( )儼存

고 가 과 역경 에 도 근 내 포 없었

후 보루 다 도 도( ) 365堡壘

간 근 여 시 것

간에 지닌 가 근 1914 2

월 강원도 림리에 삼 독21

태어났다 어 근 복 그것 그리

가지 못 다 근 곱 살 지는 산

사업에 실 고 답마 에 내 갔다 근

그림 럼 쫓 다니 가 시 것 다 상

진 것도 가 었다 러 가 에도

고 근 가 꿈꾸었다 근 가 꿈꾸게

것 보통 업 원색1926

도 만 었다lsquo rsquo

그림 가 근 경 공주- ldquo rdquo ( 2009)

zb31) 다음 이 같은 의 성 소에 해당하 않은

것은

사건 평① ② ③

④ 주 ⑤ 경

가 운 는 어 어ldquo rdquo 1965helliphellip

월 새벽 시 태 없 거웠고 는5 6 1

없 그 병원에 퇴원 집 가

는 마지막 마 고 마 내 거 다 가

죽 간신 에 실 다 사는 어느 가5 lsquo

죽 는 말 가 식 다 신rsquo

상에 각 시키는 에 실 어느( ) lsquo刻印

가는 후 민 가가 근 었다rsquo

는 간 과 진실 그 다는 에ldquo

단 평 견 가지고 다 내가 그

리는 간상 단 고 다 지 다 는 그들 가

에 는 평 지 니 그리고 어린 들

미지 겨 그린다rdquo

근 간 과 진실 그리고 싶어 가

다 근에게 그것 진리 다 거 다 없 거

고 다 없 는 것 진리다

근 진리는 후 쪽 었다 신산 삶 ( )辛酸

었 질곡 역사 에 지냈 가 눈에( )桎梏

든 것 료 단 료 게 보 것

었다 그것 그 에 겨우겨우 슬

슬 생 어가는 간들 었다 리

과 단 리 고리에 검 마

없 거리 돌 상

것 없는 등 근에게 상에 과 진실

엄 다는 사실 리는 가 실( )儼存

고 가 과 역경 에 도 근 내 포 없었

후 보루 다 도 도( ) 365堡壘

간 근 여 시 것

간에 지닌 가 근 1914 2

월 강원도 림리에 삼 독21

태어났다 어 근 복 그것 그리

가지 못 다 근 곱 살 지는 산

사업에 실 고 답마 에 내 갔다 근

그림 럼 쫓 다니 가 시 것 다 상

진 것도 가 었다 러 가 에도

고 근 가 꿈꾸었다 근 가 꿈꾸게

것 보통 업 원색1926

도 만 었다lsquo rsquo

공주 그림 가 근 경- ldquo rdquo ( 2009)

년 학 간고사 대비2013 2 현대고 대비

ECN-0102-2013-001-000076193

zb32) 위 을 작성하는 과정에서 되어 활 된 자

어 것은

신 사 료① 연보②

고③ ④ 들과 담

⑤ 에 평

는 간 과 진실 그 다는 에ldquo

단 평 견 가지고 다 내가 그

리는 간상 단 고 다 지 다 는 그들 가

에 는 평 지 니 그리고 어린 들

미지 겨 그린다rdquo

근 간 과 진실 그리고 싶어 가

다 근에게 그것 진리 다 거 다 없 거

고 다 없 는 것 진리다

근 진리는 후 쪽 었다 신산 삶 ( )辛酸

었 질곡 역사 에 지냈 가( )桎梏

눈에 든 것 료 단 료 게 보

것 었다 그것 그 에 겨우겨우

슬 슬 생 어가는 간들 었다

리 과 단 리 고리에 검 마

없 거리 돌 상

것 없는 등 근에게 상에 과

진실 엄 다는 사실 리는 가 실( )儼存

고 가 과 역경 에 도 근 내 포

없었 후 보루 다 도 도( ) 365堡壘

간 근 여 시

것 다

간에 지닌 가 근 1914 2

월 강원도 림리에 삼 독21

태어났다 어 근 복 그것 그리

가지 못 다 근 곱 살 지는 산

사업에 실 고 답마 에 내 갔다 근

그림 럼 쫓 다니 가 시 것 다 상

진 것도 가 었다 러 가 에도

고 근 가 꿈꾸었다 근 가 꿈꾸게

것 보통 업 원색1926

도 만 었다lsquo rsquo

질 루 마 가 도 린다 경건

움 느껴지는 경 다 훗 근 그림에

과 는 거 것( )裸木

만 간과 연 엮어 가는 경건 움lsquo rsquo

니었

같 가가 고 싶었 근에게 그 꿈에 다

가가는 지 다 다 가 지망생들 규 미

상 에 진 고

에 지만 근 다 다 근

미 에 운 것 보통 시 미 시간

다 그런 그에게 없는 연습 가가

통 다 가 귀 시 지 도

얻는 뛸 듯 뻤지만 마 도 가 에

듯 는 었 에 어린 근 주 에

에 그림 그리고 지우고 복( )粉板

시간 가는 게 루 보냈다

근 그 갈 가가 것 열여( )渴求

었 다가 미1932 lsquo rsquo ( lsquo

미 에 다 다는 고 마rsquo) lsquo rsquo

가 근 집 고도 지는 시골 경

그린 그림 다 후 근 에 1943 22

지 미 에 그림 고

에 걸쳐 다 미 근 가

동 는 었다

공주 그림 가 근 경- ldquo rdquo ( 2009)

zb33) 위 의 내 과 일치하는 것은

가 근 가 꿈 포 다①

근 당 가들과 께 에 다②

살 근 가 걷20③

게 었다

④ 만 통 근 역경 겨내는lsquo rsquo

느 다

⑤ 근 간 과 진실 그리 에 그 에

드러 는 간상 단 다

계 시 주 근 건강

걸었다 신 과 간에 상 다 건강

신 는 눈에도 다 근 쪽 눈 뿌 게

보 지 과에 다 다 시 지지 고 결

내 었다 시 지만 마 막막

다 늦어 결 근 쪽 눈 고 말 다

쪽 눈 근에게는 쪽 눈 었고

계 었다 그 근 는 여 그lsquo rsquo

다 근 에 같 그림 그 었다1950

시 그림 는 여 쪽lsquo rsquo

고 어 마주 고 는 그림1963

여 과 동 다 마 복

그린 듯 눈 내리 새 게 다 지

사 다 근 게 복 것

복 상과 타 는 근 상

가 떳떳 단 었고 근 그리고

간 과 진실 에 다가가 가 근다

운 었다 근 신에게 당당 지 그리고

그 다 근 그림에 단 복 보다

년 학 간고사 대비2013 2 현대고 대비

ECN-0102-2013-001-000076193

태 도 그리고 극 보다 과

얻 여 었다 과 통

근 그리고 는 재 고 에 질

만들고 특 것 다

공주 그림 가 근 경- ldquo rdquo( 2009)

zb34) 의 이유에 대해 추 한 것으 적절하 않은 것

상과 타 시도①

보다 과 얻②

근 신에게 당당 지③

④ 간 과 진실 에 다가

⑤ 태 도 얻

근 가가 었지만 그 다니 가

럼 어지지 다 복과 쟁 거쳐 시

는 가 근에게 생계 사 에

운 사 다 에 키에 건( ) 178cm死鬪

체 근 에 동 역 업( )荷役

가 생계 다 쟁

에는 동에 운 상우 주 미

죄 사 에 그림 그리는 시 다 그곳에

에 동 역 업 것에

결 것 럼 보 다 지만 그런 것만도

니었다 그림 그리는 고는 지만 매 근

는 극 간 과 별 없는 경 리 그림

벽에 그리는 것 었다 우도 리 없었다 근

트 는 우 그림 그 다 생

계 그림 단 것 다

후 근 지 신 계 리에 미

엑 리 겼다 근 곳에

건 사 크 에 미 들 ( )

상 상 그 다 근 갖 다 겪

냈다 그리고 결 그 돈

신동에 어 사리 집 마 다 마 ㄷ

루 심 쪽에는 과 엌 쪽에는 건

었다 건 주고 근 가 에

여 살 다 심 에는 지 집어

쓰고 지만 곳 근 가 에게 러웠

보 리 다 근 과 마루 업실 삼 그림

그 다 신동 마루는 근 그림에 등 는 lsquo rsquo

같 상들 지 다 시 고

에 들 폐허가

가 업실 었다

공주 그림 가 근 경- ldquo rdquo( 2009)

zb35) 위 에 대한 설 으 적절한 것은

업 시 여 훈과 감동 다①

에 주 평 드러 다②

사 사 등 식 과 ③

④ 다 근거 시 여 삶에

⑤ 살 시 사 경 께 여

습 시 다

가 시간 많지 다 청량리 생 병원( )

마지막 상 경 릿 게 들어 다 그 는

십 만 큰 가 상 말 다

지 못 들 마 갈 고 돗

도시민들 싹 싹 탔다 가 시

월에 병원에 원4 가 폐( )疲弊

진 몸도 갈 미 지 못 고 었다( )解渴 가는

얼마 지 생 에 생각

가 마감 는 신 평생 십 만에

가 과 많 닮 다고 생각 지는

가 운 는 어 어( ) ldquo rdquohelliphellip

월 새벽 시1965 5 6 1 태 없 거웠고

는 없 그 병원에 퇴원 집

가는 마지막 마 고 마 내 거 다

가 죽 간신 에 실 다 사는 어느5 lsquo

가 죽 는 말 가 식 다 신rsquo

상에 각 시키는 에 실( )刻印

어느 가는 후 민 가가 근 었다lsquo rsquo

다 는 간 과 진실 그 다는( ) ldquo

에 단 평 견 가지고 다 내

가 그리는 간상 단 고 다 지 다 는 가

에 는 평 지 니 그리고 어린 들

미지 겨 그린다rdquo

근 간 과 진실 그리고 싶어( )

가 다 근에게 그것 진리 다 거 다 없

년 학 간고사 대비2013 2 현대고 대비

ECN-0102-2013-001-000076193

거 고 다 없 는 것 진리

다 근 진리는 후 쪽 었다 신산( )辛酸 삶

었 질곡 역사 에 지냈( )桎梏

가 눈에 든 것 료 단 료 게 보

것 었다 그것 그 에 겨우겨우

슬 슬 생 어가는 간들 었다

리 과 단 리 고리에 검

마 없 거리 돌

상 것 없는 등 근에게 상에

과 진실 엄 다는 사실 리는 가 실( )儼存

고 가 과 역경 에 도 근 내 포

없었 후 보루 다( ) 堡壘 도 365

도 간 근 여

시 것 다

마 같 가가 고 싶었 근에게 그 꿈( )

에 다가가는 지 다 다 가 지망생들

규 미 상 에 진 고

에 지만 근 다 다 근

미 에 운 것 보통 시 미 시간

다 그런 그에게 없는 연습 가가

통 다 가 귀 시 지 도

얻는 뛸 듯 뻤지만 마 도 (

는 었 에 어린 근 주 에)

에 그림 그리고 지우고( )粉板

복 시간 가는 게 루 보냈다

zb36) 전 의 성 소가 아닌 것을 고르

① 평 ② 사건 ③ 경

④ ⑤ 훈

늘 지 상에 살고 는 사 들 억 도가10

고 그리 지 통 고 는 사 들( )知的

그보다 훨 많 억 도는 고 지 20

통 다 그런 지 고 2500

그리 간 보는 과 사 에

매우 달 뿐만 니 과 에 도 극

루고 었다 미 운 그런 들

살고 는 동 과 사 들 사고 식에

큰 가 다는 다

고 그리 들 우주 개별 고 독립

사 들 생각 지만 고 들 우

주 연 질 간주 다 같( ) 看做

각 도 들에게는 연 질

었지만 그리 들에게는 미 들 결 었

다 고 과 그리 들 사 같

는 동 과 사 에 도 견 다

지심리 미 마 드 겐트 는

살 들에 에 지 다

연 동 과 상 다 과 같 실험

다 크 만든 미드 도 보

여 주고 그 상 닥 고 주었다lsquo (Dax)rsquo

실 닥 는 재 지 는 것 실험 가lsquo rsquo

만들어 낸 다 그런 다 개 다 체 보

여 주었는 는 미드 지만 틱

만들었고 다 는 재료는 크 지만

달 다 그러고 어 것 닥 지 사 들에게 고 lsquo rsquo

게 니 들 주 같 고 는

체 택 고 동 들 같 재료 만들어진 체

택 다 러 는 심지어 살짜리

들에게 도 타났다 것 곧 과 동

다 상 보고 다는 것 미 다

개별 사 보고 고 동 연 질 보

고 는 것 다

동 들 주변 상 에 맞 어 동 고

에 다 사 들 태도 동에 보다 많

주 울 다 동 가 미시간 에

에 경험 다 그는 미식

경 보러 가게 었는 경 체는 매우 재미 었

주변 들 동에 질 다 그 는

들 계 어 상태 경 다

어 들 에 에 그 시 가 계 가

진 것 다 상 살펴 는 말 들 lsquo rsquo

에 그는 에 시 어 도 뒷사

생각 곧 다시 곤 것 다 그런 그에게 뒷

사 고 지 는 들 동 럼

어 웠다

생각 지도 리 드 니 벳-

zb37) 다음 위 의 내 전개 으 만 인lt gt

것은

lt gt

대조의 통해 대상이 닌 특성을 설 하고 있다

일화를 제 하여 자 의 주장을 뒷 침하고 있다

유추의 을 사 하여 독자의 의해를 돕고 있다

대상이 형성되는 과정을 간적 서에 따라 서 하고 있

① ②

③ ④

년 학 간고사 대비2013 2 현대고 대비

ECN-0102-2013-001-000076193

가 우리가 말 고 쓰는 든 단어가 사 에 는( )

것 니다 사 격에 가 는 지만

어 사 과 같 특별 는 사 니lsquo rsquo

단어 격 보 단어가 사 에

등재 어 다 리 리 사 는 단어 도 그

것 시 사 는 어 고 사 에

격 보 것 니다

러 얼 은 사전에 를 있는가 이에 대한 답lsquo rsquo

은 얼 이 유행어인가 아닌가에 따라 갈라 다 이 단어lsquo rsquo

는 년 어 자 에 랐고 쓰이고 있으2002 lsquo rsquo

유행어라고 하 에는 생 이 다 런데 계속

을 유 하 서 사전에 등재될 자격을 획득할 것인가 이

에 대한 답을 내리 는 히 어 다

여 서 가 를 고 해 볼 있다 첫 는 이 단어

를 써야 할 필 가 속적으 있는가 하는 점이다

상주의 열풍에 휩 인 사회 위 에 편 해서 퍼 말

이 얼 인데 과연 런 위 가 속될 것인가 이에lsquo rsquo

대해 필자의 생각은 정적이다 사회 위 가 뀌

런 말을 쓸 일이 없어 것이다

다음은 단어의 성이다 단어의 성이 사회적으 거

감이 없으 계속 사 될 가능성이 높다 런 에서

얼 은 좋은 조건이 아니다 익히 알 졌듯이 이lsquo rsquo

말은 얼 과 청소년층에서 속어 사 하는 이 결합lsquo rsquo lsquo rsquo

된 말이다 얼 에서 얼 을 리하는 조어 도 lsquo rsquo lsquo -rsquo

어에서는 매 낯선 이다 이것만으 도 거 감을 갖

는 사람들이 있다 더 나 속어 결합한 말이다 얼 lsquo rsquo

이 널리 퍼졌다 해도 은 여전히 청소년층의 속어lsquo rsquo

남아 있다 속어는 자연 럽게 아 자리에서나 쓰 에는

담 러 말이다 러한 담을 하고 사

역을 넓혀 가는 속어도 없 는 않다 특히 얼 은 lsquo rsquo

에도 종종 등장한다 만큼 거 감이 많이 희석되었다

고 할 있다 러나 일상의 자연 러 대화에서도 거

리낌 없이 등장하는가 게 는 되 않았다고 생

각한다

얼 이 유사어인 쌈 등을 만들어 내고lsquo rsquo lsquo rsquo

있으니 살아남을 있을 것이라고 는 견해도 있을 것

이다 러나 간이 나 서 유사어를 포함하여 든

말이 사라 사 는 많다 유사어가 많다는 것이 생 을

유 할 있는 절대적인 조건은 아니다

나 언젠가 터 사람들은 어느 단에서 얼 이 가장( )

쁜 사람을 가리켜 얼 이라고 르고 있다 이 얼lsquo rsquo lsquo rsquo

이라는 단어가 최근 어사전에 라 항간에 논란이 일고

있다 아닌 게 아니라 얼 은 유행어처럼 인다 생 lsquo rsquo

도 리 래되 않은 것 같고 언제 사라 도 알

없다 게다가 젊은이들 사이에서 주 쓰일 뿐이다 이런

단어를 사전에 는다는 게 하 이 없어 이 도

한다

러나 속단은 이다 차근차근 따져 볼 일이다

선 얼 이 일 적 유행어인 아닌 주의 게 들여다lsquo rsquo

볼 필 가 있다 유행어란 유행에 따라 빠르게 유포되었

다가 단 간 내에 소 되는 단어나 를 가리킨다

얼 은 인터넷을 통해 속히 퍼 말이다 하 만 일lsquo rsquo

적인 유행어처럼 단 간 내에 사라 않았을 뿐 아니라

현재 도 잦은 빈도 사 되고 있고 앞으 도 상당

간 사 될 것으 측된다 한 언 재단의 뉴 검 lsquo rsquo

색 사이트에 따르 얼 은 년 에 처음 나타난lsquo rsquo 2001

이후 꾸 히 사 되고 있다

이 같은 사 빈도는 얼 이 일 적 유행어 는 현lsquo rsquo

저히 다르다는 것을 여 다 장 간의 생존 만으 도

얼 은 이 한 어의 어휘 에 를 자격을 얻었다lsquo rsquo

고 할 있다 더 이 이라는 비 적 정제된 매체에

높은 빈도 쓰이고 있 않은가 사 빈도 측 에서

필통이나 연필과 같은 단어 대등하거나 더 많이 쓰lsquo rsquo lsquo rsquo

다는 것은 결코 가 게 볼 일이 아니다

이제는 사전이 언어 현 을 빠르게 하는 게 덕인

대가 되었다 세계적으 유 한 의 사전들도 경쟁

적으 어를 고 있다

하 만 얼 은 젊은이들이나 쓰는 속어라고 흠을 잡을lsquo rsquo

도 르겠다 얼 이 주 젊은 층에서 많이 쓰 lsquo rsquo

는 속어임에 틀림없다 러나 어사전에 표 적이고 품

위 있는 말만 어야 한다고 생각한다 것은 커다란

해다 당장 아 어사전이나 펼쳐 라 속어는

설과 같은 비어나 죄자들이 쓰는 은어 어

마니 같은 소 의 사람만이 쓰는 말 도 라 있

않은가 사전은 말 치에 일정 빈도 이상 나타나는 말이

라 말이든 다 할 있다

zb38) 가 나 에 대한 다음의 설( ) ( ) 않은 것은

① 가 는 얼짱 사 에 등재 것에( ) ( ) lsquo rsquo

보 고 다

② 사 등재 가는 단어 격에( )

고 고 는 언 들 언어 사 도에 고 다 ( )

③ 가 얼짱 어지만 신 과 같 매( ) ( ) lsquo rsquo

체에 도 사 는 말 는 고 다

④ 가는 얼짱 어 보고 크게 가지 근( ) lsquo rsquo 3

거 들어 뒷 고 다

⑤ 는 얼짱 어 는 다 특 다는( ) lsquo rsquo

근거 에도 크게 가지 근거 가 들어 주 2

뒷 고 다

가 늘 지 상에 살고 는 사 들 억( ) 10

도가 고 그리 지 통 고 는 사 들

그보다 훨 많 억 도는 고 지 20

통 다 그런 지 고 2500

년 학 간고사 대비2013 2 현대고 대비

ECN-0102-2013-001-000076193

그리 간 보는 과 사 에

매우 달 뿐만 니 과 에 도 극

루고 었다 미 운 그런 들

살고 는 동 과 사 들 사고 식에

큰 가 다는 다

고 그리 들 우주 개별 고 독립

사 들 생각 지만 고 들 우

주 연 질 간주 다 같 각

도 들에게는 연 질 었지

만 그리 들에게는 미 들 결 었다

고 과 그리 들 사 같 는

동 과 사 에 도 견 다

인 리학자인 츠 이마이 디드 겐트너는 두

살이 채 안 된 아이들에서 터 성인에 이르 다양한

연 대의 동양인과 서양인을 대상으 다음과 같은 험

을 했다 저 코르크 만든 피라 드 양의 도형을

여 주고 대상의 이름을 닥 라고 알 주었다lsquo (Dax)rsquo

제 닥 는 존재하 않는 것으 험자가 임의lsquo rsquo

만들어 낸 이름이다 런 다음 두 개의 다른 체를

여 주었는데 하나는 피라 드 양이 만 하얀 플라 틱

으 만들었고 다른 하나는 재 는 코르크 만 양이

달랐다 러고 나서 어떤 것이 닥 인 사람들에게 고 lsquo rsquo

르게 했더니 서양인들은 주 같은 양을 하고 있는

체를 선택했고 동양인들은 같은 재 만들어 체를

선택했다 이러한 차이는 성인은 어 두 살 리

아이들에게서도 나타났다 이것은 곧 서양인과 동양인은

서 다른 세상을 고 있다는 것을 의 한다 략 ( )

는 아주 단 하 서도 인상적인 험을 했다

험에는 동서양의 대학생들이 참여했다 는 험 참가자

들에게 컴퓨터 화 을 통해 속 장 을 담은 애니 이션

을 여 주었다 화 의 앙에는 초점의 역할을 하는 커

다란 고 한 마리가 있었고 주위에는 다른 생

들과 초 자갈 거품 등이 함 제 되었다 화 을

두 씩 후 참가자들은 자 이 것을 회상해 라는

를 았다

결과 서양인 대학생들과 동양인 대학생 두 앙

의 초점 역할을 했던 고 를 동일한 정도 언 했으

나 경 소 위 거품 초 다른 생 들 에 ( )

대해서는 동양인 대학생들이 서양인 대학생들 다 60

이상 더 많이 언 했다 뿐만 아니라 동양인 학생들은 서

양인 학생들에 비해 개 적인 고 다 전체적인 계

를 더 언 하는 경향을 다 략 또한 경의 일 ( )

를 화 킨 림을 제 하 을 때 동양인 대학생들은 대

경의 화를 알아챘 만 서양인 대학생들은 경

의 화를 거의 알아차리 했다 략 ( )

따라서 서양인들만을 대상으 연 한 화lsquo

편성 결 은 잘 된 것일 도 있다 각 과정과 인rsquo

과정의 어떤 이 화 편적이고 어떤 이

화에 따라 달라 는 는 앞으 많은 연 를 통하여 논의

되어야 한다

나 어떤 의 에서 리 두는 이 화적이다 리( )

안에는 다른 사람들과 더 친 한 계를 유 하 는 상호

의존성과 다른 사람들 터 독립적인 존재 살아가 는

독립성이 혼재한다 따라서 이 에서 어떤 특성이 더 강

하게 각되는 상황에 놓이느냐에 따라 서 다른 화적

특 을 일 있다 결 리 두는 어떤 경 에는

동양인처럼 행동하고 어떤 경 에는 서양인처럼 행동하는

것이다

zb39) 가 에 대한 다음의 설( ) 않은 것은

① 는 신 주 뒷 닥 실험과lsquo rsquo lsquo

니 실험 근거 시 다rsquo

② 동 들 상 간 공통 보다는 에 식

는 강 다

③ 들 주변 맥 에는 심 경 어 사건

과 사건 사 계에 상 민감 다

④ 는 동 과 틀린 지 고 는 것lsquo rsquo

니 다 고 다 lsquo rsquo

⑤ 가에 우리 사 들 개 시 가 원( )

집 경 말 고 는 것 개 보다는

에 고 는 것에 다

늘 지 상에 살고 는 사 들 억 도가10

고 그리 지 통 고 는 사 들( )知的

그보다 훨 많 억 도는 고 지 20

통 다 그런 지 고 2500

그리 간 보는 과 사 에

매우 달 뿐만 니 과 에 도 극

루고 었다 미 운 그런 들

살고 는 동 과 사 들 사고 식에

큰 가 다는 다

지심리 미 마 드 겐트 는 동

과 상 다 과 같 실험 다

크 만든 미드 도 보여 주고 그

상 닥 고 주었다 그런 다lsquo (Dax)rsquo

개 다 체 보여 주었는 는 미드

지만 틱 만들었고 다 는 재료는

크 지만 달 다 그러고 어 것 닥 lsquo

지 사 들에게 고 게 니 들 주 같rsquo

고 는 체 택 고 동 들 같

재료 만들어진 체 택 다 러 는

심지어 살짜리 들에게 도 타났다 것

곧 과 동 다 상 보고 다는

것 미 다 개별 사 보고 고 동

년 학 간고사 대비2013 2 현대고 대비

ECN-0102-2013-001-000076193

연 질 보고 는 것 다

동 들 주변 상 에 맞 어 동 고

에 다 사 들 태도 동에 보다

많 주 울 다 동 가 미시간

에 에 경험 다 그는 미

식 경 보러 가게 었는 경 체는 매우 재

미 었 주변 들 동에 질 다 그

는 들 계 어 상태 경

다 어 들 에 에 그 시 가 계

가 진 것 다 뒷사 고 지 는 들

동 럼 어 웠다

그는 경험에 어 얻어 동 들lsquo

각도 상 본다 는 가 우고rsquo

검 여 주 단 도 상 실험 실

시 다 그는 실험 가 들에게 컴퓨 통

담 니 보여 주었다

에는 역 는 커다 고 마리가 었

고 주 에는 다 생 들과 갈 거 등

께 시 었다 본 후 가 들

신 본 것 상 보 는 지시 다

그 결과 생들과 동 생

역 고 동 도 언

경 거 다 생 들에 ( )

는 동 생들 생들보다 60

상 많 언 다 뿐만 니 동 생들

생들에 개별 고 보다 체 계

언 는 경 보 다 경 변 시

킨 그림 시 동 생들 경

변 지만 생들 경 변

거 리지 못 다

지 지 들만 상 연 lsquo

보편 결 못 것 도 다 지각 과 과rsquo

지 과 어 보편 고 어

에 달 지는지는 많 연 통 여

어 다

리 드 니 벳 생각 지도 사- ldquo rdquo( 2004)

zb40) 위 에 대한 설 으 가장 적절한 것은

① 동 과 생 식 강 고 다

② 가지 실험 통 쓴 고 다

③ 닥 실험에 사 본질에 동 사

상에 주 다

④ 니 실험에 동 과 에 지

각 도에 가 다

⑤ 쓴 는 보편 연 에 드러 우월 에

에 근 고 다

가 동 들 주변 상 에 맞 어 동 고( )

에 다 사 들 태도 동에 보다 많

주 울 다 동 가 미시간 에

에 경험 다 그는 미식

경 보러 가게 었는 경 체는 매우 재미 었

주변 들 동에 질 다 그 는

들 계 어 상태 경 다

어 들 에 에 그 시 가 계 가

진 것 다 상 살펴lsquo 는 말 들rsquo

에 그는 에 시 어 도 뒷사

생각 곧 다시 곤 것 다 그런 그에게

뒷사 고 지 는 들 동 럼

어 웠다

그는 경험에 어 얻어( ) 동 들lsquo

각도 상 본다 는 가 우고rsquo

검 여 주 단 도 상 실험

실시 다 실험에는 동 생들 여 다

그는 실험 가 들에게 컴퓨 통

담 니 보여 주었다 에는

역 는 커다 고 마리가 었고 주 에는

다 생 들과 갈 거 등 께 시

었다 본 후 가 들 신 본 것

상 보 는 지시 다

다 그 결과 생들과 동 생( )

역 고 동 도 언

경 거 다 생 들 에 ( )

는 동 생들 생들보다 60

상 많 언 다 뿐만 니 동 생들

생들에 개별 고 보다 체 계

언 는 경 보 다 들어 동

생들 상 체 연못 럼 보 어ldquo 같rdquo

체 맥 언 시 었지만

생들 상 어 같 큰 고 가 쪽 움ldquo

직 어 같 역 고rdquo

언 시 다 경 변 시킨 그

림 시 동 생들 경 변

지만 생들 경 변 거

리지 못 다

년 학 간고사 대비2013 2 현대고 대비

ECN-0102-2013-001-000076193

게 볼 동 들 보다는 큰 그( )

림 보 에 사 과 체 맥 연결시 지각

는 경 고 체에 특 떼어 내

어 독립 보는 것 낯 어 다 에

들 사 에 고 주변 맥 에는 심 경

에 사건과 사건 사 계에 상

민감 편 다

마 지 지( ) 들만 상 연

보편 결 못 것 도 다lsquo rsquo 지각 과

과 지 과 어 보편 고 어

에 달 지는지는 많 연 통 여

어 다

리 드 니 벳 생각 지도 사- ldquo rdquo( 2004)

zb41) 의 하는 가~ 다른 것은

① ② ③

④ ⑤

얼마 그 에 동 사고 식과

사고 식 보여 주는 내 다

들 에 는 탕 고 같 게

어 겨 고 미 에 는 그 크 럼 큰 고

어리 주고 원 는 어 도 는

상 고 생각 다는 것 다 러

는 어떻게 생 것 고 과 그리 거슬

러 가 보 그 단 다

고 연 경 체 경 생 에

다 벼 사는 공동 업과 경험 많 연 역

에 고 들 연 웃과

게 지내 고 탁 연 들

들 지 연 럽게 들 다 민들

웃과 동 게 뿐만 니 는 집 과

게 다

동 시 는 생태 경 에 살 결과

들 다 사 들 사 상 에 주

울 게 었고 는 곧 체 상 과 간 사

계 시 는 낳게 었다 신 가

가 는 체에 는 원 는 동시

에 다 사 들 그 사 포 체 맥 에

다 들 간 사 연

계 체 계에 주 울 는 사고 체계

게 었다

그러 그리 연 경 그 었다 산

지 연결 는 지 건 그리고 역

에 다 런 들 업에 다 사 과

동 므 공동체에

다고 다 고 그리 들

들과는 달리 보 내 감 지 들과

지 크게 느 지 못 다 그

견 다 경우 주 쟁 통 결 는 갖

게 었다

신 사 간 계들 루어진 커다

트워크 에 게 당연 사 역시 연

계들 체 식 게 다 어 상

원 도 그 개체가 체 맥 과

계 에 고 다 게 체 맥 에 주

울 다 보 상 복 과 가변 식 게 고

상에 재 는 많 변 들 사 에 재 는 들도

게 다 들 주 태도 보

는 경우가 많다 쟁 결

통 결 보다는 통 결

는 보 다

그러 고 그리 들 개개 사 사 독

에 주 울 다 사 사 체에

어 그들 사 에 재 는 공통 규 주

고 다 상 원 에도 사

체 내 주 고 다 그들

체 여 탕 체

는 주 태도 시 고 특 사 어

주에 는지 여 그 주에 는 규

견 다 에 는 쟁 식 리

같 리 사고 체계가 달 게 었다

리 드 니 벳 생각 지도 사- ldquo rdquo( 2004)

zb42) 위 에서 사 된 설 과 가장 유사한 것은

① 크톱 컴퓨 는 본체 니 마우 루

어 다

② 곡과 시 리 는 지 과 사 루어 다는 공통

지니고 다

③ 경 고 것과는 달리

경 본 연 태 그 주변 경

④ 벽돌 능 에 사계 내내

습도가 지 다

⑤ 잰느 체 체 지닌 재 체가 없

는 재 눌 다

년 학 간고사 대비2013 2 현대고 대비

ECN-0102-2013-001-000076193

zb43) 는 립 앙 도서 이 정의 일 이다lt gt

도서 장과 이 자의 리 의 정의 연결이

적절하 않은 것은

lt gt

제 조 서 유8 ( )

도서 장은 다른 이 자의 안전을 위협하거나 도서 의①

서를 란하게 할 가 있는 자에 대하여는 도서 출입

을 제한할 있다

도서 장은 이 자가 제 조 각 호의 어느 하나의 행위를 하7②

을 때에는 이 을 하게 하거나 도서 출입을 제한할

있다

제 조자 의 대출9 ( )

도서 자 는 다음 각 호의 경 대출할 있다①

상호대차도서 간에 자 를 류하는 것을 말한다 등 다1 ( )

른 도서 과의 협 을 위하여 필 한 경

공 이 공 행 상 필 하는 경2

에 도서 장이 필 하다고 인정하는 경3

대출이 가능한 도서 자 의 위는 도서 장이 정하는②

에 따른다

제 조 상10 ( )

이 자가 도서 자 설을 더럽히거나 찢거나 뜨①

쓰게 하거나 잃어 린 경 에는 상하여야 한다

도서 장은 제 항에 따른 상 을 정하여 게 하여야1②

한다

제 조이 절차 등11 ( )

이 칙에서 정한 것 에 도서 자 설의 이 절차

이 제한 등에 필 한 사항은 도서 장이 정한다

출처 립 앙 도서- (httpwwwnlgokr)

① 는 도 리 다8

② 도 는 리 다9 1

③ 료 지 는 도 리 다9 2

④ 도 료 변상에 리10 1

⑤ 는 에 도 리 다11

3

도 다 각 같다①

공 공 다만 연1

연 간 다

매월 째 째 월2

도 도 리 그 사3

가 다고 는

도 에 미리 게1 3②

시 여 다

4

도 시간 도 여 게시 다

5

도 료 시 는 는 도①

지에 등 후

등 에 사 도②

7

는 다 각 여 는 니 다

도 료 시 상 리1 lsquo rsquo

도 료 시 훼 는2 middot

지 가 닌 곳에 식 거 담3

우는

도 보 등 보 검색열4 middot

그 에 도 질 지 여 도5

여 게시 사 는

8

도 다 거 도①

질 게 우 가 는 에 여는 도

도 가 각 어느7②

에는 지 게 거 도

9

도 료는 다 각 경우 다①

상 도 간에 료 는 것 말1 (

다 등 다 도 과 여 경우)

공 원 공 상 는 경우2

그 에 도 다고 는 경우3

가능 도 료 는 도②

는 에 다

10

년 학 간고사 대비2013 2 현대고 대비

ECN-0102-2013-001-000076193

가 도 료 시 럽 거 거①

못 쓰게 거 어 린 경우에는 변상 여

도 에 변상 여 게시1②

여 다

zb44) 위 에서 도서 장이 게 해야 할 사항에 해당하는

것을 두 쓰

년 학 간고사 대비2013 2 현대고 대비

ECN-0102-2013-001-000076193

립 도 규

1 ( )

규 립 도 립 어린 청 도(

포 다 료 시 열 시 말) (

다 에 사 규 립 도)

편 진 다

2 ( )

규 립 도 도 다 에( lsquo rsquo )

고 는 도 에 도lsquo rsquo 2 2

료 에 여 다 다만 특 료 귀

료 등 료 에 사 립 도

도 다 다( lsquo rsquo )

3 ( )

도 다 각 같다①

공 공 다만 연1

연 간 다

매월 째 째 월2

도 도 리 그 사3

가 다고 는

도 에 미리 게1 3②

시 여 다

시간4 ( )

도 시간 도 여 게시 다

등 등5 ( )

도 료 시 는 는 도①

지에 등 후

등 에 사 도②

사 료6 ( )

도 료 시 에 사 료는 도

7 ( )

는 다 각 여 는 니 다

도 료 시 상 리1 lsquo rsquo

도 료 시 훼 는2 middot

지 가 닌 곳에 식 거 담3

우는

도 보 등 보 검색열4 middot

그 에 도 질 지 여 도5

여 게시 사 는

질 지8 ( )

도 다 거 도①

질 게 우 가 는 에 여는 도

도 가 각 어느7②

에는 지 게 거 도

료9 ( )

도 료는 다 각 경우 다①

상 도 간에 료 는 것 말1 (

다 등 다 도 과 여 경우)

공 원 공 상 는 경우2

그 에 도 다고 는 경우3

가능 도 료 는 도②

는 에 다

변상10 ( )

가 도 료 시 럽 거 거①

못 쓰게 거 어 린 경우에는 변상 여

도 에 변상 여 게시1②

여 다

등 규 에 것 에 도11 ( )

료 시 등에 사

도 다

립 도- (httpwwwnlgokr)

zb45) 도서 장의 리 있는 조항으 적절하 않

은 것은

① ② ③ ④ ⑤

년 학 간고사 대비2013 2 현대고 대비

ECN-0102-2013-001-000076193

1 ( )

사가 공 는lsquo rsquo

과 여 사 원과 리

사 타 사 규

니다

개 보 보7 ( )

사는 보통신망 등 계 는 에lsquo rsquo lsquo rsquo

원 개 보 보 니다 개lsquo rsquo

보 보 사 에 는 사 개lsquo rsquo

보 취 니다 다만 사는 다 lsquo rsquo

사 계 통 공 는 경우 원 lsquo rsquo

등 개 보 당 사에 습니lsquo rsquo

원 리에8 (lsquo rsquo lsquo rsquo lsquo rsquo

)

원 에 리lsquo rsquo lsquo rsquo lsquo rsquo①

원에게 가 도 여 는lsquo rsquo 3

니다

사는 원 가 개 보 우 가lsquo rsquo lsquo rsquo lsquo rsquo②

거 사 경우 는 미 에 어 거 lsquo

사 사 운 우 가 는 경우 당rsquo lsquo rsquo

습니다lsquo rsquo

원 가 도 거lsquo rsquo lsquo rsquo lsquo rsquo 3③

가 사 고 지 경우에는 시 사에lsquo rsquo

통지 고 사 내에 니다lsquo rsquo

경우에 당 원 사에 그 사실3 lsquo rsquo lsquo rsquo④

통지 지 거 통지 도 사 내에 지 lsquo rsquo

생 경우 사는 지지 습니다lsquo rsquo

사10 (lsquo rsquo )

사는 과 지 미lsquo rsquo①

에 는 지 계 고

공 여 다 여 니다lsquo rsquo

사는 원 게lsquo rsquo lsquo rsquo lsquo rsquo②

도 개 보 신 보 포 보 보 시( )

갖 어 개 보 취 공시 고

니다

사는 과 여 원lsquo rsquo lsquo rsquo③

견 만 당 다고 경우에는

리 여 니다 원 견 만 사 lsquo rsquo

에 는 게시 거 우편 등 통 여

원에게 리 과 결과 달 니다lsquo rsquo

원11 (lsquo rsquo )

원 다 여 는 니다lsquo rsquo ①

신청 는 변경 시 허 내 등1

타 보 도2

사가 게시 보 변경3 lsquo rsquo

사가 보 보 컴퓨 그4 lsquo rsquo (

등 등 신 는 게시)

사 타 등 지 재산 에5 lsquo rsquo 3

사 타 상 거 업6 lsquo rsquo 3

는 폭 시지 상 타 공7 middot middot

에 는 보 에 공개 는 게시 는lsquo rsquo

사 동 없 리 사8 lsquo rsquo

타 거 당9

게시15 (lsquo rsquo )

원 내에 게시 는 게시 게재 는lsquo rsquo lsquo rsquo lsquo rsquo

경우 원 사가 게시 복 lsquo rsquo lsquo rsquo lsquo rsquo middot middot

등 태 언 등에 공 는

것 내에 다 원 본 게시 등 lsquo rsquo lsquo rsquo

크 능 등 여 복 는 등 태

는 것 동 것 니다

- (wwwnavercom)

zb46) 위 은 인터넷 포털사이트의 회 가입을 위한 이

약 의 일 이다 이 약 을 만드는 과정에서 생각한

내 으 적절하 않은 것은

개 보 보 가 지에 별 눠①

겠어

원 가 만들게 에②

시 주어 겠어

원들 게재 게시 다 원 크 다③

는 것 지

④ 원 지 는 뿐만 니 사가 지 는

도 께 달 지

리에 가 생 경우 사가⑤

에 다는 도 듯

1 ( )

사가 공 는lsquo rsquo

과 여 사 원과 리

사 타 사 규

년 학 간고사 대비2013 2 현대고 대비

ECN-0102-2013-001-000076193

니다

개 보 보7 ( )

사는 보통신망 등 계 는 에lsquo rsquo lsquo rsquo

원 개 보 보 니다 개lsquo rsquo

보 보 사 에 는 사 개lsquo rsquo

보 취 니다 다만 사는 다 lsquo rsquo

사 계 통 공 는 경우 원 lsquo rsquo

등 개 보 당 사에 습니lsquo rsquo

원 리에8 (lsquo rsquo lsquo rsquo lsquo rsquo

)

원 에 리lsquo rsquo lsquo rsquo lsquo rsquo①

원에게 가 도 여 는lsquo rsquo 3

니다

사는 원 가 개 보 우 가lsquo rsquo lsquo rsquo lsquo rsquo②

거 사 경우 는 미 에 어 거 lsquo

사 사 운 우 가 는 경우 당rsquo lsquo rsquo

습니다lsquo rsquo

원 가 도 거lsquo rsquo lsquo rsquo lsquo rsquo 3③

가 사 고 지 경우에는 시 사에lsquo rsquo

통지 고 사 내에 니다lsquo rsquo

경우에 당 원 사에 그 사실3 lsquo rsquo lsquo rsquo④

통지 지 거 통지 도 사 내에 지 lsquo rsquo

생 경우 사는 지지 습니다lsquo rsquo

원에 통지9 (lsquo rsquo )

사는 특 다 원에게 통지 경우lsquo rsquo lsquo rsquo

공지 게시 통 상 게시 개별 통지에7

갈 습니다

사10 (lsquo rsquo )

사는 과 지 미lsquo rsquo①

에 는 지 계 고

공 여 다 여 니다lsquo rsquo

사는 원 게lsquo rsquo lsquo rsquo lsquo rsquo②

도 개 보 신 보 포 보 보 시( )

갖 어 개 보 취 공시 고

니다

사는 과 여 원lsquo rsquo lsquo rsquo③

견 만 당 다고 경우에는

리 여 니다 원 견 만 사 lsquo rsquo

에 는 게시 거 우편 등 통 여

원에게 리 과 결과 달 니다lsquo rsquo

원11 (lsquo rsquo )

원 다 여 는 니다lsquo rsquo ①

신청 는 변경 시 허 내 등1

타 보 도2

사가 게시 보 변경3 lsquo rsquo

사가 보 보 컴퓨 그4 lsquo rsquo (

등 등 신 는 게시)

사 타 등 지 재산 에5 lsquo rsquo 3

사 타 상 거 업6 lsquo rsquo 3

는 폭 시지 상 타 공7 middot middot

에 는 보 에 공개 는 게시 는lsquo rsquo

사 동 없 리 사8 lsquo rsquo

타 거 당9

원 계 규 내lsquo rsquo lsquo②

여 공지 주 사 사가 통지 는rsquo lsquo rsquo

사 등 여 타 사 업 에 lsquo rsquo

는 여 는 니다

- (wwwnavercom)

zb47) 위 약 의 조항에서 같은 제점을 하lt gt

고 있는 조항은

lt gt

제휴 회사에 회 의 아이디 개인 정 를 전송할 있도

한 조항은 고객에게 당한 조항이다

1 7 8① ② ③

④ 9 ⑤ 10

립 도 규

1 ( )

규 립 도 립 어린 청 도(

포 다 료 시 열 시 말) (

다 에 사 규 립 도)

편 진 다

2 ( )

규 립 도 도 다 에( lsquo rsquo )

고 는 도 에 도lsquo rsquo 2 2

료 에 여 다 다만 특 료 귀

료 등 료 에 사 립 도

도 다 다( lsquo rsquo )

3 ( )

도 다 각 같다①

공 공 다만 연1

연 간 다

년 학 간고사 대비2013 2 현대고 대비

ECN-0102-2013-001-000076193

매월 째 째 월2

도 도 리 그 사3

가 다고 는

도 에 미리 게1 3②

시 여 다

시간4 ( )

도 시간 도 여 게시 다

등 등5 ( )

도 료 시 는 는 도①

지에 등 후

등 에 사 도②

사 료6 ( )

도 료 시 에 사 료는 도

7 ( )

는 다 각 여 는 니 다

도 료 시 상 리1 lsquo rsquo

도 료 시 훼 는2 middot

지 가 닌 곳에 식 거 담3

우는

도 보 등 보 검색열4 middot

그 에 도 질 지 여 도5

여 게시 사 는

질 지8 ( )

도 다 거 도①

질 게 우 가 는 에 여는 도

도 가 각 어느7②

에는 지 게 거 도

료9 ( )

도 료는 다 각 경우 다①

상 도 간에 료 는 것 말1 (

다 등 다 도 과 여 경우)

공 원 공 상 는 경우2

그 에 도 다고 는 경우3

가능 도 료 는 도②

는 에 다

변상10 ( )

가 도 료 시 럽 거 거①

못 쓰게 거 어 린 경우에는 변상 여

도 에 변상 여 게시1②

여 다

등 규 에 것 에 도11 ( )

료 시 등에 사

도 다

립 도- (httpwwwnlgokr)

zb48) 다음 정 리 의 의 으 볼 때 가장

이 적인 것은

도 시간 도 여 게시 다①

등 에 사 도②

가능 도 료 는 도 는③

에 다

④ 도 에 변상 여 게10 1

시 여 다

⑤ 도 가 각 어느7

에는 지 거 도

zb49) 를 참고하여 이 어의 성격을 설 한lt gt

것으 적절하 않은 것은

① 보 에 는 어 시 상 고 어 시lt gt lsquo rsquo

에 보여주고 다

② 진 어 어원에 견 고 다

에는 타 어 들어가는 것 다 lsquo rsquo

③ 에 들어갈 말 각각 고 어 어 신 어~

들 언어는 질 격 강 통 없었다

④ 시 우리 에 가 었지만 지 계

과 달리 들 통 사 달 어 웠

년 학 간고사 대비2013 2 현대고 대비

ECN-0102-2013-001-000076193

⑤ 크 몽골 만주 공통어가 우리 어 같

계열에 다는 에 사 특 짐

가( )

善化公主主隱 공주님

他密只嫁良置古 몰 결 고

薯童房乙 맛

夜矣卯乙抱遣去如 에 몰 고 가다

( )

始汝 會隱日恚見隱扐 만 에 본

恥隱汝衣淸隱笑 맑 웃

고 시 여 공 크다 만 다[ ] ( ) ( ) ( ) ( )始 汝 會扐

내다 에 보다 견( ) ( )恚 見 다( )隱

럽다 맑다 청 웃( ) ( ) ( ) ( )恥 衣 淸 笑

zb50) 위의 나 를 함 고 음에 답하( ) lt gt

보lt gt

( )素那或云金川 白城郡蛇山人也

운 사산

는 고 다 는( )[ ( ) ] (素那 金川 白城

사산 사 다) ( ) 郡 蛇山

삼 사- lsquo rsquo 47

에 제 된 단어 의 표 리를 조건(1) lt gt ( ) lt gt

에 맞게 서 하

건lt gt

lsquo 었고 었다 태rsquo

에 제 된 단어 동일한 표 리에(2) lt gt ( )

의해 적은 것을 나 에서 찾아 조건 에 맞게 서 하( ) lt gt

건lt gt

에 당 는 각각( ) 개 쓸 것2 단

당 는 가 여러 개 어도 개만 쓸 것 각2

개 과 도 쪽에 개만2 2

드시 지 것( )

과 동 원리 것lsquo 고

과 동 원리 것 다rsquo

태 것

가( )

素那(或云金川) 白城郡蛇山人也

소나 또는 천 이라 한다 는 성 사( ) ( ) ( )素那 金川 白城郡〔 〕

산 사람이다 현대어 풀이( ) ( )蛇山

나( )

紫布岩乎希 회

執音乎手母牛放敎遣 자 손 암쇼 노히 고

吾 不喩慙 伊賜等肹 肹 나 안디 리샤

花 折叱肹 可獻乎理音如 고 것거 도림다

다 향찰은 리말을 리 으 적은 표 이었 만 생( )

은 고 대를 넘 하고 끊어 고 말았다 랜 세

동안 갈고 닦아 체계적이었던 향찰 표 이 사라졌

을 인은 크게 두 가 나누어 생각해 볼 있다

하나는 족 사회의 한 선호도에서 찾을 있다 라 때

향찰은 주 족 계 에서 사 했을 것으 인다 한 을

알 하고서는 한자를 활 하여 리말을 리 으 표

하 란 가능하 때 이다 런데 족들은 간이 흐

를 향찰과 같은 리 표 을 익혀 사 하 다는

아 한 을 대 사 하는 쪽을 선호하게 되었다 더 이

고 초에 인재 등 을 위해 과거제도가 행되 서 한 선

호도가 더 높아졌고 결 향찰은 소 되고 말았다

또 다른 가능성은 한 어의 특성에서 찾을 있다

터 한 과 일 세 나라는 한자 화 에 속해 다

당연한 이야 겠 만 표의 자인 한자는 어를 표 하

에 매 적절하다 어의 음절은 성 ( ) ( )聲母 韻母

이 어 고 여 에 성조가 추가되어 최종 소리가 결정된

다 래서 어는 단음절을 하나의 한자 표 하 된

다 에 초성 성 종성의 세 가 소가 하나의 음절

년 학 간고사 대비2013 2 현대고 대비

ECN-0102-2013-001-000076193

을 이 는 한 어는 음절 조가 잡하고 음절의 가 많아

서 한자 차 만으 한 어의 소리를 만족 럽게 표 할

없었다 를 들어 한 어에서는 어 니 같이 음절 lsquo rsquo

이 어 단어가 얼마든 있으나 어는( ) 複數音節

자 하나 나타내 만이다lsquo [m ]rsquo 母 ǔ

한편 일 어의 표 은 핵 적 단어는 한자 적고 토는

가나라는 일 의 자 적는 이다 적인 의 를 나

타내는 은 표의 자인 한자 적고 적 계를 나

타내는 토는 표음 자 적는 셈이니 자세히 살펴

리의 향찰 표 을 쏙 빼닮았음을 알 있다 한 어 같

은 착어이 서도 일 어에만 향찰과 유사한 표 이 살아

남은 것은 일 어의 특 때 이다 일 어는 하나의 자음과

음의 결합으 음절을 이 고 침이 거의 없는 음절 언어

이다 이러한 음절의 특색에다가 토가 달한 착어라는 점

이 향찰과 유사한 표 이 살아남을 있는 비결이었다

하 만 같은 착어라도 다양한 음소 침이 달한 한

어는 향찰 표 하는 데 근 적으 한계가 있었다

zb51) 다 하여 의 행에 대한 탐 한 결과( ) lt gt 2

않은 것은

보lt gt

善花公主主隱 공주니믄 공주님( )

----------------------------------------

-

他密只嫁良置古 그 지 얼어 고 몰 결(

----------------------------------------

-

薯童房乙 맛 맛( )

夜矣卯乙抱遺去如 몰 고 가다 에 몰 고(

가다)

주동 역 동- (薯童謠『 』

에 2 ( )他密只嫁良置古

얼다 시집가다 결 다 말 lsquo rsquo

① 실질 미 지니고 므 타 타lsquo ( )rsquo lsquo [ ]

② 에 실질 미 타내고 지 는lsquo rsquo lsquo [ ]rsquo lsquo [ ]密只 密 只

계 타내는

③ 얼어는 실질 미 포 고 므 가lsquo rsquo lsquo [ ]rsquo嫁

것lsquo [ ]rsquo 良

④ 고 어간 는 실질 미 지니고 므lsquo rsquo lsquo -rsquo

것lsquo [ ]rsquo 置

⑤ 고 어미 고는 계 타내고 므lsquo rsquo lsquo- rsquo

고 것lsquo [ ]rsquo 古

가( )

엉 훈 민middot middot middot middot middot世 宗 御 製 訓 民 正 音

말 미 듕 귁에 달middot middot middot middot middot middot middot middot中 國 文 字

니 런middot middot middot middot middot middot 어린middot middot middot middot百 姓

니 고 도 내 들middot middot middot middot middot middot middot middot middot 시러middot

펴 몯middot 미middot middot 니 내middot middot middot middot middot middot middot middot 爲

어엿middot 겨 새middot middot middot 믈여듧middot middot middot middot字 니middot middot middot

사 마다 니겨 킈 middot middot middot middot middot middot middot middot middot便 安

고 미니middot middot middot middot

본 는 상( ) (象

원리에 만들어진 본) ( )形 ㄱ ㄴ ㅁ ㅅ ㅇ

에 는 가 원리에( )加劃

그리고( )ㅋ ㄷ ㅌ ㅂ ㅍ ㅈ ㅊ ㆆ ㅎ

쓰는 병 원리에 만들어진( )竝書

마지막 체( ) ( )異體ㄲ ㄸ ㅃ ㅆ ㅉ ㆅ

ᅀ 다 상 원리에 ㅇ ㄹ

지 는 삼재 상 본 본( ) ( ) ( 天地人 三才

탕 므림과 림에 ) (初ㅡ ㅣ

재)( ) ( )( )出字 再出字ㅗ ㅏ ㅜ ㅓ ㅛ ㅑ ㅜ ㅕ

병 그리고 들 에 다시( )ㅘ ㅝ ㅣ

( )ㅣ ㅢ ㅚ ㅐ ㅟ ㅔ ㆉ ㅒ ㆌ ㅖ ㅙ ㅞ

zb52) 가 에 대한 설 으 르 않은 것을( ) 두 고르

① 어쓰 규 지키고 다

② 리 고 다

③ 말 미 미 등 어 사 다lsquo rsquo

④ 개 지 다

년 학 간고사 대비2013 2 현대고 대비

ECN-0102-2013-001-000076193

⑤ 어 원 에 가 도 고 다

엉 훈 민世 宗 御 製 訓 民 正 音

말 미 듕귁에 달 니

런 어린 니 고 도middot

내 들 시러 펴 몯 미 니middot

내 어엿 겨 새 믈여듧

사 마다 니겨middot 킈 고

미니

훈민 언 본- lsquo rsquo 5 (1459 )

zb53) 위의 에 대한 현대어 풀이가 르~ 않은 것

① 우리 말 과 달

② 어리 말 고 는 것 어도

③ 신 생각 마 껏 펼 는 사 많다

④ 게 생각 여

⑤ 사 마다 게

zb54) 훈민정음 언해 에는 한 을 창제한 동 가 드러나

있다 훈민정음 창제의 정 과 내 이 잘 연결된 것

① 주 신 말 미 듕귁에 달

② 민 신 내 어 겨

③ 신 뻔 킈 고 미니

④ 실 신 사 마다 니겨

⑤ 귀 신 계 주 는 훈민 신과 거리가

가 엉 훈 민( ) middot middot middot middot middot世 宗 御 製 訓 民 正 音 

말 미 귁에 中 國 달 文 字

니 런 어린 니 百 姓

고 도 내 들 시러 펴 몯

미 니 내 어엿 爲 겨 새

믈여듧 니 사 마다 니 字

겨 킈 고 미니 便 安

훈민 언 본- lsquo ( )rsquo ( ) 5 (1459 )訓民正音 世祖

( )

[ 1 ]

동 룡 샤 마다 복( ) ( ) ( )海東 六龍 天福

시니 고 동( ) ( )古聖 同符 시니

[ 2 ]

매 니 곶 여

미 므 니 그 내 러

가 니

[ 125 ]

우 미리( )千世 샨( )定 에( )漢水北 累仁

누 개 샤 복 업 시니( ) ( ) 開國 卜年

신( )聖神 니 샤도 경 근민 샤 욱( )敬天勤民

드시리 다

님 쇼 산 가( ) ( )洛水 山行

미드니 가

어 가- lsquo ( )rsquo 27龍飛御天歌

다 우리신 니쓰고 다만 만 쓰( )

거 샹 귀쳔 다보게 러 귀

여 쓴 도 신 보 가 고 신 에

말 어 보게 각 에 사 들

고 본 몬 능통 후에

죠 죠 니

드 도 만 공 에 사

드 미 죠 고 고 여 보 죠

보다 얼마가 거시 어신고 니 첫

가 죠 니 죠

민 들 어 신 샹

귀쳔 도보고 어보 가 만 늘

고 폐 에 만쓴 죠 민

도 러보지못 고 보니 그게 엇지

심 니 리 보 가 어 운건 다

니 쳣 말마 지 니 고 그

쓰 에 가 우 지 지

몰 거 본후에 가 어 지

고 그니 쓴편지 쟝 보

년 학 간고사 대비2013 2 현대고 대비

ECN-0102-2013-001-000076193

쓴것보다 듸 보고 그 마 니 쓴 고

어 못

그런고 에 리 과 가

만 쓴 못 민 말만 듯고

고 편 그 못 보니 그사 단

병신 못 다고 그사 식 사

니 만 고 다 과 그사

만 고 다 과 업 사 보다 식 고

죠 도 고 각 과

견 고 실 직 귀쳔 간에 그

고도 다 것 몰 귀죡 보다

사 우리 신 귀쳔 다 업

시 신 보고 과 지 게 랴

시니 샹 귀쳔 간에 우리 신 걸

간 보 새지각과 새 걸 미리

독립신- lsquo (1896)rsquo

zb55) 친 어 나의 제 장( ) 2 매 함축적

의 가 가장 유사한 것은

① 지 눈 내리고 매 득 니 내 여 가

사- lsquo rsquo

② 도 어 리듯 그 게 어 다

주 사- lsquo rsquo

③ 눈 살 다 죽 어 린 과 체 여

눈 새벽 지 도 살 다

눈- lsquo rsquo

④ 삶 근심과 고단 에 돌 거니는 여 거 는

여 리 내린 살가지 에 눈 리 눈 리

택 그 생 에- lsquo rsquo

⑤ 늘 러 고 러

청룡 룡 어 개 루 우

신경림 계- lsquo rsquo

zb56) 친 를 위 가 나 에 나타난A B ( ) ( )

세 어의 특 에 의거하여 세 어 표 하

그 산 고 공 도 맑지만

A

주변에 쓰 리는 어리 사 많다

B

건lt gt

식 가 에 타 어 특징에( ) ( )

거 과 어쓰 는 고 지 말 것

A

B

zb57) 가 의( ) 달 아ㆍ 다 의 ( ) 나셔에서 알 있는

세 어 개화 어의 특 을 비 하여 조건 에lt gt

맞게 서 하

건lt gt

어에 는lsquo 개

어에 는 다 태rsquo

zb58) 은 가 는 다 에 나 는 절lt 1gt ( ) lt 2gt ( )

일 를 췌한 것이다 의 의 가 lt 1gt (1)~(2)

유사한 말을 에서 찾아 쓰lt 2gt

보lt 1gt

런 (1) 어린 니 고百 姓

도 내 들 시러 펴 몯 미

사 마다 (2) 니겨 便 安

킈 고 미니

보lt 2gt

죠 고 고 여 보 죠

보다 얼마가 거시 어신고 니 첫 가

죠 니 죠 민

들 어 신 샹 귀쳔

도보고 어보 가 만 늘 고

폐 에 만쓴 죠 민 도

러보지못 고 보니 그게 엇지 심

니 리

년 학 간고사 대비2013 2 현대고 대비

ECN-0102-2013-001-000076193

lt 1 gt

동 룡 샤 마다 복 시( ) ( ) ( )海東 六龍 天福

고 동 시니( ) ( )古聖 同符

lt 2 gt

(A) 매 니 곶

여 니

미 므 니 그 내

러 가 니

lt125 gt

우 미리 샨 에( ) ( ) ( ) 千世 定 漢水北 累

누 개 샤 복 업 시 니( ) ( ) 仁開國 卜年 聖

신( ) 神 니 샤도 경 근민 샤( ) 敬天勤民

욱 드 시 리 다

님 쇼 산 가 ( ) ( )洛水 山行

미드니 가

- lt gt龍飛御天歌

zb59) 장과 내 상 유사한 성격의 조는125

① 뫼 고 고 고 고

어 그린 많고 많고 고 고

어 러 는 울고 울고 가느니

도 견- lt gt

② 강 에 드니 몸 다

그믈 고 가니

뒷 뫼 엄 언 니( )藥

-

③ 말 없는 청산 태 없는 다

값 없는 청 없는 월

에 병 없는 몸 별 없 늙 리

-

④ 가마귀 골에 가지 마

낸 가마귀 새

청강에 것 시 몸 러 가( ) 淸江

-

⑤ 진 골에( ) 白雪

가 매 는 어느 곳에 었는고

에 갈 곳 몰( ) 夕陽

색-

zb60) 위 에 나타난 세 어의 특 으 적절하 않은

것은

① 룡 어 주격 사에 당 는 가 사( ) lsquo rsquo六龍

고 다

② 샤 어에도 어 주체 쓰 다

는 것 다

③ 매 어 달리 사 택에 어

가 지 지지 고 다

④ 므 원 상 직 어 지 다

⑤ 드시리 다 주체 과 상 께 사

고 다

수고 하셨습니다hearts hearts

년 학 간고사 대비2013 2 현대고 대비

ECN-0102-2013-001-000076193

보닷컴에 공 는 별 보는 고등

들 여 주 는

들 습니다 슷 동 지

가 복 는 것 도가

니 복 여 습 시고 거 시

니다

정답 해설

1) 정답[ ] ④

해설 다른 것은 두 특정 업이나 단 내에서 사[ ]

하는 일종의 은어 사회 언에 해당한다 러나

는 언이 아니라 단과대학을 여서 단대 사lsquo rsquo lsquo rsquo lsquo④

대학을 여서 사대라고 한 말에 해당하 일rsquo lsquo rsquo

사회에서도 널리 쓰이 사회 언이라 할

없다

2) 정답[ ] ⑤

해설 사회 언은 같은 단 내에서 쓰이는 언어이[ ] lsquo rsquo

동일 단끼리는 단결 과 친 감을 형성하는

능을 하 리적 안감이 일어나 않는다

3) 정답[ ] ③

해설 사람이라는 차 적 표현에 대한 대안적 표현이[ ]lsquo rsquo

인 아내 처 등으 볼 있다lsquo rsquo

4) 정답[ ]⑤

해설 남성은 주 격 체를 사 한다[ ]

5) 정답[ ] ⑤

해설 흑인은 검다라는 뜻을 가 고 있을 뿐 인[ ]lsquo rsquo lsquo rsquo lsquo rsquo

다 열등한 뜻을 내포하 않는다

6) 정답 살 색 첫 작품[ ] - -

해설 살색 혹은 킨색은 한 인의 피 색을 뜻[ ] lsquo rsquo lsquo rsquo

하는 것으 인종 차 을 추 고 출 이주민

의 평등 을 침해할 있어 년 표 이2005

살 색으 이름을 꾸었다 처녀작은 처녀라lsquo rsquo lsquo rsquo lsquo rsquo

는 단어가 가 고 있는 곡된 성 인 을 한 것

으 첫 작품정도 꾸어 사 하는 것이 좋다lsquo rsquo

7) 정답[ ] ⑤

해설 호는 아들에게 해체를 사 하고 있다[ ] ① ②

장 을 성하는 청자는 자 의 아 느리 아lsquo

들 세 이다 호는 아 느리에게 해rsquo ③

체를 사 하고 있다 호가 느리 아 에게 ④

사 한 해 체 아들에게 사 한 해체는 두 비lsquo rsquo lsquo rsquo

격 체에 해당한다 호는 자 의 아랫사람인 ⑤

느리에게 아들과 마찬가 해체를 사 하는 것이

상 이 만 임 을 한 느리에게 고마 과 쁨

존 의 표 를 하 위해 자 의 아 에게 말하듯

해 체를 사 하고 있다

8) 정답[ ] ③

9) 정답[ ] ⑤

10) 정답[ ] ①

해설 청자 할아 가 장의 주체 아 다 높을[ ] ( ) ( )

경 에는 압존 에 의해 장의 주체를 높이 않는lsquo rsquo

다 러 아 서가 아닌 아 는으 계 lsquo rsquo lsquo rsquo lsquo

니다 가 아닌 있 니다 표현하는 것이 르rsquo lsquo rsquo

11) 정답 당이 당을 쫒았다 당이[ ]

당에 다

해설[ ]

12) 정답[ ] ⑤

해설 서 다른 높임표현을 통해 청자에 대해 리[ ] ⑤

적 거리감을 나타내는 인 은 이 아니라 현정이

다 가 에서 현정은 에게 해 체를 사 함으 써 ( )

친근감을 드러낸다 나 에서 연 을 게을리하는 역 ( )

도 들 때 에 화가 난 현정이 선생님에게 항의하

는 장 에서는 하 체를 사 하여 리적 거리lsquo rsquo

가 어졌음을 나타내고 있다

13) 정답[ ] ①

해설 는 는 얼 빛이 날과 어찌 다르 고[ ] lsquo rsquo

라는 뜻으 전과 달리 임이 화자를 않고

있음을 알 있다

14) 정답 달리 후 가 있다 이를 통해 경[ ] lt gt

쾌한 음악성을 형성하고 노 젓는 상황을 체적으

형상화하는 역할을 한다

15) 정답[ ] ①

16) 정답[ ] ⑤

해설 다 의 자연은 를 성찰하게 하는 대상[ ] ( )⑤

이자 정의 대상이다 의 자연은 자 의 상황과 ⑤

처 를 드러내는 경으 서의 역할을 하 이

이 없다

17) 정답[ ] ③

해설 는 빈천 을 해결하고자 했으나 강산[ ] lsquo ( )rsquo 貧賤③

과 풍 을 달라는 에 거절하 다고 함으 써 자

연에 대한 애정을 드러내고 있으 는 않는

임에 대한 망을 개에게 전가 켜서 임에 대한 리

을 드러내고 있다

18) 정답[ ] ③

년 학 간고사 대비2013 2 현대고 대비

ECN-0102-2013-001-000076193

19) 정답[ ] ⑤

해설 고상한 음악가의 이름을 리말 꽝 럽[ ]

게 꿈으 써 언어유희를 통해 음을 유 하고 있

다 이는 고상한 척하는 총 를 비꼼으 써 비판적

태도를 드러내는 것이 대상을 꽝 럽게 표현

하여 총 의 허 과 사치를 풍자하고 있다

20) 정답[ ] ⑤

해설 는 작품 속 경에 대한 설 이 드러나는 것이[ ]

서 자의 주 적인 견해가 접적으 드러나는 것이

아니다

21) 정답[ ] ⑤

22) 정답[ ] ②

23) 정답[ ] ④

24) 정답[ ] ①

해설 적강 티프는 주인공의 비 한 출생이나 능[ ] ①

과 이 있는 것으 조정의 능함을 풍자하는lsquo rsquo

것과는 거리가 다

25) 정답 픔 나[ ] ( )

해설 의 음악은 고통 는 사람들을 위 하고 아픔[ ] lsquo rsquo

을 치유해 주는 능을 한다고 할 있다 의 lt gt

픔 도 소 된 이 과 더 어 살아가는 따뜻한 마음lsquo rsquo

을 상 한다

26) 정답[ ] ⑤

해설 에게 선천적으 주어 각 장애라는 역경[ ]

은 의 이라는 가사 연 을 있다lsquo rsquo

27) 정답[ ] ④

해설 는 장 란 선 에게 은 개인적인 인상을[ ]

소녀 장정 등으 표현한 것이다lsquo rsquo

28) 정답[ ] ②

해설 담자가 피 담자의 언어적 표현이나 비언어[ ]②

적 표현 하 독자는 담의 위 나 피

담자의 감정 상태를 알 있다 이를 통해 독자는

담 상황을 더 생생하게 느낄 있고 피 담자

를 더 잘 이해할 있게 된다

29) 정답[ ]③

해설 일상생활과 역도 선 서의 성과에 된 것에서[ ]

역도를 하 서 겪는 어 과 내적 고민으 화제를

전화하 위한 것이다

30) 정답[ ] ①

해설 릿속에 새겨 넣듯 이 억되도 함 세상[ ] ② ③

살이가 힘들고 고생 러 속 하여 자유를 ④

가 없는 고통의 상태를 비유적으 이르는 말

적의 침입을 막 위해 쌓은 축 켜야 할⑤

대상을 비유적으 이르는 말이다

31) 정답[ ] ④

해설 이 의 종류는 전 으 인 사건 경[ ] lsquo

비평을 성 소 삼는다rsquo

32) 정답[ ] ④

해설 근은 삼대독자 태어났음을 에서 확인할[ ]

있다 형제들과의 담은 이뤄 가 없다

33) 정답[ ] ⑤

해설 근은 가난에도 하고 화가를 꿈꾸었다[ ] (3

단 또한 다른 화가 망생들은 정 육을)

위해 상 학 학 해 유학 에 랐 만

근은 다른 을 찾아야 했다 단 세에(5 ) 18

근은 조선 전람회에 입선하 다 단 의(6 )

만종은 인간과 자연이 엮어 가는 경건한 조화 을lsquo rsquo

나타낸다

34) 정답[ ] ①

해설 근이 속에서도 창작활동을 추 않고[ ]

하는 닭은 은 세상과 타협할 르는

근이 세상의 이해를 하 위한 가장 떳떳한 단

이 때 이다

35) 정답[ ] ⑤

해설 전 은 서 자의 주 적인 평이 리는 것이[ ]

만 위 제 은 인 이 살았던 대 사회적 경

을 통해 객 적인 인 의 을 제 하고 있다

36) 정답[ ] ⑤

해설 전 은 인 사건 경 비평이라는[ ] lsquo rsquo⑤

성 이 어져 있다

37) 정답[ ] ①

해설 이 은 동양인과 서양인의 사고 에 차이가[ ]

있다는 것을 대조를 통해 설 하고 있다 또 쓴이

의 제자가 축 경 를 러 가서 경험한 일화를

통해 동양인이 서양인에 비해 주 상황에 더 많은

주의를 인다는 주장을 뒷 침하고 있다

38) 정답[ ] ④

39) 정답[ ] ②

40) 정답[ ] ②

41) 정답[ ] ④

42) 정답[ ] ③

43) 정답[ ] ④

44) 정답 도서 의 휴 일 도서 의 이 간 도서의[ ]

해설 도서 장은 임의 정한 휴 일과 도서 이[ ]

간 도서의 상 등을 게 할 의 가 있다

년 학 간고사 대비2013 2 현대고 대비

ECN-0102-2013-001-000076193

45) 정답[ ] ①

해설 제 조의 정 휴 일 의 휴 일의 사전 게[ ] 3

는 도서 장의 의 조항에 속한다

46) 정답[ ] ①

해설 개인 정 호 의 를 제 하 했 만 항[ ]

나눠서 제 하 않고 대 나열하고 있다

47) 정답[ ] ②

해설 제 조의 내 을 회사는 다른 회사 협[ ] 7 lsquo

계약을 통해 서비 를 제공하는 경 회 의 아이디

등 개인 정 를 해당 회사에 전송할 있다는 내rsquo

이 있으 의 제점을 제 할 있다②

48) 정답[ ] ④

해설 는 도서 장의 의 에 해당하고 나 는 도[ ] ④

서 장의 리에 해당한다

49) 정답[ ] ③

50) 정답 은 음독으 적었고 은 훈독으 적었[ ] (1)

다 과 동일한 표 리 적은 것은 이고 (2) ce

과 동일한 표 리 적은 것은 이다ab

51) 정답[ ] ③

52) 정답[ ] ①②

53) 정답[ ] ③

54) 정답[ ] ③

55) 정답[ ] ①

56) 정답 른 죠코 어린 노 하니라[ ] A B

57) 정답 세 어에서는 활 형이 칙적으[ ] lsquo rsquoㄹㅇ

나타났 만 개화 어에서는 활 형이 쓰 다 lsquo rsquo ㄹㄴ

58) 정답 호 가 흔[ ] (1) (2)

59) 정답[ ] ④

60) 정답[ ] ③

Page 12: 현대고대비 국어 - chamsoriedu.com 「콘텐츠산업진흥 법」외 에도 저작권 의하여 ... 다른주체에게어떤동작을하도록만드는것을나타내는

년 학 간고사 대비2013 2 현대고 대비

ECN-0102-2013-001-000076193

게 리 는 다 집 시 공도 여

고 가 에 가가 어진 만 개 공신

었 니 도 러워 마 시고 rdquo 시

택 여 니 운 신 과 신

습 늘에 죄 짓고 간 상에 내 신

다 략 지낸 후에 튿 승상 ( ) hellip hellip

니 승상 거운 마 지 못

마( )

듯 월 러 생 열다 살 었

다 에 승상 어진 사 얻고 만 에 근심 없었

다만 주 가 간신 에

죽 것 생각 마 곧 어 곤

다 그 에 주 원통 어

없 고 여 시 가 거늘 생 만

여 다

말 감격 러우 간신 에 가득 여ldquo

고 니 께 상 듣지 니 것

니다rdquo

승상 듣지 고 가

퇴 재상 공달 집에 거 고 상 지어

승지 러 께 리

( )

뒷 거리 강 승상 에게 상 리지[ ]

만 여움 사 귀 가게 다 강 승상

몸 는 연 가 헤어

리 다 산 들어간 룡사 승 만

게 다 승 만 우 다릴

과 들고 략 다 담

원 여 에게 복 고 어

공격 다 담에게 여러 가( ) 天子

복 등 여 다 단

신 리쳐 담 사 고 에게

간 후 태후 태 여 지에 고생

지 심과 강 주 여 개 다 헤

어 어 니 내 고 담 리

벼슬에 귀 누리게 다

zb21) 위 의 인 간 계를 같이 나타냈을lt gt

때 에 대한 이해 가장 적절하 ~ 않은 것은

① 계에 주 는 계 심 열

상 에 다고 다( ) 水深火熱

② 계는 견원지간 고 다( ) 犬猿之間

③ 계는 달리 막역지 계 고( )莫逆之交

④ 연결 사 컬어 재 가 고( )才子佳人

⑤ 는 생 과 볼 ( )匹夫匹婦

가 재 는 는 심 고 매사에 생( )

각 고 능 도 어 가 에게 많lsquo rsquo

도움 사 다 그는 에게 거 에

꺼리 없 거 났다고 는

매우 싫어 고 신 들

는 사 다

내가 지 리에( ) 1970

사 실에 지 월간ldquo

편집 고 어 었다rdquo

어느 없 가 쑥 다 도 어 10

후 다 산 시 럼 어 엇 어 ( ) lsquo怡山

다시 만 랴 니 그는 재 그룹 승 운rsquo

사가 고 는 고 거 누

주는 가 없는 가가 어 다시 만 게 것

었다

다 보통 것 닐러 그( ) ldquo 어낸 ( )

틀어주 가 루 러 허 에

싶어 키 틀어주 그( )

가 루 허 우간 곡 틀어 주는 루 못

는 는 고 닝께 고 지

들어 사는 고 가 다는 건 에 그 집에

rdquo

그런 단 어들 어 새벽에 떼죽 거

다 고 어 보니 죄다 허 게 집어진

는 것 었다 가 실내 꿴 뛰어 지만

없는 었다

어떻게 된 거야 한동안 넋나간 듯이 서 있던 총ldquo rdquo

가 하고많은 사람 에 하필이 유자를 겨냥하 은

말이었다 쎄유 아마 새에 고뿔이 들었던 개비네 ldquo

유rdquo

유자는 러 딴청을 하 다 야 고 가 에서 ldquo

감 가 들어 죽는 고 두 어rdquo 총 는 가 혐의

자 나 되는 것처럼 화풀이를 하 드는 것이었다( )嫌疑者

라 이 어쩌 어 유( ) ldquo rdquo ldquo rdquo

애유 이런 잔인 도 한 것들 같으니ldquo ( ) rdquo殘忍無道 helliphellip

총 는 탱천 하여 쩌 를 하 다( ) 憤氣撐天

아하니 아는 자는 다 동 하여 호통을 쳤으 하나 혈

압을 생각하여 참는 눈치 다 달리 처리헐 두 ldquo

잖은감유rdquo

총 의 성 을 덧들이 고 한 말이 아니었다 가 할

년 학 간고사 대비2013 2 현대고 대비

ECN-0102-2013-001-000076193

있는 것이 말고는 없었 때 에 게 뒷동

산을 달은 거 다

이 유자소전- lsquo rsquo

zb22) 의 상황을 속담으 표현한 것으 적절한 것은

① 루 곳 게 마 다

② 에 맞고 강에 눈 다

③ 늘 도 다

④ 도 사 다

⑤ 에 가도 신만 리 다

거리 공신 후[ ]

에 주 는 벼슬 심 늦도( )主簿

식 없어 과 께 산에 드리고 신

태몽 꾼 에 만고 웅 상 지닌 들

낳 키운다 그 후 신 들 에 역심

담 귀 등 심 여 리 귀 보내고

지 죽 는 도망 간다 그

만 고 에 에 어 니

헤어지게 다 지 가 사 들에

사 들 별 고 없 다니었다

마 마 돌 다니 걸 여 고

어 곤 다 에는 동쪽에 고 에

는 쪽에 니 가 에 리는 엽 가는

없 니 늘 다니는 었다

얼 말 죽 사 같고 림새가 말 니었

다 가슴 에 고 등 삼태

헌 에 니 달 가 도리 ( )奇男子

어 걸 었 담 만 열 도 ( ) ( )傅說 慇

고 만났고 만 갈( ) ( )武丁 伊尹

도 탕 만났( ) ( )成湯 渭水

여상 도 주 만났는 월( ) ( ) ( ) 呂尙 周 文王

같 러가 도 어느 열 살

늘과 집 삼고 사 에 쳐 거리에

어 다가 곳에 니 다 ( ) 楚

지 다가 사 보고 가에 다다( )長沙

니 망 가에는 원 리가 슬 고 가

가 내리는 사 에는 갈매 가 갈 뿐 었다

쪽 돌 보니 가 우거 고

가 사 보 었다 그곳에

가니 는 사( ) 汨羅水

는 다 주 가 쓰고 죽고

곳 었다

마 감 여 에 가 사 살펴보니

에는 삼 고 그 에( ) 屈三閭

는 만고 월 과 지 가는 그 들( )風月

가 어 었다( ) 路程記

동쪽 벽 에 새 운 어 거늘 그

보니

월 에 경 주 는 간신에게ldquo ( )敗

보고 연경 귀 가다가 에 죽 rdquo

거늘 그 보고 에 거꾸러

통곡 말

우리 연경 간 만 니ldquo ( )燕京

에 지 살 상에 엇 겠는

가 에 고 에 었 니

상에 살 것 가 도 께 지리 rdquo

고 가에 내 가니 울 리가 에 지

사 쳤는지 심 심 것 가

에는 강 주 는 재상 살고 었

니 시 에 과거에 격 여 승상 벼슬 다가 간

신 만 벼슬 그만 고 고 돌 었

다 그러 신 지 가 지 못 여 상

가 못 결 는 상 여 원 니

신 들 그 직간 꺼 다 그 에 도

담과 귀가 강 승상 가 미워 다 강 승상 마

본 에 갔다가 돌 는 에 우편 주( ) ( )本府 右便

에 다가 색 에 어리었는 청룡( ) 酒店

에 지 늘 여 통곡 고

사 는 꿈 꾸었다 마 상 게 생

각 여 새 다리다가 새벽닭 울고

가 달 갔다 가 보니 과연 어 동 가

가에 울고 는지 달 들어 그

고 사 에 어 말

는 어 어 에 어 가ldquo

닭 곳에 우느냐rdquo

니 울 그 고 답 여 말 다

는 경 동 에 사는 언 주 공 들ldquo

니다 께 간신 만 연경 귀 가

시다가 에 죽 사 에 는 닭에

도 에 죽고 니다rdquo

강 승상 말 듣고 크게 낯 변 말

것 웬 말 냐 근 동ldquo ( )老患

못 갔 니 그 사 변 여 런 변

었단 말 가 주 는 신 다 같

에 벼슬 다가 는 가 많 들어 고 돌

는 주 가 게 꿈 에 생각

겠느냐 생각지 못 다 미 지 간 지지

말고 께 가 rdquo

뒷 거리 강 승상 도움 죽 고[ ]

년 학 간고사 대비2013 2 현대고 대비

ECN-0102-2013-001-000076193

고 그 과 결 여 사 가 다 그러 강

승상 에게 울린 상 강 승상 귀 가고

과 헤어 리 승 만 게 다

승 우 다릴 과

들고 략 다 담 원

여 에게 복 고 어 (天

공격 다 담에게 여러 가 복) 子

등 여 다 단신

리쳐 담 사 고 에게 간

후 태후 태 여 지에 고생 지

심과 강 주 여 개 다 헤어

어 니 내 고 담 리 벼

슬에 귀 누리게 다

미상- lsquo ( )-劉忠烈傳

zb23) 위 과 의 서사 조를 비 한 것으 적절하lt gt

않은 것은

보lt gt

믿지 고 결 여 곱

낳 다 곱째 공주 낳 가

리게 다 리 만 고 진 공주는 lsquo rsquo

리공 미 리공 에 키워진다 월

러 과 가 죽 병에 걸 는 승에 는

어 산다고 다 여 들에게 탁

지만 거 리 는다 리 는 과

승 다 승 지 가는 에 많

만 지만 보살 도움 사 도 다

그러 승 신과 결 여 시

들어 주겠다고 다 리 는 그 결

여 들 곱 낳 후에 신

얻게 다 돌 리 는

에 과 상여 만 지만 여 과

살 낸다 훗 리 그 공 우 죽 사

승 도 는 신 다

리-lsquo rsquo-

① 복 결말에 고 다

② 웅 에 탕 고 다

③ 시 겨 내고 귀 누리는lsquo rsquo

보 리 는 월 재 신 다lt gt lsquo rsquo

④ 과 보 리 는lsquo rsquo lt gt lsquo rsquo

도움과 어 신 능 극복 고

⑤ 등 여 시 겪는lsquo rsquo

보 리 는 닌 지lt gt lsquo rsquo

림 시 겪는다

가 각 고 에( ) ( )却說

살 없었다 략 사 들 슬 에 어 lt gt

가에 내 고 가고 싶 가 고 후

워 경

사 들 별 고 없 다니었다 lt

략 얼 말 죽 사 같고 림새가 말gt

니었다 가슴 에 고 등

삼태 헌 에 니 달 가 ( )奇男子

도리어 걸 었 담 만 열 도( )傅說

고 만났고 만 갈( ) ( ) 殷 武丁

도 탕 만났( ) ( ) (伊尹 成湯 渭

여상 도 주 만났는) ( ) ( ) ( )水 呂尙 周 文王

월 같 러가 도 어느 열 살

늘과 집 삼고 사 에 쳐 거리에

어 다가 곳에 니 다 ( ) 楚

지 다가 사 보고 가에 다다( )長沙

니 망 가에는 원 리가 슬 고 가

가 내리는 사 에는 갈매 가 갈 뿐 었다

쪽 돌 보니 가 우거 고

가 사 보 었다 그곳에

가니 는 사( ) 汨羅水

는 다 주 가 쓰고 죽고

곳 었다

에는 강 주 는 재상 살고( )

었 니 시 에 과거에 격 여 승상 벼슬 다

가 간신 만 벼슬 그만 고 고 돌

었다 략 강 승상 마 본 에 갔다가 돌 lt gt ( )本府

는 에 우편 주 에 다가 색( ) ( ) 右便 酒店

에 어리었는 청룡 에 지

늘 여 통곡 고 사 는 꿈 꾸

었다 마 상 게 생각 여 새 다리다

가 새벽닭 울고 달 갔다 가

보니 과연 어 동 가 가에 울고 는지

달 들어 그 고 사 에

어 말

는 어 어 에 어 가ldquo

닭 곳에 우느냐rdquo

니 울 그 고 답 여 말 다 lt

략gt

년 학 간고사 대비2013 2 현대고 대비

ECN-0102-2013-001-000076193

생각 여 가 고 시 는ldquo ( )大人

상에 다시없는 니다 살 엇 겠습니

에 돌 가시고

가에 돌 가 니 살 마 없습니

다 략 어 없어 강 승상 가니rdquo lt gt

그곳 월계 었다

다 강 승상에게는 들 없고 다만 만( )

었다 가 낳 에 가 색

타고 내 에게 말

는 니다 미원 과ldquo ( )紫微垣

연 맺고 었는 께 강 집( )緣分

보내 에 니 게 여겨 주십시

rdquo

거늘 미 가운 낳 니 가

고 거동 단 다 시 짓 쓰 고

는 없었 니 여 가운( ) 音律

지 는 짝 룰 만 사 없었다 가 사

여 사 감 게 고 지 못 고 염 는 만다

다가 당에 거 고 식같 러

내니 고귀 상 루 말 다 어 ( )相

울 도 다 귀 사 없고 ( )富貴爵祿

웅 걸 만고 었다 승상 매우 뻐 내

당 들어가 에게 사 니 역( ) 內堂

시 매우 거워 말 다

도 마 사 는 승상께ldquo

그 게 말 시니 상 여러 말 지 말고 사

도 시다 략 시 택 여rdquo lt gt

니 다운 신 과 신 습 늘에 죄

짓고 간 상에 내 신 다

다 내고 들어가 사 살펴보니

고 것 는 다 말 어 고

는 다 어 신 에 ( )新房

에 신 과 신 가 평생 연 맺었( )緣分

니 사 주고 말 어떻게 다 헤 릴

어떻게 다 리 지낸 후에 튿 승

상 니 승상 거운 마 지 못

( ) 듯 월 러 생 열다 살

었다 에 승상 어진 사 얻고 만 에 근심

없었 다만 주 가 간신

에 죽 것 생각 마 곧 어

곤 다 그 에 주 원통

어 없 고 여 시 가 거늘 략 lt gt

략 거리

강 승상 에게 상 리지만 여움

사 귀 가게 다 강 승상 몸 는

연 과 헤어 리 다

마 각 생 강 승상 집 쪽( )

늘 보고 없 가 신 신 생각 니

없고 어 없었다 는 어떻게 도리가 없다

여 산 에 들어가 리 고 어 도 닦

고 다 그 산 보고 가

다가 곳에 다다 니 에 큰 산 었다 많

우리 골짜 가 늘 는 가운 색

에 고 갖가지 가 짝 어 ( )花草

었다 략 주 보니 lt gt ( ) (一柱門 黃

산 룡사 어 었다) lsquo rsquo 金大字

산 들어가 고승 다 그( ) ( ) 山門 高僧

거동 보니 눈 눈 듯 고

변 같 귀는 어 에 늘어 니( ) 白邊

맑고 어 골격과 신 평 니었

다 염주 에 걸고 짚고 포 ( )六環杖

삼에 어진 쓰고 생 보고 말

승 연 여 상공 시는 동 에ldquo

가 맞 지 못 니 승 십시 rdquo

생 크게 말 다

생 가 여 어 고ldquo

없 다니다가 우연 곳에 사 만 것

그 시 생 어떻게 고 습니

rdquo

승 답 여 말

어 산 승 에ldquo ( ) ( )南岳 衡山

시어 승에게 탁 내 낮 시경에 경 lsquo 12

동 에 사는 심 들 가 것 니 내쫓

지 말고 습니다 마 승rsquo

다가 상공 림새 보니 경 사 에 보

습니다rdquo

생 그 말 듣고 편 고 편( )

슬 승 들어가니 여러 승 들

가워 다 승 에 들어가

후에 그 편 니 곳 경 었다 상( ) 仙境

고 신 편 다 후 는 승과

께 병 도 탐 고 경도 게( )兵書

게 었다 게 니 지 에 가객 ( ) ( )大明天地 佳客

년 학 간고사 대비2013 2 현대고 대비

ECN-0102-2013-001-000076193

없고 산 에 리 만 본 ( ) 廣德山

신 상 사 살 는 만

우고 늘 월 신 과 늘 ( )日月聖神

산 신 들 다 니 그 재( ) 名山神靈

주 민 누가 당 겠는가 낮 공

zb24) 다 에 해당하는 내 으 적절하( ) 않은 것은

① 강 티 통 당시 능 다

② 상계 지상계 경 는 원 계 드러

③ 실에 어 없는 실 가 타 는

④ 뛰어 재주 어 가진 고

등 다

⑤ 가 직 개 여 평가 내리는

편집 평 타 다lsquo rsquo

가 본격 가 동 것 지( )

다 단 상 에2003 lsquo rsquo

들어가 드럼 연주 다 취미 생 달리

들었다는 보 우 가 들ldquo

어 틱 린 도 다 고 말 다rdquo

경 는 가 망 없( ) lsquo

티 원 고 답 다 신과 같 시각rsquo

는 습 상상 만 도 감동

다 시각 연주 동시에

열 상 는

티 원 그런 열 경 럽다는 것 다

다 역시 엄청 다 본( )

에 복 들

고쳐 가고 다 신 에 얼

마 지는 고 리가 는 지도 생님

가 훈 고 많 고쳐 다

고 말 다

그러 직도 에 지 는 다 그는

체격 지 못 게 가 큰 만

체 운동 훈 과 께 체 늘 동 50

는 게 고 말 다

에게는 꿈 다 통 누 가( )

주겠다는 것 그 꿈 다 신 극복 는

과 에 큰 경험 들도 느 게

주고 싶다는 것 다

마 슬 마다( ) ldquo 통

낼 었 것 럼 고통 는 사 들

고 겠다 고rdquo

말 다 달 루 첫 낸 lsquo rsquo

첫 드 심 집에 는 리듬 드 2

루 에 도 보고 싶다 집 에는 직(RampB) 3 4

사 곡 도 보 고 싶다고 포 다middot

zb25) 에서 가장 유사한 의 를 닌 어를lt gt

찾아 쓰

lt gt

나는 이제 너에게도 픔을 주겠다

사랑 다 소 한 픔을 주겠다

겨 거리에서 개 놓고

살아 추위 떨고 있는 할 니에게

값을 으 서 뻐하던 너를 위하여

나는 픔의 평등한 얼 을 여 주겠다

내가 어둠 속에서 너를 를 때

단 한 도 평등하게 어주 않은

가마니에 덮인 동사자가

다 얼어 죽을 때

가마니 한 장조차 덮어주 않은

한 너의 사랑을 위해

흘릴 르는 너의 눈 을 위해

나는 너에게 이제 너에게도 다림을 주겠다

지 울 포동 여고 생들17

틈 없 가득 체 에 맑 울

다 죽 듣 생들 사 에

연 는 탄 다 객들 도 는lsquo rsquo

가 보 주 공 맹 가 운 는

단 그룹사운드 루 보컬 맡고 는lsquo rsquo

시각 지 었다17 1

근 다만과 가 거lsquo rsquo lsquo

꿈 고 퇴 내가 다rsquo

간 간에 지 지 연 생들 짧lsquo rsquo lsquo rsquo

가 운 듯 리에 어

연 다 내 사 고 퇴lsquo rsquo

과 루 들 결 다시 돌lsquo rsquo

들 고 사 들 에 당당

것 니다 내 태어

볼 없었 크고 열여

년 학 간고사 대비2013 2 현대고 대비

ECN-0102-2013-001-000076193

에도 고 시 얻지 못 다

감지 없는 시각 상태 다

신 지에 고 상 원망 도

단다 어느 가 에 시각 에 ldquo

어 그런 듣고 다 보니 내가 게 lsquo

살 는지 도 눈 고 싶rsquo lsquohelliphellip

보 는 생각만 들 고 그 가 들에게rsquo

도 내고 들도 고 많 었죠 들 rdquo

었 지 새 는 에 쑥 러운 색

어났다

생에 것 단연 었다lsquo rsquo

공연에 거 꿈lsquo rsquo

는 다 특 가사 갑게 는 운 lsquo

벽 에 당당 마주 어 언 가 그 벽

고 늘 어 거운 상도

없죠 내 삶 에 웃 그 께

는 다고 다rsquo

들었 그냥 런 도 고만 여ldquo lsquo rsquo

겼죠 그런 꾸 가사 미 새 다 보

니 통 는 가사 는 생각 들 고 (

가 게는 시각 는 생각 들고 들) ( )

마다 듣고 큰 얻었어 rdquo

에 진지 게 가에 미 가

zb26) 의 에 들어갈 말 적절한 것은lt gt ~

lt gt

난 난 꿈이 있었죠

고 찢겨 남 하여도

내 가 히 과 같이 간 했던 꿈

혹 때 누 가가 뜻 를 비 음

내 등 뒤에 흘릴 때도

난 참아야 했죠 참을 있었죠

날을 위해

늘 걱정하듯 말하죠

헛된 꿈은 독이라고

세상은 끝이 정해 책처럼

이 돌이킬 없는

현 이라고 helliphellip

래 난 난 꿈이 있어

꿈을 믿어

나를 켜

저 차갑게 서 있는 이란 앞에

당당히 마주칠 있어

출처 가 거위의 꿈 작사 이적 작곡 동률- lsquo rsquo ( )

① ② ③ ④ ⑤

가 떴다는 들 만 지만( ) lsquo rsquo

늘 겸 다 에 주 연 우승 지 간에도 3

단 생님께 만 지 고 고 만ldquo rdquo

큼 늘 겸 신 계 가

고 다

에게는 꿈 다 통 누 가

주겠다는 것 그 꿈 다 신 극복 는 과

에 큰 경험 들도 느 게 주

고 싶다는 것 다

슬 마다 통 낼ldquo

었 것 럼 고통 는 사 들

고 겠다 고rdquo

말 다 달 루 첫 낸lsquo rsquo

첫 드 심 집에 는 리듬 2

루 에 도 보고 싶다(RampB) 집 에는 직34

사 곡 도 보 고 싶다고 포 다

미 는( ) (26) 어 헤헤헤 웃다가 어ldquo rdquo

허허허 웃었다ldquo rdquo ldquo rdquo 같 도 고

상 다 는 같 도 다( ) 壯丁 킹 들lsquo

다 는 역도 보 그 다 지만 그는rsquo

뷰에 지 다 운동만 지 ldquo

것 지 간에 여러 사 도 역rdquo helliphellip

었다 그런 엇 그 마 움직 는지 보 쯤

지 담 사 다 훈 없어 그는 티

지 림 었다 태 다 갔다 는 습

마 집 럼 편 게 보 다

주말에는 주 엇 보내

주말에도 별 주 에 청ldquo

고 에 가고 도 쳐

에 듣고 보 에 갈 가 별 없

어 산 시 게 고 들어 2002

거 매 여 지냅니다 시 과 지훈 rdquo

다 근 간 과 진실 그리고 싶어( )

가 다 근에게 그것 진리 다 거 다 없

거 고 다 없 는 것 진리

다 근 진리는 후 쪽 었다 신산( )辛酸 삶

었 질곡( )桎梏 역사 에 지냈 가

눈에 든 것 료 단 료 게 보

것 었다 그것 그 에 겨우겨우

슬 슬 생 어가는 간들 었다

리 과 단 리 고리에 검 마

없 거리 돌

상 것 없는 등 근에게 상

과 진실 엄 ( )儼存 다는 사실 리는 가

실 고 가 과 역경 에 도 근 내 포

없었 후 보루( )堡壘 다 도 365

도 간 근 여

시 것 다

년 학 간고사 대비2013 2 현대고 대비

ECN-0102-2013-001-000076193

다 공주 그림 가 근 경- ( ) ldquo rdquo(

2009)

zb27) 작가의 주 적인 각이 드러난 것은~

① ② ③ ④ ⑤

가 신 지에 고 상 원망( )

도 단다 어느 가 에 시각 에 ldquo

어 그런 듣고 다 보니 내가 lsquo

게 살 는지 도 눈 고 싶rsquo lsquohelliphellip

보 는 생각만 들 고 그 가 들에게rsquo

도 내고 들도 고 많 었죠 들었rdquo

지 새 는 에 쑥쓰러운 색

어났다 략 [ ]

경 는 가 망 없 티lsquo

원 고 답 다 신과 같 시각rsquo

는 습 상상 만 도 감동

다 시각 연주 동시에

열 상 는 티

원 그런 열 경 럽다는 것 다 략 [ ]

슬 마다 통 낼ldquo

었 것 럼 고통 는 사 들

고 겠다 고rdquo

말 다 달 루 첫 낸 lsquo rsquo

첫 드 심 집에 는 리듬 2

루 에 도 보고 싶다 집 에는 직(RampB) 3 4

사 곡 도 보 고 싶다고 포 다

식 누 가-

고 싶어

다 역도 미 담 고 사( )

질 주말에는 주 엇 보내[ 1]

답 주말에도 별 주 에[ ] ldquo

청 고 에 가고 도 쳐

에 듣고 보 에 갈 가 별

없어 rdquo

질 계 고 슬슬 도 는 것 닙니[ 2]

답 다 들 눈 에 보 고 뿐 보[ ] ldquo

다 열심 고 어 상에 도 들지만 상

지키는 것 들다고 에 도달

그것 지키 훨 많 rdquo

질 들 살 고 리 는[ 3]

거운 들 체 리느 는다

답 가 고 게 체 어[ ] ldquo ( ) 級

느 도 계가 니 살 는 것도 고역 지만

살 우는 것 들어 는 살

체 리 고 어도 어도 실 갔다

쑥 어 rdquo

질 거리에 슷 연 여 들[ 4]

보는 간 상 지

답 상 다 체 게 리지 못[ ] ldquo

거 주변에 는 그 거 누 보지

못 고 뻐지고 싶 에 체 리는 에

타 워 지만 는 어울 는 것보다 는

시간 운동만 는 건 니에 사복 lsquo rsquo

고 사복 는 말에 들 웃지만 늘 운동복

고 지내니 사러 갈 도 어 rdquo

질 역도가 말 단 식 운동 니[ 5]

답 가 내는 만 클 업 보[ ] ldquo

그러니 만 쓰는 식 운동 니다

만 다고 거운 것 들 는 건 니거든 연

도 고 가지 동 에 도 여러 가지

복 들

보식 역도 여 미-

zb28) 가 에 대한 설 으( ) 않은 것은

① 시각 우 지 시 에 지

고 망 가는 태도 달 고 다

② 언어 과 언어 복 사 여

담 내 생각 게 는 가

③ 직 감 그 마 것

럼 생생 게 느껴지는 과 주고 간 내

없 리 어 억 게 다

④ 담 내 식 리 여 담 삶 습

과 가 시 여 독 에게 감동과 훈 다

⑤ 직 진 담 직 누

지 못 는 독 에게 생생 상 달 주고

담 욱 게 다

zb29) 나 의 각 의 의도를 설 한 것으 적절하( ) 않

년 학 간고사 대비2013 2 현대고 대비

ECN-0102-2013-001-000076193

은 것은

① 질 담 상 보여 주 것 다1

② 질 담 과 그에 삶 태도 보여2

주 것 다

③ 질 역도 겪는 어 움에 역도3

과 것 다

④ 질 같 연 여 갖는 고민 는지 말4

주 는 것 다

⑤ 질 역도가 과 고 운동 는 것5

담 가 말 주 는 것 다

가 만진 것 다( ) 3

감 달 다고 다 억 에( ) 音感

지워 지만 당시 청 탁 리도

다고 다 드럼 웠다 4

에 갈 마다 드럼 는 리가 신 게 들

다고 다 눈 볼 가 없 니 엔ldquo

는 는 님 틱 에 여 주

다 드럼과 연 맺 과 들 주었다rdquo

식 누 가-

고 싶어

역( ) 도가 말 단 식 운동 니

가 내는 만 클 업에 보ldquo

그러니 만 쓰는 식 운동 니다 만

다고 거운 것 들 는 건 니거든 연

도 고 가지 동 에 도 여러 가지 복

들 시 는 상 상

드는 상 에 맞춰 실 에 는 여러

펼쳐집니다rdquo

략( )

늘 에 는 어 만 것 같

가 에 사 고 사 사ldquo

겠어 든 에 가 경 만 고

울 는 사 겠어 rdquo

보식 역도 여 미-

다 가 운 는 어 어( ) ldquo rdquohelliphellip

월 새벽 시 태 없 거웠고1965 5 6 1

는 없 그 병원에 퇴원 집

가는 마지막 마 고 마 내 거 다

가 죽 간신 에 실 다 사는 어느5 lsquo

가 죽 는 말 가 식 다 신rsquo

상에 각 시키는( )刻印 에 실

어느 가는 후 민 가가 근 었다lsquo rsquo

는 간 과 진실 그 다는( ) ldquo

에 단 평 견 가지고 다 내

가 그리는 간상 단 고 다 지 다 는 그들

가 에 는 평 지 니 그리고 어린

들 미지 겨 그린다rdquo

마 근 간 과 진실 그리고 싶어( )

가 다 근에게 그것 진리 다 거 다 없

거 고 다 없 는 것 진리

다 근 진리는 후 쪽 었다 신산(辛酸 삶)

었 질곡(桎梏 역사 에 지냈)

가 눈에 든 것 료 단 료 게

보 것 었다 그것 그 에 겨우겨우

슬 슬 생 어가는 간들 었

다 리 과 단 리 고리에 검

마 없 거리 돌

상 것 없는 등 근에게 상에

과 진실 엄 다는 사실 리는 가( )儼存

실 고 가 과 역경 에 도 근 내

포 없었 후 보루(堡壘 다 도)

도 간 근365

여 시 것 다

월 강원도 림리에( ) 1914 2 21

삼 독 태어났다 어 근 복

그것 그리 가지 못 다 근 곱 살

지는 산 산업에 실 고 답마 에 내

갔다 근 그림 럼 쫓 다니 가 시 것

다 상 진 것도 가 었다

러 가 에도 고 근 가 꿈꾸었다 근

가 꿈꾸게 것 보통 업

원색도1926 만lsquo rsquo 었다

공주 그림 가 근 경-

zb30) 에 대한 설 가장 른 것은~

① 역도가 과 운동 도 질

② 리는 는 다 lsquo rsquo

③ 들었지만 그럭 럭 는 다 lsquo rsquo

④ 가 게 보 시 말 다

⑤ 보 병 는 지 상 lsquo rsquo

는 말 다

년 학 간고사 대비2013 2 현대고 대비

ECN-0102-2013-001-000076193

시간 많지 다 청량리 생 병원

마지막 상 경 릿 게 들어 다 그 는 십

만 큰 가 상 말 다

지 못 들 마 갈 고 돗

도시민들 싹 싹 탔다 가 시

월에 병원에 원 가 폐 진 몸도4 ( )疲弊

갈 미 지 못 고 었다 가는 얼마( ) 解渴

지 생 에 생각 가

마감 는 신 평생 십 만에

가 과 많 닮 다고 생각 지는

가 운 는 어 어ldquo rdquo 1965helliphellip

월 새벽 시 태 없 거웠고 는5 6 1

없 그 병원에 퇴원 집 가

는 마지막 마 고 마 내 거 다 가

죽 간신 에 실 다 사는 어느 가5 lsquo

죽 는 말 가 식 다 신rsquo

상에 각 시키는 에 실 어느( ) lsquo刻印

가는 후 민 가가 근 었다rsquo

ldquo 는 간 과 진실 그 다는 에

단 평 견 가지고 다 내가 그

리는 간상 단 고 다 지 다 는 그들 가

에 는 평 지 니 그리고 어린 들

미지 겨 그린다rdquo

근 간 과 진실 그리고 싶어 가

다 근에게 그것 진리 다 거 다 없 거

고 다 없 는 것 진리다

근 진리는 후 쪽 었다 신산 삶 ( )辛酸

었 질곡 역사 에 지냈 가 눈에( )桎梏

든 것 료 단 료 게 보 것

었다 그것 그 에 겨우겨우 슬

슬 생 어가는 간들 었다 리

과 단 리 고리에 검 마

없 거리 돌 상

것 없는 등 근에게 상에 과 진실

엄 다는 사실 리는 가 실( )儼存

고 가 과 역경 에 도 근 내 포 없었

후 보루 다 도 도( ) 365堡壘

간 근 여 시 것

간에 지닌 가 근 1914 2

월 강원도 림리에 삼 독21

태어났다 어 근 복 그것 그리

가지 못 다 근 곱 살 지는 산

사업에 실 고 답마 에 내 갔다 근

그림 럼 쫓 다니 가 시 것 다 상

진 것도 가 었다 러 가 에도

고 근 가 꿈꾸었다 근 가 꿈꾸게

것 보통 업 원색1926

도 만 었다lsquo rsquo

그림 가 근 경 공주- ldquo rdquo ( 2009)

zb31) 다음 이 같은 의 성 소에 해당하 않은

것은

사건 평① ② ③

④ 주 ⑤ 경

가 운 는 어 어ldquo rdquo 1965helliphellip

월 새벽 시 태 없 거웠고 는5 6 1

없 그 병원에 퇴원 집 가

는 마지막 마 고 마 내 거 다 가

죽 간신 에 실 다 사는 어느 가5 lsquo

죽 는 말 가 식 다 신rsquo

상에 각 시키는 에 실 어느( ) lsquo刻印

가는 후 민 가가 근 었다rsquo

는 간 과 진실 그 다는 에ldquo

단 평 견 가지고 다 내가 그

리는 간상 단 고 다 지 다 는 그들 가

에 는 평 지 니 그리고 어린 들

미지 겨 그린다rdquo

근 간 과 진실 그리고 싶어 가

다 근에게 그것 진리 다 거 다 없 거

고 다 없 는 것 진리다

근 진리는 후 쪽 었다 신산 삶 ( )辛酸

었 질곡 역사 에 지냈 가 눈에( )桎梏

든 것 료 단 료 게 보 것

었다 그것 그 에 겨우겨우 슬

슬 생 어가는 간들 었다 리

과 단 리 고리에 검 마

없 거리 돌 상

것 없는 등 근에게 상에 과 진실

엄 다는 사실 리는 가 실( )儼存

고 가 과 역경 에 도 근 내 포 없었

후 보루 다 도 도( ) 365堡壘

간 근 여 시 것

간에 지닌 가 근 1914 2

월 강원도 림리에 삼 독21

태어났다 어 근 복 그것 그리

가지 못 다 근 곱 살 지는 산

사업에 실 고 답마 에 내 갔다 근

그림 럼 쫓 다니 가 시 것 다 상

진 것도 가 었다 러 가 에도

고 근 가 꿈꾸었다 근 가 꿈꾸게

것 보통 업 원색1926

도 만 었다lsquo rsquo

공주 그림 가 근 경- ldquo rdquo ( 2009)

년 학 간고사 대비2013 2 현대고 대비

ECN-0102-2013-001-000076193

zb32) 위 을 작성하는 과정에서 되어 활 된 자

어 것은

신 사 료① 연보②

고③ ④ 들과 담

⑤ 에 평

는 간 과 진실 그 다는 에ldquo

단 평 견 가지고 다 내가 그

리는 간상 단 고 다 지 다 는 그들 가

에 는 평 지 니 그리고 어린 들

미지 겨 그린다rdquo

근 간 과 진실 그리고 싶어 가

다 근에게 그것 진리 다 거 다 없 거

고 다 없 는 것 진리다

근 진리는 후 쪽 었다 신산 삶 ( )辛酸

었 질곡 역사 에 지냈 가( )桎梏

눈에 든 것 료 단 료 게 보

것 었다 그것 그 에 겨우겨우

슬 슬 생 어가는 간들 었다

리 과 단 리 고리에 검 마

없 거리 돌 상

것 없는 등 근에게 상에 과

진실 엄 다는 사실 리는 가 실( )儼存

고 가 과 역경 에 도 근 내 포

없었 후 보루 다 도 도( ) 365堡壘

간 근 여 시

것 다

간에 지닌 가 근 1914 2

월 강원도 림리에 삼 독21

태어났다 어 근 복 그것 그리

가지 못 다 근 곱 살 지는 산

사업에 실 고 답마 에 내 갔다 근

그림 럼 쫓 다니 가 시 것 다 상

진 것도 가 었다 러 가 에도

고 근 가 꿈꾸었다 근 가 꿈꾸게

것 보통 업 원색1926

도 만 었다lsquo rsquo

질 루 마 가 도 린다 경건

움 느껴지는 경 다 훗 근 그림에

과 는 거 것( )裸木

만 간과 연 엮어 가는 경건 움lsquo rsquo

니었

같 가가 고 싶었 근에게 그 꿈에 다

가가는 지 다 다 가 지망생들 규 미

상 에 진 고

에 지만 근 다 다 근

미 에 운 것 보통 시 미 시간

다 그런 그에게 없는 연습 가가

통 다 가 귀 시 지 도

얻는 뛸 듯 뻤지만 마 도 가 에

듯 는 었 에 어린 근 주 에

에 그림 그리고 지우고 복( )粉板

시간 가는 게 루 보냈다

근 그 갈 가가 것 열여( )渴求

었 다가 미1932 lsquo rsquo ( lsquo

미 에 다 다는 고 마rsquo) lsquo rsquo

가 근 집 고도 지는 시골 경

그린 그림 다 후 근 에 1943 22

지 미 에 그림 고

에 걸쳐 다 미 근 가

동 는 었다

공주 그림 가 근 경- ldquo rdquo ( 2009)

zb33) 위 의 내 과 일치하는 것은

가 근 가 꿈 포 다①

근 당 가들과 께 에 다②

살 근 가 걷20③

게 었다

④ 만 통 근 역경 겨내는lsquo rsquo

느 다

⑤ 근 간 과 진실 그리 에 그 에

드러 는 간상 단 다

계 시 주 근 건강

걸었다 신 과 간에 상 다 건강

신 는 눈에도 다 근 쪽 눈 뿌 게

보 지 과에 다 다 시 지지 고 결

내 었다 시 지만 마 막막

다 늦어 결 근 쪽 눈 고 말 다

쪽 눈 근에게는 쪽 눈 었고

계 었다 그 근 는 여 그lsquo rsquo

다 근 에 같 그림 그 었다1950

시 그림 는 여 쪽lsquo rsquo

고 어 마주 고 는 그림1963

여 과 동 다 마 복

그린 듯 눈 내리 새 게 다 지

사 다 근 게 복 것

복 상과 타 는 근 상

가 떳떳 단 었고 근 그리고

간 과 진실 에 다가가 가 근다

운 었다 근 신에게 당당 지 그리고

그 다 근 그림에 단 복 보다

년 학 간고사 대비2013 2 현대고 대비

ECN-0102-2013-001-000076193

태 도 그리고 극 보다 과

얻 여 었다 과 통

근 그리고 는 재 고 에 질

만들고 특 것 다

공주 그림 가 근 경- ldquo rdquo( 2009)

zb34) 의 이유에 대해 추 한 것으 적절하 않은 것

상과 타 시도①

보다 과 얻②

근 신에게 당당 지③

④ 간 과 진실 에 다가

⑤ 태 도 얻

근 가가 었지만 그 다니 가

럼 어지지 다 복과 쟁 거쳐 시

는 가 근에게 생계 사 에

운 사 다 에 키에 건( ) 178cm死鬪

체 근 에 동 역 업( )荷役

가 생계 다 쟁

에는 동에 운 상우 주 미

죄 사 에 그림 그리는 시 다 그곳에

에 동 역 업 것에

결 것 럼 보 다 지만 그런 것만도

니었다 그림 그리는 고는 지만 매 근

는 극 간 과 별 없는 경 리 그림

벽에 그리는 것 었다 우도 리 없었다 근

트 는 우 그림 그 다 생

계 그림 단 것 다

후 근 지 신 계 리에 미

엑 리 겼다 근 곳에

건 사 크 에 미 들 ( )

상 상 그 다 근 갖 다 겪

냈다 그리고 결 그 돈

신동에 어 사리 집 마 다 마 ㄷ

루 심 쪽에는 과 엌 쪽에는 건

었다 건 주고 근 가 에

여 살 다 심 에는 지 집어

쓰고 지만 곳 근 가 에게 러웠

보 리 다 근 과 마루 업실 삼 그림

그 다 신동 마루는 근 그림에 등 는 lsquo rsquo

같 상들 지 다 시 고

에 들 폐허가

가 업실 었다

공주 그림 가 근 경- ldquo rdquo( 2009)

zb35) 위 에 대한 설 으 적절한 것은

업 시 여 훈과 감동 다①

에 주 평 드러 다②

사 사 등 식 과 ③

④ 다 근거 시 여 삶에

⑤ 살 시 사 경 께 여

습 시 다

가 시간 많지 다 청량리 생 병원( )

마지막 상 경 릿 게 들어 다 그 는

십 만 큰 가 상 말 다

지 못 들 마 갈 고 돗

도시민들 싹 싹 탔다 가 시

월에 병원에 원4 가 폐( )疲弊

진 몸도 갈 미 지 못 고 었다( )解渴 가는

얼마 지 생 에 생각

가 마감 는 신 평생 십 만에

가 과 많 닮 다고 생각 지는

가 운 는 어 어( ) ldquo rdquohelliphellip

월 새벽 시1965 5 6 1 태 없 거웠고

는 없 그 병원에 퇴원 집

가는 마지막 마 고 마 내 거 다

가 죽 간신 에 실 다 사는 어느5 lsquo

가 죽 는 말 가 식 다 신rsquo

상에 각 시키는 에 실( )刻印

어느 가는 후 민 가가 근 었다lsquo rsquo

다 는 간 과 진실 그 다는( ) ldquo

에 단 평 견 가지고 다 내

가 그리는 간상 단 고 다 지 다 는 가

에 는 평 지 니 그리고 어린 들

미지 겨 그린다rdquo

근 간 과 진실 그리고 싶어( )

가 다 근에게 그것 진리 다 거 다 없

년 학 간고사 대비2013 2 현대고 대비

ECN-0102-2013-001-000076193

거 고 다 없 는 것 진리

다 근 진리는 후 쪽 었다 신산( )辛酸 삶

었 질곡 역사 에 지냈( )桎梏

가 눈에 든 것 료 단 료 게 보

것 었다 그것 그 에 겨우겨우

슬 슬 생 어가는 간들 었다

리 과 단 리 고리에 검

마 없 거리 돌

상 것 없는 등 근에게 상에

과 진실 엄 다는 사실 리는 가 실( )儼存

고 가 과 역경 에 도 근 내 포

없었 후 보루 다( ) 堡壘 도 365

도 간 근 여

시 것 다

마 같 가가 고 싶었 근에게 그 꿈( )

에 다가가는 지 다 다 가 지망생들

규 미 상 에 진 고

에 지만 근 다 다 근

미 에 운 것 보통 시 미 시간

다 그런 그에게 없는 연습 가가

통 다 가 귀 시 지 도

얻는 뛸 듯 뻤지만 마 도 (

는 었 에 어린 근 주 에)

에 그림 그리고 지우고( )粉板

복 시간 가는 게 루 보냈다

zb36) 전 의 성 소가 아닌 것을 고르

① 평 ② 사건 ③ 경

④ ⑤ 훈

늘 지 상에 살고 는 사 들 억 도가10

고 그리 지 통 고 는 사 들( )知的

그보다 훨 많 억 도는 고 지 20

통 다 그런 지 고 2500

그리 간 보는 과 사 에

매우 달 뿐만 니 과 에 도 극

루고 었다 미 운 그런 들

살고 는 동 과 사 들 사고 식에

큰 가 다는 다

고 그리 들 우주 개별 고 독립

사 들 생각 지만 고 들 우

주 연 질 간주 다 같( ) 看做

각 도 들에게는 연 질

었지만 그리 들에게는 미 들 결 었

다 고 과 그리 들 사 같

는 동 과 사 에 도 견 다

지심리 미 마 드 겐트 는

살 들에 에 지 다

연 동 과 상 다 과 같 실험

다 크 만든 미드 도 보

여 주고 그 상 닥 고 주었다lsquo (Dax)rsquo

실 닥 는 재 지 는 것 실험 가lsquo rsquo

만들어 낸 다 그런 다 개 다 체 보

여 주었는 는 미드 지만 틱

만들었고 다 는 재료는 크 지만

달 다 그러고 어 것 닥 지 사 들에게 고 lsquo rsquo

게 니 들 주 같 고 는

체 택 고 동 들 같 재료 만들어진 체

택 다 러 는 심지어 살짜리

들에게 도 타났다 것 곧 과 동

다 상 보고 다는 것 미 다

개별 사 보고 고 동 연 질 보

고 는 것 다

동 들 주변 상 에 맞 어 동 고

에 다 사 들 태도 동에 보다 많

주 울 다 동 가 미시간 에

에 경험 다 그는 미식

경 보러 가게 었는 경 체는 매우 재미 었

주변 들 동에 질 다 그 는

들 계 어 상태 경 다

어 들 에 에 그 시 가 계 가

진 것 다 상 살펴 는 말 들 lsquo rsquo

에 그는 에 시 어 도 뒷사

생각 곧 다시 곤 것 다 그런 그에게 뒷

사 고 지 는 들 동 럼

어 웠다

생각 지도 리 드 니 벳-

zb37) 다음 위 의 내 전개 으 만 인lt gt

것은

lt gt

대조의 통해 대상이 닌 특성을 설 하고 있다

일화를 제 하여 자 의 주장을 뒷 침하고 있다

유추의 을 사 하여 독자의 의해를 돕고 있다

대상이 형성되는 과정을 간적 서에 따라 서 하고 있

① ②

③ ④

년 학 간고사 대비2013 2 현대고 대비

ECN-0102-2013-001-000076193

가 우리가 말 고 쓰는 든 단어가 사 에 는( )

것 니다 사 격에 가 는 지만

어 사 과 같 특별 는 사 니lsquo rsquo

단어 격 보 단어가 사 에

등재 어 다 리 리 사 는 단어 도 그

것 시 사 는 어 고 사 에

격 보 것 니다

러 얼 은 사전에 를 있는가 이에 대한 답lsquo rsquo

은 얼 이 유행어인가 아닌가에 따라 갈라 다 이 단어lsquo rsquo

는 년 어 자 에 랐고 쓰이고 있으2002 lsquo rsquo

유행어라고 하 에는 생 이 다 런데 계속

을 유 하 서 사전에 등재될 자격을 획득할 것인가 이

에 대한 답을 내리 는 히 어 다

여 서 가 를 고 해 볼 있다 첫 는 이 단어

를 써야 할 필 가 속적으 있는가 하는 점이다

상주의 열풍에 휩 인 사회 위 에 편 해서 퍼 말

이 얼 인데 과연 런 위 가 속될 것인가 이에lsquo rsquo

대해 필자의 생각은 정적이다 사회 위 가 뀌

런 말을 쓸 일이 없어 것이다

다음은 단어의 성이다 단어의 성이 사회적으 거

감이 없으 계속 사 될 가능성이 높다 런 에서

얼 은 좋은 조건이 아니다 익히 알 졌듯이 이lsquo rsquo

말은 얼 과 청소년층에서 속어 사 하는 이 결합lsquo rsquo lsquo rsquo

된 말이다 얼 에서 얼 을 리하는 조어 도 lsquo rsquo lsquo -rsquo

어에서는 매 낯선 이다 이것만으 도 거 감을 갖

는 사람들이 있다 더 나 속어 결합한 말이다 얼 lsquo rsquo

이 널리 퍼졌다 해도 은 여전히 청소년층의 속어lsquo rsquo

남아 있다 속어는 자연 럽게 아 자리에서나 쓰 에는

담 러 말이다 러한 담을 하고 사

역을 넓혀 가는 속어도 없 는 않다 특히 얼 은 lsquo rsquo

에도 종종 등장한다 만큼 거 감이 많이 희석되었다

고 할 있다 러나 일상의 자연 러 대화에서도 거

리낌 없이 등장하는가 게 는 되 않았다고 생

각한다

얼 이 유사어인 쌈 등을 만들어 내고lsquo rsquo lsquo rsquo

있으니 살아남을 있을 것이라고 는 견해도 있을 것

이다 러나 간이 나 서 유사어를 포함하여 든

말이 사라 사 는 많다 유사어가 많다는 것이 생 을

유 할 있는 절대적인 조건은 아니다

나 언젠가 터 사람들은 어느 단에서 얼 이 가장( )

쁜 사람을 가리켜 얼 이라고 르고 있다 이 얼lsquo rsquo lsquo rsquo

이라는 단어가 최근 어사전에 라 항간에 논란이 일고

있다 아닌 게 아니라 얼 은 유행어처럼 인다 생 lsquo rsquo

도 리 래되 않은 것 같고 언제 사라 도 알

없다 게다가 젊은이들 사이에서 주 쓰일 뿐이다 이런

단어를 사전에 는다는 게 하 이 없어 이 도

한다

러나 속단은 이다 차근차근 따져 볼 일이다

선 얼 이 일 적 유행어인 아닌 주의 게 들여다lsquo rsquo

볼 필 가 있다 유행어란 유행에 따라 빠르게 유포되었

다가 단 간 내에 소 되는 단어나 를 가리킨다

얼 은 인터넷을 통해 속히 퍼 말이다 하 만 일lsquo rsquo

적인 유행어처럼 단 간 내에 사라 않았을 뿐 아니라

현재 도 잦은 빈도 사 되고 있고 앞으 도 상당

간 사 될 것으 측된다 한 언 재단의 뉴 검 lsquo rsquo

색 사이트에 따르 얼 은 년 에 처음 나타난lsquo rsquo 2001

이후 꾸 히 사 되고 있다

이 같은 사 빈도는 얼 이 일 적 유행어 는 현lsquo rsquo

저히 다르다는 것을 여 다 장 간의 생존 만으 도

얼 은 이 한 어의 어휘 에 를 자격을 얻었다lsquo rsquo

고 할 있다 더 이 이라는 비 적 정제된 매체에

높은 빈도 쓰이고 있 않은가 사 빈도 측 에서

필통이나 연필과 같은 단어 대등하거나 더 많이 쓰lsquo rsquo lsquo rsquo

다는 것은 결코 가 게 볼 일이 아니다

이제는 사전이 언어 현 을 빠르게 하는 게 덕인

대가 되었다 세계적으 유 한 의 사전들도 경쟁

적으 어를 고 있다

하 만 얼 은 젊은이들이나 쓰는 속어라고 흠을 잡을lsquo rsquo

도 르겠다 얼 이 주 젊은 층에서 많이 쓰 lsquo rsquo

는 속어임에 틀림없다 러나 어사전에 표 적이고 품

위 있는 말만 어야 한다고 생각한다 것은 커다란

해다 당장 아 어사전이나 펼쳐 라 속어는

설과 같은 비어나 죄자들이 쓰는 은어 어

마니 같은 소 의 사람만이 쓰는 말 도 라 있

않은가 사전은 말 치에 일정 빈도 이상 나타나는 말이

라 말이든 다 할 있다

zb38) 가 나 에 대한 다음의 설( ) ( ) 않은 것은

① 가 는 얼짱 사 에 등재 것에( ) ( ) lsquo rsquo

보 고 다

② 사 등재 가는 단어 격에( )

고 고 는 언 들 언어 사 도에 고 다 ( )

③ 가 얼짱 어지만 신 과 같 매( ) ( ) lsquo rsquo

체에 도 사 는 말 는 고 다

④ 가는 얼짱 어 보고 크게 가지 근( ) lsquo rsquo 3

거 들어 뒷 고 다

⑤ 는 얼짱 어 는 다 특 다는( ) lsquo rsquo

근거 에도 크게 가지 근거 가 들어 주 2

뒷 고 다

가 늘 지 상에 살고 는 사 들 억( ) 10

도가 고 그리 지 통 고 는 사 들

그보다 훨 많 억 도는 고 지 20

통 다 그런 지 고 2500

년 학 간고사 대비2013 2 현대고 대비

ECN-0102-2013-001-000076193

그리 간 보는 과 사 에

매우 달 뿐만 니 과 에 도 극

루고 었다 미 운 그런 들

살고 는 동 과 사 들 사고 식에

큰 가 다는 다

고 그리 들 우주 개별 고 독립

사 들 생각 지만 고 들 우

주 연 질 간주 다 같 각

도 들에게는 연 질 었지

만 그리 들에게는 미 들 결 었다

고 과 그리 들 사 같 는

동 과 사 에 도 견 다

인 리학자인 츠 이마이 디드 겐트너는 두

살이 채 안 된 아이들에서 터 성인에 이르 다양한

연 대의 동양인과 서양인을 대상으 다음과 같은 험

을 했다 저 코르크 만든 피라 드 양의 도형을

여 주고 대상의 이름을 닥 라고 알 주었다lsquo (Dax)rsquo

제 닥 는 존재하 않는 것으 험자가 임의lsquo rsquo

만들어 낸 이름이다 런 다음 두 개의 다른 체를

여 주었는데 하나는 피라 드 양이 만 하얀 플라 틱

으 만들었고 다른 하나는 재 는 코르크 만 양이

달랐다 러고 나서 어떤 것이 닥 인 사람들에게 고 lsquo rsquo

르게 했더니 서양인들은 주 같은 양을 하고 있는

체를 선택했고 동양인들은 같은 재 만들어 체를

선택했다 이러한 차이는 성인은 어 두 살 리

아이들에게서도 나타났다 이것은 곧 서양인과 동양인은

서 다른 세상을 고 있다는 것을 의 한다 략 ( )

는 아주 단 하 서도 인상적인 험을 했다

험에는 동서양의 대학생들이 참여했다 는 험 참가자

들에게 컴퓨터 화 을 통해 속 장 을 담은 애니 이션

을 여 주었다 화 의 앙에는 초점의 역할을 하는 커

다란 고 한 마리가 있었고 주위에는 다른 생

들과 초 자갈 거품 등이 함 제 되었다 화 을

두 씩 후 참가자들은 자 이 것을 회상해 라는

를 았다

결과 서양인 대학생들과 동양인 대학생 두 앙

의 초점 역할을 했던 고 를 동일한 정도 언 했으

나 경 소 위 거품 초 다른 생 들 에 ( )

대해서는 동양인 대학생들이 서양인 대학생들 다 60

이상 더 많이 언 했다 뿐만 아니라 동양인 학생들은 서

양인 학생들에 비해 개 적인 고 다 전체적인 계

를 더 언 하는 경향을 다 략 또한 경의 일 ( )

를 화 킨 림을 제 하 을 때 동양인 대학생들은 대

경의 화를 알아챘 만 서양인 대학생들은 경

의 화를 거의 알아차리 했다 략 ( )

따라서 서양인들만을 대상으 연 한 화lsquo

편성 결 은 잘 된 것일 도 있다 각 과정과 인rsquo

과정의 어떤 이 화 편적이고 어떤 이

화에 따라 달라 는 는 앞으 많은 연 를 통하여 논의

되어야 한다

나 어떤 의 에서 리 두는 이 화적이다 리( )

안에는 다른 사람들과 더 친 한 계를 유 하 는 상호

의존성과 다른 사람들 터 독립적인 존재 살아가 는

독립성이 혼재한다 따라서 이 에서 어떤 특성이 더 강

하게 각되는 상황에 놓이느냐에 따라 서 다른 화적

특 을 일 있다 결 리 두는 어떤 경 에는

동양인처럼 행동하고 어떤 경 에는 서양인처럼 행동하는

것이다

zb39) 가 에 대한 다음의 설( ) 않은 것은

① 는 신 주 뒷 닥 실험과lsquo rsquo lsquo

니 실험 근거 시 다rsquo

② 동 들 상 간 공통 보다는 에 식

는 강 다

③ 들 주변 맥 에는 심 경 어 사건

과 사건 사 계에 상 민감 다

④ 는 동 과 틀린 지 고 는 것lsquo rsquo

니 다 고 다 lsquo rsquo

⑤ 가에 우리 사 들 개 시 가 원( )

집 경 말 고 는 것 개 보다는

에 고 는 것에 다

늘 지 상에 살고 는 사 들 억 도가10

고 그리 지 통 고 는 사 들( )知的

그보다 훨 많 억 도는 고 지 20

통 다 그런 지 고 2500

그리 간 보는 과 사 에

매우 달 뿐만 니 과 에 도 극

루고 었다 미 운 그런 들

살고 는 동 과 사 들 사고 식에

큰 가 다는 다

지심리 미 마 드 겐트 는 동

과 상 다 과 같 실험 다

크 만든 미드 도 보여 주고 그

상 닥 고 주었다 그런 다lsquo (Dax)rsquo

개 다 체 보여 주었는 는 미드

지만 틱 만들었고 다 는 재료는

크 지만 달 다 그러고 어 것 닥 lsquo

지 사 들에게 고 게 니 들 주 같rsquo

고 는 체 택 고 동 들 같

재료 만들어진 체 택 다 러 는

심지어 살짜리 들에게 도 타났다 것

곧 과 동 다 상 보고 다는

것 미 다 개별 사 보고 고 동

년 학 간고사 대비2013 2 현대고 대비

ECN-0102-2013-001-000076193

연 질 보고 는 것 다

동 들 주변 상 에 맞 어 동 고

에 다 사 들 태도 동에 보다

많 주 울 다 동 가 미시간

에 에 경험 다 그는 미

식 경 보러 가게 었는 경 체는 매우 재

미 었 주변 들 동에 질 다 그

는 들 계 어 상태 경

다 어 들 에 에 그 시 가 계

가 진 것 다 뒷사 고 지 는 들

동 럼 어 웠다

그는 경험에 어 얻어 동 들lsquo

각도 상 본다 는 가 우고rsquo

검 여 주 단 도 상 실험 실

시 다 그는 실험 가 들에게 컴퓨 통

담 니 보여 주었다

에는 역 는 커다 고 마리가 었

고 주 에는 다 생 들과 갈 거 등

께 시 었다 본 후 가 들

신 본 것 상 보 는 지시 다

그 결과 생들과 동 생

역 고 동 도 언

경 거 다 생 들에 ( )

는 동 생들 생들보다 60

상 많 언 다 뿐만 니 동 생들

생들에 개별 고 보다 체 계

언 는 경 보 다 경 변 시

킨 그림 시 동 생들 경

변 지만 생들 경 변

거 리지 못 다

지 지 들만 상 연 lsquo

보편 결 못 것 도 다 지각 과 과rsquo

지 과 어 보편 고 어

에 달 지는지는 많 연 통 여

어 다

리 드 니 벳 생각 지도 사- ldquo rdquo( 2004)

zb40) 위 에 대한 설 으 가장 적절한 것은

① 동 과 생 식 강 고 다

② 가지 실험 통 쓴 고 다

③ 닥 실험에 사 본질에 동 사

상에 주 다

④ 니 실험에 동 과 에 지

각 도에 가 다

⑤ 쓴 는 보편 연 에 드러 우월 에

에 근 고 다

가 동 들 주변 상 에 맞 어 동 고( )

에 다 사 들 태도 동에 보다 많

주 울 다 동 가 미시간 에

에 경험 다 그는 미식

경 보러 가게 었는 경 체는 매우 재미 었

주변 들 동에 질 다 그 는

들 계 어 상태 경 다

어 들 에 에 그 시 가 계 가

진 것 다 상 살펴lsquo 는 말 들rsquo

에 그는 에 시 어 도 뒷사

생각 곧 다시 곤 것 다 그런 그에게

뒷사 고 지 는 들 동 럼

어 웠다

그는 경험에 어 얻어( ) 동 들lsquo

각도 상 본다 는 가 우고rsquo

검 여 주 단 도 상 실험

실시 다 실험에는 동 생들 여 다

그는 실험 가 들에게 컴퓨 통

담 니 보여 주었다 에는

역 는 커다 고 마리가 었고 주 에는

다 생 들과 갈 거 등 께 시

었다 본 후 가 들 신 본 것

상 보 는 지시 다

다 그 결과 생들과 동 생( )

역 고 동 도 언

경 거 다 생 들 에 ( )

는 동 생들 생들보다 60

상 많 언 다 뿐만 니 동 생들

생들에 개별 고 보다 체 계

언 는 경 보 다 들어 동

생들 상 체 연못 럼 보 어ldquo 같rdquo

체 맥 언 시 었지만

생들 상 어 같 큰 고 가 쪽 움ldquo

직 어 같 역 고rdquo

언 시 다 경 변 시킨 그

림 시 동 생들 경 변

지만 생들 경 변 거

리지 못 다

년 학 간고사 대비2013 2 현대고 대비

ECN-0102-2013-001-000076193

게 볼 동 들 보다는 큰 그( )

림 보 에 사 과 체 맥 연결시 지각

는 경 고 체에 특 떼어 내

어 독립 보는 것 낯 어 다 에

들 사 에 고 주변 맥 에는 심 경

에 사건과 사건 사 계에 상

민감 편 다

마 지 지( ) 들만 상 연

보편 결 못 것 도 다lsquo rsquo 지각 과

과 지 과 어 보편 고 어

에 달 지는지는 많 연 통 여

어 다

리 드 니 벳 생각 지도 사- ldquo rdquo( 2004)

zb41) 의 하는 가~ 다른 것은

① ② ③

④ ⑤

얼마 그 에 동 사고 식과

사고 식 보여 주는 내 다

들 에 는 탕 고 같 게

어 겨 고 미 에 는 그 크 럼 큰 고

어리 주고 원 는 어 도 는

상 고 생각 다는 것 다 러

는 어떻게 생 것 고 과 그리 거슬

러 가 보 그 단 다

고 연 경 체 경 생 에

다 벼 사는 공동 업과 경험 많 연 역

에 고 들 연 웃과

게 지내 고 탁 연 들

들 지 연 럽게 들 다 민들

웃과 동 게 뿐만 니 는 집 과

게 다

동 시 는 생태 경 에 살 결과

들 다 사 들 사 상 에 주

울 게 었고 는 곧 체 상 과 간 사

계 시 는 낳게 었다 신 가

가 는 체에 는 원 는 동시

에 다 사 들 그 사 포 체 맥 에

다 들 간 사 연

계 체 계에 주 울 는 사고 체계

게 었다

그러 그리 연 경 그 었다 산

지 연결 는 지 건 그리고 역

에 다 런 들 업에 다 사 과

동 므 공동체에

다고 다 고 그리 들

들과는 달리 보 내 감 지 들과

지 크게 느 지 못 다 그

견 다 경우 주 쟁 통 결 는 갖

게 었다

신 사 간 계들 루어진 커다

트워크 에 게 당연 사 역시 연

계들 체 식 게 다 어 상

원 도 그 개체가 체 맥 과

계 에 고 다 게 체 맥 에 주

울 다 보 상 복 과 가변 식 게 고

상에 재 는 많 변 들 사 에 재 는 들도

게 다 들 주 태도 보

는 경우가 많다 쟁 결

통 결 보다는 통 결

는 보 다

그러 고 그리 들 개개 사 사 독

에 주 울 다 사 사 체에

어 그들 사 에 재 는 공통 규 주

고 다 상 원 에도 사

체 내 주 고 다 그들

체 여 탕 체

는 주 태도 시 고 특 사 어

주에 는지 여 그 주에 는 규

견 다 에 는 쟁 식 리

같 리 사고 체계가 달 게 었다

리 드 니 벳 생각 지도 사- ldquo rdquo( 2004)

zb42) 위 에서 사 된 설 과 가장 유사한 것은

① 크톱 컴퓨 는 본체 니 마우 루

어 다

② 곡과 시 리 는 지 과 사 루어 다는 공통

지니고 다

③ 경 고 것과는 달리

경 본 연 태 그 주변 경

④ 벽돌 능 에 사계 내내

습도가 지 다

⑤ 잰느 체 체 지닌 재 체가 없

는 재 눌 다

년 학 간고사 대비2013 2 현대고 대비

ECN-0102-2013-001-000076193

zb43) 는 립 앙 도서 이 정의 일 이다lt gt

도서 장과 이 자의 리 의 정의 연결이

적절하 않은 것은

lt gt

제 조 서 유8 ( )

도서 장은 다른 이 자의 안전을 위협하거나 도서 의①

서를 란하게 할 가 있는 자에 대하여는 도서 출입

을 제한할 있다

도서 장은 이 자가 제 조 각 호의 어느 하나의 행위를 하7②

을 때에는 이 을 하게 하거나 도서 출입을 제한할

있다

제 조자 의 대출9 ( )

도서 자 는 다음 각 호의 경 대출할 있다①

상호대차도서 간에 자 를 류하는 것을 말한다 등 다1 ( )

른 도서 과의 협 을 위하여 필 한 경

공 이 공 행 상 필 하는 경2

에 도서 장이 필 하다고 인정하는 경3

대출이 가능한 도서 자 의 위는 도서 장이 정하는②

에 따른다

제 조 상10 ( )

이 자가 도서 자 설을 더럽히거나 찢거나 뜨①

쓰게 하거나 잃어 린 경 에는 상하여야 한다

도서 장은 제 항에 따른 상 을 정하여 게 하여야1②

한다

제 조이 절차 등11 ( )

이 칙에서 정한 것 에 도서 자 설의 이 절차

이 제한 등에 필 한 사항은 도서 장이 정한다

출처 립 앙 도서- (httpwwwnlgokr)

① 는 도 리 다8

② 도 는 리 다9 1

③ 료 지 는 도 리 다9 2

④ 도 료 변상에 리10 1

⑤ 는 에 도 리 다11

3

도 다 각 같다①

공 공 다만 연1

연 간 다

매월 째 째 월2

도 도 리 그 사3

가 다고 는

도 에 미리 게1 3②

시 여 다

4

도 시간 도 여 게시 다

5

도 료 시 는 는 도①

지에 등 후

등 에 사 도②

7

는 다 각 여 는 니 다

도 료 시 상 리1 lsquo rsquo

도 료 시 훼 는2 middot

지 가 닌 곳에 식 거 담3

우는

도 보 등 보 검색열4 middot

그 에 도 질 지 여 도5

여 게시 사 는

8

도 다 거 도①

질 게 우 가 는 에 여는 도

도 가 각 어느7②

에는 지 게 거 도

9

도 료는 다 각 경우 다①

상 도 간에 료 는 것 말1 (

다 등 다 도 과 여 경우)

공 원 공 상 는 경우2

그 에 도 다고 는 경우3

가능 도 료 는 도②

는 에 다

10

년 학 간고사 대비2013 2 현대고 대비

ECN-0102-2013-001-000076193

가 도 료 시 럽 거 거①

못 쓰게 거 어 린 경우에는 변상 여

도 에 변상 여 게시1②

여 다

zb44) 위 에서 도서 장이 게 해야 할 사항에 해당하는

것을 두 쓰

년 학 간고사 대비2013 2 현대고 대비

ECN-0102-2013-001-000076193

립 도 규

1 ( )

규 립 도 립 어린 청 도(

포 다 료 시 열 시 말) (

다 에 사 규 립 도)

편 진 다

2 ( )

규 립 도 도 다 에( lsquo rsquo )

고 는 도 에 도lsquo rsquo 2 2

료 에 여 다 다만 특 료 귀

료 등 료 에 사 립 도

도 다 다( lsquo rsquo )

3 ( )

도 다 각 같다①

공 공 다만 연1

연 간 다

매월 째 째 월2

도 도 리 그 사3

가 다고 는

도 에 미리 게1 3②

시 여 다

시간4 ( )

도 시간 도 여 게시 다

등 등5 ( )

도 료 시 는 는 도①

지에 등 후

등 에 사 도②

사 료6 ( )

도 료 시 에 사 료는 도

7 ( )

는 다 각 여 는 니 다

도 료 시 상 리1 lsquo rsquo

도 료 시 훼 는2 middot

지 가 닌 곳에 식 거 담3

우는

도 보 등 보 검색열4 middot

그 에 도 질 지 여 도5

여 게시 사 는

질 지8 ( )

도 다 거 도①

질 게 우 가 는 에 여는 도

도 가 각 어느7②

에는 지 게 거 도

료9 ( )

도 료는 다 각 경우 다①

상 도 간에 료 는 것 말1 (

다 등 다 도 과 여 경우)

공 원 공 상 는 경우2

그 에 도 다고 는 경우3

가능 도 료 는 도②

는 에 다

변상10 ( )

가 도 료 시 럽 거 거①

못 쓰게 거 어 린 경우에는 변상 여

도 에 변상 여 게시1②

여 다

등 규 에 것 에 도11 ( )

료 시 등에 사

도 다

립 도- (httpwwwnlgokr)

zb45) 도서 장의 리 있는 조항으 적절하 않

은 것은

① ② ③ ④ ⑤

년 학 간고사 대비2013 2 현대고 대비

ECN-0102-2013-001-000076193

1 ( )

사가 공 는lsquo rsquo

과 여 사 원과 리

사 타 사 규

니다

개 보 보7 ( )

사는 보통신망 등 계 는 에lsquo rsquo lsquo rsquo

원 개 보 보 니다 개lsquo rsquo

보 보 사 에 는 사 개lsquo rsquo

보 취 니다 다만 사는 다 lsquo rsquo

사 계 통 공 는 경우 원 lsquo rsquo

등 개 보 당 사에 습니lsquo rsquo

원 리에8 (lsquo rsquo lsquo rsquo lsquo rsquo

)

원 에 리lsquo rsquo lsquo rsquo lsquo rsquo①

원에게 가 도 여 는lsquo rsquo 3

니다

사는 원 가 개 보 우 가lsquo rsquo lsquo rsquo lsquo rsquo②

거 사 경우 는 미 에 어 거 lsquo

사 사 운 우 가 는 경우 당rsquo lsquo rsquo

습니다lsquo rsquo

원 가 도 거lsquo rsquo lsquo rsquo lsquo rsquo 3③

가 사 고 지 경우에는 시 사에lsquo rsquo

통지 고 사 내에 니다lsquo rsquo

경우에 당 원 사에 그 사실3 lsquo rsquo lsquo rsquo④

통지 지 거 통지 도 사 내에 지 lsquo rsquo

생 경우 사는 지지 습니다lsquo rsquo

사10 (lsquo rsquo )

사는 과 지 미lsquo rsquo①

에 는 지 계 고

공 여 다 여 니다lsquo rsquo

사는 원 게lsquo rsquo lsquo rsquo lsquo rsquo②

도 개 보 신 보 포 보 보 시( )

갖 어 개 보 취 공시 고

니다

사는 과 여 원lsquo rsquo lsquo rsquo③

견 만 당 다고 경우에는

리 여 니다 원 견 만 사 lsquo rsquo

에 는 게시 거 우편 등 통 여

원에게 리 과 결과 달 니다lsquo rsquo

원11 (lsquo rsquo )

원 다 여 는 니다lsquo rsquo ①

신청 는 변경 시 허 내 등1

타 보 도2

사가 게시 보 변경3 lsquo rsquo

사가 보 보 컴퓨 그4 lsquo rsquo (

등 등 신 는 게시)

사 타 등 지 재산 에5 lsquo rsquo 3

사 타 상 거 업6 lsquo rsquo 3

는 폭 시지 상 타 공7 middot middot

에 는 보 에 공개 는 게시 는lsquo rsquo

사 동 없 리 사8 lsquo rsquo

타 거 당9

게시15 (lsquo rsquo )

원 내에 게시 는 게시 게재 는lsquo rsquo lsquo rsquo lsquo rsquo

경우 원 사가 게시 복 lsquo rsquo lsquo rsquo lsquo rsquo middot middot

등 태 언 등에 공 는

것 내에 다 원 본 게시 등 lsquo rsquo lsquo rsquo

크 능 등 여 복 는 등 태

는 것 동 것 니다

- (wwwnavercom)

zb46) 위 은 인터넷 포털사이트의 회 가입을 위한 이

약 의 일 이다 이 약 을 만드는 과정에서 생각한

내 으 적절하 않은 것은

개 보 보 가 지에 별 눠①

겠어

원 가 만들게 에②

시 주어 겠어

원들 게재 게시 다 원 크 다③

는 것 지

④ 원 지 는 뿐만 니 사가 지 는

도 께 달 지

리에 가 생 경우 사가⑤

에 다는 도 듯

1 ( )

사가 공 는lsquo rsquo

과 여 사 원과 리

사 타 사 규

년 학 간고사 대비2013 2 현대고 대비

ECN-0102-2013-001-000076193

니다

개 보 보7 ( )

사는 보통신망 등 계 는 에lsquo rsquo lsquo rsquo

원 개 보 보 니다 개lsquo rsquo

보 보 사 에 는 사 개lsquo rsquo

보 취 니다 다만 사는 다 lsquo rsquo

사 계 통 공 는 경우 원 lsquo rsquo

등 개 보 당 사에 습니lsquo rsquo

원 리에8 (lsquo rsquo lsquo rsquo lsquo rsquo

)

원 에 리lsquo rsquo lsquo rsquo lsquo rsquo①

원에게 가 도 여 는lsquo rsquo 3

니다

사는 원 가 개 보 우 가lsquo rsquo lsquo rsquo lsquo rsquo②

거 사 경우 는 미 에 어 거 lsquo

사 사 운 우 가 는 경우 당rsquo lsquo rsquo

습니다lsquo rsquo

원 가 도 거lsquo rsquo lsquo rsquo lsquo rsquo 3③

가 사 고 지 경우에는 시 사에lsquo rsquo

통지 고 사 내에 니다lsquo rsquo

경우에 당 원 사에 그 사실3 lsquo rsquo lsquo rsquo④

통지 지 거 통지 도 사 내에 지 lsquo rsquo

생 경우 사는 지지 습니다lsquo rsquo

원에 통지9 (lsquo rsquo )

사는 특 다 원에게 통지 경우lsquo rsquo lsquo rsquo

공지 게시 통 상 게시 개별 통지에7

갈 습니다

사10 (lsquo rsquo )

사는 과 지 미lsquo rsquo①

에 는 지 계 고

공 여 다 여 니다lsquo rsquo

사는 원 게lsquo rsquo lsquo rsquo lsquo rsquo②

도 개 보 신 보 포 보 보 시( )

갖 어 개 보 취 공시 고

니다

사는 과 여 원lsquo rsquo lsquo rsquo③

견 만 당 다고 경우에는

리 여 니다 원 견 만 사 lsquo rsquo

에 는 게시 거 우편 등 통 여

원에게 리 과 결과 달 니다lsquo rsquo

원11 (lsquo rsquo )

원 다 여 는 니다lsquo rsquo ①

신청 는 변경 시 허 내 등1

타 보 도2

사가 게시 보 변경3 lsquo rsquo

사가 보 보 컴퓨 그4 lsquo rsquo (

등 등 신 는 게시)

사 타 등 지 재산 에5 lsquo rsquo 3

사 타 상 거 업6 lsquo rsquo 3

는 폭 시지 상 타 공7 middot middot

에 는 보 에 공개 는 게시 는lsquo rsquo

사 동 없 리 사8 lsquo rsquo

타 거 당9

원 계 규 내lsquo rsquo lsquo②

여 공지 주 사 사가 통지 는rsquo lsquo rsquo

사 등 여 타 사 업 에 lsquo rsquo

는 여 는 니다

- (wwwnavercom)

zb47) 위 약 의 조항에서 같은 제점을 하lt gt

고 있는 조항은

lt gt

제휴 회사에 회 의 아이디 개인 정 를 전송할 있도

한 조항은 고객에게 당한 조항이다

1 7 8① ② ③

④ 9 ⑤ 10

립 도 규

1 ( )

규 립 도 립 어린 청 도(

포 다 료 시 열 시 말) (

다 에 사 규 립 도)

편 진 다

2 ( )

규 립 도 도 다 에( lsquo rsquo )

고 는 도 에 도lsquo rsquo 2 2

료 에 여 다 다만 특 료 귀

료 등 료 에 사 립 도

도 다 다( lsquo rsquo )

3 ( )

도 다 각 같다①

공 공 다만 연1

연 간 다

년 학 간고사 대비2013 2 현대고 대비

ECN-0102-2013-001-000076193

매월 째 째 월2

도 도 리 그 사3

가 다고 는

도 에 미리 게1 3②

시 여 다

시간4 ( )

도 시간 도 여 게시 다

등 등5 ( )

도 료 시 는 는 도①

지에 등 후

등 에 사 도②

사 료6 ( )

도 료 시 에 사 료는 도

7 ( )

는 다 각 여 는 니 다

도 료 시 상 리1 lsquo rsquo

도 료 시 훼 는2 middot

지 가 닌 곳에 식 거 담3

우는

도 보 등 보 검색열4 middot

그 에 도 질 지 여 도5

여 게시 사 는

질 지8 ( )

도 다 거 도①

질 게 우 가 는 에 여는 도

도 가 각 어느7②

에는 지 게 거 도

료9 ( )

도 료는 다 각 경우 다①

상 도 간에 료 는 것 말1 (

다 등 다 도 과 여 경우)

공 원 공 상 는 경우2

그 에 도 다고 는 경우3

가능 도 료 는 도②

는 에 다

변상10 ( )

가 도 료 시 럽 거 거①

못 쓰게 거 어 린 경우에는 변상 여

도 에 변상 여 게시1②

여 다

등 규 에 것 에 도11 ( )

료 시 등에 사

도 다

립 도- (httpwwwnlgokr)

zb48) 다음 정 리 의 의 으 볼 때 가장

이 적인 것은

도 시간 도 여 게시 다①

등 에 사 도②

가능 도 료 는 도 는③

에 다

④ 도 에 변상 여 게10 1

시 여 다

⑤ 도 가 각 어느7

에는 지 거 도

zb49) 를 참고하여 이 어의 성격을 설 한lt gt

것으 적절하 않은 것은

① 보 에 는 어 시 상 고 어 시lt gt lsquo rsquo

에 보여주고 다

② 진 어 어원에 견 고 다

에는 타 어 들어가는 것 다 lsquo rsquo

③ 에 들어갈 말 각각 고 어 어 신 어~

들 언어는 질 격 강 통 없었다

④ 시 우리 에 가 었지만 지 계

과 달리 들 통 사 달 어 웠

년 학 간고사 대비2013 2 현대고 대비

ECN-0102-2013-001-000076193

⑤ 크 몽골 만주 공통어가 우리 어 같

계열에 다는 에 사 특 짐

가( )

善化公主主隱 공주님

他密只嫁良置古 몰 결 고

薯童房乙 맛

夜矣卯乙抱遣去如 에 몰 고 가다

( )

始汝 會隱日恚見隱扐 만 에 본

恥隱汝衣淸隱笑 맑 웃

고 시 여 공 크다 만 다[ ] ( ) ( ) ( ) ( )始 汝 會扐

내다 에 보다 견( ) ( )恚 見 다( )隱

럽다 맑다 청 웃( ) ( ) ( ) ( )恥 衣 淸 笑

zb50) 위의 나 를 함 고 음에 답하( ) lt gt

보lt gt

( )素那或云金川 白城郡蛇山人也

운 사산

는 고 다 는( )[ ( ) ] (素那 金川 白城

사산 사 다) ( ) 郡 蛇山

삼 사- lsquo rsquo 47

에 제 된 단어 의 표 리를 조건(1) lt gt ( ) lt gt

에 맞게 서 하

건lt gt

lsquo 었고 었다 태rsquo

에 제 된 단어 동일한 표 리에(2) lt gt ( )

의해 적은 것을 나 에서 찾아 조건 에 맞게 서 하( ) lt gt

건lt gt

에 당 는 각각( ) 개 쓸 것2 단

당 는 가 여러 개 어도 개만 쓸 것 각2

개 과 도 쪽에 개만2 2

드시 지 것( )

과 동 원리 것lsquo 고

과 동 원리 것 다rsquo

태 것

가( )

素那(或云金川) 白城郡蛇山人也

소나 또는 천 이라 한다 는 성 사( ) ( ) ( )素那 金川 白城郡〔 〕

산 사람이다 현대어 풀이( ) ( )蛇山

나( )

紫布岩乎希 회

執音乎手母牛放敎遣 자 손 암쇼 노히 고

吾 不喩慙 伊賜等肹 肹 나 안디 리샤

花 折叱肹 可獻乎理音如 고 것거 도림다

다 향찰은 리말을 리 으 적은 표 이었 만 생( )

은 고 대를 넘 하고 끊어 고 말았다 랜 세

동안 갈고 닦아 체계적이었던 향찰 표 이 사라졌

을 인은 크게 두 가 나누어 생각해 볼 있다

하나는 족 사회의 한 선호도에서 찾을 있다 라 때

향찰은 주 족 계 에서 사 했을 것으 인다 한 을

알 하고서는 한자를 활 하여 리말을 리 으 표

하 란 가능하 때 이다 런데 족들은 간이 흐

를 향찰과 같은 리 표 을 익혀 사 하 다는

아 한 을 대 사 하는 쪽을 선호하게 되었다 더 이

고 초에 인재 등 을 위해 과거제도가 행되 서 한 선

호도가 더 높아졌고 결 향찰은 소 되고 말았다

또 다른 가능성은 한 어의 특성에서 찾을 있다

터 한 과 일 세 나라는 한자 화 에 속해 다

당연한 이야 겠 만 표의 자인 한자는 어를 표 하

에 매 적절하다 어의 음절은 성 ( ) ( )聲母 韻母

이 어 고 여 에 성조가 추가되어 최종 소리가 결정된

다 래서 어는 단음절을 하나의 한자 표 하 된

다 에 초성 성 종성의 세 가 소가 하나의 음절

년 학 간고사 대비2013 2 현대고 대비

ECN-0102-2013-001-000076193

을 이 는 한 어는 음절 조가 잡하고 음절의 가 많아

서 한자 차 만으 한 어의 소리를 만족 럽게 표 할

없었다 를 들어 한 어에서는 어 니 같이 음절 lsquo rsquo

이 어 단어가 얼마든 있으나 어는( ) 複數音節

자 하나 나타내 만이다lsquo [m ]rsquo 母 ǔ

한편 일 어의 표 은 핵 적 단어는 한자 적고 토는

가나라는 일 의 자 적는 이다 적인 의 를 나

타내는 은 표의 자인 한자 적고 적 계를 나

타내는 토는 표음 자 적는 셈이니 자세히 살펴

리의 향찰 표 을 쏙 빼닮았음을 알 있다 한 어 같

은 착어이 서도 일 어에만 향찰과 유사한 표 이 살아

남은 것은 일 어의 특 때 이다 일 어는 하나의 자음과

음의 결합으 음절을 이 고 침이 거의 없는 음절 언어

이다 이러한 음절의 특색에다가 토가 달한 착어라는 점

이 향찰과 유사한 표 이 살아남을 있는 비결이었다

하 만 같은 착어라도 다양한 음소 침이 달한 한

어는 향찰 표 하는 데 근 적으 한계가 있었다

zb51) 다 하여 의 행에 대한 탐 한 결과( ) lt gt 2

않은 것은

보lt gt

善花公主主隱 공주니믄 공주님( )

----------------------------------------

-

他密只嫁良置古 그 지 얼어 고 몰 결(

----------------------------------------

-

薯童房乙 맛 맛( )

夜矣卯乙抱遺去如 몰 고 가다 에 몰 고(

가다)

주동 역 동- (薯童謠『 』

에 2 ( )他密只嫁良置古

얼다 시집가다 결 다 말 lsquo rsquo

① 실질 미 지니고 므 타 타lsquo ( )rsquo lsquo [ ]

② 에 실질 미 타내고 지 는lsquo rsquo lsquo [ ]rsquo lsquo [ ]密只 密 只

계 타내는

③ 얼어는 실질 미 포 고 므 가lsquo rsquo lsquo [ ]rsquo嫁

것lsquo [ ]rsquo 良

④ 고 어간 는 실질 미 지니고 므lsquo rsquo lsquo -rsquo

것lsquo [ ]rsquo 置

⑤ 고 어미 고는 계 타내고 므lsquo rsquo lsquo- rsquo

고 것lsquo [ ]rsquo 古

가( )

엉 훈 민middot middot middot middot middot世 宗 御 製 訓 民 正 音

말 미 듕 귁에 달middot middot middot middot middot middot middot middot中 國 文 字

니 런middot middot middot middot middot middot 어린middot middot middot middot百 姓

니 고 도 내 들middot middot middot middot middot middot middot middot middot 시러middot

펴 몯middot 미middot middot 니 내middot middot middot middot middot middot middot middot 爲

어엿middot 겨 새middot middot middot 믈여듧middot middot middot middot字 니middot middot middot

사 마다 니겨 킈 middot middot middot middot middot middot middot middot middot便 安

고 미니middot middot middot middot

본 는 상( ) (象

원리에 만들어진 본) ( )形 ㄱ ㄴ ㅁ ㅅ ㅇ

에 는 가 원리에( )加劃

그리고( )ㅋ ㄷ ㅌ ㅂ ㅍ ㅈ ㅊ ㆆ ㅎ

쓰는 병 원리에 만들어진( )竝書

마지막 체( ) ( )異體ㄲ ㄸ ㅃ ㅆ ㅉ ㆅ

ᅀ 다 상 원리에 ㅇ ㄹ

지 는 삼재 상 본 본( ) ( ) ( 天地人 三才

탕 므림과 림에 ) (初ㅡ ㅣ

재)( ) ( )( )出字 再出字ㅗ ㅏ ㅜ ㅓ ㅛ ㅑ ㅜ ㅕ

병 그리고 들 에 다시( )ㅘ ㅝ ㅣ

( )ㅣ ㅢ ㅚ ㅐ ㅟ ㅔ ㆉ ㅒ ㆌ ㅖ ㅙ ㅞ

zb52) 가 에 대한 설 으 르 않은 것을( ) 두 고르

① 어쓰 규 지키고 다

② 리 고 다

③ 말 미 미 등 어 사 다lsquo rsquo

④ 개 지 다

년 학 간고사 대비2013 2 현대고 대비

ECN-0102-2013-001-000076193

⑤ 어 원 에 가 도 고 다

엉 훈 민世 宗 御 製 訓 民 正 音

말 미 듕귁에 달 니

런 어린 니 고 도middot

내 들 시러 펴 몯 미 니middot

내 어엿 겨 새 믈여듧

사 마다 니겨middot 킈 고

미니

훈민 언 본- lsquo rsquo 5 (1459 )

zb53) 위의 에 대한 현대어 풀이가 르~ 않은 것

① 우리 말 과 달

② 어리 말 고 는 것 어도

③ 신 생각 마 껏 펼 는 사 많다

④ 게 생각 여

⑤ 사 마다 게

zb54) 훈민정음 언해 에는 한 을 창제한 동 가 드러나

있다 훈민정음 창제의 정 과 내 이 잘 연결된 것

① 주 신 말 미 듕귁에 달

② 민 신 내 어 겨

③ 신 뻔 킈 고 미니

④ 실 신 사 마다 니겨

⑤ 귀 신 계 주 는 훈민 신과 거리가

가 엉 훈 민( ) middot middot middot middot middot世 宗 御 製 訓 民 正 音 

말 미 귁에 中 國 달 文 字

니 런 어린 니 百 姓

고 도 내 들 시러 펴 몯

미 니 내 어엿 爲 겨 새

믈여듧 니 사 마다 니 字

겨 킈 고 미니 便 安

훈민 언 본- lsquo ( )rsquo ( ) 5 (1459 )訓民正音 世祖

( )

[ 1 ]

동 룡 샤 마다 복( ) ( ) ( )海東 六龍 天福

시니 고 동( ) ( )古聖 同符 시니

[ 2 ]

매 니 곶 여

미 므 니 그 내 러

가 니

[ 125 ]

우 미리( )千世 샨( )定 에( )漢水北 累仁

누 개 샤 복 업 시니( ) ( ) 開國 卜年

신( )聖神 니 샤도 경 근민 샤 욱( )敬天勤民

드시리 다

님 쇼 산 가( ) ( )洛水 山行

미드니 가

어 가- lsquo ( )rsquo 27龍飛御天歌

다 우리신 니쓰고 다만 만 쓰( )

거 샹 귀쳔 다보게 러 귀

여 쓴 도 신 보 가 고 신 에

말 어 보게 각 에 사 들

고 본 몬 능통 후에

죠 죠 니

드 도 만 공 에 사

드 미 죠 고 고 여 보 죠

보다 얼마가 거시 어신고 니 첫

가 죠 니 죠

민 들 어 신 샹

귀쳔 도보고 어보 가 만 늘

고 폐 에 만쓴 죠 민

도 러보지못 고 보니 그게 엇지

심 니 리 보 가 어 운건 다

니 쳣 말마 지 니 고 그

쓰 에 가 우 지 지

몰 거 본후에 가 어 지

고 그니 쓴편지 쟝 보

년 학 간고사 대비2013 2 현대고 대비

ECN-0102-2013-001-000076193

쓴것보다 듸 보고 그 마 니 쓴 고

어 못

그런고 에 리 과 가

만 쓴 못 민 말만 듯고

고 편 그 못 보니 그사 단

병신 못 다고 그사 식 사

니 만 고 다 과 그사

만 고 다 과 업 사 보다 식 고

죠 도 고 각 과

견 고 실 직 귀쳔 간에 그

고도 다 것 몰 귀죡 보다

사 우리 신 귀쳔 다 업

시 신 보고 과 지 게 랴

시니 샹 귀쳔 간에 우리 신 걸

간 보 새지각과 새 걸 미리

독립신- lsquo (1896)rsquo

zb55) 친 어 나의 제 장( ) 2 매 함축적

의 가 가장 유사한 것은

① 지 눈 내리고 매 득 니 내 여 가

사- lsquo rsquo

② 도 어 리듯 그 게 어 다

주 사- lsquo rsquo

③ 눈 살 다 죽 어 린 과 체 여

눈 새벽 지 도 살 다

눈- lsquo rsquo

④ 삶 근심과 고단 에 돌 거니는 여 거 는

여 리 내린 살가지 에 눈 리 눈 리

택 그 생 에- lsquo rsquo

⑤ 늘 러 고 러

청룡 룡 어 개 루 우

신경림 계- lsquo rsquo

zb56) 친 를 위 가 나 에 나타난A B ( ) ( )

세 어의 특 에 의거하여 세 어 표 하

그 산 고 공 도 맑지만

A

주변에 쓰 리는 어리 사 많다

B

건lt gt

식 가 에 타 어 특징에( ) ( )

거 과 어쓰 는 고 지 말 것

A

B

zb57) 가 의( ) 달 아ㆍ 다 의 ( ) 나셔에서 알 있는

세 어 개화 어의 특 을 비 하여 조건 에lt gt

맞게 서 하

건lt gt

어에 는lsquo 개

어에 는 다 태rsquo

zb58) 은 가 는 다 에 나 는 절lt 1gt ( ) lt 2gt ( )

일 를 췌한 것이다 의 의 가 lt 1gt (1)~(2)

유사한 말을 에서 찾아 쓰lt 2gt

보lt 1gt

런 (1) 어린 니 고百 姓

도 내 들 시러 펴 몯 미

사 마다 (2) 니겨 便 安

킈 고 미니

보lt 2gt

죠 고 고 여 보 죠

보다 얼마가 거시 어신고 니 첫 가

죠 니 죠 민

들 어 신 샹 귀쳔

도보고 어보 가 만 늘 고

폐 에 만쓴 죠 민 도

러보지못 고 보니 그게 엇지 심

니 리

년 학 간고사 대비2013 2 현대고 대비

ECN-0102-2013-001-000076193

lt 1 gt

동 룡 샤 마다 복 시( ) ( ) ( )海東 六龍 天福

고 동 시니( ) ( )古聖 同符

lt 2 gt

(A) 매 니 곶

여 니

미 므 니 그 내

러 가 니

lt125 gt

우 미리 샨 에( ) ( ) ( ) 千世 定 漢水北 累

누 개 샤 복 업 시 니( ) ( ) 仁開國 卜年 聖

신( ) 神 니 샤도 경 근민 샤( ) 敬天勤民

욱 드 시 리 다

님 쇼 산 가 ( ) ( )洛水 山行

미드니 가

- lt gt龍飛御天歌

zb59) 장과 내 상 유사한 성격의 조는125

① 뫼 고 고 고 고

어 그린 많고 많고 고 고

어 러 는 울고 울고 가느니

도 견- lt gt

② 강 에 드니 몸 다

그믈 고 가니

뒷 뫼 엄 언 니( )藥

-

③ 말 없는 청산 태 없는 다

값 없는 청 없는 월

에 병 없는 몸 별 없 늙 리

-

④ 가마귀 골에 가지 마

낸 가마귀 새

청강에 것 시 몸 러 가( ) 淸江

-

⑤ 진 골에( ) 白雪

가 매 는 어느 곳에 었는고

에 갈 곳 몰( ) 夕陽

색-

zb60) 위 에 나타난 세 어의 특 으 적절하 않은

것은

① 룡 어 주격 사에 당 는 가 사( ) lsquo rsquo六龍

고 다

② 샤 어에도 어 주체 쓰 다

는 것 다

③ 매 어 달리 사 택에 어

가 지 지지 고 다

④ 므 원 상 직 어 지 다

⑤ 드시리 다 주체 과 상 께 사

고 다

수고 하셨습니다hearts hearts

년 학 간고사 대비2013 2 현대고 대비

ECN-0102-2013-001-000076193

보닷컴에 공 는 별 보는 고등

들 여 주 는

들 습니다 슷 동 지

가 복 는 것 도가

니 복 여 습 시고 거 시

니다

정답 해설

1) 정답[ ] ④

해설 다른 것은 두 특정 업이나 단 내에서 사[ ]

하는 일종의 은어 사회 언에 해당한다 러나

는 언이 아니라 단과대학을 여서 단대 사lsquo rsquo lsquo rsquo lsquo④

대학을 여서 사대라고 한 말에 해당하 일rsquo lsquo rsquo

사회에서도 널리 쓰이 사회 언이라 할

없다

2) 정답[ ] ⑤

해설 사회 언은 같은 단 내에서 쓰이는 언어이[ ] lsquo rsquo

동일 단끼리는 단결 과 친 감을 형성하는

능을 하 리적 안감이 일어나 않는다

3) 정답[ ] ③

해설 사람이라는 차 적 표현에 대한 대안적 표현이[ ]lsquo rsquo

인 아내 처 등으 볼 있다lsquo rsquo

4) 정답[ ]⑤

해설 남성은 주 격 체를 사 한다[ ]

5) 정답[ ] ⑤

해설 흑인은 검다라는 뜻을 가 고 있을 뿐 인[ ]lsquo rsquo lsquo rsquo lsquo rsquo

다 열등한 뜻을 내포하 않는다

6) 정답 살 색 첫 작품[ ] - -

해설 살색 혹은 킨색은 한 인의 피 색을 뜻[ ] lsquo rsquo lsquo rsquo

하는 것으 인종 차 을 추 고 출 이주민

의 평등 을 침해할 있어 년 표 이2005

살 색으 이름을 꾸었다 처녀작은 처녀라lsquo rsquo lsquo rsquo lsquo rsquo

는 단어가 가 고 있는 곡된 성 인 을 한 것

으 첫 작품정도 꾸어 사 하는 것이 좋다lsquo rsquo

7) 정답[ ] ⑤

해설 호는 아들에게 해체를 사 하고 있다[ ] ① ②

장 을 성하는 청자는 자 의 아 느리 아lsquo

들 세 이다 호는 아 느리에게 해rsquo ③

체를 사 하고 있다 호가 느리 아 에게 ④

사 한 해 체 아들에게 사 한 해체는 두 비lsquo rsquo lsquo rsquo

격 체에 해당한다 호는 자 의 아랫사람인 ⑤

느리에게 아들과 마찬가 해체를 사 하는 것이

상 이 만 임 을 한 느리에게 고마 과 쁨

존 의 표 를 하 위해 자 의 아 에게 말하듯

해 체를 사 하고 있다

8) 정답[ ] ③

9) 정답[ ] ⑤

10) 정답[ ] ①

해설 청자 할아 가 장의 주체 아 다 높을[ ] ( ) ( )

경 에는 압존 에 의해 장의 주체를 높이 않는lsquo rsquo

다 러 아 서가 아닌 아 는으 계 lsquo rsquo lsquo rsquo lsquo

니다 가 아닌 있 니다 표현하는 것이 르rsquo lsquo rsquo

11) 정답 당이 당을 쫒았다 당이[ ]

당에 다

해설[ ]

12) 정답[ ] ⑤

해설 서 다른 높임표현을 통해 청자에 대해 리[ ] ⑤

적 거리감을 나타내는 인 은 이 아니라 현정이

다 가 에서 현정은 에게 해 체를 사 함으 써 ( )

친근감을 드러낸다 나 에서 연 을 게을리하는 역 ( )

도 들 때 에 화가 난 현정이 선생님에게 항의하

는 장 에서는 하 체를 사 하여 리적 거리lsquo rsquo

가 어졌음을 나타내고 있다

13) 정답[ ] ①

해설 는 는 얼 빛이 날과 어찌 다르 고[ ] lsquo rsquo

라는 뜻으 전과 달리 임이 화자를 않고

있음을 알 있다

14) 정답 달리 후 가 있다 이를 통해 경[ ] lt gt

쾌한 음악성을 형성하고 노 젓는 상황을 체적으

형상화하는 역할을 한다

15) 정답[ ] ①

16) 정답[ ] ⑤

해설 다 의 자연은 를 성찰하게 하는 대상[ ] ( )⑤

이자 정의 대상이다 의 자연은 자 의 상황과 ⑤

처 를 드러내는 경으 서의 역할을 하 이

이 없다

17) 정답[ ] ③

해설 는 빈천 을 해결하고자 했으나 강산[ ] lsquo ( )rsquo 貧賤③

과 풍 을 달라는 에 거절하 다고 함으 써 자

연에 대한 애정을 드러내고 있으 는 않는

임에 대한 망을 개에게 전가 켜서 임에 대한 리

을 드러내고 있다

18) 정답[ ] ③

년 학 간고사 대비2013 2 현대고 대비

ECN-0102-2013-001-000076193

19) 정답[ ] ⑤

해설 고상한 음악가의 이름을 리말 꽝 럽[ ]

게 꿈으 써 언어유희를 통해 음을 유 하고 있

다 이는 고상한 척하는 총 를 비꼼으 써 비판적

태도를 드러내는 것이 대상을 꽝 럽게 표현

하여 총 의 허 과 사치를 풍자하고 있다

20) 정답[ ] ⑤

해설 는 작품 속 경에 대한 설 이 드러나는 것이[ ]

서 자의 주 적인 견해가 접적으 드러나는 것이

아니다

21) 정답[ ] ⑤

22) 정답[ ] ②

23) 정답[ ] ④

24) 정답[ ] ①

해설 적강 티프는 주인공의 비 한 출생이나 능[ ] ①

과 이 있는 것으 조정의 능함을 풍자하는lsquo rsquo

것과는 거리가 다

25) 정답 픔 나[ ] ( )

해설 의 음악은 고통 는 사람들을 위 하고 아픔[ ] lsquo rsquo

을 치유해 주는 능을 한다고 할 있다 의 lt gt

픔 도 소 된 이 과 더 어 살아가는 따뜻한 마음lsquo rsquo

을 상 한다

26) 정답[ ] ⑤

해설 에게 선천적으 주어 각 장애라는 역경[ ]

은 의 이라는 가사 연 을 있다lsquo rsquo

27) 정답[ ] ④

해설 는 장 란 선 에게 은 개인적인 인상을[ ]

소녀 장정 등으 표현한 것이다lsquo rsquo

28) 정답[ ] ②

해설 담자가 피 담자의 언어적 표현이나 비언어[ ]②

적 표현 하 독자는 담의 위 나 피

담자의 감정 상태를 알 있다 이를 통해 독자는

담 상황을 더 생생하게 느낄 있고 피 담자

를 더 잘 이해할 있게 된다

29) 정답[ ]③

해설 일상생활과 역도 선 서의 성과에 된 것에서[ ]

역도를 하 서 겪는 어 과 내적 고민으 화제를

전화하 위한 것이다

30) 정답[ ] ①

해설 릿속에 새겨 넣듯 이 억되도 함 세상[ ] ② ③

살이가 힘들고 고생 러 속 하여 자유를 ④

가 없는 고통의 상태를 비유적으 이르는 말

적의 침입을 막 위해 쌓은 축 켜야 할⑤

대상을 비유적으 이르는 말이다

31) 정답[ ] ④

해설 이 의 종류는 전 으 인 사건 경[ ] lsquo

비평을 성 소 삼는다rsquo

32) 정답[ ] ④

해설 근은 삼대독자 태어났음을 에서 확인할[ ]

있다 형제들과의 담은 이뤄 가 없다

33) 정답[ ] ⑤

해설 근은 가난에도 하고 화가를 꿈꾸었다[ ] (3

단 또한 다른 화가 망생들은 정 육을)

위해 상 학 학 해 유학 에 랐 만

근은 다른 을 찾아야 했다 단 세에(5 ) 18

근은 조선 전람회에 입선하 다 단 의(6 )

만종은 인간과 자연이 엮어 가는 경건한 조화 을lsquo rsquo

나타낸다

34) 정답[ ] ①

해설 근이 속에서도 창작활동을 추 않고[ ]

하는 닭은 은 세상과 타협할 르는

근이 세상의 이해를 하 위한 가장 떳떳한 단

이 때 이다

35) 정답[ ] ⑤

해설 전 은 서 자의 주 적인 평이 리는 것이[ ]

만 위 제 은 인 이 살았던 대 사회적 경

을 통해 객 적인 인 의 을 제 하고 있다

36) 정답[ ] ⑤

해설 전 은 인 사건 경 비평이라는[ ] lsquo rsquo⑤

성 이 어져 있다

37) 정답[ ] ①

해설 이 은 동양인과 서양인의 사고 에 차이가[ ]

있다는 것을 대조를 통해 설 하고 있다 또 쓴이

의 제자가 축 경 를 러 가서 경험한 일화를

통해 동양인이 서양인에 비해 주 상황에 더 많은

주의를 인다는 주장을 뒷 침하고 있다

38) 정답[ ] ④

39) 정답[ ] ②

40) 정답[ ] ②

41) 정답[ ] ④

42) 정답[ ] ③

43) 정답[ ] ④

44) 정답 도서 의 휴 일 도서 의 이 간 도서의[ ]

해설 도서 장은 임의 정한 휴 일과 도서 이[ ]

간 도서의 상 등을 게 할 의 가 있다

년 학 간고사 대비2013 2 현대고 대비

ECN-0102-2013-001-000076193

45) 정답[ ] ①

해설 제 조의 정 휴 일 의 휴 일의 사전 게[ ] 3

는 도서 장의 의 조항에 속한다

46) 정답[ ] ①

해설 개인 정 호 의 를 제 하 했 만 항[ ]

나눠서 제 하 않고 대 나열하고 있다

47) 정답[ ] ②

해설 제 조의 내 을 회사는 다른 회사 협[ ] 7 lsquo

계약을 통해 서비 를 제공하는 경 회 의 아이디

등 개인 정 를 해당 회사에 전송할 있다는 내rsquo

이 있으 의 제점을 제 할 있다②

48) 정답[ ] ④

해설 는 도서 장의 의 에 해당하고 나 는 도[ ] ④

서 장의 리에 해당한다

49) 정답[ ] ③

50) 정답 은 음독으 적었고 은 훈독으 적었[ ] (1)

다 과 동일한 표 리 적은 것은 이고 (2) ce

과 동일한 표 리 적은 것은 이다ab

51) 정답[ ] ③

52) 정답[ ] ①②

53) 정답[ ] ③

54) 정답[ ] ③

55) 정답[ ] ①

56) 정답 른 죠코 어린 노 하니라[ ] A B

57) 정답 세 어에서는 활 형이 칙적으[ ] lsquo rsquoㄹㅇ

나타났 만 개화 어에서는 활 형이 쓰 다 lsquo rsquo ㄹㄴ

58) 정답 호 가 흔[ ] (1) (2)

59) 정답[ ] ④

60) 정답[ ] ③

Page 13: 현대고대비 국어 - chamsoriedu.com 「콘텐츠산업진흥 법」외 에도 저작권 의하여 ... 다른주체에게어떤동작을하도록만드는것을나타내는

년 학 간고사 대비2013 2 현대고 대비

ECN-0102-2013-001-000076193

있는 것이 말고는 없었 때 에 게 뒷동

산을 달은 거 다

이 유자소전- lsquo rsquo

zb22) 의 상황을 속담으 표현한 것으 적절한 것은

① 루 곳 게 마 다

② 에 맞고 강에 눈 다

③ 늘 도 다

④ 도 사 다

⑤ 에 가도 신만 리 다

거리 공신 후[ ]

에 주 는 벼슬 심 늦도( )主簿

식 없어 과 께 산에 드리고 신

태몽 꾼 에 만고 웅 상 지닌 들

낳 키운다 그 후 신 들 에 역심

담 귀 등 심 여 리 귀 보내고

지 죽 는 도망 간다 그

만 고 에 에 어 니

헤어지게 다 지 가 사 들에

사 들 별 고 없 다니었다

마 마 돌 다니 걸 여 고

어 곤 다 에는 동쪽에 고 에

는 쪽에 니 가 에 리는 엽 가는

없 니 늘 다니는 었다

얼 말 죽 사 같고 림새가 말 니었

다 가슴 에 고 등 삼태

헌 에 니 달 가 도리 ( )奇男子

어 걸 었 담 만 열 도 ( ) ( )傅說 慇

고 만났고 만 갈( ) ( )武丁 伊尹

도 탕 만났( ) ( )成湯 渭水

여상 도 주 만났는 월( ) ( ) ( ) 呂尙 周 文王

같 러가 도 어느 열 살

늘과 집 삼고 사 에 쳐 거리에

어 다가 곳에 니 다 ( ) 楚

지 다가 사 보고 가에 다다( )長沙

니 망 가에는 원 리가 슬 고 가

가 내리는 사 에는 갈매 가 갈 뿐 었다

쪽 돌 보니 가 우거 고

가 사 보 었다 그곳에

가니 는 사( ) 汨羅水

는 다 주 가 쓰고 죽고

곳 었다

마 감 여 에 가 사 살펴보니

에는 삼 고 그 에( ) 屈三閭

는 만고 월 과 지 가는 그 들( )風月

가 어 었다( ) 路程記

동쪽 벽 에 새 운 어 거늘 그

보니

월 에 경 주 는 간신에게ldquo ( )敗

보고 연경 귀 가다가 에 죽 rdquo

거늘 그 보고 에 거꾸러

통곡 말

우리 연경 간 만 니ldquo ( )燕京

에 지 살 상에 엇 겠는

가 에 고 에 었 니

상에 살 것 가 도 께 지리 rdquo

고 가에 내 가니 울 리가 에 지

사 쳤는지 심 심 것 가

에는 강 주 는 재상 살고 었

니 시 에 과거에 격 여 승상 벼슬 다가 간

신 만 벼슬 그만 고 고 돌 었

다 그러 신 지 가 지 못 여 상

가 못 결 는 상 여 원 니

신 들 그 직간 꺼 다 그 에 도

담과 귀가 강 승상 가 미워 다 강 승상 마

본 에 갔다가 돌 는 에 우편 주( ) ( )本府 右便

에 다가 색 에 어리었는 청룡( ) 酒店

에 지 늘 여 통곡 고

사 는 꿈 꾸었다 마 상 게 생

각 여 새 다리다가 새벽닭 울고

가 달 갔다 가 보니 과연 어 동 가

가에 울고 는지 달 들어 그

고 사 에 어 말

는 어 어 에 어 가ldquo

닭 곳에 우느냐rdquo

니 울 그 고 답 여 말 다

는 경 동 에 사는 언 주 공 들ldquo

니다 께 간신 만 연경 귀 가

시다가 에 죽 사 에 는 닭에

도 에 죽고 니다rdquo

강 승상 말 듣고 크게 낯 변 말

것 웬 말 냐 근 동ldquo ( )老患

못 갔 니 그 사 변 여 런 변

었단 말 가 주 는 신 다 같

에 벼슬 다가 는 가 많 들어 고 돌

는 주 가 게 꿈 에 생각

겠느냐 생각지 못 다 미 지 간 지지

말고 께 가 rdquo

뒷 거리 강 승상 도움 죽 고[ ]

년 학 간고사 대비2013 2 현대고 대비

ECN-0102-2013-001-000076193

고 그 과 결 여 사 가 다 그러 강

승상 에게 울린 상 강 승상 귀 가고

과 헤어 리 승 만 게 다

승 우 다릴 과

들고 략 다 담 원

여 에게 복 고 어 (天

공격 다 담에게 여러 가 복) 子

등 여 다 단신

리쳐 담 사 고 에게 간

후 태후 태 여 지에 고생 지

심과 강 주 여 개 다 헤어

어 니 내 고 담 리 벼

슬에 귀 누리게 다

미상- lsquo ( )-劉忠烈傳

zb23) 위 과 의 서사 조를 비 한 것으 적절하lt gt

않은 것은

보lt gt

믿지 고 결 여 곱

낳 다 곱째 공주 낳 가

리게 다 리 만 고 진 공주는 lsquo rsquo

리공 미 리공 에 키워진다 월

러 과 가 죽 병에 걸 는 승에 는

어 산다고 다 여 들에게 탁

지만 거 리 는다 리 는 과

승 다 승 지 가는 에 많

만 지만 보살 도움 사 도 다

그러 승 신과 결 여 시

들어 주겠다고 다 리 는 그 결

여 들 곱 낳 후에 신

얻게 다 돌 리 는

에 과 상여 만 지만 여 과

살 낸다 훗 리 그 공 우 죽 사

승 도 는 신 다

리-lsquo rsquo-

① 복 결말에 고 다

② 웅 에 탕 고 다

③ 시 겨 내고 귀 누리는lsquo rsquo

보 리 는 월 재 신 다lt gt lsquo rsquo

④ 과 보 리 는lsquo rsquo lt gt lsquo rsquo

도움과 어 신 능 극복 고

⑤ 등 여 시 겪는lsquo rsquo

보 리 는 닌 지lt gt lsquo rsquo

림 시 겪는다

가 각 고 에( ) ( )却說

살 없었다 략 사 들 슬 에 어 lt gt

가에 내 고 가고 싶 가 고 후

워 경

사 들 별 고 없 다니었다 lt

략 얼 말 죽 사 같고 림새가 말gt

니었다 가슴 에 고 등

삼태 헌 에 니 달 가 ( )奇男子

도리어 걸 었 담 만 열 도( )傅說

고 만났고 만 갈( ) ( ) 殷 武丁

도 탕 만났( ) ( ) (伊尹 成湯 渭

여상 도 주 만났는) ( ) ( ) ( )水 呂尙 周 文王

월 같 러가 도 어느 열 살

늘과 집 삼고 사 에 쳐 거리에

어 다가 곳에 니 다 ( ) 楚

지 다가 사 보고 가에 다다( )長沙

니 망 가에는 원 리가 슬 고 가

가 내리는 사 에는 갈매 가 갈 뿐 었다

쪽 돌 보니 가 우거 고

가 사 보 었다 그곳에

가니 는 사( ) 汨羅水

는 다 주 가 쓰고 죽고

곳 었다

에는 강 주 는 재상 살고( )

었 니 시 에 과거에 격 여 승상 벼슬 다

가 간신 만 벼슬 그만 고 고 돌

었다 략 강 승상 마 본 에 갔다가 돌 lt gt ( )本府

는 에 우편 주 에 다가 색( ) ( ) 右便 酒店

에 어리었는 청룡 에 지

늘 여 통곡 고 사 는 꿈 꾸

었다 마 상 게 생각 여 새 다리다

가 새벽닭 울고 달 갔다 가

보니 과연 어 동 가 가에 울고 는지

달 들어 그 고 사 에

어 말

는 어 어 에 어 가ldquo

닭 곳에 우느냐rdquo

니 울 그 고 답 여 말 다 lt

략gt

년 학 간고사 대비2013 2 현대고 대비

ECN-0102-2013-001-000076193

생각 여 가 고 시 는ldquo ( )大人

상에 다시없는 니다 살 엇 겠습니

에 돌 가시고

가에 돌 가 니 살 마 없습니

다 략 어 없어 강 승상 가니rdquo lt gt

그곳 월계 었다

다 강 승상에게는 들 없고 다만 만( )

었다 가 낳 에 가 색

타고 내 에게 말

는 니다 미원 과ldquo ( )紫微垣

연 맺고 었는 께 강 집( )緣分

보내 에 니 게 여겨 주십시

rdquo

거늘 미 가운 낳 니 가

고 거동 단 다 시 짓 쓰 고

는 없었 니 여 가운( ) 音律

지 는 짝 룰 만 사 없었다 가 사

여 사 감 게 고 지 못 고 염 는 만다

다가 당에 거 고 식같 러

내니 고귀 상 루 말 다 어 ( )相

울 도 다 귀 사 없고 ( )富貴爵祿

웅 걸 만고 었다 승상 매우 뻐 내

당 들어가 에게 사 니 역( ) 內堂

시 매우 거워 말 다

도 마 사 는 승상께ldquo

그 게 말 시니 상 여러 말 지 말고 사

도 시다 략 시 택 여rdquo lt gt

니 다운 신 과 신 습 늘에 죄

짓고 간 상에 내 신 다

다 내고 들어가 사 살펴보니

고 것 는 다 말 어 고

는 다 어 신 에 ( )新房

에 신 과 신 가 평생 연 맺었( )緣分

니 사 주고 말 어떻게 다 헤 릴

어떻게 다 리 지낸 후에 튿 승

상 니 승상 거운 마 지 못

( ) 듯 월 러 생 열다 살

었다 에 승상 어진 사 얻고 만 에 근심

없었 다만 주 가 간신

에 죽 것 생각 마 곧 어

곤 다 그 에 주 원통

어 없 고 여 시 가 거늘 략 lt gt

략 거리

강 승상 에게 상 리지만 여움

사 귀 가게 다 강 승상 몸 는

연 과 헤어 리 다

마 각 생 강 승상 집 쪽( )

늘 보고 없 가 신 신 생각 니

없고 어 없었다 는 어떻게 도리가 없다

여 산 에 들어가 리 고 어 도 닦

고 다 그 산 보고 가

다가 곳에 다다 니 에 큰 산 었다 많

우리 골짜 가 늘 는 가운 색

에 고 갖가지 가 짝 어 ( )花草

었다 략 주 보니 lt gt ( ) (一柱門 黃

산 룡사 어 었다) lsquo rsquo 金大字

산 들어가 고승 다 그( ) ( ) 山門 高僧

거동 보니 눈 눈 듯 고

변 같 귀는 어 에 늘어 니( ) 白邊

맑고 어 골격과 신 평 니었

다 염주 에 걸고 짚고 포 ( )六環杖

삼에 어진 쓰고 생 보고 말

승 연 여 상공 시는 동 에ldquo

가 맞 지 못 니 승 십시 rdquo

생 크게 말 다

생 가 여 어 고ldquo

없 다니다가 우연 곳에 사 만 것

그 시 생 어떻게 고 습니

rdquo

승 답 여 말

어 산 승 에ldquo ( ) ( )南岳 衡山

시어 승에게 탁 내 낮 시경에 경 lsquo 12

동 에 사는 심 들 가 것 니 내쫓

지 말고 습니다 마 승rsquo

다가 상공 림새 보니 경 사 에 보

습니다rdquo

생 그 말 듣고 편 고 편( )

슬 승 들어가니 여러 승 들

가워 다 승 에 들어가

후에 그 편 니 곳 경 었다 상( ) 仙境

고 신 편 다 후 는 승과

께 병 도 탐 고 경도 게( )兵書

게 었다 게 니 지 에 가객 ( ) ( )大明天地 佳客

년 학 간고사 대비2013 2 현대고 대비

ECN-0102-2013-001-000076193

없고 산 에 리 만 본 ( ) 廣德山

신 상 사 살 는 만

우고 늘 월 신 과 늘 ( )日月聖神

산 신 들 다 니 그 재( ) 名山神靈

주 민 누가 당 겠는가 낮 공

zb24) 다 에 해당하는 내 으 적절하( ) 않은 것은

① 강 티 통 당시 능 다

② 상계 지상계 경 는 원 계 드러

③ 실에 어 없는 실 가 타 는

④ 뛰어 재주 어 가진 고

등 다

⑤ 가 직 개 여 평가 내리는

편집 평 타 다lsquo rsquo

가 본격 가 동 것 지( )

다 단 상 에2003 lsquo rsquo

들어가 드럼 연주 다 취미 생 달리

들었다는 보 우 가 들ldquo

어 틱 린 도 다 고 말 다rdquo

경 는 가 망 없( ) lsquo

티 원 고 답 다 신과 같 시각rsquo

는 습 상상 만 도 감동

다 시각 연주 동시에

열 상 는

티 원 그런 열 경 럽다는 것 다

다 역시 엄청 다 본( )

에 복 들

고쳐 가고 다 신 에 얼

마 지는 고 리가 는 지도 생님

가 훈 고 많 고쳐 다

고 말 다

그러 직도 에 지 는 다 그는

체격 지 못 게 가 큰 만

체 운동 훈 과 께 체 늘 동 50

는 게 고 말 다

에게는 꿈 다 통 누 가( )

주겠다는 것 그 꿈 다 신 극복 는

과 에 큰 경험 들도 느 게

주고 싶다는 것 다

마 슬 마다( ) ldquo 통

낼 었 것 럼 고통 는 사 들

고 겠다 고rdquo

말 다 달 루 첫 낸 lsquo rsquo

첫 드 심 집에 는 리듬 드 2

루 에 도 보고 싶다 집 에는 직(RampB) 3 4

사 곡 도 보 고 싶다고 포 다middot

zb25) 에서 가장 유사한 의 를 닌 어를lt gt

찾아 쓰

lt gt

나는 이제 너에게도 픔을 주겠다

사랑 다 소 한 픔을 주겠다

겨 거리에서 개 놓고

살아 추위 떨고 있는 할 니에게

값을 으 서 뻐하던 너를 위하여

나는 픔의 평등한 얼 을 여 주겠다

내가 어둠 속에서 너를 를 때

단 한 도 평등하게 어주 않은

가마니에 덮인 동사자가

다 얼어 죽을 때

가마니 한 장조차 덮어주 않은

한 너의 사랑을 위해

흘릴 르는 너의 눈 을 위해

나는 너에게 이제 너에게도 다림을 주겠다

지 울 포동 여고 생들17

틈 없 가득 체 에 맑 울

다 죽 듣 생들 사 에

연 는 탄 다 객들 도 는lsquo rsquo

가 보 주 공 맹 가 운 는

단 그룹사운드 루 보컬 맡고 는lsquo rsquo

시각 지 었다17 1

근 다만과 가 거lsquo rsquo lsquo

꿈 고 퇴 내가 다rsquo

간 간에 지 지 연 생들 짧lsquo rsquo lsquo rsquo

가 운 듯 리에 어

연 다 내 사 고 퇴lsquo rsquo

과 루 들 결 다시 돌lsquo rsquo

들 고 사 들 에 당당

것 니다 내 태어

볼 없었 크고 열여

년 학 간고사 대비2013 2 현대고 대비

ECN-0102-2013-001-000076193

에도 고 시 얻지 못 다

감지 없는 시각 상태 다

신 지에 고 상 원망 도

단다 어느 가 에 시각 에 ldquo

어 그런 듣고 다 보니 내가 게 lsquo

살 는지 도 눈 고 싶rsquo lsquohelliphellip

보 는 생각만 들 고 그 가 들에게rsquo

도 내고 들도 고 많 었죠 들 rdquo

었 지 새 는 에 쑥 러운 색

어났다

생에 것 단연 었다lsquo rsquo

공연에 거 꿈lsquo rsquo

는 다 특 가사 갑게 는 운 lsquo

벽 에 당당 마주 어 언 가 그 벽

고 늘 어 거운 상도

없죠 내 삶 에 웃 그 께

는 다고 다rsquo

들었 그냥 런 도 고만 여ldquo lsquo rsquo

겼죠 그런 꾸 가사 미 새 다 보

니 통 는 가사 는 생각 들 고 (

가 게는 시각 는 생각 들고 들) ( )

마다 듣고 큰 얻었어 rdquo

에 진지 게 가에 미 가

zb26) 의 에 들어갈 말 적절한 것은lt gt ~

lt gt

난 난 꿈이 있었죠

고 찢겨 남 하여도

내 가 히 과 같이 간 했던 꿈

혹 때 누 가가 뜻 를 비 음

내 등 뒤에 흘릴 때도

난 참아야 했죠 참을 있었죠

날을 위해

늘 걱정하듯 말하죠

헛된 꿈은 독이라고

세상은 끝이 정해 책처럼

이 돌이킬 없는

현 이라고 helliphellip

래 난 난 꿈이 있어

꿈을 믿어

나를 켜

저 차갑게 서 있는 이란 앞에

당당히 마주칠 있어

출처 가 거위의 꿈 작사 이적 작곡 동률- lsquo rsquo ( )

① ② ③ ④ ⑤

가 떴다는 들 만 지만( ) lsquo rsquo

늘 겸 다 에 주 연 우승 지 간에도 3

단 생님께 만 지 고 고 만ldquo rdquo

큼 늘 겸 신 계 가

고 다

에게는 꿈 다 통 누 가

주겠다는 것 그 꿈 다 신 극복 는 과

에 큰 경험 들도 느 게 주

고 싶다는 것 다

슬 마다 통 낼ldquo

었 것 럼 고통 는 사 들

고 겠다 고rdquo

말 다 달 루 첫 낸lsquo rsquo

첫 드 심 집에 는 리듬 2

루 에 도 보고 싶다(RampB) 집 에는 직34

사 곡 도 보 고 싶다고 포 다

미 는( ) (26) 어 헤헤헤 웃다가 어ldquo rdquo

허허허 웃었다ldquo rdquo ldquo rdquo 같 도 고

상 다 는 같 도 다( ) 壯丁 킹 들lsquo

다 는 역도 보 그 다 지만 그는rsquo

뷰에 지 다 운동만 지 ldquo

것 지 간에 여러 사 도 역rdquo helliphellip

었다 그런 엇 그 마 움직 는지 보 쯤

지 담 사 다 훈 없어 그는 티

지 림 었다 태 다 갔다 는 습

마 집 럼 편 게 보 다

주말에는 주 엇 보내

주말에도 별 주 에 청ldquo

고 에 가고 도 쳐

에 듣고 보 에 갈 가 별 없

어 산 시 게 고 들어 2002

거 매 여 지냅니다 시 과 지훈 rdquo

다 근 간 과 진실 그리고 싶어( )

가 다 근에게 그것 진리 다 거 다 없

거 고 다 없 는 것 진리

다 근 진리는 후 쪽 었다 신산( )辛酸 삶

었 질곡( )桎梏 역사 에 지냈 가

눈에 든 것 료 단 료 게 보

것 었다 그것 그 에 겨우겨우

슬 슬 생 어가는 간들 었다

리 과 단 리 고리에 검 마

없 거리 돌

상 것 없는 등 근에게 상

과 진실 엄 ( )儼存 다는 사실 리는 가

실 고 가 과 역경 에 도 근 내 포

없었 후 보루( )堡壘 다 도 365

도 간 근 여

시 것 다

년 학 간고사 대비2013 2 현대고 대비

ECN-0102-2013-001-000076193

다 공주 그림 가 근 경- ( ) ldquo rdquo(

2009)

zb27) 작가의 주 적인 각이 드러난 것은~

① ② ③ ④ ⑤

가 신 지에 고 상 원망( )

도 단다 어느 가 에 시각 에 ldquo

어 그런 듣고 다 보니 내가 lsquo

게 살 는지 도 눈 고 싶rsquo lsquohelliphellip

보 는 생각만 들 고 그 가 들에게rsquo

도 내고 들도 고 많 었죠 들었rdquo

지 새 는 에 쑥쓰러운 색

어났다 략 [ ]

경 는 가 망 없 티lsquo

원 고 답 다 신과 같 시각rsquo

는 습 상상 만 도 감동

다 시각 연주 동시에

열 상 는 티

원 그런 열 경 럽다는 것 다 략 [ ]

슬 마다 통 낼ldquo

었 것 럼 고통 는 사 들

고 겠다 고rdquo

말 다 달 루 첫 낸 lsquo rsquo

첫 드 심 집에 는 리듬 2

루 에 도 보고 싶다 집 에는 직(RampB) 3 4

사 곡 도 보 고 싶다고 포 다

식 누 가-

고 싶어

다 역도 미 담 고 사( )

질 주말에는 주 엇 보내[ 1]

답 주말에도 별 주 에[ ] ldquo

청 고 에 가고 도 쳐

에 듣고 보 에 갈 가 별

없어 rdquo

질 계 고 슬슬 도 는 것 닙니[ 2]

답 다 들 눈 에 보 고 뿐 보[ ] ldquo

다 열심 고 어 상에 도 들지만 상

지키는 것 들다고 에 도달

그것 지키 훨 많 rdquo

질 들 살 고 리 는[ 3]

거운 들 체 리느 는다

답 가 고 게 체 어[ ] ldquo ( ) 級

느 도 계가 니 살 는 것도 고역 지만

살 우는 것 들어 는 살

체 리 고 어도 어도 실 갔다

쑥 어 rdquo

질 거리에 슷 연 여 들[ 4]

보는 간 상 지

답 상 다 체 게 리지 못[ ] ldquo

거 주변에 는 그 거 누 보지

못 고 뻐지고 싶 에 체 리는 에

타 워 지만 는 어울 는 것보다 는

시간 운동만 는 건 니에 사복 lsquo rsquo

고 사복 는 말에 들 웃지만 늘 운동복

고 지내니 사러 갈 도 어 rdquo

질 역도가 말 단 식 운동 니[ 5]

답 가 내는 만 클 업 보[ ] ldquo

그러니 만 쓰는 식 운동 니다

만 다고 거운 것 들 는 건 니거든 연

도 고 가지 동 에 도 여러 가지

복 들

보식 역도 여 미-

zb28) 가 에 대한 설 으( ) 않은 것은

① 시각 우 지 시 에 지

고 망 가는 태도 달 고 다

② 언어 과 언어 복 사 여

담 내 생각 게 는 가

③ 직 감 그 마 것

럼 생생 게 느껴지는 과 주고 간 내

없 리 어 억 게 다

④ 담 내 식 리 여 담 삶 습

과 가 시 여 독 에게 감동과 훈 다

⑤ 직 진 담 직 누

지 못 는 독 에게 생생 상 달 주고

담 욱 게 다

zb29) 나 의 각 의 의도를 설 한 것으 적절하( ) 않

년 학 간고사 대비2013 2 현대고 대비

ECN-0102-2013-001-000076193

은 것은

① 질 담 상 보여 주 것 다1

② 질 담 과 그에 삶 태도 보여2

주 것 다

③ 질 역도 겪는 어 움에 역도3

과 것 다

④ 질 같 연 여 갖는 고민 는지 말4

주 는 것 다

⑤ 질 역도가 과 고 운동 는 것5

담 가 말 주 는 것 다

가 만진 것 다( ) 3

감 달 다고 다 억 에( ) 音感

지워 지만 당시 청 탁 리도

다고 다 드럼 웠다 4

에 갈 마다 드럼 는 리가 신 게 들

다고 다 눈 볼 가 없 니 엔ldquo

는 는 님 틱 에 여 주

다 드럼과 연 맺 과 들 주었다rdquo

식 누 가-

고 싶어

역( ) 도가 말 단 식 운동 니

가 내는 만 클 업에 보ldquo

그러니 만 쓰는 식 운동 니다 만

다고 거운 것 들 는 건 니거든 연

도 고 가지 동 에 도 여러 가지 복

들 시 는 상 상

드는 상 에 맞춰 실 에 는 여러

펼쳐집니다rdquo

략( )

늘 에 는 어 만 것 같

가 에 사 고 사 사ldquo

겠어 든 에 가 경 만 고

울 는 사 겠어 rdquo

보식 역도 여 미-

다 가 운 는 어 어( ) ldquo rdquohelliphellip

월 새벽 시 태 없 거웠고1965 5 6 1

는 없 그 병원에 퇴원 집

가는 마지막 마 고 마 내 거 다

가 죽 간신 에 실 다 사는 어느5 lsquo

가 죽 는 말 가 식 다 신rsquo

상에 각 시키는( )刻印 에 실

어느 가는 후 민 가가 근 었다lsquo rsquo

는 간 과 진실 그 다는( ) ldquo

에 단 평 견 가지고 다 내

가 그리는 간상 단 고 다 지 다 는 그들

가 에 는 평 지 니 그리고 어린

들 미지 겨 그린다rdquo

마 근 간 과 진실 그리고 싶어( )

가 다 근에게 그것 진리 다 거 다 없

거 고 다 없 는 것 진리

다 근 진리는 후 쪽 었다 신산(辛酸 삶)

었 질곡(桎梏 역사 에 지냈)

가 눈에 든 것 료 단 료 게

보 것 었다 그것 그 에 겨우겨우

슬 슬 생 어가는 간들 었

다 리 과 단 리 고리에 검

마 없 거리 돌

상 것 없는 등 근에게 상에

과 진실 엄 다는 사실 리는 가( )儼存

실 고 가 과 역경 에 도 근 내

포 없었 후 보루(堡壘 다 도)

도 간 근365

여 시 것 다

월 강원도 림리에( ) 1914 2 21

삼 독 태어났다 어 근 복

그것 그리 가지 못 다 근 곱 살

지는 산 산업에 실 고 답마 에 내

갔다 근 그림 럼 쫓 다니 가 시 것

다 상 진 것도 가 었다

러 가 에도 고 근 가 꿈꾸었다 근

가 꿈꾸게 것 보통 업

원색도1926 만lsquo rsquo 었다

공주 그림 가 근 경-

zb30) 에 대한 설 가장 른 것은~

① 역도가 과 운동 도 질

② 리는 는 다 lsquo rsquo

③ 들었지만 그럭 럭 는 다 lsquo rsquo

④ 가 게 보 시 말 다

⑤ 보 병 는 지 상 lsquo rsquo

는 말 다

년 학 간고사 대비2013 2 현대고 대비

ECN-0102-2013-001-000076193

시간 많지 다 청량리 생 병원

마지막 상 경 릿 게 들어 다 그 는 십

만 큰 가 상 말 다

지 못 들 마 갈 고 돗

도시민들 싹 싹 탔다 가 시

월에 병원에 원 가 폐 진 몸도4 ( )疲弊

갈 미 지 못 고 었다 가는 얼마( ) 解渴

지 생 에 생각 가

마감 는 신 평생 십 만에

가 과 많 닮 다고 생각 지는

가 운 는 어 어ldquo rdquo 1965helliphellip

월 새벽 시 태 없 거웠고 는5 6 1

없 그 병원에 퇴원 집 가

는 마지막 마 고 마 내 거 다 가

죽 간신 에 실 다 사는 어느 가5 lsquo

죽 는 말 가 식 다 신rsquo

상에 각 시키는 에 실 어느( ) lsquo刻印

가는 후 민 가가 근 었다rsquo

ldquo 는 간 과 진실 그 다는 에

단 평 견 가지고 다 내가 그

리는 간상 단 고 다 지 다 는 그들 가

에 는 평 지 니 그리고 어린 들

미지 겨 그린다rdquo

근 간 과 진실 그리고 싶어 가

다 근에게 그것 진리 다 거 다 없 거

고 다 없 는 것 진리다

근 진리는 후 쪽 었다 신산 삶 ( )辛酸

었 질곡 역사 에 지냈 가 눈에( )桎梏

든 것 료 단 료 게 보 것

었다 그것 그 에 겨우겨우 슬

슬 생 어가는 간들 었다 리

과 단 리 고리에 검 마

없 거리 돌 상

것 없는 등 근에게 상에 과 진실

엄 다는 사실 리는 가 실( )儼存

고 가 과 역경 에 도 근 내 포 없었

후 보루 다 도 도( ) 365堡壘

간 근 여 시 것

간에 지닌 가 근 1914 2

월 강원도 림리에 삼 독21

태어났다 어 근 복 그것 그리

가지 못 다 근 곱 살 지는 산

사업에 실 고 답마 에 내 갔다 근

그림 럼 쫓 다니 가 시 것 다 상

진 것도 가 었다 러 가 에도

고 근 가 꿈꾸었다 근 가 꿈꾸게

것 보통 업 원색1926

도 만 었다lsquo rsquo

그림 가 근 경 공주- ldquo rdquo ( 2009)

zb31) 다음 이 같은 의 성 소에 해당하 않은

것은

사건 평① ② ③

④ 주 ⑤ 경

가 운 는 어 어ldquo rdquo 1965helliphellip

월 새벽 시 태 없 거웠고 는5 6 1

없 그 병원에 퇴원 집 가

는 마지막 마 고 마 내 거 다 가

죽 간신 에 실 다 사는 어느 가5 lsquo

죽 는 말 가 식 다 신rsquo

상에 각 시키는 에 실 어느( ) lsquo刻印

가는 후 민 가가 근 었다rsquo

는 간 과 진실 그 다는 에ldquo

단 평 견 가지고 다 내가 그

리는 간상 단 고 다 지 다 는 그들 가

에 는 평 지 니 그리고 어린 들

미지 겨 그린다rdquo

근 간 과 진실 그리고 싶어 가

다 근에게 그것 진리 다 거 다 없 거

고 다 없 는 것 진리다

근 진리는 후 쪽 었다 신산 삶 ( )辛酸

었 질곡 역사 에 지냈 가 눈에( )桎梏

든 것 료 단 료 게 보 것

었다 그것 그 에 겨우겨우 슬

슬 생 어가는 간들 었다 리

과 단 리 고리에 검 마

없 거리 돌 상

것 없는 등 근에게 상에 과 진실

엄 다는 사실 리는 가 실( )儼存

고 가 과 역경 에 도 근 내 포 없었

후 보루 다 도 도( ) 365堡壘

간 근 여 시 것

간에 지닌 가 근 1914 2

월 강원도 림리에 삼 독21

태어났다 어 근 복 그것 그리

가지 못 다 근 곱 살 지는 산

사업에 실 고 답마 에 내 갔다 근

그림 럼 쫓 다니 가 시 것 다 상

진 것도 가 었다 러 가 에도

고 근 가 꿈꾸었다 근 가 꿈꾸게

것 보통 업 원색1926

도 만 었다lsquo rsquo

공주 그림 가 근 경- ldquo rdquo ( 2009)

년 학 간고사 대비2013 2 현대고 대비

ECN-0102-2013-001-000076193

zb32) 위 을 작성하는 과정에서 되어 활 된 자

어 것은

신 사 료① 연보②

고③ ④ 들과 담

⑤ 에 평

는 간 과 진실 그 다는 에ldquo

단 평 견 가지고 다 내가 그

리는 간상 단 고 다 지 다 는 그들 가

에 는 평 지 니 그리고 어린 들

미지 겨 그린다rdquo

근 간 과 진실 그리고 싶어 가

다 근에게 그것 진리 다 거 다 없 거

고 다 없 는 것 진리다

근 진리는 후 쪽 었다 신산 삶 ( )辛酸

었 질곡 역사 에 지냈 가( )桎梏

눈에 든 것 료 단 료 게 보

것 었다 그것 그 에 겨우겨우

슬 슬 생 어가는 간들 었다

리 과 단 리 고리에 검 마

없 거리 돌 상

것 없는 등 근에게 상에 과

진실 엄 다는 사실 리는 가 실( )儼存

고 가 과 역경 에 도 근 내 포

없었 후 보루 다 도 도( ) 365堡壘

간 근 여 시

것 다

간에 지닌 가 근 1914 2

월 강원도 림리에 삼 독21

태어났다 어 근 복 그것 그리

가지 못 다 근 곱 살 지는 산

사업에 실 고 답마 에 내 갔다 근

그림 럼 쫓 다니 가 시 것 다 상

진 것도 가 었다 러 가 에도

고 근 가 꿈꾸었다 근 가 꿈꾸게

것 보통 업 원색1926

도 만 었다lsquo rsquo

질 루 마 가 도 린다 경건

움 느껴지는 경 다 훗 근 그림에

과 는 거 것( )裸木

만 간과 연 엮어 가는 경건 움lsquo rsquo

니었

같 가가 고 싶었 근에게 그 꿈에 다

가가는 지 다 다 가 지망생들 규 미

상 에 진 고

에 지만 근 다 다 근

미 에 운 것 보통 시 미 시간

다 그런 그에게 없는 연습 가가

통 다 가 귀 시 지 도

얻는 뛸 듯 뻤지만 마 도 가 에

듯 는 었 에 어린 근 주 에

에 그림 그리고 지우고 복( )粉板

시간 가는 게 루 보냈다

근 그 갈 가가 것 열여( )渴求

었 다가 미1932 lsquo rsquo ( lsquo

미 에 다 다는 고 마rsquo) lsquo rsquo

가 근 집 고도 지는 시골 경

그린 그림 다 후 근 에 1943 22

지 미 에 그림 고

에 걸쳐 다 미 근 가

동 는 었다

공주 그림 가 근 경- ldquo rdquo ( 2009)

zb33) 위 의 내 과 일치하는 것은

가 근 가 꿈 포 다①

근 당 가들과 께 에 다②

살 근 가 걷20③

게 었다

④ 만 통 근 역경 겨내는lsquo rsquo

느 다

⑤ 근 간 과 진실 그리 에 그 에

드러 는 간상 단 다

계 시 주 근 건강

걸었다 신 과 간에 상 다 건강

신 는 눈에도 다 근 쪽 눈 뿌 게

보 지 과에 다 다 시 지지 고 결

내 었다 시 지만 마 막막

다 늦어 결 근 쪽 눈 고 말 다

쪽 눈 근에게는 쪽 눈 었고

계 었다 그 근 는 여 그lsquo rsquo

다 근 에 같 그림 그 었다1950

시 그림 는 여 쪽lsquo rsquo

고 어 마주 고 는 그림1963

여 과 동 다 마 복

그린 듯 눈 내리 새 게 다 지

사 다 근 게 복 것

복 상과 타 는 근 상

가 떳떳 단 었고 근 그리고

간 과 진실 에 다가가 가 근다

운 었다 근 신에게 당당 지 그리고

그 다 근 그림에 단 복 보다

년 학 간고사 대비2013 2 현대고 대비

ECN-0102-2013-001-000076193

태 도 그리고 극 보다 과

얻 여 었다 과 통

근 그리고 는 재 고 에 질

만들고 특 것 다

공주 그림 가 근 경- ldquo rdquo( 2009)

zb34) 의 이유에 대해 추 한 것으 적절하 않은 것

상과 타 시도①

보다 과 얻②

근 신에게 당당 지③

④ 간 과 진실 에 다가

⑤ 태 도 얻

근 가가 었지만 그 다니 가

럼 어지지 다 복과 쟁 거쳐 시

는 가 근에게 생계 사 에

운 사 다 에 키에 건( ) 178cm死鬪

체 근 에 동 역 업( )荷役

가 생계 다 쟁

에는 동에 운 상우 주 미

죄 사 에 그림 그리는 시 다 그곳에

에 동 역 업 것에

결 것 럼 보 다 지만 그런 것만도

니었다 그림 그리는 고는 지만 매 근

는 극 간 과 별 없는 경 리 그림

벽에 그리는 것 었다 우도 리 없었다 근

트 는 우 그림 그 다 생

계 그림 단 것 다

후 근 지 신 계 리에 미

엑 리 겼다 근 곳에

건 사 크 에 미 들 ( )

상 상 그 다 근 갖 다 겪

냈다 그리고 결 그 돈

신동에 어 사리 집 마 다 마 ㄷ

루 심 쪽에는 과 엌 쪽에는 건

었다 건 주고 근 가 에

여 살 다 심 에는 지 집어

쓰고 지만 곳 근 가 에게 러웠

보 리 다 근 과 마루 업실 삼 그림

그 다 신동 마루는 근 그림에 등 는 lsquo rsquo

같 상들 지 다 시 고

에 들 폐허가

가 업실 었다

공주 그림 가 근 경- ldquo rdquo( 2009)

zb35) 위 에 대한 설 으 적절한 것은

업 시 여 훈과 감동 다①

에 주 평 드러 다②

사 사 등 식 과 ③

④ 다 근거 시 여 삶에

⑤ 살 시 사 경 께 여

습 시 다

가 시간 많지 다 청량리 생 병원( )

마지막 상 경 릿 게 들어 다 그 는

십 만 큰 가 상 말 다

지 못 들 마 갈 고 돗

도시민들 싹 싹 탔다 가 시

월에 병원에 원4 가 폐( )疲弊

진 몸도 갈 미 지 못 고 었다( )解渴 가는

얼마 지 생 에 생각

가 마감 는 신 평생 십 만에

가 과 많 닮 다고 생각 지는

가 운 는 어 어( ) ldquo rdquohelliphellip

월 새벽 시1965 5 6 1 태 없 거웠고

는 없 그 병원에 퇴원 집

가는 마지막 마 고 마 내 거 다

가 죽 간신 에 실 다 사는 어느5 lsquo

가 죽 는 말 가 식 다 신rsquo

상에 각 시키는 에 실( )刻印

어느 가는 후 민 가가 근 었다lsquo rsquo

다 는 간 과 진실 그 다는( ) ldquo

에 단 평 견 가지고 다 내

가 그리는 간상 단 고 다 지 다 는 가

에 는 평 지 니 그리고 어린 들

미지 겨 그린다rdquo

근 간 과 진실 그리고 싶어( )

가 다 근에게 그것 진리 다 거 다 없

년 학 간고사 대비2013 2 현대고 대비

ECN-0102-2013-001-000076193

거 고 다 없 는 것 진리

다 근 진리는 후 쪽 었다 신산( )辛酸 삶

었 질곡 역사 에 지냈( )桎梏

가 눈에 든 것 료 단 료 게 보

것 었다 그것 그 에 겨우겨우

슬 슬 생 어가는 간들 었다

리 과 단 리 고리에 검

마 없 거리 돌

상 것 없는 등 근에게 상에

과 진실 엄 다는 사실 리는 가 실( )儼存

고 가 과 역경 에 도 근 내 포

없었 후 보루 다( ) 堡壘 도 365

도 간 근 여

시 것 다

마 같 가가 고 싶었 근에게 그 꿈( )

에 다가가는 지 다 다 가 지망생들

규 미 상 에 진 고

에 지만 근 다 다 근

미 에 운 것 보통 시 미 시간

다 그런 그에게 없는 연습 가가

통 다 가 귀 시 지 도

얻는 뛸 듯 뻤지만 마 도 (

는 었 에 어린 근 주 에)

에 그림 그리고 지우고( )粉板

복 시간 가는 게 루 보냈다

zb36) 전 의 성 소가 아닌 것을 고르

① 평 ② 사건 ③ 경

④ ⑤ 훈

늘 지 상에 살고 는 사 들 억 도가10

고 그리 지 통 고 는 사 들( )知的

그보다 훨 많 억 도는 고 지 20

통 다 그런 지 고 2500

그리 간 보는 과 사 에

매우 달 뿐만 니 과 에 도 극

루고 었다 미 운 그런 들

살고 는 동 과 사 들 사고 식에

큰 가 다는 다

고 그리 들 우주 개별 고 독립

사 들 생각 지만 고 들 우

주 연 질 간주 다 같( ) 看做

각 도 들에게는 연 질

었지만 그리 들에게는 미 들 결 었

다 고 과 그리 들 사 같

는 동 과 사 에 도 견 다

지심리 미 마 드 겐트 는

살 들에 에 지 다

연 동 과 상 다 과 같 실험

다 크 만든 미드 도 보

여 주고 그 상 닥 고 주었다lsquo (Dax)rsquo

실 닥 는 재 지 는 것 실험 가lsquo rsquo

만들어 낸 다 그런 다 개 다 체 보

여 주었는 는 미드 지만 틱

만들었고 다 는 재료는 크 지만

달 다 그러고 어 것 닥 지 사 들에게 고 lsquo rsquo

게 니 들 주 같 고 는

체 택 고 동 들 같 재료 만들어진 체

택 다 러 는 심지어 살짜리

들에게 도 타났다 것 곧 과 동

다 상 보고 다는 것 미 다

개별 사 보고 고 동 연 질 보

고 는 것 다

동 들 주변 상 에 맞 어 동 고

에 다 사 들 태도 동에 보다 많

주 울 다 동 가 미시간 에

에 경험 다 그는 미식

경 보러 가게 었는 경 체는 매우 재미 었

주변 들 동에 질 다 그 는

들 계 어 상태 경 다

어 들 에 에 그 시 가 계 가

진 것 다 상 살펴 는 말 들 lsquo rsquo

에 그는 에 시 어 도 뒷사

생각 곧 다시 곤 것 다 그런 그에게 뒷

사 고 지 는 들 동 럼

어 웠다

생각 지도 리 드 니 벳-

zb37) 다음 위 의 내 전개 으 만 인lt gt

것은

lt gt

대조의 통해 대상이 닌 특성을 설 하고 있다

일화를 제 하여 자 의 주장을 뒷 침하고 있다

유추의 을 사 하여 독자의 의해를 돕고 있다

대상이 형성되는 과정을 간적 서에 따라 서 하고 있

① ②

③ ④

년 학 간고사 대비2013 2 현대고 대비

ECN-0102-2013-001-000076193

가 우리가 말 고 쓰는 든 단어가 사 에 는( )

것 니다 사 격에 가 는 지만

어 사 과 같 특별 는 사 니lsquo rsquo

단어 격 보 단어가 사 에

등재 어 다 리 리 사 는 단어 도 그

것 시 사 는 어 고 사 에

격 보 것 니다

러 얼 은 사전에 를 있는가 이에 대한 답lsquo rsquo

은 얼 이 유행어인가 아닌가에 따라 갈라 다 이 단어lsquo rsquo

는 년 어 자 에 랐고 쓰이고 있으2002 lsquo rsquo

유행어라고 하 에는 생 이 다 런데 계속

을 유 하 서 사전에 등재될 자격을 획득할 것인가 이

에 대한 답을 내리 는 히 어 다

여 서 가 를 고 해 볼 있다 첫 는 이 단어

를 써야 할 필 가 속적으 있는가 하는 점이다

상주의 열풍에 휩 인 사회 위 에 편 해서 퍼 말

이 얼 인데 과연 런 위 가 속될 것인가 이에lsquo rsquo

대해 필자의 생각은 정적이다 사회 위 가 뀌

런 말을 쓸 일이 없어 것이다

다음은 단어의 성이다 단어의 성이 사회적으 거

감이 없으 계속 사 될 가능성이 높다 런 에서

얼 은 좋은 조건이 아니다 익히 알 졌듯이 이lsquo rsquo

말은 얼 과 청소년층에서 속어 사 하는 이 결합lsquo rsquo lsquo rsquo

된 말이다 얼 에서 얼 을 리하는 조어 도 lsquo rsquo lsquo -rsquo

어에서는 매 낯선 이다 이것만으 도 거 감을 갖

는 사람들이 있다 더 나 속어 결합한 말이다 얼 lsquo rsquo

이 널리 퍼졌다 해도 은 여전히 청소년층의 속어lsquo rsquo

남아 있다 속어는 자연 럽게 아 자리에서나 쓰 에는

담 러 말이다 러한 담을 하고 사

역을 넓혀 가는 속어도 없 는 않다 특히 얼 은 lsquo rsquo

에도 종종 등장한다 만큼 거 감이 많이 희석되었다

고 할 있다 러나 일상의 자연 러 대화에서도 거

리낌 없이 등장하는가 게 는 되 않았다고 생

각한다

얼 이 유사어인 쌈 등을 만들어 내고lsquo rsquo lsquo rsquo

있으니 살아남을 있을 것이라고 는 견해도 있을 것

이다 러나 간이 나 서 유사어를 포함하여 든

말이 사라 사 는 많다 유사어가 많다는 것이 생 을

유 할 있는 절대적인 조건은 아니다

나 언젠가 터 사람들은 어느 단에서 얼 이 가장( )

쁜 사람을 가리켜 얼 이라고 르고 있다 이 얼lsquo rsquo lsquo rsquo

이라는 단어가 최근 어사전에 라 항간에 논란이 일고

있다 아닌 게 아니라 얼 은 유행어처럼 인다 생 lsquo rsquo

도 리 래되 않은 것 같고 언제 사라 도 알

없다 게다가 젊은이들 사이에서 주 쓰일 뿐이다 이런

단어를 사전에 는다는 게 하 이 없어 이 도

한다

러나 속단은 이다 차근차근 따져 볼 일이다

선 얼 이 일 적 유행어인 아닌 주의 게 들여다lsquo rsquo

볼 필 가 있다 유행어란 유행에 따라 빠르게 유포되었

다가 단 간 내에 소 되는 단어나 를 가리킨다

얼 은 인터넷을 통해 속히 퍼 말이다 하 만 일lsquo rsquo

적인 유행어처럼 단 간 내에 사라 않았을 뿐 아니라

현재 도 잦은 빈도 사 되고 있고 앞으 도 상당

간 사 될 것으 측된다 한 언 재단의 뉴 검 lsquo rsquo

색 사이트에 따르 얼 은 년 에 처음 나타난lsquo rsquo 2001

이후 꾸 히 사 되고 있다

이 같은 사 빈도는 얼 이 일 적 유행어 는 현lsquo rsquo

저히 다르다는 것을 여 다 장 간의 생존 만으 도

얼 은 이 한 어의 어휘 에 를 자격을 얻었다lsquo rsquo

고 할 있다 더 이 이라는 비 적 정제된 매체에

높은 빈도 쓰이고 있 않은가 사 빈도 측 에서

필통이나 연필과 같은 단어 대등하거나 더 많이 쓰lsquo rsquo lsquo rsquo

다는 것은 결코 가 게 볼 일이 아니다

이제는 사전이 언어 현 을 빠르게 하는 게 덕인

대가 되었다 세계적으 유 한 의 사전들도 경쟁

적으 어를 고 있다

하 만 얼 은 젊은이들이나 쓰는 속어라고 흠을 잡을lsquo rsquo

도 르겠다 얼 이 주 젊은 층에서 많이 쓰 lsquo rsquo

는 속어임에 틀림없다 러나 어사전에 표 적이고 품

위 있는 말만 어야 한다고 생각한다 것은 커다란

해다 당장 아 어사전이나 펼쳐 라 속어는

설과 같은 비어나 죄자들이 쓰는 은어 어

마니 같은 소 의 사람만이 쓰는 말 도 라 있

않은가 사전은 말 치에 일정 빈도 이상 나타나는 말이

라 말이든 다 할 있다

zb38) 가 나 에 대한 다음의 설( ) ( ) 않은 것은

① 가 는 얼짱 사 에 등재 것에( ) ( ) lsquo rsquo

보 고 다

② 사 등재 가는 단어 격에( )

고 고 는 언 들 언어 사 도에 고 다 ( )

③ 가 얼짱 어지만 신 과 같 매( ) ( ) lsquo rsquo

체에 도 사 는 말 는 고 다

④ 가는 얼짱 어 보고 크게 가지 근( ) lsquo rsquo 3

거 들어 뒷 고 다

⑤ 는 얼짱 어 는 다 특 다는( ) lsquo rsquo

근거 에도 크게 가지 근거 가 들어 주 2

뒷 고 다

가 늘 지 상에 살고 는 사 들 억( ) 10

도가 고 그리 지 통 고 는 사 들

그보다 훨 많 억 도는 고 지 20

통 다 그런 지 고 2500

년 학 간고사 대비2013 2 현대고 대비

ECN-0102-2013-001-000076193

그리 간 보는 과 사 에

매우 달 뿐만 니 과 에 도 극

루고 었다 미 운 그런 들

살고 는 동 과 사 들 사고 식에

큰 가 다는 다

고 그리 들 우주 개별 고 독립

사 들 생각 지만 고 들 우

주 연 질 간주 다 같 각

도 들에게는 연 질 었지

만 그리 들에게는 미 들 결 었다

고 과 그리 들 사 같 는

동 과 사 에 도 견 다

인 리학자인 츠 이마이 디드 겐트너는 두

살이 채 안 된 아이들에서 터 성인에 이르 다양한

연 대의 동양인과 서양인을 대상으 다음과 같은 험

을 했다 저 코르크 만든 피라 드 양의 도형을

여 주고 대상의 이름을 닥 라고 알 주었다lsquo (Dax)rsquo

제 닥 는 존재하 않는 것으 험자가 임의lsquo rsquo

만들어 낸 이름이다 런 다음 두 개의 다른 체를

여 주었는데 하나는 피라 드 양이 만 하얀 플라 틱

으 만들었고 다른 하나는 재 는 코르크 만 양이

달랐다 러고 나서 어떤 것이 닥 인 사람들에게 고 lsquo rsquo

르게 했더니 서양인들은 주 같은 양을 하고 있는

체를 선택했고 동양인들은 같은 재 만들어 체를

선택했다 이러한 차이는 성인은 어 두 살 리

아이들에게서도 나타났다 이것은 곧 서양인과 동양인은

서 다른 세상을 고 있다는 것을 의 한다 략 ( )

는 아주 단 하 서도 인상적인 험을 했다

험에는 동서양의 대학생들이 참여했다 는 험 참가자

들에게 컴퓨터 화 을 통해 속 장 을 담은 애니 이션

을 여 주었다 화 의 앙에는 초점의 역할을 하는 커

다란 고 한 마리가 있었고 주위에는 다른 생

들과 초 자갈 거품 등이 함 제 되었다 화 을

두 씩 후 참가자들은 자 이 것을 회상해 라는

를 았다

결과 서양인 대학생들과 동양인 대학생 두 앙

의 초점 역할을 했던 고 를 동일한 정도 언 했으

나 경 소 위 거품 초 다른 생 들 에 ( )

대해서는 동양인 대학생들이 서양인 대학생들 다 60

이상 더 많이 언 했다 뿐만 아니라 동양인 학생들은 서

양인 학생들에 비해 개 적인 고 다 전체적인 계

를 더 언 하는 경향을 다 략 또한 경의 일 ( )

를 화 킨 림을 제 하 을 때 동양인 대학생들은 대

경의 화를 알아챘 만 서양인 대학생들은 경

의 화를 거의 알아차리 했다 략 ( )

따라서 서양인들만을 대상으 연 한 화lsquo

편성 결 은 잘 된 것일 도 있다 각 과정과 인rsquo

과정의 어떤 이 화 편적이고 어떤 이

화에 따라 달라 는 는 앞으 많은 연 를 통하여 논의

되어야 한다

나 어떤 의 에서 리 두는 이 화적이다 리( )

안에는 다른 사람들과 더 친 한 계를 유 하 는 상호

의존성과 다른 사람들 터 독립적인 존재 살아가 는

독립성이 혼재한다 따라서 이 에서 어떤 특성이 더 강

하게 각되는 상황에 놓이느냐에 따라 서 다른 화적

특 을 일 있다 결 리 두는 어떤 경 에는

동양인처럼 행동하고 어떤 경 에는 서양인처럼 행동하는

것이다

zb39) 가 에 대한 다음의 설( ) 않은 것은

① 는 신 주 뒷 닥 실험과lsquo rsquo lsquo

니 실험 근거 시 다rsquo

② 동 들 상 간 공통 보다는 에 식

는 강 다

③ 들 주변 맥 에는 심 경 어 사건

과 사건 사 계에 상 민감 다

④ 는 동 과 틀린 지 고 는 것lsquo rsquo

니 다 고 다 lsquo rsquo

⑤ 가에 우리 사 들 개 시 가 원( )

집 경 말 고 는 것 개 보다는

에 고 는 것에 다

늘 지 상에 살고 는 사 들 억 도가10

고 그리 지 통 고 는 사 들( )知的

그보다 훨 많 억 도는 고 지 20

통 다 그런 지 고 2500

그리 간 보는 과 사 에

매우 달 뿐만 니 과 에 도 극

루고 었다 미 운 그런 들

살고 는 동 과 사 들 사고 식에

큰 가 다는 다

지심리 미 마 드 겐트 는 동

과 상 다 과 같 실험 다

크 만든 미드 도 보여 주고 그

상 닥 고 주었다 그런 다lsquo (Dax)rsquo

개 다 체 보여 주었는 는 미드

지만 틱 만들었고 다 는 재료는

크 지만 달 다 그러고 어 것 닥 lsquo

지 사 들에게 고 게 니 들 주 같rsquo

고 는 체 택 고 동 들 같

재료 만들어진 체 택 다 러 는

심지어 살짜리 들에게 도 타났다 것

곧 과 동 다 상 보고 다는

것 미 다 개별 사 보고 고 동

년 학 간고사 대비2013 2 현대고 대비

ECN-0102-2013-001-000076193

연 질 보고 는 것 다

동 들 주변 상 에 맞 어 동 고

에 다 사 들 태도 동에 보다

많 주 울 다 동 가 미시간

에 에 경험 다 그는 미

식 경 보러 가게 었는 경 체는 매우 재

미 었 주변 들 동에 질 다 그

는 들 계 어 상태 경

다 어 들 에 에 그 시 가 계

가 진 것 다 뒷사 고 지 는 들

동 럼 어 웠다

그는 경험에 어 얻어 동 들lsquo

각도 상 본다 는 가 우고rsquo

검 여 주 단 도 상 실험 실

시 다 그는 실험 가 들에게 컴퓨 통

담 니 보여 주었다

에는 역 는 커다 고 마리가 었

고 주 에는 다 생 들과 갈 거 등

께 시 었다 본 후 가 들

신 본 것 상 보 는 지시 다

그 결과 생들과 동 생

역 고 동 도 언

경 거 다 생 들에 ( )

는 동 생들 생들보다 60

상 많 언 다 뿐만 니 동 생들

생들에 개별 고 보다 체 계

언 는 경 보 다 경 변 시

킨 그림 시 동 생들 경

변 지만 생들 경 변

거 리지 못 다

지 지 들만 상 연 lsquo

보편 결 못 것 도 다 지각 과 과rsquo

지 과 어 보편 고 어

에 달 지는지는 많 연 통 여

어 다

리 드 니 벳 생각 지도 사- ldquo rdquo( 2004)

zb40) 위 에 대한 설 으 가장 적절한 것은

① 동 과 생 식 강 고 다

② 가지 실험 통 쓴 고 다

③ 닥 실험에 사 본질에 동 사

상에 주 다

④ 니 실험에 동 과 에 지

각 도에 가 다

⑤ 쓴 는 보편 연 에 드러 우월 에

에 근 고 다

가 동 들 주변 상 에 맞 어 동 고( )

에 다 사 들 태도 동에 보다 많

주 울 다 동 가 미시간 에

에 경험 다 그는 미식

경 보러 가게 었는 경 체는 매우 재미 었

주변 들 동에 질 다 그 는

들 계 어 상태 경 다

어 들 에 에 그 시 가 계 가

진 것 다 상 살펴lsquo 는 말 들rsquo

에 그는 에 시 어 도 뒷사

생각 곧 다시 곤 것 다 그런 그에게

뒷사 고 지 는 들 동 럼

어 웠다

그는 경험에 어 얻어( ) 동 들lsquo

각도 상 본다 는 가 우고rsquo

검 여 주 단 도 상 실험

실시 다 실험에는 동 생들 여 다

그는 실험 가 들에게 컴퓨 통

담 니 보여 주었다 에는

역 는 커다 고 마리가 었고 주 에는

다 생 들과 갈 거 등 께 시

었다 본 후 가 들 신 본 것

상 보 는 지시 다

다 그 결과 생들과 동 생( )

역 고 동 도 언

경 거 다 생 들 에 ( )

는 동 생들 생들보다 60

상 많 언 다 뿐만 니 동 생들

생들에 개별 고 보다 체 계

언 는 경 보 다 들어 동

생들 상 체 연못 럼 보 어ldquo 같rdquo

체 맥 언 시 었지만

생들 상 어 같 큰 고 가 쪽 움ldquo

직 어 같 역 고rdquo

언 시 다 경 변 시킨 그

림 시 동 생들 경 변

지만 생들 경 변 거

리지 못 다

년 학 간고사 대비2013 2 현대고 대비

ECN-0102-2013-001-000076193

게 볼 동 들 보다는 큰 그( )

림 보 에 사 과 체 맥 연결시 지각

는 경 고 체에 특 떼어 내

어 독립 보는 것 낯 어 다 에

들 사 에 고 주변 맥 에는 심 경

에 사건과 사건 사 계에 상

민감 편 다

마 지 지( ) 들만 상 연

보편 결 못 것 도 다lsquo rsquo 지각 과

과 지 과 어 보편 고 어

에 달 지는지는 많 연 통 여

어 다

리 드 니 벳 생각 지도 사- ldquo rdquo( 2004)

zb41) 의 하는 가~ 다른 것은

① ② ③

④ ⑤

얼마 그 에 동 사고 식과

사고 식 보여 주는 내 다

들 에 는 탕 고 같 게

어 겨 고 미 에 는 그 크 럼 큰 고

어리 주고 원 는 어 도 는

상 고 생각 다는 것 다 러

는 어떻게 생 것 고 과 그리 거슬

러 가 보 그 단 다

고 연 경 체 경 생 에

다 벼 사는 공동 업과 경험 많 연 역

에 고 들 연 웃과

게 지내 고 탁 연 들

들 지 연 럽게 들 다 민들

웃과 동 게 뿐만 니 는 집 과

게 다

동 시 는 생태 경 에 살 결과

들 다 사 들 사 상 에 주

울 게 었고 는 곧 체 상 과 간 사

계 시 는 낳게 었다 신 가

가 는 체에 는 원 는 동시

에 다 사 들 그 사 포 체 맥 에

다 들 간 사 연

계 체 계에 주 울 는 사고 체계

게 었다

그러 그리 연 경 그 었다 산

지 연결 는 지 건 그리고 역

에 다 런 들 업에 다 사 과

동 므 공동체에

다고 다 고 그리 들

들과는 달리 보 내 감 지 들과

지 크게 느 지 못 다 그

견 다 경우 주 쟁 통 결 는 갖

게 었다

신 사 간 계들 루어진 커다

트워크 에 게 당연 사 역시 연

계들 체 식 게 다 어 상

원 도 그 개체가 체 맥 과

계 에 고 다 게 체 맥 에 주

울 다 보 상 복 과 가변 식 게 고

상에 재 는 많 변 들 사 에 재 는 들도

게 다 들 주 태도 보

는 경우가 많다 쟁 결

통 결 보다는 통 결

는 보 다

그러 고 그리 들 개개 사 사 독

에 주 울 다 사 사 체에

어 그들 사 에 재 는 공통 규 주

고 다 상 원 에도 사

체 내 주 고 다 그들

체 여 탕 체

는 주 태도 시 고 특 사 어

주에 는지 여 그 주에 는 규

견 다 에 는 쟁 식 리

같 리 사고 체계가 달 게 었다

리 드 니 벳 생각 지도 사- ldquo rdquo( 2004)

zb42) 위 에서 사 된 설 과 가장 유사한 것은

① 크톱 컴퓨 는 본체 니 마우 루

어 다

② 곡과 시 리 는 지 과 사 루어 다는 공통

지니고 다

③ 경 고 것과는 달리

경 본 연 태 그 주변 경

④ 벽돌 능 에 사계 내내

습도가 지 다

⑤ 잰느 체 체 지닌 재 체가 없

는 재 눌 다

년 학 간고사 대비2013 2 현대고 대비

ECN-0102-2013-001-000076193

zb43) 는 립 앙 도서 이 정의 일 이다lt gt

도서 장과 이 자의 리 의 정의 연결이

적절하 않은 것은

lt gt

제 조 서 유8 ( )

도서 장은 다른 이 자의 안전을 위협하거나 도서 의①

서를 란하게 할 가 있는 자에 대하여는 도서 출입

을 제한할 있다

도서 장은 이 자가 제 조 각 호의 어느 하나의 행위를 하7②

을 때에는 이 을 하게 하거나 도서 출입을 제한할

있다

제 조자 의 대출9 ( )

도서 자 는 다음 각 호의 경 대출할 있다①

상호대차도서 간에 자 를 류하는 것을 말한다 등 다1 ( )

른 도서 과의 협 을 위하여 필 한 경

공 이 공 행 상 필 하는 경2

에 도서 장이 필 하다고 인정하는 경3

대출이 가능한 도서 자 의 위는 도서 장이 정하는②

에 따른다

제 조 상10 ( )

이 자가 도서 자 설을 더럽히거나 찢거나 뜨①

쓰게 하거나 잃어 린 경 에는 상하여야 한다

도서 장은 제 항에 따른 상 을 정하여 게 하여야1②

한다

제 조이 절차 등11 ( )

이 칙에서 정한 것 에 도서 자 설의 이 절차

이 제한 등에 필 한 사항은 도서 장이 정한다

출처 립 앙 도서- (httpwwwnlgokr)

① 는 도 리 다8

② 도 는 리 다9 1

③ 료 지 는 도 리 다9 2

④ 도 료 변상에 리10 1

⑤ 는 에 도 리 다11

3

도 다 각 같다①

공 공 다만 연1

연 간 다

매월 째 째 월2

도 도 리 그 사3

가 다고 는

도 에 미리 게1 3②

시 여 다

4

도 시간 도 여 게시 다

5

도 료 시 는 는 도①

지에 등 후

등 에 사 도②

7

는 다 각 여 는 니 다

도 료 시 상 리1 lsquo rsquo

도 료 시 훼 는2 middot

지 가 닌 곳에 식 거 담3

우는

도 보 등 보 검색열4 middot

그 에 도 질 지 여 도5

여 게시 사 는

8

도 다 거 도①

질 게 우 가 는 에 여는 도

도 가 각 어느7②

에는 지 게 거 도

9

도 료는 다 각 경우 다①

상 도 간에 료 는 것 말1 (

다 등 다 도 과 여 경우)

공 원 공 상 는 경우2

그 에 도 다고 는 경우3

가능 도 료 는 도②

는 에 다

10

년 학 간고사 대비2013 2 현대고 대비

ECN-0102-2013-001-000076193

가 도 료 시 럽 거 거①

못 쓰게 거 어 린 경우에는 변상 여

도 에 변상 여 게시1②

여 다

zb44) 위 에서 도서 장이 게 해야 할 사항에 해당하는

것을 두 쓰

년 학 간고사 대비2013 2 현대고 대비

ECN-0102-2013-001-000076193

립 도 규

1 ( )

규 립 도 립 어린 청 도(

포 다 료 시 열 시 말) (

다 에 사 규 립 도)

편 진 다

2 ( )

규 립 도 도 다 에( lsquo rsquo )

고 는 도 에 도lsquo rsquo 2 2

료 에 여 다 다만 특 료 귀

료 등 료 에 사 립 도

도 다 다( lsquo rsquo )

3 ( )

도 다 각 같다①

공 공 다만 연1

연 간 다

매월 째 째 월2

도 도 리 그 사3

가 다고 는

도 에 미리 게1 3②

시 여 다

시간4 ( )

도 시간 도 여 게시 다

등 등5 ( )

도 료 시 는 는 도①

지에 등 후

등 에 사 도②

사 료6 ( )

도 료 시 에 사 료는 도

7 ( )

는 다 각 여 는 니 다

도 료 시 상 리1 lsquo rsquo

도 료 시 훼 는2 middot

지 가 닌 곳에 식 거 담3

우는

도 보 등 보 검색열4 middot

그 에 도 질 지 여 도5

여 게시 사 는

질 지8 ( )

도 다 거 도①

질 게 우 가 는 에 여는 도

도 가 각 어느7②

에는 지 게 거 도

료9 ( )

도 료는 다 각 경우 다①

상 도 간에 료 는 것 말1 (

다 등 다 도 과 여 경우)

공 원 공 상 는 경우2

그 에 도 다고 는 경우3

가능 도 료 는 도②

는 에 다

변상10 ( )

가 도 료 시 럽 거 거①

못 쓰게 거 어 린 경우에는 변상 여

도 에 변상 여 게시1②

여 다

등 규 에 것 에 도11 ( )

료 시 등에 사

도 다

립 도- (httpwwwnlgokr)

zb45) 도서 장의 리 있는 조항으 적절하 않

은 것은

① ② ③ ④ ⑤

년 학 간고사 대비2013 2 현대고 대비

ECN-0102-2013-001-000076193

1 ( )

사가 공 는lsquo rsquo

과 여 사 원과 리

사 타 사 규

니다

개 보 보7 ( )

사는 보통신망 등 계 는 에lsquo rsquo lsquo rsquo

원 개 보 보 니다 개lsquo rsquo

보 보 사 에 는 사 개lsquo rsquo

보 취 니다 다만 사는 다 lsquo rsquo

사 계 통 공 는 경우 원 lsquo rsquo

등 개 보 당 사에 습니lsquo rsquo

원 리에8 (lsquo rsquo lsquo rsquo lsquo rsquo

)

원 에 리lsquo rsquo lsquo rsquo lsquo rsquo①

원에게 가 도 여 는lsquo rsquo 3

니다

사는 원 가 개 보 우 가lsquo rsquo lsquo rsquo lsquo rsquo②

거 사 경우 는 미 에 어 거 lsquo

사 사 운 우 가 는 경우 당rsquo lsquo rsquo

습니다lsquo rsquo

원 가 도 거lsquo rsquo lsquo rsquo lsquo rsquo 3③

가 사 고 지 경우에는 시 사에lsquo rsquo

통지 고 사 내에 니다lsquo rsquo

경우에 당 원 사에 그 사실3 lsquo rsquo lsquo rsquo④

통지 지 거 통지 도 사 내에 지 lsquo rsquo

생 경우 사는 지지 습니다lsquo rsquo

사10 (lsquo rsquo )

사는 과 지 미lsquo rsquo①

에 는 지 계 고

공 여 다 여 니다lsquo rsquo

사는 원 게lsquo rsquo lsquo rsquo lsquo rsquo②

도 개 보 신 보 포 보 보 시( )

갖 어 개 보 취 공시 고

니다

사는 과 여 원lsquo rsquo lsquo rsquo③

견 만 당 다고 경우에는

리 여 니다 원 견 만 사 lsquo rsquo

에 는 게시 거 우편 등 통 여

원에게 리 과 결과 달 니다lsquo rsquo

원11 (lsquo rsquo )

원 다 여 는 니다lsquo rsquo ①

신청 는 변경 시 허 내 등1

타 보 도2

사가 게시 보 변경3 lsquo rsquo

사가 보 보 컴퓨 그4 lsquo rsquo (

등 등 신 는 게시)

사 타 등 지 재산 에5 lsquo rsquo 3

사 타 상 거 업6 lsquo rsquo 3

는 폭 시지 상 타 공7 middot middot

에 는 보 에 공개 는 게시 는lsquo rsquo

사 동 없 리 사8 lsquo rsquo

타 거 당9

게시15 (lsquo rsquo )

원 내에 게시 는 게시 게재 는lsquo rsquo lsquo rsquo lsquo rsquo

경우 원 사가 게시 복 lsquo rsquo lsquo rsquo lsquo rsquo middot middot

등 태 언 등에 공 는

것 내에 다 원 본 게시 등 lsquo rsquo lsquo rsquo

크 능 등 여 복 는 등 태

는 것 동 것 니다

- (wwwnavercom)

zb46) 위 은 인터넷 포털사이트의 회 가입을 위한 이

약 의 일 이다 이 약 을 만드는 과정에서 생각한

내 으 적절하 않은 것은

개 보 보 가 지에 별 눠①

겠어

원 가 만들게 에②

시 주어 겠어

원들 게재 게시 다 원 크 다③

는 것 지

④ 원 지 는 뿐만 니 사가 지 는

도 께 달 지

리에 가 생 경우 사가⑤

에 다는 도 듯

1 ( )

사가 공 는lsquo rsquo

과 여 사 원과 리

사 타 사 규

년 학 간고사 대비2013 2 현대고 대비

ECN-0102-2013-001-000076193

니다

개 보 보7 ( )

사는 보통신망 등 계 는 에lsquo rsquo lsquo rsquo

원 개 보 보 니다 개lsquo rsquo

보 보 사 에 는 사 개lsquo rsquo

보 취 니다 다만 사는 다 lsquo rsquo

사 계 통 공 는 경우 원 lsquo rsquo

등 개 보 당 사에 습니lsquo rsquo

원 리에8 (lsquo rsquo lsquo rsquo lsquo rsquo

)

원 에 리lsquo rsquo lsquo rsquo lsquo rsquo①

원에게 가 도 여 는lsquo rsquo 3

니다

사는 원 가 개 보 우 가lsquo rsquo lsquo rsquo lsquo rsquo②

거 사 경우 는 미 에 어 거 lsquo

사 사 운 우 가 는 경우 당rsquo lsquo rsquo

습니다lsquo rsquo

원 가 도 거lsquo rsquo lsquo rsquo lsquo rsquo 3③

가 사 고 지 경우에는 시 사에lsquo rsquo

통지 고 사 내에 니다lsquo rsquo

경우에 당 원 사에 그 사실3 lsquo rsquo lsquo rsquo④

통지 지 거 통지 도 사 내에 지 lsquo rsquo

생 경우 사는 지지 습니다lsquo rsquo

원에 통지9 (lsquo rsquo )

사는 특 다 원에게 통지 경우lsquo rsquo lsquo rsquo

공지 게시 통 상 게시 개별 통지에7

갈 습니다

사10 (lsquo rsquo )

사는 과 지 미lsquo rsquo①

에 는 지 계 고

공 여 다 여 니다lsquo rsquo

사는 원 게lsquo rsquo lsquo rsquo lsquo rsquo②

도 개 보 신 보 포 보 보 시( )

갖 어 개 보 취 공시 고

니다

사는 과 여 원lsquo rsquo lsquo rsquo③

견 만 당 다고 경우에는

리 여 니다 원 견 만 사 lsquo rsquo

에 는 게시 거 우편 등 통 여

원에게 리 과 결과 달 니다lsquo rsquo

원11 (lsquo rsquo )

원 다 여 는 니다lsquo rsquo ①

신청 는 변경 시 허 내 등1

타 보 도2

사가 게시 보 변경3 lsquo rsquo

사가 보 보 컴퓨 그4 lsquo rsquo (

등 등 신 는 게시)

사 타 등 지 재산 에5 lsquo rsquo 3

사 타 상 거 업6 lsquo rsquo 3

는 폭 시지 상 타 공7 middot middot

에 는 보 에 공개 는 게시 는lsquo rsquo

사 동 없 리 사8 lsquo rsquo

타 거 당9

원 계 규 내lsquo rsquo lsquo②

여 공지 주 사 사가 통지 는rsquo lsquo rsquo

사 등 여 타 사 업 에 lsquo rsquo

는 여 는 니다

- (wwwnavercom)

zb47) 위 약 의 조항에서 같은 제점을 하lt gt

고 있는 조항은

lt gt

제휴 회사에 회 의 아이디 개인 정 를 전송할 있도

한 조항은 고객에게 당한 조항이다

1 7 8① ② ③

④ 9 ⑤ 10

립 도 규

1 ( )

규 립 도 립 어린 청 도(

포 다 료 시 열 시 말) (

다 에 사 규 립 도)

편 진 다

2 ( )

규 립 도 도 다 에( lsquo rsquo )

고 는 도 에 도lsquo rsquo 2 2

료 에 여 다 다만 특 료 귀

료 등 료 에 사 립 도

도 다 다( lsquo rsquo )

3 ( )

도 다 각 같다①

공 공 다만 연1

연 간 다

년 학 간고사 대비2013 2 현대고 대비

ECN-0102-2013-001-000076193

매월 째 째 월2

도 도 리 그 사3

가 다고 는

도 에 미리 게1 3②

시 여 다

시간4 ( )

도 시간 도 여 게시 다

등 등5 ( )

도 료 시 는 는 도①

지에 등 후

등 에 사 도②

사 료6 ( )

도 료 시 에 사 료는 도

7 ( )

는 다 각 여 는 니 다

도 료 시 상 리1 lsquo rsquo

도 료 시 훼 는2 middot

지 가 닌 곳에 식 거 담3

우는

도 보 등 보 검색열4 middot

그 에 도 질 지 여 도5

여 게시 사 는

질 지8 ( )

도 다 거 도①

질 게 우 가 는 에 여는 도

도 가 각 어느7②

에는 지 게 거 도

료9 ( )

도 료는 다 각 경우 다①

상 도 간에 료 는 것 말1 (

다 등 다 도 과 여 경우)

공 원 공 상 는 경우2

그 에 도 다고 는 경우3

가능 도 료 는 도②

는 에 다

변상10 ( )

가 도 료 시 럽 거 거①

못 쓰게 거 어 린 경우에는 변상 여

도 에 변상 여 게시1②

여 다

등 규 에 것 에 도11 ( )

료 시 등에 사

도 다

립 도- (httpwwwnlgokr)

zb48) 다음 정 리 의 의 으 볼 때 가장

이 적인 것은

도 시간 도 여 게시 다①

등 에 사 도②

가능 도 료 는 도 는③

에 다

④ 도 에 변상 여 게10 1

시 여 다

⑤ 도 가 각 어느7

에는 지 거 도

zb49) 를 참고하여 이 어의 성격을 설 한lt gt

것으 적절하 않은 것은

① 보 에 는 어 시 상 고 어 시lt gt lsquo rsquo

에 보여주고 다

② 진 어 어원에 견 고 다

에는 타 어 들어가는 것 다 lsquo rsquo

③ 에 들어갈 말 각각 고 어 어 신 어~

들 언어는 질 격 강 통 없었다

④ 시 우리 에 가 었지만 지 계

과 달리 들 통 사 달 어 웠

년 학 간고사 대비2013 2 현대고 대비

ECN-0102-2013-001-000076193

⑤ 크 몽골 만주 공통어가 우리 어 같

계열에 다는 에 사 특 짐

가( )

善化公主主隱 공주님

他密只嫁良置古 몰 결 고

薯童房乙 맛

夜矣卯乙抱遣去如 에 몰 고 가다

( )

始汝 會隱日恚見隱扐 만 에 본

恥隱汝衣淸隱笑 맑 웃

고 시 여 공 크다 만 다[ ] ( ) ( ) ( ) ( )始 汝 會扐

내다 에 보다 견( ) ( )恚 見 다( )隱

럽다 맑다 청 웃( ) ( ) ( ) ( )恥 衣 淸 笑

zb50) 위의 나 를 함 고 음에 답하( ) lt gt

보lt gt

( )素那或云金川 白城郡蛇山人也

운 사산

는 고 다 는( )[ ( ) ] (素那 金川 白城

사산 사 다) ( ) 郡 蛇山

삼 사- lsquo rsquo 47

에 제 된 단어 의 표 리를 조건(1) lt gt ( ) lt gt

에 맞게 서 하

건lt gt

lsquo 었고 었다 태rsquo

에 제 된 단어 동일한 표 리에(2) lt gt ( )

의해 적은 것을 나 에서 찾아 조건 에 맞게 서 하( ) lt gt

건lt gt

에 당 는 각각( ) 개 쓸 것2 단

당 는 가 여러 개 어도 개만 쓸 것 각2

개 과 도 쪽에 개만2 2

드시 지 것( )

과 동 원리 것lsquo 고

과 동 원리 것 다rsquo

태 것

가( )

素那(或云金川) 白城郡蛇山人也

소나 또는 천 이라 한다 는 성 사( ) ( ) ( )素那 金川 白城郡〔 〕

산 사람이다 현대어 풀이( ) ( )蛇山

나( )

紫布岩乎希 회

執音乎手母牛放敎遣 자 손 암쇼 노히 고

吾 不喩慙 伊賜等肹 肹 나 안디 리샤

花 折叱肹 可獻乎理音如 고 것거 도림다

다 향찰은 리말을 리 으 적은 표 이었 만 생( )

은 고 대를 넘 하고 끊어 고 말았다 랜 세

동안 갈고 닦아 체계적이었던 향찰 표 이 사라졌

을 인은 크게 두 가 나누어 생각해 볼 있다

하나는 족 사회의 한 선호도에서 찾을 있다 라 때

향찰은 주 족 계 에서 사 했을 것으 인다 한 을

알 하고서는 한자를 활 하여 리말을 리 으 표

하 란 가능하 때 이다 런데 족들은 간이 흐

를 향찰과 같은 리 표 을 익혀 사 하 다는

아 한 을 대 사 하는 쪽을 선호하게 되었다 더 이

고 초에 인재 등 을 위해 과거제도가 행되 서 한 선

호도가 더 높아졌고 결 향찰은 소 되고 말았다

또 다른 가능성은 한 어의 특성에서 찾을 있다

터 한 과 일 세 나라는 한자 화 에 속해 다

당연한 이야 겠 만 표의 자인 한자는 어를 표 하

에 매 적절하다 어의 음절은 성 ( ) ( )聲母 韻母

이 어 고 여 에 성조가 추가되어 최종 소리가 결정된

다 래서 어는 단음절을 하나의 한자 표 하 된

다 에 초성 성 종성의 세 가 소가 하나의 음절

년 학 간고사 대비2013 2 현대고 대비

ECN-0102-2013-001-000076193

을 이 는 한 어는 음절 조가 잡하고 음절의 가 많아

서 한자 차 만으 한 어의 소리를 만족 럽게 표 할

없었다 를 들어 한 어에서는 어 니 같이 음절 lsquo rsquo

이 어 단어가 얼마든 있으나 어는( ) 複數音節

자 하나 나타내 만이다lsquo [m ]rsquo 母 ǔ

한편 일 어의 표 은 핵 적 단어는 한자 적고 토는

가나라는 일 의 자 적는 이다 적인 의 를 나

타내는 은 표의 자인 한자 적고 적 계를 나

타내는 토는 표음 자 적는 셈이니 자세히 살펴

리의 향찰 표 을 쏙 빼닮았음을 알 있다 한 어 같

은 착어이 서도 일 어에만 향찰과 유사한 표 이 살아

남은 것은 일 어의 특 때 이다 일 어는 하나의 자음과

음의 결합으 음절을 이 고 침이 거의 없는 음절 언어

이다 이러한 음절의 특색에다가 토가 달한 착어라는 점

이 향찰과 유사한 표 이 살아남을 있는 비결이었다

하 만 같은 착어라도 다양한 음소 침이 달한 한

어는 향찰 표 하는 데 근 적으 한계가 있었다

zb51) 다 하여 의 행에 대한 탐 한 결과( ) lt gt 2

않은 것은

보lt gt

善花公主主隱 공주니믄 공주님( )

----------------------------------------

-

他密只嫁良置古 그 지 얼어 고 몰 결(

----------------------------------------

-

薯童房乙 맛 맛( )

夜矣卯乙抱遺去如 몰 고 가다 에 몰 고(

가다)

주동 역 동- (薯童謠『 』

에 2 ( )他密只嫁良置古

얼다 시집가다 결 다 말 lsquo rsquo

① 실질 미 지니고 므 타 타lsquo ( )rsquo lsquo [ ]

② 에 실질 미 타내고 지 는lsquo rsquo lsquo [ ]rsquo lsquo [ ]密只 密 只

계 타내는

③ 얼어는 실질 미 포 고 므 가lsquo rsquo lsquo [ ]rsquo嫁

것lsquo [ ]rsquo 良

④ 고 어간 는 실질 미 지니고 므lsquo rsquo lsquo -rsquo

것lsquo [ ]rsquo 置

⑤ 고 어미 고는 계 타내고 므lsquo rsquo lsquo- rsquo

고 것lsquo [ ]rsquo 古

가( )

엉 훈 민middot middot middot middot middot世 宗 御 製 訓 民 正 音

말 미 듕 귁에 달middot middot middot middot middot middot middot middot中 國 文 字

니 런middot middot middot middot middot middot 어린middot middot middot middot百 姓

니 고 도 내 들middot middot middot middot middot middot middot middot middot 시러middot

펴 몯middot 미middot middot 니 내middot middot middot middot middot middot middot middot 爲

어엿middot 겨 새middot middot middot 믈여듧middot middot middot middot字 니middot middot middot

사 마다 니겨 킈 middot middot middot middot middot middot middot middot middot便 安

고 미니middot middot middot middot

본 는 상( ) (象

원리에 만들어진 본) ( )形 ㄱ ㄴ ㅁ ㅅ ㅇ

에 는 가 원리에( )加劃

그리고( )ㅋ ㄷ ㅌ ㅂ ㅍ ㅈ ㅊ ㆆ ㅎ

쓰는 병 원리에 만들어진( )竝書

마지막 체( ) ( )異體ㄲ ㄸ ㅃ ㅆ ㅉ ㆅ

ᅀ 다 상 원리에 ㅇ ㄹ

지 는 삼재 상 본 본( ) ( ) ( 天地人 三才

탕 므림과 림에 ) (初ㅡ ㅣ

재)( ) ( )( )出字 再出字ㅗ ㅏ ㅜ ㅓ ㅛ ㅑ ㅜ ㅕ

병 그리고 들 에 다시( )ㅘ ㅝ ㅣ

( )ㅣ ㅢ ㅚ ㅐ ㅟ ㅔ ㆉ ㅒ ㆌ ㅖ ㅙ ㅞ

zb52) 가 에 대한 설 으 르 않은 것을( ) 두 고르

① 어쓰 규 지키고 다

② 리 고 다

③ 말 미 미 등 어 사 다lsquo rsquo

④ 개 지 다

년 학 간고사 대비2013 2 현대고 대비

ECN-0102-2013-001-000076193

⑤ 어 원 에 가 도 고 다

엉 훈 민世 宗 御 製 訓 民 正 音

말 미 듕귁에 달 니

런 어린 니 고 도middot

내 들 시러 펴 몯 미 니middot

내 어엿 겨 새 믈여듧

사 마다 니겨middot 킈 고

미니

훈민 언 본- lsquo rsquo 5 (1459 )

zb53) 위의 에 대한 현대어 풀이가 르~ 않은 것

① 우리 말 과 달

② 어리 말 고 는 것 어도

③ 신 생각 마 껏 펼 는 사 많다

④ 게 생각 여

⑤ 사 마다 게

zb54) 훈민정음 언해 에는 한 을 창제한 동 가 드러나

있다 훈민정음 창제의 정 과 내 이 잘 연결된 것

① 주 신 말 미 듕귁에 달

② 민 신 내 어 겨

③ 신 뻔 킈 고 미니

④ 실 신 사 마다 니겨

⑤ 귀 신 계 주 는 훈민 신과 거리가

가 엉 훈 민( ) middot middot middot middot middot世 宗 御 製 訓 民 正 音 

말 미 귁에 中 國 달 文 字

니 런 어린 니 百 姓

고 도 내 들 시러 펴 몯

미 니 내 어엿 爲 겨 새

믈여듧 니 사 마다 니 字

겨 킈 고 미니 便 安

훈민 언 본- lsquo ( )rsquo ( ) 5 (1459 )訓民正音 世祖

( )

[ 1 ]

동 룡 샤 마다 복( ) ( ) ( )海東 六龍 天福

시니 고 동( ) ( )古聖 同符 시니

[ 2 ]

매 니 곶 여

미 므 니 그 내 러

가 니

[ 125 ]

우 미리( )千世 샨( )定 에( )漢水北 累仁

누 개 샤 복 업 시니( ) ( ) 開國 卜年

신( )聖神 니 샤도 경 근민 샤 욱( )敬天勤民

드시리 다

님 쇼 산 가( ) ( )洛水 山行

미드니 가

어 가- lsquo ( )rsquo 27龍飛御天歌

다 우리신 니쓰고 다만 만 쓰( )

거 샹 귀쳔 다보게 러 귀

여 쓴 도 신 보 가 고 신 에

말 어 보게 각 에 사 들

고 본 몬 능통 후에

죠 죠 니

드 도 만 공 에 사

드 미 죠 고 고 여 보 죠

보다 얼마가 거시 어신고 니 첫

가 죠 니 죠

민 들 어 신 샹

귀쳔 도보고 어보 가 만 늘

고 폐 에 만쓴 죠 민

도 러보지못 고 보니 그게 엇지

심 니 리 보 가 어 운건 다

니 쳣 말마 지 니 고 그

쓰 에 가 우 지 지

몰 거 본후에 가 어 지

고 그니 쓴편지 쟝 보

년 학 간고사 대비2013 2 현대고 대비

ECN-0102-2013-001-000076193

쓴것보다 듸 보고 그 마 니 쓴 고

어 못

그런고 에 리 과 가

만 쓴 못 민 말만 듯고

고 편 그 못 보니 그사 단

병신 못 다고 그사 식 사

니 만 고 다 과 그사

만 고 다 과 업 사 보다 식 고

죠 도 고 각 과

견 고 실 직 귀쳔 간에 그

고도 다 것 몰 귀죡 보다

사 우리 신 귀쳔 다 업

시 신 보고 과 지 게 랴

시니 샹 귀쳔 간에 우리 신 걸

간 보 새지각과 새 걸 미리

독립신- lsquo (1896)rsquo

zb55) 친 어 나의 제 장( ) 2 매 함축적

의 가 가장 유사한 것은

① 지 눈 내리고 매 득 니 내 여 가

사- lsquo rsquo

② 도 어 리듯 그 게 어 다

주 사- lsquo rsquo

③ 눈 살 다 죽 어 린 과 체 여

눈 새벽 지 도 살 다

눈- lsquo rsquo

④ 삶 근심과 고단 에 돌 거니는 여 거 는

여 리 내린 살가지 에 눈 리 눈 리

택 그 생 에- lsquo rsquo

⑤ 늘 러 고 러

청룡 룡 어 개 루 우

신경림 계- lsquo rsquo

zb56) 친 를 위 가 나 에 나타난A B ( ) ( )

세 어의 특 에 의거하여 세 어 표 하

그 산 고 공 도 맑지만

A

주변에 쓰 리는 어리 사 많다

B

건lt gt

식 가 에 타 어 특징에( ) ( )

거 과 어쓰 는 고 지 말 것

A

B

zb57) 가 의( ) 달 아ㆍ 다 의 ( ) 나셔에서 알 있는

세 어 개화 어의 특 을 비 하여 조건 에lt gt

맞게 서 하

건lt gt

어에 는lsquo 개

어에 는 다 태rsquo

zb58) 은 가 는 다 에 나 는 절lt 1gt ( ) lt 2gt ( )

일 를 췌한 것이다 의 의 가 lt 1gt (1)~(2)

유사한 말을 에서 찾아 쓰lt 2gt

보lt 1gt

런 (1) 어린 니 고百 姓

도 내 들 시러 펴 몯 미

사 마다 (2) 니겨 便 安

킈 고 미니

보lt 2gt

죠 고 고 여 보 죠

보다 얼마가 거시 어신고 니 첫 가

죠 니 죠 민

들 어 신 샹 귀쳔

도보고 어보 가 만 늘 고

폐 에 만쓴 죠 민 도

러보지못 고 보니 그게 엇지 심

니 리

년 학 간고사 대비2013 2 현대고 대비

ECN-0102-2013-001-000076193

lt 1 gt

동 룡 샤 마다 복 시( ) ( ) ( )海東 六龍 天福

고 동 시니( ) ( )古聖 同符

lt 2 gt

(A) 매 니 곶

여 니

미 므 니 그 내

러 가 니

lt125 gt

우 미리 샨 에( ) ( ) ( ) 千世 定 漢水北 累

누 개 샤 복 업 시 니( ) ( ) 仁開國 卜年 聖

신( ) 神 니 샤도 경 근민 샤( ) 敬天勤民

욱 드 시 리 다

님 쇼 산 가 ( ) ( )洛水 山行

미드니 가

- lt gt龍飛御天歌

zb59) 장과 내 상 유사한 성격의 조는125

① 뫼 고 고 고 고

어 그린 많고 많고 고 고

어 러 는 울고 울고 가느니

도 견- lt gt

② 강 에 드니 몸 다

그믈 고 가니

뒷 뫼 엄 언 니( )藥

-

③ 말 없는 청산 태 없는 다

값 없는 청 없는 월

에 병 없는 몸 별 없 늙 리

-

④ 가마귀 골에 가지 마

낸 가마귀 새

청강에 것 시 몸 러 가( ) 淸江

-

⑤ 진 골에( ) 白雪

가 매 는 어느 곳에 었는고

에 갈 곳 몰( ) 夕陽

색-

zb60) 위 에 나타난 세 어의 특 으 적절하 않은

것은

① 룡 어 주격 사에 당 는 가 사( ) lsquo rsquo六龍

고 다

② 샤 어에도 어 주체 쓰 다

는 것 다

③ 매 어 달리 사 택에 어

가 지 지지 고 다

④ 므 원 상 직 어 지 다

⑤ 드시리 다 주체 과 상 께 사

고 다

수고 하셨습니다hearts hearts

년 학 간고사 대비2013 2 현대고 대비

ECN-0102-2013-001-000076193

보닷컴에 공 는 별 보는 고등

들 여 주 는

들 습니다 슷 동 지

가 복 는 것 도가

니 복 여 습 시고 거 시

니다

정답 해설

1) 정답[ ] ④

해설 다른 것은 두 특정 업이나 단 내에서 사[ ]

하는 일종의 은어 사회 언에 해당한다 러나

는 언이 아니라 단과대학을 여서 단대 사lsquo rsquo lsquo rsquo lsquo④

대학을 여서 사대라고 한 말에 해당하 일rsquo lsquo rsquo

사회에서도 널리 쓰이 사회 언이라 할

없다

2) 정답[ ] ⑤

해설 사회 언은 같은 단 내에서 쓰이는 언어이[ ] lsquo rsquo

동일 단끼리는 단결 과 친 감을 형성하는

능을 하 리적 안감이 일어나 않는다

3) 정답[ ] ③

해설 사람이라는 차 적 표현에 대한 대안적 표현이[ ]lsquo rsquo

인 아내 처 등으 볼 있다lsquo rsquo

4) 정답[ ]⑤

해설 남성은 주 격 체를 사 한다[ ]

5) 정답[ ] ⑤

해설 흑인은 검다라는 뜻을 가 고 있을 뿐 인[ ]lsquo rsquo lsquo rsquo lsquo rsquo

다 열등한 뜻을 내포하 않는다

6) 정답 살 색 첫 작품[ ] - -

해설 살색 혹은 킨색은 한 인의 피 색을 뜻[ ] lsquo rsquo lsquo rsquo

하는 것으 인종 차 을 추 고 출 이주민

의 평등 을 침해할 있어 년 표 이2005

살 색으 이름을 꾸었다 처녀작은 처녀라lsquo rsquo lsquo rsquo lsquo rsquo

는 단어가 가 고 있는 곡된 성 인 을 한 것

으 첫 작품정도 꾸어 사 하는 것이 좋다lsquo rsquo

7) 정답[ ] ⑤

해설 호는 아들에게 해체를 사 하고 있다[ ] ① ②

장 을 성하는 청자는 자 의 아 느리 아lsquo

들 세 이다 호는 아 느리에게 해rsquo ③

체를 사 하고 있다 호가 느리 아 에게 ④

사 한 해 체 아들에게 사 한 해체는 두 비lsquo rsquo lsquo rsquo

격 체에 해당한다 호는 자 의 아랫사람인 ⑤

느리에게 아들과 마찬가 해체를 사 하는 것이

상 이 만 임 을 한 느리에게 고마 과 쁨

존 의 표 를 하 위해 자 의 아 에게 말하듯

해 체를 사 하고 있다

8) 정답[ ] ③

9) 정답[ ] ⑤

10) 정답[ ] ①

해설 청자 할아 가 장의 주체 아 다 높을[ ] ( ) ( )

경 에는 압존 에 의해 장의 주체를 높이 않는lsquo rsquo

다 러 아 서가 아닌 아 는으 계 lsquo rsquo lsquo rsquo lsquo

니다 가 아닌 있 니다 표현하는 것이 르rsquo lsquo rsquo

11) 정답 당이 당을 쫒았다 당이[ ]

당에 다

해설[ ]

12) 정답[ ] ⑤

해설 서 다른 높임표현을 통해 청자에 대해 리[ ] ⑤

적 거리감을 나타내는 인 은 이 아니라 현정이

다 가 에서 현정은 에게 해 체를 사 함으 써 ( )

친근감을 드러낸다 나 에서 연 을 게을리하는 역 ( )

도 들 때 에 화가 난 현정이 선생님에게 항의하

는 장 에서는 하 체를 사 하여 리적 거리lsquo rsquo

가 어졌음을 나타내고 있다

13) 정답[ ] ①

해설 는 는 얼 빛이 날과 어찌 다르 고[ ] lsquo rsquo

라는 뜻으 전과 달리 임이 화자를 않고

있음을 알 있다

14) 정답 달리 후 가 있다 이를 통해 경[ ] lt gt

쾌한 음악성을 형성하고 노 젓는 상황을 체적으

형상화하는 역할을 한다

15) 정답[ ] ①

16) 정답[ ] ⑤

해설 다 의 자연은 를 성찰하게 하는 대상[ ] ( )⑤

이자 정의 대상이다 의 자연은 자 의 상황과 ⑤

처 를 드러내는 경으 서의 역할을 하 이

이 없다

17) 정답[ ] ③

해설 는 빈천 을 해결하고자 했으나 강산[ ] lsquo ( )rsquo 貧賤③

과 풍 을 달라는 에 거절하 다고 함으 써 자

연에 대한 애정을 드러내고 있으 는 않는

임에 대한 망을 개에게 전가 켜서 임에 대한 리

을 드러내고 있다

18) 정답[ ] ③

년 학 간고사 대비2013 2 현대고 대비

ECN-0102-2013-001-000076193

19) 정답[ ] ⑤

해설 고상한 음악가의 이름을 리말 꽝 럽[ ]

게 꿈으 써 언어유희를 통해 음을 유 하고 있

다 이는 고상한 척하는 총 를 비꼼으 써 비판적

태도를 드러내는 것이 대상을 꽝 럽게 표현

하여 총 의 허 과 사치를 풍자하고 있다

20) 정답[ ] ⑤

해설 는 작품 속 경에 대한 설 이 드러나는 것이[ ]

서 자의 주 적인 견해가 접적으 드러나는 것이

아니다

21) 정답[ ] ⑤

22) 정답[ ] ②

23) 정답[ ] ④

24) 정답[ ] ①

해설 적강 티프는 주인공의 비 한 출생이나 능[ ] ①

과 이 있는 것으 조정의 능함을 풍자하는lsquo rsquo

것과는 거리가 다

25) 정답 픔 나[ ] ( )

해설 의 음악은 고통 는 사람들을 위 하고 아픔[ ] lsquo rsquo

을 치유해 주는 능을 한다고 할 있다 의 lt gt

픔 도 소 된 이 과 더 어 살아가는 따뜻한 마음lsquo rsquo

을 상 한다

26) 정답[ ] ⑤

해설 에게 선천적으 주어 각 장애라는 역경[ ]

은 의 이라는 가사 연 을 있다lsquo rsquo

27) 정답[ ] ④

해설 는 장 란 선 에게 은 개인적인 인상을[ ]

소녀 장정 등으 표현한 것이다lsquo rsquo

28) 정답[ ] ②

해설 담자가 피 담자의 언어적 표현이나 비언어[ ]②

적 표현 하 독자는 담의 위 나 피

담자의 감정 상태를 알 있다 이를 통해 독자는

담 상황을 더 생생하게 느낄 있고 피 담자

를 더 잘 이해할 있게 된다

29) 정답[ ]③

해설 일상생활과 역도 선 서의 성과에 된 것에서[ ]

역도를 하 서 겪는 어 과 내적 고민으 화제를

전화하 위한 것이다

30) 정답[ ] ①

해설 릿속에 새겨 넣듯 이 억되도 함 세상[ ] ② ③

살이가 힘들고 고생 러 속 하여 자유를 ④

가 없는 고통의 상태를 비유적으 이르는 말

적의 침입을 막 위해 쌓은 축 켜야 할⑤

대상을 비유적으 이르는 말이다

31) 정답[ ] ④

해설 이 의 종류는 전 으 인 사건 경[ ] lsquo

비평을 성 소 삼는다rsquo

32) 정답[ ] ④

해설 근은 삼대독자 태어났음을 에서 확인할[ ]

있다 형제들과의 담은 이뤄 가 없다

33) 정답[ ] ⑤

해설 근은 가난에도 하고 화가를 꿈꾸었다[ ] (3

단 또한 다른 화가 망생들은 정 육을)

위해 상 학 학 해 유학 에 랐 만

근은 다른 을 찾아야 했다 단 세에(5 ) 18

근은 조선 전람회에 입선하 다 단 의(6 )

만종은 인간과 자연이 엮어 가는 경건한 조화 을lsquo rsquo

나타낸다

34) 정답[ ] ①

해설 근이 속에서도 창작활동을 추 않고[ ]

하는 닭은 은 세상과 타협할 르는

근이 세상의 이해를 하 위한 가장 떳떳한 단

이 때 이다

35) 정답[ ] ⑤

해설 전 은 서 자의 주 적인 평이 리는 것이[ ]

만 위 제 은 인 이 살았던 대 사회적 경

을 통해 객 적인 인 의 을 제 하고 있다

36) 정답[ ] ⑤

해설 전 은 인 사건 경 비평이라는[ ] lsquo rsquo⑤

성 이 어져 있다

37) 정답[ ] ①

해설 이 은 동양인과 서양인의 사고 에 차이가[ ]

있다는 것을 대조를 통해 설 하고 있다 또 쓴이

의 제자가 축 경 를 러 가서 경험한 일화를

통해 동양인이 서양인에 비해 주 상황에 더 많은

주의를 인다는 주장을 뒷 침하고 있다

38) 정답[ ] ④

39) 정답[ ] ②

40) 정답[ ] ②

41) 정답[ ] ④

42) 정답[ ] ③

43) 정답[ ] ④

44) 정답 도서 의 휴 일 도서 의 이 간 도서의[ ]

해설 도서 장은 임의 정한 휴 일과 도서 이[ ]

간 도서의 상 등을 게 할 의 가 있다

년 학 간고사 대비2013 2 현대고 대비

ECN-0102-2013-001-000076193

45) 정답[ ] ①

해설 제 조의 정 휴 일 의 휴 일의 사전 게[ ] 3

는 도서 장의 의 조항에 속한다

46) 정답[ ] ①

해설 개인 정 호 의 를 제 하 했 만 항[ ]

나눠서 제 하 않고 대 나열하고 있다

47) 정답[ ] ②

해설 제 조의 내 을 회사는 다른 회사 협[ ] 7 lsquo

계약을 통해 서비 를 제공하는 경 회 의 아이디

등 개인 정 를 해당 회사에 전송할 있다는 내rsquo

이 있으 의 제점을 제 할 있다②

48) 정답[ ] ④

해설 는 도서 장의 의 에 해당하고 나 는 도[ ] ④

서 장의 리에 해당한다

49) 정답[ ] ③

50) 정답 은 음독으 적었고 은 훈독으 적었[ ] (1)

다 과 동일한 표 리 적은 것은 이고 (2) ce

과 동일한 표 리 적은 것은 이다ab

51) 정답[ ] ③

52) 정답[ ] ①②

53) 정답[ ] ③

54) 정답[ ] ③

55) 정답[ ] ①

56) 정답 른 죠코 어린 노 하니라[ ] A B

57) 정답 세 어에서는 활 형이 칙적으[ ] lsquo rsquoㄹㅇ

나타났 만 개화 어에서는 활 형이 쓰 다 lsquo rsquo ㄹㄴ

58) 정답 호 가 흔[ ] (1) (2)

59) 정답[ ] ④

60) 정답[ ] ③

Page 14: 현대고대비 국어 - chamsoriedu.com 「콘텐츠산업진흥 법」외 에도 저작권 의하여 ... 다른주체에게어떤동작을하도록만드는것을나타내는

년 학 간고사 대비2013 2 현대고 대비

ECN-0102-2013-001-000076193

고 그 과 결 여 사 가 다 그러 강

승상 에게 울린 상 강 승상 귀 가고

과 헤어 리 승 만 게 다

승 우 다릴 과

들고 략 다 담 원

여 에게 복 고 어 (天

공격 다 담에게 여러 가 복) 子

등 여 다 단신

리쳐 담 사 고 에게 간

후 태후 태 여 지에 고생 지

심과 강 주 여 개 다 헤어

어 니 내 고 담 리 벼

슬에 귀 누리게 다

미상- lsquo ( )-劉忠烈傳

zb23) 위 과 의 서사 조를 비 한 것으 적절하lt gt

않은 것은

보lt gt

믿지 고 결 여 곱

낳 다 곱째 공주 낳 가

리게 다 리 만 고 진 공주는 lsquo rsquo

리공 미 리공 에 키워진다 월

러 과 가 죽 병에 걸 는 승에 는

어 산다고 다 여 들에게 탁

지만 거 리 는다 리 는 과

승 다 승 지 가는 에 많

만 지만 보살 도움 사 도 다

그러 승 신과 결 여 시

들어 주겠다고 다 리 는 그 결

여 들 곱 낳 후에 신

얻게 다 돌 리 는

에 과 상여 만 지만 여 과

살 낸다 훗 리 그 공 우 죽 사

승 도 는 신 다

리-lsquo rsquo-

① 복 결말에 고 다

② 웅 에 탕 고 다

③ 시 겨 내고 귀 누리는lsquo rsquo

보 리 는 월 재 신 다lt gt lsquo rsquo

④ 과 보 리 는lsquo rsquo lt gt lsquo rsquo

도움과 어 신 능 극복 고

⑤ 등 여 시 겪는lsquo rsquo

보 리 는 닌 지lt gt lsquo rsquo

림 시 겪는다

가 각 고 에( ) ( )却說

살 없었다 략 사 들 슬 에 어 lt gt

가에 내 고 가고 싶 가 고 후

워 경

사 들 별 고 없 다니었다 lt

략 얼 말 죽 사 같고 림새가 말gt

니었다 가슴 에 고 등

삼태 헌 에 니 달 가 ( )奇男子

도리어 걸 었 담 만 열 도( )傅說

고 만났고 만 갈( ) ( ) 殷 武丁

도 탕 만났( ) ( ) (伊尹 成湯 渭

여상 도 주 만났는) ( ) ( ) ( )水 呂尙 周 文王

월 같 러가 도 어느 열 살

늘과 집 삼고 사 에 쳐 거리에

어 다가 곳에 니 다 ( ) 楚

지 다가 사 보고 가에 다다( )長沙

니 망 가에는 원 리가 슬 고 가

가 내리는 사 에는 갈매 가 갈 뿐 었다

쪽 돌 보니 가 우거 고

가 사 보 었다 그곳에

가니 는 사( ) 汨羅水

는 다 주 가 쓰고 죽고

곳 었다

에는 강 주 는 재상 살고( )

었 니 시 에 과거에 격 여 승상 벼슬 다

가 간신 만 벼슬 그만 고 고 돌

었다 략 강 승상 마 본 에 갔다가 돌 lt gt ( )本府

는 에 우편 주 에 다가 색( ) ( ) 右便 酒店

에 어리었는 청룡 에 지

늘 여 통곡 고 사 는 꿈 꾸

었다 마 상 게 생각 여 새 다리다

가 새벽닭 울고 달 갔다 가

보니 과연 어 동 가 가에 울고 는지

달 들어 그 고 사 에

어 말

는 어 어 에 어 가ldquo

닭 곳에 우느냐rdquo

니 울 그 고 답 여 말 다 lt

략gt

년 학 간고사 대비2013 2 현대고 대비

ECN-0102-2013-001-000076193

생각 여 가 고 시 는ldquo ( )大人

상에 다시없는 니다 살 엇 겠습니

에 돌 가시고

가에 돌 가 니 살 마 없습니

다 략 어 없어 강 승상 가니rdquo lt gt

그곳 월계 었다

다 강 승상에게는 들 없고 다만 만( )

었다 가 낳 에 가 색

타고 내 에게 말

는 니다 미원 과ldquo ( )紫微垣

연 맺고 었는 께 강 집( )緣分

보내 에 니 게 여겨 주십시

rdquo

거늘 미 가운 낳 니 가

고 거동 단 다 시 짓 쓰 고

는 없었 니 여 가운( ) 音律

지 는 짝 룰 만 사 없었다 가 사

여 사 감 게 고 지 못 고 염 는 만다

다가 당에 거 고 식같 러

내니 고귀 상 루 말 다 어 ( )相

울 도 다 귀 사 없고 ( )富貴爵祿

웅 걸 만고 었다 승상 매우 뻐 내

당 들어가 에게 사 니 역( ) 內堂

시 매우 거워 말 다

도 마 사 는 승상께ldquo

그 게 말 시니 상 여러 말 지 말고 사

도 시다 략 시 택 여rdquo lt gt

니 다운 신 과 신 습 늘에 죄

짓고 간 상에 내 신 다

다 내고 들어가 사 살펴보니

고 것 는 다 말 어 고

는 다 어 신 에 ( )新房

에 신 과 신 가 평생 연 맺었( )緣分

니 사 주고 말 어떻게 다 헤 릴

어떻게 다 리 지낸 후에 튿 승

상 니 승상 거운 마 지 못

( ) 듯 월 러 생 열다 살

었다 에 승상 어진 사 얻고 만 에 근심

없었 다만 주 가 간신

에 죽 것 생각 마 곧 어

곤 다 그 에 주 원통

어 없 고 여 시 가 거늘 략 lt gt

략 거리

강 승상 에게 상 리지만 여움

사 귀 가게 다 강 승상 몸 는

연 과 헤어 리 다

마 각 생 강 승상 집 쪽( )

늘 보고 없 가 신 신 생각 니

없고 어 없었다 는 어떻게 도리가 없다

여 산 에 들어가 리 고 어 도 닦

고 다 그 산 보고 가

다가 곳에 다다 니 에 큰 산 었다 많

우리 골짜 가 늘 는 가운 색

에 고 갖가지 가 짝 어 ( )花草

었다 략 주 보니 lt gt ( ) (一柱門 黃

산 룡사 어 었다) lsquo rsquo 金大字

산 들어가 고승 다 그( ) ( ) 山門 高僧

거동 보니 눈 눈 듯 고

변 같 귀는 어 에 늘어 니( ) 白邊

맑고 어 골격과 신 평 니었

다 염주 에 걸고 짚고 포 ( )六環杖

삼에 어진 쓰고 생 보고 말

승 연 여 상공 시는 동 에ldquo

가 맞 지 못 니 승 십시 rdquo

생 크게 말 다

생 가 여 어 고ldquo

없 다니다가 우연 곳에 사 만 것

그 시 생 어떻게 고 습니

rdquo

승 답 여 말

어 산 승 에ldquo ( ) ( )南岳 衡山

시어 승에게 탁 내 낮 시경에 경 lsquo 12

동 에 사는 심 들 가 것 니 내쫓

지 말고 습니다 마 승rsquo

다가 상공 림새 보니 경 사 에 보

습니다rdquo

생 그 말 듣고 편 고 편( )

슬 승 들어가니 여러 승 들

가워 다 승 에 들어가

후에 그 편 니 곳 경 었다 상( ) 仙境

고 신 편 다 후 는 승과

께 병 도 탐 고 경도 게( )兵書

게 었다 게 니 지 에 가객 ( ) ( )大明天地 佳客

년 학 간고사 대비2013 2 현대고 대비

ECN-0102-2013-001-000076193

없고 산 에 리 만 본 ( ) 廣德山

신 상 사 살 는 만

우고 늘 월 신 과 늘 ( )日月聖神

산 신 들 다 니 그 재( ) 名山神靈

주 민 누가 당 겠는가 낮 공

zb24) 다 에 해당하는 내 으 적절하( ) 않은 것은

① 강 티 통 당시 능 다

② 상계 지상계 경 는 원 계 드러

③ 실에 어 없는 실 가 타 는

④ 뛰어 재주 어 가진 고

등 다

⑤ 가 직 개 여 평가 내리는

편집 평 타 다lsquo rsquo

가 본격 가 동 것 지( )

다 단 상 에2003 lsquo rsquo

들어가 드럼 연주 다 취미 생 달리

들었다는 보 우 가 들ldquo

어 틱 린 도 다 고 말 다rdquo

경 는 가 망 없( ) lsquo

티 원 고 답 다 신과 같 시각rsquo

는 습 상상 만 도 감동

다 시각 연주 동시에

열 상 는

티 원 그런 열 경 럽다는 것 다

다 역시 엄청 다 본( )

에 복 들

고쳐 가고 다 신 에 얼

마 지는 고 리가 는 지도 생님

가 훈 고 많 고쳐 다

고 말 다

그러 직도 에 지 는 다 그는

체격 지 못 게 가 큰 만

체 운동 훈 과 께 체 늘 동 50

는 게 고 말 다

에게는 꿈 다 통 누 가( )

주겠다는 것 그 꿈 다 신 극복 는

과 에 큰 경험 들도 느 게

주고 싶다는 것 다

마 슬 마다( ) ldquo 통

낼 었 것 럼 고통 는 사 들

고 겠다 고rdquo

말 다 달 루 첫 낸 lsquo rsquo

첫 드 심 집에 는 리듬 드 2

루 에 도 보고 싶다 집 에는 직(RampB) 3 4

사 곡 도 보 고 싶다고 포 다middot

zb25) 에서 가장 유사한 의 를 닌 어를lt gt

찾아 쓰

lt gt

나는 이제 너에게도 픔을 주겠다

사랑 다 소 한 픔을 주겠다

겨 거리에서 개 놓고

살아 추위 떨고 있는 할 니에게

값을 으 서 뻐하던 너를 위하여

나는 픔의 평등한 얼 을 여 주겠다

내가 어둠 속에서 너를 를 때

단 한 도 평등하게 어주 않은

가마니에 덮인 동사자가

다 얼어 죽을 때

가마니 한 장조차 덮어주 않은

한 너의 사랑을 위해

흘릴 르는 너의 눈 을 위해

나는 너에게 이제 너에게도 다림을 주겠다

지 울 포동 여고 생들17

틈 없 가득 체 에 맑 울

다 죽 듣 생들 사 에

연 는 탄 다 객들 도 는lsquo rsquo

가 보 주 공 맹 가 운 는

단 그룹사운드 루 보컬 맡고 는lsquo rsquo

시각 지 었다17 1

근 다만과 가 거lsquo rsquo lsquo

꿈 고 퇴 내가 다rsquo

간 간에 지 지 연 생들 짧lsquo rsquo lsquo rsquo

가 운 듯 리에 어

연 다 내 사 고 퇴lsquo rsquo

과 루 들 결 다시 돌lsquo rsquo

들 고 사 들 에 당당

것 니다 내 태어

볼 없었 크고 열여

년 학 간고사 대비2013 2 현대고 대비

ECN-0102-2013-001-000076193

에도 고 시 얻지 못 다

감지 없는 시각 상태 다

신 지에 고 상 원망 도

단다 어느 가 에 시각 에 ldquo

어 그런 듣고 다 보니 내가 게 lsquo

살 는지 도 눈 고 싶rsquo lsquohelliphellip

보 는 생각만 들 고 그 가 들에게rsquo

도 내고 들도 고 많 었죠 들 rdquo

었 지 새 는 에 쑥 러운 색

어났다

생에 것 단연 었다lsquo rsquo

공연에 거 꿈lsquo rsquo

는 다 특 가사 갑게 는 운 lsquo

벽 에 당당 마주 어 언 가 그 벽

고 늘 어 거운 상도

없죠 내 삶 에 웃 그 께

는 다고 다rsquo

들었 그냥 런 도 고만 여ldquo lsquo rsquo

겼죠 그런 꾸 가사 미 새 다 보

니 통 는 가사 는 생각 들 고 (

가 게는 시각 는 생각 들고 들) ( )

마다 듣고 큰 얻었어 rdquo

에 진지 게 가에 미 가

zb26) 의 에 들어갈 말 적절한 것은lt gt ~

lt gt

난 난 꿈이 있었죠

고 찢겨 남 하여도

내 가 히 과 같이 간 했던 꿈

혹 때 누 가가 뜻 를 비 음

내 등 뒤에 흘릴 때도

난 참아야 했죠 참을 있었죠

날을 위해

늘 걱정하듯 말하죠

헛된 꿈은 독이라고

세상은 끝이 정해 책처럼

이 돌이킬 없는

현 이라고 helliphellip

래 난 난 꿈이 있어

꿈을 믿어

나를 켜

저 차갑게 서 있는 이란 앞에

당당히 마주칠 있어

출처 가 거위의 꿈 작사 이적 작곡 동률- lsquo rsquo ( )

① ② ③ ④ ⑤

가 떴다는 들 만 지만( ) lsquo rsquo

늘 겸 다 에 주 연 우승 지 간에도 3

단 생님께 만 지 고 고 만ldquo rdquo

큼 늘 겸 신 계 가

고 다

에게는 꿈 다 통 누 가

주겠다는 것 그 꿈 다 신 극복 는 과

에 큰 경험 들도 느 게 주

고 싶다는 것 다

슬 마다 통 낼ldquo

었 것 럼 고통 는 사 들

고 겠다 고rdquo

말 다 달 루 첫 낸lsquo rsquo

첫 드 심 집에 는 리듬 2

루 에 도 보고 싶다(RampB) 집 에는 직34

사 곡 도 보 고 싶다고 포 다

미 는( ) (26) 어 헤헤헤 웃다가 어ldquo rdquo

허허허 웃었다ldquo rdquo ldquo rdquo 같 도 고

상 다 는 같 도 다( ) 壯丁 킹 들lsquo

다 는 역도 보 그 다 지만 그는rsquo

뷰에 지 다 운동만 지 ldquo

것 지 간에 여러 사 도 역rdquo helliphellip

었다 그런 엇 그 마 움직 는지 보 쯤

지 담 사 다 훈 없어 그는 티

지 림 었다 태 다 갔다 는 습

마 집 럼 편 게 보 다

주말에는 주 엇 보내

주말에도 별 주 에 청ldquo

고 에 가고 도 쳐

에 듣고 보 에 갈 가 별 없

어 산 시 게 고 들어 2002

거 매 여 지냅니다 시 과 지훈 rdquo

다 근 간 과 진실 그리고 싶어( )

가 다 근에게 그것 진리 다 거 다 없

거 고 다 없 는 것 진리

다 근 진리는 후 쪽 었다 신산( )辛酸 삶

었 질곡( )桎梏 역사 에 지냈 가

눈에 든 것 료 단 료 게 보

것 었다 그것 그 에 겨우겨우

슬 슬 생 어가는 간들 었다

리 과 단 리 고리에 검 마

없 거리 돌

상 것 없는 등 근에게 상

과 진실 엄 ( )儼存 다는 사실 리는 가

실 고 가 과 역경 에 도 근 내 포

없었 후 보루( )堡壘 다 도 365

도 간 근 여

시 것 다

년 학 간고사 대비2013 2 현대고 대비

ECN-0102-2013-001-000076193

다 공주 그림 가 근 경- ( ) ldquo rdquo(

2009)

zb27) 작가의 주 적인 각이 드러난 것은~

① ② ③ ④ ⑤

가 신 지에 고 상 원망( )

도 단다 어느 가 에 시각 에 ldquo

어 그런 듣고 다 보니 내가 lsquo

게 살 는지 도 눈 고 싶rsquo lsquohelliphellip

보 는 생각만 들 고 그 가 들에게rsquo

도 내고 들도 고 많 었죠 들었rdquo

지 새 는 에 쑥쓰러운 색

어났다 략 [ ]

경 는 가 망 없 티lsquo

원 고 답 다 신과 같 시각rsquo

는 습 상상 만 도 감동

다 시각 연주 동시에

열 상 는 티

원 그런 열 경 럽다는 것 다 략 [ ]

슬 마다 통 낼ldquo

었 것 럼 고통 는 사 들

고 겠다 고rdquo

말 다 달 루 첫 낸 lsquo rsquo

첫 드 심 집에 는 리듬 2

루 에 도 보고 싶다 집 에는 직(RampB) 3 4

사 곡 도 보 고 싶다고 포 다

식 누 가-

고 싶어

다 역도 미 담 고 사( )

질 주말에는 주 엇 보내[ 1]

답 주말에도 별 주 에[ ] ldquo

청 고 에 가고 도 쳐

에 듣고 보 에 갈 가 별

없어 rdquo

질 계 고 슬슬 도 는 것 닙니[ 2]

답 다 들 눈 에 보 고 뿐 보[ ] ldquo

다 열심 고 어 상에 도 들지만 상

지키는 것 들다고 에 도달

그것 지키 훨 많 rdquo

질 들 살 고 리 는[ 3]

거운 들 체 리느 는다

답 가 고 게 체 어[ ] ldquo ( ) 級

느 도 계가 니 살 는 것도 고역 지만

살 우는 것 들어 는 살

체 리 고 어도 어도 실 갔다

쑥 어 rdquo

질 거리에 슷 연 여 들[ 4]

보는 간 상 지

답 상 다 체 게 리지 못[ ] ldquo

거 주변에 는 그 거 누 보지

못 고 뻐지고 싶 에 체 리는 에

타 워 지만 는 어울 는 것보다 는

시간 운동만 는 건 니에 사복 lsquo rsquo

고 사복 는 말에 들 웃지만 늘 운동복

고 지내니 사러 갈 도 어 rdquo

질 역도가 말 단 식 운동 니[ 5]

답 가 내는 만 클 업 보[ ] ldquo

그러니 만 쓰는 식 운동 니다

만 다고 거운 것 들 는 건 니거든 연

도 고 가지 동 에 도 여러 가지

복 들

보식 역도 여 미-

zb28) 가 에 대한 설 으( ) 않은 것은

① 시각 우 지 시 에 지

고 망 가는 태도 달 고 다

② 언어 과 언어 복 사 여

담 내 생각 게 는 가

③ 직 감 그 마 것

럼 생생 게 느껴지는 과 주고 간 내

없 리 어 억 게 다

④ 담 내 식 리 여 담 삶 습

과 가 시 여 독 에게 감동과 훈 다

⑤ 직 진 담 직 누

지 못 는 독 에게 생생 상 달 주고

담 욱 게 다

zb29) 나 의 각 의 의도를 설 한 것으 적절하( ) 않

년 학 간고사 대비2013 2 현대고 대비

ECN-0102-2013-001-000076193

은 것은

① 질 담 상 보여 주 것 다1

② 질 담 과 그에 삶 태도 보여2

주 것 다

③ 질 역도 겪는 어 움에 역도3

과 것 다

④ 질 같 연 여 갖는 고민 는지 말4

주 는 것 다

⑤ 질 역도가 과 고 운동 는 것5

담 가 말 주 는 것 다

가 만진 것 다( ) 3

감 달 다고 다 억 에( ) 音感

지워 지만 당시 청 탁 리도

다고 다 드럼 웠다 4

에 갈 마다 드럼 는 리가 신 게 들

다고 다 눈 볼 가 없 니 엔ldquo

는 는 님 틱 에 여 주

다 드럼과 연 맺 과 들 주었다rdquo

식 누 가-

고 싶어

역( ) 도가 말 단 식 운동 니

가 내는 만 클 업에 보ldquo

그러니 만 쓰는 식 운동 니다 만

다고 거운 것 들 는 건 니거든 연

도 고 가지 동 에 도 여러 가지 복

들 시 는 상 상

드는 상 에 맞춰 실 에 는 여러

펼쳐집니다rdquo

략( )

늘 에 는 어 만 것 같

가 에 사 고 사 사ldquo

겠어 든 에 가 경 만 고

울 는 사 겠어 rdquo

보식 역도 여 미-

다 가 운 는 어 어( ) ldquo rdquohelliphellip

월 새벽 시 태 없 거웠고1965 5 6 1

는 없 그 병원에 퇴원 집

가는 마지막 마 고 마 내 거 다

가 죽 간신 에 실 다 사는 어느5 lsquo

가 죽 는 말 가 식 다 신rsquo

상에 각 시키는( )刻印 에 실

어느 가는 후 민 가가 근 었다lsquo rsquo

는 간 과 진실 그 다는( ) ldquo

에 단 평 견 가지고 다 내

가 그리는 간상 단 고 다 지 다 는 그들

가 에 는 평 지 니 그리고 어린

들 미지 겨 그린다rdquo

마 근 간 과 진실 그리고 싶어( )

가 다 근에게 그것 진리 다 거 다 없

거 고 다 없 는 것 진리

다 근 진리는 후 쪽 었다 신산(辛酸 삶)

었 질곡(桎梏 역사 에 지냈)

가 눈에 든 것 료 단 료 게

보 것 었다 그것 그 에 겨우겨우

슬 슬 생 어가는 간들 었

다 리 과 단 리 고리에 검

마 없 거리 돌

상 것 없는 등 근에게 상에

과 진실 엄 다는 사실 리는 가( )儼存

실 고 가 과 역경 에 도 근 내

포 없었 후 보루(堡壘 다 도)

도 간 근365

여 시 것 다

월 강원도 림리에( ) 1914 2 21

삼 독 태어났다 어 근 복

그것 그리 가지 못 다 근 곱 살

지는 산 산업에 실 고 답마 에 내

갔다 근 그림 럼 쫓 다니 가 시 것

다 상 진 것도 가 었다

러 가 에도 고 근 가 꿈꾸었다 근

가 꿈꾸게 것 보통 업

원색도1926 만lsquo rsquo 었다

공주 그림 가 근 경-

zb30) 에 대한 설 가장 른 것은~

① 역도가 과 운동 도 질

② 리는 는 다 lsquo rsquo

③ 들었지만 그럭 럭 는 다 lsquo rsquo

④ 가 게 보 시 말 다

⑤ 보 병 는 지 상 lsquo rsquo

는 말 다

년 학 간고사 대비2013 2 현대고 대비

ECN-0102-2013-001-000076193

시간 많지 다 청량리 생 병원

마지막 상 경 릿 게 들어 다 그 는 십

만 큰 가 상 말 다

지 못 들 마 갈 고 돗

도시민들 싹 싹 탔다 가 시

월에 병원에 원 가 폐 진 몸도4 ( )疲弊

갈 미 지 못 고 었다 가는 얼마( ) 解渴

지 생 에 생각 가

마감 는 신 평생 십 만에

가 과 많 닮 다고 생각 지는

가 운 는 어 어ldquo rdquo 1965helliphellip

월 새벽 시 태 없 거웠고 는5 6 1

없 그 병원에 퇴원 집 가

는 마지막 마 고 마 내 거 다 가

죽 간신 에 실 다 사는 어느 가5 lsquo

죽 는 말 가 식 다 신rsquo

상에 각 시키는 에 실 어느( ) lsquo刻印

가는 후 민 가가 근 었다rsquo

ldquo 는 간 과 진실 그 다는 에

단 평 견 가지고 다 내가 그

리는 간상 단 고 다 지 다 는 그들 가

에 는 평 지 니 그리고 어린 들

미지 겨 그린다rdquo

근 간 과 진실 그리고 싶어 가

다 근에게 그것 진리 다 거 다 없 거

고 다 없 는 것 진리다

근 진리는 후 쪽 었다 신산 삶 ( )辛酸

었 질곡 역사 에 지냈 가 눈에( )桎梏

든 것 료 단 료 게 보 것

었다 그것 그 에 겨우겨우 슬

슬 생 어가는 간들 었다 리

과 단 리 고리에 검 마

없 거리 돌 상

것 없는 등 근에게 상에 과 진실

엄 다는 사실 리는 가 실( )儼存

고 가 과 역경 에 도 근 내 포 없었

후 보루 다 도 도( ) 365堡壘

간 근 여 시 것

간에 지닌 가 근 1914 2

월 강원도 림리에 삼 독21

태어났다 어 근 복 그것 그리

가지 못 다 근 곱 살 지는 산

사업에 실 고 답마 에 내 갔다 근

그림 럼 쫓 다니 가 시 것 다 상

진 것도 가 었다 러 가 에도

고 근 가 꿈꾸었다 근 가 꿈꾸게

것 보통 업 원색1926

도 만 었다lsquo rsquo

그림 가 근 경 공주- ldquo rdquo ( 2009)

zb31) 다음 이 같은 의 성 소에 해당하 않은

것은

사건 평① ② ③

④ 주 ⑤ 경

가 운 는 어 어ldquo rdquo 1965helliphellip

월 새벽 시 태 없 거웠고 는5 6 1

없 그 병원에 퇴원 집 가

는 마지막 마 고 마 내 거 다 가

죽 간신 에 실 다 사는 어느 가5 lsquo

죽 는 말 가 식 다 신rsquo

상에 각 시키는 에 실 어느( ) lsquo刻印

가는 후 민 가가 근 었다rsquo

는 간 과 진실 그 다는 에ldquo

단 평 견 가지고 다 내가 그

리는 간상 단 고 다 지 다 는 그들 가

에 는 평 지 니 그리고 어린 들

미지 겨 그린다rdquo

근 간 과 진실 그리고 싶어 가

다 근에게 그것 진리 다 거 다 없 거

고 다 없 는 것 진리다

근 진리는 후 쪽 었다 신산 삶 ( )辛酸

었 질곡 역사 에 지냈 가 눈에( )桎梏

든 것 료 단 료 게 보 것

었다 그것 그 에 겨우겨우 슬

슬 생 어가는 간들 었다 리

과 단 리 고리에 검 마

없 거리 돌 상

것 없는 등 근에게 상에 과 진실

엄 다는 사실 리는 가 실( )儼存

고 가 과 역경 에 도 근 내 포 없었

후 보루 다 도 도( ) 365堡壘

간 근 여 시 것

간에 지닌 가 근 1914 2

월 강원도 림리에 삼 독21

태어났다 어 근 복 그것 그리

가지 못 다 근 곱 살 지는 산

사업에 실 고 답마 에 내 갔다 근

그림 럼 쫓 다니 가 시 것 다 상

진 것도 가 었다 러 가 에도

고 근 가 꿈꾸었다 근 가 꿈꾸게

것 보통 업 원색1926

도 만 었다lsquo rsquo

공주 그림 가 근 경- ldquo rdquo ( 2009)

년 학 간고사 대비2013 2 현대고 대비

ECN-0102-2013-001-000076193

zb32) 위 을 작성하는 과정에서 되어 활 된 자

어 것은

신 사 료① 연보②

고③ ④ 들과 담

⑤ 에 평

는 간 과 진실 그 다는 에ldquo

단 평 견 가지고 다 내가 그

리는 간상 단 고 다 지 다 는 그들 가

에 는 평 지 니 그리고 어린 들

미지 겨 그린다rdquo

근 간 과 진실 그리고 싶어 가

다 근에게 그것 진리 다 거 다 없 거

고 다 없 는 것 진리다

근 진리는 후 쪽 었다 신산 삶 ( )辛酸

었 질곡 역사 에 지냈 가( )桎梏

눈에 든 것 료 단 료 게 보

것 었다 그것 그 에 겨우겨우

슬 슬 생 어가는 간들 었다

리 과 단 리 고리에 검 마

없 거리 돌 상

것 없는 등 근에게 상에 과

진실 엄 다는 사실 리는 가 실( )儼存

고 가 과 역경 에 도 근 내 포

없었 후 보루 다 도 도( ) 365堡壘

간 근 여 시

것 다

간에 지닌 가 근 1914 2

월 강원도 림리에 삼 독21

태어났다 어 근 복 그것 그리

가지 못 다 근 곱 살 지는 산

사업에 실 고 답마 에 내 갔다 근

그림 럼 쫓 다니 가 시 것 다 상

진 것도 가 었다 러 가 에도

고 근 가 꿈꾸었다 근 가 꿈꾸게

것 보통 업 원색1926

도 만 었다lsquo rsquo

질 루 마 가 도 린다 경건

움 느껴지는 경 다 훗 근 그림에

과 는 거 것( )裸木

만 간과 연 엮어 가는 경건 움lsquo rsquo

니었

같 가가 고 싶었 근에게 그 꿈에 다

가가는 지 다 다 가 지망생들 규 미

상 에 진 고

에 지만 근 다 다 근

미 에 운 것 보통 시 미 시간

다 그런 그에게 없는 연습 가가

통 다 가 귀 시 지 도

얻는 뛸 듯 뻤지만 마 도 가 에

듯 는 었 에 어린 근 주 에

에 그림 그리고 지우고 복( )粉板

시간 가는 게 루 보냈다

근 그 갈 가가 것 열여( )渴求

었 다가 미1932 lsquo rsquo ( lsquo

미 에 다 다는 고 마rsquo) lsquo rsquo

가 근 집 고도 지는 시골 경

그린 그림 다 후 근 에 1943 22

지 미 에 그림 고

에 걸쳐 다 미 근 가

동 는 었다

공주 그림 가 근 경- ldquo rdquo ( 2009)

zb33) 위 의 내 과 일치하는 것은

가 근 가 꿈 포 다①

근 당 가들과 께 에 다②

살 근 가 걷20③

게 었다

④ 만 통 근 역경 겨내는lsquo rsquo

느 다

⑤ 근 간 과 진실 그리 에 그 에

드러 는 간상 단 다

계 시 주 근 건강

걸었다 신 과 간에 상 다 건강

신 는 눈에도 다 근 쪽 눈 뿌 게

보 지 과에 다 다 시 지지 고 결

내 었다 시 지만 마 막막

다 늦어 결 근 쪽 눈 고 말 다

쪽 눈 근에게는 쪽 눈 었고

계 었다 그 근 는 여 그lsquo rsquo

다 근 에 같 그림 그 었다1950

시 그림 는 여 쪽lsquo rsquo

고 어 마주 고 는 그림1963

여 과 동 다 마 복

그린 듯 눈 내리 새 게 다 지

사 다 근 게 복 것

복 상과 타 는 근 상

가 떳떳 단 었고 근 그리고

간 과 진실 에 다가가 가 근다

운 었다 근 신에게 당당 지 그리고

그 다 근 그림에 단 복 보다

년 학 간고사 대비2013 2 현대고 대비

ECN-0102-2013-001-000076193

태 도 그리고 극 보다 과

얻 여 었다 과 통

근 그리고 는 재 고 에 질

만들고 특 것 다

공주 그림 가 근 경- ldquo rdquo( 2009)

zb34) 의 이유에 대해 추 한 것으 적절하 않은 것

상과 타 시도①

보다 과 얻②

근 신에게 당당 지③

④ 간 과 진실 에 다가

⑤ 태 도 얻

근 가가 었지만 그 다니 가

럼 어지지 다 복과 쟁 거쳐 시

는 가 근에게 생계 사 에

운 사 다 에 키에 건( ) 178cm死鬪

체 근 에 동 역 업( )荷役

가 생계 다 쟁

에는 동에 운 상우 주 미

죄 사 에 그림 그리는 시 다 그곳에

에 동 역 업 것에

결 것 럼 보 다 지만 그런 것만도

니었다 그림 그리는 고는 지만 매 근

는 극 간 과 별 없는 경 리 그림

벽에 그리는 것 었다 우도 리 없었다 근

트 는 우 그림 그 다 생

계 그림 단 것 다

후 근 지 신 계 리에 미

엑 리 겼다 근 곳에

건 사 크 에 미 들 ( )

상 상 그 다 근 갖 다 겪

냈다 그리고 결 그 돈

신동에 어 사리 집 마 다 마 ㄷ

루 심 쪽에는 과 엌 쪽에는 건

었다 건 주고 근 가 에

여 살 다 심 에는 지 집어

쓰고 지만 곳 근 가 에게 러웠

보 리 다 근 과 마루 업실 삼 그림

그 다 신동 마루는 근 그림에 등 는 lsquo rsquo

같 상들 지 다 시 고

에 들 폐허가

가 업실 었다

공주 그림 가 근 경- ldquo rdquo( 2009)

zb35) 위 에 대한 설 으 적절한 것은

업 시 여 훈과 감동 다①

에 주 평 드러 다②

사 사 등 식 과 ③

④ 다 근거 시 여 삶에

⑤ 살 시 사 경 께 여

습 시 다

가 시간 많지 다 청량리 생 병원( )

마지막 상 경 릿 게 들어 다 그 는

십 만 큰 가 상 말 다

지 못 들 마 갈 고 돗

도시민들 싹 싹 탔다 가 시

월에 병원에 원4 가 폐( )疲弊

진 몸도 갈 미 지 못 고 었다( )解渴 가는

얼마 지 생 에 생각

가 마감 는 신 평생 십 만에

가 과 많 닮 다고 생각 지는

가 운 는 어 어( ) ldquo rdquohelliphellip

월 새벽 시1965 5 6 1 태 없 거웠고

는 없 그 병원에 퇴원 집

가는 마지막 마 고 마 내 거 다

가 죽 간신 에 실 다 사는 어느5 lsquo

가 죽 는 말 가 식 다 신rsquo

상에 각 시키는 에 실( )刻印

어느 가는 후 민 가가 근 었다lsquo rsquo

다 는 간 과 진실 그 다는( ) ldquo

에 단 평 견 가지고 다 내

가 그리는 간상 단 고 다 지 다 는 가

에 는 평 지 니 그리고 어린 들

미지 겨 그린다rdquo

근 간 과 진실 그리고 싶어( )

가 다 근에게 그것 진리 다 거 다 없

년 학 간고사 대비2013 2 현대고 대비

ECN-0102-2013-001-000076193

거 고 다 없 는 것 진리

다 근 진리는 후 쪽 었다 신산( )辛酸 삶

었 질곡 역사 에 지냈( )桎梏

가 눈에 든 것 료 단 료 게 보

것 었다 그것 그 에 겨우겨우

슬 슬 생 어가는 간들 었다

리 과 단 리 고리에 검

마 없 거리 돌

상 것 없는 등 근에게 상에

과 진실 엄 다는 사실 리는 가 실( )儼存

고 가 과 역경 에 도 근 내 포

없었 후 보루 다( ) 堡壘 도 365

도 간 근 여

시 것 다

마 같 가가 고 싶었 근에게 그 꿈( )

에 다가가는 지 다 다 가 지망생들

규 미 상 에 진 고

에 지만 근 다 다 근

미 에 운 것 보통 시 미 시간

다 그런 그에게 없는 연습 가가

통 다 가 귀 시 지 도

얻는 뛸 듯 뻤지만 마 도 (

는 었 에 어린 근 주 에)

에 그림 그리고 지우고( )粉板

복 시간 가는 게 루 보냈다

zb36) 전 의 성 소가 아닌 것을 고르

① 평 ② 사건 ③ 경

④ ⑤ 훈

늘 지 상에 살고 는 사 들 억 도가10

고 그리 지 통 고 는 사 들( )知的

그보다 훨 많 억 도는 고 지 20

통 다 그런 지 고 2500

그리 간 보는 과 사 에

매우 달 뿐만 니 과 에 도 극

루고 었다 미 운 그런 들

살고 는 동 과 사 들 사고 식에

큰 가 다는 다

고 그리 들 우주 개별 고 독립

사 들 생각 지만 고 들 우

주 연 질 간주 다 같( ) 看做

각 도 들에게는 연 질

었지만 그리 들에게는 미 들 결 었

다 고 과 그리 들 사 같

는 동 과 사 에 도 견 다

지심리 미 마 드 겐트 는

살 들에 에 지 다

연 동 과 상 다 과 같 실험

다 크 만든 미드 도 보

여 주고 그 상 닥 고 주었다lsquo (Dax)rsquo

실 닥 는 재 지 는 것 실험 가lsquo rsquo

만들어 낸 다 그런 다 개 다 체 보

여 주었는 는 미드 지만 틱

만들었고 다 는 재료는 크 지만

달 다 그러고 어 것 닥 지 사 들에게 고 lsquo rsquo

게 니 들 주 같 고 는

체 택 고 동 들 같 재료 만들어진 체

택 다 러 는 심지어 살짜리

들에게 도 타났다 것 곧 과 동

다 상 보고 다는 것 미 다

개별 사 보고 고 동 연 질 보

고 는 것 다

동 들 주변 상 에 맞 어 동 고

에 다 사 들 태도 동에 보다 많

주 울 다 동 가 미시간 에

에 경험 다 그는 미식

경 보러 가게 었는 경 체는 매우 재미 었

주변 들 동에 질 다 그 는

들 계 어 상태 경 다

어 들 에 에 그 시 가 계 가

진 것 다 상 살펴 는 말 들 lsquo rsquo

에 그는 에 시 어 도 뒷사

생각 곧 다시 곤 것 다 그런 그에게 뒷

사 고 지 는 들 동 럼

어 웠다

생각 지도 리 드 니 벳-

zb37) 다음 위 의 내 전개 으 만 인lt gt

것은

lt gt

대조의 통해 대상이 닌 특성을 설 하고 있다

일화를 제 하여 자 의 주장을 뒷 침하고 있다

유추의 을 사 하여 독자의 의해를 돕고 있다

대상이 형성되는 과정을 간적 서에 따라 서 하고 있

① ②

③ ④

년 학 간고사 대비2013 2 현대고 대비

ECN-0102-2013-001-000076193

가 우리가 말 고 쓰는 든 단어가 사 에 는( )

것 니다 사 격에 가 는 지만

어 사 과 같 특별 는 사 니lsquo rsquo

단어 격 보 단어가 사 에

등재 어 다 리 리 사 는 단어 도 그

것 시 사 는 어 고 사 에

격 보 것 니다

러 얼 은 사전에 를 있는가 이에 대한 답lsquo rsquo

은 얼 이 유행어인가 아닌가에 따라 갈라 다 이 단어lsquo rsquo

는 년 어 자 에 랐고 쓰이고 있으2002 lsquo rsquo

유행어라고 하 에는 생 이 다 런데 계속

을 유 하 서 사전에 등재될 자격을 획득할 것인가 이

에 대한 답을 내리 는 히 어 다

여 서 가 를 고 해 볼 있다 첫 는 이 단어

를 써야 할 필 가 속적으 있는가 하는 점이다

상주의 열풍에 휩 인 사회 위 에 편 해서 퍼 말

이 얼 인데 과연 런 위 가 속될 것인가 이에lsquo rsquo

대해 필자의 생각은 정적이다 사회 위 가 뀌

런 말을 쓸 일이 없어 것이다

다음은 단어의 성이다 단어의 성이 사회적으 거

감이 없으 계속 사 될 가능성이 높다 런 에서

얼 은 좋은 조건이 아니다 익히 알 졌듯이 이lsquo rsquo

말은 얼 과 청소년층에서 속어 사 하는 이 결합lsquo rsquo lsquo rsquo

된 말이다 얼 에서 얼 을 리하는 조어 도 lsquo rsquo lsquo -rsquo

어에서는 매 낯선 이다 이것만으 도 거 감을 갖

는 사람들이 있다 더 나 속어 결합한 말이다 얼 lsquo rsquo

이 널리 퍼졌다 해도 은 여전히 청소년층의 속어lsquo rsquo

남아 있다 속어는 자연 럽게 아 자리에서나 쓰 에는

담 러 말이다 러한 담을 하고 사

역을 넓혀 가는 속어도 없 는 않다 특히 얼 은 lsquo rsquo

에도 종종 등장한다 만큼 거 감이 많이 희석되었다

고 할 있다 러나 일상의 자연 러 대화에서도 거

리낌 없이 등장하는가 게 는 되 않았다고 생

각한다

얼 이 유사어인 쌈 등을 만들어 내고lsquo rsquo lsquo rsquo

있으니 살아남을 있을 것이라고 는 견해도 있을 것

이다 러나 간이 나 서 유사어를 포함하여 든

말이 사라 사 는 많다 유사어가 많다는 것이 생 을

유 할 있는 절대적인 조건은 아니다

나 언젠가 터 사람들은 어느 단에서 얼 이 가장( )

쁜 사람을 가리켜 얼 이라고 르고 있다 이 얼lsquo rsquo lsquo rsquo

이라는 단어가 최근 어사전에 라 항간에 논란이 일고

있다 아닌 게 아니라 얼 은 유행어처럼 인다 생 lsquo rsquo

도 리 래되 않은 것 같고 언제 사라 도 알

없다 게다가 젊은이들 사이에서 주 쓰일 뿐이다 이런

단어를 사전에 는다는 게 하 이 없어 이 도

한다

러나 속단은 이다 차근차근 따져 볼 일이다

선 얼 이 일 적 유행어인 아닌 주의 게 들여다lsquo rsquo

볼 필 가 있다 유행어란 유행에 따라 빠르게 유포되었

다가 단 간 내에 소 되는 단어나 를 가리킨다

얼 은 인터넷을 통해 속히 퍼 말이다 하 만 일lsquo rsquo

적인 유행어처럼 단 간 내에 사라 않았을 뿐 아니라

현재 도 잦은 빈도 사 되고 있고 앞으 도 상당

간 사 될 것으 측된다 한 언 재단의 뉴 검 lsquo rsquo

색 사이트에 따르 얼 은 년 에 처음 나타난lsquo rsquo 2001

이후 꾸 히 사 되고 있다

이 같은 사 빈도는 얼 이 일 적 유행어 는 현lsquo rsquo

저히 다르다는 것을 여 다 장 간의 생존 만으 도

얼 은 이 한 어의 어휘 에 를 자격을 얻었다lsquo rsquo

고 할 있다 더 이 이라는 비 적 정제된 매체에

높은 빈도 쓰이고 있 않은가 사 빈도 측 에서

필통이나 연필과 같은 단어 대등하거나 더 많이 쓰lsquo rsquo lsquo rsquo

다는 것은 결코 가 게 볼 일이 아니다

이제는 사전이 언어 현 을 빠르게 하는 게 덕인

대가 되었다 세계적으 유 한 의 사전들도 경쟁

적으 어를 고 있다

하 만 얼 은 젊은이들이나 쓰는 속어라고 흠을 잡을lsquo rsquo

도 르겠다 얼 이 주 젊은 층에서 많이 쓰 lsquo rsquo

는 속어임에 틀림없다 러나 어사전에 표 적이고 품

위 있는 말만 어야 한다고 생각한다 것은 커다란

해다 당장 아 어사전이나 펼쳐 라 속어는

설과 같은 비어나 죄자들이 쓰는 은어 어

마니 같은 소 의 사람만이 쓰는 말 도 라 있

않은가 사전은 말 치에 일정 빈도 이상 나타나는 말이

라 말이든 다 할 있다

zb38) 가 나 에 대한 다음의 설( ) ( ) 않은 것은

① 가 는 얼짱 사 에 등재 것에( ) ( ) lsquo rsquo

보 고 다

② 사 등재 가는 단어 격에( )

고 고 는 언 들 언어 사 도에 고 다 ( )

③ 가 얼짱 어지만 신 과 같 매( ) ( ) lsquo rsquo

체에 도 사 는 말 는 고 다

④ 가는 얼짱 어 보고 크게 가지 근( ) lsquo rsquo 3

거 들어 뒷 고 다

⑤ 는 얼짱 어 는 다 특 다는( ) lsquo rsquo

근거 에도 크게 가지 근거 가 들어 주 2

뒷 고 다

가 늘 지 상에 살고 는 사 들 억( ) 10

도가 고 그리 지 통 고 는 사 들

그보다 훨 많 억 도는 고 지 20

통 다 그런 지 고 2500

년 학 간고사 대비2013 2 현대고 대비

ECN-0102-2013-001-000076193

그리 간 보는 과 사 에

매우 달 뿐만 니 과 에 도 극

루고 었다 미 운 그런 들

살고 는 동 과 사 들 사고 식에

큰 가 다는 다

고 그리 들 우주 개별 고 독립

사 들 생각 지만 고 들 우

주 연 질 간주 다 같 각

도 들에게는 연 질 었지

만 그리 들에게는 미 들 결 었다

고 과 그리 들 사 같 는

동 과 사 에 도 견 다

인 리학자인 츠 이마이 디드 겐트너는 두

살이 채 안 된 아이들에서 터 성인에 이르 다양한

연 대의 동양인과 서양인을 대상으 다음과 같은 험

을 했다 저 코르크 만든 피라 드 양의 도형을

여 주고 대상의 이름을 닥 라고 알 주었다lsquo (Dax)rsquo

제 닥 는 존재하 않는 것으 험자가 임의lsquo rsquo

만들어 낸 이름이다 런 다음 두 개의 다른 체를

여 주었는데 하나는 피라 드 양이 만 하얀 플라 틱

으 만들었고 다른 하나는 재 는 코르크 만 양이

달랐다 러고 나서 어떤 것이 닥 인 사람들에게 고 lsquo rsquo

르게 했더니 서양인들은 주 같은 양을 하고 있는

체를 선택했고 동양인들은 같은 재 만들어 체를

선택했다 이러한 차이는 성인은 어 두 살 리

아이들에게서도 나타났다 이것은 곧 서양인과 동양인은

서 다른 세상을 고 있다는 것을 의 한다 략 ( )

는 아주 단 하 서도 인상적인 험을 했다

험에는 동서양의 대학생들이 참여했다 는 험 참가자

들에게 컴퓨터 화 을 통해 속 장 을 담은 애니 이션

을 여 주었다 화 의 앙에는 초점의 역할을 하는 커

다란 고 한 마리가 있었고 주위에는 다른 생

들과 초 자갈 거품 등이 함 제 되었다 화 을

두 씩 후 참가자들은 자 이 것을 회상해 라는

를 았다

결과 서양인 대학생들과 동양인 대학생 두 앙

의 초점 역할을 했던 고 를 동일한 정도 언 했으

나 경 소 위 거품 초 다른 생 들 에 ( )

대해서는 동양인 대학생들이 서양인 대학생들 다 60

이상 더 많이 언 했다 뿐만 아니라 동양인 학생들은 서

양인 학생들에 비해 개 적인 고 다 전체적인 계

를 더 언 하는 경향을 다 략 또한 경의 일 ( )

를 화 킨 림을 제 하 을 때 동양인 대학생들은 대

경의 화를 알아챘 만 서양인 대학생들은 경

의 화를 거의 알아차리 했다 략 ( )

따라서 서양인들만을 대상으 연 한 화lsquo

편성 결 은 잘 된 것일 도 있다 각 과정과 인rsquo

과정의 어떤 이 화 편적이고 어떤 이

화에 따라 달라 는 는 앞으 많은 연 를 통하여 논의

되어야 한다

나 어떤 의 에서 리 두는 이 화적이다 리( )

안에는 다른 사람들과 더 친 한 계를 유 하 는 상호

의존성과 다른 사람들 터 독립적인 존재 살아가 는

독립성이 혼재한다 따라서 이 에서 어떤 특성이 더 강

하게 각되는 상황에 놓이느냐에 따라 서 다른 화적

특 을 일 있다 결 리 두는 어떤 경 에는

동양인처럼 행동하고 어떤 경 에는 서양인처럼 행동하는

것이다

zb39) 가 에 대한 다음의 설( ) 않은 것은

① 는 신 주 뒷 닥 실험과lsquo rsquo lsquo

니 실험 근거 시 다rsquo

② 동 들 상 간 공통 보다는 에 식

는 강 다

③ 들 주변 맥 에는 심 경 어 사건

과 사건 사 계에 상 민감 다

④ 는 동 과 틀린 지 고 는 것lsquo rsquo

니 다 고 다 lsquo rsquo

⑤ 가에 우리 사 들 개 시 가 원( )

집 경 말 고 는 것 개 보다는

에 고 는 것에 다

늘 지 상에 살고 는 사 들 억 도가10

고 그리 지 통 고 는 사 들( )知的

그보다 훨 많 억 도는 고 지 20

통 다 그런 지 고 2500

그리 간 보는 과 사 에

매우 달 뿐만 니 과 에 도 극

루고 었다 미 운 그런 들

살고 는 동 과 사 들 사고 식에

큰 가 다는 다

지심리 미 마 드 겐트 는 동

과 상 다 과 같 실험 다

크 만든 미드 도 보여 주고 그

상 닥 고 주었다 그런 다lsquo (Dax)rsquo

개 다 체 보여 주었는 는 미드

지만 틱 만들었고 다 는 재료는

크 지만 달 다 그러고 어 것 닥 lsquo

지 사 들에게 고 게 니 들 주 같rsquo

고 는 체 택 고 동 들 같

재료 만들어진 체 택 다 러 는

심지어 살짜리 들에게 도 타났다 것

곧 과 동 다 상 보고 다는

것 미 다 개별 사 보고 고 동

년 학 간고사 대비2013 2 현대고 대비

ECN-0102-2013-001-000076193

연 질 보고 는 것 다

동 들 주변 상 에 맞 어 동 고

에 다 사 들 태도 동에 보다

많 주 울 다 동 가 미시간

에 에 경험 다 그는 미

식 경 보러 가게 었는 경 체는 매우 재

미 었 주변 들 동에 질 다 그

는 들 계 어 상태 경

다 어 들 에 에 그 시 가 계

가 진 것 다 뒷사 고 지 는 들

동 럼 어 웠다

그는 경험에 어 얻어 동 들lsquo

각도 상 본다 는 가 우고rsquo

검 여 주 단 도 상 실험 실

시 다 그는 실험 가 들에게 컴퓨 통

담 니 보여 주었다

에는 역 는 커다 고 마리가 었

고 주 에는 다 생 들과 갈 거 등

께 시 었다 본 후 가 들

신 본 것 상 보 는 지시 다

그 결과 생들과 동 생

역 고 동 도 언

경 거 다 생 들에 ( )

는 동 생들 생들보다 60

상 많 언 다 뿐만 니 동 생들

생들에 개별 고 보다 체 계

언 는 경 보 다 경 변 시

킨 그림 시 동 생들 경

변 지만 생들 경 변

거 리지 못 다

지 지 들만 상 연 lsquo

보편 결 못 것 도 다 지각 과 과rsquo

지 과 어 보편 고 어

에 달 지는지는 많 연 통 여

어 다

리 드 니 벳 생각 지도 사- ldquo rdquo( 2004)

zb40) 위 에 대한 설 으 가장 적절한 것은

① 동 과 생 식 강 고 다

② 가지 실험 통 쓴 고 다

③ 닥 실험에 사 본질에 동 사

상에 주 다

④ 니 실험에 동 과 에 지

각 도에 가 다

⑤ 쓴 는 보편 연 에 드러 우월 에

에 근 고 다

가 동 들 주변 상 에 맞 어 동 고( )

에 다 사 들 태도 동에 보다 많

주 울 다 동 가 미시간 에

에 경험 다 그는 미식

경 보러 가게 었는 경 체는 매우 재미 었

주변 들 동에 질 다 그 는

들 계 어 상태 경 다

어 들 에 에 그 시 가 계 가

진 것 다 상 살펴lsquo 는 말 들rsquo

에 그는 에 시 어 도 뒷사

생각 곧 다시 곤 것 다 그런 그에게

뒷사 고 지 는 들 동 럼

어 웠다

그는 경험에 어 얻어( ) 동 들lsquo

각도 상 본다 는 가 우고rsquo

검 여 주 단 도 상 실험

실시 다 실험에는 동 생들 여 다

그는 실험 가 들에게 컴퓨 통

담 니 보여 주었다 에는

역 는 커다 고 마리가 었고 주 에는

다 생 들과 갈 거 등 께 시

었다 본 후 가 들 신 본 것

상 보 는 지시 다

다 그 결과 생들과 동 생( )

역 고 동 도 언

경 거 다 생 들 에 ( )

는 동 생들 생들보다 60

상 많 언 다 뿐만 니 동 생들

생들에 개별 고 보다 체 계

언 는 경 보 다 들어 동

생들 상 체 연못 럼 보 어ldquo 같rdquo

체 맥 언 시 었지만

생들 상 어 같 큰 고 가 쪽 움ldquo

직 어 같 역 고rdquo

언 시 다 경 변 시킨 그

림 시 동 생들 경 변

지만 생들 경 변 거

리지 못 다

년 학 간고사 대비2013 2 현대고 대비

ECN-0102-2013-001-000076193

게 볼 동 들 보다는 큰 그( )

림 보 에 사 과 체 맥 연결시 지각

는 경 고 체에 특 떼어 내

어 독립 보는 것 낯 어 다 에

들 사 에 고 주변 맥 에는 심 경

에 사건과 사건 사 계에 상

민감 편 다

마 지 지( ) 들만 상 연

보편 결 못 것 도 다lsquo rsquo 지각 과

과 지 과 어 보편 고 어

에 달 지는지는 많 연 통 여

어 다

리 드 니 벳 생각 지도 사- ldquo rdquo( 2004)

zb41) 의 하는 가~ 다른 것은

① ② ③

④ ⑤

얼마 그 에 동 사고 식과

사고 식 보여 주는 내 다

들 에 는 탕 고 같 게

어 겨 고 미 에 는 그 크 럼 큰 고

어리 주고 원 는 어 도 는

상 고 생각 다는 것 다 러

는 어떻게 생 것 고 과 그리 거슬

러 가 보 그 단 다

고 연 경 체 경 생 에

다 벼 사는 공동 업과 경험 많 연 역

에 고 들 연 웃과

게 지내 고 탁 연 들

들 지 연 럽게 들 다 민들

웃과 동 게 뿐만 니 는 집 과

게 다

동 시 는 생태 경 에 살 결과

들 다 사 들 사 상 에 주

울 게 었고 는 곧 체 상 과 간 사

계 시 는 낳게 었다 신 가

가 는 체에 는 원 는 동시

에 다 사 들 그 사 포 체 맥 에

다 들 간 사 연

계 체 계에 주 울 는 사고 체계

게 었다

그러 그리 연 경 그 었다 산

지 연결 는 지 건 그리고 역

에 다 런 들 업에 다 사 과

동 므 공동체에

다고 다 고 그리 들

들과는 달리 보 내 감 지 들과

지 크게 느 지 못 다 그

견 다 경우 주 쟁 통 결 는 갖

게 었다

신 사 간 계들 루어진 커다

트워크 에 게 당연 사 역시 연

계들 체 식 게 다 어 상

원 도 그 개체가 체 맥 과

계 에 고 다 게 체 맥 에 주

울 다 보 상 복 과 가변 식 게 고

상에 재 는 많 변 들 사 에 재 는 들도

게 다 들 주 태도 보

는 경우가 많다 쟁 결

통 결 보다는 통 결

는 보 다

그러 고 그리 들 개개 사 사 독

에 주 울 다 사 사 체에

어 그들 사 에 재 는 공통 규 주

고 다 상 원 에도 사

체 내 주 고 다 그들

체 여 탕 체

는 주 태도 시 고 특 사 어

주에 는지 여 그 주에 는 규

견 다 에 는 쟁 식 리

같 리 사고 체계가 달 게 었다

리 드 니 벳 생각 지도 사- ldquo rdquo( 2004)

zb42) 위 에서 사 된 설 과 가장 유사한 것은

① 크톱 컴퓨 는 본체 니 마우 루

어 다

② 곡과 시 리 는 지 과 사 루어 다는 공통

지니고 다

③ 경 고 것과는 달리

경 본 연 태 그 주변 경

④ 벽돌 능 에 사계 내내

습도가 지 다

⑤ 잰느 체 체 지닌 재 체가 없

는 재 눌 다

년 학 간고사 대비2013 2 현대고 대비

ECN-0102-2013-001-000076193

zb43) 는 립 앙 도서 이 정의 일 이다lt gt

도서 장과 이 자의 리 의 정의 연결이

적절하 않은 것은

lt gt

제 조 서 유8 ( )

도서 장은 다른 이 자의 안전을 위협하거나 도서 의①

서를 란하게 할 가 있는 자에 대하여는 도서 출입

을 제한할 있다

도서 장은 이 자가 제 조 각 호의 어느 하나의 행위를 하7②

을 때에는 이 을 하게 하거나 도서 출입을 제한할

있다

제 조자 의 대출9 ( )

도서 자 는 다음 각 호의 경 대출할 있다①

상호대차도서 간에 자 를 류하는 것을 말한다 등 다1 ( )

른 도서 과의 협 을 위하여 필 한 경

공 이 공 행 상 필 하는 경2

에 도서 장이 필 하다고 인정하는 경3

대출이 가능한 도서 자 의 위는 도서 장이 정하는②

에 따른다

제 조 상10 ( )

이 자가 도서 자 설을 더럽히거나 찢거나 뜨①

쓰게 하거나 잃어 린 경 에는 상하여야 한다

도서 장은 제 항에 따른 상 을 정하여 게 하여야1②

한다

제 조이 절차 등11 ( )

이 칙에서 정한 것 에 도서 자 설의 이 절차

이 제한 등에 필 한 사항은 도서 장이 정한다

출처 립 앙 도서- (httpwwwnlgokr)

① 는 도 리 다8

② 도 는 리 다9 1

③ 료 지 는 도 리 다9 2

④ 도 료 변상에 리10 1

⑤ 는 에 도 리 다11

3

도 다 각 같다①

공 공 다만 연1

연 간 다

매월 째 째 월2

도 도 리 그 사3

가 다고 는

도 에 미리 게1 3②

시 여 다

4

도 시간 도 여 게시 다

5

도 료 시 는 는 도①

지에 등 후

등 에 사 도②

7

는 다 각 여 는 니 다

도 료 시 상 리1 lsquo rsquo

도 료 시 훼 는2 middot

지 가 닌 곳에 식 거 담3

우는

도 보 등 보 검색열4 middot

그 에 도 질 지 여 도5

여 게시 사 는

8

도 다 거 도①

질 게 우 가 는 에 여는 도

도 가 각 어느7②

에는 지 게 거 도

9

도 료는 다 각 경우 다①

상 도 간에 료 는 것 말1 (

다 등 다 도 과 여 경우)

공 원 공 상 는 경우2

그 에 도 다고 는 경우3

가능 도 료 는 도②

는 에 다

10

년 학 간고사 대비2013 2 현대고 대비

ECN-0102-2013-001-000076193

가 도 료 시 럽 거 거①

못 쓰게 거 어 린 경우에는 변상 여

도 에 변상 여 게시1②

여 다

zb44) 위 에서 도서 장이 게 해야 할 사항에 해당하는

것을 두 쓰

년 학 간고사 대비2013 2 현대고 대비

ECN-0102-2013-001-000076193

립 도 규

1 ( )

규 립 도 립 어린 청 도(

포 다 료 시 열 시 말) (

다 에 사 규 립 도)

편 진 다

2 ( )

규 립 도 도 다 에( lsquo rsquo )

고 는 도 에 도lsquo rsquo 2 2

료 에 여 다 다만 특 료 귀

료 등 료 에 사 립 도

도 다 다( lsquo rsquo )

3 ( )

도 다 각 같다①

공 공 다만 연1

연 간 다

매월 째 째 월2

도 도 리 그 사3

가 다고 는

도 에 미리 게1 3②

시 여 다

시간4 ( )

도 시간 도 여 게시 다

등 등5 ( )

도 료 시 는 는 도①

지에 등 후

등 에 사 도②

사 료6 ( )

도 료 시 에 사 료는 도

7 ( )

는 다 각 여 는 니 다

도 료 시 상 리1 lsquo rsquo

도 료 시 훼 는2 middot

지 가 닌 곳에 식 거 담3

우는

도 보 등 보 검색열4 middot

그 에 도 질 지 여 도5

여 게시 사 는

질 지8 ( )

도 다 거 도①

질 게 우 가 는 에 여는 도

도 가 각 어느7②

에는 지 게 거 도

료9 ( )

도 료는 다 각 경우 다①

상 도 간에 료 는 것 말1 (

다 등 다 도 과 여 경우)

공 원 공 상 는 경우2

그 에 도 다고 는 경우3

가능 도 료 는 도②

는 에 다

변상10 ( )

가 도 료 시 럽 거 거①

못 쓰게 거 어 린 경우에는 변상 여

도 에 변상 여 게시1②

여 다

등 규 에 것 에 도11 ( )

료 시 등에 사

도 다

립 도- (httpwwwnlgokr)

zb45) 도서 장의 리 있는 조항으 적절하 않

은 것은

① ② ③ ④ ⑤

년 학 간고사 대비2013 2 현대고 대비

ECN-0102-2013-001-000076193

1 ( )

사가 공 는lsquo rsquo

과 여 사 원과 리

사 타 사 규

니다

개 보 보7 ( )

사는 보통신망 등 계 는 에lsquo rsquo lsquo rsquo

원 개 보 보 니다 개lsquo rsquo

보 보 사 에 는 사 개lsquo rsquo

보 취 니다 다만 사는 다 lsquo rsquo

사 계 통 공 는 경우 원 lsquo rsquo

등 개 보 당 사에 습니lsquo rsquo

원 리에8 (lsquo rsquo lsquo rsquo lsquo rsquo

)

원 에 리lsquo rsquo lsquo rsquo lsquo rsquo①

원에게 가 도 여 는lsquo rsquo 3

니다

사는 원 가 개 보 우 가lsquo rsquo lsquo rsquo lsquo rsquo②

거 사 경우 는 미 에 어 거 lsquo

사 사 운 우 가 는 경우 당rsquo lsquo rsquo

습니다lsquo rsquo

원 가 도 거lsquo rsquo lsquo rsquo lsquo rsquo 3③

가 사 고 지 경우에는 시 사에lsquo rsquo

통지 고 사 내에 니다lsquo rsquo

경우에 당 원 사에 그 사실3 lsquo rsquo lsquo rsquo④

통지 지 거 통지 도 사 내에 지 lsquo rsquo

생 경우 사는 지지 습니다lsquo rsquo

사10 (lsquo rsquo )

사는 과 지 미lsquo rsquo①

에 는 지 계 고

공 여 다 여 니다lsquo rsquo

사는 원 게lsquo rsquo lsquo rsquo lsquo rsquo②

도 개 보 신 보 포 보 보 시( )

갖 어 개 보 취 공시 고

니다

사는 과 여 원lsquo rsquo lsquo rsquo③

견 만 당 다고 경우에는

리 여 니다 원 견 만 사 lsquo rsquo

에 는 게시 거 우편 등 통 여

원에게 리 과 결과 달 니다lsquo rsquo

원11 (lsquo rsquo )

원 다 여 는 니다lsquo rsquo ①

신청 는 변경 시 허 내 등1

타 보 도2

사가 게시 보 변경3 lsquo rsquo

사가 보 보 컴퓨 그4 lsquo rsquo (

등 등 신 는 게시)

사 타 등 지 재산 에5 lsquo rsquo 3

사 타 상 거 업6 lsquo rsquo 3

는 폭 시지 상 타 공7 middot middot

에 는 보 에 공개 는 게시 는lsquo rsquo

사 동 없 리 사8 lsquo rsquo

타 거 당9

게시15 (lsquo rsquo )

원 내에 게시 는 게시 게재 는lsquo rsquo lsquo rsquo lsquo rsquo

경우 원 사가 게시 복 lsquo rsquo lsquo rsquo lsquo rsquo middot middot

등 태 언 등에 공 는

것 내에 다 원 본 게시 등 lsquo rsquo lsquo rsquo

크 능 등 여 복 는 등 태

는 것 동 것 니다

- (wwwnavercom)

zb46) 위 은 인터넷 포털사이트의 회 가입을 위한 이

약 의 일 이다 이 약 을 만드는 과정에서 생각한

내 으 적절하 않은 것은

개 보 보 가 지에 별 눠①

겠어

원 가 만들게 에②

시 주어 겠어

원들 게재 게시 다 원 크 다③

는 것 지

④ 원 지 는 뿐만 니 사가 지 는

도 께 달 지

리에 가 생 경우 사가⑤

에 다는 도 듯

1 ( )

사가 공 는lsquo rsquo

과 여 사 원과 리

사 타 사 규

년 학 간고사 대비2013 2 현대고 대비

ECN-0102-2013-001-000076193

니다

개 보 보7 ( )

사는 보통신망 등 계 는 에lsquo rsquo lsquo rsquo

원 개 보 보 니다 개lsquo rsquo

보 보 사 에 는 사 개lsquo rsquo

보 취 니다 다만 사는 다 lsquo rsquo

사 계 통 공 는 경우 원 lsquo rsquo

등 개 보 당 사에 습니lsquo rsquo

원 리에8 (lsquo rsquo lsquo rsquo lsquo rsquo

)

원 에 리lsquo rsquo lsquo rsquo lsquo rsquo①

원에게 가 도 여 는lsquo rsquo 3

니다

사는 원 가 개 보 우 가lsquo rsquo lsquo rsquo lsquo rsquo②

거 사 경우 는 미 에 어 거 lsquo

사 사 운 우 가 는 경우 당rsquo lsquo rsquo

습니다lsquo rsquo

원 가 도 거lsquo rsquo lsquo rsquo lsquo rsquo 3③

가 사 고 지 경우에는 시 사에lsquo rsquo

통지 고 사 내에 니다lsquo rsquo

경우에 당 원 사에 그 사실3 lsquo rsquo lsquo rsquo④

통지 지 거 통지 도 사 내에 지 lsquo rsquo

생 경우 사는 지지 습니다lsquo rsquo

원에 통지9 (lsquo rsquo )

사는 특 다 원에게 통지 경우lsquo rsquo lsquo rsquo

공지 게시 통 상 게시 개별 통지에7

갈 습니다

사10 (lsquo rsquo )

사는 과 지 미lsquo rsquo①

에 는 지 계 고

공 여 다 여 니다lsquo rsquo

사는 원 게lsquo rsquo lsquo rsquo lsquo rsquo②

도 개 보 신 보 포 보 보 시( )

갖 어 개 보 취 공시 고

니다

사는 과 여 원lsquo rsquo lsquo rsquo③

견 만 당 다고 경우에는

리 여 니다 원 견 만 사 lsquo rsquo

에 는 게시 거 우편 등 통 여

원에게 리 과 결과 달 니다lsquo rsquo

원11 (lsquo rsquo )

원 다 여 는 니다lsquo rsquo ①

신청 는 변경 시 허 내 등1

타 보 도2

사가 게시 보 변경3 lsquo rsquo

사가 보 보 컴퓨 그4 lsquo rsquo (

등 등 신 는 게시)

사 타 등 지 재산 에5 lsquo rsquo 3

사 타 상 거 업6 lsquo rsquo 3

는 폭 시지 상 타 공7 middot middot

에 는 보 에 공개 는 게시 는lsquo rsquo

사 동 없 리 사8 lsquo rsquo

타 거 당9

원 계 규 내lsquo rsquo lsquo②

여 공지 주 사 사가 통지 는rsquo lsquo rsquo

사 등 여 타 사 업 에 lsquo rsquo

는 여 는 니다

- (wwwnavercom)

zb47) 위 약 의 조항에서 같은 제점을 하lt gt

고 있는 조항은

lt gt

제휴 회사에 회 의 아이디 개인 정 를 전송할 있도

한 조항은 고객에게 당한 조항이다

1 7 8① ② ③

④ 9 ⑤ 10

립 도 규

1 ( )

규 립 도 립 어린 청 도(

포 다 료 시 열 시 말) (

다 에 사 규 립 도)

편 진 다

2 ( )

규 립 도 도 다 에( lsquo rsquo )

고 는 도 에 도lsquo rsquo 2 2

료 에 여 다 다만 특 료 귀

료 등 료 에 사 립 도

도 다 다( lsquo rsquo )

3 ( )

도 다 각 같다①

공 공 다만 연1

연 간 다

년 학 간고사 대비2013 2 현대고 대비

ECN-0102-2013-001-000076193

매월 째 째 월2

도 도 리 그 사3

가 다고 는

도 에 미리 게1 3②

시 여 다

시간4 ( )

도 시간 도 여 게시 다

등 등5 ( )

도 료 시 는 는 도①

지에 등 후

등 에 사 도②

사 료6 ( )

도 료 시 에 사 료는 도

7 ( )

는 다 각 여 는 니 다

도 료 시 상 리1 lsquo rsquo

도 료 시 훼 는2 middot

지 가 닌 곳에 식 거 담3

우는

도 보 등 보 검색열4 middot

그 에 도 질 지 여 도5

여 게시 사 는

질 지8 ( )

도 다 거 도①

질 게 우 가 는 에 여는 도

도 가 각 어느7②

에는 지 게 거 도

료9 ( )

도 료는 다 각 경우 다①

상 도 간에 료 는 것 말1 (

다 등 다 도 과 여 경우)

공 원 공 상 는 경우2

그 에 도 다고 는 경우3

가능 도 료 는 도②

는 에 다

변상10 ( )

가 도 료 시 럽 거 거①

못 쓰게 거 어 린 경우에는 변상 여

도 에 변상 여 게시1②

여 다

등 규 에 것 에 도11 ( )

료 시 등에 사

도 다

립 도- (httpwwwnlgokr)

zb48) 다음 정 리 의 의 으 볼 때 가장

이 적인 것은

도 시간 도 여 게시 다①

등 에 사 도②

가능 도 료 는 도 는③

에 다

④ 도 에 변상 여 게10 1

시 여 다

⑤ 도 가 각 어느7

에는 지 거 도

zb49) 를 참고하여 이 어의 성격을 설 한lt gt

것으 적절하 않은 것은

① 보 에 는 어 시 상 고 어 시lt gt lsquo rsquo

에 보여주고 다

② 진 어 어원에 견 고 다

에는 타 어 들어가는 것 다 lsquo rsquo

③ 에 들어갈 말 각각 고 어 어 신 어~

들 언어는 질 격 강 통 없었다

④ 시 우리 에 가 었지만 지 계

과 달리 들 통 사 달 어 웠

년 학 간고사 대비2013 2 현대고 대비

ECN-0102-2013-001-000076193

⑤ 크 몽골 만주 공통어가 우리 어 같

계열에 다는 에 사 특 짐

가( )

善化公主主隱 공주님

他密只嫁良置古 몰 결 고

薯童房乙 맛

夜矣卯乙抱遣去如 에 몰 고 가다

( )

始汝 會隱日恚見隱扐 만 에 본

恥隱汝衣淸隱笑 맑 웃

고 시 여 공 크다 만 다[ ] ( ) ( ) ( ) ( )始 汝 會扐

내다 에 보다 견( ) ( )恚 見 다( )隱

럽다 맑다 청 웃( ) ( ) ( ) ( )恥 衣 淸 笑

zb50) 위의 나 를 함 고 음에 답하( ) lt gt

보lt gt

( )素那或云金川 白城郡蛇山人也

운 사산

는 고 다 는( )[ ( ) ] (素那 金川 白城

사산 사 다) ( ) 郡 蛇山

삼 사- lsquo rsquo 47

에 제 된 단어 의 표 리를 조건(1) lt gt ( ) lt gt

에 맞게 서 하

건lt gt

lsquo 었고 었다 태rsquo

에 제 된 단어 동일한 표 리에(2) lt gt ( )

의해 적은 것을 나 에서 찾아 조건 에 맞게 서 하( ) lt gt

건lt gt

에 당 는 각각( ) 개 쓸 것2 단

당 는 가 여러 개 어도 개만 쓸 것 각2

개 과 도 쪽에 개만2 2

드시 지 것( )

과 동 원리 것lsquo 고

과 동 원리 것 다rsquo

태 것

가( )

素那(或云金川) 白城郡蛇山人也

소나 또는 천 이라 한다 는 성 사( ) ( ) ( )素那 金川 白城郡〔 〕

산 사람이다 현대어 풀이( ) ( )蛇山

나( )

紫布岩乎希 회

執音乎手母牛放敎遣 자 손 암쇼 노히 고

吾 不喩慙 伊賜等肹 肹 나 안디 리샤

花 折叱肹 可獻乎理音如 고 것거 도림다

다 향찰은 리말을 리 으 적은 표 이었 만 생( )

은 고 대를 넘 하고 끊어 고 말았다 랜 세

동안 갈고 닦아 체계적이었던 향찰 표 이 사라졌

을 인은 크게 두 가 나누어 생각해 볼 있다

하나는 족 사회의 한 선호도에서 찾을 있다 라 때

향찰은 주 족 계 에서 사 했을 것으 인다 한 을

알 하고서는 한자를 활 하여 리말을 리 으 표

하 란 가능하 때 이다 런데 족들은 간이 흐

를 향찰과 같은 리 표 을 익혀 사 하 다는

아 한 을 대 사 하는 쪽을 선호하게 되었다 더 이

고 초에 인재 등 을 위해 과거제도가 행되 서 한 선

호도가 더 높아졌고 결 향찰은 소 되고 말았다

또 다른 가능성은 한 어의 특성에서 찾을 있다

터 한 과 일 세 나라는 한자 화 에 속해 다

당연한 이야 겠 만 표의 자인 한자는 어를 표 하

에 매 적절하다 어의 음절은 성 ( ) ( )聲母 韻母

이 어 고 여 에 성조가 추가되어 최종 소리가 결정된

다 래서 어는 단음절을 하나의 한자 표 하 된

다 에 초성 성 종성의 세 가 소가 하나의 음절

년 학 간고사 대비2013 2 현대고 대비

ECN-0102-2013-001-000076193

을 이 는 한 어는 음절 조가 잡하고 음절의 가 많아

서 한자 차 만으 한 어의 소리를 만족 럽게 표 할

없었다 를 들어 한 어에서는 어 니 같이 음절 lsquo rsquo

이 어 단어가 얼마든 있으나 어는( ) 複數音節

자 하나 나타내 만이다lsquo [m ]rsquo 母 ǔ

한편 일 어의 표 은 핵 적 단어는 한자 적고 토는

가나라는 일 의 자 적는 이다 적인 의 를 나

타내는 은 표의 자인 한자 적고 적 계를 나

타내는 토는 표음 자 적는 셈이니 자세히 살펴

리의 향찰 표 을 쏙 빼닮았음을 알 있다 한 어 같

은 착어이 서도 일 어에만 향찰과 유사한 표 이 살아

남은 것은 일 어의 특 때 이다 일 어는 하나의 자음과

음의 결합으 음절을 이 고 침이 거의 없는 음절 언어

이다 이러한 음절의 특색에다가 토가 달한 착어라는 점

이 향찰과 유사한 표 이 살아남을 있는 비결이었다

하 만 같은 착어라도 다양한 음소 침이 달한 한

어는 향찰 표 하는 데 근 적으 한계가 있었다

zb51) 다 하여 의 행에 대한 탐 한 결과( ) lt gt 2

않은 것은

보lt gt

善花公主主隱 공주니믄 공주님( )

----------------------------------------

-

他密只嫁良置古 그 지 얼어 고 몰 결(

----------------------------------------

-

薯童房乙 맛 맛( )

夜矣卯乙抱遺去如 몰 고 가다 에 몰 고(

가다)

주동 역 동- (薯童謠『 』

에 2 ( )他密只嫁良置古

얼다 시집가다 결 다 말 lsquo rsquo

① 실질 미 지니고 므 타 타lsquo ( )rsquo lsquo [ ]

② 에 실질 미 타내고 지 는lsquo rsquo lsquo [ ]rsquo lsquo [ ]密只 密 只

계 타내는

③ 얼어는 실질 미 포 고 므 가lsquo rsquo lsquo [ ]rsquo嫁

것lsquo [ ]rsquo 良

④ 고 어간 는 실질 미 지니고 므lsquo rsquo lsquo -rsquo

것lsquo [ ]rsquo 置

⑤ 고 어미 고는 계 타내고 므lsquo rsquo lsquo- rsquo

고 것lsquo [ ]rsquo 古

가( )

엉 훈 민middot middot middot middot middot世 宗 御 製 訓 民 正 音

말 미 듕 귁에 달middot middot middot middot middot middot middot middot中 國 文 字

니 런middot middot middot middot middot middot 어린middot middot middot middot百 姓

니 고 도 내 들middot middot middot middot middot middot middot middot middot 시러middot

펴 몯middot 미middot middot 니 내middot middot middot middot middot middot middot middot 爲

어엿middot 겨 새middot middot middot 믈여듧middot middot middot middot字 니middot middot middot

사 마다 니겨 킈 middot middot middot middot middot middot middot middot middot便 安

고 미니middot middot middot middot

본 는 상( ) (象

원리에 만들어진 본) ( )形 ㄱ ㄴ ㅁ ㅅ ㅇ

에 는 가 원리에( )加劃

그리고( )ㅋ ㄷ ㅌ ㅂ ㅍ ㅈ ㅊ ㆆ ㅎ

쓰는 병 원리에 만들어진( )竝書

마지막 체( ) ( )異體ㄲ ㄸ ㅃ ㅆ ㅉ ㆅ

ᅀ 다 상 원리에 ㅇ ㄹ

지 는 삼재 상 본 본( ) ( ) ( 天地人 三才

탕 므림과 림에 ) (初ㅡ ㅣ

재)( ) ( )( )出字 再出字ㅗ ㅏ ㅜ ㅓ ㅛ ㅑ ㅜ ㅕ

병 그리고 들 에 다시( )ㅘ ㅝ ㅣ

( )ㅣ ㅢ ㅚ ㅐ ㅟ ㅔ ㆉ ㅒ ㆌ ㅖ ㅙ ㅞ

zb52) 가 에 대한 설 으 르 않은 것을( ) 두 고르

① 어쓰 규 지키고 다

② 리 고 다

③ 말 미 미 등 어 사 다lsquo rsquo

④ 개 지 다

년 학 간고사 대비2013 2 현대고 대비

ECN-0102-2013-001-000076193

⑤ 어 원 에 가 도 고 다

엉 훈 민世 宗 御 製 訓 民 正 音

말 미 듕귁에 달 니

런 어린 니 고 도middot

내 들 시러 펴 몯 미 니middot

내 어엿 겨 새 믈여듧

사 마다 니겨middot 킈 고

미니

훈민 언 본- lsquo rsquo 5 (1459 )

zb53) 위의 에 대한 현대어 풀이가 르~ 않은 것

① 우리 말 과 달

② 어리 말 고 는 것 어도

③ 신 생각 마 껏 펼 는 사 많다

④ 게 생각 여

⑤ 사 마다 게

zb54) 훈민정음 언해 에는 한 을 창제한 동 가 드러나

있다 훈민정음 창제의 정 과 내 이 잘 연결된 것

① 주 신 말 미 듕귁에 달

② 민 신 내 어 겨

③ 신 뻔 킈 고 미니

④ 실 신 사 마다 니겨

⑤ 귀 신 계 주 는 훈민 신과 거리가

가 엉 훈 민( ) middot middot middot middot middot世 宗 御 製 訓 民 正 音 

말 미 귁에 中 國 달 文 字

니 런 어린 니 百 姓

고 도 내 들 시러 펴 몯

미 니 내 어엿 爲 겨 새

믈여듧 니 사 마다 니 字

겨 킈 고 미니 便 安

훈민 언 본- lsquo ( )rsquo ( ) 5 (1459 )訓民正音 世祖

( )

[ 1 ]

동 룡 샤 마다 복( ) ( ) ( )海東 六龍 天福

시니 고 동( ) ( )古聖 同符 시니

[ 2 ]

매 니 곶 여

미 므 니 그 내 러

가 니

[ 125 ]

우 미리( )千世 샨( )定 에( )漢水北 累仁

누 개 샤 복 업 시니( ) ( ) 開國 卜年

신( )聖神 니 샤도 경 근민 샤 욱( )敬天勤民

드시리 다

님 쇼 산 가( ) ( )洛水 山行

미드니 가

어 가- lsquo ( )rsquo 27龍飛御天歌

다 우리신 니쓰고 다만 만 쓰( )

거 샹 귀쳔 다보게 러 귀

여 쓴 도 신 보 가 고 신 에

말 어 보게 각 에 사 들

고 본 몬 능통 후에

죠 죠 니

드 도 만 공 에 사

드 미 죠 고 고 여 보 죠

보다 얼마가 거시 어신고 니 첫

가 죠 니 죠

민 들 어 신 샹

귀쳔 도보고 어보 가 만 늘

고 폐 에 만쓴 죠 민

도 러보지못 고 보니 그게 엇지

심 니 리 보 가 어 운건 다

니 쳣 말마 지 니 고 그

쓰 에 가 우 지 지

몰 거 본후에 가 어 지

고 그니 쓴편지 쟝 보

년 학 간고사 대비2013 2 현대고 대비

ECN-0102-2013-001-000076193

쓴것보다 듸 보고 그 마 니 쓴 고

어 못

그런고 에 리 과 가

만 쓴 못 민 말만 듯고

고 편 그 못 보니 그사 단

병신 못 다고 그사 식 사

니 만 고 다 과 그사

만 고 다 과 업 사 보다 식 고

죠 도 고 각 과

견 고 실 직 귀쳔 간에 그

고도 다 것 몰 귀죡 보다

사 우리 신 귀쳔 다 업

시 신 보고 과 지 게 랴

시니 샹 귀쳔 간에 우리 신 걸

간 보 새지각과 새 걸 미리

독립신- lsquo (1896)rsquo

zb55) 친 어 나의 제 장( ) 2 매 함축적

의 가 가장 유사한 것은

① 지 눈 내리고 매 득 니 내 여 가

사- lsquo rsquo

② 도 어 리듯 그 게 어 다

주 사- lsquo rsquo

③ 눈 살 다 죽 어 린 과 체 여

눈 새벽 지 도 살 다

눈- lsquo rsquo

④ 삶 근심과 고단 에 돌 거니는 여 거 는

여 리 내린 살가지 에 눈 리 눈 리

택 그 생 에- lsquo rsquo

⑤ 늘 러 고 러

청룡 룡 어 개 루 우

신경림 계- lsquo rsquo

zb56) 친 를 위 가 나 에 나타난A B ( ) ( )

세 어의 특 에 의거하여 세 어 표 하

그 산 고 공 도 맑지만

A

주변에 쓰 리는 어리 사 많다

B

건lt gt

식 가 에 타 어 특징에( ) ( )

거 과 어쓰 는 고 지 말 것

A

B

zb57) 가 의( ) 달 아ㆍ 다 의 ( ) 나셔에서 알 있는

세 어 개화 어의 특 을 비 하여 조건 에lt gt

맞게 서 하

건lt gt

어에 는lsquo 개

어에 는 다 태rsquo

zb58) 은 가 는 다 에 나 는 절lt 1gt ( ) lt 2gt ( )

일 를 췌한 것이다 의 의 가 lt 1gt (1)~(2)

유사한 말을 에서 찾아 쓰lt 2gt

보lt 1gt

런 (1) 어린 니 고百 姓

도 내 들 시러 펴 몯 미

사 마다 (2) 니겨 便 安

킈 고 미니

보lt 2gt

죠 고 고 여 보 죠

보다 얼마가 거시 어신고 니 첫 가

죠 니 죠 민

들 어 신 샹 귀쳔

도보고 어보 가 만 늘 고

폐 에 만쓴 죠 민 도

러보지못 고 보니 그게 엇지 심

니 리

년 학 간고사 대비2013 2 현대고 대비

ECN-0102-2013-001-000076193

lt 1 gt

동 룡 샤 마다 복 시( ) ( ) ( )海東 六龍 天福

고 동 시니( ) ( )古聖 同符

lt 2 gt

(A) 매 니 곶

여 니

미 므 니 그 내

러 가 니

lt125 gt

우 미리 샨 에( ) ( ) ( ) 千世 定 漢水北 累

누 개 샤 복 업 시 니( ) ( ) 仁開國 卜年 聖

신( ) 神 니 샤도 경 근민 샤( ) 敬天勤民

욱 드 시 리 다

님 쇼 산 가 ( ) ( )洛水 山行

미드니 가

- lt gt龍飛御天歌

zb59) 장과 내 상 유사한 성격의 조는125

① 뫼 고 고 고 고

어 그린 많고 많고 고 고

어 러 는 울고 울고 가느니

도 견- lt gt

② 강 에 드니 몸 다

그믈 고 가니

뒷 뫼 엄 언 니( )藥

-

③ 말 없는 청산 태 없는 다

값 없는 청 없는 월

에 병 없는 몸 별 없 늙 리

-

④ 가마귀 골에 가지 마

낸 가마귀 새

청강에 것 시 몸 러 가( ) 淸江

-

⑤ 진 골에( ) 白雪

가 매 는 어느 곳에 었는고

에 갈 곳 몰( ) 夕陽

색-

zb60) 위 에 나타난 세 어의 특 으 적절하 않은

것은

① 룡 어 주격 사에 당 는 가 사( ) lsquo rsquo六龍

고 다

② 샤 어에도 어 주체 쓰 다

는 것 다

③ 매 어 달리 사 택에 어

가 지 지지 고 다

④ 므 원 상 직 어 지 다

⑤ 드시리 다 주체 과 상 께 사

고 다

수고 하셨습니다hearts hearts

년 학 간고사 대비2013 2 현대고 대비

ECN-0102-2013-001-000076193

보닷컴에 공 는 별 보는 고등

들 여 주 는

들 습니다 슷 동 지

가 복 는 것 도가

니 복 여 습 시고 거 시

니다

정답 해설

1) 정답[ ] ④

해설 다른 것은 두 특정 업이나 단 내에서 사[ ]

하는 일종의 은어 사회 언에 해당한다 러나

는 언이 아니라 단과대학을 여서 단대 사lsquo rsquo lsquo rsquo lsquo④

대학을 여서 사대라고 한 말에 해당하 일rsquo lsquo rsquo

사회에서도 널리 쓰이 사회 언이라 할

없다

2) 정답[ ] ⑤

해설 사회 언은 같은 단 내에서 쓰이는 언어이[ ] lsquo rsquo

동일 단끼리는 단결 과 친 감을 형성하는

능을 하 리적 안감이 일어나 않는다

3) 정답[ ] ③

해설 사람이라는 차 적 표현에 대한 대안적 표현이[ ]lsquo rsquo

인 아내 처 등으 볼 있다lsquo rsquo

4) 정답[ ]⑤

해설 남성은 주 격 체를 사 한다[ ]

5) 정답[ ] ⑤

해설 흑인은 검다라는 뜻을 가 고 있을 뿐 인[ ]lsquo rsquo lsquo rsquo lsquo rsquo

다 열등한 뜻을 내포하 않는다

6) 정답 살 색 첫 작품[ ] - -

해설 살색 혹은 킨색은 한 인의 피 색을 뜻[ ] lsquo rsquo lsquo rsquo

하는 것으 인종 차 을 추 고 출 이주민

의 평등 을 침해할 있어 년 표 이2005

살 색으 이름을 꾸었다 처녀작은 처녀라lsquo rsquo lsquo rsquo lsquo rsquo

는 단어가 가 고 있는 곡된 성 인 을 한 것

으 첫 작품정도 꾸어 사 하는 것이 좋다lsquo rsquo

7) 정답[ ] ⑤

해설 호는 아들에게 해체를 사 하고 있다[ ] ① ②

장 을 성하는 청자는 자 의 아 느리 아lsquo

들 세 이다 호는 아 느리에게 해rsquo ③

체를 사 하고 있다 호가 느리 아 에게 ④

사 한 해 체 아들에게 사 한 해체는 두 비lsquo rsquo lsquo rsquo

격 체에 해당한다 호는 자 의 아랫사람인 ⑤

느리에게 아들과 마찬가 해체를 사 하는 것이

상 이 만 임 을 한 느리에게 고마 과 쁨

존 의 표 를 하 위해 자 의 아 에게 말하듯

해 체를 사 하고 있다

8) 정답[ ] ③

9) 정답[ ] ⑤

10) 정답[ ] ①

해설 청자 할아 가 장의 주체 아 다 높을[ ] ( ) ( )

경 에는 압존 에 의해 장의 주체를 높이 않는lsquo rsquo

다 러 아 서가 아닌 아 는으 계 lsquo rsquo lsquo rsquo lsquo

니다 가 아닌 있 니다 표현하는 것이 르rsquo lsquo rsquo

11) 정답 당이 당을 쫒았다 당이[ ]

당에 다

해설[ ]

12) 정답[ ] ⑤

해설 서 다른 높임표현을 통해 청자에 대해 리[ ] ⑤

적 거리감을 나타내는 인 은 이 아니라 현정이

다 가 에서 현정은 에게 해 체를 사 함으 써 ( )

친근감을 드러낸다 나 에서 연 을 게을리하는 역 ( )

도 들 때 에 화가 난 현정이 선생님에게 항의하

는 장 에서는 하 체를 사 하여 리적 거리lsquo rsquo

가 어졌음을 나타내고 있다

13) 정답[ ] ①

해설 는 는 얼 빛이 날과 어찌 다르 고[ ] lsquo rsquo

라는 뜻으 전과 달리 임이 화자를 않고

있음을 알 있다

14) 정답 달리 후 가 있다 이를 통해 경[ ] lt gt

쾌한 음악성을 형성하고 노 젓는 상황을 체적으

형상화하는 역할을 한다

15) 정답[ ] ①

16) 정답[ ] ⑤

해설 다 의 자연은 를 성찰하게 하는 대상[ ] ( )⑤

이자 정의 대상이다 의 자연은 자 의 상황과 ⑤

처 를 드러내는 경으 서의 역할을 하 이

이 없다

17) 정답[ ] ③

해설 는 빈천 을 해결하고자 했으나 강산[ ] lsquo ( )rsquo 貧賤③

과 풍 을 달라는 에 거절하 다고 함으 써 자

연에 대한 애정을 드러내고 있으 는 않는

임에 대한 망을 개에게 전가 켜서 임에 대한 리

을 드러내고 있다

18) 정답[ ] ③

년 학 간고사 대비2013 2 현대고 대비

ECN-0102-2013-001-000076193

19) 정답[ ] ⑤

해설 고상한 음악가의 이름을 리말 꽝 럽[ ]

게 꿈으 써 언어유희를 통해 음을 유 하고 있

다 이는 고상한 척하는 총 를 비꼼으 써 비판적

태도를 드러내는 것이 대상을 꽝 럽게 표현

하여 총 의 허 과 사치를 풍자하고 있다

20) 정답[ ] ⑤

해설 는 작품 속 경에 대한 설 이 드러나는 것이[ ]

서 자의 주 적인 견해가 접적으 드러나는 것이

아니다

21) 정답[ ] ⑤

22) 정답[ ] ②

23) 정답[ ] ④

24) 정답[ ] ①

해설 적강 티프는 주인공의 비 한 출생이나 능[ ] ①

과 이 있는 것으 조정의 능함을 풍자하는lsquo rsquo

것과는 거리가 다

25) 정답 픔 나[ ] ( )

해설 의 음악은 고통 는 사람들을 위 하고 아픔[ ] lsquo rsquo

을 치유해 주는 능을 한다고 할 있다 의 lt gt

픔 도 소 된 이 과 더 어 살아가는 따뜻한 마음lsquo rsquo

을 상 한다

26) 정답[ ] ⑤

해설 에게 선천적으 주어 각 장애라는 역경[ ]

은 의 이라는 가사 연 을 있다lsquo rsquo

27) 정답[ ] ④

해설 는 장 란 선 에게 은 개인적인 인상을[ ]

소녀 장정 등으 표현한 것이다lsquo rsquo

28) 정답[ ] ②

해설 담자가 피 담자의 언어적 표현이나 비언어[ ]②

적 표현 하 독자는 담의 위 나 피

담자의 감정 상태를 알 있다 이를 통해 독자는

담 상황을 더 생생하게 느낄 있고 피 담자

를 더 잘 이해할 있게 된다

29) 정답[ ]③

해설 일상생활과 역도 선 서의 성과에 된 것에서[ ]

역도를 하 서 겪는 어 과 내적 고민으 화제를

전화하 위한 것이다

30) 정답[ ] ①

해설 릿속에 새겨 넣듯 이 억되도 함 세상[ ] ② ③

살이가 힘들고 고생 러 속 하여 자유를 ④

가 없는 고통의 상태를 비유적으 이르는 말

적의 침입을 막 위해 쌓은 축 켜야 할⑤

대상을 비유적으 이르는 말이다

31) 정답[ ] ④

해설 이 의 종류는 전 으 인 사건 경[ ] lsquo

비평을 성 소 삼는다rsquo

32) 정답[ ] ④

해설 근은 삼대독자 태어났음을 에서 확인할[ ]

있다 형제들과의 담은 이뤄 가 없다

33) 정답[ ] ⑤

해설 근은 가난에도 하고 화가를 꿈꾸었다[ ] (3

단 또한 다른 화가 망생들은 정 육을)

위해 상 학 학 해 유학 에 랐 만

근은 다른 을 찾아야 했다 단 세에(5 ) 18

근은 조선 전람회에 입선하 다 단 의(6 )

만종은 인간과 자연이 엮어 가는 경건한 조화 을lsquo rsquo

나타낸다

34) 정답[ ] ①

해설 근이 속에서도 창작활동을 추 않고[ ]

하는 닭은 은 세상과 타협할 르는

근이 세상의 이해를 하 위한 가장 떳떳한 단

이 때 이다

35) 정답[ ] ⑤

해설 전 은 서 자의 주 적인 평이 리는 것이[ ]

만 위 제 은 인 이 살았던 대 사회적 경

을 통해 객 적인 인 의 을 제 하고 있다

36) 정답[ ] ⑤

해설 전 은 인 사건 경 비평이라는[ ] lsquo rsquo⑤

성 이 어져 있다

37) 정답[ ] ①

해설 이 은 동양인과 서양인의 사고 에 차이가[ ]

있다는 것을 대조를 통해 설 하고 있다 또 쓴이

의 제자가 축 경 를 러 가서 경험한 일화를

통해 동양인이 서양인에 비해 주 상황에 더 많은

주의를 인다는 주장을 뒷 침하고 있다

38) 정답[ ] ④

39) 정답[ ] ②

40) 정답[ ] ②

41) 정답[ ] ④

42) 정답[ ] ③

43) 정답[ ] ④

44) 정답 도서 의 휴 일 도서 의 이 간 도서의[ ]

해설 도서 장은 임의 정한 휴 일과 도서 이[ ]

간 도서의 상 등을 게 할 의 가 있다

년 학 간고사 대비2013 2 현대고 대비

ECN-0102-2013-001-000076193

45) 정답[ ] ①

해설 제 조의 정 휴 일 의 휴 일의 사전 게[ ] 3

는 도서 장의 의 조항에 속한다

46) 정답[ ] ①

해설 개인 정 호 의 를 제 하 했 만 항[ ]

나눠서 제 하 않고 대 나열하고 있다

47) 정답[ ] ②

해설 제 조의 내 을 회사는 다른 회사 협[ ] 7 lsquo

계약을 통해 서비 를 제공하는 경 회 의 아이디

등 개인 정 를 해당 회사에 전송할 있다는 내rsquo

이 있으 의 제점을 제 할 있다②

48) 정답[ ] ④

해설 는 도서 장의 의 에 해당하고 나 는 도[ ] ④

서 장의 리에 해당한다

49) 정답[ ] ③

50) 정답 은 음독으 적었고 은 훈독으 적었[ ] (1)

다 과 동일한 표 리 적은 것은 이고 (2) ce

과 동일한 표 리 적은 것은 이다ab

51) 정답[ ] ③

52) 정답[ ] ①②

53) 정답[ ] ③

54) 정답[ ] ③

55) 정답[ ] ①

56) 정답 른 죠코 어린 노 하니라[ ] A B

57) 정답 세 어에서는 활 형이 칙적으[ ] lsquo rsquoㄹㅇ

나타났 만 개화 어에서는 활 형이 쓰 다 lsquo rsquo ㄹㄴ

58) 정답 호 가 흔[ ] (1) (2)

59) 정답[ ] ④

60) 정답[ ] ③

Page 15: 현대고대비 국어 - chamsoriedu.com 「콘텐츠산업진흥 법」외 에도 저작권 의하여 ... 다른주체에게어떤동작을하도록만드는것을나타내는

년 학 간고사 대비2013 2 현대고 대비

ECN-0102-2013-001-000076193

생각 여 가 고 시 는ldquo ( )大人

상에 다시없는 니다 살 엇 겠습니

에 돌 가시고

가에 돌 가 니 살 마 없습니

다 략 어 없어 강 승상 가니rdquo lt gt

그곳 월계 었다

다 강 승상에게는 들 없고 다만 만( )

었다 가 낳 에 가 색

타고 내 에게 말

는 니다 미원 과ldquo ( )紫微垣

연 맺고 었는 께 강 집( )緣分

보내 에 니 게 여겨 주십시

rdquo

거늘 미 가운 낳 니 가

고 거동 단 다 시 짓 쓰 고

는 없었 니 여 가운( ) 音律

지 는 짝 룰 만 사 없었다 가 사

여 사 감 게 고 지 못 고 염 는 만다

다가 당에 거 고 식같 러

내니 고귀 상 루 말 다 어 ( )相

울 도 다 귀 사 없고 ( )富貴爵祿

웅 걸 만고 었다 승상 매우 뻐 내

당 들어가 에게 사 니 역( ) 內堂

시 매우 거워 말 다

도 마 사 는 승상께ldquo

그 게 말 시니 상 여러 말 지 말고 사

도 시다 략 시 택 여rdquo lt gt

니 다운 신 과 신 습 늘에 죄

짓고 간 상에 내 신 다

다 내고 들어가 사 살펴보니

고 것 는 다 말 어 고

는 다 어 신 에 ( )新房

에 신 과 신 가 평생 연 맺었( )緣分

니 사 주고 말 어떻게 다 헤 릴

어떻게 다 리 지낸 후에 튿 승

상 니 승상 거운 마 지 못

( ) 듯 월 러 생 열다 살

었다 에 승상 어진 사 얻고 만 에 근심

없었 다만 주 가 간신

에 죽 것 생각 마 곧 어

곤 다 그 에 주 원통

어 없 고 여 시 가 거늘 략 lt gt

략 거리

강 승상 에게 상 리지만 여움

사 귀 가게 다 강 승상 몸 는

연 과 헤어 리 다

마 각 생 강 승상 집 쪽( )

늘 보고 없 가 신 신 생각 니

없고 어 없었다 는 어떻게 도리가 없다

여 산 에 들어가 리 고 어 도 닦

고 다 그 산 보고 가

다가 곳에 다다 니 에 큰 산 었다 많

우리 골짜 가 늘 는 가운 색

에 고 갖가지 가 짝 어 ( )花草

었다 략 주 보니 lt gt ( ) (一柱門 黃

산 룡사 어 었다) lsquo rsquo 金大字

산 들어가 고승 다 그( ) ( ) 山門 高僧

거동 보니 눈 눈 듯 고

변 같 귀는 어 에 늘어 니( ) 白邊

맑고 어 골격과 신 평 니었

다 염주 에 걸고 짚고 포 ( )六環杖

삼에 어진 쓰고 생 보고 말

승 연 여 상공 시는 동 에ldquo

가 맞 지 못 니 승 십시 rdquo

생 크게 말 다

생 가 여 어 고ldquo

없 다니다가 우연 곳에 사 만 것

그 시 생 어떻게 고 습니

rdquo

승 답 여 말

어 산 승 에ldquo ( ) ( )南岳 衡山

시어 승에게 탁 내 낮 시경에 경 lsquo 12

동 에 사는 심 들 가 것 니 내쫓

지 말고 습니다 마 승rsquo

다가 상공 림새 보니 경 사 에 보

습니다rdquo

생 그 말 듣고 편 고 편( )

슬 승 들어가니 여러 승 들

가워 다 승 에 들어가

후에 그 편 니 곳 경 었다 상( ) 仙境

고 신 편 다 후 는 승과

께 병 도 탐 고 경도 게( )兵書

게 었다 게 니 지 에 가객 ( ) ( )大明天地 佳客

년 학 간고사 대비2013 2 현대고 대비

ECN-0102-2013-001-000076193

없고 산 에 리 만 본 ( ) 廣德山

신 상 사 살 는 만

우고 늘 월 신 과 늘 ( )日月聖神

산 신 들 다 니 그 재( ) 名山神靈

주 민 누가 당 겠는가 낮 공

zb24) 다 에 해당하는 내 으 적절하( ) 않은 것은

① 강 티 통 당시 능 다

② 상계 지상계 경 는 원 계 드러

③ 실에 어 없는 실 가 타 는

④ 뛰어 재주 어 가진 고

등 다

⑤ 가 직 개 여 평가 내리는

편집 평 타 다lsquo rsquo

가 본격 가 동 것 지( )

다 단 상 에2003 lsquo rsquo

들어가 드럼 연주 다 취미 생 달리

들었다는 보 우 가 들ldquo

어 틱 린 도 다 고 말 다rdquo

경 는 가 망 없( ) lsquo

티 원 고 답 다 신과 같 시각rsquo

는 습 상상 만 도 감동

다 시각 연주 동시에

열 상 는

티 원 그런 열 경 럽다는 것 다

다 역시 엄청 다 본( )

에 복 들

고쳐 가고 다 신 에 얼

마 지는 고 리가 는 지도 생님

가 훈 고 많 고쳐 다

고 말 다

그러 직도 에 지 는 다 그는

체격 지 못 게 가 큰 만

체 운동 훈 과 께 체 늘 동 50

는 게 고 말 다

에게는 꿈 다 통 누 가( )

주겠다는 것 그 꿈 다 신 극복 는

과 에 큰 경험 들도 느 게

주고 싶다는 것 다

마 슬 마다( ) ldquo 통

낼 었 것 럼 고통 는 사 들

고 겠다 고rdquo

말 다 달 루 첫 낸 lsquo rsquo

첫 드 심 집에 는 리듬 드 2

루 에 도 보고 싶다 집 에는 직(RampB) 3 4

사 곡 도 보 고 싶다고 포 다middot

zb25) 에서 가장 유사한 의 를 닌 어를lt gt

찾아 쓰

lt gt

나는 이제 너에게도 픔을 주겠다

사랑 다 소 한 픔을 주겠다

겨 거리에서 개 놓고

살아 추위 떨고 있는 할 니에게

값을 으 서 뻐하던 너를 위하여

나는 픔의 평등한 얼 을 여 주겠다

내가 어둠 속에서 너를 를 때

단 한 도 평등하게 어주 않은

가마니에 덮인 동사자가

다 얼어 죽을 때

가마니 한 장조차 덮어주 않은

한 너의 사랑을 위해

흘릴 르는 너의 눈 을 위해

나는 너에게 이제 너에게도 다림을 주겠다

지 울 포동 여고 생들17

틈 없 가득 체 에 맑 울

다 죽 듣 생들 사 에

연 는 탄 다 객들 도 는lsquo rsquo

가 보 주 공 맹 가 운 는

단 그룹사운드 루 보컬 맡고 는lsquo rsquo

시각 지 었다17 1

근 다만과 가 거lsquo rsquo lsquo

꿈 고 퇴 내가 다rsquo

간 간에 지 지 연 생들 짧lsquo rsquo lsquo rsquo

가 운 듯 리에 어

연 다 내 사 고 퇴lsquo rsquo

과 루 들 결 다시 돌lsquo rsquo

들 고 사 들 에 당당

것 니다 내 태어

볼 없었 크고 열여

년 학 간고사 대비2013 2 현대고 대비

ECN-0102-2013-001-000076193

에도 고 시 얻지 못 다

감지 없는 시각 상태 다

신 지에 고 상 원망 도

단다 어느 가 에 시각 에 ldquo

어 그런 듣고 다 보니 내가 게 lsquo

살 는지 도 눈 고 싶rsquo lsquohelliphellip

보 는 생각만 들 고 그 가 들에게rsquo

도 내고 들도 고 많 었죠 들 rdquo

었 지 새 는 에 쑥 러운 색

어났다

생에 것 단연 었다lsquo rsquo

공연에 거 꿈lsquo rsquo

는 다 특 가사 갑게 는 운 lsquo

벽 에 당당 마주 어 언 가 그 벽

고 늘 어 거운 상도

없죠 내 삶 에 웃 그 께

는 다고 다rsquo

들었 그냥 런 도 고만 여ldquo lsquo rsquo

겼죠 그런 꾸 가사 미 새 다 보

니 통 는 가사 는 생각 들 고 (

가 게는 시각 는 생각 들고 들) ( )

마다 듣고 큰 얻었어 rdquo

에 진지 게 가에 미 가

zb26) 의 에 들어갈 말 적절한 것은lt gt ~

lt gt

난 난 꿈이 있었죠

고 찢겨 남 하여도

내 가 히 과 같이 간 했던 꿈

혹 때 누 가가 뜻 를 비 음

내 등 뒤에 흘릴 때도

난 참아야 했죠 참을 있었죠

날을 위해

늘 걱정하듯 말하죠

헛된 꿈은 독이라고

세상은 끝이 정해 책처럼

이 돌이킬 없는

현 이라고 helliphellip

래 난 난 꿈이 있어

꿈을 믿어

나를 켜

저 차갑게 서 있는 이란 앞에

당당히 마주칠 있어

출처 가 거위의 꿈 작사 이적 작곡 동률- lsquo rsquo ( )

① ② ③ ④ ⑤

가 떴다는 들 만 지만( ) lsquo rsquo

늘 겸 다 에 주 연 우승 지 간에도 3

단 생님께 만 지 고 고 만ldquo rdquo

큼 늘 겸 신 계 가

고 다

에게는 꿈 다 통 누 가

주겠다는 것 그 꿈 다 신 극복 는 과

에 큰 경험 들도 느 게 주

고 싶다는 것 다

슬 마다 통 낼ldquo

었 것 럼 고통 는 사 들

고 겠다 고rdquo

말 다 달 루 첫 낸lsquo rsquo

첫 드 심 집에 는 리듬 2

루 에 도 보고 싶다(RampB) 집 에는 직34

사 곡 도 보 고 싶다고 포 다

미 는( ) (26) 어 헤헤헤 웃다가 어ldquo rdquo

허허허 웃었다ldquo rdquo ldquo rdquo 같 도 고

상 다 는 같 도 다( ) 壯丁 킹 들lsquo

다 는 역도 보 그 다 지만 그는rsquo

뷰에 지 다 운동만 지 ldquo

것 지 간에 여러 사 도 역rdquo helliphellip

었다 그런 엇 그 마 움직 는지 보 쯤

지 담 사 다 훈 없어 그는 티

지 림 었다 태 다 갔다 는 습

마 집 럼 편 게 보 다

주말에는 주 엇 보내

주말에도 별 주 에 청ldquo

고 에 가고 도 쳐

에 듣고 보 에 갈 가 별 없

어 산 시 게 고 들어 2002

거 매 여 지냅니다 시 과 지훈 rdquo

다 근 간 과 진실 그리고 싶어( )

가 다 근에게 그것 진리 다 거 다 없

거 고 다 없 는 것 진리

다 근 진리는 후 쪽 었다 신산( )辛酸 삶

었 질곡( )桎梏 역사 에 지냈 가

눈에 든 것 료 단 료 게 보

것 었다 그것 그 에 겨우겨우

슬 슬 생 어가는 간들 었다

리 과 단 리 고리에 검 마

없 거리 돌

상 것 없는 등 근에게 상

과 진실 엄 ( )儼存 다는 사실 리는 가

실 고 가 과 역경 에 도 근 내 포

없었 후 보루( )堡壘 다 도 365

도 간 근 여

시 것 다

년 학 간고사 대비2013 2 현대고 대비

ECN-0102-2013-001-000076193

다 공주 그림 가 근 경- ( ) ldquo rdquo(

2009)

zb27) 작가의 주 적인 각이 드러난 것은~

① ② ③ ④ ⑤

가 신 지에 고 상 원망( )

도 단다 어느 가 에 시각 에 ldquo

어 그런 듣고 다 보니 내가 lsquo

게 살 는지 도 눈 고 싶rsquo lsquohelliphellip

보 는 생각만 들 고 그 가 들에게rsquo

도 내고 들도 고 많 었죠 들었rdquo

지 새 는 에 쑥쓰러운 색

어났다 략 [ ]

경 는 가 망 없 티lsquo

원 고 답 다 신과 같 시각rsquo

는 습 상상 만 도 감동

다 시각 연주 동시에

열 상 는 티

원 그런 열 경 럽다는 것 다 략 [ ]

슬 마다 통 낼ldquo

었 것 럼 고통 는 사 들

고 겠다 고rdquo

말 다 달 루 첫 낸 lsquo rsquo

첫 드 심 집에 는 리듬 2

루 에 도 보고 싶다 집 에는 직(RampB) 3 4

사 곡 도 보 고 싶다고 포 다

식 누 가-

고 싶어

다 역도 미 담 고 사( )

질 주말에는 주 엇 보내[ 1]

답 주말에도 별 주 에[ ] ldquo

청 고 에 가고 도 쳐

에 듣고 보 에 갈 가 별

없어 rdquo

질 계 고 슬슬 도 는 것 닙니[ 2]

답 다 들 눈 에 보 고 뿐 보[ ] ldquo

다 열심 고 어 상에 도 들지만 상

지키는 것 들다고 에 도달

그것 지키 훨 많 rdquo

질 들 살 고 리 는[ 3]

거운 들 체 리느 는다

답 가 고 게 체 어[ ] ldquo ( ) 級

느 도 계가 니 살 는 것도 고역 지만

살 우는 것 들어 는 살

체 리 고 어도 어도 실 갔다

쑥 어 rdquo

질 거리에 슷 연 여 들[ 4]

보는 간 상 지

답 상 다 체 게 리지 못[ ] ldquo

거 주변에 는 그 거 누 보지

못 고 뻐지고 싶 에 체 리는 에

타 워 지만 는 어울 는 것보다 는

시간 운동만 는 건 니에 사복 lsquo rsquo

고 사복 는 말에 들 웃지만 늘 운동복

고 지내니 사러 갈 도 어 rdquo

질 역도가 말 단 식 운동 니[ 5]

답 가 내는 만 클 업 보[ ] ldquo

그러니 만 쓰는 식 운동 니다

만 다고 거운 것 들 는 건 니거든 연

도 고 가지 동 에 도 여러 가지

복 들

보식 역도 여 미-

zb28) 가 에 대한 설 으( ) 않은 것은

① 시각 우 지 시 에 지

고 망 가는 태도 달 고 다

② 언어 과 언어 복 사 여

담 내 생각 게 는 가

③ 직 감 그 마 것

럼 생생 게 느껴지는 과 주고 간 내

없 리 어 억 게 다

④ 담 내 식 리 여 담 삶 습

과 가 시 여 독 에게 감동과 훈 다

⑤ 직 진 담 직 누

지 못 는 독 에게 생생 상 달 주고

담 욱 게 다

zb29) 나 의 각 의 의도를 설 한 것으 적절하( ) 않

년 학 간고사 대비2013 2 현대고 대비

ECN-0102-2013-001-000076193

은 것은

① 질 담 상 보여 주 것 다1

② 질 담 과 그에 삶 태도 보여2

주 것 다

③ 질 역도 겪는 어 움에 역도3

과 것 다

④ 질 같 연 여 갖는 고민 는지 말4

주 는 것 다

⑤ 질 역도가 과 고 운동 는 것5

담 가 말 주 는 것 다

가 만진 것 다( ) 3

감 달 다고 다 억 에( ) 音感

지워 지만 당시 청 탁 리도

다고 다 드럼 웠다 4

에 갈 마다 드럼 는 리가 신 게 들

다고 다 눈 볼 가 없 니 엔ldquo

는 는 님 틱 에 여 주

다 드럼과 연 맺 과 들 주었다rdquo

식 누 가-

고 싶어

역( ) 도가 말 단 식 운동 니

가 내는 만 클 업에 보ldquo

그러니 만 쓰는 식 운동 니다 만

다고 거운 것 들 는 건 니거든 연

도 고 가지 동 에 도 여러 가지 복

들 시 는 상 상

드는 상 에 맞춰 실 에 는 여러

펼쳐집니다rdquo

략( )

늘 에 는 어 만 것 같

가 에 사 고 사 사ldquo

겠어 든 에 가 경 만 고

울 는 사 겠어 rdquo

보식 역도 여 미-

다 가 운 는 어 어( ) ldquo rdquohelliphellip

월 새벽 시 태 없 거웠고1965 5 6 1

는 없 그 병원에 퇴원 집

가는 마지막 마 고 마 내 거 다

가 죽 간신 에 실 다 사는 어느5 lsquo

가 죽 는 말 가 식 다 신rsquo

상에 각 시키는( )刻印 에 실

어느 가는 후 민 가가 근 었다lsquo rsquo

는 간 과 진실 그 다는( ) ldquo

에 단 평 견 가지고 다 내

가 그리는 간상 단 고 다 지 다 는 그들

가 에 는 평 지 니 그리고 어린

들 미지 겨 그린다rdquo

마 근 간 과 진실 그리고 싶어( )

가 다 근에게 그것 진리 다 거 다 없

거 고 다 없 는 것 진리

다 근 진리는 후 쪽 었다 신산(辛酸 삶)

었 질곡(桎梏 역사 에 지냈)

가 눈에 든 것 료 단 료 게

보 것 었다 그것 그 에 겨우겨우

슬 슬 생 어가는 간들 었

다 리 과 단 리 고리에 검

마 없 거리 돌

상 것 없는 등 근에게 상에

과 진실 엄 다는 사실 리는 가( )儼存

실 고 가 과 역경 에 도 근 내

포 없었 후 보루(堡壘 다 도)

도 간 근365

여 시 것 다

월 강원도 림리에( ) 1914 2 21

삼 독 태어났다 어 근 복

그것 그리 가지 못 다 근 곱 살

지는 산 산업에 실 고 답마 에 내

갔다 근 그림 럼 쫓 다니 가 시 것

다 상 진 것도 가 었다

러 가 에도 고 근 가 꿈꾸었다 근

가 꿈꾸게 것 보통 업

원색도1926 만lsquo rsquo 었다

공주 그림 가 근 경-

zb30) 에 대한 설 가장 른 것은~

① 역도가 과 운동 도 질

② 리는 는 다 lsquo rsquo

③ 들었지만 그럭 럭 는 다 lsquo rsquo

④ 가 게 보 시 말 다

⑤ 보 병 는 지 상 lsquo rsquo

는 말 다

년 학 간고사 대비2013 2 현대고 대비

ECN-0102-2013-001-000076193

시간 많지 다 청량리 생 병원

마지막 상 경 릿 게 들어 다 그 는 십

만 큰 가 상 말 다

지 못 들 마 갈 고 돗

도시민들 싹 싹 탔다 가 시

월에 병원에 원 가 폐 진 몸도4 ( )疲弊

갈 미 지 못 고 었다 가는 얼마( ) 解渴

지 생 에 생각 가

마감 는 신 평생 십 만에

가 과 많 닮 다고 생각 지는

가 운 는 어 어ldquo rdquo 1965helliphellip

월 새벽 시 태 없 거웠고 는5 6 1

없 그 병원에 퇴원 집 가

는 마지막 마 고 마 내 거 다 가

죽 간신 에 실 다 사는 어느 가5 lsquo

죽 는 말 가 식 다 신rsquo

상에 각 시키는 에 실 어느( ) lsquo刻印

가는 후 민 가가 근 었다rsquo

ldquo 는 간 과 진실 그 다는 에

단 평 견 가지고 다 내가 그

리는 간상 단 고 다 지 다 는 그들 가

에 는 평 지 니 그리고 어린 들

미지 겨 그린다rdquo

근 간 과 진실 그리고 싶어 가

다 근에게 그것 진리 다 거 다 없 거

고 다 없 는 것 진리다

근 진리는 후 쪽 었다 신산 삶 ( )辛酸

었 질곡 역사 에 지냈 가 눈에( )桎梏

든 것 료 단 료 게 보 것

었다 그것 그 에 겨우겨우 슬

슬 생 어가는 간들 었다 리

과 단 리 고리에 검 마

없 거리 돌 상

것 없는 등 근에게 상에 과 진실

엄 다는 사실 리는 가 실( )儼存

고 가 과 역경 에 도 근 내 포 없었

후 보루 다 도 도( ) 365堡壘

간 근 여 시 것

간에 지닌 가 근 1914 2

월 강원도 림리에 삼 독21

태어났다 어 근 복 그것 그리

가지 못 다 근 곱 살 지는 산

사업에 실 고 답마 에 내 갔다 근

그림 럼 쫓 다니 가 시 것 다 상

진 것도 가 었다 러 가 에도

고 근 가 꿈꾸었다 근 가 꿈꾸게

것 보통 업 원색1926

도 만 었다lsquo rsquo

그림 가 근 경 공주- ldquo rdquo ( 2009)

zb31) 다음 이 같은 의 성 소에 해당하 않은

것은

사건 평① ② ③

④ 주 ⑤ 경

가 운 는 어 어ldquo rdquo 1965helliphellip

월 새벽 시 태 없 거웠고 는5 6 1

없 그 병원에 퇴원 집 가

는 마지막 마 고 마 내 거 다 가

죽 간신 에 실 다 사는 어느 가5 lsquo

죽 는 말 가 식 다 신rsquo

상에 각 시키는 에 실 어느( ) lsquo刻印

가는 후 민 가가 근 었다rsquo

는 간 과 진실 그 다는 에ldquo

단 평 견 가지고 다 내가 그

리는 간상 단 고 다 지 다 는 그들 가

에 는 평 지 니 그리고 어린 들

미지 겨 그린다rdquo

근 간 과 진실 그리고 싶어 가

다 근에게 그것 진리 다 거 다 없 거

고 다 없 는 것 진리다

근 진리는 후 쪽 었다 신산 삶 ( )辛酸

었 질곡 역사 에 지냈 가 눈에( )桎梏

든 것 료 단 료 게 보 것

었다 그것 그 에 겨우겨우 슬

슬 생 어가는 간들 었다 리

과 단 리 고리에 검 마

없 거리 돌 상

것 없는 등 근에게 상에 과 진실

엄 다는 사실 리는 가 실( )儼存

고 가 과 역경 에 도 근 내 포 없었

후 보루 다 도 도( ) 365堡壘

간 근 여 시 것

간에 지닌 가 근 1914 2

월 강원도 림리에 삼 독21

태어났다 어 근 복 그것 그리

가지 못 다 근 곱 살 지는 산

사업에 실 고 답마 에 내 갔다 근

그림 럼 쫓 다니 가 시 것 다 상

진 것도 가 었다 러 가 에도

고 근 가 꿈꾸었다 근 가 꿈꾸게

것 보통 업 원색1926

도 만 었다lsquo rsquo

공주 그림 가 근 경- ldquo rdquo ( 2009)

년 학 간고사 대비2013 2 현대고 대비

ECN-0102-2013-001-000076193

zb32) 위 을 작성하는 과정에서 되어 활 된 자

어 것은

신 사 료① 연보②

고③ ④ 들과 담

⑤ 에 평

는 간 과 진실 그 다는 에ldquo

단 평 견 가지고 다 내가 그

리는 간상 단 고 다 지 다 는 그들 가

에 는 평 지 니 그리고 어린 들

미지 겨 그린다rdquo

근 간 과 진실 그리고 싶어 가

다 근에게 그것 진리 다 거 다 없 거

고 다 없 는 것 진리다

근 진리는 후 쪽 었다 신산 삶 ( )辛酸

었 질곡 역사 에 지냈 가( )桎梏

눈에 든 것 료 단 료 게 보

것 었다 그것 그 에 겨우겨우

슬 슬 생 어가는 간들 었다

리 과 단 리 고리에 검 마

없 거리 돌 상

것 없는 등 근에게 상에 과

진실 엄 다는 사실 리는 가 실( )儼存

고 가 과 역경 에 도 근 내 포

없었 후 보루 다 도 도( ) 365堡壘

간 근 여 시

것 다

간에 지닌 가 근 1914 2

월 강원도 림리에 삼 독21

태어났다 어 근 복 그것 그리

가지 못 다 근 곱 살 지는 산

사업에 실 고 답마 에 내 갔다 근

그림 럼 쫓 다니 가 시 것 다 상

진 것도 가 었다 러 가 에도

고 근 가 꿈꾸었다 근 가 꿈꾸게

것 보통 업 원색1926

도 만 었다lsquo rsquo

질 루 마 가 도 린다 경건

움 느껴지는 경 다 훗 근 그림에

과 는 거 것( )裸木

만 간과 연 엮어 가는 경건 움lsquo rsquo

니었

같 가가 고 싶었 근에게 그 꿈에 다

가가는 지 다 다 가 지망생들 규 미

상 에 진 고

에 지만 근 다 다 근

미 에 운 것 보통 시 미 시간

다 그런 그에게 없는 연습 가가

통 다 가 귀 시 지 도

얻는 뛸 듯 뻤지만 마 도 가 에

듯 는 었 에 어린 근 주 에

에 그림 그리고 지우고 복( )粉板

시간 가는 게 루 보냈다

근 그 갈 가가 것 열여( )渴求

었 다가 미1932 lsquo rsquo ( lsquo

미 에 다 다는 고 마rsquo) lsquo rsquo

가 근 집 고도 지는 시골 경

그린 그림 다 후 근 에 1943 22

지 미 에 그림 고

에 걸쳐 다 미 근 가

동 는 었다

공주 그림 가 근 경- ldquo rdquo ( 2009)

zb33) 위 의 내 과 일치하는 것은

가 근 가 꿈 포 다①

근 당 가들과 께 에 다②

살 근 가 걷20③

게 었다

④ 만 통 근 역경 겨내는lsquo rsquo

느 다

⑤ 근 간 과 진실 그리 에 그 에

드러 는 간상 단 다

계 시 주 근 건강

걸었다 신 과 간에 상 다 건강

신 는 눈에도 다 근 쪽 눈 뿌 게

보 지 과에 다 다 시 지지 고 결

내 었다 시 지만 마 막막

다 늦어 결 근 쪽 눈 고 말 다

쪽 눈 근에게는 쪽 눈 었고

계 었다 그 근 는 여 그lsquo rsquo

다 근 에 같 그림 그 었다1950

시 그림 는 여 쪽lsquo rsquo

고 어 마주 고 는 그림1963

여 과 동 다 마 복

그린 듯 눈 내리 새 게 다 지

사 다 근 게 복 것

복 상과 타 는 근 상

가 떳떳 단 었고 근 그리고

간 과 진실 에 다가가 가 근다

운 었다 근 신에게 당당 지 그리고

그 다 근 그림에 단 복 보다

년 학 간고사 대비2013 2 현대고 대비

ECN-0102-2013-001-000076193

태 도 그리고 극 보다 과

얻 여 었다 과 통

근 그리고 는 재 고 에 질

만들고 특 것 다

공주 그림 가 근 경- ldquo rdquo( 2009)

zb34) 의 이유에 대해 추 한 것으 적절하 않은 것

상과 타 시도①

보다 과 얻②

근 신에게 당당 지③

④ 간 과 진실 에 다가

⑤ 태 도 얻

근 가가 었지만 그 다니 가

럼 어지지 다 복과 쟁 거쳐 시

는 가 근에게 생계 사 에

운 사 다 에 키에 건( ) 178cm死鬪

체 근 에 동 역 업( )荷役

가 생계 다 쟁

에는 동에 운 상우 주 미

죄 사 에 그림 그리는 시 다 그곳에

에 동 역 업 것에

결 것 럼 보 다 지만 그런 것만도

니었다 그림 그리는 고는 지만 매 근

는 극 간 과 별 없는 경 리 그림

벽에 그리는 것 었다 우도 리 없었다 근

트 는 우 그림 그 다 생

계 그림 단 것 다

후 근 지 신 계 리에 미

엑 리 겼다 근 곳에

건 사 크 에 미 들 ( )

상 상 그 다 근 갖 다 겪

냈다 그리고 결 그 돈

신동에 어 사리 집 마 다 마 ㄷ

루 심 쪽에는 과 엌 쪽에는 건

었다 건 주고 근 가 에

여 살 다 심 에는 지 집어

쓰고 지만 곳 근 가 에게 러웠

보 리 다 근 과 마루 업실 삼 그림

그 다 신동 마루는 근 그림에 등 는 lsquo rsquo

같 상들 지 다 시 고

에 들 폐허가

가 업실 었다

공주 그림 가 근 경- ldquo rdquo( 2009)

zb35) 위 에 대한 설 으 적절한 것은

업 시 여 훈과 감동 다①

에 주 평 드러 다②

사 사 등 식 과 ③

④ 다 근거 시 여 삶에

⑤ 살 시 사 경 께 여

습 시 다

가 시간 많지 다 청량리 생 병원( )

마지막 상 경 릿 게 들어 다 그 는

십 만 큰 가 상 말 다

지 못 들 마 갈 고 돗

도시민들 싹 싹 탔다 가 시

월에 병원에 원4 가 폐( )疲弊

진 몸도 갈 미 지 못 고 었다( )解渴 가는

얼마 지 생 에 생각

가 마감 는 신 평생 십 만에

가 과 많 닮 다고 생각 지는

가 운 는 어 어( ) ldquo rdquohelliphellip

월 새벽 시1965 5 6 1 태 없 거웠고

는 없 그 병원에 퇴원 집

가는 마지막 마 고 마 내 거 다

가 죽 간신 에 실 다 사는 어느5 lsquo

가 죽 는 말 가 식 다 신rsquo

상에 각 시키는 에 실( )刻印

어느 가는 후 민 가가 근 었다lsquo rsquo

다 는 간 과 진실 그 다는( ) ldquo

에 단 평 견 가지고 다 내

가 그리는 간상 단 고 다 지 다 는 가

에 는 평 지 니 그리고 어린 들

미지 겨 그린다rdquo

근 간 과 진실 그리고 싶어( )

가 다 근에게 그것 진리 다 거 다 없

년 학 간고사 대비2013 2 현대고 대비

ECN-0102-2013-001-000076193

거 고 다 없 는 것 진리

다 근 진리는 후 쪽 었다 신산( )辛酸 삶

었 질곡 역사 에 지냈( )桎梏

가 눈에 든 것 료 단 료 게 보

것 었다 그것 그 에 겨우겨우

슬 슬 생 어가는 간들 었다

리 과 단 리 고리에 검

마 없 거리 돌

상 것 없는 등 근에게 상에

과 진실 엄 다는 사실 리는 가 실( )儼存

고 가 과 역경 에 도 근 내 포

없었 후 보루 다( ) 堡壘 도 365

도 간 근 여

시 것 다

마 같 가가 고 싶었 근에게 그 꿈( )

에 다가가는 지 다 다 가 지망생들

규 미 상 에 진 고

에 지만 근 다 다 근

미 에 운 것 보통 시 미 시간

다 그런 그에게 없는 연습 가가

통 다 가 귀 시 지 도

얻는 뛸 듯 뻤지만 마 도 (

는 었 에 어린 근 주 에)

에 그림 그리고 지우고( )粉板

복 시간 가는 게 루 보냈다

zb36) 전 의 성 소가 아닌 것을 고르

① 평 ② 사건 ③ 경

④ ⑤ 훈

늘 지 상에 살고 는 사 들 억 도가10

고 그리 지 통 고 는 사 들( )知的

그보다 훨 많 억 도는 고 지 20

통 다 그런 지 고 2500

그리 간 보는 과 사 에

매우 달 뿐만 니 과 에 도 극

루고 었다 미 운 그런 들

살고 는 동 과 사 들 사고 식에

큰 가 다는 다

고 그리 들 우주 개별 고 독립

사 들 생각 지만 고 들 우

주 연 질 간주 다 같( ) 看做

각 도 들에게는 연 질

었지만 그리 들에게는 미 들 결 었

다 고 과 그리 들 사 같

는 동 과 사 에 도 견 다

지심리 미 마 드 겐트 는

살 들에 에 지 다

연 동 과 상 다 과 같 실험

다 크 만든 미드 도 보

여 주고 그 상 닥 고 주었다lsquo (Dax)rsquo

실 닥 는 재 지 는 것 실험 가lsquo rsquo

만들어 낸 다 그런 다 개 다 체 보

여 주었는 는 미드 지만 틱

만들었고 다 는 재료는 크 지만

달 다 그러고 어 것 닥 지 사 들에게 고 lsquo rsquo

게 니 들 주 같 고 는

체 택 고 동 들 같 재료 만들어진 체

택 다 러 는 심지어 살짜리

들에게 도 타났다 것 곧 과 동

다 상 보고 다는 것 미 다

개별 사 보고 고 동 연 질 보

고 는 것 다

동 들 주변 상 에 맞 어 동 고

에 다 사 들 태도 동에 보다 많

주 울 다 동 가 미시간 에

에 경험 다 그는 미식

경 보러 가게 었는 경 체는 매우 재미 었

주변 들 동에 질 다 그 는

들 계 어 상태 경 다

어 들 에 에 그 시 가 계 가

진 것 다 상 살펴 는 말 들 lsquo rsquo

에 그는 에 시 어 도 뒷사

생각 곧 다시 곤 것 다 그런 그에게 뒷

사 고 지 는 들 동 럼

어 웠다

생각 지도 리 드 니 벳-

zb37) 다음 위 의 내 전개 으 만 인lt gt

것은

lt gt

대조의 통해 대상이 닌 특성을 설 하고 있다

일화를 제 하여 자 의 주장을 뒷 침하고 있다

유추의 을 사 하여 독자의 의해를 돕고 있다

대상이 형성되는 과정을 간적 서에 따라 서 하고 있

① ②

③ ④

년 학 간고사 대비2013 2 현대고 대비

ECN-0102-2013-001-000076193

가 우리가 말 고 쓰는 든 단어가 사 에 는( )

것 니다 사 격에 가 는 지만

어 사 과 같 특별 는 사 니lsquo rsquo

단어 격 보 단어가 사 에

등재 어 다 리 리 사 는 단어 도 그

것 시 사 는 어 고 사 에

격 보 것 니다

러 얼 은 사전에 를 있는가 이에 대한 답lsquo rsquo

은 얼 이 유행어인가 아닌가에 따라 갈라 다 이 단어lsquo rsquo

는 년 어 자 에 랐고 쓰이고 있으2002 lsquo rsquo

유행어라고 하 에는 생 이 다 런데 계속

을 유 하 서 사전에 등재될 자격을 획득할 것인가 이

에 대한 답을 내리 는 히 어 다

여 서 가 를 고 해 볼 있다 첫 는 이 단어

를 써야 할 필 가 속적으 있는가 하는 점이다

상주의 열풍에 휩 인 사회 위 에 편 해서 퍼 말

이 얼 인데 과연 런 위 가 속될 것인가 이에lsquo rsquo

대해 필자의 생각은 정적이다 사회 위 가 뀌

런 말을 쓸 일이 없어 것이다

다음은 단어의 성이다 단어의 성이 사회적으 거

감이 없으 계속 사 될 가능성이 높다 런 에서

얼 은 좋은 조건이 아니다 익히 알 졌듯이 이lsquo rsquo

말은 얼 과 청소년층에서 속어 사 하는 이 결합lsquo rsquo lsquo rsquo

된 말이다 얼 에서 얼 을 리하는 조어 도 lsquo rsquo lsquo -rsquo

어에서는 매 낯선 이다 이것만으 도 거 감을 갖

는 사람들이 있다 더 나 속어 결합한 말이다 얼 lsquo rsquo

이 널리 퍼졌다 해도 은 여전히 청소년층의 속어lsquo rsquo

남아 있다 속어는 자연 럽게 아 자리에서나 쓰 에는

담 러 말이다 러한 담을 하고 사

역을 넓혀 가는 속어도 없 는 않다 특히 얼 은 lsquo rsquo

에도 종종 등장한다 만큼 거 감이 많이 희석되었다

고 할 있다 러나 일상의 자연 러 대화에서도 거

리낌 없이 등장하는가 게 는 되 않았다고 생

각한다

얼 이 유사어인 쌈 등을 만들어 내고lsquo rsquo lsquo rsquo

있으니 살아남을 있을 것이라고 는 견해도 있을 것

이다 러나 간이 나 서 유사어를 포함하여 든

말이 사라 사 는 많다 유사어가 많다는 것이 생 을

유 할 있는 절대적인 조건은 아니다

나 언젠가 터 사람들은 어느 단에서 얼 이 가장( )

쁜 사람을 가리켜 얼 이라고 르고 있다 이 얼lsquo rsquo lsquo rsquo

이라는 단어가 최근 어사전에 라 항간에 논란이 일고

있다 아닌 게 아니라 얼 은 유행어처럼 인다 생 lsquo rsquo

도 리 래되 않은 것 같고 언제 사라 도 알

없다 게다가 젊은이들 사이에서 주 쓰일 뿐이다 이런

단어를 사전에 는다는 게 하 이 없어 이 도

한다

러나 속단은 이다 차근차근 따져 볼 일이다

선 얼 이 일 적 유행어인 아닌 주의 게 들여다lsquo rsquo

볼 필 가 있다 유행어란 유행에 따라 빠르게 유포되었

다가 단 간 내에 소 되는 단어나 를 가리킨다

얼 은 인터넷을 통해 속히 퍼 말이다 하 만 일lsquo rsquo

적인 유행어처럼 단 간 내에 사라 않았을 뿐 아니라

현재 도 잦은 빈도 사 되고 있고 앞으 도 상당

간 사 될 것으 측된다 한 언 재단의 뉴 검 lsquo rsquo

색 사이트에 따르 얼 은 년 에 처음 나타난lsquo rsquo 2001

이후 꾸 히 사 되고 있다

이 같은 사 빈도는 얼 이 일 적 유행어 는 현lsquo rsquo

저히 다르다는 것을 여 다 장 간의 생존 만으 도

얼 은 이 한 어의 어휘 에 를 자격을 얻었다lsquo rsquo

고 할 있다 더 이 이라는 비 적 정제된 매체에

높은 빈도 쓰이고 있 않은가 사 빈도 측 에서

필통이나 연필과 같은 단어 대등하거나 더 많이 쓰lsquo rsquo lsquo rsquo

다는 것은 결코 가 게 볼 일이 아니다

이제는 사전이 언어 현 을 빠르게 하는 게 덕인

대가 되었다 세계적으 유 한 의 사전들도 경쟁

적으 어를 고 있다

하 만 얼 은 젊은이들이나 쓰는 속어라고 흠을 잡을lsquo rsquo

도 르겠다 얼 이 주 젊은 층에서 많이 쓰 lsquo rsquo

는 속어임에 틀림없다 러나 어사전에 표 적이고 품

위 있는 말만 어야 한다고 생각한다 것은 커다란

해다 당장 아 어사전이나 펼쳐 라 속어는

설과 같은 비어나 죄자들이 쓰는 은어 어

마니 같은 소 의 사람만이 쓰는 말 도 라 있

않은가 사전은 말 치에 일정 빈도 이상 나타나는 말이

라 말이든 다 할 있다

zb38) 가 나 에 대한 다음의 설( ) ( ) 않은 것은

① 가 는 얼짱 사 에 등재 것에( ) ( ) lsquo rsquo

보 고 다

② 사 등재 가는 단어 격에( )

고 고 는 언 들 언어 사 도에 고 다 ( )

③ 가 얼짱 어지만 신 과 같 매( ) ( ) lsquo rsquo

체에 도 사 는 말 는 고 다

④ 가는 얼짱 어 보고 크게 가지 근( ) lsquo rsquo 3

거 들어 뒷 고 다

⑤ 는 얼짱 어 는 다 특 다는( ) lsquo rsquo

근거 에도 크게 가지 근거 가 들어 주 2

뒷 고 다

가 늘 지 상에 살고 는 사 들 억( ) 10

도가 고 그리 지 통 고 는 사 들

그보다 훨 많 억 도는 고 지 20

통 다 그런 지 고 2500

년 학 간고사 대비2013 2 현대고 대비

ECN-0102-2013-001-000076193

그리 간 보는 과 사 에

매우 달 뿐만 니 과 에 도 극

루고 었다 미 운 그런 들

살고 는 동 과 사 들 사고 식에

큰 가 다는 다

고 그리 들 우주 개별 고 독립

사 들 생각 지만 고 들 우

주 연 질 간주 다 같 각

도 들에게는 연 질 었지

만 그리 들에게는 미 들 결 었다

고 과 그리 들 사 같 는

동 과 사 에 도 견 다

인 리학자인 츠 이마이 디드 겐트너는 두

살이 채 안 된 아이들에서 터 성인에 이르 다양한

연 대의 동양인과 서양인을 대상으 다음과 같은 험

을 했다 저 코르크 만든 피라 드 양의 도형을

여 주고 대상의 이름을 닥 라고 알 주었다lsquo (Dax)rsquo

제 닥 는 존재하 않는 것으 험자가 임의lsquo rsquo

만들어 낸 이름이다 런 다음 두 개의 다른 체를

여 주었는데 하나는 피라 드 양이 만 하얀 플라 틱

으 만들었고 다른 하나는 재 는 코르크 만 양이

달랐다 러고 나서 어떤 것이 닥 인 사람들에게 고 lsquo rsquo

르게 했더니 서양인들은 주 같은 양을 하고 있는

체를 선택했고 동양인들은 같은 재 만들어 체를

선택했다 이러한 차이는 성인은 어 두 살 리

아이들에게서도 나타났다 이것은 곧 서양인과 동양인은

서 다른 세상을 고 있다는 것을 의 한다 략 ( )

는 아주 단 하 서도 인상적인 험을 했다

험에는 동서양의 대학생들이 참여했다 는 험 참가자

들에게 컴퓨터 화 을 통해 속 장 을 담은 애니 이션

을 여 주었다 화 의 앙에는 초점의 역할을 하는 커

다란 고 한 마리가 있었고 주위에는 다른 생

들과 초 자갈 거품 등이 함 제 되었다 화 을

두 씩 후 참가자들은 자 이 것을 회상해 라는

를 았다

결과 서양인 대학생들과 동양인 대학생 두 앙

의 초점 역할을 했던 고 를 동일한 정도 언 했으

나 경 소 위 거품 초 다른 생 들 에 ( )

대해서는 동양인 대학생들이 서양인 대학생들 다 60

이상 더 많이 언 했다 뿐만 아니라 동양인 학생들은 서

양인 학생들에 비해 개 적인 고 다 전체적인 계

를 더 언 하는 경향을 다 략 또한 경의 일 ( )

를 화 킨 림을 제 하 을 때 동양인 대학생들은 대

경의 화를 알아챘 만 서양인 대학생들은 경

의 화를 거의 알아차리 했다 략 ( )

따라서 서양인들만을 대상으 연 한 화lsquo

편성 결 은 잘 된 것일 도 있다 각 과정과 인rsquo

과정의 어떤 이 화 편적이고 어떤 이

화에 따라 달라 는 는 앞으 많은 연 를 통하여 논의

되어야 한다

나 어떤 의 에서 리 두는 이 화적이다 리( )

안에는 다른 사람들과 더 친 한 계를 유 하 는 상호

의존성과 다른 사람들 터 독립적인 존재 살아가 는

독립성이 혼재한다 따라서 이 에서 어떤 특성이 더 강

하게 각되는 상황에 놓이느냐에 따라 서 다른 화적

특 을 일 있다 결 리 두는 어떤 경 에는

동양인처럼 행동하고 어떤 경 에는 서양인처럼 행동하는

것이다

zb39) 가 에 대한 다음의 설( ) 않은 것은

① 는 신 주 뒷 닥 실험과lsquo rsquo lsquo

니 실험 근거 시 다rsquo

② 동 들 상 간 공통 보다는 에 식

는 강 다

③ 들 주변 맥 에는 심 경 어 사건

과 사건 사 계에 상 민감 다

④ 는 동 과 틀린 지 고 는 것lsquo rsquo

니 다 고 다 lsquo rsquo

⑤ 가에 우리 사 들 개 시 가 원( )

집 경 말 고 는 것 개 보다는

에 고 는 것에 다

늘 지 상에 살고 는 사 들 억 도가10

고 그리 지 통 고 는 사 들( )知的

그보다 훨 많 억 도는 고 지 20

통 다 그런 지 고 2500

그리 간 보는 과 사 에

매우 달 뿐만 니 과 에 도 극

루고 었다 미 운 그런 들

살고 는 동 과 사 들 사고 식에

큰 가 다는 다

지심리 미 마 드 겐트 는 동

과 상 다 과 같 실험 다

크 만든 미드 도 보여 주고 그

상 닥 고 주었다 그런 다lsquo (Dax)rsquo

개 다 체 보여 주었는 는 미드

지만 틱 만들었고 다 는 재료는

크 지만 달 다 그러고 어 것 닥 lsquo

지 사 들에게 고 게 니 들 주 같rsquo

고 는 체 택 고 동 들 같

재료 만들어진 체 택 다 러 는

심지어 살짜리 들에게 도 타났다 것

곧 과 동 다 상 보고 다는

것 미 다 개별 사 보고 고 동

년 학 간고사 대비2013 2 현대고 대비

ECN-0102-2013-001-000076193

연 질 보고 는 것 다

동 들 주변 상 에 맞 어 동 고

에 다 사 들 태도 동에 보다

많 주 울 다 동 가 미시간

에 에 경험 다 그는 미

식 경 보러 가게 었는 경 체는 매우 재

미 었 주변 들 동에 질 다 그

는 들 계 어 상태 경

다 어 들 에 에 그 시 가 계

가 진 것 다 뒷사 고 지 는 들

동 럼 어 웠다

그는 경험에 어 얻어 동 들lsquo

각도 상 본다 는 가 우고rsquo

검 여 주 단 도 상 실험 실

시 다 그는 실험 가 들에게 컴퓨 통

담 니 보여 주었다

에는 역 는 커다 고 마리가 었

고 주 에는 다 생 들과 갈 거 등

께 시 었다 본 후 가 들

신 본 것 상 보 는 지시 다

그 결과 생들과 동 생

역 고 동 도 언

경 거 다 생 들에 ( )

는 동 생들 생들보다 60

상 많 언 다 뿐만 니 동 생들

생들에 개별 고 보다 체 계

언 는 경 보 다 경 변 시

킨 그림 시 동 생들 경

변 지만 생들 경 변

거 리지 못 다

지 지 들만 상 연 lsquo

보편 결 못 것 도 다 지각 과 과rsquo

지 과 어 보편 고 어

에 달 지는지는 많 연 통 여

어 다

리 드 니 벳 생각 지도 사- ldquo rdquo( 2004)

zb40) 위 에 대한 설 으 가장 적절한 것은

① 동 과 생 식 강 고 다

② 가지 실험 통 쓴 고 다

③ 닥 실험에 사 본질에 동 사

상에 주 다

④ 니 실험에 동 과 에 지

각 도에 가 다

⑤ 쓴 는 보편 연 에 드러 우월 에

에 근 고 다

가 동 들 주변 상 에 맞 어 동 고( )

에 다 사 들 태도 동에 보다 많

주 울 다 동 가 미시간 에

에 경험 다 그는 미식

경 보러 가게 었는 경 체는 매우 재미 었

주변 들 동에 질 다 그 는

들 계 어 상태 경 다

어 들 에 에 그 시 가 계 가

진 것 다 상 살펴lsquo 는 말 들rsquo

에 그는 에 시 어 도 뒷사

생각 곧 다시 곤 것 다 그런 그에게

뒷사 고 지 는 들 동 럼

어 웠다

그는 경험에 어 얻어( ) 동 들lsquo

각도 상 본다 는 가 우고rsquo

검 여 주 단 도 상 실험

실시 다 실험에는 동 생들 여 다

그는 실험 가 들에게 컴퓨 통

담 니 보여 주었다 에는

역 는 커다 고 마리가 었고 주 에는

다 생 들과 갈 거 등 께 시

었다 본 후 가 들 신 본 것

상 보 는 지시 다

다 그 결과 생들과 동 생( )

역 고 동 도 언

경 거 다 생 들 에 ( )

는 동 생들 생들보다 60

상 많 언 다 뿐만 니 동 생들

생들에 개별 고 보다 체 계

언 는 경 보 다 들어 동

생들 상 체 연못 럼 보 어ldquo 같rdquo

체 맥 언 시 었지만

생들 상 어 같 큰 고 가 쪽 움ldquo

직 어 같 역 고rdquo

언 시 다 경 변 시킨 그

림 시 동 생들 경 변

지만 생들 경 변 거

리지 못 다

년 학 간고사 대비2013 2 현대고 대비

ECN-0102-2013-001-000076193

게 볼 동 들 보다는 큰 그( )

림 보 에 사 과 체 맥 연결시 지각

는 경 고 체에 특 떼어 내

어 독립 보는 것 낯 어 다 에

들 사 에 고 주변 맥 에는 심 경

에 사건과 사건 사 계에 상

민감 편 다

마 지 지( ) 들만 상 연

보편 결 못 것 도 다lsquo rsquo 지각 과

과 지 과 어 보편 고 어

에 달 지는지는 많 연 통 여

어 다

리 드 니 벳 생각 지도 사- ldquo rdquo( 2004)

zb41) 의 하는 가~ 다른 것은

① ② ③

④ ⑤

얼마 그 에 동 사고 식과

사고 식 보여 주는 내 다

들 에 는 탕 고 같 게

어 겨 고 미 에 는 그 크 럼 큰 고

어리 주고 원 는 어 도 는

상 고 생각 다는 것 다 러

는 어떻게 생 것 고 과 그리 거슬

러 가 보 그 단 다

고 연 경 체 경 생 에

다 벼 사는 공동 업과 경험 많 연 역

에 고 들 연 웃과

게 지내 고 탁 연 들

들 지 연 럽게 들 다 민들

웃과 동 게 뿐만 니 는 집 과

게 다

동 시 는 생태 경 에 살 결과

들 다 사 들 사 상 에 주

울 게 었고 는 곧 체 상 과 간 사

계 시 는 낳게 었다 신 가

가 는 체에 는 원 는 동시

에 다 사 들 그 사 포 체 맥 에

다 들 간 사 연

계 체 계에 주 울 는 사고 체계

게 었다

그러 그리 연 경 그 었다 산

지 연결 는 지 건 그리고 역

에 다 런 들 업에 다 사 과

동 므 공동체에

다고 다 고 그리 들

들과는 달리 보 내 감 지 들과

지 크게 느 지 못 다 그

견 다 경우 주 쟁 통 결 는 갖

게 었다

신 사 간 계들 루어진 커다

트워크 에 게 당연 사 역시 연

계들 체 식 게 다 어 상

원 도 그 개체가 체 맥 과

계 에 고 다 게 체 맥 에 주

울 다 보 상 복 과 가변 식 게 고

상에 재 는 많 변 들 사 에 재 는 들도

게 다 들 주 태도 보

는 경우가 많다 쟁 결

통 결 보다는 통 결

는 보 다

그러 고 그리 들 개개 사 사 독

에 주 울 다 사 사 체에

어 그들 사 에 재 는 공통 규 주

고 다 상 원 에도 사

체 내 주 고 다 그들

체 여 탕 체

는 주 태도 시 고 특 사 어

주에 는지 여 그 주에 는 규

견 다 에 는 쟁 식 리

같 리 사고 체계가 달 게 었다

리 드 니 벳 생각 지도 사- ldquo rdquo( 2004)

zb42) 위 에서 사 된 설 과 가장 유사한 것은

① 크톱 컴퓨 는 본체 니 마우 루

어 다

② 곡과 시 리 는 지 과 사 루어 다는 공통

지니고 다

③ 경 고 것과는 달리

경 본 연 태 그 주변 경

④ 벽돌 능 에 사계 내내

습도가 지 다

⑤ 잰느 체 체 지닌 재 체가 없

는 재 눌 다

년 학 간고사 대비2013 2 현대고 대비

ECN-0102-2013-001-000076193

zb43) 는 립 앙 도서 이 정의 일 이다lt gt

도서 장과 이 자의 리 의 정의 연결이

적절하 않은 것은

lt gt

제 조 서 유8 ( )

도서 장은 다른 이 자의 안전을 위협하거나 도서 의①

서를 란하게 할 가 있는 자에 대하여는 도서 출입

을 제한할 있다

도서 장은 이 자가 제 조 각 호의 어느 하나의 행위를 하7②

을 때에는 이 을 하게 하거나 도서 출입을 제한할

있다

제 조자 의 대출9 ( )

도서 자 는 다음 각 호의 경 대출할 있다①

상호대차도서 간에 자 를 류하는 것을 말한다 등 다1 ( )

른 도서 과의 협 을 위하여 필 한 경

공 이 공 행 상 필 하는 경2

에 도서 장이 필 하다고 인정하는 경3

대출이 가능한 도서 자 의 위는 도서 장이 정하는②

에 따른다

제 조 상10 ( )

이 자가 도서 자 설을 더럽히거나 찢거나 뜨①

쓰게 하거나 잃어 린 경 에는 상하여야 한다

도서 장은 제 항에 따른 상 을 정하여 게 하여야1②

한다

제 조이 절차 등11 ( )

이 칙에서 정한 것 에 도서 자 설의 이 절차

이 제한 등에 필 한 사항은 도서 장이 정한다

출처 립 앙 도서- (httpwwwnlgokr)

① 는 도 리 다8

② 도 는 리 다9 1

③ 료 지 는 도 리 다9 2

④ 도 료 변상에 리10 1

⑤ 는 에 도 리 다11

3

도 다 각 같다①

공 공 다만 연1

연 간 다

매월 째 째 월2

도 도 리 그 사3

가 다고 는

도 에 미리 게1 3②

시 여 다

4

도 시간 도 여 게시 다

5

도 료 시 는 는 도①

지에 등 후

등 에 사 도②

7

는 다 각 여 는 니 다

도 료 시 상 리1 lsquo rsquo

도 료 시 훼 는2 middot

지 가 닌 곳에 식 거 담3

우는

도 보 등 보 검색열4 middot

그 에 도 질 지 여 도5

여 게시 사 는

8

도 다 거 도①

질 게 우 가 는 에 여는 도

도 가 각 어느7②

에는 지 게 거 도

9

도 료는 다 각 경우 다①

상 도 간에 료 는 것 말1 (

다 등 다 도 과 여 경우)

공 원 공 상 는 경우2

그 에 도 다고 는 경우3

가능 도 료 는 도②

는 에 다

10

년 학 간고사 대비2013 2 현대고 대비

ECN-0102-2013-001-000076193

가 도 료 시 럽 거 거①

못 쓰게 거 어 린 경우에는 변상 여

도 에 변상 여 게시1②

여 다

zb44) 위 에서 도서 장이 게 해야 할 사항에 해당하는

것을 두 쓰

년 학 간고사 대비2013 2 현대고 대비

ECN-0102-2013-001-000076193

립 도 규

1 ( )

규 립 도 립 어린 청 도(

포 다 료 시 열 시 말) (

다 에 사 규 립 도)

편 진 다

2 ( )

규 립 도 도 다 에( lsquo rsquo )

고 는 도 에 도lsquo rsquo 2 2

료 에 여 다 다만 특 료 귀

료 등 료 에 사 립 도

도 다 다( lsquo rsquo )

3 ( )

도 다 각 같다①

공 공 다만 연1

연 간 다

매월 째 째 월2

도 도 리 그 사3

가 다고 는

도 에 미리 게1 3②

시 여 다

시간4 ( )

도 시간 도 여 게시 다

등 등5 ( )

도 료 시 는 는 도①

지에 등 후

등 에 사 도②

사 료6 ( )

도 료 시 에 사 료는 도

7 ( )

는 다 각 여 는 니 다

도 료 시 상 리1 lsquo rsquo

도 료 시 훼 는2 middot

지 가 닌 곳에 식 거 담3

우는

도 보 등 보 검색열4 middot

그 에 도 질 지 여 도5

여 게시 사 는

질 지8 ( )

도 다 거 도①

질 게 우 가 는 에 여는 도

도 가 각 어느7②

에는 지 게 거 도

료9 ( )

도 료는 다 각 경우 다①

상 도 간에 료 는 것 말1 (

다 등 다 도 과 여 경우)

공 원 공 상 는 경우2

그 에 도 다고 는 경우3

가능 도 료 는 도②

는 에 다

변상10 ( )

가 도 료 시 럽 거 거①

못 쓰게 거 어 린 경우에는 변상 여

도 에 변상 여 게시1②

여 다

등 규 에 것 에 도11 ( )

료 시 등에 사

도 다

립 도- (httpwwwnlgokr)

zb45) 도서 장의 리 있는 조항으 적절하 않

은 것은

① ② ③ ④ ⑤

년 학 간고사 대비2013 2 현대고 대비

ECN-0102-2013-001-000076193

1 ( )

사가 공 는lsquo rsquo

과 여 사 원과 리

사 타 사 규

니다

개 보 보7 ( )

사는 보통신망 등 계 는 에lsquo rsquo lsquo rsquo

원 개 보 보 니다 개lsquo rsquo

보 보 사 에 는 사 개lsquo rsquo

보 취 니다 다만 사는 다 lsquo rsquo

사 계 통 공 는 경우 원 lsquo rsquo

등 개 보 당 사에 습니lsquo rsquo

원 리에8 (lsquo rsquo lsquo rsquo lsquo rsquo

)

원 에 리lsquo rsquo lsquo rsquo lsquo rsquo①

원에게 가 도 여 는lsquo rsquo 3

니다

사는 원 가 개 보 우 가lsquo rsquo lsquo rsquo lsquo rsquo②

거 사 경우 는 미 에 어 거 lsquo

사 사 운 우 가 는 경우 당rsquo lsquo rsquo

습니다lsquo rsquo

원 가 도 거lsquo rsquo lsquo rsquo lsquo rsquo 3③

가 사 고 지 경우에는 시 사에lsquo rsquo

통지 고 사 내에 니다lsquo rsquo

경우에 당 원 사에 그 사실3 lsquo rsquo lsquo rsquo④

통지 지 거 통지 도 사 내에 지 lsquo rsquo

생 경우 사는 지지 습니다lsquo rsquo

사10 (lsquo rsquo )

사는 과 지 미lsquo rsquo①

에 는 지 계 고

공 여 다 여 니다lsquo rsquo

사는 원 게lsquo rsquo lsquo rsquo lsquo rsquo②

도 개 보 신 보 포 보 보 시( )

갖 어 개 보 취 공시 고

니다

사는 과 여 원lsquo rsquo lsquo rsquo③

견 만 당 다고 경우에는

리 여 니다 원 견 만 사 lsquo rsquo

에 는 게시 거 우편 등 통 여

원에게 리 과 결과 달 니다lsquo rsquo

원11 (lsquo rsquo )

원 다 여 는 니다lsquo rsquo ①

신청 는 변경 시 허 내 등1

타 보 도2

사가 게시 보 변경3 lsquo rsquo

사가 보 보 컴퓨 그4 lsquo rsquo (

등 등 신 는 게시)

사 타 등 지 재산 에5 lsquo rsquo 3

사 타 상 거 업6 lsquo rsquo 3

는 폭 시지 상 타 공7 middot middot

에 는 보 에 공개 는 게시 는lsquo rsquo

사 동 없 리 사8 lsquo rsquo

타 거 당9

게시15 (lsquo rsquo )

원 내에 게시 는 게시 게재 는lsquo rsquo lsquo rsquo lsquo rsquo

경우 원 사가 게시 복 lsquo rsquo lsquo rsquo lsquo rsquo middot middot

등 태 언 등에 공 는

것 내에 다 원 본 게시 등 lsquo rsquo lsquo rsquo

크 능 등 여 복 는 등 태

는 것 동 것 니다

- (wwwnavercom)

zb46) 위 은 인터넷 포털사이트의 회 가입을 위한 이

약 의 일 이다 이 약 을 만드는 과정에서 생각한

내 으 적절하 않은 것은

개 보 보 가 지에 별 눠①

겠어

원 가 만들게 에②

시 주어 겠어

원들 게재 게시 다 원 크 다③

는 것 지

④ 원 지 는 뿐만 니 사가 지 는

도 께 달 지

리에 가 생 경우 사가⑤

에 다는 도 듯

1 ( )

사가 공 는lsquo rsquo

과 여 사 원과 리

사 타 사 규

년 학 간고사 대비2013 2 현대고 대비

ECN-0102-2013-001-000076193

니다

개 보 보7 ( )

사는 보통신망 등 계 는 에lsquo rsquo lsquo rsquo

원 개 보 보 니다 개lsquo rsquo

보 보 사 에 는 사 개lsquo rsquo

보 취 니다 다만 사는 다 lsquo rsquo

사 계 통 공 는 경우 원 lsquo rsquo

등 개 보 당 사에 습니lsquo rsquo

원 리에8 (lsquo rsquo lsquo rsquo lsquo rsquo

)

원 에 리lsquo rsquo lsquo rsquo lsquo rsquo①

원에게 가 도 여 는lsquo rsquo 3

니다

사는 원 가 개 보 우 가lsquo rsquo lsquo rsquo lsquo rsquo②

거 사 경우 는 미 에 어 거 lsquo

사 사 운 우 가 는 경우 당rsquo lsquo rsquo

습니다lsquo rsquo

원 가 도 거lsquo rsquo lsquo rsquo lsquo rsquo 3③

가 사 고 지 경우에는 시 사에lsquo rsquo

통지 고 사 내에 니다lsquo rsquo

경우에 당 원 사에 그 사실3 lsquo rsquo lsquo rsquo④

통지 지 거 통지 도 사 내에 지 lsquo rsquo

생 경우 사는 지지 습니다lsquo rsquo

원에 통지9 (lsquo rsquo )

사는 특 다 원에게 통지 경우lsquo rsquo lsquo rsquo

공지 게시 통 상 게시 개별 통지에7

갈 습니다

사10 (lsquo rsquo )

사는 과 지 미lsquo rsquo①

에 는 지 계 고

공 여 다 여 니다lsquo rsquo

사는 원 게lsquo rsquo lsquo rsquo lsquo rsquo②

도 개 보 신 보 포 보 보 시( )

갖 어 개 보 취 공시 고

니다

사는 과 여 원lsquo rsquo lsquo rsquo③

견 만 당 다고 경우에는

리 여 니다 원 견 만 사 lsquo rsquo

에 는 게시 거 우편 등 통 여

원에게 리 과 결과 달 니다lsquo rsquo

원11 (lsquo rsquo )

원 다 여 는 니다lsquo rsquo ①

신청 는 변경 시 허 내 등1

타 보 도2

사가 게시 보 변경3 lsquo rsquo

사가 보 보 컴퓨 그4 lsquo rsquo (

등 등 신 는 게시)

사 타 등 지 재산 에5 lsquo rsquo 3

사 타 상 거 업6 lsquo rsquo 3

는 폭 시지 상 타 공7 middot middot

에 는 보 에 공개 는 게시 는lsquo rsquo

사 동 없 리 사8 lsquo rsquo

타 거 당9

원 계 규 내lsquo rsquo lsquo②

여 공지 주 사 사가 통지 는rsquo lsquo rsquo

사 등 여 타 사 업 에 lsquo rsquo

는 여 는 니다

- (wwwnavercom)

zb47) 위 약 의 조항에서 같은 제점을 하lt gt

고 있는 조항은

lt gt

제휴 회사에 회 의 아이디 개인 정 를 전송할 있도

한 조항은 고객에게 당한 조항이다

1 7 8① ② ③

④ 9 ⑤ 10

립 도 규

1 ( )

규 립 도 립 어린 청 도(

포 다 료 시 열 시 말) (

다 에 사 규 립 도)

편 진 다

2 ( )

규 립 도 도 다 에( lsquo rsquo )

고 는 도 에 도lsquo rsquo 2 2

료 에 여 다 다만 특 료 귀

료 등 료 에 사 립 도

도 다 다( lsquo rsquo )

3 ( )

도 다 각 같다①

공 공 다만 연1

연 간 다

년 학 간고사 대비2013 2 현대고 대비

ECN-0102-2013-001-000076193

매월 째 째 월2

도 도 리 그 사3

가 다고 는

도 에 미리 게1 3②

시 여 다

시간4 ( )

도 시간 도 여 게시 다

등 등5 ( )

도 료 시 는 는 도①

지에 등 후

등 에 사 도②

사 료6 ( )

도 료 시 에 사 료는 도

7 ( )

는 다 각 여 는 니 다

도 료 시 상 리1 lsquo rsquo

도 료 시 훼 는2 middot

지 가 닌 곳에 식 거 담3

우는

도 보 등 보 검색열4 middot

그 에 도 질 지 여 도5

여 게시 사 는

질 지8 ( )

도 다 거 도①

질 게 우 가 는 에 여는 도

도 가 각 어느7②

에는 지 게 거 도

료9 ( )

도 료는 다 각 경우 다①

상 도 간에 료 는 것 말1 (

다 등 다 도 과 여 경우)

공 원 공 상 는 경우2

그 에 도 다고 는 경우3

가능 도 료 는 도②

는 에 다

변상10 ( )

가 도 료 시 럽 거 거①

못 쓰게 거 어 린 경우에는 변상 여

도 에 변상 여 게시1②

여 다

등 규 에 것 에 도11 ( )

료 시 등에 사

도 다

립 도- (httpwwwnlgokr)

zb48) 다음 정 리 의 의 으 볼 때 가장

이 적인 것은

도 시간 도 여 게시 다①

등 에 사 도②

가능 도 료 는 도 는③

에 다

④ 도 에 변상 여 게10 1

시 여 다

⑤ 도 가 각 어느7

에는 지 거 도

zb49) 를 참고하여 이 어의 성격을 설 한lt gt

것으 적절하 않은 것은

① 보 에 는 어 시 상 고 어 시lt gt lsquo rsquo

에 보여주고 다

② 진 어 어원에 견 고 다

에는 타 어 들어가는 것 다 lsquo rsquo

③ 에 들어갈 말 각각 고 어 어 신 어~

들 언어는 질 격 강 통 없었다

④ 시 우리 에 가 었지만 지 계

과 달리 들 통 사 달 어 웠

년 학 간고사 대비2013 2 현대고 대비

ECN-0102-2013-001-000076193

⑤ 크 몽골 만주 공통어가 우리 어 같

계열에 다는 에 사 특 짐

가( )

善化公主主隱 공주님

他密只嫁良置古 몰 결 고

薯童房乙 맛

夜矣卯乙抱遣去如 에 몰 고 가다

( )

始汝 會隱日恚見隱扐 만 에 본

恥隱汝衣淸隱笑 맑 웃

고 시 여 공 크다 만 다[ ] ( ) ( ) ( ) ( )始 汝 會扐

내다 에 보다 견( ) ( )恚 見 다( )隱

럽다 맑다 청 웃( ) ( ) ( ) ( )恥 衣 淸 笑

zb50) 위의 나 를 함 고 음에 답하( ) lt gt

보lt gt

( )素那或云金川 白城郡蛇山人也

운 사산

는 고 다 는( )[ ( ) ] (素那 金川 白城

사산 사 다) ( ) 郡 蛇山

삼 사- lsquo rsquo 47

에 제 된 단어 의 표 리를 조건(1) lt gt ( ) lt gt

에 맞게 서 하

건lt gt

lsquo 었고 었다 태rsquo

에 제 된 단어 동일한 표 리에(2) lt gt ( )

의해 적은 것을 나 에서 찾아 조건 에 맞게 서 하( ) lt gt

건lt gt

에 당 는 각각( ) 개 쓸 것2 단

당 는 가 여러 개 어도 개만 쓸 것 각2

개 과 도 쪽에 개만2 2

드시 지 것( )

과 동 원리 것lsquo 고

과 동 원리 것 다rsquo

태 것

가( )

素那(或云金川) 白城郡蛇山人也

소나 또는 천 이라 한다 는 성 사( ) ( ) ( )素那 金川 白城郡〔 〕

산 사람이다 현대어 풀이( ) ( )蛇山

나( )

紫布岩乎希 회

執音乎手母牛放敎遣 자 손 암쇼 노히 고

吾 不喩慙 伊賜等肹 肹 나 안디 리샤

花 折叱肹 可獻乎理音如 고 것거 도림다

다 향찰은 리말을 리 으 적은 표 이었 만 생( )

은 고 대를 넘 하고 끊어 고 말았다 랜 세

동안 갈고 닦아 체계적이었던 향찰 표 이 사라졌

을 인은 크게 두 가 나누어 생각해 볼 있다

하나는 족 사회의 한 선호도에서 찾을 있다 라 때

향찰은 주 족 계 에서 사 했을 것으 인다 한 을

알 하고서는 한자를 활 하여 리말을 리 으 표

하 란 가능하 때 이다 런데 족들은 간이 흐

를 향찰과 같은 리 표 을 익혀 사 하 다는

아 한 을 대 사 하는 쪽을 선호하게 되었다 더 이

고 초에 인재 등 을 위해 과거제도가 행되 서 한 선

호도가 더 높아졌고 결 향찰은 소 되고 말았다

또 다른 가능성은 한 어의 특성에서 찾을 있다

터 한 과 일 세 나라는 한자 화 에 속해 다

당연한 이야 겠 만 표의 자인 한자는 어를 표 하

에 매 적절하다 어의 음절은 성 ( ) ( )聲母 韻母

이 어 고 여 에 성조가 추가되어 최종 소리가 결정된

다 래서 어는 단음절을 하나의 한자 표 하 된

다 에 초성 성 종성의 세 가 소가 하나의 음절

년 학 간고사 대비2013 2 현대고 대비

ECN-0102-2013-001-000076193

을 이 는 한 어는 음절 조가 잡하고 음절의 가 많아

서 한자 차 만으 한 어의 소리를 만족 럽게 표 할

없었다 를 들어 한 어에서는 어 니 같이 음절 lsquo rsquo

이 어 단어가 얼마든 있으나 어는( ) 複數音節

자 하나 나타내 만이다lsquo [m ]rsquo 母 ǔ

한편 일 어의 표 은 핵 적 단어는 한자 적고 토는

가나라는 일 의 자 적는 이다 적인 의 를 나

타내는 은 표의 자인 한자 적고 적 계를 나

타내는 토는 표음 자 적는 셈이니 자세히 살펴

리의 향찰 표 을 쏙 빼닮았음을 알 있다 한 어 같

은 착어이 서도 일 어에만 향찰과 유사한 표 이 살아

남은 것은 일 어의 특 때 이다 일 어는 하나의 자음과

음의 결합으 음절을 이 고 침이 거의 없는 음절 언어

이다 이러한 음절의 특색에다가 토가 달한 착어라는 점

이 향찰과 유사한 표 이 살아남을 있는 비결이었다

하 만 같은 착어라도 다양한 음소 침이 달한 한

어는 향찰 표 하는 데 근 적으 한계가 있었다

zb51) 다 하여 의 행에 대한 탐 한 결과( ) lt gt 2

않은 것은

보lt gt

善花公主主隱 공주니믄 공주님( )

----------------------------------------

-

他密只嫁良置古 그 지 얼어 고 몰 결(

----------------------------------------

-

薯童房乙 맛 맛( )

夜矣卯乙抱遺去如 몰 고 가다 에 몰 고(

가다)

주동 역 동- (薯童謠『 』

에 2 ( )他密只嫁良置古

얼다 시집가다 결 다 말 lsquo rsquo

① 실질 미 지니고 므 타 타lsquo ( )rsquo lsquo [ ]

② 에 실질 미 타내고 지 는lsquo rsquo lsquo [ ]rsquo lsquo [ ]密只 密 只

계 타내는

③ 얼어는 실질 미 포 고 므 가lsquo rsquo lsquo [ ]rsquo嫁

것lsquo [ ]rsquo 良

④ 고 어간 는 실질 미 지니고 므lsquo rsquo lsquo -rsquo

것lsquo [ ]rsquo 置

⑤ 고 어미 고는 계 타내고 므lsquo rsquo lsquo- rsquo

고 것lsquo [ ]rsquo 古

가( )

엉 훈 민middot middot middot middot middot世 宗 御 製 訓 民 正 音

말 미 듕 귁에 달middot middot middot middot middot middot middot middot中 國 文 字

니 런middot middot middot middot middot middot 어린middot middot middot middot百 姓

니 고 도 내 들middot middot middot middot middot middot middot middot middot 시러middot

펴 몯middot 미middot middot 니 내middot middot middot middot middot middot middot middot 爲

어엿middot 겨 새middot middot middot 믈여듧middot middot middot middot字 니middot middot middot

사 마다 니겨 킈 middot middot middot middot middot middot middot middot middot便 安

고 미니middot middot middot middot

본 는 상( ) (象

원리에 만들어진 본) ( )形 ㄱ ㄴ ㅁ ㅅ ㅇ

에 는 가 원리에( )加劃

그리고( )ㅋ ㄷ ㅌ ㅂ ㅍ ㅈ ㅊ ㆆ ㅎ

쓰는 병 원리에 만들어진( )竝書

마지막 체( ) ( )異體ㄲ ㄸ ㅃ ㅆ ㅉ ㆅ

ᅀ 다 상 원리에 ㅇ ㄹ

지 는 삼재 상 본 본( ) ( ) ( 天地人 三才

탕 므림과 림에 ) (初ㅡ ㅣ

재)( ) ( )( )出字 再出字ㅗ ㅏ ㅜ ㅓ ㅛ ㅑ ㅜ ㅕ

병 그리고 들 에 다시( )ㅘ ㅝ ㅣ

( )ㅣ ㅢ ㅚ ㅐ ㅟ ㅔ ㆉ ㅒ ㆌ ㅖ ㅙ ㅞ

zb52) 가 에 대한 설 으 르 않은 것을( ) 두 고르

① 어쓰 규 지키고 다

② 리 고 다

③ 말 미 미 등 어 사 다lsquo rsquo

④ 개 지 다

년 학 간고사 대비2013 2 현대고 대비

ECN-0102-2013-001-000076193

⑤ 어 원 에 가 도 고 다

엉 훈 민世 宗 御 製 訓 民 正 音

말 미 듕귁에 달 니

런 어린 니 고 도middot

내 들 시러 펴 몯 미 니middot

내 어엿 겨 새 믈여듧

사 마다 니겨middot 킈 고

미니

훈민 언 본- lsquo rsquo 5 (1459 )

zb53) 위의 에 대한 현대어 풀이가 르~ 않은 것

① 우리 말 과 달

② 어리 말 고 는 것 어도

③ 신 생각 마 껏 펼 는 사 많다

④ 게 생각 여

⑤ 사 마다 게

zb54) 훈민정음 언해 에는 한 을 창제한 동 가 드러나

있다 훈민정음 창제의 정 과 내 이 잘 연결된 것

① 주 신 말 미 듕귁에 달

② 민 신 내 어 겨

③ 신 뻔 킈 고 미니

④ 실 신 사 마다 니겨

⑤ 귀 신 계 주 는 훈민 신과 거리가

가 엉 훈 민( ) middot middot middot middot middot世 宗 御 製 訓 民 正 音 

말 미 귁에 中 國 달 文 字

니 런 어린 니 百 姓

고 도 내 들 시러 펴 몯

미 니 내 어엿 爲 겨 새

믈여듧 니 사 마다 니 字

겨 킈 고 미니 便 安

훈민 언 본- lsquo ( )rsquo ( ) 5 (1459 )訓民正音 世祖

( )

[ 1 ]

동 룡 샤 마다 복( ) ( ) ( )海東 六龍 天福

시니 고 동( ) ( )古聖 同符 시니

[ 2 ]

매 니 곶 여

미 므 니 그 내 러

가 니

[ 125 ]

우 미리( )千世 샨( )定 에( )漢水北 累仁

누 개 샤 복 업 시니( ) ( ) 開國 卜年

신( )聖神 니 샤도 경 근민 샤 욱( )敬天勤民

드시리 다

님 쇼 산 가( ) ( )洛水 山行

미드니 가

어 가- lsquo ( )rsquo 27龍飛御天歌

다 우리신 니쓰고 다만 만 쓰( )

거 샹 귀쳔 다보게 러 귀

여 쓴 도 신 보 가 고 신 에

말 어 보게 각 에 사 들

고 본 몬 능통 후에

죠 죠 니

드 도 만 공 에 사

드 미 죠 고 고 여 보 죠

보다 얼마가 거시 어신고 니 첫

가 죠 니 죠

민 들 어 신 샹

귀쳔 도보고 어보 가 만 늘

고 폐 에 만쓴 죠 민

도 러보지못 고 보니 그게 엇지

심 니 리 보 가 어 운건 다

니 쳣 말마 지 니 고 그

쓰 에 가 우 지 지

몰 거 본후에 가 어 지

고 그니 쓴편지 쟝 보

년 학 간고사 대비2013 2 현대고 대비

ECN-0102-2013-001-000076193

쓴것보다 듸 보고 그 마 니 쓴 고

어 못

그런고 에 리 과 가

만 쓴 못 민 말만 듯고

고 편 그 못 보니 그사 단

병신 못 다고 그사 식 사

니 만 고 다 과 그사

만 고 다 과 업 사 보다 식 고

죠 도 고 각 과

견 고 실 직 귀쳔 간에 그

고도 다 것 몰 귀죡 보다

사 우리 신 귀쳔 다 업

시 신 보고 과 지 게 랴

시니 샹 귀쳔 간에 우리 신 걸

간 보 새지각과 새 걸 미리

독립신- lsquo (1896)rsquo

zb55) 친 어 나의 제 장( ) 2 매 함축적

의 가 가장 유사한 것은

① 지 눈 내리고 매 득 니 내 여 가

사- lsquo rsquo

② 도 어 리듯 그 게 어 다

주 사- lsquo rsquo

③ 눈 살 다 죽 어 린 과 체 여

눈 새벽 지 도 살 다

눈- lsquo rsquo

④ 삶 근심과 고단 에 돌 거니는 여 거 는

여 리 내린 살가지 에 눈 리 눈 리

택 그 생 에- lsquo rsquo

⑤ 늘 러 고 러

청룡 룡 어 개 루 우

신경림 계- lsquo rsquo

zb56) 친 를 위 가 나 에 나타난A B ( ) ( )

세 어의 특 에 의거하여 세 어 표 하

그 산 고 공 도 맑지만

A

주변에 쓰 리는 어리 사 많다

B

건lt gt

식 가 에 타 어 특징에( ) ( )

거 과 어쓰 는 고 지 말 것

A

B

zb57) 가 의( ) 달 아ㆍ 다 의 ( ) 나셔에서 알 있는

세 어 개화 어의 특 을 비 하여 조건 에lt gt

맞게 서 하

건lt gt

어에 는lsquo 개

어에 는 다 태rsquo

zb58) 은 가 는 다 에 나 는 절lt 1gt ( ) lt 2gt ( )

일 를 췌한 것이다 의 의 가 lt 1gt (1)~(2)

유사한 말을 에서 찾아 쓰lt 2gt

보lt 1gt

런 (1) 어린 니 고百 姓

도 내 들 시러 펴 몯 미

사 마다 (2) 니겨 便 安

킈 고 미니

보lt 2gt

죠 고 고 여 보 죠

보다 얼마가 거시 어신고 니 첫 가

죠 니 죠 민

들 어 신 샹 귀쳔

도보고 어보 가 만 늘 고

폐 에 만쓴 죠 민 도

러보지못 고 보니 그게 엇지 심

니 리

년 학 간고사 대비2013 2 현대고 대비

ECN-0102-2013-001-000076193

lt 1 gt

동 룡 샤 마다 복 시( ) ( ) ( )海東 六龍 天福

고 동 시니( ) ( )古聖 同符

lt 2 gt

(A) 매 니 곶

여 니

미 므 니 그 내

러 가 니

lt125 gt

우 미리 샨 에( ) ( ) ( ) 千世 定 漢水北 累

누 개 샤 복 업 시 니( ) ( ) 仁開國 卜年 聖

신( ) 神 니 샤도 경 근민 샤( ) 敬天勤民

욱 드 시 리 다

님 쇼 산 가 ( ) ( )洛水 山行

미드니 가

- lt gt龍飛御天歌

zb59) 장과 내 상 유사한 성격의 조는125

① 뫼 고 고 고 고

어 그린 많고 많고 고 고

어 러 는 울고 울고 가느니

도 견- lt gt

② 강 에 드니 몸 다

그믈 고 가니

뒷 뫼 엄 언 니( )藥

-

③ 말 없는 청산 태 없는 다

값 없는 청 없는 월

에 병 없는 몸 별 없 늙 리

-

④ 가마귀 골에 가지 마

낸 가마귀 새

청강에 것 시 몸 러 가( ) 淸江

-

⑤ 진 골에( ) 白雪

가 매 는 어느 곳에 었는고

에 갈 곳 몰( ) 夕陽

색-

zb60) 위 에 나타난 세 어의 특 으 적절하 않은

것은

① 룡 어 주격 사에 당 는 가 사( ) lsquo rsquo六龍

고 다

② 샤 어에도 어 주체 쓰 다

는 것 다

③ 매 어 달리 사 택에 어

가 지 지지 고 다

④ 므 원 상 직 어 지 다

⑤ 드시리 다 주체 과 상 께 사

고 다

수고 하셨습니다hearts hearts

년 학 간고사 대비2013 2 현대고 대비

ECN-0102-2013-001-000076193

보닷컴에 공 는 별 보는 고등

들 여 주 는

들 습니다 슷 동 지

가 복 는 것 도가

니 복 여 습 시고 거 시

니다

정답 해설

1) 정답[ ] ④

해설 다른 것은 두 특정 업이나 단 내에서 사[ ]

하는 일종의 은어 사회 언에 해당한다 러나

는 언이 아니라 단과대학을 여서 단대 사lsquo rsquo lsquo rsquo lsquo④

대학을 여서 사대라고 한 말에 해당하 일rsquo lsquo rsquo

사회에서도 널리 쓰이 사회 언이라 할

없다

2) 정답[ ] ⑤

해설 사회 언은 같은 단 내에서 쓰이는 언어이[ ] lsquo rsquo

동일 단끼리는 단결 과 친 감을 형성하는

능을 하 리적 안감이 일어나 않는다

3) 정답[ ] ③

해설 사람이라는 차 적 표현에 대한 대안적 표현이[ ]lsquo rsquo

인 아내 처 등으 볼 있다lsquo rsquo

4) 정답[ ]⑤

해설 남성은 주 격 체를 사 한다[ ]

5) 정답[ ] ⑤

해설 흑인은 검다라는 뜻을 가 고 있을 뿐 인[ ]lsquo rsquo lsquo rsquo lsquo rsquo

다 열등한 뜻을 내포하 않는다

6) 정답 살 색 첫 작품[ ] - -

해설 살색 혹은 킨색은 한 인의 피 색을 뜻[ ] lsquo rsquo lsquo rsquo

하는 것으 인종 차 을 추 고 출 이주민

의 평등 을 침해할 있어 년 표 이2005

살 색으 이름을 꾸었다 처녀작은 처녀라lsquo rsquo lsquo rsquo lsquo rsquo

는 단어가 가 고 있는 곡된 성 인 을 한 것

으 첫 작품정도 꾸어 사 하는 것이 좋다lsquo rsquo

7) 정답[ ] ⑤

해설 호는 아들에게 해체를 사 하고 있다[ ] ① ②

장 을 성하는 청자는 자 의 아 느리 아lsquo

들 세 이다 호는 아 느리에게 해rsquo ③

체를 사 하고 있다 호가 느리 아 에게 ④

사 한 해 체 아들에게 사 한 해체는 두 비lsquo rsquo lsquo rsquo

격 체에 해당한다 호는 자 의 아랫사람인 ⑤

느리에게 아들과 마찬가 해체를 사 하는 것이

상 이 만 임 을 한 느리에게 고마 과 쁨

존 의 표 를 하 위해 자 의 아 에게 말하듯

해 체를 사 하고 있다

8) 정답[ ] ③

9) 정답[ ] ⑤

10) 정답[ ] ①

해설 청자 할아 가 장의 주체 아 다 높을[ ] ( ) ( )

경 에는 압존 에 의해 장의 주체를 높이 않는lsquo rsquo

다 러 아 서가 아닌 아 는으 계 lsquo rsquo lsquo rsquo lsquo

니다 가 아닌 있 니다 표현하는 것이 르rsquo lsquo rsquo

11) 정답 당이 당을 쫒았다 당이[ ]

당에 다

해설[ ]

12) 정답[ ] ⑤

해설 서 다른 높임표현을 통해 청자에 대해 리[ ] ⑤

적 거리감을 나타내는 인 은 이 아니라 현정이

다 가 에서 현정은 에게 해 체를 사 함으 써 ( )

친근감을 드러낸다 나 에서 연 을 게을리하는 역 ( )

도 들 때 에 화가 난 현정이 선생님에게 항의하

는 장 에서는 하 체를 사 하여 리적 거리lsquo rsquo

가 어졌음을 나타내고 있다

13) 정답[ ] ①

해설 는 는 얼 빛이 날과 어찌 다르 고[ ] lsquo rsquo

라는 뜻으 전과 달리 임이 화자를 않고

있음을 알 있다

14) 정답 달리 후 가 있다 이를 통해 경[ ] lt gt

쾌한 음악성을 형성하고 노 젓는 상황을 체적으

형상화하는 역할을 한다

15) 정답[ ] ①

16) 정답[ ] ⑤

해설 다 의 자연은 를 성찰하게 하는 대상[ ] ( )⑤

이자 정의 대상이다 의 자연은 자 의 상황과 ⑤

처 를 드러내는 경으 서의 역할을 하 이

이 없다

17) 정답[ ] ③

해설 는 빈천 을 해결하고자 했으나 강산[ ] lsquo ( )rsquo 貧賤③

과 풍 을 달라는 에 거절하 다고 함으 써 자

연에 대한 애정을 드러내고 있으 는 않는

임에 대한 망을 개에게 전가 켜서 임에 대한 리

을 드러내고 있다

18) 정답[ ] ③

년 학 간고사 대비2013 2 현대고 대비

ECN-0102-2013-001-000076193

19) 정답[ ] ⑤

해설 고상한 음악가의 이름을 리말 꽝 럽[ ]

게 꿈으 써 언어유희를 통해 음을 유 하고 있

다 이는 고상한 척하는 총 를 비꼼으 써 비판적

태도를 드러내는 것이 대상을 꽝 럽게 표현

하여 총 의 허 과 사치를 풍자하고 있다

20) 정답[ ] ⑤

해설 는 작품 속 경에 대한 설 이 드러나는 것이[ ]

서 자의 주 적인 견해가 접적으 드러나는 것이

아니다

21) 정답[ ] ⑤

22) 정답[ ] ②

23) 정답[ ] ④

24) 정답[ ] ①

해설 적강 티프는 주인공의 비 한 출생이나 능[ ] ①

과 이 있는 것으 조정의 능함을 풍자하는lsquo rsquo

것과는 거리가 다

25) 정답 픔 나[ ] ( )

해설 의 음악은 고통 는 사람들을 위 하고 아픔[ ] lsquo rsquo

을 치유해 주는 능을 한다고 할 있다 의 lt gt

픔 도 소 된 이 과 더 어 살아가는 따뜻한 마음lsquo rsquo

을 상 한다

26) 정답[ ] ⑤

해설 에게 선천적으 주어 각 장애라는 역경[ ]

은 의 이라는 가사 연 을 있다lsquo rsquo

27) 정답[ ] ④

해설 는 장 란 선 에게 은 개인적인 인상을[ ]

소녀 장정 등으 표현한 것이다lsquo rsquo

28) 정답[ ] ②

해설 담자가 피 담자의 언어적 표현이나 비언어[ ]②

적 표현 하 독자는 담의 위 나 피

담자의 감정 상태를 알 있다 이를 통해 독자는

담 상황을 더 생생하게 느낄 있고 피 담자

를 더 잘 이해할 있게 된다

29) 정답[ ]③

해설 일상생활과 역도 선 서의 성과에 된 것에서[ ]

역도를 하 서 겪는 어 과 내적 고민으 화제를

전화하 위한 것이다

30) 정답[ ] ①

해설 릿속에 새겨 넣듯 이 억되도 함 세상[ ] ② ③

살이가 힘들고 고생 러 속 하여 자유를 ④

가 없는 고통의 상태를 비유적으 이르는 말

적의 침입을 막 위해 쌓은 축 켜야 할⑤

대상을 비유적으 이르는 말이다

31) 정답[ ] ④

해설 이 의 종류는 전 으 인 사건 경[ ] lsquo

비평을 성 소 삼는다rsquo

32) 정답[ ] ④

해설 근은 삼대독자 태어났음을 에서 확인할[ ]

있다 형제들과의 담은 이뤄 가 없다

33) 정답[ ] ⑤

해설 근은 가난에도 하고 화가를 꿈꾸었다[ ] (3

단 또한 다른 화가 망생들은 정 육을)

위해 상 학 학 해 유학 에 랐 만

근은 다른 을 찾아야 했다 단 세에(5 ) 18

근은 조선 전람회에 입선하 다 단 의(6 )

만종은 인간과 자연이 엮어 가는 경건한 조화 을lsquo rsquo

나타낸다

34) 정답[ ] ①

해설 근이 속에서도 창작활동을 추 않고[ ]

하는 닭은 은 세상과 타협할 르는

근이 세상의 이해를 하 위한 가장 떳떳한 단

이 때 이다

35) 정답[ ] ⑤

해설 전 은 서 자의 주 적인 평이 리는 것이[ ]

만 위 제 은 인 이 살았던 대 사회적 경

을 통해 객 적인 인 의 을 제 하고 있다

36) 정답[ ] ⑤

해설 전 은 인 사건 경 비평이라는[ ] lsquo rsquo⑤

성 이 어져 있다

37) 정답[ ] ①

해설 이 은 동양인과 서양인의 사고 에 차이가[ ]

있다는 것을 대조를 통해 설 하고 있다 또 쓴이

의 제자가 축 경 를 러 가서 경험한 일화를

통해 동양인이 서양인에 비해 주 상황에 더 많은

주의를 인다는 주장을 뒷 침하고 있다

38) 정답[ ] ④

39) 정답[ ] ②

40) 정답[ ] ②

41) 정답[ ] ④

42) 정답[ ] ③

43) 정답[ ] ④

44) 정답 도서 의 휴 일 도서 의 이 간 도서의[ ]

해설 도서 장은 임의 정한 휴 일과 도서 이[ ]

간 도서의 상 등을 게 할 의 가 있다

년 학 간고사 대비2013 2 현대고 대비

ECN-0102-2013-001-000076193

45) 정답[ ] ①

해설 제 조의 정 휴 일 의 휴 일의 사전 게[ ] 3

는 도서 장의 의 조항에 속한다

46) 정답[ ] ①

해설 개인 정 호 의 를 제 하 했 만 항[ ]

나눠서 제 하 않고 대 나열하고 있다

47) 정답[ ] ②

해설 제 조의 내 을 회사는 다른 회사 협[ ] 7 lsquo

계약을 통해 서비 를 제공하는 경 회 의 아이디

등 개인 정 를 해당 회사에 전송할 있다는 내rsquo

이 있으 의 제점을 제 할 있다②

48) 정답[ ] ④

해설 는 도서 장의 의 에 해당하고 나 는 도[ ] ④

서 장의 리에 해당한다

49) 정답[ ] ③

50) 정답 은 음독으 적었고 은 훈독으 적었[ ] (1)

다 과 동일한 표 리 적은 것은 이고 (2) ce

과 동일한 표 리 적은 것은 이다ab

51) 정답[ ] ③

52) 정답[ ] ①②

53) 정답[ ] ③

54) 정답[ ] ③

55) 정답[ ] ①

56) 정답 른 죠코 어린 노 하니라[ ] A B

57) 정답 세 어에서는 활 형이 칙적으[ ] lsquo rsquoㄹㅇ

나타났 만 개화 어에서는 활 형이 쓰 다 lsquo rsquo ㄹㄴ

58) 정답 호 가 흔[ ] (1) (2)

59) 정답[ ] ④

60) 정답[ ] ③

Page 16: 현대고대비 국어 - chamsoriedu.com 「콘텐츠산업진흥 법」외 에도 저작권 의하여 ... 다른주체에게어떤동작을하도록만드는것을나타내는

년 학 간고사 대비2013 2 현대고 대비

ECN-0102-2013-001-000076193

없고 산 에 리 만 본 ( ) 廣德山

신 상 사 살 는 만

우고 늘 월 신 과 늘 ( )日月聖神

산 신 들 다 니 그 재( ) 名山神靈

주 민 누가 당 겠는가 낮 공

zb24) 다 에 해당하는 내 으 적절하( ) 않은 것은

① 강 티 통 당시 능 다

② 상계 지상계 경 는 원 계 드러

③ 실에 어 없는 실 가 타 는

④ 뛰어 재주 어 가진 고

등 다

⑤ 가 직 개 여 평가 내리는

편집 평 타 다lsquo rsquo

가 본격 가 동 것 지( )

다 단 상 에2003 lsquo rsquo

들어가 드럼 연주 다 취미 생 달리

들었다는 보 우 가 들ldquo

어 틱 린 도 다 고 말 다rdquo

경 는 가 망 없( ) lsquo

티 원 고 답 다 신과 같 시각rsquo

는 습 상상 만 도 감동

다 시각 연주 동시에

열 상 는

티 원 그런 열 경 럽다는 것 다

다 역시 엄청 다 본( )

에 복 들

고쳐 가고 다 신 에 얼

마 지는 고 리가 는 지도 생님

가 훈 고 많 고쳐 다

고 말 다

그러 직도 에 지 는 다 그는

체격 지 못 게 가 큰 만

체 운동 훈 과 께 체 늘 동 50

는 게 고 말 다

에게는 꿈 다 통 누 가( )

주겠다는 것 그 꿈 다 신 극복 는

과 에 큰 경험 들도 느 게

주고 싶다는 것 다

마 슬 마다( ) ldquo 통

낼 었 것 럼 고통 는 사 들

고 겠다 고rdquo

말 다 달 루 첫 낸 lsquo rsquo

첫 드 심 집에 는 리듬 드 2

루 에 도 보고 싶다 집 에는 직(RampB) 3 4

사 곡 도 보 고 싶다고 포 다middot

zb25) 에서 가장 유사한 의 를 닌 어를lt gt

찾아 쓰

lt gt

나는 이제 너에게도 픔을 주겠다

사랑 다 소 한 픔을 주겠다

겨 거리에서 개 놓고

살아 추위 떨고 있는 할 니에게

값을 으 서 뻐하던 너를 위하여

나는 픔의 평등한 얼 을 여 주겠다

내가 어둠 속에서 너를 를 때

단 한 도 평등하게 어주 않은

가마니에 덮인 동사자가

다 얼어 죽을 때

가마니 한 장조차 덮어주 않은

한 너의 사랑을 위해

흘릴 르는 너의 눈 을 위해

나는 너에게 이제 너에게도 다림을 주겠다

지 울 포동 여고 생들17

틈 없 가득 체 에 맑 울

다 죽 듣 생들 사 에

연 는 탄 다 객들 도 는lsquo rsquo

가 보 주 공 맹 가 운 는

단 그룹사운드 루 보컬 맡고 는lsquo rsquo

시각 지 었다17 1

근 다만과 가 거lsquo rsquo lsquo

꿈 고 퇴 내가 다rsquo

간 간에 지 지 연 생들 짧lsquo rsquo lsquo rsquo

가 운 듯 리에 어

연 다 내 사 고 퇴lsquo rsquo

과 루 들 결 다시 돌lsquo rsquo

들 고 사 들 에 당당

것 니다 내 태어

볼 없었 크고 열여

년 학 간고사 대비2013 2 현대고 대비

ECN-0102-2013-001-000076193

에도 고 시 얻지 못 다

감지 없는 시각 상태 다

신 지에 고 상 원망 도

단다 어느 가 에 시각 에 ldquo

어 그런 듣고 다 보니 내가 게 lsquo

살 는지 도 눈 고 싶rsquo lsquohelliphellip

보 는 생각만 들 고 그 가 들에게rsquo

도 내고 들도 고 많 었죠 들 rdquo

었 지 새 는 에 쑥 러운 색

어났다

생에 것 단연 었다lsquo rsquo

공연에 거 꿈lsquo rsquo

는 다 특 가사 갑게 는 운 lsquo

벽 에 당당 마주 어 언 가 그 벽

고 늘 어 거운 상도

없죠 내 삶 에 웃 그 께

는 다고 다rsquo

들었 그냥 런 도 고만 여ldquo lsquo rsquo

겼죠 그런 꾸 가사 미 새 다 보

니 통 는 가사 는 생각 들 고 (

가 게는 시각 는 생각 들고 들) ( )

마다 듣고 큰 얻었어 rdquo

에 진지 게 가에 미 가

zb26) 의 에 들어갈 말 적절한 것은lt gt ~

lt gt

난 난 꿈이 있었죠

고 찢겨 남 하여도

내 가 히 과 같이 간 했던 꿈

혹 때 누 가가 뜻 를 비 음

내 등 뒤에 흘릴 때도

난 참아야 했죠 참을 있었죠

날을 위해

늘 걱정하듯 말하죠

헛된 꿈은 독이라고

세상은 끝이 정해 책처럼

이 돌이킬 없는

현 이라고 helliphellip

래 난 난 꿈이 있어

꿈을 믿어

나를 켜

저 차갑게 서 있는 이란 앞에

당당히 마주칠 있어

출처 가 거위의 꿈 작사 이적 작곡 동률- lsquo rsquo ( )

① ② ③ ④ ⑤

가 떴다는 들 만 지만( ) lsquo rsquo

늘 겸 다 에 주 연 우승 지 간에도 3

단 생님께 만 지 고 고 만ldquo rdquo

큼 늘 겸 신 계 가

고 다

에게는 꿈 다 통 누 가

주겠다는 것 그 꿈 다 신 극복 는 과

에 큰 경험 들도 느 게 주

고 싶다는 것 다

슬 마다 통 낼ldquo

었 것 럼 고통 는 사 들

고 겠다 고rdquo

말 다 달 루 첫 낸lsquo rsquo

첫 드 심 집에 는 리듬 2

루 에 도 보고 싶다(RampB) 집 에는 직34

사 곡 도 보 고 싶다고 포 다

미 는( ) (26) 어 헤헤헤 웃다가 어ldquo rdquo

허허허 웃었다ldquo rdquo ldquo rdquo 같 도 고

상 다 는 같 도 다( ) 壯丁 킹 들lsquo

다 는 역도 보 그 다 지만 그는rsquo

뷰에 지 다 운동만 지 ldquo

것 지 간에 여러 사 도 역rdquo helliphellip

었다 그런 엇 그 마 움직 는지 보 쯤

지 담 사 다 훈 없어 그는 티

지 림 었다 태 다 갔다 는 습

마 집 럼 편 게 보 다

주말에는 주 엇 보내

주말에도 별 주 에 청ldquo

고 에 가고 도 쳐

에 듣고 보 에 갈 가 별 없

어 산 시 게 고 들어 2002

거 매 여 지냅니다 시 과 지훈 rdquo

다 근 간 과 진실 그리고 싶어( )

가 다 근에게 그것 진리 다 거 다 없

거 고 다 없 는 것 진리

다 근 진리는 후 쪽 었다 신산( )辛酸 삶

었 질곡( )桎梏 역사 에 지냈 가

눈에 든 것 료 단 료 게 보

것 었다 그것 그 에 겨우겨우

슬 슬 생 어가는 간들 었다

리 과 단 리 고리에 검 마

없 거리 돌

상 것 없는 등 근에게 상

과 진실 엄 ( )儼存 다는 사실 리는 가

실 고 가 과 역경 에 도 근 내 포

없었 후 보루( )堡壘 다 도 365

도 간 근 여

시 것 다

년 학 간고사 대비2013 2 현대고 대비

ECN-0102-2013-001-000076193

다 공주 그림 가 근 경- ( ) ldquo rdquo(

2009)

zb27) 작가의 주 적인 각이 드러난 것은~

① ② ③ ④ ⑤

가 신 지에 고 상 원망( )

도 단다 어느 가 에 시각 에 ldquo

어 그런 듣고 다 보니 내가 lsquo

게 살 는지 도 눈 고 싶rsquo lsquohelliphellip

보 는 생각만 들 고 그 가 들에게rsquo

도 내고 들도 고 많 었죠 들었rdquo

지 새 는 에 쑥쓰러운 색

어났다 략 [ ]

경 는 가 망 없 티lsquo

원 고 답 다 신과 같 시각rsquo

는 습 상상 만 도 감동

다 시각 연주 동시에

열 상 는 티

원 그런 열 경 럽다는 것 다 략 [ ]

슬 마다 통 낼ldquo

었 것 럼 고통 는 사 들

고 겠다 고rdquo

말 다 달 루 첫 낸 lsquo rsquo

첫 드 심 집에 는 리듬 2

루 에 도 보고 싶다 집 에는 직(RampB) 3 4

사 곡 도 보 고 싶다고 포 다

식 누 가-

고 싶어

다 역도 미 담 고 사( )

질 주말에는 주 엇 보내[ 1]

답 주말에도 별 주 에[ ] ldquo

청 고 에 가고 도 쳐

에 듣고 보 에 갈 가 별

없어 rdquo

질 계 고 슬슬 도 는 것 닙니[ 2]

답 다 들 눈 에 보 고 뿐 보[ ] ldquo

다 열심 고 어 상에 도 들지만 상

지키는 것 들다고 에 도달

그것 지키 훨 많 rdquo

질 들 살 고 리 는[ 3]

거운 들 체 리느 는다

답 가 고 게 체 어[ ] ldquo ( ) 級

느 도 계가 니 살 는 것도 고역 지만

살 우는 것 들어 는 살

체 리 고 어도 어도 실 갔다

쑥 어 rdquo

질 거리에 슷 연 여 들[ 4]

보는 간 상 지

답 상 다 체 게 리지 못[ ] ldquo

거 주변에 는 그 거 누 보지

못 고 뻐지고 싶 에 체 리는 에

타 워 지만 는 어울 는 것보다 는

시간 운동만 는 건 니에 사복 lsquo rsquo

고 사복 는 말에 들 웃지만 늘 운동복

고 지내니 사러 갈 도 어 rdquo

질 역도가 말 단 식 운동 니[ 5]

답 가 내는 만 클 업 보[ ] ldquo

그러니 만 쓰는 식 운동 니다

만 다고 거운 것 들 는 건 니거든 연

도 고 가지 동 에 도 여러 가지

복 들

보식 역도 여 미-

zb28) 가 에 대한 설 으( ) 않은 것은

① 시각 우 지 시 에 지

고 망 가는 태도 달 고 다

② 언어 과 언어 복 사 여

담 내 생각 게 는 가

③ 직 감 그 마 것

럼 생생 게 느껴지는 과 주고 간 내

없 리 어 억 게 다

④ 담 내 식 리 여 담 삶 습

과 가 시 여 독 에게 감동과 훈 다

⑤ 직 진 담 직 누

지 못 는 독 에게 생생 상 달 주고

담 욱 게 다

zb29) 나 의 각 의 의도를 설 한 것으 적절하( ) 않

년 학 간고사 대비2013 2 현대고 대비

ECN-0102-2013-001-000076193

은 것은

① 질 담 상 보여 주 것 다1

② 질 담 과 그에 삶 태도 보여2

주 것 다

③ 질 역도 겪는 어 움에 역도3

과 것 다

④ 질 같 연 여 갖는 고민 는지 말4

주 는 것 다

⑤ 질 역도가 과 고 운동 는 것5

담 가 말 주 는 것 다

가 만진 것 다( ) 3

감 달 다고 다 억 에( ) 音感

지워 지만 당시 청 탁 리도

다고 다 드럼 웠다 4

에 갈 마다 드럼 는 리가 신 게 들

다고 다 눈 볼 가 없 니 엔ldquo

는 는 님 틱 에 여 주

다 드럼과 연 맺 과 들 주었다rdquo

식 누 가-

고 싶어

역( ) 도가 말 단 식 운동 니

가 내는 만 클 업에 보ldquo

그러니 만 쓰는 식 운동 니다 만

다고 거운 것 들 는 건 니거든 연

도 고 가지 동 에 도 여러 가지 복

들 시 는 상 상

드는 상 에 맞춰 실 에 는 여러

펼쳐집니다rdquo

략( )

늘 에 는 어 만 것 같

가 에 사 고 사 사ldquo

겠어 든 에 가 경 만 고

울 는 사 겠어 rdquo

보식 역도 여 미-

다 가 운 는 어 어( ) ldquo rdquohelliphellip

월 새벽 시 태 없 거웠고1965 5 6 1

는 없 그 병원에 퇴원 집

가는 마지막 마 고 마 내 거 다

가 죽 간신 에 실 다 사는 어느5 lsquo

가 죽 는 말 가 식 다 신rsquo

상에 각 시키는( )刻印 에 실

어느 가는 후 민 가가 근 었다lsquo rsquo

는 간 과 진실 그 다는( ) ldquo

에 단 평 견 가지고 다 내

가 그리는 간상 단 고 다 지 다 는 그들

가 에 는 평 지 니 그리고 어린

들 미지 겨 그린다rdquo

마 근 간 과 진실 그리고 싶어( )

가 다 근에게 그것 진리 다 거 다 없

거 고 다 없 는 것 진리

다 근 진리는 후 쪽 었다 신산(辛酸 삶)

었 질곡(桎梏 역사 에 지냈)

가 눈에 든 것 료 단 료 게

보 것 었다 그것 그 에 겨우겨우

슬 슬 생 어가는 간들 었

다 리 과 단 리 고리에 검

마 없 거리 돌

상 것 없는 등 근에게 상에

과 진실 엄 다는 사실 리는 가( )儼存

실 고 가 과 역경 에 도 근 내

포 없었 후 보루(堡壘 다 도)

도 간 근365

여 시 것 다

월 강원도 림리에( ) 1914 2 21

삼 독 태어났다 어 근 복

그것 그리 가지 못 다 근 곱 살

지는 산 산업에 실 고 답마 에 내

갔다 근 그림 럼 쫓 다니 가 시 것

다 상 진 것도 가 었다

러 가 에도 고 근 가 꿈꾸었다 근

가 꿈꾸게 것 보통 업

원색도1926 만lsquo rsquo 었다

공주 그림 가 근 경-

zb30) 에 대한 설 가장 른 것은~

① 역도가 과 운동 도 질

② 리는 는 다 lsquo rsquo

③ 들었지만 그럭 럭 는 다 lsquo rsquo

④ 가 게 보 시 말 다

⑤ 보 병 는 지 상 lsquo rsquo

는 말 다

년 학 간고사 대비2013 2 현대고 대비

ECN-0102-2013-001-000076193

시간 많지 다 청량리 생 병원

마지막 상 경 릿 게 들어 다 그 는 십

만 큰 가 상 말 다

지 못 들 마 갈 고 돗

도시민들 싹 싹 탔다 가 시

월에 병원에 원 가 폐 진 몸도4 ( )疲弊

갈 미 지 못 고 었다 가는 얼마( ) 解渴

지 생 에 생각 가

마감 는 신 평생 십 만에

가 과 많 닮 다고 생각 지는

가 운 는 어 어ldquo rdquo 1965helliphellip

월 새벽 시 태 없 거웠고 는5 6 1

없 그 병원에 퇴원 집 가

는 마지막 마 고 마 내 거 다 가

죽 간신 에 실 다 사는 어느 가5 lsquo

죽 는 말 가 식 다 신rsquo

상에 각 시키는 에 실 어느( ) lsquo刻印

가는 후 민 가가 근 었다rsquo

ldquo 는 간 과 진실 그 다는 에

단 평 견 가지고 다 내가 그

리는 간상 단 고 다 지 다 는 그들 가

에 는 평 지 니 그리고 어린 들

미지 겨 그린다rdquo

근 간 과 진실 그리고 싶어 가

다 근에게 그것 진리 다 거 다 없 거

고 다 없 는 것 진리다

근 진리는 후 쪽 었다 신산 삶 ( )辛酸

었 질곡 역사 에 지냈 가 눈에( )桎梏

든 것 료 단 료 게 보 것

었다 그것 그 에 겨우겨우 슬

슬 생 어가는 간들 었다 리

과 단 리 고리에 검 마

없 거리 돌 상

것 없는 등 근에게 상에 과 진실

엄 다는 사실 리는 가 실( )儼存

고 가 과 역경 에 도 근 내 포 없었

후 보루 다 도 도( ) 365堡壘

간 근 여 시 것

간에 지닌 가 근 1914 2

월 강원도 림리에 삼 독21

태어났다 어 근 복 그것 그리

가지 못 다 근 곱 살 지는 산

사업에 실 고 답마 에 내 갔다 근

그림 럼 쫓 다니 가 시 것 다 상

진 것도 가 었다 러 가 에도

고 근 가 꿈꾸었다 근 가 꿈꾸게

것 보통 업 원색1926

도 만 었다lsquo rsquo

그림 가 근 경 공주- ldquo rdquo ( 2009)

zb31) 다음 이 같은 의 성 소에 해당하 않은

것은

사건 평① ② ③

④ 주 ⑤ 경

가 운 는 어 어ldquo rdquo 1965helliphellip

월 새벽 시 태 없 거웠고 는5 6 1

없 그 병원에 퇴원 집 가

는 마지막 마 고 마 내 거 다 가

죽 간신 에 실 다 사는 어느 가5 lsquo

죽 는 말 가 식 다 신rsquo

상에 각 시키는 에 실 어느( ) lsquo刻印

가는 후 민 가가 근 었다rsquo

는 간 과 진실 그 다는 에ldquo

단 평 견 가지고 다 내가 그

리는 간상 단 고 다 지 다 는 그들 가

에 는 평 지 니 그리고 어린 들

미지 겨 그린다rdquo

근 간 과 진실 그리고 싶어 가

다 근에게 그것 진리 다 거 다 없 거

고 다 없 는 것 진리다

근 진리는 후 쪽 었다 신산 삶 ( )辛酸

었 질곡 역사 에 지냈 가 눈에( )桎梏

든 것 료 단 료 게 보 것

었다 그것 그 에 겨우겨우 슬

슬 생 어가는 간들 었다 리

과 단 리 고리에 검 마

없 거리 돌 상

것 없는 등 근에게 상에 과 진실

엄 다는 사실 리는 가 실( )儼存

고 가 과 역경 에 도 근 내 포 없었

후 보루 다 도 도( ) 365堡壘

간 근 여 시 것

간에 지닌 가 근 1914 2

월 강원도 림리에 삼 독21

태어났다 어 근 복 그것 그리

가지 못 다 근 곱 살 지는 산

사업에 실 고 답마 에 내 갔다 근

그림 럼 쫓 다니 가 시 것 다 상

진 것도 가 었다 러 가 에도

고 근 가 꿈꾸었다 근 가 꿈꾸게

것 보통 업 원색1926

도 만 었다lsquo rsquo

공주 그림 가 근 경- ldquo rdquo ( 2009)

년 학 간고사 대비2013 2 현대고 대비

ECN-0102-2013-001-000076193

zb32) 위 을 작성하는 과정에서 되어 활 된 자

어 것은

신 사 료① 연보②

고③ ④ 들과 담

⑤ 에 평

는 간 과 진실 그 다는 에ldquo

단 평 견 가지고 다 내가 그

리는 간상 단 고 다 지 다 는 그들 가

에 는 평 지 니 그리고 어린 들

미지 겨 그린다rdquo

근 간 과 진실 그리고 싶어 가

다 근에게 그것 진리 다 거 다 없 거

고 다 없 는 것 진리다

근 진리는 후 쪽 었다 신산 삶 ( )辛酸

었 질곡 역사 에 지냈 가( )桎梏

눈에 든 것 료 단 료 게 보

것 었다 그것 그 에 겨우겨우

슬 슬 생 어가는 간들 었다

리 과 단 리 고리에 검 마

없 거리 돌 상

것 없는 등 근에게 상에 과

진실 엄 다는 사실 리는 가 실( )儼存

고 가 과 역경 에 도 근 내 포

없었 후 보루 다 도 도( ) 365堡壘

간 근 여 시

것 다

간에 지닌 가 근 1914 2

월 강원도 림리에 삼 독21

태어났다 어 근 복 그것 그리

가지 못 다 근 곱 살 지는 산

사업에 실 고 답마 에 내 갔다 근

그림 럼 쫓 다니 가 시 것 다 상

진 것도 가 었다 러 가 에도

고 근 가 꿈꾸었다 근 가 꿈꾸게

것 보통 업 원색1926

도 만 었다lsquo rsquo

질 루 마 가 도 린다 경건

움 느껴지는 경 다 훗 근 그림에

과 는 거 것( )裸木

만 간과 연 엮어 가는 경건 움lsquo rsquo

니었

같 가가 고 싶었 근에게 그 꿈에 다

가가는 지 다 다 가 지망생들 규 미

상 에 진 고

에 지만 근 다 다 근

미 에 운 것 보통 시 미 시간

다 그런 그에게 없는 연습 가가

통 다 가 귀 시 지 도

얻는 뛸 듯 뻤지만 마 도 가 에

듯 는 었 에 어린 근 주 에

에 그림 그리고 지우고 복( )粉板

시간 가는 게 루 보냈다

근 그 갈 가가 것 열여( )渴求

었 다가 미1932 lsquo rsquo ( lsquo

미 에 다 다는 고 마rsquo) lsquo rsquo

가 근 집 고도 지는 시골 경

그린 그림 다 후 근 에 1943 22

지 미 에 그림 고

에 걸쳐 다 미 근 가

동 는 었다

공주 그림 가 근 경- ldquo rdquo ( 2009)

zb33) 위 의 내 과 일치하는 것은

가 근 가 꿈 포 다①

근 당 가들과 께 에 다②

살 근 가 걷20③

게 었다

④ 만 통 근 역경 겨내는lsquo rsquo

느 다

⑤ 근 간 과 진실 그리 에 그 에

드러 는 간상 단 다

계 시 주 근 건강

걸었다 신 과 간에 상 다 건강

신 는 눈에도 다 근 쪽 눈 뿌 게

보 지 과에 다 다 시 지지 고 결

내 었다 시 지만 마 막막

다 늦어 결 근 쪽 눈 고 말 다

쪽 눈 근에게는 쪽 눈 었고

계 었다 그 근 는 여 그lsquo rsquo

다 근 에 같 그림 그 었다1950

시 그림 는 여 쪽lsquo rsquo

고 어 마주 고 는 그림1963

여 과 동 다 마 복

그린 듯 눈 내리 새 게 다 지

사 다 근 게 복 것

복 상과 타 는 근 상

가 떳떳 단 었고 근 그리고

간 과 진실 에 다가가 가 근다

운 었다 근 신에게 당당 지 그리고

그 다 근 그림에 단 복 보다

년 학 간고사 대비2013 2 현대고 대비

ECN-0102-2013-001-000076193

태 도 그리고 극 보다 과

얻 여 었다 과 통

근 그리고 는 재 고 에 질

만들고 특 것 다

공주 그림 가 근 경- ldquo rdquo( 2009)

zb34) 의 이유에 대해 추 한 것으 적절하 않은 것

상과 타 시도①

보다 과 얻②

근 신에게 당당 지③

④ 간 과 진실 에 다가

⑤ 태 도 얻

근 가가 었지만 그 다니 가

럼 어지지 다 복과 쟁 거쳐 시

는 가 근에게 생계 사 에

운 사 다 에 키에 건( ) 178cm死鬪

체 근 에 동 역 업( )荷役

가 생계 다 쟁

에는 동에 운 상우 주 미

죄 사 에 그림 그리는 시 다 그곳에

에 동 역 업 것에

결 것 럼 보 다 지만 그런 것만도

니었다 그림 그리는 고는 지만 매 근

는 극 간 과 별 없는 경 리 그림

벽에 그리는 것 었다 우도 리 없었다 근

트 는 우 그림 그 다 생

계 그림 단 것 다

후 근 지 신 계 리에 미

엑 리 겼다 근 곳에

건 사 크 에 미 들 ( )

상 상 그 다 근 갖 다 겪

냈다 그리고 결 그 돈

신동에 어 사리 집 마 다 마 ㄷ

루 심 쪽에는 과 엌 쪽에는 건

었다 건 주고 근 가 에

여 살 다 심 에는 지 집어

쓰고 지만 곳 근 가 에게 러웠

보 리 다 근 과 마루 업실 삼 그림

그 다 신동 마루는 근 그림에 등 는 lsquo rsquo

같 상들 지 다 시 고

에 들 폐허가

가 업실 었다

공주 그림 가 근 경- ldquo rdquo( 2009)

zb35) 위 에 대한 설 으 적절한 것은

업 시 여 훈과 감동 다①

에 주 평 드러 다②

사 사 등 식 과 ③

④ 다 근거 시 여 삶에

⑤ 살 시 사 경 께 여

습 시 다

가 시간 많지 다 청량리 생 병원( )

마지막 상 경 릿 게 들어 다 그 는

십 만 큰 가 상 말 다

지 못 들 마 갈 고 돗

도시민들 싹 싹 탔다 가 시

월에 병원에 원4 가 폐( )疲弊

진 몸도 갈 미 지 못 고 었다( )解渴 가는

얼마 지 생 에 생각

가 마감 는 신 평생 십 만에

가 과 많 닮 다고 생각 지는

가 운 는 어 어( ) ldquo rdquohelliphellip

월 새벽 시1965 5 6 1 태 없 거웠고

는 없 그 병원에 퇴원 집

가는 마지막 마 고 마 내 거 다

가 죽 간신 에 실 다 사는 어느5 lsquo

가 죽 는 말 가 식 다 신rsquo

상에 각 시키는 에 실( )刻印

어느 가는 후 민 가가 근 었다lsquo rsquo

다 는 간 과 진실 그 다는( ) ldquo

에 단 평 견 가지고 다 내

가 그리는 간상 단 고 다 지 다 는 가

에 는 평 지 니 그리고 어린 들

미지 겨 그린다rdquo

근 간 과 진실 그리고 싶어( )

가 다 근에게 그것 진리 다 거 다 없

년 학 간고사 대비2013 2 현대고 대비

ECN-0102-2013-001-000076193

거 고 다 없 는 것 진리

다 근 진리는 후 쪽 었다 신산( )辛酸 삶

었 질곡 역사 에 지냈( )桎梏

가 눈에 든 것 료 단 료 게 보

것 었다 그것 그 에 겨우겨우

슬 슬 생 어가는 간들 었다

리 과 단 리 고리에 검

마 없 거리 돌

상 것 없는 등 근에게 상에

과 진실 엄 다는 사실 리는 가 실( )儼存

고 가 과 역경 에 도 근 내 포

없었 후 보루 다( ) 堡壘 도 365

도 간 근 여

시 것 다

마 같 가가 고 싶었 근에게 그 꿈( )

에 다가가는 지 다 다 가 지망생들

규 미 상 에 진 고

에 지만 근 다 다 근

미 에 운 것 보통 시 미 시간

다 그런 그에게 없는 연습 가가

통 다 가 귀 시 지 도

얻는 뛸 듯 뻤지만 마 도 (

는 었 에 어린 근 주 에)

에 그림 그리고 지우고( )粉板

복 시간 가는 게 루 보냈다

zb36) 전 의 성 소가 아닌 것을 고르

① 평 ② 사건 ③ 경

④ ⑤ 훈

늘 지 상에 살고 는 사 들 억 도가10

고 그리 지 통 고 는 사 들( )知的

그보다 훨 많 억 도는 고 지 20

통 다 그런 지 고 2500

그리 간 보는 과 사 에

매우 달 뿐만 니 과 에 도 극

루고 었다 미 운 그런 들

살고 는 동 과 사 들 사고 식에

큰 가 다는 다

고 그리 들 우주 개별 고 독립

사 들 생각 지만 고 들 우

주 연 질 간주 다 같( ) 看做

각 도 들에게는 연 질

었지만 그리 들에게는 미 들 결 었

다 고 과 그리 들 사 같

는 동 과 사 에 도 견 다

지심리 미 마 드 겐트 는

살 들에 에 지 다

연 동 과 상 다 과 같 실험

다 크 만든 미드 도 보

여 주고 그 상 닥 고 주었다lsquo (Dax)rsquo

실 닥 는 재 지 는 것 실험 가lsquo rsquo

만들어 낸 다 그런 다 개 다 체 보

여 주었는 는 미드 지만 틱

만들었고 다 는 재료는 크 지만

달 다 그러고 어 것 닥 지 사 들에게 고 lsquo rsquo

게 니 들 주 같 고 는

체 택 고 동 들 같 재료 만들어진 체

택 다 러 는 심지어 살짜리

들에게 도 타났다 것 곧 과 동

다 상 보고 다는 것 미 다

개별 사 보고 고 동 연 질 보

고 는 것 다

동 들 주변 상 에 맞 어 동 고

에 다 사 들 태도 동에 보다 많

주 울 다 동 가 미시간 에

에 경험 다 그는 미식

경 보러 가게 었는 경 체는 매우 재미 었

주변 들 동에 질 다 그 는

들 계 어 상태 경 다

어 들 에 에 그 시 가 계 가

진 것 다 상 살펴 는 말 들 lsquo rsquo

에 그는 에 시 어 도 뒷사

생각 곧 다시 곤 것 다 그런 그에게 뒷

사 고 지 는 들 동 럼

어 웠다

생각 지도 리 드 니 벳-

zb37) 다음 위 의 내 전개 으 만 인lt gt

것은

lt gt

대조의 통해 대상이 닌 특성을 설 하고 있다

일화를 제 하여 자 의 주장을 뒷 침하고 있다

유추의 을 사 하여 독자의 의해를 돕고 있다

대상이 형성되는 과정을 간적 서에 따라 서 하고 있

① ②

③ ④

년 학 간고사 대비2013 2 현대고 대비

ECN-0102-2013-001-000076193

가 우리가 말 고 쓰는 든 단어가 사 에 는( )

것 니다 사 격에 가 는 지만

어 사 과 같 특별 는 사 니lsquo rsquo

단어 격 보 단어가 사 에

등재 어 다 리 리 사 는 단어 도 그

것 시 사 는 어 고 사 에

격 보 것 니다

러 얼 은 사전에 를 있는가 이에 대한 답lsquo rsquo

은 얼 이 유행어인가 아닌가에 따라 갈라 다 이 단어lsquo rsquo

는 년 어 자 에 랐고 쓰이고 있으2002 lsquo rsquo

유행어라고 하 에는 생 이 다 런데 계속

을 유 하 서 사전에 등재될 자격을 획득할 것인가 이

에 대한 답을 내리 는 히 어 다

여 서 가 를 고 해 볼 있다 첫 는 이 단어

를 써야 할 필 가 속적으 있는가 하는 점이다

상주의 열풍에 휩 인 사회 위 에 편 해서 퍼 말

이 얼 인데 과연 런 위 가 속될 것인가 이에lsquo rsquo

대해 필자의 생각은 정적이다 사회 위 가 뀌

런 말을 쓸 일이 없어 것이다

다음은 단어의 성이다 단어의 성이 사회적으 거

감이 없으 계속 사 될 가능성이 높다 런 에서

얼 은 좋은 조건이 아니다 익히 알 졌듯이 이lsquo rsquo

말은 얼 과 청소년층에서 속어 사 하는 이 결합lsquo rsquo lsquo rsquo

된 말이다 얼 에서 얼 을 리하는 조어 도 lsquo rsquo lsquo -rsquo

어에서는 매 낯선 이다 이것만으 도 거 감을 갖

는 사람들이 있다 더 나 속어 결합한 말이다 얼 lsquo rsquo

이 널리 퍼졌다 해도 은 여전히 청소년층의 속어lsquo rsquo

남아 있다 속어는 자연 럽게 아 자리에서나 쓰 에는

담 러 말이다 러한 담을 하고 사

역을 넓혀 가는 속어도 없 는 않다 특히 얼 은 lsquo rsquo

에도 종종 등장한다 만큼 거 감이 많이 희석되었다

고 할 있다 러나 일상의 자연 러 대화에서도 거

리낌 없이 등장하는가 게 는 되 않았다고 생

각한다

얼 이 유사어인 쌈 등을 만들어 내고lsquo rsquo lsquo rsquo

있으니 살아남을 있을 것이라고 는 견해도 있을 것

이다 러나 간이 나 서 유사어를 포함하여 든

말이 사라 사 는 많다 유사어가 많다는 것이 생 을

유 할 있는 절대적인 조건은 아니다

나 언젠가 터 사람들은 어느 단에서 얼 이 가장( )

쁜 사람을 가리켜 얼 이라고 르고 있다 이 얼lsquo rsquo lsquo rsquo

이라는 단어가 최근 어사전에 라 항간에 논란이 일고

있다 아닌 게 아니라 얼 은 유행어처럼 인다 생 lsquo rsquo

도 리 래되 않은 것 같고 언제 사라 도 알

없다 게다가 젊은이들 사이에서 주 쓰일 뿐이다 이런

단어를 사전에 는다는 게 하 이 없어 이 도

한다

러나 속단은 이다 차근차근 따져 볼 일이다

선 얼 이 일 적 유행어인 아닌 주의 게 들여다lsquo rsquo

볼 필 가 있다 유행어란 유행에 따라 빠르게 유포되었

다가 단 간 내에 소 되는 단어나 를 가리킨다

얼 은 인터넷을 통해 속히 퍼 말이다 하 만 일lsquo rsquo

적인 유행어처럼 단 간 내에 사라 않았을 뿐 아니라

현재 도 잦은 빈도 사 되고 있고 앞으 도 상당

간 사 될 것으 측된다 한 언 재단의 뉴 검 lsquo rsquo

색 사이트에 따르 얼 은 년 에 처음 나타난lsquo rsquo 2001

이후 꾸 히 사 되고 있다

이 같은 사 빈도는 얼 이 일 적 유행어 는 현lsquo rsquo

저히 다르다는 것을 여 다 장 간의 생존 만으 도

얼 은 이 한 어의 어휘 에 를 자격을 얻었다lsquo rsquo

고 할 있다 더 이 이라는 비 적 정제된 매체에

높은 빈도 쓰이고 있 않은가 사 빈도 측 에서

필통이나 연필과 같은 단어 대등하거나 더 많이 쓰lsquo rsquo lsquo rsquo

다는 것은 결코 가 게 볼 일이 아니다

이제는 사전이 언어 현 을 빠르게 하는 게 덕인

대가 되었다 세계적으 유 한 의 사전들도 경쟁

적으 어를 고 있다

하 만 얼 은 젊은이들이나 쓰는 속어라고 흠을 잡을lsquo rsquo

도 르겠다 얼 이 주 젊은 층에서 많이 쓰 lsquo rsquo

는 속어임에 틀림없다 러나 어사전에 표 적이고 품

위 있는 말만 어야 한다고 생각한다 것은 커다란

해다 당장 아 어사전이나 펼쳐 라 속어는

설과 같은 비어나 죄자들이 쓰는 은어 어

마니 같은 소 의 사람만이 쓰는 말 도 라 있

않은가 사전은 말 치에 일정 빈도 이상 나타나는 말이

라 말이든 다 할 있다

zb38) 가 나 에 대한 다음의 설( ) ( ) 않은 것은

① 가 는 얼짱 사 에 등재 것에( ) ( ) lsquo rsquo

보 고 다

② 사 등재 가는 단어 격에( )

고 고 는 언 들 언어 사 도에 고 다 ( )

③ 가 얼짱 어지만 신 과 같 매( ) ( ) lsquo rsquo

체에 도 사 는 말 는 고 다

④ 가는 얼짱 어 보고 크게 가지 근( ) lsquo rsquo 3

거 들어 뒷 고 다

⑤ 는 얼짱 어 는 다 특 다는( ) lsquo rsquo

근거 에도 크게 가지 근거 가 들어 주 2

뒷 고 다

가 늘 지 상에 살고 는 사 들 억( ) 10

도가 고 그리 지 통 고 는 사 들

그보다 훨 많 억 도는 고 지 20

통 다 그런 지 고 2500

년 학 간고사 대비2013 2 현대고 대비

ECN-0102-2013-001-000076193

그리 간 보는 과 사 에

매우 달 뿐만 니 과 에 도 극

루고 었다 미 운 그런 들

살고 는 동 과 사 들 사고 식에

큰 가 다는 다

고 그리 들 우주 개별 고 독립

사 들 생각 지만 고 들 우

주 연 질 간주 다 같 각

도 들에게는 연 질 었지

만 그리 들에게는 미 들 결 었다

고 과 그리 들 사 같 는

동 과 사 에 도 견 다

인 리학자인 츠 이마이 디드 겐트너는 두

살이 채 안 된 아이들에서 터 성인에 이르 다양한

연 대의 동양인과 서양인을 대상으 다음과 같은 험

을 했다 저 코르크 만든 피라 드 양의 도형을

여 주고 대상의 이름을 닥 라고 알 주었다lsquo (Dax)rsquo

제 닥 는 존재하 않는 것으 험자가 임의lsquo rsquo

만들어 낸 이름이다 런 다음 두 개의 다른 체를

여 주었는데 하나는 피라 드 양이 만 하얀 플라 틱

으 만들었고 다른 하나는 재 는 코르크 만 양이

달랐다 러고 나서 어떤 것이 닥 인 사람들에게 고 lsquo rsquo

르게 했더니 서양인들은 주 같은 양을 하고 있는

체를 선택했고 동양인들은 같은 재 만들어 체를

선택했다 이러한 차이는 성인은 어 두 살 리

아이들에게서도 나타났다 이것은 곧 서양인과 동양인은

서 다른 세상을 고 있다는 것을 의 한다 략 ( )

는 아주 단 하 서도 인상적인 험을 했다

험에는 동서양의 대학생들이 참여했다 는 험 참가자

들에게 컴퓨터 화 을 통해 속 장 을 담은 애니 이션

을 여 주었다 화 의 앙에는 초점의 역할을 하는 커

다란 고 한 마리가 있었고 주위에는 다른 생

들과 초 자갈 거품 등이 함 제 되었다 화 을

두 씩 후 참가자들은 자 이 것을 회상해 라는

를 았다

결과 서양인 대학생들과 동양인 대학생 두 앙

의 초점 역할을 했던 고 를 동일한 정도 언 했으

나 경 소 위 거품 초 다른 생 들 에 ( )

대해서는 동양인 대학생들이 서양인 대학생들 다 60

이상 더 많이 언 했다 뿐만 아니라 동양인 학생들은 서

양인 학생들에 비해 개 적인 고 다 전체적인 계

를 더 언 하는 경향을 다 략 또한 경의 일 ( )

를 화 킨 림을 제 하 을 때 동양인 대학생들은 대

경의 화를 알아챘 만 서양인 대학생들은 경

의 화를 거의 알아차리 했다 략 ( )

따라서 서양인들만을 대상으 연 한 화lsquo

편성 결 은 잘 된 것일 도 있다 각 과정과 인rsquo

과정의 어떤 이 화 편적이고 어떤 이

화에 따라 달라 는 는 앞으 많은 연 를 통하여 논의

되어야 한다

나 어떤 의 에서 리 두는 이 화적이다 리( )

안에는 다른 사람들과 더 친 한 계를 유 하 는 상호

의존성과 다른 사람들 터 독립적인 존재 살아가 는

독립성이 혼재한다 따라서 이 에서 어떤 특성이 더 강

하게 각되는 상황에 놓이느냐에 따라 서 다른 화적

특 을 일 있다 결 리 두는 어떤 경 에는

동양인처럼 행동하고 어떤 경 에는 서양인처럼 행동하는

것이다

zb39) 가 에 대한 다음의 설( ) 않은 것은

① 는 신 주 뒷 닥 실험과lsquo rsquo lsquo

니 실험 근거 시 다rsquo

② 동 들 상 간 공통 보다는 에 식

는 강 다

③ 들 주변 맥 에는 심 경 어 사건

과 사건 사 계에 상 민감 다

④ 는 동 과 틀린 지 고 는 것lsquo rsquo

니 다 고 다 lsquo rsquo

⑤ 가에 우리 사 들 개 시 가 원( )

집 경 말 고 는 것 개 보다는

에 고 는 것에 다

늘 지 상에 살고 는 사 들 억 도가10

고 그리 지 통 고 는 사 들( )知的

그보다 훨 많 억 도는 고 지 20

통 다 그런 지 고 2500

그리 간 보는 과 사 에

매우 달 뿐만 니 과 에 도 극

루고 었다 미 운 그런 들

살고 는 동 과 사 들 사고 식에

큰 가 다는 다

지심리 미 마 드 겐트 는 동

과 상 다 과 같 실험 다

크 만든 미드 도 보여 주고 그

상 닥 고 주었다 그런 다lsquo (Dax)rsquo

개 다 체 보여 주었는 는 미드

지만 틱 만들었고 다 는 재료는

크 지만 달 다 그러고 어 것 닥 lsquo

지 사 들에게 고 게 니 들 주 같rsquo

고 는 체 택 고 동 들 같

재료 만들어진 체 택 다 러 는

심지어 살짜리 들에게 도 타났다 것

곧 과 동 다 상 보고 다는

것 미 다 개별 사 보고 고 동

년 학 간고사 대비2013 2 현대고 대비

ECN-0102-2013-001-000076193

연 질 보고 는 것 다

동 들 주변 상 에 맞 어 동 고

에 다 사 들 태도 동에 보다

많 주 울 다 동 가 미시간

에 에 경험 다 그는 미

식 경 보러 가게 었는 경 체는 매우 재

미 었 주변 들 동에 질 다 그

는 들 계 어 상태 경

다 어 들 에 에 그 시 가 계

가 진 것 다 뒷사 고 지 는 들

동 럼 어 웠다

그는 경험에 어 얻어 동 들lsquo

각도 상 본다 는 가 우고rsquo

검 여 주 단 도 상 실험 실

시 다 그는 실험 가 들에게 컴퓨 통

담 니 보여 주었다

에는 역 는 커다 고 마리가 었

고 주 에는 다 생 들과 갈 거 등

께 시 었다 본 후 가 들

신 본 것 상 보 는 지시 다

그 결과 생들과 동 생

역 고 동 도 언

경 거 다 생 들에 ( )

는 동 생들 생들보다 60

상 많 언 다 뿐만 니 동 생들

생들에 개별 고 보다 체 계

언 는 경 보 다 경 변 시

킨 그림 시 동 생들 경

변 지만 생들 경 변

거 리지 못 다

지 지 들만 상 연 lsquo

보편 결 못 것 도 다 지각 과 과rsquo

지 과 어 보편 고 어

에 달 지는지는 많 연 통 여

어 다

리 드 니 벳 생각 지도 사- ldquo rdquo( 2004)

zb40) 위 에 대한 설 으 가장 적절한 것은

① 동 과 생 식 강 고 다

② 가지 실험 통 쓴 고 다

③ 닥 실험에 사 본질에 동 사

상에 주 다

④ 니 실험에 동 과 에 지

각 도에 가 다

⑤ 쓴 는 보편 연 에 드러 우월 에

에 근 고 다

가 동 들 주변 상 에 맞 어 동 고( )

에 다 사 들 태도 동에 보다 많

주 울 다 동 가 미시간 에

에 경험 다 그는 미식

경 보러 가게 었는 경 체는 매우 재미 었

주변 들 동에 질 다 그 는

들 계 어 상태 경 다

어 들 에 에 그 시 가 계 가

진 것 다 상 살펴lsquo 는 말 들rsquo

에 그는 에 시 어 도 뒷사

생각 곧 다시 곤 것 다 그런 그에게

뒷사 고 지 는 들 동 럼

어 웠다

그는 경험에 어 얻어( ) 동 들lsquo

각도 상 본다 는 가 우고rsquo

검 여 주 단 도 상 실험

실시 다 실험에는 동 생들 여 다

그는 실험 가 들에게 컴퓨 통

담 니 보여 주었다 에는

역 는 커다 고 마리가 었고 주 에는

다 생 들과 갈 거 등 께 시

었다 본 후 가 들 신 본 것

상 보 는 지시 다

다 그 결과 생들과 동 생( )

역 고 동 도 언

경 거 다 생 들 에 ( )

는 동 생들 생들보다 60

상 많 언 다 뿐만 니 동 생들

생들에 개별 고 보다 체 계

언 는 경 보 다 들어 동

생들 상 체 연못 럼 보 어ldquo 같rdquo

체 맥 언 시 었지만

생들 상 어 같 큰 고 가 쪽 움ldquo

직 어 같 역 고rdquo

언 시 다 경 변 시킨 그

림 시 동 생들 경 변

지만 생들 경 변 거

리지 못 다

년 학 간고사 대비2013 2 현대고 대비

ECN-0102-2013-001-000076193

게 볼 동 들 보다는 큰 그( )

림 보 에 사 과 체 맥 연결시 지각

는 경 고 체에 특 떼어 내

어 독립 보는 것 낯 어 다 에

들 사 에 고 주변 맥 에는 심 경

에 사건과 사건 사 계에 상

민감 편 다

마 지 지( ) 들만 상 연

보편 결 못 것 도 다lsquo rsquo 지각 과

과 지 과 어 보편 고 어

에 달 지는지는 많 연 통 여

어 다

리 드 니 벳 생각 지도 사- ldquo rdquo( 2004)

zb41) 의 하는 가~ 다른 것은

① ② ③

④ ⑤

얼마 그 에 동 사고 식과

사고 식 보여 주는 내 다

들 에 는 탕 고 같 게

어 겨 고 미 에 는 그 크 럼 큰 고

어리 주고 원 는 어 도 는

상 고 생각 다는 것 다 러

는 어떻게 생 것 고 과 그리 거슬

러 가 보 그 단 다

고 연 경 체 경 생 에

다 벼 사는 공동 업과 경험 많 연 역

에 고 들 연 웃과

게 지내 고 탁 연 들

들 지 연 럽게 들 다 민들

웃과 동 게 뿐만 니 는 집 과

게 다

동 시 는 생태 경 에 살 결과

들 다 사 들 사 상 에 주

울 게 었고 는 곧 체 상 과 간 사

계 시 는 낳게 었다 신 가

가 는 체에 는 원 는 동시

에 다 사 들 그 사 포 체 맥 에

다 들 간 사 연

계 체 계에 주 울 는 사고 체계

게 었다

그러 그리 연 경 그 었다 산

지 연결 는 지 건 그리고 역

에 다 런 들 업에 다 사 과

동 므 공동체에

다고 다 고 그리 들

들과는 달리 보 내 감 지 들과

지 크게 느 지 못 다 그

견 다 경우 주 쟁 통 결 는 갖

게 었다

신 사 간 계들 루어진 커다

트워크 에 게 당연 사 역시 연

계들 체 식 게 다 어 상

원 도 그 개체가 체 맥 과

계 에 고 다 게 체 맥 에 주

울 다 보 상 복 과 가변 식 게 고

상에 재 는 많 변 들 사 에 재 는 들도

게 다 들 주 태도 보

는 경우가 많다 쟁 결

통 결 보다는 통 결

는 보 다

그러 고 그리 들 개개 사 사 독

에 주 울 다 사 사 체에

어 그들 사 에 재 는 공통 규 주

고 다 상 원 에도 사

체 내 주 고 다 그들

체 여 탕 체

는 주 태도 시 고 특 사 어

주에 는지 여 그 주에 는 규

견 다 에 는 쟁 식 리

같 리 사고 체계가 달 게 었다

리 드 니 벳 생각 지도 사- ldquo rdquo( 2004)

zb42) 위 에서 사 된 설 과 가장 유사한 것은

① 크톱 컴퓨 는 본체 니 마우 루

어 다

② 곡과 시 리 는 지 과 사 루어 다는 공통

지니고 다

③ 경 고 것과는 달리

경 본 연 태 그 주변 경

④ 벽돌 능 에 사계 내내

습도가 지 다

⑤ 잰느 체 체 지닌 재 체가 없

는 재 눌 다

년 학 간고사 대비2013 2 현대고 대비

ECN-0102-2013-001-000076193

zb43) 는 립 앙 도서 이 정의 일 이다lt gt

도서 장과 이 자의 리 의 정의 연결이

적절하 않은 것은

lt gt

제 조 서 유8 ( )

도서 장은 다른 이 자의 안전을 위협하거나 도서 의①

서를 란하게 할 가 있는 자에 대하여는 도서 출입

을 제한할 있다

도서 장은 이 자가 제 조 각 호의 어느 하나의 행위를 하7②

을 때에는 이 을 하게 하거나 도서 출입을 제한할

있다

제 조자 의 대출9 ( )

도서 자 는 다음 각 호의 경 대출할 있다①

상호대차도서 간에 자 를 류하는 것을 말한다 등 다1 ( )

른 도서 과의 협 을 위하여 필 한 경

공 이 공 행 상 필 하는 경2

에 도서 장이 필 하다고 인정하는 경3

대출이 가능한 도서 자 의 위는 도서 장이 정하는②

에 따른다

제 조 상10 ( )

이 자가 도서 자 설을 더럽히거나 찢거나 뜨①

쓰게 하거나 잃어 린 경 에는 상하여야 한다

도서 장은 제 항에 따른 상 을 정하여 게 하여야1②

한다

제 조이 절차 등11 ( )

이 칙에서 정한 것 에 도서 자 설의 이 절차

이 제한 등에 필 한 사항은 도서 장이 정한다

출처 립 앙 도서- (httpwwwnlgokr)

① 는 도 리 다8

② 도 는 리 다9 1

③ 료 지 는 도 리 다9 2

④ 도 료 변상에 리10 1

⑤ 는 에 도 리 다11

3

도 다 각 같다①

공 공 다만 연1

연 간 다

매월 째 째 월2

도 도 리 그 사3

가 다고 는

도 에 미리 게1 3②

시 여 다

4

도 시간 도 여 게시 다

5

도 료 시 는 는 도①

지에 등 후

등 에 사 도②

7

는 다 각 여 는 니 다

도 료 시 상 리1 lsquo rsquo

도 료 시 훼 는2 middot

지 가 닌 곳에 식 거 담3

우는

도 보 등 보 검색열4 middot

그 에 도 질 지 여 도5

여 게시 사 는

8

도 다 거 도①

질 게 우 가 는 에 여는 도

도 가 각 어느7②

에는 지 게 거 도

9

도 료는 다 각 경우 다①

상 도 간에 료 는 것 말1 (

다 등 다 도 과 여 경우)

공 원 공 상 는 경우2

그 에 도 다고 는 경우3

가능 도 료 는 도②

는 에 다

10

년 학 간고사 대비2013 2 현대고 대비

ECN-0102-2013-001-000076193

가 도 료 시 럽 거 거①

못 쓰게 거 어 린 경우에는 변상 여

도 에 변상 여 게시1②

여 다

zb44) 위 에서 도서 장이 게 해야 할 사항에 해당하는

것을 두 쓰

년 학 간고사 대비2013 2 현대고 대비

ECN-0102-2013-001-000076193

립 도 규

1 ( )

규 립 도 립 어린 청 도(

포 다 료 시 열 시 말) (

다 에 사 규 립 도)

편 진 다

2 ( )

규 립 도 도 다 에( lsquo rsquo )

고 는 도 에 도lsquo rsquo 2 2

료 에 여 다 다만 특 료 귀

료 등 료 에 사 립 도

도 다 다( lsquo rsquo )

3 ( )

도 다 각 같다①

공 공 다만 연1

연 간 다

매월 째 째 월2

도 도 리 그 사3

가 다고 는

도 에 미리 게1 3②

시 여 다

시간4 ( )

도 시간 도 여 게시 다

등 등5 ( )

도 료 시 는 는 도①

지에 등 후

등 에 사 도②

사 료6 ( )

도 료 시 에 사 료는 도

7 ( )

는 다 각 여 는 니 다

도 료 시 상 리1 lsquo rsquo

도 료 시 훼 는2 middot

지 가 닌 곳에 식 거 담3

우는

도 보 등 보 검색열4 middot

그 에 도 질 지 여 도5

여 게시 사 는

질 지8 ( )

도 다 거 도①

질 게 우 가 는 에 여는 도

도 가 각 어느7②

에는 지 게 거 도

료9 ( )

도 료는 다 각 경우 다①

상 도 간에 료 는 것 말1 (

다 등 다 도 과 여 경우)

공 원 공 상 는 경우2

그 에 도 다고 는 경우3

가능 도 료 는 도②

는 에 다

변상10 ( )

가 도 료 시 럽 거 거①

못 쓰게 거 어 린 경우에는 변상 여

도 에 변상 여 게시1②

여 다

등 규 에 것 에 도11 ( )

료 시 등에 사

도 다

립 도- (httpwwwnlgokr)

zb45) 도서 장의 리 있는 조항으 적절하 않

은 것은

① ② ③ ④ ⑤

년 학 간고사 대비2013 2 현대고 대비

ECN-0102-2013-001-000076193

1 ( )

사가 공 는lsquo rsquo

과 여 사 원과 리

사 타 사 규

니다

개 보 보7 ( )

사는 보통신망 등 계 는 에lsquo rsquo lsquo rsquo

원 개 보 보 니다 개lsquo rsquo

보 보 사 에 는 사 개lsquo rsquo

보 취 니다 다만 사는 다 lsquo rsquo

사 계 통 공 는 경우 원 lsquo rsquo

등 개 보 당 사에 습니lsquo rsquo

원 리에8 (lsquo rsquo lsquo rsquo lsquo rsquo

)

원 에 리lsquo rsquo lsquo rsquo lsquo rsquo①

원에게 가 도 여 는lsquo rsquo 3

니다

사는 원 가 개 보 우 가lsquo rsquo lsquo rsquo lsquo rsquo②

거 사 경우 는 미 에 어 거 lsquo

사 사 운 우 가 는 경우 당rsquo lsquo rsquo

습니다lsquo rsquo

원 가 도 거lsquo rsquo lsquo rsquo lsquo rsquo 3③

가 사 고 지 경우에는 시 사에lsquo rsquo

통지 고 사 내에 니다lsquo rsquo

경우에 당 원 사에 그 사실3 lsquo rsquo lsquo rsquo④

통지 지 거 통지 도 사 내에 지 lsquo rsquo

생 경우 사는 지지 습니다lsquo rsquo

사10 (lsquo rsquo )

사는 과 지 미lsquo rsquo①

에 는 지 계 고

공 여 다 여 니다lsquo rsquo

사는 원 게lsquo rsquo lsquo rsquo lsquo rsquo②

도 개 보 신 보 포 보 보 시( )

갖 어 개 보 취 공시 고

니다

사는 과 여 원lsquo rsquo lsquo rsquo③

견 만 당 다고 경우에는

리 여 니다 원 견 만 사 lsquo rsquo

에 는 게시 거 우편 등 통 여

원에게 리 과 결과 달 니다lsquo rsquo

원11 (lsquo rsquo )

원 다 여 는 니다lsquo rsquo ①

신청 는 변경 시 허 내 등1

타 보 도2

사가 게시 보 변경3 lsquo rsquo

사가 보 보 컴퓨 그4 lsquo rsquo (

등 등 신 는 게시)

사 타 등 지 재산 에5 lsquo rsquo 3

사 타 상 거 업6 lsquo rsquo 3

는 폭 시지 상 타 공7 middot middot

에 는 보 에 공개 는 게시 는lsquo rsquo

사 동 없 리 사8 lsquo rsquo

타 거 당9

게시15 (lsquo rsquo )

원 내에 게시 는 게시 게재 는lsquo rsquo lsquo rsquo lsquo rsquo

경우 원 사가 게시 복 lsquo rsquo lsquo rsquo lsquo rsquo middot middot

등 태 언 등에 공 는

것 내에 다 원 본 게시 등 lsquo rsquo lsquo rsquo

크 능 등 여 복 는 등 태

는 것 동 것 니다

- (wwwnavercom)

zb46) 위 은 인터넷 포털사이트의 회 가입을 위한 이

약 의 일 이다 이 약 을 만드는 과정에서 생각한

내 으 적절하 않은 것은

개 보 보 가 지에 별 눠①

겠어

원 가 만들게 에②

시 주어 겠어

원들 게재 게시 다 원 크 다③

는 것 지

④ 원 지 는 뿐만 니 사가 지 는

도 께 달 지

리에 가 생 경우 사가⑤

에 다는 도 듯

1 ( )

사가 공 는lsquo rsquo

과 여 사 원과 리

사 타 사 규

년 학 간고사 대비2013 2 현대고 대비

ECN-0102-2013-001-000076193

니다

개 보 보7 ( )

사는 보통신망 등 계 는 에lsquo rsquo lsquo rsquo

원 개 보 보 니다 개lsquo rsquo

보 보 사 에 는 사 개lsquo rsquo

보 취 니다 다만 사는 다 lsquo rsquo

사 계 통 공 는 경우 원 lsquo rsquo

등 개 보 당 사에 습니lsquo rsquo

원 리에8 (lsquo rsquo lsquo rsquo lsquo rsquo

)

원 에 리lsquo rsquo lsquo rsquo lsquo rsquo①

원에게 가 도 여 는lsquo rsquo 3

니다

사는 원 가 개 보 우 가lsquo rsquo lsquo rsquo lsquo rsquo②

거 사 경우 는 미 에 어 거 lsquo

사 사 운 우 가 는 경우 당rsquo lsquo rsquo

습니다lsquo rsquo

원 가 도 거lsquo rsquo lsquo rsquo lsquo rsquo 3③

가 사 고 지 경우에는 시 사에lsquo rsquo

통지 고 사 내에 니다lsquo rsquo

경우에 당 원 사에 그 사실3 lsquo rsquo lsquo rsquo④

통지 지 거 통지 도 사 내에 지 lsquo rsquo

생 경우 사는 지지 습니다lsquo rsquo

원에 통지9 (lsquo rsquo )

사는 특 다 원에게 통지 경우lsquo rsquo lsquo rsquo

공지 게시 통 상 게시 개별 통지에7

갈 습니다

사10 (lsquo rsquo )

사는 과 지 미lsquo rsquo①

에 는 지 계 고

공 여 다 여 니다lsquo rsquo

사는 원 게lsquo rsquo lsquo rsquo lsquo rsquo②

도 개 보 신 보 포 보 보 시( )

갖 어 개 보 취 공시 고

니다

사는 과 여 원lsquo rsquo lsquo rsquo③

견 만 당 다고 경우에는

리 여 니다 원 견 만 사 lsquo rsquo

에 는 게시 거 우편 등 통 여

원에게 리 과 결과 달 니다lsquo rsquo

원11 (lsquo rsquo )

원 다 여 는 니다lsquo rsquo ①

신청 는 변경 시 허 내 등1

타 보 도2

사가 게시 보 변경3 lsquo rsquo

사가 보 보 컴퓨 그4 lsquo rsquo (

등 등 신 는 게시)

사 타 등 지 재산 에5 lsquo rsquo 3

사 타 상 거 업6 lsquo rsquo 3

는 폭 시지 상 타 공7 middot middot

에 는 보 에 공개 는 게시 는lsquo rsquo

사 동 없 리 사8 lsquo rsquo

타 거 당9

원 계 규 내lsquo rsquo lsquo②

여 공지 주 사 사가 통지 는rsquo lsquo rsquo

사 등 여 타 사 업 에 lsquo rsquo

는 여 는 니다

- (wwwnavercom)

zb47) 위 약 의 조항에서 같은 제점을 하lt gt

고 있는 조항은

lt gt

제휴 회사에 회 의 아이디 개인 정 를 전송할 있도

한 조항은 고객에게 당한 조항이다

1 7 8① ② ③

④ 9 ⑤ 10

립 도 규

1 ( )

규 립 도 립 어린 청 도(

포 다 료 시 열 시 말) (

다 에 사 규 립 도)

편 진 다

2 ( )

규 립 도 도 다 에( lsquo rsquo )

고 는 도 에 도lsquo rsquo 2 2

료 에 여 다 다만 특 료 귀

료 등 료 에 사 립 도

도 다 다( lsquo rsquo )

3 ( )

도 다 각 같다①

공 공 다만 연1

연 간 다

년 학 간고사 대비2013 2 현대고 대비

ECN-0102-2013-001-000076193

매월 째 째 월2

도 도 리 그 사3

가 다고 는

도 에 미리 게1 3②

시 여 다

시간4 ( )

도 시간 도 여 게시 다

등 등5 ( )

도 료 시 는 는 도①

지에 등 후

등 에 사 도②

사 료6 ( )

도 료 시 에 사 료는 도

7 ( )

는 다 각 여 는 니 다

도 료 시 상 리1 lsquo rsquo

도 료 시 훼 는2 middot

지 가 닌 곳에 식 거 담3

우는

도 보 등 보 검색열4 middot

그 에 도 질 지 여 도5

여 게시 사 는

질 지8 ( )

도 다 거 도①

질 게 우 가 는 에 여는 도

도 가 각 어느7②

에는 지 게 거 도

료9 ( )

도 료는 다 각 경우 다①

상 도 간에 료 는 것 말1 (

다 등 다 도 과 여 경우)

공 원 공 상 는 경우2

그 에 도 다고 는 경우3

가능 도 료 는 도②

는 에 다

변상10 ( )

가 도 료 시 럽 거 거①

못 쓰게 거 어 린 경우에는 변상 여

도 에 변상 여 게시1②

여 다

등 규 에 것 에 도11 ( )

료 시 등에 사

도 다

립 도- (httpwwwnlgokr)

zb48) 다음 정 리 의 의 으 볼 때 가장

이 적인 것은

도 시간 도 여 게시 다①

등 에 사 도②

가능 도 료 는 도 는③

에 다

④ 도 에 변상 여 게10 1

시 여 다

⑤ 도 가 각 어느7

에는 지 거 도

zb49) 를 참고하여 이 어의 성격을 설 한lt gt

것으 적절하 않은 것은

① 보 에 는 어 시 상 고 어 시lt gt lsquo rsquo

에 보여주고 다

② 진 어 어원에 견 고 다

에는 타 어 들어가는 것 다 lsquo rsquo

③ 에 들어갈 말 각각 고 어 어 신 어~

들 언어는 질 격 강 통 없었다

④ 시 우리 에 가 었지만 지 계

과 달리 들 통 사 달 어 웠

년 학 간고사 대비2013 2 현대고 대비

ECN-0102-2013-001-000076193

⑤ 크 몽골 만주 공통어가 우리 어 같

계열에 다는 에 사 특 짐

가( )

善化公主主隱 공주님

他密只嫁良置古 몰 결 고

薯童房乙 맛

夜矣卯乙抱遣去如 에 몰 고 가다

( )

始汝 會隱日恚見隱扐 만 에 본

恥隱汝衣淸隱笑 맑 웃

고 시 여 공 크다 만 다[ ] ( ) ( ) ( ) ( )始 汝 會扐

내다 에 보다 견( ) ( )恚 見 다( )隱

럽다 맑다 청 웃( ) ( ) ( ) ( )恥 衣 淸 笑

zb50) 위의 나 를 함 고 음에 답하( ) lt gt

보lt gt

( )素那或云金川 白城郡蛇山人也

운 사산

는 고 다 는( )[ ( ) ] (素那 金川 白城

사산 사 다) ( ) 郡 蛇山

삼 사- lsquo rsquo 47

에 제 된 단어 의 표 리를 조건(1) lt gt ( ) lt gt

에 맞게 서 하

건lt gt

lsquo 었고 었다 태rsquo

에 제 된 단어 동일한 표 리에(2) lt gt ( )

의해 적은 것을 나 에서 찾아 조건 에 맞게 서 하( ) lt gt

건lt gt

에 당 는 각각( ) 개 쓸 것2 단

당 는 가 여러 개 어도 개만 쓸 것 각2

개 과 도 쪽에 개만2 2

드시 지 것( )

과 동 원리 것lsquo 고

과 동 원리 것 다rsquo

태 것

가( )

素那(或云金川) 白城郡蛇山人也

소나 또는 천 이라 한다 는 성 사( ) ( ) ( )素那 金川 白城郡〔 〕

산 사람이다 현대어 풀이( ) ( )蛇山

나( )

紫布岩乎希 회

執音乎手母牛放敎遣 자 손 암쇼 노히 고

吾 不喩慙 伊賜等肹 肹 나 안디 리샤

花 折叱肹 可獻乎理音如 고 것거 도림다

다 향찰은 리말을 리 으 적은 표 이었 만 생( )

은 고 대를 넘 하고 끊어 고 말았다 랜 세

동안 갈고 닦아 체계적이었던 향찰 표 이 사라졌

을 인은 크게 두 가 나누어 생각해 볼 있다

하나는 족 사회의 한 선호도에서 찾을 있다 라 때

향찰은 주 족 계 에서 사 했을 것으 인다 한 을

알 하고서는 한자를 활 하여 리말을 리 으 표

하 란 가능하 때 이다 런데 족들은 간이 흐

를 향찰과 같은 리 표 을 익혀 사 하 다는

아 한 을 대 사 하는 쪽을 선호하게 되었다 더 이

고 초에 인재 등 을 위해 과거제도가 행되 서 한 선

호도가 더 높아졌고 결 향찰은 소 되고 말았다

또 다른 가능성은 한 어의 특성에서 찾을 있다

터 한 과 일 세 나라는 한자 화 에 속해 다

당연한 이야 겠 만 표의 자인 한자는 어를 표 하

에 매 적절하다 어의 음절은 성 ( ) ( )聲母 韻母

이 어 고 여 에 성조가 추가되어 최종 소리가 결정된

다 래서 어는 단음절을 하나의 한자 표 하 된

다 에 초성 성 종성의 세 가 소가 하나의 음절

년 학 간고사 대비2013 2 현대고 대비

ECN-0102-2013-001-000076193

을 이 는 한 어는 음절 조가 잡하고 음절의 가 많아

서 한자 차 만으 한 어의 소리를 만족 럽게 표 할

없었다 를 들어 한 어에서는 어 니 같이 음절 lsquo rsquo

이 어 단어가 얼마든 있으나 어는( ) 複數音節

자 하나 나타내 만이다lsquo [m ]rsquo 母 ǔ

한편 일 어의 표 은 핵 적 단어는 한자 적고 토는

가나라는 일 의 자 적는 이다 적인 의 를 나

타내는 은 표의 자인 한자 적고 적 계를 나

타내는 토는 표음 자 적는 셈이니 자세히 살펴

리의 향찰 표 을 쏙 빼닮았음을 알 있다 한 어 같

은 착어이 서도 일 어에만 향찰과 유사한 표 이 살아

남은 것은 일 어의 특 때 이다 일 어는 하나의 자음과

음의 결합으 음절을 이 고 침이 거의 없는 음절 언어

이다 이러한 음절의 특색에다가 토가 달한 착어라는 점

이 향찰과 유사한 표 이 살아남을 있는 비결이었다

하 만 같은 착어라도 다양한 음소 침이 달한 한

어는 향찰 표 하는 데 근 적으 한계가 있었다

zb51) 다 하여 의 행에 대한 탐 한 결과( ) lt gt 2

않은 것은

보lt gt

善花公主主隱 공주니믄 공주님( )

----------------------------------------

-

他密只嫁良置古 그 지 얼어 고 몰 결(

----------------------------------------

-

薯童房乙 맛 맛( )

夜矣卯乙抱遺去如 몰 고 가다 에 몰 고(

가다)

주동 역 동- (薯童謠『 』

에 2 ( )他密只嫁良置古

얼다 시집가다 결 다 말 lsquo rsquo

① 실질 미 지니고 므 타 타lsquo ( )rsquo lsquo [ ]

② 에 실질 미 타내고 지 는lsquo rsquo lsquo [ ]rsquo lsquo [ ]密只 密 只

계 타내는

③ 얼어는 실질 미 포 고 므 가lsquo rsquo lsquo [ ]rsquo嫁

것lsquo [ ]rsquo 良

④ 고 어간 는 실질 미 지니고 므lsquo rsquo lsquo -rsquo

것lsquo [ ]rsquo 置

⑤ 고 어미 고는 계 타내고 므lsquo rsquo lsquo- rsquo

고 것lsquo [ ]rsquo 古

가( )

엉 훈 민middot middot middot middot middot世 宗 御 製 訓 民 正 音

말 미 듕 귁에 달middot middot middot middot middot middot middot middot中 國 文 字

니 런middot middot middot middot middot middot 어린middot middot middot middot百 姓

니 고 도 내 들middot middot middot middot middot middot middot middot middot 시러middot

펴 몯middot 미middot middot 니 내middot middot middot middot middot middot middot middot 爲

어엿middot 겨 새middot middot middot 믈여듧middot middot middot middot字 니middot middot middot

사 마다 니겨 킈 middot middot middot middot middot middot middot middot middot便 安

고 미니middot middot middot middot

본 는 상( ) (象

원리에 만들어진 본) ( )形 ㄱ ㄴ ㅁ ㅅ ㅇ

에 는 가 원리에( )加劃

그리고( )ㅋ ㄷ ㅌ ㅂ ㅍ ㅈ ㅊ ㆆ ㅎ

쓰는 병 원리에 만들어진( )竝書

마지막 체( ) ( )異體ㄲ ㄸ ㅃ ㅆ ㅉ ㆅ

ᅀ 다 상 원리에 ㅇ ㄹ

지 는 삼재 상 본 본( ) ( ) ( 天地人 三才

탕 므림과 림에 ) (初ㅡ ㅣ

재)( ) ( )( )出字 再出字ㅗ ㅏ ㅜ ㅓ ㅛ ㅑ ㅜ ㅕ

병 그리고 들 에 다시( )ㅘ ㅝ ㅣ

( )ㅣ ㅢ ㅚ ㅐ ㅟ ㅔ ㆉ ㅒ ㆌ ㅖ ㅙ ㅞ

zb52) 가 에 대한 설 으 르 않은 것을( ) 두 고르

① 어쓰 규 지키고 다

② 리 고 다

③ 말 미 미 등 어 사 다lsquo rsquo

④ 개 지 다

년 학 간고사 대비2013 2 현대고 대비

ECN-0102-2013-001-000076193

⑤ 어 원 에 가 도 고 다

엉 훈 민世 宗 御 製 訓 民 正 音

말 미 듕귁에 달 니

런 어린 니 고 도middot

내 들 시러 펴 몯 미 니middot

내 어엿 겨 새 믈여듧

사 마다 니겨middot 킈 고

미니

훈민 언 본- lsquo rsquo 5 (1459 )

zb53) 위의 에 대한 현대어 풀이가 르~ 않은 것

① 우리 말 과 달

② 어리 말 고 는 것 어도

③ 신 생각 마 껏 펼 는 사 많다

④ 게 생각 여

⑤ 사 마다 게

zb54) 훈민정음 언해 에는 한 을 창제한 동 가 드러나

있다 훈민정음 창제의 정 과 내 이 잘 연결된 것

① 주 신 말 미 듕귁에 달

② 민 신 내 어 겨

③ 신 뻔 킈 고 미니

④ 실 신 사 마다 니겨

⑤ 귀 신 계 주 는 훈민 신과 거리가

가 엉 훈 민( ) middot middot middot middot middot世 宗 御 製 訓 民 正 音 

말 미 귁에 中 國 달 文 字

니 런 어린 니 百 姓

고 도 내 들 시러 펴 몯

미 니 내 어엿 爲 겨 새

믈여듧 니 사 마다 니 字

겨 킈 고 미니 便 安

훈민 언 본- lsquo ( )rsquo ( ) 5 (1459 )訓民正音 世祖

( )

[ 1 ]

동 룡 샤 마다 복( ) ( ) ( )海東 六龍 天福

시니 고 동( ) ( )古聖 同符 시니

[ 2 ]

매 니 곶 여

미 므 니 그 내 러

가 니

[ 125 ]

우 미리( )千世 샨( )定 에( )漢水北 累仁

누 개 샤 복 업 시니( ) ( ) 開國 卜年

신( )聖神 니 샤도 경 근민 샤 욱( )敬天勤民

드시리 다

님 쇼 산 가( ) ( )洛水 山行

미드니 가

어 가- lsquo ( )rsquo 27龍飛御天歌

다 우리신 니쓰고 다만 만 쓰( )

거 샹 귀쳔 다보게 러 귀

여 쓴 도 신 보 가 고 신 에

말 어 보게 각 에 사 들

고 본 몬 능통 후에

죠 죠 니

드 도 만 공 에 사

드 미 죠 고 고 여 보 죠

보다 얼마가 거시 어신고 니 첫

가 죠 니 죠

민 들 어 신 샹

귀쳔 도보고 어보 가 만 늘

고 폐 에 만쓴 죠 민

도 러보지못 고 보니 그게 엇지

심 니 리 보 가 어 운건 다

니 쳣 말마 지 니 고 그

쓰 에 가 우 지 지

몰 거 본후에 가 어 지

고 그니 쓴편지 쟝 보

년 학 간고사 대비2013 2 현대고 대비

ECN-0102-2013-001-000076193

쓴것보다 듸 보고 그 마 니 쓴 고

어 못

그런고 에 리 과 가

만 쓴 못 민 말만 듯고

고 편 그 못 보니 그사 단

병신 못 다고 그사 식 사

니 만 고 다 과 그사

만 고 다 과 업 사 보다 식 고

죠 도 고 각 과

견 고 실 직 귀쳔 간에 그

고도 다 것 몰 귀죡 보다

사 우리 신 귀쳔 다 업

시 신 보고 과 지 게 랴

시니 샹 귀쳔 간에 우리 신 걸

간 보 새지각과 새 걸 미리

독립신- lsquo (1896)rsquo

zb55) 친 어 나의 제 장( ) 2 매 함축적

의 가 가장 유사한 것은

① 지 눈 내리고 매 득 니 내 여 가

사- lsquo rsquo

② 도 어 리듯 그 게 어 다

주 사- lsquo rsquo

③ 눈 살 다 죽 어 린 과 체 여

눈 새벽 지 도 살 다

눈- lsquo rsquo

④ 삶 근심과 고단 에 돌 거니는 여 거 는

여 리 내린 살가지 에 눈 리 눈 리

택 그 생 에- lsquo rsquo

⑤ 늘 러 고 러

청룡 룡 어 개 루 우

신경림 계- lsquo rsquo

zb56) 친 를 위 가 나 에 나타난A B ( ) ( )

세 어의 특 에 의거하여 세 어 표 하

그 산 고 공 도 맑지만

A

주변에 쓰 리는 어리 사 많다

B

건lt gt

식 가 에 타 어 특징에( ) ( )

거 과 어쓰 는 고 지 말 것

A

B

zb57) 가 의( ) 달 아ㆍ 다 의 ( ) 나셔에서 알 있는

세 어 개화 어의 특 을 비 하여 조건 에lt gt

맞게 서 하

건lt gt

어에 는lsquo 개

어에 는 다 태rsquo

zb58) 은 가 는 다 에 나 는 절lt 1gt ( ) lt 2gt ( )

일 를 췌한 것이다 의 의 가 lt 1gt (1)~(2)

유사한 말을 에서 찾아 쓰lt 2gt

보lt 1gt

런 (1) 어린 니 고百 姓

도 내 들 시러 펴 몯 미

사 마다 (2) 니겨 便 安

킈 고 미니

보lt 2gt

죠 고 고 여 보 죠

보다 얼마가 거시 어신고 니 첫 가

죠 니 죠 민

들 어 신 샹 귀쳔

도보고 어보 가 만 늘 고

폐 에 만쓴 죠 민 도

러보지못 고 보니 그게 엇지 심

니 리

년 학 간고사 대비2013 2 현대고 대비

ECN-0102-2013-001-000076193

lt 1 gt

동 룡 샤 마다 복 시( ) ( ) ( )海東 六龍 天福

고 동 시니( ) ( )古聖 同符

lt 2 gt

(A) 매 니 곶

여 니

미 므 니 그 내

러 가 니

lt125 gt

우 미리 샨 에( ) ( ) ( ) 千世 定 漢水北 累

누 개 샤 복 업 시 니( ) ( ) 仁開國 卜年 聖

신( ) 神 니 샤도 경 근민 샤( ) 敬天勤民

욱 드 시 리 다

님 쇼 산 가 ( ) ( )洛水 山行

미드니 가

- lt gt龍飛御天歌

zb59) 장과 내 상 유사한 성격의 조는125

① 뫼 고 고 고 고

어 그린 많고 많고 고 고

어 러 는 울고 울고 가느니

도 견- lt gt

② 강 에 드니 몸 다

그믈 고 가니

뒷 뫼 엄 언 니( )藥

-

③ 말 없는 청산 태 없는 다

값 없는 청 없는 월

에 병 없는 몸 별 없 늙 리

-

④ 가마귀 골에 가지 마

낸 가마귀 새

청강에 것 시 몸 러 가( ) 淸江

-

⑤ 진 골에( ) 白雪

가 매 는 어느 곳에 었는고

에 갈 곳 몰( ) 夕陽

색-

zb60) 위 에 나타난 세 어의 특 으 적절하 않은

것은

① 룡 어 주격 사에 당 는 가 사( ) lsquo rsquo六龍

고 다

② 샤 어에도 어 주체 쓰 다

는 것 다

③ 매 어 달리 사 택에 어

가 지 지지 고 다

④ 므 원 상 직 어 지 다

⑤ 드시리 다 주체 과 상 께 사

고 다

수고 하셨습니다hearts hearts

년 학 간고사 대비2013 2 현대고 대비

ECN-0102-2013-001-000076193

보닷컴에 공 는 별 보는 고등

들 여 주 는

들 습니다 슷 동 지

가 복 는 것 도가

니 복 여 습 시고 거 시

니다

정답 해설

1) 정답[ ] ④

해설 다른 것은 두 특정 업이나 단 내에서 사[ ]

하는 일종의 은어 사회 언에 해당한다 러나

는 언이 아니라 단과대학을 여서 단대 사lsquo rsquo lsquo rsquo lsquo④

대학을 여서 사대라고 한 말에 해당하 일rsquo lsquo rsquo

사회에서도 널리 쓰이 사회 언이라 할

없다

2) 정답[ ] ⑤

해설 사회 언은 같은 단 내에서 쓰이는 언어이[ ] lsquo rsquo

동일 단끼리는 단결 과 친 감을 형성하는

능을 하 리적 안감이 일어나 않는다

3) 정답[ ] ③

해설 사람이라는 차 적 표현에 대한 대안적 표현이[ ]lsquo rsquo

인 아내 처 등으 볼 있다lsquo rsquo

4) 정답[ ]⑤

해설 남성은 주 격 체를 사 한다[ ]

5) 정답[ ] ⑤

해설 흑인은 검다라는 뜻을 가 고 있을 뿐 인[ ]lsquo rsquo lsquo rsquo lsquo rsquo

다 열등한 뜻을 내포하 않는다

6) 정답 살 색 첫 작품[ ] - -

해설 살색 혹은 킨색은 한 인의 피 색을 뜻[ ] lsquo rsquo lsquo rsquo

하는 것으 인종 차 을 추 고 출 이주민

의 평등 을 침해할 있어 년 표 이2005

살 색으 이름을 꾸었다 처녀작은 처녀라lsquo rsquo lsquo rsquo lsquo rsquo

는 단어가 가 고 있는 곡된 성 인 을 한 것

으 첫 작품정도 꾸어 사 하는 것이 좋다lsquo rsquo

7) 정답[ ] ⑤

해설 호는 아들에게 해체를 사 하고 있다[ ] ① ②

장 을 성하는 청자는 자 의 아 느리 아lsquo

들 세 이다 호는 아 느리에게 해rsquo ③

체를 사 하고 있다 호가 느리 아 에게 ④

사 한 해 체 아들에게 사 한 해체는 두 비lsquo rsquo lsquo rsquo

격 체에 해당한다 호는 자 의 아랫사람인 ⑤

느리에게 아들과 마찬가 해체를 사 하는 것이

상 이 만 임 을 한 느리에게 고마 과 쁨

존 의 표 를 하 위해 자 의 아 에게 말하듯

해 체를 사 하고 있다

8) 정답[ ] ③

9) 정답[ ] ⑤

10) 정답[ ] ①

해설 청자 할아 가 장의 주체 아 다 높을[ ] ( ) ( )

경 에는 압존 에 의해 장의 주체를 높이 않는lsquo rsquo

다 러 아 서가 아닌 아 는으 계 lsquo rsquo lsquo rsquo lsquo

니다 가 아닌 있 니다 표현하는 것이 르rsquo lsquo rsquo

11) 정답 당이 당을 쫒았다 당이[ ]

당에 다

해설[ ]

12) 정답[ ] ⑤

해설 서 다른 높임표현을 통해 청자에 대해 리[ ] ⑤

적 거리감을 나타내는 인 은 이 아니라 현정이

다 가 에서 현정은 에게 해 체를 사 함으 써 ( )

친근감을 드러낸다 나 에서 연 을 게을리하는 역 ( )

도 들 때 에 화가 난 현정이 선생님에게 항의하

는 장 에서는 하 체를 사 하여 리적 거리lsquo rsquo

가 어졌음을 나타내고 있다

13) 정답[ ] ①

해설 는 는 얼 빛이 날과 어찌 다르 고[ ] lsquo rsquo

라는 뜻으 전과 달리 임이 화자를 않고

있음을 알 있다

14) 정답 달리 후 가 있다 이를 통해 경[ ] lt gt

쾌한 음악성을 형성하고 노 젓는 상황을 체적으

형상화하는 역할을 한다

15) 정답[ ] ①

16) 정답[ ] ⑤

해설 다 의 자연은 를 성찰하게 하는 대상[ ] ( )⑤

이자 정의 대상이다 의 자연은 자 의 상황과 ⑤

처 를 드러내는 경으 서의 역할을 하 이

이 없다

17) 정답[ ] ③

해설 는 빈천 을 해결하고자 했으나 강산[ ] lsquo ( )rsquo 貧賤③

과 풍 을 달라는 에 거절하 다고 함으 써 자

연에 대한 애정을 드러내고 있으 는 않는

임에 대한 망을 개에게 전가 켜서 임에 대한 리

을 드러내고 있다

18) 정답[ ] ③

년 학 간고사 대비2013 2 현대고 대비

ECN-0102-2013-001-000076193

19) 정답[ ] ⑤

해설 고상한 음악가의 이름을 리말 꽝 럽[ ]

게 꿈으 써 언어유희를 통해 음을 유 하고 있

다 이는 고상한 척하는 총 를 비꼼으 써 비판적

태도를 드러내는 것이 대상을 꽝 럽게 표현

하여 총 의 허 과 사치를 풍자하고 있다

20) 정답[ ] ⑤

해설 는 작품 속 경에 대한 설 이 드러나는 것이[ ]

서 자의 주 적인 견해가 접적으 드러나는 것이

아니다

21) 정답[ ] ⑤

22) 정답[ ] ②

23) 정답[ ] ④

24) 정답[ ] ①

해설 적강 티프는 주인공의 비 한 출생이나 능[ ] ①

과 이 있는 것으 조정의 능함을 풍자하는lsquo rsquo

것과는 거리가 다

25) 정답 픔 나[ ] ( )

해설 의 음악은 고통 는 사람들을 위 하고 아픔[ ] lsquo rsquo

을 치유해 주는 능을 한다고 할 있다 의 lt gt

픔 도 소 된 이 과 더 어 살아가는 따뜻한 마음lsquo rsquo

을 상 한다

26) 정답[ ] ⑤

해설 에게 선천적으 주어 각 장애라는 역경[ ]

은 의 이라는 가사 연 을 있다lsquo rsquo

27) 정답[ ] ④

해설 는 장 란 선 에게 은 개인적인 인상을[ ]

소녀 장정 등으 표현한 것이다lsquo rsquo

28) 정답[ ] ②

해설 담자가 피 담자의 언어적 표현이나 비언어[ ]②

적 표현 하 독자는 담의 위 나 피

담자의 감정 상태를 알 있다 이를 통해 독자는

담 상황을 더 생생하게 느낄 있고 피 담자

를 더 잘 이해할 있게 된다

29) 정답[ ]③

해설 일상생활과 역도 선 서의 성과에 된 것에서[ ]

역도를 하 서 겪는 어 과 내적 고민으 화제를

전화하 위한 것이다

30) 정답[ ] ①

해설 릿속에 새겨 넣듯 이 억되도 함 세상[ ] ② ③

살이가 힘들고 고생 러 속 하여 자유를 ④

가 없는 고통의 상태를 비유적으 이르는 말

적의 침입을 막 위해 쌓은 축 켜야 할⑤

대상을 비유적으 이르는 말이다

31) 정답[ ] ④

해설 이 의 종류는 전 으 인 사건 경[ ] lsquo

비평을 성 소 삼는다rsquo

32) 정답[ ] ④

해설 근은 삼대독자 태어났음을 에서 확인할[ ]

있다 형제들과의 담은 이뤄 가 없다

33) 정답[ ] ⑤

해설 근은 가난에도 하고 화가를 꿈꾸었다[ ] (3

단 또한 다른 화가 망생들은 정 육을)

위해 상 학 학 해 유학 에 랐 만

근은 다른 을 찾아야 했다 단 세에(5 ) 18

근은 조선 전람회에 입선하 다 단 의(6 )

만종은 인간과 자연이 엮어 가는 경건한 조화 을lsquo rsquo

나타낸다

34) 정답[ ] ①

해설 근이 속에서도 창작활동을 추 않고[ ]

하는 닭은 은 세상과 타협할 르는

근이 세상의 이해를 하 위한 가장 떳떳한 단

이 때 이다

35) 정답[ ] ⑤

해설 전 은 서 자의 주 적인 평이 리는 것이[ ]

만 위 제 은 인 이 살았던 대 사회적 경

을 통해 객 적인 인 의 을 제 하고 있다

36) 정답[ ] ⑤

해설 전 은 인 사건 경 비평이라는[ ] lsquo rsquo⑤

성 이 어져 있다

37) 정답[ ] ①

해설 이 은 동양인과 서양인의 사고 에 차이가[ ]

있다는 것을 대조를 통해 설 하고 있다 또 쓴이

의 제자가 축 경 를 러 가서 경험한 일화를

통해 동양인이 서양인에 비해 주 상황에 더 많은

주의를 인다는 주장을 뒷 침하고 있다

38) 정답[ ] ④

39) 정답[ ] ②

40) 정답[ ] ②

41) 정답[ ] ④

42) 정답[ ] ③

43) 정답[ ] ④

44) 정답 도서 의 휴 일 도서 의 이 간 도서의[ ]

해설 도서 장은 임의 정한 휴 일과 도서 이[ ]

간 도서의 상 등을 게 할 의 가 있다

년 학 간고사 대비2013 2 현대고 대비

ECN-0102-2013-001-000076193

45) 정답[ ] ①

해설 제 조의 정 휴 일 의 휴 일의 사전 게[ ] 3

는 도서 장의 의 조항에 속한다

46) 정답[ ] ①

해설 개인 정 호 의 를 제 하 했 만 항[ ]

나눠서 제 하 않고 대 나열하고 있다

47) 정답[ ] ②

해설 제 조의 내 을 회사는 다른 회사 협[ ] 7 lsquo

계약을 통해 서비 를 제공하는 경 회 의 아이디

등 개인 정 를 해당 회사에 전송할 있다는 내rsquo

이 있으 의 제점을 제 할 있다②

48) 정답[ ] ④

해설 는 도서 장의 의 에 해당하고 나 는 도[ ] ④

서 장의 리에 해당한다

49) 정답[ ] ③

50) 정답 은 음독으 적었고 은 훈독으 적었[ ] (1)

다 과 동일한 표 리 적은 것은 이고 (2) ce

과 동일한 표 리 적은 것은 이다ab

51) 정답[ ] ③

52) 정답[ ] ①②

53) 정답[ ] ③

54) 정답[ ] ③

55) 정답[ ] ①

56) 정답 른 죠코 어린 노 하니라[ ] A B

57) 정답 세 어에서는 활 형이 칙적으[ ] lsquo rsquoㄹㅇ

나타났 만 개화 어에서는 활 형이 쓰 다 lsquo rsquo ㄹㄴ

58) 정답 호 가 흔[ ] (1) (2)

59) 정답[ ] ④

60) 정답[ ] ③

Page 17: 현대고대비 국어 - chamsoriedu.com 「콘텐츠산업진흥 법」외 에도 저작권 의하여 ... 다른주체에게어떤동작을하도록만드는것을나타내는

년 학 간고사 대비2013 2 현대고 대비

ECN-0102-2013-001-000076193

에도 고 시 얻지 못 다

감지 없는 시각 상태 다

신 지에 고 상 원망 도

단다 어느 가 에 시각 에 ldquo

어 그런 듣고 다 보니 내가 게 lsquo

살 는지 도 눈 고 싶rsquo lsquohelliphellip

보 는 생각만 들 고 그 가 들에게rsquo

도 내고 들도 고 많 었죠 들 rdquo

었 지 새 는 에 쑥 러운 색

어났다

생에 것 단연 었다lsquo rsquo

공연에 거 꿈lsquo rsquo

는 다 특 가사 갑게 는 운 lsquo

벽 에 당당 마주 어 언 가 그 벽

고 늘 어 거운 상도

없죠 내 삶 에 웃 그 께

는 다고 다rsquo

들었 그냥 런 도 고만 여ldquo lsquo rsquo

겼죠 그런 꾸 가사 미 새 다 보

니 통 는 가사 는 생각 들 고 (

가 게는 시각 는 생각 들고 들) ( )

마다 듣고 큰 얻었어 rdquo

에 진지 게 가에 미 가

zb26) 의 에 들어갈 말 적절한 것은lt gt ~

lt gt

난 난 꿈이 있었죠

고 찢겨 남 하여도

내 가 히 과 같이 간 했던 꿈

혹 때 누 가가 뜻 를 비 음

내 등 뒤에 흘릴 때도

난 참아야 했죠 참을 있었죠

날을 위해

늘 걱정하듯 말하죠

헛된 꿈은 독이라고

세상은 끝이 정해 책처럼

이 돌이킬 없는

현 이라고 helliphellip

래 난 난 꿈이 있어

꿈을 믿어

나를 켜

저 차갑게 서 있는 이란 앞에

당당히 마주칠 있어

출처 가 거위의 꿈 작사 이적 작곡 동률- lsquo rsquo ( )

① ② ③ ④ ⑤

가 떴다는 들 만 지만( ) lsquo rsquo

늘 겸 다 에 주 연 우승 지 간에도 3

단 생님께 만 지 고 고 만ldquo rdquo

큼 늘 겸 신 계 가

고 다

에게는 꿈 다 통 누 가

주겠다는 것 그 꿈 다 신 극복 는 과

에 큰 경험 들도 느 게 주

고 싶다는 것 다

슬 마다 통 낼ldquo

었 것 럼 고통 는 사 들

고 겠다 고rdquo

말 다 달 루 첫 낸lsquo rsquo

첫 드 심 집에 는 리듬 2

루 에 도 보고 싶다(RampB) 집 에는 직34

사 곡 도 보 고 싶다고 포 다

미 는( ) (26) 어 헤헤헤 웃다가 어ldquo rdquo

허허허 웃었다ldquo rdquo ldquo rdquo 같 도 고

상 다 는 같 도 다( ) 壯丁 킹 들lsquo

다 는 역도 보 그 다 지만 그는rsquo

뷰에 지 다 운동만 지 ldquo

것 지 간에 여러 사 도 역rdquo helliphellip

었다 그런 엇 그 마 움직 는지 보 쯤

지 담 사 다 훈 없어 그는 티

지 림 었다 태 다 갔다 는 습

마 집 럼 편 게 보 다

주말에는 주 엇 보내

주말에도 별 주 에 청ldquo

고 에 가고 도 쳐

에 듣고 보 에 갈 가 별 없

어 산 시 게 고 들어 2002

거 매 여 지냅니다 시 과 지훈 rdquo

다 근 간 과 진실 그리고 싶어( )

가 다 근에게 그것 진리 다 거 다 없

거 고 다 없 는 것 진리

다 근 진리는 후 쪽 었다 신산( )辛酸 삶

었 질곡( )桎梏 역사 에 지냈 가

눈에 든 것 료 단 료 게 보

것 었다 그것 그 에 겨우겨우

슬 슬 생 어가는 간들 었다

리 과 단 리 고리에 검 마

없 거리 돌

상 것 없는 등 근에게 상

과 진실 엄 ( )儼存 다는 사실 리는 가

실 고 가 과 역경 에 도 근 내 포

없었 후 보루( )堡壘 다 도 365

도 간 근 여

시 것 다

년 학 간고사 대비2013 2 현대고 대비

ECN-0102-2013-001-000076193

다 공주 그림 가 근 경- ( ) ldquo rdquo(

2009)

zb27) 작가의 주 적인 각이 드러난 것은~

① ② ③ ④ ⑤

가 신 지에 고 상 원망( )

도 단다 어느 가 에 시각 에 ldquo

어 그런 듣고 다 보니 내가 lsquo

게 살 는지 도 눈 고 싶rsquo lsquohelliphellip

보 는 생각만 들 고 그 가 들에게rsquo

도 내고 들도 고 많 었죠 들었rdquo

지 새 는 에 쑥쓰러운 색

어났다 략 [ ]

경 는 가 망 없 티lsquo

원 고 답 다 신과 같 시각rsquo

는 습 상상 만 도 감동

다 시각 연주 동시에

열 상 는 티

원 그런 열 경 럽다는 것 다 략 [ ]

슬 마다 통 낼ldquo

었 것 럼 고통 는 사 들

고 겠다 고rdquo

말 다 달 루 첫 낸 lsquo rsquo

첫 드 심 집에 는 리듬 2

루 에 도 보고 싶다 집 에는 직(RampB) 3 4

사 곡 도 보 고 싶다고 포 다

식 누 가-

고 싶어

다 역도 미 담 고 사( )

질 주말에는 주 엇 보내[ 1]

답 주말에도 별 주 에[ ] ldquo

청 고 에 가고 도 쳐

에 듣고 보 에 갈 가 별

없어 rdquo

질 계 고 슬슬 도 는 것 닙니[ 2]

답 다 들 눈 에 보 고 뿐 보[ ] ldquo

다 열심 고 어 상에 도 들지만 상

지키는 것 들다고 에 도달

그것 지키 훨 많 rdquo

질 들 살 고 리 는[ 3]

거운 들 체 리느 는다

답 가 고 게 체 어[ ] ldquo ( ) 級

느 도 계가 니 살 는 것도 고역 지만

살 우는 것 들어 는 살

체 리 고 어도 어도 실 갔다

쑥 어 rdquo

질 거리에 슷 연 여 들[ 4]

보는 간 상 지

답 상 다 체 게 리지 못[ ] ldquo

거 주변에 는 그 거 누 보지

못 고 뻐지고 싶 에 체 리는 에

타 워 지만 는 어울 는 것보다 는

시간 운동만 는 건 니에 사복 lsquo rsquo

고 사복 는 말에 들 웃지만 늘 운동복

고 지내니 사러 갈 도 어 rdquo

질 역도가 말 단 식 운동 니[ 5]

답 가 내는 만 클 업 보[ ] ldquo

그러니 만 쓰는 식 운동 니다

만 다고 거운 것 들 는 건 니거든 연

도 고 가지 동 에 도 여러 가지

복 들

보식 역도 여 미-

zb28) 가 에 대한 설 으( ) 않은 것은

① 시각 우 지 시 에 지

고 망 가는 태도 달 고 다

② 언어 과 언어 복 사 여

담 내 생각 게 는 가

③ 직 감 그 마 것

럼 생생 게 느껴지는 과 주고 간 내

없 리 어 억 게 다

④ 담 내 식 리 여 담 삶 습

과 가 시 여 독 에게 감동과 훈 다

⑤ 직 진 담 직 누

지 못 는 독 에게 생생 상 달 주고

담 욱 게 다

zb29) 나 의 각 의 의도를 설 한 것으 적절하( ) 않

년 학 간고사 대비2013 2 현대고 대비

ECN-0102-2013-001-000076193

은 것은

① 질 담 상 보여 주 것 다1

② 질 담 과 그에 삶 태도 보여2

주 것 다

③ 질 역도 겪는 어 움에 역도3

과 것 다

④ 질 같 연 여 갖는 고민 는지 말4

주 는 것 다

⑤ 질 역도가 과 고 운동 는 것5

담 가 말 주 는 것 다

가 만진 것 다( ) 3

감 달 다고 다 억 에( ) 音感

지워 지만 당시 청 탁 리도

다고 다 드럼 웠다 4

에 갈 마다 드럼 는 리가 신 게 들

다고 다 눈 볼 가 없 니 엔ldquo

는 는 님 틱 에 여 주

다 드럼과 연 맺 과 들 주었다rdquo

식 누 가-

고 싶어

역( ) 도가 말 단 식 운동 니

가 내는 만 클 업에 보ldquo

그러니 만 쓰는 식 운동 니다 만

다고 거운 것 들 는 건 니거든 연

도 고 가지 동 에 도 여러 가지 복

들 시 는 상 상

드는 상 에 맞춰 실 에 는 여러

펼쳐집니다rdquo

략( )

늘 에 는 어 만 것 같

가 에 사 고 사 사ldquo

겠어 든 에 가 경 만 고

울 는 사 겠어 rdquo

보식 역도 여 미-

다 가 운 는 어 어( ) ldquo rdquohelliphellip

월 새벽 시 태 없 거웠고1965 5 6 1

는 없 그 병원에 퇴원 집

가는 마지막 마 고 마 내 거 다

가 죽 간신 에 실 다 사는 어느5 lsquo

가 죽 는 말 가 식 다 신rsquo

상에 각 시키는( )刻印 에 실

어느 가는 후 민 가가 근 었다lsquo rsquo

는 간 과 진실 그 다는( ) ldquo

에 단 평 견 가지고 다 내

가 그리는 간상 단 고 다 지 다 는 그들

가 에 는 평 지 니 그리고 어린

들 미지 겨 그린다rdquo

마 근 간 과 진실 그리고 싶어( )

가 다 근에게 그것 진리 다 거 다 없

거 고 다 없 는 것 진리

다 근 진리는 후 쪽 었다 신산(辛酸 삶)

었 질곡(桎梏 역사 에 지냈)

가 눈에 든 것 료 단 료 게

보 것 었다 그것 그 에 겨우겨우

슬 슬 생 어가는 간들 었

다 리 과 단 리 고리에 검

마 없 거리 돌

상 것 없는 등 근에게 상에

과 진실 엄 다는 사실 리는 가( )儼存

실 고 가 과 역경 에 도 근 내

포 없었 후 보루(堡壘 다 도)

도 간 근365

여 시 것 다

월 강원도 림리에( ) 1914 2 21

삼 독 태어났다 어 근 복

그것 그리 가지 못 다 근 곱 살

지는 산 산업에 실 고 답마 에 내

갔다 근 그림 럼 쫓 다니 가 시 것

다 상 진 것도 가 었다

러 가 에도 고 근 가 꿈꾸었다 근

가 꿈꾸게 것 보통 업

원색도1926 만lsquo rsquo 었다

공주 그림 가 근 경-

zb30) 에 대한 설 가장 른 것은~

① 역도가 과 운동 도 질

② 리는 는 다 lsquo rsquo

③ 들었지만 그럭 럭 는 다 lsquo rsquo

④ 가 게 보 시 말 다

⑤ 보 병 는 지 상 lsquo rsquo

는 말 다

년 학 간고사 대비2013 2 현대고 대비

ECN-0102-2013-001-000076193

시간 많지 다 청량리 생 병원

마지막 상 경 릿 게 들어 다 그 는 십

만 큰 가 상 말 다

지 못 들 마 갈 고 돗

도시민들 싹 싹 탔다 가 시

월에 병원에 원 가 폐 진 몸도4 ( )疲弊

갈 미 지 못 고 었다 가는 얼마( ) 解渴

지 생 에 생각 가

마감 는 신 평생 십 만에

가 과 많 닮 다고 생각 지는

가 운 는 어 어ldquo rdquo 1965helliphellip

월 새벽 시 태 없 거웠고 는5 6 1

없 그 병원에 퇴원 집 가

는 마지막 마 고 마 내 거 다 가

죽 간신 에 실 다 사는 어느 가5 lsquo

죽 는 말 가 식 다 신rsquo

상에 각 시키는 에 실 어느( ) lsquo刻印

가는 후 민 가가 근 었다rsquo

ldquo 는 간 과 진실 그 다는 에

단 평 견 가지고 다 내가 그

리는 간상 단 고 다 지 다 는 그들 가

에 는 평 지 니 그리고 어린 들

미지 겨 그린다rdquo

근 간 과 진실 그리고 싶어 가

다 근에게 그것 진리 다 거 다 없 거

고 다 없 는 것 진리다

근 진리는 후 쪽 었다 신산 삶 ( )辛酸

었 질곡 역사 에 지냈 가 눈에( )桎梏

든 것 료 단 료 게 보 것

었다 그것 그 에 겨우겨우 슬

슬 생 어가는 간들 었다 리

과 단 리 고리에 검 마

없 거리 돌 상

것 없는 등 근에게 상에 과 진실

엄 다는 사실 리는 가 실( )儼存

고 가 과 역경 에 도 근 내 포 없었

후 보루 다 도 도( ) 365堡壘

간 근 여 시 것

간에 지닌 가 근 1914 2

월 강원도 림리에 삼 독21

태어났다 어 근 복 그것 그리

가지 못 다 근 곱 살 지는 산

사업에 실 고 답마 에 내 갔다 근

그림 럼 쫓 다니 가 시 것 다 상

진 것도 가 었다 러 가 에도

고 근 가 꿈꾸었다 근 가 꿈꾸게

것 보통 업 원색1926

도 만 었다lsquo rsquo

그림 가 근 경 공주- ldquo rdquo ( 2009)

zb31) 다음 이 같은 의 성 소에 해당하 않은

것은

사건 평① ② ③

④ 주 ⑤ 경

가 운 는 어 어ldquo rdquo 1965helliphellip

월 새벽 시 태 없 거웠고 는5 6 1

없 그 병원에 퇴원 집 가

는 마지막 마 고 마 내 거 다 가

죽 간신 에 실 다 사는 어느 가5 lsquo

죽 는 말 가 식 다 신rsquo

상에 각 시키는 에 실 어느( ) lsquo刻印

가는 후 민 가가 근 었다rsquo

는 간 과 진실 그 다는 에ldquo

단 평 견 가지고 다 내가 그

리는 간상 단 고 다 지 다 는 그들 가

에 는 평 지 니 그리고 어린 들

미지 겨 그린다rdquo

근 간 과 진실 그리고 싶어 가

다 근에게 그것 진리 다 거 다 없 거

고 다 없 는 것 진리다

근 진리는 후 쪽 었다 신산 삶 ( )辛酸

었 질곡 역사 에 지냈 가 눈에( )桎梏

든 것 료 단 료 게 보 것

었다 그것 그 에 겨우겨우 슬

슬 생 어가는 간들 었다 리

과 단 리 고리에 검 마

없 거리 돌 상

것 없는 등 근에게 상에 과 진실

엄 다는 사실 리는 가 실( )儼存

고 가 과 역경 에 도 근 내 포 없었

후 보루 다 도 도( ) 365堡壘

간 근 여 시 것

간에 지닌 가 근 1914 2

월 강원도 림리에 삼 독21

태어났다 어 근 복 그것 그리

가지 못 다 근 곱 살 지는 산

사업에 실 고 답마 에 내 갔다 근

그림 럼 쫓 다니 가 시 것 다 상

진 것도 가 었다 러 가 에도

고 근 가 꿈꾸었다 근 가 꿈꾸게

것 보통 업 원색1926

도 만 었다lsquo rsquo

공주 그림 가 근 경- ldquo rdquo ( 2009)

년 학 간고사 대비2013 2 현대고 대비

ECN-0102-2013-001-000076193

zb32) 위 을 작성하는 과정에서 되어 활 된 자

어 것은

신 사 료① 연보②

고③ ④ 들과 담

⑤ 에 평

는 간 과 진실 그 다는 에ldquo

단 평 견 가지고 다 내가 그

리는 간상 단 고 다 지 다 는 그들 가

에 는 평 지 니 그리고 어린 들

미지 겨 그린다rdquo

근 간 과 진실 그리고 싶어 가

다 근에게 그것 진리 다 거 다 없 거

고 다 없 는 것 진리다

근 진리는 후 쪽 었다 신산 삶 ( )辛酸

었 질곡 역사 에 지냈 가( )桎梏

눈에 든 것 료 단 료 게 보

것 었다 그것 그 에 겨우겨우

슬 슬 생 어가는 간들 었다

리 과 단 리 고리에 검 마

없 거리 돌 상

것 없는 등 근에게 상에 과

진실 엄 다는 사실 리는 가 실( )儼存

고 가 과 역경 에 도 근 내 포

없었 후 보루 다 도 도( ) 365堡壘

간 근 여 시

것 다

간에 지닌 가 근 1914 2

월 강원도 림리에 삼 독21

태어났다 어 근 복 그것 그리

가지 못 다 근 곱 살 지는 산

사업에 실 고 답마 에 내 갔다 근

그림 럼 쫓 다니 가 시 것 다 상

진 것도 가 었다 러 가 에도

고 근 가 꿈꾸었다 근 가 꿈꾸게

것 보통 업 원색1926

도 만 었다lsquo rsquo

질 루 마 가 도 린다 경건

움 느껴지는 경 다 훗 근 그림에

과 는 거 것( )裸木

만 간과 연 엮어 가는 경건 움lsquo rsquo

니었

같 가가 고 싶었 근에게 그 꿈에 다

가가는 지 다 다 가 지망생들 규 미

상 에 진 고

에 지만 근 다 다 근

미 에 운 것 보통 시 미 시간

다 그런 그에게 없는 연습 가가

통 다 가 귀 시 지 도

얻는 뛸 듯 뻤지만 마 도 가 에

듯 는 었 에 어린 근 주 에

에 그림 그리고 지우고 복( )粉板

시간 가는 게 루 보냈다

근 그 갈 가가 것 열여( )渴求

었 다가 미1932 lsquo rsquo ( lsquo

미 에 다 다는 고 마rsquo) lsquo rsquo

가 근 집 고도 지는 시골 경

그린 그림 다 후 근 에 1943 22

지 미 에 그림 고

에 걸쳐 다 미 근 가

동 는 었다

공주 그림 가 근 경- ldquo rdquo ( 2009)

zb33) 위 의 내 과 일치하는 것은

가 근 가 꿈 포 다①

근 당 가들과 께 에 다②

살 근 가 걷20③

게 었다

④ 만 통 근 역경 겨내는lsquo rsquo

느 다

⑤ 근 간 과 진실 그리 에 그 에

드러 는 간상 단 다

계 시 주 근 건강

걸었다 신 과 간에 상 다 건강

신 는 눈에도 다 근 쪽 눈 뿌 게

보 지 과에 다 다 시 지지 고 결

내 었다 시 지만 마 막막

다 늦어 결 근 쪽 눈 고 말 다

쪽 눈 근에게는 쪽 눈 었고

계 었다 그 근 는 여 그lsquo rsquo

다 근 에 같 그림 그 었다1950

시 그림 는 여 쪽lsquo rsquo

고 어 마주 고 는 그림1963

여 과 동 다 마 복

그린 듯 눈 내리 새 게 다 지

사 다 근 게 복 것

복 상과 타 는 근 상

가 떳떳 단 었고 근 그리고

간 과 진실 에 다가가 가 근다

운 었다 근 신에게 당당 지 그리고

그 다 근 그림에 단 복 보다

년 학 간고사 대비2013 2 현대고 대비

ECN-0102-2013-001-000076193

태 도 그리고 극 보다 과

얻 여 었다 과 통

근 그리고 는 재 고 에 질

만들고 특 것 다

공주 그림 가 근 경- ldquo rdquo( 2009)

zb34) 의 이유에 대해 추 한 것으 적절하 않은 것

상과 타 시도①

보다 과 얻②

근 신에게 당당 지③

④ 간 과 진실 에 다가

⑤ 태 도 얻

근 가가 었지만 그 다니 가

럼 어지지 다 복과 쟁 거쳐 시

는 가 근에게 생계 사 에

운 사 다 에 키에 건( ) 178cm死鬪

체 근 에 동 역 업( )荷役

가 생계 다 쟁

에는 동에 운 상우 주 미

죄 사 에 그림 그리는 시 다 그곳에

에 동 역 업 것에

결 것 럼 보 다 지만 그런 것만도

니었다 그림 그리는 고는 지만 매 근

는 극 간 과 별 없는 경 리 그림

벽에 그리는 것 었다 우도 리 없었다 근

트 는 우 그림 그 다 생

계 그림 단 것 다

후 근 지 신 계 리에 미

엑 리 겼다 근 곳에

건 사 크 에 미 들 ( )

상 상 그 다 근 갖 다 겪

냈다 그리고 결 그 돈

신동에 어 사리 집 마 다 마 ㄷ

루 심 쪽에는 과 엌 쪽에는 건

었다 건 주고 근 가 에

여 살 다 심 에는 지 집어

쓰고 지만 곳 근 가 에게 러웠

보 리 다 근 과 마루 업실 삼 그림

그 다 신동 마루는 근 그림에 등 는 lsquo rsquo

같 상들 지 다 시 고

에 들 폐허가

가 업실 었다

공주 그림 가 근 경- ldquo rdquo( 2009)

zb35) 위 에 대한 설 으 적절한 것은

업 시 여 훈과 감동 다①

에 주 평 드러 다②

사 사 등 식 과 ③

④ 다 근거 시 여 삶에

⑤ 살 시 사 경 께 여

습 시 다

가 시간 많지 다 청량리 생 병원( )

마지막 상 경 릿 게 들어 다 그 는

십 만 큰 가 상 말 다

지 못 들 마 갈 고 돗

도시민들 싹 싹 탔다 가 시

월에 병원에 원4 가 폐( )疲弊

진 몸도 갈 미 지 못 고 었다( )解渴 가는

얼마 지 생 에 생각

가 마감 는 신 평생 십 만에

가 과 많 닮 다고 생각 지는

가 운 는 어 어( ) ldquo rdquohelliphellip

월 새벽 시1965 5 6 1 태 없 거웠고

는 없 그 병원에 퇴원 집

가는 마지막 마 고 마 내 거 다

가 죽 간신 에 실 다 사는 어느5 lsquo

가 죽 는 말 가 식 다 신rsquo

상에 각 시키는 에 실( )刻印

어느 가는 후 민 가가 근 었다lsquo rsquo

다 는 간 과 진실 그 다는( ) ldquo

에 단 평 견 가지고 다 내

가 그리는 간상 단 고 다 지 다 는 가

에 는 평 지 니 그리고 어린 들

미지 겨 그린다rdquo

근 간 과 진실 그리고 싶어( )

가 다 근에게 그것 진리 다 거 다 없

년 학 간고사 대비2013 2 현대고 대비

ECN-0102-2013-001-000076193

거 고 다 없 는 것 진리

다 근 진리는 후 쪽 었다 신산( )辛酸 삶

었 질곡 역사 에 지냈( )桎梏

가 눈에 든 것 료 단 료 게 보

것 었다 그것 그 에 겨우겨우

슬 슬 생 어가는 간들 었다

리 과 단 리 고리에 검

마 없 거리 돌

상 것 없는 등 근에게 상에

과 진실 엄 다는 사실 리는 가 실( )儼存

고 가 과 역경 에 도 근 내 포

없었 후 보루 다( ) 堡壘 도 365

도 간 근 여

시 것 다

마 같 가가 고 싶었 근에게 그 꿈( )

에 다가가는 지 다 다 가 지망생들

규 미 상 에 진 고

에 지만 근 다 다 근

미 에 운 것 보통 시 미 시간

다 그런 그에게 없는 연습 가가

통 다 가 귀 시 지 도

얻는 뛸 듯 뻤지만 마 도 (

는 었 에 어린 근 주 에)

에 그림 그리고 지우고( )粉板

복 시간 가는 게 루 보냈다

zb36) 전 의 성 소가 아닌 것을 고르

① 평 ② 사건 ③ 경

④ ⑤ 훈

늘 지 상에 살고 는 사 들 억 도가10

고 그리 지 통 고 는 사 들( )知的

그보다 훨 많 억 도는 고 지 20

통 다 그런 지 고 2500

그리 간 보는 과 사 에

매우 달 뿐만 니 과 에 도 극

루고 었다 미 운 그런 들

살고 는 동 과 사 들 사고 식에

큰 가 다는 다

고 그리 들 우주 개별 고 독립

사 들 생각 지만 고 들 우

주 연 질 간주 다 같( ) 看做

각 도 들에게는 연 질

었지만 그리 들에게는 미 들 결 었

다 고 과 그리 들 사 같

는 동 과 사 에 도 견 다

지심리 미 마 드 겐트 는

살 들에 에 지 다

연 동 과 상 다 과 같 실험

다 크 만든 미드 도 보

여 주고 그 상 닥 고 주었다lsquo (Dax)rsquo

실 닥 는 재 지 는 것 실험 가lsquo rsquo

만들어 낸 다 그런 다 개 다 체 보

여 주었는 는 미드 지만 틱

만들었고 다 는 재료는 크 지만

달 다 그러고 어 것 닥 지 사 들에게 고 lsquo rsquo

게 니 들 주 같 고 는

체 택 고 동 들 같 재료 만들어진 체

택 다 러 는 심지어 살짜리

들에게 도 타났다 것 곧 과 동

다 상 보고 다는 것 미 다

개별 사 보고 고 동 연 질 보

고 는 것 다

동 들 주변 상 에 맞 어 동 고

에 다 사 들 태도 동에 보다 많

주 울 다 동 가 미시간 에

에 경험 다 그는 미식

경 보러 가게 었는 경 체는 매우 재미 었

주변 들 동에 질 다 그 는

들 계 어 상태 경 다

어 들 에 에 그 시 가 계 가

진 것 다 상 살펴 는 말 들 lsquo rsquo

에 그는 에 시 어 도 뒷사

생각 곧 다시 곤 것 다 그런 그에게 뒷

사 고 지 는 들 동 럼

어 웠다

생각 지도 리 드 니 벳-

zb37) 다음 위 의 내 전개 으 만 인lt gt

것은

lt gt

대조의 통해 대상이 닌 특성을 설 하고 있다

일화를 제 하여 자 의 주장을 뒷 침하고 있다

유추의 을 사 하여 독자의 의해를 돕고 있다

대상이 형성되는 과정을 간적 서에 따라 서 하고 있

① ②

③ ④

년 학 간고사 대비2013 2 현대고 대비

ECN-0102-2013-001-000076193

가 우리가 말 고 쓰는 든 단어가 사 에 는( )

것 니다 사 격에 가 는 지만

어 사 과 같 특별 는 사 니lsquo rsquo

단어 격 보 단어가 사 에

등재 어 다 리 리 사 는 단어 도 그

것 시 사 는 어 고 사 에

격 보 것 니다

러 얼 은 사전에 를 있는가 이에 대한 답lsquo rsquo

은 얼 이 유행어인가 아닌가에 따라 갈라 다 이 단어lsquo rsquo

는 년 어 자 에 랐고 쓰이고 있으2002 lsquo rsquo

유행어라고 하 에는 생 이 다 런데 계속

을 유 하 서 사전에 등재될 자격을 획득할 것인가 이

에 대한 답을 내리 는 히 어 다

여 서 가 를 고 해 볼 있다 첫 는 이 단어

를 써야 할 필 가 속적으 있는가 하는 점이다

상주의 열풍에 휩 인 사회 위 에 편 해서 퍼 말

이 얼 인데 과연 런 위 가 속될 것인가 이에lsquo rsquo

대해 필자의 생각은 정적이다 사회 위 가 뀌

런 말을 쓸 일이 없어 것이다

다음은 단어의 성이다 단어의 성이 사회적으 거

감이 없으 계속 사 될 가능성이 높다 런 에서

얼 은 좋은 조건이 아니다 익히 알 졌듯이 이lsquo rsquo

말은 얼 과 청소년층에서 속어 사 하는 이 결합lsquo rsquo lsquo rsquo

된 말이다 얼 에서 얼 을 리하는 조어 도 lsquo rsquo lsquo -rsquo

어에서는 매 낯선 이다 이것만으 도 거 감을 갖

는 사람들이 있다 더 나 속어 결합한 말이다 얼 lsquo rsquo

이 널리 퍼졌다 해도 은 여전히 청소년층의 속어lsquo rsquo

남아 있다 속어는 자연 럽게 아 자리에서나 쓰 에는

담 러 말이다 러한 담을 하고 사

역을 넓혀 가는 속어도 없 는 않다 특히 얼 은 lsquo rsquo

에도 종종 등장한다 만큼 거 감이 많이 희석되었다

고 할 있다 러나 일상의 자연 러 대화에서도 거

리낌 없이 등장하는가 게 는 되 않았다고 생

각한다

얼 이 유사어인 쌈 등을 만들어 내고lsquo rsquo lsquo rsquo

있으니 살아남을 있을 것이라고 는 견해도 있을 것

이다 러나 간이 나 서 유사어를 포함하여 든

말이 사라 사 는 많다 유사어가 많다는 것이 생 을

유 할 있는 절대적인 조건은 아니다

나 언젠가 터 사람들은 어느 단에서 얼 이 가장( )

쁜 사람을 가리켜 얼 이라고 르고 있다 이 얼lsquo rsquo lsquo rsquo

이라는 단어가 최근 어사전에 라 항간에 논란이 일고

있다 아닌 게 아니라 얼 은 유행어처럼 인다 생 lsquo rsquo

도 리 래되 않은 것 같고 언제 사라 도 알

없다 게다가 젊은이들 사이에서 주 쓰일 뿐이다 이런

단어를 사전에 는다는 게 하 이 없어 이 도

한다

러나 속단은 이다 차근차근 따져 볼 일이다

선 얼 이 일 적 유행어인 아닌 주의 게 들여다lsquo rsquo

볼 필 가 있다 유행어란 유행에 따라 빠르게 유포되었

다가 단 간 내에 소 되는 단어나 를 가리킨다

얼 은 인터넷을 통해 속히 퍼 말이다 하 만 일lsquo rsquo

적인 유행어처럼 단 간 내에 사라 않았을 뿐 아니라

현재 도 잦은 빈도 사 되고 있고 앞으 도 상당

간 사 될 것으 측된다 한 언 재단의 뉴 검 lsquo rsquo

색 사이트에 따르 얼 은 년 에 처음 나타난lsquo rsquo 2001

이후 꾸 히 사 되고 있다

이 같은 사 빈도는 얼 이 일 적 유행어 는 현lsquo rsquo

저히 다르다는 것을 여 다 장 간의 생존 만으 도

얼 은 이 한 어의 어휘 에 를 자격을 얻었다lsquo rsquo

고 할 있다 더 이 이라는 비 적 정제된 매체에

높은 빈도 쓰이고 있 않은가 사 빈도 측 에서

필통이나 연필과 같은 단어 대등하거나 더 많이 쓰lsquo rsquo lsquo rsquo

다는 것은 결코 가 게 볼 일이 아니다

이제는 사전이 언어 현 을 빠르게 하는 게 덕인

대가 되었다 세계적으 유 한 의 사전들도 경쟁

적으 어를 고 있다

하 만 얼 은 젊은이들이나 쓰는 속어라고 흠을 잡을lsquo rsquo

도 르겠다 얼 이 주 젊은 층에서 많이 쓰 lsquo rsquo

는 속어임에 틀림없다 러나 어사전에 표 적이고 품

위 있는 말만 어야 한다고 생각한다 것은 커다란

해다 당장 아 어사전이나 펼쳐 라 속어는

설과 같은 비어나 죄자들이 쓰는 은어 어

마니 같은 소 의 사람만이 쓰는 말 도 라 있

않은가 사전은 말 치에 일정 빈도 이상 나타나는 말이

라 말이든 다 할 있다

zb38) 가 나 에 대한 다음의 설( ) ( ) 않은 것은

① 가 는 얼짱 사 에 등재 것에( ) ( ) lsquo rsquo

보 고 다

② 사 등재 가는 단어 격에( )

고 고 는 언 들 언어 사 도에 고 다 ( )

③ 가 얼짱 어지만 신 과 같 매( ) ( ) lsquo rsquo

체에 도 사 는 말 는 고 다

④ 가는 얼짱 어 보고 크게 가지 근( ) lsquo rsquo 3

거 들어 뒷 고 다

⑤ 는 얼짱 어 는 다 특 다는( ) lsquo rsquo

근거 에도 크게 가지 근거 가 들어 주 2

뒷 고 다

가 늘 지 상에 살고 는 사 들 억( ) 10

도가 고 그리 지 통 고 는 사 들

그보다 훨 많 억 도는 고 지 20

통 다 그런 지 고 2500

년 학 간고사 대비2013 2 현대고 대비

ECN-0102-2013-001-000076193

그리 간 보는 과 사 에

매우 달 뿐만 니 과 에 도 극

루고 었다 미 운 그런 들

살고 는 동 과 사 들 사고 식에

큰 가 다는 다

고 그리 들 우주 개별 고 독립

사 들 생각 지만 고 들 우

주 연 질 간주 다 같 각

도 들에게는 연 질 었지

만 그리 들에게는 미 들 결 었다

고 과 그리 들 사 같 는

동 과 사 에 도 견 다

인 리학자인 츠 이마이 디드 겐트너는 두

살이 채 안 된 아이들에서 터 성인에 이르 다양한

연 대의 동양인과 서양인을 대상으 다음과 같은 험

을 했다 저 코르크 만든 피라 드 양의 도형을

여 주고 대상의 이름을 닥 라고 알 주었다lsquo (Dax)rsquo

제 닥 는 존재하 않는 것으 험자가 임의lsquo rsquo

만들어 낸 이름이다 런 다음 두 개의 다른 체를

여 주었는데 하나는 피라 드 양이 만 하얀 플라 틱

으 만들었고 다른 하나는 재 는 코르크 만 양이

달랐다 러고 나서 어떤 것이 닥 인 사람들에게 고 lsquo rsquo

르게 했더니 서양인들은 주 같은 양을 하고 있는

체를 선택했고 동양인들은 같은 재 만들어 체를

선택했다 이러한 차이는 성인은 어 두 살 리

아이들에게서도 나타났다 이것은 곧 서양인과 동양인은

서 다른 세상을 고 있다는 것을 의 한다 략 ( )

는 아주 단 하 서도 인상적인 험을 했다

험에는 동서양의 대학생들이 참여했다 는 험 참가자

들에게 컴퓨터 화 을 통해 속 장 을 담은 애니 이션

을 여 주었다 화 의 앙에는 초점의 역할을 하는 커

다란 고 한 마리가 있었고 주위에는 다른 생

들과 초 자갈 거품 등이 함 제 되었다 화 을

두 씩 후 참가자들은 자 이 것을 회상해 라는

를 았다

결과 서양인 대학생들과 동양인 대학생 두 앙

의 초점 역할을 했던 고 를 동일한 정도 언 했으

나 경 소 위 거품 초 다른 생 들 에 ( )

대해서는 동양인 대학생들이 서양인 대학생들 다 60

이상 더 많이 언 했다 뿐만 아니라 동양인 학생들은 서

양인 학생들에 비해 개 적인 고 다 전체적인 계

를 더 언 하는 경향을 다 략 또한 경의 일 ( )

를 화 킨 림을 제 하 을 때 동양인 대학생들은 대

경의 화를 알아챘 만 서양인 대학생들은 경

의 화를 거의 알아차리 했다 략 ( )

따라서 서양인들만을 대상으 연 한 화lsquo

편성 결 은 잘 된 것일 도 있다 각 과정과 인rsquo

과정의 어떤 이 화 편적이고 어떤 이

화에 따라 달라 는 는 앞으 많은 연 를 통하여 논의

되어야 한다

나 어떤 의 에서 리 두는 이 화적이다 리( )

안에는 다른 사람들과 더 친 한 계를 유 하 는 상호

의존성과 다른 사람들 터 독립적인 존재 살아가 는

독립성이 혼재한다 따라서 이 에서 어떤 특성이 더 강

하게 각되는 상황에 놓이느냐에 따라 서 다른 화적

특 을 일 있다 결 리 두는 어떤 경 에는

동양인처럼 행동하고 어떤 경 에는 서양인처럼 행동하는

것이다

zb39) 가 에 대한 다음의 설( ) 않은 것은

① 는 신 주 뒷 닥 실험과lsquo rsquo lsquo

니 실험 근거 시 다rsquo

② 동 들 상 간 공통 보다는 에 식

는 강 다

③ 들 주변 맥 에는 심 경 어 사건

과 사건 사 계에 상 민감 다

④ 는 동 과 틀린 지 고 는 것lsquo rsquo

니 다 고 다 lsquo rsquo

⑤ 가에 우리 사 들 개 시 가 원( )

집 경 말 고 는 것 개 보다는

에 고 는 것에 다

늘 지 상에 살고 는 사 들 억 도가10

고 그리 지 통 고 는 사 들( )知的

그보다 훨 많 억 도는 고 지 20

통 다 그런 지 고 2500

그리 간 보는 과 사 에

매우 달 뿐만 니 과 에 도 극

루고 었다 미 운 그런 들

살고 는 동 과 사 들 사고 식에

큰 가 다는 다

지심리 미 마 드 겐트 는 동

과 상 다 과 같 실험 다

크 만든 미드 도 보여 주고 그

상 닥 고 주었다 그런 다lsquo (Dax)rsquo

개 다 체 보여 주었는 는 미드

지만 틱 만들었고 다 는 재료는

크 지만 달 다 그러고 어 것 닥 lsquo

지 사 들에게 고 게 니 들 주 같rsquo

고 는 체 택 고 동 들 같

재료 만들어진 체 택 다 러 는

심지어 살짜리 들에게 도 타났다 것

곧 과 동 다 상 보고 다는

것 미 다 개별 사 보고 고 동

년 학 간고사 대비2013 2 현대고 대비

ECN-0102-2013-001-000076193

연 질 보고 는 것 다

동 들 주변 상 에 맞 어 동 고

에 다 사 들 태도 동에 보다

많 주 울 다 동 가 미시간

에 에 경험 다 그는 미

식 경 보러 가게 었는 경 체는 매우 재

미 었 주변 들 동에 질 다 그

는 들 계 어 상태 경

다 어 들 에 에 그 시 가 계

가 진 것 다 뒷사 고 지 는 들

동 럼 어 웠다

그는 경험에 어 얻어 동 들lsquo

각도 상 본다 는 가 우고rsquo

검 여 주 단 도 상 실험 실

시 다 그는 실험 가 들에게 컴퓨 통

담 니 보여 주었다

에는 역 는 커다 고 마리가 었

고 주 에는 다 생 들과 갈 거 등

께 시 었다 본 후 가 들

신 본 것 상 보 는 지시 다

그 결과 생들과 동 생

역 고 동 도 언

경 거 다 생 들에 ( )

는 동 생들 생들보다 60

상 많 언 다 뿐만 니 동 생들

생들에 개별 고 보다 체 계

언 는 경 보 다 경 변 시

킨 그림 시 동 생들 경

변 지만 생들 경 변

거 리지 못 다

지 지 들만 상 연 lsquo

보편 결 못 것 도 다 지각 과 과rsquo

지 과 어 보편 고 어

에 달 지는지는 많 연 통 여

어 다

리 드 니 벳 생각 지도 사- ldquo rdquo( 2004)

zb40) 위 에 대한 설 으 가장 적절한 것은

① 동 과 생 식 강 고 다

② 가지 실험 통 쓴 고 다

③ 닥 실험에 사 본질에 동 사

상에 주 다

④ 니 실험에 동 과 에 지

각 도에 가 다

⑤ 쓴 는 보편 연 에 드러 우월 에

에 근 고 다

가 동 들 주변 상 에 맞 어 동 고( )

에 다 사 들 태도 동에 보다 많

주 울 다 동 가 미시간 에

에 경험 다 그는 미식

경 보러 가게 었는 경 체는 매우 재미 었

주변 들 동에 질 다 그 는

들 계 어 상태 경 다

어 들 에 에 그 시 가 계 가

진 것 다 상 살펴lsquo 는 말 들rsquo

에 그는 에 시 어 도 뒷사

생각 곧 다시 곤 것 다 그런 그에게

뒷사 고 지 는 들 동 럼

어 웠다

그는 경험에 어 얻어( ) 동 들lsquo

각도 상 본다 는 가 우고rsquo

검 여 주 단 도 상 실험

실시 다 실험에는 동 생들 여 다

그는 실험 가 들에게 컴퓨 통

담 니 보여 주었다 에는

역 는 커다 고 마리가 었고 주 에는

다 생 들과 갈 거 등 께 시

었다 본 후 가 들 신 본 것

상 보 는 지시 다

다 그 결과 생들과 동 생( )

역 고 동 도 언

경 거 다 생 들 에 ( )

는 동 생들 생들보다 60

상 많 언 다 뿐만 니 동 생들

생들에 개별 고 보다 체 계

언 는 경 보 다 들어 동

생들 상 체 연못 럼 보 어ldquo 같rdquo

체 맥 언 시 었지만

생들 상 어 같 큰 고 가 쪽 움ldquo

직 어 같 역 고rdquo

언 시 다 경 변 시킨 그

림 시 동 생들 경 변

지만 생들 경 변 거

리지 못 다

년 학 간고사 대비2013 2 현대고 대비

ECN-0102-2013-001-000076193

게 볼 동 들 보다는 큰 그( )

림 보 에 사 과 체 맥 연결시 지각

는 경 고 체에 특 떼어 내

어 독립 보는 것 낯 어 다 에

들 사 에 고 주변 맥 에는 심 경

에 사건과 사건 사 계에 상

민감 편 다

마 지 지( ) 들만 상 연

보편 결 못 것 도 다lsquo rsquo 지각 과

과 지 과 어 보편 고 어

에 달 지는지는 많 연 통 여

어 다

리 드 니 벳 생각 지도 사- ldquo rdquo( 2004)

zb41) 의 하는 가~ 다른 것은

① ② ③

④ ⑤

얼마 그 에 동 사고 식과

사고 식 보여 주는 내 다

들 에 는 탕 고 같 게

어 겨 고 미 에 는 그 크 럼 큰 고

어리 주고 원 는 어 도 는

상 고 생각 다는 것 다 러

는 어떻게 생 것 고 과 그리 거슬

러 가 보 그 단 다

고 연 경 체 경 생 에

다 벼 사는 공동 업과 경험 많 연 역

에 고 들 연 웃과

게 지내 고 탁 연 들

들 지 연 럽게 들 다 민들

웃과 동 게 뿐만 니 는 집 과

게 다

동 시 는 생태 경 에 살 결과

들 다 사 들 사 상 에 주

울 게 었고 는 곧 체 상 과 간 사

계 시 는 낳게 었다 신 가

가 는 체에 는 원 는 동시

에 다 사 들 그 사 포 체 맥 에

다 들 간 사 연

계 체 계에 주 울 는 사고 체계

게 었다

그러 그리 연 경 그 었다 산

지 연결 는 지 건 그리고 역

에 다 런 들 업에 다 사 과

동 므 공동체에

다고 다 고 그리 들

들과는 달리 보 내 감 지 들과

지 크게 느 지 못 다 그

견 다 경우 주 쟁 통 결 는 갖

게 었다

신 사 간 계들 루어진 커다

트워크 에 게 당연 사 역시 연

계들 체 식 게 다 어 상

원 도 그 개체가 체 맥 과

계 에 고 다 게 체 맥 에 주

울 다 보 상 복 과 가변 식 게 고

상에 재 는 많 변 들 사 에 재 는 들도

게 다 들 주 태도 보

는 경우가 많다 쟁 결

통 결 보다는 통 결

는 보 다

그러 고 그리 들 개개 사 사 독

에 주 울 다 사 사 체에

어 그들 사 에 재 는 공통 규 주

고 다 상 원 에도 사

체 내 주 고 다 그들

체 여 탕 체

는 주 태도 시 고 특 사 어

주에 는지 여 그 주에 는 규

견 다 에 는 쟁 식 리

같 리 사고 체계가 달 게 었다

리 드 니 벳 생각 지도 사- ldquo rdquo( 2004)

zb42) 위 에서 사 된 설 과 가장 유사한 것은

① 크톱 컴퓨 는 본체 니 마우 루

어 다

② 곡과 시 리 는 지 과 사 루어 다는 공통

지니고 다

③ 경 고 것과는 달리

경 본 연 태 그 주변 경

④ 벽돌 능 에 사계 내내

습도가 지 다

⑤ 잰느 체 체 지닌 재 체가 없

는 재 눌 다

년 학 간고사 대비2013 2 현대고 대비

ECN-0102-2013-001-000076193

zb43) 는 립 앙 도서 이 정의 일 이다lt gt

도서 장과 이 자의 리 의 정의 연결이

적절하 않은 것은

lt gt

제 조 서 유8 ( )

도서 장은 다른 이 자의 안전을 위협하거나 도서 의①

서를 란하게 할 가 있는 자에 대하여는 도서 출입

을 제한할 있다

도서 장은 이 자가 제 조 각 호의 어느 하나의 행위를 하7②

을 때에는 이 을 하게 하거나 도서 출입을 제한할

있다

제 조자 의 대출9 ( )

도서 자 는 다음 각 호의 경 대출할 있다①

상호대차도서 간에 자 를 류하는 것을 말한다 등 다1 ( )

른 도서 과의 협 을 위하여 필 한 경

공 이 공 행 상 필 하는 경2

에 도서 장이 필 하다고 인정하는 경3

대출이 가능한 도서 자 의 위는 도서 장이 정하는②

에 따른다

제 조 상10 ( )

이 자가 도서 자 설을 더럽히거나 찢거나 뜨①

쓰게 하거나 잃어 린 경 에는 상하여야 한다

도서 장은 제 항에 따른 상 을 정하여 게 하여야1②

한다

제 조이 절차 등11 ( )

이 칙에서 정한 것 에 도서 자 설의 이 절차

이 제한 등에 필 한 사항은 도서 장이 정한다

출처 립 앙 도서- (httpwwwnlgokr)

① 는 도 리 다8

② 도 는 리 다9 1

③ 료 지 는 도 리 다9 2

④ 도 료 변상에 리10 1

⑤ 는 에 도 리 다11

3

도 다 각 같다①

공 공 다만 연1

연 간 다

매월 째 째 월2

도 도 리 그 사3

가 다고 는

도 에 미리 게1 3②

시 여 다

4

도 시간 도 여 게시 다

5

도 료 시 는 는 도①

지에 등 후

등 에 사 도②

7

는 다 각 여 는 니 다

도 료 시 상 리1 lsquo rsquo

도 료 시 훼 는2 middot

지 가 닌 곳에 식 거 담3

우는

도 보 등 보 검색열4 middot

그 에 도 질 지 여 도5

여 게시 사 는

8

도 다 거 도①

질 게 우 가 는 에 여는 도

도 가 각 어느7②

에는 지 게 거 도

9

도 료는 다 각 경우 다①

상 도 간에 료 는 것 말1 (

다 등 다 도 과 여 경우)

공 원 공 상 는 경우2

그 에 도 다고 는 경우3

가능 도 료 는 도②

는 에 다

10

년 학 간고사 대비2013 2 현대고 대비

ECN-0102-2013-001-000076193

가 도 료 시 럽 거 거①

못 쓰게 거 어 린 경우에는 변상 여

도 에 변상 여 게시1②

여 다

zb44) 위 에서 도서 장이 게 해야 할 사항에 해당하는

것을 두 쓰

년 학 간고사 대비2013 2 현대고 대비

ECN-0102-2013-001-000076193

립 도 규

1 ( )

규 립 도 립 어린 청 도(

포 다 료 시 열 시 말) (

다 에 사 규 립 도)

편 진 다

2 ( )

규 립 도 도 다 에( lsquo rsquo )

고 는 도 에 도lsquo rsquo 2 2

료 에 여 다 다만 특 료 귀

료 등 료 에 사 립 도

도 다 다( lsquo rsquo )

3 ( )

도 다 각 같다①

공 공 다만 연1

연 간 다

매월 째 째 월2

도 도 리 그 사3

가 다고 는

도 에 미리 게1 3②

시 여 다

시간4 ( )

도 시간 도 여 게시 다

등 등5 ( )

도 료 시 는 는 도①

지에 등 후

등 에 사 도②

사 료6 ( )

도 료 시 에 사 료는 도

7 ( )

는 다 각 여 는 니 다

도 료 시 상 리1 lsquo rsquo

도 료 시 훼 는2 middot

지 가 닌 곳에 식 거 담3

우는

도 보 등 보 검색열4 middot

그 에 도 질 지 여 도5

여 게시 사 는

질 지8 ( )

도 다 거 도①

질 게 우 가 는 에 여는 도

도 가 각 어느7②

에는 지 게 거 도

료9 ( )

도 료는 다 각 경우 다①

상 도 간에 료 는 것 말1 (

다 등 다 도 과 여 경우)

공 원 공 상 는 경우2

그 에 도 다고 는 경우3

가능 도 료 는 도②

는 에 다

변상10 ( )

가 도 료 시 럽 거 거①

못 쓰게 거 어 린 경우에는 변상 여

도 에 변상 여 게시1②

여 다

등 규 에 것 에 도11 ( )

료 시 등에 사

도 다

립 도- (httpwwwnlgokr)

zb45) 도서 장의 리 있는 조항으 적절하 않

은 것은

① ② ③ ④ ⑤

년 학 간고사 대비2013 2 현대고 대비

ECN-0102-2013-001-000076193

1 ( )

사가 공 는lsquo rsquo

과 여 사 원과 리

사 타 사 규

니다

개 보 보7 ( )

사는 보통신망 등 계 는 에lsquo rsquo lsquo rsquo

원 개 보 보 니다 개lsquo rsquo

보 보 사 에 는 사 개lsquo rsquo

보 취 니다 다만 사는 다 lsquo rsquo

사 계 통 공 는 경우 원 lsquo rsquo

등 개 보 당 사에 습니lsquo rsquo

원 리에8 (lsquo rsquo lsquo rsquo lsquo rsquo

)

원 에 리lsquo rsquo lsquo rsquo lsquo rsquo①

원에게 가 도 여 는lsquo rsquo 3

니다

사는 원 가 개 보 우 가lsquo rsquo lsquo rsquo lsquo rsquo②

거 사 경우 는 미 에 어 거 lsquo

사 사 운 우 가 는 경우 당rsquo lsquo rsquo

습니다lsquo rsquo

원 가 도 거lsquo rsquo lsquo rsquo lsquo rsquo 3③

가 사 고 지 경우에는 시 사에lsquo rsquo

통지 고 사 내에 니다lsquo rsquo

경우에 당 원 사에 그 사실3 lsquo rsquo lsquo rsquo④

통지 지 거 통지 도 사 내에 지 lsquo rsquo

생 경우 사는 지지 습니다lsquo rsquo

사10 (lsquo rsquo )

사는 과 지 미lsquo rsquo①

에 는 지 계 고

공 여 다 여 니다lsquo rsquo

사는 원 게lsquo rsquo lsquo rsquo lsquo rsquo②

도 개 보 신 보 포 보 보 시( )

갖 어 개 보 취 공시 고

니다

사는 과 여 원lsquo rsquo lsquo rsquo③

견 만 당 다고 경우에는

리 여 니다 원 견 만 사 lsquo rsquo

에 는 게시 거 우편 등 통 여

원에게 리 과 결과 달 니다lsquo rsquo

원11 (lsquo rsquo )

원 다 여 는 니다lsquo rsquo ①

신청 는 변경 시 허 내 등1

타 보 도2

사가 게시 보 변경3 lsquo rsquo

사가 보 보 컴퓨 그4 lsquo rsquo (

등 등 신 는 게시)

사 타 등 지 재산 에5 lsquo rsquo 3

사 타 상 거 업6 lsquo rsquo 3

는 폭 시지 상 타 공7 middot middot

에 는 보 에 공개 는 게시 는lsquo rsquo

사 동 없 리 사8 lsquo rsquo

타 거 당9

게시15 (lsquo rsquo )

원 내에 게시 는 게시 게재 는lsquo rsquo lsquo rsquo lsquo rsquo

경우 원 사가 게시 복 lsquo rsquo lsquo rsquo lsquo rsquo middot middot

등 태 언 등에 공 는

것 내에 다 원 본 게시 등 lsquo rsquo lsquo rsquo

크 능 등 여 복 는 등 태

는 것 동 것 니다

- (wwwnavercom)

zb46) 위 은 인터넷 포털사이트의 회 가입을 위한 이

약 의 일 이다 이 약 을 만드는 과정에서 생각한

내 으 적절하 않은 것은

개 보 보 가 지에 별 눠①

겠어

원 가 만들게 에②

시 주어 겠어

원들 게재 게시 다 원 크 다③

는 것 지

④ 원 지 는 뿐만 니 사가 지 는

도 께 달 지

리에 가 생 경우 사가⑤

에 다는 도 듯

1 ( )

사가 공 는lsquo rsquo

과 여 사 원과 리

사 타 사 규

년 학 간고사 대비2013 2 현대고 대비

ECN-0102-2013-001-000076193

니다

개 보 보7 ( )

사는 보통신망 등 계 는 에lsquo rsquo lsquo rsquo

원 개 보 보 니다 개lsquo rsquo

보 보 사 에 는 사 개lsquo rsquo

보 취 니다 다만 사는 다 lsquo rsquo

사 계 통 공 는 경우 원 lsquo rsquo

등 개 보 당 사에 습니lsquo rsquo

원 리에8 (lsquo rsquo lsquo rsquo lsquo rsquo

)

원 에 리lsquo rsquo lsquo rsquo lsquo rsquo①

원에게 가 도 여 는lsquo rsquo 3

니다

사는 원 가 개 보 우 가lsquo rsquo lsquo rsquo lsquo rsquo②

거 사 경우 는 미 에 어 거 lsquo

사 사 운 우 가 는 경우 당rsquo lsquo rsquo

습니다lsquo rsquo

원 가 도 거lsquo rsquo lsquo rsquo lsquo rsquo 3③

가 사 고 지 경우에는 시 사에lsquo rsquo

통지 고 사 내에 니다lsquo rsquo

경우에 당 원 사에 그 사실3 lsquo rsquo lsquo rsquo④

통지 지 거 통지 도 사 내에 지 lsquo rsquo

생 경우 사는 지지 습니다lsquo rsquo

원에 통지9 (lsquo rsquo )

사는 특 다 원에게 통지 경우lsquo rsquo lsquo rsquo

공지 게시 통 상 게시 개별 통지에7

갈 습니다

사10 (lsquo rsquo )

사는 과 지 미lsquo rsquo①

에 는 지 계 고

공 여 다 여 니다lsquo rsquo

사는 원 게lsquo rsquo lsquo rsquo lsquo rsquo②

도 개 보 신 보 포 보 보 시( )

갖 어 개 보 취 공시 고

니다

사는 과 여 원lsquo rsquo lsquo rsquo③

견 만 당 다고 경우에는

리 여 니다 원 견 만 사 lsquo rsquo

에 는 게시 거 우편 등 통 여

원에게 리 과 결과 달 니다lsquo rsquo

원11 (lsquo rsquo )

원 다 여 는 니다lsquo rsquo ①

신청 는 변경 시 허 내 등1

타 보 도2

사가 게시 보 변경3 lsquo rsquo

사가 보 보 컴퓨 그4 lsquo rsquo (

등 등 신 는 게시)

사 타 등 지 재산 에5 lsquo rsquo 3

사 타 상 거 업6 lsquo rsquo 3

는 폭 시지 상 타 공7 middot middot

에 는 보 에 공개 는 게시 는lsquo rsquo

사 동 없 리 사8 lsquo rsquo

타 거 당9

원 계 규 내lsquo rsquo lsquo②

여 공지 주 사 사가 통지 는rsquo lsquo rsquo

사 등 여 타 사 업 에 lsquo rsquo

는 여 는 니다

- (wwwnavercom)

zb47) 위 약 의 조항에서 같은 제점을 하lt gt

고 있는 조항은

lt gt

제휴 회사에 회 의 아이디 개인 정 를 전송할 있도

한 조항은 고객에게 당한 조항이다

1 7 8① ② ③

④ 9 ⑤ 10

립 도 규

1 ( )

규 립 도 립 어린 청 도(

포 다 료 시 열 시 말) (

다 에 사 규 립 도)

편 진 다

2 ( )

규 립 도 도 다 에( lsquo rsquo )

고 는 도 에 도lsquo rsquo 2 2

료 에 여 다 다만 특 료 귀

료 등 료 에 사 립 도

도 다 다( lsquo rsquo )

3 ( )

도 다 각 같다①

공 공 다만 연1

연 간 다

년 학 간고사 대비2013 2 현대고 대비

ECN-0102-2013-001-000076193

매월 째 째 월2

도 도 리 그 사3

가 다고 는

도 에 미리 게1 3②

시 여 다

시간4 ( )

도 시간 도 여 게시 다

등 등5 ( )

도 료 시 는 는 도①

지에 등 후

등 에 사 도②

사 료6 ( )

도 료 시 에 사 료는 도

7 ( )

는 다 각 여 는 니 다

도 료 시 상 리1 lsquo rsquo

도 료 시 훼 는2 middot

지 가 닌 곳에 식 거 담3

우는

도 보 등 보 검색열4 middot

그 에 도 질 지 여 도5

여 게시 사 는

질 지8 ( )

도 다 거 도①

질 게 우 가 는 에 여는 도

도 가 각 어느7②

에는 지 게 거 도

료9 ( )

도 료는 다 각 경우 다①

상 도 간에 료 는 것 말1 (

다 등 다 도 과 여 경우)

공 원 공 상 는 경우2

그 에 도 다고 는 경우3

가능 도 료 는 도②

는 에 다

변상10 ( )

가 도 료 시 럽 거 거①

못 쓰게 거 어 린 경우에는 변상 여

도 에 변상 여 게시1②

여 다

등 규 에 것 에 도11 ( )

료 시 등에 사

도 다

립 도- (httpwwwnlgokr)

zb48) 다음 정 리 의 의 으 볼 때 가장

이 적인 것은

도 시간 도 여 게시 다①

등 에 사 도②

가능 도 료 는 도 는③

에 다

④ 도 에 변상 여 게10 1

시 여 다

⑤ 도 가 각 어느7

에는 지 거 도

zb49) 를 참고하여 이 어의 성격을 설 한lt gt

것으 적절하 않은 것은

① 보 에 는 어 시 상 고 어 시lt gt lsquo rsquo

에 보여주고 다

② 진 어 어원에 견 고 다

에는 타 어 들어가는 것 다 lsquo rsquo

③ 에 들어갈 말 각각 고 어 어 신 어~

들 언어는 질 격 강 통 없었다

④ 시 우리 에 가 었지만 지 계

과 달리 들 통 사 달 어 웠

년 학 간고사 대비2013 2 현대고 대비

ECN-0102-2013-001-000076193

⑤ 크 몽골 만주 공통어가 우리 어 같

계열에 다는 에 사 특 짐

가( )

善化公主主隱 공주님

他密只嫁良置古 몰 결 고

薯童房乙 맛

夜矣卯乙抱遣去如 에 몰 고 가다

( )

始汝 會隱日恚見隱扐 만 에 본

恥隱汝衣淸隱笑 맑 웃

고 시 여 공 크다 만 다[ ] ( ) ( ) ( ) ( )始 汝 會扐

내다 에 보다 견( ) ( )恚 見 다( )隱

럽다 맑다 청 웃( ) ( ) ( ) ( )恥 衣 淸 笑

zb50) 위의 나 를 함 고 음에 답하( ) lt gt

보lt gt

( )素那或云金川 白城郡蛇山人也

운 사산

는 고 다 는( )[ ( ) ] (素那 金川 白城

사산 사 다) ( ) 郡 蛇山

삼 사- lsquo rsquo 47

에 제 된 단어 의 표 리를 조건(1) lt gt ( ) lt gt

에 맞게 서 하

건lt gt

lsquo 었고 었다 태rsquo

에 제 된 단어 동일한 표 리에(2) lt gt ( )

의해 적은 것을 나 에서 찾아 조건 에 맞게 서 하( ) lt gt

건lt gt

에 당 는 각각( ) 개 쓸 것2 단

당 는 가 여러 개 어도 개만 쓸 것 각2

개 과 도 쪽에 개만2 2

드시 지 것( )

과 동 원리 것lsquo 고

과 동 원리 것 다rsquo

태 것

가( )

素那(或云金川) 白城郡蛇山人也

소나 또는 천 이라 한다 는 성 사( ) ( ) ( )素那 金川 白城郡〔 〕

산 사람이다 현대어 풀이( ) ( )蛇山

나( )

紫布岩乎希 회

執音乎手母牛放敎遣 자 손 암쇼 노히 고

吾 不喩慙 伊賜等肹 肹 나 안디 리샤

花 折叱肹 可獻乎理音如 고 것거 도림다

다 향찰은 리말을 리 으 적은 표 이었 만 생( )

은 고 대를 넘 하고 끊어 고 말았다 랜 세

동안 갈고 닦아 체계적이었던 향찰 표 이 사라졌

을 인은 크게 두 가 나누어 생각해 볼 있다

하나는 족 사회의 한 선호도에서 찾을 있다 라 때

향찰은 주 족 계 에서 사 했을 것으 인다 한 을

알 하고서는 한자를 활 하여 리말을 리 으 표

하 란 가능하 때 이다 런데 족들은 간이 흐

를 향찰과 같은 리 표 을 익혀 사 하 다는

아 한 을 대 사 하는 쪽을 선호하게 되었다 더 이

고 초에 인재 등 을 위해 과거제도가 행되 서 한 선

호도가 더 높아졌고 결 향찰은 소 되고 말았다

또 다른 가능성은 한 어의 특성에서 찾을 있다

터 한 과 일 세 나라는 한자 화 에 속해 다

당연한 이야 겠 만 표의 자인 한자는 어를 표 하

에 매 적절하다 어의 음절은 성 ( ) ( )聲母 韻母

이 어 고 여 에 성조가 추가되어 최종 소리가 결정된

다 래서 어는 단음절을 하나의 한자 표 하 된

다 에 초성 성 종성의 세 가 소가 하나의 음절

년 학 간고사 대비2013 2 현대고 대비

ECN-0102-2013-001-000076193

을 이 는 한 어는 음절 조가 잡하고 음절의 가 많아

서 한자 차 만으 한 어의 소리를 만족 럽게 표 할

없었다 를 들어 한 어에서는 어 니 같이 음절 lsquo rsquo

이 어 단어가 얼마든 있으나 어는( ) 複數音節

자 하나 나타내 만이다lsquo [m ]rsquo 母 ǔ

한편 일 어의 표 은 핵 적 단어는 한자 적고 토는

가나라는 일 의 자 적는 이다 적인 의 를 나

타내는 은 표의 자인 한자 적고 적 계를 나

타내는 토는 표음 자 적는 셈이니 자세히 살펴

리의 향찰 표 을 쏙 빼닮았음을 알 있다 한 어 같

은 착어이 서도 일 어에만 향찰과 유사한 표 이 살아

남은 것은 일 어의 특 때 이다 일 어는 하나의 자음과

음의 결합으 음절을 이 고 침이 거의 없는 음절 언어

이다 이러한 음절의 특색에다가 토가 달한 착어라는 점

이 향찰과 유사한 표 이 살아남을 있는 비결이었다

하 만 같은 착어라도 다양한 음소 침이 달한 한

어는 향찰 표 하는 데 근 적으 한계가 있었다

zb51) 다 하여 의 행에 대한 탐 한 결과( ) lt gt 2

않은 것은

보lt gt

善花公主主隱 공주니믄 공주님( )

----------------------------------------

-

他密只嫁良置古 그 지 얼어 고 몰 결(

----------------------------------------

-

薯童房乙 맛 맛( )

夜矣卯乙抱遺去如 몰 고 가다 에 몰 고(

가다)

주동 역 동- (薯童謠『 』

에 2 ( )他密只嫁良置古

얼다 시집가다 결 다 말 lsquo rsquo

① 실질 미 지니고 므 타 타lsquo ( )rsquo lsquo [ ]

② 에 실질 미 타내고 지 는lsquo rsquo lsquo [ ]rsquo lsquo [ ]密只 密 只

계 타내는

③ 얼어는 실질 미 포 고 므 가lsquo rsquo lsquo [ ]rsquo嫁

것lsquo [ ]rsquo 良

④ 고 어간 는 실질 미 지니고 므lsquo rsquo lsquo -rsquo

것lsquo [ ]rsquo 置

⑤ 고 어미 고는 계 타내고 므lsquo rsquo lsquo- rsquo

고 것lsquo [ ]rsquo 古

가( )

엉 훈 민middot middot middot middot middot世 宗 御 製 訓 民 正 音

말 미 듕 귁에 달middot middot middot middot middot middot middot middot中 國 文 字

니 런middot middot middot middot middot middot 어린middot middot middot middot百 姓

니 고 도 내 들middot middot middot middot middot middot middot middot middot 시러middot

펴 몯middot 미middot middot 니 내middot middot middot middot middot middot middot middot 爲

어엿middot 겨 새middot middot middot 믈여듧middot middot middot middot字 니middot middot middot

사 마다 니겨 킈 middot middot middot middot middot middot middot middot middot便 安

고 미니middot middot middot middot

본 는 상( ) (象

원리에 만들어진 본) ( )形 ㄱ ㄴ ㅁ ㅅ ㅇ

에 는 가 원리에( )加劃

그리고( )ㅋ ㄷ ㅌ ㅂ ㅍ ㅈ ㅊ ㆆ ㅎ

쓰는 병 원리에 만들어진( )竝書

마지막 체( ) ( )異體ㄲ ㄸ ㅃ ㅆ ㅉ ㆅ

ᅀ 다 상 원리에 ㅇ ㄹ

지 는 삼재 상 본 본( ) ( ) ( 天地人 三才

탕 므림과 림에 ) (初ㅡ ㅣ

재)( ) ( )( )出字 再出字ㅗ ㅏ ㅜ ㅓ ㅛ ㅑ ㅜ ㅕ

병 그리고 들 에 다시( )ㅘ ㅝ ㅣ

( )ㅣ ㅢ ㅚ ㅐ ㅟ ㅔ ㆉ ㅒ ㆌ ㅖ ㅙ ㅞ

zb52) 가 에 대한 설 으 르 않은 것을( ) 두 고르

① 어쓰 규 지키고 다

② 리 고 다

③ 말 미 미 등 어 사 다lsquo rsquo

④ 개 지 다

년 학 간고사 대비2013 2 현대고 대비

ECN-0102-2013-001-000076193

⑤ 어 원 에 가 도 고 다

엉 훈 민世 宗 御 製 訓 民 正 音

말 미 듕귁에 달 니

런 어린 니 고 도middot

내 들 시러 펴 몯 미 니middot

내 어엿 겨 새 믈여듧

사 마다 니겨middot 킈 고

미니

훈민 언 본- lsquo rsquo 5 (1459 )

zb53) 위의 에 대한 현대어 풀이가 르~ 않은 것

① 우리 말 과 달

② 어리 말 고 는 것 어도

③ 신 생각 마 껏 펼 는 사 많다

④ 게 생각 여

⑤ 사 마다 게

zb54) 훈민정음 언해 에는 한 을 창제한 동 가 드러나

있다 훈민정음 창제의 정 과 내 이 잘 연결된 것

① 주 신 말 미 듕귁에 달

② 민 신 내 어 겨

③ 신 뻔 킈 고 미니

④ 실 신 사 마다 니겨

⑤ 귀 신 계 주 는 훈민 신과 거리가

가 엉 훈 민( ) middot middot middot middot middot世 宗 御 製 訓 民 正 音 

말 미 귁에 中 國 달 文 字

니 런 어린 니 百 姓

고 도 내 들 시러 펴 몯

미 니 내 어엿 爲 겨 새

믈여듧 니 사 마다 니 字

겨 킈 고 미니 便 安

훈민 언 본- lsquo ( )rsquo ( ) 5 (1459 )訓民正音 世祖

( )

[ 1 ]

동 룡 샤 마다 복( ) ( ) ( )海東 六龍 天福

시니 고 동( ) ( )古聖 同符 시니

[ 2 ]

매 니 곶 여

미 므 니 그 내 러

가 니

[ 125 ]

우 미리( )千世 샨( )定 에( )漢水北 累仁

누 개 샤 복 업 시니( ) ( ) 開國 卜年

신( )聖神 니 샤도 경 근민 샤 욱( )敬天勤民

드시리 다

님 쇼 산 가( ) ( )洛水 山行

미드니 가

어 가- lsquo ( )rsquo 27龍飛御天歌

다 우리신 니쓰고 다만 만 쓰( )

거 샹 귀쳔 다보게 러 귀

여 쓴 도 신 보 가 고 신 에

말 어 보게 각 에 사 들

고 본 몬 능통 후에

죠 죠 니

드 도 만 공 에 사

드 미 죠 고 고 여 보 죠

보다 얼마가 거시 어신고 니 첫

가 죠 니 죠

민 들 어 신 샹

귀쳔 도보고 어보 가 만 늘

고 폐 에 만쓴 죠 민

도 러보지못 고 보니 그게 엇지

심 니 리 보 가 어 운건 다

니 쳣 말마 지 니 고 그

쓰 에 가 우 지 지

몰 거 본후에 가 어 지

고 그니 쓴편지 쟝 보

년 학 간고사 대비2013 2 현대고 대비

ECN-0102-2013-001-000076193

쓴것보다 듸 보고 그 마 니 쓴 고

어 못

그런고 에 리 과 가

만 쓴 못 민 말만 듯고

고 편 그 못 보니 그사 단

병신 못 다고 그사 식 사

니 만 고 다 과 그사

만 고 다 과 업 사 보다 식 고

죠 도 고 각 과

견 고 실 직 귀쳔 간에 그

고도 다 것 몰 귀죡 보다

사 우리 신 귀쳔 다 업

시 신 보고 과 지 게 랴

시니 샹 귀쳔 간에 우리 신 걸

간 보 새지각과 새 걸 미리

독립신- lsquo (1896)rsquo

zb55) 친 어 나의 제 장( ) 2 매 함축적

의 가 가장 유사한 것은

① 지 눈 내리고 매 득 니 내 여 가

사- lsquo rsquo

② 도 어 리듯 그 게 어 다

주 사- lsquo rsquo

③ 눈 살 다 죽 어 린 과 체 여

눈 새벽 지 도 살 다

눈- lsquo rsquo

④ 삶 근심과 고단 에 돌 거니는 여 거 는

여 리 내린 살가지 에 눈 리 눈 리

택 그 생 에- lsquo rsquo

⑤ 늘 러 고 러

청룡 룡 어 개 루 우

신경림 계- lsquo rsquo

zb56) 친 를 위 가 나 에 나타난A B ( ) ( )

세 어의 특 에 의거하여 세 어 표 하

그 산 고 공 도 맑지만

A

주변에 쓰 리는 어리 사 많다

B

건lt gt

식 가 에 타 어 특징에( ) ( )

거 과 어쓰 는 고 지 말 것

A

B

zb57) 가 의( ) 달 아ㆍ 다 의 ( ) 나셔에서 알 있는

세 어 개화 어의 특 을 비 하여 조건 에lt gt

맞게 서 하

건lt gt

어에 는lsquo 개

어에 는 다 태rsquo

zb58) 은 가 는 다 에 나 는 절lt 1gt ( ) lt 2gt ( )

일 를 췌한 것이다 의 의 가 lt 1gt (1)~(2)

유사한 말을 에서 찾아 쓰lt 2gt

보lt 1gt

런 (1) 어린 니 고百 姓

도 내 들 시러 펴 몯 미

사 마다 (2) 니겨 便 安

킈 고 미니

보lt 2gt

죠 고 고 여 보 죠

보다 얼마가 거시 어신고 니 첫 가

죠 니 죠 민

들 어 신 샹 귀쳔

도보고 어보 가 만 늘 고

폐 에 만쓴 죠 민 도

러보지못 고 보니 그게 엇지 심

니 리

년 학 간고사 대비2013 2 현대고 대비

ECN-0102-2013-001-000076193

lt 1 gt

동 룡 샤 마다 복 시( ) ( ) ( )海東 六龍 天福

고 동 시니( ) ( )古聖 同符

lt 2 gt

(A) 매 니 곶

여 니

미 므 니 그 내

러 가 니

lt125 gt

우 미리 샨 에( ) ( ) ( ) 千世 定 漢水北 累

누 개 샤 복 업 시 니( ) ( ) 仁開國 卜年 聖

신( ) 神 니 샤도 경 근민 샤( ) 敬天勤民

욱 드 시 리 다

님 쇼 산 가 ( ) ( )洛水 山行

미드니 가

- lt gt龍飛御天歌

zb59) 장과 내 상 유사한 성격의 조는125

① 뫼 고 고 고 고

어 그린 많고 많고 고 고

어 러 는 울고 울고 가느니

도 견- lt gt

② 강 에 드니 몸 다

그믈 고 가니

뒷 뫼 엄 언 니( )藥

-

③ 말 없는 청산 태 없는 다

값 없는 청 없는 월

에 병 없는 몸 별 없 늙 리

-

④ 가마귀 골에 가지 마

낸 가마귀 새

청강에 것 시 몸 러 가( ) 淸江

-

⑤ 진 골에( ) 白雪

가 매 는 어느 곳에 었는고

에 갈 곳 몰( ) 夕陽

색-

zb60) 위 에 나타난 세 어의 특 으 적절하 않은

것은

① 룡 어 주격 사에 당 는 가 사( ) lsquo rsquo六龍

고 다

② 샤 어에도 어 주체 쓰 다

는 것 다

③ 매 어 달리 사 택에 어

가 지 지지 고 다

④ 므 원 상 직 어 지 다

⑤ 드시리 다 주체 과 상 께 사

고 다

수고 하셨습니다hearts hearts

년 학 간고사 대비2013 2 현대고 대비

ECN-0102-2013-001-000076193

보닷컴에 공 는 별 보는 고등

들 여 주 는

들 습니다 슷 동 지

가 복 는 것 도가

니 복 여 습 시고 거 시

니다

정답 해설

1) 정답[ ] ④

해설 다른 것은 두 특정 업이나 단 내에서 사[ ]

하는 일종의 은어 사회 언에 해당한다 러나

는 언이 아니라 단과대학을 여서 단대 사lsquo rsquo lsquo rsquo lsquo④

대학을 여서 사대라고 한 말에 해당하 일rsquo lsquo rsquo

사회에서도 널리 쓰이 사회 언이라 할

없다

2) 정답[ ] ⑤

해설 사회 언은 같은 단 내에서 쓰이는 언어이[ ] lsquo rsquo

동일 단끼리는 단결 과 친 감을 형성하는

능을 하 리적 안감이 일어나 않는다

3) 정답[ ] ③

해설 사람이라는 차 적 표현에 대한 대안적 표현이[ ]lsquo rsquo

인 아내 처 등으 볼 있다lsquo rsquo

4) 정답[ ]⑤

해설 남성은 주 격 체를 사 한다[ ]

5) 정답[ ] ⑤

해설 흑인은 검다라는 뜻을 가 고 있을 뿐 인[ ]lsquo rsquo lsquo rsquo lsquo rsquo

다 열등한 뜻을 내포하 않는다

6) 정답 살 색 첫 작품[ ] - -

해설 살색 혹은 킨색은 한 인의 피 색을 뜻[ ] lsquo rsquo lsquo rsquo

하는 것으 인종 차 을 추 고 출 이주민

의 평등 을 침해할 있어 년 표 이2005

살 색으 이름을 꾸었다 처녀작은 처녀라lsquo rsquo lsquo rsquo lsquo rsquo

는 단어가 가 고 있는 곡된 성 인 을 한 것

으 첫 작품정도 꾸어 사 하는 것이 좋다lsquo rsquo

7) 정답[ ] ⑤

해설 호는 아들에게 해체를 사 하고 있다[ ] ① ②

장 을 성하는 청자는 자 의 아 느리 아lsquo

들 세 이다 호는 아 느리에게 해rsquo ③

체를 사 하고 있다 호가 느리 아 에게 ④

사 한 해 체 아들에게 사 한 해체는 두 비lsquo rsquo lsquo rsquo

격 체에 해당한다 호는 자 의 아랫사람인 ⑤

느리에게 아들과 마찬가 해체를 사 하는 것이

상 이 만 임 을 한 느리에게 고마 과 쁨

존 의 표 를 하 위해 자 의 아 에게 말하듯

해 체를 사 하고 있다

8) 정답[ ] ③

9) 정답[ ] ⑤

10) 정답[ ] ①

해설 청자 할아 가 장의 주체 아 다 높을[ ] ( ) ( )

경 에는 압존 에 의해 장의 주체를 높이 않는lsquo rsquo

다 러 아 서가 아닌 아 는으 계 lsquo rsquo lsquo rsquo lsquo

니다 가 아닌 있 니다 표현하는 것이 르rsquo lsquo rsquo

11) 정답 당이 당을 쫒았다 당이[ ]

당에 다

해설[ ]

12) 정답[ ] ⑤

해설 서 다른 높임표현을 통해 청자에 대해 리[ ] ⑤

적 거리감을 나타내는 인 은 이 아니라 현정이

다 가 에서 현정은 에게 해 체를 사 함으 써 ( )

친근감을 드러낸다 나 에서 연 을 게을리하는 역 ( )

도 들 때 에 화가 난 현정이 선생님에게 항의하

는 장 에서는 하 체를 사 하여 리적 거리lsquo rsquo

가 어졌음을 나타내고 있다

13) 정답[ ] ①

해설 는 는 얼 빛이 날과 어찌 다르 고[ ] lsquo rsquo

라는 뜻으 전과 달리 임이 화자를 않고

있음을 알 있다

14) 정답 달리 후 가 있다 이를 통해 경[ ] lt gt

쾌한 음악성을 형성하고 노 젓는 상황을 체적으

형상화하는 역할을 한다

15) 정답[ ] ①

16) 정답[ ] ⑤

해설 다 의 자연은 를 성찰하게 하는 대상[ ] ( )⑤

이자 정의 대상이다 의 자연은 자 의 상황과 ⑤

처 를 드러내는 경으 서의 역할을 하 이

이 없다

17) 정답[ ] ③

해설 는 빈천 을 해결하고자 했으나 강산[ ] lsquo ( )rsquo 貧賤③

과 풍 을 달라는 에 거절하 다고 함으 써 자

연에 대한 애정을 드러내고 있으 는 않는

임에 대한 망을 개에게 전가 켜서 임에 대한 리

을 드러내고 있다

18) 정답[ ] ③

년 학 간고사 대비2013 2 현대고 대비

ECN-0102-2013-001-000076193

19) 정답[ ] ⑤

해설 고상한 음악가의 이름을 리말 꽝 럽[ ]

게 꿈으 써 언어유희를 통해 음을 유 하고 있

다 이는 고상한 척하는 총 를 비꼼으 써 비판적

태도를 드러내는 것이 대상을 꽝 럽게 표현

하여 총 의 허 과 사치를 풍자하고 있다

20) 정답[ ] ⑤

해설 는 작품 속 경에 대한 설 이 드러나는 것이[ ]

서 자의 주 적인 견해가 접적으 드러나는 것이

아니다

21) 정답[ ] ⑤

22) 정답[ ] ②

23) 정답[ ] ④

24) 정답[ ] ①

해설 적강 티프는 주인공의 비 한 출생이나 능[ ] ①

과 이 있는 것으 조정의 능함을 풍자하는lsquo rsquo

것과는 거리가 다

25) 정답 픔 나[ ] ( )

해설 의 음악은 고통 는 사람들을 위 하고 아픔[ ] lsquo rsquo

을 치유해 주는 능을 한다고 할 있다 의 lt gt

픔 도 소 된 이 과 더 어 살아가는 따뜻한 마음lsquo rsquo

을 상 한다

26) 정답[ ] ⑤

해설 에게 선천적으 주어 각 장애라는 역경[ ]

은 의 이라는 가사 연 을 있다lsquo rsquo

27) 정답[ ] ④

해설 는 장 란 선 에게 은 개인적인 인상을[ ]

소녀 장정 등으 표현한 것이다lsquo rsquo

28) 정답[ ] ②

해설 담자가 피 담자의 언어적 표현이나 비언어[ ]②

적 표현 하 독자는 담의 위 나 피

담자의 감정 상태를 알 있다 이를 통해 독자는

담 상황을 더 생생하게 느낄 있고 피 담자

를 더 잘 이해할 있게 된다

29) 정답[ ]③

해설 일상생활과 역도 선 서의 성과에 된 것에서[ ]

역도를 하 서 겪는 어 과 내적 고민으 화제를

전화하 위한 것이다

30) 정답[ ] ①

해설 릿속에 새겨 넣듯 이 억되도 함 세상[ ] ② ③

살이가 힘들고 고생 러 속 하여 자유를 ④

가 없는 고통의 상태를 비유적으 이르는 말

적의 침입을 막 위해 쌓은 축 켜야 할⑤

대상을 비유적으 이르는 말이다

31) 정답[ ] ④

해설 이 의 종류는 전 으 인 사건 경[ ] lsquo

비평을 성 소 삼는다rsquo

32) 정답[ ] ④

해설 근은 삼대독자 태어났음을 에서 확인할[ ]

있다 형제들과의 담은 이뤄 가 없다

33) 정답[ ] ⑤

해설 근은 가난에도 하고 화가를 꿈꾸었다[ ] (3

단 또한 다른 화가 망생들은 정 육을)

위해 상 학 학 해 유학 에 랐 만

근은 다른 을 찾아야 했다 단 세에(5 ) 18

근은 조선 전람회에 입선하 다 단 의(6 )

만종은 인간과 자연이 엮어 가는 경건한 조화 을lsquo rsquo

나타낸다

34) 정답[ ] ①

해설 근이 속에서도 창작활동을 추 않고[ ]

하는 닭은 은 세상과 타협할 르는

근이 세상의 이해를 하 위한 가장 떳떳한 단

이 때 이다

35) 정답[ ] ⑤

해설 전 은 서 자의 주 적인 평이 리는 것이[ ]

만 위 제 은 인 이 살았던 대 사회적 경

을 통해 객 적인 인 의 을 제 하고 있다

36) 정답[ ] ⑤

해설 전 은 인 사건 경 비평이라는[ ] lsquo rsquo⑤

성 이 어져 있다

37) 정답[ ] ①

해설 이 은 동양인과 서양인의 사고 에 차이가[ ]

있다는 것을 대조를 통해 설 하고 있다 또 쓴이

의 제자가 축 경 를 러 가서 경험한 일화를

통해 동양인이 서양인에 비해 주 상황에 더 많은

주의를 인다는 주장을 뒷 침하고 있다

38) 정답[ ] ④

39) 정답[ ] ②

40) 정답[ ] ②

41) 정답[ ] ④

42) 정답[ ] ③

43) 정답[ ] ④

44) 정답 도서 의 휴 일 도서 의 이 간 도서의[ ]

해설 도서 장은 임의 정한 휴 일과 도서 이[ ]

간 도서의 상 등을 게 할 의 가 있다

년 학 간고사 대비2013 2 현대고 대비

ECN-0102-2013-001-000076193

45) 정답[ ] ①

해설 제 조의 정 휴 일 의 휴 일의 사전 게[ ] 3

는 도서 장의 의 조항에 속한다

46) 정답[ ] ①

해설 개인 정 호 의 를 제 하 했 만 항[ ]

나눠서 제 하 않고 대 나열하고 있다

47) 정답[ ] ②

해설 제 조의 내 을 회사는 다른 회사 협[ ] 7 lsquo

계약을 통해 서비 를 제공하는 경 회 의 아이디

등 개인 정 를 해당 회사에 전송할 있다는 내rsquo

이 있으 의 제점을 제 할 있다②

48) 정답[ ] ④

해설 는 도서 장의 의 에 해당하고 나 는 도[ ] ④

서 장의 리에 해당한다

49) 정답[ ] ③

50) 정답 은 음독으 적었고 은 훈독으 적었[ ] (1)

다 과 동일한 표 리 적은 것은 이고 (2) ce

과 동일한 표 리 적은 것은 이다ab

51) 정답[ ] ③

52) 정답[ ] ①②

53) 정답[ ] ③

54) 정답[ ] ③

55) 정답[ ] ①

56) 정답 른 죠코 어린 노 하니라[ ] A B

57) 정답 세 어에서는 활 형이 칙적으[ ] lsquo rsquoㄹㅇ

나타났 만 개화 어에서는 활 형이 쓰 다 lsquo rsquo ㄹㄴ

58) 정답 호 가 흔[ ] (1) (2)

59) 정답[ ] ④

60) 정답[ ] ③

Page 18: 현대고대비 국어 - chamsoriedu.com 「콘텐츠산업진흥 법」외 에도 저작권 의하여 ... 다른주체에게어떤동작을하도록만드는것을나타내는

년 학 간고사 대비2013 2 현대고 대비

ECN-0102-2013-001-000076193

다 공주 그림 가 근 경- ( ) ldquo rdquo(

2009)

zb27) 작가의 주 적인 각이 드러난 것은~

① ② ③ ④ ⑤

가 신 지에 고 상 원망( )

도 단다 어느 가 에 시각 에 ldquo

어 그런 듣고 다 보니 내가 lsquo

게 살 는지 도 눈 고 싶rsquo lsquohelliphellip

보 는 생각만 들 고 그 가 들에게rsquo

도 내고 들도 고 많 었죠 들었rdquo

지 새 는 에 쑥쓰러운 색

어났다 략 [ ]

경 는 가 망 없 티lsquo

원 고 답 다 신과 같 시각rsquo

는 습 상상 만 도 감동

다 시각 연주 동시에

열 상 는 티

원 그런 열 경 럽다는 것 다 략 [ ]

슬 마다 통 낼ldquo

었 것 럼 고통 는 사 들

고 겠다 고rdquo

말 다 달 루 첫 낸 lsquo rsquo

첫 드 심 집에 는 리듬 2

루 에 도 보고 싶다 집 에는 직(RampB) 3 4

사 곡 도 보 고 싶다고 포 다

식 누 가-

고 싶어

다 역도 미 담 고 사( )

질 주말에는 주 엇 보내[ 1]

답 주말에도 별 주 에[ ] ldquo

청 고 에 가고 도 쳐

에 듣고 보 에 갈 가 별

없어 rdquo

질 계 고 슬슬 도 는 것 닙니[ 2]

답 다 들 눈 에 보 고 뿐 보[ ] ldquo

다 열심 고 어 상에 도 들지만 상

지키는 것 들다고 에 도달

그것 지키 훨 많 rdquo

질 들 살 고 리 는[ 3]

거운 들 체 리느 는다

답 가 고 게 체 어[ ] ldquo ( ) 級

느 도 계가 니 살 는 것도 고역 지만

살 우는 것 들어 는 살

체 리 고 어도 어도 실 갔다

쑥 어 rdquo

질 거리에 슷 연 여 들[ 4]

보는 간 상 지

답 상 다 체 게 리지 못[ ] ldquo

거 주변에 는 그 거 누 보지

못 고 뻐지고 싶 에 체 리는 에

타 워 지만 는 어울 는 것보다 는

시간 운동만 는 건 니에 사복 lsquo rsquo

고 사복 는 말에 들 웃지만 늘 운동복

고 지내니 사러 갈 도 어 rdquo

질 역도가 말 단 식 운동 니[ 5]

답 가 내는 만 클 업 보[ ] ldquo

그러니 만 쓰는 식 운동 니다

만 다고 거운 것 들 는 건 니거든 연

도 고 가지 동 에 도 여러 가지

복 들

보식 역도 여 미-

zb28) 가 에 대한 설 으( ) 않은 것은

① 시각 우 지 시 에 지

고 망 가는 태도 달 고 다

② 언어 과 언어 복 사 여

담 내 생각 게 는 가

③ 직 감 그 마 것

럼 생생 게 느껴지는 과 주고 간 내

없 리 어 억 게 다

④ 담 내 식 리 여 담 삶 습

과 가 시 여 독 에게 감동과 훈 다

⑤ 직 진 담 직 누

지 못 는 독 에게 생생 상 달 주고

담 욱 게 다

zb29) 나 의 각 의 의도를 설 한 것으 적절하( ) 않

년 학 간고사 대비2013 2 현대고 대비

ECN-0102-2013-001-000076193

은 것은

① 질 담 상 보여 주 것 다1

② 질 담 과 그에 삶 태도 보여2

주 것 다

③ 질 역도 겪는 어 움에 역도3

과 것 다

④ 질 같 연 여 갖는 고민 는지 말4

주 는 것 다

⑤ 질 역도가 과 고 운동 는 것5

담 가 말 주 는 것 다

가 만진 것 다( ) 3

감 달 다고 다 억 에( ) 音感

지워 지만 당시 청 탁 리도

다고 다 드럼 웠다 4

에 갈 마다 드럼 는 리가 신 게 들

다고 다 눈 볼 가 없 니 엔ldquo

는 는 님 틱 에 여 주

다 드럼과 연 맺 과 들 주었다rdquo

식 누 가-

고 싶어

역( ) 도가 말 단 식 운동 니

가 내는 만 클 업에 보ldquo

그러니 만 쓰는 식 운동 니다 만

다고 거운 것 들 는 건 니거든 연

도 고 가지 동 에 도 여러 가지 복

들 시 는 상 상

드는 상 에 맞춰 실 에 는 여러

펼쳐집니다rdquo

략( )

늘 에 는 어 만 것 같

가 에 사 고 사 사ldquo

겠어 든 에 가 경 만 고

울 는 사 겠어 rdquo

보식 역도 여 미-

다 가 운 는 어 어( ) ldquo rdquohelliphellip

월 새벽 시 태 없 거웠고1965 5 6 1

는 없 그 병원에 퇴원 집

가는 마지막 마 고 마 내 거 다

가 죽 간신 에 실 다 사는 어느5 lsquo

가 죽 는 말 가 식 다 신rsquo

상에 각 시키는( )刻印 에 실

어느 가는 후 민 가가 근 었다lsquo rsquo

는 간 과 진실 그 다는( ) ldquo

에 단 평 견 가지고 다 내

가 그리는 간상 단 고 다 지 다 는 그들

가 에 는 평 지 니 그리고 어린

들 미지 겨 그린다rdquo

마 근 간 과 진실 그리고 싶어( )

가 다 근에게 그것 진리 다 거 다 없

거 고 다 없 는 것 진리

다 근 진리는 후 쪽 었다 신산(辛酸 삶)

었 질곡(桎梏 역사 에 지냈)

가 눈에 든 것 료 단 료 게

보 것 었다 그것 그 에 겨우겨우

슬 슬 생 어가는 간들 었

다 리 과 단 리 고리에 검

마 없 거리 돌

상 것 없는 등 근에게 상에

과 진실 엄 다는 사실 리는 가( )儼存

실 고 가 과 역경 에 도 근 내

포 없었 후 보루(堡壘 다 도)

도 간 근365

여 시 것 다

월 강원도 림리에( ) 1914 2 21

삼 독 태어났다 어 근 복

그것 그리 가지 못 다 근 곱 살

지는 산 산업에 실 고 답마 에 내

갔다 근 그림 럼 쫓 다니 가 시 것

다 상 진 것도 가 었다

러 가 에도 고 근 가 꿈꾸었다 근

가 꿈꾸게 것 보통 업

원색도1926 만lsquo rsquo 었다

공주 그림 가 근 경-

zb30) 에 대한 설 가장 른 것은~

① 역도가 과 운동 도 질

② 리는 는 다 lsquo rsquo

③ 들었지만 그럭 럭 는 다 lsquo rsquo

④ 가 게 보 시 말 다

⑤ 보 병 는 지 상 lsquo rsquo

는 말 다

년 학 간고사 대비2013 2 현대고 대비

ECN-0102-2013-001-000076193

시간 많지 다 청량리 생 병원

마지막 상 경 릿 게 들어 다 그 는 십

만 큰 가 상 말 다

지 못 들 마 갈 고 돗

도시민들 싹 싹 탔다 가 시

월에 병원에 원 가 폐 진 몸도4 ( )疲弊

갈 미 지 못 고 었다 가는 얼마( ) 解渴

지 생 에 생각 가

마감 는 신 평생 십 만에

가 과 많 닮 다고 생각 지는

가 운 는 어 어ldquo rdquo 1965helliphellip

월 새벽 시 태 없 거웠고 는5 6 1

없 그 병원에 퇴원 집 가

는 마지막 마 고 마 내 거 다 가

죽 간신 에 실 다 사는 어느 가5 lsquo

죽 는 말 가 식 다 신rsquo

상에 각 시키는 에 실 어느( ) lsquo刻印

가는 후 민 가가 근 었다rsquo

ldquo 는 간 과 진실 그 다는 에

단 평 견 가지고 다 내가 그

리는 간상 단 고 다 지 다 는 그들 가

에 는 평 지 니 그리고 어린 들

미지 겨 그린다rdquo

근 간 과 진실 그리고 싶어 가

다 근에게 그것 진리 다 거 다 없 거

고 다 없 는 것 진리다

근 진리는 후 쪽 었다 신산 삶 ( )辛酸

었 질곡 역사 에 지냈 가 눈에( )桎梏

든 것 료 단 료 게 보 것

었다 그것 그 에 겨우겨우 슬

슬 생 어가는 간들 었다 리

과 단 리 고리에 검 마

없 거리 돌 상

것 없는 등 근에게 상에 과 진실

엄 다는 사실 리는 가 실( )儼存

고 가 과 역경 에 도 근 내 포 없었

후 보루 다 도 도( ) 365堡壘

간 근 여 시 것

간에 지닌 가 근 1914 2

월 강원도 림리에 삼 독21

태어났다 어 근 복 그것 그리

가지 못 다 근 곱 살 지는 산

사업에 실 고 답마 에 내 갔다 근

그림 럼 쫓 다니 가 시 것 다 상

진 것도 가 었다 러 가 에도

고 근 가 꿈꾸었다 근 가 꿈꾸게

것 보통 업 원색1926

도 만 었다lsquo rsquo

그림 가 근 경 공주- ldquo rdquo ( 2009)

zb31) 다음 이 같은 의 성 소에 해당하 않은

것은

사건 평① ② ③

④ 주 ⑤ 경

가 운 는 어 어ldquo rdquo 1965helliphellip

월 새벽 시 태 없 거웠고 는5 6 1

없 그 병원에 퇴원 집 가

는 마지막 마 고 마 내 거 다 가

죽 간신 에 실 다 사는 어느 가5 lsquo

죽 는 말 가 식 다 신rsquo

상에 각 시키는 에 실 어느( ) lsquo刻印

가는 후 민 가가 근 었다rsquo

는 간 과 진실 그 다는 에ldquo

단 평 견 가지고 다 내가 그

리는 간상 단 고 다 지 다 는 그들 가

에 는 평 지 니 그리고 어린 들

미지 겨 그린다rdquo

근 간 과 진실 그리고 싶어 가

다 근에게 그것 진리 다 거 다 없 거

고 다 없 는 것 진리다

근 진리는 후 쪽 었다 신산 삶 ( )辛酸

었 질곡 역사 에 지냈 가 눈에( )桎梏

든 것 료 단 료 게 보 것

었다 그것 그 에 겨우겨우 슬

슬 생 어가는 간들 었다 리

과 단 리 고리에 검 마

없 거리 돌 상

것 없는 등 근에게 상에 과 진실

엄 다는 사실 리는 가 실( )儼存

고 가 과 역경 에 도 근 내 포 없었

후 보루 다 도 도( ) 365堡壘

간 근 여 시 것

간에 지닌 가 근 1914 2

월 강원도 림리에 삼 독21

태어났다 어 근 복 그것 그리

가지 못 다 근 곱 살 지는 산

사업에 실 고 답마 에 내 갔다 근

그림 럼 쫓 다니 가 시 것 다 상

진 것도 가 었다 러 가 에도

고 근 가 꿈꾸었다 근 가 꿈꾸게

것 보통 업 원색1926

도 만 었다lsquo rsquo

공주 그림 가 근 경- ldquo rdquo ( 2009)

년 학 간고사 대비2013 2 현대고 대비

ECN-0102-2013-001-000076193

zb32) 위 을 작성하는 과정에서 되어 활 된 자

어 것은

신 사 료① 연보②

고③ ④ 들과 담

⑤ 에 평

는 간 과 진실 그 다는 에ldquo

단 평 견 가지고 다 내가 그

리는 간상 단 고 다 지 다 는 그들 가

에 는 평 지 니 그리고 어린 들

미지 겨 그린다rdquo

근 간 과 진실 그리고 싶어 가

다 근에게 그것 진리 다 거 다 없 거

고 다 없 는 것 진리다

근 진리는 후 쪽 었다 신산 삶 ( )辛酸

었 질곡 역사 에 지냈 가( )桎梏

눈에 든 것 료 단 료 게 보

것 었다 그것 그 에 겨우겨우

슬 슬 생 어가는 간들 었다

리 과 단 리 고리에 검 마

없 거리 돌 상

것 없는 등 근에게 상에 과

진실 엄 다는 사실 리는 가 실( )儼存

고 가 과 역경 에 도 근 내 포

없었 후 보루 다 도 도( ) 365堡壘

간 근 여 시

것 다

간에 지닌 가 근 1914 2

월 강원도 림리에 삼 독21

태어났다 어 근 복 그것 그리

가지 못 다 근 곱 살 지는 산

사업에 실 고 답마 에 내 갔다 근

그림 럼 쫓 다니 가 시 것 다 상

진 것도 가 었다 러 가 에도

고 근 가 꿈꾸었다 근 가 꿈꾸게

것 보통 업 원색1926

도 만 었다lsquo rsquo

질 루 마 가 도 린다 경건

움 느껴지는 경 다 훗 근 그림에

과 는 거 것( )裸木

만 간과 연 엮어 가는 경건 움lsquo rsquo

니었

같 가가 고 싶었 근에게 그 꿈에 다

가가는 지 다 다 가 지망생들 규 미

상 에 진 고

에 지만 근 다 다 근

미 에 운 것 보통 시 미 시간

다 그런 그에게 없는 연습 가가

통 다 가 귀 시 지 도

얻는 뛸 듯 뻤지만 마 도 가 에

듯 는 었 에 어린 근 주 에

에 그림 그리고 지우고 복( )粉板

시간 가는 게 루 보냈다

근 그 갈 가가 것 열여( )渴求

었 다가 미1932 lsquo rsquo ( lsquo

미 에 다 다는 고 마rsquo) lsquo rsquo

가 근 집 고도 지는 시골 경

그린 그림 다 후 근 에 1943 22

지 미 에 그림 고

에 걸쳐 다 미 근 가

동 는 었다

공주 그림 가 근 경- ldquo rdquo ( 2009)

zb33) 위 의 내 과 일치하는 것은

가 근 가 꿈 포 다①

근 당 가들과 께 에 다②

살 근 가 걷20③

게 었다

④ 만 통 근 역경 겨내는lsquo rsquo

느 다

⑤ 근 간 과 진실 그리 에 그 에

드러 는 간상 단 다

계 시 주 근 건강

걸었다 신 과 간에 상 다 건강

신 는 눈에도 다 근 쪽 눈 뿌 게

보 지 과에 다 다 시 지지 고 결

내 었다 시 지만 마 막막

다 늦어 결 근 쪽 눈 고 말 다

쪽 눈 근에게는 쪽 눈 었고

계 었다 그 근 는 여 그lsquo rsquo

다 근 에 같 그림 그 었다1950

시 그림 는 여 쪽lsquo rsquo

고 어 마주 고 는 그림1963

여 과 동 다 마 복

그린 듯 눈 내리 새 게 다 지

사 다 근 게 복 것

복 상과 타 는 근 상

가 떳떳 단 었고 근 그리고

간 과 진실 에 다가가 가 근다

운 었다 근 신에게 당당 지 그리고

그 다 근 그림에 단 복 보다

년 학 간고사 대비2013 2 현대고 대비

ECN-0102-2013-001-000076193

태 도 그리고 극 보다 과

얻 여 었다 과 통

근 그리고 는 재 고 에 질

만들고 특 것 다

공주 그림 가 근 경- ldquo rdquo( 2009)

zb34) 의 이유에 대해 추 한 것으 적절하 않은 것

상과 타 시도①

보다 과 얻②

근 신에게 당당 지③

④ 간 과 진실 에 다가

⑤ 태 도 얻

근 가가 었지만 그 다니 가

럼 어지지 다 복과 쟁 거쳐 시

는 가 근에게 생계 사 에

운 사 다 에 키에 건( ) 178cm死鬪

체 근 에 동 역 업( )荷役

가 생계 다 쟁

에는 동에 운 상우 주 미

죄 사 에 그림 그리는 시 다 그곳에

에 동 역 업 것에

결 것 럼 보 다 지만 그런 것만도

니었다 그림 그리는 고는 지만 매 근

는 극 간 과 별 없는 경 리 그림

벽에 그리는 것 었다 우도 리 없었다 근

트 는 우 그림 그 다 생

계 그림 단 것 다

후 근 지 신 계 리에 미

엑 리 겼다 근 곳에

건 사 크 에 미 들 ( )

상 상 그 다 근 갖 다 겪

냈다 그리고 결 그 돈

신동에 어 사리 집 마 다 마 ㄷ

루 심 쪽에는 과 엌 쪽에는 건

었다 건 주고 근 가 에

여 살 다 심 에는 지 집어

쓰고 지만 곳 근 가 에게 러웠

보 리 다 근 과 마루 업실 삼 그림

그 다 신동 마루는 근 그림에 등 는 lsquo rsquo

같 상들 지 다 시 고

에 들 폐허가

가 업실 었다

공주 그림 가 근 경- ldquo rdquo( 2009)

zb35) 위 에 대한 설 으 적절한 것은

업 시 여 훈과 감동 다①

에 주 평 드러 다②

사 사 등 식 과 ③

④ 다 근거 시 여 삶에

⑤ 살 시 사 경 께 여

습 시 다

가 시간 많지 다 청량리 생 병원( )

마지막 상 경 릿 게 들어 다 그 는

십 만 큰 가 상 말 다

지 못 들 마 갈 고 돗

도시민들 싹 싹 탔다 가 시

월에 병원에 원4 가 폐( )疲弊

진 몸도 갈 미 지 못 고 었다( )解渴 가는

얼마 지 생 에 생각

가 마감 는 신 평생 십 만에

가 과 많 닮 다고 생각 지는

가 운 는 어 어( ) ldquo rdquohelliphellip

월 새벽 시1965 5 6 1 태 없 거웠고

는 없 그 병원에 퇴원 집

가는 마지막 마 고 마 내 거 다

가 죽 간신 에 실 다 사는 어느5 lsquo

가 죽 는 말 가 식 다 신rsquo

상에 각 시키는 에 실( )刻印

어느 가는 후 민 가가 근 었다lsquo rsquo

다 는 간 과 진실 그 다는( ) ldquo

에 단 평 견 가지고 다 내

가 그리는 간상 단 고 다 지 다 는 가

에 는 평 지 니 그리고 어린 들

미지 겨 그린다rdquo

근 간 과 진실 그리고 싶어( )

가 다 근에게 그것 진리 다 거 다 없

년 학 간고사 대비2013 2 현대고 대비

ECN-0102-2013-001-000076193

거 고 다 없 는 것 진리

다 근 진리는 후 쪽 었다 신산( )辛酸 삶

었 질곡 역사 에 지냈( )桎梏

가 눈에 든 것 료 단 료 게 보

것 었다 그것 그 에 겨우겨우

슬 슬 생 어가는 간들 었다

리 과 단 리 고리에 검

마 없 거리 돌

상 것 없는 등 근에게 상에

과 진실 엄 다는 사실 리는 가 실( )儼存

고 가 과 역경 에 도 근 내 포

없었 후 보루 다( ) 堡壘 도 365

도 간 근 여

시 것 다

마 같 가가 고 싶었 근에게 그 꿈( )

에 다가가는 지 다 다 가 지망생들

규 미 상 에 진 고

에 지만 근 다 다 근

미 에 운 것 보통 시 미 시간

다 그런 그에게 없는 연습 가가

통 다 가 귀 시 지 도

얻는 뛸 듯 뻤지만 마 도 (

는 었 에 어린 근 주 에)

에 그림 그리고 지우고( )粉板

복 시간 가는 게 루 보냈다

zb36) 전 의 성 소가 아닌 것을 고르

① 평 ② 사건 ③ 경

④ ⑤ 훈

늘 지 상에 살고 는 사 들 억 도가10

고 그리 지 통 고 는 사 들( )知的

그보다 훨 많 억 도는 고 지 20

통 다 그런 지 고 2500

그리 간 보는 과 사 에

매우 달 뿐만 니 과 에 도 극

루고 었다 미 운 그런 들

살고 는 동 과 사 들 사고 식에

큰 가 다는 다

고 그리 들 우주 개별 고 독립

사 들 생각 지만 고 들 우

주 연 질 간주 다 같( ) 看做

각 도 들에게는 연 질

었지만 그리 들에게는 미 들 결 었

다 고 과 그리 들 사 같

는 동 과 사 에 도 견 다

지심리 미 마 드 겐트 는

살 들에 에 지 다

연 동 과 상 다 과 같 실험

다 크 만든 미드 도 보

여 주고 그 상 닥 고 주었다lsquo (Dax)rsquo

실 닥 는 재 지 는 것 실험 가lsquo rsquo

만들어 낸 다 그런 다 개 다 체 보

여 주었는 는 미드 지만 틱

만들었고 다 는 재료는 크 지만

달 다 그러고 어 것 닥 지 사 들에게 고 lsquo rsquo

게 니 들 주 같 고 는

체 택 고 동 들 같 재료 만들어진 체

택 다 러 는 심지어 살짜리

들에게 도 타났다 것 곧 과 동

다 상 보고 다는 것 미 다

개별 사 보고 고 동 연 질 보

고 는 것 다

동 들 주변 상 에 맞 어 동 고

에 다 사 들 태도 동에 보다 많

주 울 다 동 가 미시간 에

에 경험 다 그는 미식

경 보러 가게 었는 경 체는 매우 재미 었

주변 들 동에 질 다 그 는

들 계 어 상태 경 다

어 들 에 에 그 시 가 계 가

진 것 다 상 살펴 는 말 들 lsquo rsquo

에 그는 에 시 어 도 뒷사

생각 곧 다시 곤 것 다 그런 그에게 뒷

사 고 지 는 들 동 럼

어 웠다

생각 지도 리 드 니 벳-

zb37) 다음 위 의 내 전개 으 만 인lt gt

것은

lt gt

대조의 통해 대상이 닌 특성을 설 하고 있다

일화를 제 하여 자 의 주장을 뒷 침하고 있다

유추의 을 사 하여 독자의 의해를 돕고 있다

대상이 형성되는 과정을 간적 서에 따라 서 하고 있

① ②

③ ④

년 학 간고사 대비2013 2 현대고 대비

ECN-0102-2013-001-000076193

가 우리가 말 고 쓰는 든 단어가 사 에 는( )

것 니다 사 격에 가 는 지만

어 사 과 같 특별 는 사 니lsquo rsquo

단어 격 보 단어가 사 에

등재 어 다 리 리 사 는 단어 도 그

것 시 사 는 어 고 사 에

격 보 것 니다

러 얼 은 사전에 를 있는가 이에 대한 답lsquo rsquo

은 얼 이 유행어인가 아닌가에 따라 갈라 다 이 단어lsquo rsquo

는 년 어 자 에 랐고 쓰이고 있으2002 lsquo rsquo

유행어라고 하 에는 생 이 다 런데 계속

을 유 하 서 사전에 등재될 자격을 획득할 것인가 이

에 대한 답을 내리 는 히 어 다

여 서 가 를 고 해 볼 있다 첫 는 이 단어

를 써야 할 필 가 속적으 있는가 하는 점이다

상주의 열풍에 휩 인 사회 위 에 편 해서 퍼 말

이 얼 인데 과연 런 위 가 속될 것인가 이에lsquo rsquo

대해 필자의 생각은 정적이다 사회 위 가 뀌

런 말을 쓸 일이 없어 것이다

다음은 단어의 성이다 단어의 성이 사회적으 거

감이 없으 계속 사 될 가능성이 높다 런 에서

얼 은 좋은 조건이 아니다 익히 알 졌듯이 이lsquo rsquo

말은 얼 과 청소년층에서 속어 사 하는 이 결합lsquo rsquo lsquo rsquo

된 말이다 얼 에서 얼 을 리하는 조어 도 lsquo rsquo lsquo -rsquo

어에서는 매 낯선 이다 이것만으 도 거 감을 갖

는 사람들이 있다 더 나 속어 결합한 말이다 얼 lsquo rsquo

이 널리 퍼졌다 해도 은 여전히 청소년층의 속어lsquo rsquo

남아 있다 속어는 자연 럽게 아 자리에서나 쓰 에는

담 러 말이다 러한 담을 하고 사

역을 넓혀 가는 속어도 없 는 않다 특히 얼 은 lsquo rsquo

에도 종종 등장한다 만큼 거 감이 많이 희석되었다

고 할 있다 러나 일상의 자연 러 대화에서도 거

리낌 없이 등장하는가 게 는 되 않았다고 생

각한다

얼 이 유사어인 쌈 등을 만들어 내고lsquo rsquo lsquo rsquo

있으니 살아남을 있을 것이라고 는 견해도 있을 것

이다 러나 간이 나 서 유사어를 포함하여 든

말이 사라 사 는 많다 유사어가 많다는 것이 생 을

유 할 있는 절대적인 조건은 아니다

나 언젠가 터 사람들은 어느 단에서 얼 이 가장( )

쁜 사람을 가리켜 얼 이라고 르고 있다 이 얼lsquo rsquo lsquo rsquo

이라는 단어가 최근 어사전에 라 항간에 논란이 일고

있다 아닌 게 아니라 얼 은 유행어처럼 인다 생 lsquo rsquo

도 리 래되 않은 것 같고 언제 사라 도 알

없다 게다가 젊은이들 사이에서 주 쓰일 뿐이다 이런

단어를 사전에 는다는 게 하 이 없어 이 도

한다

러나 속단은 이다 차근차근 따져 볼 일이다

선 얼 이 일 적 유행어인 아닌 주의 게 들여다lsquo rsquo

볼 필 가 있다 유행어란 유행에 따라 빠르게 유포되었

다가 단 간 내에 소 되는 단어나 를 가리킨다

얼 은 인터넷을 통해 속히 퍼 말이다 하 만 일lsquo rsquo

적인 유행어처럼 단 간 내에 사라 않았을 뿐 아니라

현재 도 잦은 빈도 사 되고 있고 앞으 도 상당

간 사 될 것으 측된다 한 언 재단의 뉴 검 lsquo rsquo

색 사이트에 따르 얼 은 년 에 처음 나타난lsquo rsquo 2001

이후 꾸 히 사 되고 있다

이 같은 사 빈도는 얼 이 일 적 유행어 는 현lsquo rsquo

저히 다르다는 것을 여 다 장 간의 생존 만으 도

얼 은 이 한 어의 어휘 에 를 자격을 얻었다lsquo rsquo

고 할 있다 더 이 이라는 비 적 정제된 매체에

높은 빈도 쓰이고 있 않은가 사 빈도 측 에서

필통이나 연필과 같은 단어 대등하거나 더 많이 쓰lsquo rsquo lsquo rsquo

다는 것은 결코 가 게 볼 일이 아니다

이제는 사전이 언어 현 을 빠르게 하는 게 덕인

대가 되었다 세계적으 유 한 의 사전들도 경쟁

적으 어를 고 있다

하 만 얼 은 젊은이들이나 쓰는 속어라고 흠을 잡을lsquo rsquo

도 르겠다 얼 이 주 젊은 층에서 많이 쓰 lsquo rsquo

는 속어임에 틀림없다 러나 어사전에 표 적이고 품

위 있는 말만 어야 한다고 생각한다 것은 커다란

해다 당장 아 어사전이나 펼쳐 라 속어는

설과 같은 비어나 죄자들이 쓰는 은어 어

마니 같은 소 의 사람만이 쓰는 말 도 라 있

않은가 사전은 말 치에 일정 빈도 이상 나타나는 말이

라 말이든 다 할 있다

zb38) 가 나 에 대한 다음의 설( ) ( ) 않은 것은

① 가 는 얼짱 사 에 등재 것에( ) ( ) lsquo rsquo

보 고 다

② 사 등재 가는 단어 격에( )

고 고 는 언 들 언어 사 도에 고 다 ( )

③ 가 얼짱 어지만 신 과 같 매( ) ( ) lsquo rsquo

체에 도 사 는 말 는 고 다

④ 가는 얼짱 어 보고 크게 가지 근( ) lsquo rsquo 3

거 들어 뒷 고 다

⑤ 는 얼짱 어 는 다 특 다는( ) lsquo rsquo

근거 에도 크게 가지 근거 가 들어 주 2

뒷 고 다

가 늘 지 상에 살고 는 사 들 억( ) 10

도가 고 그리 지 통 고 는 사 들

그보다 훨 많 억 도는 고 지 20

통 다 그런 지 고 2500

년 학 간고사 대비2013 2 현대고 대비

ECN-0102-2013-001-000076193

그리 간 보는 과 사 에

매우 달 뿐만 니 과 에 도 극

루고 었다 미 운 그런 들

살고 는 동 과 사 들 사고 식에

큰 가 다는 다

고 그리 들 우주 개별 고 독립

사 들 생각 지만 고 들 우

주 연 질 간주 다 같 각

도 들에게는 연 질 었지

만 그리 들에게는 미 들 결 었다

고 과 그리 들 사 같 는

동 과 사 에 도 견 다

인 리학자인 츠 이마이 디드 겐트너는 두

살이 채 안 된 아이들에서 터 성인에 이르 다양한

연 대의 동양인과 서양인을 대상으 다음과 같은 험

을 했다 저 코르크 만든 피라 드 양의 도형을

여 주고 대상의 이름을 닥 라고 알 주었다lsquo (Dax)rsquo

제 닥 는 존재하 않는 것으 험자가 임의lsquo rsquo

만들어 낸 이름이다 런 다음 두 개의 다른 체를

여 주었는데 하나는 피라 드 양이 만 하얀 플라 틱

으 만들었고 다른 하나는 재 는 코르크 만 양이

달랐다 러고 나서 어떤 것이 닥 인 사람들에게 고 lsquo rsquo

르게 했더니 서양인들은 주 같은 양을 하고 있는

체를 선택했고 동양인들은 같은 재 만들어 체를

선택했다 이러한 차이는 성인은 어 두 살 리

아이들에게서도 나타났다 이것은 곧 서양인과 동양인은

서 다른 세상을 고 있다는 것을 의 한다 략 ( )

는 아주 단 하 서도 인상적인 험을 했다

험에는 동서양의 대학생들이 참여했다 는 험 참가자

들에게 컴퓨터 화 을 통해 속 장 을 담은 애니 이션

을 여 주었다 화 의 앙에는 초점의 역할을 하는 커

다란 고 한 마리가 있었고 주위에는 다른 생

들과 초 자갈 거품 등이 함 제 되었다 화 을

두 씩 후 참가자들은 자 이 것을 회상해 라는

를 았다

결과 서양인 대학생들과 동양인 대학생 두 앙

의 초점 역할을 했던 고 를 동일한 정도 언 했으

나 경 소 위 거품 초 다른 생 들 에 ( )

대해서는 동양인 대학생들이 서양인 대학생들 다 60

이상 더 많이 언 했다 뿐만 아니라 동양인 학생들은 서

양인 학생들에 비해 개 적인 고 다 전체적인 계

를 더 언 하는 경향을 다 략 또한 경의 일 ( )

를 화 킨 림을 제 하 을 때 동양인 대학생들은 대

경의 화를 알아챘 만 서양인 대학생들은 경

의 화를 거의 알아차리 했다 략 ( )

따라서 서양인들만을 대상으 연 한 화lsquo

편성 결 은 잘 된 것일 도 있다 각 과정과 인rsquo

과정의 어떤 이 화 편적이고 어떤 이

화에 따라 달라 는 는 앞으 많은 연 를 통하여 논의

되어야 한다

나 어떤 의 에서 리 두는 이 화적이다 리( )

안에는 다른 사람들과 더 친 한 계를 유 하 는 상호

의존성과 다른 사람들 터 독립적인 존재 살아가 는

독립성이 혼재한다 따라서 이 에서 어떤 특성이 더 강

하게 각되는 상황에 놓이느냐에 따라 서 다른 화적

특 을 일 있다 결 리 두는 어떤 경 에는

동양인처럼 행동하고 어떤 경 에는 서양인처럼 행동하는

것이다

zb39) 가 에 대한 다음의 설( ) 않은 것은

① 는 신 주 뒷 닥 실험과lsquo rsquo lsquo

니 실험 근거 시 다rsquo

② 동 들 상 간 공통 보다는 에 식

는 강 다

③ 들 주변 맥 에는 심 경 어 사건

과 사건 사 계에 상 민감 다

④ 는 동 과 틀린 지 고 는 것lsquo rsquo

니 다 고 다 lsquo rsquo

⑤ 가에 우리 사 들 개 시 가 원( )

집 경 말 고 는 것 개 보다는

에 고 는 것에 다

늘 지 상에 살고 는 사 들 억 도가10

고 그리 지 통 고 는 사 들( )知的

그보다 훨 많 억 도는 고 지 20

통 다 그런 지 고 2500

그리 간 보는 과 사 에

매우 달 뿐만 니 과 에 도 극

루고 었다 미 운 그런 들

살고 는 동 과 사 들 사고 식에

큰 가 다는 다

지심리 미 마 드 겐트 는 동

과 상 다 과 같 실험 다

크 만든 미드 도 보여 주고 그

상 닥 고 주었다 그런 다lsquo (Dax)rsquo

개 다 체 보여 주었는 는 미드

지만 틱 만들었고 다 는 재료는

크 지만 달 다 그러고 어 것 닥 lsquo

지 사 들에게 고 게 니 들 주 같rsquo

고 는 체 택 고 동 들 같

재료 만들어진 체 택 다 러 는

심지어 살짜리 들에게 도 타났다 것

곧 과 동 다 상 보고 다는

것 미 다 개별 사 보고 고 동

년 학 간고사 대비2013 2 현대고 대비

ECN-0102-2013-001-000076193

연 질 보고 는 것 다

동 들 주변 상 에 맞 어 동 고

에 다 사 들 태도 동에 보다

많 주 울 다 동 가 미시간

에 에 경험 다 그는 미

식 경 보러 가게 었는 경 체는 매우 재

미 었 주변 들 동에 질 다 그

는 들 계 어 상태 경

다 어 들 에 에 그 시 가 계

가 진 것 다 뒷사 고 지 는 들

동 럼 어 웠다

그는 경험에 어 얻어 동 들lsquo

각도 상 본다 는 가 우고rsquo

검 여 주 단 도 상 실험 실

시 다 그는 실험 가 들에게 컴퓨 통

담 니 보여 주었다

에는 역 는 커다 고 마리가 었

고 주 에는 다 생 들과 갈 거 등

께 시 었다 본 후 가 들

신 본 것 상 보 는 지시 다

그 결과 생들과 동 생

역 고 동 도 언

경 거 다 생 들에 ( )

는 동 생들 생들보다 60

상 많 언 다 뿐만 니 동 생들

생들에 개별 고 보다 체 계

언 는 경 보 다 경 변 시

킨 그림 시 동 생들 경

변 지만 생들 경 변

거 리지 못 다

지 지 들만 상 연 lsquo

보편 결 못 것 도 다 지각 과 과rsquo

지 과 어 보편 고 어

에 달 지는지는 많 연 통 여

어 다

리 드 니 벳 생각 지도 사- ldquo rdquo( 2004)

zb40) 위 에 대한 설 으 가장 적절한 것은

① 동 과 생 식 강 고 다

② 가지 실험 통 쓴 고 다

③ 닥 실험에 사 본질에 동 사

상에 주 다

④ 니 실험에 동 과 에 지

각 도에 가 다

⑤ 쓴 는 보편 연 에 드러 우월 에

에 근 고 다

가 동 들 주변 상 에 맞 어 동 고( )

에 다 사 들 태도 동에 보다 많

주 울 다 동 가 미시간 에

에 경험 다 그는 미식

경 보러 가게 었는 경 체는 매우 재미 었

주변 들 동에 질 다 그 는

들 계 어 상태 경 다

어 들 에 에 그 시 가 계 가

진 것 다 상 살펴lsquo 는 말 들rsquo

에 그는 에 시 어 도 뒷사

생각 곧 다시 곤 것 다 그런 그에게

뒷사 고 지 는 들 동 럼

어 웠다

그는 경험에 어 얻어( ) 동 들lsquo

각도 상 본다 는 가 우고rsquo

검 여 주 단 도 상 실험

실시 다 실험에는 동 생들 여 다

그는 실험 가 들에게 컴퓨 통

담 니 보여 주었다 에는

역 는 커다 고 마리가 었고 주 에는

다 생 들과 갈 거 등 께 시

었다 본 후 가 들 신 본 것

상 보 는 지시 다

다 그 결과 생들과 동 생( )

역 고 동 도 언

경 거 다 생 들 에 ( )

는 동 생들 생들보다 60

상 많 언 다 뿐만 니 동 생들

생들에 개별 고 보다 체 계

언 는 경 보 다 들어 동

생들 상 체 연못 럼 보 어ldquo 같rdquo

체 맥 언 시 었지만

생들 상 어 같 큰 고 가 쪽 움ldquo

직 어 같 역 고rdquo

언 시 다 경 변 시킨 그

림 시 동 생들 경 변

지만 생들 경 변 거

리지 못 다

년 학 간고사 대비2013 2 현대고 대비

ECN-0102-2013-001-000076193

게 볼 동 들 보다는 큰 그( )

림 보 에 사 과 체 맥 연결시 지각

는 경 고 체에 특 떼어 내

어 독립 보는 것 낯 어 다 에

들 사 에 고 주변 맥 에는 심 경

에 사건과 사건 사 계에 상

민감 편 다

마 지 지( ) 들만 상 연

보편 결 못 것 도 다lsquo rsquo 지각 과

과 지 과 어 보편 고 어

에 달 지는지는 많 연 통 여

어 다

리 드 니 벳 생각 지도 사- ldquo rdquo( 2004)

zb41) 의 하는 가~ 다른 것은

① ② ③

④ ⑤

얼마 그 에 동 사고 식과

사고 식 보여 주는 내 다

들 에 는 탕 고 같 게

어 겨 고 미 에 는 그 크 럼 큰 고

어리 주고 원 는 어 도 는

상 고 생각 다는 것 다 러

는 어떻게 생 것 고 과 그리 거슬

러 가 보 그 단 다

고 연 경 체 경 생 에

다 벼 사는 공동 업과 경험 많 연 역

에 고 들 연 웃과

게 지내 고 탁 연 들

들 지 연 럽게 들 다 민들

웃과 동 게 뿐만 니 는 집 과

게 다

동 시 는 생태 경 에 살 결과

들 다 사 들 사 상 에 주

울 게 었고 는 곧 체 상 과 간 사

계 시 는 낳게 었다 신 가

가 는 체에 는 원 는 동시

에 다 사 들 그 사 포 체 맥 에

다 들 간 사 연

계 체 계에 주 울 는 사고 체계

게 었다

그러 그리 연 경 그 었다 산

지 연결 는 지 건 그리고 역

에 다 런 들 업에 다 사 과

동 므 공동체에

다고 다 고 그리 들

들과는 달리 보 내 감 지 들과

지 크게 느 지 못 다 그

견 다 경우 주 쟁 통 결 는 갖

게 었다

신 사 간 계들 루어진 커다

트워크 에 게 당연 사 역시 연

계들 체 식 게 다 어 상

원 도 그 개체가 체 맥 과

계 에 고 다 게 체 맥 에 주

울 다 보 상 복 과 가변 식 게 고

상에 재 는 많 변 들 사 에 재 는 들도

게 다 들 주 태도 보

는 경우가 많다 쟁 결

통 결 보다는 통 결

는 보 다

그러 고 그리 들 개개 사 사 독

에 주 울 다 사 사 체에

어 그들 사 에 재 는 공통 규 주

고 다 상 원 에도 사

체 내 주 고 다 그들

체 여 탕 체

는 주 태도 시 고 특 사 어

주에 는지 여 그 주에 는 규

견 다 에 는 쟁 식 리

같 리 사고 체계가 달 게 었다

리 드 니 벳 생각 지도 사- ldquo rdquo( 2004)

zb42) 위 에서 사 된 설 과 가장 유사한 것은

① 크톱 컴퓨 는 본체 니 마우 루

어 다

② 곡과 시 리 는 지 과 사 루어 다는 공통

지니고 다

③ 경 고 것과는 달리

경 본 연 태 그 주변 경

④ 벽돌 능 에 사계 내내

습도가 지 다

⑤ 잰느 체 체 지닌 재 체가 없

는 재 눌 다

년 학 간고사 대비2013 2 현대고 대비

ECN-0102-2013-001-000076193

zb43) 는 립 앙 도서 이 정의 일 이다lt gt

도서 장과 이 자의 리 의 정의 연결이

적절하 않은 것은

lt gt

제 조 서 유8 ( )

도서 장은 다른 이 자의 안전을 위협하거나 도서 의①

서를 란하게 할 가 있는 자에 대하여는 도서 출입

을 제한할 있다

도서 장은 이 자가 제 조 각 호의 어느 하나의 행위를 하7②

을 때에는 이 을 하게 하거나 도서 출입을 제한할

있다

제 조자 의 대출9 ( )

도서 자 는 다음 각 호의 경 대출할 있다①

상호대차도서 간에 자 를 류하는 것을 말한다 등 다1 ( )

른 도서 과의 협 을 위하여 필 한 경

공 이 공 행 상 필 하는 경2

에 도서 장이 필 하다고 인정하는 경3

대출이 가능한 도서 자 의 위는 도서 장이 정하는②

에 따른다

제 조 상10 ( )

이 자가 도서 자 설을 더럽히거나 찢거나 뜨①

쓰게 하거나 잃어 린 경 에는 상하여야 한다

도서 장은 제 항에 따른 상 을 정하여 게 하여야1②

한다

제 조이 절차 등11 ( )

이 칙에서 정한 것 에 도서 자 설의 이 절차

이 제한 등에 필 한 사항은 도서 장이 정한다

출처 립 앙 도서- (httpwwwnlgokr)

① 는 도 리 다8

② 도 는 리 다9 1

③ 료 지 는 도 리 다9 2

④ 도 료 변상에 리10 1

⑤ 는 에 도 리 다11

3

도 다 각 같다①

공 공 다만 연1

연 간 다

매월 째 째 월2

도 도 리 그 사3

가 다고 는

도 에 미리 게1 3②

시 여 다

4

도 시간 도 여 게시 다

5

도 료 시 는 는 도①

지에 등 후

등 에 사 도②

7

는 다 각 여 는 니 다

도 료 시 상 리1 lsquo rsquo

도 료 시 훼 는2 middot

지 가 닌 곳에 식 거 담3

우는

도 보 등 보 검색열4 middot

그 에 도 질 지 여 도5

여 게시 사 는

8

도 다 거 도①

질 게 우 가 는 에 여는 도

도 가 각 어느7②

에는 지 게 거 도

9

도 료는 다 각 경우 다①

상 도 간에 료 는 것 말1 (

다 등 다 도 과 여 경우)

공 원 공 상 는 경우2

그 에 도 다고 는 경우3

가능 도 료 는 도②

는 에 다

10

년 학 간고사 대비2013 2 현대고 대비

ECN-0102-2013-001-000076193

가 도 료 시 럽 거 거①

못 쓰게 거 어 린 경우에는 변상 여

도 에 변상 여 게시1②

여 다

zb44) 위 에서 도서 장이 게 해야 할 사항에 해당하는

것을 두 쓰

년 학 간고사 대비2013 2 현대고 대비

ECN-0102-2013-001-000076193

립 도 규

1 ( )

규 립 도 립 어린 청 도(

포 다 료 시 열 시 말) (

다 에 사 규 립 도)

편 진 다

2 ( )

규 립 도 도 다 에( lsquo rsquo )

고 는 도 에 도lsquo rsquo 2 2

료 에 여 다 다만 특 료 귀

료 등 료 에 사 립 도

도 다 다( lsquo rsquo )

3 ( )

도 다 각 같다①

공 공 다만 연1

연 간 다

매월 째 째 월2

도 도 리 그 사3

가 다고 는

도 에 미리 게1 3②

시 여 다

시간4 ( )

도 시간 도 여 게시 다

등 등5 ( )

도 료 시 는 는 도①

지에 등 후

등 에 사 도②

사 료6 ( )

도 료 시 에 사 료는 도

7 ( )

는 다 각 여 는 니 다

도 료 시 상 리1 lsquo rsquo

도 료 시 훼 는2 middot

지 가 닌 곳에 식 거 담3

우는

도 보 등 보 검색열4 middot

그 에 도 질 지 여 도5

여 게시 사 는

질 지8 ( )

도 다 거 도①

질 게 우 가 는 에 여는 도

도 가 각 어느7②

에는 지 게 거 도

료9 ( )

도 료는 다 각 경우 다①

상 도 간에 료 는 것 말1 (

다 등 다 도 과 여 경우)

공 원 공 상 는 경우2

그 에 도 다고 는 경우3

가능 도 료 는 도②

는 에 다

변상10 ( )

가 도 료 시 럽 거 거①

못 쓰게 거 어 린 경우에는 변상 여

도 에 변상 여 게시1②

여 다

등 규 에 것 에 도11 ( )

료 시 등에 사

도 다

립 도- (httpwwwnlgokr)

zb45) 도서 장의 리 있는 조항으 적절하 않

은 것은

① ② ③ ④ ⑤

년 학 간고사 대비2013 2 현대고 대비

ECN-0102-2013-001-000076193

1 ( )

사가 공 는lsquo rsquo

과 여 사 원과 리

사 타 사 규

니다

개 보 보7 ( )

사는 보통신망 등 계 는 에lsquo rsquo lsquo rsquo

원 개 보 보 니다 개lsquo rsquo

보 보 사 에 는 사 개lsquo rsquo

보 취 니다 다만 사는 다 lsquo rsquo

사 계 통 공 는 경우 원 lsquo rsquo

등 개 보 당 사에 습니lsquo rsquo

원 리에8 (lsquo rsquo lsquo rsquo lsquo rsquo

)

원 에 리lsquo rsquo lsquo rsquo lsquo rsquo①

원에게 가 도 여 는lsquo rsquo 3

니다

사는 원 가 개 보 우 가lsquo rsquo lsquo rsquo lsquo rsquo②

거 사 경우 는 미 에 어 거 lsquo

사 사 운 우 가 는 경우 당rsquo lsquo rsquo

습니다lsquo rsquo

원 가 도 거lsquo rsquo lsquo rsquo lsquo rsquo 3③

가 사 고 지 경우에는 시 사에lsquo rsquo

통지 고 사 내에 니다lsquo rsquo

경우에 당 원 사에 그 사실3 lsquo rsquo lsquo rsquo④

통지 지 거 통지 도 사 내에 지 lsquo rsquo

생 경우 사는 지지 습니다lsquo rsquo

사10 (lsquo rsquo )

사는 과 지 미lsquo rsquo①

에 는 지 계 고

공 여 다 여 니다lsquo rsquo

사는 원 게lsquo rsquo lsquo rsquo lsquo rsquo②

도 개 보 신 보 포 보 보 시( )

갖 어 개 보 취 공시 고

니다

사는 과 여 원lsquo rsquo lsquo rsquo③

견 만 당 다고 경우에는

리 여 니다 원 견 만 사 lsquo rsquo

에 는 게시 거 우편 등 통 여

원에게 리 과 결과 달 니다lsquo rsquo

원11 (lsquo rsquo )

원 다 여 는 니다lsquo rsquo ①

신청 는 변경 시 허 내 등1

타 보 도2

사가 게시 보 변경3 lsquo rsquo

사가 보 보 컴퓨 그4 lsquo rsquo (

등 등 신 는 게시)

사 타 등 지 재산 에5 lsquo rsquo 3

사 타 상 거 업6 lsquo rsquo 3

는 폭 시지 상 타 공7 middot middot

에 는 보 에 공개 는 게시 는lsquo rsquo

사 동 없 리 사8 lsquo rsquo

타 거 당9

게시15 (lsquo rsquo )

원 내에 게시 는 게시 게재 는lsquo rsquo lsquo rsquo lsquo rsquo

경우 원 사가 게시 복 lsquo rsquo lsquo rsquo lsquo rsquo middot middot

등 태 언 등에 공 는

것 내에 다 원 본 게시 등 lsquo rsquo lsquo rsquo

크 능 등 여 복 는 등 태

는 것 동 것 니다

- (wwwnavercom)

zb46) 위 은 인터넷 포털사이트의 회 가입을 위한 이

약 의 일 이다 이 약 을 만드는 과정에서 생각한

내 으 적절하 않은 것은

개 보 보 가 지에 별 눠①

겠어

원 가 만들게 에②

시 주어 겠어

원들 게재 게시 다 원 크 다③

는 것 지

④ 원 지 는 뿐만 니 사가 지 는

도 께 달 지

리에 가 생 경우 사가⑤

에 다는 도 듯

1 ( )

사가 공 는lsquo rsquo

과 여 사 원과 리

사 타 사 규

년 학 간고사 대비2013 2 현대고 대비

ECN-0102-2013-001-000076193

니다

개 보 보7 ( )

사는 보통신망 등 계 는 에lsquo rsquo lsquo rsquo

원 개 보 보 니다 개lsquo rsquo

보 보 사 에 는 사 개lsquo rsquo

보 취 니다 다만 사는 다 lsquo rsquo

사 계 통 공 는 경우 원 lsquo rsquo

등 개 보 당 사에 습니lsquo rsquo

원 리에8 (lsquo rsquo lsquo rsquo lsquo rsquo

)

원 에 리lsquo rsquo lsquo rsquo lsquo rsquo①

원에게 가 도 여 는lsquo rsquo 3

니다

사는 원 가 개 보 우 가lsquo rsquo lsquo rsquo lsquo rsquo②

거 사 경우 는 미 에 어 거 lsquo

사 사 운 우 가 는 경우 당rsquo lsquo rsquo

습니다lsquo rsquo

원 가 도 거lsquo rsquo lsquo rsquo lsquo rsquo 3③

가 사 고 지 경우에는 시 사에lsquo rsquo

통지 고 사 내에 니다lsquo rsquo

경우에 당 원 사에 그 사실3 lsquo rsquo lsquo rsquo④

통지 지 거 통지 도 사 내에 지 lsquo rsquo

생 경우 사는 지지 습니다lsquo rsquo

원에 통지9 (lsquo rsquo )

사는 특 다 원에게 통지 경우lsquo rsquo lsquo rsquo

공지 게시 통 상 게시 개별 통지에7

갈 습니다

사10 (lsquo rsquo )

사는 과 지 미lsquo rsquo①

에 는 지 계 고

공 여 다 여 니다lsquo rsquo

사는 원 게lsquo rsquo lsquo rsquo lsquo rsquo②

도 개 보 신 보 포 보 보 시( )

갖 어 개 보 취 공시 고

니다

사는 과 여 원lsquo rsquo lsquo rsquo③

견 만 당 다고 경우에는

리 여 니다 원 견 만 사 lsquo rsquo

에 는 게시 거 우편 등 통 여

원에게 리 과 결과 달 니다lsquo rsquo

원11 (lsquo rsquo )

원 다 여 는 니다lsquo rsquo ①

신청 는 변경 시 허 내 등1

타 보 도2

사가 게시 보 변경3 lsquo rsquo

사가 보 보 컴퓨 그4 lsquo rsquo (

등 등 신 는 게시)

사 타 등 지 재산 에5 lsquo rsquo 3

사 타 상 거 업6 lsquo rsquo 3

는 폭 시지 상 타 공7 middot middot

에 는 보 에 공개 는 게시 는lsquo rsquo

사 동 없 리 사8 lsquo rsquo

타 거 당9

원 계 규 내lsquo rsquo lsquo②

여 공지 주 사 사가 통지 는rsquo lsquo rsquo

사 등 여 타 사 업 에 lsquo rsquo

는 여 는 니다

- (wwwnavercom)

zb47) 위 약 의 조항에서 같은 제점을 하lt gt

고 있는 조항은

lt gt

제휴 회사에 회 의 아이디 개인 정 를 전송할 있도

한 조항은 고객에게 당한 조항이다

1 7 8① ② ③

④ 9 ⑤ 10

립 도 규

1 ( )

규 립 도 립 어린 청 도(

포 다 료 시 열 시 말) (

다 에 사 규 립 도)

편 진 다

2 ( )

규 립 도 도 다 에( lsquo rsquo )

고 는 도 에 도lsquo rsquo 2 2

료 에 여 다 다만 특 료 귀

료 등 료 에 사 립 도

도 다 다( lsquo rsquo )

3 ( )

도 다 각 같다①

공 공 다만 연1

연 간 다

년 학 간고사 대비2013 2 현대고 대비

ECN-0102-2013-001-000076193

매월 째 째 월2

도 도 리 그 사3

가 다고 는

도 에 미리 게1 3②

시 여 다

시간4 ( )

도 시간 도 여 게시 다

등 등5 ( )

도 료 시 는 는 도①

지에 등 후

등 에 사 도②

사 료6 ( )

도 료 시 에 사 료는 도

7 ( )

는 다 각 여 는 니 다

도 료 시 상 리1 lsquo rsquo

도 료 시 훼 는2 middot

지 가 닌 곳에 식 거 담3

우는

도 보 등 보 검색열4 middot

그 에 도 질 지 여 도5

여 게시 사 는

질 지8 ( )

도 다 거 도①

질 게 우 가 는 에 여는 도

도 가 각 어느7②

에는 지 게 거 도

료9 ( )

도 료는 다 각 경우 다①

상 도 간에 료 는 것 말1 (

다 등 다 도 과 여 경우)

공 원 공 상 는 경우2

그 에 도 다고 는 경우3

가능 도 료 는 도②

는 에 다

변상10 ( )

가 도 료 시 럽 거 거①

못 쓰게 거 어 린 경우에는 변상 여

도 에 변상 여 게시1②

여 다

등 규 에 것 에 도11 ( )

료 시 등에 사

도 다

립 도- (httpwwwnlgokr)

zb48) 다음 정 리 의 의 으 볼 때 가장

이 적인 것은

도 시간 도 여 게시 다①

등 에 사 도②

가능 도 료 는 도 는③

에 다

④ 도 에 변상 여 게10 1

시 여 다

⑤ 도 가 각 어느7

에는 지 거 도

zb49) 를 참고하여 이 어의 성격을 설 한lt gt

것으 적절하 않은 것은

① 보 에 는 어 시 상 고 어 시lt gt lsquo rsquo

에 보여주고 다

② 진 어 어원에 견 고 다

에는 타 어 들어가는 것 다 lsquo rsquo

③ 에 들어갈 말 각각 고 어 어 신 어~

들 언어는 질 격 강 통 없었다

④ 시 우리 에 가 었지만 지 계

과 달리 들 통 사 달 어 웠

년 학 간고사 대비2013 2 현대고 대비

ECN-0102-2013-001-000076193

⑤ 크 몽골 만주 공통어가 우리 어 같

계열에 다는 에 사 특 짐

가( )

善化公主主隱 공주님

他密只嫁良置古 몰 결 고

薯童房乙 맛

夜矣卯乙抱遣去如 에 몰 고 가다

( )

始汝 會隱日恚見隱扐 만 에 본

恥隱汝衣淸隱笑 맑 웃

고 시 여 공 크다 만 다[ ] ( ) ( ) ( ) ( )始 汝 會扐

내다 에 보다 견( ) ( )恚 見 다( )隱

럽다 맑다 청 웃( ) ( ) ( ) ( )恥 衣 淸 笑

zb50) 위의 나 를 함 고 음에 답하( ) lt gt

보lt gt

( )素那或云金川 白城郡蛇山人也

운 사산

는 고 다 는( )[ ( ) ] (素那 金川 白城

사산 사 다) ( ) 郡 蛇山

삼 사- lsquo rsquo 47

에 제 된 단어 의 표 리를 조건(1) lt gt ( ) lt gt

에 맞게 서 하

건lt gt

lsquo 었고 었다 태rsquo

에 제 된 단어 동일한 표 리에(2) lt gt ( )

의해 적은 것을 나 에서 찾아 조건 에 맞게 서 하( ) lt gt

건lt gt

에 당 는 각각( ) 개 쓸 것2 단

당 는 가 여러 개 어도 개만 쓸 것 각2

개 과 도 쪽에 개만2 2

드시 지 것( )

과 동 원리 것lsquo 고

과 동 원리 것 다rsquo

태 것

가( )

素那(或云金川) 白城郡蛇山人也

소나 또는 천 이라 한다 는 성 사( ) ( ) ( )素那 金川 白城郡〔 〕

산 사람이다 현대어 풀이( ) ( )蛇山

나( )

紫布岩乎希 회

執音乎手母牛放敎遣 자 손 암쇼 노히 고

吾 不喩慙 伊賜等肹 肹 나 안디 리샤

花 折叱肹 可獻乎理音如 고 것거 도림다

다 향찰은 리말을 리 으 적은 표 이었 만 생( )

은 고 대를 넘 하고 끊어 고 말았다 랜 세

동안 갈고 닦아 체계적이었던 향찰 표 이 사라졌

을 인은 크게 두 가 나누어 생각해 볼 있다

하나는 족 사회의 한 선호도에서 찾을 있다 라 때

향찰은 주 족 계 에서 사 했을 것으 인다 한 을

알 하고서는 한자를 활 하여 리말을 리 으 표

하 란 가능하 때 이다 런데 족들은 간이 흐

를 향찰과 같은 리 표 을 익혀 사 하 다는

아 한 을 대 사 하는 쪽을 선호하게 되었다 더 이

고 초에 인재 등 을 위해 과거제도가 행되 서 한 선

호도가 더 높아졌고 결 향찰은 소 되고 말았다

또 다른 가능성은 한 어의 특성에서 찾을 있다

터 한 과 일 세 나라는 한자 화 에 속해 다

당연한 이야 겠 만 표의 자인 한자는 어를 표 하

에 매 적절하다 어의 음절은 성 ( ) ( )聲母 韻母

이 어 고 여 에 성조가 추가되어 최종 소리가 결정된

다 래서 어는 단음절을 하나의 한자 표 하 된

다 에 초성 성 종성의 세 가 소가 하나의 음절

년 학 간고사 대비2013 2 현대고 대비

ECN-0102-2013-001-000076193

을 이 는 한 어는 음절 조가 잡하고 음절의 가 많아

서 한자 차 만으 한 어의 소리를 만족 럽게 표 할

없었다 를 들어 한 어에서는 어 니 같이 음절 lsquo rsquo

이 어 단어가 얼마든 있으나 어는( ) 複數音節

자 하나 나타내 만이다lsquo [m ]rsquo 母 ǔ

한편 일 어의 표 은 핵 적 단어는 한자 적고 토는

가나라는 일 의 자 적는 이다 적인 의 를 나

타내는 은 표의 자인 한자 적고 적 계를 나

타내는 토는 표음 자 적는 셈이니 자세히 살펴

리의 향찰 표 을 쏙 빼닮았음을 알 있다 한 어 같

은 착어이 서도 일 어에만 향찰과 유사한 표 이 살아

남은 것은 일 어의 특 때 이다 일 어는 하나의 자음과

음의 결합으 음절을 이 고 침이 거의 없는 음절 언어

이다 이러한 음절의 특색에다가 토가 달한 착어라는 점

이 향찰과 유사한 표 이 살아남을 있는 비결이었다

하 만 같은 착어라도 다양한 음소 침이 달한 한

어는 향찰 표 하는 데 근 적으 한계가 있었다

zb51) 다 하여 의 행에 대한 탐 한 결과( ) lt gt 2

않은 것은

보lt gt

善花公主主隱 공주니믄 공주님( )

----------------------------------------

-

他密只嫁良置古 그 지 얼어 고 몰 결(

----------------------------------------

-

薯童房乙 맛 맛( )

夜矣卯乙抱遺去如 몰 고 가다 에 몰 고(

가다)

주동 역 동- (薯童謠『 』

에 2 ( )他密只嫁良置古

얼다 시집가다 결 다 말 lsquo rsquo

① 실질 미 지니고 므 타 타lsquo ( )rsquo lsquo [ ]

② 에 실질 미 타내고 지 는lsquo rsquo lsquo [ ]rsquo lsquo [ ]密只 密 只

계 타내는

③ 얼어는 실질 미 포 고 므 가lsquo rsquo lsquo [ ]rsquo嫁

것lsquo [ ]rsquo 良

④ 고 어간 는 실질 미 지니고 므lsquo rsquo lsquo -rsquo

것lsquo [ ]rsquo 置

⑤ 고 어미 고는 계 타내고 므lsquo rsquo lsquo- rsquo

고 것lsquo [ ]rsquo 古

가( )

엉 훈 민middot middot middot middot middot世 宗 御 製 訓 民 正 音

말 미 듕 귁에 달middot middot middot middot middot middot middot middot中 國 文 字

니 런middot middot middot middot middot middot 어린middot middot middot middot百 姓

니 고 도 내 들middot middot middot middot middot middot middot middot middot 시러middot

펴 몯middot 미middot middot 니 내middot middot middot middot middot middot middot middot 爲

어엿middot 겨 새middot middot middot 믈여듧middot middot middot middot字 니middot middot middot

사 마다 니겨 킈 middot middot middot middot middot middot middot middot middot便 安

고 미니middot middot middot middot

본 는 상( ) (象

원리에 만들어진 본) ( )形 ㄱ ㄴ ㅁ ㅅ ㅇ

에 는 가 원리에( )加劃

그리고( )ㅋ ㄷ ㅌ ㅂ ㅍ ㅈ ㅊ ㆆ ㅎ

쓰는 병 원리에 만들어진( )竝書

마지막 체( ) ( )異體ㄲ ㄸ ㅃ ㅆ ㅉ ㆅ

ᅀ 다 상 원리에 ㅇ ㄹ

지 는 삼재 상 본 본( ) ( ) ( 天地人 三才

탕 므림과 림에 ) (初ㅡ ㅣ

재)( ) ( )( )出字 再出字ㅗ ㅏ ㅜ ㅓ ㅛ ㅑ ㅜ ㅕ

병 그리고 들 에 다시( )ㅘ ㅝ ㅣ

( )ㅣ ㅢ ㅚ ㅐ ㅟ ㅔ ㆉ ㅒ ㆌ ㅖ ㅙ ㅞ

zb52) 가 에 대한 설 으 르 않은 것을( ) 두 고르

① 어쓰 규 지키고 다

② 리 고 다

③ 말 미 미 등 어 사 다lsquo rsquo

④ 개 지 다

년 학 간고사 대비2013 2 현대고 대비

ECN-0102-2013-001-000076193

⑤ 어 원 에 가 도 고 다

엉 훈 민世 宗 御 製 訓 民 正 音

말 미 듕귁에 달 니

런 어린 니 고 도middot

내 들 시러 펴 몯 미 니middot

내 어엿 겨 새 믈여듧

사 마다 니겨middot 킈 고

미니

훈민 언 본- lsquo rsquo 5 (1459 )

zb53) 위의 에 대한 현대어 풀이가 르~ 않은 것

① 우리 말 과 달

② 어리 말 고 는 것 어도

③ 신 생각 마 껏 펼 는 사 많다

④ 게 생각 여

⑤ 사 마다 게

zb54) 훈민정음 언해 에는 한 을 창제한 동 가 드러나

있다 훈민정음 창제의 정 과 내 이 잘 연결된 것

① 주 신 말 미 듕귁에 달

② 민 신 내 어 겨

③ 신 뻔 킈 고 미니

④ 실 신 사 마다 니겨

⑤ 귀 신 계 주 는 훈민 신과 거리가

가 엉 훈 민( ) middot middot middot middot middot世 宗 御 製 訓 民 正 音 

말 미 귁에 中 國 달 文 字

니 런 어린 니 百 姓

고 도 내 들 시러 펴 몯

미 니 내 어엿 爲 겨 새

믈여듧 니 사 마다 니 字

겨 킈 고 미니 便 安

훈민 언 본- lsquo ( )rsquo ( ) 5 (1459 )訓民正音 世祖

( )

[ 1 ]

동 룡 샤 마다 복( ) ( ) ( )海東 六龍 天福

시니 고 동( ) ( )古聖 同符 시니

[ 2 ]

매 니 곶 여

미 므 니 그 내 러

가 니

[ 125 ]

우 미리( )千世 샨( )定 에( )漢水北 累仁

누 개 샤 복 업 시니( ) ( ) 開國 卜年

신( )聖神 니 샤도 경 근민 샤 욱( )敬天勤民

드시리 다

님 쇼 산 가( ) ( )洛水 山行

미드니 가

어 가- lsquo ( )rsquo 27龍飛御天歌

다 우리신 니쓰고 다만 만 쓰( )

거 샹 귀쳔 다보게 러 귀

여 쓴 도 신 보 가 고 신 에

말 어 보게 각 에 사 들

고 본 몬 능통 후에

죠 죠 니

드 도 만 공 에 사

드 미 죠 고 고 여 보 죠

보다 얼마가 거시 어신고 니 첫

가 죠 니 죠

민 들 어 신 샹

귀쳔 도보고 어보 가 만 늘

고 폐 에 만쓴 죠 민

도 러보지못 고 보니 그게 엇지

심 니 리 보 가 어 운건 다

니 쳣 말마 지 니 고 그

쓰 에 가 우 지 지

몰 거 본후에 가 어 지

고 그니 쓴편지 쟝 보

년 학 간고사 대비2013 2 현대고 대비

ECN-0102-2013-001-000076193

쓴것보다 듸 보고 그 마 니 쓴 고

어 못

그런고 에 리 과 가

만 쓴 못 민 말만 듯고

고 편 그 못 보니 그사 단

병신 못 다고 그사 식 사

니 만 고 다 과 그사

만 고 다 과 업 사 보다 식 고

죠 도 고 각 과

견 고 실 직 귀쳔 간에 그

고도 다 것 몰 귀죡 보다

사 우리 신 귀쳔 다 업

시 신 보고 과 지 게 랴

시니 샹 귀쳔 간에 우리 신 걸

간 보 새지각과 새 걸 미리

독립신- lsquo (1896)rsquo

zb55) 친 어 나의 제 장( ) 2 매 함축적

의 가 가장 유사한 것은

① 지 눈 내리고 매 득 니 내 여 가

사- lsquo rsquo

② 도 어 리듯 그 게 어 다

주 사- lsquo rsquo

③ 눈 살 다 죽 어 린 과 체 여

눈 새벽 지 도 살 다

눈- lsquo rsquo

④ 삶 근심과 고단 에 돌 거니는 여 거 는

여 리 내린 살가지 에 눈 리 눈 리

택 그 생 에- lsquo rsquo

⑤ 늘 러 고 러

청룡 룡 어 개 루 우

신경림 계- lsquo rsquo

zb56) 친 를 위 가 나 에 나타난A B ( ) ( )

세 어의 특 에 의거하여 세 어 표 하

그 산 고 공 도 맑지만

A

주변에 쓰 리는 어리 사 많다

B

건lt gt

식 가 에 타 어 특징에( ) ( )

거 과 어쓰 는 고 지 말 것

A

B

zb57) 가 의( ) 달 아ㆍ 다 의 ( ) 나셔에서 알 있는

세 어 개화 어의 특 을 비 하여 조건 에lt gt

맞게 서 하

건lt gt

어에 는lsquo 개

어에 는 다 태rsquo

zb58) 은 가 는 다 에 나 는 절lt 1gt ( ) lt 2gt ( )

일 를 췌한 것이다 의 의 가 lt 1gt (1)~(2)

유사한 말을 에서 찾아 쓰lt 2gt

보lt 1gt

런 (1) 어린 니 고百 姓

도 내 들 시러 펴 몯 미

사 마다 (2) 니겨 便 安

킈 고 미니

보lt 2gt

죠 고 고 여 보 죠

보다 얼마가 거시 어신고 니 첫 가

죠 니 죠 민

들 어 신 샹 귀쳔

도보고 어보 가 만 늘 고

폐 에 만쓴 죠 민 도

러보지못 고 보니 그게 엇지 심

니 리

년 학 간고사 대비2013 2 현대고 대비

ECN-0102-2013-001-000076193

lt 1 gt

동 룡 샤 마다 복 시( ) ( ) ( )海東 六龍 天福

고 동 시니( ) ( )古聖 同符

lt 2 gt

(A) 매 니 곶

여 니

미 므 니 그 내

러 가 니

lt125 gt

우 미리 샨 에( ) ( ) ( ) 千世 定 漢水北 累

누 개 샤 복 업 시 니( ) ( ) 仁開國 卜年 聖

신( ) 神 니 샤도 경 근민 샤( ) 敬天勤民

욱 드 시 리 다

님 쇼 산 가 ( ) ( )洛水 山行

미드니 가

- lt gt龍飛御天歌

zb59) 장과 내 상 유사한 성격의 조는125

① 뫼 고 고 고 고

어 그린 많고 많고 고 고

어 러 는 울고 울고 가느니

도 견- lt gt

② 강 에 드니 몸 다

그믈 고 가니

뒷 뫼 엄 언 니( )藥

-

③ 말 없는 청산 태 없는 다

값 없는 청 없는 월

에 병 없는 몸 별 없 늙 리

-

④ 가마귀 골에 가지 마

낸 가마귀 새

청강에 것 시 몸 러 가( ) 淸江

-

⑤ 진 골에( ) 白雪

가 매 는 어느 곳에 었는고

에 갈 곳 몰( ) 夕陽

색-

zb60) 위 에 나타난 세 어의 특 으 적절하 않은

것은

① 룡 어 주격 사에 당 는 가 사( ) lsquo rsquo六龍

고 다

② 샤 어에도 어 주체 쓰 다

는 것 다

③ 매 어 달리 사 택에 어

가 지 지지 고 다

④ 므 원 상 직 어 지 다

⑤ 드시리 다 주체 과 상 께 사

고 다

수고 하셨습니다hearts hearts

년 학 간고사 대비2013 2 현대고 대비

ECN-0102-2013-001-000076193

보닷컴에 공 는 별 보는 고등

들 여 주 는

들 습니다 슷 동 지

가 복 는 것 도가

니 복 여 습 시고 거 시

니다

정답 해설

1) 정답[ ] ④

해설 다른 것은 두 특정 업이나 단 내에서 사[ ]

하는 일종의 은어 사회 언에 해당한다 러나

는 언이 아니라 단과대학을 여서 단대 사lsquo rsquo lsquo rsquo lsquo④

대학을 여서 사대라고 한 말에 해당하 일rsquo lsquo rsquo

사회에서도 널리 쓰이 사회 언이라 할

없다

2) 정답[ ] ⑤

해설 사회 언은 같은 단 내에서 쓰이는 언어이[ ] lsquo rsquo

동일 단끼리는 단결 과 친 감을 형성하는

능을 하 리적 안감이 일어나 않는다

3) 정답[ ] ③

해설 사람이라는 차 적 표현에 대한 대안적 표현이[ ]lsquo rsquo

인 아내 처 등으 볼 있다lsquo rsquo

4) 정답[ ]⑤

해설 남성은 주 격 체를 사 한다[ ]

5) 정답[ ] ⑤

해설 흑인은 검다라는 뜻을 가 고 있을 뿐 인[ ]lsquo rsquo lsquo rsquo lsquo rsquo

다 열등한 뜻을 내포하 않는다

6) 정답 살 색 첫 작품[ ] - -

해설 살색 혹은 킨색은 한 인의 피 색을 뜻[ ] lsquo rsquo lsquo rsquo

하는 것으 인종 차 을 추 고 출 이주민

의 평등 을 침해할 있어 년 표 이2005

살 색으 이름을 꾸었다 처녀작은 처녀라lsquo rsquo lsquo rsquo lsquo rsquo

는 단어가 가 고 있는 곡된 성 인 을 한 것

으 첫 작품정도 꾸어 사 하는 것이 좋다lsquo rsquo

7) 정답[ ] ⑤

해설 호는 아들에게 해체를 사 하고 있다[ ] ① ②

장 을 성하는 청자는 자 의 아 느리 아lsquo

들 세 이다 호는 아 느리에게 해rsquo ③

체를 사 하고 있다 호가 느리 아 에게 ④

사 한 해 체 아들에게 사 한 해체는 두 비lsquo rsquo lsquo rsquo

격 체에 해당한다 호는 자 의 아랫사람인 ⑤

느리에게 아들과 마찬가 해체를 사 하는 것이

상 이 만 임 을 한 느리에게 고마 과 쁨

존 의 표 를 하 위해 자 의 아 에게 말하듯

해 체를 사 하고 있다

8) 정답[ ] ③

9) 정답[ ] ⑤

10) 정답[ ] ①

해설 청자 할아 가 장의 주체 아 다 높을[ ] ( ) ( )

경 에는 압존 에 의해 장의 주체를 높이 않는lsquo rsquo

다 러 아 서가 아닌 아 는으 계 lsquo rsquo lsquo rsquo lsquo

니다 가 아닌 있 니다 표현하는 것이 르rsquo lsquo rsquo

11) 정답 당이 당을 쫒았다 당이[ ]

당에 다

해설[ ]

12) 정답[ ] ⑤

해설 서 다른 높임표현을 통해 청자에 대해 리[ ] ⑤

적 거리감을 나타내는 인 은 이 아니라 현정이

다 가 에서 현정은 에게 해 체를 사 함으 써 ( )

친근감을 드러낸다 나 에서 연 을 게을리하는 역 ( )

도 들 때 에 화가 난 현정이 선생님에게 항의하

는 장 에서는 하 체를 사 하여 리적 거리lsquo rsquo

가 어졌음을 나타내고 있다

13) 정답[ ] ①

해설 는 는 얼 빛이 날과 어찌 다르 고[ ] lsquo rsquo

라는 뜻으 전과 달리 임이 화자를 않고

있음을 알 있다

14) 정답 달리 후 가 있다 이를 통해 경[ ] lt gt

쾌한 음악성을 형성하고 노 젓는 상황을 체적으

형상화하는 역할을 한다

15) 정답[ ] ①

16) 정답[ ] ⑤

해설 다 의 자연은 를 성찰하게 하는 대상[ ] ( )⑤

이자 정의 대상이다 의 자연은 자 의 상황과 ⑤

처 를 드러내는 경으 서의 역할을 하 이

이 없다

17) 정답[ ] ③

해설 는 빈천 을 해결하고자 했으나 강산[ ] lsquo ( )rsquo 貧賤③

과 풍 을 달라는 에 거절하 다고 함으 써 자

연에 대한 애정을 드러내고 있으 는 않는

임에 대한 망을 개에게 전가 켜서 임에 대한 리

을 드러내고 있다

18) 정답[ ] ③

년 학 간고사 대비2013 2 현대고 대비

ECN-0102-2013-001-000076193

19) 정답[ ] ⑤

해설 고상한 음악가의 이름을 리말 꽝 럽[ ]

게 꿈으 써 언어유희를 통해 음을 유 하고 있

다 이는 고상한 척하는 총 를 비꼼으 써 비판적

태도를 드러내는 것이 대상을 꽝 럽게 표현

하여 총 의 허 과 사치를 풍자하고 있다

20) 정답[ ] ⑤

해설 는 작품 속 경에 대한 설 이 드러나는 것이[ ]

서 자의 주 적인 견해가 접적으 드러나는 것이

아니다

21) 정답[ ] ⑤

22) 정답[ ] ②

23) 정답[ ] ④

24) 정답[ ] ①

해설 적강 티프는 주인공의 비 한 출생이나 능[ ] ①

과 이 있는 것으 조정의 능함을 풍자하는lsquo rsquo

것과는 거리가 다

25) 정답 픔 나[ ] ( )

해설 의 음악은 고통 는 사람들을 위 하고 아픔[ ] lsquo rsquo

을 치유해 주는 능을 한다고 할 있다 의 lt gt

픔 도 소 된 이 과 더 어 살아가는 따뜻한 마음lsquo rsquo

을 상 한다

26) 정답[ ] ⑤

해설 에게 선천적으 주어 각 장애라는 역경[ ]

은 의 이라는 가사 연 을 있다lsquo rsquo

27) 정답[ ] ④

해설 는 장 란 선 에게 은 개인적인 인상을[ ]

소녀 장정 등으 표현한 것이다lsquo rsquo

28) 정답[ ] ②

해설 담자가 피 담자의 언어적 표현이나 비언어[ ]②

적 표현 하 독자는 담의 위 나 피

담자의 감정 상태를 알 있다 이를 통해 독자는

담 상황을 더 생생하게 느낄 있고 피 담자

를 더 잘 이해할 있게 된다

29) 정답[ ]③

해설 일상생활과 역도 선 서의 성과에 된 것에서[ ]

역도를 하 서 겪는 어 과 내적 고민으 화제를

전화하 위한 것이다

30) 정답[ ] ①

해설 릿속에 새겨 넣듯 이 억되도 함 세상[ ] ② ③

살이가 힘들고 고생 러 속 하여 자유를 ④

가 없는 고통의 상태를 비유적으 이르는 말

적의 침입을 막 위해 쌓은 축 켜야 할⑤

대상을 비유적으 이르는 말이다

31) 정답[ ] ④

해설 이 의 종류는 전 으 인 사건 경[ ] lsquo

비평을 성 소 삼는다rsquo

32) 정답[ ] ④

해설 근은 삼대독자 태어났음을 에서 확인할[ ]

있다 형제들과의 담은 이뤄 가 없다

33) 정답[ ] ⑤

해설 근은 가난에도 하고 화가를 꿈꾸었다[ ] (3

단 또한 다른 화가 망생들은 정 육을)

위해 상 학 학 해 유학 에 랐 만

근은 다른 을 찾아야 했다 단 세에(5 ) 18

근은 조선 전람회에 입선하 다 단 의(6 )

만종은 인간과 자연이 엮어 가는 경건한 조화 을lsquo rsquo

나타낸다

34) 정답[ ] ①

해설 근이 속에서도 창작활동을 추 않고[ ]

하는 닭은 은 세상과 타협할 르는

근이 세상의 이해를 하 위한 가장 떳떳한 단

이 때 이다

35) 정답[ ] ⑤

해설 전 은 서 자의 주 적인 평이 리는 것이[ ]

만 위 제 은 인 이 살았던 대 사회적 경

을 통해 객 적인 인 의 을 제 하고 있다

36) 정답[ ] ⑤

해설 전 은 인 사건 경 비평이라는[ ] lsquo rsquo⑤

성 이 어져 있다

37) 정답[ ] ①

해설 이 은 동양인과 서양인의 사고 에 차이가[ ]

있다는 것을 대조를 통해 설 하고 있다 또 쓴이

의 제자가 축 경 를 러 가서 경험한 일화를

통해 동양인이 서양인에 비해 주 상황에 더 많은

주의를 인다는 주장을 뒷 침하고 있다

38) 정답[ ] ④

39) 정답[ ] ②

40) 정답[ ] ②

41) 정답[ ] ④

42) 정답[ ] ③

43) 정답[ ] ④

44) 정답 도서 의 휴 일 도서 의 이 간 도서의[ ]

해설 도서 장은 임의 정한 휴 일과 도서 이[ ]

간 도서의 상 등을 게 할 의 가 있다

년 학 간고사 대비2013 2 현대고 대비

ECN-0102-2013-001-000076193

45) 정답[ ] ①

해설 제 조의 정 휴 일 의 휴 일의 사전 게[ ] 3

는 도서 장의 의 조항에 속한다

46) 정답[ ] ①

해설 개인 정 호 의 를 제 하 했 만 항[ ]

나눠서 제 하 않고 대 나열하고 있다

47) 정답[ ] ②

해설 제 조의 내 을 회사는 다른 회사 협[ ] 7 lsquo

계약을 통해 서비 를 제공하는 경 회 의 아이디

등 개인 정 를 해당 회사에 전송할 있다는 내rsquo

이 있으 의 제점을 제 할 있다②

48) 정답[ ] ④

해설 는 도서 장의 의 에 해당하고 나 는 도[ ] ④

서 장의 리에 해당한다

49) 정답[ ] ③

50) 정답 은 음독으 적었고 은 훈독으 적었[ ] (1)

다 과 동일한 표 리 적은 것은 이고 (2) ce

과 동일한 표 리 적은 것은 이다ab

51) 정답[ ] ③

52) 정답[ ] ①②

53) 정답[ ] ③

54) 정답[ ] ③

55) 정답[ ] ①

56) 정답 른 죠코 어린 노 하니라[ ] A B

57) 정답 세 어에서는 활 형이 칙적으[ ] lsquo rsquoㄹㅇ

나타났 만 개화 어에서는 활 형이 쓰 다 lsquo rsquo ㄹㄴ

58) 정답 호 가 흔[ ] (1) (2)

59) 정답[ ] ④

60) 정답[ ] ③

Page 19: 현대고대비 국어 - chamsoriedu.com 「콘텐츠산업진흥 법」외 에도 저작권 의하여 ... 다른주체에게어떤동작을하도록만드는것을나타내는

년 학 간고사 대비2013 2 현대고 대비

ECN-0102-2013-001-000076193

은 것은

① 질 담 상 보여 주 것 다1

② 질 담 과 그에 삶 태도 보여2

주 것 다

③ 질 역도 겪는 어 움에 역도3

과 것 다

④ 질 같 연 여 갖는 고민 는지 말4

주 는 것 다

⑤ 질 역도가 과 고 운동 는 것5

담 가 말 주 는 것 다

가 만진 것 다( ) 3

감 달 다고 다 억 에( ) 音感

지워 지만 당시 청 탁 리도

다고 다 드럼 웠다 4

에 갈 마다 드럼 는 리가 신 게 들

다고 다 눈 볼 가 없 니 엔ldquo

는 는 님 틱 에 여 주

다 드럼과 연 맺 과 들 주었다rdquo

식 누 가-

고 싶어

역( ) 도가 말 단 식 운동 니

가 내는 만 클 업에 보ldquo

그러니 만 쓰는 식 운동 니다 만

다고 거운 것 들 는 건 니거든 연

도 고 가지 동 에 도 여러 가지 복

들 시 는 상 상

드는 상 에 맞춰 실 에 는 여러

펼쳐집니다rdquo

략( )

늘 에 는 어 만 것 같

가 에 사 고 사 사ldquo

겠어 든 에 가 경 만 고

울 는 사 겠어 rdquo

보식 역도 여 미-

다 가 운 는 어 어( ) ldquo rdquohelliphellip

월 새벽 시 태 없 거웠고1965 5 6 1

는 없 그 병원에 퇴원 집

가는 마지막 마 고 마 내 거 다

가 죽 간신 에 실 다 사는 어느5 lsquo

가 죽 는 말 가 식 다 신rsquo

상에 각 시키는( )刻印 에 실

어느 가는 후 민 가가 근 었다lsquo rsquo

는 간 과 진실 그 다는( ) ldquo

에 단 평 견 가지고 다 내

가 그리는 간상 단 고 다 지 다 는 그들

가 에 는 평 지 니 그리고 어린

들 미지 겨 그린다rdquo

마 근 간 과 진실 그리고 싶어( )

가 다 근에게 그것 진리 다 거 다 없

거 고 다 없 는 것 진리

다 근 진리는 후 쪽 었다 신산(辛酸 삶)

었 질곡(桎梏 역사 에 지냈)

가 눈에 든 것 료 단 료 게

보 것 었다 그것 그 에 겨우겨우

슬 슬 생 어가는 간들 었

다 리 과 단 리 고리에 검

마 없 거리 돌

상 것 없는 등 근에게 상에

과 진실 엄 다는 사실 리는 가( )儼存

실 고 가 과 역경 에 도 근 내

포 없었 후 보루(堡壘 다 도)

도 간 근365

여 시 것 다

월 강원도 림리에( ) 1914 2 21

삼 독 태어났다 어 근 복

그것 그리 가지 못 다 근 곱 살

지는 산 산업에 실 고 답마 에 내

갔다 근 그림 럼 쫓 다니 가 시 것

다 상 진 것도 가 었다

러 가 에도 고 근 가 꿈꾸었다 근

가 꿈꾸게 것 보통 업

원색도1926 만lsquo rsquo 었다

공주 그림 가 근 경-

zb30) 에 대한 설 가장 른 것은~

① 역도가 과 운동 도 질

② 리는 는 다 lsquo rsquo

③ 들었지만 그럭 럭 는 다 lsquo rsquo

④ 가 게 보 시 말 다

⑤ 보 병 는 지 상 lsquo rsquo

는 말 다

년 학 간고사 대비2013 2 현대고 대비

ECN-0102-2013-001-000076193

시간 많지 다 청량리 생 병원

마지막 상 경 릿 게 들어 다 그 는 십

만 큰 가 상 말 다

지 못 들 마 갈 고 돗

도시민들 싹 싹 탔다 가 시

월에 병원에 원 가 폐 진 몸도4 ( )疲弊

갈 미 지 못 고 었다 가는 얼마( ) 解渴

지 생 에 생각 가

마감 는 신 평생 십 만에

가 과 많 닮 다고 생각 지는

가 운 는 어 어ldquo rdquo 1965helliphellip

월 새벽 시 태 없 거웠고 는5 6 1

없 그 병원에 퇴원 집 가

는 마지막 마 고 마 내 거 다 가

죽 간신 에 실 다 사는 어느 가5 lsquo

죽 는 말 가 식 다 신rsquo

상에 각 시키는 에 실 어느( ) lsquo刻印

가는 후 민 가가 근 었다rsquo

ldquo 는 간 과 진실 그 다는 에

단 평 견 가지고 다 내가 그

리는 간상 단 고 다 지 다 는 그들 가

에 는 평 지 니 그리고 어린 들

미지 겨 그린다rdquo

근 간 과 진실 그리고 싶어 가

다 근에게 그것 진리 다 거 다 없 거

고 다 없 는 것 진리다

근 진리는 후 쪽 었다 신산 삶 ( )辛酸

었 질곡 역사 에 지냈 가 눈에( )桎梏

든 것 료 단 료 게 보 것

었다 그것 그 에 겨우겨우 슬

슬 생 어가는 간들 었다 리

과 단 리 고리에 검 마

없 거리 돌 상

것 없는 등 근에게 상에 과 진실

엄 다는 사실 리는 가 실( )儼存

고 가 과 역경 에 도 근 내 포 없었

후 보루 다 도 도( ) 365堡壘

간 근 여 시 것

간에 지닌 가 근 1914 2

월 강원도 림리에 삼 독21

태어났다 어 근 복 그것 그리

가지 못 다 근 곱 살 지는 산

사업에 실 고 답마 에 내 갔다 근

그림 럼 쫓 다니 가 시 것 다 상

진 것도 가 었다 러 가 에도

고 근 가 꿈꾸었다 근 가 꿈꾸게

것 보통 업 원색1926

도 만 었다lsquo rsquo

그림 가 근 경 공주- ldquo rdquo ( 2009)

zb31) 다음 이 같은 의 성 소에 해당하 않은

것은

사건 평① ② ③

④ 주 ⑤ 경

가 운 는 어 어ldquo rdquo 1965helliphellip

월 새벽 시 태 없 거웠고 는5 6 1

없 그 병원에 퇴원 집 가

는 마지막 마 고 마 내 거 다 가

죽 간신 에 실 다 사는 어느 가5 lsquo

죽 는 말 가 식 다 신rsquo

상에 각 시키는 에 실 어느( ) lsquo刻印

가는 후 민 가가 근 었다rsquo

는 간 과 진실 그 다는 에ldquo

단 평 견 가지고 다 내가 그

리는 간상 단 고 다 지 다 는 그들 가

에 는 평 지 니 그리고 어린 들

미지 겨 그린다rdquo

근 간 과 진실 그리고 싶어 가

다 근에게 그것 진리 다 거 다 없 거

고 다 없 는 것 진리다

근 진리는 후 쪽 었다 신산 삶 ( )辛酸

었 질곡 역사 에 지냈 가 눈에( )桎梏

든 것 료 단 료 게 보 것

었다 그것 그 에 겨우겨우 슬

슬 생 어가는 간들 었다 리

과 단 리 고리에 검 마

없 거리 돌 상

것 없는 등 근에게 상에 과 진실

엄 다는 사실 리는 가 실( )儼存

고 가 과 역경 에 도 근 내 포 없었

후 보루 다 도 도( ) 365堡壘

간 근 여 시 것

간에 지닌 가 근 1914 2

월 강원도 림리에 삼 독21

태어났다 어 근 복 그것 그리

가지 못 다 근 곱 살 지는 산

사업에 실 고 답마 에 내 갔다 근

그림 럼 쫓 다니 가 시 것 다 상

진 것도 가 었다 러 가 에도

고 근 가 꿈꾸었다 근 가 꿈꾸게

것 보통 업 원색1926

도 만 었다lsquo rsquo

공주 그림 가 근 경- ldquo rdquo ( 2009)

년 학 간고사 대비2013 2 현대고 대비

ECN-0102-2013-001-000076193

zb32) 위 을 작성하는 과정에서 되어 활 된 자

어 것은

신 사 료① 연보②

고③ ④ 들과 담

⑤ 에 평

는 간 과 진실 그 다는 에ldquo

단 평 견 가지고 다 내가 그

리는 간상 단 고 다 지 다 는 그들 가

에 는 평 지 니 그리고 어린 들

미지 겨 그린다rdquo

근 간 과 진실 그리고 싶어 가

다 근에게 그것 진리 다 거 다 없 거

고 다 없 는 것 진리다

근 진리는 후 쪽 었다 신산 삶 ( )辛酸

었 질곡 역사 에 지냈 가( )桎梏

눈에 든 것 료 단 료 게 보

것 었다 그것 그 에 겨우겨우

슬 슬 생 어가는 간들 었다

리 과 단 리 고리에 검 마

없 거리 돌 상

것 없는 등 근에게 상에 과

진실 엄 다는 사실 리는 가 실( )儼存

고 가 과 역경 에 도 근 내 포

없었 후 보루 다 도 도( ) 365堡壘

간 근 여 시

것 다

간에 지닌 가 근 1914 2

월 강원도 림리에 삼 독21

태어났다 어 근 복 그것 그리

가지 못 다 근 곱 살 지는 산

사업에 실 고 답마 에 내 갔다 근

그림 럼 쫓 다니 가 시 것 다 상

진 것도 가 었다 러 가 에도

고 근 가 꿈꾸었다 근 가 꿈꾸게

것 보통 업 원색1926

도 만 었다lsquo rsquo

질 루 마 가 도 린다 경건

움 느껴지는 경 다 훗 근 그림에

과 는 거 것( )裸木

만 간과 연 엮어 가는 경건 움lsquo rsquo

니었

같 가가 고 싶었 근에게 그 꿈에 다

가가는 지 다 다 가 지망생들 규 미

상 에 진 고

에 지만 근 다 다 근

미 에 운 것 보통 시 미 시간

다 그런 그에게 없는 연습 가가

통 다 가 귀 시 지 도

얻는 뛸 듯 뻤지만 마 도 가 에

듯 는 었 에 어린 근 주 에

에 그림 그리고 지우고 복( )粉板

시간 가는 게 루 보냈다

근 그 갈 가가 것 열여( )渴求

었 다가 미1932 lsquo rsquo ( lsquo

미 에 다 다는 고 마rsquo) lsquo rsquo

가 근 집 고도 지는 시골 경

그린 그림 다 후 근 에 1943 22

지 미 에 그림 고

에 걸쳐 다 미 근 가

동 는 었다

공주 그림 가 근 경- ldquo rdquo ( 2009)

zb33) 위 의 내 과 일치하는 것은

가 근 가 꿈 포 다①

근 당 가들과 께 에 다②

살 근 가 걷20③

게 었다

④ 만 통 근 역경 겨내는lsquo rsquo

느 다

⑤ 근 간 과 진실 그리 에 그 에

드러 는 간상 단 다

계 시 주 근 건강

걸었다 신 과 간에 상 다 건강

신 는 눈에도 다 근 쪽 눈 뿌 게

보 지 과에 다 다 시 지지 고 결

내 었다 시 지만 마 막막

다 늦어 결 근 쪽 눈 고 말 다

쪽 눈 근에게는 쪽 눈 었고

계 었다 그 근 는 여 그lsquo rsquo

다 근 에 같 그림 그 었다1950

시 그림 는 여 쪽lsquo rsquo

고 어 마주 고 는 그림1963

여 과 동 다 마 복

그린 듯 눈 내리 새 게 다 지

사 다 근 게 복 것

복 상과 타 는 근 상

가 떳떳 단 었고 근 그리고

간 과 진실 에 다가가 가 근다

운 었다 근 신에게 당당 지 그리고

그 다 근 그림에 단 복 보다

년 학 간고사 대비2013 2 현대고 대비

ECN-0102-2013-001-000076193

태 도 그리고 극 보다 과

얻 여 었다 과 통

근 그리고 는 재 고 에 질

만들고 특 것 다

공주 그림 가 근 경- ldquo rdquo( 2009)

zb34) 의 이유에 대해 추 한 것으 적절하 않은 것

상과 타 시도①

보다 과 얻②

근 신에게 당당 지③

④ 간 과 진실 에 다가

⑤ 태 도 얻

근 가가 었지만 그 다니 가

럼 어지지 다 복과 쟁 거쳐 시

는 가 근에게 생계 사 에

운 사 다 에 키에 건( ) 178cm死鬪

체 근 에 동 역 업( )荷役

가 생계 다 쟁

에는 동에 운 상우 주 미

죄 사 에 그림 그리는 시 다 그곳에

에 동 역 업 것에

결 것 럼 보 다 지만 그런 것만도

니었다 그림 그리는 고는 지만 매 근

는 극 간 과 별 없는 경 리 그림

벽에 그리는 것 었다 우도 리 없었다 근

트 는 우 그림 그 다 생

계 그림 단 것 다

후 근 지 신 계 리에 미

엑 리 겼다 근 곳에

건 사 크 에 미 들 ( )

상 상 그 다 근 갖 다 겪

냈다 그리고 결 그 돈

신동에 어 사리 집 마 다 마 ㄷ

루 심 쪽에는 과 엌 쪽에는 건

었다 건 주고 근 가 에

여 살 다 심 에는 지 집어

쓰고 지만 곳 근 가 에게 러웠

보 리 다 근 과 마루 업실 삼 그림

그 다 신동 마루는 근 그림에 등 는 lsquo rsquo

같 상들 지 다 시 고

에 들 폐허가

가 업실 었다

공주 그림 가 근 경- ldquo rdquo( 2009)

zb35) 위 에 대한 설 으 적절한 것은

업 시 여 훈과 감동 다①

에 주 평 드러 다②

사 사 등 식 과 ③

④ 다 근거 시 여 삶에

⑤ 살 시 사 경 께 여

습 시 다

가 시간 많지 다 청량리 생 병원( )

마지막 상 경 릿 게 들어 다 그 는

십 만 큰 가 상 말 다

지 못 들 마 갈 고 돗

도시민들 싹 싹 탔다 가 시

월에 병원에 원4 가 폐( )疲弊

진 몸도 갈 미 지 못 고 었다( )解渴 가는

얼마 지 생 에 생각

가 마감 는 신 평생 십 만에

가 과 많 닮 다고 생각 지는

가 운 는 어 어( ) ldquo rdquohelliphellip

월 새벽 시1965 5 6 1 태 없 거웠고

는 없 그 병원에 퇴원 집

가는 마지막 마 고 마 내 거 다

가 죽 간신 에 실 다 사는 어느5 lsquo

가 죽 는 말 가 식 다 신rsquo

상에 각 시키는 에 실( )刻印

어느 가는 후 민 가가 근 었다lsquo rsquo

다 는 간 과 진실 그 다는( ) ldquo

에 단 평 견 가지고 다 내

가 그리는 간상 단 고 다 지 다 는 가

에 는 평 지 니 그리고 어린 들

미지 겨 그린다rdquo

근 간 과 진실 그리고 싶어( )

가 다 근에게 그것 진리 다 거 다 없

년 학 간고사 대비2013 2 현대고 대비

ECN-0102-2013-001-000076193

거 고 다 없 는 것 진리

다 근 진리는 후 쪽 었다 신산( )辛酸 삶

었 질곡 역사 에 지냈( )桎梏

가 눈에 든 것 료 단 료 게 보

것 었다 그것 그 에 겨우겨우

슬 슬 생 어가는 간들 었다

리 과 단 리 고리에 검

마 없 거리 돌

상 것 없는 등 근에게 상에

과 진실 엄 다는 사실 리는 가 실( )儼存

고 가 과 역경 에 도 근 내 포

없었 후 보루 다( ) 堡壘 도 365

도 간 근 여

시 것 다

마 같 가가 고 싶었 근에게 그 꿈( )

에 다가가는 지 다 다 가 지망생들

규 미 상 에 진 고

에 지만 근 다 다 근

미 에 운 것 보통 시 미 시간

다 그런 그에게 없는 연습 가가

통 다 가 귀 시 지 도

얻는 뛸 듯 뻤지만 마 도 (

는 었 에 어린 근 주 에)

에 그림 그리고 지우고( )粉板

복 시간 가는 게 루 보냈다

zb36) 전 의 성 소가 아닌 것을 고르

① 평 ② 사건 ③ 경

④ ⑤ 훈

늘 지 상에 살고 는 사 들 억 도가10

고 그리 지 통 고 는 사 들( )知的

그보다 훨 많 억 도는 고 지 20

통 다 그런 지 고 2500

그리 간 보는 과 사 에

매우 달 뿐만 니 과 에 도 극

루고 었다 미 운 그런 들

살고 는 동 과 사 들 사고 식에

큰 가 다는 다

고 그리 들 우주 개별 고 독립

사 들 생각 지만 고 들 우

주 연 질 간주 다 같( ) 看做

각 도 들에게는 연 질

었지만 그리 들에게는 미 들 결 었

다 고 과 그리 들 사 같

는 동 과 사 에 도 견 다

지심리 미 마 드 겐트 는

살 들에 에 지 다

연 동 과 상 다 과 같 실험

다 크 만든 미드 도 보

여 주고 그 상 닥 고 주었다lsquo (Dax)rsquo

실 닥 는 재 지 는 것 실험 가lsquo rsquo

만들어 낸 다 그런 다 개 다 체 보

여 주었는 는 미드 지만 틱

만들었고 다 는 재료는 크 지만

달 다 그러고 어 것 닥 지 사 들에게 고 lsquo rsquo

게 니 들 주 같 고 는

체 택 고 동 들 같 재료 만들어진 체

택 다 러 는 심지어 살짜리

들에게 도 타났다 것 곧 과 동

다 상 보고 다는 것 미 다

개별 사 보고 고 동 연 질 보

고 는 것 다

동 들 주변 상 에 맞 어 동 고

에 다 사 들 태도 동에 보다 많

주 울 다 동 가 미시간 에

에 경험 다 그는 미식

경 보러 가게 었는 경 체는 매우 재미 었

주변 들 동에 질 다 그 는

들 계 어 상태 경 다

어 들 에 에 그 시 가 계 가

진 것 다 상 살펴 는 말 들 lsquo rsquo

에 그는 에 시 어 도 뒷사

생각 곧 다시 곤 것 다 그런 그에게 뒷

사 고 지 는 들 동 럼

어 웠다

생각 지도 리 드 니 벳-

zb37) 다음 위 의 내 전개 으 만 인lt gt

것은

lt gt

대조의 통해 대상이 닌 특성을 설 하고 있다

일화를 제 하여 자 의 주장을 뒷 침하고 있다

유추의 을 사 하여 독자의 의해를 돕고 있다

대상이 형성되는 과정을 간적 서에 따라 서 하고 있

① ②

③ ④

년 학 간고사 대비2013 2 현대고 대비

ECN-0102-2013-001-000076193

가 우리가 말 고 쓰는 든 단어가 사 에 는( )

것 니다 사 격에 가 는 지만

어 사 과 같 특별 는 사 니lsquo rsquo

단어 격 보 단어가 사 에

등재 어 다 리 리 사 는 단어 도 그

것 시 사 는 어 고 사 에

격 보 것 니다

러 얼 은 사전에 를 있는가 이에 대한 답lsquo rsquo

은 얼 이 유행어인가 아닌가에 따라 갈라 다 이 단어lsquo rsquo

는 년 어 자 에 랐고 쓰이고 있으2002 lsquo rsquo

유행어라고 하 에는 생 이 다 런데 계속

을 유 하 서 사전에 등재될 자격을 획득할 것인가 이

에 대한 답을 내리 는 히 어 다

여 서 가 를 고 해 볼 있다 첫 는 이 단어

를 써야 할 필 가 속적으 있는가 하는 점이다

상주의 열풍에 휩 인 사회 위 에 편 해서 퍼 말

이 얼 인데 과연 런 위 가 속될 것인가 이에lsquo rsquo

대해 필자의 생각은 정적이다 사회 위 가 뀌

런 말을 쓸 일이 없어 것이다

다음은 단어의 성이다 단어의 성이 사회적으 거

감이 없으 계속 사 될 가능성이 높다 런 에서

얼 은 좋은 조건이 아니다 익히 알 졌듯이 이lsquo rsquo

말은 얼 과 청소년층에서 속어 사 하는 이 결합lsquo rsquo lsquo rsquo

된 말이다 얼 에서 얼 을 리하는 조어 도 lsquo rsquo lsquo -rsquo

어에서는 매 낯선 이다 이것만으 도 거 감을 갖

는 사람들이 있다 더 나 속어 결합한 말이다 얼 lsquo rsquo

이 널리 퍼졌다 해도 은 여전히 청소년층의 속어lsquo rsquo

남아 있다 속어는 자연 럽게 아 자리에서나 쓰 에는

담 러 말이다 러한 담을 하고 사

역을 넓혀 가는 속어도 없 는 않다 특히 얼 은 lsquo rsquo

에도 종종 등장한다 만큼 거 감이 많이 희석되었다

고 할 있다 러나 일상의 자연 러 대화에서도 거

리낌 없이 등장하는가 게 는 되 않았다고 생

각한다

얼 이 유사어인 쌈 등을 만들어 내고lsquo rsquo lsquo rsquo

있으니 살아남을 있을 것이라고 는 견해도 있을 것

이다 러나 간이 나 서 유사어를 포함하여 든

말이 사라 사 는 많다 유사어가 많다는 것이 생 을

유 할 있는 절대적인 조건은 아니다

나 언젠가 터 사람들은 어느 단에서 얼 이 가장( )

쁜 사람을 가리켜 얼 이라고 르고 있다 이 얼lsquo rsquo lsquo rsquo

이라는 단어가 최근 어사전에 라 항간에 논란이 일고

있다 아닌 게 아니라 얼 은 유행어처럼 인다 생 lsquo rsquo

도 리 래되 않은 것 같고 언제 사라 도 알

없다 게다가 젊은이들 사이에서 주 쓰일 뿐이다 이런

단어를 사전에 는다는 게 하 이 없어 이 도

한다

러나 속단은 이다 차근차근 따져 볼 일이다

선 얼 이 일 적 유행어인 아닌 주의 게 들여다lsquo rsquo

볼 필 가 있다 유행어란 유행에 따라 빠르게 유포되었

다가 단 간 내에 소 되는 단어나 를 가리킨다

얼 은 인터넷을 통해 속히 퍼 말이다 하 만 일lsquo rsquo

적인 유행어처럼 단 간 내에 사라 않았을 뿐 아니라

현재 도 잦은 빈도 사 되고 있고 앞으 도 상당

간 사 될 것으 측된다 한 언 재단의 뉴 검 lsquo rsquo

색 사이트에 따르 얼 은 년 에 처음 나타난lsquo rsquo 2001

이후 꾸 히 사 되고 있다

이 같은 사 빈도는 얼 이 일 적 유행어 는 현lsquo rsquo

저히 다르다는 것을 여 다 장 간의 생존 만으 도

얼 은 이 한 어의 어휘 에 를 자격을 얻었다lsquo rsquo

고 할 있다 더 이 이라는 비 적 정제된 매체에

높은 빈도 쓰이고 있 않은가 사 빈도 측 에서

필통이나 연필과 같은 단어 대등하거나 더 많이 쓰lsquo rsquo lsquo rsquo

다는 것은 결코 가 게 볼 일이 아니다

이제는 사전이 언어 현 을 빠르게 하는 게 덕인

대가 되었다 세계적으 유 한 의 사전들도 경쟁

적으 어를 고 있다

하 만 얼 은 젊은이들이나 쓰는 속어라고 흠을 잡을lsquo rsquo

도 르겠다 얼 이 주 젊은 층에서 많이 쓰 lsquo rsquo

는 속어임에 틀림없다 러나 어사전에 표 적이고 품

위 있는 말만 어야 한다고 생각한다 것은 커다란

해다 당장 아 어사전이나 펼쳐 라 속어는

설과 같은 비어나 죄자들이 쓰는 은어 어

마니 같은 소 의 사람만이 쓰는 말 도 라 있

않은가 사전은 말 치에 일정 빈도 이상 나타나는 말이

라 말이든 다 할 있다

zb38) 가 나 에 대한 다음의 설( ) ( ) 않은 것은

① 가 는 얼짱 사 에 등재 것에( ) ( ) lsquo rsquo

보 고 다

② 사 등재 가는 단어 격에( )

고 고 는 언 들 언어 사 도에 고 다 ( )

③ 가 얼짱 어지만 신 과 같 매( ) ( ) lsquo rsquo

체에 도 사 는 말 는 고 다

④ 가는 얼짱 어 보고 크게 가지 근( ) lsquo rsquo 3

거 들어 뒷 고 다

⑤ 는 얼짱 어 는 다 특 다는( ) lsquo rsquo

근거 에도 크게 가지 근거 가 들어 주 2

뒷 고 다

가 늘 지 상에 살고 는 사 들 억( ) 10

도가 고 그리 지 통 고 는 사 들

그보다 훨 많 억 도는 고 지 20

통 다 그런 지 고 2500

년 학 간고사 대비2013 2 현대고 대비

ECN-0102-2013-001-000076193

그리 간 보는 과 사 에

매우 달 뿐만 니 과 에 도 극

루고 었다 미 운 그런 들

살고 는 동 과 사 들 사고 식에

큰 가 다는 다

고 그리 들 우주 개별 고 독립

사 들 생각 지만 고 들 우

주 연 질 간주 다 같 각

도 들에게는 연 질 었지

만 그리 들에게는 미 들 결 었다

고 과 그리 들 사 같 는

동 과 사 에 도 견 다

인 리학자인 츠 이마이 디드 겐트너는 두

살이 채 안 된 아이들에서 터 성인에 이르 다양한

연 대의 동양인과 서양인을 대상으 다음과 같은 험

을 했다 저 코르크 만든 피라 드 양의 도형을

여 주고 대상의 이름을 닥 라고 알 주었다lsquo (Dax)rsquo

제 닥 는 존재하 않는 것으 험자가 임의lsquo rsquo

만들어 낸 이름이다 런 다음 두 개의 다른 체를

여 주었는데 하나는 피라 드 양이 만 하얀 플라 틱

으 만들었고 다른 하나는 재 는 코르크 만 양이

달랐다 러고 나서 어떤 것이 닥 인 사람들에게 고 lsquo rsquo

르게 했더니 서양인들은 주 같은 양을 하고 있는

체를 선택했고 동양인들은 같은 재 만들어 체를

선택했다 이러한 차이는 성인은 어 두 살 리

아이들에게서도 나타났다 이것은 곧 서양인과 동양인은

서 다른 세상을 고 있다는 것을 의 한다 략 ( )

는 아주 단 하 서도 인상적인 험을 했다

험에는 동서양의 대학생들이 참여했다 는 험 참가자

들에게 컴퓨터 화 을 통해 속 장 을 담은 애니 이션

을 여 주었다 화 의 앙에는 초점의 역할을 하는 커

다란 고 한 마리가 있었고 주위에는 다른 생

들과 초 자갈 거품 등이 함 제 되었다 화 을

두 씩 후 참가자들은 자 이 것을 회상해 라는

를 았다

결과 서양인 대학생들과 동양인 대학생 두 앙

의 초점 역할을 했던 고 를 동일한 정도 언 했으

나 경 소 위 거품 초 다른 생 들 에 ( )

대해서는 동양인 대학생들이 서양인 대학생들 다 60

이상 더 많이 언 했다 뿐만 아니라 동양인 학생들은 서

양인 학생들에 비해 개 적인 고 다 전체적인 계

를 더 언 하는 경향을 다 략 또한 경의 일 ( )

를 화 킨 림을 제 하 을 때 동양인 대학생들은 대

경의 화를 알아챘 만 서양인 대학생들은 경

의 화를 거의 알아차리 했다 략 ( )

따라서 서양인들만을 대상으 연 한 화lsquo

편성 결 은 잘 된 것일 도 있다 각 과정과 인rsquo

과정의 어떤 이 화 편적이고 어떤 이

화에 따라 달라 는 는 앞으 많은 연 를 통하여 논의

되어야 한다

나 어떤 의 에서 리 두는 이 화적이다 리( )

안에는 다른 사람들과 더 친 한 계를 유 하 는 상호

의존성과 다른 사람들 터 독립적인 존재 살아가 는

독립성이 혼재한다 따라서 이 에서 어떤 특성이 더 강

하게 각되는 상황에 놓이느냐에 따라 서 다른 화적

특 을 일 있다 결 리 두는 어떤 경 에는

동양인처럼 행동하고 어떤 경 에는 서양인처럼 행동하는

것이다

zb39) 가 에 대한 다음의 설( ) 않은 것은

① 는 신 주 뒷 닥 실험과lsquo rsquo lsquo

니 실험 근거 시 다rsquo

② 동 들 상 간 공통 보다는 에 식

는 강 다

③ 들 주변 맥 에는 심 경 어 사건

과 사건 사 계에 상 민감 다

④ 는 동 과 틀린 지 고 는 것lsquo rsquo

니 다 고 다 lsquo rsquo

⑤ 가에 우리 사 들 개 시 가 원( )

집 경 말 고 는 것 개 보다는

에 고 는 것에 다

늘 지 상에 살고 는 사 들 억 도가10

고 그리 지 통 고 는 사 들( )知的

그보다 훨 많 억 도는 고 지 20

통 다 그런 지 고 2500

그리 간 보는 과 사 에

매우 달 뿐만 니 과 에 도 극

루고 었다 미 운 그런 들

살고 는 동 과 사 들 사고 식에

큰 가 다는 다

지심리 미 마 드 겐트 는 동

과 상 다 과 같 실험 다

크 만든 미드 도 보여 주고 그

상 닥 고 주었다 그런 다lsquo (Dax)rsquo

개 다 체 보여 주었는 는 미드

지만 틱 만들었고 다 는 재료는

크 지만 달 다 그러고 어 것 닥 lsquo

지 사 들에게 고 게 니 들 주 같rsquo

고 는 체 택 고 동 들 같

재료 만들어진 체 택 다 러 는

심지어 살짜리 들에게 도 타났다 것

곧 과 동 다 상 보고 다는

것 미 다 개별 사 보고 고 동

년 학 간고사 대비2013 2 현대고 대비

ECN-0102-2013-001-000076193

연 질 보고 는 것 다

동 들 주변 상 에 맞 어 동 고

에 다 사 들 태도 동에 보다

많 주 울 다 동 가 미시간

에 에 경험 다 그는 미

식 경 보러 가게 었는 경 체는 매우 재

미 었 주변 들 동에 질 다 그

는 들 계 어 상태 경

다 어 들 에 에 그 시 가 계

가 진 것 다 뒷사 고 지 는 들

동 럼 어 웠다

그는 경험에 어 얻어 동 들lsquo

각도 상 본다 는 가 우고rsquo

검 여 주 단 도 상 실험 실

시 다 그는 실험 가 들에게 컴퓨 통

담 니 보여 주었다

에는 역 는 커다 고 마리가 었

고 주 에는 다 생 들과 갈 거 등

께 시 었다 본 후 가 들

신 본 것 상 보 는 지시 다

그 결과 생들과 동 생

역 고 동 도 언

경 거 다 생 들에 ( )

는 동 생들 생들보다 60

상 많 언 다 뿐만 니 동 생들

생들에 개별 고 보다 체 계

언 는 경 보 다 경 변 시

킨 그림 시 동 생들 경

변 지만 생들 경 변

거 리지 못 다

지 지 들만 상 연 lsquo

보편 결 못 것 도 다 지각 과 과rsquo

지 과 어 보편 고 어

에 달 지는지는 많 연 통 여

어 다

리 드 니 벳 생각 지도 사- ldquo rdquo( 2004)

zb40) 위 에 대한 설 으 가장 적절한 것은

① 동 과 생 식 강 고 다

② 가지 실험 통 쓴 고 다

③ 닥 실험에 사 본질에 동 사

상에 주 다

④ 니 실험에 동 과 에 지

각 도에 가 다

⑤ 쓴 는 보편 연 에 드러 우월 에

에 근 고 다

가 동 들 주변 상 에 맞 어 동 고( )

에 다 사 들 태도 동에 보다 많

주 울 다 동 가 미시간 에

에 경험 다 그는 미식

경 보러 가게 었는 경 체는 매우 재미 었

주변 들 동에 질 다 그 는

들 계 어 상태 경 다

어 들 에 에 그 시 가 계 가

진 것 다 상 살펴lsquo 는 말 들rsquo

에 그는 에 시 어 도 뒷사

생각 곧 다시 곤 것 다 그런 그에게

뒷사 고 지 는 들 동 럼

어 웠다

그는 경험에 어 얻어( ) 동 들lsquo

각도 상 본다 는 가 우고rsquo

검 여 주 단 도 상 실험

실시 다 실험에는 동 생들 여 다

그는 실험 가 들에게 컴퓨 통

담 니 보여 주었다 에는

역 는 커다 고 마리가 었고 주 에는

다 생 들과 갈 거 등 께 시

었다 본 후 가 들 신 본 것

상 보 는 지시 다

다 그 결과 생들과 동 생( )

역 고 동 도 언

경 거 다 생 들 에 ( )

는 동 생들 생들보다 60

상 많 언 다 뿐만 니 동 생들

생들에 개별 고 보다 체 계

언 는 경 보 다 들어 동

생들 상 체 연못 럼 보 어ldquo 같rdquo

체 맥 언 시 었지만

생들 상 어 같 큰 고 가 쪽 움ldquo

직 어 같 역 고rdquo

언 시 다 경 변 시킨 그

림 시 동 생들 경 변

지만 생들 경 변 거

리지 못 다

년 학 간고사 대비2013 2 현대고 대비

ECN-0102-2013-001-000076193

게 볼 동 들 보다는 큰 그( )

림 보 에 사 과 체 맥 연결시 지각

는 경 고 체에 특 떼어 내

어 독립 보는 것 낯 어 다 에

들 사 에 고 주변 맥 에는 심 경

에 사건과 사건 사 계에 상

민감 편 다

마 지 지( ) 들만 상 연

보편 결 못 것 도 다lsquo rsquo 지각 과

과 지 과 어 보편 고 어

에 달 지는지는 많 연 통 여

어 다

리 드 니 벳 생각 지도 사- ldquo rdquo( 2004)

zb41) 의 하는 가~ 다른 것은

① ② ③

④ ⑤

얼마 그 에 동 사고 식과

사고 식 보여 주는 내 다

들 에 는 탕 고 같 게

어 겨 고 미 에 는 그 크 럼 큰 고

어리 주고 원 는 어 도 는

상 고 생각 다는 것 다 러

는 어떻게 생 것 고 과 그리 거슬

러 가 보 그 단 다

고 연 경 체 경 생 에

다 벼 사는 공동 업과 경험 많 연 역

에 고 들 연 웃과

게 지내 고 탁 연 들

들 지 연 럽게 들 다 민들

웃과 동 게 뿐만 니 는 집 과

게 다

동 시 는 생태 경 에 살 결과

들 다 사 들 사 상 에 주

울 게 었고 는 곧 체 상 과 간 사

계 시 는 낳게 었다 신 가

가 는 체에 는 원 는 동시

에 다 사 들 그 사 포 체 맥 에

다 들 간 사 연

계 체 계에 주 울 는 사고 체계

게 었다

그러 그리 연 경 그 었다 산

지 연결 는 지 건 그리고 역

에 다 런 들 업에 다 사 과

동 므 공동체에

다고 다 고 그리 들

들과는 달리 보 내 감 지 들과

지 크게 느 지 못 다 그

견 다 경우 주 쟁 통 결 는 갖

게 었다

신 사 간 계들 루어진 커다

트워크 에 게 당연 사 역시 연

계들 체 식 게 다 어 상

원 도 그 개체가 체 맥 과

계 에 고 다 게 체 맥 에 주

울 다 보 상 복 과 가변 식 게 고

상에 재 는 많 변 들 사 에 재 는 들도

게 다 들 주 태도 보

는 경우가 많다 쟁 결

통 결 보다는 통 결

는 보 다

그러 고 그리 들 개개 사 사 독

에 주 울 다 사 사 체에

어 그들 사 에 재 는 공통 규 주

고 다 상 원 에도 사

체 내 주 고 다 그들

체 여 탕 체

는 주 태도 시 고 특 사 어

주에 는지 여 그 주에 는 규

견 다 에 는 쟁 식 리

같 리 사고 체계가 달 게 었다

리 드 니 벳 생각 지도 사- ldquo rdquo( 2004)

zb42) 위 에서 사 된 설 과 가장 유사한 것은

① 크톱 컴퓨 는 본체 니 마우 루

어 다

② 곡과 시 리 는 지 과 사 루어 다는 공통

지니고 다

③ 경 고 것과는 달리

경 본 연 태 그 주변 경

④ 벽돌 능 에 사계 내내

습도가 지 다

⑤ 잰느 체 체 지닌 재 체가 없

는 재 눌 다

년 학 간고사 대비2013 2 현대고 대비

ECN-0102-2013-001-000076193

zb43) 는 립 앙 도서 이 정의 일 이다lt gt

도서 장과 이 자의 리 의 정의 연결이

적절하 않은 것은

lt gt

제 조 서 유8 ( )

도서 장은 다른 이 자의 안전을 위협하거나 도서 의①

서를 란하게 할 가 있는 자에 대하여는 도서 출입

을 제한할 있다

도서 장은 이 자가 제 조 각 호의 어느 하나의 행위를 하7②

을 때에는 이 을 하게 하거나 도서 출입을 제한할

있다

제 조자 의 대출9 ( )

도서 자 는 다음 각 호의 경 대출할 있다①

상호대차도서 간에 자 를 류하는 것을 말한다 등 다1 ( )

른 도서 과의 협 을 위하여 필 한 경

공 이 공 행 상 필 하는 경2

에 도서 장이 필 하다고 인정하는 경3

대출이 가능한 도서 자 의 위는 도서 장이 정하는②

에 따른다

제 조 상10 ( )

이 자가 도서 자 설을 더럽히거나 찢거나 뜨①

쓰게 하거나 잃어 린 경 에는 상하여야 한다

도서 장은 제 항에 따른 상 을 정하여 게 하여야1②

한다

제 조이 절차 등11 ( )

이 칙에서 정한 것 에 도서 자 설의 이 절차

이 제한 등에 필 한 사항은 도서 장이 정한다

출처 립 앙 도서- (httpwwwnlgokr)

① 는 도 리 다8

② 도 는 리 다9 1

③ 료 지 는 도 리 다9 2

④ 도 료 변상에 리10 1

⑤ 는 에 도 리 다11

3

도 다 각 같다①

공 공 다만 연1

연 간 다

매월 째 째 월2

도 도 리 그 사3

가 다고 는

도 에 미리 게1 3②

시 여 다

4

도 시간 도 여 게시 다

5

도 료 시 는 는 도①

지에 등 후

등 에 사 도②

7

는 다 각 여 는 니 다

도 료 시 상 리1 lsquo rsquo

도 료 시 훼 는2 middot

지 가 닌 곳에 식 거 담3

우는

도 보 등 보 검색열4 middot

그 에 도 질 지 여 도5

여 게시 사 는

8

도 다 거 도①

질 게 우 가 는 에 여는 도

도 가 각 어느7②

에는 지 게 거 도

9

도 료는 다 각 경우 다①

상 도 간에 료 는 것 말1 (

다 등 다 도 과 여 경우)

공 원 공 상 는 경우2

그 에 도 다고 는 경우3

가능 도 료 는 도②

는 에 다

10

년 학 간고사 대비2013 2 현대고 대비

ECN-0102-2013-001-000076193

가 도 료 시 럽 거 거①

못 쓰게 거 어 린 경우에는 변상 여

도 에 변상 여 게시1②

여 다

zb44) 위 에서 도서 장이 게 해야 할 사항에 해당하는

것을 두 쓰

년 학 간고사 대비2013 2 현대고 대비

ECN-0102-2013-001-000076193

립 도 규

1 ( )

규 립 도 립 어린 청 도(

포 다 료 시 열 시 말) (

다 에 사 규 립 도)

편 진 다

2 ( )

규 립 도 도 다 에( lsquo rsquo )

고 는 도 에 도lsquo rsquo 2 2

료 에 여 다 다만 특 료 귀

료 등 료 에 사 립 도

도 다 다( lsquo rsquo )

3 ( )

도 다 각 같다①

공 공 다만 연1

연 간 다

매월 째 째 월2

도 도 리 그 사3

가 다고 는

도 에 미리 게1 3②

시 여 다

시간4 ( )

도 시간 도 여 게시 다

등 등5 ( )

도 료 시 는 는 도①

지에 등 후

등 에 사 도②

사 료6 ( )

도 료 시 에 사 료는 도

7 ( )

는 다 각 여 는 니 다

도 료 시 상 리1 lsquo rsquo

도 료 시 훼 는2 middot

지 가 닌 곳에 식 거 담3

우는

도 보 등 보 검색열4 middot

그 에 도 질 지 여 도5

여 게시 사 는

질 지8 ( )

도 다 거 도①

질 게 우 가 는 에 여는 도

도 가 각 어느7②

에는 지 게 거 도

료9 ( )

도 료는 다 각 경우 다①

상 도 간에 료 는 것 말1 (

다 등 다 도 과 여 경우)

공 원 공 상 는 경우2

그 에 도 다고 는 경우3

가능 도 료 는 도②

는 에 다

변상10 ( )

가 도 료 시 럽 거 거①

못 쓰게 거 어 린 경우에는 변상 여

도 에 변상 여 게시1②

여 다

등 규 에 것 에 도11 ( )

료 시 등에 사

도 다

립 도- (httpwwwnlgokr)

zb45) 도서 장의 리 있는 조항으 적절하 않

은 것은

① ② ③ ④ ⑤

년 학 간고사 대비2013 2 현대고 대비

ECN-0102-2013-001-000076193

1 ( )

사가 공 는lsquo rsquo

과 여 사 원과 리

사 타 사 규

니다

개 보 보7 ( )

사는 보통신망 등 계 는 에lsquo rsquo lsquo rsquo

원 개 보 보 니다 개lsquo rsquo

보 보 사 에 는 사 개lsquo rsquo

보 취 니다 다만 사는 다 lsquo rsquo

사 계 통 공 는 경우 원 lsquo rsquo

등 개 보 당 사에 습니lsquo rsquo

원 리에8 (lsquo rsquo lsquo rsquo lsquo rsquo

)

원 에 리lsquo rsquo lsquo rsquo lsquo rsquo①

원에게 가 도 여 는lsquo rsquo 3

니다

사는 원 가 개 보 우 가lsquo rsquo lsquo rsquo lsquo rsquo②

거 사 경우 는 미 에 어 거 lsquo

사 사 운 우 가 는 경우 당rsquo lsquo rsquo

습니다lsquo rsquo

원 가 도 거lsquo rsquo lsquo rsquo lsquo rsquo 3③

가 사 고 지 경우에는 시 사에lsquo rsquo

통지 고 사 내에 니다lsquo rsquo

경우에 당 원 사에 그 사실3 lsquo rsquo lsquo rsquo④

통지 지 거 통지 도 사 내에 지 lsquo rsquo

생 경우 사는 지지 습니다lsquo rsquo

사10 (lsquo rsquo )

사는 과 지 미lsquo rsquo①

에 는 지 계 고

공 여 다 여 니다lsquo rsquo

사는 원 게lsquo rsquo lsquo rsquo lsquo rsquo②

도 개 보 신 보 포 보 보 시( )

갖 어 개 보 취 공시 고

니다

사는 과 여 원lsquo rsquo lsquo rsquo③

견 만 당 다고 경우에는

리 여 니다 원 견 만 사 lsquo rsquo

에 는 게시 거 우편 등 통 여

원에게 리 과 결과 달 니다lsquo rsquo

원11 (lsquo rsquo )

원 다 여 는 니다lsquo rsquo ①

신청 는 변경 시 허 내 등1

타 보 도2

사가 게시 보 변경3 lsquo rsquo

사가 보 보 컴퓨 그4 lsquo rsquo (

등 등 신 는 게시)

사 타 등 지 재산 에5 lsquo rsquo 3

사 타 상 거 업6 lsquo rsquo 3

는 폭 시지 상 타 공7 middot middot

에 는 보 에 공개 는 게시 는lsquo rsquo

사 동 없 리 사8 lsquo rsquo

타 거 당9

게시15 (lsquo rsquo )

원 내에 게시 는 게시 게재 는lsquo rsquo lsquo rsquo lsquo rsquo

경우 원 사가 게시 복 lsquo rsquo lsquo rsquo lsquo rsquo middot middot

등 태 언 등에 공 는

것 내에 다 원 본 게시 등 lsquo rsquo lsquo rsquo

크 능 등 여 복 는 등 태

는 것 동 것 니다

- (wwwnavercom)

zb46) 위 은 인터넷 포털사이트의 회 가입을 위한 이

약 의 일 이다 이 약 을 만드는 과정에서 생각한

내 으 적절하 않은 것은

개 보 보 가 지에 별 눠①

겠어

원 가 만들게 에②

시 주어 겠어

원들 게재 게시 다 원 크 다③

는 것 지

④ 원 지 는 뿐만 니 사가 지 는

도 께 달 지

리에 가 생 경우 사가⑤

에 다는 도 듯

1 ( )

사가 공 는lsquo rsquo

과 여 사 원과 리

사 타 사 규

년 학 간고사 대비2013 2 현대고 대비

ECN-0102-2013-001-000076193

니다

개 보 보7 ( )

사는 보통신망 등 계 는 에lsquo rsquo lsquo rsquo

원 개 보 보 니다 개lsquo rsquo

보 보 사 에 는 사 개lsquo rsquo

보 취 니다 다만 사는 다 lsquo rsquo

사 계 통 공 는 경우 원 lsquo rsquo

등 개 보 당 사에 습니lsquo rsquo

원 리에8 (lsquo rsquo lsquo rsquo lsquo rsquo

)

원 에 리lsquo rsquo lsquo rsquo lsquo rsquo①

원에게 가 도 여 는lsquo rsquo 3

니다

사는 원 가 개 보 우 가lsquo rsquo lsquo rsquo lsquo rsquo②

거 사 경우 는 미 에 어 거 lsquo

사 사 운 우 가 는 경우 당rsquo lsquo rsquo

습니다lsquo rsquo

원 가 도 거lsquo rsquo lsquo rsquo lsquo rsquo 3③

가 사 고 지 경우에는 시 사에lsquo rsquo

통지 고 사 내에 니다lsquo rsquo

경우에 당 원 사에 그 사실3 lsquo rsquo lsquo rsquo④

통지 지 거 통지 도 사 내에 지 lsquo rsquo

생 경우 사는 지지 습니다lsquo rsquo

원에 통지9 (lsquo rsquo )

사는 특 다 원에게 통지 경우lsquo rsquo lsquo rsquo

공지 게시 통 상 게시 개별 통지에7

갈 습니다

사10 (lsquo rsquo )

사는 과 지 미lsquo rsquo①

에 는 지 계 고

공 여 다 여 니다lsquo rsquo

사는 원 게lsquo rsquo lsquo rsquo lsquo rsquo②

도 개 보 신 보 포 보 보 시( )

갖 어 개 보 취 공시 고

니다

사는 과 여 원lsquo rsquo lsquo rsquo③

견 만 당 다고 경우에는

리 여 니다 원 견 만 사 lsquo rsquo

에 는 게시 거 우편 등 통 여

원에게 리 과 결과 달 니다lsquo rsquo

원11 (lsquo rsquo )

원 다 여 는 니다lsquo rsquo ①

신청 는 변경 시 허 내 등1

타 보 도2

사가 게시 보 변경3 lsquo rsquo

사가 보 보 컴퓨 그4 lsquo rsquo (

등 등 신 는 게시)

사 타 등 지 재산 에5 lsquo rsquo 3

사 타 상 거 업6 lsquo rsquo 3

는 폭 시지 상 타 공7 middot middot

에 는 보 에 공개 는 게시 는lsquo rsquo

사 동 없 리 사8 lsquo rsquo

타 거 당9

원 계 규 내lsquo rsquo lsquo②

여 공지 주 사 사가 통지 는rsquo lsquo rsquo

사 등 여 타 사 업 에 lsquo rsquo

는 여 는 니다

- (wwwnavercom)

zb47) 위 약 의 조항에서 같은 제점을 하lt gt

고 있는 조항은

lt gt

제휴 회사에 회 의 아이디 개인 정 를 전송할 있도

한 조항은 고객에게 당한 조항이다

1 7 8① ② ③

④ 9 ⑤ 10

립 도 규

1 ( )

규 립 도 립 어린 청 도(

포 다 료 시 열 시 말) (

다 에 사 규 립 도)

편 진 다

2 ( )

규 립 도 도 다 에( lsquo rsquo )

고 는 도 에 도lsquo rsquo 2 2

료 에 여 다 다만 특 료 귀

료 등 료 에 사 립 도

도 다 다( lsquo rsquo )

3 ( )

도 다 각 같다①

공 공 다만 연1

연 간 다

년 학 간고사 대비2013 2 현대고 대비

ECN-0102-2013-001-000076193

매월 째 째 월2

도 도 리 그 사3

가 다고 는

도 에 미리 게1 3②

시 여 다

시간4 ( )

도 시간 도 여 게시 다

등 등5 ( )

도 료 시 는 는 도①

지에 등 후

등 에 사 도②

사 료6 ( )

도 료 시 에 사 료는 도

7 ( )

는 다 각 여 는 니 다

도 료 시 상 리1 lsquo rsquo

도 료 시 훼 는2 middot

지 가 닌 곳에 식 거 담3

우는

도 보 등 보 검색열4 middot

그 에 도 질 지 여 도5

여 게시 사 는

질 지8 ( )

도 다 거 도①

질 게 우 가 는 에 여는 도

도 가 각 어느7②

에는 지 게 거 도

료9 ( )

도 료는 다 각 경우 다①

상 도 간에 료 는 것 말1 (

다 등 다 도 과 여 경우)

공 원 공 상 는 경우2

그 에 도 다고 는 경우3

가능 도 료 는 도②

는 에 다

변상10 ( )

가 도 료 시 럽 거 거①

못 쓰게 거 어 린 경우에는 변상 여

도 에 변상 여 게시1②

여 다

등 규 에 것 에 도11 ( )

료 시 등에 사

도 다

립 도- (httpwwwnlgokr)

zb48) 다음 정 리 의 의 으 볼 때 가장

이 적인 것은

도 시간 도 여 게시 다①

등 에 사 도②

가능 도 료 는 도 는③

에 다

④ 도 에 변상 여 게10 1

시 여 다

⑤ 도 가 각 어느7

에는 지 거 도

zb49) 를 참고하여 이 어의 성격을 설 한lt gt

것으 적절하 않은 것은

① 보 에 는 어 시 상 고 어 시lt gt lsquo rsquo

에 보여주고 다

② 진 어 어원에 견 고 다

에는 타 어 들어가는 것 다 lsquo rsquo

③ 에 들어갈 말 각각 고 어 어 신 어~

들 언어는 질 격 강 통 없었다

④ 시 우리 에 가 었지만 지 계

과 달리 들 통 사 달 어 웠

년 학 간고사 대비2013 2 현대고 대비

ECN-0102-2013-001-000076193

⑤ 크 몽골 만주 공통어가 우리 어 같

계열에 다는 에 사 특 짐

가( )

善化公主主隱 공주님

他密只嫁良置古 몰 결 고

薯童房乙 맛

夜矣卯乙抱遣去如 에 몰 고 가다

( )

始汝 會隱日恚見隱扐 만 에 본

恥隱汝衣淸隱笑 맑 웃

고 시 여 공 크다 만 다[ ] ( ) ( ) ( ) ( )始 汝 會扐

내다 에 보다 견( ) ( )恚 見 다( )隱

럽다 맑다 청 웃( ) ( ) ( ) ( )恥 衣 淸 笑

zb50) 위의 나 를 함 고 음에 답하( ) lt gt

보lt gt

( )素那或云金川 白城郡蛇山人也

운 사산

는 고 다 는( )[ ( ) ] (素那 金川 白城

사산 사 다) ( ) 郡 蛇山

삼 사- lsquo rsquo 47

에 제 된 단어 의 표 리를 조건(1) lt gt ( ) lt gt

에 맞게 서 하

건lt gt

lsquo 었고 었다 태rsquo

에 제 된 단어 동일한 표 리에(2) lt gt ( )

의해 적은 것을 나 에서 찾아 조건 에 맞게 서 하( ) lt gt

건lt gt

에 당 는 각각( ) 개 쓸 것2 단

당 는 가 여러 개 어도 개만 쓸 것 각2

개 과 도 쪽에 개만2 2

드시 지 것( )

과 동 원리 것lsquo 고

과 동 원리 것 다rsquo

태 것

가( )

素那(或云金川) 白城郡蛇山人也

소나 또는 천 이라 한다 는 성 사( ) ( ) ( )素那 金川 白城郡〔 〕

산 사람이다 현대어 풀이( ) ( )蛇山

나( )

紫布岩乎希 회

執音乎手母牛放敎遣 자 손 암쇼 노히 고

吾 不喩慙 伊賜等肹 肹 나 안디 리샤

花 折叱肹 可獻乎理音如 고 것거 도림다

다 향찰은 리말을 리 으 적은 표 이었 만 생( )

은 고 대를 넘 하고 끊어 고 말았다 랜 세

동안 갈고 닦아 체계적이었던 향찰 표 이 사라졌

을 인은 크게 두 가 나누어 생각해 볼 있다

하나는 족 사회의 한 선호도에서 찾을 있다 라 때

향찰은 주 족 계 에서 사 했을 것으 인다 한 을

알 하고서는 한자를 활 하여 리말을 리 으 표

하 란 가능하 때 이다 런데 족들은 간이 흐

를 향찰과 같은 리 표 을 익혀 사 하 다는

아 한 을 대 사 하는 쪽을 선호하게 되었다 더 이

고 초에 인재 등 을 위해 과거제도가 행되 서 한 선

호도가 더 높아졌고 결 향찰은 소 되고 말았다

또 다른 가능성은 한 어의 특성에서 찾을 있다

터 한 과 일 세 나라는 한자 화 에 속해 다

당연한 이야 겠 만 표의 자인 한자는 어를 표 하

에 매 적절하다 어의 음절은 성 ( ) ( )聲母 韻母

이 어 고 여 에 성조가 추가되어 최종 소리가 결정된

다 래서 어는 단음절을 하나의 한자 표 하 된

다 에 초성 성 종성의 세 가 소가 하나의 음절

년 학 간고사 대비2013 2 현대고 대비

ECN-0102-2013-001-000076193

을 이 는 한 어는 음절 조가 잡하고 음절의 가 많아

서 한자 차 만으 한 어의 소리를 만족 럽게 표 할

없었다 를 들어 한 어에서는 어 니 같이 음절 lsquo rsquo

이 어 단어가 얼마든 있으나 어는( ) 複數音節

자 하나 나타내 만이다lsquo [m ]rsquo 母 ǔ

한편 일 어의 표 은 핵 적 단어는 한자 적고 토는

가나라는 일 의 자 적는 이다 적인 의 를 나

타내는 은 표의 자인 한자 적고 적 계를 나

타내는 토는 표음 자 적는 셈이니 자세히 살펴

리의 향찰 표 을 쏙 빼닮았음을 알 있다 한 어 같

은 착어이 서도 일 어에만 향찰과 유사한 표 이 살아

남은 것은 일 어의 특 때 이다 일 어는 하나의 자음과

음의 결합으 음절을 이 고 침이 거의 없는 음절 언어

이다 이러한 음절의 특색에다가 토가 달한 착어라는 점

이 향찰과 유사한 표 이 살아남을 있는 비결이었다

하 만 같은 착어라도 다양한 음소 침이 달한 한

어는 향찰 표 하는 데 근 적으 한계가 있었다

zb51) 다 하여 의 행에 대한 탐 한 결과( ) lt gt 2

않은 것은

보lt gt

善花公主主隱 공주니믄 공주님( )

----------------------------------------

-

他密只嫁良置古 그 지 얼어 고 몰 결(

----------------------------------------

-

薯童房乙 맛 맛( )

夜矣卯乙抱遺去如 몰 고 가다 에 몰 고(

가다)

주동 역 동- (薯童謠『 』

에 2 ( )他密只嫁良置古

얼다 시집가다 결 다 말 lsquo rsquo

① 실질 미 지니고 므 타 타lsquo ( )rsquo lsquo [ ]

② 에 실질 미 타내고 지 는lsquo rsquo lsquo [ ]rsquo lsquo [ ]密只 密 只

계 타내는

③ 얼어는 실질 미 포 고 므 가lsquo rsquo lsquo [ ]rsquo嫁

것lsquo [ ]rsquo 良

④ 고 어간 는 실질 미 지니고 므lsquo rsquo lsquo -rsquo

것lsquo [ ]rsquo 置

⑤ 고 어미 고는 계 타내고 므lsquo rsquo lsquo- rsquo

고 것lsquo [ ]rsquo 古

가( )

엉 훈 민middot middot middot middot middot世 宗 御 製 訓 民 正 音

말 미 듕 귁에 달middot middot middot middot middot middot middot middot中 國 文 字

니 런middot middot middot middot middot middot 어린middot middot middot middot百 姓

니 고 도 내 들middot middot middot middot middot middot middot middot middot 시러middot

펴 몯middot 미middot middot 니 내middot middot middot middot middot middot middot middot 爲

어엿middot 겨 새middot middot middot 믈여듧middot middot middot middot字 니middot middot middot

사 마다 니겨 킈 middot middot middot middot middot middot middot middot middot便 安

고 미니middot middot middot middot

본 는 상( ) (象

원리에 만들어진 본) ( )形 ㄱ ㄴ ㅁ ㅅ ㅇ

에 는 가 원리에( )加劃

그리고( )ㅋ ㄷ ㅌ ㅂ ㅍ ㅈ ㅊ ㆆ ㅎ

쓰는 병 원리에 만들어진( )竝書

마지막 체( ) ( )異體ㄲ ㄸ ㅃ ㅆ ㅉ ㆅ

ᅀ 다 상 원리에 ㅇ ㄹ

지 는 삼재 상 본 본( ) ( ) ( 天地人 三才

탕 므림과 림에 ) (初ㅡ ㅣ

재)( ) ( )( )出字 再出字ㅗ ㅏ ㅜ ㅓ ㅛ ㅑ ㅜ ㅕ

병 그리고 들 에 다시( )ㅘ ㅝ ㅣ

( )ㅣ ㅢ ㅚ ㅐ ㅟ ㅔ ㆉ ㅒ ㆌ ㅖ ㅙ ㅞ

zb52) 가 에 대한 설 으 르 않은 것을( ) 두 고르

① 어쓰 규 지키고 다

② 리 고 다

③ 말 미 미 등 어 사 다lsquo rsquo

④ 개 지 다

년 학 간고사 대비2013 2 현대고 대비

ECN-0102-2013-001-000076193

⑤ 어 원 에 가 도 고 다

엉 훈 민世 宗 御 製 訓 民 正 音

말 미 듕귁에 달 니

런 어린 니 고 도middot

내 들 시러 펴 몯 미 니middot

내 어엿 겨 새 믈여듧

사 마다 니겨middot 킈 고

미니

훈민 언 본- lsquo rsquo 5 (1459 )

zb53) 위의 에 대한 현대어 풀이가 르~ 않은 것

① 우리 말 과 달

② 어리 말 고 는 것 어도

③ 신 생각 마 껏 펼 는 사 많다

④ 게 생각 여

⑤ 사 마다 게

zb54) 훈민정음 언해 에는 한 을 창제한 동 가 드러나

있다 훈민정음 창제의 정 과 내 이 잘 연결된 것

① 주 신 말 미 듕귁에 달

② 민 신 내 어 겨

③ 신 뻔 킈 고 미니

④ 실 신 사 마다 니겨

⑤ 귀 신 계 주 는 훈민 신과 거리가

가 엉 훈 민( ) middot middot middot middot middot世 宗 御 製 訓 民 正 音 

말 미 귁에 中 國 달 文 字

니 런 어린 니 百 姓

고 도 내 들 시러 펴 몯

미 니 내 어엿 爲 겨 새

믈여듧 니 사 마다 니 字

겨 킈 고 미니 便 安

훈민 언 본- lsquo ( )rsquo ( ) 5 (1459 )訓民正音 世祖

( )

[ 1 ]

동 룡 샤 마다 복( ) ( ) ( )海東 六龍 天福

시니 고 동( ) ( )古聖 同符 시니

[ 2 ]

매 니 곶 여

미 므 니 그 내 러

가 니

[ 125 ]

우 미리( )千世 샨( )定 에( )漢水北 累仁

누 개 샤 복 업 시니( ) ( ) 開國 卜年

신( )聖神 니 샤도 경 근민 샤 욱( )敬天勤民

드시리 다

님 쇼 산 가( ) ( )洛水 山行

미드니 가

어 가- lsquo ( )rsquo 27龍飛御天歌

다 우리신 니쓰고 다만 만 쓰( )

거 샹 귀쳔 다보게 러 귀

여 쓴 도 신 보 가 고 신 에

말 어 보게 각 에 사 들

고 본 몬 능통 후에

죠 죠 니

드 도 만 공 에 사

드 미 죠 고 고 여 보 죠

보다 얼마가 거시 어신고 니 첫

가 죠 니 죠

민 들 어 신 샹

귀쳔 도보고 어보 가 만 늘

고 폐 에 만쓴 죠 민

도 러보지못 고 보니 그게 엇지

심 니 리 보 가 어 운건 다

니 쳣 말마 지 니 고 그

쓰 에 가 우 지 지

몰 거 본후에 가 어 지

고 그니 쓴편지 쟝 보

년 학 간고사 대비2013 2 현대고 대비

ECN-0102-2013-001-000076193

쓴것보다 듸 보고 그 마 니 쓴 고

어 못

그런고 에 리 과 가

만 쓴 못 민 말만 듯고

고 편 그 못 보니 그사 단

병신 못 다고 그사 식 사

니 만 고 다 과 그사

만 고 다 과 업 사 보다 식 고

죠 도 고 각 과

견 고 실 직 귀쳔 간에 그

고도 다 것 몰 귀죡 보다

사 우리 신 귀쳔 다 업

시 신 보고 과 지 게 랴

시니 샹 귀쳔 간에 우리 신 걸

간 보 새지각과 새 걸 미리

독립신- lsquo (1896)rsquo

zb55) 친 어 나의 제 장( ) 2 매 함축적

의 가 가장 유사한 것은

① 지 눈 내리고 매 득 니 내 여 가

사- lsquo rsquo

② 도 어 리듯 그 게 어 다

주 사- lsquo rsquo

③ 눈 살 다 죽 어 린 과 체 여

눈 새벽 지 도 살 다

눈- lsquo rsquo

④ 삶 근심과 고단 에 돌 거니는 여 거 는

여 리 내린 살가지 에 눈 리 눈 리

택 그 생 에- lsquo rsquo

⑤ 늘 러 고 러

청룡 룡 어 개 루 우

신경림 계- lsquo rsquo

zb56) 친 를 위 가 나 에 나타난A B ( ) ( )

세 어의 특 에 의거하여 세 어 표 하

그 산 고 공 도 맑지만

A

주변에 쓰 리는 어리 사 많다

B

건lt gt

식 가 에 타 어 특징에( ) ( )

거 과 어쓰 는 고 지 말 것

A

B

zb57) 가 의( ) 달 아ㆍ 다 의 ( ) 나셔에서 알 있는

세 어 개화 어의 특 을 비 하여 조건 에lt gt

맞게 서 하

건lt gt

어에 는lsquo 개

어에 는 다 태rsquo

zb58) 은 가 는 다 에 나 는 절lt 1gt ( ) lt 2gt ( )

일 를 췌한 것이다 의 의 가 lt 1gt (1)~(2)

유사한 말을 에서 찾아 쓰lt 2gt

보lt 1gt

런 (1) 어린 니 고百 姓

도 내 들 시러 펴 몯 미

사 마다 (2) 니겨 便 安

킈 고 미니

보lt 2gt

죠 고 고 여 보 죠

보다 얼마가 거시 어신고 니 첫 가

죠 니 죠 민

들 어 신 샹 귀쳔

도보고 어보 가 만 늘 고

폐 에 만쓴 죠 민 도

러보지못 고 보니 그게 엇지 심

니 리

년 학 간고사 대비2013 2 현대고 대비

ECN-0102-2013-001-000076193

lt 1 gt

동 룡 샤 마다 복 시( ) ( ) ( )海東 六龍 天福

고 동 시니( ) ( )古聖 同符

lt 2 gt

(A) 매 니 곶

여 니

미 므 니 그 내

러 가 니

lt125 gt

우 미리 샨 에( ) ( ) ( ) 千世 定 漢水北 累

누 개 샤 복 업 시 니( ) ( ) 仁開國 卜年 聖

신( ) 神 니 샤도 경 근민 샤( ) 敬天勤民

욱 드 시 리 다

님 쇼 산 가 ( ) ( )洛水 山行

미드니 가

- lt gt龍飛御天歌

zb59) 장과 내 상 유사한 성격의 조는125

① 뫼 고 고 고 고

어 그린 많고 많고 고 고

어 러 는 울고 울고 가느니

도 견- lt gt

② 강 에 드니 몸 다

그믈 고 가니

뒷 뫼 엄 언 니( )藥

-

③ 말 없는 청산 태 없는 다

값 없는 청 없는 월

에 병 없는 몸 별 없 늙 리

-

④ 가마귀 골에 가지 마

낸 가마귀 새

청강에 것 시 몸 러 가( ) 淸江

-

⑤ 진 골에( ) 白雪

가 매 는 어느 곳에 었는고

에 갈 곳 몰( ) 夕陽

색-

zb60) 위 에 나타난 세 어의 특 으 적절하 않은

것은

① 룡 어 주격 사에 당 는 가 사( ) lsquo rsquo六龍

고 다

② 샤 어에도 어 주체 쓰 다

는 것 다

③ 매 어 달리 사 택에 어

가 지 지지 고 다

④ 므 원 상 직 어 지 다

⑤ 드시리 다 주체 과 상 께 사

고 다

수고 하셨습니다hearts hearts

년 학 간고사 대비2013 2 현대고 대비

ECN-0102-2013-001-000076193

보닷컴에 공 는 별 보는 고등

들 여 주 는

들 습니다 슷 동 지

가 복 는 것 도가

니 복 여 습 시고 거 시

니다

정답 해설

1) 정답[ ] ④

해설 다른 것은 두 특정 업이나 단 내에서 사[ ]

하는 일종의 은어 사회 언에 해당한다 러나

는 언이 아니라 단과대학을 여서 단대 사lsquo rsquo lsquo rsquo lsquo④

대학을 여서 사대라고 한 말에 해당하 일rsquo lsquo rsquo

사회에서도 널리 쓰이 사회 언이라 할

없다

2) 정답[ ] ⑤

해설 사회 언은 같은 단 내에서 쓰이는 언어이[ ] lsquo rsquo

동일 단끼리는 단결 과 친 감을 형성하는

능을 하 리적 안감이 일어나 않는다

3) 정답[ ] ③

해설 사람이라는 차 적 표현에 대한 대안적 표현이[ ]lsquo rsquo

인 아내 처 등으 볼 있다lsquo rsquo

4) 정답[ ]⑤

해설 남성은 주 격 체를 사 한다[ ]

5) 정답[ ] ⑤

해설 흑인은 검다라는 뜻을 가 고 있을 뿐 인[ ]lsquo rsquo lsquo rsquo lsquo rsquo

다 열등한 뜻을 내포하 않는다

6) 정답 살 색 첫 작품[ ] - -

해설 살색 혹은 킨색은 한 인의 피 색을 뜻[ ] lsquo rsquo lsquo rsquo

하는 것으 인종 차 을 추 고 출 이주민

의 평등 을 침해할 있어 년 표 이2005

살 색으 이름을 꾸었다 처녀작은 처녀라lsquo rsquo lsquo rsquo lsquo rsquo

는 단어가 가 고 있는 곡된 성 인 을 한 것

으 첫 작품정도 꾸어 사 하는 것이 좋다lsquo rsquo

7) 정답[ ] ⑤

해설 호는 아들에게 해체를 사 하고 있다[ ] ① ②

장 을 성하는 청자는 자 의 아 느리 아lsquo

들 세 이다 호는 아 느리에게 해rsquo ③

체를 사 하고 있다 호가 느리 아 에게 ④

사 한 해 체 아들에게 사 한 해체는 두 비lsquo rsquo lsquo rsquo

격 체에 해당한다 호는 자 의 아랫사람인 ⑤

느리에게 아들과 마찬가 해체를 사 하는 것이

상 이 만 임 을 한 느리에게 고마 과 쁨

존 의 표 를 하 위해 자 의 아 에게 말하듯

해 체를 사 하고 있다

8) 정답[ ] ③

9) 정답[ ] ⑤

10) 정답[ ] ①

해설 청자 할아 가 장의 주체 아 다 높을[ ] ( ) ( )

경 에는 압존 에 의해 장의 주체를 높이 않는lsquo rsquo

다 러 아 서가 아닌 아 는으 계 lsquo rsquo lsquo rsquo lsquo

니다 가 아닌 있 니다 표현하는 것이 르rsquo lsquo rsquo

11) 정답 당이 당을 쫒았다 당이[ ]

당에 다

해설[ ]

12) 정답[ ] ⑤

해설 서 다른 높임표현을 통해 청자에 대해 리[ ] ⑤

적 거리감을 나타내는 인 은 이 아니라 현정이

다 가 에서 현정은 에게 해 체를 사 함으 써 ( )

친근감을 드러낸다 나 에서 연 을 게을리하는 역 ( )

도 들 때 에 화가 난 현정이 선생님에게 항의하

는 장 에서는 하 체를 사 하여 리적 거리lsquo rsquo

가 어졌음을 나타내고 있다

13) 정답[ ] ①

해설 는 는 얼 빛이 날과 어찌 다르 고[ ] lsquo rsquo

라는 뜻으 전과 달리 임이 화자를 않고

있음을 알 있다

14) 정답 달리 후 가 있다 이를 통해 경[ ] lt gt

쾌한 음악성을 형성하고 노 젓는 상황을 체적으

형상화하는 역할을 한다

15) 정답[ ] ①

16) 정답[ ] ⑤

해설 다 의 자연은 를 성찰하게 하는 대상[ ] ( )⑤

이자 정의 대상이다 의 자연은 자 의 상황과 ⑤

처 를 드러내는 경으 서의 역할을 하 이

이 없다

17) 정답[ ] ③

해설 는 빈천 을 해결하고자 했으나 강산[ ] lsquo ( )rsquo 貧賤③

과 풍 을 달라는 에 거절하 다고 함으 써 자

연에 대한 애정을 드러내고 있으 는 않는

임에 대한 망을 개에게 전가 켜서 임에 대한 리

을 드러내고 있다

18) 정답[ ] ③

년 학 간고사 대비2013 2 현대고 대비

ECN-0102-2013-001-000076193

19) 정답[ ] ⑤

해설 고상한 음악가의 이름을 리말 꽝 럽[ ]

게 꿈으 써 언어유희를 통해 음을 유 하고 있

다 이는 고상한 척하는 총 를 비꼼으 써 비판적

태도를 드러내는 것이 대상을 꽝 럽게 표현

하여 총 의 허 과 사치를 풍자하고 있다

20) 정답[ ] ⑤

해설 는 작품 속 경에 대한 설 이 드러나는 것이[ ]

서 자의 주 적인 견해가 접적으 드러나는 것이

아니다

21) 정답[ ] ⑤

22) 정답[ ] ②

23) 정답[ ] ④

24) 정답[ ] ①

해설 적강 티프는 주인공의 비 한 출생이나 능[ ] ①

과 이 있는 것으 조정의 능함을 풍자하는lsquo rsquo

것과는 거리가 다

25) 정답 픔 나[ ] ( )

해설 의 음악은 고통 는 사람들을 위 하고 아픔[ ] lsquo rsquo

을 치유해 주는 능을 한다고 할 있다 의 lt gt

픔 도 소 된 이 과 더 어 살아가는 따뜻한 마음lsquo rsquo

을 상 한다

26) 정답[ ] ⑤

해설 에게 선천적으 주어 각 장애라는 역경[ ]

은 의 이라는 가사 연 을 있다lsquo rsquo

27) 정답[ ] ④

해설 는 장 란 선 에게 은 개인적인 인상을[ ]

소녀 장정 등으 표현한 것이다lsquo rsquo

28) 정답[ ] ②

해설 담자가 피 담자의 언어적 표현이나 비언어[ ]②

적 표현 하 독자는 담의 위 나 피

담자의 감정 상태를 알 있다 이를 통해 독자는

담 상황을 더 생생하게 느낄 있고 피 담자

를 더 잘 이해할 있게 된다

29) 정답[ ]③

해설 일상생활과 역도 선 서의 성과에 된 것에서[ ]

역도를 하 서 겪는 어 과 내적 고민으 화제를

전화하 위한 것이다

30) 정답[ ] ①

해설 릿속에 새겨 넣듯 이 억되도 함 세상[ ] ② ③

살이가 힘들고 고생 러 속 하여 자유를 ④

가 없는 고통의 상태를 비유적으 이르는 말

적의 침입을 막 위해 쌓은 축 켜야 할⑤

대상을 비유적으 이르는 말이다

31) 정답[ ] ④

해설 이 의 종류는 전 으 인 사건 경[ ] lsquo

비평을 성 소 삼는다rsquo

32) 정답[ ] ④

해설 근은 삼대독자 태어났음을 에서 확인할[ ]

있다 형제들과의 담은 이뤄 가 없다

33) 정답[ ] ⑤

해설 근은 가난에도 하고 화가를 꿈꾸었다[ ] (3

단 또한 다른 화가 망생들은 정 육을)

위해 상 학 학 해 유학 에 랐 만

근은 다른 을 찾아야 했다 단 세에(5 ) 18

근은 조선 전람회에 입선하 다 단 의(6 )

만종은 인간과 자연이 엮어 가는 경건한 조화 을lsquo rsquo

나타낸다

34) 정답[ ] ①

해설 근이 속에서도 창작활동을 추 않고[ ]

하는 닭은 은 세상과 타협할 르는

근이 세상의 이해를 하 위한 가장 떳떳한 단

이 때 이다

35) 정답[ ] ⑤

해설 전 은 서 자의 주 적인 평이 리는 것이[ ]

만 위 제 은 인 이 살았던 대 사회적 경

을 통해 객 적인 인 의 을 제 하고 있다

36) 정답[ ] ⑤

해설 전 은 인 사건 경 비평이라는[ ] lsquo rsquo⑤

성 이 어져 있다

37) 정답[ ] ①

해설 이 은 동양인과 서양인의 사고 에 차이가[ ]

있다는 것을 대조를 통해 설 하고 있다 또 쓴이

의 제자가 축 경 를 러 가서 경험한 일화를

통해 동양인이 서양인에 비해 주 상황에 더 많은

주의를 인다는 주장을 뒷 침하고 있다

38) 정답[ ] ④

39) 정답[ ] ②

40) 정답[ ] ②

41) 정답[ ] ④

42) 정답[ ] ③

43) 정답[ ] ④

44) 정답 도서 의 휴 일 도서 의 이 간 도서의[ ]

해설 도서 장은 임의 정한 휴 일과 도서 이[ ]

간 도서의 상 등을 게 할 의 가 있다

년 학 간고사 대비2013 2 현대고 대비

ECN-0102-2013-001-000076193

45) 정답[ ] ①

해설 제 조의 정 휴 일 의 휴 일의 사전 게[ ] 3

는 도서 장의 의 조항에 속한다

46) 정답[ ] ①

해설 개인 정 호 의 를 제 하 했 만 항[ ]

나눠서 제 하 않고 대 나열하고 있다

47) 정답[ ] ②

해설 제 조의 내 을 회사는 다른 회사 협[ ] 7 lsquo

계약을 통해 서비 를 제공하는 경 회 의 아이디

등 개인 정 를 해당 회사에 전송할 있다는 내rsquo

이 있으 의 제점을 제 할 있다②

48) 정답[ ] ④

해설 는 도서 장의 의 에 해당하고 나 는 도[ ] ④

서 장의 리에 해당한다

49) 정답[ ] ③

50) 정답 은 음독으 적었고 은 훈독으 적었[ ] (1)

다 과 동일한 표 리 적은 것은 이고 (2) ce

과 동일한 표 리 적은 것은 이다ab

51) 정답[ ] ③

52) 정답[ ] ①②

53) 정답[ ] ③

54) 정답[ ] ③

55) 정답[ ] ①

56) 정답 른 죠코 어린 노 하니라[ ] A B

57) 정답 세 어에서는 활 형이 칙적으[ ] lsquo rsquoㄹㅇ

나타났 만 개화 어에서는 활 형이 쓰 다 lsquo rsquo ㄹㄴ

58) 정답 호 가 흔[ ] (1) (2)

59) 정답[ ] ④

60) 정답[ ] ③

Page 20: 현대고대비 국어 - chamsoriedu.com 「콘텐츠산업진흥 법」외 에도 저작권 의하여 ... 다른주체에게어떤동작을하도록만드는것을나타내는

년 학 간고사 대비2013 2 현대고 대비

ECN-0102-2013-001-000076193

시간 많지 다 청량리 생 병원

마지막 상 경 릿 게 들어 다 그 는 십

만 큰 가 상 말 다

지 못 들 마 갈 고 돗

도시민들 싹 싹 탔다 가 시

월에 병원에 원 가 폐 진 몸도4 ( )疲弊

갈 미 지 못 고 었다 가는 얼마( ) 解渴

지 생 에 생각 가

마감 는 신 평생 십 만에

가 과 많 닮 다고 생각 지는

가 운 는 어 어ldquo rdquo 1965helliphellip

월 새벽 시 태 없 거웠고 는5 6 1

없 그 병원에 퇴원 집 가

는 마지막 마 고 마 내 거 다 가

죽 간신 에 실 다 사는 어느 가5 lsquo

죽 는 말 가 식 다 신rsquo

상에 각 시키는 에 실 어느( ) lsquo刻印

가는 후 민 가가 근 었다rsquo

ldquo 는 간 과 진실 그 다는 에

단 평 견 가지고 다 내가 그

리는 간상 단 고 다 지 다 는 그들 가

에 는 평 지 니 그리고 어린 들

미지 겨 그린다rdquo

근 간 과 진실 그리고 싶어 가

다 근에게 그것 진리 다 거 다 없 거

고 다 없 는 것 진리다

근 진리는 후 쪽 었다 신산 삶 ( )辛酸

었 질곡 역사 에 지냈 가 눈에( )桎梏

든 것 료 단 료 게 보 것

었다 그것 그 에 겨우겨우 슬

슬 생 어가는 간들 었다 리

과 단 리 고리에 검 마

없 거리 돌 상

것 없는 등 근에게 상에 과 진실

엄 다는 사실 리는 가 실( )儼存

고 가 과 역경 에 도 근 내 포 없었

후 보루 다 도 도( ) 365堡壘

간 근 여 시 것

간에 지닌 가 근 1914 2

월 강원도 림리에 삼 독21

태어났다 어 근 복 그것 그리

가지 못 다 근 곱 살 지는 산

사업에 실 고 답마 에 내 갔다 근

그림 럼 쫓 다니 가 시 것 다 상

진 것도 가 었다 러 가 에도

고 근 가 꿈꾸었다 근 가 꿈꾸게

것 보통 업 원색1926

도 만 었다lsquo rsquo

그림 가 근 경 공주- ldquo rdquo ( 2009)

zb31) 다음 이 같은 의 성 소에 해당하 않은

것은

사건 평① ② ③

④ 주 ⑤ 경

가 운 는 어 어ldquo rdquo 1965helliphellip

월 새벽 시 태 없 거웠고 는5 6 1

없 그 병원에 퇴원 집 가

는 마지막 마 고 마 내 거 다 가

죽 간신 에 실 다 사는 어느 가5 lsquo

죽 는 말 가 식 다 신rsquo

상에 각 시키는 에 실 어느( ) lsquo刻印

가는 후 민 가가 근 었다rsquo

는 간 과 진실 그 다는 에ldquo

단 평 견 가지고 다 내가 그

리는 간상 단 고 다 지 다 는 그들 가

에 는 평 지 니 그리고 어린 들

미지 겨 그린다rdquo

근 간 과 진실 그리고 싶어 가

다 근에게 그것 진리 다 거 다 없 거

고 다 없 는 것 진리다

근 진리는 후 쪽 었다 신산 삶 ( )辛酸

었 질곡 역사 에 지냈 가 눈에( )桎梏

든 것 료 단 료 게 보 것

었다 그것 그 에 겨우겨우 슬

슬 생 어가는 간들 었다 리

과 단 리 고리에 검 마

없 거리 돌 상

것 없는 등 근에게 상에 과 진실

엄 다는 사실 리는 가 실( )儼存

고 가 과 역경 에 도 근 내 포 없었

후 보루 다 도 도( ) 365堡壘

간 근 여 시 것

간에 지닌 가 근 1914 2

월 강원도 림리에 삼 독21

태어났다 어 근 복 그것 그리

가지 못 다 근 곱 살 지는 산

사업에 실 고 답마 에 내 갔다 근

그림 럼 쫓 다니 가 시 것 다 상

진 것도 가 었다 러 가 에도

고 근 가 꿈꾸었다 근 가 꿈꾸게

것 보통 업 원색1926

도 만 었다lsquo rsquo

공주 그림 가 근 경- ldquo rdquo ( 2009)

년 학 간고사 대비2013 2 현대고 대비

ECN-0102-2013-001-000076193

zb32) 위 을 작성하는 과정에서 되어 활 된 자

어 것은

신 사 료① 연보②

고③ ④ 들과 담

⑤ 에 평

는 간 과 진실 그 다는 에ldquo

단 평 견 가지고 다 내가 그

리는 간상 단 고 다 지 다 는 그들 가

에 는 평 지 니 그리고 어린 들

미지 겨 그린다rdquo

근 간 과 진실 그리고 싶어 가

다 근에게 그것 진리 다 거 다 없 거

고 다 없 는 것 진리다

근 진리는 후 쪽 었다 신산 삶 ( )辛酸

었 질곡 역사 에 지냈 가( )桎梏

눈에 든 것 료 단 료 게 보

것 었다 그것 그 에 겨우겨우

슬 슬 생 어가는 간들 었다

리 과 단 리 고리에 검 마

없 거리 돌 상

것 없는 등 근에게 상에 과

진실 엄 다는 사실 리는 가 실( )儼存

고 가 과 역경 에 도 근 내 포

없었 후 보루 다 도 도( ) 365堡壘

간 근 여 시

것 다

간에 지닌 가 근 1914 2

월 강원도 림리에 삼 독21

태어났다 어 근 복 그것 그리

가지 못 다 근 곱 살 지는 산

사업에 실 고 답마 에 내 갔다 근

그림 럼 쫓 다니 가 시 것 다 상

진 것도 가 었다 러 가 에도

고 근 가 꿈꾸었다 근 가 꿈꾸게

것 보통 업 원색1926

도 만 었다lsquo rsquo

질 루 마 가 도 린다 경건

움 느껴지는 경 다 훗 근 그림에

과 는 거 것( )裸木

만 간과 연 엮어 가는 경건 움lsquo rsquo

니었

같 가가 고 싶었 근에게 그 꿈에 다

가가는 지 다 다 가 지망생들 규 미

상 에 진 고

에 지만 근 다 다 근

미 에 운 것 보통 시 미 시간

다 그런 그에게 없는 연습 가가

통 다 가 귀 시 지 도

얻는 뛸 듯 뻤지만 마 도 가 에

듯 는 었 에 어린 근 주 에

에 그림 그리고 지우고 복( )粉板

시간 가는 게 루 보냈다

근 그 갈 가가 것 열여( )渴求

었 다가 미1932 lsquo rsquo ( lsquo

미 에 다 다는 고 마rsquo) lsquo rsquo

가 근 집 고도 지는 시골 경

그린 그림 다 후 근 에 1943 22

지 미 에 그림 고

에 걸쳐 다 미 근 가

동 는 었다

공주 그림 가 근 경- ldquo rdquo ( 2009)

zb33) 위 의 내 과 일치하는 것은

가 근 가 꿈 포 다①

근 당 가들과 께 에 다②

살 근 가 걷20③

게 었다

④ 만 통 근 역경 겨내는lsquo rsquo

느 다

⑤ 근 간 과 진실 그리 에 그 에

드러 는 간상 단 다

계 시 주 근 건강

걸었다 신 과 간에 상 다 건강

신 는 눈에도 다 근 쪽 눈 뿌 게

보 지 과에 다 다 시 지지 고 결

내 었다 시 지만 마 막막

다 늦어 결 근 쪽 눈 고 말 다

쪽 눈 근에게는 쪽 눈 었고

계 었다 그 근 는 여 그lsquo rsquo

다 근 에 같 그림 그 었다1950

시 그림 는 여 쪽lsquo rsquo

고 어 마주 고 는 그림1963

여 과 동 다 마 복

그린 듯 눈 내리 새 게 다 지

사 다 근 게 복 것

복 상과 타 는 근 상

가 떳떳 단 었고 근 그리고

간 과 진실 에 다가가 가 근다

운 었다 근 신에게 당당 지 그리고

그 다 근 그림에 단 복 보다

년 학 간고사 대비2013 2 현대고 대비

ECN-0102-2013-001-000076193

태 도 그리고 극 보다 과

얻 여 었다 과 통

근 그리고 는 재 고 에 질

만들고 특 것 다

공주 그림 가 근 경- ldquo rdquo( 2009)

zb34) 의 이유에 대해 추 한 것으 적절하 않은 것

상과 타 시도①

보다 과 얻②

근 신에게 당당 지③

④ 간 과 진실 에 다가

⑤ 태 도 얻

근 가가 었지만 그 다니 가

럼 어지지 다 복과 쟁 거쳐 시

는 가 근에게 생계 사 에

운 사 다 에 키에 건( ) 178cm死鬪

체 근 에 동 역 업( )荷役

가 생계 다 쟁

에는 동에 운 상우 주 미

죄 사 에 그림 그리는 시 다 그곳에

에 동 역 업 것에

결 것 럼 보 다 지만 그런 것만도

니었다 그림 그리는 고는 지만 매 근

는 극 간 과 별 없는 경 리 그림

벽에 그리는 것 었다 우도 리 없었다 근

트 는 우 그림 그 다 생

계 그림 단 것 다

후 근 지 신 계 리에 미

엑 리 겼다 근 곳에

건 사 크 에 미 들 ( )

상 상 그 다 근 갖 다 겪

냈다 그리고 결 그 돈

신동에 어 사리 집 마 다 마 ㄷ

루 심 쪽에는 과 엌 쪽에는 건

었다 건 주고 근 가 에

여 살 다 심 에는 지 집어

쓰고 지만 곳 근 가 에게 러웠

보 리 다 근 과 마루 업실 삼 그림

그 다 신동 마루는 근 그림에 등 는 lsquo rsquo

같 상들 지 다 시 고

에 들 폐허가

가 업실 었다

공주 그림 가 근 경- ldquo rdquo( 2009)

zb35) 위 에 대한 설 으 적절한 것은

업 시 여 훈과 감동 다①

에 주 평 드러 다②

사 사 등 식 과 ③

④ 다 근거 시 여 삶에

⑤ 살 시 사 경 께 여

습 시 다

가 시간 많지 다 청량리 생 병원( )

마지막 상 경 릿 게 들어 다 그 는

십 만 큰 가 상 말 다

지 못 들 마 갈 고 돗

도시민들 싹 싹 탔다 가 시

월에 병원에 원4 가 폐( )疲弊

진 몸도 갈 미 지 못 고 었다( )解渴 가는

얼마 지 생 에 생각

가 마감 는 신 평생 십 만에

가 과 많 닮 다고 생각 지는

가 운 는 어 어( ) ldquo rdquohelliphellip

월 새벽 시1965 5 6 1 태 없 거웠고

는 없 그 병원에 퇴원 집

가는 마지막 마 고 마 내 거 다

가 죽 간신 에 실 다 사는 어느5 lsquo

가 죽 는 말 가 식 다 신rsquo

상에 각 시키는 에 실( )刻印

어느 가는 후 민 가가 근 었다lsquo rsquo

다 는 간 과 진실 그 다는( ) ldquo

에 단 평 견 가지고 다 내

가 그리는 간상 단 고 다 지 다 는 가

에 는 평 지 니 그리고 어린 들

미지 겨 그린다rdquo

근 간 과 진실 그리고 싶어( )

가 다 근에게 그것 진리 다 거 다 없

년 학 간고사 대비2013 2 현대고 대비

ECN-0102-2013-001-000076193

거 고 다 없 는 것 진리

다 근 진리는 후 쪽 었다 신산( )辛酸 삶

었 질곡 역사 에 지냈( )桎梏

가 눈에 든 것 료 단 료 게 보

것 었다 그것 그 에 겨우겨우

슬 슬 생 어가는 간들 었다

리 과 단 리 고리에 검

마 없 거리 돌

상 것 없는 등 근에게 상에

과 진실 엄 다는 사실 리는 가 실( )儼存

고 가 과 역경 에 도 근 내 포

없었 후 보루 다( ) 堡壘 도 365

도 간 근 여

시 것 다

마 같 가가 고 싶었 근에게 그 꿈( )

에 다가가는 지 다 다 가 지망생들

규 미 상 에 진 고

에 지만 근 다 다 근

미 에 운 것 보통 시 미 시간

다 그런 그에게 없는 연습 가가

통 다 가 귀 시 지 도

얻는 뛸 듯 뻤지만 마 도 (

는 었 에 어린 근 주 에)

에 그림 그리고 지우고( )粉板

복 시간 가는 게 루 보냈다

zb36) 전 의 성 소가 아닌 것을 고르

① 평 ② 사건 ③ 경

④ ⑤ 훈

늘 지 상에 살고 는 사 들 억 도가10

고 그리 지 통 고 는 사 들( )知的

그보다 훨 많 억 도는 고 지 20

통 다 그런 지 고 2500

그리 간 보는 과 사 에

매우 달 뿐만 니 과 에 도 극

루고 었다 미 운 그런 들

살고 는 동 과 사 들 사고 식에

큰 가 다는 다

고 그리 들 우주 개별 고 독립

사 들 생각 지만 고 들 우

주 연 질 간주 다 같( ) 看做

각 도 들에게는 연 질

었지만 그리 들에게는 미 들 결 었

다 고 과 그리 들 사 같

는 동 과 사 에 도 견 다

지심리 미 마 드 겐트 는

살 들에 에 지 다

연 동 과 상 다 과 같 실험

다 크 만든 미드 도 보

여 주고 그 상 닥 고 주었다lsquo (Dax)rsquo

실 닥 는 재 지 는 것 실험 가lsquo rsquo

만들어 낸 다 그런 다 개 다 체 보

여 주었는 는 미드 지만 틱

만들었고 다 는 재료는 크 지만

달 다 그러고 어 것 닥 지 사 들에게 고 lsquo rsquo

게 니 들 주 같 고 는

체 택 고 동 들 같 재료 만들어진 체

택 다 러 는 심지어 살짜리

들에게 도 타났다 것 곧 과 동

다 상 보고 다는 것 미 다

개별 사 보고 고 동 연 질 보

고 는 것 다

동 들 주변 상 에 맞 어 동 고

에 다 사 들 태도 동에 보다 많

주 울 다 동 가 미시간 에

에 경험 다 그는 미식

경 보러 가게 었는 경 체는 매우 재미 었

주변 들 동에 질 다 그 는

들 계 어 상태 경 다

어 들 에 에 그 시 가 계 가

진 것 다 상 살펴 는 말 들 lsquo rsquo

에 그는 에 시 어 도 뒷사

생각 곧 다시 곤 것 다 그런 그에게 뒷

사 고 지 는 들 동 럼

어 웠다

생각 지도 리 드 니 벳-

zb37) 다음 위 의 내 전개 으 만 인lt gt

것은

lt gt

대조의 통해 대상이 닌 특성을 설 하고 있다

일화를 제 하여 자 의 주장을 뒷 침하고 있다

유추의 을 사 하여 독자의 의해를 돕고 있다

대상이 형성되는 과정을 간적 서에 따라 서 하고 있

① ②

③ ④

년 학 간고사 대비2013 2 현대고 대비

ECN-0102-2013-001-000076193

가 우리가 말 고 쓰는 든 단어가 사 에 는( )

것 니다 사 격에 가 는 지만

어 사 과 같 특별 는 사 니lsquo rsquo

단어 격 보 단어가 사 에

등재 어 다 리 리 사 는 단어 도 그

것 시 사 는 어 고 사 에

격 보 것 니다

러 얼 은 사전에 를 있는가 이에 대한 답lsquo rsquo

은 얼 이 유행어인가 아닌가에 따라 갈라 다 이 단어lsquo rsquo

는 년 어 자 에 랐고 쓰이고 있으2002 lsquo rsquo

유행어라고 하 에는 생 이 다 런데 계속

을 유 하 서 사전에 등재될 자격을 획득할 것인가 이

에 대한 답을 내리 는 히 어 다

여 서 가 를 고 해 볼 있다 첫 는 이 단어

를 써야 할 필 가 속적으 있는가 하는 점이다

상주의 열풍에 휩 인 사회 위 에 편 해서 퍼 말

이 얼 인데 과연 런 위 가 속될 것인가 이에lsquo rsquo

대해 필자의 생각은 정적이다 사회 위 가 뀌

런 말을 쓸 일이 없어 것이다

다음은 단어의 성이다 단어의 성이 사회적으 거

감이 없으 계속 사 될 가능성이 높다 런 에서

얼 은 좋은 조건이 아니다 익히 알 졌듯이 이lsquo rsquo

말은 얼 과 청소년층에서 속어 사 하는 이 결합lsquo rsquo lsquo rsquo

된 말이다 얼 에서 얼 을 리하는 조어 도 lsquo rsquo lsquo -rsquo

어에서는 매 낯선 이다 이것만으 도 거 감을 갖

는 사람들이 있다 더 나 속어 결합한 말이다 얼 lsquo rsquo

이 널리 퍼졌다 해도 은 여전히 청소년층의 속어lsquo rsquo

남아 있다 속어는 자연 럽게 아 자리에서나 쓰 에는

담 러 말이다 러한 담을 하고 사

역을 넓혀 가는 속어도 없 는 않다 특히 얼 은 lsquo rsquo

에도 종종 등장한다 만큼 거 감이 많이 희석되었다

고 할 있다 러나 일상의 자연 러 대화에서도 거

리낌 없이 등장하는가 게 는 되 않았다고 생

각한다

얼 이 유사어인 쌈 등을 만들어 내고lsquo rsquo lsquo rsquo

있으니 살아남을 있을 것이라고 는 견해도 있을 것

이다 러나 간이 나 서 유사어를 포함하여 든

말이 사라 사 는 많다 유사어가 많다는 것이 생 을

유 할 있는 절대적인 조건은 아니다

나 언젠가 터 사람들은 어느 단에서 얼 이 가장( )

쁜 사람을 가리켜 얼 이라고 르고 있다 이 얼lsquo rsquo lsquo rsquo

이라는 단어가 최근 어사전에 라 항간에 논란이 일고

있다 아닌 게 아니라 얼 은 유행어처럼 인다 생 lsquo rsquo

도 리 래되 않은 것 같고 언제 사라 도 알

없다 게다가 젊은이들 사이에서 주 쓰일 뿐이다 이런

단어를 사전에 는다는 게 하 이 없어 이 도

한다

러나 속단은 이다 차근차근 따져 볼 일이다

선 얼 이 일 적 유행어인 아닌 주의 게 들여다lsquo rsquo

볼 필 가 있다 유행어란 유행에 따라 빠르게 유포되었

다가 단 간 내에 소 되는 단어나 를 가리킨다

얼 은 인터넷을 통해 속히 퍼 말이다 하 만 일lsquo rsquo

적인 유행어처럼 단 간 내에 사라 않았을 뿐 아니라

현재 도 잦은 빈도 사 되고 있고 앞으 도 상당

간 사 될 것으 측된다 한 언 재단의 뉴 검 lsquo rsquo

색 사이트에 따르 얼 은 년 에 처음 나타난lsquo rsquo 2001

이후 꾸 히 사 되고 있다

이 같은 사 빈도는 얼 이 일 적 유행어 는 현lsquo rsquo

저히 다르다는 것을 여 다 장 간의 생존 만으 도

얼 은 이 한 어의 어휘 에 를 자격을 얻었다lsquo rsquo

고 할 있다 더 이 이라는 비 적 정제된 매체에

높은 빈도 쓰이고 있 않은가 사 빈도 측 에서

필통이나 연필과 같은 단어 대등하거나 더 많이 쓰lsquo rsquo lsquo rsquo

다는 것은 결코 가 게 볼 일이 아니다

이제는 사전이 언어 현 을 빠르게 하는 게 덕인

대가 되었다 세계적으 유 한 의 사전들도 경쟁

적으 어를 고 있다

하 만 얼 은 젊은이들이나 쓰는 속어라고 흠을 잡을lsquo rsquo

도 르겠다 얼 이 주 젊은 층에서 많이 쓰 lsquo rsquo

는 속어임에 틀림없다 러나 어사전에 표 적이고 품

위 있는 말만 어야 한다고 생각한다 것은 커다란

해다 당장 아 어사전이나 펼쳐 라 속어는

설과 같은 비어나 죄자들이 쓰는 은어 어

마니 같은 소 의 사람만이 쓰는 말 도 라 있

않은가 사전은 말 치에 일정 빈도 이상 나타나는 말이

라 말이든 다 할 있다

zb38) 가 나 에 대한 다음의 설( ) ( ) 않은 것은

① 가 는 얼짱 사 에 등재 것에( ) ( ) lsquo rsquo

보 고 다

② 사 등재 가는 단어 격에( )

고 고 는 언 들 언어 사 도에 고 다 ( )

③ 가 얼짱 어지만 신 과 같 매( ) ( ) lsquo rsquo

체에 도 사 는 말 는 고 다

④ 가는 얼짱 어 보고 크게 가지 근( ) lsquo rsquo 3

거 들어 뒷 고 다

⑤ 는 얼짱 어 는 다 특 다는( ) lsquo rsquo

근거 에도 크게 가지 근거 가 들어 주 2

뒷 고 다

가 늘 지 상에 살고 는 사 들 억( ) 10

도가 고 그리 지 통 고 는 사 들

그보다 훨 많 억 도는 고 지 20

통 다 그런 지 고 2500

년 학 간고사 대비2013 2 현대고 대비

ECN-0102-2013-001-000076193

그리 간 보는 과 사 에

매우 달 뿐만 니 과 에 도 극

루고 었다 미 운 그런 들

살고 는 동 과 사 들 사고 식에

큰 가 다는 다

고 그리 들 우주 개별 고 독립

사 들 생각 지만 고 들 우

주 연 질 간주 다 같 각

도 들에게는 연 질 었지

만 그리 들에게는 미 들 결 었다

고 과 그리 들 사 같 는

동 과 사 에 도 견 다

인 리학자인 츠 이마이 디드 겐트너는 두

살이 채 안 된 아이들에서 터 성인에 이르 다양한

연 대의 동양인과 서양인을 대상으 다음과 같은 험

을 했다 저 코르크 만든 피라 드 양의 도형을

여 주고 대상의 이름을 닥 라고 알 주었다lsquo (Dax)rsquo

제 닥 는 존재하 않는 것으 험자가 임의lsquo rsquo

만들어 낸 이름이다 런 다음 두 개의 다른 체를

여 주었는데 하나는 피라 드 양이 만 하얀 플라 틱

으 만들었고 다른 하나는 재 는 코르크 만 양이

달랐다 러고 나서 어떤 것이 닥 인 사람들에게 고 lsquo rsquo

르게 했더니 서양인들은 주 같은 양을 하고 있는

체를 선택했고 동양인들은 같은 재 만들어 체를

선택했다 이러한 차이는 성인은 어 두 살 리

아이들에게서도 나타났다 이것은 곧 서양인과 동양인은

서 다른 세상을 고 있다는 것을 의 한다 략 ( )

는 아주 단 하 서도 인상적인 험을 했다

험에는 동서양의 대학생들이 참여했다 는 험 참가자

들에게 컴퓨터 화 을 통해 속 장 을 담은 애니 이션

을 여 주었다 화 의 앙에는 초점의 역할을 하는 커

다란 고 한 마리가 있었고 주위에는 다른 생

들과 초 자갈 거품 등이 함 제 되었다 화 을

두 씩 후 참가자들은 자 이 것을 회상해 라는

를 았다

결과 서양인 대학생들과 동양인 대학생 두 앙

의 초점 역할을 했던 고 를 동일한 정도 언 했으

나 경 소 위 거품 초 다른 생 들 에 ( )

대해서는 동양인 대학생들이 서양인 대학생들 다 60

이상 더 많이 언 했다 뿐만 아니라 동양인 학생들은 서

양인 학생들에 비해 개 적인 고 다 전체적인 계

를 더 언 하는 경향을 다 략 또한 경의 일 ( )

를 화 킨 림을 제 하 을 때 동양인 대학생들은 대

경의 화를 알아챘 만 서양인 대학생들은 경

의 화를 거의 알아차리 했다 략 ( )

따라서 서양인들만을 대상으 연 한 화lsquo

편성 결 은 잘 된 것일 도 있다 각 과정과 인rsquo

과정의 어떤 이 화 편적이고 어떤 이

화에 따라 달라 는 는 앞으 많은 연 를 통하여 논의

되어야 한다

나 어떤 의 에서 리 두는 이 화적이다 리( )

안에는 다른 사람들과 더 친 한 계를 유 하 는 상호

의존성과 다른 사람들 터 독립적인 존재 살아가 는

독립성이 혼재한다 따라서 이 에서 어떤 특성이 더 강

하게 각되는 상황에 놓이느냐에 따라 서 다른 화적

특 을 일 있다 결 리 두는 어떤 경 에는

동양인처럼 행동하고 어떤 경 에는 서양인처럼 행동하는

것이다

zb39) 가 에 대한 다음의 설( ) 않은 것은

① 는 신 주 뒷 닥 실험과lsquo rsquo lsquo

니 실험 근거 시 다rsquo

② 동 들 상 간 공통 보다는 에 식

는 강 다

③ 들 주변 맥 에는 심 경 어 사건

과 사건 사 계에 상 민감 다

④ 는 동 과 틀린 지 고 는 것lsquo rsquo

니 다 고 다 lsquo rsquo

⑤ 가에 우리 사 들 개 시 가 원( )

집 경 말 고 는 것 개 보다는

에 고 는 것에 다

늘 지 상에 살고 는 사 들 억 도가10

고 그리 지 통 고 는 사 들( )知的

그보다 훨 많 억 도는 고 지 20

통 다 그런 지 고 2500

그리 간 보는 과 사 에

매우 달 뿐만 니 과 에 도 극

루고 었다 미 운 그런 들

살고 는 동 과 사 들 사고 식에

큰 가 다는 다

지심리 미 마 드 겐트 는 동

과 상 다 과 같 실험 다

크 만든 미드 도 보여 주고 그

상 닥 고 주었다 그런 다lsquo (Dax)rsquo

개 다 체 보여 주었는 는 미드

지만 틱 만들었고 다 는 재료는

크 지만 달 다 그러고 어 것 닥 lsquo

지 사 들에게 고 게 니 들 주 같rsquo

고 는 체 택 고 동 들 같

재료 만들어진 체 택 다 러 는

심지어 살짜리 들에게 도 타났다 것

곧 과 동 다 상 보고 다는

것 미 다 개별 사 보고 고 동

년 학 간고사 대비2013 2 현대고 대비

ECN-0102-2013-001-000076193

연 질 보고 는 것 다

동 들 주변 상 에 맞 어 동 고

에 다 사 들 태도 동에 보다

많 주 울 다 동 가 미시간

에 에 경험 다 그는 미

식 경 보러 가게 었는 경 체는 매우 재

미 었 주변 들 동에 질 다 그

는 들 계 어 상태 경

다 어 들 에 에 그 시 가 계

가 진 것 다 뒷사 고 지 는 들

동 럼 어 웠다

그는 경험에 어 얻어 동 들lsquo

각도 상 본다 는 가 우고rsquo

검 여 주 단 도 상 실험 실

시 다 그는 실험 가 들에게 컴퓨 통

담 니 보여 주었다

에는 역 는 커다 고 마리가 었

고 주 에는 다 생 들과 갈 거 등

께 시 었다 본 후 가 들

신 본 것 상 보 는 지시 다

그 결과 생들과 동 생

역 고 동 도 언

경 거 다 생 들에 ( )

는 동 생들 생들보다 60

상 많 언 다 뿐만 니 동 생들

생들에 개별 고 보다 체 계

언 는 경 보 다 경 변 시

킨 그림 시 동 생들 경

변 지만 생들 경 변

거 리지 못 다

지 지 들만 상 연 lsquo

보편 결 못 것 도 다 지각 과 과rsquo

지 과 어 보편 고 어

에 달 지는지는 많 연 통 여

어 다

리 드 니 벳 생각 지도 사- ldquo rdquo( 2004)

zb40) 위 에 대한 설 으 가장 적절한 것은

① 동 과 생 식 강 고 다

② 가지 실험 통 쓴 고 다

③ 닥 실험에 사 본질에 동 사

상에 주 다

④ 니 실험에 동 과 에 지

각 도에 가 다

⑤ 쓴 는 보편 연 에 드러 우월 에

에 근 고 다

가 동 들 주변 상 에 맞 어 동 고( )

에 다 사 들 태도 동에 보다 많

주 울 다 동 가 미시간 에

에 경험 다 그는 미식

경 보러 가게 었는 경 체는 매우 재미 었

주변 들 동에 질 다 그 는

들 계 어 상태 경 다

어 들 에 에 그 시 가 계 가

진 것 다 상 살펴lsquo 는 말 들rsquo

에 그는 에 시 어 도 뒷사

생각 곧 다시 곤 것 다 그런 그에게

뒷사 고 지 는 들 동 럼

어 웠다

그는 경험에 어 얻어( ) 동 들lsquo

각도 상 본다 는 가 우고rsquo

검 여 주 단 도 상 실험

실시 다 실험에는 동 생들 여 다

그는 실험 가 들에게 컴퓨 통

담 니 보여 주었다 에는

역 는 커다 고 마리가 었고 주 에는

다 생 들과 갈 거 등 께 시

었다 본 후 가 들 신 본 것

상 보 는 지시 다

다 그 결과 생들과 동 생( )

역 고 동 도 언

경 거 다 생 들 에 ( )

는 동 생들 생들보다 60

상 많 언 다 뿐만 니 동 생들

생들에 개별 고 보다 체 계

언 는 경 보 다 들어 동

생들 상 체 연못 럼 보 어ldquo 같rdquo

체 맥 언 시 었지만

생들 상 어 같 큰 고 가 쪽 움ldquo

직 어 같 역 고rdquo

언 시 다 경 변 시킨 그

림 시 동 생들 경 변

지만 생들 경 변 거

리지 못 다

년 학 간고사 대비2013 2 현대고 대비

ECN-0102-2013-001-000076193

게 볼 동 들 보다는 큰 그( )

림 보 에 사 과 체 맥 연결시 지각

는 경 고 체에 특 떼어 내

어 독립 보는 것 낯 어 다 에

들 사 에 고 주변 맥 에는 심 경

에 사건과 사건 사 계에 상

민감 편 다

마 지 지( ) 들만 상 연

보편 결 못 것 도 다lsquo rsquo 지각 과

과 지 과 어 보편 고 어

에 달 지는지는 많 연 통 여

어 다

리 드 니 벳 생각 지도 사- ldquo rdquo( 2004)

zb41) 의 하는 가~ 다른 것은

① ② ③

④ ⑤

얼마 그 에 동 사고 식과

사고 식 보여 주는 내 다

들 에 는 탕 고 같 게

어 겨 고 미 에 는 그 크 럼 큰 고

어리 주고 원 는 어 도 는

상 고 생각 다는 것 다 러

는 어떻게 생 것 고 과 그리 거슬

러 가 보 그 단 다

고 연 경 체 경 생 에

다 벼 사는 공동 업과 경험 많 연 역

에 고 들 연 웃과

게 지내 고 탁 연 들

들 지 연 럽게 들 다 민들

웃과 동 게 뿐만 니 는 집 과

게 다

동 시 는 생태 경 에 살 결과

들 다 사 들 사 상 에 주

울 게 었고 는 곧 체 상 과 간 사

계 시 는 낳게 었다 신 가

가 는 체에 는 원 는 동시

에 다 사 들 그 사 포 체 맥 에

다 들 간 사 연

계 체 계에 주 울 는 사고 체계

게 었다

그러 그리 연 경 그 었다 산

지 연결 는 지 건 그리고 역

에 다 런 들 업에 다 사 과

동 므 공동체에

다고 다 고 그리 들

들과는 달리 보 내 감 지 들과

지 크게 느 지 못 다 그

견 다 경우 주 쟁 통 결 는 갖

게 었다

신 사 간 계들 루어진 커다

트워크 에 게 당연 사 역시 연

계들 체 식 게 다 어 상

원 도 그 개체가 체 맥 과

계 에 고 다 게 체 맥 에 주

울 다 보 상 복 과 가변 식 게 고

상에 재 는 많 변 들 사 에 재 는 들도

게 다 들 주 태도 보

는 경우가 많다 쟁 결

통 결 보다는 통 결

는 보 다

그러 고 그리 들 개개 사 사 독

에 주 울 다 사 사 체에

어 그들 사 에 재 는 공통 규 주

고 다 상 원 에도 사

체 내 주 고 다 그들

체 여 탕 체

는 주 태도 시 고 특 사 어

주에 는지 여 그 주에 는 규

견 다 에 는 쟁 식 리

같 리 사고 체계가 달 게 었다

리 드 니 벳 생각 지도 사- ldquo rdquo( 2004)

zb42) 위 에서 사 된 설 과 가장 유사한 것은

① 크톱 컴퓨 는 본체 니 마우 루

어 다

② 곡과 시 리 는 지 과 사 루어 다는 공통

지니고 다

③ 경 고 것과는 달리

경 본 연 태 그 주변 경

④ 벽돌 능 에 사계 내내

습도가 지 다

⑤ 잰느 체 체 지닌 재 체가 없

는 재 눌 다

년 학 간고사 대비2013 2 현대고 대비

ECN-0102-2013-001-000076193

zb43) 는 립 앙 도서 이 정의 일 이다lt gt

도서 장과 이 자의 리 의 정의 연결이

적절하 않은 것은

lt gt

제 조 서 유8 ( )

도서 장은 다른 이 자의 안전을 위협하거나 도서 의①

서를 란하게 할 가 있는 자에 대하여는 도서 출입

을 제한할 있다

도서 장은 이 자가 제 조 각 호의 어느 하나의 행위를 하7②

을 때에는 이 을 하게 하거나 도서 출입을 제한할

있다

제 조자 의 대출9 ( )

도서 자 는 다음 각 호의 경 대출할 있다①

상호대차도서 간에 자 를 류하는 것을 말한다 등 다1 ( )

른 도서 과의 협 을 위하여 필 한 경

공 이 공 행 상 필 하는 경2

에 도서 장이 필 하다고 인정하는 경3

대출이 가능한 도서 자 의 위는 도서 장이 정하는②

에 따른다

제 조 상10 ( )

이 자가 도서 자 설을 더럽히거나 찢거나 뜨①

쓰게 하거나 잃어 린 경 에는 상하여야 한다

도서 장은 제 항에 따른 상 을 정하여 게 하여야1②

한다

제 조이 절차 등11 ( )

이 칙에서 정한 것 에 도서 자 설의 이 절차

이 제한 등에 필 한 사항은 도서 장이 정한다

출처 립 앙 도서- (httpwwwnlgokr)

① 는 도 리 다8

② 도 는 리 다9 1

③ 료 지 는 도 리 다9 2

④ 도 료 변상에 리10 1

⑤ 는 에 도 리 다11

3

도 다 각 같다①

공 공 다만 연1

연 간 다

매월 째 째 월2

도 도 리 그 사3

가 다고 는

도 에 미리 게1 3②

시 여 다

4

도 시간 도 여 게시 다

5

도 료 시 는 는 도①

지에 등 후

등 에 사 도②

7

는 다 각 여 는 니 다

도 료 시 상 리1 lsquo rsquo

도 료 시 훼 는2 middot

지 가 닌 곳에 식 거 담3

우는

도 보 등 보 검색열4 middot

그 에 도 질 지 여 도5

여 게시 사 는

8

도 다 거 도①

질 게 우 가 는 에 여는 도

도 가 각 어느7②

에는 지 게 거 도

9

도 료는 다 각 경우 다①

상 도 간에 료 는 것 말1 (

다 등 다 도 과 여 경우)

공 원 공 상 는 경우2

그 에 도 다고 는 경우3

가능 도 료 는 도②

는 에 다

10

년 학 간고사 대비2013 2 현대고 대비

ECN-0102-2013-001-000076193

가 도 료 시 럽 거 거①

못 쓰게 거 어 린 경우에는 변상 여

도 에 변상 여 게시1②

여 다

zb44) 위 에서 도서 장이 게 해야 할 사항에 해당하는

것을 두 쓰

년 학 간고사 대비2013 2 현대고 대비

ECN-0102-2013-001-000076193

립 도 규

1 ( )

규 립 도 립 어린 청 도(

포 다 료 시 열 시 말) (

다 에 사 규 립 도)

편 진 다

2 ( )

규 립 도 도 다 에( lsquo rsquo )

고 는 도 에 도lsquo rsquo 2 2

료 에 여 다 다만 특 료 귀

료 등 료 에 사 립 도

도 다 다( lsquo rsquo )

3 ( )

도 다 각 같다①

공 공 다만 연1

연 간 다

매월 째 째 월2

도 도 리 그 사3

가 다고 는

도 에 미리 게1 3②

시 여 다

시간4 ( )

도 시간 도 여 게시 다

등 등5 ( )

도 료 시 는 는 도①

지에 등 후

등 에 사 도②

사 료6 ( )

도 료 시 에 사 료는 도

7 ( )

는 다 각 여 는 니 다

도 료 시 상 리1 lsquo rsquo

도 료 시 훼 는2 middot

지 가 닌 곳에 식 거 담3

우는

도 보 등 보 검색열4 middot

그 에 도 질 지 여 도5

여 게시 사 는

질 지8 ( )

도 다 거 도①

질 게 우 가 는 에 여는 도

도 가 각 어느7②

에는 지 게 거 도

료9 ( )

도 료는 다 각 경우 다①

상 도 간에 료 는 것 말1 (

다 등 다 도 과 여 경우)

공 원 공 상 는 경우2

그 에 도 다고 는 경우3

가능 도 료 는 도②

는 에 다

변상10 ( )

가 도 료 시 럽 거 거①

못 쓰게 거 어 린 경우에는 변상 여

도 에 변상 여 게시1②

여 다

등 규 에 것 에 도11 ( )

료 시 등에 사

도 다

립 도- (httpwwwnlgokr)

zb45) 도서 장의 리 있는 조항으 적절하 않

은 것은

① ② ③ ④ ⑤

년 학 간고사 대비2013 2 현대고 대비

ECN-0102-2013-001-000076193

1 ( )

사가 공 는lsquo rsquo

과 여 사 원과 리

사 타 사 규

니다

개 보 보7 ( )

사는 보통신망 등 계 는 에lsquo rsquo lsquo rsquo

원 개 보 보 니다 개lsquo rsquo

보 보 사 에 는 사 개lsquo rsquo

보 취 니다 다만 사는 다 lsquo rsquo

사 계 통 공 는 경우 원 lsquo rsquo

등 개 보 당 사에 습니lsquo rsquo

원 리에8 (lsquo rsquo lsquo rsquo lsquo rsquo

)

원 에 리lsquo rsquo lsquo rsquo lsquo rsquo①

원에게 가 도 여 는lsquo rsquo 3

니다

사는 원 가 개 보 우 가lsquo rsquo lsquo rsquo lsquo rsquo②

거 사 경우 는 미 에 어 거 lsquo

사 사 운 우 가 는 경우 당rsquo lsquo rsquo

습니다lsquo rsquo

원 가 도 거lsquo rsquo lsquo rsquo lsquo rsquo 3③

가 사 고 지 경우에는 시 사에lsquo rsquo

통지 고 사 내에 니다lsquo rsquo

경우에 당 원 사에 그 사실3 lsquo rsquo lsquo rsquo④

통지 지 거 통지 도 사 내에 지 lsquo rsquo

생 경우 사는 지지 습니다lsquo rsquo

사10 (lsquo rsquo )

사는 과 지 미lsquo rsquo①

에 는 지 계 고

공 여 다 여 니다lsquo rsquo

사는 원 게lsquo rsquo lsquo rsquo lsquo rsquo②

도 개 보 신 보 포 보 보 시( )

갖 어 개 보 취 공시 고

니다

사는 과 여 원lsquo rsquo lsquo rsquo③

견 만 당 다고 경우에는

리 여 니다 원 견 만 사 lsquo rsquo

에 는 게시 거 우편 등 통 여

원에게 리 과 결과 달 니다lsquo rsquo

원11 (lsquo rsquo )

원 다 여 는 니다lsquo rsquo ①

신청 는 변경 시 허 내 등1

타 보 도2

사가 게시 보 변경3 lsquo rsquo

사가 보 보 컴퓨 그4 lsquo rsquo (

등 등 신 는 게시)

사 타 등 지 재산 에5 lsquo rsquo 3

사 타 상 거 업6 lsquo rsquo 3

는 폭 시지 상 타 공7 middot middot

에 는 보 에 공개 는 게시 는lsquo rsquo

사 동 없 리 사8 lsquo rsquo

타 거 당9

게시15 (lsquo rsquo )

원 내에 게시 는 게시 게재 는lsquo rsquo lsquo rsquo lsquo rsquo

경우 원 사가 게시 복 lsquo rsquo lsquo rsquo lsquo rsquo middot middot

등 태 언 등에 공 는

것 내에 다 원 본 게시 등 lsquo rsquo lsquo rsquo

크 능 등 여 복 는 등 태

는 것 동 것 니다

- (wwwnavercom)

zb46) 위 은 인터넷 포털사이트의 회 가입을 위한 이

약 의 일 이다 이 약 을 만드는 과정에서 생각한

내 으 적절하 않은 것은

개 보 보 가 지에 별 눠①

겠어

원 가 만들게 에②

시 주어 겠어

원들 게재 게시 다 원 크 다③

는 것 지

④ 원 지 는 뿐만 니 사가 지 는

도 께 달 지

리에 가 생 경우 사가⑤

에 다는 도 듯

1 ( )

사가 공 는lsquo rsquo

과 여 사 원과 리

사 타 사 규

년 학 간고사 대비2013 2 현대고 대비

ECN-0102-2013-001-000076193

니다

개 보 보7 ( )

사는 보통신망 등 계 는 에lsquo rsquo lsquo rsquo

원 개 보 보 니다 개lsquo rsquo

보 보 사 에 는 사 개lsquo rsquo

보 취 니다 다만 사는 다 lsquo rsquo

사 계 통 공 는 경우 원 lsquo rsquo

등 개 보 당 사에 습니lsquo rsquo

원 리에8 (lsquo rsquo lsquo rsquo lsquo rsquo

)

원 에 리lsquo rsquo lsquo rsquo lsquo rsquo①

원에게 가 도 여 는lsquo rsquo 3

니다

사는 원 가 개 보 우 가lsquo rsquo lsquo rsquo lsquo rsquo②

거 사 경우 는 미 에 어 거 lsquo

사 사 운 우 가 는 경우 당rsquo lsquo rsquo

습니다lsquo rsquo

원 가 도 거lsquo rsquo lsquo rsquo lsquo rsquo 3③

가 사 고 지 경우에는 시 사에lsquo rsquo

통지 고 사 내에 니다lsquo rsquo

경우에 당 원 사에 그 사실3 lsquo rsquo lsquo rsquo④

통지 지 거 통지 도 사 내에 지 lsquo rsquo

생 경우 사는 지지 습니다lsquo rsquo

원에 통지9 (lsquo rsquo )

사는 특 다 원에게 통지 경우lsquo rsquo lsquo rsquo

공지 게시 통 상 게시 개별 통지에7

갈 습니다

사10 (lsquo rsquo )

사는 과 지 미lsquo rsquo①

에 는 지 계 고

공 여 다 여 니다lsquo rsquo

사는 원 게lsquo rsquo lsquo rsquo lsquo rsquo②

도 개 보 신 보 포 보 보 시( )

갖 어 개 보 취 공시 고

니다

사는 과 여 원lsquo rsquo lsquo rsquo③

견 만 당 다고 경우에는

리 여 니다 원 견 만 사 lsquo rsquo

에 는 게시 거 우편 등 통 여

원에게 리 과 결과 달 니다lsquo rsquo

원11 (lsquo rsquo )

원 다 여 는 니다lsquo rsquo ①

신청 는 변경 시 허 내 등1

타 보 도2

사가 게시 보 변경3 lsquo rsquo

사가 보 보 컴퓨 그4 lsquo rsquo (

등 등 신 는 게시)

사 타 등 지 재산 에5 lsquo rsquo 3

사 타 상 거 업6 lsquo rsquo 3

는 폭 시지 상 타 공7 middot middot

에 는 보 에 공개 는 게시 는lsquo rsquo

사 동 없 리 사8 lsquo rsquo

타 거 당9

원 계 규 내lsquo rsquo lsquo②

여 공지 주 사 사가 통지 는rsquo lsquo rsquo

사 등 여 타 사 업 에 lsquo rsquo

는 여 는 니다

- (wwwnavercom)

zb47) 위 약 의 조항에서 같은 제점을 하lt gt

고 있는 조항은

lt gt

제휴 회사에 회 의 아이디 개인 정 를 전송할 있도

한 조항은 고객에게 당한 조항이다

1 7 8① ② ③

④ 9 ⑤ 10

립 도 규

1 ( )

규 립 도 립 어린 청 도(

포 다 료 시 열 시 말) (

다 에 사 규 립 도)

편 진 다

2 ( )

규 립 도 도 다 에( lsquo rsquo )

고 는 도 에 도lsquo rsquo 2 2

료 에 여 다 다만 특 료 귀

료 등 료 에 사 립 도

도 다 다( lsquo rsquo )

3 ( )

도 다 각 같다①

공 공 다만 연1

연 간 다

년 학 간고사 대비2013 2 현대고 대비

ECN-0102-2013-001-000076193

매월 째 째 월2

도 도 리 그 사3

가 다고 는

도 에 미리 게1 3②

시 여 다

시간4 ( )

도 시간 도 여 게시 다

등 등5 ( )

도 료 시 는 는 도①

지에 등 후

등 에 사 도②

사 료6 ( )

도 료 시 에 사 료는 도

7 ( )

는 다 각 여 는 니 다

도 료 시 상 리1 lsquo rsquo

도 료 시 훼 는2 middot

지 가 닌 곳에 식 거 담3

우는

도 보 등 보 검색열4 middot

그 에 도 질 지 여 도5

여 게시 사 는

질 지8 ( )

도 다 거 도①

질 게 우 가 는 에 여는 도

도 가 각 어느7②

에는 지 게 거 도

료9 ( )

도 료는 다 각 경우 다①

상 도 간에 료 는 것 말1 (

다 등 다 도 과 여 경우)

공 원 공 상 는 경우2

그 에 도 다고 는 경우3

가능 도 료 는 도②

는 에 다

변상10 ( )

가 도 료 시 럽 거 거①

못 쓰게 거 어 린 경우에는 변상 여

도 에 변상 여 게시1②

여 다

등 규 에 것 에 도11 ( )

료 시 등에 사

도 다

립 도- (httpwwwnlgokr)

zb48) 다음 정 리 의 의 으 볼 때 가장

이 적인 것은

도 시간 도 여 게시 다①

등 에 사 도②

가능 도 료 는 도 는③

에 다

④ 도 에 변상 여 게10 1

시 여 다

⑤ 도 가 각 어느7

에는 지 거 도

zb49) 를 참고하여 이 어의 성격을 설 한lt gt

것으 적절하 않은 것은

① 보 에 는 어 시 상 고 어 시lt gt lsquo rsquo

에 보여주고 다

② 진 어 어원에 견 고 다

에는 타 어 들어가는 것 다 lsquo rsquo

③ 에 들어갈 말 각각 고 어 어 신 어~

들 언어는 질 격 강 통 없었다

④ 시 우리 에 가 었지만 지 계

과 달리 들 통 사 달 어 웠

년 학 간고사 대비2013 2 현대고 대비

ECN-0102-2013-001-000076193

⑤ 크 몽골 만주 공통어가 우리 어 같

계열에 다는 에 사 특 짐

가( )

善化公主主隱 공주님

他密只嫁良置古 몰 결 고

薯童房乙 맛

夜矣卯乙抱遣去如 에 몰 고 가다

( )

始汝 會隱日恚見隱扐 만 에 본

恥隱汝衣淸隱笑 맑 웃

고 시 여 공 크다 만 다[ ] ( ) ( ) ( ) ( )始 汝 會扐

내다 에 보다 견( ) ( )恚 見 다( )隱

럽다 맑다 청 웃( ) ( ) ( ) ( )恥 衣 淸 笑

zb50) 위의 나 를 함 고 음에 답하( ) lt gt

보lt gt

( )素那或云金川 白城郡蛇山人也

운 사산

는 고 다 는( )[ ( ) ] (素那 金川 白城

사산 사 다) ( ) 郡 蛇山

삼 사- lsquo rsquo 47

에 제 된 단어 의 표 리를 조건(1) lt gt ( ) lt gt

에 맞게 서 하

건lt gt

lsquo 었고 었다 태rsquo

에 제 된 단어 동일한 표 리에(2) lt gt ( )

의해 적은 것을 나 에서 찾아 조건 에 맞게 서 하( ) lt gt

건lt gt

에 당 는 각각( ) 개 쓸 것2 단

당 는 가 여러 개 어도 개만 쓸 것 각2

개 과 도 쪽에 개만2 2

드시 지 것( )

과 동 원리 것lsquo 고

과 동 원리 것 다rsquo

태 것

가( )

素那(或云金川) 白城郡蛇山人也

소나 또는 천 이라 한다 는 성 사( ) ( ) ( )素那 金川 白城郡〔 〕

산 사람이다 현대어 풀이( ) ( )蛇山

나( )

紫布岩乎希 회

執音乎手母牛放敎遣 자 손 암쇼 노히 고

吾 不喩慙 伊賜等肹 肹 나 안디 리샤

花 折叱肹 可獻乎理音如 고 것거 도림다

다 향찰은 리말을 리 으 적은 표 이었 만 생( )

은 고 대를 넘 하고 끊어 고 말았다 랜 세

동안 갈고 닦아 체계적이었던 향찰 표 이 사라졌

을 인은 크게 두 가 나누어 생각해 볼 있다

하나는 족 사회의 한 선호도에서 찾을 있다 라 때

향찰은 주 족 계 에서 사 했을 것으 인다 한 을

알 하고서는 한자를 활 하여 리말을 리 으 표

하 란 가능하 때 이다 런데 족들은 간이 흐

를 향찰과 같은 리 표 을 익혀 사 하 다는

아 한 을 대 사 하는 쪽을 선호하게 되었다 더 이

고 초에 인재 등 을 위해 과거제도가 행되 서 한 선

호도가 더 높아졌고 결 향찰은 소 되고 말았다

또 다른 가능성은 한 어의 특성에서 찾을 있다

터 한 과 일 세 나라는 한자 화 에 속해 다

당연한 이야 겠 만 표의 자인 한자는 어를 표 하

에 매 적절하다 어의 음절은 성 ( ) ( )聲母 韻母

이 어 고 여 에 성조가 추가되어 최종 소리가 결정된

다 래서 어는 단음절을 하나의 한자 표 하 된

다 에 초성 성 종성의 세 가 소가 하나의 음절

년 학 간고사 대비2013 2 현대고 대비

ECN-0102-2013-001-000076193

을 이 는 한 어는 음절 조가 잡하고 음절의 가 많아

서 한자 차 만으 한 어의 소리를 만족 럽게 표 할

없었다 를 들어 한 어에서는 어 니 같이 음절 lsquo rsquo

이 어 단어가 얼마든 있으나 어는( ) 複數音節

자 하나 나타내 만이다lsquo [m ]rsquo 母 ǔ

한편 일 어의 표 은 핵 적 단어는 한자 적고 토는

가나라는 일 의 자 적는 이다 적인 의 를 나

타내는 은 표의 자인 한자 적고 적 계를 나

타내는 토는 표음 자 적는 셈이니 자세히 살펴

리의 향찰 표 을 쏙 빼닮았음을 알 있다 한 어 같

은 착어이 서도 일 어에만 향찰과 유사한 표 이 살아

남은 것은 일 어의 특 때 이다 일 어는 하나의 자음과

음의 결합으 음절을 이 고 침이 거의 없는 음절 언어

이다 이러한 음절의 특색에다가 토가 달한 착어라는 점

이 향찰과 유사한 표 이 살아남을 있는 비결이었다

하 만 같은 착어라도 다양한 음소 침이 달한 한

어는 향찰 표 하는 데 근 적으 한계가 있었다

zb51) 다 하여 의 행에 대한 탐 한 결과( ) lt gt 2

않은 것은

보lt gt

善花公主主隱 공주니믄 공주님( )

----------------------------------------

-

他密只嫁良置古 그 지 얼어 고 몰 결(

----------------------------------------

-

薯童房乙 맛 맛( )

夜矣卯乙抱遺去如 몰 고 가다 에 몰 고(

가다)

주동 역 동- (薯童謠『 』

에 2 ( )他密只嫁良置古

얼다 시집가다 결 다 말 lsquo rsquo

① 실질 미 지니고 므 타 타lsquo ( )rsquo lsquo [ ]

② 에 실질 미 타내고 지 는lsquo rsquo lsquo [ ]rsquo lsquo [ ]密只 密 只

계 타내는

③ 얼어는 실질 미 포 고 므 가lsquo rsquo lsquo [ ]rsquo嫁

것lsquo [ ]rsquo 良

④ 고 어간 는 실질 미 지니고 므lsquo rsquo lsquo -rsquo

것lsquo [ ]rsquo 置

⑤ 고 어미 고는 계 타내고 므lsquo rsquo lsquo- rsquo

고 것lsquo [ ]rsquo 古

가( )

엉 훈 민middot middot middot middot middot世 宗 御 製 訓 民 正 音

말 미 듕 귁에 달middot middot middot middot middot middot middot middot中 國 文 字

니 런middot middot middot middot middot middot 어린middot middot middot middot百 姓

니 고 도 내 들middot middot middot middot middot middot middot middot middot 시러middot

펴 몯middot 미middot middot 니 내middot middot middot middot middot middot middot middot 爲

어엿middot 겨 새middot middot middot 믈여듧middot middot middot middot字 니middot middot middot

사 마다 니겨 킈 middot middot middot middot middot middot middot middot middot便 安

고 미니middot middot middot middot

본 는 상( ) (象

원리에 만들어진 본) ( )形 ㄱ ㄴ ㅁ ㅅ ㅇ

에 는 가 원리에( )加劃

그리고( )ㅋ ㄷ ㅌ ㅂ ㅍ ㅈ ㅊ ㆆ ㅎ

쓰는 병 원리에 만들어진( )竝書

마지막 체( ) ( )異體ㄲ ㄸ ㅃ ㅆ ㅉ ㆅ

ᅀ 다 상 원리에 ㅇ ㄹ

지 는 삼재 상 본 본( ) ( ) ( 天地人 三才

탕 므림과 림에 ) (初ㅡ ㅣ

재)( ) ( )( )出字 再出字ㅗ ㅏ ㅜ ㅓ ㅛ ㅑ ㅜ ㅕ

병 그리고 들 에 다시( )ㅘ ㅝ ㅣ

( )ㅣ ㅢ ㅚ ㅐ ㅟ ㅔ ㆉ ㅒ ㆌ ㅖ ㅙ ㅞ

zb52) 가 에 대한 설 으 르 않은 것을( ) 두 고르

① 어쓰 규 지키고 다

② 리 고 다

③ 말 미 미 등 어 사 다lsquo rsquo

④ 개 지 다

년 학 간고사 대비2013 2 현대고 대비

ECN-0102-2013-001-000076193

⑤ 어 원 에 가 도 고 다

엉 훈 민世 宗 御 製 訓 民 正 音

말 미 듕귁에 달 니

런 어린 니 고 도middot

내 들 시러 펴 몯 미 니middot

내 어엿 겨 새 믈여듧

사 마다 니겨middot 킈 고

미니

훈민 언 본- lsquo rsquo 5 (1459 )

zb53) 위의 에 대한 현대어 풀이가 르~ 않은 것

① 우리 말 과 달

② 어리 말 고 는 것 어도

③ 신 생각 마 껏 펼 는 사 많다

④ 게 생각 여

⑤ 사 마다 게

zb54) 훈민정음 언해 에는 한 을 창제한 동 가 드러나

있다 훈민정음 창제의 정 과 내 이 잘 연결된 것

① 주 신 말 미 듕귁에 달

② 민 신 내 어 겨

③ 신 뻔 킈 고 미니

④ 실 신 사 마다 니겨

⑤ 귀 신 계 주 는 훈민 신과 거리가

가 엉 훈 민( ) middot middot middot middot middot世 宗 御 製 訓 民 正 音 

말 미 귁에 中 國 달 文 字

니 런 어린 니 百 姓

고 도 내 들 시러 펴 몯

미 니 내 어엿 爲 겨 새

믈여듧 니 사 마다 니 字

겨 킈 고 미니 便 安

훈민 언 본- lsquo ( )rsquo ( ) 5 (1459 )訓民正音 世祖

( )

[ 1 ]

동 룡 샤 마다 복( ) ( ) ( )海東 六龍 天福

시니 고 동( ) ( )古聖 同符 시니

[ 2 ]

매 니 곶 여

미 므 니 그 내 러

가 니

[ 125 ]

우 미리( )千世 샨( )定 에( )漢水北 累仁

누 개 샤 복 업 시니( ) ( ) 開國 卜年

신( )聖神 니 샤도 경 근민 샤 욱( )敬天勤民

드시리 다

님 쇼 산 가( ) ( )洛水 山行

미드니 가

어 가- lsquo ( )rsquo 27龍飛御天歌

다 우리신 니쓰고 다만 만 쓰( )

거 샹 귀쳔 다보게 러 귀

여 쓴 도 신 보 가 고 신 에

말 어 보게 각 에 사 들

고 본 몬 능통 후에

죠 죠 니

드 도 만 공 에 사

드 미 죠 고 고 여 보 죠

보다 얼마가 거시 어신고 니 첫

가 죠 니 죠

민 들 어 신 샹

귀쳔 도보고 어보 가 만 늘

고 폐 에 만쓴 죠 민

도 러보지못 고 보니 그게 엇지

심 니 리 보 가 어 운건 다

니 쳣 말마 지 니 고 그

쓰 에 가 우 지 지

몰 거 본후에 가 어 지

고 그니 쓴편지 쟝 보

년 학 간고사 대비2013 2 현대고 대비

ECN-0102-2013-001-000076193

쓴것보다 듸 보고 그 마 니 쓴 고

어 못

그런고 에 리 과 가

만 쓴 못 민 말만 듯고

고 편 그 못 보니 그사 단

병신 못 다고 그사 식 사

니 만 고 다 과 그사

만 고 다 과 업 사 보다 식 고

죠 도 고 각 과

견 고 실 직 귀쳔 간에 그

고도 다 것 몰 귀죡 보다

사 우리 신 귀쳔 다 업

시 신 보고 과 지 게 랴

시니 샹 귀쳔 간에 우리 신 걸

간 보 새지각과 새 걸 미리

독립신- lsquo (1896)rsquo

zb55) 친 어 나의 제 장( ) 2 매 함축적

의 가 가장 유사한 것은

① 지 눈 내리고 매 득 니 내 여 가

사- lsquo rsquo

② 도 어 리듯 그 게 어 다

주 사- lsquo rsquo

③ 눈 살 다 죽 어 린 과 체 여

눈 새벽 지 도 살 다

눈- lsquo rsquo

④ 삶 근심과 고단 에 돌 거니는 여 거 는

여 리 내린 살가지 에 눈 리 눈 리

택 그 생 에- lsquo rsquo

⑤ 늘 러 고 러

청룡 룡 어 개 루 우

신경림 계- lsquo rsquo

zb56) 친 를 위 가 나 에 나타난A B ( ) ( )

세 어의 특 에 의거하여 세 어 표 하

그 산 고 공 도 맑지만

A

주변에 쓰 리는 어리 사 많다

B

건lt gt

식 가 에 타 어 특징에( ) ( )

거 과 어쓰 는 고 지 말 것

A

B

zb57) 가 의( ) 달 아ㆍ 다 의 ( ) 나셔에서 알 있는

세 어 개화 어의 특 을 비 하여 조건 에lt gt

맞게 서 하

건lt gt

어에 는lsquo 개

어에 는 다 태rsquo

zb58) 은 가 는 다 에 나 는 절lt 1gt ( ) lt 2gt ( )

일 를 췌한 것이다 의 의 가 lt 1gt (1)~(2)

유사한 말을 에서 찾아 쓰lt 2gt

보lt 1gt

런 (1) 어린 니 고百 姓

도 내 들 시러 펴 몯 미

사 마다 (2) 니겨 便 安

킈 고 미니

보lt 2gt

죠 고 고 여 보 죠

보다 얼마가 거시 어신고 니 첫 가

죠 니 죠 민

들 어 신 샹 귀쳔

도보고 어보 가 만 늘 고

폐 에 만쓴 죠 민 도

러보지못 고 보니 그게 엇지 심

니 리

년 학 간고사 대비2013 2 현대고 대비

ECN-0102-2013-001-000076193

lt 1 gt

동 룡 샤 마다 복 시( ) ( ) ( )海東 六龍 天福

고 동 시니( ) ( )古聖 同符

lt 2 gt

(A) 매 니 곶

여 니

미 므 니 그 내

러 가 니

lt125 gt

우 미리 샨 에( ) ( ) ( ) 千世 定 漢水北 累

누 개 샤 복 업 시 니( ) ( ) 仁開國 卜年 聖

신( ) 神 니 샤도 경 근민 샤( ) 敬天勤民

욱 드 시 리 다

님 쇼 산 가 ( ) ( )洛水 山行

미드니 가

- lt gt龍飛御天歌

zb59) 장과 내 상 유사한 성격의 조는125

① 뫼 고 고 고 고

어 그린 많고 많고 고 고

어 러 는 울고 울고 가느니

도 견- lt gt

② 강 에 드니 몸 다

그믈 고 가니

뒷 뫼 엄 언 니( )藥

-

③ 말 없는 청산 태 없는 다

값 없는 청 없는 월

에 병 없는 몸 별 없 늙 리

-

④ 가마귀 골에 가지 마

낸 가마귀 새

청강에 것 시 몸 러 가( ) 淸江

-

⑤ 진 골에( ) 白雪

가 매 는 어느 곳에 었는고

에 갈 곳 몰( ) 夕陽

색-

zb60) 위 에 나타난 세 어의 특 으 적절하 않은

것은

① 룡 어 주격 사에 당 는 가 사( ) lsquo rsquo六龍

고 다

② 샤 어에도 어 주체 쓰 다

는 것 다

③ 매 어 달리 사 택에 어

가 지 지지 고 다

④ 므 원 상 직 어 지 다

⑤ 드시리 다 주체 과 상 께 사

고 다

수고 하셨습니다hearts hearts

년 학 간고사 대비2013 2 현대고 대비

ECN-0102-2013-001-000076193

보닷컴에 공 는 별 보는 고등

들 여 주 는

들 습니다 슷 동 지

가 복 는 것 도가

니 복 여 습 시고 거 시

니다

정답 해설

1) 정답[ ] ④

해설 다른 것은 두 특정 업이나 단 내에서 사[ ]

하는 일종의 은어 사회 언에 해당한다 러나

는 언이 아니라 단과대학을 여서 단대 사lsquo rsquo lsquo rsquo lsquo④

대학을 여서 사대라고 한 말에 해당하 일rsquo lsquo rsquo

사회에서도 널리 쓰이 사회 언이라 할

없다

2) 정답[ ] ⑤

해설 사회 언은 같은 단 내에서 쓰이는 언어이[ ] lsquo rsquo

동일 단끼리는 단결 과 친 감을 형성하는

능을 하 리적 안감이 일어나 않는다

3) 정답[ ] ③

해설 사람이라는 차 적 표현에 대한 대안적 표현이[ ]lsquo rsquo

인 아내 처 등으 볼 있다lsquo rsquo

4) 정답[ ]⑤

해설 남성은 주 격 체를 사 한다[ ]

5) 정답[ ] ⑤

해설 흑인은 검다라는 뜻을 가 고 있을 뿐 인[ ]lsquo rsquo lsquo rsquo lsquo rsquo

다 열등한 뜻을 내포하 않는다

6) 정답 살 색 첫 작품[ ] - -

해설 살색 혹은 킨색은 한 인의 피 색을 뜻[ ] lsquo rsquo lsquo rsquo

하는 것으 인종 차 을 추 고 출 이주민

의 평등 을 침해할 있어 년 표 이2005

살 색으 이름을 꾸었다 처녀작은 처녀라lsquo rsquo lsquo rsquo lsquo rsquo

는 단어가 가 고 있는 곡된 성 인 을 한 것

으 첫 작품정도 꾸어 사 하는 것이 좋다lsquo rsquo

7) 정답[ ] ⑤

해설 호는 아들에게 해체를 사 하고 있다[ ] ① ②

장 을 성하는 청자는 자 의 아 느리 아lsquo

들 세 이다 호는 아 느리에게 해rsquo ③

체를 사 하고 있다 호가 느리 아 에게 ④

사 한 해 체 아들에게 사 한 해체는 두 비lsquo rsquo lsquo rsquo

격 체에 해당한다 호는 자 의 아랫사람인 ⑤

느리에게 아들과 마찬가 해체를 사 하는 것이

상 이 만 임 을 한 느리에게 고마 과 쁨

존 의 표 를 하 위해 자 의 아 에게 말하듯

해 체를 사 하고 있다

8) 정답[ ] ③

9) 정답[ ] ⑤

10) 정답[ ] ①

해설 청자 할아 가 장의 주체 아 다 높을[ ] ( ) ( )

경 에는 압존 에 의해 장의 주체를 높이 않는lsquo rsquo

다 러 아 서가 아닌 아 는으 계 lsquo rsquo lsquo rsquo lsquo

니다 가 아닌 있 니다 표현하는 것이 르rsquo lsquo rsquo

11) 정답 당이 당을 쫒았다 당이[ ]

당에 다

해설[ ]

12) 정답[ ] ⑤

해설 서 다른 높임표현을 통해 청자에 대해 리[ ] ⑤

적 거리감을 나타내는 인 은 이 아니라 현정이

다 가 에서 현정은 에게 해 체를 사 함으 써 ( )

친근감을 드러낸다 나 에서 연 을 게을리하는 역 ( )

도 들 때 에 화가 난 현정이 선생님에게 항의하

는 장 에서는 하 체를 사 하여 리적 거리lsquo rsquo

가 어졌음을 나타내고 있다

13) 정답[ ] ①

해설 는 는 얼 빛이 날과 어찌 다르 고[ ] lsquo rsquo

라는 뜻으 전과 달리 임이 화자를 않고

있음을 알 있다

14) 정답 달리 후 가 있다 이를 통해 경[ ] lt gt

쾌한 음악성을 형성하고 노 젓는 상황을 체적으

형상화하는 역할을 한다

15) 정답[ ] ①

16) 정답[ ] ⑤

해설 다 의 자연은 를 성찰하게 하는 대상[ ] ( )⑤

이자 정의 대상이다 의 자연은 자 의 상황과 ⑤

처 를 드러내는 경으 서의 역할을 하 이

이 없다

17) 정답[ ] ③

해설 는 빈천 을 해결하고자 했으나 강산[ ] lsquo ( )rsquo 貧賤③

과 풍 을 달라는 에 거절하 다고 함으 써 자

연에 대한 애정을 드러내고 있으 는 않는

임에 대한 망을 개에게 전가 켜서 임에 대한 리

을 드러내고 있다

18) 정답[ ] ③

년 학 간고사 대비2013 2 현대고 대비

ECN-0102-2013-001-000076193

19) 정답[ ] ⑤

해설 고상한 음악가의 이름을 리말 꽝 럽[ ]

게 꿈으 써 언어유희를 통해 음을 유 하고 있

다 이는 고상한 척하는 총 를 비꼼으 써 비판적

태도를 드러내는 것이 대상을 꽝 럽게 표현

하여 총 의 허 과 사치를 풍자하고 있다

20) 정답[ ] ⑤

해설 는 작품 속 경에 대한 설 이 드러나는 것이[ ]

서 자의 주 적인 견해가 접적으 드러나는 것이

아니다

21) 정답[ ] ⑤

22) 정답[ ] ②

23) 정답[ ] ④

24) 정답[ ] ①

해설 적강 티프는 주인공의 비 한 출생이나 능[ ] ①

과 이 있는 것으 조정의 능함을 풍자하는lsquo rsquo

것과는 거리가 다

25) 정답 픔 나[ ] ( )

해설 의 음악은 고통 는 사람들을 위 하고 아픔[ ] lsquo rsquo

을 치유해 주는 능을 한다고 할 있다 의 lt gt

픔 도 소 된 이 과 더 어 살아가는 따뜻한 마음lsquo rsquo

을 상 한다

26) 정답[ ] ⑤

해설 에게 선천적으 주어 각 장애라는 역경[ ]

은 의 이라는 가사 연 을 있다lsquo rsquo

27) 정답[ ] ④

해설 는 장 란 선 에게 은 개인적인 인상을[ ]

소녀 장정 등으 표현한 것이다lsquo rsquo

28) 정답[ ] ②

해설 담자가 피 담자의 언어적 표현이나 비언어[ ]②

적 표현 하 독자는 담의 위 나 피

담자의 감정 상태를 알 있다 이를 통해 독자는

담 상황을 더 생생하게 느낄 있고 피 담자

를 더 잘 이해할 있게 된다

29) 정답[ ]③

해설 일상생활과 역도 선 서의 성과에 된 것에서[ ]

역도를 하 서 겪는 어 과 내적 고민으 화제를

전화하 위한 것이다

30) 정답[ ] ①

해설 릿속에 새겨 넣듯 이 억되도 함 세상[ ] ② ③

살이가 힘들고 고생 러 속 하여 자유를 ④

가 없는 고통의 상태를 비유적으 이르는 말

적의 침입을 막 위해 쌓은 축 켜야 할⑤

대상을 비유적으 이르는 말이다

31) 정답[ ] ④

해설 이 의 종류는 전 으 인 사건 경[ ] lsquo

비평을 성 소 삼는다rsquo

32) 정답[ ] ④

해설 근은 삼대독자 태어났음을 에서 확인할[ ]

있다 형제들과의 담은 이뤄 가 없다

33) 정답[ ] ⑤

해설 근은 가난에도 하고 화가를 꿈꾸었다[ ] (3

단 또한 다른 화가 망생들은 정 육을)

위해 상 학 학 해 유학 에 랐 만

근은 다른 을 찾아야 했다 단 세에(5 ) 18

근은 조선 전람회에 입선하 다 단 의(6 )

만종은 인간과 자연이 엮어 가는 경건한 조화 을lsquo rsquo

나타낸다

34) 정답[ ] ①

해설 근이 속에서도 창작활동을 추 않고[ ]

하는 닭은 은 세상과 타협할 르는

근이 세상의 이해를 하 위한 가장 떳떳한 단

이 때 이다

35) 정답[ ] ⑤

해설 전 은 서 자의 주 적인 평이 리는 것이[ ]

만 위 제 은 인 이 살았던 대 사회적 경

을 통해 객 적인 인 의 을 제 하고 있다

36) 정답[ ] ⑤

해설 전 은 인 사건 경 비평이라는[ ] lsquo rsquo⑤

성 이 어져 있다

37) 정답[ ] ①

해설 이 은 동양인과 서양인의 사고 에 차이가[ ]

있다는 것을 대조를 통해 설 하고 있다 또 쓴이

의 제자가 축 경 를 러 가서 경험한 일화를

통해 동양인이 서양인에 비해 주 상황에 더 많은

주의를 인다는 주장을 뒷 침하고 있다

38) 정답[ ] ④

39) 정답[ ] ②

40) 정답[ ] ②

41) 정답[ ] ④

42) 정답[ ] ③

43) 정답[ ] ④

44) 정답 도서 의 휴 일 도서 의 이 간 도서의[ ]

해설 도서 장은 임의 정한 휴 일과 도서 이[ ]

간 도서의 상 등을 게 할 의 가 있다

년 학 간고사 대비2013 2 현대고 대비

ECN-0102-2013-001-000076193

45) 정답[ ] ①

해설 제 조의 정 휴 일 의 휴 일의 사전 게[ ] 3

는 도서 장의 의 조항에 속한다

46) 정답[ ] ①

해설 개인 정 호 의 를 제 하 했 만 항[ ]

나눠서 제 하 않고 대 나열하고 있다

47) 정답[ ] ②

해설 제 조의 내 을 회사는 다른 회사 협[ ] 7 lsquo

계약을 통해 서비 를 제공하는 경 회 의 아이디

등 개인 정 를 해당 회사에 전송할 있다는 내rsquo

이 있으 의 제점을 제 할 있다②

48) 정답[ ] ④

해설 는 도서 장의 의 에 해당하고 나 는 도[ ] ④

서 장의 리에 해당한다

49) 정답[ ] ③

50) 정답 은 음독으 적었고 은 훈독으 적었[ ] (1)

다 과 동일한 표 리 적은 것은 이고 (2) ce

과 동일한 표 리 적은 것은 이다ab

51) 정답[ ] ③

52) 정답[ ] ①②

53) 정답[ ] ③

54) 정답[ ] ③

55) 정답[ ] ①

56) 정답 른 죠코 어린 노 하니라[ ] A B

57) 정답 세 어에서는 활 형이 칙적으[ ] lsquo rsquoㄹㅇ

나타났 만 개화 어에서는 활 형이 쓰 다 lsquo rsquo ㄹㄴ

58) 정답 호 가 흔[ ] (1) (2)

59) 정답[ ] ④

60) 정답[ ] ③

Page 21: 현대고대비 국어 - chamsoriedu.com 「콘텐츠산업진흥 법」외 에도 저작권 의하여 ... 다른주체에게어떤동작을하도록만드는것을나타내는

년 학 간고사 대비2013 2 현대고 대비

ECN-0102-2013-001-000076193

zb32) 위 을 작성하는 과정에서 되어 활 된 자

어 것은

신 사 료① 연보②

고③ ④ 들과 담

⑤ 에 평

는 간 과 진실 그 다는 에ldquo

단 평 견 가지고 다 내가 그

리는 간상 단 고 다 지 다 는 그들 가

에 는 평 지 니 그리고 어린 들

미지 겨 그린다rdquo

근 간 과 진실 그리고 싶어 가

다 근에게 그것 진리 다 거 다 없 거

고 다 없 는 것 진리다

근 진리는 후 쪽 었다 신산 삶 ( )辛酸

었 질곡 역사 에 지냈 가( )桎梏

눈에 든 것 료 단 료 게 보

것 었다 그것 그 에 겨우겨우

슬 슬 생 어가는 간들 었다

리 과 단 리 고리에 검 마

없 거리 돌 상

것 없는 등 근에게 상에 과

진실 엄 다는 사실 리는 가 실( )儼存

고 가 과 역경 에 도 근 내 포

없었 후 보루 다 도 도( ) 365堡壘

간 근 여 시

것 다

간에 지닌 가 근 1914 2

월 강원도 림리에 삼 독21

태어났다 어 근 복 그것 그리

가지 못 다 근 곱 살 지는 산

사업에 실 고 답마 에 내 갔다 근

그림 럼 쫓 다니 가 시 것 다 상

진 것도 가 었다 러 가 에도

고 근 가 꿈꾸었다 근 가 꿈꾸게

것 보통 업 원색1926

도 만 었다lsquo rsquo

질 루 마 가 도 린다 경건

움 느껴지는 경 다 훗 근 그림에

과 는 거 것( )裸木

만 간과 연 엮어 가는 경건 움lsquo rsquo

니었

같 가가 고 싶었 근에게 그 꿈에 다

가가는 지 다 다 가 지망생들 규 미

상 에 진 고

에 지만 근 다 다 근

미 에 운 것 보통 시 미 시간

다 그런 그에게 없는 연습 가가

통 다 가 귀 시 지 도

얻는 뛸 듯 뻤지만 마 도 가 에

듯 는 었 에 어린 근 주 에

에 그림 그리고 지우고 복( )粉板

시간 가는 게 루 보냈다

근 그 갈 가가 것 열여( )渴求

었 다가 미1932 lsquo rsquo ( lsquo

미 에 다 다는 고 마rsquo) lsquo rsquo

가 근 집 고도 지는 시골 경

그린 그림 다 후 근 에 1943 22

지 미 에 그림 고

에 걸쳐 다 미 근 가

동 는 었다

공주 그림 가 근 경- ldquo rdquo ( 2009)

zb33) 위 의 내 과 일치하는 것은

가 근 가 꿈 포 다①

근 당 가들과 께 에 다②

살 근 가 걷20③

게 었다

④ 만 통 근 역경 겨내는lsquo rsquo

느 다

⑤ 근 간 과 진실 그리 에 그 에

드러 는 간상 단 다

계 시 주 근 건강

걸었다 신 과 간에 상 다 건강

신 는 눈에도 다 근 쪽 눈 뿌 게

보 지 과에 다 다 시 지지 고 결

내 었다 시 지만 마 막막

다 늦어 결 근 쪽 눈 고 말 다

쪽 눈 근에게는 쪽 눈 었고

계 었다 그 근 는 여 그lsquo rsquo

다 근 에 같 그림 그 었다1950

시 그림 는 여 쪽lsquo rsquo

고 어 마주 고 는 그림1963

여 과 동 다 마 복

그린 듯 눈 내리 새 게 다 지

사 다 근 게 복 것

복 상과 타 는 근 상

가 떳떳 단 었고 근 그리고

간 과 진실 에 다가가 가 근다

운 었다 근 신에게 당당 지 그리고

그 다 근 그림에 단 복 보다

년 학 간고사 대비2013 2 현대고 대비

ECN-0102-2013-001-000076193

태 도 그리고 극 보다 과

얻 여 었다 과 통

근 그리고 는 재 고 에 질

만들고 특 것 다

공주 그림 가 근 경- ldquo rdquo( 2009)

zb34) 의 이유에 대해 추 한 것으 적절하 않은 것

상과 타 시도①

보다 과 얻②

근 신에게 당당 지③

④ 간 과 진실 에 다가

⑤ 태 도 얻

근 가가 었지만 그 다니 가

럼 어지지 다 복과 쟁 거쳐 시

는 가 근에게 생계 사 에

운 사 다 에 키에 건( ) 178cm死鬪

체 근 에 동 역 업( )荷役

가 생계 다 쟁

에는 동에 운 상우 주 미

죄 사 에 그림 그리는 시 다 그곳에

에 동 역 업 것에

결 것 럼 보 다 지만 그런 것만도

니었다 그림 그리는 고는 지만 매 근

는 극 간 과 별 없는 경 리 그림

벽에 그리는 것 었다 우도 리 없었다 근

트 는 우 그림 그 다 생

계 그림 단 것 다

후 근 지 신 계 리에 미

엑 리 겼다 근 곳에

건 사 크 에 미 들 ( )

상 상 그 다 근 갖 다 겪

냈다 그리고 결 그 돈

신동에 어 사리 집 마 다 마 ㄷ

루 심 쪽에는 과 엌 쪽에는 건

었다 건 주고 근 가 에

여 살 다 심 에는 지 집어

쓰고 지만 곳 근 가 에게 러웠

보 리 다 근 과 마루 업실 삼 그림

그 다 신동 마루는 근 그림에 등 는 lsquo rsquo

같 상들 지 다 시 고

에 들 폐허가

가 업실 었다

공주 그림 가 근 경- ldquo rdquo( 2009)

zb35) 위 에 대한 설 으 적절한 것은

업 시 여 훈과 감동 다①

에 주 평 드러 다②

사 사 등 식 과 ③

④ 다 근거 시 여 삶에

⑤ 살 시 사 경 께 여

습 시 다

가 시간 많지 다 청량리 생 병원( )

마지막 상 경 릿 게 들어 다 그 는

십 만 큰 가 상 말 다

지 못 들 마 갈 고 돗

도시민들 싹 싹 탔다 가 시

월에 병원에 원4 가 폐( )疲弊

진 몸도 갈 미 지 못 고 었다( )解渴 가는

얼마 지 생 에 생각

가 마감 는 신 평생 십 만에

가 과 많 닮 다고 생각 지는

가 운 는 어 어( ) ldquo rdquohelliphellip

월 새벽 시1965 5 6 1 태 없 거웠고

는 없 그 병원에 퇴원 집

가는 마지막 마 고 마 내 거 다

가 죽 간신 에 실 다 사는 어느5 lsquo

가 죽 는 말 가 식 다 신rsquo

상에 각 시키는 에 실( )刻印

어느 가는 후 민 가가 근 었다lsquo rsquo

다 는 간 과 진실 그 다는( ) ldquo

에 단 평 견 가지고 다 내

가 그리는 간상 단 고 다 지 다 는 가

에 는 평 지 니 그리고 어린 들

미지 겨 그린다rdquo

근 간 과 진실 그리고 싶어( )

가 다 근에게 그것 진리 다 거 다 없

년 학 간고사 대비2013 2 현대고 대비

ECN-0102-2013-001-000076193

거 고 다 없 는 것 진리

다 근 진리는 후 쪽 었다 신산( )辛酸 삶

었 질곡 역사 에 지냈( )桎梏

가 눈에 든 것 료 단 료 게 보

것 었다 그것 그 에 겨우겨우

슬 슬 생 어가는 간들 었다

리 과 단 리 고리에 검

마 없 거리 돌

상 것 없는 등 근에게 상에

과 진실 엄 다는 사실 리는 가 실( )儼存

고 가 과 역경 에 도 근 내 포

없었 후 보루 다( ) 堡壘 도 365

도 간 근 여

시 것 다

마 같 가가 고 싶었 근에게 그 꿈( )

에 다가가는 지 다 다 가 지망생들

규 미 상 에 진 고

에 지만 근 다 다 근

미 에 운 것 보통 시 미 시간

다 그런 그에게 없는 연습 가가

통 다 가 귀 시 지 도

얻는 뛸 듯 뻤지만 마 도 (

는 었 에 어린 근 주 에)

에 그림 그리고 지우고( )粉板

복 시간 가는 게 루 보냈다

zb36) 전 의 성 소가 아닌 것을 고르

① 평 ② 사건 ③ 경

④ ⑤ 훈

늘 지 상에 살고 는 사 들 억 도가10

고 그리 지 통 고 는 사 들( )知的

그보다 훨 많 억 도는 고 지 20

통 다 그런 지 고 2500

그리 간 보는 과 사 에

매우 달 뿐만 니 과 에 도 극

루고 었다 미 운 그런 들

살고 는 동 과 사 들 사고 식에

큰 가 다는 다

고 그리 들 우주 개별 고 독립

사 들 생각 지만 고 들 우

주 연 질 간주 다 같( ) 看做

각 도 들에게는 연 질

었지만 그리 들에게는 미 들 결 었

다 고 과 그리 들 사 같

는 동 과 사 에 도 견 다

지심리 미 마 드 겐트 는

살 들에 에 지 다

연 동 과 상 다 과 같 실험

다 크 만든 미드 도 보

여 주고 그 상 닥 고 주었다lsquo (Dax)rsquo

실 닥 는 재 지 는 것 실험 가lsquo rsquo

만들어 낸 다 그런 다 개 다 체 보

여 주었는 는 미드 지만 틱

만들었고 다 는 재료는 크 지만

달 다 그러고 어 것 닥 지 사 들에게 고 lsquo rsquo

게 니 들 주 같 고 는

체 택 고 동 들 같 재료 만들어진 체

택 다 러 는 심지어 살짜리

들에게 도 타났다 것 곧 과 동

다 상 보고 다는 것 미 다

개별 사 보고 고 동 연 질 보

고 는 것 다

동 들 주변 상 에 맞 어 동 고

에 다 사 들 태도 동에 보다 많

주 울 다 동 가 미시간 에

에 경험 다 그는 미식

경 보러 가게 었는 경 체는 매우 재미 었

주변 들 동에 질 다 그 는

들 계 어 상태 경 다

어 들 에 에 그 시 가 계 가

진 것 다 상 살펴 는 말 들 lsquo rsquo

에 그는 에 시 어 도 뒷사

생각 곧 다시 곤 것 다 그런 그에게 뒷

사 고 지 는 들 동 럼

어 웠다

생각 지도 리 드 니 벳-

zb37) 다음 위 의 내 전개 으 만 인lt gt

것은

lt gt

대조의 통해 대상이 닌 특성을 설 하고 있다

일화를 제 하여 자 의 주장을 뒷 침하고 있다

유추의 을 사 하여 독자의 의해를 돕고 있다

대상이 형성되는 과정을 간적 서에 따라 서 하고 있

① ②

③ ④

년 학 간고사 대비2013 2 현대고 대비

ECN-0102-2013-001-000076193

가 우리가 말 고 쓰는 든 단어가 사 에 는( )

것 니다 사 격에 가 는 지만

어 사 과 같 특별 는 사 니lsquo rsquo

단어 격 보 단어가 사 에

등재 어 다 리 리 사 는 단어 도 그

것 시 사 는 어 고 사 에

격 보 것 니다

러 얼 은 사전에 를 있는가 이에 대한 답lsquo rsquo

은 얼 이 유행어인가 아닌가에 따라 갈라 다 이 단어lsquo rsquo

는 년 어 자 에 랐고 쓰이고 있으2002 lsquo rsquo

유행어라고 하 에는 생 이 다 런데 계속

을 유 하 서 사전에 등재될 자격을 획득할 것인가 이

에 대한 답을 내리 는 히 어 다

여 서 가 를 고 해 볼 있다 첫 는 이 단어

를 써야 할 필 가 속적으 있는가 하는 점이다

상주의 열풍에 휩 인 사회 위 에 편 해서 퍼 말

이 얼 인데 과연 런 위 가 속될 것인가 이에lsquo rsquo

대해 필자의 생각은 정적이다 사회 위 가 뀌

런 말을 쓸 일이 없어 것이다

다음은 단어의 성이다 단어의 성이 사회적으 거

감이 없으 계속 사 될 가능성이 높다 런 에서

얼 은 좋은 조건이 아니다 익히 알 졌듯이 이lsquo rsquo

말은 얼 과 청소년층에서 속어 사 하는 이 결합lsquo rsquo lsquo rsquo

된 말이다 얼 에서 얼 을 리하는 조어 도 lsquo rsquo lsquo -rsquo

어에서는 매 낯선 이다 이것만으 도 거 감을 갖

는 사람들이 있다 더 나 속어 결합한 말이다 얼 lsquo rsquo

이 널리 퍼졌다 해도 은 여전히 청소년층의 속어lsquo rsquo

남아 있다 속어는 자연 럽게 아 자리에서나 쓰 에는

담 러 말이다 러한 담을 하고 사

역을 넓혀 가는 속어도 없 는 않다 특히 얼 은 lsquo rsquo

에도 종종 등장한다 만큼 거 감이 많이 희석되었다

고 할 있다 러나 일상의 자연 러 대화에서도 거

리낌 없이 등장하는가 게 는 되 않았다고 생

각한다

얼 이 유사어인 쌈 등을 만들어 내고lsquo rsquo lsquo rsquo

있으니 살아남을 있을 것이라고 는 견해도 있을 것

이다 러나 간이 나 서 유사어를 포함하여 든

말이 사라 사 는 많다 유사어가 많다는 것이 생 을

유 할 있는 절대적인 조건은 아니다

나 언젠가 터 사람들은 어느 단에서 얼 이 가장( )

쁜 사람을 가리켜 얼 이라고 르고 있다 이 얼lsquo rsquo lsquo rsquo

이라는 단어가 최근 어사전에 라 항간에 논란이 일고

있다 아닌 게 아니라 얼 은 유행어처럼 인다 생 lsquo rsquo

도 리 래되 않은 것 같고 언제 사라 도 알

없다 게다가 젊은이들 사이에서 주 쓰일 뿐이다 이런

단어를 사전에 는다는 게 하 이 없어 이 도

한다

러나 속단은 이다 차근차근 따져 볼 일이다

선 얼 이 일 적 유행어인 아닌 주의 게 들여다lsquo rsquo

볼 필 가 있다 유행어란 유행에 따라 빠르게 유포되었

다가 단 간 내에 소 되는 단어나 를 가리킨다

얼 은 인터넷을 통해 속히 퍼 말이다 하 만 일lsquo rsquo

적인 유행어처럼 단 간 내에 사라 않았을 뿐 아니라

현재 도 잦은 빈도 사 되고 있고 앞으 도 상당

간 사 될 것으 측된다 한 언 재단의 뉴 검 lsquo rsquo

색 사이트에 따르 얼 은 년 에 처음 나타난lsquo rsquo 2001

이후 꾸 히 사 되고 있다

이 같은 사 빈도는 얼 이 일 적 유행어 는 현lsquo rsquo

저히 다르다는 것을 여 다 장 간의 생존 만으 도

얼 은 이 한 어의 어휘 에 를 자격을 얻었다lsquo rsquo

고 할 있다 더 이 이라는 비 적 정제된 매체에

높은 빈도 쓰이고 있 않은가 사 빈도 측 에서

필통이나 연필과 같은 단어 대등하거나 더 많이 쓰lsquo rsquo lsquo rsquo

다는 것은 결코 가 게 볼 일이 아니다

이제는 사전이 언어 현 을 빠르게 하는 게 덕인

대가 되었다 세계적으 유 한 의 사전들도 경쟁

적으 어를 고 있다

하 만 얼 은 젊은이들이나 쓰는 속어라고 흠을 잡을lsquo rsquo

도 르겠다 얼 이 주 젊은 층에서 많이 쓰 lsquo rsquo

는 속어임에 틀림없다 러나 어사전에 표 적이고 품

위 있는 말만 어야 한다고 생각한다 것은 커다란

해다 당장 아 어사전이나 펼쳐 라 속어는

설과 같은 비어나 죄자들이 쓰는 은어 어

마니 같은 소 의 사람만이 쓰는 말 도 라 있

않은가 사전은 말 치에 일정 빈도 이상 나타나는 말이

라 말이든 다 할 있다

zb38) 가 나 에 대한 다음의 설( ) ( ) 않은 것은

① 가 는 얼짱 사 에 등재 것에( ) ( ) lsquo rsquo

보 고 다

② 사 등재 가는 단어 격에( )

고 고 는 언 들 언어 사 도에 고 다 ( )

③ 가 얼짱 어지만 신 과 같 매( ) ( ) lsquo rsquo

체에 도 사 는 말 는 고 다

④ 가는 얼짱 어 보고 크게 가지 근( ) lsquo rsquo 3

거 들어 뒷 고 다

⑤ 는 얼짱 어 는 다 특 다는( ) lsquo rsquo

근거 에도 크게 가지 근거 가 들어 주 2

뒷 고 다

가 늘 지 상에 살고 는 사 들 억( ) 10

도가 고 그리 지 통 고 는 사 들

그보다 훨 많 억 도는 고 지 20

통 다 그런 지 고 2500

년 학 간고사 대비2013 2 현대고 대비

ECN-0102-2013-001-000076193

그리 간 보는 과 사 에

매우 달 뿐만 니 과 에 도 극

루고 었다 미 운 그런 들

살고 는 동 과 사 들 사고 식에

큰 가 다는 다

고 그리 들 우주 개별 고 독립

사 들 생각 지만 고 들 우

주 연 질 간주 다 같 각

도 들에게는 연 질 었지

만 그리 들에게는 미 들 결 었다

고 과 그리 들 사 같 는

동 과 사 에 도 견 다

인 리학자인 츠 이마이 디드 겐트너는 두

살이 채 안 된 아이들에서 터 성인에 이르 다양한

연 대의 동양인과 서양인을 대상으 다음과 같은 험

을 했다 저 코르크 만든 피라 드 양의 도형을

여 주고 대상의 이름을 닥 라고 알 주었다lsquo (Dax)rsquo

제 닥 는 존재하 않는 것으 험자가 임의lsquo rsquo

만들어 낸 이름이다 런 다음 두 개의 다른 체를

여 주었는데 하나는 피라 드 양이 만 하얀 플라 틱

으 만들었고 다른 하나는 재 는 코르크 만 양이

달랐다 러고 나서 어떤 것이 닥 인 사람들에게 고 lsquo rsquo

르게 했더니 서양인들은 주 같은 양을 하고 있는

체를 선택했고 동양인들은 같은 재 만들어 체를

선택했다 이러한 차이는 성인은 어 두 살 리

아이들에게서도 나타났다 이것은 곧 서양인과 동양인은

서 다른 세상을 고 있다는 것을 의 한다 략 ( )

는 아주 단 하 서도 인상적인 험을 했다

험에는 동서양의 대학생들이 참여했다 는 험 참가자

들에게 컴퓨터 화 을 통해 속 장 을 담은 애니 이션

을 여 주었다 화 의 앙에는 초점의 역할을 하는 커

다란 고 한 마리가 있었고 주위에는 다른 생

들과 초 자갈 거품 등이 함 제 되었다 화 을

두 씩 후 참가자들은 자 이 것을 회상해 라는

를 았다

결과 서양인 대학생들과 동양인 대학생 두 앙

의 초점 역할을 했던 고 를 동일한 정도 언 했으

나 경 소 위 거품 초 다른 생 들 에 ( )

대해서는 동양인 대학생들이 서양인 대학생들 다 60

이상 더 많이 언 했다 뿐만 아니라 동양인 학생들은 서

양인 학생들에 비해 개 적인 고 다 전체적인 계

를 더 언 하는 경향을 다 략 또한 경의 일 ( )

를 화 킨 림을 제 하 을 때 동양인 대학생들은 대

경의 화를 알아챘 만 서양인 대학생들은 경

의 화를 거의 알아차리 했다 략 ( )

따라서 서양인들만을 대상으 연 한 화lsquo

편성 결 은 잘 된 것일 도 있다 각 과정과 인rsquo

과정의 어떤 이 화 편적이고 어떤 이

화에 따라 달라 는 는 앞으 많은 연 를 통하여 논의

되어야 한다

나 어떤 의 에서 리 두는 이 화적이다 리( )

안에는 다른 사람들과 더 친 한 계를 유 하 는 상호

의존성과 다른 사람들 터 독립적인 존재 살아가 는

독립성이 혼재한다 따라서 이 에서 어떤 특성이 더 강

하게 각되는 상황에 놓이느냐에 따라 서 다른 화적

특 을 일 있다 결 리 두는 어떤 경 에는

동양인처럼 행동하고 어떤 경 에는 서양인처럼 행동하는

것이다

zb39) 가 에 대한 다음의 설( ) 않은 것은

① 는 신 주 뒷 닥 실험과lsquo rsquo lsquo

니 실험 근거 시 다rsquo

② 동 들 상 간 공통 보다는 에 식

는 강 다

③ 들 주변 맥 에는 심 경 어 사건

과 사건 사 계에 상 민감 다

④ 는 동 과 틀린 지 고 는 것lsquo rsquo

니 다 고 다 lsquo rsquo

⑤ 가에 우리 사 들 개 시 가 원( )

집 경 말 고 는 것 개 보다는

에 고 는 것에 다

늘 지 상에 살고 는 사 들 억 도가10

고 그리 지 통 고 는 사 들( )知的

그보다 훨 많 억 도는 고 지 20

통 다 그런 지 고 2500

그리 간 보는 과 사 에

매우 달 뿐만 니 과 에 도 극

루고 었다 미 운 그런 들

살고 는 동 과 사 들 사고 식에

큰 가 다는 다

지심리 미 마 드 겐트 는 동

과 상 다 과 같 실험 다

크 만든 미드 도 보여 주고 그

상 닥 고 주었다 그런 다lsquo (Dax)rsquo

개 다 체 보여 주었는 는 미드

지만 틱 만들었고 다 는 재료는

크 지만 달 다 그러고 어 것 닥 lsquo

지 사 들에게 고 게 니 들 주 같rsquo

고 는 체 택 고 동 들 같

재료 만들어진 체 택 다 러 는

심지어 살짜리 들에게 도 타났다 것

곧 과 동 다 상 보고 다는

것 미 다 개별 사 보고 고 동

년 학 간고사 대비2013 2 현대고 대비

ECN-0102-2013-001-000076193

연 질 보고 는 것 다

동 들 주변 상 에 맞 어 동 고

에 다 사 들 태도 동에 보다

많 주 울 다 동 가 미시간

에 에 경험 다 그는 미

식 경 보러 가게 었는 경 체는 매우 재

미 었 주변 들 동에 질 다 그

는 들 계 어 상태 경

다 어 들 에 에 그 시 가 계

가 진 것 다 뒷사 고 지 는 들

동 럼 어 웠다

그는 경험에 어 얻어 동 들lsquo

각도 상 본다 는 가 우고rsquo

검 여 주 단 도 상 실험 실

시 다 그는 실험 가 들에게 컴퓨 통

담 니 보여 주었다

에는 역 는 커다 고 마리가 었

고 주 에는 다 생 들과 갈 거 등

께 시 었다 본 후 가 들

신 본 것 상 보 는 지시 다

그 결과 생들과 동 생

역 고 동 도 언

경 거 다 생 들에 ( )

는 동 생들 생들보다 60

상 많 언 다 뿐만 니 동 생들

생들에 개별 고 보다 체 계

언 는 경 보 다 경 변 시

킨 그림 시 동 생들 경

변 지만 생들 경 변

거 리지 못 다

지 지 들만 상 연 lsquo

보편 결 못 것 도 다 지각 과 과rsquo

지 과 어 보편 고 어

에 달 지는지는 많 연 통 여

어 다

리 드 니 벳 생각 지도 사- ldquo rdquo( 2004)

zb40) 위 에 대한 설 으 가장 적절한 것은

① 동 과 생 식 강 고 다

② 가지 실험 통 쓴 고 다

③ 닥 실험에 사 본질에 동 사

상에 주 다

④ 니 실험에 동 과 에 지

각 도에 가 다

⑤ 쓴 는 보편 연 에 드러 우월 에

에 근 고 다

가 동 들 주변 상 에 맞 어 동 고( )

에 다 사 들 태도 동에 보다 많

주 울 다 동 가 미시간 에

에 경험 다 그는 미식

경 보러 가게 었는 경 체는 매우 재미 었

주변 들 동에 질 다 그 는

들 계 어 상태 경 다

어 들 에 에 그 시 가 계 가

진 것 다 상 살펴lsquo 는 말 들rsquo

에 그는 에 시 어 도 뒷사

생각 곧 다시 곤 것 다 그런 그에게

뒷사 고 지 는 들 동 럼

어 웠다

그는 경험에 어 얻어( ) 동 들lsquo

각도 상 본다 는 가 우고rsquo

검 여 주 단 도 상 실험

실시 다 실험에는 동 생들 여 다

그는 실험 가 들에게 컴퓨 통

담 니 보여 주었다 에는

역 는 커다 고 마리가 었고 주 에는

다 생 들과 갈 거 등 께 시

었다 본 후 가 들 신 본 것

상 보 는 지시 다

다 그 결과 생들과 동 생( )

역 고 동 도 언

경 거 다 생 들 에 ( )

는 동 생들 생들보다 60

상 많 언 다 뿐만 니 동 생들

생들에 개별 고 보다 체 계

언 는 경 보 다 들어 동

생들 상 체 연못 럼 보 어ldquo 같rdquo

체 맥 언 시 었지만

생들 상 어 같 큰 고 가 쪽 움ldquo

직 어 같 역 고rdquo

언 시 다 경 변 시킨 그

림 시 동 생들 경 변

지만 생들 경 변 거

리지 못 다

년 학 간고사 대비2013 2 현대고 대비

ECN-0102-2013-001-000076193

게 볼 동 들 보다는 큰 그( )

림 보 에 사 과 체 맥 연결시 지각

는 경 고 체에 특 떼어 내

어 독립 보는 것 낯 어 다 에

들 사 에 고 주변 맥 에는 심 경

에 사건과 사건 사 계에 상

민감 편 다

마 지 지( ) 들만 상 연

보편 결 못 것 도 다lsquo rsquo 지각 과

과 지 과 어 보편 고 어

에 달 지는지는 많 연 통 여

어 다

리 드 니 벳 생각 지도 사- ldquo rdquo( 2004)

zb41) 의 하는 가~ 다른 것은

① ② ③

④ ⑤

얼마 그 에 동 사고 식과

사고 식 보여 주는 내 다

들 에 는 탕 고 같 게

어 겨 고 미 에 는 그 크 럼 큰 고

어리 주고 원 는 어 도 는

상 고 생각 다는 것 다 러

는 어떻게 생 것 고 과 그리 거슬

러 가 보 그 단 다

고 연 경 체 경 생 에

다 벼 사는 공동 업과 경험 많 연 역

에 고 들 연 웃과

게 지내 고 탁 연 들

들 지 연 럽게 들 다 민들

웃과 동 게 뿐만 니 는 집 과

게 다

동 시 는 생태 경 에 살 결과

들 다 사 들 사 상 에 주

울 게 었고 는 곧 체 상 과 간 사

계 시 는 낳게 었다 신 가

가 는 체에 는 원 는 동시

에 다 사 들 그 사 포 체 맥 에

다 들 간 사 연

계 체 계에 주 울 는 사고 체계

게 었다

그러 그리 연 경 그 었다 산

지 연결 는 지 건 그리고 역

에 다 런 들 업에 다 사 과

동 므 공동체에

다고 다 고 그리 들

들과는 달리 보 내 감 지 들과

지 크게 느 지 못 다 그

견 다 경우 주 쟁 통 결 는 갖

게 었다

신 사 간 계들 루어진 커다

트워크 에 게 당연 사 역시 연

계들 체 식 게 다 어 상

원 도 그 개체가 체 맥 과

계 에 고 다 게 체 맥 에 주

울 다 보 상 복 과 가변 식 게 고

상에 재 는 많 변 들 사 에 재 는 들도

게 다 들 주 태도 보

는 경우가 많다 쟁 결

통 결 보다는 통 결

는 보 다

그러 고 그리 들 개개 사 사 독

에 주 울 다 사 사 체에

어 그들 사 에 재 는 공통 규 주

고 다 상 원 에도 사

체 내 주 고 다 그들

체 여 탕 체

는 주 태도 시 고 특 사 어

주에 는지 여 그 주에 는 규

견 다 에 는 쟁 식 리

같 리 사고 체계가 달 게 었다

리 드 니 벳 생각 지도 사- ldquo rdquo( 2004)

zb42) 위 에서 사 된 설 과 가장 유사한 것은

① 크톱 컴퓨 는 본체 니 마우 루

어 다

② 곡과 시 리 는 지 과 사 루어 다는 공통

지니고 다

③ 경 고 것과는 달리

경 본 연 태 그 주변 경

④ 벽돌 능 에 사계 내내

습도가 지 다

⑤ 잰느 체 체 지닌 재 체가 없

는 재 눌 다

년 학 간고사 대비2013 2 현대고 대비

ECN-0102-2013-001-000076193

zb43) 는 립 앙 도서 이 정의 일 이다lt gt

도서 장과 이 자의 리 의 정의 연결이

적절하 않은 것은

lt gt

제 조 서 유8 ( )

도서 장은 다른 이 자의 안전을 위협하거나 도서 의①

서를 란하게 할 가 있는 자에 대하여는 도서 출입

을 제한할 있다

도서 장은 이 자가 제 조 각 호의 어느 하나의 행위를 하7②

을 때에는 이 을 하게 하거나 도서 출입을 제한할

있다

제 조자 의 대출9 ( )

도서 자 는 다음 각 호의 경 대출할 있다①

상호대차도서 간에 자 를 류하는 것을 말한다 등 다1 ( )

른 도서 과의 협 을 위하여 필 한 경

공 이 공 행 상 필 하는 경2

에 도서 장이 필 하다고 인정하는 경3

대출이 가능한 도서 자 의 위는 도서 장이 정하는②

에 따른다

제 조 상10 ( )

이 자가 도서 자 설을 더럽히거나 찢거나 뜨①

쓰게 하거나 잃어 린 경 에는 상하여야 한다

도서 장은 제 항에 따른 상 을 정하여 게 하여야1②

한다

제 조이 절차 등11 ( )

이 칙에서 정한 것 에 도서 자 설의 이 절차

이 제한 등에 필 한 사항은 도서 장이 정한다

출처 립 앙 도서- (httpwwwnlgokr)

① 는 도 리 다8

② 도 는 리 다9 1

③ 료 지 는 도 리 다9 2

④ 도 료 변상에 리10 1

⑤ 는 에 도 리 다11

3

도 다 각 같다①

공 공 다만 연1

연 간 다

매월 째 째 월2

도 도 리 그 사3

가 다고 는

도 에 미리 게1 3②

시 여 다

4

도 시간 도 여 게시 다

5

도 료 시 는 는 도①

지에 등 후

등 에 사 도②

7

는 다 각 여 는 니 다

도 료 시 상 리1 lsquo rsquo

도 료 시 훼 는2 middot

지 가 닌 곳에 식 거 담3

우는

도 보 등 보 검색열4 middot

그 에 도 질 지 여 도5

여 게시 사 는

8

도 다 거 도①

질 게 우 가 는 에 여는 도

도 가 각 어느7②

에는 지 게 거 도

9

도 료는 다 각 경우 다①

상 도 간에 료 는 것 말1 (

다 등 다 도 과 여 경우)

공 원 공 상 는 경우2

그 에 도 다고 는 경우3

가능 도 료 는 도②

는 에 다

10

년 학 간고사 대비2013 2 현대고 대비

ECN-0102-2013-001-000076193

가 도 료 시 럽 거 거①

못 쓰게 거 어 린 경우에는 변상 여

도 에 변상 여 게시1②

여 다

zb44) 위 에서 도서 장이 게 해야 할 사항에 해당하는

것을 두 쓰

년 학 간고사 대비2013 2 현대고 대비

ECN-0102-2013-001-000076193

립 도 규

1 ( )

규 립 도 립 어린 청 도(

포 다 료 시 열 시 말) (

다 에 사 규 립 도)

편 진 다

2 ( )

규 립 도 도 다 에( lsquo rsquo )

고 는 도 에 도lsquo rsquo 2 2

료 에 여 다 다만 특 료 귀

료 등 료 에 사 립 도

도 다 다( lsquo rsquo )

3 ( )

도 다 각 같다①

공 공 다만 연1

연 간 다

매월 째 째 월2

도 도 리 그 사3

가 다고 는

도 에 미리 게1 3②

시 여 다

시간4 ( )

도 시간 도 여 게시 다

등 등5 ( )

도 료 시 는 는 도①

지에 등 후

등 에 사 도②

사 료6 ( )

도 료 시 에 사 료는 도

7 ( )

는 다 각 여 는 니 다

도 료 시 상 리1 lsquo rsquo

도 료 시 훼 는2 middot

지 가 닌 곳에 식 거 담3

우는

도 보 등 보 검색열4 middot

그 에 도 질 지 여 도5

여 게시 사 는

질 지8 ( )

도 다 거 도①

질 게 우 가 는 에 여는 도

도 가 각 어느7②

에는 지 게 거 도

료9 ( )

도 료는 다 각 경우 다①

상 도 간에 료 는 것 말1 (

다 등 다 도 과 여 경우)

공 원 공 상 는 경우2

그 에 도 다고 는 경우3

가능 도 료 는 도②

는 에 다

변상10 ( )

가 도 료 시 럽 거 거①

못 쓰게 거 어 린 경우에는 변상 여

도 에 변상 여 게시1②

여 다

등 규 에 것 에 도11 ( )

료 시 등에 사

도 다

립 도- (httpwwwnlgokr)

zb45) 도서 장의 리 있는 조항으 적절하 않

은 것은

① ② ③ ④ ⑤

년 학 간고사 대비2013 2 현대고 대비

ECN-0102-2013-001-000076193

1 ( )

사가 공 는lsquo rsquo

과 여 사 원과 리

사 타 사 규

니다

개 보 보7 ( )

사는 보통신망 등 계 는 에lsquo rsquo lsquo rsquo

원 개 보 보 니다 개lsquo rsquo

보 보 사 에 는 사 개lsquo rsquo

보 취 니다 다만 사는 다 lsquo rsquo

사 계 통 공 는 경우 원 lsquo rsquo

등 개 보 당 사에 습니lsquo rsquo

원 리에8 (lsquo rsquo lsquo rsquo lsquo rsquo

)

원 에 리lsquo rsquo lsquo rsquo lsquo rsquo①

원에게 가 도 여 는lsquo rsquo 3

니다

사는 원 가 개 보 우 가lsquo rsquo lsquo rsquo lsquo rsquo②

거 사 경우 는 미 에 어 거 lsquo

사 사 운 우 가 는 경우 당rsquo lsquo rsquo

습니다lsquo rsquo

원 가 도 거lsquo rsquo lsquo rsquo lsquo rsquo 3③

가 사 고 지 경우에는 시 사에lsquo rsquo

통지 고 사 내에 니다lsquo rsquo

경우에 당 원 사에 그 사실3 lsquo rsquo lsquo rsquo④

통지 지 거 통지 도 사 내에 지 lsquo rsquo

생 경우 사는 지지 습니다lsquo rsquo

사10 (lsquo rsquo )

사는 과 지 미lsquo rsquo①

에 는 지 계 고

공 여 다 여 니다lsquo rsquo

사는 원 게lsquo rsquo lsquo rsquo lsquo rsquo②

도 개 보 신 보 포 보 보 시( )

갖 어 개 보 취 공시 고

니다

사는 과 여 원lsquo rsquo lsquo rsquo③

견 만 당 다고 경우에는

리 여 니다 원 견 만 사 lsquo rsquo

에 는 게시 거 우편 등 통 여

원에게 리 과 결과 달 니다lsquo rsquo

원11 (lsquo rsquo )

원 다 여 는 니다lsquo rsquo ①

신청 는 변경 시 허 내 등1

타 보 도2

사가 게시 보 변경3 lsquo rsquo

사가 보 보 컴퓨 그4 lsquo rsquo (

등 등 신 는 게시)

사 타 등 지 재산 에5 lsquo rsquo 3

사 타 상 거 업6 lsquo rsquo 3

는 폭 시지 상 타 공7 middot middot

에 는 보 에 공개 는 게시 는lsquo rsquo

사 동 없 리 사8 lsquo rsquo

타 거 당9

게시15 (lsquo rsquo )

원 내에 게시 는 게시 게재 는lsquo rsquo lsquo rsquo lsquo rsquo

경우 원 사가 게시 복 lsquo rsquo lsquo rsquo lsquo rsquo middot middot

등 태 언 등에 공 는

것 내에 다 원 본 게시 등 lsquo rsquo lsquo rsquo

크 능 등 여 복 는 등 태

는 것 동 것 니다

- (wwwnavercom)

zb46) 위 은 인터넷 포털사이트의 회 가입을 위한 이

약 의 일 이다 이 약 을 만드는 과정에서 생각한

내 으 적절하 않은 것은

개 보 보 가 지에 별 눠①

겠어

원 가 만들게 에②

시 주어 겠어

원들 게재 게시 다 원 크 다③

는 것 지

④ 원 지 는 뿐만 니 사가 지 는

도 께 달 지

리에 가 생 경우 사가⑤

에 다는 도 듯

1 ( )

사가 공 는lsquo rsquo

과 여 사 원과 리

사 타 사 규

년 학 간고사 대비2013 2 현대고 대비

ECN-0102-2013-001-000076193

니다

개 보 보7 ( )

사는 보통신망 등 계 는 에lsquo rsquo lsquo rsquo

원 개 보 보 니다 개lsquo rsquo

보 보 사 에 는 사 개lsquo rsquo

보 취 니다 다만 사는 다 lsquo rsquo

사 계 통 공 는 경우 원 lsquo rsquo

등 개 보 당 사에 습니lsquo rsquo

원 리에8 (lsquo rsquo lsquo rsquo lsquo rsquo

)

원 에 리lsquo rsquo lsquo rsquo lsquo rsquo①

원에게 가 도 여 는lsquo rsquo 3

니다

사는 원 가 개 보 우 가lsquo rsquo lsquo rsquo lsquo rsquo②

거 사 경우 는 미 에 어 거 lsquo

사 사 운 우 가 는 경우 당rsquo lsquo rsquo

습니다lsquo rsquo

원 가 도 거lsquo rsquo lsquo rsquo lsquo rsquo 3③

가 사 고 지 경우에는 시 사에lsquo rsquo

통지 고 사 내에 니다lsquo rsquo

경우에 당 원 사에 그 사실3 lsquo rsquo lsquo rsquo④

통지 지 거 통지 도 사 내에 지 lsquo rsquo

생 경우 사는 지지 습니다lsquo rsquo

원에 통지9 (lsquo rsquo )

사는 특 다 원에게 통지 경우lsquo rsquo lsquo rsquo

공지 게시 통 상 게시 개별 통지에7

갈 습니다

사10 (lsquo rsquo )

사는 과 지 미lsquo rsquo①

에 는 지 계 고

공 여 다 여 니다lsquo rsquo

사는 원 게lsquo rsquo lsquo rsquo lsquo rsquo②

도 개 보 신 보 포 보 보 시( )

갖 어 개 보 취 공시 고

니다

사는 과 여 원lsquo rsquo lsquo rsquo③

견 만 당 다고 경우에는

리 여 니다 원 견 만 사 lsquo rsquo

에 는 게시 거 우편 등 통 여

원에게 리 과 결과 달 니다lsquo rsquo

원11 (lsquo rsquo )

원 다 여 는 니다lsquo rsquo ①

신청 는 변경 시 허 내 등1

타 보 도2

사가 게시 보 변경3 lsquo rsquo

사가 보 보 컴퓨 그4 lsquo rsquo (

등 등 신 는 게시)

사 타 등 지 재산 에5 lsquo rsquo 3

사 타 상 거 업6 lsquo rsquo 3

는 폭 시지 상 타 공7 middot middot

에 는 보 에 공개 는 게시 는lsquo rsquo

사 동 없 리 사8 lsquo rsquo

타 거 당9

원 계 규 내lsquo rsquo lsquo②

여 공지 주 사 사가 통지 는rsquo lsquo rsquo

사 등 여 타 사 업 에 lsquo rsquo

는 여 는 니다

- (wwwnavercom)

zb47) 위 약 의 조항에서 같은 제점을 하lt gt

고 있는 조항은

lt gt

제휴 회사에 회 의 아이디 개인 정 를 전송할 있도

한 조항은 고객에게 당한 조항이다

1 7 8① ② ③

④ 9 ⑤ 10

립 도 규

1 ( )

규 립 도 립 어린 청 도(

포 다 료 시 열 시 말) (

다 에 사 규 립 도)

편 진 다

2 ( )

규 립 도 도 다 에( lsquo rsquo )

고 는 도 에 도lsquo rsquo 2 2

료 에 여 다 다만 특 료 귀

료 등 료 에 사 립 도

도 다 다( lsquo rsquo )

3 ( )

도 다 각 같다①

공 공 다만 연1

연 간 다

년 학 간고사 대비2013 2 현대고 대비

ECN-0102-2013-001-000076193

매월 째 째 월2

도 도 리 그 사3

가 다고 는

도 에 미리 게1 3②

시 여 다

시간4 ( )

도 시간 도 여 게시 다

등 등5 ( )

도 료 시 는 는 도①

지에 등 후

등 에 사 도②

사 료6 ( )

도 료 시 에 사 료는 도

7 ( )

는 다 각 여 는 니 다

도 료 시 상 리1 lsquo rsquo

도 료 시 훼 는2 middot

지 가 닌 곳에 식 거 담3

우는

도 보 등 보 검색열4 middot

그 에 도 질 지 여 도5

여 게시 사 는

질 지8 ( )

도 다 거 도①

질 게 우 가 는 에 여는 도

도 가 각 어느7②

에는 지 게 거 도

료9 ( )

도 료는 다 각 경우 다①

상 도 간에 료 는 것 말1 (

다 등 다 도 과 여 경우)

공 원 공 상 는 경우2

그 에 도 다고 는 경우3

가능 도 료 는 도②

는 에 다

변상10 ( )

가 도 료 시 럽 거 거①

못 쓰게 거 어 린 경우에는 변상 여

도 에 변상 여 게시1②

여 다

등 규 에 것 에 도11 ( )

료 시 등에 사

도 다

립 도- (httpwwwnlgokr)

zb48) 다음 정 리 의 의 으 볼 때 가장

이 적인 것은

도 시간 도 여 게시 다①

등 에 사 도②

가능 도 료 는 도 는③

에 다

④ 도 에 변상 여 게10 1

시 여 다

⑤ 도 가 각 어느7

에는 지 거 도

zb49) 를 참고하여 이 어의 성격을 설 한lt gt

것으 적절하 않은 것은

① 보 에 는 어 시 상 고 어 시lt gt lsquo rsquo

에 보여주고 다

② 진 어 어원에 견 고 다

에는 타 어 들어가는 것 다 lsquo rsquo

③ 에 들어갈 말 각각 고 어 어 신 어~

들 언어는 질 격 강 통 없었다

④ 시 우리 에 가 었지만 지 계

과 달리 들 통 사 달 어 웠

년 학 간고사 대비2013 2 현대고 대비

ECN-0102-2013-001-000076193

⑤ 크 몽골 만주 공통어가 우리 어 같

계열에 다는 에 사 특 짐

가( )

善化公主主隱 공주님

他密只嫁良置古 몰 결 고

薯童房乙 맛

夜矣卯乙抱遣去如 에 몰 고 가다

( )

始汝 會隱日恚見隱扐 만 에 본

恥隱汝衣淸隱笑 맑 웃

고 시 여 공 크다 만 다[ ] ( ) ( ) ( ) ( )始 汝 會扐

내다 에 보다 견( ) ( )恚 見 다( )隱

럽다 맑다 청 웃( ) ( ) ( ) ( )恥 衣 淸 笑

zb50) 위의 나 를 함 고 음에 답하( ) lt gt

보lt gt

( )素那或云金川 白城郡蛇山人也

운 사산

는 고 다 는( )[ ( ) ] (素那 金川 白城

사산 사 다) ( ) 郡 蛇山

삼 사- lsquo rsquo 47

에 제 된 단어 의 표 리를 조건(1) lt gt ( ) lt gt

에 맞게 서 하

건lt gt

lsquo 었고 었다 태rsquo

에 제 된 단어 동일한 표 리에(2) lt gt ( )

의해 적은 것을 나 에서 찾아 조건 에 맞게 서 하( ) lt gt

건lt gt

에 당 는 각각( ) 개 쓸 것2 단

당 는 가 여러 개 어도 개만 쓸 것 각2

개 과 도 쪽에 개만2 2

드시 지 것( )

과 동 원리 것lsquo 고

과 동 원리 것 다rsquo

태 것

가( )

素那(或云金川) 白城郡蛇山人也

소나 또는 천 이라 한다 는 성 사( ) ( ) ( )素那 金川 白城郡〔 〕

산 사람이다 현대어 풀이( ) ( )蛇山

나( )

紫布岩乎希 회

執音乎手母牛放敎遣 자 손 암쇼 노히 고

吾 不喩慙 伊賜等肹 肹 나 안디 리샤

花 折叱肹 可獻乎理音如 고 것거 도림다

다 향찰은 리말을 리 으 적은 표 이었 만 생( )

은 고 대를 넘 하고 끊어 고 말았다 랜 세

동안 갈고 닦아 체계적이었던 향찰 표 이 사라졌

을 인은 크게 두 가 나누어 생각해 볼 있다

하나는 족 사회의 한 선호도에서 찾을 있다 라 때

향찰은 주 족 계 에서 사 했을 것으 인다 한 을

알 하고서는 한자를 활 하여 리말을 리 으 표

하 란 가능하 때 이다 런데 족들은 간이 흐

를 향찰과 같은 리 표 을 익혀 사 하 다는

아 한 을 대 사 하는 쪽을 선호하게 되었다 더 이

고 초에 인재 등 을 위해 과거제도가 행되 서 한 선

호도가 더 높아졌고 결 향찰은 소 되고 말았다

또 다른 가능성은 한 어의 특성에서 찾을 있다

터 한 과 일 세 나라는 한자 화 에 속해 다

당연한 이야 겠 만 표의 자인 한자는 어를 표 하

에 매 적절하다 어의 음절은 성 ( ) ( )聲母 韻母

이 어 고 여 에 성조가 추가되어 최종 소리가 결정된

다 래서 어는 단음절을 하나의 한자 표 하 된

다 에 초성 성 종성의 세 가 소가 하나의 음절

년 학 간고사 대비2013 2 현대고 대비

ECN-0102-2013-001-000076193

을 이 는 한 어는 음절 조가 잡하고 음절의 가 많아

서 한자 차 만으 한 어의 소리를 만족 럽게 표 할

없었다 를 들어 한 어에서는 어 니 같이 음절 lsquo rsquo

이 어 단어가 얼마든 있으나 어는( ) 複數音節

자 하나 나타내 만이다lsquo [m ]rsquo 母 ǔ

한편 일 어의 표 은 핵 적 단어는 한자 적고 토는

가나라는 일 의 자 적는 이다 적인 의 를 나

타내는 은 표의 자인 한자 적고 적 계를 나

타내는 토는 표음 자 적는 셈이니 자세히 살펴

리의 향찰 표 을 쏙 빼닮았음을 알 있다 한 어 같

은 착어이 서도 일 어에만 향찰과 유사한 표 이 살아

남은 것은 일 어의 특 때 이다 일 어는 하나의 자음과

음의 결합으 음절을 이 고 침이 거의 없는 음절 언어

이다 이러한 음절의 특색에다가 토가 달한 착어라는 점

이 향찰과 유사한 표 이 살아남을 있는 비결이었다

하 만 같은 착어라도 다양한 음소 침이 달한 한

어는 향찰 표 하는 데 근 적으 한계가 있었다

zb51) 다 하여 의 행에 대한 탐 한 결과( ) lt gt 2

않은 것은

보lt gt

善花公主主隱 공주니믄 공주님( )

----------------------------------------

-

他密只嫁良置古 그 지 얼어 고 몰 결(

----------------------------------------

-

薯童房乙 맛 맛( )

夜矣卯乙抱遺去如 몰 고 가다 에 몰 고(

가다)

주동 역 동- (薯童謠『 』

에 2 ( )他密只嫁良置古

얼다 시집가다 결 다 말 lsquo rsquo

① 실질 미 지니고 므 타 타lsquo ( )rsquo lsquo [ ]

② 에 실질 미 타내고 지 는lsquo rsquo lsquo [ ]rsquo lsquo [ ]密只 密 只

계 타내는

③ 얼어는 실질 미 포 고 므 가lsquo rsquo lsquo [ ]rsquo嫁

것lsquo [ ]rsquo 良

④ 고 어간 는 실질 미 지니고 므lsquo rsquo lsquo -rsquo

것lsquo [ ]rsquo 置

⑤ 고 어미 고는 계 타내고 므lsquo rsquo lsquo- rsquo

고 것lsquo [ ]rsquo 古

가( )

엉 훈 민middot middot middot middot middot世 宗 御 製 訓 民 正 音

말 미 듕 귁에 달middot middot middot middot middot middot middot middot中 國 文 字

니 런middot middot middot middot middot middot 어린middot middot middot middot百 姓

니 고 도 내 들middot middot middot middot middot middot middot middot middot 시러middot

펴 몯middot 미middot middot 니 내middot middot middot middot middot middot middot middot 爲

어엿middot 겨 새middot middot middot 믈여듧middot middot middot middot字 니middot middot middot

사 마다 니겨 킈 middot middot middot middot middot middot middot middot middot便 安

고 미니middot middot middot middot

본 는 상( ) (象

원리에 만들어진 본) ( )形 ㄱ ㄴ ㅁ ㅅ ㅇ

에 는 가 원리에( )加劃

그리고( )ㅋ ㄷ ㅌ ㅂ ㅍ ㅈ ㅊ ㆆ ㅎ

쓰는 병 원리에 만들어진( )竝書

마지막 체( ) ( )異體ㄲ ㄸ ㅃ ㅆ ㅉ ㆅ

ᅀ 다 상 원리에 ㅇ ㄹ

지 는 삼재 상 본 본( ) ( ) ( 天地人 三才

탕 므림과 림에 ) (初ㅡ ㅣ

재)( ) ( )( )出字 再出字ㅗ ㅏ ㅜ ㅓ ㅛ ㅑ ㅜ ㅕ

병 그리고 들 에 다시( )ㅘ ㅝ ㅣ

( )ㅣ ㅢ ㅚ ㅐ ㅟ ㅔ ㆉ ㅒ ㆌ ㅖ ㅙ ㅞ

zb52) 가 에 대한 설 으 르 않은 것을( ) 두 고르

① 어쓰 규 지키고 다

② 리 고 다

③ 말 미 미 등 어 사 다lsquo rsquo

④ 개 지 다

년 학 간고사 대비2013 2 현대고 대비

ECN-0102-2013-001-000076193

⑤ 어 원 에 가 도 고 다

엉 훈 민世 宗 御 製 訓 民 正 音

말 미 듕귁에 달 니

런 어린 니 고 도middot

내 들 시러 펴 몯 미 니middot

내 어엿 겨 새 믈여듧

사 마다 니겨middot 킈 고

미니

훈민 언 본- lsquo rsquo 5 (1459 )

zb53) 위의 에 대한 현대어 풀이가 르~ 않은 것

① 우리 말 과 달

② 어리 말 고 는 것 어도

③ 신 생각 마 껏 펼 는 사 많다

④ 게 생각 여

⑤ 사 마다 게

zb54) 훈민정음 언해 에는 한 을 창제한 동 가 드러나

있다 훈민정음 창제의 정 과 내 이 잘 연결된 것

① 주 신 말 미 듕귁에 달

② 민 신 내 어 겨

③ 신 뻔 킈 고 미니

④ 실 신 사 마다 니겨

⑤ 귀 신 계 주 는 훈민 신과 거리가

가 엉 훈 민( ) middot middot middot middot middot世 宗 御 製 訓 民 正 音 

말 미 귁에 中 國 달 文 字

니 런 어린 니 百 姓

고 도 내 들 시러 펴 몯

미 니 내 어엿 爲 겨 새

믈여듧 니 사 마다 니 字

겨 킈 고 미니 便 安

훈민 언 본- lsquo ( )rsquo ( ) 5 (1459 )訓民正音 世祖

( )

[ 1 ]

동 룡 샤 마다 복( ) ( ) ( )海東 六龍 天福

시니 고 동( ) ( )古聖 同符 시니

[ 2 ]

매 니 곶 여

미 므 니 그 내 러

가 니

[ 125 ]

우 미리( )千世 샨( )定 에( )漢水北 累仁

누 개 샤 복 업 시니( ) ( ) 開國 卜年

신( )聖神 니 샤도 경 근민 샤 욱( )敬天勤民

드시리 다

님 쇼 산 가( ) ( )洛水 山行

미드니 가

어 가- lsquo ( )rsquo 27龍飛御天歌

다 우리신 니쓰고 다만 만 쓰( )

거 샹 귀쳔 다보게 러 귀

여 쓴 도 신 보 가 고 신 에

말 어 보게 각 에 사 들

고 본 몬 능통 후에

죠 죠 니

드 도 만 공 에 사

드 미 죠 고 고 여 보 죠

보다 얼마가 거시 어신고 니 첫

가 죠 니 죠

민 들 어 신 샹

귀쳔 도보고 어보 가 만 늘

고 폐 에 만쓴 죠 민

도 러보지못 고 보니 그게 엇지

심 니 리 보 가 어 운건 다

니 쳣 말마 지 니 고 그

쓰 에 가 우 지 지

몰 거 본후에 가 어 지

고 그니 쓴편지 쟝 보

년 학 간고사 대비2013 2 현대고 대비

ECN-0102-2013-001-000076193

쓴것보다 듸 보고 그 마 니 쓴 고

어 못

그런고 에 리 과 가

만 쓴 못 민 말만 듯고

고 편 그 못 보니 그사 단

병신 못 다고 그사 식 사

니 만 고 다 과 그사

만 고 다 과 업 사 보다 식 고

죠 도 고 각 과

견 고 실 직 귀쳔 간에 그

고도 다 것 몰 귀죡 보다

사 우리 신 귀쳔 다 업

시 신 보고 과 지 게 랴

시니 샹 귀쳔 간에 우리 신 걸

간 보 새지각과 새 걸 미리

독립신- lsquo (1896)rsquo

zb55) 친 어 나의 제 장( ) 2 매 함축적

의 가 가장 유사한 것은

① 지 눈 내리고 매 득 니 내 여 가

사- lsquo rsquo

② 도 어 리듯 그 게 어 다

주 사- lsquo rsquo

③ 눈 살 다 죽 어 린 과 체 여

눈 새벽 지 도 살 다

눈- lsquo rsquo

④ 삶 근심과 고단 에 돌 거니는 여 거 는

여 리 내린 살가지 에 눈 리 눈 리

택 그 생 에- lsquo rsquo

⑤ 늘 러 고 러

청룡 룡 어 개 루 우

신경림 계- lsquo rsquo

zb56) 친 를 위 가 나 에 나타난A B ( ) ( )

세 어의 특 에 의거하여 세 어 표 하

그 산 고 공 도 맑지만

A

주변에 쓰 리는 어리 사 많다

B

건lt gt

식 가 에 타 어 특징에( ) ( )

거 과 어쓰 는 고 지 말 것

A

B

zb57) 가 의( ) 달 아ㆍ 다 의 ( ) 나셔에서 알 있는

세 어 개화 어의 특 을 비 하여 조건 에lt gt

맞게 서 하

건lt gt

어에 는lsquo 개

어에 는 다 태rsquo

zb58) 은 가 는 다 에 나 는 절lt 1gt ( ) lt 2gt ( )

일 를 췌한 것이다 의 의 가 lt 1gt (1)~(2)

유사한 말을 에서 찾아 쓰lt 2gt

보lt 1gt

런 (1) 어린 니 고百 姓

도 내 들 시러 펴 몯 미

사 마다 (2) 니겨 便 安

킈 고 미니

보lt 2gt

죠 고 고 여 보 죠

보다 얼마가 거시 어신고 니 첫 가

죠 니 죠 민

들 어 신 샹 귀쳔

도보고 어보 가 만 늘 고

폐 에 만쓴 죠 민 도

러보지못 고 보니 그게 엇지 심

니 리

년 학 간고사 대비2013 2 현대고 대비

ECN-0102-2013-001-000076193

lt 1 gt

동 룡 샤 마다 복 시( ) ( ) ( )海東 六龍 天福

고 동 시니( ) ( )古聖 同符

lt 2 gt

(A) 매 니 곶

여 니

미 므 니 그 내

러 가 니

lt125 gt

우 미리 샨 에( ) ( ) ( ) 千世 定 漢水北 累

누 개 샤 복 업 시 니( ) ( ) 仁開國 卜年 聖

신( ) 神 니 샤도 경 근민 샤( ) 敬天勤民

욱 드 시 리 다

님 쇼 산 가 ( ) ( )洛水 山行

미드니 가

- lt gt龍飛御天歌

zb59) 장과 내 상 유사한 성격의 조는125

① 뫼 고 고 고 고

어 그린 많고 많고 고 고

어 러 는 울고 울고 가느니

도 견- lt gt

② 강 에 드니 몸 다

그믈 고 가니

뒷 뫼 엄 언 니( )藥

-

③ 말 없는 청산 태 없는 다

값 없는 청 없는 월

에 병 없는 몸 별 없 늙 리

-

④ 가마귀 골에 가지 마

낸 가마귀 새

청강에 것 시 몸 러 가( ) 淸江

-

⑤ 진 골에( ) 白雪

가 매 는 어느 곳에 었는고

에 갈 곳 몰( ) 夕陽

색-

zb60) 위 에 나타난 세 어의 특 으 적절하 않은

것은

① 룡 어 주격 사에 당 는 가 사( ) lsquo rsquo六龍

고 다

② 샤 어에도 어 주체 쓰 다

는 것 다

③ 매 어 달리 사 택에 어

가 지 지지 고 다

④ 므 원 상 직 어 지 다

⑤ 드시리 다 주체 과 상 께 사

고 다

수고 하셨습니다hearts hearts

년 학 간고사 대비2013 2 현대고 대비

ECN-0102-2013-001-000076193

보닷컴에 공 는 별 보는 고등

들 여 주 는

들 습니다 슷 동 지

가 복 는 것 도가

니 복 여 습 시고 거 시

니다

정답 해설

1) 정답[ ] ④

해설 다른 것은 두 특정 업이나 단 내에서 사[ ]

하는 일종의 은어 사회 언에 해당한다 러나

는 언이 아니라 단과대학을 여서 단대 사lsquo rsquo lsquo rsquo lsquo④

대학을 여서 사대라고 한 말에 해당하 일rsquo lsquo rsquo

사회에서도 널리 쓰이 사회 언이라 할

없다

2) 정답[ ] ⑤

해설 사회 언은 같은 단 내에서 쓰이는 언어이[ ] lsquo rsquo

동일 단끼리는 단결 과 친 감을 형성하는

능을 하 리적 안감이 일어나 않는다

3) 정답[ ] ③

해설 사람이라는 차 적 표현에 대한 대안적 표현이[ ]lsquo rsquo

인 아내 처 등으 볼 있다lsquo rsquo

4) 정답[ ]⑤

해설 남성은 주 격 체를 사 한다[ ]

5) 정답[ ] ⑤

해설 흑인은 검다라는 뜻을 가 고 있을 뿐 인[ ]lsquo rsquo lsquo rsquo lsquo rsquo

다 열등한 뜻을 내포하 않는다

6) 정답 살 색 첫 작품[ ] - -

해설 살색 혹은 킨색은 한 인의 피 색을 뜻[ ] lsquo rsquo lsquo rsquo

하는 것으 인종 차 을 추 고 출 이주민

의 평등 을 침해할 있어 년 표 이2005

살 색으 이름을 꾸었다 처녀작은 처녀라lsquo rsquo lsquo rsquo lsquo rsquo

는 단어가 가 고 있는 곡된 성 인 을 한 것

으 첫 작품정도 꾸어 사 하는 것이 좋다lsquo rsquo

7) 정답[ ] ⑤

해설 호는 아들에게 해체를 사 하고 있다[ ] ① ②

장 을 성하는 청자는 자 의 아 느리 아lsquo

들 세 이다 호는 아 느리에게 해rsquo ③

체를 사 하고 있다 호가 느리 아 에게 ④

사 한 해 체 아들에게 사 한 해체는 두 비lsquo rsquo lsquo rsquo

격 체에 해당한다 호는 자 의 아랫사람인 ⑤

느리에게 아들과 마찬가 해체를 사 하는 것이

상 이 만 임 을 한 느리에게 고마 과 쁨

존 의 표 를 하 위해 자 의 아 에게 말하듯

해 체를 사 하고 있다

8) 정답[ ] ③

9) 정답[ ] ⑤

10) 정답[ ] ①

해설 청자 할아 가 장의 주체 아 다 높을[ ] ( ) ( )

경 에는 압존 에 의해 장의 주체를 높이 않는lsquo rsquo

다 러 아 서가 아닌 아 는으 계 lsquo rsquo lsquo rsquo lsquo

니다 가 아닌 있 니다 표현하는 것이 르rsquo lsquo rsquo

11) 정답 당이 당을 쫒았다 당이[ ]

당에 다

해설[ ]

12) 정답[ ] ⑤

해설 서 다른 높임표현을 통해 청자에 대해 리[ ] ⑤

적 거리감을 나타내는 인 은 이 아니라 현정이

다 가 에서 현정은 에게 해 체를 사 함으 써 ( )

친근감을 드러낸다 나 에서 연 을 게을리하는 역 ( )

도 들 때 에 화가 난 현정이 선생님에게 항의하

는 장 에서는 하 체를 사 하여 리적 거리lsquo rsquo

가 어졌음을 나타내고 있다

13) 정답[ ] ①

해설 는 는 얼 빛이 날과 어찌 다르 고[ ] lsquo rsquo

라는 뜻으 전과 달리 임이 화자를 않고

있음을 알 있다

14) 정답 달리 후 가 있다 이를 통해 경[ ] lt gt

쾌한 음악성을 형성하고 노 젓는 상황을 체적으

형상화하는 역할을 한다

15) 정답[ ] ①

16) 정답[ ] ⑤

해설 다 의 자연은 를 성찰하게 하는 대상[ ] ( )⑤

이자 정의 대상이다 의 자연은 자 의 상황과 ⑤

처 를 드러내는 경으 서의 역할을 하 이

이 없다

17) 정답[ ] ③

해설 는 빈천 을 해결하고자 했으나 강산[ ] lsquo ( )rsquo 貧賤③

과 풍 을 달라는 에 거절하 다고 함으 써 자

연에 대한 애정을 드러내고 있으 는 않는

임에 대한 망을 개에게 전가 켜서 임에 대한 리

을 드러내고 있다

18) 정답[ ] ③

년 학 간고사 대비2013 2 현대고 대비

ECN-0102-2013-001-000076193

19) 정답[ ] ⑤

해설 고상한 음악가의 이름을 리말 꽝 럽[ ]

게 꿈으 써 언어유희를 통해 음을 유 하고 있

다 이는 고상한 척하는 총 를 비꼼으 써 비판적

태도를 드러내는 것이 대상을 꽝 럽게 표현

하여 총 의 허 과 사치를 풍자하고 있다

20) 정답[ ] ⑤

해설 는 작품 속 경에 대한 설 이 드러나는 것이[ ]

서 자의 주 적인 견해가 접적으 드러나는 것이

아니다

21) 정답[ ] ⑤

22) 정답[ ] ②

23) 정답[ ] ④

24) 정답[ ] ①

해설 적강 티프는 주인공의 비 한 출생이나 능[ ] ①

과 이 있는 것으 조정의 능함을 풍자하는lsquo rsquo

것과는 거리가 다

25) 정답 픔 나[ ] ( )

해설 의 음악은 고통 는 사람들을 위 하고 아픔[ ] lsquo rsquo

을 치유해 주는 능을 한다고 할 있다 의 lt gt

픔 도 소 된 이 과 더 어 살아가는 따뜻한 마음lsquo rsquo

을 상 한다

26) 정답[ ] ⑤

해설 에게 선천적으 주어 각 장애라는 역경[ ]

은 의 이라는 가사 연 을 있다lsquo rsquo

27) 정답[ ] ④

해설 는 장 란 선 에게 은 개인적인 인상을[ ]

소녀 장정 등으 표현한 것이다lsquo rsquo

28) 정답[ ] ②

해설 담자가 피 담자의 언어적 표현이나 비언어[ ]②

적 표현 하 독자는 담의 위 나 피

담자의 감정 상태를 알 있다 이를 통해 독자는

담 상황을 더 생생하게 느낄 있고 피 담자

를 더 잘 이해할 있게 된다

29) 정답[ ]③

해설 일상생활과 역도 선 서의 성과에 된 것에서[ ]

역도를 하 서 겪는 어 과 내적 고민으 화제를

전화하 위한 것이다

30) 정답[ ] ①

해설 릿속에 새겨 넣듯 이 억되도 함 세상[ ] ② ③

살이가 힘들고 고생 러 속 하여 자유를 ④

가 없는 고통의 상태를 비유적으 이르는 말

적의 침입을 막 위해 쌓은 축 켜야 할⑤

대상을 비유적으 이르는 말이다

31) 정답[ ] ④

해설 이 의 종류는 전 으 인 사건 경[ ] lsquo

비평을 성 소 삼는다rsquo

32) 정답[ ] ④

해설 근은 삼대독자 태어났음을 에서 확인할[ ]

있다 형제들과의 담은 이뤄 가 없다

33) 정답[ ] ⑤

해설 근은 가난에도 하고 화가를 꿈꾸었다[ ] (3

단 또한 다른 화가 망생들은 정 육을)

위해 상 학 학 해 유학 에 랐 만

근은 다른 을 찾아야 했다 단 세에(5 ) 18

근은 조선 전람회에 입선하 다 단 의(6 )

만종은 인간과 자연이 엮어 가는 경건한 조화 을lsquo rsquo

나타낸다

34) 정답[ ] ①

해설 근이 속에서도 창작활동을 추 않고[ ]

하는 닭은 은 세상과 타협할 르는

근이 세상의 이해를 하 위한 가장 떳떳한 단

이 때 이다

35) 정답[ ] ⑤

해설 전 은 서 자의 주 적인 평이 리는 것이[ ]

만 위 제 은 인 이 살았던 대 사회적 경

을 통해 객 적인 인 의 을 제 하고 있다

36) 정답[ ] ⑤

해설 전 은 인 사건 경 비평이라는[ ] lsquo rsquo⑤

성 이 어져 있다

37) 정답[ ] ①

해설 이 은 동양인과 서양인의 사고 에 차이가[ ]

있다는 것을 대조를 통해 설 하고 있다 또 쓴이

의 제자가 축 경 를 러 가서 경험한 일화를

통해 동양인이 서양인에 비해 주 상황에 더 많은

주의를 인다는 주장을 뒷 침하고 있다

38) 정답[ ] ④

39) 정답[ ] ②

40) 정답[ ] ②

41) 정답[ ] ④

42) 정답[ ] ③

43) 정답[ ] ④

44) 정답 도서 의 휴 일 도서 의 이 간 도서의[ ]

해설 도서 장은 임의 정한 휴 일과 도서 이[ ]

간 도서의 상 등을 게 할 의 가 있다

년 학 간고사 대비2013 2 현대고 대비

ECN-0102-2013-001-000076193

45) 정답[ ] ①

해설 제 조의 정 휴 일 의 휴 일의 사전 게[ ] 3

는 도서 장의 의 조항에 속한다

46) 정답[ ] ①

해설 개인 정 호 의 를 제 하 했 만 항[ ]

나눠서 제 하 않고 대 나열하고 있다

47) 정답[ ] ②

해설 제 조의 내 을 회사는 다른 회사 협[ ] 7 lsquo

계약을 통해 서비 를 제공하는 경 회 의 아이디

등 개인 정 를 해당 회사에 전송할 있다는 내rsquo

이 있으 의 제점을 제 할 있다②

48) 정답[ ] ④

해설 는 도서 장의 의 에 해당하고 나 는 도[ ] ④

서 장의 리에 해당한다

49) 정답[ ] ③

50) 정답 은 음독으 적었고 은 훈독으 적었[ ] (1)

다 과 동일한 표 리 적은 것은 이고 (2) ce

과 동일한 표 리 적은 것은 이다ab

51) 정답[ ] ③

52) 정답[ ] ①②

53) 정답[ ] ③

54) 정답[ ] ③

55) 정답[ ] ①

56) 정답 른 죠코 어린 노 하니라[ ] A B

57) 정답 세 어에서는 활 형이 칙적으[ ] lsquo rsquoㄹㅇ

나타났 만 개화 어에서는 활 형이 쓰 다 lsquo rsquo ㄹㄴ

58) 정답 호 가 흔[ ] (1) (2)

59) 정답[ ] ④

60) 정답[ ] ③

Page 22: 현대고대비 국어 - chamsoriedu.com 「콘텐츠산업진흥 법」외 에도 저작권 의하여 ... 다른주체에게어떤동작을하도록만드는것을나타내는

년 학 간고사 대비2013 2 현대고 대비

ECN-0102-2013-001-000076193

태 도 그리고 극 보다 과

얻 여 었다 과 통

근 그리고 는 재 고 에 질

만들고 특 것 다

공주 그림 가 근 경- ldquo rdquo( 2009)

zb34) 의 이유에 대해 추 한 것으 적절하 않은 것

상과 타 시도①

보다 과 얻②

근 신에게 당당 지③

④ 간 과 진실 에 다가

⑤ 태 도 얻

근 가가 었지만 그 다니 가

럼 어지지 다 복과 쟁 거쳐 시

는 가 근에게 생계 사 에

운 사 다 에 키에 건( ) 178cm死鬪

체 근 에 동 역 업( )荷役

가 생계 다 쟁

에는 동에 운 상우 주 미

죄 사 에 그림 그리는 시 다 그곳에

에 동 역 업 것에

결 것 럼 보 다 지만 그런 것만도

니었다 그림 그리는 고는 지만 매 근

는 극 간 과 별 없는 경 리 그림

벽에 그리는 것 었다 우도 리 없었다 근

트 는 우 그림 그 다 생

계 그림 단 것 다

후 근 지 신 계 리에 미

엑 리 겼다 근 곳에

건 사 크 에 미 들 ( )

상 상 그 다 근 갖 다 겪

냈다 그리고 결 그 돈

신동에 어 사리 집 마 다 마 ㄷ

루 심 쪽에는 과 엌 쪽에는 건

었다 건 주고 근 가 에

여 살 다 심 에는 지 집어

쓰고 지만 곳 근 가 에게 러웠

보 리 다 근 과 마루 업실 삼 그림

그 다 신동 마루는 근 그림에 등 는 lsquo rsquo

같 상들 지 다 시 고

에 들 폐허가

가 업실 었다

공주 그림 가 근 경- ldquo rdquo( 2009)

zb35) 위 에 대한 설 으 적절한 것은

업 시 여 훈과 감동 다①

에 주 평 드러 다②

사 사 등 식 과 ③

④ 다 근거 시 여 삶에

⑤ 살 시 사 경 께 여

습 시 다

가 시간 많지 다 청량리 생 병원( )

마지막 상 경 릿 게 들어 다 그 는

십 만 큰 가 상 말 다

지 못 들 마 갈 고 돗

도시민들 싹 싹 탔다 가 시

월에 병원에 원4 가 폐( )疲弊

진 몸도 갈 미 지 못 고 었다( )解渴 가는

얼마 지 생 에 생각

가 마감 는 신 평생 십 만에

가 과 많 닮 다고 생각 지는

가 운 는 어 어( ) ldquo rdquohelliphellip

월 새벽 시1965 5 6 1 태 없 거웠고

는 없 그 병원에 퇴원 집

가는 마지막 마 고 마 내 거 다

가 죽 간신 에 실 다 사는 어느5 lsquo

가 죽 는 말 가 식 다 신rsquo

상에 각 시키는 에 실( )刻印

어느 가는 후 민 가가 근 었다lsquo rsquo

다 는 간 과 진실 그 다는( ) ldquo

에 단 평 견 가지고 다 내

가 그리는 간상 단 고 다 지 다 는 가

에 는 평 지 니 그리고 어린 들

미지 겨 그린다rdquo

근 간 과 진실 그리고 싶어( )

가 다 근에게 그것 진리 다 거 다 없

년 학 간고사 대비2013 2 현대고 대비

ECN-0102-2013-001-000076193

거 고 다 없 는 것 진리

다 근 진리는 후 쪽 었다 신산( )辛酸 삶

었 질곡 역사 에 지냈( )桎梏

가 눈에 든 것 료 단 료 게 보

것 었다 그것 그 에 겨우겨우

슬 슬 생 어가는 간들 었다

리 과 단 리 고리에 검

마 없 거리 돌

상 것 없는 등 근에게 상에

과 진실 엄 다는 사실 리는 가 실( )儼存

고 가 과 역경 에 도 근 내 포

없었 후 보루 다( ) 堡壘 도 365

도 간 근 여

시 것 다

마 같 가가 고 싶었 근에게 그 꿈( )

에 다가가는 지 다 다 가 지망생들

규 미 상 에 진 고

에 지만 근 다 다 근

미 에 운 것 보통 시 미 시간

다 그런 그에게 없는 연습 가가

통 다 가 귀 시 지 도

얻는 뛸 듯 뻤지만 마 도 (

는 었 에 어린 근 주 에)

에 그림 그리고 지우고( )粉板

복 시간 가는 게 루 보냈다

zb36) 전 의 성 소가 아닌 것을 고르

① 평 ② 사건 ③ 경

④ ⑤ 훈

늘 지 상에 살고 는 사 들 억 도가10

고 그리 지 통 고 는 사 들( )知的

그보다 훨 많 억 도는 고 지 20

통 다 그런 지 고 2500

그리 간 보는 과 사 에

매우 달 뿐만 니 과 에 도 극

루고 었다 미 운 그런 들

살고 는 동 과 사 들 사고 식에

큰 가 다는 다

고 그리 들 우주 개별 고 독립

사 들 생각 지만 고 들 우

주 연 질 간주 다 같( ) 看做

각 도 들에게는 연 질

었지만 그리 들에게는 미 들 결 었

다 고 과 그리 들 사 같

는 동 과 사 에 도 견 다

지심리 미 마 드 겐트 는

살 들에 에 지 다

연 동 과 상 다 과 같 실험

다 크 만든 미드 도 보

여 주고 그 상 닥 고 주었다lsquo (Dax)rsquo

실 닥 는 재 지 는 것 실험 가lsquo rsquo

만들어 낸 다 그런 다 개 다 체 보

여 주었는 는 미드 지만 틱

만들었고 다 는 재료는 크 지만

달 다 그러고 어 것 닥 지 사 들에게 고 lsquo rsquo

게 니 들 주 같 고 는

체 택 고 동 들 같 재료 만들어진 체

택 다 러 는 심지어 살짜리

들에게 도 타났다 것 곧 과 동

다 상 보고 다는 것 미 다

개별 사 보고 고 동 연 질 보

고 는 것 다

동 들 주변 상 에 맞 어 동 고

에 다 사 들 태도 동에 보다 많

주 울 다 동 가 미시간 에

에 경험 다 그는 미식

경 보러 가게 었는 경 체는 매우 재미 었

주변 들 동에 질 다 그 는

들 계 어 상태 경 다

어 들 에 에 그 시 가 계 가

진 것 다 상 살펴 는 말 들 lsquo rsquo

에 그는 에 시 어 도 뒷사

생각 곧 다시 곤 것 다 그런 그에게 뒷

사 고 지 는 들 동 럼

어 웠다

생각 지도 리 드 니 벳-

zb37) 다음 위 의 내 전개 으 만 인lt gt

것은

lt gt

대조의 통해 대상이 닌 특성을 설 하고 있다

일화를 제 하여 자 의 주장을 뒷 침하고 있다

유추의 을 사 하여 독자의 의해를 돕고 있다

대상이 형성되는 과정을 간적 서에 따라 서 하고 있

① ②

③ ④

년 학 간고사 대비2013 2 현대고 대비

ECN-0102-2013-001-000076193

가 우리가 말 고 쓰는 든 단어가 사 에 는( )

것 니다 사 격에 가 는 지만

어 사 과 같 특별 는 사 니lsquo rsquo

단어 격 보 단어가 사 에

등재 어 다 리 리 사 는 단어 도 그

것 시 사 는 어 고 사 에

격 보 것 니다

러 얼 은 사전에 를 있는가 이에 대한 답lsquo rsquo

은 얼 이 유행어인가 아닌가에 따라 갈라 다 이 단어lsquo rsquo

는 년 어 자 에 랐고 쓰이고 있으2002 lsquo rsquo

유행어라고 하 에는 생 이 다 런데 계속

을 유 하 서 사전에 등재될 자격을 획득할 것인가 이

에 대한 답을 내리 는 히 어 다

여 서 가 를 고 해 볼 있다 첫 는 이 단어

를 써야 할 필 가 속적으 있는가 하는 점이다

상주의 열풍에 휩 인 사회 위 에 편 해서 퍼 말

이 얼 인데 과연 런 위 가 속될 것인가 이에lsquo rsquo

대해 필자의 생각은 정적이다 사회 위 가 뀌

런 말을 쓸 일이 없어 것이다

다음은 단어의 성이다 단어의 성이 사회적으 거

감이 없으 계속 사 될 가능성이 높다 런 에서

얼 은 좋은 조건이 아니다 익히 알 졌듯이 이lsquo rsquo

말은 얼 과 청소년층에서 속어 사 하는 이 결합lsquo rsquo lsquo rsquo

된 말이다 얼 에서 얼 을 리하는 조어 도 lsquo rsquo lsquo -rsquo

어에서는 매 낯선 이다 이것만으 도 거 감을 갖

는 사람들이 있다 더 나 속어 결합한 말이다 얼 lsquo rsquo

이 널리 퍼졌다 해도 은 여전히 청소년층의 속어lsquo rsquo

남아 있다 속어는 자연 럽게 아 자리에서나 쓰 에는

담 러 말이다 러한 담을 하고 사

역을 넓혀 가는 속어도 없 는 않다 특히 얼 은 lsquo rsquo

에도 종종 등장한다 만큼 거 감이 많이 희석되었다

고 할 있다 러나 일상의 자연 러 대화에서도 거

리낌 없이 등장하는가 게 는 되 않았다고 생

각한다

얼 이 유사어인 쌈 등을 만들어 내고lsquo rsquo lsquo rsquo

있으니 살아남을 있을 것이라고 는 견해도 있을 것

이다 러나 간이 나 서 유사어를 포함하여 든

말이 사라 사 는 많다 유사어가 많다는 것이 생 을

유 할 있는 절대적인 조건은 아니다

나 언젠가 터 사람들은 어느 단에서 얼 이 가장( )

쁜 사람을 가리켜 얼 이라고 르고 있다 이 얼lsquo rsquo lsquo rsquo

이라는 단어가 최근 어사전에 라 항간에 논란이 일고

있다 아닌 게 아니라 얼 은 유행어처럼 인다 생 lsquo rsquo

도 리 래되 않은 것 같고 언제 사라 도 알

없다 게다가 젊은이들 사이에서 주 쓰일 뿐이다 이런

단어를 사전에 는다는 게 하 이 없어 이 도

한다

러나 속단은 이다 차근차근 따져 볼 일이다

선 얼 이 일 적 유행어인 아닌 주의 게 들여다lsquo rsquo

볼 필 가 있다 유행어란 유행에 따라 빠르게 유포되었

다가 단 간 내에 소 되는 단어나 를 가리킨다

얼 은 인터넷을 통해 속히 퍼 말이다 하 만 일lsquo rsquo

적인 유행어처럼 단 간 내에 사라 않았을 뿐 아니라

현재 도 잦은 빈도 사 되고 있고 앞으 도 상당

간 사 될 것으 측된다 한 언 재단의 뉴 검 lsquo rsquo

색 사이트에 따르 얼 은 년 에 처음 나타난lsquo rsquo 2001

이후 꾸 히 사 되고 있다

이 같은 사 빈도는 얼 이 일 적 유행어 는 현lsquo rsquo

저히 다르다는 것을 여 다 장 간의 생존 만으 도

얼 은 이 한 어의 어휘 에 를 자격을 얻었다lsquo rsquo

고 할 있다 더 이 이라는 비 적 정제된 매체에

높은 빈도 쓰이고 있 않은가 사 빈도 측 에서

필통이나 연필과 같은 단어 대등하거나 더 많이 쓰lsquo rsquo lsquo rsquo

다는 것은 결코 가 게 볼 일이 아니다

이제는 사전이 언어 현 을 빠르게 하는 게 덕인

대가 되었다 세계적으 유 한 의 사전들도 경쟁

적으 어를 고 있다

하 만 얼 은 젊은이들이나 쓰는 속어라고 흠을 잡을lsquo rsquo

도 르겠다 얼 이 주 젊은 층에서 많이 쓰 lsquo rsquo

는 속어임에 틀림없다 러나 어사전에 표 적이고 품

위 있는 말만 어야 한다고 생각한다 것은 커다란

해다 당장 아 어사전이나 펼쳐 라 속어는

설과 같은 비어나 죄자들이 쓰는 은어 어

마니 같은 소 의 사람만이 쓰는 말 도 라 있

않은가 사전은 말 치에 일정 빈도 이상 나타나는 말이

라 말이든 다 할 있다

zb38) 가 나 에 대한 다음의 설( ) ( ) 않은 것은

① 가 는 얼짱 사 에 등재 것에( ) ( ) lsquo rsquo

보 고 다

② 사 등재 가는 단어 격에( )

고 고 는 언 들 언어 사 도에 고 다 ( )

③ 가 얼짱 어지만 신 과 같 매( ) ( ) lsquo rsquo

체에 도 사 는 말 는 고 다

④ 가는 얼짱 어 보고 크게 가지 근( ) lsquo rsquo 3

거 들어 뒷 고 다

⑤ 는 얼짱 어 는 다 특 다는( ) lsquo rsquo

근거 에도 크게 가지 근거 가 들어 주 2

뒷 고 다

가 늘 지 상에 살고 는 사 들 억( ) 10

도가 고 그리 지 통 고 는 사 들

그보다 훨 많 억 도는 고 지 20

통 다 그런 지 고 2500

년 학 간고사 대비2013 2 현대고 대비

ECN-0102-2013-001-000076193

그리 간 보는 과 사 에

매우 달 뿐만 니 과 에 도 극

루고 었다 미 운 그런 들

살고 는 동 과 사 들 사고 식에

큰 가 다는 다

고 그리 들 우주 개별 고 독립

사 들 생각 지만 고 들 우

주 연 질 간주 다 같 각

도 들에게는 연 질 었지

만 그리 들에게는 미 들 결 었다

고 과 그리 들 사 같 는

동 과 사 에 도 견 다

인 리학자인 츠 이마이 디드 겐트너는 두

살이 채 안 된 아이들에서 터 성인에 이르 다양한

연 대의 동양인과 서양인을 대상으 다음과 같은 험

을 했다 저 코르크 만든 피라 드 양의 도형을

여 주고 대상의 이름을 닥 라고 알 주었다lsquo (Dax)rsquo

제 닥 는 존재하 않는 것으 험자가 임의lsquo rsquo

만들어 낸 이름이다 런 다음 두 개의 다른 체를

여 주었는데 하나는 피라 드 양이 만 하얀 플라 틱

으 만들었고 다른 하나는 재 는 코르크 만 양이

달랐다 러고 나서 어떤 것이 닥 인 사람들에게 고 lsquo rsquo

르게 했더니 서양인들은 주 같은 양을 하고 있는

체를 선택했고 동양인들은 같은 재 만들어 체를

선택했다 이러한 차이는 성인은 어 두 살 리

아이들에게서도 나타났다 이것은 곧 서양인과 동양인은

서 다른 세상을 고 있다는 것을 의 한다 략 ( )

는 아주 단 하 서도 인상적인 험을 했다

험에는 동서양의 대학생들이 참여했다 는 험 참가자

들에게 컴퓨터 화 을 통해 속 장 을 담은 애니 이션

을 여 주었다 화 의 앙에는 초점의 역할을 하는 커

다란 고 한 마리가 있었고 주위에는 다른 생

들과 초 자갈 거품 등이 함 제 되었다 화 을

두 씩 후 참가자들은 자 이 것을 회상해 라는

를 았다

결과 서양인 대학생들과 동양인 대학생 두 앙

의 초점 역할을 했던 고 를 동일한 정도 언 했으

나 경 소 위 거품 초 다른 생 들 에 ( )

대해서는 동양인 대학생들이 서양인 대학생들 다 60

이상 더 많이 언 했다 뿐만 아니라 동양인 학생들은 서

양인 학생들에 비해 개 적인 고 다 전체적인 계

를 더 언 하는 경향을 다 략 또한 경의 일 ( )

를 화 킨 림을 제 하 을 때 동양인 대학생들은 대

경의 화를 알아챘 만 서양인 대학생들은 경

의 화를 거의 알아차리 했다 략 ( )

따라서 서양인들만을 대상으 연 한 화lsquo

편성 결 은 잘 된 것일 도 있다 각 과정과 인rsquo

과정의 어떤 이 화 편적이고 어떤 이

화에 따라 달라 는 는 앞으 많은 연 를 통하여 논의

되어야 한다

나 어떤 의 에서 리 두는 이 화적이다 리( )

안에는 다른 사람들과 더 친 한 계를 유 하 는 상호

의존성과 다른 사람들 터 독립적인 존재 살아가 는

독립성이 혼재한다 따라서 이 에서 어떤 특성이 더 강

하게 각되는 상황에 놓이느냐에 따라 서 다른 화적

특 을 일 있다 결 리 두는 어떤 경 에는

동양인처럼 행동하고 어떤 경 에는 서양인처럼 행동하는

것이다

zb39) 가 에 대한 다음의 설( ) 않은 것은

① 는 신 주 뒷 닥 실험과lsquo rsquo lsquo

니 실험 근거 시 다rsquo

② 동 들 상 간 공통 보다는 에 식

는 강 다

③ 들 주변 맥 에는 심 경 어 사건

과 사건 사 계에 상 민감 다

④ 는 동 과 틀린 지 고 는 것lsquo rsquo

니 다 고 다 lsquo rsquo

⑤ 가에 우리 사 들 개 시 가 원( )

집 경 말 고 는 것 개 보다는

에 고 는 것에 다

늘 지 상에 살고 는 사 들 억 도가10

고 그리 지 통 고 는 사 들( )知的

그보다 훨 많 억 도는 고 지 20

통 다 그런 지 고 2500

그리 간 보는 과 사 에

매우 달 뿐만 니 과 에 도 극

루고 었다 미 운 그런 들

살고 는 동 과 사 들 사고 식에

큰 가 다는 다

지심리 미 마 드 겐트 는 동

과 상 다 과 같 실험 다

크 만든 미드 도 보여 주고 그

상 닥 고 주었다 그런 다lsquo (Dax)rsquo

개 다 체 보여 주었는 는 미드

지만 틱 만들었고 다 는 재료는

크 지만 달 다 그러고 어 것 닥 lsquo

지 사 들에게 고 게 니 들 주 같rsquo

고 는 체 택 고 동 들 같

재료 만들어진 체 택 다 러 는

심지어 살짜리 들에게 도 타났다 것

곧 과 동 다 상 보고 다는

것 미 다 개별 사 보고 고 동

년 학 간고사 대비2013 2 현대고 대비

ECN-0102-2013-001-000076193

연 질 보고 는 것 다

동 들 주변 상 에 맞 어 동 고

에 다 사 들 태도 동에 보다

많 주 울 다 동 가 미시간

에 에 경험 다 그는 미

식 경 보러 가게 었는 경 체는 매우 재

미 었 주변 들 동에 질 다 그

는 들 계 어 상태 경

다 어 들 에 에 그 시 가 계

가 진 것 다 뒷사 고 지 는 들

동 럼 어 웠다

그는 경험에 어 얻어 동 들lsquo

각도 상 본다 는 가 우고rsquo

검 여 주 단 도 상 실험 실

시 다 그는 실험 가 들에게 컴퓨 통

담 니 보여 주었다

에는 역 는 커다 고 마리가 었

고 주 에는 다 생 들과 갈 거 등

께 시 었다 본 후 가 들

신 본 것 상 보 는 지시 다

그 결과 생들과 동 생

역 고 동 도 언

경 거 다 생 들에 ( )

는 동 생들 생들보다 60

상 많 언 다 뿐만 니 동 생들

생들에 개별 고 보다 체 계

언 는 경 보 다 경 변 시

킨 그림 시 동 생들 경

변 지만 생들 경 변

거 리지 못 다

지 지 들만 상 연 lsquo

보편 결 못 것 도 다 지각 과 과rsquo

지 과 어 보편 고 어

에 달 지는지는 많 연 통 여

어 다

리 드 니 벳 생각 지도 사- ldquo rdquo( 2004)

zb40) 위 에 대한 설 으 가장 적절한 것은

① 동 과 생 식 강 고 다

② 가지 실험 통 쓴 고 다

③ 닥 실험에 사 본질에 동 사

상에 주 다

④ 니 실험에 동 과 에 지

각 도에 가 다

⑤ 쓴 는 보편 연 에 드러 우월 에

에 근 고 다

가 동 들 주변 상 에 맞 어 동 고( )

에 다 사 들 태도 동에 보다 많

주 울 다 동 가 미시간 에

에 경험 다 그는 미식

경 보러 가게 었는 경 체는 매우 재미 었

주변 들 동에 질 다 그 는

들 계 어 상태 경 다

어 들 에 에 그 시 가 계 가

진 것 다 상 살펴lsquo 는 말 들rsquo

에 그는 에 시 어 도 뒷사

생각 곧 다시 곤 것 다 그런 그에게

뒷사 고 지 는 들 동 럼

어 웠다

그는 경험에 어 얻어( ) 동 들lsquo

각도 상 본다 는 가 우고rsquo

검 여 주 단 도 상 실험

실시 다 실험에는 동 생들 여 다

그는 실험 가 들에게 컴퓨 통

담 니 보여 주었다 에는

역 는 커다 고 마리가 었고 주 에는

다 생 들과 갈 거 등 께 시

었다 본 후 가 들 신 본 것

상 보 는 지시 다

다 그 결과 생들과 동 생( )

역 고 동 도 언

경 거 다 생 들 에 ( )

는 동 생들 생들보다 60

상 많 언 다 뿐만 니 동 생들

생들에 개별 고 보다 체 계

언 는 경 보 다 들어 동

생들 상 체 연못 럼 보 어ldquo 같rdquo

체 맥 언 시 었지만

생들 상 어 같 큰 고 가 쪽 움ldquo

직 어 같 역 고rdquo

언 시 다 경 변 시킨 그

림 시 동 생들 경 변

지만 생들 경 변 거

리지 못 다

년 학 간고사 대비2013 2 현대고 대비

ECN-0102-2013-001-000076193

게 볼 동 들 보다는 큰 그( )

림 보 에 사 과 체 맥 연결시 지각

는 경 고 체에 특 떼어 내

어 독립 보는 것 낯 어 다 에

들 사 에 고 주변 맥 에는 심 경

에 사건과 사건 사 계에 상

민감 편 다

마 지 지( ) 들만 상 연

보편 결 못 것 도 다lsquo rsquo 지각 과

과 지 과 어 보편 고 어

에 달 지는지는 많 연 통 여

어 다

리 드 니 벳 생각 지도 사- ldquo rdquo( 2004)

zb41) 의 하는 가~ 다른 것은

① ② ③

④ ⑤

얼마 그 에 동 사고 식과

사고 식 보여 주는 내 다

들 에 는 탕 고 같 게

어 겨 고 미 에 는 그 크 럼 큰 고

어리 주고 원 는 어 도 는

상 고 생각 다는 것 다 러

는 어떻게 생 것 고 과 그리 거슬

러 가 보 그 단 다

고 연 경 체 경 생 에

다 벼 사는 공동 업과 경험 많 연 역

에 고 들 연 웃과

게 지내 고 탁 연 들

들 지 연 럽게 들 다 민들

웃과 동 게 뿐만 니 는 집 과

게 다

동 시 는 생태 경 에 살 결과

들 다 사 들 사 상 에 주

울 게 었고 는 곧 체 상 과 간 사

계 시 는 낳게 었다 신 가

가 는 체에 는 원 는 동시

에 다 사 들 그 사 포 체 맥 에

다 들 간 사 연

계 체 계에 주 울 는 사고 체계

게 었다

그러 그리 연 경 그 었다 산

지 연결 는 지 건 그리고 역

에 다 런 들 업에 다 사 과

동 므 공동체에

다고 다 고 그리 들

들과는 달리 보 내 감 지 들과

지 크게 느 지 못 다 그

견 다 경우 주 쟁 통 결 는 갖

게 었다

신 사 간 계들 루어진 커다

트워크 에 게 당연 사 역시 연

계들 체 식 게 다 어 상

원 도 그 개체가 체 맥 과

계 에 고 다 게 체 맥 에 주

울 다 보 상 복 과 가변 식 게 고

상에 재 는 많 변 들 사 에 재 는 들도

게 다 들 주 태도 보

는 경우가 많다 쟁 결

통 결 보다는 통 결

는 보 다

그러 고 그리 들 개개 사 사 독

에 주 울 다 사 사 체에

어 그들 사 에 재 는 공통 규 주

고 다 상 원 에도 사

체 내 주 고 다 그들

체 여 탕 체

는 주 태도 시 고 특 사 어

주에 는지 여 그 주에 는 규

견 다 에 는 쟁 식 리

같 리 사고 체계가 달 게 었다

리 드 니 벳 생각 지도 사- ldquo rdquo( 2004)

zb42) 위 에서 사 된 설 과 가장 유사한 것은

① 크톱 컴퓨 는 본체 니 마우 루

어 다

② 곡과 시 리 는 지 과 사 루어 다는 공통

지니고 다

③ 경 고 것과는 달리

경 본 연 태 그 주변 경

④ 벽돌 능 에 사계 내내

습도가 지 다

⑤ 잰느 체 체 지닌 재 체가 없

는 재 눌 다

년 학 간고사 대비2013 2 현대고 대비

ECN-0102-2013-001-000076193

zb43) 는 립 앙 도서 이 정의 일 이다lt gt

도서 장과 이 자의 리 의 정의 연결이

적절하 않은 것은

lt gt

제 조 서 유8 ( )

도서 장은 다른 이 자의 안전을 위협하거나 도서 의①

서를 란하게 할 가 있는 자에 대하여는 도서 출입

을 제한할 있다

도서 장은 이 자가 제 조 각 호의 어느 하나의 행위를 하7②

을 때에는 이 을 하게 하거나 도서 출입을 제한할

있다

제 조자 의 대출9 ( )

도서 자 는 다음 각 호의 경 대출할 있다①

상호대차도서 간에 자 를 류하는 것을 말한다 등 다1 ( )

른 도서 과의 협 을 위하여 필 한 경

공 이 공 행 상 필 하는 경2

에 도서 장이 필 하다고 인정하는 경3

대출이 가능한 도서 자 의 위는 도서 장이 정하는②

에 따른다

제 조 상10 ( )

이 자가 도서 자 설을 더럽히거나 찢거나 뜨①

쓰게 하거나 잃어 린 경 에는 상하여야 한다

도서 장은 제 항에 따른 상 을 정하여 게 하여야1②

한다

제 조이 절차 등11 ( )

이 칙에서 정한 것 에 도서 자 설의 이 절차

이 제한 등에 필 한 사항은 도서 장이 정한다

출처 립 앙 도서- (httpwwwnlgokr)

① 는 도 리 다8

② 도 는 리 다9 1

③ 료 지 는 도 리 다9 2

④ 도 료 변상에 리10 1

⑤ 는 에 도 리 다11

3

도 다 각 같다①

공 공 다만 연1

연 간 다

매월 째 째 월2

도 도 리 그 사3

가 다고 는

도 에 미리 게1 3②

시 여 다

4

도 시간 도 여 게시 다

5

도 료 시 는 는 도①

지에 등 후

등 에 사 도②

7

는 다 각 여 는 니 다

도 료 시 상 리1 lsquo rsquo

도 료 시 훼 는2 middot

지 가 닌 곳에 식 거 담3

우는

도 보 등 보 검색열4 middot

그 에 도 질 지 여 도5

여 게시 사 는

8

도 다 거 도①

질 게 우 가 는 에 여는 도

도 가 각 어느7②

에는 지 게 거 도

9

도 료는 다 각 경우 다①

상 도 간에 료 는 것 말1 (

다 등 다 도 과 여 경우)

공 원 공 상 는 경우2

그 에 도 다고 는 경우3

가능 도 료 는 도②

는 에 다

10

년 학 간고사 대비2013 2 현대고 대비

ECN-0102-2013-001-000076193

가 도 료 시 럽 거 거①

못 쓰게 거 어 린 경우에는 변상 여

도 에 변상 여 게시1②

여 다

zb44) 위 에서 도서 장이 게 해야 할 사항에 해당하는

것을 두 쓰

년 학 간고사 대비2013 2 현대고 대비

ECN-0102-2013-001-000076193

립 도 규

1 ( )

규 립 도 립 어린 청 도(

포 다 료 시 열 시 말) (

다 에 사 규 립 도)

편 진 다

2 ( )

규 립 도 도 다 에( lsquo rsquo )

고 는 도 에 도lsquo rsquo 2 2

료 에 여 다 다만 특 료 귀

료 등 료 에 사 립 도

도 다 다( lsquo rsquo )

3 ( )

도 다 각 같다①

공 공 다만 연1

연 간 다

매월 째 째 월2

도 도 리 그 사3

가 다고 는

도 에 미리 게1 3②

시 여 다

시간4 ( )

도 시간 도 여 게시 다

등 등5 ( )

도 료 시 는 는 도①

지에 등 후

등 에 사 도②

사 료6 ( )

도 료 시 에 사 료는 도

7 ( )

는 다 각 여 는 니 다

도 료 시 상 리1 lsquo rsquo

도 료 시 훼 는2 middot

지 가 닌 곳에 식 거 담3

우는

도 보 등 보 검색열4 middot

그 에 도 질 지 여 도5

여 게시 사 는

질 지8 ( )

도 다 거 도①

질 게 우 가 는 에 여는 도

도 가 각 어느7②

에는 지 게 거 도

료9 ( )

도 료는 다 각 경우 다①

상 도 간에 료 는 것 말1 (

다 등 다 도 과 여 경우)

공 원 공 상 는 경우2

그 에 도 다고 는 경우3

가능 도 료 는 도②

는 에 다

변상10 ( )

가 도 료 시 럽 거 거①

못 쓰게 거 어 린 경우에는 변상 여

도 에 변상 여 게시1②

여 다

등 규 에 것 에 도11 ( )

료 시 등에 사

도 다

립 도- (httpwwwnlgokr)

zb45) 도서 장의 리 있는 조항으 적절하 않

은 것은

① ② ③ ④ ⑤

년 학 간고사 대비2013 2 현대고 대비

ECN-0102-2013-001-000076193

1 ( )

사가 공 는lsquo rsquo

과 여 사 원과 리

사 타 사 규

니다

개 보 보7 ( )

사는 보통신망 등 계 는 에lsquo rsquo lsquo rsquo

원 개 보 보 니다 개lsquo rsquo

보 보 사 에 는 사 개lsquo rsquo

보 취 니다 다만 사는 다 lsquo rsquo

사 계 통 공 는 경우 원 lsquo rsquo

등 개 보 당 사에 습니lsquo rsquo

원 리에8 (lsquo rsquo lsquo rsquo lsquo rsquo

)

원 에 리lsquo rsquo lsquo rsquo lsquo rsquo①

원에게 가 도 여 는lsquo rsquo 3

니다

사는 원 가 개 보 우 가lsquo rsquo lsquo rsquo lsquo rsquo②

거 사 경우 는 미 에 어 거 lsquo

사 사 운 우 가 는 경우 당rsquo lsquo rsquo

습니다lsquo rsquo

원 가 도 거lsquo rsquo lsquo rsquo lsquo rsquo 3③

가 사 고 지 경우에는 시 사에lsquo rsquo

통지 고 사 내에 니다lsquo rsquo

경우에 당 원 사에 그 사실3 lsquo rsquo lsquo rsquo④

통지 지 거 통지 도 사 내에 지 lsquo rsquo

생 경우 사는 지지 습니다lsquo rsquo

사10 (lsquo rsquo )

사는 과 지 미lsquo rsquo①

에 는 지 계 고

공 여 다 여 니다lsquo rsquo

사는 원 게lsquo rsquo lsquo rsquo lsquo rsquo②

도 개 보 신 보 포 보 보 시( )

갖 어 개 보 취 공시 고

니다

사는 과 여 원lsquo rsquo lsquo rsquo③

견 만 당 다고 경우에는

리 여 니다 원 견 만 사 lsquo rsquo

에 는 게시 거 우편 등 통 여

원에게 리 과 결과 달 니다lsquo rsquo

원11 (lsquo rsquo )

원 다 여 는 니다lsquo rsquo ①

신청 는 변경 시 허 내 등1

타 보 도2

사가 게시 보 변경3 lsquo rsquo

사가 보 보 컴퓨 그4 lsquo rsquo (

등 등 신 는 게시)

사 타 등 지 재산 에5 lsquo rsquo 3

사 타 상 거 업6 lsquo rsquo 3

는 폭 시지 상 타 공7 middot middot

에 는 보 에 공개 는 게시 는lsquo rsquo

사 동 없 리 사8 lsquo rsquo

타 거 당9

게시15 (lsquo rsquo )

원 내에 게시 는 게시 게재 는lsquo rsquo lsquo rsquo lsquo rsquo

경우 원 사가 게시 복 lsquo rsquo lsquo rsquo lsquo rsquo middot middot

등 태 언 등에 공 는

것 내에 다 원 본 게시 등 lsquo rsquo lsquo rsquo

크 능 등 여 복 는 등 태

는 것 동 것 니다

- (wwwnavercom)

zb46) 위 은 인터넷 포털사이트의 회 가입을 위한 이

약 의 일 이다 이 약 을 만드는 과정에서 생각한

내 으 적절하 않은 것은

개 보 보 가 지에 별 눠①

겠어

원 가 만들게 에②

시 주어 겠어

원들 게재 게시 다 원 크 다③

는 것 지

④ 원 지 는 뿐만 니 사가 지 는

도 께 달 지

리에 가 생 경우 사가⑤

에 다는 도 듯

1 ( )

사가 공 는lsquo rsquo

과 여 사 원과 리

사 타 사 규

년 학 간고사 대비2013 2 현대고 대비

ECN-0102-2013-001-000076193

니다

개 보 보7 ( )

사는 보통신망 등 계 는 에lsquo rsquo lsquo rsquo

원 개 보 보 니다 개lsquo rsquo

보 보 사 에 는 사 개lsquo rsquo

보 취 니다 다만 사는 다 lsquo rsquo

사 계 통 공 는 경우 원 lsquo rsquo

등 개 보 당 사에 습니lsquo rsquo

원 리에8 (lsquo rsquo lsquo rsquo lsquo rsquo

)

원 에 리lsquo rsquo lsquo rsquo lsquo rsquo①

원에게 가 도 여 는lsquo rsquo 3

니다

사는 원 가 개 보 우 가lsquo rsquo lsquo rsquo lsquo rsquo②

거 사 경우 는 미 에 어 거 lsquo

사 사 운 우 가 는 경우 당rsquo lsquo rsquo

습니다lsquo rsquo

원 가 도 거lsquo rsquo lsquo rsquo lsquo rsquo 3③

가 사 고 지 경우에는 시 사에lsquo rsquo

통지 고 사 내에 니다lsquo rsquo

경우에 당 원 사에 그 사실3 lsquo rsquo lsquo rsquo④

통지 지 거 통지 도 사 내에 지 lsquo rsquo

생 경우 사는 지지 습니다lsquo rsquo

원에 통지9 (lsquo rsquo )

사는 특 다 원에게 통지 경우lsquo rsquo lsquo rsquo

공지 게시 통 상 게시 개별 통지에7

갈 습니다

사10 (lsquo rsquo )

사는 과 지 미lsquo rsquo①

에 는 지 계 고

공 여 다 여 니다lsquo rsquo

사는 원 게lsquo rsquo lsquo rsquo lsquo rsquo②

도 개 보 신 보 포 보 보 시( )

갖 어 개 보 취 공시 고

니다

사는 과 여 원lsquo rsquo lsquo rsquo③

견 만 당 다고 경우에는

리 여 니다 원 견 만 사 lsquo rsquo

에 는 게시 거 우편 등 통 여

원에게 리 과 결과 달 니다lsquo rsquo

원11 (lsquo rsquo )

원 다 여 는 니다lsquo rsquo ①

신청 는 변경 시 허 내 등1

타 보 도2

사가 게시 보 변경3 lsquo rsquo

사가 보 보 컴퓨 그4 lsquo rsquo (

등 등 신 는 게시)

사 타 등 지 재산 에5 lsquo rsquo 3

사 타 상 거 업6 lsquo rsquo 3

는 폭 시지 상 타 공7 middot middot

에 는 보 에 공개 는 게시 는lsquo rsquo

사 동 없 리 사8 lsquo rsquo

타 거 당9

원 계 규 내lsquo rsquo lsquo②

여 공지 주 사 사가 통지 는rsquo lsquo rsquo

사 등 여 타 사 업 에 lsquo rsquo

는 여 는 니다

- (wwwnavercom)

zb47) 위 약 의 조항에서 같은 제점을 하lt gt

고 있는 조항은

lt gt

제휴 회사에 회 의 아이디 개인 정 를 전송할 있도

한 조항은 고객에게 당한 조항이다

1 7 8① ② ③

④ 9 ⑤ 10

립 도 규

1 ( )

규 립 도 립 어린 청 도(

포 다 료 시 열 시 말) (

다 에 사 규 립 도)

편 진 다

2 ( )

규 립 도 도 다 에( lsquo rsquo )

고 는 도 에 도lsquo rsquo 2 2

료 에 여 다 다만 특 료 귀

료 등 료 에 사 립 도

도 다 다( lsquo rsquo )

3 ( )

도 다 각 같다①

공 공 다만 연1

연 간 다

년 학 간고사 대비2013 2 현대고 대비

ECN-0102-2013-001-000076193

매월 째 째 월2

도 도 리 그 사3

가 다고 는

도 에 미리 게1 3②

시 여 다

시간4 ( )

도 시간 도 여 게시 다

등 등5 ( )

도 료 시 는 는 도①

지에 등 후

등 에 사 도②

사 료6 ( )

도 료 시 에 사 료는 도

7 ( )

는 다 각 여 는 니 다

도 료 시 상 리1 lsquo rsquo

도 료 시 훼 는2 middot

지 가 닌 곳에 식 거 담3

우는

도 보 등 보 검색열4 middot

그 에 도 질 지 여 도5

여 게시 사 는

질 지8 ( )

도 다 거 도①

질 게 우 가 는 에 여는 도

도 가 각 어느7②

에는 지 게 거 도

료9 ( )

도 료는 다 각 경우 다①

상 도 간에 료 는 것 말1 (

다 등 다 도 과 여 경우)

공 원 공 상 는 경우2

그 에 도 다고 는 경우3

가능 도 료 는 도②

는 에 다

변상10 ( )

가 도 료 시 럽 거 거①

못 쓰게 거 어 린 경우에는 변상 여

도 에 변상 여 게시1②

여 다

등 규 에 것 에 도11 ( )

료 시 등에 사

도 다

립 도- (httpwwwnlgokr)

zb48) 다음 정 리 의 의 으 볼 때 가장

이 적인 것은

도 시간 도 여 게시 다①

등 에 사 도②

가능 도 료 는 도 는③

에 다

④ 도 에 변상 여 게10 1

시 여 다

⑤ 도 가 각 어느7

에는 지 거 도

zb49) 를 참고하여 이 어의 성격을 설 한lt gt

것으 적절하 않은 것은

① 보 에 는 어 시 상 고 어 시lt gt lsquo rsquo

에 보여주고 다

② 진 어 어원에 견 고 다

에는 타 어 들어가는 것 다 lsquo rsquo

③ 에 들어갈 말 각각 고 어 어 신 어~

들 언어는 질 격 강 통 없었다

④ 시 우리 에 가 었지만 지 계

과 달리 들 통 사 달 어 웠

년 학 간고사 대비2013 2 현대고 대비

ECN-0102-2013-001-000076193

⑤ 크 몽골 만주 공통어가 우리 어 같

계열에 다는 에 사 특 짐

가( )

善化公主主隱 공주님

他密只嫁良置古 몰 결 고

薯童房乙 맛

夜矣卯乙抱遣去如 에 몰 고 가다

( )

始汝 會隱日恚見隱扐 만 에 본

恥隱汝衣淸隱笑 맑 웃

고 시 여 공 크다 만 다[ ] ( ) ( ) ( ) ( )始 汝 會扐

내다 에 보다 견( ) ( )恚 見 다( )隱

럽다 맑다 청 웃( ) ( ) ( ) ( )恥 衣 淸 笑

zb50) 위의 나 를 함 고 음에 답하( ) lt gt

보lt gt

( )素那或云金川 白城郡蛇山人也

운 사산

는 고 다 는( )[ ( ) ] (素那 金川 白城

사산 사 다) ( ) 郡 蛇山

삼 사- lsquo rsquo 47

에 제 된 단어 의 표 리를 조건(1) lt gt ( ) lt gt

에 맞게 서 하

건lt gt

lsquo 었고 었다 태rsquo

에 제 된 단어 동일한 표 리에(2) lt gt ( )

의해 적은 것을 나 에서 찾아 조건 에 맞게 서 하( ) lt gt

건lt gt

에 당 는 각각( ) 개 쓸 것2 단

당 는 가 여러 개 어도 개만 쓸 것 각2

개 과 도 쪽에 개만2 2

드시 지 것( )

과 동 원리 것lsquo 고

과 동 원리 것 다rsquo

태 것

가( )

素那(或云金川) 白城郡蛇山人也

소나 또는 천 이라 한다 는 성 사( ) ( ) ( )素那 金川 白城郡〔 〕

산 사람이다 현대어 풀이( ) ( )蛇山

나( )

紫布岩乎希 회

執音乎手母牛放敎遣 자 손 암쇼 노히 고

吾 不喩慙 伊賜等肹 肹 나 안디 리샤

花 折叱肹 可獻乎理音如 고 것거 도림다

다 향찰은 리말을 리 으 적은 표 이었 만 생( )

은 고 대를 넘 하고 끊어 고 말았다 랜 세

동안 갈고 닦아 체계적이었던 향찰 표 이 사라졌

을 인은 크게 두 가 나누어 생각해 볼 있다

하나는 족 사회의 한 선호도에서 찾을 있다 라 때

향찰은 주 족 계 에서 사 했을 것으 인다 한 을

알 하고서는 한자를 활 하여 리말을 리 으 표

하 란 가능하 때 이다 런데 족들은 간이 흐

를 향찰과 같은 리 표 을 익혀 사 하 다는

아 한 을 대 사 하는 쪽을 선호하게 되었다 더 이

고 초에 인재 등 을 위해 과거제도가 행되 서 한 선

호도가 더 높아졌고 결 향찰은 소 되고 말았다

또 다른 가능성은 한 어의 특성에서 찾을 있다

터 한 과 일 세 나라는 한자 화 에 속해 다

당연한 이야 겠 만 표의 자인 한자는 어를 표 하

에 매 적절하다 어의 음절은 성 ( ) ( )聲母 韻母

이 어 고 여 에 성조가 추가되어 최종 소리가 결정된

다 래서 어는 단음절을 하나의 한자 표 하 된

다 에 초성 성 종성의 세 가 소가 하나의 음절

년 학 간고사 대비2013 2 현대고 대비

ECN-0102-2013-001-000076193

을 이 는 한 어는 음절 조가 잡하고 음절의 가 많아

서 한자 차 만으 한 어의 소리를 만족 럽게 표 할

없었다 를 들어 한 어에서는 어 니 같이 음절 lsquo rsquo

이 어 단어가 얼마든 있으나 어는( ) 複數音節

자 하나 나타내 만이다lsquo [m ]rsquo 母 ǔ

한편 일 어의 표 은 핵 적 단어는 한자 적고 토는

가나라는 일 의 자 적는 이다 적인 의 를 나

타내는 은 표의 자인 한자 적고 적 계를 나

타내는 토는 표음 자 적는 셈이니 자세히 살펴

리의 향찰 표 을 쏙 빼닮았음을 알 있다 한 어 같

은 착어이 서도 일 어에만 향찰과 유사한 표 이 살아

남은 것은 일 어의 특 때 이다 일 어는 하나의 자음과

음의 결합으 음절을 이 고 침이 거의 없는 음절 언어

이다 이러한 음절의 특색에다가 토가 달한 착어라는 점

이 향찰과 유사한 표 이 살아남을 있는 비결이었다

하 만 같은 착어라도 다양한 음소 침이 달한 한

어는 향찰 표 하는 데 근 적으 한계가 있었다

zb51) 다 하여 의 행에 대한 탐 한 결과( ) lt gt 2

않은 것은

보lt gt

善花公主主隱 공주니믄 공주님( )

----------------------------------------

-

他密只嫁良置古 그 지 얼어 고 몰 결(

----------------------------------------

-

薯童房乙 맛 맛( )

夜矣卯乙抱遺去如 몰 고 가다 에 몰 고(

가다)

주동 역 동- (薯童謠『 』

에 2 ( )他密只嫁良置古

얼다 시집가다 결 다 말 lsquo rsquo

① 실질 미 지니고 므 타 타lsquo ( )rsquo lsquo [ ]

② 에 실질 미 타내고 지 는lsquo rsquo lsquo [ ]rsquo lsquo [ ]密只 密 只

계 타내는

③ 얼어는 실질 미 포 고 므 가lsquo rsquo lsquo [ ]rsquo嫁

것lsquo [ ]rsquo 良

④ 고 어간 는 실질 미 지니고 므lsquo rsquo lsquo -rsquo

것lsquo [ ]rsquo 置

⑤ 고 어미 고는 계 타내고 므lsquo rsquo lsquo- rsquo

고 것lsquo [ ]rsquo 古

가( )

엉 훈 민middot middot middot middot middot世 宗 御 製 訓 民 正 音

말 미 듕 귁에 달middot middot middot middot middot middot middot middot中 國 文 字

니 런middot middot middot middot middot middot 어린middot middot middot middot百 姓

니 고 도 내 들middot middot middot middot middot middot middot middot middot 시러middot

펴 몯middot 미middot middot 니 내middot middot middot middot middot middot middot middot 爲

어엿middot 겨 새middot middot middot 믈여듧middot middot middot middot字 니middot middot middot

사 마다 니겨 킈 middot middot middot middot middot middot middot middot middot便 安

고 미니middot middot middot middot

본 는 상( ) (象

원리에 만들어진 본) ( )形 ㄱ ㄴ ㅁ ㅅ ㅇ

에 는 가 원리에( )加劃

그리고( )ㅋ ㄷ ㅌ ㅂ ㅍ ㅈ ㅊ ㆆ ㅎ

쓰는 병 원리에 만들어진( )竝書

마지막 체( ) ( )異體ㄲ ㄸ ㅃ ㅆ ㅉ ㆅ

ᅀ 다 상 원리에 ㅇ ㄹ

지 는 삼재 상 본 본( ) ( ) ( 天地人 三才

탕 므림과 림에 ) (初ㅡ ㅣ

재)( ) ( )( )出字 再出字ㅗ ㅏ ㅜ ㅓ ㅛ ㅑ ㅜ ㅕ

병 그리고 들 에 다시( )ㅘ ㅝ ㅣ

( )ㅣ ㅢ ㅚ ㅐ ㅟ ㅔ ㆉ ㅒ ㆌ ㅖ ㅙ ㅞ

zb52) 가 에 대한 설 으 르 않은 것을( ) 두 고르

① 어쓰 규 지키고 다

② 리 고 다

③ 말 미 미 등 어 사 다lsquo rsquo

④ 개 지 다

년 학 간고사 대비2013 2 현대고 대비

ECN-0102-2013-001-000076193

⑤ 어 원 에 가 도 고 다

엉 훈 민世 宗 御 製 訓 民 正 音

말 미 듕귁에 달 니

런 어린 니 고 도middot

내 들 시러 펴 몯 미 니middot

내 어엿 겨 새 믈여듧

사 마다 니겨middot 킈 고

미니

훈민 언 본- lsquo rsquo 5 (1459 )

zb53) 위의 에 대한 현대어 풀이가 르~ 않은 것

① 우리 말 과 달

② 어리 말 고 는 것 어도

③ 신 생각 마 껏 펼 는 사 많다

④ 게 생각 여

⑤ 사 마다 게

zb54) 훈민정음 언해 에는 한 을 창제한 동 가 드러나

있다 훈민정음 창제의 정 과 내 이 잘 연결된 것

① 주 신 말 미 듕귁에 달

② 민 신 내 어 겨

③ 신 뻔 킈 고 미니

④ 실 신 사 마다 니겨

⑤ 귀 신 계 주 는 훈민 신과 거리가

가 엉 훈 민( ) middot middot middot middot middot世 宗 御 製 訓 民 正 音 

말 미 귁에 中 國 달 文 字

니 런 어린 니 百 姓

고 도 내 들 시러 펴 몯

미 니 내 어엿 爲 겨 새

믈여듧 니 사 마다 니 字

겨 킈 고 미니 便 安

훈민 언 본- lsquo ( )rsquo ( ) 5 (1459 )訓民正音 世祖

( )

[ 1 ]

동 룡 샤 마다 복( ) ( ) ( )海東 六龍 天福

시니 고 동( ) ( )古聖 同符 시니

[ 2 ]

매 니 곶 여

미 므 니 그 내 러

가 니

[ 125 ]

우 미리( )千世 샨( )定 에( )漢水北 累仁

누 개 샤 복 업 시니( ) ( ) 開國 卜年

신( )聖神 니 샤도 경 근민 샤 욱( )敬天勤民

드시리 다

님 쇼 산 가( ) ( )洛水 山行

미드니 가

어 가- lsquo ( )rsquo 27龍飛御天歌

다 우리신 니쓰고 다만 만 쓰( )

거 샹 귀쳔 다보게 러 귀

여 쓴 도 신 보 가 고 신 에

말 어 보게 각 에 사 들

고 본 몬 능통 후에

죠 죠 니

드 도 만 공 에 사

드 미 죠 고 고 여 보 죠

보다 얼마가 거시 어신고 니 첫

가 죠 니 죠

민 들 어 신 샹

귀쳔 도보고 어보 가 만 늘

고 폐 에 만쓴 죠 민

도 러보지못 고 보니 그게 엇지

심 니 리 보 가 어 운건 다

니 쳣 말마 지 니 고 그

쓰 에 가 우 지 지

몰 거 본후에 가 어 지

고 그니 쓴편지 쟝 보

년 학 간고사 대비2013 2 현대고 대비

ECN-0102-2013-001-000076193

쓴것보다 듸 보고 그 마 니 쓴 고

어 못

그런고 에 리 과 가

만 쓴 못 민 말만 듯고

고 편 그 못 보니 그사 단

병신 못 다고 그사 식 사

니 만 고 다 과 그사

만 고 다 과 업 사 보다 식 고

죠 도 고 각 과

견 고 실 직 귀쳔 간에 그

고도 다 것 몰 귀죡 보다

사 우리 신 귀쳔 다 업

시 신 보고 과 지 게 랴

시니 샹 귀쳔 간에 우리 신 걸

간 보 새지각과 새 걸 미리

독립신- lsquo (1896)rsquo

zb55) 친 어 나의 제 장( ) 2 매 함축적

의 가 가장 유사한 것은

① 지 눈 내리고 매 득 니 내 여 가

사- lsquo rsquo

② 도 어 리듯 그 게 어 다

주 사- lsquo rsquo

③ 눈 살 다 죽 어 린 과 체 여

눈 새벽 지 도 살 다

눈- lsquo rsquo

④ 삶 근심과 고단 에 돌 거니는 여 거 는

여 리 내린 살가지 에 눈 리 눈 리

택 그 생 에- lsquo rsquo

⑤ 늘 러 고 러

청룡 룡 어 개 루 우

신경림 계- lsquo rsquo

zb56) 친 를 위 가 나 에 나타난A B ( ) ( )

세 어의 특 에 의거하여 세 어 표 하

그 산 고 공 도 맑지만

A

주변에 쓰 리는 어리 사 많다

B

건lt gt

식 가 에 타 어 특징에( ) ( )

거 과 어쓰 는 고 지 말 것

A

B

zb57) 가 의( ) 달 아ㆍ 다 의 ( ) 나셔에서 알 있는

세 어 개화 어의 특 을 비 하여 조건 에lt gt

맞게 서 하

건lt gt

어에 는lsquo 개

어에 는 다 태rsquo

zb58) 은 가 는 다 에 나 는 절lt 1gt ( ) lt 2gt ( )

일 를 췌한 것이다 의 의 가 lt 1gt (1)~(2)

유사한 말을 에서 찾아 쓰lt 2gt

보lt 1gt

런 (1) 어린 니 고百 姓

도 내 들 시러 펴 몯 미

사 마다 (2) 니겨 便 安

킈 고 미니

보lt 2gt

죠 고 고 여 보 죠

보다 얼마가 거시 어신고 니 첫 가

죠 니 죠 민

들 어 신 샹 귀쳔

도보고 어보 가 만 늘 고

폐 에 만쓴 죠 민 도

러보지못 고 보니 그게 엇지 심

니 리

년 학 간고사 대비2013 2 현대고 대비

ECN-0102-2013-001-000076193

lt 1 gt

동 룡 샤 마다 복 시( ) ( ) ( )海東 六龍 天福

고 동 시니( ) ( )古聖 同符

lt 2 gt

(A) 매 니 곶

여 니

미 므 니 그 내

러 가 니

lt125 gt

우 미리 샨 에( ) ( ) ( ) 千世 定 漢水北 累

누 개 샤 복 업 시 니( ) ( ) 仁開國 卜年 聖

신( ) 神 니 샤도 경 근민 샤( ) 敬天勤民

욱 드 시 리 다

님 쇼 산 가 ( ) ( )洛水 山行

미드니 가

- lt gt龍飛御天歌

zb59) 장과 내 상 유사한 성격의 조는125

① 뫼 고 고 고 고

어 그린 많고 많고 고 고

어 러 는 울고 울고 가느니

도 견- lt gt

② 강 에 드니 몸 다

그믈 고 가니

뒷 뫼 엄 언 니( )藥

-

③ 말 없는 청산 태 없는 다

값 없는 청 없는 월

에 병 없는 몸 별 없 늙 리

-

④ 가마귀 골에 가지 마

낸 가마귀 새

청강에 것 시 몸 러 가( ) 淸江

-

⑤ 진 골에( ) 白雪

가 매 는 어느 곳에 었는고

에 갈 곳 몰( ) 夕陽

색-

zb60) 위 에 나타난 세 어의 특 으 적절하 않은

것은

① 룡 어 주격 사에 당 는 가 사( ) lsquo rsquo六龍

고 다

② 샤 어에도 어 주체 쓰 다

는 것 다

③ 매 어 달리 사 택에 어

가 지 지지 고 다

④ 므 원 상 직 어 지 다

⑤ 드시리 다 주체 과 상 께 사

고 다

수고 하셨습니다hearts hearts

년 학 간고사 대비2013 2 현대고 대비

ECN-0102-2013-001-000076193

보닷컴에 공 는 별 보는 고등

들 여 주 는

들 습니다 슷 동 지

가 복 는 것 도가

니 복 여 습 시고 거 시

니다

정답 해설

1) 정답[ ] ④

해설 다른 것은 두 특정 업이나 단 내에서 사[ ]

하는 일종의 은어 사회 언에 해당한다 러나

는 언이 아니라 단과대학을 여서 단대 사lsquo rsquo lsquo rsquo lsquo④

대학을 여서 사대라고 한 말에 해당하 일rsquo lsquo rsquo

사회에서도 널리 쓰이 사회 언이라 할

없다

2) 정답[ ] ⑤

해설 사회 언은 같은 단 내에서 쓰이는 언어이[ ] lsquo rsquo

동일 단끼리는 단결 과 친 감을 형성하는

능을 하 리적 안감이 일어나 않는다

3) 정답[ ] ③

해설 사람이라는 차 적 표현에 대한 대안적 표현이[ ]lsquo rsquo

인 아내 처 등으 볼 있다lsquo rsquo

4) 정답[ ]⑤

해설 남성은 주 격 체를 사 한다[ ]

5) 정답[ ] ⑤

해설 흑인은 검다라는 뜻을 가 고 있을 뿐 인[ ]lsquo rsquo lsquo rsquo lsquo rsquo

다 열등한 뜻을 내포하 않는다

6) 정답 살 색 첫 작품[ ] - -

해설 살색 혹은 킨색은 한 인의 피 색을 뜻[ ] lsquo rsquo lsquo rsquo

하는 것으 인종 차 을 추 고 출 이주민

의 평등 을 침해할 있어 년 표 이2005

살 색으 이름을 꾸었다 처녀작은 처녀라lsquo rsquo lsquo rsquo lsquo rsquo

는 단어가 가 고 있는 곡된 성 인 을 한 것

으 첫 작품정도 꾸어 사 하는 것이 좋다lsquo rsquo

7) 정답[ ] ⑤

해설 호는 아들에게 해체를 사 하고 있다[ ] ① ②

장 을 성하는 청자는 자 의 아 느리 아lsquo

들 세 이다 호는 아 느리에게 해rsquo ③

체를 사 하고 있다 호가 느리 아 에게 ④

사 한 해 체 아들에게 사 한 해체는 두 비lsquo rsquo lsquo rsquo

격 체에 해당한다 호는 자 의 아랫사람인 ⑤

느리에게 아들과 마찬가 해체를 사 하는 것이

상 이 만 임 을 한 느리에게 고마 과 쁨

존 의 표 를 하 위해 자 의 아 에게 말하듯

해 체를 사 하고 있다

8) 정답[ ] ③

9) 정답[ ] ⑤

10) 정답[ ] ①

해설 청자 할아 가 장의 주체 아 다 높을[ ] ( ) ( )

경 에는 압존 에 의해 장의 주체를 높이 않는lsquo rsquo

다 러 아 서가 아닌 아 는으 계 lsquo rsquo lsquo rsquo lsquo

니다 가 아닌 있 니다 표현하는 것이 르rsquo lsquo rsquo

11) 정답 당이 당을 쫒았다 당이[ ]

당에 다

해설[ ]

12) 정답[ ] ⑤

해설 서 다른 높임표현을 통해 청자에 대해 리[ ] ⑤

적 거리감을 나타내는 인 은 이 아니라 현정이

다 가 에서 현정은 에게 해 체를 사 함으 써 ( )

친근감을 드러낸다 나 에서 연 을 게을리하는 역 ( )

도 들 때 에 화가 난 현정이 선생님에게 항의하

는 장 에서는 하 체를 사 하여 리적 거리lsquo rsquo

가 어졌음을 나타내고 있다

13) 정답[ ] ①

해설 는 는 얼 빛이 날과 어찌 다르 고[ ] lsquo rsquo

라는 뜻으 전과 달리 임이 화자를 않고

있음을 알 있다

14) 정답 달리 후 가 있다 이를 통해 경[ ] lt gt

쾌한 음악성을 형성하고 노 젓는 상황을 체적으

형상화하는 역할을 한다

15) 정답[ ] ①

16) 정답[ ] ⑤

해설 다 의 자연은 를 성찰하게 하는 대상[ ] ( )⑤

이자 정의 대상이다 의 자연은 자 의 상황과 ⑤

처 를 드러내는 경으 서의 역할을 하 이

이 없다

17) 정답[ ] ③

해설 는 빈천 을 해결하고자 했으나 강산[ ] lsquo ( )rsquo 貧賤③

과 풍 을 달라는 에 거절하 다고 함으 써 자

연에 대한 애정을 드러내고 있으 는 않는

임에 대한 망을 개에게 전가 켜서 임에 대한 리

을 드러내고 있다

18) 정답[ ] ③

년 학 간고사 대비2013 2 현대고 대비

ECN-0102-2013-001-000076193

19) 정답[ ] ⑤

해설 고상한 음악가의 이름을 리말 꽝 럽[ ]

게 꿈으 써 언어유희를 통해 음을 유 하고 있

다 이는 고상한 척하는 총 를 비꼼으 써 비판적

태도를 드러내는 것이 대상을 꽝 럽게 표현

하여 총 의 허 과 사치를 풍자하고 있다

20) 정답[ ] ⑤

해설 는 작품 속 경에 대한 설 이 드러나는 것이[ ]

서 자의 주 적인 견해가 접적으 드러나는 것이

아니다

21) 정답[ ] ⑤

22) 정답[ ] ②

23) 정답[ ] ④

24) 정답[ ] ①

해설 적강 티프는 주인공의 비 한 출생이나 능[ ] ①

과 이 있는 것으 조정의 능함을 풍자하는lsquo rsquo

것과는 거리가 다

25) 정답 픔 나[ ] ( )

해설 의 음악은 고통 는 사람들을 위 하고 아픔[ ] lsquo rsquo

을 치유해 주는 능을 한다고 할 있다 의 lt gt

픔 도 소 된 이 과 더 어 살아가는 따뜻한 마음lsquo rsquo

을 상 한다

26) 정답[ ] ⑤

해설 에게 선천적으 주어 각 장애라는 역경[ ]

은 의 이라는 가사 연 을 있다lsquo rsquo

27) 정답[ ] ④

해설 는 장 란 선 에게 은 개인적인 인상을[ ]

소녀 장정 등으 표현한 것이다lsquo rsquo

28) 정답[ ] ②

해설 담자가 피 담자의 언어적 표현이나 비언어[ ]②

적 표현 하 독자는 담의 위 나 피

담자의 감정 상태를 알 있다 이를 통해 독자는

담 상황을 더 생생하게 느낄 있고 피 담자

를 더 잘 이해할 있게 된다

29) 정답[ ]③

해설 일상생활과 역도 선 서의 성과에 된 것에서[ ]

역도를 하 서 겪는 어 과 내적 고민으 화제를

전화하 위한 것이다

30) 정답[ ] ①

해설 릿속에 새겨 넣듯 이 억되도 함 세상[ ] ② ③

살이가 힘들고 고생 러 속 하여 자유를 ④

가 없는 고통의 상태를 비유적으 이르는 말

적의 침입을 막 위해 쌓은 축 켜야 할⑤

대상을 비유적으 이르는 말이다

31) 정답[ ] ④

해설 이 의 종류는 전 으 인 사건 경[ ] lsquo

비평을 성 소 삼는다rsquo

32) 정답[ ] ④

해설 근은 삼대독자 태어났음을 에서 확인할[ ]

있다 형제들과의 담은 이뤄 가 없다

33) 정답[ ] ⑤

해설 근은 가난에도 하고 화가를 꿈꾸었다[ ] (3

단 또한 다른 화가 망생들은 정 육을)

위해 상 학 학 해 유학 에 랐 만

근은 다른 을 찾아야 했다 단 세에(5 ) 18

근은 조선 전람회에 입선하 다 단 의(6 )

만종은 인간과 자연이 엮어 가는 경건한 조화 을lsquo rsquo

나타낸다

34) 정답[ ] ①

해설 근이 속에서도 창작활동을 추 않고[ ]

하는 닭은 은 세상과 타협할 르는

근이 세상의 이해를 하 위한 가장 떳떳한 단

이 때 이다

35) 정답[ ] ⑤

해설 전 은 서 자의 주 적인 평이 리는 것이[ ]

만 위 제 은 인 이 살았던 대 사회적 경

을 통해 객 적인 인 의 을 제 하고 있다

36) 정답[ ] ⑤

해설 전 은 인 사건 경 비평이라는[ ] lsquo rsquo⑤

성 이 어져 있다

37) 정답[ ] ①

해설 이 은 동양인과 서양인의 사고 에 차이가[ ]

있다는 것을 대조를 통해 설 하고 있다 또 쓴이

의 제자가 축 경 를 러 가서 경험한 일화를

통해 동양인이 서양인에 비해 주 상황에 더 많은

주의를 인다는 주장을 뒷 침하고 있다

38) 정답[ ] ④

39) 정답[ ] ②

40) 정답[ ] ②

41) 정답[ ] ④

42) 정답[ ] ③

43) 정답[ ] ④

44) 정답 도서 의 휴 일 도서 의 이 간 도서의[ ]

해설 도서 장은 임의 정한 휴 일과 도서 이[ ]

간 도서의 상 등을 게 할 의 가 있다

년 학 간고사 대비2013 2 현대고 대비

ECN-0102-2013-001-000076193

45) 정답[ ] ①

해설 제 조의 정 휴 일 의 휴 일의 사전 게[ ] 3

는 도서 장의 의 조항에 속한다

46) 정답[ ] ①

해설 개인 정 호 의 를 제 하 했 만 항[ ]

나눠서 제 하 않고 대 나열하고 있다

47) 정답[ ] ②

해설 제 조의 내 을 회사는 다른 회사 협[ ] 7 lsquo

계약을 통해 서비 를 제공하는 경 회 의 아이디

등 개인 정 를 해당 회사에 전송할 있다는 내rsquo

이 있으 의 제점을 제 할 있다②

48) 정답[ ] ④

해설 는 도서 장의 의 에 해당하고 나 는 도[ ] ④

서 장의 리에 해당한다

49) 정답[ ] ③

50) 정답 은 음독으 적었고 은 훈독으 적었[ ] (1)

다 과 동일한 표 리 적은 것은 이고 (2) ce

과 동일한 표 리 적은 것은 이다ab

51) 정답[ ] ③

52) 정답[ ] ①②

53) 정답[ ] ③

54) 정답[ ] ③

55) 정답[ ] ①

56) 정답 른 죠코 어린 노 하니라[ ] A B

57) 정답 세 어에서는 활 형이 칙적으[ ] lsquo rsquoㄹㅇ

나타났 만 개화 어에서는 활 형이 쓰 다 lsquo rsquo ㄹㄴ

58) 정답 호 가 흔[ ] (1) (2)

59) 정답[ ] ④

60) 정답[ ] ③

Page 23: 현대고대비 국어 - chamsoriedu.com 「콘텐츠산업진흥 법」외 에도 저작권 의하여 ... 다른주체에게어떤동작을하도록만드는것을나타내는

년 학 간고사 대비2013 2 현대고 대비

ECN-0102-2013-001-000076193

거 고 다 없 는 것 진리

다 근 진리는 후 쪽 었다 신산( )辛酸 삶

었 질곡 역사 에 지냈( )桎梏

가 눈에 든 것 료 단 료 게 보

것 었다 그것 그 에 겨우겨우

슬 슬 생 어가는 간들 었다

리 과 단 리 고리에 검

마 없 거리 돌

상 것 없는 등 근에게 상에

과 진실 엄 다는 사실 리는 가 실( )儼存

고 가 과 역경 에 도 근 내 포

없었 후 보루 다( ) 堡壘 도 365

도 간 근 여

시 것 다

마 같 가가 고 싶었 근에게 그 꿈( )

에 다가가는 지 다 다 가 지망생들

규 미 상 에 진 고

에 지만 근 다 다 근

미 에 운 것 보통 시 미 시간

다 그런 그에게 없는 연습 가가

통 다 가 귀 시 지 도

얻는 뛸 듯 뻤지만 마 도 (

는 었 에 어린 근 주 에)

에 그림 그리고 지우고( )粉板

복 시간 가는 게 루 보냈다

zb36) 전 의 성 소가 아닌 것을 고르

① 평 ② 사건 ③ 경

④ ⑤ 훈

늘 지 상에 살고 는 사 들 억 도가10

고 그리 지 통 고 는 사 들( )知的

그보다 훨 많 억 도는 고 지 20

통 다 그런 지 고 2500

그리 간 보는 과 사 에

매우 달 뿐만 니 과 에 도 극

루고 었다 미 운 그런 들

살고 는 동 과 사 들 사고 식에

큰 가 다는 다

고 그리 들 우주 개별 고 독립

사 들 생각 지만 고 들 우

주 연 질 간주 다 같( ) 看做

각 도 들에게는 연 질

었지만 그리 들에게는 미 들 결 었

다 고 과 그리 들 사 같

는 동 과 사 에 도 견 다

지심리 미 마 드 겐트 는

살 들에 에 지 다

연 동 과 상 다 과 같 실험

다 크 만든 미드 도 보

여 주고 그 상 닥 고 주었다lsquo (Dax)rsquo

실 닥 는 재 지 는 것 실험 가lsquo rsquo

만들어 낸 다 그런 다 개 다 체 보

여 주었는 는 미드 지만 틱

만들었고 다 는 재료는 크 지만

달 다 그러고 어 것 닥 지 사 들에게 고 lsquo rsquo

게 니 들 주 같 고 는

체 택 고 동 들 같 재료 만들어진 체

택 다 러 는 심지어 살짜리

들에게 도 타났다 것 곧 과 동

다 상 보고 다는 것 미 다

개별 사 보고 고 동 연 질 보

고 는 것 다

동 들 주변 상 에 맞 어 동 고

에 다 사 들 태도 동에 보다 많

주 울 다 동 가 미시간 에

에 경험 다 그는 미식

경 보러 가게 었는 경 체는 매우 재미 었

주변 들 동에 질 다 그 는

들 계 어 상태 경 다

어 들 에 에 그 시 가 계 가

진 것 다 상 살펴 는 말 들 lsquo rsquo

에 그는 에 시 어 도 뒷사

생각 곧 다시 곤 것 다 그런 그에게 뒷

사 고 지 는 들 동 럼

어 웠다

생각 지도 리 드 니 벳-

zb37) 다음 위 의 내 전개 으 만 인lt gt

것은

lt gt

대조의 통해 대상이 닌 특성을 설 하고 있다

일화를 제 하여 자 의 주장을 뒷 침하고 있다

유추의 을 사 하여 독자의 의해를 돕고 있다

대상이 형성되는 과정을 간적 서에 따라 서 하고 있

① ②

③ ④

년 학 간고사 대비2013 2 현대고 대비

ECN-0102-2013-001-000076193

가 우리가 말 고 쓰는 든 단어가 사 에 는( )

것 니다 사 격에 가 는 지만

어 사 과 같 특별 는 사 니lsquo rsquo

단어 격 보 단어가 사 에

등재 어 다 리 리 사 는 단어 도 그

것 시 사 는 어 고 사 에

격 보 것 니다

러 얼 은 사전에 를 있는가 이에 대한 답lsquo rsquo

은 얼 이 유행어인가 아닌가에 따라 갈라 다 이 단어lsquo rsquo

는 년 어 자 에 랐고 쓰이고 있으2002 lsquo rsquo

유행어라고 하 에는 생 이 다 런데 계속

을 유 하 서 사전에 등재될 자격을 획득할 것인가 이

에 대한 답을 내리 는 히 어 다

여 서 가 를 고 해 볼 있다 첫 는 이 단어

를 써야 할 필 가 속적으 있는가 하는 점이다

상주의 열풍에 휩 인 사회 위 에 편 해서 퍼 말

이 얼 인데 과연 런 위 가 속될 것인가 이에lsquo rsquo

대해 필자의 생각은 정적이다 사회 위 가 뀌

런 말을 쓸 일이 없어 것이다

다음은 단어의 성이다 단어의 성이 사회적으 거

감이 없으 계속 사 될 가능성이 높다 런 에서

얼 은 좋은 조건이 아니다 익히 알 졌듯이 이lsquo rsquo

말은 얼 과 청소년층에서 속어 사 하는 이 결합lsquo rsquo lsquo rsquo

된 말이다 얼 에서 얼 을 리하는 조어 도 lsquo rsquo lsquo -rsquo

어에서는 매 낯선 이다 이것만으 도 거 감을 갖

는 사람들이 있다 더 나 속어 결합한 말이다 얼 lsquo rsquo

이 널리 퍼졌다 해도 은 여전히 청소년층의 속어lsquo rsquo

남아 있다 속어는 자연 럽게 아 자리에서나 쓰 에는

담 러 말이다 러한 담을 하고 사

역을 넓혀 가는 속어도 없 는 않다 특히 얼 은 lsquo rsquo

에도 종종 등장한다 만큼 거 감이 많이 희석되었다

고 할 있다 러나 일상의 자연 러 대화에서도 거

리낌 없이 등장하는가 게 는 되 않았다고 생

각한다

얼 이 유사어인 쌈 등을 만들어 내고lsquo rsquo lsquo rsquo

있으니 살아남을 있을 것이라고 는 견해도 있을 것

이다 러나 간이 나 서 유사어를 포함하여 든

말이 사라 사 는 많다 유사어가 많다는 것이 생 을

유 할 있는 절대적인 조건은 아니다

나 언젠가 터 사람들은 어느 단에서 얼 이 가장( )

쁜 사람을 가리켜 얼 이라고 르고 있다 이 얼lsquo rsquo lsquo rsquo

이라는 단어가 최근 어사전에 라 항간에 논란이 일고

있다 아닌 게 아니라 얼 은 유행어처럼 인다 생 lsquo rsquo

도 리 래되 않은 것 같고 언제 사라 도 알

없다 게다가 젊은이들 사이에서 주 쓰일 뿐이다 이런

단어를 사전에 는다는 게 하 이 없어 이 도

한다

러나 속단은 이다 차근차근 따져 볼 일이다

선 얼 이 일 적 유행어인 아닌 주의 게 들여다lsquo rsquo

볼 필 가 있다 유행어란 유행에 따라 빠르게 유포되었

다가 단 간 내에 소 되는 단어나 를 가리킨다

얼 은 인터넷을 통해 속히 퍼 말이다 하 만 일lsquo rsquo

적인 유행어처럼 단 간 내에 사라 않았을 뿐 아니라

현재 도 잦은 빈도 사 되고 있고 앞으 도 상당

간 사 될 것으 측된다 한 언 재단의 뉴 검 lsquo rsquo

색 사이트에 따르 얼 은 년 에 처음 나타난lsquo rsquo 2001

이후 꾸 히 사 되고 있다

이 같은 사 빈도는 얼 이 일 적 유행어 는 현lsquo rsquo

저히 다르다는 것을 여 다 장 간의 생존 만으 도

얼 은 이 한 어의 어휘 에 를 자격을 얻었다lsquo rsquo

고 할 있다 더 이 이라는 비 적 정제된 매체에

높은 빈도 쓰이고 있 않은가 사 빈도 측 에서

필통이나 연필과 같은 단어 대등하거나 더 많이 쓰lsquo rsquo lsquo rsquo

다는 것은 결코 가 게 볼 일이 아니다

이제는 사전이 언어 현 을 빠르게 하는 게 덕인

대가 되었다 세계적으 유 한 의 사전들도 경쟁

적으 어를 고 있다

하 만 얼 은 젊은이들이나 쓰는 속어라고 흠을 잡을lsquo rsquo

도 르겠다 얼 이 주 젊은 층에서 많이 쓰 lsquo rsquo

는 속어임에 틀림없다 러나 어사전에 표 적이고 품

위 있는 말만 어야 한다고 생각한다 것은 커다란

해다 당장 아 어사전이나 펼쳐 라 속어는

설과 같은 비어나 죄자들이 쓰는 은어 어

마니 같은 소 의 사람만이 쓰는 말 도 라 있

않은가 사전은 말 치에 일정 빈도 이상 나타나는 말이

라 말이든 다 할 있다

zb38) 가 나 에 대한 다음의 설( ) ( ) 않은 것은

① 가 는 얼짱 사 에 등재 것에( ) ( ) lsquo rsquo

보 고 다

② 사 등재 가는 단어 격에( )

고 고 는 언 들 언어 사 도에 고 다 ( )

③ 가 얼짱 어지만 신 과 같 매( ) ( ) lsquo rsquo

체에 도 사 는 말 는 고 다

④ 가는 얼짱 어 보고 크게 가지 근( ) lsquo rsquo 3

거 들어 뒷 고 다

⑤ 는 얼짱 어 는 다 특 다는( ) lsquo rsquo

근거 에도 크게 가지 근거 가 들어 주 2

뒷 고 다

가 늘 지 상에 살고 는 사 들 억( ) 10

도가 고 그리 지 통 고 는 사 들

그보다 훨 많 억 도는 고 지 20

통 다 그런 지 고 2500

년 학 간고사 대비2013 2 현대고 대비

ECN-0102-2013-001-000076193

그리 간 보는 과 사 에

매우 달 뿐만 니 과 에 도 극

루고 었다 미 운 그런 들

살고 는 동 과 사 들 사고 식에

큰 가 다는 다

고 그리 들 우주 개별 고 독립

사 들 생각 지만 고 들 우

주 연 질 간주 다 같 각

도 들에게는 연 질 었지

만 그리 들에게는 미 들 결 었다

고 과 그리 들 사 같 는

동 과 사 에 도 견 다

인 리학자인 츠 이마이 디드 겐트너는 두

살이 채 안 된 아이들에서 터 성인에 이르 다양한

연 대의 동양인과 서양인을 대상으 다음과 같은 험

을 했다 저 코르크 만든 피라 드 양의 도형을

여 주고 대상의 이름을 닥 라고 알 주었다lsquo (Dax)rsquo

제 닥 는 존재하 않는 것으 험자가 임의lsquo rsquo

만들어 낸 이름이다 런 다음 두 개의 다른 체를

여 주었는데 하나는 피라 드 양이 만 하얀 플라 틱

으 만들었고 다른 하나는 재 는 코르크 만 양이

달랐다 러고 나서 어떤 것이 닥 인 사람들에게 고 lsquo rsquo

르게 했더니 서양인들은 주 같은 양을 하고 있는

체를 선택했고 동양인들은 같은 재 만들어 체를

선택했다 이러한 차이는 성인은 어 두 살 리

아이들에게서도 나타났다 이것은 곧 서양인과 동양인은

서 다른 세상을 고 있다는 것을 의 한다 략 ( )

는 아주 단 하 서도 인상적인 험을 했다

험에는 동서양의 대학생들이 참여했다 는 험 참가자

들에게 컴퓨터 화 을 통해 속 장 을 담은 애니 이션

을 여 주었다 화 의 앙에는 초점의 역할을 하는 커

다란 고 한 마리가 있었고 주위에는 다른 생

들과 초 자갈 거품 등이 함 제 되었다 화 을

두 씩 후 참가자들은 자 이 것을 회상해 라는

를 았다

결과 서양인 대학생들과 동양인 대학생 두 앙

의 초점 역할을 했던 고 를 동일한 정도 언 했으

나 경 소 위 거품 초 다른 생 들 에 ( )

대해서는 동양인 대학생들이 서양인 대학생들 다 60

이상 더 많이 언 했다 뿐만 아니라 동양인 학생들은 서

양인 학생들에 비해 개 적인 고 다 전체적인 계

를 더 언 하는 경향을 다 략 또한 경의 일 ( )

를 화 킨 림을 제 하 을 때 동양인 대학생들은 대

경의 화를 알아챘 만 서양인 대학생들은 경

의 화를 거의 알아차리 했다 략 ( )

따라서 서양인들만을 대상으 연 한 화lsquo

편성 결 은 잘 된 것일 도 있다 각 과정과 인rsquo

과정의 어떤 이 화 편적이고 어떤 이

화에 따라 달라 는 는 앞으 많은 연 를 통하여 논의

되어야 한다

나 어떤 의 에서 리 두는 이 화적이다 리( )

안에는 다른 사람들과 더 친 한 계를 유 하 는 상호

의존성과 다른 사람들 터 독립적인 존재 살아가 는

독립성이 혼재한다 따라서 이 에서 어떤 특성이 더 강

하게 각되는 상황에 놓이느냐에 따라 서 다른 화적

특 을 일 있다 결 리 두는 어떤 경 에는

동양인처럼 행동하고 어떤 경 에는 서양인처럼 행동하는

것이다

zb39) 가 에 대한 다음의 설( ) 않은 것은

① 는 신 주 뒷 닥 실험과lsquo rsquo lsquo

니 실험 근거 시 다rsquo

② 동 들 상 간 공통 보다는 에 식

는 강 다

③ 들 주변 맥 에는 심 경 어 사건

과 사건 사 계에 상 민감 다

④ 는 동 과 틀린 지 고 는 것lsquo rsquo

니 다 고 다 lsquo rsquo

⑤ 가에 우리 사 들 개 시 가 원( )

집 경 말 고 는 것 개 보다는

에 고 는 것에 다

늘 지 상에 살고 는 사 들 억 도가10

고 그리 지 통 고 는 사 들( )知的

그보다 훨 많 억 도는 고 지 20

통 다 그런 지 고 2500

그리 간 보는 과 사 에

매우 달 뿐만 니 과 에 도 극

루고 었다 미 운 그런 들

살고 는 동 과 사 들 사고 식에

큰 가 다는 다

지심리 미 마 드 겐트 는 동

과 상 다 과 같 실험 다

크 만든 미드 도 보여 주고 그

상 닥 고 주었다 그런 다lsquo (Dax)rsquo

개 다 체 보여 주었는 는 미드

지만 틱 만들었고 다 는 재료는

크 지만 달 다 그러고 어 것 닥 lsquo

지 사 들에게 고 게 니 들 주 같rsquo

고 는 체 택 고 동 들 같

재료 만들어진 체 택 다 러 는

심지어 살짜리 들에게 도 타났다 것

곧 과 동 다 상 보고 다는

것 미 다 개별 사 보고 고 동

년 학 간고사 대비2013 2 현대고 대비

ECN-0102-2013-001-000076193

연 질 보고 는 것 다

동 들 주변 상 에 맞 어 동 고

에 다 사 들 태도 동에 보다

많 주 울 다 동 가 미시간

에 에 경험 다 그는 미

식 경 보러 가게 었는 경 체는 매우 재

미 었 주변 들 동에 질 다 그

는 들 계 어 상태 경

다 어 들 에 에 그 시 가 계

가 진 것 다 뒷사 고 지 는 들

동 럼 어 웠다

그는 경험에 어 얻어 동 들lsquo

각도 상 본다 는 가 우고rsquo

검 여 주 단 도 상 실험 실

시 다 그는 실험 가 들에게 컴퓨 통

담 니 보여 주었다

에는 역 는 커다 고 마리가 었

고 주 에는 다 생 들과 갈 거 등

께 시 었다 본 후 가 들

신 본 것 상 보 는 지시 다

그 결과 생들과 동 생

역 고 동 도 언

경 거 다 생 들에 ( )

는 동 생들 생들보다 60

상 많 언 다 뿐만 니 동 생들

생들에 개별 고 보다 체 계

언 는 경 보 다 경 변 시

킨 그림 시 동 생들 경

변 지만 생들 경 변

거 리지 못 다

지 지 들만 상 연 lsquo

보편 결 못 것 도 다 지각 과 과rsquo

지 과 어 보편 고 어

에 달 지는지는 많 연 통 여

어 다

리 드 니 벳 생각 지도 사- ldquo rdquo( 2004)

zb40) 위 에 대한 설 으 가장 적절한 것은

① 동 과 생 식 강 고 다

② 가지 실험 통 쓴 고 다

③ 닥 실험에 사 본질에 동 사

상에 주 다

④ 니 실험에 동 과 에 지

각 도에 가 다

⑤ 쓴 는 보편 연 에 드러 우월 에

에 근 고 다

가 동 들 주변 상 에 맞 어 동 고( )

에 다 사 들 태도 동에 보다 많

주 울 다 동 가 미시간 에

에 경험 다 그는 미식

경 보러 가게 었는 경 체는 매우 재미 었

주변 들 동에 질 다 그 는

들 계 어 상태 경 다

어 들 에 에 그 시 가 계 가

진 것 다 상 살펴lsquo 는 말 들rsquo

에 그는 에 시 어 도 뒷사

생각 곧 다시 곤 것 다 그런 그에게

뒷사 고 지 는 들 동 럼

어 웠다

그는 경험에 어 얻어( ) 동 들lsquo

각도 상 본다 는 가 우고rsquo

검 여 주 단 도 상 실험

실시 다 실험에는 동 생들 여 다

그는 실험 가 들에게 컴퓨 통

담 니 보여 주었다 에는

역 는 커다 고 마리가 었고 주 에는

다 생 들과 갈 거 등 께 시

었다 본 후 가 들 신 본 것

상 보 는 지시 다

다 그 결과 생들과 동 생( )

역 고 동 도 언

경 거 다 생 들 에 ( )

는 동 생들 생들보다 60

상 많 언 다 뿐만 니 동 생들

생들에 개별 고 보다 체 계

언 는 경 보 다 들어 동

생들 상 체 연못 럼 보 어ldquo 같rdquo

체 맥 언 시 었지만

생들 상 어 같 큰 고 가 쪽 움ldquo

직 어 같 역 고rdquo

언 시 다 경 변 시킨 그

림 시 동 생들 경 변

지만 생들 경 변 거

리지 못 다

년 학 간고사 대비2013 2 현대고 대비

ECN-0102-2013-001-000076193

게 볼 동 들 보다는 큰 그( )

림 보 에 사 과 체 맥 연결시 지각

는 경 고 체에 특 떼어 내

어 독립 보는 것 낯 어 다 에

들 사 에 고 주변 맥 에는 심 경

에 사건과 사건 사 계에 상

민감 편 다

마 지 지( ) 들만 상 연

보편 결 못 것 도 다lsquo rsquo 지각 과

과 지 과 어 보편 고 어

에 달 지는지는 많 연 통 여

어 다

리 드 니 벳 생각 지도 사- ldquo rdquo( 2004)

zb41) 의 하는 가~ 다른 것은

① ② ③

④ ⑤

얼마 그 에 동 사고 식과

사고 식 보여 주는 내 다

들 에 는 탕 고 같 게

어 겨 고 미 에 는 그 크 럼 큰 고

어리 주고 원 는 어 도 는

상 고 생각 다는 것 다 러

는 어떻게 생 것 고 과 그리 거슬

러 가 보 그 단 다

고 연 경 체 경 생 에

다 벼 사는 공동 업과 경험 많 연 역

에 고 들 연 웃과

게 지내 고 탁 연 들

들 지 연 럽게 들 다 민들

웃과 동 게 뿐만 니 는 집 과

게 다

동 시 는 생태 경 에 살 결과

들 다 사 들 사 상 에 주

울 게 었고 는 곧 체 상 과 간 사

계 시 는 낳게 었다 신 가

가 는 체에 는 원 는 동시

에 다 사 들 그 사 포 체 맥 에

다 들 간 사 연

계 체 계에 주 울 는 사고 체계

게 었다

그러 그리 연 경 그 었다 산

지 연결 는 지 건 그리고 역

에 다 런 들 업에 다 사 과

동 므 공동체에

다고 다 고 그리 들

들과는 달리 보 내 감 지 들과

지 크게 느 지 못 다 그

견 다 경우 주 쟁 통 결 는 갖

게 었다

신 사 간 계들 루어진 커다

트워크 에 게 당연 사 역시 연

계들 체 식 게 다 어 상

원 도 그 개체가 체 맥 과

계 에 고 다 게 체 맥 에 주

울 다 보 상 복 과 가변 식 게 고

상에 재 는 많 변 들 사 에 재 는 들도

게 다 들 주 태도 보

는 경우가 많다 쟁 결

통 결 보다는 통 결

는 보 다

그러 고 그리 들 개개 사 사 독

에 주 울 다 사 사 체에

어 그들 사 에 재 는 공통 규 주

고 다 상 원 에도 사

체 내 주 고 다 그들

체 여 탕 체

는 주 태도 시 고 특 사 어

주에 는지 여 그 주에 는 규

견 다 에 는 쟁 식 리

같 리 사고 체계가 달 게 었다

리 드 니 벳 생각 지도 사- ldquo rdquo( 2004)

zb42) 위 에서 사 된 설 과 가장 유사한 것은

① 크톱 컴퓨 는 본체 니 마우 루

어 다

② 곡과 시 리 는 지 과 사 루어 다는 공통

지니고 다

③ 경 고 것과는 달리

경 본 연 태 그 주변 경

④ 벽돌 능 에 사계 내내

습도가 지 다

⑤ 잰느 체 체 지닌 재 체가 없

는 재 눌 다

년 학 간고사 대비2013 2 현대고 대비

ECN-0102-2013-001-000076193

zb43) 는 립 앙 도서 이 정의 일 이다lt gt

도서 장과 이 자의 리 의 정의 연결이

적절하 않은 것은

lt gt

제 조 서 유8 ( )

도서 장은 다른 이 자의 안전을 위협하거나 도서 의①

서를 란하게 할 가 있는 자에 대하여는 도서 출입

을 제한할 있다

도서 장은 이 자가 제 조 각 호의 어느 하나의 행위를 하7②

을 때에는 이 을 하게 하거나 도서 출입을 제한할

있다

제 조자 의 대출9 ( )

도서 자 는 다음 각 호의 경 대출할 있다①

상호대차도서 간에 자 를 류하는 것을 말한다 등 다1 ( )

른 도서 과의 협 을 위하여 필 한 경

공 이 공 행 상 필 하는 경2

에 도서 장이 필 하다고 인정하는 경3

대출이 가능한 도서 자 의 위는 도서 장이 정하는②

에 따른다

제 조 상10 ( )

이 자가 도서 자 설을 더럽히거나 찢거나 뜨①

쓰게 하거나 잃어 린 경 에는 상하여야 한다

도서 장은 제 항에 따른 상 을 정하여 게 하여야1②

한다

제 조이 절차 등11 ( )

이 칙에서 정한 것 에 도서 자 설의 이 절차

이 제한 등에 필 한 사항은 도서 장이 정한다

출처 립 앙 도서- (httpwwwnlgokr)

① 는 도 리 다8

② 도 는 리 다9 1

③ 료 지 는 도 리 다9 2

④ 도 료 변상에 리10 1

⑤ 는 에 도 리 다11

3

도 다 각 같다①

공 공 다만 연1

연 간 다

매월 째 째 월2

도 도 리 그 사3

가 다고 는

도 에 미리 게1 3②

시 여 다

4

도 시간 도 여 게시 다

5

도 료 시 는 는 도①

지에 등 후

등 에 사 도②

7

는 다 각 여 는 니 다

도 료 시 상 리1 lsquo rsquo

도 료 시 훼 는2 middot

지 가 닌 곳에 식 거 담3

우는

도 보 등 보 검색열4 middot

그 에 도 질 지 여 도5

여 게시 사 는

8

도 다 거 도①

질 게 우 가 는 에 여는 도

도 가 각 어느7②

에는 지 게 거 도

9

도 료는 다 각 경우 다①

상 도 간에 료 는 것 말1 (

다 등 다 도 과 여 경우)

공 원 공 상 는 경우2

그 에 도 다고 는 경우3

가능 도 료 는 도②

는 에 다

10

년 학 간고사 대비2013 2 현대고 대비

ECN-0102-2013-001-000076193

가 도 료 시 럽 거 거①

못 쓰게 거 어 린 경우에는 변상 여

도 에 변상 여 게시1②

여 다

zb44) 위 에서 도서 장이 게 해야 할 사항에 해당하는

것을 두 쓰

년 학 간고사 대비2013 2 현대고 대비

ECN-0102-2013-001-000076193

립 도 규

1 ( )

규 립 도 립 어린 청 도(

포 다 료 시 열 시 말) (

다 에 사 규 립 도)

편 진 다

2 ( )

규 립 도 도 다 에( lsquo rsquo )

고 는 도 에 도lsquo rsquo 2 2

료 에 여 다 다만 특 료 귀

료 등 료 에 사 립 도

도 다 다( lsquo rsquo )

3 ( )

도 다 각 같다①

공 공 다만 연1

연 간 다

매월 째 째 월2

도 도 리 그 사3

가 다고 는

도 에 미리 게1 3②

시 여 다

시간4 ( )

도 시간 도 여 게시 다

등 등5 ( )

도 료 시 는 는 도①

지에 등 후

등 에 사 도②

사 료6 ( )

도 료 시 에 사 료는 도

7 ( )

는 다 각 여 는 니 다

도 료 시 상 리1 lsquo rsquo

도 료 시 훼 는2 middot

지 가 닌 곳에 식 거 담3

우는

도 보 등 보 검색열4 middot

그 에 도 질 지 여 도5

여 게시 사 는

질 지8 ( )

도 다 거 도①

질 게 우 가 는 에 여는 도

도 가 각 어느7②

에는 지 게 거 도

료9 ( )

도 료는 다 각 경우 다①

상 도 간에 료 는 것 말1 (

다 등 다 도 과 여 경우)

공 원 공 상 는 경우2

그 에 도 다고 는 경우3

가능 도 료 는 도②

는 에 다

변상10 ( )

가 도 료 시 럽 거 거①

못 쓰게 거 어 린 경우에는 변상 여

도 에 변상 여 게시1②

여 다

등 규 에 것 에 도11 ( )

료 시 등에 사

도 다

립 도- (httpwwwnlgokr)

zb45) 도서 장의 리 있는 조항으 적절하 않

은 것은

① ② ③ ④ ⑤

년 학 간고사 대비2013 2 현대고 대비

ECN-0102-2013-001-000076193

1 ( )

사가 공 는lsquo rsquo

과 여 사 원과 리

사 타 사 규

니다

개 보 보7 ( )

사는 보통신망 등 계 는 에lsquo rsquo lsquo rsquo

원 개 보 보 니다 개lsquo rsquo

보 보 사 에 는 사 개lsquo rsquo

보 취 니다 다만 사는 다 lsquo rsquo

사 계 통 공 는 경우 원 lsquo rsquo

등 개 보 당 사에 습니lsquo rsquo

원 리에8 (lsquo rsquo lsquo rsquo lsquo rsquo

)

원 에 리lsquo rsquo lsquo rsquo lsquo rsquo①

원에게 가 도 여 는lsquo rsquo 3

니다

사는 원 가 개 보 우 가lsquo rsquo lsquo rsquo lsquo rsquo②

거 사 경우 는 미 에 어 거 lsquo

사 사 운 우 가 는 경우 당rsquo lsquo rsquo

습니다lsquo rsquo

원 가 도 거lsquo rsquo lsquo rsquo lsquo rsquo 3③

가 사 고 지 경우에는 시 사에lsquo rsquo

통지 고 사 내에 니다lsquo rsquo

경우에 당 원 사에 그 사실3 lsquo rsquo lsquo rsquo④

통지 지 거 통지 도 사 내에 지 lsquo rsquo

생 경우 사는 지지 습니다lsquo rsquo

사10 (lsquo rsquo )

사는 과 지 미lsquo rsquo①

에 는 지 계 고

공 여 다 여 니다lsquo rsquo

사는 원 게lsquo rsquo lsquo rsquo lsquo rsquo②

도 개 보 신 보 포 보 보 시( )

갖 어 개 보 취 공시 고

니다

사는 과 여 원lsquo rsquo lsquo rsquo③

견 만 당 다고 경우에는

리 여 니다 원 견 만 사 lsquo rsquo

에 는 게시 거 우편 등 통 여

원에게 리 과 결과 달 니다lsquo rsquo

원11 (lsquo rsquo )

원 다 여 는 니다lsquo rsquo ①

신청 는 변경 시 허 내 등1

타 보 도2

사가 게시 보 변경3 lsquo rsquo

사가 보 보 컴퓨 그4 lsquo rsquo (

등 등 신 는 게시)

사 타 등 지 재산 에5 lsquo rsquo 3

사 타 상 거 업6 lsquo rsquo 3

는 폭 시지 상 타 공7 middot middot

에 는 보 에 공개 는 게시 는lsquo rsquo

사 동 없 리 사8 lsquo rsquo

타 거 당9

게시15 (lsquo rsquo )

원 내에 게시 는 게시 게재 는lsquo rsquo lsquo rsquo lsquo rsquo

경우 원 사가 게시 복 lsquo rsquo lsquo rsquo lsquo rsquo middot middot

등 태 언 등에 공 는

것 내에 다 원 본 게시 등 lsquo rsquo lsquo rsquo

크 능 등 여 복 는 등 태

는 것 동 것 니다

- (wwwnavercom)

zb46) 위 은 인터넷 포털사이트의 회 가입을 위한 이

약 의 일 이다 이 약 을 만드는 과정에서 생각한

내 으 적절하 않은 것은

개 보 보 가 지에 별 눠①

겠어

원 가 만들게 에②

시 주어 겠어

원들 게재 게시 다 원 크 다③

는 것 지

④ 원 지 는 뿐만 니 사가 지 는

도 께 달 지

리에 가 생 경우 사가⑤

에 다는 도 듯

1 ( )

사가 공 는lsquo rsquo

과 여 사 원과 리

사 타 사 규

년 학 간고사 대비2013 2 현대고 대비

ECN-0102-2013-001-000076193

니다

개 보 보7 ( )

사는 보통신망 등 계 는 에lsquo rsquo lsquo rsquo

원 개 보 보 니다 개lsquo rsquo

보 보 사 에 는 사 개lsquo rsquo

보 취 니다 다만 사는 다 lsquo rsquo

사 계 통 공 는 경우 원 lsquo rsquo

등 개 보 당 사에 습니lsquo rsquo

원 리에8 (lsquo rsquo lsquo rsquo lsquo rsquo

)

원 에 리lsquo rsquo lsquo rsquo lsquo rsquo①

원에게 가 도 여 는lsquo rsquo 3

니다

사는 원 가 개 보 우 가lsquo rsquo lsquo rsquo lsquo rsquo②

거 사 경우 는 미 에 어 거 lsquo

사 사 운 우 가 는 경우 당rsquo lsquo rsquo

습니다lsquo rsquo

원 가 도 거lsquo rsquo lsquo rsquo lsquo rsquo 3③

가 사 고 지 경우에는 시 사에lsquo rsquo

통지 고 사 내에 니다lsquo rsquo

경우에 당 원 사에 그 사실3 lsquo rsquo lsquo rsquo④

통지 지 거 통지 도 사 내에 지 lsquo rsquo

생 경우 사는 지지 습니다lsquo rsquo

원에 통지9 (lsquo rsquo )

사는 특 다 원에게 통지 경우lsquo rsquo lsquo rsquo

공지 게시 통 상 게시 개별 통지에7

갈 습니다

사10 (lsquo rsquo )

사는 과 지 미lsquo rsquo①

에 는 지 계 고

공 여 다 여 니다lsquo rsquo

사는 원 게lsquo rsquo lsquo rsquo lsquo rsquo②

도 개 보 신 보 포 보 보 시( )

갖 어 개 보 취 공시 고

니다

사는 과 여 원lsquo rsquo lsquo rsquo③

견 만 당 다고 경우에는

리 여 니다 원 견 만 사 lsquo rsquo

에 는 게시 거 우편 등 통 여

원에게 리 과 결과 달 니다lsquo rsquo

원11 (lsquo rsquo )

원 다 여 는 니다lsquo rsquo ①

신청 는 변경 시 허 내 등1

타 보 도2

사가 게시 보 변경3 lsquo rsquo

사가 보 보 컴퓨 그4 lsquo rsquo (

등 등 신 는 게시)

사 타 등 지 재산 에5 lsquo rsquo 3

사 타 상 거 업6 lsquo rsquo 3

는 폭 시지 상 타 공7 middot middot

에 는 보 에 공개 는 게시 는lsquo rsquo

사 동 없 리 사8 lsquo rsquo

타 거 당9

원 계 규 내lsquo rsquo lsquo②

여 공지 주 사 사가 통지 는rsquo lsquo rsquo

사 등 여 타 사 업 에 lsquo rsquo

는 여 는 니다

- (wwwnavercom)

zb47) 위 약 의 조항에서 같은 제점을 하lt gt

고 있는 조항은

lt gt

제휴 회사에 회 의 아이디 개인 정 를 전송할 있도

한 조항은 고객에게 당한 조항이다

1 7 8① ② ③

④ 9 ⑤ 10

립 도 규

1 ( )

규 립 도 립 어린 청 도(

포 다 료 시 열 시 말) (

다 에 사 규 립 도)

편 진 다

2 ( )

규 립 도 도 다 에( lsquo rsquo )

고 는 도 에 도lsquo rsquo 2 2

료 에 여 다 다만 특 료 귀

료 등 료 에 사 립 도

도 다 다( lsquo rsquo )

3 ( )

도 다 각 같다①

공 공 다만 연1

연 간 다

년 학 간고사 대비2013 2 현대고 대비

ECN-0102-2013-001-000076193

매월 째 째 월2

도 도 리 그 사3

가 다고 는

도 에 미리 게1 3②

시 여 다

시간4 ( )

도 시간 도 여 게시 다

등 등5 ( )

도 료 시 는 는 도①

지에 등 후

등 에 사 도②

사 료6 ( )

도 료 시 에 사 료는 도

7 ( )

는 다 각 여 는 니 다

도 료 시 상 리1 lsquo rsquo

도 료 시 훼 는2 middot

지 가 닌 곳에 식 거 담3

우는

도 보 등 보 검색열4 middot

그 에 도 질 지 여 도5

여 게시 사 는

질 지8 ( )

도 다 거 도①

질 게 우 가 는 에 여는 도

도 가 각 어느7②

에는 지 게 거 도

료9 ( )

도 료는 다 각 경우 다①

상 도 간에 료 는 것 말1 (

다 등 다 도 과 여 경우)

공 원 공 상 는 경우2

그 에 도 다고 는 경우3

가능 도 료 는 도②

는 에 다

변상10 ( )

가 도 료 시 럽 거 거①

못 쓰게 거 어 린 경우에는 변상 여

도 에 변상 여 게시1②

여 다

등 규 에 것 에 도11 ( )

료 시 등에 사

도 다

립 도- (httpwwwnlgokr)

zb48) 다음 정 리 의 의 으 볼 때 가장

이 적인 것은

도 시간 도 여 게시 다①

등 에 사 도②

가능 도 료 는 도 는③

에 다

④ 도 에 변상 여 게10 1

시 여 다

⑤ 도 가 각 어느7

에는 지 거 도

zb49) 를 참고하여 이 어의 성격을 설 한lt gt

것으 적절하 않은 것은

① 보 에 는 어 시 상 고 어 시lt gt lsquo rsquo

에 보여주고 다

② 진 어 어원에 견 고 다

에는 타 어 들어가는 것 다 lsquo rsquo

③ 에 들어갈 말 각각 고 어 어 신 어~

들 언어는 질 격 강 통 없었다

④ 시 우리 에 가 었지만 지 계

과 달리 들 통 사 달 어 웠

년 학 간고사 대비2013 2 현대고 대비

ECN-0102-2013-001-000076193

⑤ 크 몽골 만주 공통어가 우리 어 같

계열에 다는 에 사 특 짐

가( )

善化公主主隱 공주님

他密只嫁良置古 몰 결 고

薯童房乙 맛

夜矣卯乙抱遣去如 에 몰 고 가다

( )

始汝 會隱日恚見隱扐 만 에 본

恥隱汝衣淸隱笑 맑 웃

고 시 여 공 크다 만 다[ ] ( ) ( ) ( ) ( )始 汝 會扐

내다 에 보다 견( ) ( )恚 見 다( )隱

럽다 맑다 청 웃( ) ( ) ( ) ( )恥 衣 淸 笑

zb50) 위의 나 를 함 고 음에 답하( ) lt gt

보lt gt

( )素那或云金川 白城郡蛇山人也

운 사산

는 고 다 는( )[ ( ) ] (素那 金川 白城

사산 사 다) ( ) 郡 蛇山

삼 사- lsquo rsquo 47

에 제 된 단어 의 표 리를 조건(1) lt gt ( ) lt gt

에 맞게 서 하

건lt gt

lsquo 었고 었다 태rsquo

에 제 된 단어 동일한 표 리에(2) lt gt ( )

의해 적은 것을 나 에서 찾아 조건 에 맞게 서 하( ) lt gt

건lt gt

에 당 는 각각( ) 개 쓸 것2 단

당 는 가 여러 개 어도 개만 쓸 것 각2

개 과 도 쪽에 개만2 2

드시 지 것( )

과 동 원리 것lsquo 고

과 동 원리 것 다rsquo

태 것

가( )

素那(或云金川) 白城郡蛇山人也

소나 또는 천 이라 한다 는 성 사( ) ( ) ( )素那 金川 白城郡〔 〕

산 사람이다 현대어 풀이( ) ( )蛇山

나( )

紫布岩乎希 회

執音乎手母牛放敎遣 자 손 암쇼 노히 고

吾 不喩慙 伊賜等肹 肹 나 안디 리샤

花 折叱肹 可獻乎理音如 고 것거 도림다

다 향찰은 리말을 리 으 적은 표 이었 만 생( )

은 고 대를 넘 하고 끊어 고 말았다 랜 세

동안 갈고 닦아 체계적이었던 향찰 표 이 사라졌

을 인은 크게 두 가 나누어 생각해 볼 있다

하나는 족 사회의 한 선호도에서 찾을 있다 라 때

향찰은 주 족 계 에서 사 했을 것으 인다 한 을

알 하고서는 한자를 활 하여 리말을 리 으 표

하 란 가능하 때 이다 런데 족들은 간이 흐

를 향찰과 같은 리 표 을 익혀 사 하 다는

아 한 을 대 사 하는 쪽을 선호하게 되었다 더 이

고 초에 인재 등 을 위해 과거제도가 행되 서 한 선

호도가 더 높아졌고 결 향찰은 소 되고 말았다

또 다른 가능성은 한 어의 특성에서 찾을 있다

터 한 과 일 세 나라는 한자 화 에 속해 다

당연한 이야 겠 만 표의 자인 한자는 어를 표 하

에 매 적절하다 어의 음절은 성 ( ) ( )聲母 韻母

이 어 고 여 에 성조가 추가되어 최종 소리가 결정된

다 래서 어는 단음절을 하나의 한자 표 하 된

다 에 초성 성 종성의 세 가 소가 하나의 음절

년 학 간고사 대비2013 2 현대고 대비

ECN-0102-2013-001-000076193

을 이 는 한 어는 음절 조가 잡하고 음절의 가 많아

서 한자 차 만으 한 어의 소리를 만족 럽게 표 할

없었다 를 들어 한 어에서는 어 니 같이 음절 lsquo rsquo

이 어 단어가 얼마든 있으나 어는( ) 複數音節

자 하나 나타내 만이다lsquo [m ]rsquo 母 ǔ

한편 일 어의 표 은 핵 적 단어는 한자 적고 토는

가나라는 일 의 자 적는 이다 적인 의 를 나

타내는 은 표의 자인 한자 적고 적 계를 나

타내는 토는 표음 자 적는 셈이니 자세히 살펴

리의 향찰 표 을 쏙 빼닮았음을 알 있다 한 어 같

은 착어이 서도 일 어에만 향찰과 유사한 표 이 살아

남은 것은 일 어의 특 때 이다 일 어는 하나의 자음과

음의 결합으 음절을 이 고 침이 거의 없는 음절 언어

이다 이러한 음절의 특색에다가 토가 달한 착어라는 점

이 향찰과 유사한 표 이 살아남을 있는 비결이었다

하 만 같은 착어라도 다양한 음소 침이 달한 한

어는 향찰 표 하는 데 근 적으 한계가 있었다

zb51) 다 하여 의 행에 대한 탐 한 결과( ) lt gt 2

않은 것은

보lt gt

善花公主主隱 공주니믄 공주님( )

----------------------------------------

-

他密只嫁良置古 그 지 얼어 고 몰 결(

----------------------------------------

-

薯童房乙 맛 맛( )

夜矣卯乙抱遺去如 몰 고 가다 에 몰 고(

가다)

주동 역 동- (薯童謠『 』

에 2 ( )他密只嫁良置古

얼다 시집가다 결 다 말 lsquo rsquo

① 실질 미 지니고 므 타 타lsquo ( )rsquo lsquo [ ]

② 에 실질 미 타내고 지 는lsquo rsquo lsquo [ ]rsquo lsquo [ ]密只 密 只

계 타내는

③ 얼어는 실질 미 포 고 므 가lsquo rsquo lsquo [ ]rsquo嫁

것lsquo [ ]rsquo 良

④ 고 어간 는 실질 미 지니고 므lsquo rsquo lsquo -rsquo

것lsquo [ ]rsquo 置

⑤ 고 어미 고는 계 타내고 므lsquo rsquo lsquo- rsquo

고 것lsquo [ ]rsquo 古

가( )

엉 훈 민middot middot middot middot middot世 宗 御 製 訓 民 正 音

말 미 듕 귁에 달middot middot middot middot middot middot middot middot中 國 文 字

니 런middot middot middot middot middot middot 어린middot middot middot middot百 姓

니 고 도 내 들middot middot middot middot middot middot middot middot middot 시러middot

펴 몯middot 미middot middot 니 내middot middot middot middot middot middot middot middot 爲

어엿middot 겨 새middot middot middot 믈여듧middot middot middot middot字 니middot middot middot

사 마다 니겨 킈 middot middot middot middot middot middot middot middot middot便 安

고 미니middot middot middot middot

본 는 상( ) (象

원리에 만들어진 본) ( )形 ㄱ ㄴ ㅁ ㅅ ㅇ

에 는 가 원리에( )加劃

그리고( )ㅋ ㄷ ㅌ ㅂ ㅍ ㅈ ㅊ ㆆ ㅎ

쓰는 병 원리에 만들어진( )竝書

마지막 체( ) ( )異體ㄲ ㄸ ㅃ ㅆ ㅉ ㆅ

ᅀ 다 상 원리에 ㅇ ㄹ

지 는 삼재 상 본 본( ) ( ) ( 天地人 三才

탕 므림과 림에 ) (初ㅡ ㅣ

재)( ) ( )( )出字 再出字ㅗ ㅏ ㅜ ㅓ ㅛ ㅑ ㅜ ㅕ

병 그리고 들 에 다시( )ㅘ ㅝ ㅣ

( )ㅣ ㅢ ㅚ ㅐ ㅟ ㅔ ㆉ ㅒ ㆌ ㅖ ㅙ ㅞ

zb52) 가 에 대한 설 으 르 않은 것을( ) 두 고르

① 어쓰 규 지키고 다

② 리 고 다

③ 말 미 미 등 어 사 다lsquo rsquo

④ 개 지 다

년 학 간고사 대비2013 2 현대고 대비

ECN-0102-2013-001-000076193

⑤ 어 원 에 가 도 고 다

엉 훈 민世 宗 御 製 訓 民 正 音

말 미 듕귁에 달 니

런 어린 니 고 도middot

내 들 시러 펴 몯 미 니middot

내 어엿 겨 새 믈여듧

사 마다 니겨middot 킈 고

미니

훈민 언 본- lsquo rsquo 5 (1459 )

zb53) 위의 에 대한 현대어 풀이가 르~ 않은 것

① 우리 말 과 달

② 어리 말 고 는 것 어도

③ 신 생각 마 껏 펼 는 사 많다

④ 게 생각 여

⑤ 사 마다 게

zb54) 훈민정음 언해 에는 한 을 창제한 동 가 드러나

있다 훈민정음 창제의 정 과 내 이 잘 연결된 것

① 주 신 말 미 듕귁에 달

② 민 신 내 어 겨

③ 신 뻔 킈 고 미니

④ 실 신 사 마다 니겨

⑤ 귀 신 계 주 는 훈민 신과 거리가

가 엉 훈 민( ) middot middot middot middot middot世 宗 御 製 訓 民 正 音 

말 미 귁에 中 國 달 文 字

니 런 어린 니 百 姓

고 도 내 들 시러 펴 몯

미 니 내 어엿 爲 겨 새

믈여듧 니 사 마다 니 字

겨 킈 고 미니 便 安

훈민 언 본- lsquo ( )rsquo ( ) 5 (1459 )訓民正音 世祖

( )

[ 1 ]

동 룡 샤 마다 복( ) ( ) ( )海東 六龍 天福

시니 고 동( ) ( )古聖 同符 시니

[ 2 ]

매 니 곶 여

미 므 니 그 내 러

가 니

[ 125 ]

우 미리( )千世 샨( )定 에( )漢水北 累仁

누 개 샤 복 업 시니( ) ( ) 開國 卜年

신( )聖神 니 샤도 경 근민 샤 욱( )敬天勤民

드시리 다

님 쇼 산 가( ) ( )洛水 山行

미드니 가

어 가- lsquo ( )rsquo 27龍飛御天歌

다 우리신 니쓰고 다만 만 쓰( )

거 샹 귀쳔 다보게 러 귀

여 쓴 도 신 보 가 고 신 에

말 어 보게 각 에 사 들

고 본 몬 능통 후에

죠 죠 니

드 도 만 공 에 사

드 미 죠 고 고 여 보 죠

보다 얼마가 거시 어신고 니 첫

가 죠 니 죠

민 들 어 신 샹

귀쳔 도보고 어보 가 만 늘

고 폐 에 만쓴 죠 민

도 러보지못 고 보니 그게 엇지

심 니 리 보 가 어 운건 다

니 쳣 말마 지 니 고 그

쓰 에 가 우 지 지

몰 거 본후에 가 어 지

고 그니 쓴편지 쟝 보

년 학 간고사 대비2013 2 현대고 대비

ECN-0102-2013-001-000076193

쓴것보다 듸 보고 그 마 니 쓴 고

어 못

그런고 에 리 과 가

만 쓴 못 민 말만 듯고

고 편 그 못 보니 그사 단

병신 못 다고 그사 식 사

니 만 고 다 과 그사

만 고 다 과 업 사 보다 식 고

죠 도 고 각 과

견 고 실 직 귀쳔 간에 그

고도 다 것 몰 귀죡 보다

사 우리 신 귀쳔 다 업

시 신 보고 과 지 게 랴

시니 샹 귀쳔 간에 우리 신 걸

간 보 새지각과 새 걸 미리

독립신- lsquo (1896)rsquo

zb55) 친 어 나의 제 장( ) 2 매 함축적

의 가 가장 유사한 것은

① 지 눈 내리고 매 득 니 내 여 가

사- lsquo rsquo

② 도 어 리듯 그 게 어 다

주 사- lsquo rsquo

③ 눈 살 다 죽 어 린 과 체 여

눈 새벽 지 도 살 다

눈- lsquo rsquo

④ 삶 근심과 고단 에 돌 거니는 여 거 는

여 리 내린 살가지 에 눈 리 눈 리

택 그 생 에- lsquo rsquo

⑤ 늘 러 고 러

청룡 룡 어 개 루 우

신경림 계- lsquo rsquo

zb56) 친 를 위 가 나 에 나타난A B ( ) ( )

세 어의 특 에 의거하여 세 어 표 하

그 산 고 공 도 맑지만

A

주변에 쓰 리는 어리 사 많다

B

건lt gt

식 가 에 타 어 특징에( ) ( )

거 과 어쓰 는 고 지 말 것

A

B

zb57) 가 의( ) 달 아ㆍ 다 의 ( ) 나셔에서 알 있는

세 어 개화 어의 특 을 비 하여 조건 에lt gt

맞게 서 하

건lt gt

어에 는lsquo 개

어에 는 다 태rsquo

zb58) 은 가 는 다 에 나 는 절lt 1gt ( ) lt 2gt ( )

일 를 췌한 것이다 의 의 가 lt 1gt (1)~(2)

유사한 말을 에서 찾아 쓰lt 2gt

보lt 1gt

런 (1) 어린 니 고百 姓

도 내 들 시러 펴 몯 미

사 마다 (2) 니겨 便 安

킈 고 미니

보lt 2gt

죠 고 고 여 보 죠

보다 얼마가 거시 어신고 니 첫 가

죠 니 죠 민

들 어 신 샹 귀쳔

도보고 어보 가 만 늘 고

폐 에 만쓴 죠 민 도

러보지못 고 보니 그게 엇지 심

니 리

년 학 간고사 대비2013 2 현대고 대비

ECN-0102-2013-001-000076193

lt 1 gt

동 룡 샤 마다 복 시( ) ( ) ( )海東 六龍 天福

고 동 시니( ) ( )古聖 同符

lt 2 gt

(A) 매 니 곶

여 니

미 므 니 그 내

러 가 니

lt125 gt

우 미리 샨 에( ) ( ) ( ) 千世 定 漢水北 累

누 개 샤 복 업 시 니( ) ( ) 仁開國 卜年 聖

신( ) 神 니 샤도 경 근민 샤( ) 敬天勤民

욱 드 시 리 다

님 쇼 산 가 ( ) ( )洛水 山行

미드니 가

- lt gt龍飛御天歌

zb59) 장과 내 상 유사한 성격의 조는125

① 뫼 고 고 고 고

어 그린 많고 많고 고 고

어 러 는 울고 울고 가느니

도 견- lt gt

② 강 에 드니 몸 다

그믈 고 가니

뒷 뫼 엄 언 니( )藥

-

③ 말 없는 청산 태 없는 다

값 없는 청 없는 월

에 병 없는 몸 별 없 늙 리

-

④ 가마귀 골에 가지 마

낸 가마귀 새

청강에 것 시 몸 러 가( ) 淸江

-

⑤ 진 골에( ) 白雪

가 매 는 어느 곳에 었는고

에 갈 곳 몰( ) 夕陽

색-

zb60) 위 에 나타난 세 어의 특 으 적절하 않은

것은

① 룡 어 주격 사에 당 는 가 사( ) lsquo rsquo六龍

고 다

② 샤 어에도 어 주체 쓰 다

는 것 다

③ 매 어 달리 사 택에 어

가 지 지지 고 다

④ 므 원 상 직 어 지 다

⑤ 드시리 다 주체 과 상 께 사

고 다

수고 하셨습니다hearts hearts

년 학 간고사 대비2013 2 현대고 대비

ECN-0102-2013-001-000076193

보닷컴에 공 는 별 보는 고등

들 여 주 는

들 습니다 슷 동 지

가 복 는 것 도가

니 복 여 습 시고 거 시

니다

정답 해설

1) 정답[ ] ④

해설 다른 것은 두 특정 업이나 단 내에서 사[ ]

하는 일종의 은어 사회 언에 해당한다 러나

는 언이 아니라 단과대학을 여서 단대 사lsquo rsquo lsquo rsquo lsquo④

대학을 여서 사대라고 한 말에 해당하 일rsquo lsquo rsquo

사회에서도 널리 쓰이 사회 언이라 할

없다

2) 정답[ ] ⑤

해설 사회 언은 같은 단 내에서 쓰이는 언어이[ ] lsquo rsquo

동일 단끼리는 단결 과 친 감을 형성하는

능을 하 리적 안감이 일어나 않는다

3) 정답[ ] ③

해설 사람이라는 차 적 표현에 대한 대안적 표현이[ ]lsquo rsquo

인 아내 처 등으 볼 있다lsquo rsquo

4) 정답[ ]⑤

해설 남성은 주 격 체를 사 한다[ ]

5) 정답[ ] ⑤

해설 흑인은 검다라는 뜻을 가 고 있을 뿐 인[ ]lsquo rsquo lsquo rsquo lsquo rsquo

다 열등한 뜻을 내포하 않는다

6) 정답 살 색 첫 작품[ ] - -

해설 살색 혹은 킨색은 한 인의 피 색을 뜻[ ] lsquo rsquo lsquo rsquo

하는 것으 인종 차 을 추 고 출 이주민

의 평등 을 침해할 있어 년 표 이2005

살 색으 이름을 꾸었다 처녀작은 처녀라lsquo rsquo lsquo rsquo lsquo rsquo

는 단어가 가 고 있는 곡된 성 인 을 한 것

으 첫 작품정도 꾸어 사 하는 것이 좋다lsquo rsquo

7) 정답[ ] ⑤

해설 호는 아들에게 해체를 사 하고 있다[ ] ① ②

장 을 성하는 청자는 자 의 아 느리 아lsquo

들 세 이다 호는 아 느리에게 해rsquo ③

체를 사 하고 있다 호가 느리 아 에게 ④

사 한 해 체 아들에게 사 한 해체는 두 비lsquo rsquo lsquo rsquo

격 체에 해당한다 호는 자 의 아랫사람인 ⑤

느리에게 아들과 마찬가 해체를 사 하는 것이

상 이 만 임 을 한 느리에게 고마 과 쁨

존 의 표 를 하 위해 자 의 아 에게 말하듯

해 체를 사 하고 있다

8) 정답[ ] ③

9) 정답[ ] ⑤

10) 정답[ ] ①

해설 청자 할아 가 장의 주체 아 다 높을[ ] ( ) ( )

경 에는 압존 에 의해 장의 주체를 높이 않는lsquo rsquo

다 러 아 서가 아닌 아 는으 계 lsquo rsquo lsquo rsquo lsquo

니다 가 아닌 있 니다 표현하는 것이 르rsquo lsquo rsquo

11) 정답 당이 당을 쫒았다 당이[ ]

당에 다

해설[ ]

12) 정답[ ] ⑤

해설 서 다른 높임표현을 통해 청자에 대해 리[ ] ⑤

적 거리감을 나타내는 인 은 이 아니라 현정이

다 가 에서 현정은 에게 해 체를 사 함으 써 ( )

친근감을 드러낸다 나 에서 연 을 게을리하는 역 ( )

도 들 때 에 화가 난 현정이 선생님에게 항의하

는 장 에서는 하 체를 사 하여 리적 거리lsquo rsquo

가 어졌음을 나타내고 있다

13) 정답[ ] ①

해설 는 는 얼 빛이 날과 어찌 다르 고[ ] lsquo rsquo

라는 뜻으 전과 달리 임이 화자를 않고

있음을 알 있다

14) 정답 달리 후 가 있다 이를 통해 경[ ] lt gt

쾌한 음악성을 형성하고 노 젓는 상황을 체적으

형상화하는 역할을 한다

15) 정답[ ] ①

16) 정답[ ] ⑤

해설 다 의 자연은 를 성찰하게 하는 대상[ ] ( )⑤

이자 정의 대상이다 의 자연은 자 의 상황과 ⑤

처 를 드러내는 경으 서의 역할을 하 이

이 없다

17) 정답[ ] ③

해설 는 빈천 을 해결하고자 했으나 강산[ ] lsquo ( )rsquo 貧賤③

과 풍 을 달라는 에 거절하 다고 함으 써 자

연에 대한 애정을 드러내고 있으 는 않는

임에 대한 망을 개에게 전가 켜서 임에 대한 리

을 드러내고 있다

18) 정답[ ] ③

년 학 간고사 대비2013 2 현대고 대비

ECN-0102-2013-001-000076193

19) 정답[ ] ⑤

해설 고상한 음악가의 이름을 리말 꽝 럽[ ]

게 꿈으 써 언어유희를 통해 음을 유 하고 있

다 이는 고상한 척하는 총 를 비꼼으 써 비판적

태도를 드러내는 것이 대상을 꽝 럽게 표현

하여 총 의 허 과 사치를 풍자하고 있다

20) 정답[ ] ⑤

해설 는 작품 속 경에 대한 설 이 드러나는 것이[ ]

서 자의 주 적인 견해가 접적으 드러나는 것이

아니다

21) 정답[ ] ⑤

22) 정답[ ] ②

23) 정답[ ] ④

24) 정답[ ] ①

해설 적강 티프는 주인공의 비 한 출생이나 능[ ] ①

과 이 있는 것으 조정의 능함을 풍자하는lsquo rsquo

것과는 거리가 다

25) 정답 픔 나[ ] ( )

해설 의 음악은 고통 는 사람들을 위 하고 아픔[ ] lsquo rsquo

을 치유해 주는 능을 한다고 할 있다 의 lt gt

픔 도 소 된 이 과 더 어 살아가는 따뜻한 마음lsquo rsquo

을 상 한다

26) 정답[ ] ⑤

해설 에게 선천적으 주어 각 장애라는 역경[ ]

은 의 이라는 가사 연 을 있다lsquo rsquo

27) 정답[ ] ④

해설 는 장 란 선 에게 은 개인적인 인상을[ ]

소녀 장정 등으 표현한 것이다lsquo rsquo

28) 정답[ ] ②

해설 담자가 피 담자의 언어적 표현이나 비언어[ ]②

적 표현 하 독자는 담의 위 나 피

담자의 감정 상태를 알 있다 이를 통해 독자는

담 상황을 더 생생하게 느낄 있고 피 담자

를 더 잘 이해할 있게 된다

29) 정답[ ]③

해설 일상생활과 역도 선 서의 성과에 된 것에서[ ]

역도를 하 서 겪는 어 과 내적 고민으 화제를

전화하 위한 것이다

30) 정답[ ] ①

해설 릿속에 새겨 넣듯 이 억되도 함 세상[ ] ② ③

살이가 힘들고 고생 러 속 하여 자유를 ④

가 없는 고통의 상태를 비유적으 이르는 말

적의 침입을 막 위해 쌓은 축 켜야 할⑤

대상을 비유적으 이르는 말이다

31) 정답[ ] ④

해설 이 의 종류는 전 으 인 사건 경[ ] lsquo

비평을 성 소 삼는다rsquo

32) 정답[ ] ④

해설 근은 삼대독자 태어났음을 에서 확인할[ ]

있다 형제들과의 담은 이뤄 가 없다

33) 정답[ ] ⑤

해설 근은 가난에도 하고 화가를 꿈꾸었다[ ] (3

단 또한 다른 화가 망생들은 정 육을)

위해 상 학 학 해 유학 에 랐 만

근은 다른 을 찾아야 했다 단 세에(5 ) 18

근은 조선 전람회에 입선하 다 단 의(6 )

만종은 인간과 자연이 엮어 가는 경건한 조화 을lsquo rsquo

나타낸다

34) 정답[ ] ①

해설 근이 속에서도 창작활동을 추 않고[ ]

하는 닭은 은 세상과 타협할 르는

근이 세상의 이해를 하 위한 가장 떳떳한 단

이 때 이다

35) 정답[ ] ⑤

해설 전 은 서 자의 주 적인 평이 리는 것이[ ]

만 위 제 은 인 이 살았던 대 사회적 경

을 통해 객 적인 인 의 을 제 하고 있다

36) 정답[ ] ⑤

해설 전 은 인 사건 경 비평이라는[ ] lsquo rsquo⑤

성 이 어져 있다

37) 정답[ ] ①

해설 이 은 동양인과 서양인의 사고 에 차이가[ ]

있다는 것을 대조를 통해 설 하고 있다 또 쓴이

의 제자가 축 경 를 러 가서 경험한 일화를

통해 동양인이 서양인에 비해 주 상황에 더 많은

주의를 인다는 주장을 뒷 침하고 있다

38) 정답[ ] ④

39) 정답[ ] ②

40) 정답[ ] ②

41) 정답[ ] ④

42) 정답[ ] ③

43) 정답[ ] ④

44) 정답 도서 의 휴 일 도서 의 이 간 도서의[ ]

해설 도서 장은 임의 정한 휴 일과 도서 이[ ]

간 도서의 상 등을 게 할 의 가 있다

년 학 간고사 대비2013 2 현대고 대비

ECN-0102-2013-001-000076193

45) 정답[ ] ①

해설 제 조의 정 휴 일 의 휴 일의 사전 게[ ] 3

는 도서 장의 의 조항에 속한다

46) 정답[ ] ①

해설 개인 정 호 의 를 제 하 했 만 항[ ]

나눠서 제 하 않고 대 나열하고 있다

47) 정답[ ] ②

해설 제 조의 내 을 회사는 다른 회사 협[ ] 7 lsquo

계약을 통해 서비 를 제공하는 경 회 의 아이디

등 개인 정 를 해당 회사에 전송할 있다는 내rsquo

이 있으 의 제점을 제 할 있다②

48) 정답[ ] ④

해설 는 도서 장의 의 에 해당하고 나 는 도[ ] ④

서 장의 리에 해당한다

49) 정답[ ] ③

50) 정답 은 음독으 적었고 은 훈독으 적었[ ] (1)

다 과 동일한 표 리 적은 것은 이고 (2) ce

과 동일한 표 리 적은 것은 이다ab

51) 정답[ ] ③

52) 정답[ ] ①②

53) 정답[ ] ③

54) 정답[ ] ③

55) 정답[ ] ①

56) 정답 른 죠코 어린 노 하니라[ ] A B

57) 정답 세 어에서는 활 형이 칙적으[ ] lsquo rsquoㄹㅇ

나타났 만 개화 어에서는 활 형이 쓰 다 lsquo rsquo ㄹㄴ

58) 정답 호 가 흔[ ] (1) (2)

59) 정답[ ] ④

60) 정답[ ] ③

Page 24: 현대고대비 국어 - chamsoriedu.com 「콘텐츠산업진흥 법」외 에도 저작권 의하여 ... 다른주체에게어떤동작을하도록만드는것을나타내는

년 학 간고사 대비2013 2 현대고 대비

ECN-0102-2013-001-000076193

가 우리가 말 고 쓰는 든 단어가 사 에 는( )

것 니다 사 격에 가 는 지만

어 사 과 같 특별 는 사 니lsquo rsquo

단어 격 보 단어가 사 에

등재 어 다 리 리 사 는 단어 도 그

것 시 사 는 어 고 사 에

격 보 것 니다

러 얼 은 사전에 를 있는가 이에 대한 답lsquo rsquo

은 얼 이 유행어인가 아닌가에 따라 갈라 다 이 단어lsquo rsquo

는 년 어 자 에 랐고 쓰이고 있으2002 lsquo rsquo

유행어라고 하 에는 생 이 다 런데 계속

을 유 하 서 사전에 등재될 자격을 획득할 것인가 이

에 대한 답을 내리 는 히 어 다

여 서 가 를 고 해 볼 있다 첫 는 이 단어

를 써야 할 필 가 속적으 있는가 하는 점이다

상주의 열풍에 휩 인 사회 위 에 편 해서 퍼 말

이 얼 인데 과연 런 위 가 속될 것인가 이에lsquo rsquo

대해 필자의 생각은 정적이다 사회 위 가 뀌

런 말을 쓸 일이 없어 것이다

다음은 단어의 성이다 단어의 성이 사회적으 거

감이 없으 계속 사 될 가능성이 높다 런 에서

얼 은 좋은 조건이 아니다 익히 알 졌듯이 이lsquo rsquo

말은 얼 과 청소년층에서 속어 사 하는 이 결합lsquo rsquo lsquo rsquo

된 말이다 얼 에서 얼 을 리하는 조어 도 lsquo rsquo lsquo -rsquo

어에서는 매 낯선 이다 이것만으 도 거 감을 갖

는 사람들이 있다 더 나 속어 결합한 말이다 얼 lsquo rsquo

이 널리 퍼졌다 해도 은 여전히 청소년층의 속어lsquo rsquo

남아 있다 속어는 자연 럽게 아 자리에서나 쓰 에는

담 러 말이다 러한 담을 하고 사

역을 넓혀 가는 속어도 없 는 않다 특히 얼 은 lsquo rsquo

에도 종종 등장한다 만큼 거 감이 많이 희석되었다

고 할 있다 러나 일상의 자연 러 대화에서도 거

리낌 없이 등장하는가 게 는 되 않았다고 생

각한다

얼 이 유사어인 쌈 등을 만들어 내고lsquo rsquo lsquo rsquo

있으니 살아남을 있을 것이라고 는 견해도 있을 것

이다 러나 간이 나 서 유사어를 포함하여 든

말이 사라 사 는 많다 유사어가 많다는 것이 생 을

유 할 있는 절대적인 조건은 아니다

나 언젠가 터 사람들은 어느 단에서 얼 이 가장( )

쁜 사람을 가리켜 얼 이라고 르고 있다 이 얼lsquo rsquo lsquo rsquo

이라는 단어가 최근 어사전에 라 항간에 논란이 일고

있다 아닌 게 아니라 얼 은 유행어처럼 인다 생 lsquo rsquo

도 리 래되 않은 것 같고 언제 사라 도 알

없다 게다가 젊은이들 사이에서 주 쓰일 뿐이다 이런

단어를 사전에 는다는 게 하 이 없어 이 도

한다

러나 속단은 이다 차근차근 따져 볼 일이다

선 얼 이 일 적 유행어인 아닌 주의 게 들여다lsquo rsquo

볼 필 가 있다 유행어란 유행에 따라 빠르게 유포되었

다가 단 간 내에 소 되는 단어나 를 가리킨다

얼 은 인터넷을 통해 속히 퍼 말이다 하 만 일lsquo rsquo

적인 유행어처럼 단 간 내에 사라 않았을 뿐 아니라

현재 도 잦은 빈도 사 되고 있고 앞으 도 상당

간 사 될 것으 측된다 한 언 재단의 뉴 검 lsquo rsquo

색 사이트에 따르 얼 은 년 에 처음 나타난lsquo rsquo 2001

이후 꾸 히 사 되고 있다

이 같은 사 빈도는 얼 이 일 적 유행어 는 현lsquo rsquo

저히 다르다는 것을 여 다 장 간의 생존 만으 도

얼 은 이 한 어의 어휘 에 를 자격을 얻었다lsquo rsquo

고 할 있다 더 이 이라는 비 적 정제된 매체에

높은 빈도 쓰이고 있 않은가 사 빈도 측 에서

필통이나 연필과 같은 단어 대등하거나 더 많이 쓰lsquo rsquo lsquo rsquo

다는 것은 결코 가 게 볼 일이 아니다

이제는 사전이 언어 현 을 빠르게 하는 게 덕인

대가 되었다 세계적으 유 한 의 사전들도 경쟁

적으 어를 고 있다

하 만 얼 은 젊은이들이나 쓰는 속어라고 흠을 잡을lsquo rsquo

도 르겠다 얼 이 주 젊은 층에서 많이 쓰 lsquo rsquo

는 속어임에 틀림없다 러나 어사전에 표 적이고 품

위 있는 말만 어야 한다고 생각한다 것은 커다란

해다 당장 아 어사전이나 펼쳐 라 속어는

설과 같은 비어나 죄자들이 쓰는 은어 어

마니 같은 소 의 사람만이 쓰는 말 도 라 있

않은가 사전은 말 치에 일정 빈도 이상 나타나는 말이

라 말이든 다 할 있다

zb38) 가 나 에 대한 다음의 설( ) ( ) 않은 것은

① 가 는 얼짱 사 에 등재 것에( ) ( ) lsquo rsquo

보 고 다

② 사 등재 가는 단어 격에( )

고 고 는 언 들 언어 사 도에 고 다 ( )

③ 가 얼짱 어지만 신 과 같 매( ) ( ) lsquo rsquo

체에 도 사 는 말 는 고 다

④ 가는 얼짱 어 보고 크게 가지 근( ) lsquo rsquo 3

거 들어 뒷 고 다

⑤ 는 얼짱 어 는 다 특 다는( ) lsquo rsquo

근거 에도 크게 가지 근거 가 들어 주 2

뒷 고 다

가 늘 지 상에 살고 는 사 들 억( ) 10

도가 고 그리 지 통 고 는 사 들

그보다 훨 많 억 도는 고 지 20

통 다 그런 지 고 2500

년 학 간고사 대비2013 2 현대고 대비

ECN-0102-2013-001-000076193

그리 간 보는 과 사 에

매우 달 뿐만 니 과 에 도 극

루고 었다 미 운 그런 들

살고 는 동 과 사 들 사고 식에

큰 가 다는 다

고 그리 들 우주 개별 고 독립

사 들 생각 지만 고 들 우

주 연 질 간주 다 같 각

도 들에게는 연 질 었지

만 그리 들에게는 미 들 결 었다

고 과 그리 들 사 같 는

동 과 사 에 도 견 다

인 리학자인 츠 이마이 디드 겐트너는 두

살이 채 안 된 아이들에서 터 성인에 이르 다양한

연 대의 동양인과 서양인을 대상으 다음과 같은 험

을 했다 저 코르크 만든 피라 드 양의 도형을

여 주고 대상의 이름을 닥 라고 알 주었다lsquo (Dax)rsquo

제 닥 는 존재하 않는 것으 험자가 임의lsquo rsquo

만들어 낸 이름이다 런 다음 두 개의 다른 체를

여 주었는데 하나는 피라 드 양이 만 하얀 플라 틱

으 만들었고 다른 하나는 재 는 코르크 만 양이

달랐다 러고 나서 어떤 것이 닥 인 사람들에게 고 lsquo rsquo

르게 했더니 서양인들은 주 같은 양을 하고 있는

체를 선택했고 동양인들은 같은 재 만들어 체를

선택했다 이러한 차이는 성인은 어 두 살 리

아이들에게서도 나타났다 이것은 곧 서양인과 동양인은

서 다른 세상을 고 있다는 것을 의 한다 략 ( )

는 아주 단 하 서도 인상적인 험을 했다

험에는 동서양의 대학생들이 참여했다 는 험 참가자

들에게 컴퓨터 화 을 통해 속 장 을 담은 애니 이션

을 여 주었다 화 의 앙에는 초점의 역할을 하는 커

다란 고 한 마리가 있었고 주위에는 다른 생

들과 초 자갈 거품 등이 함 제 되었다 화 을

두 씩 후 참가자들은 자 이 것을 회상해 라는

를 았다

결과 서양인 대학생들과 동양인 대학생 두 앙

의 초점 역할을 했던 고 를 동일한 정도 언 했으

나 경 소 위 거품 초 다른 생 들 에 ( )

대해서는 동양인 대학생들이 서양인 대학생들 다 60

이상 더 많이 언 했다 뿐만 아니라 동양인 학생들은 서

양인 학생들에 비해 개 적인 고 다 전체적인 계

를 더 언 하는 경향을 다 략 또한 경의 일 ( )

를 화 킨 림을 제 하 을 때 동양인 대학생들은 대

경의 화를 알아챘 만 서양인 대학생들은 경

의 화를 거의 알아차리 했다 략 ( )

따라서 서양인들만을 대상으 연 한 화lsquo

편성 결 은 잘 된 것일 도 있다 각 과정과 인rsquo

과정의 어떤 이 화 편적이고 어떤 이

화에 따라 달라 는 는 앞으 많은 연 를 통하여 논의

되어야 한다

나 어떤 의 에서 리 두는 이 화적이다 리( )

안에는 다른 사람들과 더 친 한 계를 유 하 는 상호

의존성과 다른 사람들 터 독립적인 존재 살아가 는

독립성이 혼재한다 따라서 이 에서 어떤 특성이 더 강

하게 각되는 상황에 놓이느냐에 따라 서 다른 화적

특 을 일 있다 결 리 두는 어떤 경 에는

동양인처럼 행동하고 어떤 경 에는 서양인처럼 행동하는

것이다

zb39) 가 에 대한 다음의 설( ) 않은 것은

① 는 신 주 뒷 닥 실험과lsquo rsquo lsquo

니 실험 근거 시 다rsquo

② 동 들 상 간 공통 보다는 에 식

는 강 다

③ 들 주변 맥 에는 심 경 어 사건

과 사건 사 계에 상 민감 다

④ 는 동 과 틀린 지 고 는 것lsquo rsquo

니 다 고 다 lsquo rsquo

⑤ 가에 우리 사 들 개 시 가 원( )

집 경 말 고 는 것 개 보다는

에 고 는 것에 다

늘 지 상에 살고 는 사 들 억 도가10

고 그리 지 통 고 는 사 들( )知的

그보다 훨 많 억 도는 고 지 20

통 다 그런 지 고 2500

그리 간 보는 과 사 에

매우 달 뿐만 니 과 에 도 극

루고 었다 미 운 그런 들

살고 는 동 과 사 들 사고 식에

큰 가 다는 다

지심리 미 마 드 겐트 는 동

과 상 다 과 같 실험 다

크 만든 미드 도 보여 주고 그

상 닥 고 주었다 그런 다lsquo (Dax)rsquo

개 다 체 보여 주었는 는 미드

지만 틱 만들었고 다 는 재료는

크 지만 달 다 그러고 어 것 닥 lsquo

지 사 들에게 고 게 니 들 주 같rsquo

고 는 체 택 고 동 들 같

재료 만들어진 체 택 다 러 는

심지어 살짜리 들에게 도 타났다 것

곧 과 동 다 상 보고 다는

것 미 다 개별 사 보고 고 동

년 학 간고사 대비2013 2 현대고 대비

ECN-0102-2013-001-000076193

연 질 보고 는 것 다

동 들 주변 상 에 맞 어 동 고

에 다 사 들 태도 동에 보다

많 주 울 다 동 가 미시간

에 에 경험 다 그는 미

식 경 보러 가게 었는 경 체는 매우 재

미 었 주변 들 동에 질 다 그

는 들 계 어 상태 경

다 어 들 에 에 그 시 가 계

가 진 것 다 뒷사 고 지 는 들

동 럼 어 웠다

그는 경험에 어 얻어 동 들lsquo

각도 상 본다 는 가 우고rsquo

검 여 주 단 도 상 실험 실

시 다 그는 실험 가 들에게 컴퓨 통

담 니 보여 주었다

에는 역 는 커다 고 마리가 었

고 주 에는 다 생 들과 갈 거 등

께 시 었다 본 후 가 들

신 본 것 상 보 는 지시 다

그 결과 생들과 동 생

역 고 동 도 언

경 거 다 생 들에 ( )

는 동 생들 생들보다 60

상 많 언 다 뿐만 니 동 생들

생들에 개별 고 보다 체 계

언 는 경 보 다 경 변 시

킨 그림 시 동 생들 경

변 지만 생들 경 변

거 리지 못 다

지 지 들만 상 연 lsquo

보편 결 못 것 도 다 지각 과 과rsquo

지 과 어 보편 고 어

에 달 지는지는 많 연 통 여

어 다

리 드 니 벳 생각 지도 사- ldquo rdquo( 2004)

zb40) 위 에 대한 설 으 가장 적절한 것은

① 동 과 생 식 강 고 다

② 가지 실험 통 쓴 고 다

③ 닥 실험에 사 본질에 동 사

상에 주 다

④ 니 실험에 동 과 에 지

각 도에 가 다

⑤ 쓴 는 보편 연 에 드러 우월 에

에 근 고 다

가 동 들 주변 상 에 맞 어 동 고( )

에 다 사 들 태도 동에 보다 많

주 울 다 동 가 미시간 에

에 경험 다 그는 미식

경 보러 가게 었는 경 체는 매우 재미 었

주변 들 동에 질 다 그 는

들 계 어 상태 경 다

어 들 에 에 그 시 가 계 가

진 것 다 상 살펴lsquo 는 말 들rsquo

에 그는 에 시 어 도 뒷사

생각 곧 다시 곤 것 다 그런 그에게

뒷사 고 지 는 들 동 럼

어 웠다

그는 경험에 어 얻어( ) 동 들lsquo

각도 상 본다 는 가 우고rsquo

검 여 주 단 도 상 실험

실시 다 실험에는 동 생들 여 다

그는 실험 가 들에게 컴퓨 통

담 니 보여 주었다 에는

역 는 커다 고 마리가 었고 주 에는

다 생 들과 갈 거 등 께 시

었다 본 후 가 들 신 본 것

상 보 는 지시 다

다 그 결과 생들과 동 생( )

역 고 동 도 언

경 거 다 생 들 에 ( )

는 동 생들 생들보다 60

상 많 언 다 뿐만 니 동 생들

생들에 개별 고 보다 체 계

언 는 경 보 다 들어 동

생들 상 체 연못 럼 보 어ldquo 같rdquo

체 맥 언 시 었지만

생들 상 어 같 큰 고 가 쪽 움ldquo

직 어 같 역 고rdquo

언 시 다 경 변 시킨 그

림 시 동 생들 경 변

지만 생들 경 변 거

리지 못 다

년 학 간고사 대비2013 2 현대고 대비

ECN-0102-2013-001-000076193

게 볼 동 들 보다는 큰 그( )

림 보 에 사 과 체 맥 연결시 지각

는 경 고 체에 특 떼어 내

어 독립 보는 것 낯 어 다 에

들 사 에 고 주변 맥 에는 심 경

에 사건과 사건 사 계에 상

민감 편 다

마 지 지( ) 들만 상 연

보편 결 못 것 도 다lsquo rsquo 지각 과

과 지 과 어 보편 고 어

에 달 지는지는 많 연 통 여

어 다

리 드 니 벳 생각 지도 사- ldquo rdquo( 2004)

zb41) 의 하는 가~ 다른 것은

① ② ③

④ ⑤

얼마 그 에 동 사고 식과

사고 식 보여 주는 내 다

들 에 는 탕 고 같 게

어 겨 고 미 에 는 그 크 럼 큰 고

어리 주고 원 는 어 도 는

상 고 생각 다는 것 다 러

는 어떻게 생 것 고 과 그리 거슬

러 가 보 그 단 다

고 연 경 체 경 생 에

다 벼 사는 공동 업과 경험 많 연 역

에 고 들 연 웃과

게 지내 고 탁 연 들

들 지 연 럽게 들 다 민들

웃과 동 게 뿐만 니 는 집 과

게 다

동 시 는 생태 경 에 살 결과

들 다 사 들 사 상 에 주

울 게 었고 는 곧 체 상 과 간 사

계 시 는 낳게 었다 신 가

가 는 체에 는 원 는 동시

에 다 사 들 그 사 포 체 맥 에

다 들 간 사 연

계 체 계에 주 울 는 사고 체계

게 었다

그러 그리 연 경 그 었다 산

지 연결 는 지 건 그리고 역

에 다 런 들 업에 다 사 과

동 므 공동체에

다고 다 고 그리 들

들과는 달리 보 내 감 지 들과

지 크게 느 지 못 다 그

견 다 경우 주 쟁 통 결 는 갖

게 었다

신 사 간 계들 루어진 커다

트워크 에 게 당연 사 역시 연

계들 체 식 게 다 어 상

원 도 그 개체가 체 맥 과

계 에 고 다 게 체 맥 에 주

울 다 보 상 복 과 가변 식 게 고

상에 재 는 많 변 들 사 에 재 는 들도

게 다 들 주 태도 보

는 경우가 많다 쟁 결

통 결 보다는 통 결

는 보 다

그러 고 그리 들 개개 사 사 독

에 주 울 다 사 사 체에

어 그들 사 에 재 는 공통 규 주

고 다 상 원 에도 사

체 내 주 고 다 그들

체 여 탕 체

는 주 태도 시 고 특 사 어

주에 는지 여 그 주에 는 규

견 다 에 는 쟁 식 리

같 리 사고 체계가 달 게 었다

리 드 니 벳 생각 지도 사- ldquo rdquo( 2004)

zb42) 위 에서 사 된 설 과 가장 유사한 것은

① 크톱 컴퓨 는 본체 니 마우 루

어 다

② 곡과 시 리 는 지 과 사 루어 다는 공통

지니고 다

③ 경 고 것과는 달리

경 본 연 태 그 주변 경

④ 벽돌 능 에 사계 내내

습도가 지 다

⑤ 잰느 체 체 지닌 재 체가 없

는 재 눌 다

년 학 간고사 대비2013 2 현대고 대비

ECN-0102-2013-001-000076193

zb43) 는 립 앙 도서 이 정의 일 이다lt gt

도서 장과 이 자의 리 의 정의 연결이

적절하 않은 것은

lt gt

제 조 서 유8 ( )

도서 장은 다른 이 자의 안전을 위협하거나 도서 의①

서를 란하게 할 가 있는 자에 대하여는 도서 출입

을 제한할 있다

도서 장은 이 자가 제 조 각 호의 어느 하나의 행위를 하7②

을 때에는 이 을 하게 하거나 도서 출입을 제한할

있다

제 조자 의 대출9 ( )

도서 자 는 다음 각 호의 경 대출할 있다①

상호대차도서 간에 자 를 류하는 것을 말한다 등 다1 ( )

른 도서 과의 협 을 위하여 필 한 경

공 이 공 행 상 필 하는 경2

에 도서 장이 필 하다고 인정하는 경3

대출이 가능한 도서 자 의 위는 도서 장이 정하는②

에 따른다

제 조 상10 ( )

이 자가 도서 자 설을 더럽히거나 찢거나 뜨①

쓰게 하거나 잃어 린 경 에는 상하여야 한다

도서 장은 제 항에 따른 상 을 정하여 게 하여야1②

한다

제 조이 절차 등11 ( )

이 칙에서 정한 것 에 도서 자 설의 이 절차

이 제한 등에 필 한 사항은 도서 장이 정한다

출처 립 앙 도서- (httpwwwnlgokr)

① 는 도 리 다8

② 도 는 리 다9 1

③ 료 지 는 도 리 다9 2

④ 도 료 변상에 리10 1

⑤ 는 에 도 리 다11

3

도 다 각 같다①

공 공 다만 연1

연 간 다

매월 째 째 월2

도 도 리 그 사3

가 다고 는

도 에 미리 게1 3②

시 여 다

4

도 시간 도 여 게시 다

5

도 료 시 는 는 도①

지에 등 후

등 에 사 도②

7

는 다 각 여 는 니 다

도 료 시 상 리1 lsquo rsquo

도 료 시 훼 는2 middot

지 가 닌 곳에 식 거 담3

우는

도 보 등 보 검색열4 middot

그 에 도 질 지 여 도5

여 게시 사 는

8

도 다 거 도①

질 게 우 가 는 에 여는 도

도 가 각 어느7②

에는 지 게 거 도

9

도 료는 다 각 경우 다①

상 도 간에 료 는 것 말1 (

다 등 다 도 과 여 경우)

공 원 공 상 는 경우2

그 에 도 다고 는 경우3

가능 도 료 는 도②

는 에 다

10

년 학 간고사 대비2013 2 현대고 대비

ECN-0102-2013-001-000076193

가 도 료 시 럽 거 거①

못 쓰게 거 어 린 경우에는 변상 여

도 에 변상 여 게시1②

여 다

zb44) 위 에서 도서 장이 게 해야 할 사항에 해당하는

것을 두 쓰

년 학 간고사 대비2013 2 현대고 대비

ECN-0102-2013-001-000076193

립 도 규

1 ( )

규 립 도 립 어린 청 도(

포 다 료 시 열 시 말) (

다 에 사 규 립 도)

편 진 다

2 ( )

규 립 도 도 다 에( lsquo rsquo )

고 는 도 에 도lsquo rsquo 2 2

료 에 여 다 다만 특 료 귀

료 등 료 에 사 립 도

도 다 다( lsquo rsquo )

3 ( )

도 다 각 같다①

공 공 다만 연1

연 간 다

매월 째 째 월2

도 도 리 그 사3

가 다고 는

도 에 미리 게1 3②

시 여 다

시간4 ( )

도 시간 도 여 게시 다

등 등5 ( )

도 료 시 는 는 도①

지에 등 후

등 에 사 도②

사 료6 ( )

도 료 시 에 사 료는 도

7 ( )

는 다 각 여 는 니 다

도 료 시 상 리1 lsquo rsquo

도 료 시 훼 는2 middot

지 가 닌 곳에 식 거 담3

우는

도 보 등 보 검색열4 middot

그 에 도 질 지 여 도5

여 게시 사 는

질 지8 ( )

도 다 거 도①

질 게 우 가 는 에 여는 도

도 가 각 어느7②

에는 지 게 거 도

료9 ( )

도 료는 다 각 경우 다①

상 도 간에 료 는 것 말1 (

다 등 다 도 과 여 경우)

공 원 공 상 는 경우2

그 에 도 다고 는 경우3

가능 도 료 는 도②

는 에 다

변상10 ( )

가 도 료 시 럽 거 거①

못 쓰게 거 어 린 경우에는 변상 여

도 에 변상 여 게시1②

여 다

등 규 에 것 에 도11 ( )

료 시 등에 사

도 다

립 도- (httpwwwnlgokr)

zb45) 도서 장의 리 있는 조항으 적절하 않

은 것은

① ② ③ ④ ⑤

년 학 간고사 대비2013 2 현대고 대비

ECN-0102-2013-001-000076193

1 ( )

사가 공 는lsquo rsquo

과 여 사 원과 리

사 타 사 규

니다

개 보 보7 ( )

사는 보통신망 등 계 는 에lsquo rsquo lsquo rsquo

원 개 보 보 니다 개lsquo rsquo

보 보 사 에 는 사 개lsquo rsquo

보 취 니다 다만 사는 다 lsquo rsquo

사 계 통 공 는 경우 원 lsquo rsquo

등 개 보 당 사에 습니lsquo rsquo

원 리에8 (lsquo rsquo lsquo rsquo lsquo rsquo

)

원 에 리lsquo rsquo lsquo rsquo lsquo rsquo①

원에게 가 도 여 는lsquo rsquo 3

니다

사는 원 가 개 보 우 가lsquo rsquo lsquo rsquo lsquo rsquo②

거 사 경우 는 미 에 어 거 lsquo

사 사 운 우 가 는 경우 당rsquo lsquo rsquo

습니다lsquo rsquo

원 가 도 거lsquo rsquo lsquo rsquo lsquo rsquo 3③

가 사 고 지 경우에는 시 사에lsquo rsquo

통지 고 사 내에 니다lsquo rsquo

경우에 당 원 사에 그 사실3 lsquo rsquo lsquo rsquo④

통지 지 거 통지 도 사 내에 지 lsquo rsquo

생 경우 사는 지지 습니다lsquo rsquo

사10 (lsquo rsquo )

사는 과 지 미lsquo rsquo①

에 는 지 계 고

공 여 다 여 니다lsquo rsquo

사는 원 게lsquo rsquo lsquo rsquo lsquo rsquo②

도 개 보 신 보 포 보 보 시( )

갖 어 개 보 취 공시 고

니다

사는 과 여 원lsquo rsquo lsquo rsquo③

견 만 당 다고 경우에는

리 여 니다 원 견 만 사 lsquo rsquo

에 는 게시 거 우편 등 통 여

원에게 리 과 결과 달 니다lsquo rsquo

원11 (lsquo rsquo )

원 다 여 는 니다lsquo rsquo ①

신청 는 변경 시 허 내 등1

타 보 도2

사가 게시 보 변경3 lsquo rsquo

사가 보 보 컴퓨 그4 lsquo rsquo (

등 등 신 는 게시)

사 타 등 지 재산 에5 lsquo rsquo 3

사 타 상 거 업6 lsquo rsquo 3

는 폭 시지 상 타 공7 middot middot

에 는 보 에 공개 는 게시 는lsquo rsquo

사 동 없 리 사8 lsquo rsquo

타 거 당9

게시15 (lsquo rsquo )

원 내에 게시 는 게시 게재 는lsquo rsquo lsquo rsquo lsquo rsquo

경우 원 사가 게시 복 lsquo rsquo lsquo rsquo lsquo rsquo middot middot

등 태 언 등에 공 는

것 내에 다 원 본 게시 등 lsquo rsquo lsquo rsquo

크 능 등 여 복 는 등 태

는 것 동 것 니다

- (wwwnavercom)

zb46) 위 은 인터넷 포털사이트의 회 가입을 위한 이

약 의 일 이다 이 약 을 만드는 과정에서 생각한

내 으 적절하 않은 것은

개 보 보 가 지에 별 눠①

겠어

원 가 만들게 에②

시 주어 겠어

원들 게재 게시 다 원 크 다③

는 것 지

④ 원 지 는 뿐만 니 사가 지 는

도 께 달 지

리에 가 생 경우 사가⑤

에 다는 도 듯

1 ( )

사가 공 는lsquo rsquo

과 여 사 원과 리

사 타 사 규

년 학 간고사 대비2013 2 현대고 대비

ECN-0102-2013-001-000076193

니다

개 보 보7 ( )

사는 보통신망 등 계 는 에lsquo rsquo lsquo rsquo

원 개 보 보 니다 개lsquo rsquo

보 보 사 에 는 사 개lsquo rsquo

보 취 니다 다만 사는 다 lsquo rsquo

사 계 통 공 는 경우 원 lsquo rsquo

등 개 보 당 사에 습니lsquo rsquo

원 리에8 (lsquo rsquo lsquo rsquo lsquo rsquo

)

원 에 리lsquo rsquo lsquo rsquo lsquo rsquo①

원에게 가 도 여 는lsquo rsquo 3

니다

사는 원 가 개 보 우 가lsquo rsquo lsquo rsquo lsquo rsquo②

거 사 경우 는 미 에 어 거 lsquo

사 사 운 우 가 는 경우 당rsquo lsquo rsquo

습니다lsquo rsquo

원 가 도 거lsquo rsquo lsquo rsquo lsquo rsquo 3③

가 사 고 지 경우에는 시 사에lsquo rsquo

통지 고 사 내에 니다lsquo rsquo

경우에 당 원 사에 그 사실3 lsquo rsquo lsquo rsquo④

통지 지 거 통지 도 사 내에 지 lsquo rsquo

생 경우 사는 지지 습니다lsquo rsquo

원에 통지9 (lsquo rsquo )

사는 특 다 원에게 통지 경우lsquo rsquo lsquo rsquo

공지 게시 통 상 게시 개별 통지에7

갈 습니다

사10 (lsquo rsquo )

사는 과 지 미lsquo rsquo①

에 는 지 계 고

공 여 다 여 니다lsquo rsquo

사는 원 게lsquo rsquo lsquo rsquo lsquo rsquo②

도 개 보 신 보 포 보 보 시( )

갖 어 개 보 취 공시 고

니다

사는 과 여 원lsquo rsquo lsquo rsquo③

견 만 당 다고 경우에는

리 여 니다 원 견 만 사 lsquo rsquo

에 는 게시 거 우편 등 통 여

원에게 리 과 결과 달 니다lsquo rsquo

원11 (lsquo rsquo )

원 다 여 는 니다lsquo rsquo ①

신청 는 변경 시 허 내 등1

타 보 도2

사가 게시 보 변경3 lsquo rsquo

사가 보 보 컴퓨 그4 lsquo rsquo (

등 등 신 는 게시)

사 타 등 지 재산 에5 lsquo rsquo 3

사 타 상 거 업6 lsquo rsquo 3

는 폭 시지 상 타 공7 middot middot

에 는 보 에 공개 는 게시 는lsquo rsquo

사 동 없 리 사8 lsquo rsquo

타 거 당9

원 계 규 내lsquo rsquo lsquo②

여 공지 주 사 사가 통지 는rsquo lsquo rsquo

사 등 여 타 사 업 에 lsquo rsquo

는 여 는 니다

- (wwwnavercom)

zb47) 위 약 의 조항에서 같은 제점을 하lt gt

고 있는 조항은

lt gt

제휴 회사에 회 의 아이디 개인 정 를 전송할 있도

한 조항은 고객에게 당한 조항이다

1 7 8① ② ③

④ 9 ⑤ 10

립 도 규

1 ( )

규 립 도 립 어린 청 도(

포 다 료 시 열 시 말) (

다 에 사 규 립 도)

편 진 다

2 ( )

규 립 도 도 다 에( lsquo rsquo )

고 는 도 에 도lsquo rsquo 2 2

료 에 여 다 다만 특 료 귀

료 등 료 에 사 립 도

도 다 다( lsquo rsquo )

3 ( )

도 다 각 같다①

공 공 다만 연1

연 간 다

년 학 간고사 대비2013 2 현대고 대비

ECN-0102-2013-001-000076193

매월 째 째 월2

도 도 리 그 사3

가 다고 는

도 에 미리 게1 3②

시 여 다

시간4 ( )

도 시간 도 여 게시 다

등 등5 ( )

도 료 시 는 는 도①

지에 등 후

등 에 사 도②

사 료6 ( )

도 료 시 에 사 료는 도

7 ( )

는 다 각 여 는 니 다

도 료 시 상 리1 lsquo rsquo

도 료 시 훼 는2 middot

지 가 닌 곳에 식 거 담3

우는

도 보 등 보 검색열4 middot

그 에 도 질 지 여 도5

여 게시 사 는

질 지8 ( )

도 다 거 도①

질 게 우 가 는 에 여는 도

도 가 각 어느7②

에는 지 게 거 도

료9 ( )

도 료는 다 각 경우 다①

상 도 간에 료 는 것 말1 (

다 등 다 도 과 여 경우)

공 원 공 상 는 경우2

그 에 도 다고 는 경우3

가능 도 료 는 도②

는 에 다

변상10 ( )

가 도 료 시 럽 거 거①

못 쓰게 거 어 린 경우에는 변상 여

도 에 변상 여 게시1②

여 다

등 규 에 것 에 도11 ( )

료 시 등에 사

도 다

립 도- (httpwwwnlgokr)

zb48) 다음 정 리 의 의 으 볼 때 가장

이 적인 것은

도 시간 도 여 게시 다①

등 에 사 도②

가능 도 료 는 도 는③

에 다

④ 도 에 변상 여 게10 1

시 여 다

⑤ 도 가 각 어느7

에는 지 거 도

zb49) 를 참고하여 이 어의 성격을 설 한lt gt

것으 적절하 않은 것은

① 보 에 는 어 시 상 고 어 시lt gt lsquo rsquo

에 보여주고 다

② 진 어 어원에 견 고 다

에는 타 어 들어가는 것 다 lsquo rsquo

③ 에 들어갈 말 각각 고 어 어 신 어~

들 언어는 질 격 강 통 없었다

④ 시 우리 에 가 었지만 지 계

과 달리 들 통 사 달 어 웠

년 학 간고사 대비2013 2 현대고 대비

ECN-0102-2013-001-000076193

⑤ 크 몽골 만주 공통어가 우리 어 같

계열에 다는 에 사 특 짐

가( )

善化公主主隱 공주님

他密只嫁良置古 몰 결 고

薯童房乙 맛

夜矣卯乙抱遣去如 에 몰 고 가다

( )

始汝 會隱日恚見隱扐 만 에 본

恥隱汝衣淸隱笑 맑 웃

고 시 여 공 크다 만 다[ ] ( ) ( ) ( ) ( )始 汝 會扐

내다 에 보다 견( ) ( )恚 見 다( )隱

럽다 맑다 청 웃( ) ( ) ( ) ( )恥 衣 淸 笑

zb50) 위의 나 를 함 고 음에 답하( ) lt gt

보lt gt

( )素那或云金川 白城郡蛇山人也

운 사산

는 고 다 는( )[ ( ) ] (素那 金川 白城

사산 사 다) ( ) 郡 蛇山

삼 사- lsquo rsquo 47

에 제 된 단어 의 표 리를 조건(1) lt gt ( ) lt gt

에 맞게 서 하

건lt gt

lsquo 었고 었다 태rsquo

에 제 된 단어 동일한 표 리에(2) lt gt ( )

의해 적은 것을 나 에서 찾아 조건 에 맞게 서 하( ) lt gt

건lt gt

에 당 는 각각( ) 개 쓸 것2 단

당 는 가 여러 개 어도 개만 쓸 것 각2

개 과 도 쪽에 개만2 2

드시 지 것( )

과 동 원리 것lsquo 고

과 동 원리 것 다rsquo

태 것

가( )

素那(或云金川) 白城郡蛇山人也

소나 또는 천 이라 한다 는 성 사( ) ( ) ( )素那 金川 白城郡〔 〕

산 사람이다 현대어 풀이( ) ( )蛇山

나( )

紫布岩乎希 회

執音乎手母牛放敎遣 자 손 암쇼 노히 고

吾 不喩慙 伊賜等肹 肹 나 안디 리샤

花 折叱肹 可獻乎理音如 고 것거 도림다

다 향찰은 리말을 리 으 적은 표 이었 만 생( )

은 고 대를 넘 하고 끊어 고 말았다 랜 세

동안 갈고 닦아 체계적이었던 향찰 표 이 사라졌

을 인은 크게 두 가 나누어 생각해 볼 있다

하나는 족 사회의 한 선호도에서 찾을 있다 라 때

향찰은 주 족 계 에서 사 했을 것으 인다 한 을

알 하고서는 한자를 활 하여 리말을 리 으 표

하 란 가능하 때 이다 런데 족들은 간이 흐

를 향찰과 같은 리 표 을 익혀 사 하 다는

아 한 을 대 사 하는 쪽을 선호하게 되었다 더 이

고 초에 인재 등 을 위해 과거제도가 행되 서 한 선

호도가 더 높아졌고 결 향찰은 소 되고 말았다

또 다른 가능성은 한 어의 특성에서 찾을 있다

터 한 과 일 세 나라는 한자 화 에 속해 다

당연한 이야 겠 만 표의 자인 한자는 어를 표 하

에 매 적절하다 어의 음절은 성 ( ) ( )聲母 韻母

이 어 고 여 에 성조가 추가되어 최종 소리가 결정된

다 래서 어는 단음절을 하나의 한자 표 하 된

다 에 초성 성 종성의 세 가 소가 하나의 음절

년 학 간고사 대비2013 2 현대고 대비

ECN-0102-2013-001-000076193

을 이 는 한 어는 음절 조가 잡하고 음절의 가 많아

서 한자 차 만으 한 어의 소리를 만족 럽게 표 할

없었다 를 들어 한 어에서는 어 니 같이 음절 lsquo rsquo

이 어 단어가 얼마든 있으나 어는( ) 複數音節

자 하나 나타내 만이다lsquo [m ]rsquo 母 ǔ

한편 일 어의 표 은 핵 적 단어는 한자 적고 토는

가나라는 일 의 자 적는 이다 적인 의 를 나

타내는 은 표의 자인 한자 적고 적 계를 나

타내는 토는 표음 자 적는 셈이니 자세히 살펴

리의 향찰 표 을 쏙 빼닮았음을 알 있다 한 어 같

은 착어이 서도 일 어에만 향찰과 유사한 표 이 살아

남은 것은 일 어의 특 때 이다 일 어는 하나의 자음과

음의 결합으 음절을 이 고 침이 거의 없는 음절 언어

이다 이러한 음절의 특색에다가 토가 달한 착어라는 점

이 향찰과 유사한 표 이 살아남을 있는 비결이었다

하 만 같은 착어라도 다양한 음소 침이 달한 한

어는 향찰 표 하는 데 근 적으 한계가 있었다

zb51) 다 하여 의 행에 대한 탐 한 결과( ) lt gt 2

않은 것은

보lt gt

善花公主主隱 공주니믄 공주님( )

----------------------------------------

-

他密只嫁良置古 그 지 얼어 고 몰 결(

----------------------------------------

-

薯童房乙 맛 맛( )

夜矣卯乙抱遺去如 몰 고 가다 에 몰 고(

가다)

주동 역 동- (薯童謠『 』

에 2 ( )他密只嫁良置古

얼다 시집가다 결 다 말 lsquo rsquo

① 실질 미 지니고 므 타 타lsquo ( )rsquo lsquo [ ]

② 에 실질 미 타내고 지 는lsquo rsquo lsquo [ ]rsquo lsquo [ ]密只 密 只

계 타내는

③ 얼어는 실질 미 포 고 므 가lsquo rsquo lsquo [ ]rsquo嫁

것lsquo [ ]rsquo 良

④ 고 어간 는 실질 미 지니고 므lsquo rsquo lsquo -rsquo

것lsquo [ ]rsquo 置

⑤ 고 어미 고는 계 타내고 므lsquo rsquo lsquo- rsquo

고 것lsquo [ ]rsquo 古

가( )

엉 훈 민middot middot middot middot middot世 宗 御 製 訓 民 正 音

말 미 듕 귁에 달middot middot middot middot middot middot middot middot中 國 文 字

니 런middot middot middot middot middot middot 어린middot middot middot middot百 姓

니 고 도 내 들middot middot middot middot middot middot middot middot middot 시러middot

펴 몯middot 미middot middot 니 내middot middot middot middot middot middot middot middot 爲

어엿middot 겨 새middot middot middot 믈여듧middot middot middot middot字 니middot middot middot

사 마다 니겨 킈 middot middot middot middot middot middot middot middot middot便 安

고 미니middot middot middot middot

본 는 상( ) (象

원리에 만들어진 본) ( )形 ㄱ ㄴ ㅁ ㅅ ㅇ

에 는 가 원리에( )加劃

그리고( )ㅋ ㄷ ㅌ ㅂ ㅍ ㅈ ㅊ ㆆ ㅎ

쓰는 병 원리에 만들어진( )竝書

마지막 체( ) ( )異體ㄲ ㄸ ㅃ ㅆ ㅉ ㆅ

ᅀ 다 상 원리에 ㅇ ㄹ

지 는 삼재 상 본 본( ) ( ) ( 天地人 三才

탕 므림과 림에 ) (初ㅡ ㅣ

재)( ) ( )( )出字 再出字ㅗ ㅏ ㅜ ㅓ ㅛ ㅑ ㅜ ㅕ

병 그리고 들 에 다시( )ㅘ ㅝ ㅣ

( )ㅣ ㅢ ㅚ ㅐ ㅟ ㅔ ㆉ ㅒ ㆌ ㅖ ㅙ ㅞ

zb52) 가 에 대한 설 으 르 않은 것을( ) 두 고르

① 어쓰 규 지키고 다

② 리 고 다

③ 말 미 미 등 어 사 다lsquo rsquo

④ 개 지 다

년 학 간고사 대비2013 2 현대고 대비

ECN-0102-2013-001-000076193

⑤ 어 원 에 가 도 고 다

엉 훈 민世 宗 御 製 訓 民 正 音

말 미 듕귁에 달 니

런 어린 니 고 도middot

내 들 시러 펴 몯 미 니middot

내 어엿 겨 새 믈여듧

사 마다 니겨middot 킈 고

미니

훈민 언 본- lsquo rsquo 5 (1459 )

zb53) 위의 에 대한 현대어 풀이가 르~ 않은 것

① 우리 말 과 달

② 어리 말 고 는 것 어도

③ 신 생각 마 껏 펼 는 사 많다

④ 게 생각 여

⑤ 사 마다 게

zb54) 훈민정음 언해 에는 한 을 창제한 동 가 드러나

있다 훈민정음 창제의 정 과 내 이 잘 연결된 것

① 주 신 말 미 듕귁에 달

② 민 신 내 어 겨

③ 신 뻔 킈 고 미니

④ 실 신 사 마다 니겨

⑤ 귀 신 계 주 는 훈민 신과 거리가

가 엉 훈 민( ) middot middot middot middot middot世 宗 御 製 訓 民 正 音 

말 미 귁에 中 國 달 文 字

니 런 어린 니 百 姓

고 도 내 들 시러 펴 몯

미 니 내 어엿 爲 겨 새

믈여듧 니 사 마다 니 字

겨 킈 고 미니 便 安

훈민 언 본- lsquo ( )rsquo ( ) 5 (1459 )訓民正音 世祖

( )

[ 1 ]

동 룡 샤 마다 복( ) ( ) ( )海東 六龍 天福

시니 고 동( ) ( )古聖 同符 시니

[ 2 ]

매 니 곶 여

미 므 니 그 내 러

가 니

[ 125 ]

우 미리( )千世 샨( )定 에( )漢水北 累仁

누 개 샤 복 업 시니( ) ( ) 開國 卜年

신( )聖神 니 샤도 경 근민 샤 욱( )敬天勤民

드시리 다

님 쇼 산 가( ) ( )洛水 山行

미드니 가

어 가- lsquo ( )rsquo 27龍飛御天歌

다 우리신 니쓰고 다만 만 쓰( )

거 샹 귀쳔 다보게 러 귀

여 쓴 도 신 보 가 고 신 에

말 어 보게 각 에 사 들

고 본 몬 능통 후에

죠 죠 니

드 도 만 공 에 사

드 미 죠 고 고 여 보 죠

보다 얼마가 거시 어신고 니 첫

가 죠 니 죠

민 들 어 신 샹

귀쳔 도보고 어보 가 만 늘

고 폐 에 만쓴 죠 민

도 러보지못 고 보니 그게 엇지

심 니 리 보 가 어 운건 다

니 쳣 말마 지 니 고 그

쓰 에 가 우 지 지

몰 거 본후에 가 어 지

고 그니 쓴편지 쟝 보

년 학 간고사 대비2013 2 현대고 대비

ECN-0102-2013-001-000076193

쓴것보다 듸 보고 그 마 니 쓴 고

어 못

그런고 에 리 과 가

만 쓴 못 민 말만 듯고

고 편 그 못 보니 그사 단

병신 못 다고 그사 식 사

니 만 고 다 과 그사

만 고 다 과 업 사 보다 식 고

죠 도 고 각 과

견 고 실 직 귀쳔 간에 그

고도 다 것 몰 귀죡 보다

사 우리 신 귀쳔 다 업

시 신 보고 과 지 게 랴

시니 샹 귀쳔 간에 우리 신 걸

간 보 새지각과 새 걸 미리

독립신- lsquo (1896)rsquo

zb55) 친 어 나의 제 장( ) 2 매 함축적

의 가 가장 유사한 것은

① 지 눈 내리고 매 득 니 내 여 가

사- lsquo rsquo

② 도 어 리듯 그 게 어 다

주 사- lsquo rsquo

③ 눈 살 다 죽 어 린 과 체 여

눈 새벽 지 도 살 다

눈- lsquo rsquo

④ 삶 근심과 고단 에 돌 거니는 여 거 는

여 리 내린 살가지 에 눈 리 눈 리

택 그 생 에- lsquo rsquo

⑤ 늘 러 고 러

청룡 룡 어 개 루 우

신경림 계- lsquo rsquo

zb56) 친 를 위 가 나 에 나타난A B ( ) ( )

세 어의 특 에 의거하여 세 어 표 하

그 산 고 공 도 맑지만

A

주변에 쓰 리는 어리 사 많다

B

건lt gt

식 가 에 타 어 특징에( ) ( )

거 과 어쓰 는 고 지 말 것

A

B

zb57) 가 의( ) 달 아ㆍ 다 의 ( ) 나셔에서 알 있는

세 어 개화 어의 특 을 비 하여 조건 에lt gt

맞게 서 하

건lt gt

어에 는lsquo 개

어에 는 다 태rsquo

zb58) 은 가 는 다 에 나 는 절lt 1gt ( ) lt 2gt ( )

일 를 췌한 것이다 의 의 가 lt 1gt (1)~(2)

유사한 말을 에서 찾아 쓰lt 2gt

보lt 1gt

런 (1) 어린 니 고百 姓

도 내 들 시러 펴 몯 미

사 마다 (2) 니겨 便 安

킈 고 미니

보lt 2gt

죠 고 고 여 보 죠

보다 얼마가 거시 어신고 니 첫 가

죠 니 죠 민

들 어 신 샹 귀쳔

도보고 어보 가 만 늘 고

폐 에 만쓴 죠 민 도

러보지못 고 보니 그게 엇지 심

니 리

년 학 간고사 대비2013 2 현대고 대비

ECN-0102-2013-001-000076193

lt 1 gt

동 룡 샤 마다 복 시( ) ( ) ( )海東 六龍 天福

고 동 시니( ) ( )古聖 同符

lt 2 gt

(A) 매 니 곶

여 니

미 므 니 그 내

러 가 니

lt125 gt

우 미리 샨 에( ) ( ) ( ) 千世 定 漢水北 累

누 개 샤 복 업 시 니( ) ( ) 仁開國 卜年 聖

신( ) 神 니 샤도 경 근민 샤( ) 敬天勤民

욱 드 시 리 다

님 쇼 산 가 ( ) ( )洛水 山行

미드니 가

- lt gt龍飛御天歌

zb59) 장과 내 상 유사한 성격의 조는125

① 뫼 고 고 고 고

어 그린 많고 많고 고 고

어 러 는 울고 울고 가느니

도 견- lt gt

② 강 에 드니 몸 다

그믈 고 가니

뒷 뫼 엄 언 니( )藥

-

③ 말 없는 청산 태 없는 다

값 없는 청 없는 월

에 병 없는 몸 별 없 늙 리

-

④ 가마귀 골에 가지 마

낸 가마귀 새

청강에 것 시 몸 러 가( ) 淸江

-

⑤ 진 골에( ) 白雪

가 매 는 어느 곳에 었는고

에 갈 곳 몰( ) 夕陽

색-

zb60) 위 에 나타난 세 어의 특 으 적절하 않은

것은

① 룡 어 주격 사에 당 는 가 사( ) lsquo rsquo六龍

고 다

② 샤 어에도 어 주체 쓰 다

는 것 다

③ 매 어 달리 사 택에 어

가 지 지지 고 다

④ 므 원 상 직 어 지 다

⑤ 드시리 다 주체 과 상 께 사

고 다

수고 하셨습니다hearts hearts

년 학 간고사 대비2013 2 현대고 대비

ECN-0102-2013-001-000076193

보닷컴에 공 는 별 보는 고등

들 여 주 는

들 습니다 슷 동 지

가 복 는 것 도가

니 복 여 습 시고 거 시

니다

정답 해설

1) 정답[ ] ④

해설 다른 것은 두 특정 업이나 단 내에서 사[ ]

하는 일종의 은어 사회 언에 해당한다 러나

는 언이 아니라 단과대학을 여서 단대 사lsquo rsquo lsquo rsquo lsquo④

대학을 여서 사대라고 한 말에 해당하 일rsquo lsquo rsquo

사회에서도 널리 쓰이 사회 언이라 할

없다

2) 정답[ ] ⑤

해설 사회 언은 같은 단 내에서 쓰이는 언어이[ ] lsquo rsquo

동일 단끼리는 단결 과 친 감을 형성하는

능을 하 리적 안감이 일어나 않는다

3) 정답[ ] ③

해설 사람이라는 차 적 표현에 대한 대안적 표현이[ ]lsquo rsquo

인 아내 처 등으 볼 있다lsquo rsquo

4) 정답[ ]⑤

해설 남성은 주 격 체를 사 한다[ ]

5) 정답[ ] ⑤

해설 흑인은 검다라는 뜻을 가 고 있을 뿐 인[ ]lsquo rsquo lsquo rsquo lsquo rsquo

다 열등한 뜻을 내포하 않는다

6) 정답 살 색 첫 작품[ ] - -

해설 살색 혹은 킨색은 한 인의 피 색을 뜻[ ] lsquo rsquo lsquo rsquo

하는 것으 인종 차 을 추 고 출 이주민

의 평등 을 침해할 있어 년 표 이2005

살 색으 이름을 꾸었다 처녀작은 처녀라lsquo rsquo lsquo rsquo lsquo rsquo

는 단어가 가 고 있는 곡된 성 인 을 한 것

으 첫 작품정도 꾸어 사 하는 것이 좋다lsquo rsquo

7) 정답[ ] ⑤

해설 호는 아들에게 해체를 사 하고 있다[ ] ① ②

장 을 성하는 청자는 자 의 아 느리 아lsquo

들 세 이다 호는 아 느리에게 해rsquo ③

체를 사 하고 있다 호가 느리 아 에게 ④

사 한 해 체 아들에게 사 한 해체는 두 비lsquo rsquo lsquo rsquo

격 체에 해당한다 호는 자 의 아랫사람인 ⑤

느리에게 아들과 마찬가 해체를 사 하는 것이

상 이 만 임 을 한 느리에게 고마 과 쁨

존 의 표 를 하 위해 자 의 아 에게 말하듯

해 체를 사 하고 있다

8) 정답[ ] ③

9) 정답[ ] ⑤

10) 정답[ ] ①

해설 청자 할아 가 장의 주체 아 다 높을[ ] ( ) ( )

경 에는 압존 에 의해 장의 주체를 높이 않는lsquo rsquo

다 러 아 서가 아닌 아 는으 계 lsquo rsquo lsquo rsquo lsquo

니다 가 아닌 있 니다 표현하는 것이 르rsquo lsquo rsquo

11) 정답 당이 당을 쫒았다 당이[ ]

당에 다

해설[ ]

12) 정답[ ] ⑤

해설 서 다른 높임표현을 통해 청자에 대해 리[ ] ⑤

적 거리감을 나타내는 인 은 이 아니라 현정이

다 가 에서 현정은 에게 해 체를 사 함으 써 ( )

친근감을 드러낸다 나 에서 연 을 게을리하는 역 ( )

도 들 때 에 화가 난 현정이 선생님에게 항의하

는 장 에서는 하 체를 사 하여 리적 거리lsquo rsquo

가 어졌음을 나타내고 있다

13) 정답[ ] ①

해설 는 는 얼 빛이 날과 어찌 다르 고[ ] lsquo rsquo

라는 뜻으 전과 달리 임이 화자를 않고

있음을 알 있다

14) 정답 달리 후 가 있다 이를 통해 경[ ] lt gt

쾌한 음악성을 형성하고 노 젓는 상황을 체적으

형상화하는 역할을 한다

15) 정답[ ] ①

16) 정답[ ] ⑤

해설 다 의 자연은 를 성찰하게 하는 대상[ ] ( )⑤

이자 정의 대상이다 의 자연은 자 의 상황과 ⑤

처 를 드러내는 경으 서의 역할을 하 이

이 없다

17) 정답[ ] ③

해설 는 빈천 을 해결하고자 했으나 강산[ ] lsquo ( )rsquo 貧賤③

과 풍 을 달라는 에 거절하 다고 함으 써 자

연에 대한 애정을 드러내고 있으 는 않는

임에 대한 망을 개에게 전가 켜서 임에 대한 리

을 드러내고 있다

18) 정답[ ] ③

년 학 간고사 대비2013 2 현대고 대비

ECN-0102-2013-001-000076193

19) 정답[ ] ⑤

해설 고상한 음악가의 이름을 리말 꽝 럽[ ]

게 꿈으 써 언어유희를 통해 음을 유 하고 있

다 이는 고상한 척하는 총 를 비꼼으 써 비판적

태도를 드러내는 것이 대상을 꽝 럽게 표현

하여 총 의 허 과 사치를 풍자하고 있다

20) 정답[ ] ⑤

해설 는 작품 속 경에 대한 설 이 드러나는 것이[ ]

서 자의 주 적인 견해가 접적으 드러나는 것이

아니다

21) 정답[ ] ⑤

22) 정답[ ] ②

23) 정답[ ] ④

24) 정답[ ] ①

해설 적강 티프는 주인공의 비 한 출생이나 능[ ] ①

과 이 있는 것으 조정의 능함을 풍자하는lsquo rsquo

것과는 거리가 다

25) 정답 픔 나[ ] ( )

해설 의 음악은 고통 는 사람들을 위 하고 아픔[ ] lsquo rsquo

을 치유해 주는 능을 한다고 할 있다 의 lt gt

픔 도 소 된 이 과 더 어 살아가는 따뜻한 마음lsquo rsquo

을 상 한다

26) 정답[ ] ⑤

해설 에게 선천적으 주어 각 장애라는 역경[ ]

은 의 이라는 가사 연 을 있다lsquo rsquo

27) 정답[ ] ④

해설 는 장 란 선 에게 은 개인적인 인상을[ ]

소녀 장정 등으 표현한 것이다lsquo rsquo

28) 정답[ ] ②

해설 담자가 피 담자의 언어적 표현이나 비언어[ ]②

적 표현 하 독자는 담의 위 나 피

담자의 감정 상태를 알 있다 이를 통해 독자는

담 상황을 더 생생하게 느낄 있고 피 담자

를 더 잘 이해할 있게 된다

29) 정답[ ]③

해설 일상생활과 역도 선 서의 성과에 된 것에서[ ]

역도를 하 서 겪는 어 과 내적 고민으 화제를

전화하 위한 것이다

30) 정답[ ] ①

해설 릿속에 새겨 넣듯 이 억되도 함 세상[ ] ② ③

살이가 힘들고 고생 러 속 하여 자유를 ④

가 없는 고통의 상태를 비유적으 이르는 말

적의 침입을 막 위해 쌓은 축 켜야 할⑤

대상을 비유적으 이르는 말이다

31) 정답[ ] ④

해설 이 의 종류는 전 으 인 사건 경[ ] lsquo

비평을 성 소 삼는다rsquo

32) 정답[ ] ④

해설 근은 삼대독자 태어났음을 에서 확인할[ ]

있다 형제들과의 담은 이뤄 가 없다

33) 정답[ ] ⑤

해설 근은 가난에도 하고 화가를 꿈꾸었다[ ] (3

단 또한 다른 화가 망생들은 정 육을)

위해 상 학 학 해 유학 에 랐 만

근은 다른 을 찾아야 했다 단 세에(5 ) 18

근은 조선 전람회에 입선하 다 단 의(6 )

만종은 인간과 자연이 엮어 가는 경건한 조화 을lsquo rsquo

나타낸다

34) 정답[ ] ①

해설 근이 속에서도 창작활동을 추 않고[ ]

하는 닭은 은 세상과 타협할 르는

근이 세상의 이해를 하 위한 가장 떳떳한 단

이 때 이다

35) 정답[ ] ⑤

해설 전 은 서 자의 주 적인 평이 리는 것이[ ]

만 위 제 은 인 이 살았던 대 사회적 경

을 통해 객 적인 인 의 을 제 하고 있다

36) 정답[ ] ⑤

해설 전 은 인 사건 경 비평이라는[ ] lsquo rsquo⑤

성 이 어져 있다

37) 정답[ ] ①

해설 이 은 동양인과 서양인의 사고 에 차이가[ ]

있다는 것을 대조를 통해 설 하고 있다 또 쓴이

의 제자가 축 경 를 러 가서 경험한 일화를

통해 동양인이 서양인에 비해 주 상황에 더 많은

주의를 인다는 주장을 뒷 침하고 있다

38) 정답[ ] ④

39) 정답[ ] ②

40) 정답[ ] ②

41) 정답[ ] ④

42) 정답[ ] ③

43) 정답[ ] ④

44) 정답 도서 의 휴 일 도서 의 이 간 도서의[ ]

해설 도서 장은 임의 정한 휴 일과 도서 이[ ]

간 도서의 상 등을 게 할 의 가 있다

년 학 간고사 대비2013 2 현대고 대비

ECN-0102-2013-001-000076193

45) 정답[ ] ①

해설 제 조의 정 휴 일 의 휴 일의 사전 게[ ] 3

는 도서 장의 의 조항에 속한다

46) 정답[ ] ①

해설 개인 정 호 의 를 제 하 했 만 항[ ]

나눠서 제 하 않고 대 나열하고 있다

47) 정답[ ] ②

해설 제 조의 내 을 회사는 다른 회사 협[ ] 7 lsquo

계약을 통해 서비 를 제공하는 경 회 의 아이디

등 개인 정 를 해당 회사에 전송할 있다는 내rsquo

이 있으 의 제점을 제 할 있다②

48) 정답[ ] ④

해설 는 도서 장의 의 에 해당하고 나 는 도[ ] ④

서 장의 리에 해당한다

49) 정답[ ] ③

50) 정답 은 음독으 적었고 은 훈독으 적었[ ] (1)

다 과 동일한 표 리 적은 것은 이고 (2) ce

과 동일한 표 리 적은 것은 이다ab

51) 정답[ ] ③

52) 정답[ ] ①②

53) 정답[ ] ③

54) 정답[ ] ③

55) 정답[ ] ①

56) 정답 른 죠코 어린 노 하니라[ ] A B

57) 정답 세 어에서는 활 형이 칙적으[ ] lsquo rsquoㄹㅇ

나타났 만 개화 어에서는 활 형이 쓰 다 lsquo rsquo ㄹㄴ

58) 정답 호 가 흔[ ] (1) (2)

59) 정답[ ] ④

60) 정답[ ] ③

Page 25: 현대고대비 국어 - chamsoriedu.com 「콘텐츠산업진흥 법」외 에도 저작권 의하여 ... 다른주체에게어떤동작을하도록만드는것을나타내는

년 학 간고사 대비2013 2 현대고 대비

ECN-0102-2013-001-000076193

그리 간 보는 과 사 에

매우 달 뿐만 니 과 에 도 극

루고 었다 미 운 그런 들

살고 는 동 과 사 들 사고 식에

큰 가 다는 다

고 그리 들 우주 개별 고 독립

사 들 생각 지만 고 들 우

주 연 질 간주 다 같 각

도 들에게는 연 질 었지

만 그리 들에게는 미 들 결 었다

고 과 그리 들 사 같 는

동 과 사 에 도 견 다

인 리학자인 츠 이마이 디드 겐트너는 두

살이 채 안 된 아이들에서 터 성인에 이르 다양한

연 대의 동양인과 서양인을 대상으 다음과 같은 험

을 했다 저 코르크 만든 피라 드 양의 도형을

여 주고 대상의 이름을 닥 라고 알 주었다lsquo (Dax)rsquo

제 닥 는 존재하 않는 것으 험자가 임의lsquo rsquo

만들어 낸 이름이다 런 다음 두 개의 다른 체를

여 주었는데 하나는 피라 드 양이 만 하얀 플라 틱

으 만들었고 다른 하나는 재 는 코르크 만 양이

달랐다 러고 나서 어떤 것이 닥 인 사람들에게 고 lsquo rsquo

르게 했더니 서양인들은 주 같은 양을 하고 있는

체를 선택했고 동양인들은 같은 재 만들어 체를

선택했다 이러한 차이는 성인은 어 두 살 리

아이들에게서도 나타났다 이것은 곧 서양인과 동양인은

서 다른 세상을 고 있다는 것을 의 한다 략 ( )

는 아주 단 하 서도 인상적인 험을 했다

험에는 동서양의 대학생들이 참여했다 는 험 참가자

들에게 컴퓨터 화 을 통해 속 장 을 담은 애니 이션

을 여 주었다 화 의 앙에는 초점의 역할을 하는 커

다란 고 한 마리가 있었고 주위에는 다른 생

들과 초 자갈 거품 등이 함 제 되었다 화 을

두 씩 후 참가자들은 자 이 것을 회상해 라는

를 았다

결과 서양인 대학생들과 동양인 대학생 두 앙

의 초점 역할을 했던 고 를 동일한 정도 언 했으

나 경 소 위 거품 초 다른 생 들 에 ( )

대해서는 동양인 대학생들이 서양인 대학생들 다 60

이상 더 많이 언 했다 뿐만 아니라 동양인 학생들은 서

양인 학생들에 비해 개 적인 고 다 전체적인 계

를 더 언 하는 경향을 다 략 또한 경의 일 ( )

를 화 킨 림을 제 하 을 때 동양인 대학생들은 대

경의 화를 알아챘 만 서양인 대학생들은 경

의 화를 거의 알아차리 했다 략 ( )

따라서 서양인들만을 대상으 연 한 화lsquo

편성 결 은 잘 된 것일 도 있다 각 과정과 인rsquo

과정의 어떤 이 화 편적이고 어떤 이

화에 따라 달라 는 는 앞으 많은 연 를 통하여 논의

되어야 한다

나 어떤 의 에서 리 두는 이 화적이다 리( )

안에는 다른 사람들과 더 친 한 계를 유 하 는 상호

의존성과 다른 사람들 터 독립적인 존재 살아가 는

독립성이 혼재한다 따라서 이 에서 어떤 특성이 더 강

하게 각되는 상황에 놓이느냐에 따라 서 다른 화적

특 을 일 있다 결 리 두는 어떤 경 에는

동양인처럼 행동하고 어떤 경 에는 서양인처럼 행동하는

것이다

zb39) 가 에 대한 다음의 설( ) 않은 것은

① 는 신 주 뒷 닥 실험과lsquo rsquo lsquo

니 실험 근거 시 다rsquo

② 동 들 상 간 공통 보다는 에 식

는 강 다

③ 들 주변 맥 에는 심 경 어 사건

과 사건 사 계에 상 민감 다

④ 는 동 과 틀린 지 고 는 것lsquo rsquo

니 다 고 다 lsquo rsquo

⑤ 가에 우리 사 들 개 시 가 원( )

집 경 말 고 는 것 개 보다는

에 고 는 것에 다

늘 지 상에 살고 는 사 들 억 도가10

고 그리 지 통 고 는 사 들( )知的

그보다 훨 많 억 도는 고 지 20

통 다 그런 지 고 2500

그리 간 보는 과 사 에

매우 달 뿐만 니 과 에 도 극

루고 었다 미 운 그런 들

살고 는 동 과 사 들 사고 식에

큰 가 다는 다

지심리 미 마 드 겐트 는 동

과 상 다 과 같 실험 다

크 만든 미드 도 보여 주고 그

상 닥 고 주었다 그런 다lsquo (Dax)rsquo

개 다 체 보여 주었는 는 미드

지만 틱 만들었고 다 는 재료는

크 지만 달 다 그러고 어 것 닥 lsquo

지 사 들에게 고 게 니 들 주 같rsquo

고 는 체 택 고 동 들 같

재료 만들어진 체 택 다 러 는

심지어 살짜리 들에게 도 타났다 것

곧 과 동 다 상 보고 다는

것 미 다 개별 사 보고 고 동

년 학 간고사 대비2013 2 현대고 대비

ECN-0102-2013-001-000076193

연 질 보고 는 것 다

동 들 주변 상 에 맞 어 동 고

에 다 사 들 태도 동에 보다

많 주 울 다 동 가 미시간

에 에 경험 다 그는 미

식 경 보러 가게 었는 경 체는 매우 재

미 었 주변 들 동에 질 다 그

는 들 계 어 상태 경

다 어 들 에 에 그 시 가 계

가 진 것 다 뒷사 고 지 는 들

동 럼 어 웠다

그는 경험에 어 얻어 동 들lsquo

각도 상 본다 는 가 우고rsquo

검 여 주 단 도 상 실험 실

시 다 그는 실험 가 들에게 컴퓨 통

담 니 보여 주었다

에는 역 는 커다 고 마리가 었

고 주 에는 다 생 들과 갈 거 등

께 시 었다 본 후 가 들

신 본 것 상 보 는 지시 다

그 결과 생들과 동 생

역 고 동 도 언

경 거 다 생 들에 ( )

는 동 생들 생들보다 60

상 많 언 다 뿐만 니 동 생들

생들에 개별 고 보다 체 계

언 는 경 보 다 경 변 시

킨 그림 시 동 생들 경

변 지만 생들 경 변

거 리지 못 다

지 지 들만 상 연 lsquo

보편 결 못 것 도 다 지각 과 과rsquo

지 과 어 보편 고 어

에 달 지는지는 많 연 통 여

어 다

리 드 니 벳 생각 지도 사- ldquo rdquo( 2004)

zb40) 위 에 대한 설 으 가장 적절한 것은

① 동 과 생 식 강 고 다

② 가지 실험 통 쓴 고 다

③ 닥 실험에 사 본질에 동 사

상에 주 다

④ 니 실험에 동 과 에 지

각 도에 가 다

⑤ 쓴 는 보편 연 에 드러 우월 에

에 근 고 다

가 동 들 주변 상 에 맞 어 동 고( )

에 다 사 들 태도 동에 보다 많

주 울 다 동 가 미시간 에

에 경험 다 그는 미식

경 보러 가게 었는 경 체는 매우 재미 었

주변 들 동에 질 다 그 는

들 계 어 상태 경 다

어 들 에 에 그 시 가 계 가

진 것 다 상 살펴lsquo 는 말 들rsquo

에 그는 에 시 어 도 뒷사

생각 곧 다시 곤 것 다 그런 그에게

뒷사 고 지 는 들 동 럼

어 웠다

그는 경험에 어 얻어( ) 동 들lsquo

각도 상 본다 는 가 우고rsquo

검 여 주 단 도 상 실험

실시 다 실험에는 동 생들 여 다

그는 실험 가 들에게 컴퓨 통

담 니 보여 주었다 에는

역 는 커다 고 마리가 었고 주 에는

다 생 들과 갈 거 등 께 시

었다 본 후 가 들 신 본 것

상 보 는 지시 다

다 그 결과 생들과 동 생( )

역 고 동 도 언

경 거 다 생 들 에 ( )

는 동 생들 생들보다 60

상 많 언 다 뿐만 니 동 생들

생들에 개별 고 보다 체 계

언 는 경 보 다 들어 동

생들 상 체 연못 럼 보 어ldquo 같rdquo

체 맥 언 시 었지만

생들 상 어 같 큰 고 가 쪽 움ldquo

직 어 같 역 고rdquo

언 시 다 경 변 시킨 그

림 시 동 생들 경 변

지만 생들 경 변 거

리지 못 다

년 학 간고사 대비2013 2 현대고 대비

ECN-0102-2013-001-000076193

게 볼 동 들 보다는 큰 그( )

림 보 에 사 과 체 맥 연결시 지각

는 경 고 체에 특 떼어 내

어 독립 보는 것 낯 어 다 에

들 사 에 고 주변 맥 에는 심 경

에 사건과 사건 사 계에 상

민감 편 다

마 지 지( ) 들만 상 연

보편 결 못 것 도 다lsquo rsquo 지각 과

과 지 과 어 보편 고 어

에 달 지는지는 많 연 통 여

어 다

리 드 니 벳 생각 지도 사- ldquo rdquo( 2004)

zb41) 의 하는 가~ 다른 것은

① ② ③

④ ⑤

얼마 그 에 동 사고 식과

사고 식 보여 주는 내 다

들 에 는 탕 고 같 게

어 겨 고 미 에 는 그 크 럼 큰 고

어리 주고 원 는 어 도 는

상 고 생각 다는 것 다 러

는 어떻게 생 것 고 과 그리 거슬

러 가 보 그 단 다

고 연 경 체 경 생 에

다 벼 사는 공동 업과 경험 많 연 역

에 고 들 연 웃과

게 지내 고 탁 연 들

들 지 연 럽게 들 다 민들

웃과 동 게 뿐만 니 는 집 과

게 다

동 시 는 생태 경 에 살 결과

들 다 사 들 사 상 에 주

울 게 었고 는 곧 체 상 과 간 사

계 시 는 낳게 었다 신 가

가 는 체에 는 원 는 동시

에 다 사 들 그 사 포 체 맥 에

다 들 간 사 연

계 체 계에 주 울 는 사고 체계

게 었다

그러 그리 연 경 그 었다 산

지 연결 는 지 건 그리고 역

에 다 런 들 업에 다 사 과

동 므 공동체에

다고 다 고 그리 들

들과는 달리 보 내 감 지 들과

지 크게 느 지 못 다 그

견 다 경우 주 쟁 통 결 는 갖

게 었다

신 사 간 계들 루어진 커다

트워크 에 게 당연 사 역시 연

계들 체 식 게 다 어 상

원 도 그 개체가 체 맥 과

계 에 고 다 게 체 맥 에 주

울 다 보 상 복 과 가변 식 게 고

상에 재 는 많 변 들 사 에 재 는 들도

게 다 들 주 태도 보

는 경우가 많다 쟁 결

통 결 보다는 통 결

는 보 다

그러 고 그리 들 개개 사 사 독

에 주 울 다 사 사 체에

어 그들 사 에 재 는 공통 규 주

고 다 상 원 에도 사

체 내 주 고 다 그들

체 여 탕 체

는 주 태도 시 고 특 사 어

주에 는지 여 그 주에 는 규

견 다 에 는 쟁 식 리

같 리 사고 체계가 달 게 었다

리 드 니 벳 생각 지도 사- ldquo rdquo( 2004)

zb42) 위 에서 사 된 설 과 가장 유사한 것은

① 크톱 컴퓨 는 본체 니 마우 루

어 다

② 곡과 시 리 는 지 과 사 루어 다는 공통

지니고 다

③ 경 고 것과는 달리

경 본 연 태 그 주변 경

④ 벽돌 능 에 사계 내내

습도가 지 다

⑤ 잰느 체 체 지닌 재 체가 없

는 재 눌 다

년 학 간고사 대비2013 2 현대고 대비

ECN-0102-2013-001-000076193

zb43) 는 립 앙 도서 이 정의 일 이다lt gt

도서 장과 이 자의 리 의 정의 연결이

적절하 않은 것은

lt gt

제 조 서 유8 ( )

도서 장은 다른 이 자의 안전을 위협하거나 도서 의①

서를 란하게 할 가 있는 자에 대하여는 도서 출입

을 제한할 있다

도서 장은 이 자가 제 조 각 호의 어느 하나의 행위를 하7②

을 때에는 이 을 하게 하거나 도서 출입을 제한할

있다

제 조자 의 대출9 ( )

도서 자 는 다음 각 호의 경 대출할 있다①

상호대차도서 간에 자 를 류하는 것을 말한다 등 다1 ( )

른 도서 과의 협 을 위하여 필 한 경

공 이 공 행 상 필 하는 경2

에 도서 장이 필 하다고 인정하는 경3

대출이 가능한 도서 자 의 위는 도서 장이 정하는②

에 따른다

제 조 상10 ( )

이 자가 도서 자 설을 더럽히거나 찢거나 뜨①

쓰게 하거나 잃어 린 경 에는 상하여야 한다

도서 장은 제 항에 따른 상 을 정하여 게 하여야1②

한다

제 조이 절차 등11 ( )

이 칙에서 정한 것 에 도서 자 설의 이 절차

이 제한 등에 필 한 사항은 도서 장이 정한다

출처 립 앙 도서- (httpwwwnlgokr)

① 는 도 리 다8

② 도 는 리 다9 1

③ 료 지 는 도 리 다9 2

④ 도 료 변상에 리10 1

⑤ 는 에 도 리 다11

3

도 다 각 같다①

공 공 다만 연1

연 간 다

매월 째 째 월2

도 도 리 그 사3

가 다고 는

도 에 미리 게1 3②

시 여 다

4

도 시간 도 여 게시 다

5

도 료 시 는 는 도①

지에 등 후

등 에 사 도②

7

는 다 각 여 는 니 다

도 료 시 상 리1 lsquo rsquo

도 료 시 훼 는2 middot

지 가 닌 곳에 식 거 담3

우는

도 보 등 보 검색열4 middot

그 에 도 질 지 여 도5

여 게시 사 는

8

도 다 거 도①

질 게 우 가 는 에 여는 도

도 가 각 어느7②

에는 지 게 거 도

9

도 료는 다 각 경우 다①

상 도 간에 료 는 것 말1 (

다 등 다 도 과 여 경우)

공 원 공 상 는 경우2

그 에 도 다고 는 경우3

가능 도 료 는 도②

는 에 다

10

년 학 간고사 대비2013 2 현대고 대비

ECN-0102-2013-001-000076193

가 도 료 시 럽 거 거①

못 쓰게 거 어 린 경우에는 변상 여

도 에 변상 여 게시1②

여 다

zb44) 위 에서 도서 장이 게 해야 할 사항에 해당하는

것을 두 쓰

년 학 간고사 대비2013 2 현대고 대비

ECN-0102-2013-001-000076193

립 도 규

1 ( )

규 립 도 립 어린 청 도(

포 다 료 시 열 시 말) (

다 에 사 규 립 도)

편 진 다

2 ( )

규 립 도 도 다 에( lsquo rsquo )

고 는 도 에 도lsquo rsquo 2 2

료 에 여 다 다만 특 료 귀

료 등 료 에 사 립 도

도 다 다( lsquo rsquo )

3 ( )

도 다 각 같다①

공 공 다만 연1

연 간 다

매월 째 째 월2

도 도 리 그 사3

가 다고 는

도 에 미리 게1 3②

시 여 다

시간4 ( )

도 시간 도 여 게시 다

등 등5 ( )

도 료 시 는 는 도①

지에 등 후

등 에 사 도②

사 료6 ( )

도 료 시 에 사 료는 도

7 ( )

는 다 각 여 는 니 다

도 료 시 상 리1 lsquo rsquo

도 료 시 훼 는2 middot

지 가 닌 곳에 식 거 담3

우는

도 보 등 보 검색열4 middot

그 에 도 질 지 여 도5

여 게시 사 는

질 지8 ( )

도 다 거 도①

질 게 우 가 는 에 여는 도

도 가 각 어느7②

에는 지 게 거 도

료9 ( )

도 료는 다 각 경우 다①

상 도 간에 료 는 것 말1 (

다 등 다 도 과 여 경우)

공 원 공 상 는 경우2

그 에 도 다고 는 경우3

가능 도 료 는 도②

는 에 다

변상10 ( )

가 도 료 시 럽 거 거①

못 쓰게 거 어 린 경우에는 변상 여

도 에 변상 여 게시1②

여 다

등 규 에 것 에 도11 ( )

료 시 등에 사

도 다

립 도- (httpwwwnlgokr)

zb45) 도서 장의 리 있는 조항으 적절하 않

은 것은

① ② ③ ④ ⑤

년 학 간고사 대비2013 2 현대고 대비

ECN-0102-2013-001-000076193

1 ( )

사가 공 는lsquo rsquo

과 여 사 원과 리

사 타 사 규

니다

개 보 보7 ( )

사는 보통신망 등 계 는 에lsquo rsquo lsquo rsquo

원 개 보 보 니다 개lsquo rsquo

보 보 사 에 는 사 개lsquo rsquo

보 취 니다 다만 사는 다 lsquo rsquo

사 계 통 공 는 경우 원 lsquo rsquo

등 개 보 당 사에 습니lsquo rsquo

원 리에8 (lsquo rsquo lsquo rsquo lsquo rsquo

)

원 에 리lsquo rsquo lsquo rsquo lsquo rsquo①

원에게 가 도 여 는lsquo rsquo 3

니다

사는 원 가 개 보 우 가lsquo rsquo lsquo rsquo lsquo rsquo②

거 사 경우 는 미 에 어 거 lsquo

사 사 운 우 가 는 경우 당rsquo lsquo rsquo

습니다lsquo rsquo

원 가 도 거lsquo rsquo lsquo rsquo lsquo rsquo 3③

가 사 고 지 경우에는 시 사에lsquo rsquo

통지 고 사 내에 니다lsquo rsquo

경우에 당 원 사에 그 사실3 lsquo rsquo lsquo rsquo④

통지 지 거 통지 도 사 내에 지 lsquo rsquo

생 경우 사는 지지 습니다lsquo rsquo

사10 (lsquo rsquo )

사는 과 지 미lsquo rsquo①

에 는 지 계 고

공 여 다 여 니다lsquo rsquo

사는 원 게lsquo rsquo lsquo rsquo lsquo rsquo②

도 개 보 신 보 포 보 보 시( )

갖 어 개 보 취 공시 고

니다

사는 과 여 원lsquo rsquo lsquo rsquo③

견 만 당 다고 경우에는

리 여 니다 원 견 만 사 lsquo rsquo

에 는 게시 거 우편 등 통 여

원에게 리 과 결과 달 니다lsquo rsquo

원11 (lsquo rsquo )

원 다 여 는 니다lsquo rsquo ①

신청 는 변경 시 허 내 등1

타 보 도2

사가 게시 보 변경3 lsquo rsquo

사가 보 보 컴퓨 그4 lsquo rsquo (

등 등 신 는 게시)

사 타 등 지 재산 에5 lsquo rsquo 3

사 타 상 거 업6 lsquo rsquo 3

는 폭 시지 상 타 공7 middot middot

에 는 보 에 공개 는 게시 는lsquo rsquo

사 동 없 리 사8 lsquo rsquo

타 거 당9

게시15 (lsquo rsquo )

원 내에 게시 는 게시 게재 는lsquo rsquo lsquo rsquo lsquo rsquo

경우 원 사가 게시 복 lsquo rsquo lsquo rsquo lsquo rsquo middot middot

등 태 언 등에 공 는

것 내에 다 원 본 게시 등 lsquo rsquo lsquo rsquo

크 능 등 여 복 는 등 태

는 것 동 것 니다

- (wwwnavercom)

zb46) 위 은 인터넷 포털사이트의 회 가입을 위한 이

약 의 일 이다 이 약 을 만드는 과정에서 생각한

내 으 적절하 않은 것은

개 보 보 가 지에 별 눠①

겠어

원 가 만들게 에②

시 주어 겠어

원들 게재 게시 다 원 크 다③

는 것 지

④ 원 지 는 뿐만 니 사가 지 는

도 께 달 지

리에 가 생 경우 사가⑤

에 다는 도 듯

1 ( )

사가 공 는lsquo rsquo

과 여 사 원과 리

사 타 사 규

년 학 간고사 대비2013 2 현대고 대비

ECN-0102-2013-001-000076193

니다

개 보 보7 ( )

사는 보통신망 등 계 는 에lsquo rsquo lsquo rsquo

원 개 보 보 니다 개lsquo rsquo

보 보 사 에 는 사 개lsquo rsquo

보 취 니다 다만 사는 다 lsquo rsquo

사 계 통 공 는 경우 원 lsquo rsquo

등 개 보 당 사에 습니lsquo rsquo

원 리에8 (lsquo rsquo lsquo rsquo lsquo rsquo

)

원 에 리lsquo rsquo lsquo rsquo lsquo rsquo①

원에게 가 도 여 는lsquo rsquo 3

니다

사는 원 가 개 보 우 가lsquo rsquo lsquo rsquo lsquo rsquo②

거 사 경우 는 미 에 어 거 lsquo

사 사 운 우 가 는 경우 당rsquo lsquo rsquo

습니다lsquo rsquo

원 가 도 거lsquo rsquo lsquo rsquo lsquo rsquo 3③

가 사 고 지 경우에는 시 사에lsquo rsquo

통지 고 사 내에 니다lsquo rsquo

경우에 당 원 사에 그 사실3 lsquo rsquo lsquo rsquo④

통지 지 거 통지 도 사 내에 지 lsquo rsquo

생 경우 사는 지지 습니다lsquo rsquo

원에 통지9 (lsquo rsquo )

사는 특 다 원에게 통지 경우lsquo rsquo lsquo rsquo

공지 게시 통 상 게시 개별 통지에7

갈 습니다

사10 (lsquo rsquo )

사는 과 지 미lsquo rsquo①

에 는 지 계 고

공 여 다 여 니다lsquo rsquo

사는 원 게lsquo rsquo lsquo rsquo lsquo rsquo②

도 개 보 신 보 포 보 보 시( )

갖 어 개 보 취 공시 고

니다

사는 과 여 원lsquo rsquo lsquo rsquo③

견 만 당 다고 경우에는

리 여 니다 원 견 만 사 lsquo rsquo

에 는 게시 거 우편 등 통 여

원에게 리 과 결과 달 니다lsquo rsquo

원11 (lsquo rsquo )

원 다 여 는 니다lsquo rsquo ①

신청 는 변경 시 허 내 등1

타 보 도2

사가 게시 보 변경3 lsquo rsquo

사가 보 보 컴퓨 그4 lsquo rsquo (

등 등 신 는 게시)

사 타 등 지 재산 에5 lsquo rsquo 3

사 타 상 거 업6 lsquo rsquo 3

는 폭 시지 상 타 공7 middot middot

에 는 보 에 공개 는 게시 는lsquo rsquo

사 동 없 리 사8 lsquo rsquo

타 거 당9

원 계 규 내lsquo rsquo lsquo②

여 공지 주 사 사가 통지 는rsquo lsquo rsquo

사 등 여 타 사 업 에 lsquo rsquo

는 여 는 니다

- (wwwnavercom)

zb47) 위 약 의 조항에서 같은 제점을 하lt gt

고 있는 조항은

lt gt

제휴 회사에 회 의 아이디 개인 정 를 전송할 있도

한 조항은 고객에게 당한 조항이다

1 7 8① ② ③

④ 9 ⑤ 10

립 도 규

1 ( )

규 립 도 립 어린 청 도(

포 다 료 시 열 시 말) (

다 에 사 규 립 도)

편 진 다

2 ( )

규 립 도 도 다 에( lsquo rsquo )

고 는 도 에 도lsquo rsquo 2 2

료 에 여 다 다만 특 료 귀

료 등 료 에 사 립 도

도 다 다( lsquo rsquo )

3 ( )

도 다 각 같다①

공 공 다만 연1

연 간 다

년 학 간고사 대비2013 2 현대고 대비

ECN-0102-2013-001-000076193

매월 째 째 월2

도 도 리 그 사3

가 다고 는

도 에 미리 게1 3②

시 여 다

시간4 ( )

도 시간 도 여 게시 다

등 등5 ( )

도 료 시 는 는 도①

지에 등 후

등 에 사 도②

사 료6 ( )

도 료 시 에 사 료는 도

7 ( )

는 다 각 여 는 니 다

도 료 시 상 리1 lsquo rsquo

도 료 시 훼 는2 middot

지 가 닌 곳에 식 거 담3

우는

도 보 등 보 검색열4 middot

그 에 도 질 지 여 도5

여 게시 사 는

질 지8 ( )

도 다 거 도①

질 게 우 가 는 에 여는 도

도 가 각 어느7②

에는 지 게 거 도

료9 ( )

도 료는 다 각 경우 다①

상 도 간에 료 는 것 말1 (

다 등 다 도 과 여 경우)

공 원 공 상 는 경우2

그 에 도 다고 는 경우3

가능 도 료 는 도②

는 에 다

변상10 ( )

가 도 료 시 럽 거 거①

못 쓰게 거 어 린 경우에는 변상 여

도 에 변상 여 게시1②

여 다

등 규 에 것 에 도11 ( )

료 시 등에 사

도 다

립 도- (httpwwwnlgokr)

zb48) 다음 정 리 의 의 으 볼 때 가장

이 적인 것은

도 시간 도 여 게시 다①

등 에 사 도②

가능 도 료 는 도 는③

에 다

④ 도 에 변상 여 게10 1

시 여 다

⑤ 도 가 각 어느7

에는 지 거 도

zb49) 를 참고하여 이 어의 성격을 설 한lt gt

것으 적절하 않은 것은

① 보 에 는 어 시 상 고 어 시lt gt lsquo rsquo

에 보여주고 다

② 진 어 어원에 견 고 다

에는 타 어 들어가는 것 다 lsquo rsquo

③ 에 들어갈 말 각각 고 어 어 신 어~

들 언어는 질 격 강 통 없었다

④ 시 우리 에 가 었지만 지 계

과 달리 들 통 사 달 어 웠

년 학 간고사 대비2013 2 현대고 대비

ECN-0102-2013-001-000076193

⑤ 크 몽골 만주 공통어가 우리 어 같

계열에 다는 에 사 특 짐

가( )

善化公主主隱 공주님

他密只嫁良置古 몰 결 고

薯童房乙 맛

夜矣卯乙抱遣去如 에 몰 고 가다

( )

始汝 會隱日恚見隱扐 만 에 본

恥隱汝衣淸隱笑 맑 웃

고 시 여 공 크다 만 다[ ] ( ) ( ) ( ) ( )始 汝 會扐

내다 에 보다 견( ) ( )恚 見 다( )隱

럽다 맑다 청 웃( ) ( ) ( ) ( )恥 衣 淸 笑

zb50) 위의 나 를 함 고 음에 답하( ) lt gt

보lt gt

( )素那或云金川 白城郡蛇山人也

운 사산

는 고 다 는( )[ ( ) ] (素那 金川 白城

사산 사 다) ( ) 郡 蛇山

삼 사- lsquo rsquo 47

에 제 된 단어 의 표 리를 조건(1) lt gt ( ) lt gt

에 맞게 서 하

건lt gt

lsquo 었고 었다 태rsquo

에 제 된 단어 동일한 표 리에(2) lt gt ( )

의해 적은 것을 나 에서 찾아 조건 에 맞게 서 하( ) lt gt

건lt gt

에 당 는 각각( ) 개 쓸 것2 단

당 는 가 여러 개 어도 개만 쓸 것 각2

개 과 도 쪽에 개만2 2

드시 지 것( )

과 동 원리 것lsquo 고

과 동 원리 것 다rsquo

태 것

가( )

素那(或云金川) 白城郡蛇山人也

소나 또는 천 이라 한다 는 성 사( ) ( ) ( )素那 金川 白城郡〔 〕

산 사람이다 현대어 풀이( ) ( )蛇山

나( )

紫布岩乎希 회

執音乎手母牛放敎遣 자 손 암쇼 노히 고

吾 不喩慙 伊賜等肹 肹 나 안디 리샤

花 折叱肹 可獻乎理音如 고 것거 도림다

다 향찰은 리말을 리 으 적은 표 이었 만 생( )

은 고 대를 넘 하고 끊어 고 말았다 랜 세

동안 갈고 닦아 체계적이었던 향찰 표 이 사라졌

을 인은 크게 두 가 나누어 생각해 볼 있다

하나는 족 사회의 한 선호도에서 찾을 있다 라 때

향찰은 주 족 계 에서 사 했을 것으 인다 한 을

알 하고서는 한자를 활 하여 리말을 리 으 표

하 란 가능하 때 이다 런데 족들은 간이 흐

를 향찰과 같은 리 표 을 익혀 사 하 다는

아 한 을 대 사 하는 쪽을 선호하게 되었다 더 이

고 초에 인재 등 을 위해 과거제도가 행되 서 한 선

호도가 더 높아졌고 결 향찰은 소 되고 말았다

또 다른 가능성은 한 어의 특성에서 찾을 있다

터 한 과 일 세 나라는 한자 화 에 속해 다

당연한 이야 겠 만 표의 자인 한자는 어를 표 하

에 매 적절하다 어의 음절은 성 ( ) ( )聲母 韻母

이 어 고 여 에 성조가 추가되어 최종 소리가 결정된

다 래서 어는 단음절을 하나의 한자 표 하 된

다 에 초성 성 종성의 세 가 소가 하나의 음절

년 학 간고사 대비2013 2 현대고 대비

ECN-0102-2013-001-000076193

을 이 는 한 어는 음절 조가 잡하고 음절의 가 많아

서 한자 차 만으 한 어의 소리를 만족 럽게 표 할

없었다 를 들어 한 어에서는 어 니 같이 음절 lsquo rsquo

이 어 단어가 얼마든 있으나 어는( ) 複數音節

자 하나 나타내 만이다lsquo [m ]rsquo 母 ǔ

한편 일 어의 표 은 핵 적 단어는 한자 적고 토는

가나라는 일 의 자 적는 이다 적인 의 를 나

타내는 은 표의 자인 한자 적고 적 계를 나

타내는 토는 표음 자 적는 셈이니 자세히 살펴

리의 향찰 표 을 쏙 빼닮았음을 알 있다 한 어 같

은 착어이 서도 일 어에만 향찰과 유사한 표 이 살아

남은 것은 일 어의 특 때 이다 일 어는 하나의 자음과

음의 결합으 음절을 이 고 침이 거의 없는 음절 언어

이다 이러한 음절의 특색에다가 토가 달한 착어라는 점

이 향찰과 유사한 표 이 살아남을 있는 비결이었다

하 만 같은 착어라도 다양한 음소 침이 달한 한

어는 향찰 표 하는 데 근 적으 한계가 있었다

zb51) 다 하여 의 행에 대한 탐 한 결과( ) lt gt 2

않은 것은

보lt gt

善花公主主隱 공주니믄 공주님( )

----------------------------------------

-

他密只嫁良置古 그 지 얼어 고 몰 결(

----------------------------------------

-

薯童房乙 맛 맛( )

夜矣卯乙抱遺去如 몰 고 가다 에 몰 고(

가다)

주동 역 동- (薯童謠『 』

에 2 ( )他密只嫁良置古

얼다 시집가다 결 다 말 lsquo rsquo

① 실질 미 지니고 므 타 타lsquo ( )rsquo lsquo [ ]

② 에 실질 미 타내고 지 는lsquo rsquo lsquo [ ]rsquo lsquo [ ]密只 密 只

계 타내는

③ 얼어는 실질 미 포 고 므 가lsquo rsquo lsquo [ ]rsquo嫁

것lsquo [ ]rsquo 良

④ 고 어간 는 실질 미 지니고 므lsquo rsquo lsquo -rsquo

것lsquo [ ]rsquo 置

⑤ 고 어미 고는 계 타내고 므lsquo rsquo lsquo- rsquo

고 것lsquo [ ]rsquo 古

가( )

엉 훈 민middot middot middot middot middot世 宗 御 製 訓 民 正 音

말 미 듕 귁에 달middot middot middot middot middot middot middot middot中 國 文 字

니 런middot middot middot middot middot middot 어린middot middot middot middot百 姓

니 고 도 내 들middot middot middot middot middot middot middot middot middot 시러middot

펴 몯middot 미middot middot 니 내middot middot middot middot middot middot middot middot 爲

어엿middot 겨 새middot middot middot 믈여듧middot middot middot middot字 니middot middot middot

사 마다 니겨 킈 middot middot middot middot middot middot middot middot middot便 安

고 미니middot middot middot middot

본 는 상( ) (象

원리에 만들어진 본) ( )形 ㄱ ㄴ ㅁ ㅅ ㅇ

에 는 가 원리에( )加劃

그리고( )ㅋ ㄷ ㅌ ㅂ ㅍ ㅈ ㅊ ㆆ ㅎ

쓰는 병 원리에 만들어진( )竝書

마지막 체( ) ( )異體ㄲ ㄸ ㅃ ㅆ ㅉ ㆅ

ᅀ 다 상 원리에 ㅇ ㄹ

지 는 삼재 상 본 본( ) ( ) ( 天地人 三才

탕 므림과 림에 ) (初ㅡ ㅣ

재)( ) ( )( )出字 再出字ㅗ ㅏ ㅜ ㅓ ㅛ ㅑ ㅜ ㅕ

병 그리고 들 에 다시( )ㅘ ㅝ ㅣ

( )ㅣ ㅢ ㅚ ㅐ ㅟ ㅔ ㆉ ㅒ ㆌ ㅖ ㅙ ㅞ

zb52) 가 에 대한 설 으 르 않은 것을( ) 두 고르

① 어쓰 규 지키고 다

② 리 고 다

③ 말 미 미 등 어 사 다lsquo rsquo

④ 개 지 다

년 학 간고사 대비2013 2 현대고 대비

ECN-0102-2013-001-000076193

⑤ 어 원 에 가 도 고 다

엉 훈 민世 宗 御 製 訓 民 正 音

말 미 듕귁에 달 니

런 어린 니 고 도middot

내 들 시러 펴 몯 미 니middot

내 어엿 겨 새 믈여듧

사 마다 니겨middot 킈 고

미니

훈민 언 본- lsquo rsquo 5 (1459 )

zb53) 위의 에 대한 현대어 풀이가 르~ 않은 것

① 우리 말 과 달

② 어리 말 고 는 것 어도

③ 신 생각 마 껏 펼 는 사 많다

④ 게 생각 여

⑤ 사 마다 게

zb54) 훈민정음 언해 에는 한 을 창제한 동 가 드러나

있다 훈민정음 창제의 정 과 내 이 잘 연결된 것

① 주 신 말 미 듕귁에 달

② 민 신 내 어 겨

③ 신 뻔 킈 고 미니

④ 실 신 사 마다 니겨

⑤ 귀 신 계 주 는 훈민 신과 거리가

가 엉 훈 민( ) middot middot middot middot middot世 宗 御 製 訓 民 正 音 

말 미 귁에 中 國 달 文 字

니 런 어린 니 百 姓

고 도 내 들 시러 펴 몯

미 니 내 어엿 爲 겨 새

믈여듧 니 사 마다 니 字

겨 킈 고 미니 便 安

훈민 언 본- lsquo ( )rsquo ( ) 5 (1459 )訓民正音 世祖

( )

[ 1 ]

동 룡 샤 마다 복( ) ( ) ( )海東 六龍 天福

시니 고 동( ) ( )古聖 同符 시니

[ 2 ]

매 니 곶 여

미 므 니 그 내 러

가 니

[ 125 ]

우 미리( )千世 샨( )定 에( )漢水北 累仁

누 개 샤 복 업 시니( ) ( ) 開國 卜年

신( )聖神 니 샤도 경 근민 샤 욱( )敬天勤民

드시리 다

님 쇼 산 가( ) ( )洛水 山行

미드니 가

어 가- lsquo ( )rsquo 27龍飛御天歌

다 우리신 니쓰고 다만 만 쓰( )

거 샹 귀쳔 다보게 러 귀

여 쓴 도 신 보 가 고 신 에

말 어 보게 각 에 사 들

고 본 몬 능통 후에

죠 죠 니

드 도 만 공 에 사

드 미 죠 고 고 여 보 죠

보다 얼마가 거시 어신고 니 첫

가 죠 니 죠

민 들 어 신 샹

귀쳔 도보고 어보 가 만 늘

고 폐 에 만쓴 죠 민

도 러보지못 고 보니 그게 엇지

심 니 리 보 가 어 운건 다

니 쳣 말마 지 니 고 그

쓰 에 가 우 지 지

몰 거 본후에 가 어 지

고 그니 쓴편지 쟝 보

년 학 간고사 대비2013 2 현대고 대비

ECN-0102-2013-001-000076193

쓴것보다 듸 보고 그 마 니 쓴 고

어 못

그런고 에 리 과 가

만 쓴 못 민 말만 듯고

고 편 그 못 보니 그사 단

병신 못 다고 그사 식 사

니 만 고 다 과 그사

만 고 다 과 업 사 보다 식 고

죠 도 고 각 과

견 고 실 직 귀쳔 간에 그

고도 다 것 몰 귀죡 보다

사 우리 신 귀쳔 다 업

시 신 보고 과 지 게 랴

시니 샹 귀쳔 간에 우리 신 걸

간 보 새지각과 새 걸 미리

독립신- lsquo (1896)rsquo

zb55) 친 어 나의 제 장( ) 2 매 함축적

의 가 가장 유사한 것은

① 지 눈 내리고 매 득 니 내 여 가

사- lsquo rsquo

② 도 어 리듯 그 게 어 다

주 사- lsquo rsquo

③ 눈 살 다 죽 어 린 과 체 여

눈 새벽 지 도 살 다

눈- lsquo rsquo

④ 삶 근심과 고단 에 돌 거니는 여 거 는

여 리 내린 살가지 에 눈 리 눈 리

택 그 생 에- lsquo rsquo

⑤ 늘 러 고 러

청룡 룡 어 개 루 우

신경림 계- lsquo rsquo

zb56) 친 를 위 가 나 에 나타난A B ( ) ( )

세 어의 특 에 의거하여 세 어 표 하

그 산 고 공 도 맑지만

A

주변에 쓰 리는 어리 사 많다

B

건lt gt

식 가 에 타 어 특징에( ) ( )

거 과 어쓰 는 고 지 말 것

A

B

zb57) 가 의( ) 달 아ㆍ 다 의 ( ) 나셔에서 알 있는

세 어 개화 어의 특 을 비 하여 조건 에lt gt

맞게 서 하

건lt gt

어에 는lsquo 개

어에 는 다 태rsquo

zb58) 은 가 는 다 에 나 는 절lt 1gt ( ) lt 2gt ( )

일 를 췌한 것이다 의 의 가 lt 1gt (1)~(2)

유사한 말을 에서 찾아 쓰lt 2gt

보lt 1gt

런 (1) 어린 니 고百 姓

도 내 들 시러 펴 몯 미

사 마다 (2) 니겨 便 安

킈 고 미니

보lt 2gt

죠 고 고 여 보 죠

보다 얼마가 거시 어신고 니 첫 가

죠 니 죠 민

들 어 신 샹 귀쳔

도보고 어보 가 만 늘 고

폐 에 만쓴 죠 민 도

러보지못 고 보니 그게 엇지 심

니 리

년 학 간고사 대비2013 2 현대고 대비

ECN-0102-2013-001-000076193

lt 1 gt

동 룡 샤 마다 복 시( ) ( ) ( )海東 六龍 天福

고 동 시니( ) ( )古聖 同符

lt 2 gt

(A) 매 니 곶

여 니

미 므 니 그 내

러 가 니

lt125 gt

우 미리 샨 에( ) ( ) ( ) 千世 定 漢水北 累

누 개 샤 복 업 시 니( ) ( ) 仁開國 卜年 聖

신( ) 神 니 샤도 경 근민 샤( ) 敬天勤民

욱 드 시 리 다

님 쇼 산 가 ( ) ( )洛水 山行

미드니 가

- lt gt龍飛御天歌

zb59) 장과 내 상 유사한 성격의 조는125

① 뫼 고 고 고 고

어 그린 많고 많고 고 고

어 러 는 울고 울고 가느니

도 견- lt gt

② 강 에 드니 몸 다

그믈 고 가니

뒷 뫼 엄 언 니( )藥

-

③ 말 없는 청산 태 없는 다

값 없는 청 없는 월

에 병 없는 몸 별 없 늙 리

-

④ 가마귀 골에 가지 마

낸 가마귀 새

청강에 것 시 몸 러 가( ) 淸江

-

⑤ 진 골에( ) 白雪

가 매 는 어느 곳에 었는고

에 갈 곳 몰( ) 夕陽

색-

zb60) 위 에 나타난 세 어의 특 으 적절하 않은

것은

① 룡 어 주격 사에 당 는 가 사( ) lsquo rsquo六龍

고 다

② 샤 어에도 어 주체 쓰 다

는 것 다

③ 매 어 달리 사 택에 어

가 지 지지 고 다

④ 므 원 상 직 어 지 다

⑤ 드시리 다 주체 과 상 께 사

고 다

수고 하셨습니다hearts hearts

년 학 간고사 대비2013 2 현대고 대비

ECN-0102-2013-001-000076193

보닷컴에 공 는 별 보는 고등

들 여 주 는

들 습니다 슷 동 지

가 복 는 것 도가

니 복 여 습 시고 거 시

니다

정답 해설

1) 정답[ ] ④

해설 다른 것은 두 특정 업이나 단 내에서 사[ ]

하는 일종의 은어 사회 언에 해당한다 러나

는 언이 아니라 단과대학을 여서 단대 사lsquo rsquo lsquo rsquo lsquo④

대학을 여서 사대라고 한 말에 해당하 일rsquo lsquo rsquo

사회에서도 널리 쓰이 사회 언이라 할

없다

2) 정답[ ] ⑤

해설 사회 언은 같은 단 내에서 쓰이는 언어이[ ] lsquo rsquo

동일 단끼리는 단결 과 친 감을 형성하는

능을 하 리적 안감이 일어나 않는다

3) 정답[ ] ③

해설 사람이라는 차 적 표현에 대한 대안적 표현이[ ]lsquo rsquo

인 아내 처 등으 볼 있다lsquo rsquo

4) 정답[ ]⑤

해설 남성은 주 격 체를 사 한다[ ]

5) 정답[ ] ⑤

해설 흑인은 검다라는 뜻을 가 고 있을 뿐 인[ ]lsquo rsquo lsquo rsquo lsquo rsquo

다 열등한 뜻을 내포하 않는다

6) 정답 살 색 첫 작품[ ] - -

해설 살색 혹은 킨색은 한 인의 피 색을 뜻[ ] lsquo rsquo lsquo rsquo

하는 것으 인종 차 을 추 고 출 이주민

의 평등 을 침해할 있어 년 표 이2005

살 색으 이름을 꾸었다 처녀작은 처녀라lsquo rsquo lsquo rsquo lsquo rsquo

는 단어가 가 고 있는 곡된 성 인 을 한 것

으 첫 작품정도 꾸어 사 하는 것이 좋다lsquo rsquo

7) 정답[ ] ⑤

해설 호는 아들에게 해체를 사 하고 있다[ ] ① ②

장 을 성하는 청자는 자 의 아 느리 아lsquo

들 세 이다 호는 아 느리에게 해rsquo ③

체를 사 하고 있다 호가 느리 아 에게 ④

사 한 해 체 아들에게 사 한 해체는 두 비lsquo rsquo lsquo rsquo

격 체에 해당한다 호는 자 의 아랫사람인 ⑤

느리에게 아들과 마찬가 해체를 사 하는 것이

상 이 만 임 을 한 느리에게 고마 과 쁨

존 의 표 를 하 위해 자 의 아 에게 말하듯

해 체를 사 하고 있다

8) 정답[ ] ③

9) 정답[ ] ⑤

10) 정답[ ] ①

해설 청자 할아 가 장의 주체 아 다 높을[ ] ( ) ( )

경 에는 압존 에 의해 장의 주체를 높이 않는lsquo rsquo

다 러 아 서가 아닌 아 는으 계 lsquo rsquo lsquo rsquo lsquo

니다 가 아닌 있 니다 표현하는 것이 르rsquo lsquo rsquo

11) 정답 당이 당을 쫒았다 당이[ ]

당에 다

해설[ ]

12) 정답[ ] ⑤

해설 서 다른 높임표현을 통해 청자에 대해 리[ ] ⑤

적 거리감을 나타내는 인 은 이 아니라 현정이

다 가 에서 현정은 에게 해 체를 사 함으 써 ( )

친근감을 드러낸다 나 에서 연 을 게을리하는 역 ( )

도 들 때 에 화가 난 현정이 선생님에게 항의하

는 장 에서는 하 체를 사 하여 리적 거리lsquo rsquo

가 어졌음을 나타내고 있다

13) 정답[ ] ①

해설 는 는 얼 빛이 날과 어찌 다르 고[ ] lsquo rsquo

라는 뜻으 전과 달리 임이 화자를 않고

있음을 알 있다

14) 정답 달리 후 가 있다 이를 통해 경[ ] lt gt

쾌한 음악성을 형성하고 노 젓는 상황을 체적으

형상화하는 역할을 한다

15) 정답[ ] ①

16) 정답[ ] ⑤

해설 다 의 자연은 를 성찰하게 하는 대상[ ] ( )⑤

이자 정의 대상이다 의 자연은 자 의 상황과 ⑤

처 를 드러내는 경으 서의 역할을 하 이

이 없다

17) 정답[ ] ③

해설 는 빈천 을 해결하고자 했으나 강산[ ] lsquo ( )rsquo 貧賤③

과 풍 을 달라는 에 거절하 다고 함으 써 자

연에 대한 애정을 드러내고 있으 는 않는

임에 대한 망을 개에게 전가 켜서 임에 대한 리

을 드러내고 있다

18) 정답[ ] ③

년 학 간고사 대비2013 2 현대고 대비

ECN-0102-2013-001-000076193

19) 정답[ ] ⑤

해설 고상한 음악가의 이름을 리말 꽝 럽[ ]

게 꿈으 써 언어유희를 통해 음을 유 하고 있

다 이는 고상한 척하는 총 를 비꼼으 써 비판적

태도를 드러내는 것이 대상을 꽝 럽게 표현

하여 총 의 허 과 사치를 풍자하고 있다

20) 정답[ ] ⑤

해설 는 작품 속 경에 대한 설 이 드러나는 것이[ ]

서 자의 주 적인 견해가 접적으 드러나는 것이

아니다

21) 정답[ ] ⑤

22) 정답[ ] ②

23) 정답[ ] ④

24) 정답[ ] ①

해설 적강 티프는 주인공의 비 한 출생이나 능[ ] ①

과 이 있는 것으 조정의 능함을 풍자하는lsquo rsquo

것과는 거리가 다

25) 정답 픔 나[ ] ( )

해설 의 음악은 고통 는 사람들을 위 하고 아픔[ ] lsquo rsquo

을 치유해 주는 능을 한다고 할 있다 의 lt gt

픔 도 소 된 이 과 더 어 살아가는 따뜻한 마음lsquo rsquo

을 상 한다

26) 정답[ ] ⑤

해설 에게 선천적으 주어 각 장애라는 역경[ ]

은 의 이라는 가사 연 을 있다lsquo rsquo

27) 정답[ ] ④

해설 는 장 란 선 에게 은 개인적인 인상을[ ]

소녀 장정 등으 표현한 것이다lsquo rsquo

28) 정답[ ] ②

해설 담자가 피 담자의 언어적 표현이나 비언어[ ]②

적 표현 하 독자는 담의 위 나 피

담자의 감정 상태를 알 있다 이를 통해 독자는

담 상황을 더 생생하게 느낄 있고 피 담자

를 더 잘 이해할 있게 된다

29) 정답[ ]③

해설 일상생활과 역도 선 서의 성과에 된 것에서[ ]

역도를 하 서 겪는 어 과 내적 고민으 화제를

전화하 위한 것이다

30) 정답[ ] ①

해설 릿속에 새겨 넣듯 이 억되도 함 세상[ ] ② ③

살이가 힘들고 고생 러 속 하여 자유를 ④

가 없는 고통의 상태를 비유적으 이르는 말

적의 침입을 막 위해 쌓은 축 켜야 할⑤

대상을 비유적으 이르는 말이다

31) 정답[ ] ④

해설 이 의 종류는 전 으 인 사건 경[ ] lsquo

비평을 성 소 삼는다rsquo

32) 정답[ ] ④

해설 근은 삼대독자 태어났음을 에서 확인할[ ]

있다 형제들과의 담은 이뤄 가 없다

33) 정답[ ] ⑤

해설 근은 가난에도 하고 화가를 꿈꾸었다[ ] (3

단 또한 다른 화가 망생들은 정 육을)

위해 상 학 학 해 유학 에 랐 만

근은 다른 을 찾아야 했다 단 세에(5 ) 18

근은 조선 전람회에 입선하 다 단 의(6 )

만종은 인간과 자연이 엮어 가는 경건한 조화 을lsquo rsquo

나타낸다

34) 정답[ ] ①

해설 근이 속에서도 창작활동을 추 않고[ ]

하는 닭은 은 세상과 타협할 르는

근이 세상의 이해를 하 위한 가장 떳떳한 단

이 때 이다

35) 정답[ ] ⑤

해설 전 은 서 자의 주 적인 평이 리는 것이[ ]

만 위 제 은 인 이 살았던 대 사회적 경

을 통해 객 적인 인 의 을 제 하고 있다

36) 정답[ ] ⑤

해설 전 은 인 사건 경 비평이라는[ ] lsquo rsquo⑤

성 이 어져 있다

37) 정답[ ] ①

해설 이 은 동양인과 서양인의 사고 에 차이가[ ]

있다는 것을 대조를 통해 설 하고 있다 또 쓴이

의 제자가 축 경 를 러 가서 경험한 일화를

통해 동양인이 서양인에 비해 주 상황에 더 많은

주의를 인다는 주장을 뒷 침하고 있다

38) 정답[ ] ④

39) 정답[ ] ②

40) 정답[ ] ②

41) 정답[ ] ④

42) 정답[ ] ③

43) 정답[ ] ④

44) 정답 도서 의 휴 일 도서 의 이 간 도서의[ ]

해설 도서 장은 임의 정한 휴 일과 도서 이[ ]

간 도서의 상 등을 게 할 의 가 있다

년 학 간고사 대비2013 2 현대고 대비

ECN-0102-2013-001-000076193

45) 정답[ ] ①

해설 제 조의 정 휴 일 의 휴 일의 사전 게[ ] 3

는 도서 장의 의 조항에 속한다

46) 정답[ ] ①

해설 개인 정 호 의 를 제 하 했 만 항[ ]

나눠서 제 하 않고 대 나열하고 있다

47) 정답[ ] ②

해설 제 조의 내 을 회사는 다른 회사 협[ ] 7 lsquo

계약을 통해 서비 를 제공하는 경 회 의 아이디

등 개인 정 를 해당 회사에 전송할 있다는 내rsquo

이 있으 의 제점을 제 할 있다②

48) 정답[ ] ④

해설 는 도서 장의 의 에 해당하고 나 는 도[ ] ④

서 장의 리에 해당한다

49) 정답[ ] ③

50) 정답 은 음독으 적었고 은 훈독으 적었[ ] (1)

다 과 동일한 표 리 적은 것은 이고 (2) ce

과 동일한 표 리 적은 것은 이다ab

51) 정답[ ] ③

52) 정답[ ] ①②

53) 정답[ ] ③

54) 정답[ ] ③

55) 정답[ ] ①

56) 정답 른 죠코 어린 노 하니라[ ] A B

57) 정답 세 어에서는 활 형이 칙적으[ ] lsquo rsquoㄹㅇ

나타났 만 개화 어에서는 활 형이 쓰 다 lsquo rsquo ㄹㄴ

58) 정답 호 가 흔[ ] (1) (2)

59) 정답[ ] ④

60) 정답[ ] ③

Page 26: 현대고대비 국어 - chamsoriedu.com 「콘텐츠산업진흥 법」외 에도 저작권 의하여 ... 다른주체에게어떤동작을하도록만드는것을나타내는

년 학 간고사 대비2013 2 현대고 대비

ECN-0102-2013-001-000076193

연 질 보고 는 것 다

동 들 주변 상 에 맞 어 동 고

에 다 사 들 태도 동에 보다

많 주 울 다 동 가 미시간

에 에 경험 다 그는 미

식 경 보러 가게 었는 경 체는 매우 재

미 었 주변 들 동에 질 다 그

는 들 계 어 상태 경

다 어 들 에 에 그 시 가 계

가 진 것 다 뒷사 고 지 는 들

동 럼 어 웠다

그는 경험에 어 얻어 동 들lsquo

각도 상 본다 는 가 우고rsquo

검 여 주 단 도 상 실험 실

시 다 그는 실험 가 들에게 컴퓨 통

담 니 보여 주었다

에는 역 는 커다 고 마리가 었

고 주 에는 다 생 들과 갈 거 등

께 시 었다 본 후 가 들

신 본 것 상 보 는 지시 다

그 결과 생들과 동 생

역 고 동 도 언

경 거 다 생 들에 ( )

는 동 생들 생들보다 60

상 많 언 다 뿐만 니 동 생들

생들에 개별 고 보다 체 계

언 는 경 보 다 경 변 시

킨 그림 시 동 생들 경

변 지만 생들 경 변

거 리지 못 다

지 지 들만 상 연 lsquo

보편 결 못 것 도 다 지각 과 과rsquo

지 과 어 보편 고 어

에 달 지는지는 많 연 통 여

어 다

리 드 니 벳 생각 지도 사- ldquo rdquo( 2004)

zb40) 위 에 대한 설 으 가장 적절한 것은

① 동 과 생 식 강 고 다

② 가지 실험 통 쓴 고 다

③ 닥 실험에 사 본질에 동 사

상에 주 다

④ 니 실험에 동 과 에 지

각 도에 가 다

⑤ 쓴 는 보편 연 에 드러 우월 에

에 근 고 다

가 동 들 주변 상 에 맞 어 동 고( )

에 다 사 들 태도 동에 보다 많

주 울 다 동 가 미시간 에

에 경험 다 그는 미식

경 보러 가게 었는 경 체는 매우 재미 었

주변 들 동에 질 다 그 는

들 계 어 상태 경 다

어 들 에 에 그 시 가 계 가

진 것 다 상 살펴lsquo 는 말 들rsquo

에 그는 에 시 어 도 뒷사

생각 곧 다시 곤 것 다 그런 그에게

뒷사 고 지 는 들 동 럼

어 웠다

그는 경험에 어 얻어( ) 동 들lsquo

각도 상 본다 는 가 우고rsquo

검 여 주 단 도 상 실험

실시 다 실험에는 동 생들 여 다

그는 실험 가 들에게 컴퓨 통

담 니 보여 주었다 에는

역 는 커다 고 마리가 었고 주 에는

다 생 들과 갈 거 등 께 시

었다 본 후 가 들 신 본 것

상 보 는 지시 다

다 그 결과 생들과 동 생( )

역 고 동 도 언

경 거 다 생 들 에 ( )

는 동 생들 생들보다 60

상 많 언 다 뿐만 니 동 생들

생들에 개별 고 보다 체 계

언 는 경 보 다 들어 동

생들 상 체 연못 럼 보 어ldquo 같rdquo

체 맥 언 시 었지만

생들 상 어 같 큰 고 가 쪽 움ldquo

직 어 같 역 고rdquo

언 시 다 경 변 시킨 그

림 시 동 생들 경 변

지만 생들 경 변 거

리지 못 다

년 학 간고사 대비2013 2 현대고 대비

ECN-0102-2013-001-000076193

게 볼 동 들 보다는 큰 그( )

림 보 에 사 과 체 맥 연결시 지각

는 경 고 체에 특 떼어 내

어 독립 보는 것 낯 어 다 에

들 사 에 고 주변 맥 에는 심 경

에 사건과 사건 사 계에 상

민감 편 다

마 지 지( ) 들만 상 연

보편 결 못 것 도 다lsquo rsquo 지각 과

과 지 과 어 보편 고 어

에 달 지는지는 많 연 통 여

어 다

리 드 니 벳 생각 지도 사- ldquo rdquo( 2004)

zb41) 의 하는 가~ 다른 것은

① ② ③

④ ⑤

얼마 그 에 동 사고 식과

사고 식 보여 주는 내 다

들 에 는 탕 고 같 게

어 겨 고 미 에 는 그 크 럼 큰 고

어리 주고 원 는 어 도 는

상 고 생각 다는 것 다 러

는 어떻게 생 것 고 과 그리 거슬

러 가 보 그 단 다

고 연 경 체 경 생 에

다 벼 사는 공동 업과 경험 많 연 역

에 고 들 연 웃과

게 지내 고 탁 연 들

들 지 연 럽게 들 다 민들

웃과 동 게 뿐만 니 는 집 과

게 다

동 시 는 생태 경 에 살 결과

들 다 사 들 사 상 에 주

울 게 었고 는 곧 체 상 과 간 사

계 시 는 낳게 었다 신 가

가 는 체에 는 원 는 동시

에 다 사 들 그 사 포 체 맥 에

다 들 간 사 연

계 체 계에 주 울 는 사고 체계

게 었다

그러 그리 연 경 그 었다 산

지 연결 는 지 건 그리고 역

에 다 런 들 업에 다 사 과

동 므 공동체에

다고 다 고 그리 들

들과는 달리 보 내 감 지 들과

지 크게 느 지 못 다 그

견 다 경우 주 쟁 통 결 는 갖

게 었다

신 사 간 계들 루어진 커다

트워크 에 게 당연 사 역시 연

계들 체 식 게 다 어 상

원 도 그 개체가 체 맥 과

계 에 고 다 게 체 맥 에 주

울 다 보 상 복 과 가변 식 게 고

상에 재 는 많 변 들 사 에 재 는 들도

게 다 들 주 태도 보

는 경우가 많다 쟁 결

통 결 보다는 통 결

는 보 다

그러 고 그리 들 개개 사 사 독

에 주 울 다 사 사 체에

어 그들 사 에 재 는 공통 규 주

고 다 상 원 에도 사

체 내 주 고 다 그들

체 여 탕 체

는 주 태도 시 고 특 사 어

주에 는지 여 그 주에 는 규

견 다 에 는 쟁 식 리

같 리 사고 체계가 달 게 었다

리 드 니 벳 생각 지도 사- ldquo rdquo( 2004)

zb42) 위 에서 사 된 설 과 가장 유사한 것은

① 크톱 컴퓨 는 본체 니 마우 루

어 다

② 곡과 시 리 는 지 과 사 루어 다는 공통

지니고 다

③ 경 고 것과는 달리

경 본 연 태 그 주변 경

④ 벽돌 능 에 사계 내내

습도가 지 다

⑤ 잰느 체 체 지닌 재 체가 없

는 재 눌 다

년 학 간고사 대비2013 2 현대고 대비

ECN-0102-2013-001-000076193

zb43) 는 립 앙 도서 이 정의 일 이다lt gt

도서 장과 이 자의 리 의 정의 연결이

적절하 않은 것은

lt gt

제 조 서 유8 ( )

도서 장은 다른 이 자의 안전을 위협하거나 도서 의①

서를 란하게 할 가 있는 자에 대하여는 도서 출입

을 제한할 있다

도서 장은 이 자가 제 조 각 호의 어느 하나의 행위를 하7②

을 때에는 이 을 하게 하거나 도서 출입을 제한할

있다

제 조자 의 대출9 ( )

도서 자 는 다음 각 호의 경 대출할 있다①

상호대차도서 간에 자 를 류하는 것을 말한다 등 다1 ( )

른 도서 과의 협 을 위하여 필 한 경

공 이 공 행 상 필 하는 경2

에 도서 장이 필 하다고 인정하는 경3

대출이 가능한 도서 자 의 위는 도서 장이 정하는②

에 따른다

제 조 상10 ( )

이 자가 도서 자 설을 더럽히거나 찢거나 뜨①

쓰게 하거나 잃어 린 경 에는 상하여야 한다

도서 장은 제 항에 따른 상 을 정하여 게 하여야1②

한다

제 조이 절차 등11 ( )

이 칙에서 정한 것 에 도서 자 설의 이 절차

이 제한 등에 필 한 사항은 도서 장이 정한다

출처 립 앙 도서- (httpwwwnlgokr)

① 는 도 리 다8

② 도 는 리 다9 1

③ 료 지 는 도 리 다9 2

④ 도 료 변상에 리10 1

⑤ 는 에 도 리 다11

3

도 다 각 같다①

공 공 다만 연1

연 간 다

매월 째 째 월2

도 도 리 그 사3

가 다고 는

도 에 미리 게1 3②

시 여 다

4

도 시간 도 여 게시 다

5

도 료 시 는 는 도①

지에 등 후

등 에 사 도②

7

는 다 각 여 는 니 다

도 료 시 상 리1 lsquo rsquo

도 료 시 훼 는2 middot

지 가 닌 곳에 식 거 담3

우는

도 보 등 보 검색열4 middot

그 에 도 질 지 여 도5

여 게시 사 는

8

도 다 거 도①

질 게 우 가 는 에 여는 도

도 가 각 어느7②

에는 지 게 거 도

9

도 료는 다 각 경우 다①

상 도 간에 료 는 것 말1 (

다 등 다 도 과 여 경우)

공 원 공 상 는 경우2

그 에 도 다고 는 경우3

가능 도 료 는 도②

는 에 다

10

년 학 간고사 대비2013 2 현대고 대비

ECN-0102-2013-001-000076193

가 도 료 시 럽 거 거①

못 쓰게 거 어 린 경우에는 변상 여

도 에 변상 여 게시1②

여 다

zb44) 위 에서 도서 장이 게 해야 할 사항에 해당하는

것을 두 쓰

년 학 간고사 대비2013 2 현대고 대비

ECN-0102-2013-001-000076193

립 도 규

1 ( )

규 립 도 립 어린 청 도(

포 다 료 시 열 시 말) (

다 에 사 규 립 도)

편 진 다

2 ( )

규 립 도 도 다 에( lsquo rsquo )

고 는 도 에 도lsquo rsquo 2 2

료 에 여 다 다만 특 료 귀

료 등 료 에 사 립 도

도 다 다( lsquo rsquo )

3 ( )

도 다 각 같다①

공 공 다만 연1

연 간 다

매월 째 째 월2

도 도 리 그 사3

가 다고 는

도 에 미리 게1 3②

시 여 다

시간4 ( )

도 시간 도 여 게시 다

등 등5 ( )

도 료 시 는 는 도①

지에 등 후

등 에 사 도②

사 료6 ( )

도 료 시 에 사 료는 도

7 ( )

는 다 각 여 는 니 다

도 료 시 상 리1 lsquo rsquo

도 료 시 훼 는2 middot

지 가 닌 곳에 식 거 담3

우는

도 보 등 보 검색열4 middot

그 에 도 질 지 여 도5

여 게시 사 는

질 지8 ( )

도 다 거 도①

질 게 우 가 는 에 여는 도

도 가 각 어느7②

에는 지 게 거 도

료9 ( )

도 료는 다 각 경우 다①

상 도 간에 료 는 것 말1 (

다 등 다 도 과 여 경우)

공 원 공 상 는 경우2

그 에 도 다고 는 경우3

가능 도 료 는 도②

는 에 다

변상10 ( )

가 도 료 시 럽 거 거①

못 쓰게 거 어 린 경우에는 변상 여

도 에 변상 여 게시1②

여 다

등 규 에 것 에 도11 ( )

료 시 등에 사

도 다

립 도- (httpwwwnlgokr)

zb45) 도서 장의 리 있는 조항으 적절하 않

은 것은

① ② ③ ④ ⑤

년 학 간고사 대비2013 2 현대고 대비

ECN-0102-2013-001-000076193

1 ( )

사가 공 는lsquo rsquo

과 여 사 원과 리

사 타 사 규

니다

개 보 보7 ( )

사는 보통신망 등 계 는 에lsquo rsquo lsquo rsquo

원 개 보 보 니다 개lsquo rsquo

보 보 사 에 는 사 개lsquo rsquo

보 취 니다 다만 사는 다 lsquo rsquo

사 계 통 공 는 경우 원 lsquo rsquo

등 개 보 당 사에 습니lsquo rsquo

원 리에8 (lsquo rsquo lsquo rsquo lsquo rsquo

)

원 에 리lsquo rsquo lsquo rsquo lsquo rsquo①

원에게 가 도 여 는lsquo rsquo 3

니다

사는 원 가 개 보 우 가lsquo rsquo lsquo rsquo lsquo rsquo②

거 사 경우 는 미 에 어 거 lsquo

사 사 운 우 가 는 경우 당rsquo lsquo rsquo

습니다lsquo rsquo

원 가 도 거lsquo rsquo lsquo rsquo lsquo rsquo 3③

가 사 고 지 경우에는 시 사에lsquo rsquo

통지 고 사 내에 니다lsquo rsquo

경우에 당 원 사에 그 사실3 lsquo rsquo lsquo rsquo④

통지 지 거 통지 도 사 내에 지 lsquo rsquo

생 경우 사는 지지 습니다lsquo rsquo

사10 (lsquo rsquo )

사는 과 지 미lsquo rsquo①

에 는 지 계 고

공 여 다 여 니다lsquo rsquo

사는 원 게lsquo rsquo lsquo rsquo lsquo rsquo②

도 개 보 신 보 포 보 보 시( )

갖 어 개 보 취 공시 고

니다

사는 과 여 원lsquo rsquo lsquo rsquo③

견 만 당 다고 경우에는

리 여 니다 원 견 만 사 lsquo rsquo

에 는 게시 거 우편 등 통 여

원에게 리 과 결과 달 니다lsquo rsquo

원11 (lsquo rsquo )

원 다 여 는 니다lsquo rsquo ①

신청 는 변경 시 허 내 등1

타 보 도2

사가 게시 보 변경3 lsquo rsquo

사가 보 보 컴퓨 그4 lsquo rsquo (

등 등 신 는 게시)

사 타 등 지 재산 에5 lsquo rsquo 3

사 타 상 거 업6 lsquo rsquo 3

는 폭 시지 상 타 공7 middot middot

에 는 보 에 공개 는 게시 는lsquo rsquo

사 동 없 리 사8 lsquo rsquo

타 거 당9

게시15 (lsquo rsquo )

원 내에 게시 는 게시 게재 는lsquo rsquo lsquo rsquo lsquo rsquo

경우 원 사가 게시 복 lsquo rsquo lsquo rsquo lsquo rsquo middot middot

등 태 언 등에 공 는

것 내에 다 원 본 게시 등 lsquo rsquo lsquo rsquo

크 능 등 여 복 는 등 태

는 것 동 것 니다

- (wwwnavercom)

zb46) 위 은 인터넷 포털사이트의 회 가입을 위한 이

약 의 일 이다 이 약 을 만드는 과정에서 생각한

내 으 적절하 않은 것은

개 보 보 가 지에 별 눠①

겠어

원 가 만들게 에②

시 주어 겠어

원들 게재 게시 다 원 크 다③

는 것 지

④ 원 지 는 뿐만 니 사가 지 는

도 께 달 지

리에 가 생 경우 사가⑤

에 다는 도 듯

1 ( )

사가 공 는lsquo rsquo

과 여 사 원과 리

사 타 사 규

년 학 간고사 대비2013 2 현대고 대비

ECN-0102-2013-001-000076193

니다

개 보 보7 ( )

사는 보통신망 등 계 는 에lsquo rsquo lsquo rsquo

원 개 보 보 니다 개lsquo rsquo

보 보 사 에 는 사 개lsquo rsquo

보 취 니다 다만 사는 다 lsquo rsquo

사 계 통 공 는 경우 원 lsquo rsquo

등 개 보 당 사에 습니lsquo rsquo

원 리에8 (lsquo rsquo lsquo rsquo lsquo rsquo

)

원 에 리lsquo rsquo lsquo rsquo lsquo rsquo①

원에게 가 도 여 는lsquo rsquo 3

니다

사는 원 가 개 보 우 가lsquo rsquo lsquo rsquo lsquo rsquo②

거 사 경우 는 미 에 어 거 lsquo

사 사 운 우 가 는 경우 당rsquo lsquo rsquo

습니다lsquo rsquo

원 가 도 거lsquo rsquo lsquo rsquo lsquo rsquo 3③

가 사 고 지 경우에는 시 사에lsquo rsquo

통지 고 사 내에 니다lsquo rsquo

경우에 당 원 사에 그 사실3 lsquo rsquo lsquo rsquo④

통지 지 거 통지 도 사 내에 지 lsquo rsquo

생 경우 사는 지지 습니다lsquo rsquo

원에 통지9 (lsquo rsquo )

사는 특 다 원에게 통지 경우lsquo rsquo lsquo rsquo

공지 게시 통 상 게시 개별 통지에7

갈 습니다

사10 (lsquo rsquo )

사는 과 지 미lsquo rsquo①

에 는 지 계 고

공 여 다 여 니다lsquo rsquo

사는 원 게lsquo rsquo lsquo rsquo lsquo rsquo②

도 개 보 신 보 포 보 보 시( )

갖 어 개 보 취 공시 고

니다

사는 과 여 원lsquo rsquo lsquo rsquo③

견 만 당 다고 경우에는

리 여 니다 원 견 만 사 lsquo rsquo

에 는 게시 거 우편 등 통 여

원에게 리 과 결과 달 니다lsquo rsquo

원11 (lsquo rsquo )

원 다 여 는 니다lsquo rsquo ①

신청 는 변경 시 허 내 등1

타 보 도2

사가 게시 보 변경3 lsquo rsquo

사가 보 보 컴퓨 그4 lsquo rsquo (

등 등 신 는 게시)

사 타 등 지 재산 에5 lsquo rsquo 3

사 타 상 거 업6 lsquo rsquo 3

는 폭 시지 상 타 공7 middot middot

에 는 보 에 공개 는 게시 는lsquo rsquo

사 동 없 리 사8 lsquo rsquo

타 거 당9

원 계 규 내lsquo rsquo lsquo②

여 공지 주 사 사가 통지 는rsquo lsquo rsquo

사 등 여 타 사 업 에 lsquo rsquo

는 여 는 니다

- (wwwnavercom)

zb47) 위 약 의 조항에서 같은 제점을 하lt gt

고 있는 조항은

lt gt

제휴 회사에 회 의 아이디 개인 정 를 전송할 있도

한 조항은 고객에게 당한 조항이다

1 7 8① ② ③

④ 9 ⑤ 10

립 도 규

1 ( )

규 립 도 립 어린 청 도(

포 다 료 시 열 시 말) (

다 에 사 규 립 도)

편 진 다

2 ( )

규 립 도 도 다 에( lsquo rsquo )

고 는 도 에 도lsquo rsquo 2 2

료 에 여 다 다만 특 료 귀

료 등 료 에 사 립 도

도 다 다( lsquo rsquo )

3 ( )

도 다 각 같다①

공 공 다만 연1

연 간 다

년 학 간고사 대비2013 2 현대고 대비

ECN-0102-2013-001-000076193

매월 째 째 월2

도 도 리 그 사3

가 다고 는

도 에 미리 게1 3②

시 여 다

시간4 ( )

도 시간 도 여 게시 다

등 등5 ( )

도 료 시 는 는 도①

지에 등 후

등 에 사 도②

사 료6 ( )

도 료 시 에 사 료는 도

7 ( )

는 다 각 여 는 니 다

도 료 시 상 리1 lsquo rsquo

도 료 시 훼 는2 middot

지 가 닌 곳에 식 거 담3

우는

도 보 등 보 검색열4 middot

그 에 도 질 지 여 도5

여 게시 사 는

질 지8 ( )

도 다 거 도①

질 게 우 가 는 에 여는 도

도 가 각 어느7②

에는 지 게 거 도

료9 ( )

도 료는 다 각 경우 다①

상 도 간에 료 는 것 말1 (

다 등 다 도 과 여 경우)

공 원 공 상 는 경우2

그 에 도 다고 는 경우3

가능 도 료 는 도②

는 에 다

변상10 ( )

가 도 료 시 럽 거 거①

못 쓰게 거 어 린 경우에는 변상 여

도 에 변상 여 게시1②

여 다

등 규 에 것 에 도11 ( )

료 시 등에 사

도 다

립 도- (httpwwwnlgokr)

zb48) 다음 정 리 의 의 으 볼 때 가장

이 적인 것은

도 시간 도 여 게시 다①

등 에 사 도②

가능 도 료 는 도 는③

에 다

④ 도 에 변상 여 게10 1

시 여 다

⑤ 도 가 각 어느7

에는 지 거 도

zb49) 를 참고하여 이 어의 성격을 설 한lt gt

것으 적절하 않은 것은

① 보 에 는 어 시 상 고 어 시lt gt lsquo rsquo

에 보여주고 다

② 진 어 어원에 견 고 다

에는 타 어 들어가는 것 다 lsquo rsquo

③ 에 들어갈 말 각각 고 어 어 신 어~

들 언어는 질 격 강 통 없었다

④ 시 우리 에 가 었지만 지 계

과 달리 들 통 사 달 어 웠

년 학 간고사 대비2013 2 현대고 대비

ECN-0102-2013-001-000076193

⑤ 크 몽골 만주 공통어가 우리 어 같

계열에 다는 에 사 특 짐

가( )

善化公主主隱 공주님

他密只嫁良置古 몰 결 고

薯童房乙 맛

夜矣卯乙抱遣去如 에 몰 고 가다

( )

始汝 會隱日恚見隱扐 만 에 본

恥隱汝衣淸隱笑 맑 웃

고 시 여 공 크다 만 다[ ] ( ) ( ) ( ) ( )始 汝 會扐

내다 에 보다 견( ) ( )恚 見 다( )隱

럽다 맑다 청 웃( ) ( ) ( ) ( )恥 衣 淸 笑

zb50) 위의 나 를 함 고 음에 답하( ) lt gt

보lt gt

( )素那或云金川 白城郡蛇山人也

운 사산

는 고 다 는( )[ ( ) ] (素那 金川 白城

사산 사 다) ( ) 郡 蛇山

삼 사- lsquo rsquo 47

에 제 된 단어 의 표 리를 조건(1) lt gt ( ) lt gt

에 맞게 서 하

건lt gt

lsquo 었고 었다 태rsquo

에 제 된 단어 동일한 표 리에(2) lt gt ( )

의해 적은 것을 나 에서 찾아 조건 에 맞게 서 하( ) lt gt

건lt gt

에 당 는 각각( ) 개 쓸 것2 단

당 는 가 여러 개 어도 개만 쓸 것 각2

개 과 도 쪽에 개만2 2

드시 지 것( )

과 동 원리 것lsquo 고

과 동 원리 것 다rsquo

태 것

가( )

素那(或云金川) 白城郡蛇山人也

소나 또는 천 이라 한다 는 성 사( ) ( ) ( )素那 金川 白城郡〔 〕

산 사람이다 현대어 풀이( ) ( )蛇山

나( )

紫布岩乎希 회

執音乎手母牛放敎遣 자 손 암쇼 노히 고

吾 不喩慙 伊賜等肹 肹 나 안디 리샤

花 折叱肹 可獻乎理音如 고 것거 도림다

다 향찰은 리말을 리 으 적은 표 이었 만 생( )

은 고 대를 넘 하고 끊어 고 말았다 랜 세

동안 갈고 닦아 체계적이었던 향찰 표 이 사라졌

을 인은 크게 두 가 나누어 생각해 볼 있다

하나는 족 사회의 한 선호도에서 찾을 있다 라 때

향찰은 주 족 계 에서 사 했을 것으 인다 한 을

알 하고서는 한자를 활 하여 리말을 리 으 표

하 란 가능하 때 이다 런데 족들은 간이 흐

를 향찰과 같은 리 표 을 익혀 사 하 다는

아 한 을 대 사 하는 쪽을 선호하게 되었다 더 이

고 초에 인재 등 을 위해 과거제도가 행되 서 한 선

호도가 더 높아졌고 결 향찰은 소 되고 말았다

또 다른 가능성은 한 어의 특성에서 찾을 있다

터 한 과 일 세 나라는 한자 화 에 속해 다

당연한 이야 겠 만 표의 자인 한자는 어를 표 하

에 매 적절하다 어의 음절은 성 ( ) ( )聲母 韻母

이 어 고 여 에 성조가 추가되어 최종 소리가 결정된

다 래서 어는 단음절을 하나의 한자 표 하 된

다 에 초성 성 종성의 세 가 소가 하나의 음절

년 학 간고사 대비2013 2 현대고 대비

ECN-0102-2013-001-000076193

을 이 는 한 어는 음절 조가 잡하고 음절의 가 많아

서 한자 차 만으 한 어의 소리를 만족 럽게 표 할

없었다 를 들어 한 어에서는 어 니 같이 음절 lsquo rsquo

이 어 단어가 얼마든 있으나 어는( ) 複數音節

자 하나 나타내 만이다lsquo [m ]rsquo 母 ǔ

한편 일 어의 표 은 핵 적 단어는 한자 적고 토는

가나라는 일 의 자 적는 이다 적인 의 를 나

타내는 은 표의 자인 한자 적고 적 계를 나

타내는 토는 표음 자 적는 셈이니 자세히 살펴

리의 향찰 표 을 쏙 빼닮았음을 알 있다 한 어 같

은 착어이 서도 일 어에만 향찰과 유사한 표 이 살아

남은 것은 일 어의 특 때 이다 일 어는 하나의 자음과

음의 결합으 음절을 이 고 침이 거의 없는 음절 언어

이다 이러한 음절의 특색에다가 토가 달한 착어라는 점

이 향찰과 유사한 표 이 살아남을 있는 비결이었다

하 만 같은 착어라도 다양한 음소 침이 달한 한

어는 향찰 표 하는 데 근 적으 한계가 있었다

zb51) 다 하여 의 행에 대한 탐 한 결과( ) lt gt 2

않은 것은

보lt gt

善花公主主隱 공주니믄 공주님( )

----------------------------------------

-

他密只嫁良置古 그 지 얼어 고 몰 결(

----------------------------------------

-

薯童房乙 맛 맛( )

夜矣卯乙抱遺去如 몰 고 가다 에 몰 고(

가다)

주동 역 동- (薯童謠『 』

에 2 ( )他密只嫁良置古

얼다 시집가다 결 다 말 lsquo rsquo

① 실질 미 지니고 므 타 타lsquo ( )rsquo lsquo [ ]

② 에 실질 미 타내고 지 는lsquo rsquo lsquo [ ]rsquo lsquo [ ]密只 密 只

계 타내는

③ 얼어는 실질 미 포 고 므 가lsquo rsquo lsquo [ ]rsquo嫁

것lsquo [ ]rsquo 良

④ 고 어간 는 실질 미 지니고 므lsquo rsquo lsquo -rsquo

것lsquo [ ]rsquo 置

⑤ 고 어미 고는 계 타내고 므lsquo rsquo lsquo- rsquo

고 것lsquo [ ]rsquo 古

가( )

엉 훈 민middot middot middot middot middot世 宗 御 製 訓 民 正 音

말 미 듕 귁에 달middot middot middot middot middot middot middot middot中 國 文 字

니 런middot middot middot middot middot middot 어린middot middot middot middot百 姓

니 고 도 내 들middot middot middot middot middot middot middot middot middot 시러middot

펴 몯middot 미middot middot 니 내middot middot middot middot middot middot middot middot 爲

어엿middot 겨 새middot middot middot 믈여듧middot middot middot middot字 니middot middot middot

사 마다 니겨 킈 middot middot middot middot middot middot middot middot middot便 安

고 미니middot middot middot middot

본 는 상( ) (象

원리에 만들어진 본) ( )形 ㄱ ㄴ ㅁ ㅅ ㅇ

에 는 가 원리에( )加劃

그리고( )ㅋ ㄷ ㅌ ㅂ ㅍ ㅈ ㅊ ㆆ ㅎ

쓰는 병 원리에 만들어진( )竝書

마지막 체( ) ( )異體ㄲ ㄸ ㅃ ㅆ ㅉ ㆅ

ᅀ 다 상 원리에 ㅇ ㄹ

지 는 삼재 상 본 본( ) ( ) ( 天地人 三才

탕 므림과 림에 ) (初ㅡ ㅣ

재)( ) ( )( )出字 再出字ㅗ ㅏ ㅜ ㅓ ㅛ ㅑ ㅜ ㅕ

병 그리고 들 에 다시( )ㅘ ㅝ ㅣ

( )ㅣ ㅢ ㅚ ㅐ ㅟ ㅔ ㆉ ㅒ ㆌ ㅖ ㅙ ㅞ

zb52) 가 에 대한 설 으 르 않은 것을( ) 두 고르

① 어쓰 규 지키고 다

② 리 고 다

③ 말 미 미 등 어 사 다lsquo rsquo

④ 개 지 다

년 학 간고사 대비2013 2 현대고 대비

ECN-0102-2013-001-000076193

⑤ 어 원 에 가 도 고 다

엉 훈 민世 宗 御 製 訓 民 正 音

말 미 듕귁에 달 니

런 어린 니 고 도middot

내 들 시러 펴 몯 미 니middot

내 어엿 겨 새 믈여듧

사 마다 니겨middot 킈 고

미니

훈민 언 본- lsquo rsquo 5 (1459 )

zb53) 위의 에 대한 현대어 풀이가 르~ 않은 것

① 우리 말 과 달

② 어리 말 고 는 것 어도

③ 신 생각 마 껏 펼 는 사 많다

④ 게 생각 여

⑤ 사 마다 게

zb54) 훈민정음 언해 에는 한 을 창제한 동 가 드러나

있다 훈민정음 창제의 정 과 내 이 잘 연결된 것

① 주 신 말 미 듕귁에 달

② 민 신 내 어 겨

③ 신 뻔 킈 고 미니

④ 실 신 사 마다 니겨

⑤ 귀 신 계 주 는 훈민 신과 거리가

가 엉 훈 민( ) middot middot middot middot middot世 宗 御 製 訓 民 正 音 

말 미 귁에 中 國 달 文 字

니 런 어린 니 百 姓

고 도 내 들 시러 펴 몯

미 니 내 어엿 爲 겨 새

믈여듧 니 사 마다 니 字

겨 킈 고 미니 便 安

훈민 언 본- lsquo ( )rsquo ( ) 5 (1459 )訓民正音 世祖

( )

[ 1 ]

동 룡 샤 마다 복( ) ( ) ( )海東 六龍 天福

시니 고 동( ) ( )古聖 同符 시니

[ 2 ]

매 니 곶 여

미 므 니 그 내 러

가 니

[ 125 ]

우 미리( )千世 샨( )定 에( )漢水北 累仁

누 개 샤 복 업 시니( ) ( ) 開國 卜年

신( )聖神 니 샤도 경 근민 샤 욱( )敬天勤民

드시리 다

님 쇼 산 가( ) ( )洛水 山行

미드니 가

어 가- lsquo ( )rsquo 27龍飛御天歌

다 우리신 니쓰고 다만 만 쓰( )

거 샹 귀쳔 다보게 러 귀

여 쓴 도 신 보 가 고 신 에

말 어 보게 각 에 사 들

고 본 몬 능통 후에

죠 죠 니

드 도 만 공 에 사

드 미 죠 고 고 여 보 죠

보다 얼마가 거시 어신고 니 첫

가 죠 니 죠

민 들 어 신 샹

귀쳔 도보고 어보 가 만 늘

고 폐 에 만쓴 죠 민

도 러보지못 고 보니 그게 엇지

심 니 리 보 가 어 운건 다

니 쳣 말마 지 니 고 그

쓰 에 가 우 지 지

몰 거 본후에 가 어 지

고 그니 쓴편지 쟝 보

년 학 간고사 대비2013 2 현대고 대비

ECN-0102-2013-001-000076193

쓴것보다 듸 보고 그 마 니 쓴 고

어 못

그런고 에 리 과 가

만 쓴 못 민 말만 듯고

고 편 그 못 보니 그사 단

병신 못 다고 그사 식 사

니 만 고 다 과 그사

만 고 다 과 업 사 보다 식 고

죠 도 고 각 과

견 고 실 직 귀쳔 간에 그

고도 다 것 몰 귀죡 보다

사 우리 신 귀쳔 다 업

시 신 보고 과 지 게 랴

시니 샹 귀쳔 간에 우리 신 걸

간 보 새지각과 새 걸 미리

독립신- lsquo (1896)rsquo

zb55) 친 어 나의 제 장( ) 2 매 함축적

의 가 가장 유사한 것은

① 지 눈 내리고 매 득 니 내 여 가

사- lsquo rsquo

② 도 어 리듯 그 게 어 다

주 사- lsquo rsquo

③ 눈 살 다 죽 어 린 과 체 여

눈 새벽 지 도 살 다

눈- lsquo rsquo

④ 삶 근심과 고단 에 돌 거니는 여 거 는

여 리 내린 살가지 에 눈 리 눈 리

택 그 생 에- lsquo rsquo

⑤ 늘 러 고 러

청룡 룡 어 개 루 우

신경림 계- lsquo rsquo

zb56) 친 를 위 가 나 에 나타난A B ( ) ( )

세 어의 특 에 의거하여 세 어 표 하

그 산 고 공 도 맑지만

A

주변에 쓰 리는 어리 사 많다

B

건lt gt

식 가 에 타 어 특징에( ) ( )

거 과 어쓰 는 고 지 말 것

A

B

zb57) 가 의( ) 달 아ㆍ 다 의 ( ) 나셔에서 알 있는

세 어 개화 어의 특 을 비 하여 조건 에lt gt

맞게 서 하

건lt gt

어에 는lsquo 개

어에 는 다 태rsquo

zb58) 은 가 는 다 에 나 는 절lt 1gt ( ) lt 2gt ( )

일 를 췌한 것이다 의 의 가 lt 1gt (1)~(2)

유사한 말을 에서 찾아 쓰lt 2gt

보lt 1gt

런 (1) 어린 니 고百 姓

도 내 들 시러 펴 몯 미

사 마다 (2) 니겨 便 安

킈 고 미니

보lt 2gt

죠 고 고 여 보 죠

보다 얼마가 거시 어신고 니 첫 가

죠 니 죠 민

들 어 신 샹 귀쳔

도보고 어보 가 만 늘 고

폐 에 만쓴 죠 민 도

러보지못 고 보니 그게 엇지 심

니 리

년 학 간고사 대비2013 2 현대고 대비

ECN-0102-2013-001-000076193

lt 1 gt

동 룡 샤 마다 복 시( ) ( ) ( )海東 六龍 天福

고 동 시니( ) ( )古聖 同符

lt 2 gt

(A) 매 니 곶

여 니

미 므 니 그 내

러 가 니

lt125 gt

우 미리 샨 에( ) ( ) ( ) 千世 定 漢水北 累

누 개 샤 복 업 시 니( ) ( ) 仁開國 卜年 聖

신( ) 神 니 샤도 경 근민 샤( ) 敬天勤民

욱 드 시 리 다

님 쇼 산 가 ( ) ( )洛水 山行

미드니 가

- lt gt龍飛御天歌

zb59) 장과 내 상 유사한 성격의 조는125

① 뫼 고 고 고 고

어 그린 많고 많고 고 고

어 러 는 울고 울고 가느니

도 견- lt gt

② 강 에 드니 몸 다

그믈 고 가니

뒷 뫼 엄 언 니( )藥

-

③ 말 없는 청산 태 없는 다

값 없는 청 없는 월

에 병 없는 몸 별 없 늙 리

-

④ 가마귀 골에 가지 마

낸 가마귀 새

청강에 것 시 몸 러 가( ) 淸江

-

⑤ 진 골에( ) 白雪

가 매 는 어느 곳에 었는고

에 갈 곳 몰( ) 夕陽

색-

zb60) 위 에 나타난 세 어의 특 으 적절하 않은

것은

① 룡 어 주격 사에 당 는 가 사( ) lsquo rsquo六龍

고 다

② 샤 어에도 어 주체 쓰 다

는 것 다

③ 매 어 달리 사 택에 어

가 지 지지 고 다

④ 므 원 상 직 어 지 다

⑤ 드시리 다 주체 과 상 께 사

고 다

수고 하셨습니다hearts hearts

년 학 간고사 대비2013 2 현대고 대비

ECN-0102-2013-001-000076193

보닷컴에 공 는 별 보는 고등

들 여 주 는

들 습니다 슷 동 지

가 복 는 것 도가

니 복 여 습 시고 거 시

니다

정답 해설

1) 정답[ ] ④

해설 다른 것은 두 특정 업이나 단 내에서 사[ ]

하는 일종의 은어 사회 언에 해당한다 러나

는 언이 아니라 단과대학을 여서 단대 사lsquo rsquo lsquo rsquo lsquo④

대학을 여서 사대라고 한 말에 해당하 일rsquo lsquo rsquo

사회에서도 널리 쓰이 사회 언이라 할

없다

2) 정답[ ] ⑤

해설 사회 언은 같은 단 내에서 쓰이는 언어이[ ] lsquo rsquo

동일 단끼리는 단결 과 친 감을 형성하는

능을 하 리적 안감이 일어나 않는다

3) 정답[ ] ③

해설 사람이라는 차 적 표현에 대한 대안적 표현이[ ]lsquo rsquo

인 아내 처 등으 볼 있다lsquo rsquo

4) 정답[ ]⑤

해설 남성은 주 격 체를 사 한다[ ]

5) 정답[ ] ⑤

해설 흑인은 검다라는 뜻을 가 고 있을 뿐 인[ ]lsquo rsquo lsquo rsquo lsquo rsquo

다 열등한 뜻을 내포하 않는다

6) 정답 살 색 첫 작품[ ] - -

해설 살색 혹은 킨색은 한 인의 피 색을 뜻[ ] lsquo rsquo lsquo rsquo

하는 것으 인종 차 을 추 고 출 이주민

의 평등 을 침해할 있어 년 표 이2005

살 색으 이름을 꾸었다 처녀작은 처녀라lsquo rsquo lsquo rsquo lsquo rsquo

는 단어가 가 고 있는 곡된 성 인 을 한 것

으 첫 작품정도 꾸어 사 하는 것이 좋다lsquo rsquo

7) 정답[ ] ⑤

해설 호는 아들에게 해체를 사 하고 있다[ ] ① ②

장 을 성하는 청자는 자 의 아 느리 아lsquo

들 세 이다 호는 아 느리에게 해rsquo ③

체를 사 하고 있다 호가 느리 아 에게 ④

사 한 해 체 아들에게 사 한 해체는 두 비lsquo rsquo lsquo rsquo

격 체에 해당한다 호는 자 의 아랫사람인 ⑤

느리에게 아들과 마찬가 해체를 사 하는 것이

상 이 만 임 을 한 느리에게 고마 과 쁨

존 의 표 를 하 위해 자 의 아 에게 말하듯

해 체를 사 하고 있다

8) 정답[ ] ③

9) 정답[ ] ⑤

10) 정답[ ] ①

해설 청자 할아 가 장의 주체 아 다 높을[ ] ( ) ( )

경 에는 압존 에 의해 장의 주체를 높이 않는lsquo rsquo

다 러 아 서가 아닌 아 는으 계 lsquo rsquo lsquo rsquo lsquo

니다 가 아닌 있 니다 표현하는 것이 르rsquo lsquo rsquo

11) 정답 당이 당을 쫒았다 당이[ ]

당에 다

해설[ ]

12) 정답[ ] ⑤

해설 서 다른 높임표현을 통해 청자에 대해 리[ ] ⑤

적 거리감을 나타내는 인 은 이 아니라 현정이

다 가 에서 현정은 에게 해 체를 사 함으 써 ( )

친근감을 드러낸다 나 에서 연 을 게을리하는 역 ( )

도 들 때 에 화가 난 현정이 선생님에게 항의하

는 장 에서는 하 체를 사 하여 리적 거리lsquo rsquo

가 어졌음을 나타내고 있다

13) 정답[ ] ①

해설 는 는 얼 빛이 날과 어찌 다르 고[ ] lsquo rsquo

라는 뜻으 전과 달리 임이 화자를 않고

있음을 알 있다

14) 정답 달리 후 가 있다 이를 통해 경[ ] lt gt

쾌한 음악성을 형성하고 노 젓는 상황을 체적으

형상화하는 역할을 한다

15) 정답[ ] ①

16) 정답[ ] ⑤

해설 다 의 자연은 를 성찰하게 하는 대상[ ] ( )⑤

이자 정의 대상이다 의 자연은 자 의 상황과 ⑤

처 를 드러내는 경으 서의 역할을 하 이

이 없다

17) 정답[ ] ③

해설 는 빈천 을 해결하고자 했으나 강산[ ] lsquo ( )rsquo 貧賤③

과 풍 을 달라는 에 거절하 다고 함으 써 자

연에 대한 애정을 드러내고 있으 는 않는

임에 대한 망을 개에게 전가 켜서 임에 대한 리

을 드러내고 있다

18) 정답[ ] ③

년 학 간고사 대비2013 2 현대고 대비

ECN-0102-2013-001-000076193

19) 정답[ ] ⑤

해설 고상한 음악가의 이름을 리말 꽝 럽[ ]

게 꿈으 써 언어유희를 통해 음을 유 하고 있

다 이는 고상한 척하는 총 를 비꼼으 써 비판적

태도를 드러내는 것이 대상을 꽝 럽게 표현

하여 총 의 허 과 사치를 풍자하고 있다

20) 정답[ ] ⑤

해설 는 작품 속 경에 대한 설 이 드러나는 것이[ ]

서 자의 주 적인 견해가 접적으 드러나는 것이

아니다

21) 정답[ ] ⑤

22) 정답[ ] ②

23) 정답[ ] ④

24) 정답[ ] ①

해설 적강 티프는 주인공의 비 한 출생이나 능[ ] ①

과 이 있는 것으 조정의 능함을 풍자하는lsquo rsquo

것과는 거리가 다

25) 정답 픔 나[ ] ( )

해설 의 음악은 고통 는 사람들을 위 하고 아픔[ ] lsquo rsquo

을 치유해 주는 능을 한다고 할 있다 의 lt gt

픔 도 소 된 이 과 더 어 살아가는 따뜻한 마음lsquo rsquo

을 상 한다

26) 정답[ ] ⑤

해설 에게 선천적으 주어 각 장애라는 역경[ ]

은 의 이라는 가사 연 을 있다lsquo rsquo

27) 정답[ ] ④

해설 는 장 란 선 에게 은 개인적인 인상을[ ]

소녀 장정 등으 표현한 것이다lsquo rsquo

28) 정답[ ] ②

해설 담자가 피 담자의 언어적 표현이나 비언어[ ]②

적 표현 하 독자는 담의 위 나 피

담자의 감정 상태를 알 있다 이를 통해 독자는

담 상황을 더 생생하게 느낄 있고 피 담자

를 더 잘 이해할 있게 된다

29) 정답[ ]③

해설 일상생활과 역도 선 서의 성과에 된 것에서[ ]

역도를 하 서 겪는 어 과 내적 고민으 화제를

전화하 위한 것이다

30) 정답[ ] ①

해설 릿속에 새겨 넣듯 이 억되도 함 세상[ ] ② ③

살이가 힘들고 고생 러 속 하여 자유를 ④

가 없는 고통의 상태를 비유적으 이르는 말

적의 침입을 막 위해 쌓은 축 켜야 할⑤

대상을 비유적으 이르는 말이다

31) 정답[ ] ④

해설 이 의 종류는 전 으 인 사건 경[ ] lsquo

비평을 성 소 삼는다rsquo

32) 정답[ ] ④

해설 근은 삼대독자 태어났음을 에서 확인할[ ]

있다 형제들과의 담은 이뤄 가 없다

33) 정답[ ] ⑤

해설 근은 가난에도 하고 화가를 꿈꾸었다[ ] (3

단 또한 다른 화가 망생들은 정 육을)

위해 상 학 학 해 유학 에 랐 만

근은 다른 을 찾아야 했다 단 세에(5 ) 18

근은 조선 전람회에 입선하 다 단 의(6 )

만종은 인간과 자연이 엮어 가는 경건한 조화 을lsquo rsquo

나타낸다

34) 정답[ ] ①

해설 근이 속에서도 창작활동을 추 않고[ ]

하는 닭은 은 세상과 타협할 르는

근이 세상의 이해를 하 위한 가장 떳떳한 단

이 때 이다

35) 정답[ ] ⑤

해설 전 은 서 자의 주 적인 평이 리는 것이[ ]

만 위 제 은 인 이 살았던 대 사회적 경

을 통해 객 적인 인 의 을 제 하고 있다

36) 정답[ ] ⑤

해설 전 은 인 사건 경 비평이라는[ ] lsquo rsquo⑤

성 이 어져 있다

37) 정답[ ] ①

해설 이 은 동양인과 서양인의 사고 에 차이가[ ]

있다는 것을 대조를 통해 설 하고 있다 또 쓴이

의 제자가 축 경 를 러 가서 경험한 일화를

통해 동양인이 서양인에 비해 주 상황에 더 많은

주의를 인다는 주장을 뒷 침하고 있다

38) 정답[ ] ④

39) 정답[ ] ②

40) 정답[ ] ②

41) 정답[ ] ④

42) 정답[ ] ③

43) 정답[ ] ④

44) 정답 도서 의 휴 일 도서 의 이 간 도서의[ ]

해설 도서 장은 임의 정한 휴 일과 도서 이[ ]

간 도서의 상 등을 게 할 의 가 있다

년 학 간고사 대비2013 2 현대고 대비

ECN-0102-2013-001-000076193

45) 정답[ ] ①

해설 제 조의 정 휴 일 의 휴 일의 사전 게[ ] 3

는 도서 장의 의 조항에 속한다

46) 정답[ ] ①

해설 개인 정 호 의 를 제 하 했 만 항[ ]

나눠서 제 하 않고 대 나열하고 있다

47) 정답[ ] ②

해설 제 조의 내 을 회사는 다른 회사 협[ ] 7 lsquo

계약을 통해 서비 를 제공하는 경 회 의 아이디

등 개인 정 를 해당 회사에 전송할 있다는 내rsquo

이 있으 의 제점을 제 할 있다②

48) 정답[ ] ④

해설 는 도서 장의 의 에 해당하고 나 는 도[ ] ④

서 장의 리에 해당한다

49) 정답[ ] ③

50) 정답 은 음독으 적었고 은 훈독으 적었[ ] (1)

다 과 동일한 표 리 적은 것은 이고 (2) ce

과 동일한 표 리 적은 것은 이다ab

51) 정답[ ] ③

52) 정답[ ] ①②

53) 정답[ ] ③

54) 정답[ ] ③

55) 정답[ ] ①

56) 정답 른 죠코 어린 노 하니라[ ] A B

57) 정답 세 어에서는 활 형이 칙적으[ ] lsquo rsquoㄹㅇ

나타났 만 개화 어에서는 활 형이 쓰 다 lsquo rsquo ㄹㄴ

58) 정답 호 가 흔[ ] (1) (2)

59) 정답[ ] ④

60) 정답[ ] ③

Page 27: 현대고대비 국어 - chamsoriedu.com 「콘텐츠산업진흥 법」외 에도 저작권 의하여 ... 다른주체에게어떤동작을하도록만드는것을나타내는

년 학 간고사 대비2013 2 현대고 대비

ECN-0102-2013-001-000076193

게 볼 동 들 보다는 큰 그( )

림 보 에 사 과 체 맥 연결시 지각

는 경 고 체에 특 떼어 내

어 독립 보는 것 낯 어 다 에

들 사 에 고 주변 맥 에는 심 경

에 사건과 사건 사 계에 상

민감 편 다

마 지 지( ) 들만 상 연

보편 결 못 것 도 다lsquo rsquo 지각 과

과 지 과 어 보편 고 어

에 달 지는지는 많 연 통 여

어 다

리 드 니 벳 생각 지도 사- ldquo rdquo( 2004)

zb41) 의 하는 가~ 다른 것은

① ② ③

④ ⑤

얼마 그 에 동 사고 식과

사고 식 보여 주는 내 다

들 에 는 탕 고 같 게

어 겨 고 미 에 는 그 크 럼 큰 고

어리 주고 원 는 어 도 는

상 고 생각 다는 것 다 러

는 어떻게 생 것 고 과 그리 거슬

러 가 보 그 단 다

고 연 경 체 경 생 에

다 벼 사는 공동 업과 경험 많 연 역

에 고 들 연 웃과

게 지내 고 탁 연 들

들 지 연 럽게 들 다 민들

웃과 동 게 뿐만 니 는 집 과

게 다

동 시 는 생태 경 에 살 결과

들 다 사 들 사 상 에 주

울 게 었고 는 곧 체 상 과 간 사

계 시 는 낳게 었다 신 가

가 는 체에 는 원 는 동시

에 다 사 들 그 사 포 체 맥 에

다 들 간 사 연

계 체 계에 주 울 는 사고 체계

게 었다

그러 그리 연 경 그 었다 산

지 연결 는 지 건 그리고 역

에 다 런 들 업에 다 사 과

동 므 공동체에

다고 다 고 그리 들

들과는 달리 보 내 감 지 들과

지 크게 느 지 못 다 그

견 다 경우 주 쟁 통 결 는 갖

게 었다

신 사 간 계들 루어진 커다

트워크 에 게 당연 사 역시 연

계들 체 식 게 다 어 상

원 도 그 개체가 체 맥 과

계 에 고 다 게 체 맥 에 주

울 다 보 상 복 과 가변 식 게 고

상에 재 는 많 변 들 사 에 재 는 들도

게 다 들 주 태도 보

는 경우가 많다 쟁 결

통 결 보다는 통 결

는 보 다

그러 고 그리 들 개개 사 사 독

에 주 울 다 사 사 체에

어 그들 사 에 재 는 공통 규 주

고 다 상 원 에도 사

체 내 주 고 다 그들

체 여 탕 체

는 주 태도 시 고 특 사 어

주에 는지 여 그 주에 는 규

견 다 에 는 쟁 식 리

같 리 사고 체계가 달 게 었다

리 드 니 벳 생각 지도 사- ldquo rdquo( 2004)

zb42) 위 에서 사 된 설 과 가장 유사한 것은

① 크톱 컴퓨 는 본체 니 마우 루

어 다

② 곡과 시 리 는 지 과 사 루어 다는 공통

지니고 다

③ 경 고 것과는 달리

경 본 연 태 그 주변 경

④ 벽돌 능 에 사계 내내

습도가 지 다

⑤ 잰느 체 체 지닌 재 체가 없

는 재 눌 다

년 학 간고사 대비2013 2 현대고 대비

ECN-0102-2013-001-000076193

zb43) 는 립 앙 도서 이 정의 일 이다lt gt

도서 장과 이 자의 리 의 정의 연결이

적절하 않은 것은

lt gt

제 조 서 유8 ( )

도서 장은 다른 이 자의 안전을 위협하거나 도서 의①

서를 란하게 할 가 있는 자에 대하여는 도서 출입

을 제한할 있다

도서 장은 이 자가 제 조 각 호의 어느 하나의 행위를 하7②

을 때에는 이 을 하게 하거나 도서 출입을 제한할

있다

제 조자 의 대출9 ( )

도서 자 는 다음 각 호의 경 대출할 있다①

상호대차도서 간에 자 를 류하는 것을 말한다 등 다1 ( )

른 도서 과의 협 을 위하여 필 한 경

공 이 공 행 상 필 하는 경2

에 도서 장이 필 하다고 인정하는 경3

대출이 가능한 도서 자 의 위는 도서 장이 정하는②

에 따른다

제 조 상10 ( )

이 자가 도서 자 설을 더럽히거나 찢거나 뜨①

쓰게 하거나 잃어 린 경 에는 상하여야 한다

도서 장은 제 항에 따른 상 을 정하여 게 하여야1②

한다

제 조이 절차 등11 ( )

이 칙에서 정한 것 에 도서 자 설의 이 절차

이 제한 등에 필 한 사항은 도서 장이 정한다

출처 립 앙 도서- (httpwwwnlgokr)

① 는 도 리 다8

② 도 는 리 다9 1

③ 료 지 는 도 리 다9 2

④ 도 료 변상에 리10 1

⑤ 는 에 도 리 다11

3

도 다 각 같다①

공 공 다만 연1

연 간 다

매월 째 째 월2

도 도 리 그 사3

가 다고 는

도 에 미리 게1 3②

시 여 다

4

도 시간 도 여 게시 다

5

도 료 시 는 는 도①

지에 등 후

등 에 사 도②

7

는 다 각 여 는 니 다

도 료 시 상 리1 lsquo rsquo

도 료 시 훼 는2 middot

지 가 닌 곳에 식 거 담3

우는

도 보 등 보 검색열4 middot

그 에 도 질 지 여 도5

여 게시 사 는

8

도 다 거 도①

질 게 우 가 는 에 여는 도

도 가 각 어느7②

에는 지 게 거 도

9

도 료는 다 각 경우 다①

상 도 간에 료 는 것 말1 (

다 등 다 도 과 여 경우)

공 원 공 상 는 경우2

그 에 도 다고 는 경우3

가능 도 료 는 도②

는 에 다

10

년 학 간고사 대비2013 2 현대고 대비

ECN-0102-2013-001-000076193

가 도 료 시 럽 거 거①

못 쓰게 거 어 린 경우에는 변상 여

도 에 변상 여 게시1②

여 다

zb44) 위 에서 도서 장이 게 해야 할 사항에 해당하는

것을 두 쓰

년 학 간고사 대비2013 2 현대고 대비

ECN-0102-2013-001-000076193

립 도 규

1 ( )

규 립 도 립 어린 청 도(

포 다 료 시 열 시 말) (

다 에 사 규 립 도)

편 진 다

2 ( )

규 립 도 도 다 에( lsquo rsquo )

고 는 도 에 도lsquo rsquo 2 2

료 에 여 다 다만 특 료 귀

료 등 료 에 사 립 도

도 다 다( lsquo rsquo )

3 ( )

도 다 각 같다①

공 공 다만 연1

연 간 다

매월 째 째 월2

도 도 리 그 사3

가 다고 는

도 에 미리 게1 3②

시 여 다

시간4 ( )

도 시간 도 여 게시 다

등 등5 ( )

도 료 시 는 는 도①

지에 등 후

등 에 사 도②

사 료6 ( )

도 료 시 에 사 료는 도

7 ( )

는 다 각 여 는 니 다

도 료 시 상 리1 lsquo rsquo

도 료 시 훼 는2 middot

지 가 닌 곳에 식 거 담3

우는

도 보 등 보 검색열4 middot

그 에 도 질 지 여 도5

여 게시 사 는

질 지8 ( )

도 다 거 도①

질 게 우 가 는 에 여는 도

도 가 각 어느7②

에는 지 게 거 도

료9 ( )

도 료는 다 각 경우 다①

상 도 간에 료 는 것 말1 (

다 등 다 도 과 여 경우)

공 원 공 상 는 경우2

그 에 도 다고 는 경우3

가능 도 료 는 도②

는 에 다

변상10 ( )

가 도 료 시 럽 거 거①

못 쓰게 거 어 린 경우에는 변상 여

도 에 변상 여 게시1②

여 다

등 규 에 것 에 도11 ( )

료 시 등에 사

도 다

립 도- (httpwwwnlgokr)

zb45) 도서 장의 리 있는 조항으 적절하 않

은 것은

① ② ③ ④ ⑤

년 학 간고사 대비2013 2 현대고 대비

ECN-0102-2013-001-000076193

1 ( )

사가 공 는lsquo rsquo

과 여 사 원과 리

사 타 사 규

니다

개 보 보7 ( )

사는 보통신망 등 계 는 에lsquo rsquo lsquo rsquo

원 개 보 보 니다 개lsquo rsquo

보 보 사 에 는 사 개lsquo rsquo

보 취 니다 다만 사는 다 lsquo rsquo

사 계 통 공 는 경우 원 lsquo rsquo

등 개 보 당 사에 습니lsquo rsquo

원 리에8 (lsquo rsquo lsquo rsquo lsquo rsquo

)

원 에 리lsquo rsquo lsquo rsquo lsquo rsquo①

원에게 가 도 여 는lsquo rsquo 3

니다

사는 원 가 개 보 우 가lsquo rsquo lsquo rsquo lsquo rsquo②

거 사 경우 는 미 에 어 거 lsquo

사 사 운 우 가 는 경우 당rsquo lsquo rsquo

습니다lsquo rsquo

원 가 도 거lsquo rsquo lsquo rsquo lsquo rsquo 3③

가 사 고 지 경우에는 시 사에lsquo rsquo

통지 고 사 내에 니다lsquo rsquo

경우에 당 원 사에 그 사실3 lsquo rsquo lsquo rsquo④

통지 지 거 통지 도 사 내에 지 lsquo rsquo

생 경우 사는 지지 습니다lsquo rsquo

사10 (lsquo rsquo )

사는 과 지 미lsquo rsquo①

에 는 지 계 고

공 여 다 여 니다lsquo rsquo

사는 원 게lsquo rsquo lsquo rsquo lsquo rsquo②

도 개 보 신 보 포 보 보 시( )

갖 어 개 보 취 공시 고

니다

사는 과 여 원lsquo rsquo lsquo rsquo③

견 만 당 다고 경우에는

리 여 니다 원 견 만 사 lsquo rsquo

에 는 게시 거 우편 등 통 여

원에게 리 과 결과 달 니다lsquo rsquo

원11 (lsquo rsquo )

원 다 여 는 니다lsquo rsquo ①

신청 는 변경 시 허 내 등1

타 보 도2

사가 게시 보 변경3 lsquo rsquo

사가 보 보 컴퓨 그4 lsquo rsquo (

등 등 신 는 게시)

사 타 등 지 재산 에5 lsquo rsquo 3

사 타 상 거 업6 lsquo rsquo 3

는 폭 시지 상 타 공7 middot middot

에 는 보 에 공개 는 게시 는lsquo rsquo

사 동 없 리 사8 lsquo rsquo

타 거 당9

게시15 (lsquo rsquo )

원 내에 게시 는 게시 게재 는lsquo rsquo lsquo rsquo lsquo rsquo

경우 원 사가 게시 복 lsquo rsquo lsquo rsquo lsquo rsquo middot middot

등 태 언 등에 공 는

것 내에 다 원 본 게시 등 lsquo rsquo lsquo rsquo

크 능 등 여 복 는 등 태

는 것 동 것 니다

- (wwwnavercom)

zb46) 위 은 인터넷 포털사이트의 회 가입을 위한 이

약 의 일 이다 이 약 을 만드는 과정에서 생각한

내 으 적절하 않은 것은

개 보 보 가 지에 별 눠①

겠어

원 가 만들게 에②

시 주어 겠어

원들 게재 게시 다 원 크 다③

는 것 지

④ 원 지 는 뿐만 니 사가 지 는

도 께 달 지

리에 가 생 경우 사가⑤

에 다는 도 듯

1 ( )

사가 공 는lsquo rsquo

과 여 사 원과 리

사 타 사 규

년 학 간고사 대비2013 2 현대고 대비

ECN-0102-2013-001-000076193

니다

개 보 보7 ( )

사는 보통신망 등 계 는 에lsquo rsquo lsquo rsquo

원 개 보 보 니다 개lsquo rsquo

보 보 사 에 는 사 개lsquo rsquo

보 취 니다 다만 사는 다 lsquo rsquo

사 계 통 공 는 경우 원 lsquo rsquo

등 개 보 당 사에 습니lsquo rsquo

원 리에8 (lsquo rsquo lsquo rsquo lsquo rsquo

)

원 에 리lsquo rsquo lsquo rsquo lsquo rsquo①

원에게 가 도 여 는lsquo rsquo 3

니다

사는 원 가 개 보 우 가lsquo rsquo lsquo rsquo lsquo rsquo②

거 사 경우 는 미 에 어 거 lsquo

사 사 운 우 가 는 경우 당rsquo lsquo rsquo

습니다lsquo rsquo

원 가 도 거lsquo rsquo lsquo rsquo lsquo rsquo 3③

가 사 고 지 경우에는 시 사에lsquo rsquo

통지 고 사 내에 니다lsquo rsquo

경우에 당 원 사에 그 사실3 lsquo rsquo lsquo rsquo④

통지 지 거 통지 도 사 내에 지 lsquo rsquo

생 경우 사는 지지 습니다lsquo rsquo

원에 통지9 (lsquo rsquo )

사는 특 다 원에게 통지 경우lsquo rsquo lsquo rsquo

공지 게시 통 상 게시 개별 통지에7

갈 습니다

사10 (lsquo rsquo )

사는 과 지 미lsquo rsquo①

에 는 지 계 고

공 여 다 여 니다lsquo rsquo

사는 원 게lsquo rsquo lsquo rsquo lsquo rsquo②

도 개 보 신 보 포 보 보 시( )

갖 어 개 보 취 공시 고

니다

사는 과 여 원lsquo rsquo lsquo rsquo③

견 만 당 다고 경우에는

리 여 니다 원 견 만 사 lsquo rsquo

에 는 게시 거 우편 등 통 여

원에게 리 과 결과 달 니다lsquo rsquo

원11 (lsquo rsquo )

원 다 여 는 니다lsquo rsquo ①

신청 는 변경 시 허 내 등1

타 보 도2

사가 게시 보 변경3 lsquo rsquo

사가 보 보 컴퓨 그4 lsquo rsquo (

등 등 신 는 게시)

사 타 등 지 재산 에5 lsquo rsquo 3

사 타 상 거 업6 lsquo rsquo 3

는 폭 시지 상 타 공7 middot middot

에 는 보 에 공개 는 게시 는lsquo rsquo

사 동 없 리 사8 lsquo rsquo

타 거 당9

원 계 규 내lsquo rsquo lsquo②

여 공지 주 사 사가 통지 는rsquo lsquo rsquo

사 등 여 타 사 업 에 lsquo rsquo

는 여 는 니다

- (wwwnavercom)

zb47) 위 약 의 조항에서 같은 제점을 하lt gt

고 있는 조항은

lt gt

제휴 회사에 회 의 아이디 개인 정 를 전송할 있도

한 조항은 고객에게 당한 조항이다

1 7 8① ② ③

④ 9 ⑤ 10

립 도 규

1 ( )

규 립 도 립 어린 청 도(

포 다 료 시 열 시 말) (

다 에 사 규 립 도)

편 진 다

2 ( )

규 립 도 도 다 에( lsquo rsquo )

고 는 도 에 도lsquo rsquo 2 2

료 에 여 다 다만 특 료 귀

료 등 료 에 사 립 도

도 다 다( lsquo rsquo )

3 ( )

도 다 각 같다①

공 공 다만 연1

연 간 다

년 학 간고사 대비2013 2 현대고 대비

ECN-0102-2013-001-000076193

매월 째 째 월2

도 도 리 그 사3

가 다고 는

도 에 미리 게1 3②

시 여 다

시간4 ( )

도 시간 도 여 게시 다

등 등5 ( )

도 료 시 는 는 도①

지에 등 후

등 에 사 도②

사 료6 ( )

도 료 시 에 사 료는 도

7 ( )

는 다 각 여 는 니 다

도 료 시 상 리1 lsquo rsquo

도 료 시 훼 는2 middot

지 가 닌 곳에 식 거 담3

우는

도 보 등 보 검색열4 middot

그 에 도 질 지 여 도5

여 게시 사 는

질 지8 ( )

도 다 거 도①

질 게 우 가 는 에 여는 도

도 가 각 어느7②

에는 지 게 거 도

료9 ( )

도 료는 다 각 경우 다①

상 도 간에 료 는 것 말1 (

다 등 다 도 과 여 경우)

공 원 공 상 는 경우2

그 에 도 다고 는 경우3

가능 도 료 는 도②

는 에 다

변상10 ( )

가 도 료 시 럽 거 거①

못 쓰게 거 어 린 경우에는 변상 여

도 에 변상 여 게시1②

여 다

등 규 에 것 에 도11 ( )

료 시 등에 사

도 다

립 도- (httpwwwnlgokr)

zb48) 다음 정 리 의 의 으 볼 때 가장

이 적인 것은

도 시간 도 여 게시 다①

등 에 사 도②

가능 도 료 는 도 는③

에 다

④ 도 에 변상 여 게10 1

시 여 다

⑤ 도 가 각 어느7

에는 지 거 도

zb49) 를 참고하여 이 어의 성격을 설 한lt gt

것으 적절하 않은 것은

① 보 에 는 어 시 상 고 어 시lt gt lsquo rsquo

에 보여주고 다

② 진 어 어원에 견 고 다

에는 타 어 들어가는 것 다 lsquo rsquo

③ 에 들어갈 말 각각 고 어 어 신 어~

들 언어는 질 격 강 통 없었다

④ 시 우리 에 가 었지만 지 계

과 달리 들 통 사 달 어 웠

년 학 간고사 대비2013 2 현대고 대비

ECN-0102-2013-001-000076193

⑤ 크 몽골 만주 공통어가 우리 어 같

계열에 다는 에 사 특 짐

가( )

善化公主主隱 공주님

他密只嫁良置古 몰 결 고

薯童房乙 맛

夜矣卯乙抱遣去如 에 몰 고 가다

( )

始汝 會隱日恚見隱扐 만 에 본

恥隱汝衣淸隱笑 맑 웃

고 시 여 공 크다 만 다[ ] ( ) ( ) ( ) ( )始 汝 會扐

내다 에 보다 견( ) ( )恚 見 다( )隱

럽다 맑다 청 웃( ) ( ) ( ) ( )恥 衣 淸 笑

zb50) 위의 나 를 함 고 음에 답하( ) lt gt

보lt gt

( )素那或云金川 白城郡蛇山人也

운 사산

는 고 다 는( )[ ( ) ] (素那 金川 白城

사산 사 다) ( ) 郡 蛇山

삼 사- lsquo rsquo 47

에 제 된 단어 의 표 리를 조건(1) lt gt ( ) lt gt

에 맞게 서 하

건lt gt

lsquo 었고 었다 태rsquo

에 제 된 단어 동일한 표 리에(2) lt gt ( )

의해 적은 것을 나 에서 찾아 조건 에 맞게 서 하( ) lt gt

건lt gt

에 당 는 각각( ) 개 쓸 것2 단

당 는 가 여러 개 어도 개만 쓸 것 각2

개 과 도 쪽에 개만2 2

드시 지 것( )

과 동 원리 것lsquo 고

과 동 원리 것 다rsquo

태 것

가( )

素那(或云金川) 白城郡蛇山人也

소나 또는 천 이라 한다 는 성 사( ) ( ) ( )素那 金川 白城郡〔 〕

산 사람이다 현대어 풀이( ) ( )蛇山

나( )

紫布岩乎希 회

執音乎手母牛放敎遣 자 손 암쇼 노히 고

吾 不喩慙 伊賜等肹 肹 나 안디 리샤

花 折叱肹 可獻乎理音如 고 것거 도림다

다 향찰은 리말을 리 으 적은 표 이었 만 생( )

은 고 대를 넘 하고 끊어 고 말았다 랜 세

동안 갈고 닦아 체계적이었던 향찰 표 이 사라졌

을 인은 크게 두 가 나누어 생각해 볼 있다

하나는 족 사회의 한 선호도에서 찾을 있다 라 때

향찰은 주 족 계 에서 사 했을 것으 인다 한 을

알 하고서는 한자를 활 하여 리말을 리 으 표

하 란 가능하 때 이다 런데 족들은 간이 흐

를 향찰과 같은 리 표 을 익혀 사 하 다는

아 한 을 대 사 하는 쪽을 선호하게 되었다 더 이

고 초에 인재 등 을 위해 과거제도가 행되 서 한 선

호도가 더 높아졌고 결 향찰은 소 되고 말았다

또 다른 가능성은 한 어의 특성에서 찾을 있다

터 한 과 일 세 나라는 한자 화 에 속해 다

당연한 이야 겠 만 표의 자인 한자는 어를 표 하

에 매 적절하다 어의 음절은 성 ( ) ( )聲母 韻母

이 어 고 여 에 성조가 추가되어 최종 소리가 결정된

다 래서 어는 단음절을 하나의 한자 표 하 된

다 에 초성 성 종성의 세 가 소가 하나의 음절

년 학 간고사 대비2013 2 현대고 대비

ECN-0102-2013-001-000076193

을 이 는 한 어는 음절 조가 잡하고 음절의 가 많아

서 한자 차 만으 한 어의 소리를 만족 럽게 표 할

없었다 를 들어 한 어에서는 어 니 같이 음절 lsquo rsquo

이 어 단어가 얼마든 있으나 어는( ) 複數音節

자 하나 나타내 만이다lsquo [m ]rsquo 母 ǔ

한편 일 어의 표 은 핵 적 단어는 한자 적고 토는

가나라는 일 의 자 적는 이다 적인 의 를 나

타내는 은 표의 자인 한자 적고 적 계를 나

타내는 토는 표음 자 적는 셈이니 자세히 살펴

리의 향찰 표 을 쏙 빼닮았음을 알 있다 한 어 같

은 착어이 서도 일 어에만 향찰과 유사한 표 이 살아

남은 것은 일 어의 특 때 이다 일 어는 하나의 자음과

음의 결합으 음절을 이 고 침이 거의 없는 음절 언어

이다 이러한 음절의 특색에다가 토가 달한 착어라는 점

이 향찰과 유사한 표 이 살아남을 있는 비결이었다

하 만 같은 착어라도 다양한 음소 침이 달한 한

어는 향찰 표 하는 데 근 적으 한계가 있었다

zb51) 다 하여 의 행에 대한 탐 한 결과( ) lt gt 2

않은 것은

보lt gt

善花公主主隱 공주니믄 공주님( )

----------------------------------------

-

他密只嫁良置古 그 지 얼어 고 몰 결(

----------------------------------------

-

薯童房乙 맛 맛( )

夜矣卯乙抱遺去如 몰 고 가다 에 몰 고(

가다)

주동 역 동- (薯童謠『 』

에 2 ( )他密只嫁良置古

얼다 시집가다 결 다 말 lsquo rsquo

① 실질 미 지니고 므 타 타lsquo ( )rsquo lsquo [ ]

② 에 실질 미 타내고 지 는lsquo rsquo lsquo [ ]rsquo lsquo [ ]密只 密 只

계 타내는

③ 얼어는 실질 미 포 고 므 가lsquo rsquo lsquo [ ]rsquo嫁

것lsquo [ ]rsquo 良

④ 고 어간 는 실질 미 지니고 므lsquo rsquo lsquo -rsquo

것lsquo [ ]rsquo 置

⑤ 고 어미 고는 계 타내고 므lsquo rsquo lsquo- rsquo

고 것lsquo [ ]rsquo 古

가( )

엉 훈 민middot middot middot middot middot世 宗 御 製 訓 民 正 音

말 미 듕 귁에 달middot middot middot middot middot middot middot middot中 國 文 字

니 런middot middot middot middot middot middot 어린middot middot middot middot百 姓

니 고 도 내 들middot middot middot middot middot middot middot middot middot 시러middot

펴 몯middot 미middot middot 니 내middot middot middot middot middot middot middot middot 爲

어엿middot 겨 새middot middot middot 믈여듧middot middot middot middot字 니middot middot middot

사 마다 니겨 킈 middot middot middot middot middot middot middot middot middot便 安

고 미니middot middot middot middot

본 는 상( ) (象

원리에 만들어진 본) ( )形 ㄱ ㄴ ㅁ ㅅ ㅇ

에 는 가 원리에( )加劃

그리고( )ㅋ ㄷ ㅌ ㅂ ㅍ ㅈ ㅊ ㆆ ㅎ

쓰는 병 원리에 만들어진( )竝書

마지막 체( ) ( )異體ㄲ ㄸ ㅃ ㅆ ㅉ ㆅ

ᅀ 다 상 원리에 ㅇ ㄹ

지 는 삼재 상 본 본( ) ( ) ( 天地人 三才

탕 므림과 림에 ) (初ㅡ ㅣ

재)( ) ( )( )出字 再出字ㅗ ㅏ ㅜ ㅓ ㅛ ㅑ ㅜ ㅕ

병 그리고 들 에 다시( )ㅘ ㅝ ㅣ

( )ㅣ ㅢ ㅚ ㅐ ㅟ ㅔ ㆉ ㅒ ㆌ ㅖ ㅙ ㅞ

zb52) 가 에 대한 설 으 르 않은 것을( ) 두 고르

① 어쓰 규 지키고 다

② 리 고 다

③ 말 미 미 등 어 사 다lsquo rsquo

④ 개 지 다

년 학 간고사 대비2013 2 현대고 대비

ECN-0102-2013-001-000076193

⑤ 어 원 에 가 도 고 다

엉 훈 민世 宗 御 製 訓 民 正 音

말 미 듕귁에 달 니

런 어린 니 고 도middot

내 들 시러 펴 몯 미 니middot

내 어엿 겨 새 믈여듧

사 마다 니겨middot 킈 고

미니

훈민 언 본- lsquo rsquo 5 (1459 )

zb53) 위의 에 대한 현대어 풀이가 르~ 않은 것

① 우리 말 과 달

② 어리 말 고 는 것 어도

③ 신 생각 마 껏 펼 는 사 많다

④ 게 생각 여

⑤ 사 마다 게

zb54) 훈민정음 언해 에는 한 을 창제한 동 가 드러나

있다 훈민정음 창제의 정 과 내 이 잘 연결된 것

① 주 신 말 미 듕귁에 달

② 민 신 내 어 겨

③ 신 뻔 킈 고 미니

④ 실 신 사 마다 니겨

⑤ 귀 신 계 주 는 훈민 신과 거리가

가 엉 훈 민( ) middot middot middot middot middot世 宗 御 製 訓 民 正 音 

말 미 귁에 中 國 달 文 字

니 런 어린 니 百 姓

고 도 내 들 시러 펴 몯

미 니 내 어엿 爲 겨 새

믈여듧 니 사 마다 니 字

겨 킈 고 미니 便 安

훈민 언 본- lsquo ( )rsquo ( ) 5 (1459 )訓民正音 世祖

( )

[ 1 ]

동 룡 샤 마다 복( ) ( ) ( )海東 六龍 天福

시니 고 동( ) ( )古聖 同符 시니

[ 2 ]

매 니 곶 여

미 므 니 그 내 러

가 니

[ 125 ]

우 미리( )千世 샨( )定 에( )漢水北 累仁

누 개 샤 복 업 시니( ) ( ) 開國 卜年

신( )聖神 니 샤도 경 근민 샤 욱( )敬天勤民

드시리 다

님 쇼 산 가( ) ( )洛水 山行

미드니 가

어 가- lsquo ( )rsquo 27龍飛御天歌

다 우리신 니쓰고 다만 만 쓰( )

거 샹 귀쳔 다보게 러 귀

여 쓴 도 신 보 가 고 신 에

말 어 보게 각 에 사 들

고 본 몬 능통 후에

죠 죠 니

드 도 만 공 에 사

드 미 죠 고 고 여 보 죠

보다 얼마가 거시 어신고 니 첫

가 죠 니 죠

민 들 어 신 샹

귀쳔 도보고 어보 가 만 늘

고 폐 에 만쓴 죠 민

도 러보지못 고 보니 그게 엇지

심 니 리 보 가 어 운건 다

니 쳣 말마 지 니 고 그

쓰 에 가 우 지 지

몰 거 본후에 가 어 지

고 그니 쓴편지 쟝 보

년 학 간고사 대비2013 2 현대고 대비

ECN-0102-2013-001-000076193

쓴것보다 듸 보고 그 마 니 쓴 고

어 못

그런고 에 리 과 가

만 쓴 못 민 말만 듯고

고 편 그 못 보니 그사 단

병신 못 다고 그사 식 사

니 만 고 다 과 그사

만 고 다 과 업 사 보다 식 고

죠 도 고 각 과

견 고 실 직 귀쳔 간에 그

고도 다 것 몰 귀죡 보다

사 우리 신 귀쳔 다 업

시 신 보고 과 지 게 랴

시니 샹 귀쳔 간에 우리 신 걸

간 보 새지각과 새 걸 미리

독립신- lsquo (1896)rsquo

zb55) 친 어 나의 제 장( ) 2 매 함축적

의 가 가장 유사한 것은

① 지 눈 내리고 매 득 니 내 여 가

사- lsquo rsquo

② 도 어 리듯 그 게 어 다

주 사- lsquo rsquo

③ 눈 살 다 죽 어 린 과 체 여

눈 새벽 지 도 살 다

눈- lsquo rsquo

④ 삶 근심과 고단 에 돌 거니는 여 거 는

여 리 내린 살가지 에 눈 리 눈 리

택 그 생 에- lsquo rsquo

⑤ 늘 러 고 러

청룡 룡 어 개 루 우

신경림 계- lsquo rsquo

zb56) 친 를 위 가 나 에 나타난A B ( ) ( )

세 어의 특 에 의거하여 세 어 표 하

그 산 고 공 도 맑지만

A

주변에 쓰 리는 어리 사 많다

B

건lt gt

식 가 에 타 어 특징에( ) ( )

거 과 어쓰 는 고 지 말 것

A

B

zb57) 가 의( ) 달 아ㆍ 다 의 ( ) 나셔에서 알 있는

세 어 개화 어의 특 을 비 하여 조건 에lt gt

맞게 서 하

건lt gt

어에 는lsquo 개

어에 는 다 태rsquo

zb58) 은 가 는 다 에 나 는 절lt 1gt ( ) lt 2gt ( )

일 를 췌한 것이다 의 의 가 lt 1gt (1)~(2)

유사한 말을 에서 찾아 쓰lt 2gt

보lt 1gt

런 (1) 어린 니 고百 姓

도 내 들 시러 펴 몯 미

사 마다 (2) 니겨 便 安

킈 고 미니

보lt 2gt

죠 고 고 여 보 죠

보다 얼마가 거시 어신고 니 첫 가

죠 니 죠 민

들 어 신 샹 귀쳔

도보고 어보 가 만 늘 고

폐 에 만쓴 죠 민 도

러보지못 고 보니 그게 엇지 심

니 리

년 학 간고사 대비2013 2 현대고 대비

ECN-0102-2013-001-000076193

lt 1 gt

동 룡 샤 마다 복 시( ) ( ) ( )海東 六龍 天福

고 동 시니( ) ( )古聖 同符

lt 2 gt

(A) 매 니 곶

여 니

미 므 니 그 내

러 가 니

lt125 gt

우 미리 샨 에( ) ( ) ( ) 千世 定 漢水北 累

누 개 샤 복 업 시 니( ) ( ) 仁開國 卜年 聖

신( ) 神 니 샤도 경 근민 샤( ) 敬天勤民

욱 드 시 리 다

님 쇼 산 가 ( ) ( )洛水 山行

미드니 가

- lt gt龍飛御天歌

zb59) 장과 내 상 유사한 성격의 조는125

① 뫼 고 고 고 고

어 그린 많고 많고 고 고

어 러 는 울고 울고 가느니

도 견- lt gt

② 강 에 드니 몸 다

그믈 고 가니

뒷 뫼 엄 언 니( )藥

-

③ 말 없는 청산 태 없는 다

값 없는 청 없는 월

에 병 없는 몸 별 없 늙 리

-

④ 가마귀 골에 가지 마

낸 가마귀 새

청강에 것 시 몸 러 가( ) 淸江

-

⑤ 진 골에( ) 白雪

가 매 는 어느 곳에 었는고

에 갈 곳 몰( ) 夕陽

색-

zb60) 위 에 나타난 세 어의 특 으 적절하 않은

것은

① 룡 어 주격 사에 당 는 가 사( ) lsquo rsquo六龍

고 다

② 샤 어에도 어 주체 쓰 다

는 것 다

③ 매 어 달리 사 택에 어

가 지 지지 고 다

④ 므 원 상 직 어 지 다

⑤ 드시리 다 주체 과 상 께 사

고 다

수고 하셨습니다hearts hearts

년 학 간고사 대비2013 2 현대고 대비

ECN-0102-2013-001-000076193

보닷컴에 공 는 별 보는 고등

들 여 주 는

들 습니다 슷 동 지

가 복 는 것 도가

니 복 여 습 시고 거 시

니다

정답 해설

1) 정답[ ] ④

해설 다른 것은 두 특정 업이나 단 내에서 사[ ]

하는 일종의 은어 사회 언에 해당한다 러나

는 언이 아니라 단과대학을 여서 단대 사lsquo rsquo lsquo rsquo lsquo④

대학을 여서 사대라고 한 말에 해당하 일rsquo lsquo rsquo

사회에서도 널리 쓰이 사회 언이라 할

없다

2) 정답[ ] ⑤

해설 사회 언은 같은 단 내에서 쓰이는 언어이[ ] lsquo rsquo

동일 단끼리는 단결 과 친 감을 형성하는

능을 하 리적 안감이 일어나 않는다

3) 정답[ ] ③

해설 사람이라는 차 적 표현에 대한 대안적 표현이[ ]lsquo rsquo

인 아내 처 등으 볼 있다lsquo rsquo

4) 정답[ ]⑤

해설 남성은 주 격 체를 사 한다[ ]

5) 정답[ ] ⑤

해설 흑인은 검다라는 뜻을 가 고 있을 뿐 인[ ]lsquo rsquo lsquo rsquo lsquo rsquo

다 열등한 뜻을 내포하 않는다

6) 정답 살 색 첫 작품[ ] - -

해설 살색 혹은 킨색은 한 인의 피 색을 뜻[ ] lsquo rsquo lsquo rsquo

하는 것으 인종 차 을 추 고 출 이주민

의 평등 을 침해할 있어 년 표 이2005

살 색으 이름을 꾸었다 처녀작은 처녀라lsquo rsquo lsquo rsquo lsquo rsquo

는 단어가 가 고 있는 곡된 성 인 을 한 것

으 첫 작품정도 꾸어 사 하는 것이 좋다lsquo rsquo

7) 정답[ ] ⑤

해설 호는 아들에게 해체를 사 하고 있다[ ] ① ②

장 을 성하는 청자는 자 의 아 느리 아lsquo

들 세 이다 호는 아 느리에게 해rsquo ③

체를 사 하고 있다 호가 느리 아 에게 ④

사 한 해 체 아들에게 사 한 해체는 두 비lsquo rsquo lsquo rsquo

격 체에 해당한다 호는 자 의 아랫사람인 ⑤

느리에게 아들과 마찬가 해체를 사 하는 것이

상 이 만 임 을 한 느리에게 고마 과 쁨

존 의 표 를 하 위해 자 의 아 에게 말하듯

해 체를 사 하고 있다

8) 정답[ ] ③

9) 정답[ ] ⑤

10) 정답[ ] ①

해설 청자 할아 가 장의 주체 아 다 높을[ ] ( ) ( )

경 에는 압존 에 의해 장의 주체를 높이 않는lsquo rsquo

다 러 아 서가 아닌 아 는으 계 lsquo rsquo lsquo rsquo lsquo

니다 가 아닌 있 니다 표현하는 것이 르rsquo lsquo rsquo

11) 정답 당이 당을 쫒았다 당이[ ]

당에 다

해설[ ]

12) 정답[ ] ⑤

해설 서 다른 높임표현을 통해 청자에 대해 리[ ] ⑤

적 거리감을 나타내는 인 은 이 아니라 현정이

다 가 에서 현정은 에게 해 체를 사 함으 써 ( )

친근감을 드러낸다 나 에서 연 을 게을리하는 역 ( )

도 들 때 에 화가 난 현정이 선생님에게 항의하

는 장 에서는 하 체를 사 하여 리적 거리lsquo rsquo

가 어졌음을 나타내고 있다

13) 정답[ ] ①

해설 는 는 얼 빛이 날과 어찌 다르 고[ ] lsquo rsquo

라는 뜻으 전과 달리 임이 화자를 않고

있음을 알 있다

14) 정답 달리 후 가 있다 이를 통해 경[ ] lt gt

쾌한 음악성을 형성하고 노 젓는 상황을 체적으

형상화하는 역할을 한다

15) 정답[ ] ①

16) 정답[ ] ⑤

해설 다 의 자연은 를 성찰하게 하는 대상[ ] ( )⑤

이자 정의 대상이다 의 자연은 자 의 상황과 ⑤

처 를 드러내는 경으 서의 역할을 하 이

이 없다

17) 정답[ ] ③

해설 는 빈천 을 해결하고자 했으나 강산[ ] lsquo ( )rsquo 貧賤③

과 풍 을 달라는 에 거절하 다고 함으 써 자

연에 대한 애정을 드러내고 있으 는 않는

임에 대한 망을 개에게 전가 켜서 임에 대한 리

을 드러내고 있다

18) 정답[ ] ③

년 학 간고사 대비2013 2 현대고 대비

ECN-0102-2013-001-000076193

19) 정답[ ] ⑤

해설 고상한 음악가의 이름을 리말 꽝 럽[ ]

게 꿈으 써 언어유희를 통해 음을 유 하고 있

다 이는 고상한 척하는 총 를 비꼼으 써 비판적

태도를 드러내는 것이 대상을 꽝 럽게 표현

하여 총 의 허 과 사치를 풍자하고 있다

20) 정답[ ] ⑤

해설 는 작품 속 경에 대한 설 이 드러나는 것이[ ]

서 자의 주 적인 견해가 접적으 드러나는 것이

아니다

21) 정답[ ] ⑤

22) 정답[ ] ②

23) 정답[ ] ④

24) 정답[ ] ①

해설 적강 티프는 주인공의 비 한 출생이나 능[ ] ①

과 이 있는 것으 조정의 능함을 풍자하는lsquo rsquo

것과는 거리가 다

25) 정답 픔 나[ ] ( )

해설 의 음악은 고통 는 사람들을 위 하고 아픔[ ] lsquo rsquo

을 치유해 주는 능을 한다고 할 있다 의 lt gt

픔 도 소 된 이 과 더 어 살아가는 따뜻한 마음lsquo rsquo

을 상 한다

26) 정답[ ] ⑤

해설 에게 선천적으 주어 각 장애라는 역경[ ]

은 의 이라는 가사 연 을 있다lsquo rsquo

27) 정답[ ] ④

해설 는 장 란 선 에게 은 개인적인 인상을[ ]

소녀 장정 등으 표현한 것이다lsquo rsquo

28) 정답[ ] ②

해설 담자가 피 담자의 언어적 표현이나 비언어[ ]②

적 표현 하 독자는 담의 위 나 피

담자의 감정 상태를 알 있다 이를 통해 독자는

담 상황을 더 생생하게 느낄 있고 피 담자

를 더 잘 이해할 있게 된다

29) 정답[ ]③

해설 일상생활과 역도 선 서의 성과에 된 것에서[ ]

역도를 하 서 겪는 어 과 내적 고민으 화제를

전화하 위한 것이다

30) 정답[ ] ①

해설 릿속에 새겨 넣듯 이 억되도 함 세상[ ] ② ③

살이가 힘들고 고생 러 속 하여 자유를 ④

가 없는 고통의 상태를 비유적으 이르는 말

적의 침입을 막 위해 쌓은 축 켜야 할⑤

대상을 비유적으 이르는 말이다

31) 정답[ ] ④

해설 이 의 종류는 전 으 인 사건 경[ ] lsquo

비평을 성 소 삼는다rsquo

32) 정답[ ] ④

해설 근은 삼대독자 태어났음을 에서 확인할[ ]

있다 형제들과의 담은 이뤄 가 없다

33) 정답[ ] ⑤

해설 근은 가난에도 하고 화가를 꿈꾸었다[ ] (3

단 또한 다른 화가 망생들은 정 육을)

위해 상 학 학 해 유학 에 랐 만

근은 다른 을 찾아야 했다 단 세에(5 ) 18

근은 조선 전람회에 입선하 다 단 의(6 )

만종은 인간과 자연이 엮어 가는 경건한 조화 을lsquo rsquo

나타낸다

34) 정답[ ] ①

해설 근이 속에서도 창작활동을 추 않고[ ]

하는 닭은 은 세상과 타협할 르는

근이 세상의 이해를 하 위한 가장 떳떳한 단

이 때 이다

35) 정답[ ] ⑤

해설 전 은 서 자의 주 적인 평이 리는 것이[ ]

만 위 제 은 인 이 살았던 대 사회적 경

을 통해 객 적인 인 의 을 제 하고 있다

36) 정답[ ] ⑤

해설 전 은 인 사건 경 비평이라는[ ] lsquo rsquo⑤

성 이 어져 있다

37) 정답[ ] ①

해설 이 은 동양인과 서양인의 사고 에 차이가[ ]

있다는 것을 대조를 통해 설 하고 있다 또 쓴이

의 제자가 축 경 를 러 가서 경험한 일화를

통해 동양인이 서양인에 비해 주 상황에 더 많은

주의를 인다는 주장을 뒷 침하고 있다

38) 정답[ ] ④

39) 정답[ ] ②

40) 정답[ ] ②

41) 정답[ ] ④

42) 정답[ ] ③

43) 정답[ ] ④

44) 정답 도서 의 휴 일 도서 의 이 간 도서의[ ]

해설 도서 장은 임의 정한 휴 일과 도서 이[ ]

간 도서의 상 등을 게 할 의 가 있다

년 학 간고사 대비2013 2 현대고 대비

ECN-0102-2013-001-000076193

45) 정답[ ] ①

해설 제 조의 정 휴 일 의 휴 일의 사전 게[ ] 3

는 도서 장의 의 조항에 속한다

46) 정답[ ] ①

해설 개인 정 호 의 를 제 하 했 만 항[ ]

나눠서 제 하 않고 대 나열하고 있다

47) 정답[ ] ②

해설 제 조의 내 을 회사는 다른 회사 협[ ] 7 lsquo

계약을 통해 서비 를 제공하는 경 회 의 아이디

등 개인 정 를 해당 회사에 전송할 있다는 내rsquo

이 있으 의 제점을 제 할 있다②

48) 정답[ ] ④

해설 는 도서 장의 의 에 해당하고 나 는 도[ ] ④

서 장의 리에 해당한다

49) 정답[ ] ③

50) 정답 은 음독으 적었고 은 훈독으 적었[ ] (1)

다 과 동일한 표 리 적은 것은 이고 (2) ce

과 동일한 표 리 적은 것은 이다ab

51) 정답[ ] ③

52) 정답[ ] ①②

53) 정답[ ] ③

54) 정답[ ] ③

55) 정답[ ] ①

56) 정답 른 죠코 어린 노 하니라[ ] A B

57) 정답 세 어에서는 활 형이 칙적으[ ] lsquo rsquoㄹㅇ

나타났 만 개화 어에서는 활 형이 쓰 다 lsquo rsquo ㄹㄴ

58) 정답 호 가 흔[ ] (1) (2)

59) 정답[ ] ④

60) 정답[ ] ③

Page 28: 현대고대비 국어 - chamsoriedu.com 「콘텐츠산업진흥 법」외 에도 저작권 의하여 ... 다른주체에게어떤동작을하도록만드는것을나타내는

년 학 간고사 대비2013 2 현대고 대비

ECN-0102-2013-001-000076193

zb43) 는 립 앙 도서 이 정의 일 이다lt gt

도서 장과 이 자의 리 의 정의 연결이

적절하 않은 것은

lt gt

제 조 서 유8 ( )

도서 장은 다른 이 자의 안전을 위협하거나 도서 의①

서를 란하게 할 가 있는 자에 대하여는 도서 출입

을 제한할 있다

도서 장은 이 자가 제 조 각 호의 어느 하나의 행위를 하7②

을 때에는 이 을 하게 하거나 도서 출입을 제한할

있다

제 조자 의 대출9 ( )

도서 자 는 다음 각 호의 경 대출할 있다①

상호대차도서 간에 자 를 류하는 것을 말한다 등 다1 ( )

른 도서 과의 협 을 위하여 필 한 경

공 이 공 행 상 필 하는 경2

에 도서 장이 필 하다고 인정하는 경3

대출이 가능한 도서 자 의 위는 도서 장이 정하는②

에 따른다

제 조 상10 ( )

이 자가 도서 자 설을 더럽히거나 찢거나 뜨①

쓰게 하거나 잃어 린 경 에는 상하여야 한다

도서 장은 제 항에 따른 상 을 정하여 게 하여야1②

한다

제 조이 절차 등11 ( )

이 칙에서 정한 것 에 도서 자 설의 이 절차

이 제한 등에 필 한 사항은 도서 장이 정한다

출처 립 앙 도서- (httpwwwnlgokr)

① 는 도 리 다8

② 도 는 리 다9 1

③ 료 지 는 도 리 다9 2

④ 도 료 변상에 리10 1

⑤ 는 에 도 리 다11

3

도 다 각 같다①

공 공 다만 연1

연 간 다

매월 째 째 월2

도 도 리 그 사3

가 다고 는

도 에 미리 게1 3②

시 여 다

4

도 시간 도 여 게시 다

5

도 료 시 는 는 도①

지에 등 후

등 에 사 도②

7

는 다 각 여 는 니 다

도 료 시 상 리1 lsquo rsquo

도 료 시 훼 는2 middot

지 가 닌 곳에 식 거 담3

우는

도 보 등 보 검색열4 middot

그 에 도 질 지 여 도5

여 게시 사 는

8

도 다 거 도①

질 게 우 가 는 에 여는 도

도 가 각 어느7②

에는 지 게 거 도

9

도 료는 다 각 경우 다①

상 도 간에 료 는 것 말1 (

다 등 다 도 과 여 경우)

공 원 공 상 는 경우2

그 에 도 다고 는 경우3

가능 도 료 는 도②

는 에 다

10

년 학 간고사 대비2013 2 현대고 대비

ECN-0102-2013-001-000076193

가 도 료 시 럽 거 거①

못 쓰게 거 어 린 경우에는 변상 여

도 에 변상 여 게시1②

여 다

zb44) 위 에서 도서 장이 게 해야 할 사항에 해당하는

것을 두 쓰

년 학 간고사 대비2013 2 현대고 대비

ECN-0102-2013-001-000076193

립 도 규

1 ( )

규 립 도 립 어린 청 도(

포 다 료 시 열 시 말) (

다 에 사 규 립 도)

편 진 다

2 ( )

규 립 도 도 다 에( lsquo rsquo )

고 는 도 에 도lsquo rsquo 2 2

료 에 여 다 다만 특 료 귀

료 등 료 에 사 립 도

도 다 다( lsquo rsquo )

3 ( )

도 다 각 같다①

공 공 다만 연1

연 간 다

매월 째 째 월2

도 도 리 그 사3

가 다고 는

도 에 미리 게1 3②

시 여 다

시간4 ( )

도 시간 도 여 게시 다

등 등5 ( )

도 료 시 는 는 도①

지에 등 후

등 에 사 도②

사 료6 ( )

도 료 시 에 사 료는 도

7 ( )

는 다 각 여 는 니 다

도 료 시 상 리1 lsquo rsquo

도 료 시 훼 는2 middot

지 가 닌 곳에 식 거 담3

우는

도 보 등 보 검색열4 middot

그 에 도 질 지 여 도5

여 게시 사 는

질 지8 ( )

도 다 거 도①

질 게 우 가 는 에 여는 도

도 가 각 어느7②

에는 지 게 거 도

료9 ( )

도 료는 다 각 경우 다①

상 도 간에 료 는 것 말1 (

다 등 다 도 과 여 경우)

공 원 공 상 는 경우2

그 에 도 다고 는 경우3

가능 도 료 는 도②

는 에 다

변상10 ( )

가 도 료 시 럽 거 거①

못 쓰게 거 어 린 경우에는 변상 여

도 에 변상 여 게시1②

여 다

등 규 에 것 에 도11 ( )

료 시 등에 사

도 다

립 도- (httpwwwnlgokr)

zb45) 도서 장의 리 있는 조항으 적절하 않

은 것은

① ② ③ ④ ⑤

년 학 간고사 대비2013 2 현대고 대비

ECN-0102-2013-001-000076193

1 ( )

사가 공 는lsquo rsquo

과 여 사 원과 리

사 타 사 규

니다

개 보 보7 ( )

사는 보통신망 등 계 는 에lsquo rsquo lsquo rsquo

원 개 보 보 니다 개lsquo rsquo

보 보 사 에 는 사 개lsquo rsquo

보 취 니다 다만 사는 다 lsquo rsquo

사 계 통 공 는 경우 원 lsquo rsquo

등 개 보 당 사에 습니lsquo rsquo

원 리에8 (lsquo rsquo lsquo rsquo lsquo rsquo

)

원 에 리lsquo rsquo lsquo rsquo lsquo rsquo①

원에게 가 도 여 는lsquo rsquo 3

니다

사는 원 가 개 보 우 가lsquo rsquo lsquo rsquo lsquo rsquo②

거 사 경우 는 미 에 어 거 lsquo

사 사 운 우 가 는 경우 당rsquo lsquo rsquo

습니다lsquo rsquo

원 가 도 거lsquo rsquo lsquo rsquo lsquo rsquo 3③

가 사 고 지 경우에는 시 사에lsquo rsquo

통지 고 사 내에 니다lsquo rsquo

경우에 당 원 사에 그 사실3 lsquo rsquo lsquo rsquo④

통지 지 거 통지 도 사 내에 지 lsquo rsquo

생 경우 사는 지지 습니다lsquo rsquo

사10 (lsquo rsquo )

사는 과 지 미lsquo rsquo①

에 는 지 계 고

공 여 다 여 니다lsquo rsquo

사는 원 게lsquo rsquo lsquo rsquo lsquo rsquo②

도 개 보 신 보 포 보 보 시( )

갖 어 개 보 취 공시 고

니다

사는 과 여 원lsquo rsquo lsquo rsquo③

견 만 당 다고 경우에는

리 여 니다 원 견 만 사 lsquo rsquo

에 는 게시 거 우편 등 통 여

원에게 리 과 결과 달 니다lsquo rsquo

원11 (lsquo rsquo )

원 다 여 는 니다lsquo rsquo ①

신청 는 변경 시 허 내 등1

타 보 도2

사가 게시 보 변경3 lsquo rsquo

사가 보 보 컴퓨 그4 lsquo rsquo (

등 등 신 는 게시)

사 타 등 지 재산 에5 lsquo rsquo 3

사 타 상 거 업6 lsquo rsquo 3

는 폭 시지 상 타 공7 middot middot

에 는 보 에 공개 는 게시 는lsquo rsquo

사 동 없 리 사8 lsquo rsquo

타 거 당9

게시15 (lsquo rsquo )

원 내에 게시 는 게시 게재 는lsquo rsquo lsquo rsquo lsquo rsquo

경우 원 사가 게시 복 lsquo rsquo lsquo rsquo lsquo rsquo middot middot

등 태 언 등에 공 는

것 내에 다 원 본 게시 등 lsquo rsquo lsquo rsquo

크 능 등 여 복 는 등 태

는 것 동 것 니다

- (wwwnavercom)

zb46) 위 은 인터넷 포털사이트의 회 가입을 위한 이

약 의 일 이다 이 약 을 만드는 과정에서 생각한

내 으 적절하 않은 것은

개 보 보 가 지에 별 눠①

겠어

원 가 만들게 에②

시 주어 겠어

원들 게재 게시 다 원 크 다③

는 것 지

④ 원 지 는 뿐만 니 사가 지 는

도 께 달 지

리에 가 생 경우 사가⑤

에 다는 도 듯

1 ( )

사가 공 는lsquo rsquo

과 여 사 원과 리

사 타 사 규

년 학 간고사 대비2013 2 현대고 대비

ECN-0102-2013-001-000076193

니다

개 보 보7 ( )

사는 보통신망 등 계 는 에lsquo rsquo lsquo rsquo

원 개 보 보 니다 개lsquo rsquo

보 보 사 에 는 사 개lsquo rsquo

보 취 니다 다만 사는 다 lsquo rsquo

사 계 통 공 는 경우 원 lsquo rsquo

등 개 보 당 사에 습니lsquo rsquo

원 리에8 (lsquo rsquo lsquo rsquo lsquo rsquo

)

원 에 리lsquo rsquo lsquo rsquo lsquo rsquo①

원에게 가 도 여 는lsquo rsquo 3

니다

사는 원 가 개 보 우 가lsquo rsquo lsquo rsquo lsquo rsquo②

거 사 경우 는 미 에 어 거 lsquo

사 사 운 우 가 는 경우 당rsquo lsquo rsquo

습니다lsquo rsquo

원 가 도 거lsquo rsquo lsquo rsquo lsquo rsquo 3③

가 사 고 지 경우에는 시 사에lsquo rsquo

통지 고 사 내에 니다lsquo rsquo

경우에 당 원 사에 그 사실3 lsquo rsquo lsquo rsquo④

통지 지 거 통지 도 사 내에 지 lsquo rsquo

생 경우 사는 지지 습니다lsquo rsquo

원에 통지9 (lsquo rsquo )

사는 특 다 원에게 통지 경우lsquo rsquo lsquo rsquo

공지 게시 통 상 게시 개별 통지에7

갈 습니다

사10 (lsquo rsquo )

사는 과 지 미lsquo rsquo①

에 는 지 계 고

공 여 다 여 니다lsquo rsquo

사는 원 게lsquo rsquo lsquo rsquo lsquo rsquo②

도 개 보 신 보 포 보 보 시( )

갖 어 개 보 취 공시 고

니다

사는 과 여 원lsquo rsquo lsquo rsquo③

견 만 당 다고 경우에는

리 여 니다 원 견 만 사 lsquo rsquo

에 는 게시 거 우편 등 통 여

원에게 리 과 결과 달 니다lsquo rsquo

원11 (lsquo rsquo )

원 다 여 는 니다lsquo rsquo ①

신청 는 변경 시 허 내 등1

타 보 도2

사가 게시 보 변경3 lsquo rsquo

사가 보 보 컴퓨 그4 lsquo rsquo (

등 등 신 는 게시)

사 타 등 지 재산 에5 lsquo rsquo 3

사 타 상 거 업6 lsquo rsquo 3

는 폭 시지 상 타 공7 middot middot

에 는 보 에 공개 는 게시 는lsquo rsquo

사 동 없 리 사8 lsquo rsquo

타 거 당9

원 계 규 내lsquo rsquo lsquo②

여 공지 주 사 사가 통지 는rsquo lsquo rsquo

사 등 여 타 사 업 에 lsquo rsquo

는 여 는 니다

- (wwwnavercom)

zb47) 위 약 의 조항에서 같은 제점을 하lt gt

고 있는 조항은

lt gt

제휴 회사에 회 의 아이디 개인 정 를 전송할 있도

한 조항은 고객에게 당한 조항이다

1 7 8① ② ③

④ 9 ⑤ 10

립 도 규

1 ( )

규 립 도 립 어린 청 도(

포 다 료 시 열 시 말) (

다 에 사 규 립 도)

편 진 다

2 ( )

규 립 도 도 다 에( lsquo rsquo )

고 는 도 에 도lsquo rsquo 2 2

료 에 여 다 다만 특 료 귀

료 등 료 에 사 립 도

도 다 다( lsquo rsquo )

3 ( )

도 다 각 같다①

공 공 다만 연1

연 간 다

년 학 간고사 대비2013 2 현대고 대비

ECN-0102-2013-001-000076193

매월 째 째 월2

도 도 리 그 사3

가 다고 는

도 에 미리 게1 3②

시 여 다

시간4 ( )

도 시간 도 여 게시 다

등 등5 ( )

도 료 시 는 는 도①

지에 등 후

등 에 사 도②

사 료6 ( )

도 료 시 에 사 료는 도

7 ( )

는 다 각 여 는 니 다

도 료 시 상 리1 lsquo rsquo

도 료 시 훼 는2 middot

지 가 닌 곳에 식 거 담3

우는

도 보 등 보 검색열4 middot

그 에 도 질 지 여 도5

여 게시 사 는

질 지8 ( )

도 다 거 도①

질 게 우 가 는 에 여는 도

도 가 각 어느7②

에는 지 게 거 도

료9 ( )

도 료는 다 각 경우 다①

상 도 간에 료 는 것 말1 (

다 등 다 도 과 여 경우)

공 원 공 상 는 경우2

그 에 도 다고 는 경우3

가능 도 료 는 도②

는 에 다

변상10 ( )

가 도 료 시 럽 거 거①

못 쓰게 거 어 린 경우에는 변상 여

도 에 변상 여 게시1②

여 다

등 규 에 것 에 도11 ( )

료 시 등에 사

도 다

립 도- (httpwwwnlgokr)

zb48) 다음 정 리 의 의 으 볼 때 가장

이 적인 것은

도 시간 도 여 게시 다①

등 에 사 도②

가능 도 료 는 도 는③

에 다

④ 도 에 변상 여 게10 1

시 여 다

⑤ 도 가 각 어느7

에는 지 거 도

zb49) 를 참고하여 이 어의 성격을 설 한lt gt

것으 적절하 않은 것은

① 보 에 는 어 시 상 고 어 시lt gt lsquo rsquo

에 보여주고 다

② 진 어 어원에 견 고 다

에는 타 어 들어가는 것 다 lsquo rsquo

③ 에 들어갈 말 각각 고 어 어 신 어~

들 언어는 질 격 강 통 없었다

④ 시 우리 에 가 었지만 지 계

과 달리 들 통 사 달 어 웠

년 학 간고사 대비2013 2 현대고 대비

ECN-0102-2013-001-000076193

⑤ 크 몽골 만주 공통어가 우리 어 같

계열에 다는 에 사 특 짐

가( )

善化公主主隱 공주님

他密只嫁良置古 몰 결 고

薯童房乙 맛

夜矣卯乙抱遣去如 에 몰 고 가다

( )

始汝 會隱日恚見隱扐 만 에 본

恥隱汝衣淸隱笑 맑 웃

고 시 여 공 크다 만 다[ ] ( ) ( ) ( ) ( )始 汝 會扐

내다 에 보다 견( ) ( )恚 見 다( )隱

럽다 맑다 청 웃( ) ( ) ( ) ( )恥 衣 淸 笑

zb50) 위의 나 를 함 고 음에 답하( ) lt gt

보lt gt

( )素那或云金川 白城郡蛇山人也

운 사산

는 고 다 는( )[ ( ) ] (素那 金川 白城

사산 사 다) ( ) 郡 蛇山

삼 사- lsquo rsquo 47

에 제 된 단어 의 표 리를 조건(1) lt gt ( ) lt gt

에 맞게 서 하

건lt gt

lsquo 었고 었다 태rsquo

에 제 된 단어 동일한 표 리에(2) lt gt ( )

의해 적은 것을 나 에서 찾아 조건 에 맞게 서 하( ) lt gt

건lt gt

에 당 는 각각( ) 개 쓸 것2 단

당 는 가 여러 개 어도 개만 쓸 것 각2

개 과 도 쪽에 개만2 2

드시 지 것( )

과 동 원리 것lsquo 고

과 동 원리 것 다rsquo

태 것

가( )

素那(或云金川) 白城郡蛇山人也

소나 또는 천 이라 한다 는 성 사( ) ( ) ( )素那 金川 白城郡〔 〕

산 사람이다 현대어 풀이( ) ( )蛇山

나( )

紫布岩乎希 회

執音乎手母牛放敎遣 자 손 암쇼 노히 고

吾 不喩慙 伊賜等肹 肹 나 안디 리샤

花 折叱肹 可獻乎理音如 고 것거 도림다

다 향찰은 리말을 리 으 적은 표 이었 만 생( )

은 고 대를 넘 하고 끊어 고 말았다 랜 세

동안 갈고 닦아 체계적이었던 향찰 표 이 사라졌

을 인은 크게 두 가 나누어 생각해 볼 있다

하나는 족 사회의 한 선호도에서 찾을 있다 라 때

향찰은 주 족 계 에서 사 했을 것으 인다 한 을

알 하고서는 한자를 활 하여 리말을 리 으 표

하 란 가능하 때 이다 런데 족들은 간이 흐

를 향찰과 같은 리 표 을 익혀 사 하 다는

아 한 을 대 사 하는 쪽을 선호하게 되었다 더 이

고 초에 인재 등 을 위해 과거제도가 행되 서 한 선

호도가 더 높아졌고 결 향찰은 소 되고 말았다

또 다른 가능성은 한 어의 특성에서 찾을 있다

터 한 과 일 세 나라는 한자 화 에 속해 다

당연한 이야 겠 만 표의 자인 한자는 어를 표 하

에 매 적절하다 어의 음절은 성 ( ) ( )聲母 韻母

이 어 고 여 에 성조가 추가되어 최종 소리가 결정된

다 래서 어는 단음절을 하나의 한자 표 하 된

다 에 초성 성 종성의 세 가 소가 하나의 음절

년 학 간고사 대비2013 2 현대고 대비

ECN-0102-2013-001-000076193

을 이 는 한 어는 음절 조가 잡하고 음절의 가 많아

서 한자 차 만으 한 어의 소리를 만족 럽게 표 할

없었다 를 들어 한 어에서는 어 니 같이 음절 lsquo rsquo

이 어 단어가 얼마든 있으나 어는( ) 複數音節

자 하나 나타내 만이다lsquo [m ]rsquo 母 ǔ

한편 일 어의 표 은 핵 적 단어는 한자 적고 토는

가나라는 일 의 자 적는 이다 적인 의 를 나

타내는 은 표의 자인 한자 적고 적 계를 나

타내는 토는 표음 자 적는 셈이니 자세히 살펴

리의 향찰 표 을 쏙 빼닮았음을 알 있다 한 어 같

은 착어이 서도 일 어에만 향찰과 유사한 표 이 살아

남은 것은 일 어의 특 때 이다 일 어는 하나의 자음과

음의 결합으 음절을 이 고 침이 거의 없는 음절 언어

이다 이러한 음절의 특색에다가 토가 달한 착어라는 점

이 향찰과 유사한 표 이 살아남을 있는 비결이었다

하 만 같은 착어라도 다양한 음소 침이 달한 한

어는 향찰 표 하는 데 근 적으 한계가 있었다

zb51) 다 하여 의 행에 대한 탐 한 결과( ) lt gt 2

않은 것은

보lt gt

善花公主主隱 공주니믄 공주님( )

----------------------------------------

-

他密只嫁良置古 그 지 얼어 고 몰 결(

----------------------------------------

-

薯童房乙 맛 맛( )

夜矣卯乙抱遺去如 몰 고 가다 에 몰 고(

가다)

주동 역 동- (薯童謠『 』

에 2 ( )他密只嫁良置古

얼다 시집가다 결 다 말 lsquo rsquo

① 실질 미 지니고 므 타 타lsquo ( )rsquo lsquo [ ]

② 에 실질 미 타내고 지 는lsquo rsquo lsquo [ ]rsquo lsquo [ ]密只 密 只

계 타내는

③ 얼어는 실질 미 포 고 므 가lsquo rsquo lsquo [ ]rsquo嫁

것lsquo [ ]rsquo 良

④ 고 어간 는 실질 미 지니고 므lsquo rsquo lsquo -rsquo

것lsquo [ ]rsquo 置

⑤ 고 어미 고는 계 타내고 므lsquo rsquo lsquo- rsquo

고 것lsquo [ ]rsquo 古

가( )

엉 훈 민middot middot middot middot middot世 宗 御 製 訓 民 正 音

말 미 듕 귁에 달middot middot middot middot middot middot middot middot中 國 文 字

니 런middot middot middot middot middot middot 어린middot middot middot middot百 姓

니 고 도 내 들middot middot middot middot middot middot middot middot middot 시러middot

펴 몯middot 미middot middot 니 내middot middot middot middot middot middot middot middot 爲

어엿middot 겨 새middot middot middot 믈여듧middot middot middot middot字 니middot middot middot

사 마다 니겨 킈 middot middot middot middot middot middot middot middot middot便 安

고 미니middot middot middot middot

본 는 상( ) (象

원리에 만들어진 본) ( )形 ㄱ ㄴ ㅁ ㅅ ㅇ

에 는 가 원리에( )加劃

그리고( )ㅋ ㄷ ㅌ ㅂ ㅍ ㅈ ㅊ ㆆ ㅎ

쓰는 병 원리에 만들어진( )竝書

마지막 체( ) ( )異體ㄲ ㄸ ㅃ ㅆ ㅉ ㆅ

ᅀ 다 상 원리에 ㅇ ㄹ

지 는 삼재 상 본 본( ) ( ) ( 天地人 三才

탕 므림과 림에 ) (初ㅡ ㅣ

재)( ) ( )( )出字 再出字ㅗ ㅏ ㅜ ㅓ ㅛ ㅑ ㅜ ㅕ

병 그리고 들 에 다시( )ㅘ ㅝ ㅣ

( )ㅣ ㅢ ㅚ ㅐ ㅟ ㅔ ㆉ ㅒ ㆌ ㅖ ㅙ ㅞ

zb52) 가 에 대한 설 으 르 않은 것을( ) 두 고르

① 어쓰 규 지키고 다

② 리 고 다

③ 말 미 미 등 어 사 다lsquo rsquo

④ 개 지 다

년 학 간고사 대비2013 2 현대고 대비

ECN-0102-2013-001-000076193

⑤ 어 원 에 가 도 고 다

엉 훈 민世 宗 御 製 訓 民 正 音

말 미 듕귁에 달 니

런 어린 니 고 도middot

내 들 시러 펴 몯 미 니middot

내 어엿 겨 새 믈여듧

사 마다 니겨middot 킈 고

미니

훈민 언 본- lsquo rsquo 5 (1459 )

zb53) 위의 에 대한 현대어 풀이가 르~ 않은 것

① 우리 말 과 달

② 어리 말 고 는 것 어도

③ 신 생각 마 껏 펼 는 사 많다

④ 게 생각 여

⑤ 사 마다 게

zb54) 훈민정음 언해 에는 한 을 창제한 동 가 드러나

있다 훈민정음 창제의 정 과 내 이 잘 연결된 것

① 주 신 말 미 듕귁에 달

② 민 신 내 어 겨

③ 신 뻔 킈 고 미니

④ 실 신 사 마다 니겨

⑤ 귀 신 계 주 는 훈민 신과 거리가

가 엉 훈 민( ) middot middot middot middot middot世 宗 御 製 訓 民 正 音 

말 미 귁에 中 國 달 文 字

니 런 어린 니 百 姓

고 도 내 들 시러 펴 몯

미 니 내 어엿 爲 겨 새

믈여듧 니 사 마다 니 字

겨 킈 고 미니 便 安

훈민 언 본- lsquo ( )rsquo ( ) 5 (1459 )訓民正音 世祖

( )

[ 1 ]

동 룡 샤 마다 복( ) ( ) ( )海東 六龍 天福

시니 고 동( ) ( )古聖 同符 시니

[ 2 ]

매 니 곶 여

미 므 니 그 내 러

가 니

[ 125 ]

우 미리( )千世 샨( )定 에( )漢水北 累仁

누 개 샤 복 업 시니( ) ( ) 開國 卜年

신( )聖神 니 샤도 경 근민 샤 욱( )敬天勤民

드시리 다

님 쇼 산 가( ) ( )洛水 山行

미드니 가

어 가- lsquo ( )rsquo 27龍飛御天歌

다 우리신 니쓰고 다만 만 쓰( )

거 샹 귀쳔 다보게 러 귀

여 쓴 도 신 보 가 고 신 에

말 어 보게 각 에 사 들

고 본 몬 능통 후에

죠 죠 니

드 도 만 공 에 사

드 미 죠 고 고 여 보 죠

보다 얼마가 거시 어신고 니 첫

가 죠 니 죠

민 들 어 신 샹

귀쳔 도보고 어보 가 만 늘

고 폐 에 만쓴 죠 민

도 러보지못 고 보니 그게 엇지

심 니 리 보 가 어 운건 다

니 쳣 말마 지 니 고 그

쓰 에 가 우 지 지

몰 거 본후에 가 어 지

고 그니 쓴편지 쟝 보

년 학 간고사 대비2013 2 현대고 대비

ECN-0102-2013-001-000076193

쓴것보다 듸 보고 그 마 니 쓴 고

어 못

그런고 에 리 과 가

만 쓴 못 민 말만 듯고

고 편 그 못 보니 그사 단

병신 못 다고 그사 식 사

니 만 고 다 과 그사

만 고 다 과 업 사 보다 식 고

죠 도 고 각 과

견 고 실 직 귀쳔 간에 그

고도 다 것 몰 귀죡 보다

사 우리 신 귀쳔 다 업

시 신 보고 과 지 게 랴

시니 샹 귀쳔 간에 우리 신 걸

간 보 새지각과 새 걸 미리

독립신- lsquo (1896)rsquo

zb55) 친 어 나의 제 장( ) 2 매 함축적

의 가 가장 유사한 것은

① 지 눈 내리고 매 득 니 내 여 가

사- lsquo rsquo

② 도 어 리듯 그 게 어 다

주 사- lsquo rsquo

③ 눈 살 다 죽 어 린 과 체 여

눈 새벽 지 도 살 다

눈- lsquo rsquo

④ 삶 근심과 고단 에 돌 거니는 여 거 는

여 리 내린 살가지 에 눈 리 눈 리

택 그 생 에- lsquo rsquo

⑤ 늘 러 고 러

청룡 룡 어 개 루 우

신경림 계- lsquo rsquo

zb56) 친 를 위 가 나 에 나타난A B ( ) ( )

세 어의 특 에 의거하여 세 어 표 하

그 산 고 공 도 맑지만

A

주변에 쓰 리는 어리 사 많다

B

건lt gt

식 가 에 타 어 특징에( ) ( )

거 과 어쓰 는 고 지 말 것

A

B

zb57) 가 의( ) 달 아ㆍ 다 의 ( ) 나셔에서 알 있는

세 어 개화 어의 특 을 비 하여 조건 에lt gt

맞게 서 하

건lt gt

어에 는lsquo 개

어에 는 다 태rsquo

zb58) 은 가 는 다 에 나 는 절lt 1gt ( ) lt 2gt ( )

일 를 췌한 것이다 의 의 가 lt 1gt (1)~(2)

유사한 말을 에서 찾아 쓰lt 2gt

보lt 1gt

런 (1) 어린 니 고百 姓

도 내 들 시러 펴 몯 미

사 마다 (2) 니겨 便 安

킈 고 미니

보lt 2gt

죠 고 고 여 보 죠

보다 얼마가 거시 어신고 니 첫 가

죠 니 죠 민

들 어 신 샹 귀쳔

도보고 어보 가 만 늘 고

폐 에 만쓴 죠 민 도

러보지못 고 보니 그게 엇지 심

니 리

년 학 간고사 대비2013 2 현대고 대비

ECN-0102-2013-001-000076193

lt 1 gt

동 룡 샤 마다 복 시( ) ( ) ( )海東 六龍 天福

고 동 시니( ) ( )古聖 同符

lt 2 gt

(A) 매 니 곶

여 니

미 므 니 그 내

러 가 니

lt125 gt

우 미리 샨 에( ) ( ) ( ) 千世 定 漢水北 累

누 개 샤 복 업 시 니( ) ( ) 仁開國 卜年 聖

신( ) 神 니 샤도 경 근민 샤( ) 敬天勤民

욱 드 시 리 다

님 쇼 산 가 ( ) ( )洛水 山行

미드니 가

- lt gt龍飛御天歌

zb59) 장과 내 상 유사한 성격의 조는125

① 뫼 고 고 고 고

어 그린 많고 많고 고 고

어 러 는 울고 울고 가느니

도 견- lt gt

② 강 에 드니 몸 다

그믈 고 가니

뒷 뫼 엄 언 니( )藥

-

③ 말 없는 청산 태 없는 다

값 없는 청 없는 월

에 병 없는 몸 별 없 늙 리

-

④ 가마귀 골에 가지 마

낸 가마귀 새

청강에 것 시 몸 러 가( ) 淸江

-

⑤ 진 골에( ) 白雪

가 매 는 어느 곳에 었는고

에 갈 곳 몰( ) 夕陽

색-

zb60) 위 에 나타난 세 어의 특 으 적절하 않은

것은

① 룡 어 주격 사에 당 는 가 사( ) lsquo rsquo六龍

고 다

② 샤 어에도 어 주체 쓰 다

는 것 다

③ 매 어 달리 사 택에 어

가 지 지지 고 다

④ 므 원 상 직 어 지 다

⑤ 드시리 다 주체 과 상 께 사

고 다

수고 하셨습니다hearts hearts

년 학 간고사 대비2013 2 현대고 대비

ECN-0102-2013-001-000076193

보닷컴에 공 는 별 보는 고등

들 여 주 는

들 습니다 슷 동 지

가 복 는 것 도가

니 복 여 습 시고 거 시

니다

정답 해설

1) 정답[ ] ④

해설 다른 것은 두 특정 업이나 단 내에서 사[ ]

하는 일종의 은어 사회 언에 해당한다 러나

는 언이 아니라 단과대학을 여서 단대 사lsquo rsquo lsquo rsquo lsquo④

대학을 여서 사대라고 한 말에 해당하 일rsquo lsquo rsquo

사회에서도 널리 쓰이 사회 언이라 할

없다

2) 정답[ ] ⑤

해설 사회 언은 같은 단 내에서 쓰이는 언어이[ ] lsquo rsquo

동일 단끼리는 단결 과 친 감을 형성하는

능을 하 리적 안감이 일어나 않는다

3) 정답[ ] ③

해설 사람이라는 차 적 표현에 대한 대안적 표현이[ ]lsquo rsquo

인 아내 처 등으 볼 있다lsquo rsquo

4) 정답[ ]⑤

해설 남성은 주 격 체를 사 한다[ ]

5) 정답[ ] ⑤

해설 흑인은 검다라는 뜻을 가 고 있을 뿐 인[ ]lsquo rsquo lsquo rsquo lsquo rsquo

다 열등한 뜻을 내포하 않는다

6) 정답 살 색 첫 작품[ ] - -

해설 살색 혹은 킨색은 한 인의 피 색을 뜻[ ] lsquo rsquo lsquo rsquo

하는 것으 인종 차 을 추 고 출 이주민

의 평등 을 침해할 있어 년 표 이2005

살 색으 이름을 꾸었다 처녀작은 처녀라lsquo rsquo lsquo rsquo lsquo rsquo

는 단어가 가 고 있는 곡된 성 인 을 한 것

으 첫 작품정도 꾸어 사 하는 것이 좋다lsquo rsquo

7) 정답[ ] ⑤

해설 호는 아들에게 해체를 사 하고 있다[ ] ① ②

장 을 성하는 청자는 자 의 아 느리 아lsquo

들 세 이다 호는 아 느리에게 해rsquo ③

체를 사 하고 있다 호가 느리 아 에게 ④

사 한 해 체 아들에게 사 한 해체는 두 비lsquo rsquo lsquo rsquo

격 체에 해당한다 호는 자 의 아랫사람인 ⑤

느리에게 아들과 마찬가 해체를 사 하는 것이

상 이 만 임 을 한 느리에게 고마 과 쁨

존 의 표 를 하 위해 자 의 아 에게 말하듯

해 체를 사 하고 있다

8) 정답[ ] ③

9) 정답[ ] ⑤

10) 정답[ ] ①

해설 청자 할아 가 장의 주체 아 다 높을[ ] ( ) ( )

경 에는 압존 에 의해 장의 주체를 높이 않는lsquo rsquo

다 러 아 서가 아닌 아 는으 계 lsquo rsquo lsquo rsquo lsquo

니다 가 아닌 있 니다 표현하는 것이 르rsquo lsquo rsquo

11) 정답 당이 당을 쫒았다 당이[ ]

당에 다

해설[ ]

12) 정답[ ] ⑤

해설 서 다른 높임표현을 통해 청자에 대해 리[ ] ⑤

적 거리감을 나타내는 인 은 이 아니라 현정이

다 가 에서 현정은 에게 해 체를 사 함으 써 ( )

친근감을 드러낸다 나 에서 연 을 게을리하는 역 ( )

도 들 때 에 화가 난 현정이 선생님에게 항의하

는 장 에서는 하 체를 사 하여 리적 거리lsquo rsquo

가 어졌음을 나타내고 있다

13) 정답[ ] ①

해설 는 는 얼 빛이 날과 어찌 다르 고[ ] lsquo rsquo

라는 뜻으 전과 달리 임이 화자를 않고

있음을 알 있다

14) 정답 달리 후 가 있다 이를 통해 경[ ] lt gt

쾌한 음악성을 형성하고 노 젓는 상황을 체적으

형상화하는 역할을 한다

15) 정답[ ] ①

16) 정답[ ] ⑤

해설 다 의 자연은 를 성찰하게 하는 대상[ ] ( )⑤

이자 정의 대상이다 의 자연은 자 의 상황과 ⑤

처 를 드러내는 경으 서의 역할을 하 이

이 없다

17) 정답[ ] ③

해설 는 빈천 을 해결하고자 했으나 강산[ ] lsquo ( )rsquo 貧賤③

과 풍 을 달라는 에 거절하 다고 함으 써 자

연에 대한 애정을 드러내고 있으 는 않는

임에 대한 망을 개에게 전가 켜서 임에 대한 리

을 드러내고 있다

18) 정답[ ] ③

년 학 간고사 대비2013 2 현대고 대비

ECN-0102-2013-001-000076193

19) 정답[ ] ⑤

해설 고상한 음악가의 이름을 리말 꽝 럽[ ]

게 꿈으 써 언어유희를 통해 음을 유 하고 있

다 이는 고상한 척하는 총 를 비꼼으 써 비판적

태도를 드러내는 것이 대상을 꽝 럽게 표현

하여 총 의 허 과 사치를 풍자하고 있다

20) 정답[ ] ⑤

해설 는 작품 속 경에 대한 설 이 드러나는 것이[ ]

서 자의 주 적인 견해가 접적으 드러나는 것이

아니다

21) 정답[ ] ⑤

22) 정답[ ] ②

23) 정답[ ] ④

24) 정답[ ] ①

해설 적강 티프는 주인공의 비 한 출생이나 능[ ] ①

과 이 있는 것으 조정의 능함을 풍자하는lsquo rsquo

것과는 거리가 다

25) 정답 픔 나[ ] ( )

해설 의 음악은 고통 는 사람들을 위 하고 아픔[ ] lsquo rsquo

을 치유해 주는 능을 한다고 할 있다 의 lt gt

픔 도 소 된 이 과 더 어 살아가는 따뜻한 마음lsquo rsquo

을 상 한다

26) 정답[ ] ⑤

해설 에게 선천적으 주어 각 장애라는 역경[ ]

은 의 이라는 가사 연 을 있다lsquo rsquo

27) 정답[ ] ④

해설 는 장 란 선 에게 은 개인적인 인상을[ ]

소녀 장정 등으 표현한 것이다lsquo rsquo

28) 정답[ ] ②

해설 담자가 피 담자의 언어적 표현이나 비언어[ ]②

적 표현 하 독자는 담의 위 나 피

담자의 감정 상태를 알 있다 이를 통해 독자는

담 상황을 더 생생하게 느낄 있고 피 담자

를 더 잘 이해할 있게 된다

29) 정답[ ]③

해설 일상생활과 역도 선 서의 성과에 된 것에서[ ]

역도를 하 서 겪는 어 과 내적 고민으 화제를

전화하 위한 것이다

30) 정답[ ] ①

해설 릿속에 새겨 넣듯 이 억되도 함 세상[ ] ② ③

살이가 힘들고 고생 러 속 하여 자유를 ④

가 없는 고통의 상태를 비유적으 이르는 말

적의 침입을 막 위해 쌓은 축 켜야 할⑤

대상을 비유적으 이르는 말이다

31) 정답[ ] ④

해설 이 의 종류는 전 으 인 사건 경[ ] lsquo

비평을 성 소 삼는다rsquo

32) 정답[ ] ④

해설 근은 삼대독자 태어났음을 에서 확인할[ ]

있다 형제들과의 담은 이뤄 가 없다

33) 정답[ ] ⑤

해설 근은 가난에도 하고 화가를 꿈꾸었다[ ] (3

단 또한 다른 화가 망생들은 정 육을)

위해 상 학 학 해 유학 에 랐 만

근은 다른 을 찾아야 했다 단 세에(5 ) 18

근은 조선 전람회에 입선하 다 단 의(6 )

만종은 인간과 자연이 엮어 가는 경건한 조화 을lsquo rsquo

나타낸다

34) 정답[ ] ①

해설 근이 속에서도 창작활동을 추 않고[ ]

하는 닭은 은 세상과 타협할 르는

근이 세상의 이해를 하 위한 가장 떳떳한 단

이 때 이다

35) 정답[ ] ⑤

해설 전 은 서 자의 주 적인 평이 리는 것이[ ]

만 위 제 은 인 이 살았던 대 사회적 경

을 통해 객 적인 인 의 을 제 하고 있다

36) 정답[ ] ⑤

해설 전 은 인 사건 경 비평이라는[ ] lsquo rsquo⑤

성 이 어져 있다

37) 정답[ ] ①

해설 이 은 동양인과 서양인의 사고 에 차이가[ ]

있다는 것을 대조를 통해 설 하고 있다 또 쓴이

의 제자가 축 경 를 러 가서 경험한 일화를

통해 동양인이 서양인에 비해 주 상황에 더 많은

주의를 인다는 주장을 뒷 침하고 있다

38) 정답[ ] ④

39) 정답[ ] ②

40) 정답[ ] ②

41) 정답[ ] ④

42) 정답[ ] ③

43) 정답[ ] ④

44) 정답 도서 의 휴 일 도서 의 이 간 도서의[ ]

해설 도서 장은 임의 정한 휴 일과 도서 이[ ]

간 도서의 상 등을 게 할 의 가 있다

년 학 간고사 대비2013 2 현대고 대비

ECN-0102-2013-001-000076193

45) 정답[ ] ①

해설 제 조의 정 휴 일 의 휴 일의 사전 게[ ] 3

는 도서 장의 의 조항에 속한다

46) 정답[ ] ①

해설 개인 정 호 의 를 제 하 했 만 항[ ]

나눠서 제 하 않고 대 나열하고 있다

47) 정답[ ] ②

해설 제 조의 내 을 회사는 다른 회사 협[ ] 7 lsquo

계약을 통해 서비 를 제공하는 경 회 의 아이디

등 개인 정 를 해당 회사에 전송할 있다는 내rsquo

이 있으 의 제점을 제 할 있다②

48) 정답[ ] ④

해설 는 도서 장의 의 에 해당하고 나 는 도[ ] ④

서 장의 리에 해당한다

49) 정답[ ] ③

50) 정답 은 음독으 적었고 은 훈독으 적었[ ] (1)

다 과 동일한 표 리 적은 것은 이고 (2) ce

과 동일한 표 리 적은 것은 이다ab

51) 정답[ ] ③

52) 정답[ ] ①②

53) 정답[ ] ③

54) 정답[ ] ③

55) 정답[ ] ①

56) 정답 른 죠코 어린 노 하니라[ ] A B

57) 정답 세 어에서는 활 형이 칙적으[ ] lsquo rsquoㄹㅇ

나타났 만 개화 어에서는 활 형이 쓰 다 lsquo rsquo ㄹㄴ

58) 정답 호 가 흔[ ] (1) (2)

59) 정답[ ] ④

60) 정답[ ] ③

Page 29: 현대고대비 국어 - chamsoriedu.com 「콘텐츠산업진흥 법」외 에도 저작권 의하여 ... 다른주체에게어떤동작을하도록만드는것을나타내는

년 학 간고사 대비2013 2 현대고 대비

ECN-0102-2013-001-000076193

가 도 료 시 럽 거 거①

못 쓰게 거 어 린 경우에는 변상 여

도 에 변상 여 게시1②

여 다

zb44) 위 에서 도서 장이 게 해야 할 사항에 해당하는

것을 두 쓰

년 학 간고사 대비2013 2 현대고 대비

ECN-0102-2013-001-000076193

립 도 규

1 ( )

규 립 도 립 어린 청 도(

포 다 료 시 열 시 말) (

다 에 사 규 립 도)

편 진 다

2 ( )

규 립 도 도 다 에( lsquo rsquo )

고 는 도 에 도lsquo rsquo 2 2

료 에 여 다 다만 특 료 귀

료 등 료 에 사 립 도

도 다 다( lsquo rsquo )

3 ( )

도 다 각 같다①

공 공 다만 연1

연 간 다

매월 째 째 월2

도 도 리 그 사3

가 다고 는

도 에 미리 게1 3②

시 여 다

시간4 ( )

도 시간 도 여 게시 다

등 등5 ( )

도 료 시 는 는 도①

지에 등 후

등 에 사 도②

사 료6 ( )

도 료 시 에 사 료는 도

7 ( )

는 다 각 여 는 니 다

도 료 시 상 리1 lsquo rsquo

도 료 시 훼 는2 middot

지 가 닌 곳에 식 거 담3

우는

도 보 등 보 검색열4 middot

그 에 도 질 지 여 도5

여 게시 사 는

질 지8 ( )

도 다 거 도①

질 게 우 가 는 에 여는 도

도 가 각 어느7②

에는 지 게 거 도

료9 ( )

도 료는 다 각 경우 다①

상 도 간에 료 는 것 말1 (

다 등 다 도 과 여 경우)

공 원 공 상 는 경우2

그 에 도 다고 는 경우3

가능 도 료 는 도②

는 에 다

변상10 ( )

가 도 료 시 럽 거 거①

못 쓰게 거 어 린 경우에는 변상 여

도 에 변상 여 게시1②

여 다

등 규 에 것 에 도11 ( )

료 시 등에 사

도 다

립 도- (httpwwwnlgokr)

zb45) 도서 장의 리 있는 조항으 적절하 않

은 것은

① ② ③ ④ ⑤

년 학 간고사 대비2013 2 현대고 대비

ECN-0102-2013-001-000076193

1 ( )

사가 공 는lsquo rsquo

과 여 사 원과 리

사 타 사 규

니다

개 보 보7 ( )

사는 보통신망 등 계 는 에lsquo rsquo lsquo rsquo

원 개 보 보 니다 개lsquo rsquo

보 보 사 에 는 사 개lsquo rsquo

보 취 니다 다만 사는 다 lsquo rsquo

사 계 통 공 는 경우 원 lsquo rsquo

등 개 보 당 사에 습니lsquo rsquo

원 리에8 (lsquo rsquo lsquo rsquo lsquo rsquo

)

원 에 리lsquo rsquo lsquo rsquo lsquo rsquo①

원에게 가 도 여 는lsquo rsquo 3

니다

사는 원 가 개 보 우 가lsquo rsquo lsquo rsquo lsquo rsquo②

거 사 경우 는 미 에 어 거 lsquo

사 사 운 우 가 는 경우 당rsquo lsquo rsquo

습니다lsquo rsquo

원 가 도 거lsquo rsquo lsquo rsquo lsquo rsquo 3③

가 사 고 지 경우에는 시 사에lsquo rsquo

통지 고 사 내에 니다lsquo rsquo

경우에 당 원 사에 그 사실3 lsquo rsquo lsquo rsquo④

통지 지 거 통지 도 사 내에 지 lsquo rsquo

생 경우 사는 지지 습니다lsquo rsquo

사10 (lsquo rsquo )

사는 과 지 미lsquo rsquo①

에 는 지 계 고

공 여 다 여 니다lsquo rsquo

사는 원 게lsquo rsquo lsquo rsquo lsquo rsquo②

도 개 보 신 보 포 보 보 시( )

갖 어 개 보 취 공시 고

니다

사는 과 여 원lsquo rsquo lsquo rsquo③

견 만 당 다고 경우에는

리 여 니다 원 견 만 사 lsquo rsquo

에 는 게시 거 우편 등 통 여

원에게 리 과 결과 달 니다lsquo rsquo

원11 (lsquo rsquo )

원 다 여 는 니다lsquo rsquo ①

신청 는 변경 시 허 내 등1

타 보 도2

사가 게시 보 변경3 lsquo rsquo

사가 보 보 컴퓨 그4 lsquo rsquo (

등 등 신 는 게시)

사 타 등 지 재산 에5 lsquo rsquo 3

사 타 상 거 업6 lsquo rsquo 3

는 폭 시지 상 타 공7 middot middot

에 는 보 에 공개 는 게시 는lsquo rsquo

사 동 없 리 사8 lsquo rsquo

타 거 당9

게시15 (lsquo rsquo )

원 내에 게시 는 게시 게재 는lsquo rsquo lsquo rsquo lsquo rsquo

경우 원 사가 게시 복 lsquo rsquo lsquo rsquo lsquo rsquo middot middot

등 태 언 등에 공 는

것 내에 다 원 본 게시 등 lsquo rsquo lsquo rsquo

크 능 등 여 복 는 등 태

는 것 동 것 니다

- (wwwnavercom)

zb46) 위 은 인터넷 포털사이트의 회 가입을 위한 이

약 의 일 이다 이 약 을 만드는 과정에서 생각한

내 으 적절하 않은 것은

개 보 보 가 지에 별 눠①

겠어

원 가 만들게 에②

시 주어 겠어

원들 게재 게시 다 원 크 다③

는 것 지

④ 원 지 는 뿐만 니 사가 지 는

도 께 달 지

리에 가 생 경우 사가⑤

에 다는 도 듯

1 ( )

사가 공 는lsquo rsquo

과 여 사 원과 리

사 타 사 규

년 학 간고사 대비2013 2 현대고 대비

ECN-0102-2013-001-000076193

니다

개 보 보7 ( )

사는 보통신망 등 계 는 에lsquo rsquo lsquo rsquo

원 개 보 보 니다 개lsquo rsquo

보 보 사 에 는 사 개lsquo rsquo

보 취 니다 다만 사는 다 lsquo rsquo

사 계 통 공 는 경우 원 lsquo rsquo

등 개 보 당 사에 습니lsquo rsquo

원 리에8 (lsquo rsquo lsquo rsquo lsquo rsquo

)

원 에 리lsquo rsquo lsquo rsquo lsquo rsquo①

원에게 가 도 여 는lsquo rsquo 3

니다

사는 원 가 개 보 우 가lsquo rsquo lsquo rsquo lsquo rsquo②

거 사 경우 는 미 에 어 거 lsquo

사 사 운 우 가 는 경우 당rsquo lsquo rsquo

습니다lsquo rsquo

원 가 도 거lsquo rsquo lsquo rsquo lsquo rsquo 3③

가 사 고 지 경우에는 시 사에lsquo rsquo

통지 고 사 내에 니다lsquo rsquo

경우에 당 원 사에 그 사실3 lsquo rsquo lsquo rsquo④

통지 지 거 통지 도 사 내에 지 lsquo rsquo

생 경우 사는 지지 습니다lsquo rsquo

원에 통지9 (lsquo rsquo )

사는 특 다 원에게 통지 경우lsquo rsquo lsquo rsquo

공지 게시 통 상 게시 개별 통지에7

갈 습니다

사10 (lsquo rsquo )

사는 과 지 미lsquo rsquo①

에 는 지 계 고

공 여 다 여 니다lsquo rsquo

사는 원 게lsquo rsquo lsquo rsquo lsquo rsquo②

도 개 보 신 보 포 보 보 시( )

갖 어 개 보 취 공시 고

니다

사는 과 여 원lsquo rsquo lsquo rsquo③

견 만 당 다고 경우에는

리 여 니다 원 견 만 사 lsquo rsquo

에 는 게시 거 우편 등 통 여

원에게 리 과 결과 달 니다lsquo rsquo

원11 (lsquo rsquo )

원 다 여 는 니다lsquo rsquo ①

신청 는 변경 시 허 내 등1

타 보 도2

사가 게시 보 변경3 lsquo rsquo

사가 보 보 컴퓨 그4 lsquo rsquo (

등 등 신 는 게시)

사 타 등 지 재산 에5 lsquo rsquo 3

사 타 상 거 업6 lsquo rsquo 3

는 폭 시지 상 타 공7 middot middot

에 는 보 에 공개 는 게시 는lsquo rsquo

사 동 없 리 사8 lsquo rsquo

타 거 당9

원 계 규 내lsquo rsquo lsquo②

여 공지 주 사 사가 통지 는rsquo lsquo rsquo

사 등 여 타 사 업 에 lsquo rsquo

는 여 는 니다

- (wwwnavercom)

zb47) 위 약 의 조항에서 같은 제점을 하lt gt

고 있는 조항은

lt gt

제휴 회사에 회 의 아이디 개인 정 를 전송할 있도

한 조항은 고객에게 당한 조항이다

1 7 8① ② ③

④ 9 ⑤ 10

립 도 규

1 ( )

규 립 도 립 어린 청 도(

포 다 료 시 열 시 말) (

다 에 사 규 립 도)

편 진 다

2 ( )

규 립 도 도 다 에( lsquo rsquo )

고 는 도 에 도lsquo rsquo 2 2

료 에 여 다 다만 특 료 귀

료 등 료 에 사 립 도

도 다 다( lsquo rsquo )

3 ( )

도 다 각 같다①

공 공 다만 연1

연 간 다

년 학 간고사 대비2013 2 현대고 대비

ECN-0102-2013-001-000076193

매월 째 째 월2

도 도 리 그 사3

가 다고 는

도 에 미리 게1 3②

시 여 다

시간4 ( )

도 시간 도 여 게시 다

등 등5 ( )

도 료 시 는 는 도①

지에 등 후

등 에 사 도②

사 료6 ( )

도 료 시 에 사 료는 도

7 ( )

는 다 각 여 는 니 다

도 료 시 상 리1 lsquo rsquo

도 료 시 훼 는2 middot

지 가 닌 곳에 식 거 담3

우는

도 보 등 보 검색열4 middot

그 에 도 질 지 여 도5

여 게시 사 는

질 지8 ( )

도 다 거 도①

질 게 우 가 는 에 여는 도

도 가 각 어느7②

에는 지 게 거 도

료9 ( )

도 료는 다 각 경우 다①

상 도 간에 료 는 것 말1 (

다 등 다 도 과 여 경우)

공 원 공 상 는 경우2

그 에 도 다고 는 경우3

가능 도 료 는 도②

는 에 다

변상10 ( )

가 도 료 시 럽 거 거①

못 쓰게 거 어 린 경우에는 변상 여

도 에 변상 여 게시1②

여 다

등 규 에 것 에 도11 ( )

료 시 등에 사

도 다

립 도- (httpwwwnlgokr)

zb48) 다음 정 리 의 의 으 볼 때 가장

이 적인 것은

도 시간 도 여 게시 다①

등 에 사 도②

가능 도 료 는 도 는③

에 다

④ 도 에 변상 여 게10 1

시 여 다

⑤ 도 가 각 어느7

에는 지 거 도

zb49) 를 참고하여 이 어의 성격을 설 한lt gt

것으 적절하 않은 것은

① 보 에 는 어 시 상 고 어 시lt gt lsquo rsquo

에 보여주고 다

② 진 어 어원에 견 고 다

에는 타 어 들어가는 것 다 lsquo rsquo

③ 에 들어갈 말 각각 고 어 어 신 어~

들 언어는 질 격 강 통 없었다

④ 시 우리 에 가 었지만 지 계

과 달리 들 통 사 달 어 웠

년 학 간고사 대비2013 2 현대고 대비

ECN-0102-2013-001-000076193

⑤ 크 몽골 만주 공통어가 우리 어 같

계열에 다는 에 사 특 짐

가( )

善化公主主隱 공주님

他密只嫁良置古 몰 결 고

薯童房乙 맛

夜矣卯乙抱遣去如 에 몰 고 가다

( )

始汝 會隱日恚見隱扐 만 에 본

恥隱汝衣淸隱笑 맑 웃

고 시 여 공 크다 만 다[ ] ( ) ( ) ( ) ( )始 汝 會扐

내다 에 보다 견( ) ( )恚 見 다( )隱

럽다 맑다 청 웃( ) ( ) ( ) ( )恥 衣 淸 笑

zb50) 위의 나 를 함 고 음에 답하( ) lt gt

보lt gt

( )素那或云金川 白城郡蛇山人也

운 사산

는 고 다 는( )[ ( ) ] (素那 金川 白城

사산 사 다) ( ) 郡 蛇山

삼 사- lsquo rsquo 47

에 제 된 단어 의 표 리를 조건(1) lt gt ( ) lt gt

에 맞게 서 하

건lt gt

lsquo 었고 었다 태rsquo

에 제 된 단어 동일한 표 리에(2) lt gt ( )

의해 적은 것을 나 에서 찾아 조건 에 맞게 서 하( ) lt gt

건lt gt

에 당 는 각각( ) 개 쓸 것2 단

당 는 가 여러 개 어도 개만 쓸 것 각2

개 과 도 쪽에 개만2 2

드시 지 것( )

과 동 원리 것lsquo 고

과 동 원리 것 다rsquo

태 것

가( )

素那(或云金川) 白城郡蛇山人也

소나 또는 천 이라 한다 는 성 사( ) ( ) ( )素那 金川 白城郡〔 〕

산 사람이다 현대어 풀이( ) ( )蛇山

나( )

紫布岩乎希 회

執音乎手母牛放敎遣 자 손 암쇼 노히 고

吾 不喩慙 伊賜等肹 肹 나 안디 리샤

花 折叱肹 可獻乎理音如 고 것거 도림다

다 향찰은 리말을 리 으 적은 표 이었 만 생( )

은 고 대를 넘 하고 끊어 고 말았다 랜 세

동안 갈고 닦아 체계적이었던 향찰 표 이 사라졌

을 인은 크게 두 가 나누어 생각해 볼 있다

하나는 족 사회의 한 선호도에서 찾을 있다 라 때

향찰은 주 족 계 에서 사 했을 것으 인다 한 을

알 하고서는 한자를 활 하여 리말을 리 으 표

하 란 가능하 때 이다 런데 족들은 간이 흐

를 향찰과 같은 리 표 을 익혀 사 하 다는

아 한 을 대 사 하는 쪽을 선호하게 되었다 더 이

고 초에 인재 등 을 위해 과거제도가 행되 서 한 선

호도가 더 높아졌고 결 향찰은 소 되고 말았다

또 다른 가능성은 한 어의 특성에서 찾을 있다

터 한 과 일 세 나라는 한자 화 에 속해 다

당연한 이야 겠 만 표의 자인 한자는 어를 표 하

에 매 적절하다 어의 음절은 성 ( ) ( )聲母 韻母

이 어 고 여 에 성조가 추가되어 최종 소리가 결정된

다 래서 어는 단음절을 하나의 한자 표 하 된

다 에 초성 성 종성의 세 가 소가 하나의 음절

년 학 간고사 대비2013 2 현대고 대비

ECN-0102-2013-001-000076193

을 이 는 한 어는 음절 조가 잡하고 음절의 가 많아

서 한자 차 만으 한 어의 소리를 만족 럽게 표 할

없었다 를 들어 한 어에서는 어 니 같이 음절 lsquo rsquo

이 어 단어가 얼마든 있으나 어는( ) 複數音節

자 하나 나타내 만이다lsquo [m ]rsquo 母 ǔ

한편 일 어의 표 은 핵 적 단어는 한자 적고 토는

가나라는 일 의 자 적는 이다 적인 의 를 나

타내는 은 표의 자인 한자 적고 적 계를 나

타내는 토는 표음 자 적는 셈이니 자세히 살펴

리의 향찰 표 을 쏙 빼닮았음을 알 있다 한 어 같

은 착어이 서도 일 어에만 향찰과 유사한 표 이 살아

남은 것은 일 어의 특 때 이다 일 어는 하나의 자음과

음의 결합으 음절을 이 고 침이 거의 없는 음절 언어

이다 이러한 음절의 특색에다가 토가 달한 착어라는 점

이 향찰과 유사한 표 이 살아남을 있는 비결이었다

하 만 같은 착어라도 다양한 음소 침이 달한 한

어는 향찰 표 하는 데 근 적으 한계가 있었다

zb51) 다 하여 의 행에 대한 탐 한 결과( ) lt gt 2

않은 것은

보lt gt

善花公主主隱 공주니믄 공주님( )

----------------------------------------

-

他密只嫁良置古 그 지 얼어 고 몰 결(

----------------------------------------

-

薯童房乙 맛 맛( )

夜矣卯乙抱遺去如 몰 고 가다 에 몰 고(

가다)

주동 역 동- (薯童謠『 』

에 2 ( )他密只嫁良置古

얼다 시집가다 결 다 말 lsquo rsquo

① 실질 미 지니고 므 타 타lsquo ( )rsquo lsquo [ ]

② 에 실질 미 타내고 지 는lsquo rsquo lsquo [ ]rsquo lsquo [ ]密只 密 只

계 타내는

③ 얼어는 실질 미 포 고 므 가lsquo rsquo lsquo [ ]rsquo嫁

것lsquo [ ]rsquo 良

④ 고 어간 는 실질 미 지니고 므lsquo rsquo lsquo -rsquo

것lsquo [ ]rsquo 置

⑤ 고 어미 고는 계 타내고 므lsquo rsquo lsquo- rsquo

고 것lsquo [ ]rsquo 古

가( )

엉 훈 민middot middot middot middot middot世 宗 御 製 訓 民 正 音

말 미 듕 귁에 달middot middot middot middot middot middot middot middot中 國 文 字

니 런middot middot middot middot middot middot 어린middot middot middot middot百 姓

니 고 도 내 들middot middot middot middot middot middot middot middot middot 시러middot

펴 몯middot 미middot middot 니 내middot middot middot middot middot middot middot middot 爲

어엿middot 겨 새middot middot middot 믈여듧middot middot middot middot字 니middot middot middot

사 마다 니겨 킈 middot middot middot middot middot middot middot middot middot便 安

고 미니middot middot middot middot

본 는 상( ) (象

원리에 만들어진 본) ( )形 ㄱ ㄴ ㅁ ㅅ ㅇ

에 는 가 원리에( )加劃

그리고( )ㅋ ㄷ ㅌ ㅂ ㅍ ㅈ ㅊ ㆆ ㅎ

쓰는 병 원리에 만들어진( )竝書

마지막 체( ) ( )異體ㄲ ㄸ ㅃ ㅆ ㅉ ㆅ

ᅀ 다 상 원리에 ㅇ ㄹ

지 는 삼재 상 본 본( ) ( ) ( 天地人 三才

탕 므림과 림에 ) (初ㅡ ㅣ

재)( ) ( )( )出字 再出字ㅗ ㅏ ㅜ ㅓ ㅛ ㅑ ㅜ ㅕ

병 그리고 들 에 다시( )ㅘ ㅝ ㅣ

( )ㅣ ㅢ ㅚ ㅐ ㅟ ㅔ ㆉ ㅒ ㆌ ㅖ ㅙ ㅞ

zb52) 가 에 대한 설 으 르 않은 것을( ) 두 고르

① 어쓰 규 지키고 다

② 리 고 다

③ 말 미 미 등 어 사 다lsquo rsquo

④ 개 지 다

년 학 간고사 대비2013 2 현대고 대비

ECN-0102-2013-001-000076193

⑤ 어 원 에 가 도 고 다

엉 훈 민世 宗 御 製 訓 民 正 音

말 미 듕귁에 달 니

런 어린 니 고 도middot

내 들 시러 펴 몯 미 니middot

내 어엿 겨 새 믈여듧

사 마다 니겨middot 킈 고

미니

훈민 언 본- lsquo rsquo 5 (1459 )

zb53) 위의 에 대한 현대어 풀이가 르~ 않은 것

① 우리 말 과 달

② 어리 말 고 는 것 어도

③ 신 생각 마 껏 펼 는 사 많다

④ 게 생각 여

⑤ 사 마다 게

zb54) 훈민정음 언해 에는 한 을 창제한 동 가 드러나

있다 훈민정음 창제의 정 과 내 이 잘 연결된 것

① 주 신 말 미 듕귁에 달

② 민 신 내 어 겨

③ 신 뻔 킈 고 미니

④ 실 신 사 마다 니겨

⑤ 귀 신 계 주 는 훈민 신과 거리가

가 엉 훈 민( ) middot middot middot middot middot世 宗 御 製 訓 民 正 音 

말 미 귁에 中 國 달 文 字

니 런 어린 니 百 姓

고 도 내 들 시러 펴 몯

미 니 내 어엿 爲 겨 새

믈여듧 니 사 마다 니 字

겨 킈 고 미니 便 安

훈민 언 본- lsquo ( )rsquo ( ) 5 (1459 )訓民正音 世祖

( )

[ 1 ]

동 룡 샤 마다 복( ) ( ) ( )海東 六龍 天福

시니 고 동( ) ( )古聖 同符 시니

[ 2 ]

매 니 곶 여

미 므 니 그 내 러

가 니

[ 125 ]

우 미리( )千世 샨( )定 에( )漢水北 累仁

누 개 샤 복 업 시니( ) ( ) 開國 卜年

신( )聖神 니 샤도 경 근민 샤 욱( )敬天勤民

드시리 다

님 쇼 산 가( ) ( )洛水 山行

미드니 가

어 가- lsquo ( )rsquo 27龍飛御天歌

다 우리신 니쓰고 다만 만 쓰( )

거 샹 귀쳔 다보게 러 귀

여 쓴 도 신 보 가 고 신 에

말 어 보게 각 에 사 들

고 본 몬 능통 후에

죠 죠 니

드 도 만 공 에 사

드 미 죠 고 고 여 보 죠

보다 얼마가 거시 어신고 니 첫

가 죠 니 죠

민 들 어 신 샹

귀쳔 도보고 어보 가 만 늘

고 폐 에 만쓴 죠 민

도 러보지못 고 보니 그게 엇지

심 니 리 보 가 어 운건 다

니 쳣 말마 지 니 고 그

쓰 에 가 우 지 지

몰 거 본후에 가 어 지

고 그니 쓴편지 쟝 보

년 학 간고사 대비2013 2 현대고 대비

ECN-0102-2013-001-000076193

쓴것보다 듸 보고 그 마 니 쓴 고

어 못

그런고 에 리 과 가

만 쓴 못 민 말만 듯고

고 편 그 못 보니 그사 단

병신 못 다고 그사 식 사

니 만 고 다 과 그사

만 고 다 과 업 사 보다 식 고

죠 도 고 각 과

견 고 실 직 귀쳔 간에 그

고도 다 것 몰 귀죡 보다

사 우리 신 귀쳔 다 업

시 신 보고 과 지 게 랴

시니 샹 귀쳔 간에 우리 신 걸

간 보 새지각과 새 걸 미리

독립신- lsquo (1896)rsquo

zb55) 친 어 나의 제 장( ) 2 매 함축적

의 가 가장 유사한 것은

① 지 눈 내리고 매 득 니 내 여 가

사- lsquo rsquo

② 도 어 리듯 그 게 어 다

주 사- lsquo rsquo

③ 눈 살 다 죽 어 린 과 체 여

눈 새벽 지 도 살 다

눈- lsquo rsquo

④ 삶 근심과 고단 에 돌 거니는 여 거 는

여 리 내린 살가지 에 눈 리 눈 리

택 그 생 에- lsquo rsquo

⑤ 늘 러 고 러

청룡 룡 어 개 루 우

신경림 계- lsquo rsquo

zb56) 친 를 위 가 나 에 나타난A B ( ) ( )

세 어의 특 에 의거하여 세 어 표 하

그 산 고 공 도 맑지만

A

주변에 쓰 리는 어리 사 많다

B

건lt gt

식 가 에 타 어 특징에( ) ( )

거 과 어쓰 는 고 지 말 것

A

B

zb57) 가 의( ) 달 아ㆍ 다 의 ( ) 나셔에서 알 있는

세 어 개화 어의 특 을 비 하여 조건 에lt gt

맞게 서 하

건lt gt

어에 는lsquo 개

어에 는 다 태rsquo

zb58) 은 가 는 다 에 나 는 절lt 1gt ( ) lt 2gt ( )

일 를 췌한 것이다 의 의 가 lt 1gt (1)~(2)

유사한 말을 에서 찾아 쓰lt 2gt

보lt 1gt

런 (1) 어린 니 고百 姓

도 내 들 시러 펴 몯 미

사 마다 (2) 니겨 便 安

킈 고 미니

보lt 2gt

죠 고 고 여 보 죠

보다 얼마가 거시 어신고 니 첫 가

죠 니 죠 민

들 어 신 샹 귀쳔

도보고 어보 가 만 늘 고

폐 에 만쓴 죠 민 도

러보지못 고 보니 그게 엇지 심

니 리

년 학 간고사 대비2013 2 현대고 대비

ECN-0102-2013-001-000076193

lt 1 gt

동 룡 샤 마다 복 시( ) ( ) ( )海東 六龍 天福

고 동 시니( ) ( )古聖 同符

lt 2 gt

(A) 매 니 곶

여 니

미 므 니 그 내

러 가 니

lt125 gt

우 미리 샨 에( ) ( ) ( ) 千世 定 漢水北 累

누 개 샤 복 업 시 니( ) ( ) 仁開國 卜年 聖

신( ) 神 니 샤도 경 근민 샤( ) 敬天勤民

욱 드 시 리 다

님 쇼 산 가 ( ) ( )洛水 山行

미드니 가

- lt gt龍飛御天歌

zb59) 장과 내 상 유사한 성격의 조는125

① 뫼 고 고 고 고

어 그린 많고 많고 고 고

어 러 는 울고 울고 가느니

도 견- lt gt

② 강 에 드니 몸 다

그믈 고 가니

뒷 뫼 엄 언 니( )藥

-

③ 말 없는 청산 태 없는 다

값 없는 청 없는 월

에 병 없는 몸 별 없 늙 리

-

④ 가마귀 골에 가지 마

낸 가마귀 새

청강에 것 시 몸 러 가( ) 淸江

-

⑤ 진 골에( ) 白雪

가 매 는 어느 곳에 었는고

에 갈 곳 몰( ) 夕陽

색-

zb60) 위 에 나타난 세 어의 특 으 적절하 않은

것은

① 룡 어 주격 사에 당 는 가 사( ) lsquo rsquo六龍

고 다

② 샤 어에도 어 주체 쓰 다

는 것 다

③ 매 어 달리 사 택에 어

가 지 지지 고 다

④ 므 원 상 직 어 지 다

⑤ 드시리 다 주체 과 상 께 사

고 다

수고 하셨습니다hearts hearts

년 학 간고사 대비2013 2 현대고 대비

ECN-0102-2013-001-000076193

보닷컴에 공 는 별 보는 고등

들 여 주 는

들 습니다 슷 동 지

가 복 는 것 도가

니 복 여 습 시고 거 시

니다

정답 해설

1) 정답[ ] ④

해설 다른 것은 두 특정 업이나 단 내에서 사[ ]

하는 일종의 은어 사회 언에 해당한다 러나

는 언이 아니라 단과대학을 여서 단대 사lsquo rsquo lsquo rsquo lsquo④

대학을 여서 사대라고 한 말에 해당하 일rsquo lsquo rsquo

사회에서도 널리 쓰이 사회 언이라 할

없다

2) 정답[ ] ⑤

해설 사회 언은 같은 단 내에서 쓰이는 언어이[ ] lsquo rsquo

동일 단끼리는 단결 과 친 감을 형성하는

능을 하 리적 안감이 일어나 않는다

3) 정답[ ] ③

해설 사람이라는 차 적 표현에 대한 대안적 표현이[ ]lsquo rsquo

인 아내 처 등으 볼 있다lsquo rsquo

4) 정답[ ]⑤

해설 남성은 주 격 체를 사 한다[ ]

5) 정답[ ] ⑤

해설 흑인은 검다라는 뜻을 가 고 있을 뿐 인[ ]lsquo rsquo lsquo rsquo lsquo rsquo

다 열등한 뜻을 내포하 않는다

6) 정답 살 색 첫 작품[ ] - -

해설 살색 혹은 킨색은 한 인의 피 색을 뜻[ ] lsquo rsquo lsquo rsquo

하는 것으 인종 차 을 추 고 출 이주민

의 평등 을 침해할 있어 년 표 이2005

살 색으 이름을 꾸었다 처녀작은 처녀라lsquo rsquo lsquo rsquo lsquo rsquo

는 단어가 가 고 있는 곡된 성 인 을 한 것

으 첫 작품정도 꾸어 사 하는 것이 좋다lsquo rsquo

7) 정답[ ] ⑤

해설 호는 아들에게 해체를 사 하고 있다[ ] ① ②

장 을 성하는 청자는 자 의 아 느리 아lsquo

들 세 이다 호는 아 느리에게 해rsquo ③

체를 사 하고 있다 호가 느리 아 에게 ④

사 한 해 체 아들에게 사 한 해체는 두 비lsquo rsquo lsquo rsquo

격 체에 해당한다 호는 자 의 아랫사람인 ⑤

느리에게 아들과 마찬가 해체를 사 하는 것이

상 이 만 임 을 한 느리에게 고마 과 쁨

존 의 표 를 하 위해 자 의 아 에게 말하듯

해 체를 사 하고 있다

8) 정답[ ] ③

9) 정답[ ] ⑤

10) 정답[ ] ①

해설 청자 할아 가 장의 주체 아 다 높을[ ] ( ) ( )

경 에는 압존 에 의해 장의 주체를 높이 않는lsquo rsquo

다 러 아 서가 아닌 아 는으 계 lsquo rsquo lsquo rsquo lsquo

니다 가 아닌 있 니다 표현하는 것이 르rsquo lsquo rsquo

11) 정답 당이 당을 쫒았다 당이[ ]

당에 다

해설[ ]

12) 정답[ ] ⑤

해설 서 다른 높임표현을 통해 청자에 대해 리[ ] ⑤

적 거리감을 나타내는 인 은 이 아니라 현정이

다 가 에서 현정은 에게 해 체를 사 함으 써 ( )

친근감을 드러낸다 나 에서 연 을 게을리하는 역 ( )

도 들 때 에 화가 난 현정이 선생님에게 항의하

는 장 에서는 하 체를 사 하여 리적 거리lsquo rsquo

가 어졌음을 나타내고 있다

13) 정답[ ] ①

해설 는 는 얼 빛이 날과 어찌 다르 고[ ] lsquo rsquo

라는 뜻으 전과 달리 임이 화자를 않고

있음을 알 있다

14) 정답 달리 후 가 있다 이를 통해 경[ ] lt gt

쾌한 음악성을 형성하고 노 젓는 상황을 체적으

형상화하는 역할을 한다

15) 정답[ ] ①

16) 정답[ ] ⑤

해설 다 의 자연은 를 성찰하게 하는 대상[ ] ( )⑤

이자 정의 대상이다 의 자연은 자 의 상황과 ⑤

처 를 드러내는 경으 서의 역할을 하 이

이 없다

17) 정답[ ] ③

해설 는 빈천 을 해결하고자 했으나 강산[ ] lsquo ( )rsquo 貧賤③

과 풍 을 달라는 에 거절하 다고 함으 써 자

연에 대한 애정을 드러내고 있으 는 않는

임에 대한 망을 개에게 전가 켜서 임에 대한 리

을 드러내고 있다

18) 정답[ ] ③

년 학 간고사 대비2013 2 현대고 대비

ECN-0102-2013-001-000076193

19) 정답[ ] ⑤

해설 고상한 음악가의 이름을 리말 꽝 럽[ ]

게 꿈으 써 언어유희를 통해 음을 유 하고 있

다 이는 고상한 척하는 총 를 비꼼으 써 비판적

태도를 드러내는 것이 대상을 꽝 럽게 표현

하여 총 의 허 과 사치를 풍자하고 있다

20) 정답[ ] ⑤

해설 는 작품 속 경에 대한 설 이 드러나는 것이[ ]

서 자의 주 적인 견해가 접적으 드러나는 것이

아니다

21) 정답[ ] ⑤

22) 정답[ ] ②

23) 정답[ ] ④

24) 정답[ ] ①

해설 적강 티프는 주인공의 비 한 출생이나 능[ ] ①

과 이 있는 것으 조정의 능함을 풍자하는lsquo rsquo

것과는 거리가 다

25) 정답 픔 나[ ] ( )

해설 의 음악은 고통 는 사람들을 위 하고 아픔[ ] lsquo rsquo

을 치유해 주는 능을 한다고 할 있다 의 lt gt

픔 도 소 된 이 과 더 어 살아가는 따뜻한 마음lsquo rsquo

을 상 한다

26) 정답[ ] ⑤

해설 에게 선천적으 주어 각 장애라는 역경[ ]

은 의 이라는 가사 연 을 있다lsquo rsquo

27) 정답[ ] ④

해설 는 장 란 선 에게 은 개인적인 인상을[ ]

소녀 장정 등으 표현한 것이다lsquo rsquo

28) 정답[ ] ②

해설 담자가 피 담자의 언어적 표현이나 비언어[ ]②

적 표현 하 독자는 담의 위 나 피

담자의 감정 상태를 알 있다 이를 통해 독자는

담 상황을 더 생생하게 느낄 있고 피 담자

를 더 잘 이해할 있게 된다

29) 정답[ ]③

해설 일상생활과 역도 선 서의 성과에 된 것에서[ ]

역도를 하 서 겪는 어 과 내적 고민으 화제를

전화하 위한 것이다

30) 정답[ ] ①

해설 릿속에 새겨 넣듯 이 억되도 함 세상[ ] ② ③

살이가 힘들고 고생 러 속 하여 자유를 ④

가 없는 고통의 상태를 비유적으 이르는 말

적의 침입을 막 위해 쌓은 축 켜야 할⑤

대상을 비유적으 이르는 말이다

31) 정답[ ] ④

해설 이 의 종류는 전 으 인 사건 경[ ] lsquo

비평을 성 소 삼는다rsquo

32) 정답[ ] ④

해설 근은 삼대독자 태어났음을 에서 확인할[ ]

있다 형제들과의 담은 이뤄 가 없다

33) 정답[ ] ⑤

해설 근은 가난에도 하고 화가를 꿈꾸었다[ ] (3

단 또한 다른 화가 망생들은 정 육을)

위해 상 학 학 해 유학 에 랐 만

근은 다른 을 찾아야 했다 단 세에(5 ) 18

근은 조선 전람회에 입선하 다 단 의(6 )

만종은 인간과 자연이 엮어 가는 경건한 조화 을lsquo rsquo

나타낸다

34) 정답[ ] ①

해설 근이 속에서도 창작활동을 추 않고[ ]

하는 닭은 은 세상과 타협할 르는

근이 세상의 이해를 하 위한 가장 떳떳한 단

이 때 이다

35) 정답[ ] ⑤

해설 전 은 서 자의 주 적인 평이 리는 것이[ ]

만 위 제 은 인 이 살았던 대 사회적 경

을 통해 객 적인 인 의 을 제 하고 있다

36) 정답[ ] ⑤

해설 전 은 인 사건 경 비평이라는[ ] lsquo rsquo⑤

성 이 어져 있다

37) 정답[ ] ①

해설 이 은 동양인과 서양인의 사고 에 차이가[ ]

있다는 것을 대조를 통해 설 하고 있다 또 쓴이

의 제자가 축 경 를 러 가서 경험한 일화를

통해 동양인이 서양인에 비해 주 상황에 더 많은

주의를 인다는 주장을 뒷 침하고 있다

38) 정답[ ] ④

39) 정답[ ] ②

40) 정답[ ] ②

41) 정답[ ] ④

42) 정답[ ] ③

43) 정답[ ] ④

44) 정답 도서 의 휴 일 도서 의 이 간 도서의[ ]

해설 도서 장은 임의 정한 휴 일과 도서 이[ ]

간 도서의 상 등을 게 할 의 가 있다

년 학 간고사 대비2013 2 현대고 대비

ECN-0102-2013-001-000076193

45) 정답[ ] ①

해설 제 조의 정 휴 일 의 휴 일의 사전 게[ ] 3

는 도서 장의 의 조항에 속한다

46) 정답[ ] ①

해설 개인 정 호 의 를 제 하 했 만 항[ ]

나눠서 제 하 않고 대 나열하고 있다

47) 정답[ ] ②

해설 제 조의 내 을 회사는 다른 회사 협[ ] 7 lsquo

계약을 통해 서비 를 제공하는 경 회 의 아이디

등 개인 정 를 해당 회사에 전송할 있다는 내rsquo

이 있으 의 제점을 제 할 있다②

48) 정답[ ] ④

해설 는 도서 장의 의 에 해당하고 나 는 도[ ] ④

서 장의 리에 해당한다

49) 정답[ ] ③

50) 정답 은 음독으 적었고 은 훈독으 적었[ ] (1)

다 과 동일한 표 리 적은 것은 이고 (2) ce

과 동일한 표 리 적은 것은 이다ab

51) 정답[ ] ③

52) 정답[ ] ①②

53) 정답[ ] ③

54) 정답[ ] ③

55) 정답[ ] ①

56) 정답 른 죠코 어린 노 하니라[ ] A B

57) 정답 세 어에서는 활 형이 칙적으[ ] lsquo rsquoㄹㅇ

나타났 만 개화 어에서는 활 형이 쓰 다 lsquo rsquo ㄹㄴ

58) 정답 호 가 흔[ ] (1) (2)

59) 정답[ ] ④

60) 정답[ ] ③

Page 30: 현대고대비 국어 - chamsoriedu.com 「콘텐츠산업진흥 법」외 에도 저작권 의하여 ... 다른주체에게어떤동작을하도록만드는것을나타내는

년 학 간고사 대비2013 2 현대고 대비

ECN-0102-2013-001-000076193

립 도 규

1 ( )

규 립 도 립 어린 청 도(

포 다 료 시 열 시 말) (

다 에 사 규 립 도)

편 진 다

2 ( )

규 립 도 도 다 에( lsquo rsquo )

고 는 도 에 도lsquo rsquo 2 2

료 에 여 다 다만 특 료 귀

료 등 료 에 사 립 도

도 다 다( lsquo rsquo )

3 ( )

도 다 각 같다①

공 공 다만 연1

연 간 다

매월 째 째 월2

도 도 리 그 사3

가 다고 는

도 에 미리 게1 3②

시 여 다

시간4 ( )

도 시간 도 여 게시 다

등 등5 ( )

도 료 시 는 는 도①

지에 등 후

등 에 사 도②

사 료6 ( )

도 료 시 에 사 료는 도

7 ( )

는 다 각 여 는 니 다

도 료 시 상 리1 lsquo rsquo

도 료 시 훼 는2 middot

지 가 닌 곳에 식 거 담3

우는

도 보 등 보 검색열4 middot

그 에 도 질 지 여 도5

여 게시 사 는

질 지8 ( )

도 다 거 도①

질 게 우 가 는 에 여는 도

도 가 각 어느7②

에는 지 게 거 도

료9 ( )

도 료는 다 각 경우 다①

상 도 간에 료 는 것 말1 (

다 등 다 도 과 여 경우)

공 원 공 상 는 경우2

그 에 도 다고 는 경우3

가능 도 료 는 도②

는 에 다

변상10 ( )

가 도 료 시 럽 거 거①

못 쓰게 거 어 린 경우에는 변상 여

도 에 변상 여 게시1②

여 다

등 규 에 것 에 도11 ( )

료 시 등에 사

도 다

립 도- (httpwwwnlgokr)

zb45) 도서 장의 리 있는 조항으 적절하 않

은 것은

① ② ③ ④ ⑤

년 학 간고사 대비2013 2 현대고 대비

ECN-0102-2013-001-000076193

1 ( )

사가 공 는lsquo rsquo

과 여 사 원과 리

사 타 사 규

니다

개 보 보7 ( )

사는 보통신망 등 계 는 에lsquo rsquo lsquo rsquo

원 개 보 보 니다 개lsquo rsquo

보 보 사 에 는 사 개lsquo rsquo

보 취 니다 다만 사는 다 lsquo rsquo

사 계 통 공 는 경우 원 lsquo rsquo

등 개 보 당 사에 습니lsquo rsquo

원 리에8 (lsquo rsquo lsquo rsquo lsquo rsquo

)

원 에 리lsquo rsquo lsquo rsquo lsquo rsquo①

원에게 가 도 여 는lsquo rsquo 3

니다

사는 원 가 개 보 우 가lsquo rsquo lsquo rsquo lsquo rsquo②

거 사 경우 는 미 에 어 거 lsquo

사 사 운 우 가 는 경우 당rsquo lsquo rsquo

습니다lsquo rsquo

원 가 도 거lsquo rsquo lsquo rsquo lsquo rsquo 3③

가 사 고 지 경우에는 시 사에lsquo rsquo

통지 고 사 내에 니다lsquo rsquo

경우에 당 원 사에 그 사실3 lsquo rsquo lsquo rsquo④

통지 지 거 통지 도 사 내에 지 lsquo rsquo

생 경우 사는 지지 습니다lsquo rsquo

사10 (lsquo rsquo )

사는 과 지 미lsquo rsquo①

에 는 지 계 고

공 여 다 여 니다lsquo rsquo

사는 원 게lsquo rsquo lsquo rsquo lsquo rsquo②

도 개 보 신 보 포 보 보 시( )

갖 어 개 보 취 공시 고

니다

사는 과 여 원lsquo rsquo lsquo rsquo③

견 만 당 다고 경우에는

리 여 니다 원 견 만 사 lsquo rsquo

에 는 게시 거 우편 등 통 여

원에게 리 과 결과 달 니다lsquo rsquo

원11 (lsquo rsquo )

원 다 여 는 니다lsquo rsquo ①

신청 는 변경 시 허 내 등1

타 보 도2

사가 게시 보 변경3 lsquo rsquo

사가 보 보 컴퓨 그4 lsquo rsquo (

등 등 신 는 게시)

사 타 등 지 재산 에5 lsquo rsquo 3

사 타 상 거 업6 lsquo rsquo 3

는 폭 시지 상 타 공7 middot middot

에 는 보 에 공개 는 게시 는lsquo rsquo

사 동 없 리 사8 lsquo rsquo

타 거 당9

게시15 (lsquo rsquo )

원 내에 게시 는 게시 게재 는lsquo rsquo lsquo rsquo lsquo rsquo

경우 원 사가 게시 복 lsquo rsquo lsquo rsquo lsquo rsquo middot middot

등 태 언 등에 공 는

것 내에 다 원 본 게시 등 lsquo rsquo lsquo rsquo

크 능 등 여 복 는 등 태

는 것 동 것 니다

- (wwwnavercom)

zb46) 위 은 인터넷 포털사이트의 회 가입을 위한 이

약 의 일 이다 이 약 을 만드는 과정에서 생각한

내 으 적절하 않은 것은

개 보 보 가 지에 별 눠①

겠어

원 가 만들게 에②

시 주어 겠어

원들 게재 게시 다 원 크 다③

는 것 지

④ 원 지 는 뿐만 니 사가 지 는

도 께 달 지

리에 가 생 경우 사가⑤

에 다는 도 듯

1 ( )

사가 공 는lsquo rsquo

과 여 사 원과 리

사 타 사 규

년 학 간고사 대비2013 2 현대고 대비

ECN-0102-2013-001-000076193

니다

개 보 보7 ( )

사는 보통신망 등 계 는 에lsquo rsquo lsquo rsquo

원 개 보 보 니다 개lsquo rsquo

보 보 사 에 는 사 개lsquo rsquo

보 취 니다 다만 사는 다 lsquo rsquo

사 계 통 공 는 경우 원 lsquo rsquo

등 개 보 당 사에 습니lsquo rsquo

원 리에8 (lsquo rsquo lsquo rsquo lsquo rsquo

)

원 에 리lsquo rsquo lsquo rsquo lsquo rsquo①

원에게 가 도 여 는lsquo rsquo 3

니다

사는 원 가 개 보 우 가lsquo rsquo lsquo rsquo lsquo rsquo②

거 사 경우 는 미 에 어 거 lsquo

사 사 운 우 가 는 경우 당rsquo lsquo rsquo

습니다lsquo rsquo

원 가 도 거lsquo rsquo lsquo rsquo lsquo rsquo 3③

가 사 고 지 경우에는 시 사에lsquo rsquo

통지 고 사 내에 니다lsquo rsquo

경우에 당 원 사에 그 사실3 lsquo rsquo lsquo rsquo④

통지 지 거 통지 도 사 내에 지 lsquo rsquo

생 경우 사는 지지 습니다lsquo rsquo

원에 통지9 (lsquo rsquo )

사는 특 다 원에게 통지 경우lsquo rsquo lsquo rsquo

공지 게시 통 상 게시 개별 통지에7

갈 습니다

사10 (lsquo rsquo )

사는 과 지 미lsquo rsquo①

에 는 지 계 고

공 여 다 여 니다lsquo rsquo

사는 원 게lsquo rsquo lsquo rsquo lsquo rsquo②

도 개 보 신 보 포 보 보 시( )

갖 어 개 보 취 공시 고

니다

사는 과 여 원lsquo rsquo lsquo rsquo③

견 만 당 다고 경우에는

리 여 니다 원 견 만 사 lsquo rsquo

에 는 게시 거 우편 등 통 여

원에게 리 과 결과 달 니다lsquo rsquo

원11 (lsquo rsquo )

원 다 여 는 니다lsquo rsquo ①

신청 는 변경 시 허 내 등1

타 보 도2

사가 게시 보 변경3 lsquo rsquo

사가 보 보 컴퓨 그4 lsquo rsquo (

등 등 신 는 게시)

사 타 등 지 재산 에5 lsquo rsquo 3

사 타 상 거 업6 lsquo rsquo 3

는 폭 시지 상 타 공7 middot middot

에 는 보 에 공개 는 게시 는lsquo rsquo

사 동 없 리 사8 lsquo rsquo

타 거 당9

원 계 규 내lsquo rsquo lsquo②

여 공지 주 사 사가 통지 는rsquo lsquo rsquo

사 등 여 타 사 업 에 lsquo rsquo

는 여 는 니다

- (wwwnavercom)

zb47) 위 약 의 조항에서 같은 제점을 하lt gt

고 있는 조항은

lt gt

제휴 회사에 회 의 아이디 개인 정 를 전송할 있도

한 조항은 고객에게 당한 조항이다

1 7 8① ② ③

④ 9 ⑤ 10

립 도 규

1 ( )

규 립 도 립 어린 청 도(

포 다 료 시 열 시 말) (

다 에 사 규 립 도)

편 진 다

2 ( )

규 립 도 도 다 에( lsquo rsquo )

고 는 도 에 도lsquo rsquo 2 2

료 에 여 다 다만 특 료 귀

료 등 료 에 사 립 도

도 다 다( lsquo rsquo )

3 ( )

도 다 각 같다①

공 공 다만 연1

연 간 다

년 학 간고사 대비2013 2 현대고 대비

ECN-0102-2013-001-000076193

매월 째 째 월2

도 도 리 그 사3

가 다고 는

도 에 미리 게1 3②

시 여 다

시간4 ( )

도 시간 도 여 게시 다

등 등5 ( )

도 료 시 는 는 도①

지에 등 후

등 에 사 도②

사 료6 ( )

도 료 시 에 사 료는 도

7 ( )

는 다 각 여 는 니 다

도 료 시 상 리1 lsquo rsquo

도 료 시 훼 는2 middot

지 가 닌 곳에 식 거 담3

우는

도 보 등 보 검색열4 middot

그 에 도 질 지 여 도5

여 게시 사 는

질 지8 ( )

도 다 거 도①

질 게 우 가 는 에 여는 도

도 가 각 어느7②

에는 지 게 거 도

료9 ( )

도 료는 다 각 경우 다①

상 도 간에 료 는 것 말1 (

다 등 다 도 과 여 경우)

공 원 공 상 는 경우2

그 에 도 다고 는 경우3

가능 도 료 는 도②

는 에 다

변상10 ( )

가 도 료 시 럽 거 거①

못 쓰게 거 어 린 경우에는 변상 여

도 에 변상 여 게시1②

여 다

등 규 에 것 에 도11 ( )

료 시 등에 사

도 다

립 도- (httpwwwnlgokr)

zb48) 다음 정 리 의 의 으 볼 때 가장

이 적인 것은

도 시간 도 여 게시 다①

등 에 사 도②

가능 도 료 는 도 는③

에 다

④ 도 에 변상 여 게10 1

시 여 다

⑤ 도 가 각 어느7

에는 지 거 도

zb49) 를 참고하여 이 어의 성격을 설 한lt gt

것으 적절하 않은 것은

① 보 에 는 어 시 상 고 어 시lt gt lsquo rsquo

에 보여주고 다

② 진 어 어원에 견 고 다

에는 타 어 들어가는 것 다 lsquo rsquo

③ 에 들어갈 말 각각 고 어 어 신 어~

들 언어는 질 격 강 통 없었다

④ 시 우리 에 가 었지만 지 계

과 달리 들 통 사 달 어 웠

년 학 간고사 대비2013 2 현대고 대비

ECN-0102-2013-001-000076193

⑤ 크 몽골 만주 공통어가 우리 어 같

계열에 다는 에 사 특 짐

가( )

善化公主主隱 공주님

他密只嫁良置古 몰 결 고

薯童房乙 맛

夜矣卯乙抱遣去如 에 몰 고 가다

( )

始汝 會隱日恚見隱扐 만 에 본

恥隱汝衣淸隱笑 맑 웃

고 시 여 공 크다 만 다[ ] ( ) ( ) ( ) ( )始 汝 會扐

내다 에 보다 견( ) ( )恚 見 다( )隱

럽다 맑다 청 웃( ) ( ) ( ) ( )恥 衣 淸 笑

zb50) 위의 나 를 함 고 음에 답하( ) lt gt

보lt gt

( )素那或云金川 白城郡蛇山人也

운 사산

는 고 다 는( )[ ( ) ] (素那 金川 白城

사산 사 다) ( ) 郡 蛇山

삼 사- lsquo rsquo 47

에 제 된 단어 의 표 리를 조건(1) lt gt ( ) lt gt

에 맞게 서 하

건lt gt

lsquo 었고 었다 태rsquo

에 제 된 단어 동일한 표 리에(2) lt gt ( )

의해 적은 것을 나 에서 찾아 조건 에 맞게 서 하( ) lt gt

건lt gt

에 당 는 각각( ) 개 쓸 것2 단

당 는 가 여러 개 어도 개만 쓸 것 각2

개 과 도 쪽에 개만2 2

드시 지 것( )

과 동 원리 것lsquo 고

과 동 원리 것 다rsquo

태 것

가( )

素那(或云金川) 白城郡蛇山人也

소나 또는 천 이라 한다 는 성 사( ) ( ) ( )素那 金川 白城郡〔 〕

산 사람이다 현대어 풀이( ) ( )蛇山

나( )

紫布岩乎希 회

執音乎手母牛放敎遣 자 손 암쇼 노히 고

吾 不喩慙 伊賜等肹 肹 나 안디 리샤

花 折叱肹 可獻乎理音如 고 것거 도림다

다 향찰은 리말을 리 으 적은 표 이었 만 생( )

은 고 대를 넘 하고 끊어 고 말았다 랜 세

동안 갈고 닦아 체계적이었던 향찰 표 이 사라졌

을 인은 크게 두 가 나누어 생각해 볼 있다

하나는 족 사회의 한 선호도에서 찾을 있다 라 때

향찰은 주 족 계 에서 사 했을 것으 인다 한 을

알 하고서는 한자를 활 하여 리말을 리 으 표

하 란 가능하 때 이다 런데 족들은 간이 흐

를 향찰과 같은 리 표 을 익혀 사 하 다는

아 한 을 대 사 하는 쪽을 선호하게 되었다 더 이

고 초에 인재 등 을 위해 과거제도가 행되 서 한 선

호도가 더 높아졌고 결 향찰은 소 되고 말았다

또 다른 가능성은 한 어의 특성에서 찾을 있다

터 한 과 일 세 나라는 한자 화 에 속해 다

당연한 이야 겠 만 표의 자인 한자는 어를 표 하

에 매 적절하다 어의 음절은 성 ( ) ( )聲母 韻母

이 어 고 여 에 성조가 추가되어 최종 소리가 결정된

다 래서 어는 단음절을 하나의 한자 표 하 된

다 에 초성 성 종성의 세 가 소가 하나의 음절

년 학 간고사 대비2013 2 현대고 대비

ECN-0102-2013-001-000076193

을 이 는 한 어는 음절 조가 잡하고 음절의 가 많아

서 한자 차 만으 한 어의 소리를 만족 럽게 표 할

없었다 를 들어 한 어에서는 어 니 같이 음절 lsquo rsquo

이 어 단어가 얼마든 있으나 어는( ) 複數音節

자 하나 나타내 만이다lsquo [m ]rsquo 母 ǔ

한편 일 어의 표 은 핵 적 단어는 한자 적고 토는

가나라는 일 의 자 적는 이다 적인 의 를 나

타내는 은 표의 자인 한자 적고 적 계를 나

타내는 토는 표음 자 적는 셈이니 자세히 살펴

리의 향찰 표 을 쏙 빼닮았음을 알 있다 한 어 같

은 착어이 서도 일 어에만 향찰과 유사한 표 이 살아

남은 것은 일 어의 특 때 이다 일 어는 하나의 자음과

음의 결합으 음절을 이 고 침이 거의 없는 음절 언어

이다 이러한 음절의 특색에다가 토가 달한 착어라는 점

이 향찰과 유사한 표 이 살아남을 있는 비결이었다

하 만 같은 착어라도 다양한 음소 침이 달한 한

어는 향찰 표 하는 데 근 적으 한계가 있었다

zb51) 다 하여 의 행에 대한 탐 한 결과( ) lt gt 2

않은 것은

보lt gt

善花公主主隱 공주니믄 공주님( )

----------------------------------------

-

他密只嫁良置古 그 지 얼어 고 몰 결(

----------------------------------------

-

薯童房乙 맛 맛( )

夜矣卯乙抱遺去如 몰 고 가다 에 몰 고(

가다)

주동 역 동- (薯童謠『 』

에 2 ( )他密只嫁良置古

얼다 시집가다 결 다 말 lsquo rsquo

① 실질 미 지니고 므 타 타lsquo ( )rsquo lsquo [ ]

② 에 실질 미 타내고 지 는lsquo rsquo lsquo [ ]rsquo lsquo [ ]密只 密 只

계 타내는

③ 얼어는 실질 미 포 고 므 가lsquo rsquo lsquo [ ]rsquo嫁

것lsquo [ ]rsquo 良

④ 고 어간 는 실질 미 지니고 므lsquo rsquo lsquo -rsquo

것lsquo [ ]rsquo 置

⑤ 고 어미 고는 계 타내고 므lsquo rsquo lsquo- rsquo

고 것lsquo [ ]rsquo 古

가( )

엉 훈 민middot middot middot middot middot世 宗 御 製 訓 民 正 音

말 미 듕 귁에 달middot middot middot middot middot middot middot middot中 國 文 字

니 런middot middot middot middot middot middot 어린middot middot middot middot百 姓

니 고 도 내 들middot middot middot middot middot middot middot middot middot 시러middot

펴 몯middot 미middot middot 니 내middot middot middot middot middot middot middot middot 爲

어엿middot 겨 새middot middot middot 믈여듧middot middot middot middot字 니middot middot middot

사 마다 니겨 킈 middot middot middot middot middot middot middot middot middot便 安

고 미니middot middot middot middot

본 는 상( ) (象

원리에 만들어진 본) ( )形 ㄱ ㄴ ㅁ ㅅ ㅇ

에 는 가 원리에( )加劃

그리고( )ㅋ ㄷ ㅌ ㅂ ㅍ ㅈ ㅊ ㆆ ㅎ

쓰는 병 원리에 만들어진( )竝書

마지막 체( ) ( )異體ㄲ ㄸ ㅃ ㅆ ㅉ ㆅ

ᅀ 다 상 원리에 ㅇ ㄹ

지 는 삼재 상 본 본( ) ( ) ( 天地人 三才

탕 므림과 림에 ) (初ㅡ ㅣ

재)( ) ( )( )出字 再出字ㅗ ㅏ ㅜ ㅓ ㅛ ㅑ ㅜ ㅕ

병 그리고 들 에 다시( )ㅘ ㅝ ㅣ

( )ㅣ ㅢ ㅚ ㅐ ㅟ ㅔ ㆉ ㅒ ㆌ ㅖ ㅙ ㅞ

zb52) 가 에 대한 설 으 르 않은 것을( ) 두 고르

① 어쓰 규 지키고 다

② 리 고 다

③ 말 미 미 등 어 사 다lsquo rsquo

④ 개 지 다

년 학 간고사 대비2013 2 현대고 대비

ECN-0102-2013-001-000076193

⑤ 어 원 에 가 도 고 다

엉 훈 민世 宗 御 製 訓 民 正 音

말 미 듕귁에 달 니

런 어린 니 고 도middot

내 들 시러 펴 몯 미 니middot

내 어엿 겨 새 믈여듧

사 마다 니겨middot 킈 고

미니

훈민 언 본- lsquo rsquo 5 (1459 )

zb53) 위의 에 대한 현대어 풀이가 르~ 않은 것

① 우리 말 과 달

② 어리 말 고 는 것 어도

③ 신 생각 마 껏 펼 는 사 많다

④ 게 생각 여

⑤ 사 마다 게

zb54) 훈민정음 언해 에는 한 을 창제한 동 가 드러나

있다 훈민정음 창제의 정 과 내 이 잘 연결된 것

① 주 신 말 미 듕귁에 달

② 민 신 내 어 겨

③ 신 뻔 킈 고 미니

④ 실 신 사 마다 니겨

⑤ 귀 신 계 주 는 훈민 신과 거리가

가 엉 훈 민( ) middot middot middot middot middot世 宗 御 製 訓 民 正 音 

말 미 귁에 中 國 달 文 字

니 런 어린 니 百 姓

고 도 내 들 시러 펴 몯

미 니 내 어엿 爲 겨 새

믈여듧 니 사 마다 니 字

겨 킈 고 미니 便 安

훈민 언 본- lsquo ( )rsquo ( ) 5 (1459 )訓民正音 世祖

( )

[ 1 ]

동 룡 샤 마다 복( ) ( ) ( )海東 六龍 天福

시니 고 동( ) ( )古聖 同符 시니

[ 2 ]

매 니 곶 여

미 므 니 그 내 러

가 니

[ 125 ]

우 미리( )千世 샨( )定 에( )漢水北 累仁

누 개 샤 복 업 시니( ) ( ) 開國 卜年

신( )聖神 니 샤도 경 근민 샤 욱( )敬天勤民

드시리 다

님 쇼 산 가( ) ( )洛水 山行

미드니 가

어 가- lsquo ( )rsquo 27龍飛御天歌

다 우리신 니쓰고 다만 만 쓰( )

거 샹 귀쳔 다보게 러 귀

여 쓴 도 신 보 가 고 신 에

말 어 보게 각 에 사 들

고 본 몬 능통 후에

죠 죠 니

드 도 만 공 에 사

드 미 죠 고 고 여 보 죠

보다 얼마가 거시 어신고 니 첫

가 죠 니 죠

민 들 어 신 샹

귀쳔 도보고 어보 가 만 늘

고 폐 에 만쓴 죠 민

도 러보지못 고 보니 그게 엇지

심 니 리 보 가 어 운건 다

니 쳣 말마 지 니 고 그

쓰 에 가 우 지 지

몰 거 본후에 가 어 지

고 그니 쓴편지 쟝 보

년 학 간고사 대비2013 2 현대고 대비

ECN-0102-2013-001-000076193

쓴것보다 듸 보고 그 마 니 쓴 고

어 못

그런고 에 리 과 가

만 쓴 못 민 말만 듯고

고 편 그 못 보니 그사 단

병신 못 다고 그사 식 사

니 만 고 다 과 그사

만 고 다 과 업 사 보다 식 고

죠 도 고 각 과

견 고 실 직 귀쳔 간에 그

고도 다 것 몰 귀죡 보다

사 우리 신 귀쳔 다 업

시 신 보고 과 지 게 랴

시니 샹 귀쳔 간에 우리 신 걸

간 보 새지각과 새 걸 미리

독립신- lsquo (1896)rsquo

zb55) 친 어 나의 제 장( ) 2 매 함축적

의 가 가장 유사한 것은

① 지 눈 내리고 매 득 니 내 여 가

사- lsquo rsquo

② 도 어 리듯 그 게 어 다

주 사- lsquo rsquo

③ 눈 살 다 죽 어 린 과 체 여

눈 새벽 지 도 살 다

눈- lsquo rsquo

④ 삶 근심과 고단 에 돌 거니는 여 거 는

여 리 내린 살가지 에 눈 리 눈 리

택 그 생 에- lsquo rsquo

⑤ 늘 러 고 러

청룡 룡 어 개 루 우

신경림 계- lsquo rsquo

zb56) 친 를 위 가 나 에 나타난A B ( ) ( )

세 어의 특 에 의거하여 세 어 표 하

그 산 고 공 도 맑지만

A

주변에 쓰 리는 어리 사 많다

B

건lt gt

식 가 에 타 어 특징에( ) ( )

거 과 어쓰 는 고 지 말 것

A

B

zb57) 가 의( ) 달 아ㆍ 다 의 ( ) 나셔에서 알 있는

세 어 개화 어의 특 을 비 하여 조건 에lt gt

맞게 서 하

건lt gt

어에 는lsquo 개

어에 는 다 태rsquo

zb58) 은 가 는 다 에 나 는 절lt 1gt ( ) lt 2gt ( )

일 를 췌한 것이다 의 의 가 lt 1gt (1)~(2)

유사한 말을 에서 찾아 쓰lt 2gt

보lt 1gt

런 (1) 어린 니 고百 姓

도 내 들 시러 펴 몯 미

사 마다 (2) 니겨 便 安

킈 고 미니

보lt 2gt

죠 고 고 여 보 죠

보다 얼마가 거시 어신고 니 첫 가

죠 니 죠 민

들 어 신 샹 귀쳔

도보고 어보 가 만 늘 고

폐 에 만쓴 죠 민 도

러보지못 고 보니 그게 엇지 심

니 리

년 학 간고사 대비2013 2 현대고 대비

ECN-0102-2013-001-000076193

lt 1 gt

동 룡 샤 마다 복 시( ) ( ) ( )海東 六龍 天福

고 동 시니( ) ( )古聖 同符

lt 2 gt

(A) 매 니 곶

여 니

미 므 니 그 내

러 가 니

lt125 gt

우 미리 샨 에( ) ( ) ( ) 千世 定 漢水北 累

누 개 샤 복 업 시 니( ) ( ) 仁開國 卜年 聖

신( ) 神 니 샤도 경 근민 샤( ) 敬天勤民

욱 드 시 리 다

님 쇼 산 가 ( ) ( )洛水 山行

미드니 가

- lt gt龍飛御天歌

zb59) 장과 내 상 유사한 성격의 조는125

① 뫼 고 고 고 고

어 그린 많고 많고 고 고

어 러 는 울고 울고 가느니

도 견- lt gt

② 강 에 드니 몸 다

그믈 고 가니

뒷 뫼 엄 언 니( )藥

-

③ 말 없는 청산 태 없는 다

값 없는 청 없는 월

에 병 없는 몸 별 없 늙 리

-

④ 가마귀 골에 가지 마

낸 가마귀 새

청강에 것 시 몸 러 가( ) 淸江

-

⑤ 진 골에( ) 白雪

가 매 는 어느 곳에 었는고

에 갈 곳 몰( ) 夕陽

색-

zb60) 위 에 나타난 세 어의 특 으 적절하 않은

것은

① 룡 어 주격 사에 당 는 가 사( ) lsquo rsquo六龍

고 다

② 샤 어에도 어 주체 쓰 다

는 것 다

③ 매 어 달리 사 택에 어

가 지 지지 고 다

④ 므 원 상 직 어 지 다

⑤ 드시리 다 주체 과 상 께 사

고 다

수고 하셨습니다hearts hearts

년 학 간고사 대비2013 2 현대고 대비

ECN-0102-2013-001-000076193

보닷컴에 공 는 별 보는 고등

들 여 주 는

들 습니다 슷 동 지

가 복 는 것 도가

니 복 여 습 시고 거 시

니다

정답 해설

1) 정답[ ] ④

해설 다른 것은 두 특정 업이나 단 내에서 사[ ]

하는 일종의 은어 사회 언에 해당한다 러나

는 언이 아니라 단과대학을 여서 단대 사lsquo rsquo lsquo rsquo lsquo④

대학을 여서 사대라고 한 말에 해당하 일rsquo lsquo rsquo

사회에서도 널리 쓰이 사회 언이라 할

없다

2) 정답[ ] ⑤

해설 사회 언은 같은 단 내에서 쓰이는 언어이[ ] lsquo rsquo

동일 단끼리는 단결 과 친 감을 형성하는

능을 하 리적 안감이 일어나 않는다

3) 정답[ ] ③

해설 사람이라는 차 적 표현에 대한 대안적 표현이[ ]lsquo rsquo

인 아내 처 등으 볼 있다lsquo rsquo

4) 정답[ ]⑤

해설 남성은 주 격 체를 사 한다[ ]

5) 정답[ ] ⑤

해설 흑인은 검다라는 뜻을 가 고 있을 뿐 인[ ]lsquo rsquo lsquo rsquo lsquo rsquo

다 열등한 뜻을 내포하 않는다

6) 정답 살 색 첫 작품[ ] - -

해설 살색 혹은 킨색은 한 인의 피 색을 뜻[ ] lsquo rsquo lsquo rsquo

하는 것으 인종 차 을 추 고 출 이주민

의 평등 을 침해할 있어 년 표 이2005

살 색으 이름을 꾸었다 처녀작은 처녀라lsquo rsquo lsquo rsquo lsquo rsquo

는 단어가 가 고 있는 곡된 성 인 을 한 것

으 첫 작품정도 꾸어 사 하는 것이 좋다lsquo rsquo

7) 정답[ ] ⑤

해설 호는 아들에게 해체를 사 하고 있다[ ] ① ②

장 을 성하는 청자는 자 의 아 느리 아lsquo

들 세 이다 호는 아 느리에게 해rsquo ③

체를 사 하고 있다 호가 느리 아 에게 ④

사 한 해 체 아들에게 사 한 해체는 두 비lsquo rsquo lsquo rsquo

격 체에 해당한다 호는 자 의 아랫사람인 ⑤

느리에게 아들과 마찬가 해체를 사 하는 것이

상 이 만 임 을 한 느리에게 고마 과 쁨

존 의 표 를 하 위해 자 의 아 에게 말하듯

해 체를 사 하고 있다

8) 정답[ ] ③

9) 정답[ ] ⑤

10) 정답[ ] ①

해설 청자 할아 가 장의 주체 아 다 높을[ ] ( ) ( )

경 에는 압존 에 의해 장의 주체를 높이 않는lsquo rsquo

다 러 아 서가 아닌 아 는으 계 lsquo rsquo lsquo rsquo lsquo

니다 가 아닌 있 니다 표현하는 것이 르rsquo lsquo rsquo

11) 정답 당이 당을 쫒았다 당이[ ]

당에 다

해설[ ]

12) 정답[ ] ⑤

해설 서 다른 높임표현을 통해 청자에 대해 리[ ] ⑤

적 거리감을 나타내는 인 은 이 아니라 현정이

다 가 에서 현정은 에게 해 체를 사 함으 써 ( )

친근감을 드러낸다 나 에서 연 을 게을리하는 역 ( )

도 들 때 에 화가 난 현정이 선생님에게 항의하

는 장 에서는 하 체를 사 하여 리적 거리lsquo rsquo

가 어졌음을 나타내고 있다

13) 정답[ ] ①

해설 는 는 얼 빛이 날과 어찌 다르 고[ ] lsquo rsquo

라는 뜻으 전과 달리 임이 화자를 않고

있음을 알 있다

14) 정답 달리 후 가 있다 이를 통해 경[ ] lt gt

쾌한 음악성을 형성하고 노 젓는 상황을 체적으

형상화하는 역할을 한다

15) 정답[ ] ①

16) 정답[ ] ⑤

해설 다 의 자연은 를 성찰하게 하는 대상[ ] ( )⑤

이자 정의 대상이다 의 자연은 자 의 상황과 ⑤

처 를 드러내는 경으 서의 역할을 하 이

이 없다

17) 정답[ ] ③

해설 는 빈천 을 해결하고자 했으나 강산[ ] lsquo ( )rsquo 貧賤③

과 풍 을 달라는 에 거절하 다고 함으 써 자

연에 대한 애정을 드러내고 있으 는 않는

임에 대한 망을 개에게 전가 켜서 임에 대한 리

을 드러내고 있다

18) 정답[ ] ③

년 학 간고사 대비2013 2 현대고 대비

ECN-0102-2013-001-000076193

19) 정답[ ] ⑤

해설 고상한 음악가의 이름을 리말 꽝 럽[ ]

게 꿈으 써 언어유희를 통해 음을 유 하고 있

다 이는 고상한 척하는 총 를 비꼼으 써 비판적

태도를 드러내는 것이 대상을 꽝 럽게 표현

하여 총 의 허 과 사치를 풍자하고 있다

20) 정답[ ] ⑤

해설 는 작품 속 경에 대한 설 이 드러나는 것이[ ]

서 자의 주 적인 견해가 접적으 드러나는 것이

아니다

21) 정답[ ] ⑤

22) 정답[ ] ②

23) 정답[ ] ④

24) 정답[ ] ①

해설 적강 티프는 주인공의 비 한 출생이나 능[ ] ①

과 이 있는 것으 조정의 능함을 풍자하는lsquo rsquo

것과는 거리가 다

25) 정답 픔 나[ ] ( )

해설 의 음악은 고통 는 사람들을 위 하고 아픔[ ] lsquo rsquo

을 치유해 주는 능을 한다고 할 있다 의 lt gt

픔 도 소 된 이 과 더 어 살아가는 따뜻한 마음lsquo rsquo

을 상 한다

26) 정답[ ] ⑤

해설 에게 선천적으 주어 각 장애라는 역경[ ]

은 의 이라는 가사 연 을 있다lsquo rsquo

27) 정답[ ] ④

해설 는 장 란 선 에게 은 개인적인 인상을[ ]

소녀 장정 등으 표현한 것이다lsquo rsquo

28) 정답[ ] ②

해설 담자가 피 담자의 언어적 표현이나 비언어[ ]②

적 표현 하 독자는 담의 위 나 피

담자의 감정 상태를 알 있다 이를 통해 독자는

담 상황을 더 생생하게 느낄 있고 피 담자

를 더 잘 이해할 있게 된다

29) 정답[ ]③

해설 일상생활과 역도 선 서의 성과에 된 것에서[ ]

역도를 하 서 겪는 어 과 내적 고민으 화제를

전화하 위한 것이다

30) 정답[ ] ①

해설 릿속에 새겨 넣듯 이 억되도 함 세상[ ] ② ③

살이가 힘들고 고생 러 속 하여 자유를 ④

가 없는 고통의 상태를 비유적으 이르는 말

적의 침입을 막 위해 쌓은 축 켜야 할⑤

대상을 비유적으 이르는 말이다

31) 정답[ ] ④

해설 이 의 종류는 전 으 인 사건 경[ ] lsquo

비평을 성 소 삼는다rsquo

32) 정답[ ] ④

해설 근은 삼대독자 태어났음을 에서 확인할[ ]

있다 형제들과의 담은 이뤄 가 없다

33) 정답[ ] ⑤

해설 근은 가난에도 하고 화가를 꿈꾸었다[ ] (3

단 또한 다른 화가 망생들은 정 육을)

위해 상 학 학 해 유학 에 랐 만

근은 다른 을 찾아야 했다 단 세에(5 ) 18

근은 조선 전람회에 입선하 다 단 의(6 )

만종은 인간과 자연이 엮어 가는 경건한 조화 을lsquo rsquo

나타낸다

34) 정답[ ] ①

해설 근이 속에서도 창작활동을 추 않고[ ]

하는 닭은 은 세상과 타협할 르는

근이 세상의 이해를 하 위한 가장 떳떳한 단

이 때 이다

35) 정답[ ] ⑤

해설 전 은 서 자의 주 적인 평이 리는 것이[ ]

만 위 제 은 인 이 살았던 대 사회적 경

을 통해 객 적인 인 의 을 제 하고 있다

36) 정답[ ] ⑤

해설 전 은 인 사건 경 비평이라는[ ] lsquo rsquo⑤

성 이 어져 있다

37) 정답[ ] ①

해설 이 은 동양인과 서양인의 사고 에 차이가[ ]

있다는 것을 대조를 통해 설 하고 있다 또 쓴이

의 제자가 축 경 를 러 가서 경험한 일화를

통해 동양인이 서양인에 비해 주 상황에 더 많은

주의를 인다는 주장을 뒷 침하고 있다

38) 정답[ ] ④

39) 정답[ ] ②

40) 정답[ ] ②

41) 정답[ ] ④

42) 정답[ ] ③

43) 정답[ ] ④

44) 정답 도서 의 휴 일 도서 의 이 간 도서의[ ]

해설 도서 장은 임의 정한 휴 일과 도서 이[ ]

간 도서의 상 등을 게 할 의 가 있다

년 학 간고사 대비2013 2 현대고 대비

ECN-0102-2013-001-000076193

45) 정답[ ] ①

해설 제 조의 정 휴 일 의 휴 일의 사전 게[ ] 3

는 도서 장의 의 조항에 속한다

46) 정답[ ] ①

해설 개인 정 호 의 를 제 하 했 만 항[ ]

나눠서 제 하 않고 대 나열하고 있다

47) 정답[ ] ②

해설 제 조의 내 을 회사는 다른 회사 협[ ] 7 lsquo

계약을 통해 서비 를 제공하는 경 회 의 아이디

등 개인 정 를 해당 회사에 전송할 있다는 내rsquo

이 있으 의 제점을 제 할 있다②

48) 정답[ ] ④

해설 는 도서 장의 의 에 해당하고 나 는 도[ ] ④

서 장의 리에 해당한다

49) 정답[ ] ③

50) 정답 은 음독으 적었고 은 훈독으 적었[ ] (1)

다 과 동일한 표 리 적은 것은 이고 (2) ce

과 동일한 표 리 적은 것은 이다ab

51) 정답[ ] ③

52) 정답[ ] ①②

53) 정답[ ] ③

54) 정답[ ] ③

55) 정답[ ] ①

56) 정답 른 죠코 어린 노 하니라[ ] A B

57) 정답 세 어에서는 활 형이 칙적으[ ] lsquo rsquoㄹㅇ

나타났 만 개화 어에서는 활 형이 쓰 다 lsquo rsquo ㄹㄴ

58) 정답 호 가 흔[ ] (1) (2)

59) 정답[ ] ④

60) 정답[ ] ③

Page 31: 현대고대비 국어 - chamsoriedu.com 「콘텐츠산업진흥 법」외 에도 저작권 의하여 ... 다른주체에게어떤동작을하도록만드는것을나타내는

년 학 간고사 대비2013 2 현대고 대비

ECN-0102-2013-001-000076193

1 ( )

사가 공 는lsquo rsquo

과 여 사 원과 리

사 타 사 규

니다

개 보 보7 ( )

사는 보통신망 등 계 는 에lsquo rsquo lsquo rsquo

원 개 보 보 니다 개lsquo rsquo

보 보 사 에 는 사 개lsquo rsquo

보 취 니다 다만 사는 다 lsquo rsquo

사 계 통 공 는 경우 원 lsquo rsquo

등 개 보 당 사에 습니lsquo rsquo

원 리에8 (lsquo rsquo lsquo rsquo lsquo rsquo

)

원 에 리lsquo rsquo lsquo rsquo lsquo rsquo①

원에게 가 도 여 는lsquo rsquo 3

니다

사는 원 가 개 보 우 가lsquo rsquo lsquo rsquo lsquo rsquo②

거 사 경우 는 미 에 어 거 lsquo

사 사 운 우 가 는 경우 당rsquo lsquo rsquo

습니다lsquo rsquo

원 가 도 거lsquo rsquo lsquo rsquo lsquo rsquo 3③

가 사 고 지 경우에는 시 사에lsquo rsquo

통지 고 사 내에 니다lsquo rsquo

경우에 당 원 사에 그 사실3 lsquo rsquo lsquo rsquo④

통지 지 거 통지 도 사 내에 지 lsquo rsquo

생 경우 사는 지지 습니다lsquo rsquo

사10 (lsquo rsquo )

사는 과 지 미lsquo rsquo①

에 는 지 계 고

공 여 다 여 니다lsquo rsquo

사는 원 게lsquo rsquo lsquo rsquo lsquo rsquo②

도 개 보 신 보 포 보 보 시( )

갖 어 개 보 취 공시 고

니다

사는 과 여 원lsquo rsquo lsquo rsquo③

견 만 당 다고 경우에는

리 여 니다 원 견 만 사 lsquo rsquo

에 는 게시 거 우편 등 통 여

원에게 리 과 결과 달 니다lsquo rsquo

원11 (lsquo rsquo )

원 다 여 는 니다lsquo rsquo ①

신청 는 변경 시 허 내 등1

타 보 도2

사가 게시 보 변경3 lsquo rsquo

사가 보 보 컴퓨 그4 lsquo rsquo (

등 등 신 는 게시)

사 타 등 지 재산 에5 lsquo rsquo 3

사 타 상 거 업6 lsquo rsquo 3

는 폭 시지 상 타 공7 middot middot

에 는 보 에 공개 는 게시 는lsquo rsquo

사 동 없 리 사8 lsquo rsquo

타 거 당9

게시15 (lsquo rsquo )

원 내에 게시 는 게시 게재 는lsquo rsquo lsquo rsquo lsquo rsquo

경우 원 사가 게시 복 lsquo rsquo lsquo rsquo lsquo rsquo middot middot

등 태 언 등에 공 는

것 내에 다 원 본 게시 등 lsquo rsquo lsquo rsquo

크 능 등 여 복 는 등 태

는 것 동 것 니다

- (wwwnavercom)

zb46) 위 은 인터넷 포털사이트의 회 가입을 위한 이

약 의 일 이다 이 약 을 만드는 과정에서 생각한

내 으 적절하 않은 것은

개 보 보 가 지에 별 눠①

겠어

원 가 만들게 에②

시 주어 겠어

원들 게재 게시 다 원 크 다③

는 것 지

④ 원 지 는 뿐만 니 사가 지 는

도 께 달 지

리에 가 생 경우 사가⑤

에 다는 도 듯

1 ( )

사가 공 는lsquo rsquo

과 여 사 원과 리

사 타 사 규

년 학 간고사 대비2013 2 현대고 대비

ECN-0102-2013-001-000076193

니다

개 보 보7 ( )

사는 보통신망 등 계 는 에lsquo rsquo lsquo rsquo

원 개 보 보 니다 개lsquo rsquo

보 보 사 에 는 사 개lsquo rsquo

보 취 니다 다만 사는 다 lsquo rsquo

사 계 통 공 는 경우 원 lsquo rsquo

등 개 보 당 사에 습니lsquo rsquo

원 리에8 (lsquo rsquo lsquo rsquo lsquo rsquo

)

원 에 리lsquo rsquo lsquo rsquo lsquo rsquo①

원에게 가 도 여 는lsquo rsquo 3

니다

사는 원 가 개 보 우 가lsquo rsquo lsquo rsquo lsquo rsquo②

거 사 경우 는 미 에 어 거 lsquo

사 사 운 우 가 는 경우 당rsquo lsquo rsquo

습니다lsquo rsquo

원 가 도 거lsquo rsquo lsquo rsquo lsquo rsquo 3③

가 사 고 지 경우에는 시 사에lsquo rsquo

통지 고 사 내에 니다lsquo rsquo

경우에 당 원 사에 그 사실3 lsquo rsquo lsquo rsquo④

통지 지 거 통지 도 사 내에 지 lsquo rsquo

생 경우 사는 지지 습니다lsquo rsquo

원에 통지9 (lsquo rsquo )

사는 특 다 원에게 통지 경우lsquo rsquo lsquo rsquo

공지 게시 통 상 게시 개별 통지에7

갈 습니다

사10 (lsquo rsquo )

사는 과 지 미lsquo rsquo①

에 는 지 계 고

공 여 다 여 니다lsquo rsquo

사는 원 게lsquo rsquo lsquo rsquo lsquo rsquo②

도 개 보 신 보 포 보 보 시( )

갖 어 개 보 취 공시 고

니다

사는 과 여 원lsquo rsquo lsquo rsquo③

견 만 당 다고 경우에는

리 여 니다 원 견 만 사 lsquo rsquo

에 는 게시 거 우편 등 통 여

원에게 리 과 결과 달 니다lsquo rsquo

원11 (lsquo rsquo )

원 다 여 는 니다lsquo rsquo ①

신청 는 변경 시 허 내 등1

타 보 도2

사가 게시 보 변경3 lsquo rsquo

사가 보 보 컴퓨 그4 lsquo rsquo (

등 등 신 는 게시)

사 타 등 지 재산 에5 lsquo rsquo 3

사 타 상 거 업6 lsquo rsquo 3

는 폭 시지 상 타 공7 middot middot

에 는 보 에 공개 는 게시 는lsquo rsquo

사 동 없 리 사8 lsquo rsquo

타 거 당9

원 계 규 내lsquo rsquo lsquo②

여 공지 주 사 사가 통지 는rsquo lsquo rsquo

사 등 여 타 사 업 에 lsquo rsquo

는 여 는 니다

- (wwwnavercom)

zb47) 위 약 의 조항에서 같은 제점을 하lt gt

고 있는 조항은

lt gt

제휴 회사에 회 의 아이디 개인 정 를 전송할 있도

한 조항은 고객에게 당한 조항이다

1 7 8① ② ③

④ 9 ⑤ 10

립 도 규

1 ( )

규 립 도 립 어린 청 도(

포 다 료 시 열 시 말) (

다 에 사 규 립 도)

편 진 다

2 ( )

규 립 도 도 다 에( lsquo rsquo )

고 는 도 에 도lsquo rsquo 2 2

료 에 여 다 다만 특 료 귀

료 등 료 에 사 립 도

도 다 다( lsquo rsquo )

3 ( )

도 다 각 같다①

공 공 다만 연1

연 간 다

년 학 간고사 대비2013 2 현대고 대비

ECN-0102-2013-001-000076193

매월 째 째 월2

도 도 리 그 사3

가 다고 는

도 에 미리 게1 3②

시 여 다

시간4 ( )

도 시간 도 여 게시 다

등 등5 ( )

도 료 시 는 는 도①

지에 등 후

등 에 사 도②

사 료6 ( )

도 료 시 에 사 료는 도

7 ( )

는 다 각 여 는 니 다

도 료 시 상 리1 lsquo rsquo

도 료 시 훼 는2 middot

지 가 닌 곳에 식 거 담3

우는

도 보 등 보 검색열4 middot

그 에 도 질 지 여 도5

여 게시 사 는

질 지8 ( )

도 다 거 도①

질 게 우 가 는 에 여는 도

도 가 각 어느7②

에는 지 게 거 도

료9 ( )

도 료는 다 각 경우 다①

상 도 간에 료 는 것 말1 (

다 등 다 도 과 여 경우)

공 원 공 상 는 경우2

그 에 도 다고 는 경우3

가능 도 료 는 도②

는 에 다

변상10 ( )

가 도 료 시 럽 거 거①

못 쓰게 거 어 린 경우에는 변상 여

도 에 변상 여 게시1②

여 다

등 규 에 것 에 도11 ( )

료 시 등에 사

도 다

립 도- (httpwwwnlgokr)

zb48) 다음 정 리 의 의 으 볼 때 가장

이 적인 것은

도 시간 도 여 게시 다①

등 에 사 도②

가능 도 료 는 도 는③

에 다

④ 도 에 변상 여 게10 1

시 여 다

⑤ 도 가 각 어느7

에는 지 거 도

zb49) 를 참고하여 이 어의 성격을 설 한lt gt

것으 적절하 않은 것은

① 보 에 는 어 시 상 고 어 시lt gt lsquo rsquo

에 보여주고 다

② 진 어 어원에 견 고 다

에는 타 어 들어가는 것 다 lsquo rsquo

③ 에 들어갈 말 각각 고 어 어 신 어~

들 언어는 질 격 강 통 없었다

④ 시 우리 에 가 었지만 지 계

과 달리 들 통 사 달 어 웠

년 학 간고사 대비2013 2 현대고 대비

ECN-0102-2013-001-000076193

⑤ 크 몽골 만주 공통어가 우리 어 같

계열에 다는 에 사 특 짐

가( )

善化公主主隱 공주님

他密只嫁良置古 몰 결 고

薯童房乙 맛

夜矣卯乙抱遣去如 에 몰 고 가다

( )

始汝 會隱日恚見隱扐 만 에 본

恥隱汝衣淸隱笑 맑 웃

고 시 여 공 크다 만 다[ ] ( ) ( ) ( ) ( )始 汝 會扐

내다 에 보다 견( ) ( )恚 見 다( )隱

럽다 맑다 청 웃( ) ( ) ( ) ( )恥 衣 淸 笑

zb50) 위의 나 를 함 고 음에 답하( ) lt gt

보lt gt

( )素那或云金川 白城郡蛇山人也

운 사산

는 고 다 는( )[ ( ) ] (素那 金川 白城

사산 사 다) ( ) 郡 蛇山

삼 사- lsquo rsquo 47

에 제 된 단어 의 표 리를 조건(1) lt gt ( ) lt gt

에 맞게 서 하

건lt gt

lsquo 었고 었다 태rsquo

에 제 된 단어 동일한 표 리에(2) lt gt ( )

의해 적은 것을 나 에서 찾아 조건 에 맞게 서 하( ) lt gt

건lt gt

에 당 는 각각( ) 개 쓸 것2 단

당 는 가 여러 개 어도 개만 쓸 것 각2

개 과 도 쪽에 개만2 2

드시 지 것( )

과 동 원리 것lsquo 고

과 동 원리 것 다rsquo

태 것

가( )

素那(或云金川) 白城郡蛇山人也

소나 또는 천 이라 한다 는 성 사( ) ( ) ( )素那 金川 白城郡〔 〕

산 사람이다 현대어 풀이( ) ( )蛇山

나( )

紫布岩乎希 회

執音乎手母牛放敎遣 자 손 암쇼 노히 고

吾 不喩慙 伊賜等肹 肹 나 안디 리샤

花 折叱肹 可獻乎理音如 고 것거 도림다

다 향찰은 리말을 리 으 적은 표 이었 만 생( )

은 고 대를 넘 하고 끊어 고 말았다 랜 세

동안 갈고 닦아 체계적이었던 향찰 표 이 사라졌

을 인은 크게 두 가 나누어 생각해 볼 있다

하나는 족 사회의 한 선호도에서 찾을 있다 라 때

향찰은 주 족 계 에서 사 했을 것으 인다 한 을

알 하고서는 한자를 활 하여 리말을 리 으 표

하 란 가능하 때 이다 런데 족들은 간이 흐

를 향찰과 같은 리 표 을 익혀 사 하 다는

아 한 을 대 사 하는 쪽을 선호하게 되었다 더 이

고 초에 인재 등 을 위해 과거제도가 행되 서 한 선

호도가 더 높아졌고 결 향찰은 소 되고 말았다

또 다른 가능성은 한 어의 특성에서 찾을 있다

터 한 과 일 세 나라는 한자 화 에 속해 다

당연한 이야 겠 만 표의 자인 한자는 어를 표 하

에 매 적절하다 어의 음절은 성 ( ) ( )聲母 韻母

이 어 고 여 에 성조가 추가되어 최종 소리가 결정된

다 래서 어는 단음절을 하나의 한자 표 하 된

다 에 초성 성 종성의 세 가 소가 하나의 음절

년 학 간고사 대비2013 2 현대고 대비

ECN-0102-2013-001-000076193

을 이 는 한 어는 음절 조가 잡하고 음절의 가 많아

서 한자 차 만으 한 어의 소리를 만족 럽게 표 할

없었다 를 들어 한 어에서는 어 니 같이 음절 lsquo rsquo

이 어 단어가 얼마든 있으나 어는( ) 複數音節

자 하나 나타내 만이다lsquo [m ]rsquo 母 ǔ

한편 일 어의 표 은 핵 적 단어는 한자 적고 토는

가나라는 일 의 자 적는 이다 적인 의 를 나

타내는 은 표의 자인 한자 적고 적 계를 나

타내는 토는 표음 자 적는 셈이니 자세히 살펴

리의 향찰 표 을 쏙 빼닮았음을 알 있다 한 어 같

은 착어이 서도 일 어에만 향찰과 유사한 표 이 살아

남은 것은 일 어의 특 때 이다 일 어는 하나의 자음과

음의 결합으 음절을 이 고 침이 거의 없는 음절 언어

이다 이러한 음절의 특색에다가 토가 달한 착어라는 점

이 향찰과 유사한 표 이 살아남을 있는 비결이었다

하 만 같은 착어라도 다양한 음소 침이 달한 한

어는 향찰 표 하는 데 근 적으 한계가 있었다

zb51) 다 하여 의 행에 대한 탐 한 결과( ) lt gt 2

않은 것은

보lt gt

善花公主主隱 공주니믄 공주님( )

----------------------------------------

-

他密只嫁良置古 그 지 얼어 고 몰 결(

----------------------------------------

-

薯童房乙 맛 맛( )

夜矣卯乙抱遺去如 몰 고 가다 에 몰 고(

가다)

주동 역 동- (薯童謠『 』

에 2 ( )他密只嫁良置古

얼다 시집가다 결 다 말 lsquo rsquo

① 실질 미 지니고 므 타 타lsquo ( )rsquo lsquo [ ]

② 에 실질 미 타내고 지 는lsquo rsquo lsquo [ ]rsquo lsquo [ ]密只 密 只

계 타내는

③ 얼어는 실질 미 포 고 므 가lsquo rsquo lsquo [ ]rsquo嫁

것lsquo [ ]rsquo 良

④ 고 어간 는 실질 미 지니고 므lsquo rsquo lsquo -rsquo

것lsquo [ ]rsquo 置

⑤ 고 어미 고는 계 타내고 므lsquo rsquo lsquo- rsquo

고 것lsquo [ ]rsquo 古

가( )

엉 훈 민middot middot middot middot middot世 宗 御 製 訓 民 正 音

말 미 듕 귁에 달middot middot middot middot middot middot middot middot中 國 文 字

니 런middot middot middot middot middot middot 어린middot middot middot middot百 姓

니 고 도 내 들middot middot middot middot middot middot middot middot middot 시러middot

펴 몯middot 미middot middot 니 내middot middot middot middot middot middot middot middot 爲

어엿middot 겨 새middot middot middot 믈여듧middot middot middot middot字 니middot middot middot

사 마다 니겨 킈 middot middot middot middot middot middot middot middot middot便 安

고 미니middot middot middot middot

본 는 상( ) (象

원리에 만들어진 본) ( )形 ㄱ ㄴ ㅁ ㅅ ㅇ

에 는 가 원리에( )加劃

그리고( )ㅋ ㄷ ㅌ ㅂ ㅍ ㅈ ㅊ ㆆ ㅎ

쓰는 병 원리에 만들어진( )竝書

마지막 체( ) ( )異體ㄲ ㄸ ㅃ ㅆ ㅉ ㆅ

ᅀ 다 상 원리에 ㅇ ㄹ

지 는 삼재 상 본 본( ) ( ) ( 天地人 三才

탕 므림과 림에 ) (初ㅡ ㅣ

재)( ) ( )( )出字 再出字ㅗ ㅏ ㅜ ㅓ ㅛ ㅑ ㅜ ㅕ

병 그리고 들 에 다시( )ㅘ ㅝ ㅣ

( )ㅣ ㅢ ㅚ ㅐ ㅟ ㅔ ㆉ ㅒ ㆌ ㅖ ㅙ ㅞ

zb52) 가 에 대한 설 으 르 않은 것을( ) 두 고르

① 어쓰 규 지키고 다

② 리 고 다

③ 말 미 미 등 어 사 다lsquo rsquo

④ 개 지 다

년 학 간고사 대비2013 2 현대고 대비

ECN-0102-2013-001-000076193

⑤ 어 원 에 가 도 고 다

엉 훈 민世 宗 御 製 訓 民 正 音

말 미 듕귁에 달 니

런 어린 니 고 도middot

내 들 시러 펴 몯 미 니middot

내 어엿 겨 새 믈여듧

사 마다 니겨middot 킈 고

미니

훈민 언 본- lsquo rsquo 5 (1459 )

zb53) 위의 에 대한 현대어 풀이가 르~ 않은 것

① 우리 말 과 달

② 어리 말 고 는 것 어도

③ 신 생각 마 껏 펼 는 사 많다

④ 게 생각 여

⑤ 사 마다 게

zb54) 훈민정음 언해 에는 한 을 창제한 동 가 드러나

있다 훈민정음 창제의 정 과 내 이 잘 연결된 것

① 주 신 말 미 듕귁에 달

② 민 신 내 어 겨

③ 신 뻔 킈 고 미니

④ 실 신 사 마다 니겨

⑤ 귀 신 계 주 는 훈민 신과 거리가

가 엉 훈 민( ) middot middot middot middot middot世 宗 御 製 訓 民 正 音 

말 미 귁에 中 國 달 文 字

니 런 어린 니 百 姓

고 도 내 들 시러 펴 몯

미 니 내 어엿 爲 겨 새

믈여듧 니 사 마다 니 字

겨 킈 고 미니 便 安

훈민 언 본- lsquo ( )rsquo ( ) 5 (1459 )訓民正音 世祖

( )

[ 1 ]

동 룡 샤 마다 복( ) ( ) ( )海東 六龍 天福

시니 고 동( ) ( )古聖 同符 시니

[ 2 ]

매 니 곶 여

미 므 니 그 내 러

가 니

[ 125 ]

우 미리( )千世 샨( )定 에( )漢水北 累仁

누 개 샤 복 업 시니( ) ( ) 開國 卜年

신( )聖神 니 샤도 경 근민 샤 욱( )敬天勤民

드시리 다

님 쇼 산 가( ) ( )洛水 山行

미드니 가

어 가- lsquo ( )rsquo 27龍飛御天歌

다 우리신 니쓰고 다만 만 쓰( )

거 샹 귀쳔 다보게 러 귀

여 쓴 도 신 보 가 고 신 에

말 어 보게 각 에 사 들

고 본 몬 능통 후에

죠 죠 니

드 도 만 공 에 사

드 미 죠 고 고 여 보 죠

보다 얼마가 거시 어신고 니 첫

가 죠 니 죠

민 들 어 신 샹

귀쳔 도보고 어보 가 만 늘

고 폐 에 만쓴 죠 민

도 러보지못 고 보니 그게 엇지

심 니 리 보 가 어 운건 다

니 쳣 말마 지 니 고 그

쓰 에 가 우 지 지

몰 거 본후에 가 어 지

고 그니 쓴편지 쟝 보

년 학 간고사 대비2013 2 현대고 대비

ECN-0102-2013-001-000076193

쓴것보다 듸 보고 그 마 니 쓴 고

어 못

그런고 에 리 과 가

만 쓴 못 민 말만 듯고

고 편 그 못 보니 그사 단

병신 못 다고 그사 식 사

니 만 고 다 과 그사

만 고 다 과 업 사 보다 식 고

죠 도 고 각 과

견 고 실 직 귀쳔 간에 그

고도 다 것 몰 귀죡 보다

사 우리 신 귀쳔 다 업

시 신 보고 과 지 게 랴

시니 샹 귀쳔 간에 우리 신 걸

간 보 새지각과 새 걸 미리

독립신- lsquo (1896)rsquo

zb55) 친 어 나의 제 장( ) 2 매 함축적

의 가 가장 유사한 것은

① 지 눈 내리고 매 득 니 내 여 가

사- lsquo rsquo

② 도 어 리듯 그 게 어 다

주 사- lsquo rsquo

③ 눈 살 다 죽 어 린 과 체 여

눈 새벽 지 도 살 다

눈- lsquo rsquo

④ 삶 근심과 고단 에 돌 거니는 여 거 는

여 리 내린 살가지 에 눈 리 눈 리

택 그 생 에- lsquo rsquo

⑤ 늘 러 고 러

청룡 룡 어 개 루 우

신경림 계- lsquo rsquo

zb56) 친 를 위 가 나 에 나타난A B ( ) ( )

세 어의 특 에 의거하여 세 어 표 하

그 산 고 공 도 맑지만

A

주변에 쓰 리는 어리 사 많다

B

건lt gt

식 가 에 타 어 특징에( ) ( )

거 과 어쓰 는 고 지 말 것

A

B

zb57) 가 의( ) 달 아ㆍ 다 의 ( ) 나셔에서 알 있는

세 어 개화 어의 특 을 비 하여 조건 에lt gt

맞게 서 하

건lt gt

어에 는lsquo 개

어에 는 다 태rsquo

zb58) 은 가 는 다 에 나 는 절lt 1gt ( ) lt 2gt ( )

일 를 췌한 것이다 의 의 가 lt 1gt (1)~(2)

유사한 말을 에서 찾아 쓰lt 2gt

보lt 1gt

런 (1) 어린 니 고百 姓

도 내 들 시러 펴 몯 미

사 마다 (2) 니겨 便 安

킈 고 미니

보lt 2gt

죠 고 고 여 보 죠

보다 얼마가 거시 어신고 니 첫 가

죠 니 죠 민

들 어 신 샹 귀쳔

도보고 어보 가 만 늘 고

폐 에 만쓴 죠 민 도

러보지못 고 보니 그게 엇지 심

니 리

년 학 간고사 대비2013 2 현대고 대비

ECN-0102-2013-001-000076193

lt 1 gt

동 룡 샤 마다 복 시( ) ( ) ( )海東 六龍 天福

고 동 시니( ) ( )古聖 同符

lt 2 gt

(A) 매 니 곶

여 니

미 므 니 그 내

러 가 니

lt125 gt

우 미리 샨 에( ) ( ) ( ) 千世 定 漢水北 累

누 개 샤 복 업 시 니( ) ( ) 仁開國 卜年 聖

신( ) 神 니 샤도 경 근민 샤( ) 敬天勤民

욱 드 시 리 다

님 쇼 산 가 ( ) ( )洛水 山行

미드니 가

- lt gt龍飛御天歌

zb59) 장과 내 상 유사한 성격의 조는125

① 뫼 고 고 고 고

어 그린 많고 많고 고 고

어 러 는 울고 울고 가느니

도 견- lt gt

② 강 에 드니 몸 다

그믈 고 가니

뒷 뫼 엄 언 니( )藥

-

③ 말 없는 청산 태 없는 다

값 없는 청 없는 월

에 병 없는 몸 별 없 늙 리

-

④ 가마귀 골에 가지 마

낸 가마귀 새

청강에 것 시 몸 러 가( ) 淸江

-

⑤ 진 골에( ) 白雪

가 매 는 어느 곳에 었는고

에 갈 곳 몰( ) 夕陽

색-

zb60) 위 에 나타난 세 어의 특 으 적절하 않은

것은

① 룡 어 주격 사에 당 는 가 사( ) lsquo rsquo六龍

고 다

② 샤 어에도 어 주체 쓰 다

는 것 다

③ 매 어 달리 사 택에 어

가 지 지지 고 다

④ 므 원 상 직 어 지 다

⑤ 드시리 다 주체 과 상 께 사

고 다

수고 하셨습니다hearts hearts

년 학 간고사 대비2013 2 현대고 대비

ECN-0102-2013-001-000076193

보닷컴에 공 는 별 보는 고등

들 여 주 는

들 습니다 슷 동 지

가 복 는 것 도가

니 복 여 습 시고 거 시

니다

정답 해설

1) 정답[ ] ④

해설 다른 것은 두 특정 업이나 단 내에서 사[ ]

하는 일종의 은어 사회 언에 해당한다 러나

는 언이 아니라 단과대학을 여서 단대 사lsquo rsquo lsquo rsquo lsquo④

대학을 여서 사대라고 한 말에 해당하 일rsquo lsquo rsquo

사회에서도 널리 쓰이 사회 언이라 할

없다

2) 정답[ ] ⑤

해설 사회 언은 같은 단 내에서 쓰이는 언어이[ ] lsquo rsquo

동일 단끼리는 단결 과 친 감을 형성하는

능을 하 리적 안감이 일어나 않는다

3) 정답[ ] ③

해설 사람이라는 차 적 표현에 대한 대안적 표현이[ ]lsquo rsquo

인 아내 처 등으 볼 있다lsquo rsquo

4) 정답[ ]⑤

해설 남성은 주 격 체를 사 한다[ ]

5) 정답[ ] ⑤

해설 흑인은 검다라는 뜻을 가 고 있을 뿐 인[ ]lsquo rsquo lsquo rsquo lsquo rsquo

다 열등한 뜻을 내포하 않는다

6) 정답 살 색 첫 작품[ ] - -

해설 살색 혹은 킨색은 한 인의 피 색을 뜻[ ] lsquo rsquo lsquo rsquo

하는 것으 인종 차 을 추 고 출 이주민

의 평등 을 침해할 있어 년 표 이2005

살 색으 이름을 꾸었다 처녀작은 처녀라lsquo rsquo lsquo rsquo lsquo rsquo

는 단어가 가 고 있는 곡된 성 인 을 한 것

으 첫 작품정도 꾸어 사 하는 것이 좋다lsquo rsquo

7) 정답[ ] ⑤

해설 호는 아들에게 해체를 사 하고 있다[ ] ① ②

장 을 성하는 청자는 자 의 아 느리 아lsquo

들 세 이다 호는 아 느리에게 해rsquo ③

체를 사 하고 있다 호가 느리 아 에게 ④

사 한 해 체 아들에게 사 한 해체는 두 비lsquo rsquo lsquo rsquo

격 체에 해당한다 호는 자 의 아랫사람인 ⑤

느리에게 아들과 마찬가 해체를 사 하는 것이

상 이 만 임 을 한 느리에게 고마 과 쁨

존 의 표 를 하 위해 자 의 아 에게 말하듯

해 체를 사 하고 있다

8) 정답[ ] ③

9) 정답[ ] ⑤

10) 정답[ ] ①

해설 청자 할아 가 장의 주체 아 다 높을[ ] ( ) ( )

경 에는 압존 에 의해 장의 주체를 높이 않는lsquo rsquo

다 러 아 서가 아닌 아 는으 계 lsquo rsquo lsquo rsquo lsquo

니다 가 아닌 있 니다 표현하는 것이 르rsquo lsquo rsquo

11) 정답 당이 당을 쫒았다 당이[ ]

당에 다

해설[ ]

12) 정답[ ] ⑤

해설 서 다른 높임표현을 통해 청자에 대해 리[ ] ⑤

적 거리감을 나타내는 인 은 이 아니라 현정이

다 가 에서 현정은 에게 해 체를 사 함으 써 ( )

친근감을 드러낸다 나 에서 연 을 게을리하는 역 ( )

도 들 때 에 화가 난 현정이 선생님에게 항의하

는 장 에서는 하 체를 사 하여 리적 거리lsquo rsquo

가 어졌음을 나타내고 있다

13) 정답[ ] ①

해설 는 는 얼 빛이 날과 어찌 다르 고[ ] lsquo rsquo

라는 뜻으 전과 달리 임이 화자를 않고

있음을 알 있다

14) 정답 달리 후 가 있다 이를 통해 경[ ] lt gt

쾌한 음악성을 형성하고 노 젓는 상황을 체적으

형상화하는 역할을 한다

15) 정답[ ] ①

16) 정답[ ] ⑤

해설 다 의 자연은 를 성찰하게 하는 대상[ ] ( )⑤

이자 정의 대상이다 의 자연은 자 의 상황과 ⑤

처 를 드러내는 경으 서의 역할을 하 이

이 없다

17) 정답[ ] ③

해설 는 빈천 을 해결하고자 했으나 강산[ ] lsquo ( )rsquo 貧賤③

과 풍 을 달라는 에 거절하 다고 함으 써 자

연에 대한 애정을 드러내고 있으 는 않는

임에 대한 망을 개에게 전가 켜서 임에 대한 리

을 드러내고 있다

18) 정답[ ] ③

년 학 간고사 대비2013 2 현대고 대비

ECN-0102-2013-001-000076193

19) 정답[ ] ⑤

해설 고상한 음악가의 이름을 리말 꽝 럽[ ]

게 꿈으 써 언어유희를 통해 음을 유 하고 있

다 이는 고상한 척하는 총 를 비꼼으 써 비판적

태도를 드러내는 것이 대상을 꽝 럽게 표현

하여 총 의 허 과 사치를 풍자하고 있다

20) 정답[ ] ⑤

해설 는 작품 속 경에 대한 설 이 드러나는 것이[ ]

서 자의 주 적인 견해가 접적으 드러나는 것이

아니다

21) 정답[ ] ⑤

22) 정답[ ] ②

23) 정답[ ] ④

24) 정답[ ] ①

해설 적강 티프는 주인공의 비 한 출생이나 능[ ] ①

과 이 있는 것으 조정의 능함을 풍자하는lsquo rsquo

것과는 거리가 다

25) 정답 픔 나[ ] ( )

해설 의 음악은 고통 는 사람들을 위 하고 아픔[ ] lsquo rsquo

을 치유해 주는 능을 한다고 할 있다 의 lt gt

픔 도 소 된 이 과 더 어 살아가는 따뜻한 마음lsquo rsquo

을 상 한다

26) 정답[ ] ⑤

해설 에게 선천적으 주어 각 장애라는 역경[ ]

은 의 이라는 가사 연 을 있다lsquo rsquo

27) 정답[ ] ④

해설 는 장 란 선 에게 은 개인적인 인상을[ ]

소녀 장정 등으 표현한 것이다lsquo rsquo

28) 정답[ ] ②

해설 담자가 피 담자의 언어적 표현이나 비언어[ ]②

적 표현 하 독자는 담의 위 나 피

담자의 감정 상태를 알 있다 이를 통해 독자는

담 상황을 더 생생하게 느낄 있고 피 담자

를 더 잘 이해할 있게 된다

29) 정답[ ]③

해설 일상생활과 역도 선 서의 성과에 된 것에서[ ]

역도를 하 서 겪는 어 과 내적 고민으 화제를

전화하 위한 것이다

30) 정답[ ] ①

해설 릿속에 새겨 넣듯 이 억되도 함 세상[ ] ② ③

살이가 힘들고 고생 러 속 하여 자유를 ④

가 없는 고통의 상태를 비유적으 이르는 말

적의 침입을 막 위해 쌓은 축 켜야 할⑤

대상을 비유적으 이르는 말이다

31) 정답[ ] ④

해설 이 의 종류는 전 으 인 사건 경[ ] lsquo

비평을 성 소 삼는다rsquo

32) 정답[ ] ④

해설 근은 삼대독자 태어났음을 에서 확인할[ ]

있다 형제들과의 담은 이뤄 가 없다

33) 정답[ ] ⑤

해설 근은 가난에도 하고 화가를 꿈꾸었다[ ] (3

단 또한 다른 화가 망생들은 정 육을)

위해 상 학 학 해 유학 에 랐 만

근은 다른 을 찾아야 했다 단 세에(5 ) 18

근은 조선 전람회에 입선하 다 단 의(6 )

만종은 인간과 자연이 엮어 가는 경건한 조화 을lsquo rsquo

나타낸다

34) 정답[ ] ①

해설 근이 속에서도 창작활동을 추 않고[ ]

하는 닭은 은 세상과 타협할 르는

근이 세상의 이해를 하 위한 가장 떳떳한 단

이 때 이다

35) 정답[ ] ⑤

해설 전 은 서 자의 주 적인 평이 리는 것이[ ]

만 위 제 은 인 이 살았던 대 사회적 경

을 통해 객 적인 인 의 을 제 하고 있다

36) 정답[ ] ⑤

해설 전 은 인 사건 경 비평이라는[ ] lsquo rsquo⑤

성 이 어져 있다

37) 정답[ ] ①

해설 이 은 동양인과 서양인의 사고 에 차이가[ ]

있다는 것을 대조를 통해 설 하고 있다 또 쓴이

의 제자가 축 경 를 러 가서 경험한 일화를

통해 동양인이 서양인에 비해 주 상황에 더 많은

주의를 인다는 주장을 뒷 침하고 있다

38) 정답[ ] ④

39) 정답[ ] ②

40) 정답[ ] ②

41) 정답[ ] ④

42) 정답[ ] ③

43) 정답[ ] ④

44) 정답 도서 의 휴 일 도서 의 이 간 도서의[ ]

해설 도서 장은 임의 정한 휴 일과 도서 이[ ]

간 도서의 상 등을 게 할 의 가 있다

년 학 간고사 대비2013 2 현대고 대비

ECN-0102-2013-001-000076193

45) 정답[ ] ①

해설 제 조의 정 휴 일 의 휴 일의 사전 게[ ] 3

는 도서 장의 의 조항에 속한다

46) 정답[ ] ①

해설 개인 정 호 의 를 제 하 했 만 항[ ]

나눠서 제 하 않고 대 나열하고 있다

47) 정답[ ] ②

해설 제 조의 내 을 회사는 다른 회사 협[ ] 7 lsquo

계약을 통해 서비 를 제공하는 경 회 의 아이디

등 개인 정 를 해당 회사에 전송할 있다는 내rsquo

이 있으 의 제점을 제 할 있다②

48) 정답[ ] ④

해설 는 도서 장의 의 에 해당하고 나 는 도[ ] ④

서 장의 리에 해당한다

49) 정답[ ] ③

50) 정답 은 음독으 적었고 은 훈독으 적었[ ] (1)

다 과 동일한 표 리 적은 것은 이고 (2) ce

과 동일한 표 리 적은 것은 이다ab

51) 정답[ ] ③

52) 정답[ ] ①②

53) 정답[ ] ③

54) 정답[ ] ③

55) 정답[ ] ①

56) 정답 른 죠코 어린 노 하니라[ ] A B

57) 정답 세 어에서는 활 형이 칙적으[ ] lsquo rsquoㄹㅇ

나타났 만 개화 어에서는 활 형이 쓰 다 lsquo rsquo ㄹㄴ

58) 정답 호 가 흔[ ] (1) (2)

59) 정답[ ] ④

60) 정답[ ] ③

Page 32: 현대고대비 국어 - chamsoriedu.com 「콘텐츠산업진흥 법」외 에도 저작권 의하여 ... 다른주체에게어떤동작을하도록만드는것을나타내는

년 학 간고사 대비2013 2 현대고 대비

ECN-0102-2013-001-000076193

니다

개 보 보7 ( )

사는 보통신망 등 계 는 에lsquo rsquo lsquo rsquo

원 개 보 보 니다 개lsquo rsquo

보 보 사 에 는 사 개lsquo rsquo

보 취 니다 다만 사는 다 lsquo rsquo

사 계 통 공 는 경우 원 lsquo rsquo

등 개 보 당 사에 습니lsquo rsquo

원 리에8 (lsquo rsquo lsquo rsquo lsquo rsquo

)

원 에 리lsquo rsquo lsquo rsquo lsquo rsquo①

원에게 가 도 여 는lsquo rsquo 3

니다

사는 원 가 개 보 우 가lsquo rsquo lsquo rsquo lsquo rsquo②

거 사 경우 는 미 에 어 거 lsquo

사 사 운 우 가 는 경우 당rsquo lsquo rsquo

습니다lsquo rsquo

원 가 도 거lsquo rsquo lsquo rsquo lsquo rsquo 3③

가 사 고 지 경우에는 시 사에lsquo rsquo

통지 고 사 내에 니다lsquo rsquo

경우에 당 원 사에 그 사실3 lsquo rsquo lsquo rsquo④

통지 지 거 통지 도 사 내에 지 lsquo rsquo

생 경우 사는 지지 습니다lsquo rsquo

원에 통지9 (lsquo rsquo )

사는 특 다 원에게 통지 경우lsquo rsquo lsquo rsquo

공지 게시 통 상 게시 개별 통지에7

갈 습니다

사10 (lsquo rsquo )

사는 과 지 미lsquo rsquo①

에 는 지 계 고

공 여 다 여 니다lsquo rsquo

사는 원 게lsquo rsquo lsquo rsquo lsquo rsquo②

도 개 보 신 보 포 보 보 시( )

갖 어 개 보 취 공시 고

니다

사는 과 여 원lsquo rsquo lsquo rsquo③

견 만 당 다고 경우에는

리 여 니다 원 견 만 사 lsquo rsquo

에 는 게시 거 우편 등 통 여

원에게 리 과 결과 달 니다lsquo rsquo

원11 (lsquo rsquo )

원 다 여 는 니다lsquo rsquo ①

신청 는 변경 시 허 내 등1

타 보 도2

사가 게시 보 변경3 lsquo rsquo

사가 보 보 컴퓨 그4 lsquo rsquo (

등 등 신 는 게시)

사 타 등 지 재산 에5 lsquo rsquo 3

사 타 상 거 업6 lsquo rsquo 3

는 폭 시지 상 타 공7 middot middot

에 는 보 에 공개 는 게시 는lsquo rsquo

사 동 없 리 사8 lsquo rsquo

타 거 당9

원 계 규 내lsquo rsquo lsquo②

여 공지 주 사 사가 통지 는rsquo lsquo rsquo

사 등 여 타 사 업 에 lsquo rsquo

는 여 는 니다

- (wwwnavercom)

zb47) 위 약 의 조항에서 같은 제점을 하lt gt

고 있는 조항은

lt gt

제휴 회사에 회 의 아이디 개인 정 를 전송할 있도

한 조항은 고객에게 당한 조항이다

1 7 8① ② ③

④ 9 ⑤ 10

립 도 규

1 ( )

규 립 도 립 어린 청 도(

포 다 료 시 열 시 말) (

다 에 사 규 립 도)

편 진 다

2 ( )

규 립 도 도 다 에( lsquo rsquo )

고 는 도 에 도lsquo rsquo 2 2

료 에 여 다 다만 특 료 귀

료 등 료 에 사 립 도

도 다 다( lsquo rsquo )

3 ( )

도 다 각 같다①

공 공 다만 연1

연 간 다

년 학 간고사 대비2013 2 현대고 대비

ECN-0102-2013-001-000076193

매월 째 째 월2

도 도 리 그 사3

가 다고 는

도 에 미리 게1 3②

시 여 다

시간4 ( )

도 시간 도 여 게시 다

등 등5 ( )

도 료 시 는 는 도①

지에 등 후

등 에 사 도②

사 료6 ( )

도 료 시 에 사 료는 도

7 ( )

는 다 각 여 는 니 다

도 료 시 상 리1 lsquo rsquo

도 료 시 훼 는2 middot

지 가 닌 곳에 식 거 담3

우는

도 보 등 보 검색열4 middot

그 에 도 질 지 여 도5

여 게시 사 는

질 지8 ( )

도 다 거 도①

질 게 우 가 는 에 여는 도

도 가 각 어느7②

에는 지 게 거 도

료9 ( )

도 료는 다 각 경우 다①

상 도 간에 료 는 것 말1 (

다 등 다 도 과 여 경우)

공 원 공 상 는 경우2

그 에 도 다고 는 경우3

가능 도 료 는 도②

는 에 다

변상10 ( )

가 도 료 시 럽 거 거①

못 쓰게 거 어 린 경우에는 변상 여

도 에 변상 여 게시1②

여 다

등 규 에 것 에 도11 ( )

료 시 등에 사

도 다

립 도- (httpwwwnlgokr)

zb48) 다음 정 리 의 의 으 볼 때 가장

이 적인 것은

도 시간 도 여 게시 다①

등 에 사 도②

가능 도 료 는 도 는③

에 다

④ 도 에 변상 여 게10 1

시 여 다

⑤ 도 가 각 어느7

에는 지 거 도

zb49) 를 참고하여 이 어의 성격을 설 한lt gt

것으 적절하 않은 것은

① 보 에 는 어 시 상 고 어 시lt gt lsquo rsquo

에 보여주고 다

② 진 어 어원에 견 고 다

에는 타 어 들어가는 것 다 lsquo rsquo

③ 에 들어갈 말 각각 고 어 어 신 어~

들 언어는 질 격 강 통 없었다

④ 시 우리 에 가 었지만 지 계

과 달리 들 통 사 달 어 웠

년 학 간고사 대비2013 2 현대고 대비

ECN-0102-2013-001-000076193

⑤ 크 몽골 만주 공통어가 우리 어 같

계열에 다는 에 사 특 짐

가( )

善化公主主隱 공주님

他密只嫁良置古 몰 결 고

薯童房乙 맛

夜矣卯乙抱遣去如 에 몰 고 가다

( )

始汝 會隱日恚見隱扐 만 에 본

恥隱汝衣淸隱笑 맑 웃

고 시 여 공 크다 만 다[ ] ( ) ( ) ( ) ( )始 汝 會扐

내다 에 보다 견( ) ( )恚 見 다( )隱

럽다 맑다 청 웃( ) ( ) ( ) ( )恥 衣 淸 笑

zb50) 위의 나 를 함 고 음에 답하( ) lt gt

보lt gt

( )素那或云金川 白城郡蛇山人也

운 사산

는 고 다 는( )[ ( ) ] (素那 金川 白城

사산 사 다) ( ) 郡 蛇山

삼 사- lsquo rsquo 47

에 제 된 단어 의 표 리를 조건(1) lt gt ( ) lt gt

에 맞게 서 하

건lt gt

lsquo 었고 었다 태rsquo

에 제 된 단어 동일한 표 리에(2) lt gt ( )

의해 적은 것을 나 에서 찾아 조건 에 맞게 서 하( ) lt gt

건lt gt

에 당 는 각각( ) 개 쓸 것2 단

당 는 가 여러 개 어도 개만 쓸 것 각2

개 과 도 쪽에 개만2 2

드시 지 것( )

과 동 원리 것lsquo 고

과 동 원리 것 다rsquo

태 것

가( )

素那(或云金川) 白城郡蛇山人也

소나 또는 천 이라 한다 는 성 사( ) ( ) ( )素那 金川 白城郡〔 〕

산 사람이다 현대어 풀이( ) ( )蛇山

나( )

紫布岩乎希 회

執音乎手母牛放敎遣 자 손 암쇼 노히 고

吾 不喩慙 伊賜等肹 肹 나 안디 리샤

花 折叱肹 可獻乎理音如 고 것거 도림다

다 향찰은 리말을 리 으 적은 표 이었 만 생( )

은 고 대를 넘 하고 끊어 고 말았다 랜 세

동안 갈고 닦아 체계적이었던 향찰 표 이 사라졌

을 인은 크게 두 가 나누어 생각해 볼 있다

하나는 족 사회의 한 선호도에서 찾을 있다 라 때

향찰은 주 족 계 에서 사 했을 것으 인다 한 을

알 하고서는 한자를 활 하여 리말을 리 으 표

하 란 가능하 때 이다 런데 족들은 간이 흐

를 향찰과 같은 리 표 을 익혀 사 하 다는

아 한 을 대 사 하는 쪽을 선호하게 되었다 더 이

고 초에 인재 등 을 위해 과거제도가 행되 서 한 선

호도가 더 높아졌고 결 향찰은 소 되고 말았다

또 다른 가능성은 한 어의 특성에서 찾을 있다

터 한 과 일 세 나라는 한자 화 에 속해 다

당연한 이야 겠 만 표의 자인 한자는 어를 표 하

에 매 적절하다 어의 음절은 성 ( ) ( )聲母 韻母

이 어 고 여 에 성조가 추가되어 최종 소리가 결정된

다 래서 어는 단음절을 하나의 한자 표 하 된

다 에 초성 성 종성의 세 가 소가 하나의 음절

년 학 간고사 대비2013 2 현대고 대비

ECN-0102-2013-001-000076193

을 이 는 한 어는 음절 조가 잡하고 음절의 가 많아

서 한자 차 만으 한 어의 소리를 만족 럽게 표 할

없었다 를 들어 한 어에서는 어 니 같이 음절 lsquo rsquo

이 어 단어가 얼마든 있으나 어는( ) 複數音節

자 하나 나타내 만이다lsquo [m ]rsquo 母 ǔ

한편 일 어의 표 은 핵 적 단어는 한자 적고 토는

가나라는 일 의 자 적는 이다 적인 의 를 나

타내는 은 표의 자인 한자 적고 적 계를 나

타내는 토는 표음 자 적는 셈이니 자세히 살펴

리의 향찰 표 을 쏙 빼닮았음을 알 있다 한 어 같

은 착어이 서도 일 어에만 향찰과 유사한 표 이 살아

남은 것은 일 어의 특 때 이다 일 어는 하나의 자음과

음의 결합으 음절을 이 고 침이 거의 없는 음절 언어

이다 이러한 음절의 특색에다가 토가 달한 착어라는 점

이 향찰과 유사한 표 이 살아남을 있는 비결이었다

하 만 같은 착어라도 다양한 음소 침이 달한 한

어는 향찰 표 하는 데 근 적으 한계가 있었다

zb51) 다 하여 의 행에 대한 탐 한 결과( ) lt gt 2

않은 것은

보lt gt

善花公主主隱 공주니믄 공주님( )

----------------------------------------

-

他密只嫁良置古 그 지 얼어 고 몰 결(

----------------------------------------

-

薯童房乙 맛 맛( )

夜矣卯乙抱遺去如 몰 고 가다 에 몰 고(

가다)

주동 역 동- (薯童謠『 』

에 2 ( )他密只嫁良置古

얼다 시집가다 결 다 말 lsquo rsquo

① 실질 미 지니고 므 타 타lsquo ( )rsquo lsquo [ ]

② 에 실질 미 타내고 지 는lsquo rsquo lsquo [ ]rsquo lsquo [ ]密只 密 只

계 타내는

③ 얼어는 실질 미 포 고 므 가lsquo rsquo lsquo [ ]rsquo嫁

것lsquo [ ]rsquo 良

④ 고 어간 는 실질 미 지니고 므lsquo rsquo lsquo -rsquo

것lsquo [ ]rsquo 置

⑤ 고 어미 고는 계 타내고 므lsquo rsquo lsquo- rsquo

고 것lsquo [ ]rsquo 古

가( )

엉 훈 민middot middot middot middot middot世 宗 御 製 訓 民 正 音

말 미 듕 귁에 달middot middot middot middot middot middot middot middot中 國 文 字

니 런middot middot middot middot middot middot 어린middot middot middot middot百 姓

니 고 도 내 들middot middot middot middot middot middot middot middot middot 시러middot

펴 몯middot 미middot middot 니 내middot middot middot middot middot middot middot middot 爲

어엿middot 겨 새middot middot middot 믈여듧middot middot middot middot字 니middot middot middot

사 마다 니겨 킈 middot middot middot middot middot middot middot middot middot便 安

고 미니middot middot middot middot

본 는 상( ) (象

원리에 만들어진 본) ( )形 ㄱ ㄴ ㅁ ㅅ ㅇ

에 는 가 원리에( )加劃

그리고( )ㅋ ㄷ ㅌ ㅂ ㅍ ㅈ ㅊ ㆆ ㅎ

쓰는 병 원리에 만들어진( )竝書

마지막 체( ) ( )異體ㄲ ㄸ ㅃ ㅆ ㅉ ㆅ

ᅀ 다 상 원리에 ㅇ ㄹ

지 는 삼재 상 본 본( ) ( ) ( 天地人 三才

탕 므림과 림에 ) (初ㅡ ㅣ

재)( ) ( )( )出字 再出字ㅗ ㅏ ㅜ ㅓ ㅛ ㅑ ㅜ ㅕ

병 그리고 들 에 다시( )ㅘ ㅝ ㅣ

( )ㅣ ㅢ ㅚ ㅐ ㅟ ㅔ ㆉ ㅒ ㆌ ㅖ ㅙ ㅞ

zb52) 가 에 대한 설 으 르 않은 것을( ) 두 고르

① 어쓰 규 지키고 다

② 리 고 다

③ 말 미 미 등 어 사 다lsquo rsquo

④ 개 지 다

년 학 간고사 대비2013 2 현대고 대비

ECN-0102-2013-001-000076193

⑤ 어 원 에 가 도 고 다

엉 훈 민世 宗 御 製 訓 民 正 音

말 미 듕귁에 달 니

런 어린 니 고 도middot

내 들 시러 펴 몯 미 니middot

내 어엿 겨 새 믈여듧

사 마다 니겨middot 킈 고

미니

훈민 언 본- lsquo rsquo 5 (1459 )

zb53) 위의 에 대한 현대어 풀이가 르~ 않은 것

① 우리 말 과 달

② 어리 말 고 는 것 어도

③ 신 생각 마 껏 펼 는 사 많다

④ 게 생각 여

⑤ 사 마다 게

zb54) 훈민정음 언해 에는 한 을 창제한 동 가 드러나

있다 훈민정음 창제의 정 과 내 이 잘 연결된 것

① 주 신 말 미 듕귁에 달

② 민 신 내 어 겨

③ 신 뻔 킈 고 미니

④ 실 신 사 마다 니겨

⑤ 귀 신 계 주 는 훈민 신과 거리가

가 엉 훈 민( ) middot middot middot middot middot世 宗 御 製 訓 民 正 音 

말 미 귁에 中 國 달 文 字

니 런 어린 니 百 姓

고 도 내 들 시러 펴 몯

미 니 내 어엿 爲 겨 새

믈여듧 니 사 마다 니 字

겨 킈 고 미니 便 安

훈민 언 본- lsquo ( )rsquo ( ) 5 (1459 )訓民正音 世祖

( )

[ 1 ]

동 룡 샤 마다 복( ) ( ) ( )海東 六龍 天福

시니 고 동( ) ( )古聖 同符 시니

[ 2 ]

매 니 곶 여

미 므 니 그 내 러

가 니

[ 125 ]

우 미리( )千世 샨( )定 에( )漢水北 累仁

누 개 샤 복 업 시니( ) ( ) 開國 卜年

신( )聖神 니 샤도 경 근민 샤 욱( )敬天勤民

드시리 다

님 쇼 산 가( ) ( )洛水 山行

미드니 가

어 가- lsquo ( )rsquo 27龍飛御天歌

다 우리신 니쓰고 다만 만 쓰( )

거 샹 귀쳔 다보게 러 귀

여 쓴 도 신 보 가 고 신 에

말 어 보게 각 에 사 들

고 본 몬 능통 후에

죠 죠 니

드 도 만 공 에 사

드 미 죠 고 고 여 보 죠

보다 얼마가 거시 어신고 니 첫

가 죠 니 죠

민 들 어 신 샹

귀쳔 도보고 어보 가 만 늘

고 폐 에 만쓴 죠 민

도 러보지못 고 보니 그게 엇지

심 니 리 보 가 어 운건 다

니 쳣 말마 지 니 고 그

쓰 에 가 우 지 지

몰 거 본후에 가 어 지

고 그니 쓴편지 쟝 보

년 학 간고사 대비2013 2 현대고 대비

ECN-0102-2013-001-000076193

쓴것보다 듸 보고 그 마 니 쓴 고

어 못

그런고 에 리 과 가

만 쓴 못 민 말만 듯고

고 편 그 못 보니 그사 단

병신 못 다고 그사 식 사

니 만 고 다 과 그사

만 고 다 과 업 사 보다 식 고

죠 도 고 각 과

견 고 실 직 귀쳔 간에 그

고도 다 것 몰 귀죡 보다

사 우리 신 귀쳔 다 업

시 신 보고 과 지 게 랴

시니 샹 귀쳔 간에 우리 신 걸

간 보 새지각과 새 걸 미리

독립신- lsquo (1896)rsquo

zb55) 친 어 나의 제 장( ) 2 매 함축적

의 가 가장 유사한 것은

① 지 눈 내리고 매 득 니 내 여 가

사- lsquo rsquo

② 도 어 리듯 그 게 어 다

주 사- lsquo rsquo

③ 눈 살 다 죽 어 린 과 체 여

눈 새벽 지 도 살 다

눈- lsquo rsquo

④ 삶 근심과 고단 에 돌 거니는 여 거 는

여 리 내린 살가지 에 눈 리 눈 리

택 그 생 에- lsquo rsquo

⑤ 늘 러 고 러

청룡 룡 어 개 루 우

신경림 계- lsquo rsquo

zb56) 친 를 위 가 나 에 나타난A B ( ) ( )

세 어의 특 에 의거하여 세 어 표 하

그 산 고 공 도 맑지만

A

주변에 쓰 리는 어리 사 많다

B

건lt gt

식 가 에 타 어 특징에( ) ( )

거 과 어쓰 는 고 지 말 것

A

B

zb57) 가 의( ) 달 아ㆍ 다 의 ( ) 나셔에서 알 있는

세 어 개화 어의 특 을 비 하여 조건 에lt gt

맞게 서 하

건lt gt

어에 는lsquo 개

어에 는 다 태rsquo

zb58) 은 가 는 다 에 나 는 절lt 1gt ( ) lt 2gt ( )

일 를 췌한 것이다 의 의 가 lt 1gt (1)~(2)

유사한 말을 에서 찾아 쓰lt 2gt

보lt 1gt

런 (1) 어린 니 고百 姓

도 내 들 시러 펴 몯 미

사 마다 (2) 니겨 便 安

킈 고 미니

보lt 2gt

죠 고 고 여 보 죠

보다 얼마가 거시 어신고 니 첫 가

죠 니 죠 민

들 어 신 샹 귀쳔

도보고 어보 가 만 늘 고

폐 에 만쓴 죠 민 도

러보지못 고 보니 그게 엇지 심

니 리

년 학 간고사 대비2013 2 현대고 대비

ECN-0102-2013-001-000076193

lt 1 gt

동 룡 샤 마다 복 시( ) ( ) ( )海東 六龍 天福

고 동 시니( ) ( )古聖 同符

lt 2 gt

(A) 매 니 곶

여 니

미 므 니 그 내

러 가 니

lt125 gt

우 미리 샨 에( ) ( ) ( ) 千世 定 漢水北 累

누 개 샤 복 업 시 니( ) ( ) 仁開國 卜年 聖

신( ) 神 니 샤도 경 근민 샤( ) 敬天勤民

욱 드 시 리 다

님 쇼 산 가 ( ) ( )洛水 山行

미드니 가

- lt gt龍飛御天歌

zb59) 장과 내 상 유사한 성격의 조는125

① 뫼 고 고 고 고

어 그린 많고 많고 고 고

어 러 는 울고 울고 가느니

도 견- lt gt

② 강 에 드니 몸 다

그믈 고 가니

뒷 뫼 엄 언 니( )藥

-

③ 말 없는 청산 태 없는 다

값 없는 청 없는 월

에 병 없는 몸 별 없 늙 리

-

④ 가마귀 골에 가지 마

낸 가마귀 새

청강에 것 시 몸 러 가( ) 淸江

-

⑤ 진 골에( ) 白雪

가 매 는 어느 곳에 었는고

에 갈 곳 몰( ) 夕陽

색-

zb60) 위 에 나타난 세 어의 특 으 적절하 않은

것은

① 룡 어 주격 사에 당 는 가 사( ) lsquo rsquo六龍

고 다

② 샤 어에도 어 주체 쓰 다

는 것 다

③ 매 어 달리 사 택에 어

가 지 지지 고 다

④ 므 원 상 직 어 지 다

⑤ 드시리 다 주체 과 상 께 사

고 다

수고 하셨습니다hearts hearts

년 학 간고사 대비2013 2 현대고 대비

ECN-0102-2013-001-000076193

보닷컴에 공 는 별 보는 고등

들 여 주 는

들 습니다 슷 동 지

가 복 는 것 도가

니 복 여 습 시고 거 시

니다

정답 해설

1) 정답[ ] ④

해설 다른 것은 두 특정 업이나 단 내에서 사[ ]

하는 일종의 은어 사회 언에 해당한다 러나

는 언이 아니라 단과대학을 여서 단대 사lsquo rsquo lsquo rsquo lsquo④

대학을 여서 사대라고 한 말에 해당하 일rsquo lsquo rsquo

사회에서도 널리 쓰이 사회 언이라 할

없다

2) 정답[ ] ⑤

해설 사회 언은 같은 단 내에서 쓰이는 언어이[ ] lsquo rsquo

동일 단끼리는 단결 과 친 감을 형성하는

능을 하 리적 안감이 일어나 않는다

3) 정답[ ] ③

해설 사람이라는 차 적 표현에 대한 대안적 표현이[ ]lsquo rsquo

인 아내 처 등으 볼 있다lsquo rsquo

4) 정답[ ]⑤

해설 남성은 주 격 체를 사 한다[ ]

5) 정답[ ] ⑤

해설 흑인은 검다라는 뜻을 가 고 있을 뿐 인[ ]lsquo rsquo lsquo rsquo lsquo rsquo

다 열등한 뜻을 내포하 않는다

6) 정답 살 색 첫 작품[ ] - -

해설 살색 혹은 킨색은 한 인의 피 색을 뜻[ ] lsquo rsquo lsquo rsquo

하는 것으 인종 차 을 추 고 출 이주민

의 평등 을 침해할 있어 년 표 이2005

살 색으 이름을 꾸었다 처녀작은 처녀라lsquo rsquo lsquo rsquo lsquo rsquo

는 단어가 가 고 있는 곡된 성 인 을 한 것

으 첫 작품정도 꾸어 사 하는 것이 좋다lsquo rsquo

7) 정답[ ] ⑤

해설 호는 아들에게 해체를 사 하고 있다[ ] ① ②

장 을 성하는 청자는 자 의 아 느리 아lsquo

들 세 이다 호는 아 느리에게 해rsquo ③

체를 사 하고 있다 호가 느리 아 에게 ④

사 한 해 체 아들에게 사 한 해체는 두 비lsquo rsquo lsquo rsquo

격 체에 해당한다 호는 자 의 아랫사람인 ⑤

느리에게 아들과 마찬가 해체를 사 하는 것이

상 이 만 임 을 한 느리에게 고마 과 쁨

존 의 표 를 하 위해 자 의 아 에게 말하듯

해 체를 사 하고 있다

8) 정답[ ] ③

9) 정답[ ] ⑤

10) 정답[ ] ①

해설 청자 할아 가 장의 주체 아 다 높을[ ] ( ) ( )

경 에는 압존 에 의해 장의 주체를 높이 않는lsquo rsquo

다 러 아 서가 아닌 아 는으 계 lsquo rsquo lsquo rsquo lsquo

니다 가 아닌 있 니다 표현하는 것이 르rsquo lsquo rsquo

11) 정답 당이 당을 쫒았다 당이[ ]

당에 다

해설[ ]

12) 정답[ ] ⑤

해설 서 다른 높임표현을 통해 청자에 대해 리[ ] ⑤

적 거리감을 나타내는 인 은 이 아니라 현정이

다 가 에서 현정은 에게 해 체를 사 함으 써 ( )

친근감을 드러낸다 나 에서 연 을 게을리하는 역 ( )

도 들 때 에 화가 난 현정이 선생님에게 항의하

는 장 에서는 하 체를 사 하여 리적 거리lsquo rsquo

가 어졌음을 나타내고 있다

13) 정답[ ] ①

해설 는 는 얼 빛이 날과 어찌 다르 고[ ] lsquo rsquo

라는 뜻으 전과 달리 임이 화자를 않고

있음을 알 있다

14) 정답 달리 후 가 있다 이를 통해 경[ ] lt gt

쾌한 음악성을 형성하고 노 젓는 상황을 체적으

형상화하는 역할을 한다

15) 정답[ ] ①

16) 정답[ ] ⑤

해설 다 의 자연은 를 성찰하게 하는 대상[ ] ( )⑤

이자 정의 대상이다 의 자연은 자 의 상황과 ⑤

처 를 드러내는 경으 서의 역할을 하 이

이 없다

17) 정답[ ] ③

해설 는 빈천 을 해결하고자 했으나 강산[ ] lsquo ( )rsquo 貧賤③

과 풍 을 달라는 에 거절하 다고 함으 써 자

연에 대한 애정을 드러내고 있으 는 않는

임에 대한 망을 개에게 전가 켜서 임에 대한 리

을 드러내고 있다

18) 정답[ ] ③

년 학 간고사 대비2013 2 현대고 대비

ECN-0102-2013-001-000076193

19) 정답[ ] ⑤

해설 고상한 음악가의 이름을 리말 꽝 럽[ ]

게 꿈으 써 언어유희를 통해 음을 유 하고 있

다 이는 고상한 척하는 총 를 비꼼으 써 비판적

태도를 드러내는 것이 대상을 꽝 럽게 표현

하여 총 의 허 과 사치를 풍자하고 있다

20) 정답[ ] ⑤

해설 는 작품 속 경에 대한 설 이 드러나는 것이[ ]

서 자의 주 적인 견해가 접적으 드러나는 것이

아니다

21) 정답[ ] ⑤

22) 정답[ ] ②

23) 정답[ ] ④

24) 정답[ ] ①

해설 적강 티프는 주인공의 비 한 출생이나 능[ ] ①

과 이 있는 것으 조정의 능함을 풍자하는lsquo rsquo

것과는 거리가 다

25) 정답 픔 나[ ] ( )

해설 의 음악은 고통 는 사람들을 위 하고 아픔[ ] lsquo rsquo

을 치유해 주는 능을 한다고 할 있다 의 lt gt

픔 도 소 된 이 과 더 어 살아가는 따뜻한 마음lsquo rsquo

을 상 한다

26) 정답[ ] ⑤

해설 에게 선천적으 주어 각 장애라는 역경[ ]

은 의 이라는 가사 연 을 있다lsquo rsquo

27) 정답[ ] ④

해설 는 장 란 선 에게 은 개인적인 인상을[ ]

소녀 장정 등으 표현한 것이다lsquo rsquo

28) 정답[ ] ②

해설 담자가 피 담자의 언어적 표현이나 비언어[ ]②

적 표현 하 독자는 담의 위 나 피

담자의 감정 상태를 알 있다 이를 통해 독자는

담 상황을 더 생생하게 느낄 있고 피 담자

를 더 잘 이해할 있게 된다

29) 정답[ ]③

해설 일상생활과 역도 선 서의 성과에 된 것에서[ ]

역도를 하 서 겪는 어 과 내적 고민으 화제를

전화하 위한 것이다

30) 정답[ ] ①

해설 릿속에 새겨 넣듯 이 억되도 함 세상[ ] ② ③

살이가 힘들고 고생 러 속 하여 자유를 ④

가 없는 고통의 상태를 비유적으 이르는 말

적의 침입을 막 위해 쌓은 축 켜야 할⑤

대상을 비유적으 이르는 말이다

31) 정답[ ] ④

해설 이 의 종류는 전 으 인 사건 경[ ] lsquo

비평을 성 소 삼는다rsquo

32) 정답[ ] ④

해설 근은 삼대독자 태어났음을 에서 확인할[ ]

있다 형제들과의 담은 이뤄 가 없다

33) 정답[ ] ⑤

해설 근은 가난에도 하고 화가를 꿈꾸었다[ ] (3

단 또한 다른 화가 망생들은 정 육을)

위해 상 학 학 해 유학 에 랐 만

근은 다른 을 찾아야 했다 단 세에(5 ) 18

근은 조선 전람회에 입선하 다 단 의(6 )

만종은 인간과 자연이 엮어 가는 경건한 조화 을lsquo rsquo

나타낸다

34) 정답[ ] ①

해설 근이 속에서도 창작활동을 추 않고[ ]

하는 닭은 은 세상과 타협할 르는

근이 세상의 이해를 하 위한 가장 떳떳한 단

이 때 이다

35) 정답[ ] ⑤

해설 전 은 서 자의 주 적인 평이 리는 것이[ ]

만 위 제 은 인 이 살았던 대 사회적 경

을 통해 객 적인 인 의 을 제 하고 있다

36) 정답[ ] ⑤

해설 전 은 인 사건 경 비평이라는[ ] lsquo rsquo⑤

성 이 어져 있다

37) 정답[ ] ①

해설 이 은 동양인과 서양인의 사고 에 차이가[ ]

있다는 것을 대조를 통해 설 하고 있다 또 쓴이

의 제자가 축 경 를 러 가서 경험한 일화를

통해 동양인이 서양인에 비해 주 상황에 더 많은

주의를 인다는 주장을 뒷 침하고 있다

38) 정답[ ] ④

39) 정답[ ] ②

40) 정답[ ] ②

41) 정답[ ] ④

42) 정답[ ] ③

43) 정답[ ] ④

44) 정답 도서 의 휴 일 도서 의 이 간 도서의[ ]

해설 도서 장은 임의 정한 휴 일과 도서 이[ ]

간 도서의 상 등을 게 할 의 가 있다

년 학 간고사 대비2013 2 현대고 대비

ECN-0102-2013-001-000076193

45) 정답[ ] ①

해설 제 조의 정 휴 일 의 휴 일의 사전 게[ ] 3

는 도서 장의 의 조항에 속한다

46) 정답[ ] ①

해설 개인 정 호 의 를 제 하 했 만 항[ ]

나눠서 제 하 않고 대 나열하고 있다

47) 정답[ ] ②

해설 제 조의 내 을 회사는 다른 회사 협[ ] 7 lsquo

계약을 통해 서비 를 제공하는 경 회 의 아이디

등 개인 정 를 해당 회사에 전송할 있다는 내rsquo

이 있으 의 제점을 제 할 있다②

48) 정답[ ] ④

해설 는 도서 장의 의 에 해당하고 나 는 도[ ] ④

서 장의 리에 해당한다

49) 정답[ ] ③

50) 정답 은 음독으 적었고 은 훈독으 적었[ ] (1)

다 과 동일한 표 리 적은 것은 이고 (2) ce

과 동일한 표 리 적은 것은 이다ab

51) 정답[ ] ③

52) 정답[ ] ①②

53) 정답[ ] ③

54) 정답[ ] ③

55) 정답[ ] ①

56) 정답 른 죠코 어린 노 하니라[ ] A B

57) 정답 세 어에서는 활 형이 칙적으[ ] lsquo rsquoㄹㅇ

나타났 만 개화 어에서는 활 형이 쓰 다 lsquo rsquo ㄹㄴ

58) 정답 호 가 흔[ ] (1) (2)

59) 정답[ ] ④

60) 정답[ ] ③

Page 33: 현대고대비 국어 - chamsoriedu.com 「콘텐츠산업진흥 법」외 에도 저작권 의하여 ... 다른주체에게어떤동작을하도록만드는것을나타내는

년 학 간고사 대비2013 2 현대고 대비

ECN-0102-2013-001-000076193

매월 째 째 월2

도 도 리 그 사3

가 다고 는

도 에 미리 게1 3②

시 여 다

시간4 ( )

도 시간 도 여 게시 다

등 등5 ( )

도 료 시 는 는 도①

지에 등 후

등 에 사 도②

사 료6 ( )

도 료 시 에 사 료는 도

7 ( )

는 다 각 여 는 니 다

도 료 시 상 리1 lsquo rsquo

도 료 시 훼 는2 middot

지 가 닌 곳에 식 거 담3

우는

도 보 등 보 검색열4 middot

그 에 도 질 지 여 도5

여 게시 사 는

질 지8 ( )

도 다 거 도①

질 게 우 가 는 에 여는 도

도 가 각 어느7②

에는 지 게 거 도

료9 ( )

도 료는 다 각 경우 다①

상 도 간에 료 는 것 말1 (

다 등 다 도 과 여 경우)

공 원 공 상 는 경우2

그 에 도 다고 는 경우3

가능 도 료 는 도②

는 에 다

변상10 ( )

가 도 료 시 럽 거 거①

못 쓰게 거 어 린 경우에는 변상 여

도 에 변상 여 게시1②

여 다

등 규 에 것 에 도11 ( )

료 시 등에 사

도 다

립 도- (httpwwwnlgokr)

zb48) 다음 정 리 의 의 으 볼 때 가장

이 적인 것은

도 시간 도 여 게시 다①

등 에 사 도②

가능 도 료 는 도 는③

에 다

④ 도 에 변상 여 게10 1

시 여 다

⑤ 도 가 각 어느7

에는 지 거 도

zb49) 를 참고하여 이 어의 성격을 설 한lt gt

것으 적절하 않은 것은

① 보 에 는 어 시 상 고 어 시lt gt lsquo rsquo

에 보여주고 다

② 진 어 어원에 견 고 다

에는 타 어 들어가는 것 다 lsquo rsquo

③ 에 들어갈 말 각각 고 어 어 신 어~

들 언어는 질 격 강 통 없었다

④ 시 우리 에 가 었지만 지 계

과 달리 들 통 사 달 어 웠

년 학 간고사 대비2013 2 현대고 대비

ECN-0102-2013-001-000076193

⑤ 크 몽골 만주 공통어가 우리 어 같

계열에 다는 에 사 특 짐

가( )

善化公主主隱 공주님

他密只嫁良置古 몰 결 고

薯童房乙 맛

夜矣卯乙抱遣去如 에 몰 고 가다

( )

始汝 會隱日恚見隱扐 만 에 본

恥隱汝衣淸隱笑 맑 웃

고 시 여 공 크다 만 다[ ] ( ) ( ) ( ) ( )始 汝 會扐

내다 에 보다 견( ) ( )恚 見 다( )隱

럽다 맑다 청 웃( ) ( ) ( ) ( )恥 衣 淸 笑

zb50) 위의 나 를 함 고 음에 답하( ) lt gt

보lt gt

( )素那或云金川 白城郡蛇山人也

운 사산

는 고 다 는( )[ ( ) ] (素那 金川 白城

사산 사 다) ( ) 郡 蛇山

삼 사- lsquo rsquo 47

에 제 된 단어 의 표 리를 조건(1) lt gt ( ) lt gt

에 맞게 서 하

건lt gt

lsquo 었고 었다 태rsquo

에 제 된 단어 동일한 표 리에(2) lt gt ( )

의해 적은 것을 나 에서 찾아 조건 에 맞게 서 하( ) lt gt

건lt gt

에 당 는 각각( ) 개 쓸 것2 단

당 는 가 여러 개 어도 개만 쓸 것 각2

개 과 도 쪽에 개만2 2

드시 지 것( )

과 동 원리 것lsquo 고

과 동 원리 것 다rsquo

태 것

가( )

素那(或云金川) 白城郡蛇山人也

소나 또는 천 이라 한다 는 성 사( ) ( ) ( )素那 金川 白城郡〔 〕

산 사람이다 현대어 풀이( ) ( )蛇山

나( )

紫布岩乎希 회

執音乎手母牛放敎遣 자 손 암쇼 노히 고

吾 不喩慙 伊賜等肹 肹 나 안디 리샤

花 折叱肹 可獻乎理音如 고 것거 도림다

다 향찰은 리말을 리 으 적은 표 이었 만 생( )

은 고 대를 넘 하고 끊어 고 말았다 랜 세

동안 갈고 닦아 체계적이었던 향찰 표 이 사라졌

을 인은 크게 두 가 나누어 생각해 볼 있다

하나는 족 사회의 한 선호도에서 찾을 있다 라 때

향찰은 주 족 계 에서 사 했을 것으 인다 한 을

알 하고서는 한자를 활 하여 리말을 리 으 표

하 란 가능하 때 이다 런데 족들은 간이 흐

를 향찰과 같은 리 표 을 익혀 사 하 다는

아 한 을 대 사 하는 쪽을 선호하게 되었다 더 이

고 초에 인재 등 을 위해 과거제도가 행되 서 한 선

호도가 더 높아졌고 결 향찰은 소 되고 말았다

또 다른 가능성은 한 어의 특성에서 찾을 있다

터 한 과 일 세 나라는 한자 화 에 속해 다

당연한 이야 겠 만 표의 자인 한자는 어를 표 하

에 매 적절하다 어의 음절은 성 ( ) ( )聲母 韻母

이 어 고 여 에 성조가 추가되어 최종 소리가 결정된

다 래서 어는 단음절을 하나의 한자 표 하 된

다 에 초성 성 종성의 세 가 소가 하나의 음절

년 학 간고사 대비2013 2 현대고 대비

ECN-0102-2013-001-000076193

을 이 는 한 어는 음절 조가 잡하고 음절의 가 많아

서 한자 차 만으 한 어의 소리를 만족 럽게 표 할

없었다 를 들어 한 어에서는 어 니 같이 음절 lsquo rsquo

이 어 단어가 얼마든 있으나 어는( ) 複數音節

자 하나 나타내 만이다lsquo [m ]rsquo 母 ǔ

한편 일 어의 표 은 핵 적 단어는 한자 적고 토는

가나라는 일 의 자 적는 이다 적인 의 를 나

타내는 은 표의 자인 한자 적고 적 계를 나

타내는 토는 표음 자 적는 셈이니 자세히 살펴

리의 향찰 표 을 쏙 빼닮았음을 알 있다 한 어 같

은 착어이 서도 일 어에만 향찰과 유사한 표 이 살아

남은 것은 일 어의 특 때 이다 일 어는 하나의 자음과

음의 결합으 음절을 이 고 침이 거의 없는 음절 언어

이다 이러한 음절의 특색에다가 토가 달한 착어라는 점

이 향찰과 유사한 표 이 살아남을 있는 비결이었다

하 만 같은 착어라도 다양한 음소 침이 달한 한

어는 향찰 표 하는 데 근 적으 한계가 있었다

zb51) 다 하여 의 행에 대한 탐 한 결과( ) lt gt 2

않은 것은

보lt gt

善花公主主隱 공주니믄 공주님( )

----------------------------------------

-

他密只嫁良置古 그 지 얼어 고 몰 결(

----------------------------------------

-

薯童房乙 맛 맛( )

夜矣卯乙抱遺去如 몰 고 가다 에 몰 고(

가다)

주동 역 동- (薯童謠『 』

에 2 ( )他密只嫁良置古

얼다 시집가다 결 다 말 lsquo rsquo

① 실질 미 지니고 므 타 타lsquo ( )rsquo lsquo [ ]

② 에 실질 미 타내고 지 는lsquo rsquo lsquo [ ]rsquo lsquo [ ]密只 密 只

계 타내는

③ 얼어는 실질 미 포 고 므 가lsquo rsquo lsquo [ ]rsquo嫁

것lsquo [ ]rsquo 良

④ 고 어간 는 실질 미 지니고 므lsquo rsquo lsquo -rsquo

것lsquo [ ]rsquo 置

⑤ 고 어미 고는 계 타내고 므lsquo rsquo lsquo- rsquo

고 것lsquo [ ]rsquo 古

가( )

엉 훈 민middot middot middot middot middot世 宗 御 製 訓 民 正 音

말 미 듕 귁에 달middot middot middot middot middot middot middot middot中 國 文 字

니 런middot middot middot middot middot middot 어린middot middot middot middot百 姓

니 고 도 내 들middot middot middot middot middot middot middot middot middot 시러middot

펴 몯middot 미middot middot 니 내middot middot middot middot middot middot middot middot 爲

어엿middot 겨 새middot middot middot 믈여듧middot middot middot middot字 니middot middot middot

사 마다 니겨 킈 middot middot middot middot middot middot middot middot middot便 安

고 미니middot middot middot middot

본 는 상( ) (象

원리에 만들어진 본) ( )形 ㄱ ㄴ ㅁ ㅅ ㅇ

에 는 가 원리에( )加劃

그리고( )ㅋ ㄷ ㅌ ㅂ ㅍ ㅈ ㅊ ㆆ ㅎ

쓰는 병 원리에 만들어진( )竝書

마지막 체( ) ( )異體ㄲ ㄸ ㅃ ㅆ ㅉ ㆅ

ᅀ 다 상 원리에 ㅇ ㄹ

지 는 삼재 상 본 본( ) ( ) ( 天地人 三才

탕 므림과 림에 ) (初ㅡ ㅣ

재)( ) ( )( )出字 再出字ㅗ ㅏ ㅜ ㅓ ㅛ ㅑ ㅜ ㅕ

병 그리고 들 에 다시( )ㅘ ㅝ ㅣ

( )ㅣ ㅢ ㅚ ㅐ ㅟ ㅔ ㆉ ㅒ ㆌ ㅖ ㅙ ㅞ

zb52) 가 에 대한 설 으 르 않은 것을( ) 두 고르

① 어쓰 규 지키고 다

② 리 고 다

③ 말 미 미 등 어 사 다lsquo rsquo

④ 개 지 다

년 학 간고사 대비2013 2 현대고 대비

ECN-0102-2013-001-000076193

⑤ 어 원 에 가 도 고 다

엉 훈 민世 宗 御 製 訓 民 正 音

말 미 듕귁에 달 니

런 어린 니 고 도middot

내 들 시러 펴 몯 미 니middot

내 어엿 겨 새 믈여듧

사 마다 니겨middot 킈 고

미니

훈민 언 본- lsquo rsquo 5 (1459 )

zb53) 위의 에 대한 현대어 풀이가 르~ 않은 것

① 우리 말 과 달

② 어리 말 고 는 것 어도

③ 신 생각 마 껏 펼 는 사 많다

④ 게 생각 여

⑤ 사 마다 게

zb54) 훈민정음 언해 에는 한 을 창제한 동 가 드러나

있다 훈민정음 창제의 정 과 내 이 잘 연결된 것

① 주 신 말 미 듕귁에 달

② 민 신 내 어 겨

③ 신 뻔 킈 고 미니

④ 실 신 사 마다 니겨

⑤ 귀 신 계 주 는 훈민 신과 거리가

가 엉 훈 민( ) middot middot middot middot middot世 宗 御 製 訓 民 正 音 

말 미 귁에 中 國 달 文 字

니 런 어린 니 百 姓

고 도 내 들 시러 펴 몯

미 니 내 어엿 爲 겨 새

믈여듧 니 사 마다 니 字

겨 킈 고 미니 便 安

훈민 언 본- lsquo ( )rsquo ( ) 5 (1459 )訓民正音 世祖

( )

[ 1 ]

동 룡 샤 마다 복( ) ( ) ( )海東 六龍 天福

시니 고 동( ) ( )古聖 同符 시니

[ 2 ]

매 니 곶 여

미 므 니 그 내 러

가 니

[ 125 ]

우 미리( )千世 샨( )定 에( )漢水北 累仁

누 개 샤 복 업 시니( ) ( ) 開國 卜年

신( )聖神 니 샤도 경 근민 샤 욱( )敬天勤民

드시리 다

님 쇼 산 가( ) ( )洛水 山行

미드니 가

어 가- lsquo ( )rsquo 27龍飛御天歌

다 우리신 니쓰고 다만 만 쓰( )

거 샹 귀쳔 다보게 러 귀

여 쓴 도 신 보 가 고 신 에

말 어 보게 각 에 사 들

고 본 몬 능통 후에

죠 죠 니

드 도 만 공 에 사

드 미 죠 고 고 여 보 죠

보다 얼마가 거시 어신고 니 첫

가 죠 니 죠

민 들 어 신 샹

귀쳔 도보고 어보 가 만 늘

고 폐 에 만쓴 죠 민

도 러보지못 고 보니 그게 엇지

심 니 리 보 가 어 운건 다

니 쳣 말마 지 니 고 그

쓰 에 가 우 지 지

몰 거 본후에 가 어 지

고 그니 쓴편지 쟝 보

년 학 간고사 대비2013 2 현대고 대비

ECN-0102-2013-001-000076193

쓴것보다 듸 보고 그 마 니 쓴 고

어 못

그런고 에 리 과 가

만 쓴 못 민 말만 듯고

고 편 그 못 보니 그사 단

병신 못 다고 그사 식 사

니 만 고 다 과 그사

만 고 다 과 업 사 보다 식 고

죠 도 고 각 과

견 고 실 직 귀쳔 간에 그

고도 다 것 몰 귀죡 보다

사 우리 신 귀쳔 다 업

시 신 보고 과 지 게 랴

시니 샹 귀쳔 간에 우리 신 걸

간 보 새지각과 새 걸 미리

독립신- lsquo (1896)rsquo

zb55) 친 어 나의 제 장( ) 2 매 함축적

의 가 가장 유사한 것은

① 지 눈 내리고 매 득 니 내 여 가

사- lsquo rsquo

② 도 어 리듯 그 게 어 다

주 사- lsquo rsquo

③ 눈 살 다 죽 어 린 과 체 여

눈 새벽 지 도 살 다

눈- lsquo rsquo

④ 삶 근심과 고단 에 돌 거니는 여 거 는

여 리 내린 살가지 에 눈 리 눈 리

택 그 생 에- lsquo rsquo

⑤ 늘 러 고 러

청룡 룡 어 개 루 우

신경림 계- lsquo rsquo

zb56) 친 를 위 가 나 에 나타난A B ( ) ( )

세 어의 특 에 의거하여 세 어 표 하

그 산 고 공 도 맑지만

A

주변에 쓰 리는 어리 사 많다

B

건lt gt

식 가 에 타 어 특징에( ) ( )

거 과 어쓰 는 고 지 말 것

A

B

zb57) 가 의( ) 달 아ㆍ 다 의 ( ) 나셔에서 알 있는

세 어 개화 어의 특 을 비 하여 조건 에lt gt

맞게 서 하

건lt gt

어에 는lsquo 개

어에 는 다 태rsquo

zb58) 은 가 는 다 에 나 는 절lt 1gt ( ) lt 2gt ( )

일 를 췌한 것이다 의 의 가 lt 1gt (1)~(2)

유사한 말을 에서 찾아 쓰lt 2gt

보lt 1gt

런 (1) 어린 니 고百 姓

도 내 들 시러 펴 몯 미

사 마다 (2) 니겨 便 安

킈 고 미니

보lt 2gt

죠 고 고 여 보 죠

보다 얼마가 거시 어신고 니 첫 가

죠 니 죠 민

들 어 신 샹 귀쳔

도보고 어보 가 만 늘 고

폐 에 만쓴 죠 민 도

러보지못 고 보니 그게 엇지 심

니 리

년 학 간고사 대비2013 2 현대고 대비

ECN-0102-2013-001-000076193

lt 1 gt

동 룡 샤 마다 복 시( ) ( ) ( )海東 六龍 天福

고 동 시니( ) ( )古聖 同符

lt 2 gt

(A) 매 니 곶

여 니

미 므 니 그 내

러 가 니

lt125 gt

우 미리 샨 에( ) ( ) ( ) 千世 定 漢水北 累

누 개 샤 복 업 시 니( ) ( ) 仁開國 卜年 聖

신( ) 神 니 샤도 경 근민 샤( ) 敬天勤民

욱 드 시 리 다

님 쇼 산 가 ( ) ( )洛水 山行

미드니 가

- lt gt龍飛御天歌

zb59) 장과 내 상 유사한 성격의 조는125

① 뫼 고 고 고 고

어 그린 많고 많고 고 고

어 러 는 울고 울고 가느니

도 견- lt gt

② 강 에 드니 몸 다

그믈 고 가니

뒷 뫼 엄 언 니( )藥

-

③ 말 없는 청산 태 없는 다

값 없는 청 없는 월

에 병 없는 몸 별 없 늙 리

-

④ 가마귀 골에 가지 마

낸 가마귀 새

청강에 것 시 몸 러 가( ) 淸江

-

⑤ 진 골에( ) 白雪

가 매 는 어느 곳에 었는고

에 갈 곳 몰( ) 夕陽

색-

zb60) 위 에 나타난 세 어의 특 으 적절하 않은

것은

① 룡 어 주격 사에 당 는 가 사( ) lsquo rsquo六龍

고 다

② 샤 어에도 어 주체 쓰 다

는 것 다

③ 매 어 달리 사 택에 어

가 지 지지 고 다

④ 므 원 상 직 어 지 다

⑤ 드시리 다 주체 과 상 께 사

고 다

수고 하셨습니다hearts hearts

년 학 간고사 대비2013 2 현대고 대비

ECN-0102-2013-001-000076193

보닷컴에 공 는 별 보는 고등

들 여 주 는

들 습니다 슷 동 지

가 복 는 것 도가

니 복 여 습 시고 거 시

니다

정답 해설

1) 정답[ ] ④

해설 다른 것은 두 특정 업이나 단 내에서 사[ ]

하는 일종의 은어 사회 언에 해당한다 러나

는 언이 아니라 단과대학을 여서 단대 사lsquo rsquo lsquo rsquo lsquo④

대학을 여서 사대라고 한 말에 해당하 일rsquo lsquo rsquo

사회에서도 널리 쓰이 사회 언이라 할

없다

2) 정답[ ] ⑤

해설 사회 언은 같은 단 내에서 쓰이는 언어이[ ] lsquo rsquo

동일 단끼리는 단결 과 친 감을 형성하는

능을 하 리적 안감이 일어나 않는다

3) 정답[ ] ③

해설 사람이라는 차 적 표현에 대한 대안적 표현이[ ]lsquo rsquo

인 아내 처 등으 볼 있다lsquo rsquo

4) 정답[ ]⑤

해설 남성은 주 격 체를 사 한다[ ]

5) 정답[ ] ⑤

해설 흑인은 검다라는 뜻을 가 고 있을 뿐 인[ ]lsquo rsquo lsquo rsquo lsquo rsquo

다 열등한 뜻을 내포하 않는다

6) 정답 살 색 첫 작품[ ] - -

해설 살색 혹은 킨색은 한 인의 피 색을 뜻[ ] lsquo rsquo lsquo rsquo

하는 것으 인종 차 을 추 고 출 이주민

의 평등 을 침해할 있어 년 표 이2005

살 색으 이름을 꾸었다 처녀작은 처녀라lsquo rsquo lsquo rsquo lsquo rsquo

는 단어가 가 고 있는 곡된 성 인 을 한 것

으 첫 작품정도 꾸어 사 하는 것이 좋다lsquo rsquo

7) 정답[ ] ⑤

해설 호는 아들에게 해체를 사 하고 있다[ ] ① ②

장 을 성하는 청자는 자 의 아 느리 아lsquo

들 세 이다 호는 아 느리에게 해rsquo ③

체를 사 하고 있다 호가 느리 아 에게 ④

사 한 해 체 아들에게 사 한 해체는 두 비lsquo rsquo lsquo rsquo

격 체에 해당한다 호는 자 의 아랫사람인 ⑤

느리에게 아들과 마찬가 해체를 사 하는 것이

상 이 만 임 을 한 느리에게 고마 과 쁨

존 의 표 를 하 위해 자 의 아 에게 말하듯

해 체를 사 하고 있다

8) 정답[ ] ③

9) 정답[ ] ⑤

10) 정답[ ] ①

해설 청자 할아 가 장의 주체 아 다 높을[ ] ( ) ( )

경 에는 압존 에 의해 장의 주체를 높이 않는lsquo rsquo

다 러 아 서가 아닌 아 는으 계 lsquo rsquo lsquo rsquo lsquo

니다 가 아닌 있 니다 표현하는 것이 르rsquo lsquo rsquo

11) 정답 당이 당을 쫒았다 당이[ ]

당에 다

해설[ ]

12) 정답[ ] ⑤

해설 서 다른 높임표현을 통해 청자에 대해 리[ ] ⑤

적 거리감을 나타내는 인 은 이 아니라 현정이

다 가 에서 현정은 에게 해 체를 사 함으 써 ( )

친근감을 드러낸다 나 에서 연 을 게을리하는 역 ( )

도 들 때 에 화가 난 현정이 선생님에게 항의하

는 장 에서는 하 체를 사 하여 리적 거리lsquo rsquo

가 어졌음을 나타내고 있다

13) 정답[ ] ①

해설 는 는 얼 빛이 날과 어찌 다르 고[ ] lsquo rsquo

라는 뜻으 전과 달리 임이 화자를 않고

있음을 알 있다

14) 정답 달리 후 가 있다 이를 통해 경[ ] lt gt

쾌한 음악성을 형성하고 노 젓는 상황을 체적으

형상화하는 역할을 한다

15) 정답[ ] ①

16) 정답[ ] ⑤

해설 다 의 자연은 를 성찰하게 하는 대상[ ] ( )⑤

이자 정의 대상이다 의 자연은 자 의 상황과 ⑤

처 를 드러내는 경으 서의 역할을 하 이

이 없다

17) 정답[ ] ③

해설 는 빈천 을 해결하고자 했으나 강산[ ] lsquo ( )rsquo 貧賤③

과 풍 을 달라는 에 거절하 다고 함으 써 자

연에 대한 애정을 드러내고 있으 는 않는

임에 대한 망을 개에게 전가 켜서 임에 대한 리

을 드러내고 있다

18) 정답[ ] ③

년 학 간고사 대비2013 2 현대고 대비

ECN-0102-2013-001-000076193

19) 정답[ ] ⑤

해설 고상한 음악가의 이름을 리말 꽝 럽[ ]

게 꿈으 써 언어유희를 통해 음을 유 하고 있

다 이는 고상한 척하는 총 를 비꼼으 써 비판적

태도를 드러내는 것이 대상을 꽝 럽게 표현

하여 총 의 허 과 사치를 풍자하고 있다

20) 정답[ ] ⑤

해설 는 작품 속 경에 대한 설 이 드러나는 것이[ ]

서 자의 주 적인 견해가 접적으 드러나는 것이

아니다

21) 정답[ ] ⑤

22) 정답[ ] ②

23) 정답[ ] ④

24) 정답[ ] ①

해설 적강 티프는 주인공의 비 한 출생이나 능[ ] ①

과 이 있는 것으 조정의 능함을 풍자하는lsquo rsquo

것과는 거리가 다

25) 정답 픔 나[ ] ( )

해설 의 음악은 고통 는 사람들을 위 하고 아픔[ ] lsquo rsquo

을 치유해 주는 능을 한다고 할 있다 의 lt gt

픔 도 소 된 이 과 더 어 살아가는 따뜻한 마음lsquo rsquo

을 상 한다

26) 정답[ ] ⑤

해설 에게 선천적으 주어 각 장애라는 역경[ ]

은 의 이라는 가사 연 을 있다lsquo rsquo

27) 정답[ ] ④

해설 는 장 란 선 에게 은 개인적인 인상을[ ]

소녀 장정 등으 표현한 것이다lsquo rsquo

28) 정답[ ] ②

해설 담자가 피 담자의 언어적 표현이나 비언어[ ]②

적 표현 하 독자는 담의 위 나 피

담자의 감정 상태를 알 있다 이를 통해 독자는

담 상황을 더 생생하게 느낄 있고 피 담자

를 더 잘 이해할 있게 된다

29) 정답[ ]③

해설 일상생활과 역도 선 서의 성과에 된 것에서[ ]

역도를 하 서 겪는 어 과 내적 고민으 화제를

전화하 위한 것이다

30) 정답[ ] ①

해설 릿속에 새겨 넣듯 이 억되도 함 세상[ ] ② ③

살이가 힘들고 고생 러 속 하여 자유를 ④

가 없는 고통의 상태를 비유적으 이르는 말

적의 침입을 막 위해 쌓은 축 켜야 할⑤

대상을 비유적으 이르는 말이다

31) 정답[ ] ④

해설 이 의 종류는 전 으 인 사건 경[ ] lsquo

비평을 성 소 삼는다rsquo

32) 정답[ ] ④

해설 근은 삼대독자 태어났음을 에서 확인할[ ]

있다 형제들과의 담은 이뤄 가 없다

33) 정답[ ] ⑤

해설 근은 가난에도 하고 화가를 꿈꾸었다[ ] (3

단 또한 다른 화가 망생들은 정 육을)

위해 상 학 학 해 유학 에 랐 만

근은 다른 을 찾아야 했다 단 세에(5 ) 18

근은 조선 전람회에 입선하 다 단 의(6 )

만종은 인간과 자연이 엮어 가는 경건한 조화 을lsquo rsquo

나타낸다

34) 정답[ ] ①

해설 근이 속에서도 창작활동을 추 않고[ ]

하는 닭은 은 세상과 타협할 르는

근이 세상의 이해를 하 위한 가장 떳떳한 단

이 때 이다

35) 정답[ ] ⑤

해설 전 은 서 자의 주 적인 평이 리는 것이[ ]

만 위 제 은 인 이 살았던 대 사회적 경

을 통해 객 적인 인 의 을 제 하고 있다

36) 정답[ ] ⑤

해설 전 은 인 사건 경 비평이라는[ ] lsquo rsquo⑤

성 이 어져 있다

37) 정답[ ] ①

해설 이 은 동양인과 서양인의 사고 에 차이가[ ]

있다는 것을 대조를 통해 설 하고 있다 또 쓴이

의 제자가 축 경 를 러 가서 경험한 일화를

통해 동양인이 서양인에 비해 주 상황에 더 많은

주의를 인다는 주장을 뒷 침하고 있다

38) 정답[ ] ④

39) 정답[ ] ②

40) 정답[ ] ②

41) 정답[ ] ④

42) 정답[ ] ③

43) 정답[ ] ④

44) 정답 도서 의 휴 일 도서 의 이 간 도서의[ ]

해설 도서 장은 임의 정한 휴 일과 도서 이[ ]

간 도서의 상 등을 게 할 의 가 있다

년 학 간고사 대비2013 2 현대고 대비

ECN-0102-2013-001-000076193

45) 정답[ ] ①

해설 제 조의 정 휴 일 의 휴 일의 사전 게[ ] 3

는 도서 장의 의 조항에 속한다

46) 정답[ ] ①

해설 개인 정 호 의 를 제 하 했 만 항[ ]

나눠서 제 하 않고 대 나열하고 있다

47) 정답[ ] ②

해설 제 조의 내 을 회사는 다른 회사 협[ ] 7 lsquo

계약을 통해 서비 를 제공하는 경 회 의 아이디

등 개인 정 를 해당 회사에 전송할 있다는 내rsquo

이 있으 의 제점을 제 할 있다②

48) 정답[ ] ④

해설 는 도서 장의 의 에 해당하고 나 는 도[ ] ④

서 장의 리에 해당한다

49) 정답[ ] ③

50) 정답 은 음독으 적었고 은 훈독으 적었[ ] (1)

다 과 동일한 표 리 적은 것은 이고 (2) ce

과 동일한 표 리 적은 것은 이다ab

51) 정답[ ] ③

52) 정답[ ] ①②

53) 정답[ ] ③

54) 정답[ ] ③

55) 정답[ ] ①

56) 정답 른 죠코 어린 노 하니라[ ] A B

57) 정답 세 어에서는 활 형이 칙적으[ ] lsquo rsquoㄹㅇ

나타났 만 개화 어에서는 활 형이 쓰 다 lsquo rsquo ㄹㄴ

58) 정답 호 가 흔[ ] (1) (2)

59) 정답[ ] ④

60) 정답[ ] ③

Page 34: 현대고대비 국어 - chamsoriedu.com 「콘텐츠산업진흥 법」외 에도 저작권 의하여 ... 다른주체에게어떤동작을하도록만드는것을나타내는

년 학 간고사 대비2013 2 현대고 대비

ECN-0102-2013-001-000076193

⑤ 크 몽골 만주 공통어가 우리 어 같

계열에 다는 에 사 특 짐

가( )

善化公主主隱 공주님

他密只嫁良置古 몰 결 고

薯童房乙 맛

夜矣卯乙抱遣去如 에 몰 고 가다

( )

始汝 會隱日恚見隱扐 만 에 본

恥隱汝衣淸隱笑 맑 웃

고 시 여 공 크다 만 다[ ] ( ) ( ) ( ) ( )始 汝 會扐

내다 에 보다 견( ) ( )恚 見 다( )隱

럽다 맑다 청 웃( ) ( ) ( ) ( )恥 衣 淸 笑

zb50) 위의 나 를 함 고 음에 답하( ) lt gt

보lt gt

( )素那或云金川 白城郡蛇山人也

운 사산

는 고 다 는( )[ ( ) ] (素那 金川 白城

사산 사 다) ( ) 郡 蛇山

삼 사- lsquo rsquo 47

에 제 된 단어 의 표 리를 조건(1) lt gt ( ) lt gt

에 맞게 서 하

건lt gt

lsquo 었고 었다 태rsquo

에 제 된 단어 동일한 표 리에(2) lt gt ( )

의해 적은 것을 나 에서 찾아 조건 에 맞게 서 하( ) lt gt

건lt gt

에 당 는 각각( ) 개 쓸 것2 단

당 는 가 여러 개 어도 개만 쓸 것 각2

개 과 도 쪽에 개만2 2

드시 지 것( )

과 동 원리 것lsquo 고

과 동 원리 것 다rsquo

태 것

가( )

素那(或云金川) 白城郡蛇山人也

소나 또는 천 이라 한다 는 성 사( ) ( ) ( )素那 金川 白城郡〔 〕

산 사람이다 현대어 풀이( ) ( )蛇山

나( )

紫布岩乎希 회

執音乎手母牛放敎遣 자 손 암쇼 노히 고

吾 不喩慙 伊賜等肹 肹 나 안디 리샤

花 折叱肹 可獻乎理音如 고 것거 도림다

다 향찰은 리말을 리 으 적은 표 이었 만 생( )

은 고 대를 넘 하고 끊어 고 말았다 랜 세

동안 갈고 닦아 체계적이었던 향찰 표 이 사라졌

을 인은 크게 두 가 나누어 생각해 볼 있다

하나는 족 사회의 한 선호도에서 찾을 있다 라 때

향찰은 주 족 계 에서 사 했을 것으 인다 한 을

알 하고서는 한자를 활 하여 리말을 리 으 표

하 란 가능하 때 이다 런데 족들은 간이 흐

를 향찰과 같은 리 표 을 익혀 사 하 다는

아 한 을 대 사 하는 쪽을 선호하게 되었다 더 이

고 초에 인재 등 을 위해 과거제도가 행되 서 한 선

호도가 더 높아졌고 결 향찰은 소 되고 말았다

또 다른 가능성은 한 어의 특성에서 찾을 있다

터 한 과 일 세 나라는 한자 화 에 속해 다

당연한 이야 겠 만 표의 자인 한자는 어를 표 하

에 매 적절하다 어의 음절은 성 ( ) ( )聲母 韻母

이 어 고 여 에 성조가 추가되어 최종 소리가 결정된

다 래서 어는 단음절을 하나의 한자 표 하 된

다 에 초성 성 종성의 세 가 소가 하나의 음절

년 학 간고사 대비2013 2 현대고 대비

ECN-0102-2013-001-000076193

을 이 는 한 어는 음절 조가 잡하고 음절의 가 많아

서 한자 차 만으 한 어의 소리를 만족 럽게 표 할

없었다 를 들어 한 어에서는 어 니 같이 음절 lsquo rsquo

이 어 단어가 얼마든 있으나 어는( ) 複數音節

자 하나 나타내 만이다lsquo [m ]rsquo 母 ǔ

한편 일 어의 표 은 핵 적 단어는 한자 적고 토는

가나라는 일 의 자 적는 이다 적인 의 를 나

타내는 은 표의 자인 한자 적고 적 계를 나

타내는 토는 표음 자 적는 셈이니 자세히 살펴

리의 향찰 표 을 쏙 빼닮았음을 알 있다 한 어 같

은 착어이 서도 일 어에만 향찰과 유사한 표 이 살아

남은 것은 일 어의 특 때 이다 일 어는 하나의 자음과

음의 결합으 음절을 이 고 침이 거의 없는 음절 언어

이다 이러한 음절의 특색에다가 토가 달한 착어라는 점

이 향찰과 유사한 표 이 살아남을 있는 비결이었다

하 만 같은 착어라도 다양한 음소 침이 달한 한

어는 향찰 표 하는 데 근 적으 한계가 있었다

zb51) 다 하여 의 행에 대한 탐 한 결과( ) lt gt 2

않은 것은

보lt gt

善花公主主隱 공주니믄 공주님( )

----------------------------------------

-

他密只嫁良置古 그 지 얼어 고 몰 결(

----------------------------------------

-

薯童房乙 맛 맛( )

夜矣卯乙抱遺去如 몰 고 가다 에 몰 고(

가다)

주동 역 동- (薯童謠『 』

에 2 ( )他密只嫁良置古

얼다 시집가다 결 다 말 lsquo rsquo

① 실질 미 지니고 므 타 타lsquo ( )rsquo lsquo [ ]

② 에 실질 미 타내고 지 는lsquo rsquo lsquo [ ]rsquo lsquo [ ]密只 密 只

계 타내는

③ 얼어는 실질 미 포 고 므 가lsquo rsquo lsquo [ ]rsquo嫁

것lsquo [ ]rsquo 良

④ 고 어간 는 실질 미 지니고 므lsquo rsquo lsquo -rsquo

것lsquo [ ]rsquo 置

⑤ 고 어미 고는 계 타내고 므lsquo rsquo lsquo- rsquo

고 것lsquo [ ]rsquo 古

가( )

엉 훈 민middot middot middot middot middot世 宗 御 製 訓 民 正 音

말 미 듕 귁에 달middot middot middot middot middot middot middot middot中 國 文 字

니 런middot middot middot middot middot middot 어린middot middot middot middot百 姓

니 고 도 내 들middot middot middot middot middot middot middot middot middot 시러middot

펴 몯middot 미middot middot 니 내middot middot middot middot middot middot middot middot 爲

어엿middot 겨 새middot middot middot 믈여듧middot middot middot middot字 니middot middot middot

사 마다 니겨 킈 middot middot middot middot middot middot middot middot middot便 安

고 미니middot middot middot middot

본 는 상( ) (象

원리에 만들어진 본) ( )形 ㄱ ㄴ ㅁ ㅅ ㅇ

에 는 가 원리에( )加劃

그리고( )ㅋ ㄷ ㅌ ㅂ ㅍ ㅈ ㅊ ㆆ ㅎ

쓰는 병 원리에 만들어진( )竝書

마지막 체( ) ( )異體ㄲ ㄸ ㅃ ㅆ ㅉ ㆅ

ᅀ 다 상 원리에 ㅇ ㄹ

지 는 삼재 상 본 본( ) ( ) ( 天地人 三才

탕 므림과 림에 ) (初ㅡ ㅣ

재)( ) ( )( )出字 再出字ㅗ ㅏ ㅜ ㅓ ㅛ ㅑ ㅜ ㅕ

병 그리고 들 에 다시( )ㅘ ㅝ ㅣ

( )ㅣ ㅢ ㅚ ㅐ ㅟ ㅔ ㆉ ㅒ ㆌ ㅖ ㅙ ㅞ

zb52) 가 에 대한 설 으 르 않은 것을( ) 두 고르

① 어쓰 규 지키고 다

② 리 고 다

③ 말 미 미 등 어 사 다lsquo rsquo

④ 개 지 다

년 학 간고사 대비2013 2 현대고 대비

ECN-0102-2013-001-000076193

⑤ 어 원 에 가 도 고 다

엉 훈 민世 宗 御 製 訓 民 正 音

말 미 듕귁에 달 니

런 어린 니 고 도middot

내 들 시러 펴 몯 미 니middot

내 어엿 겨 새 믈여듧

사 마다 니겨middot 킈 고

미니

훈민 언 본- lsquo rsquo 5 (1459 )

zb53) 위의 에 대한 현대어 풀이가 르~ 않은 것

① 우리 말 과 달

② 어리 말 고 는 것 어도

③ 신 생각 마 껏 펼 는 사 많다

④ 게 생각 여

⑤ 사 마다 게

zb54) 훈민정음 언해 에는 한 을 창제한 동 가 드러나

있다 훈민정음 창제의 정 과 내 이 잘 연결된 것

① 주 신 말 미 듕귁에 달

② 민 신 내 어 겨

③ 신 뻔 킈 고 미니

④ 실 신 사 마다 니겨

⑤ 귀 신 계 주 는 훈민 신과 거리가

가 엉 훈 민( ) middot middot middot middot middot世 宗 御 製 訓 民 正 音 

말 미 귁에 中 國 달 文 字

니 런 어린 니 百 姓

고 도 내 들 시러 펴 몯

미 니 내 어엿 爲 겨 새

믈여듧 니 사 마다 니 字

겨 킈 고 미니 便 安

훈민 언 본- lsquo ( )rsquo ( ) 5 (1459 )訓民正音 世祖

( )

[ 1 ]

동 룡 샤 마다 복( ) ( ) ( )海東 六龍 天福

시니 고 동( ) ( )古聖 同符 시니

[ 2 ]

매 니 곶 여

미 므 니 그 내 러

가 니

[ 125 ]

우 미리( )千世 샨( )定 에( )漢水北 累仁

누 개 샤 복 업 시니( ) ( ) 開國 卜年

신( )聖神 니 샤도 경 근민 샤 욱( )敬天勤民

드시리 다

님 쇼 산 가( ) ( )洛水 山行

미드니 가

어 가- lsquo ( )rsquo 27龍飛御天歌

다 우리신 니쓰고 다만 만 쓰( )

거 샹 귀쳔 다보게 러 귀

여 쓴 도 신 보 가 고 신 에

말 어 보게 각 에 사 들

고 본 몬 능통 후에

죠 죠 니

드 도 만 공 에 사

드 미 죠 고 고 여 보 죠

보다 얼마가 거시 어신고 니 첫

가 죠 니 죠

민 들 어 신 샹

귀쳔 도보고 어보 가 만 늘

고 폐 에 만쓴 죠 민

도 러보지못 고 보니 그게 엇지

심 니 리 보 가 어 운건 다

니 쳣 말마 지 니 고 그

쓰 에 가 우 지 지

몰 거 본후에 가 어 지

고 그니 쓴편지 쟝 보

년 학 간고사 대비2013 2 현대고 대비

ECN-0102-2013-001-000076193

쓴것보다 듸 보고 그 마 니 쓴 고

어 못

그런고 에 리 과 가

만 쓴 못 민 말만 듯고

고 편 그 못 보니 그사 단

병신 못 다고 그사 식 사

니 만 고 다 과 그사

만 고 다 과 업 사 보다 식 고

죠 도 고 각 과

견 고 실 직 귀쳔 간에 그

고도 다 것 몰 귀죡 보다

사 우리 신 귀쳔 다 업

시 신 보고 과 지 게 랴

시니 샹 귀쳔 간에 우리 신 걸

간 보 새지각과 새 걸 미리

독립신- lsquo (1896)rsquo

zb55) 친 어 나의 제 장( ) 2 매 함축적

의 가 가장 유사한 것은

① 지 눈 내리고 매 득 니 내 여 가

사- lsquo rsquo

② 도 어 리듯 그 게 어 다

주 사- lsquo rsquo

③ 눈 살 다 죽 어 린 과 체 여

눈 새벽 지 도 살 다

눈- lsquo rsquo

④ 삶 근심과 고단 에 돌 거니는 여 거 는

여 리 내린 살가지 에 눈 리 눈 리

택 그 생 에- lsquo rsquo

⑤ 늘 러 고 러

청룡 룡 어 개 루 우

신경림 계- lsquo rsquo

zb56) 친 를 위 가 나 에 나타난A B ( ) ( )

세 어의 특 에 의거하여 세 어 표 하

그 산 고 공 도 맑지만

A

주변에 쓰 리는 어리 사 많다

B

건lt gt

식 가 에 타 어 특징에( ) ( )

거 과 어쓰 는 고 지 말 것

A

B

zb57) 가 의( ) 달 아ㆍ 다 의 ( ) 나셔에서 알 있는

세 어 개화 어의 특 을 비 하여 조건 에lt gt

맞게 서 하

건lt gt

어에 는lsquo 개

어에 는 다 태rsquo

zb58) 은 가 는 다 에 나 는 절lt 1gt ( ) lt 2gt ( )

일 를 췌한 것이다 의 의 가 lt 1gt (1)~(2)

유사한 말을 에서 찾아 쓰lt 2gt

보lt 1gt

런 (1) 어린 니 고百 姓

도 내 들 시러 펴 몯 미

사 마다 (2) 니겨 便 安

킈 고 미니

보lt 2gt

죠 고 고 여 보 죠

보다 얼마가 거시 어신고 니 첫 가

죠 니 죠 민

들 어 신 샹 귀쳔

도보고 어보 가 만 늘 고

폐 에 만쓴 죠 민 도

러보지못 고 보니 그게 엇지 심

니 리

년 학 간고사 대비2013 2 현대고 대비

ECN-0102-2013-001-000076193

lt 1 gt

동 룡 샤 마다 복 시( ) ( ) ( )海東 六龍 天福

고 동 시니( ) ( )古聖 同符

lt 2 gt

(A) 매 니 곶

여 니

미 므 니 그 내

러 가 니

lt125 gt

우 미리 샨 에( ) ( ) ( ) 千世 定 漢水北 累

누 개 샤 복 업 시 니( ) ( ) 仁開國 卜年 聖

신( ) 神 니 샤도 경 근민 샤( ) 敬天勤民

욱 드 시 리 다

님 쇼 산 가 ( ) ( )洛水 山行

미드니 가

- lt gt龍飛御天歌

zb59) 장과 내 상 유사한 성격의 조는125

① 뫼 고 고 고 고

어 그린 많고 많고 고 고

어 러 는 울고 울고 가느니

도 견- lt gt

② 강 에 드니 몸 다

그믈 고 가니

뒷 뫼 엄 언 니( )藥

-

③ 말 없는 청산 태 없는 다

값 없는 청 없는 월

에 병 없는 몸 별 없 늙 리

-

④ 가마귀 골에 가지 마

낸 가마귀 새

청강에 것 시 몸 러 가( ) 淸江

-

⑤ 진 골에( ) 白雪

가 매 는 어느 곳에 었는고

에 갈 곳 몰( ) 夕陽

색-

zb60) 위 에 나타난 세 어의 특 으 적절하 않은

것은

① 룡 어 주격 사에 당 는 가 사( ) lsquo rsquo六龍

고 다

② 샤 어에도 어 주체 쓰 다

는 것 다

③ 매 어 달리 사 택에 어

가 지 지지 고 다

④ 므 원 상 직 어 지 다

⑤ 드시리 다 주체 과 상 께 사

고 다

수고 하셨습니다hearts hearts

년 학 간고사 대비2013 2 현대고 대비

ECN-0102-2013-001-000076193

보닷컴에 공 는 별 보는 고등

들 여 주 는

들 습니다 슷 동 지

가 복 는 것 도가

니 복 여 습 시고 거 시

니다

정답 해설

1) 정답[ ] ④

해설 다른 것은 두 특정 업이나 단 내에서 사[ ]

하는 일종의 은어 사회 언에 해당한다 러나

는 언이 아니라 단과대학을 여서 단대 사lsquo rsquo lsquo rsquo lsquo④

대학을 여서 사대라고 한 말에 해당하 일rsquo lsquo rsquo

사회에서도 널리 쓰이 사회 언이라 할

없다

2) 정답[ ] ⑤

해설 사회 언은 같은 단 내에서 쓰이는 언어이[ ] lsquo rsquo

동일 단끼리는 단결 과 친 감을 형성하는

능을 하 리적 안감이 일어나 않는다

3) 정답[ ] ③

해설 사람이라는 차 적 표현에 대한 대안적 표현이[ ]lsquo rsquo

인 아내 처 등으 볼 있다lsquo rsquo

4) 정답[ ]⑤

해설 남성은 주 격 체를 사 한다[ ]

5) 정답[ ] ⑤

해설 흑인은 검다라는 뜻을 가 고 있을 뿐 인[ ]lsquo rsquo lsquo rsquo lsquo rsquo

다 열등한 뜻을 내포하 않는다

6) 정답 살 색 첫 작품[ ] - -

해설 살색 혹은 킨색은 한 인의 피 색을 뜻[ ] lsquo rsquo lsquo rsquo

하는 것으 인종 차 을 추 고 출 이주민

의 평등 을 침해할 있어 년 표 이2005

살 색으 이름을 꾸었다 처녀작은 처녀라lsquo rsquo lsquo rsquo lsquo rsquo

는 단어가 가 고 있는 곡된 성 인 을 한 것

으 첫 작품정도 꾸어 사 하는 것이 좋다lsquo rsquo

7) 정답[ ] ⑤

해설 호는 아들에게 해체를 사 하고 있다[ ] ① ②

장 을 성하는 청자는 자 의 아 느리 아lsquo

들 세 이다 호는 아 느리에게 해rsquo ③

체를 사 하고 있다 호가 느리 아 에게 ④

사 한 해 체 아들에게 사 한 해체는 두 비lsquo rsquo lsquo rsquo

격 체에 해당한다 호는 자 의 아랫사람인 ⑤

느리에게 아들과 마찬가 해체를 사 하는 것이

상 이 만 임 을 한 느리에게 고마 과 쁨

존 의 표 를 하 위해 자 의 아 에게 말하듯

해 체를 사 하고 있다

8) 정답[ ] ③

9) 정답[ ] ⑤

10) 정답[ ] ①

해설 청자 할아 가 장의 주체 아 다 높을[ ] ( ) ( )

경 에는 압존 에 의해 장의 주체를 높이 않는lsquo rsquo

다 러 아 서가 아닌 아 는으 계 lsquo rsquo lsquo rsquo lsquo

니다 가 아닌 있 니다 표현하는 것이 르rsquo lsquo rsquo

11) 정답 당이 당을 쫒았다 당이[ ]

당에 다

해설[ ]

12) 정답[ ] ⑤

해설 서 다른 높임표현을 통해 청자에 대해 리[ ] ⑤

적 거리감을 나타내는 인 은 이 아니라 현정이

다 가 에서 현정은 에게 해 체를 사 함으 써 ( )

친근감을 드러낸다 나 에서 연 을 게을리하는 역 ( )

도 들 때 에 화가 난 현정이 선생님에게 항의하

는 장 에서는 하 체를 사 하여 리적 거리lsquo rsquo

가 어졌음을 나타내고 있다

13) 정답[ ] ①

해설 는 는 얼 빛이 날과 어찌 다르 고[ ] lsquo rsquo

라는 뜻으 전과 달리 임이 화자를 않고

있음을 알 있다

14) 정답 달리 후 가 있다 이를 통해 경[ ] lt gt

쾌한 음악성을 형성하고 노 젓는 상황을 체적으

형상화하는 역할을 한다

15) 정답[ ] ①

16) 정답[ ] ⑤

해설 다 의 자연은 를 성찰하게 하는 대상[ ] ( )⑤

이자 정의 대상이다 의 자연은 자 의 상황과 ⑤

처 를 드러내는 경으 서의 역할을 하 이

이 없다

17) 정답[ ] ③

해설 는 빈천 을 해결하고자 했으나 강산[ ] lsquo ( )rsquo 貧賤③

과 풍 을 달라는 에 거절하 다고 함으 써 자

연에 대한 애정을 드러내고 있으 는 않는

임에 대한 망을 개에게 전가 켜서 임에 대한 리

을 드러내고 있다

18) 정답[ ] ③

년 학 간고사 대비2013 2 현대고 대비

ECN-0102-2013-001-000076193

19) 정답[ ] ⑤

해설 고상한 음악가의 이름을 리말 꽝 럽[ ]

게 꿈으 써 언어유희를 통해 음을 유 하고 있

다 이는 고상한 척하는 총 를 비꼼으 써 비판적

태도를 드러내는 것이 대상을 꽝 럽게 표현

하여 총 의 허 과 사치를 풍자하고 있다

20) 정답[ ] ⑤

해설 는 작품 속 경에 대한 설 이 드러나는 것이[ ]

서 자의 주 적인 견해가 접적으 드러나는 것이

아니다

21) 정답[ ] ⑤

22) 정답[ ] ②

23) 정답[ ] ④

24) 정답[ ] ①

해설 적강 티프는 주인공의 비 한 출생이나 능[ ] ①

과 이 있는 것으 조정의 능함을 풍자하는lsquo rsquo

것과는 거리가 다

25) 정답 픔 나[ ] ( )

해설 의 음악은 고통 는 사람들을 위 하고 아픔[ ] lsquo rsquo

을 치유해 주는 능을 한다고 할 있다 의 lt gt

픔 도 소 된 이 과 더 어 살아가는 따뜻한 마음lsquo rsquo

을 상 한다

26) 정답[ ] ⑤

해설 에게 선천적으 주어 각 장애라는 역경[ ]

은 의 이라는 가사 연 을 있다lsquo rsquo

27) 정답[ ] ④

해설 는 장 란 선 에게 은 개인적인 인상을[ ]

소녀 장정 등으 표현한 것이다lsquo rsquo

28) 정답[ ] ②

해설 담자가 피 담자의 언어적 표현이나 비언어[ ]②

적 표현 하 독자는 담의 위 나 피

담자의 감정 상태를 알 있다 이를 통해 독자는

담 상황을 더 생생하게 느낄 있고 피 담자

를 더 잘 이해할 있게 된다

29) 정답[ ]③

해설 일상생활과 역도 선 서의 성과에 된 것에서[ ]

역도를 하 서 겪는 어 과 내적 고민으 화제를

전화하 위한 것이다

30) 정답[ ] ①

해설 릿속에 새겨 넣듯 이 억되도 함 세상[ ] ② ③

살이가 힘들고 고생 러 속 하여 자유를 ④

가 없는 고통의 상태를 비유적으 이르는 말

적의 침입을 막 위해 쌓은 축 켜야 할⑤

대상을 비유적으 이르는 말이다

31) 정답[ ] ④

해설 이 의 종류는 전 으 인 사건 경[ ] lsquo

비평을 성 소 삼는다rsquo

32) 정답[ ] ④

해설 근은 삼대독자 태어났음을 에서 확인할[ ]

있다 형제들과의 담은 이뤄 가 없다

33) 정답[ ] ⑤

해설 근은 가난에도 하고 화가를 꿈꾸었다[ ] (3

단 또한 다른 화가 망생들은 정 육을)

위해 상 학 학 해 유학 에 랐 만

근은 다른 을 찾아야 했다 단 세에(5 ) 18

근은 조선 전람회에 입선하 다 단 의(6 )

만종은 인간과 자연이 엮어 가는 경건한 조화 을lsquo rsquo

나타낸다

34) 정답[ ] ①

해설 근이 속에서도 창작활동을 추 않고[ ]

하는 닭은 은 세상과 타협할 르는

근이 세상의 이해를 하 위한 가장 떳떳한 단

이 때 이다

35) 정답[ ] ⑤

해설 전 은 서 자의 주 적인 평이 리는 것이[ ]

만 위 제 은 인 이 살았던 대 사회적 경

을 통해 객 적인 인 의 을 제 하고 있다

36) 정답[ ] ⑤

해설 전 은 인 사건 경 비평이라는[ ] lsquo rsquo⑤

성 이 어져 있다

37) 정답[ ] ①

해설 이 은 동양인과 서양인의 사고 에 차이가[ ]

있다는 것을 대조를 통해 설 하고 있다 또 쓴이

의 제자가 축 경 를 러 가서 경험한 일화를

통해 동양인이 서양인에 비해 주 상황에 더 많은

주의를 인다는 주장을 뒷 침하고 있다

38) 정답[ ] ④

39) 정답[ ] ②

40) 정답[ ] ②

41) 정답[ ] ④

42) 정답[ ] ③

43) 정답[ ] ④

44) 정답 도서 의 휴 일 도서 의 이 간 도서의[ ]

해설 도서 장은 임의 정한 휴 일과 도서 이[ ]

간 도서의 상 등을 게 할 의 가 있다

년 학 간고사 대비2013 2 현대고 대비

ECN-0102-2013-001-000076193

45) 정답[ ] ①

해설 제 조의 정 휴 일 의 휴 일의 사전 게[ ] 3

는 도서 장의 의 조항에 속한다

46) 정답[ ] ①

해설 개인 정 호 의 를 제 하 했 만 항[ ]

나눠서 제 하 않고 대 나열하고 있다

47) 정답[ ] ②

해설 제 조의 내 을 회사는 다른 회사 협[ ] 7 lsquo

계약을 통해 서비 를 제공하는 경 회 의 아이디

등 개인 정 를 해당 회사에 전송할 있다는 내rsquo

이 있으 의 제점을 제 할 있다②

48) 정답[ ] ④

해설 는 도서 장의 의 에 해당하고 나 는 도[ ] ④

서 장의 리에 해당한다

49) 정답[ ] ③

50) 정답 은 음독으 적었고 은 훈독으 적었[ ] (1)

다 과 동일한 표 리 적은 것은 이고 (2) ce

과 동일한 표 리 적은 것은 이다ab

51) 정답[ ] ③

52) 정답[ ] ①②

53) 정답[ ] ③

54) 정답[ ] ③

55) 정답[ ] ①

56) 정답 른 죠코 어린 노 하니라[ ] A B

57) 정답 세 어에서는 활 형이 칙적으[ ] lsquo rsquoㄹㅇ

나타났 만 개화 어에서는 활 형이 쓰 다 lsquo rsquo ㄹㄴ

58) 정답 호 가 흔[ ] (1) (2)

59) 정답[ ] ④

60) 정답[ ] ③

Page 35: 현대고대비 국어 - chamsoriedu.com 「콘텐츠산업진흥 법」외 에도 저작권 의하여 ... 다른주체에게어떤동작을하도록만드는것을나타내는

년 학 간고사 대비2013 2 현대고 대비

ECN-0102-2013-001-000076193

을 이 는 한 어는 음절 조가 잡하고 음절의 가 많아

서 한자 차 만으 한 어의 소리를 만족 럽게 표 할

없었다 를 들어 한 어에서는 어 니 같이 음절 lsquo rsquo

이 어 단어가 얼마든 있으나 어는( ) 複數音節

자 하나 나타내 만이다lsquo [m ]rsquo 母 ǔ

한편 일 어의 표 은 핵 적 단어는 한자 적고 토는

가나라는 일 의 자 적는 이다 적인 의 를 나

타내는 은 표의 자인 한자 적고 적 계를 나

타내는 토는 표음 자 적는 셈이니 자세히 살펴

리의 향찰 표 을 쏙 빼닮았음을 알 있다 한 어 같

은 착어이 서도 일 어에만 향찰과 유사한 표 이 살아

남은 것은 일 어의 특 때 이다 일 어는 하나의 자음과

음의 결합으 음절을 이 고 침이 거의 없는 음절 언어

이다 이러한 음절의 특색에다가 토가 달한 착어라는 점

이 향찰과 유사한 표 이 살아남을 있는 비결이었다

하 만 같은 착어라도 다양한 음소 침이 달한 한

어는 향찰 표 하는 데 근 적으 한계가 있었다

zb51) 다 하여 의 행에 대한 탐 한 결과( ) lt gt 2

않은 것은

보lt gt

善花公主主隱 공주니믄 공주님( )

----------------------------------------

-

他密只嫁良置古 그 지 얼어 고 몰 결(

----------------------------------------

-

薯童房乙 맛 맛( )

夜矣卯乙抱遺去如 몰 고 가다 에 몰 고(

가다)

주동 역 동- (薯童謠『 』

에 2 ( )他密只嫁良置古

얼다 시집가다 결 다 말 lsquo rsquo

① 실질 미 지니고 므 타 타lsquo ( )rsquo lsquo [ ]

② 에 실질 미 타내고 지 는lsquo rsquo lsquo [ ]rsquo lsquo [ ]密只 密 只

계 타내는

③ 얼어는 실질 미 포 고 므 가lsquo rsquo lsquo [ ]rsquo嫁

것lsquo [ ]rsquo 良

④ 고 어간 는 실질 미 지니고 므lsquo rsquo lsquo -rsquo

것lsquo [ ]rsquo 置

⑤ 고 어미 고는 계 타내고 므lsquo rsquo lsquo- rsquo

고 것lsquo [ ]rsquo 古

가( )

엉 훈 민middot middot middot middot middot世 宗 御 製 訓 民 正 音

말 미 듕 귁에 달middot middot middot middot middot middot middot middot中 國 文 字

니 런middot middot middot middot middot middot 어린middot middot middot middot百 姓

니 고 도 내 들middot middot middot middot middot middot middot middot middot 시러middot

펴 몯middot 미middot middot 니 내middot middot middot middot middot middot middot middot 爲

어엿middot 겨 새middot middot middot 믈여듧middot middot middot middot字 니middot middot middot

사 마다 니겨 킈 middot middot middot middot middot middot middot middot middot便 安

고 미니middot middot middot middot

본 는 상( ) (象

원리에 만들어진 본) ( )形 ㄱ ㄴ ㅁ ㅅ ㅇ

에 는 가 원리에( )加劃

그리고( )ㅋ ㄷ ㅌ ㅂ ㅍ ㅈ ㅊ ㆆ ㅎ

쓰는 병 원리에 만들어진( )竝書

마지막 체( ) ( )異體ㄲ ㄸ ㅃ ㅆ ㅉ ㆅ

ᅀ 다 상 원리에 ㅇ ㄹ

지 는 삼재 상 본 본( ) ( ) ( 天地人 三才

탕 므림과 림에 ) (初ㅡ ㅣ

재)( ) ( )( )出字 再出字ㅗ ㅏ ㅜ ㅓ ㅛ ㅑ ㅜ ㅕ

병 그리고 들 에 다시( )ㅘ ㅝ ㅣ

( )ㅣ ㅢ ㅚ ㅐ ㅟ ㅔ ㆉ ㅒ ㆌ ㅖ ㅙ ㅞ

zb52) 가 에 대한 설 으 르 않은 것을( ) 두 고르

① 어쓰 규 지키고 다

② 리 고 다

③ 말 미 미 등 어 사 다lsquo rsquo

④ 개 지 다

년 학 간고사 대비2013 2 현대고 대비

ECN-0102-2013-001-000076193

⑤ 어 원 에 가 도 고 다

엉 훈 민世 宗 御 製 訓 民 正 音

말 미 듕귁에 달 니

런 어린 니 고 도middot

내 들 시러 펴 몯 미 니middot

내 어엿 겨 새 믈여듧

사 마다 니겨middot 킈 고

미니

훈민 언 본- lsquo rsquo 5 (1459 )

zb53) 위의 에 대한 현대어 풀이가 르~ 않은 것

① 우리 말 과 달

② 어리 말 고 는 것 어도

③ 신 생각 마 껏 펼 는 사 많다

④ 게 생각 여

⑤ 사 마다 게

zb54) 훈민정음 언해 에는 한 을 창제한 동 가 드러나

있다 훈민정음 창제의 정 과 내 이 잘 연결된 것

① 주 신 말 미 듕귁에 달

② 민 신 내 어 겨

③ 신 뻔 킈 고 미니

④ 실 신 사 마다 니겨

⑤ 귀 신 계 주 는 훈민 신과 거리가

가 엉 훈 민( ) middot middot middot middot middot世 宗 御 製 訓 民 正 音 

말 미 귁에 中 國 달 文 字

니 런 어린 니 百 姓

고 도 내 들 시러 펴 몯

미 니 내 어엿 爲 겨 새

믈여듧 니 사 마다 니 字

겨 킈 고 미니 便 安

훈민 언 본- lsquo ( )rsquo ( ) 5 (1459 )訓民正音 世祖

( )

[ 1 ]

동 룡 샤 마다 복( ) ( ) ( )海東 六龍 天福

시니 고 동( ) ( )古聖 同符 시니

[ 2 ]

매 니 곶 여

미 므 니 그 내 러

가 니

[ 125 ]

우 미리( )千世 샨( )定 에( )漢水北 累仁

누 개 샤 복 업 시니( ) ( ) 開國 卜年

신( )聖神 니 샤도 경 근민 샤 욱( )敬天勤民

드시리 다

님 쇼 산 가( ) ( )洛水 山行

미드니 가

어 가- lsquo ( )rsquo 27龍飛御天歌

다 우리신 니쓰고 다만 만 쓰( )

거 샹 귀쳔 다보게 러 귀

여 쓴 도 신 보 가 고 신 에

말 어 보게 각 에 사 들

고 본 몬 능통 후에

죠 죠 니

드 도 만 공 에 사

드 미 죠 고 고 여 보 죠

보다 얼마가 거시 어신고 니 첫

가 죠 니 죠

민 들 어 신 샹

귀쳔 도보고 어보 가 만 늘

고 폐 에 만쓴 죠 민

도 러보지못 고 보니 그게 엇지

심 니 리 보 가 어 운건 다

니 쳣 말마 지 니 고 그

쓰 에 가 우 지 지

몰 거 본후에 가 어 지

고 그니 쓴편지 쟝 보

년 학 간고사 대비2013 2 현대고 대비

ECN-0102-2013-001-000076193

쓴것보다 듸 보고 그 마 니 쓴 고

어 못

그런고 에 리 과 가

만 쓴 못 민 말만 듯고

고 편 그 못 보니 그사 단

병신 못 다고 그사 식 사

니 만 고 다 과 그사

만 고 다 과 업 사 보다 식 고

죠 도 고 각 과

견 고 실 직 귀쳔 간에 그

고도 다 것 몰 귀죡 보다

사 우리 신 귀쳔 다 업

시 신 보고 과 지 게 랴

시니 샹 귀쳔 간에 우리 신 걸

간 보 새지각과 새 걸 미리

독립신- lsquo (1896)rsquo

zb55) 친 어 나의 제 장( ) 2 매 함축적

의 가 가장 유사한 것은

① 지 눈 내리고 매 득 니 내 여 가

사- lsquo rsquo

② 도 어 리듯 그 게 어 다

주 사- lsquo rsquo

③ 눈 살 다 죽 어 린 과 체 여

눈 새벽 지 도 살 다

눈- lsquo rsquo

④ 삶 근심과 고단 에 돌 거니는 여 거 는

여 리 내린 살가지 에 눈 리 눈 리

택 그 생 에- lsquo rsquo

⑤ 늘 러 고 러

청룡 룡 어 개 루 우

신경림 계- lsquo rsquo

zb56) 친 를 위 가 나 에 나타난A B ( ) ( )

세 어의 특 에 의거하여 세 어 표 하

그 산 고 공 도 맑지만

A

주변에 쓰 리는 어리 사 많다

B

건lt gt

식 가 에 타 어 특징에( ) ( )

거 과 어쓰 는 고 지 말 것

A

B

zb57) 가 의( ) 달 아ㆍ 다 의 ( ) 나셔에서 알 있는

세 어 개화 어의 특 을 비 하여 조건 에lt gt

맞게 서 하

건lt gt

어에 는lsquo 개

어에 는 다 태rsquo

zb58) 은 가 는 다 에 나 는 절lt 1gt ( ) lt 2gt ( )

일 를 췌한 것이다 의 의 가 lt 1gt (1)~(2)

유사한 말을 에서 찾아 쓰lt 2gt

보lt 1gt

런 (1) 어린 니 고百 姓

도 내 들 시러 펴 몯 미

사 마다 (2) 니겨 便 安

킈 고 미니

보lt 2gt

죠 고 고 여 보 죠

보다 얼마가 거시 어신고 니 첫 가

죠 니 죠 민

들 어 신 샹 귀쳔

도보고 어보 가 만 늘 고

폐 에 만쓴 죠 민 도

러보지못 고 보니 그게 엇지 심

니 리

년 학 간고사 대비2013 2 현대고 대비

ECN-0102-2013-001-000076193

lt 1 gt

동 룡 샤 마다 복 시( ) ( ) ( )海東 六龍 天福

고 동 시니( ) ( )古聖 同符

lt 2 gt

(A) 매 니 곶

여 니

미 므 니 그 내

러 가 니

lt125 gt

우 미리 샨 에( ) ( ) ( ) 千世 定 漢水北 累

누 개 샤 복 업 시 니( ) ( ) 仁開國 卜年 聖

신( ) 神 니 샤도 경 근민 샤( ) 敬天勤民

욱 드 시 리 다

님 쇼 산 가 ( ) ( )洛水 山行

미드니 가

- lt gt龍飛御天歌

zb59) 장과 내 상 유사한 성격의 조는125

① 뫼 고 고 고 고

어 그린 많고 많고 고 고

어 러 는 울고 울고 가느니

도 견- lt gt

② 강 에 드니 몸 다

그믈 고 가니

뒷 뫼 엄 언 니( )藥

-

③ 말 없는 청산 태 없는 다

값 없는 청 없는 월

에 병 없는 몸 별 없 늙 리

-

④ 가마귀 골에 가지 마

낸 가마귀 새

청강에 것 시 몸 러 가( ) 淸江

-

⑤ 진 골에( ) 白雪

가 매 는 어느 곳에 었는고

에 갈 곳 몰( ) 夕陽

색-

zb60) 위 에 나타난 세 어의 특 으 적절하 않은

것은

① 룡 어 주격 사에 당 는 가 사( ) lsquo rsquo六龍

고 다

② 샤 어에도 어 주체 쓰 다

는 것 다

③ 매 어 달리 사 택에 어

가 지 지지 고 다

④ 므 원 상 직 어 지 다

⑤ 드시리 다 주체 과 상 께 사

고 다

수고 하셨습니다hearts hearts

년 학 간고사 대비2013 2 현대고 대비

ECN-0102-2013-001-000076193

보닷컴에 공 는 별 보는 고등

들 여 주 는

들 습니다 슷 동 지

가 복 는 것 도가

니 복 여 습 시고 거 시

니다

정답 해설

1) 정답[ ] ④

해설 다른 것은 두 특정 업이나 단 내에서 사[ ]

하는 일종의 은어 사회 언에 해당한다 러나

는 언이 아니라 단과대학을 여서 단대 사lsquo rsquo lsquo rsquo lsquo④

대학을 여서 사대라고 한 말에 해당하 일rsquo lsquo rsquo

사회에서도 널리 쓰이 사회 언이라 할

없다

2) 정답[ ] ⑤

해설 사회 언은 같은 단 내에서 쓰이는 언어이[ ] lsquo rsquo

동일 단끼리는 단결 과 친 감을 형성하는

능을 하 리적 안감이 일어나 않는다

3) 정답[ ] ③

해설 사람이라는 차 적 표현에 대한 대안적 표현이[ ]lsquo rsquo

인 아내 처 등으 볼 있다lsquo rsquo

4) 정답[ ]⑤

해설 남성은 주 격 체를 사 한다[ ]

5) 정답[ ] ⑤

해설 흑인은 검다라는 뜻을 가 고 있을 뿐 인[ ]lsquo rsquo lsquo rsquo lsquo rsquo

다 열등한 뜻을 내포하 않는다

6) 정답 살 색 첫 작품[ ] - -

해설 살색 혹은 킨색은 한 인의 피 색을 뜻[ ] lsquo rsquo lsquo rsquo

하는 것으 인종 차 을 추 고 출 이주민

의 평등 을 침해할 있어 년 표 이2005

살 색으 이름을 꾸었다 처녀작은 처녀라lsquo rsquo lsquo rsquo lsquo rsquo

는 단어가 가 고 있는 곡된 성 인 을 한 것

으 첫 작품정도 꾸어 사 하는 것이 좋다lsquo rsquo

7) 정답[ ] ⑤

해설 호는 아들에게 해체를 사 하고 있다[ ] ① ②

장 을 성하는 청자는 자 의 아 느리 아lsquo

들 세 이다 호는 아 느리에게 해rsquo ③

체를 사 하고 있다 호가 느리 아 에게 ④

사 한 해 체 아들에게 사 한 해체는 두 비lsquo rsquo lsquo rsquo

격 체에 해당한다 호는 자 의 아랫사람인 ⑤

느리에게 아들과 마찬가 해체를 사 하는 것이

상 이 만 임 을 한 느리에게 고마 과 쁨

존 의 표 를 하 위해 자 의 아 에게 말하듯

해 체를 사 하고 있다

8) 정답[ ] ③

9) 정답[ ] ⑤

10) 정답[ ] ①

해설 청자 할아 가 장의 주체 아 다 높을[ ] ( ) ( )

경 에는 압존 에 의해 장의 주체를 높이 않는lsquo rsquo

다 러 아 서가 아닌 아 는으 계 lsquo rsquo lsquo rsquo lsquo

니다 가 아닌 있 니다 표현하는 것이 르rsquo lsquo rsquo

11) 정답 당이 당을 쫒았다 당이[ ]

당에 다

해설[ ]

12) 정답[ ] ⑤

해설 서 다른 높임표현을 통해 청자에 대해 리[ ] ⑤

적 거리감을 나타내는 인 은 이 아니라 현정이

다 가 에서 현정은 에게 해 체를 사 함으 써 ( )

친근감을 드러낸다 나 에서 연 을 게을리하는 역 ( )

도 들 때 에 화가 난 현정이 선생님에게 항의하

는 장 에서는 하 체를 사 하여 리적 거리lsquo rsquo

가 어졌음을 나타내고 있다

13) 정답[ ] ①

해설 는 는 얼 빛이 날과 어찌 다르 고[ ] lsquo rsquo

라는 뜻으 전과 달리 임이 화자를 않고

있음을 알 있다

14) 정답 달리 후 가 있다 이를 통해 경[ ] lt gt

쾌한 음악성을 형성하고 노 젓는 상황을 체적으

형상화하는 역할을 한다

15) 정답[ ] ①

16) 정답[ ] ⑤

해설 다 의 자연은 를 성찰하게 하는 대상[ ] ( )⑤

이자 정의 대상이다 의 자연은 자 의 상황과 ⑤

처 를 드러내는 경으 서의 역할을 하 이

이 없다

17) 정답[ ] ③

해설 는 빈천 을 해결하고자 했으나 강산[ ] lsquo ( )rsquo 貧賤③

과 풍 을 달라는 에 거절하 다고 함으 써 자

연에 대한 애정을 드러내고 있으 는 않는

임에 대한 망을 개에게 전가 켜서 임에 대한 리

을 드러내고 있다

18) 정답[ ] ③

년 학 간고사 대비2013 2 현대고 대비

ECN-0102-2013-001-000076193

19) 정답[ ] ⑤

해설 고상한 음악가의 이름을 리말 꽝 럽[ ]

게 꿈으 써 언어유희를 통해 음을 유 하고 있

다 이는 고상한 척하는 총 를 비꼼으 써 비판적

태도를 드러내는 것이 대상을 꽝 럽게 표현

하여 총 의 허 과 사치를 풍자하고 있다

20) 정답[ ] ⑤

해설 는 작품 속 경에 대한 설 이 드러나는 것이[ ]

서 자의 주 적인 견해가 접적으 드러나는 것이

아니다

21) 정답[ ] ⑤

22) 정답[ ] ②

23) 정답[ ] ④

24) 정답[ ] ①

해설 적강 티프는 주인공의 비 한 출생이나 능[ ] ①

과 이 있는 것으 조정의 능함을 풍자하는lsquo rsquo

것과는 거리가 다

25) 정답 픔 나[ ] ( )

해설 의 음악은 고통 는 사람들을 위 하고 아픔[ ] lsquo rsquo

을 치유해 주는 능을 한다고 할 있다 의 lt gt

픔 도 소 된 이 과 더 어 살아가는 따뜻한 마음lsquo rsquo

을 상 한다

26) 정답[ ] ⑤

해설 에게 선천적으 주어 각 장애라는 역경[ ]

은 의 이라는 가사 연 을 있다lsquo rsquo

27) 정답[ ] ④

해설 는 장 란 선 에게 은 개인적인 인상을[ ]

소녀 장정 등으 표현한 것이다lsquo rsquo

28) 정답[ ] ②

해설 담자가 피 담자의 언어적 표현이나 비언어[ ]②

적 표현 하 독자는 담의 위 나 피

담자의 감정 상태를 알 있다 이를 통해 독자는

담 상황을 더 생생하게 느낄 있고 피 담자

를 더 잘 이해할 있게 된다

29) 정답[ ]③

해설 일상생활과 역도 선 서의 성과에 된 것에서[ ]

역도를 하 서 겪는 어 과 내적 고민으 화제를

전화하 위한 것이다

30) 정답[ ] ①

해설 릿속에 새겨 넣듯 이 억되도 함 세상[ ] ② ③

살이가 힘들고 고생 러 속 하여 자유를 ④

가 없는 고통의 상태를 비유적으 이르는 말

적의 침입을 막 위해 쌓은 축 켜야 할⑤

대상을 비유적으 이르는 말이다

31) 정답[ ] ④

해설 이 의 종류는 전 으 인 사건 경[ ] lsquo

비평을 성 소 삼는다rsquo

32) 정답[ ] ④

해설 근은 삼대독자 태어났음을 에서 확인할[ ]

있다 형제들과의 담은 이뤄 가 없다

33) 정답[ ] ⑤

해설 근은 가난에도 하고 화가를 꿈꾸었다[ ] (3

단 또한 다른 화가 망생들은 정 육을)

위해 상 학 학 해 유학 에 랐 만

근은 다른 을 찾아야 했다 단 세에(5 ) 18

근은 조선 전람회에 입선하 다 단 의(6 )

만종은 인간과 자연이 엮어 가는 경건한 조화 을lsquo rsquo

나타낸다

34) 정답[ ] ①

해설 근이 속에서도 창작활동을 추 않고[ ]

하는 닭은 은 세상과 타협할 르는

근이 세상의 이해를 하 위한 가장 떳떳한 단

이 때 이다

35) 정답[ ] ⑤

해설 전 은 서 자의 주 적인 평이 리는 것이[ ]

만 위 제 은 인 이 살았던 대 사회적 경

을 통해 객 적인 인 의 을 제 하고 있다

36) 정답[ ] ⑤

해설 전 은 인 사건 경 비평이라는[ ] lsquo rsquo⑤

성 이 어져 있다

37) 정답[ ] ①

해설 이 은 동양인과 서양인의 사고 에 차이가[ ]

있다는 것을 대조를 통해 설 하고 있다 또 쓴이

의 제자가 축 경 를 러 가서 경험한 일화를

통해 동양인이 서양인에 비해 주 상황에 더 많은

주의를 인다는 주장을 뒷 침하고 있다

38) 정답[ ] ④

39) 정답[ ] ②

40) 정답[ ] ②

41) 정답[ ] ④

42) 정답[ ] ③

43) 정답[ ] ④

44) 정답 도서 의 휴 일 도서 의 이 간 도서의[ ]

해설 도서 장은 임의 정한 휴 일과 도서 이[ ]

간 도서의 상 등을 게 할 의 가 있다

년 학 간고사 대비2013 2 현대고 대비

ECN-0102-2013-001-000076193

45) 정답[ ] ①

해설 제 조의 정 휴 일 의 휴 일의 사전 게[ ] 3

는 도서 장의 의 조항에 속한다

46) 정답[ ] ①

해설 개인 정 호 의 를 제 하 했 만 항[ ]

나눠서 제 하 않고 대 나열하고 있다

47) 정답[ ] ②

해설 제 조의 내 을 회사는 다른 회사 협[ ] 7 lsquo

계약을 통해 서비 를 제공하는 경 회 의 아이디

등 개인 정 를 해당 회사에 전송할 있다는 내rsquo

이 있으 의 제점을 제 할 있다②

48) 정답[ ] ④

해설 는 도서 장의 의 에 해당하고 나 는 도[ ] ④

서 장의 리에 해당한다

49) 정답[ ] ③

50) 정답 은 음독으 적었고 은 훈독으 적었[ ] (1)

다 과 동일한 표 리 적은 것은 이고 (2) ce

과 동일한 표 리 적은 것은 이다ab

51) 정답[ ] ③

52) 정답[ ] ①②

53) 정답[ ] ③

54) 정답[ ] ③

55) 정답[ ] ①

56) 정답 른 죠코 어린 노 하니라[ ] A B

57) 정답 세 어에서는 활 형이 칙적으[ ] lsquo rsquoㄹㅇ

나타났 만 개화 어에서는 활 형이 쓰 다 lsquo rsquo ㄹㄴ

58) 정답 호 가 흔[ ] (1) (2)

59) 정답[ ] ④

60) 정답[ ] ③

Page 36: 현대고대비 국어 - chamsoriedu.com 「콘텐츠산업진흥 법」외 에도 저작권 의하여 ... 다른주체에게어떤동작을하도록만드는것을나타내는

년 학 간고사 대비2013 2 현대고 대비

ECN-0102-2013-001-000076193

⑤ 어 원 에 가 도 고 다

엉 훈 민世 宗 御 製 訓 民 正 音

말 미 듕귁에 달 니

런 어린 니 고 도middot

내 들 시러 펴 몯 미 니middot

내 어엿 겨 새 믈여듧

사 마다 니겨middot 킈 고

미니

훈민 언 본- lsquo rsquo 5 (1459 )

zb53) 위의 에 대한 현대어 풀이가 르~ 않은 것

① 우리 말 과 달

② 어리 말 고 는 것 어도

③ 신 생각 마 껏 펼 는 사 많다

④ 게 생각 여

⑤ 사 마다 게

zb54) 훈민정음 언해 에는 한 을 창제한 동 가 드러나

있다 훈민정음 창제의 정 과 내 이 잘 연결된 것

① 주 신 말 미 듕귁에 달

② 민 신 내 어 겨

③ 신 뻔 킈 고 미니

④ 실 신 사 마다 니겨

⑤ 귀 신 계 주 는 훈민 신과 거리가

가 엉 훈 민( ) middot middot middot middot middot世 宗 御 製 訓 民 正 音 

말 미 귁에 中 國 달 文 字

니 런 어린 니 百 姓

고 도 내 들 시러 펴 몯

미 니 내 어엿 爲 겨 새

믈여듧 니 사 마다 니 字

겨 킈 고 미니 便 安

훈민 언 본- lsquo ( )rsquo ( ) 5 (1459 )訓民正音 世祖

( )

[ 1 ]

동 룡 샤 마다 복( ) ( ) ( )海東 六龍 天福

시니 고 동( ) ( )古聖 同符 시니

[ 2 ]

매 니 곶 여

미 므 니 그 내 러

가 니

[ 125 ]

우 미리( )千世 샨( )定 에( )漢水北 累仁

누 개 샤 복 업 시니( ) ( ) 開國 卜年

신( )聖神 니 샤도 경 근민 샤 욱( )敬天勤民

드시리 다

님 쇼 산 가( ) ( )洛水 山行

미드니 가

어 가- lsquo ( )rsquo 27龍飛御天歌

다 우리신 니쓰고 다만 만 쓰( )

거 샹 귀쳔 다보게 러 귀

여 쓴 도 신 보 가 고 신 에

말 어 보게 각 에 사 들

고 본 몬 능통 후에

죠 죠 니

드 도 만 공 에 사

드 미 죠 고 고 여 보 죠

보다 얼마가 거시 어신고 니 첫

가 죠 니 죠

민 들 어 신 샹

귀쳔 도보고 어보 가 만 늘

고 폐 에 만쓴 죠 민

도 러보지못 고 보니 그게 엇지

심 니 리 보 가 어 운건 다

니 쳣 말마 지 니 고 그

쓰 에 가 우 지 지

몰 거 본후에 가 어 지

고 그니 쓴편지 쟝 보

년 학 간고사 대비2013 2 현대고 대비

ECN-0102-2013-001-000076193

쓴것보다 듸 보고 그 마 니 쓴 고

어 못

그런고 에 리 과 가

만 쓴 못 민 말만 듯고

고 편 그 못 보니 그사 단

병신 못 다고 그사 식 사

니 만 고 다 과 그사

만 고 다 과 업 사 보다 식 고

죠 도 고 각 과

견 고 실 직 귀쳔 간에 그

고도 다 것 몰 귀죡 보다

사 우리 신 귀쳔 다 업

시 신 보고 과 지 게 랴

시니 샹 귀쳔 간에 우리 신 걸

간 보 새지각과 새 걸 미리

독립신- lsquo (1896)rsquo

zb55) 친 어 나의 제 장( ) 2 매 함축적

의 가 가장 유사한 것은

① 지 눈 내리고 매 득 니 내 여 가

사- lsquo rsquo

② 도 어 리듯 그 게 어 다

주 사- lsquo rsquo

③ 눈 살 다 죽 어 린 과 체 여

눈 새벽 지 도 살 다

눈- lsquo rsquo

④ 삶 근심과 고단 에 돌 거니는 여 거 는

여 리 내린 살가지 에 눈 리 눈 리

택 그 생 에- lsquo rsquo

⑤ 늘 러 고 러

청룡 룡 어 개 루 우

신경림 계- lsquo rsquo

zb56) 친 를 위 가 나 에 나타난A B ( ) ( )

세 어의 특 에 의거하여 세 어 표 하

그 산 고 공 도 맑지만

A

주변에 쓰 리는 어리 사 많다

B

건lt gt

식 가 에 타 어 특징에( ) ( )

거 과 어쓰 는 고 지 말 것

A

B

zb57) 가 의( ) 달 아ㆍ 다 의 ( ) 나셔에서 알 있는

세 어 개화 어의 특 을 비 하여 조건 에lt gt

맞게 서 하

건lt gt

어에 는lsquo 개

어에 는 다 태rsquo

zb58) 은 가 는 다 에 나 는 절lt 1gt ( ) lt 2gt ( )

일 를 췌한 것이다 의 의 가 lt 1gt (1)~(2)

유사한 말을 에서 찾아 쓰lt 2gt

보lt 1gt

런 (1) 어린 니 고百 姓

도 내 들 시러 펴 몯 미

사 마다 (2) 니겨 便 安

킈 고 미니

보lt 2gt

죠 고 고 여 보 죠

보다 얼마가 거시 어신고 니 첫 가

죠 니 죠 민

들 어 신 샹 귀쳔

도보고 어보 가 만 늘 고

폐 에 만쓴 죠 민 도

러보지못 고 보니 그게 엇지 심

니 리

년 학 간고사 대비2013 2 현대고 대비

ECN-0102-2013-001-000076193

lt 1 gt

동 룡 샤 마다 복 시( ) ( ) ( )海東 六龍 天福

고 동 시니( ) ( )古聖 同符

lt 2 gt

(A) 매 니 곶

여 니

미 므 니 그 내

러 가 니

lt125 gt

우 미리 샨 에( ) ( ) ( ) 千世 定 漢水北 累

누 개 샤 복 업 시 니( ) ( ) 仁開國 卜年 聖

신( ) 神 니 샤도 경 근민 샤( ) 敬天勤民

욱 드 시 리 다

님 쇼 산 가 ( ) ( )洛水 山行

미드니 가

- lt gt龍飛御天歌

zb59) 장과 내 상 유사한 성격의 조는125

① 뫼 고 고 고 고

어 그린 많고 많고 고 고

어 러 는 울고 울고 가느니

도 견- lt gt

② 강 에 드니 몸 다

그믈 고 가니

뒷 뫼 엄 언 니( )藥

-

③ 말 없는 청산 태 없는 다

값 없는 청 없는 월

에 병 없는 몸 별 없 늙 리

-

④ 가마귀 골에 가지 마

낸 가마귀 새

청강에 것 시 몸 러 가( ) 淸江

-

⑤ 진 골에( ) 白雪

가 매 는 어느 곳에 었는고

에 갈 곳 몰( ) 夕陽

색-

zb60) 위 에 나타난 세 어의 특 으 적절하 않은

것은

① 룡 어 주격 사에 당 는 가 사( ) lsquo rsquo六龍

고 다

② 샤 어에도 어 주체 쓰 다

는 것 다

③ 매 어 달리 사 택에 어

가 지 지지 고 다

④ 므 원 상 직 어 지 다

⑤ 드시리 다 주체 과 상 께 사

고 다

수고 하셨습니다hearts hearts

년 학 간고사 대비2013 2 현대고 대비

ECN-0102-2013-001-000076193

보닷컴에 공 는 별 보는 고등

들 여 주 는

들 습니다 슷 동 지

가 복 는 것 도가

니 복 여 습 시고 거 시

니다

정답 해설

1) 정답[ ] ④

해설 다른 것은 두 특정 업이나 단 내에서 사[ ]

하는 일종의 은어 사회 언에 해당한다 러나

는 언이 아니라 단과대학을 여서 단대 사lsquo rsquo lsquo rsquo lsquo④

대학을 여서 사대라고 한 말에 해당하 일rsquo lsquo rsquo

사회에서도 널리 쓰이 사회 언이라 할

없다

2) 정답[ ] ⑤

해설 사회 언은 같은 단 내에서 쓰이는 언어이[ ] lsquo rsquo

동일 단끼리는 단결 과 친 감을 형성하는

능을 하 리적 안감이 일어나 않는다

3) 정답[ ] ③

해설 사람이라는 차 적 표현에 대한 대안적 표현이[ ]lsquo rsquo

인 아내 처 등으 볼 있다lsquo rsquo

4) 정답[ ]⑤

해설 남성은 주 격 체를 사 한다[ ]

5) 정답[ ] ⑤

해설 흑인은 검다라는 뜻을 가 고 있을 뿐 인[ ]lsquo rsquo lsquo rsquo lsquo rsquo

다 열등한 뜻을 내포하 않는다

6) 정답 살 색 첫 작품[ ] - -

해설 살색 혹은 킨색은 한 인의 피 색을 뜻[ ] lsquo rsquo lsquo rsquo

하는 것으 인종 차 을 추 고 출 이주민

의 평등 을 침해할 있어 년 표 이2005

살 색으 이름을 꾸었다 처녀작은 처녀라lsquo rsquo lsquo rsquo lsquo rsquo

는 단어가 가 고 있는 곡된 성 인 을 한 것

으 첫 작품정도 꾸어 사 하는 것이 좋다lsquo rsquo

7) 정답[ ] ⑤

해설 호는 아들에게 해체를 사 하고 있다[ ] ① ②

장 을 성하는 청자는 자 의 아 느리 아lsquo

들 세 이다 호는 아 느리에게 해rsquo ③

체를 사 하고 있다 호가 느리 아 에게 ④

사 한 해 체 아들에게 사 한 해체는 두 비lsquo rsquo lsquo rsquo

격 체에 해당한다 호는 자 의 아랫사람인 ⑤

느리에게 아들과 마찬가 해체를 사 하는 것이

상 이 만 임 을 한 느리에게 고마 과 쁨

존 의 표 를 하 위해 자 의 아 에게 말하듯

해 체를 사 하고 있다

8) 정답[ ] ③

9) 정답[ ] ⑤

10) 정답[ ] ①

해설 청자 할아 가 장의 주체 아 다 높을[ ] ( ) ( )

경 에는 압존 에 의해 장의 주체를 높이 않는lsquo rsquo

다 러 아 서가 아닌 아 는으 계 lsquo rsquo lsquo rsquo lsquo

니다 가 아닌 있 니다 표현하는 것이 르rsquo lsquo rsquo

11) 정답 당이 당을 쫒았다 당이[ ]

당에 다

해설[ ]

12) 정답[ ] ⑤

해설 서 다른 높임표현을 통해 청자에 대해 리[ ] ⑤

적 거리감을 나타내는 인 은 이 아니라 현정이

다 가 에서 현정은 에게 해 체를 사 함으 써 ( )

친근감을 드러낸다 나 에서 연 을 게을리하는 역 ( )

도 들 때 에 화가 난 현정이 선생님에게 항의하

는 장 에서는 하 체를 사 하여 리적 거리lsquo rsquo

가 어졌음을 나타내고 있다

13) 정답[ ] ①

해설 는 는 얼 빛이 날과 어찌 다르 고[ ] lsquo rsquo

라는 뜻으 전과 달리 임이 화자를 않고

있음을 알 있다

14) 정답 달리 후 가 있다 이를 통해 경[ ] lt gt

쾌한 음악성을 형성하고 노 젓는 상황을 체적으

형상화하는 역할을 한다

15) 정답[ ] ①

16) 정답[ ] ⑤

해설 다 의 자연은 를 성찰하게 하는 대상[ ] ( )⑤

이자 정의 대상이다 의 자연은 자 의 상황과 ⑤

처 를 드러내는 경으 서의 역할을 하 이

이 없다

17) 정답[ ] ③

해설 는 빈천 을 해결하고자 했으나 강산[ ] lsquo ( )rsquo 貧賤③

과 풍 을 달라는 에 거절하 다고 함으 써 자

연에 대한 애정을 드러내고 있으 는 않는

임에 대한 망을 개에게 전가 켜서 임에 대한 리

을 드러내고 있다

18) 정답[ ] ③

년 학 간고사 대비2013 2 현대고 대비

ECN-0102-2013-001-000076193

19) 정답[ ] ⑤

해설 고상한 음악가의 이름을 리말 꽝 럽[ ]

게 꿈으 써 언어유희를 통해 음을 유 하고 있

다 이는 고상한 척하는 총 를 비꼼으 써 비판적

태도를 드러내는 것이 대상을 꽝 럽게 표현

하여 총 의 허 과 사치를 풍자하고 있다

20) 정답[ ] ⑤

해설 는 작품 속 경에 대한 설 이 드러나는 것이[ ]

서 자의 주 적인 견해가 접적으 드러나는 것이

아니다

21) 정답[ ] ⑤

22) 정답[ ] ②

23) 정답[ ] ④

24) 정답[ ] ①

해설 적강 티프는 주인공의 비 한 출생이나 능[ ] ①

과 이 있는 것으 조정의 능함을 풍자하는lsquo rsquo

것과는 거리가 다

25) 정답 픔 나[ ] ( )

해설 의 음악은 고통 는 사람들을 위 하고 아픔[ ] lsquo rsquo

을 치유해 주는 능을 한다고 할 있다 의 lt gt

픔 도 소 된 이 과 더 어 살아가는 따뜻한 마음lsquo rsquo

을 상 한다

26) 정답[ ] ⑤

해설 에게 선천적으 주어 각 장애라는 역경[ ]

은 의 이라는 가사 연 을 있다lsquo rsquo

27) 정답[ ] ④

해설 는 장 란 선 에게 은 개인적인 인상을[ ]

소녀 장정 등으 표현한 것이다lsquo rsquo

28) 정답[ ] ②

해설 담자가 피 담자의 언어적 표현이나 비언어[ ]②

적 표현 하 독자는 담의 위 나 피

담자의 감정 상태를 알 있다 이를 통해 독자는

담 상황을 더 생생하게 느낄 있고 피 담자

를 더 잘 이해할 있게 된다

29) 정답[ ]③

해설 일상생활과 역도 선 서의 성과에 된 것에서[ ]

역도를 하 서 겪는 어 과 내적 고민으 화제를

전화하 위한 것이다

30) 정답[ ] ①

해설 릿속에 새겨 넣듯 이 억되도 함 세상[ ] ② ③

살이가 힘들고 고생 러 속 하여 자유를 ④

가 없는 고통의 상태를 비유적으 이르는 말

적의 침입을 막 위해 쌓은 축 켜야 할⑤

대상을 비유적으 이르는 말이다

31) 정답[ ] ④

해설 이 의 종류는 전 으 인 사건 경[ ] lsquo

비평을 성 소 삼는다rsquo

32) 정답[ ] ④

해설 근은 삼대독자 태어났음을 에서 확인할[ ]

있다 형제들과의 담은 이뤄 가 없다

33) 정답[ ] ⑤

해설 근은 가난에도 하고 화가를 꿈꾸었다[ ] (3

단 또한 다른 화가 망생들은 정 육을)

위해 상 학 학 해 유학 에 랐 만

근은 다른 을 찾아야 했다 단 세에(5 ) 18

근은 조선 전람회에 입선하 다 단 의(6 )

만종은 인간과 자연이 엮어 가는 경건한 조화 을lsquo rsquo

나타낸다

34) 정답[ ] ①

해설 근이 속에서도 창작활동을 추 않고[ ]

하는 닭은 은 세상과 타협할 르는

근이 세상의 이해를 하 위한 가장 떳떳한 단

이 때 이다

35) 정답[ ] ⑤

해설 전 은 서 자의 주 적인 평이 리는 것이[ ]

만 위 제 은 인 이 살았던 대 사회적 경

을 통해 객 적인 인 의 을 제 하고 있다

36) 정답[ ] ⑤

해설 전 은 인 사건 경 비평이라는[ ] lsquo rsquo⑤

성 이 어져 있다

37) 정답[ ] ①

해설 이 은 동양인과 서양인의 사고 에 차이가[ ]

있다는 것을 대조를 통해 설 하고 있다 또 쓴이

의 제자가 축 경 를 러 가서 경험한 일화를

통해 동양인이 서양인에 비해 주 상황에 더 많은

주의를 인다는 주장을 뒷 침하고 있다

38) 정답[ ] ④

39) 정답[ ] ②

40) 정답[ ] ②

41) 정답[ ] ④

42) 정답[ ] ③

43) 정답[ ] ④

44) 정답 도서 의 휴 일 도서 의 이 간 도서의[ ]

해설 도서 장은 임의 정한 휴 일과 도서 이[ ]

간 도서의 상 등을 게 할 의 가 있다

년 학 간고사 대비2013 2 현대고 대비

ECN-0102-2013-001-000076193

45) 정답[ ] ①

해설 제 조의 정 휴 일 의 휴 일의 사전 게[ ] 3

는 도서 장의 의 조항에 속한다

46) 정답[ ] ①

해설 개인 정 호 의 를 제 하 했 만 항[ ]

나눠서 제 하 않고 대 나열하고 있다

47) 정답[ ] ②

해설 제 조의 내 을 회사는 다른 회사 협[ ] 7 lsquo

계약을 통해 서비 를 제공하는 경 회 의 아이디

등 개인 정 를 해당 회사에 전송할 있다는 내rsquo

이 있으 의 제점을 제 할 있다②

48) 정답[ ] ④

해설 는 도서 장의 의 에 해당하고 나 는 도[ ] ④

서 장의 리에 해당한다

49) 정답[ ] ③

50) 정답 은 음독으 적었고 은 훈독으 적었[ ] (1)

다 과 동일한 표 리 적은 것은 이고 (2) ce

과 동일한 표 리 적은 것은 이다ab

51) 정답[ ] ③

52) 정답[ ] ①②

53) 정답[ ] ③

54) 정답[ ] ③

55) 정답[ ] ①

56) 정답 른 죠코 어린 노 하니라[ ] A B

57) 정답 세 어에서는 활 형이 칙적으[ ] lsquo rsquoㄹㅇ

나타났 만 개화 어에서는 활 형이 쓰 다 lsquo rsquo ㄹㄴ

58) 정답 호 가 흔[ ] (1) (2)

59) 정답[ ] ④

60) 정답[ ] ③

Page 37: 현대고대비 국어 - chamsoriedu.com 「콘텐츠산업진흥 법」외 에도 저작권 의하여 ... 다른주체에게어떤동작을하도록만드는것을나타내는

년 학 간고사 대비2013 2 현대고 대비

ECN-0102-2013-001-000076193

쓴것보다 듸 보고 그 마 니 쓴 고

어 못

그런고 에 리 과 가

만 쓴 못 민 말만 듯고

고 편 그 못 보니 그사 단

병신 못 다고 그사 식 사

니 만 고 다 과 그사

만 고 다 과 업 사 보다 식 고

죠 도 고 각 과

견 고 실 직 귀쳔 간에 그

고도 다 것 몰 귀죡 보다

사 우리 신 귀쳔 다 업

시 신 보고 과 지 게 랴

시니 샹 귀쳔 간에 우리 신 걸

간 보 새지각과 새 걸 미리

독립신- lsquo (1896)rsquo

zb55) 친 어 나의 제 장( ) 2 매 함축적

의 가 가장 유사한 것은

① 지 눈 내리고 매 득 니 내 여 가

사- lsquo rsquo

② 도 어 리듯 그 게 어 다

주 사- lsquo rsquo

③ 눈 살 다 죽 어 린 과 체 여

눈 새벽 지 도 살 다

눈- lsquo rsquo

④ 삶 근심과 고단 에 돌 거니는 여 거 는

여 리 내린 살가지 에 눈 리 눈 리

택 그 생 에- lsquo rsquo

⑤ 늘 러 고 러

청룡 룡 어 개 루 우

신경림 계- lsquo rsquo

zb56) 친 를 위 가 나 에 나타난A B ( ) ( )

세 어의 특 에 의거하여 세 어 표 하

그 산 고 공 도 맑지만

A

주변에 쓰 리는 어리 사 많다

B

건lt gt

식 가 에 타 어 특징에( ) ( )

거 과 어쓰 는 고 지 말 것

A

B

zb57) 가 의( ) 달 아ㆍ 다 의 ( ) 나셔에서 알 있는

세 어 개화 어의 특 을 비 하여 조건 에lt gt

맞게 서 하

건lt gt

어에 는lsquo 개

어에 는 다 태rsquo

zb58) 은 가 는 다 에 나 는 절lt 1gt ( ) lt 2gt ( )

일 를 췌한 것이다 의 의 가 lt 1gt (1)~(2)

유사한 말을 에서 찾아 쓰lt 2gt

보lt 1gt

런 (1) 어린 니 고百 姓

도 내 들 시러 펴 몯 미

사 마다 (2) 니겨 便 安

킈 고 미니

보lt 2gt

죠 고 고 여 보 죠

보다 얼마가 거시 어신고 니 첫 가

죠 니 죠 민

들 어 신 샹 귀쳔

도보고 어보 가 만 늘 고

폐 에 만쓴 죠 민 도

러보지못 고 보니 그게 엇지 심

니 리

년 학 간고사 대비2013 2 현대고 대비

ECN-0102-2013-001-000076193

lt 1 gt

동 룡 샤 마다 복 시( ) ( ) ( )海東 六龍 天福

고 동 시니( ) ( )古聖 同符

lt 2 gt

(A) 매 니 곶

여 니

미 므 니 그 내

러 가 니

lt125 gt

우 미리 샨 에( ) ( ) ( ) 千世 定 漢水北 累

누 개 샤 복 업 시 니( ) ( ) 仁開國 卜年 聖

신( ) 神 니 샤도 경 근민 샤( ) 敬天勤民

욱 드 시 리 다

님 쇼 산 가 ( ) ( )洛水 山行

미드니 가

- lt gt龍飛御天歌

zb59) 장과 내 상 유사한 성격의 조는125

① 뫼 고 고 고 고

어 그린 많고 많고 고 고

어 러 는 울고 울고 가느니

도 견- lt gt

② 강 에 드니 몸 다

그믈 고 가니

뒷 뫼 엄 언 니( )藥

-

③ 말 없는 청산 태 없는 다

값 없는 청 없는 월

에 병 없는 몸 별 없 늙 리

-

④ 가마귀 골에 가지 마

낸 가마귀 새

청강에 것 시 몸 러 가( ) 淸江

-

⑤ 진 골에( ) 白雪

가 매 는 어느 곳에 었는고

에 갈 곳 몰( ) 夕陽

색-

zb60) 위 에 나타난 세 어의 특 으 적절하 않은

것은

① 룡 어 주격 사에 당 는 가 사( ) lsquo rsquo六龍

고 다

② 샤 어에도 어 주체 쓰 다

는 것 다

③ 매 어 달리 사 택에 어

가 지 지지 고 다

④ 므 원 상 직 어 지 다

⑤ 드시리 다 주체 과 상 께 사

고 다

수고 하셨습니다hearts hearts

년 학 간고사 대비2013 2 현대고 대비

ECN-0102-2013-001-000076193

보닷컴에 공 는 별 보는 고등

들 여 주 는

들 습니다 슷 동 지

가 복 는 것 도가

니 복 여 습 시고 거 시

니다

정답 해설

1) 정답[ ] ④

해설 다른 것은 두 특정 업이나 단 내에서 사[ ]

하는 일종의 은어 사회 언에 해당한다 러나

는 언이 아니라 단과대학을 여서 단대 사lsquo rsquo lsquo rsquo lsquo④

대학을 여서 사대라고 한 말에 해당하 일rsquo lsquo rsquo

사회에서도 널리 쓰이 사회 언이라 할

없다

2) 정답[ ] ⑤

해설 사회 언은 같은 단 내에서 쓰이는 언어이[ ] lsquo rsquo

동일 단끼리는 단결 과 친 감을 형성하는

능을 하 리적 안감이 일어나 않는다

3) 정답[ ] ③

해설 사람이라는 차 적 표현에 대한 대안적 표현이[ ]lsquo rsquo

인 아내 처 등으 볼 있다lsquo rsquo

4) 정답[ ]⑤

해설 남성은 주 격 체를 사 한다[ ]

5) 정답[ ] ⑤

해설 흑인은 검다라는 뜻을 가 고 있을 뿐 인[ ]lsquo rsquo lsquo rsquo lsquo rsquo

다 열등한 뜻을 내포하 않는다

6) 정답 살 색 첫 작품[ ] - -

해설 살색 혹은 킨색은 한 인의 피 색을 뜻[ ] lsquo rsquo lsquo rsquo

하는 것으 인종 차 을 추 고 출 이주민

의 평등 을 침해할 있어 년 표 이2005

살 색으 이름을 꾸었다 처녀작은 처녀라lsquo rsquo lsquo rsquo lsquo rsquo

는 단어가 가 고 있는 곡된 성 인 을 한 것

으 첫 작품정도 꾸어 사 하는 것이 좋다lsquo rsquo

7) 정답[ ] ⑤

해설 호는 아들에게 해체를 사 하고 있다[ ] ① ②

장 을 성하는 청자는 자 의 아 느리 아lsquo

들 세 이다 호는 아 느리에게 해rsquo ③

체를 사 하고 있다 호가 느리 아 에게 ④

사 한 해 체 아들에게 사 한 해체는 두 비lsquo rsquo lsquo rsquo

격 체에 해당한다 호는 자 의 아랫사람인 ⑤

느리에게 아들과 마찬가 해체를 사 하는 것이

상 이 만 임 을 한 느리에게 고마 과 쁨

존 의 표 를 하 위해 자 의 아 에게 말하듯

해 체를 사 하고 있다

8) 정답[ ] ③

9) 정답[ ] ⑤

10) 정답[ ] ①

해설 청자 할아 가 장의 주체 아 다 높을[ ] ( ) ( )

경 에는 압존 에 의해 장의 주체를 높이 않는lsquo rsquo

다 러 아 서가 아닌 아 는으 계 lsquo rsquo lsquo rsquo lsquo

니다 가 아닌 있 니다 표현하는 것이 르rsquo lsquo rsquo

11) 정답 당이 당을 쫒았다 당이[ ]

당에 다

해설[ ]

12) 정답[ ] ⑤

해설 서 다른 높임표현을 통해 청자에 대해 리[ ] ⑤

적 거리감을 나타내는 인 은 이 아니라 현정이

다 가 에서 현정은 에게 해 체를 사 함으 써 ( )

친근감을 드러낸다 나 에서 연 을 게을리하는 역 ( )

도 들 때 에 화가 난 현정이 선생님에게 항의하

는 장 에서는 하 체를 사 하여 리적 거리lsquo rsquo

가 어졌음을 나타내고 있다

13) 정답[ ] ①

해설 는 는 얼 빛이 날과 어찌 다르 고[ ] lsquo rsquo

라는 뜻으 전과 달리 임이 화자를 않고

있음을 알 있다

14) 정답 달리 후 가 있다 이를 통해 경[ ] lt gt

쾌한 음악성을 형성하고 노 젓는 상황을 체적으

형상화하는 역할을 한다

15) 정답[ ] ①

16) 정답[ ] ⑤

해설 다 의 자연은 를 성찰하게 하는 대상[ ] ( )⑤

이자 정의 대상이다 의 자연은 자 의 상황과 ⑤

처 를 드러내는 경으 서의 역할을 하 이

이 없다

17) 정답[ ] ③

해설 는 빈천 을 해결하고자 했으나 강산[ ] lsquo ( )rsquo 貧賤③

과 풍 을 달라는 에 거절하 다고 함으 써 자

연에 대한 애정을 드러내고 있으 는 않는

임에 대한 망을 개에게 전가 켜서 임에 대한 리

을 드러내고 있다

18) 정답[ ] ③

년 학 간고사 대비2013 2 현대고 대비

ECN-0102-2013-001-000076193

19) 정답[ ] ⑤

해설 고상한 음악가의 이름을 리말 꽝 럽[ ]

게 꿈으 써 언어유희를 통해 음을 유 하고 있

다 이는 고상한 척하는 총 를 비꼼으 써 비판적

태도를 드러내는 것이 대상을 꽝 럽게 표현

하여 총 의 허 과 사치를 풍자하고 있다

20) 정답[ ] ⑤

해설 는 작품 속 경에 대한 설 이 드러나는 것이[ ]

서 자의 주 적인 견해가 접적으 드러나는 것이

아니다

21) 정답[ ] ⑤

22) 정답[ ] ②

23) 정답[ ] ④

24) 정답[ ] ①

해설 적강 티프는 주인공의 비 한 출생이나 능[ ] ①

과 이 있는 것으 조정의 능함을 풍자하는lsquo rsquo

것과는 거리가 다

25) 정답 픔 나[ ] ( )

해설 의 음악은 고통 는 사람들을 위 하고 아픔[ ] lsquo rsquo

을 치유해 주는 능을 한다고 할 있다 의 lt gt

픔 도 소 된 이 과 더 어 살아가는 따뜻한 마음lsquo rsquo

을 상 한다

26) 정답[ ] ⑤

해설 에게 선천적으 주어 각 장애라는 역경[ ]

은 의 이라는 가사 연 을 있다lsquo rsquo

27) 정답[ ] ④

해설 는 장 란 선 에게 은 개인적인 인상을[ ]

소녀 장정 등으 표현한 것이다lsquo rsquo

28) 정답[ ] ②

해설 담자가 피 담자의 언어적 표현이나 비언어[ ]②

적 표현 하 독자는 담의 위 나 피

담자의 감정 상태를 알 있다 이를 통해 독자는

담 상황을 더 생생하게 느낄 있고 피 담자

를 더 잘 이해할 있게 된다

29) 정답[ ]③

해설 일상생활과 역도 선 서의 성과에 된 것에서[ ]

역도를 하 서 겪는 어 과 내적 고민으 화제를

전화하 위한 것이다

30) 정답[ ] ①

해설 릿속에 새겨 넣듯 이 억되도 함 세상[ ] ② ③

살이가 힘들고 고생 러 속 하여 자유를 ④

가 없는 고통의 상태를 비유적으 이르는 말

적의 침입을 막 위해 쌓은 축 켜야 할⑤

대상을 비유적으 이르는 말이다

31) 정답[ ] ④

해설 이 의 종류는 전 으 인 사건 경[ ] lsquo

비평을 성 소 삼는다rsquo

32) 정답[ ] ④

해설 근은 삼대독자 태어났음을 에서 확인할[ ]

있다 형제들과의 담은 이뤄 가 없다

33) 정답[ ] ⑤

해설 근은 가난에도 하고 화가를 꿈꾸었다[ ] (3

단 또한 다른 화가 망생들은 정 육을)

위해 상 학 학 해 유학 에 랐 만

근은 다른 을 찾아야 했다 단 세에(5 ) 18

근은 조선 전람회에 입선하 다 단 의(6 )

만종은 인간과 자연이 엮어 가는 경건한 조화 을lsquo rsquo

나타낸다

34) 정답[ ] ①

해설 근이 속에서도 창작활동을 추 않고[ ]

하는 닭은 은 세상과 타협할 르는

근이 세상의 이해를 하 위한 가장 떳떳한 단

이 때 이다

35) 정답[ ] ⑤

해설 전 은 서 자의 주 적인 평이 리는 것이[ ]

만 위 제 은 인 이 살았던 대 사회적 경

을 통해 객 적인 인 의 을 제 하고 있다

36) 정답[ ] ⑤

해설 전 은 인 사건 경 비평이라는[ ] lsquo rsquo⑤

성 이 어져 있다

37) 정답[ ] ①

해설 이 은 동양인과 서양인의 사고 에 차이가[ ]

있다는 것을 대조를 통해 설 하고 있다 또 쓴이

의 제자가 축 경 를 러 가서 경험한 일화를

통해 동양인이 서양인에 비해 주 상황에 더 많은

주의를 인다는 주장을 뒷 침하고 있다

38) 정답[ ] ④

39) 정답[ ] ②

40) 정답[ ] ②

41) 정답[ ] ④

42) 정답[ ] ③

43) 정답[ ] ④

44) 정답 도서 의 휴 일 도서 의 이 간 도서의[ ]

해설 도서 장은 임의 정한 휴 일과 도서 이[ ]

간 도서의 상 등을 게 할 의 가 있다

년 학 간고사 대비2013 2 현대고 대비

ECN-0102-2013-001-000076193

45) 정답[ ] ①

해설 제 조의 정 휴 일 의 휴 일의 사전 게[ ] 3

는 도서 장의 의 조항에 속한다

46) 정답[ ] ①

해설 개인 정 호 의 를 제 하 했 만 항[ ]

나눠서 제 하 않고 대 나열하고 있다

47) 정답[ ] ②

해설 제 조의 내 을 회사는 다른 회사 협[ ] 7 lsquo

계약을 통해 서비 를 제공하는 경 회 의 아이디

등 개인 정 를 해당 회사에 전송할 있다는 내rsquo

이 있으 의 제점을 제 할 있다②

48) 정답[ ] ④

해설 는 도서 장의 의 에 해당하고 나 는 도[ ] ④

서 장의 리에 해당한다

49) 정답[ ] ③

50) 정답 은 음독으 적었고 은 훈독으 적었[ ] (1)

다 과 동일한 표 리 적은 것은 이고 (2) ce

과 동일한 표 리 적은 것은 이다ab

51) 정답[ ] ③

52) 정답[ ] ①②

53) 정답[ ] ③

54) 정답[ ] ③

55) 정답[ ] ①

56) 정답 른 죠코 어린 노 하니라[ ] A B

57) 정답 세 어에서는 활 형이 칙적으[ ] lsquo rsquoㄹㅇ

나타났 만 개화 어에서는 활 형이 쓰 다 lsquo rsquo ㄹㄴ

58) 정답 호 가 흔[ ] (1) (2)

59) 정답[ ] ④

60) 정답[ ] ③

Page 38: 현대고대비 국어 - chamsoriedu.com 「콘텐츠산업진흥 법」외 에도 저작권 의하여 ... 다른주체에게어떤동작을하도록만드는것을나타내는

년 학 간고사 대비2013 2 현대고 대비

ECN-0102-2013-001-000076193

lt 1 gt

동 룡 샤 마다 복 시( ) ( ) ( )海東 六龍 天福

고 동 시니( ) ( )古聖 同符

lt 2 gt

(A) 매 니 곶

여 니

미 므 니 그 내

러 가 니

lt125 gt

우 미리 샨 에( ) ( ) ( ) 千世 定 漢水北 累

누 개 샤 복 업 시 니( ) ( ) 仁開國 卜年 聖

신( ) 神 니 샤도 경 근민 샤( ) 敬天勤民

욱 드 시 리 다

님 쇼 산 가 ( ) ( )洛水 山行

미드니 가

- lt gt龍飛御天歌

zb59) 장과 내 상 유사한 성격의 조는125

① 뫼 고 고 고 고

어 그린 많고 많고 고 고

어 러 는 울고 울고 가느니

도 견- lt gt

② 강 에 드니 몸 다

그믈 고 가니

뒷 뫼 엄 언 니( )藥

-

③ 말 없는 청산 태 없는 다

값 없는 청 없는 월

에 병 없는 몸 별 없 늙 리

-

④ 가마귀 골에 가지 마

낸 가마귀 새

청강에 것 시 몸 러 가( ) 淸江

-

⑤ 진 골에( ) 白雪

가 매 는 어느 곳에 었는고

에 갈 곳 몰( ) 夕陽

색-

zb60) 위 에 나타난 세 어의 특 으 적절하 않은

것은

① 룡 어 주격 사에 당 는 가 사( ) lsquo rsquo六龍

고 다

② 샤 어에도 어 주체 쓰 다

는 것 다

③ 매 어 달리 사 택에 어

가 지 지지 고 다

④ 므 원 상 직 어 지 다

⑤ 드시리 다 주체 과 상 께 사

고 다

수고 하셨습니다hearts hearts

년 학 간고사 대비2013 2 현대고 대비

ECN-0102-2013-001-000076193

보닷컴에 공 는 별 보는 고등

들 여 주 는

들 습니다 슷 동 지

가 복 는 것 도가

니 복 여 습 시고 거 시

니다

정답 해설

1) 정답[ ] ④

해설 다른 것은 두 특정 업이나 단 내에서 사[ ]

하는 일종의 은어 사회 언에 해당한다 러나

는 언이 아니라 단과대학을 여서 단대 사lsquo rsquo lsquo rsquo lsquo④

대학을 여서 사대라고 한 말에 해당하 일rsquo lsquo rsquo

사회에서도 널리 쓰이 사회 언이라 할

없다

2) 정답[ ] ⑤

해설 사회 언은 같은 단 내에서 쓰이는 언어이[ ] lsquo rsquo

동일 단끼리는 단결 과 친 감을 형성하는

능을 하 리적 안감이 일어나 않는다

3) 정답[ ] ③

해설 사람이라는 차 적 표현에 대한 대안적 표현이[ ]lsquo rsquo

인 아내 처 등으 볼 있다lsquo rsquo

4) 정답[ ]⑤

해설 남성은 주 격 체를 사 한다[ ]

5) 정답[ ] ⑤

해설 흑인은 검다라는 뜻을 가 고 있을 뿐 인[ ]lsquo rsquo lsquo rsquo lsquo rsquo

다 열등한 뜻을 내포하 않는다

6) 정답 살 색 첫 작품[ ] - -

해설 살색 혹은 킨색은 한 인의 피 색을 뜻[ ] lsquo rsquo lsquo rsquo

하는 것으 인종 차 을 추 고 출 이주민

의 평등 을 침해할 있어 년 표 이2005

살 색으 이름을 꾸었다 처녀작은 처녀라lsquo rsquo lsquo rsquo lsquo rsquo

는 단어가 가 고 있는 곡된 성 인 을 한 것

으 첫 작품정도 꾸어 사 하는 것이 좋다lsquo rsquo

7) 정답[ ] ⑤

해설 호는 아들에게 해체를 사 하고 있다[ ] ① ②

장 을 성하는 청자는 자 의 아 느리 아lsquo

들 세 이다 호는 아 느리에게 해rsquo ③

체를 사 하고 있다 호가 느리 아 에게 ④

사 한 해 체 아들에게 사 한 해체는 두 비lsquo rsquo lsquo rsquo

격 체에 해당한다 호는 자 의 아랫사람인 ⑤

느리에게 아들과 마찬가 해체를 사 하는 것이

상 이 만 임 을 한 느리에게 고마 과 쁨

존 의 표 를 하 위해 자 의 아 에게 말하듯

해 체를 사 하고 있다

8) 정답[ ] ③

9) 정답[ ] ⑤

10) 정답[ ] ①

해설 청자 할아 가 장의 주체 아 다 높을[ ] ( ) ( )

경 에는 압존 에 의해 장의 주체를 높이 않는lsquo rsquo

다 러 아 서가 아닌 아 는으 계 lsquo rsquo lsquo rsquo lsquo

니다 가 아닌 있 니다 표현하는 것이 르rsquo lsquo rsquo

11) 정답 당이 당을 쫒았다 당이[ ]

당에 다

해설[ ]

12) 정답[ ] ⑤

해설 서 다른 높임표현을 통해 청자에 대해 리[ ] ⑤

적 거리감을 나타내는 인 은 이 아니라 현정이

다 가 에서 현정은 에게 해 체를 사 함으 써 ( )

친근감을 드러낸다 나 에서 연 을 게을리하는 역 ( )

도 들 때 에 화가 난 현정이 선생님에게 항의하

는 장 에서는 하 체를 사 하여 리적 거리lsquo rsquo

가 어졌음을 나타내고 있다

13) 정답[ ] ①

해설 는 는 얼 빛이 날과 어찌 다르 고[ ] lsquo rsquo

라는 뜻으 전과 달리 임이 화자를 않고

있음을 알 있다

14) 정답 달리 후 가 있다 이를 통해 경[ ] lt gt

쾌한 음악성을 형성하고 노 젓는 상황을 체적으

형상화하는 역할을 한다

15) 정답[ ] ①

16) 정답[ ] ⑤

해설 다 의 자연은 를 성찰하게 하는 대상[ ] ( )⑤

이자 정의 대상이다 의 자연은 자 의 상황과 ⑤

처 를 드러내는 경으 서의 역할을 하 이

이 없다

17) 정답[ ] ③

해설 는 빈천 을 해결하고자 했으나 강산[ ] lsquo ( )rsquo 貧賤③

과 풍 을 달라는 에 거절하 다고 함으 써 자

연에 대한 애정을 드러내고 있으 는 않는

임에 대한 망을 개에게 전가 켜서 임에 대한 리

을 드러내고 있다

18) 정답[ ] ③

년 학 간고사 대비2013 2 현대고 대비

ECN-0102-2013-001-000076193

19) 정답[ ] ⑤

해설 고상한 음악가의 이름을 리말 꽝 럽[ ]

게 꿈으 써 언어유희를 통해 음을 유 하고 있

다 이는 고상한 척하는 총 를 비꼼으 써 비판적

태도를 드러내는 것이 대상을 꽝 럽게 표현

하여 총 의 허 과 사치를 풍자하고 있다

20) 정답[ ] ⑤

해설 는 작품 속 경에 대한 설 이 드러나는 것이[ ]

서 자의 주 적인 견해가 접적으 드러나는 것이

아니다

21) 정답[ ] ⑤

22) 정답[ ] ②

23) 정답[ ] ④

24) 정답[ ] ①

해설 적강 티프는 주인공의 비 한 출생이나 능[ ] ①

과 이 있는 것으 조정의 능함을 풍자하는lsquo rsquo

것과는 거리가 다

25) 정답 픔 나[ ] ( )

해설 의 음악은 고통 는 사람들을 위 하고 아픔[ ] lsquo rsquo

을 치유해 주는 능을 한다고 할 있다 의 lt gt

픔 도 소 된 이 과 더 어 살아가는 따뜻한 마음lsquo rsquo

을 상 한다

26) 정답[ ] ⑤

해설 에게 선천적으 주어 각 장애라는 역경[ ]

은 의 이라는 가사 연 을 있다lsquo rsquo

27) 정답[ ] ④

해설 는 장 란 선 에게 은 개인적인 인상을[ ]

소녀 장정 등으 표현한 것이다lsquo rsquo

28) 정답[ ] ②

해설 담자가 피 담자의 언어적 표현이나 비언어[ ]②

적 표현 하 독자는 담의 위 나 피

담자의 감정 상태를 알 있다 이를 통해 독자는

담 상황을 더 생생하게 느낄 있고 피 담자

를 더 잘 이해할 있게 된다

29) 정답[ ]③

해설 일상생활과 역도 선 서의 성과에 된 것에서[ ]

역도를 하 서 겪는 어 과 내적 고민으 화제를

전화하 위한 것이다

30) 정답[ ] ①

해설 릿속에 새겨 넣듯 이 억되도 함 세상[ ] ② ③

살이가 힘들고 고생 러 속 하여 자유를 ④

가 없는 고통의 상태를 비유적으 이르는 말

적의 침입을 막 위해 쌓은 축 켜야 할⑤

대상을 비유적으 이르는 말이다

31) 정답[ ] ④

해설 이 의 종류는 전 으 인 사건 경[ ] lsquo

비평을 성 소 삼는다rsquo

32) 정답[ ] ④

해설 근은 삼대독자 태어났음을 에서 확인할[ ]

있다 형제들과의 담은 이뤄 가 없다

33) 정답[ ] ⑤

해설 근은 가난에도 하고 화가를 꿈꾸었다[ ] (3

단 또한 다른 화가 망생들은 정 육을)

위해 상 학 학 해 유학 에 랐 만

근은 다른 을 찾아야 했다 단 세에(5 ) 18

근은 조선 전람회에 입선하 다 단 의(6 )

만종은 인간과 자연이 엮어 가는 경건한 조화 을lsquo rsquo

나타낸다

34) 정답[ ] ①

해설 근이 속에서도 창작활동을 추 않고[ ]

하는 닭은 은 세상과 타협할 르는

근이 세상의 이해를 하 위한 가장 떳떳한 단

이 때 이다

35) 정답[ ] ⑤

해설 전 은 서 자의 주 적인 평이 리는 것이[ ]

만 위 제 은 인 이 살았던 대 사회적 경

을 통해 객 적인 인 의 을 제 하고 있다

36) 정답[ ] ⑤

해설 전 은 인 사건 경 비평이라는[ ] lsquo rsquo⑤

성 이 어져 있다

37) 정답[ ] ①

해설 이 은 동양인과 서양인의 사고 에 차이가[ ]

있다는 것을 대조를 통해 설 하고 있다 또 쓴이

의 제자가 축 경 를 러 가서 경험한 일화를

통해 동양인이 서양인에 비해 주 상황에 더 많은

주의를 인다는 주장을 뒷 침하고 있다

38) 정답[ ] ④

39) 정답[ ] ②

40) 정답[ ] ②

41) 정답[ ] ④

42) 정답[ ] ③

43) 정답[ ] ④

44) 정답 도서 의 휴 일 도서 의 이 간 도서의[ ]

해설 도서 장은 임의 정한 휴 일과 도서 이[ ]

간 도서의 상 등을 게 할 의 가 있다

년 학 간고사 대비2013 2 현대고 대비

ECN-0102-2013-001-000076193

45) 정답[ ] ①

해설 제 조의 정 휴 일 의 휴 일의 사전 게[ ] 3

는 도서 장의 의 조항에 속한다

46) 정답[ ] ①

해설 개인 정 호 의 를 제 하 했 만 항[ ]

나눠서 제 하 않고 대 나열하고 있다

47) 정답[ ] ②

해설 제 조의 내 을 회사는 다른 회사 협[ ] 7 lsquo

계약을 통해 서비 를 제공하는 경 회 의 아이디

등 개인 정 를 해당 회사에 전송할 있다는 내rsquo

이 있으 의 제점을 제 할 있다②

48) 정답[ ] ④

해설 는 도서 장의 의 에 해당하고 나 는 도[ ] ④

서 장의 리에 해당한다

49) 정답[ ] ③

50) 정답 은 음독으 적었고 은 훈독으 적었[ ] (1)

다 과 동일한 표 리 적은 것은 이고 (2) ce

과 동일한 표 리 적은 것은 이다ab

51) 정답[ ] ③

52) 정답[ ] ①②

53) 정답[ ] ③

54) 정답[ ] ③

55) 정답[ ] ①

56) 정답 른 죠코 어린 노 하니라[ ] A B

57) 정답 세 어에서는 활 형이 칙적으[ ] lsquo rsquoㄹㅇ

나타났 만 개화 어에서는 활 형이 쓰 다 lsquo rsquo ㄹㄴ

58) 정답 호 가 흔[ ] (1) (2)

59) 정답[ ] ④

60) 정답[ ] ③

Page 39: 현대고대비 국어 - chamsoriedu.com 「콘텐츠산업진흥 법」외 에도 저작권 의하여 ... 다른주체에게어떤동작을하도록만드는것을나타내는

년 학 간고사 대비2013 2 현대고 대비

ECN-0102-2013-001-000076193

보닷컴에 공 는 별 보는 고등

들 여 주 는

들 습니다 슷 동 지

가 복 는 것 도가

니 복 여 습 시고 거 시

니다

정답 해설

1) 정답[ ] ④

해설 다른 것은 두 특정 업이나 단 내에서 사[ ]

하는 일종의 은어 사회 언에 해당한다 러나

는 언이 아니라 단과대학을 여서 단대 사lsquo rsquo lsquo rsquo lsquo④

대학을 여서 사대라고 한 말에 해당하 일rsquo lsquo rsquo

사회에서도 널리 쓰이 사회 언이라 할

없다

2) 정답[ ] ⑤

해설 사회 언은 같은 단 내에서 쓰이는 언어이[ ] lsquo rsquo

동일 단끼리는 단결 과 친 감을 형성하는

능을 하 리적 안감이 일어나 않는다

3) 정답[ ] ③

해설 사람이라는 차 적 표현에 대한 대안적 표현이[ ]lsquo rsquo

인 아내 처 등으 볼 있다lsquo rsquo

4) 정답[ ]⑤

해설 남성은 주 격 체를 사 한다[ ]

5) 정답[ ] ⑤

해설 흑인은 검다라는 뜻을 가 고 있을 뿐 인[ ]lsquo rsquo lsquo rsquo lsquo rsquo

다 열등한 뜻을 내포하 않는다

6) 정답 살 색 첫 작품[ ] - -

해설 살색 혹은 킨색은 한 인의 피 색을 뜻[ ] lsquo rsquo lsquo rsquo

하는 것으 인종 차 을 추 고 출 이주민

의 평등 을 침해할 있어 년 표 이2005

살 색으 이름을 꾸었다 처녀작은 처녀라lsquo rsquo lsquo rsquo lsquo rsquo

는 단어가 가 고 있는 곡된 성 인 을 한 것

으 첫 작품정도 꾸어 사 하는 것이 좋다lsquo rsquo

7) 정답[ ] ⑤

해설 호는 아들에게 해체를 사 하고 있다[ ] ① ②

장 을 성하는 청자는 자 의 아 느리 아lsquo

들 세 이다 호는 아 느리에게 해rsquo ③

체를 사 하고 있다 호가 느리 아 에게 ④

사 한 해 체 아들에게 사 한 해체는 두 비lsquo rsquo lsquo rsquo

격 체에 해당한다 호는 자 의 아랫사람인 ⑤

느리에게 아들과 마찬가 해체를 사 하는 것이

상 이 만 임 을 한 느리에게 고마 과 쁨

존 의 표 를 하 위해 자 의 아 에게 말하듯

해 체를 사 하고 있다

8) 정답[ ] ③

9) 정답[ ] ⑤

10) 정답[ ] ①

해설 청자 할아 가 장의 주체 아 다 높을[ ] ( ) ( )

경 에는 압존 에 의해 장의 주체를 높이 않는lsquo rsquo

다 러 아 서가 아닌 아 는으 계 lsquo rsquo lsquo rsquo lsquo

니다 가 아닌 있 니다 표현하는 것이 르rsquo lsquo rsquo

11) 정답 당이 당을 쫒았다 당이[ ]

당에 다

해설[ ]

12) 정답[ ] ⑤

해설 서 다른 높임표현을 통해 청자에 대해 리[ ] ⑤

적 거리감을 나타내는 인 은 이 아니라 현정이

다 가 에서 현정은 에게 해 체를 사 함으 써 ( )

친근감을 드러낸다 나 에서 연 을 게을리하는 역 ( )

도 들 때 에 화가 난 현정이 선생님에게 항의하

는 장 에서는 하 체를 사 하여 리적 거리lsquo rsquo

가 어졌음을 나타내고 있다

13) 정답[ ] ①

해설 는 는 얼 빛이 날과 어찌 다르 고[ ] lsquo rsquo

라는 뜻으 전과 달리 임이 화자를 않고

있음을 알 있다

14) 정답 달리 후 가 있다 이를 통해 경[ ] lt gt

쾌한 음악성을 형성하고 노 젓는 상황을 체적으

형상화하는 역할을 한다

15) 정답[ ] ①

16) 정답[ ] ⑤

해설 다 의 자연은 를 성찰하게 하는 대상[ ] ( )⑤

이자 정의 대상이다 의 자연은 자 의 상황과 ⑤

처 를 드러내는 경으 서의 역할을 하 이

이 없다

17) 정답[ ] ③

해설 는 빈천 을 해결하고자 했으나 강산[ ] lsquo ( )rsquo 貧賤③

과 풍 을 달라는 에 거절하 다고 함으 써 자

연에 대한 애정을 드러내고 있으 는 않는

임에 대한 망을 개에게 전가 켜서 임에 대한 리

을 드러내고 있다

18) 정답[ ] ③

년 학 간고사 대비2013 2 현대고 대비

ECN-0102-2013-001-000076193

19) 정답[ ] ⑤

해설 고상한 음악가의 이름을 리말 꽝 럽[ ]

게 꿈으 써 언어유희를 통해 음을 유 하고 있

다 이는 고상한 척하는 총 를 비꼼으 써 비판적

태도를 드러내는 것이 대상을 꽝 럽게 표현

하여 총 의 허 과 사치를 풍자하고 있다

20) 정답[ ] ⑤

해설 는 작품 속 경에 대한 설 이 드러나는 것이[ ]

서 자의 주 적인 견해가 접적으 드러나는 것이

아니다

21) 정답[ ] ⑤

22) 정답[ ] ②

23) 정답[ ] ④

24) 정답[ ] ①

해설 적강 티프는 주인공의 비 한 출생이나 능[ ] ①

과 이 있는 것으 조정의 능함을 풍자하는lsquo rsquo

것과는 거리가 다

25) 정답 픔 나[ ] ( )

해설 의 음악은 고통 는 사람들을 위 하고 아픔[ ] lsquo rsquo

을 치유해 주는 능을 한다고 할 있다 의 lt gt

픔 도 소 된 이 과 더 어 살아가는 따뜻한 마음lsquo rsquo

을 상 한다

26) 정답[ ] ⑤

해설 에게 선천적으 주어 각 장애라는 역경[ ]

은 의 이라는 가사 연 을 있다lsquo rsquo

27) 정답[ ] ④

해설 는 장 란 선 에게 은 개인적인 인상을[ ]

소녀 장정 등으 표현한 것이다lsquo rsquo

28) 정답[ ] ②

해설 담자가 피 담자의 언어적 표현이나 비언어[ ]②

적 표현 하 독자는 담의 위 나 피

담자의 감정 상태를 알 있다 이를 통해 독자는

담 상황을 더 생생하게 느낄 있고 피 담자

를 더 잘 이해할 있게 된다

29) 정답[ ]③

해설 일상생활과 역도 선 서의 성과에 된 것에서[ ]

역도를 하 서 겪는 어 과 내적 고민으 화제를

전화하 위한 것이다

30) 정답[ ] ①

해설 릿속에 새겨 넣듯 이 억되도 함 세상[ ] ② ③

살이가 힘들고 고생 러 속 하여 자유를 ④

가 없는 고통의 상태를 비유적으 이르는 말

적의 침입을 막 위해 쌓은 축 켜야 할⑤

대상을 비유적으 이르는 말이다

31) 정답[ ] ④

해설 이 의 종류는 전 으 인 사건 경[ ] lsquo

비평을 성 소 삼는다rsquo

32) 정답[ ] ④

해설 근은 삼대독자 태어났음을 에서 확인할[ ]

있다 형제들과의 담은 이뤄 가 없다

33) 정답[ ] ⑤

해설 근은 가난에도 하고 화가를 꿈꾸었다[ ] (3

단 또한 다른 화가 망생들은 정 육을)

위해 상 학 학 해 유학 에 랐 만

근은 다른 을 찾아야 했다 단 세에(5 ) 18

근은 조선 전람회에 입선하 다 단 의(6 )

만종은 인간과 자연이 엮어 가는 경건한 조화 을lsquo rsquo

나타낸다

34) 정답[ ] ①

해설 근이 속에서도 창작활동을 추 않고[ ]

하는 닭은 은 세상과 타협할 르는

근이 세상의 이해를 하 위한 가장 떳떳한 단

이 때 이다

35) 정답[ ] ⑤

해설 전 은 서 자의 주 적인 평이 리는 것이[ ]

만 위 제 은 인 이 살았던 대 사회적 경

을 통해 객 적인 인 의 을 제 하고 있다

36) 정답[ ] ⑤

해설 전 은 인 사건 경 비평이라는[ ] lsquo rsquo⑤

성 이 어져 있다

37) 정답[ ] ①

해설 이 은 동양인과 서양인의 사고 에 차이가[ ]

있다는 것을 대조를 통해 설 하고 있다 또 쓴이

의 제자가 축 경 를 러 가서 경험한 일화를

통해 동양인이 서양인에 비해 주 상황에 더 많은

주의를 인다는 주장을 뒷 침하고 있다

38) 정답[ ] ④

39) 정답[ ] ②

40) 정답[ ] ②

41) 정답[ ] ④

42) 정답[ ] ③

43) 정답[ ] ④

44) 정답 도서 의 휴 일 도서 의 이 간 도서의[ ]

해설 도서 장은 임의 정한 휴 일과 도서 이[ ]

간 도서의 상 등을 게 할 의 가 있다

년 학 간고사 대비2013 2 현대고 대비

ECN-0102-2013-001-000076193

45) 정답[ ] ①

해설 제 조의 정 휴 일 의 휴 일의 사전 게[ ] 3

는 도서 장의 의 조항에 속한다

46) 정답[ ] ①

해설 개인 정 호 의 를 제 하 했 만 항[ ]

나눠서 제 하 않고 대 나열하고 있다

47) 정답[ ] ②

해설 제 조의 내 을 회사는 다른 회사 협[ ] 7 lsquo

계약을 통해 서비 를 제공하는 경 회 의 아이디

등 개인 정 를 해당 회사에 전송할 있다는 내rsquo

이 있으 의 제점을 제 할 있다②

48) 정답[ ] ④

해설 는 도서 장의 의 에 해당하고 나 는 도[ ] ④

서 장의 리에 해당한다

49) 정답[ ] ③

50) 정답 은 음독으 적었고 은 훈독으 적었[ ] (1)

다 과 동일한 표 리 적은 것은 이고 (2) ce

과 동일한 표 리 적은 것은 이다ab

51) 정답[ ] ③

52) 정답[ ] ①②

53) 정답[ ] ③

54) 정답[ ] ③

55) 정답[ ] ①

56) 정답 른 죠코 어린 노 하니라[ ] A B

57) 정답 세 어에서는 활 형이 칙적으[ ] lsquo rsquoㄹㅇ

나타났 만 개화 어에서는 활 형이 쓰 다 lsquo rsquo ㄹㄴ

58) 정답 호 가 흔[ ] (1) (2)

59) 정답[ ] ④

60) 정답[ ] ③

Page 40: 현대고대비 국어 - chamsoriedu.com 「콘텐츠산업진흥 법」외 에도 저작권 의하여 ... 다른주체에게어떤동작을하도록만드는것을나타내는

년 학 간고사 대비2013 2 현대고 대비

ECN-0102-2013-001-000076193

19) 정답[ ] ⑤

해설 고상한 음악가의 이름을 리말 꽝 럽[ ]

게 꿈으 써 언어유희를 통해 음을 유 하고 있

다 이는 고상한 척하는 총 를 비꼼으 써 비판적

태도를 드러내는 것이 대상을 꽝 럽게 표현

하여 총 의 허 과 사치를 풍자하고 있다

20) 정답[ ] ⑤

해설 는 작품 속 경에 대한 설 이 드러나는 것이[ ]

서 자의 주 적인 견해가 접적으 드러나는 것이

아니다

21) 정답[ ] ⑤

22) 정답[ ] ②

23) 정답[ ] ④

24) 정답[ ] ①

해설 적강 티프는 주인공의 비 한 출생이나 능[ ] ①

과 이 있는 것으 조정의 능함을 풍자하는lsquo rsquo

것과는 거리가 다

25) 정답 픔 나[ ] ( )

해설 의 음악은 고통 는 사람들을 위 하고 아픔[ ] lsquo rsquo

을 치유해 주는 능을 한다고 할 있다 의 lt gt

픔 도 소 된 이 과 더 어 살아가는 따뜻한 마음lsquo rsquo

을 상 한다

26) 정답[ ] ⑤

해설 에게 선천적으 주어 각 장애라는 역경[ ]

은 의 이라는 가사 연 을 있다lsquo rsquo

27) 정답[ ] ④

해설 는 장 란 선 에게 은 개인적인 인상을[ ]

소녀 장정 등으 표현한 것이다lsquo rsquo

28) 정답[ ] ②

해설 담자가 피 담자의 언어적 표현이나 비언어[ ]②

적 표현 하 독자는 담의 위 나 피

담자의 감정 상태를 알 있다 이를 통해 독자는

담 상황을 더 생생하게 느낄 있고 피 담자

를 더 잘 이해할 있게 된다

29) 정답[ ]③

해설 일상생활과 역도 선 서의 성과에 된 것에서[ ]

역도를 하 서 겪는 어 과 내적 고민으 화제를

전화하 위한 것이다

30) 정답[ ] ①

해설 릿속에 새겨 넣듯 이 억되도 함 세상[ ] ② ③

살이가 힘들고 고생 러 속 하여 자유를 ④

가 없는 고통의 상태를 비유적으 이르는 말

적의 침입을 막 위해 쌓은 축 켜야 할⑤

대상을 비유적으 이르는 말이다

31) 정답[ ] ④

해설 이 의 종류는 전 으 인 사건 경[ ] lsquo

비평을 성 소 삼는다rsquo

32) 정답[ ] ④

해설 근은 삼대독자 태어났음을 에서 확인할[ ]

있다 형제들과의 담은 이뤄 가 없다

33) 정답[ ] ⑤

해설 근은 가난에도 하고 화가를 꿈꾸었다[ ] (3

단 또한 다른 화가 망생들은 정 육을)

위해 상 학 학 해 유학 에 랐 만

근은 다른 을 찾아야 했다 단 세에(5 ) 18

근은 조선 전람회에 입선하 다 단 의(6 )

만종은 인간과 자연이 엮어 가는 경건한 조화 을lsquo rsquo

나타낸다

34) 정답[ ] ①

해설 근이 속에서도 창작활동을 추 않고[ ]

하는 닭은 은 세상과 타협할 르는

근이 세상의 이해를 하 위한 가장 떳떳한 단

이 때 이다

35) 정답[ ] ⑤

해설 전 은 서 자의 주 적인 평이 리는 것이[ ]

만 위 제 은 인 이 살았던 대 사회적 경

을 통해 객 적인 인 의 을 제 하고 있다

36) 정답[ ] ⑤

해설 전 은 인 사건 경 비평이라는[ ] lsquo rsquo⑤

성 이 어져 있다

37) 정답[ ] ①

해설 이 은 동양인과 서양인의 사고 에 차이가[ ]

있다는 것을 대조를 통해 설 하고 있다 또 쓴이

의 제자가 축 경 를 러 가서 경험한 일화를

통해 동양인이 서양인에 비해 주 상황에 더 많은

주의를 인다는 주장을 뒷 침하고 있다

38) 정답[ ] ④

39) 정답[ ] ②

40) 정답[ ] ②

41) 정답[ ] ④

42) 정답[ ] ③

43) 정답[ ] ④

44) 정답 도서 의 휴 일 도서 의 이 간 도서의[ ]

해설 도서 장은 임의 정한 휴 일과 도서 이[ ]

간 도서의 상 등을 게 할 의 가 있다

년 학 간고사 대비2013 2 현대고 대비

ECN-0102-2013-001-000076193

45) 정답[ ] ①

해설 제 조의 정 휴 일 의 휴 일의 사전 게[ ] 3

는 도서 장의 의 조항에 속한다

46) 정답[ ] ①

해설 개인 정 호 의 를 제 하 했 만 항[ ]

나눠서 제 하 않고 대 나열하고 있다

47) 정답[ ] ②

해설 제 조의 내 을 회사는 다른 회사 협[ ] 7 lsquo

계약을 통해 서비 를 제공하는 경 회 의 아이디

등 개인 정 를 해당 회사에 전송할 있다는 내rsquo

이 있으 의 제점을 제 할 있다②

48) 정답[ ] ④

해설 는 도서 장의 의 에 해당하고 나 는 도[ ] ④

서 장의 리에 해당한다

49) 정답[ ] ③

50) 정답 은 음독으 적었고 은 훈독으 적었[ ] (1)

다 과 동일한 표 리 적은 것은 이고 (2) ce

과 동일한 표 리 적은 것은 이다ab

51) 정답[ ] ③

52) 정답[ ] ①②

53) 정답[ ] ③

54) 정답[ ] ③

55) 정답[ ] ①

56) 정답 른 죠코 어린 노 하니라[ ] A B

57) 정답 세 어에서는 활 형이 칙적으[ ] lsquo rsquoㄹㅇ

나타났 만 개화 어에서는 활 형이 쓰 다 lsquo rsquo ㄹㄴ

58) 정답 호 가 흔[ ] (1) (2)

59) 정답[ ] ④

60) 정답[ ] ③

Page 41: 현대고대비 국어 - chamsoriedu.com 「콘텐츠산업진흥 법」외 에도 저작권 의하여 ... 다른주체에게어떤동작을하도록만드는것을나타내는

년 학 간고사 대비2013 2 현대고 대비

ECN-0102-2013-001-000076193

45) 정답[ ] ①

해설 제 조의 정 휴 일 의 휴 일의 사전 게[ ] 3

는 도서 장의 의 조항에 속한다

46) 정답[ ] ①

해설 개인 정 호 의 를 제 하 했 만 항[ ]

나눠서 제 하 않고 대 나열하고 있다

47) 정답[ ] ②

해설 제 조의 내 을 회사는 다른 회사 협[ ] 7 lsquo

계약을 통해 서비 를 제공하는 경 회 의 아이디

등 개인 정 를 해당 회사에 전송할 있다는 내rsquo

이 있으 의 제점을 제 할 있다②

48) 정답[ ] ④

해설 는 도서 장의 의 에 해당하고 나 는 도[ ] ④

서 장의 리에 해당한다

49) 정답[ ] ③

50) 정답 은 음독으 적었고 은 훈독으 적었[ ] (1)

다 과 동일한 표 리 적은 것은 이고 (2) ce

과 동일한 표 리 적은 것은 이다ab

51) 정답[ ] ③

52) 정답[ ] ①②

53) 정답[ ] ③

54) 정답[ ] ③

55) 정답[ ] ①

56) 정답 른 죠코 어린 노 하니라[ ] A B

57) 정답 세 어에서는 활 형이 칙적으[ ] lsquo rsquoㄹㅇ

나타났 만 개화 어에서는 활 형이 쓰 다 lsquo rsquo ㄹㄴ

58) 정답 호 가 흔[ ] (1) (2)

59) 정답[ ] ④

60) 정답[ ] ③